Canadian Fundamentals of Nursing 6th Edition Potter Test Bank
Canadian Fundamentals of Nursing 6th Edition Potter Test Bank Chapter 01: Health and Wellness Potter et al: Canadian Fundamentals of Nursing, 6th Edition MULTIPLE CHOICE 1. The nurse is using the population health promotion model to develop actions for improving
health. After asking, “On what should we take action?”; “How should we take action?”; and “Why should we take action?” the nurse will ask which of the following questions? a. “With whom should we act?” b. “When should we take action?” c. “Which government should take action?” d. “Where should we first act?” ANS: A
The next question to ask when using the population health model approach is “With whom should we act?” The other choices are not questions included in this model. DIF: Apply REF: 13, Figure 1-5 OBJ: Contrast distinguishing features of health promotion and disease prevention. TOP: Implementation MSC: CPNRE: Foundations of Practice 2. The principle “Health promotion is multisectoral” means which of the following? a. Relationships between individual, social, and environmental factors must be
recognized. b. Physical, mental, social, ecological, cultural, and spiritual aspects of health must
be recognized.
N R I G B.C M
c. In order to change unhealthU y liS vingNanT d workiO ng conditions, areas other than health
must also be involved. d. Health promotion involves the use of knowledge from disciplines such as social,
economic, political, environmental, medical, and nursing sciences, as well as from first-hand experience. ANS: C
The statement “Health promotion is multisectoral” is the principle explained by the necessity to involve areas other than health in order to change unhealthy living and working conditions. DIF: Understand REF: 11 OBJ: Contrast distinguishing features of health promotion and disease prevention. TOP: Planning MSC: CPNRE: Foundations of Practice 3. According to the World Health Organization, what is the best description of “health”? a. Simply the absence of disease. b. Involving the total person and environment. c. Strictly personal in nature. d. Status of pathological state. ANS: B
Canadian Fundamentals of Nursing 6th Edition Potter Test Bank The WHO defines health as “…the extent to which an individual or group is able, on the one hand, to realize aspirations and satisfy needs; and, on the other hand, to change or cope with the environment. Health is, therefore, seen as a resource for everyday life, not the objective of living; it is a positive concept emphasizing social and personal resources, as well as physical capacities.” Nurses’ attitudes toward health and illness should account for the total person, as well as the environment in which the person lives. People free of disease are not equally healthy. Views of health have broadened to include mental, social, and spiritual well-being, as well as a focus on health at family and community levels. Conditions of life, rather than pathological states, are what determine health. DIF: Knowledge REF: 2 OBJ: Discuss ways that definitions of health have been conceptualized. TOP: Evaluate MSC: CPNRE: Foundations of Practice 4. What priority strategy for health promotion in Canada is optional but seen as important to
incorporate in nursing education curricula? a. Knowledge of disease prevention. b. Strategies for health promotion. c. Policy advocacy. d. Concepts of determinants of health. ANS: C
Increasingly, policy advocacy is incorporated into nursing role statements and nursing education curricula. Nurses should think about policies that have contributed to health problems, policies that would help to alleviate health problems, and how nursing champions public policies. Disease prevention is an integral part of nursing curricula. Health promotion is a fundamental part of nursing curricula. B.C M
N R I G U S N T
O
DIF: Understand REF: 11 | 12 OBJ: Analyze how the nature and scope of nursing practice are influenced by different conceptualizations of health and health determinants. TOP: Planning MSC: CPNRE: Foundations of Practice 5. Which of the following is a prerequisite for health, as identified by the Ottawa Charter for
Health Promotion? a. Education. b. Social support. c. Self-esteem. d. Physical environment. ANS: A
Education is one of the nine prerequisites for health that were identified in the Ottawa Charter for Health Promotion. Lack of social support and low self-esteem were identified as a psychosocial risk factors by Labonte (1993). Dangerous physical environments were identified as socioenvironmental risk factors by Labonte (1993). DIF: Understand REF: 4 OBJ: Discuss contributions of the following Canadian publications to conceptualizations of health and health determinants: Lalonde Report, Ottawa Charter, Epp Report, Strategies for Population Health, Jakarta Declaration, Bangkok Charter, Toronto Charter. TOP: Planning MSC: CPNRE: Foundations of Practice
Canadian Fundamentals of Nursing 6th Edition Potter Test Bank 6. The determinant of health with the greatest effect on the health of Canadians is which of the
following? a. Education. b. Health services. c. Social support networks. d. Income and social status. ANS: D
Income, income distribution, and social status are the determinants of health that influence most other determinants. Some investigators suggest that literacy and education are important influences on health status because they affect many other health determinants. Approximately 25% of a population’s health status is attributed to the quality of its health care services. Social support affects health, health behaviours, and health care utilization but is not the most influential determinant of health. DIF: Understand REF: 6 OBJ: Discuss key health determinants and their interrelationships and how they influence health. TOP: Planning MSC: CPNRE: Foundations of Practice 7. A paraplegic patient in the hospital for an electrolyte imbalance is receiving care at which
prevention level? a. Primary prevention. b. Secondary prevention. c. Tertiary prevention. d. Health promotion. ANS: B
The secondary prevention levNelUfR ocu onTeBa. rlyCde tection of disease once pathogenesis has SIsesNG OM occurred, so that prompt treatment can be initiated to halt disease and limit disability. The primary prevention level focuses on health promotion, specific protection measures such as immunizations, and the reduction of risk factors such as smoking. The tertiary prevention level focuses on minimizing residual disability. DIF: Apply REF: 11 OBJ: Contrast distinguishing features of health promotion and disease prevention. TOP: Implementation MSC: CPNRE: Foundations of Practice 8. The nurse incorporates levels of prevention on the basis of patient needs and the type of
nursing care provided. Which of the following is an example of tertiary level preventive caregiving? a. Teaching a patient how to irrigate a new temporary colostomy. b. Providing a lesson on hygiene for an elementary school class. c. Informing a patient that immunizations for her infant are available through the health department. d. Arranging for a hospice nurse to visit with the family of a patient with cancer. ANS: D
Canadian Fundamentals of Nursing 6th Edition Potter Test Bank Tertiary prevention is provided when a defect or disability is permanent and irreversible. At this level, the hospice nurse aims to help the patient and his or her family to achieve a high level of function, despite the limitations caused by the patient’s illness. Teaching a patient how to irrigate a new colostomy is an example of secondary prevention. If the colostomy is to be permanent, care may later move to the tertiary level of prevention. Providing a lesson on hygiene for an elementary school class and informing a patient about available immunizations are examples of primary prevention. DIF: Apply REF: 11 TOP: Implementation
OBJ: Discuss the three levels of disease prevention. MSC: CPNRE: Foundations of Practice
9. The nurse is working on a committee to evaluate the need for increasing the levels of fluoride
in the drinking water of the community. In doing so, the nurse is fostering which concept? a. Anticipatory prevention. b. Primary prevention. c. Secondary prevention. d. Tertiary prevention. ANS: B
Fluoridation of municipal drinking water and fortification of homogenized milk with vitamin D are examples of primary prevention strategies. With active strategies of health promotion, individuals are motivated to adopt specific health programs such as weight reduction and smoking cessation programs. Anticipatory prevention is not a known concept. Secondary prevention promotes early detection of disease (e.g., screening). Tertiary prevention activities are initiated in the convalescence phase of disease. DIF: Apply REF: 11 OBJ: Di scuss the three levels of disease prevention. NURSINGMTSC: B.CPC M Foundations of Practice TOP: Implementation ONRE: 10. The nurse is working in a clinic that is designed to provide health education and
immunizations. As such, this clinic focuses on which type of prevention? a. Primary prevention. b. Secondary prevention. c. Tertiary prevention. d. Diagnosis and prompt intervention. ANS: A
Primary prevention precedes disease or dysfunction and is applied to people considered physically and emotionally healthy. Health promotion includes health education programs, immunizations, and physical and nutritional fitness activities. Secondary prevention focuses on individuals who are experiencing health problems or illnesses and who are at risk for developing complications or worsening conditions; activities are directed at diagnosis and prompt intervention. Tertiary prevention is provided when a defect or disability is permanent and irreversible. It involves minimizing the effects of long-term disease or disability through interventions directed at preventing complications and deterioration. DIF: Understand REF: 11 TOP: Implementation
OBJ: Discuss the three levels of disease prevention. MSC: CPNRE: Foundations of Practice
Canadian Fundamentals of Nursing 6th Edition Potter Test Bank 11. The patient is admitted to the emergency department of the local hospital from home with
reports of chest discomfort and shortness of breath. She is administered oxygen and breathing treatments, laboratory tests and blood gas measurements are performed, and electrocardiography is conducted. What level of preventive care is this patient receiving? a. Primary prevention. b. Secondary prevention. c. Tertiary prevention. d. Health promotion. ANS: B
Secondary prevention focuses on individuals who are experiencing health problems or illnesses and who are at risk for developing complications or worsening conditions. Activities are directed at diagnosis and prompt intervention. Primary prevention precedes disease or dysfunction and is applied to people considered physically and emotionally healthy. Health promotion includes health education programs, immunizations, and physical and nutritional fitness activities. Tertiary prevention is provided when a defect or disability is permanent and irreversible. It involves minimizing the effects of long-term disease or disability through interventions directed at preventing complications and deterioration. DIF: Apply REF: 11 TOP: Implementation
OBJ: Discuss the three levels of disease prevention. MSC: CPNRE: Foundations of Practice
12. A patient is admitted to a rehabilitation facility after a stroke. The patient has right-sided
paralysis and is unable to speak. The patient will be receiving physiotherapy and speech therapy. What are these examples of? a. Primary prevention. b. Secondary prevention. NURSINGTB.COM c. Tertiary prevention. d. Health promotion. ANS: C
Tertiary prevention is provided when a defect or disability is permanent and irreversible. It involves minimizing the effects of long-term disease or disability through interventions directed at preventing complications and deterioration. Secondary prevention focuses on individuals who are experiencing health problems or illnesses and who are at risk for developing complications or worsening conditions. Activities are directed at diagnosis and prompt intervention. Primary prevention precedes disease or dysfunction and is applied to people considered physically and emotionally healthy. Health promotion includes health education programs, immunizations, and physical and nutritional fitness activities. DIF: Apply REF: 11 TOP: Implementation
OBJ: Discuss the three levels of disease prevention. MSC: CPNRE: Foundations of Practice
13. Risk factors can be placed in the following interrelated categories: genetic and physiological
factors, age, physical environment, and lifestyle. The presence of any of these risk factors means which of the following? a. A person with the risk factor will get the disease. b. The chances of getting the disease are increased. c. The disease is guaranteed not to develop if the risk factor is controlled. d. Risk modification will have no effect on disease prevention.
Canadian Fundamentals of Nursing 6th Edition Potter Test Bank ANS: B
The presence of risk factors does not mean that a disease will develop, but risk factors increase the chances that the individual will experience a particular disease or dysfunction. Control of risk factors does not guarantee that a disease will not develop. However, risk factor identification assists patients in visualizing areas in life that can be modified or even eliminated to promote wellness and prevent illness. DIF: Knowledge REF: 3 OBJ: Identify factors that have led to each approach to health. MSC: CPNRE: Foundations of Practice
TOP: Assessment
14. Since the early 1990s, which group has had the highest amount of absenteeism of all workers
in Canada? a. “White collar sector” workers. b. Nurses. c. Workers in the trades. d. Transport and equipment operators. ANS: B
There is considerable concern regarding negative workplace conditions in the health care sector. Nurses have had the highest or second-highest rate of absenteeism of all workers in Canada since the early 1990s. Rates of absenteeism for the “white collar sector,” for workers in the trades, and for transport and equipment operators are not available. DIF: Analyze REF: 7 OBJ: Analyze how the nature and scope of nursing practice are influenced by different conceptualizations of health and health determinants. TOP: Planning MSC: CPNRE: Foundations ofNPU raR ctice SINGTB.COM 15. Which of the following is a true statement about nutrition in Canada, according to the
research? a. Canadians have increased their total fat and salt consumption. b. Canadians report that their children eat the recommended daily number of fruits and vegetables. c. Fifty percent of children aged 2 to 17 years were overweight or obese. d. The 2004 Canadian Community Health Survey (CCHS) revealed that 40% of adult Canadians were obese (body mass index of 30 or more) and 50% were overweight. ANS: A
One quarter of Canadians overall, and one third of teenagers aged 14 to 18 years, reported eating at a fast-food outlet the previous day; such foods are high in fats and salts. Of children aged 4 to 8, 70% ate fewer than the minimum servings of fruits and vegetables daily. Of children aged 2 to 17 years, 26% were overweight or obese, not 50%. The 2004 Canadian Community Health Survey (CCHS) revealed that 23% of adult Canadians were obese (body mass index of 30 or more), not 40%, and 36% were overweight, not 50%. DIF: Apply REF: 8 | 9 OBJ: Discuss key health determinants and their interrelationships and how they influence health. TOP: Planning MSC: CPNRE: Foundations of Practice
Canadian Fundamentals of Nursing 6th Edition Potter Test Bank 16. One of the five health promotion strategies, as identified by the Ottawa Charter for Health
Promotion, is which of the following? a. Creating supportive environments. b. Strengthening educational opportunities. c. Developing a medical approach. d. Minimizing stressful situations. ANS: A
“Creating supportive environments” is one of the five broad health promotion strategies identified by the Ottawa Charter for Health Promotion. The other strategies are building healthy public policy, creating supportive environments, strengthening community action, developing personal skills, and reorienting health services. “Strengthening educational opportunities,” “Developing a medical approach,” and “Minimizing stressful situations” are not among the five strategies. DIF: Apply REF: 11 | 12 OBJ: Discuss contributions of the following Canadian publications to conceptualizations of health and health determinants: Lalonde Report, Ottawa Charter, Epp Report, Strategies for Population Health, Jakarta Declaration, Bangkok Charter, Toronto Charter. TOP: Planning MSC: CPNRE: Foundations of Practice 17. Which of the following is an example of tertiary prevention? a. Reduction of risk factors, such as smoking. b. Breast self-examination and testicular self-examination. c. Cardiac rehabilitation programs. d. Blood pressure screening to detect hypertension. ANS: C
NURSINGTB.COM
Tertiary prevention activities are provided in the convalescence stage of disease and are directed toward minimizing residual disability and helping people live productively with limitations. An example is a cardiac rehabilitation program after a myocardial infarction. Breast self-examination and testicular self-examination are examples of secondary prevention, as is blood pressure screening to detect hypertension. Reducing risk factors, such as smoking, is an example of primary prevention. DIF: Understand REF: 11 TOP: Implementation
OBJ: Describe the three levels of disease prevention. MSC: CPNRE: Foundations of Practice
18. When the nurse is educating an adult patient about health promotion activities, which of the
following is the most important internal patient factor for the nurse to consider? a. Emotional wellness. b. Developmental stage. c. Professed spirituality. d. Levels of education and literacy. ANS: D
Canadian Fundamentals of Nursing 6th Edition Potter Test Bank Levels of education and literacy are important influences to consider when the nurse is educating an adult patient concerning health promotion activities. Literacy can influence health both directly (e.g., medication use, safety practices) and indirectly through use of services, lifestyles, income, work environments, and stress levels. Spirituality is reflected in how a person lives his or her life, including the values and beliefs exercised, the relationships established with family and friends, and the ability to find hope and meaning in life; however, it is not the most important factor to consider. The nurse must consider the patient’s level of growth and development when using the patient’s health beliefs and practices as a basis for planning care, but this is not the most important patient factor to consider. In this case, the patient is at the adult developmental stage. The patient’s emotional wellness—degree of stress, depression, or fear, for example—can influence health beliefs and practices. The manner in which a person handles stress throughout each phase of life will influence the way he or she reacts to illness. However, this is not the best available option. DIF: Analyze REF: 7 OBJ: Discuss key health determinants and their interrelationships and how they influence health. TOP: Implementation MSC: CPNRE: Foundations of Practice 19. When discussing the effect of a known risk factor on a patient’s health, what would the nurse
say? a. “It doesn’t mean that you’ll get the disease, just that the odds are greater for you.” b. “Now that you know the possibility is there, you can take steps to prevent it.” c. “This risk factor can be managed by making a change to your lifestyle.” d. “You’re lucky because you have the benefit of being able to do something about it.” ANS: A
NURSINGTB.COM
The presence of risk factors does not mean that a disease will develop, but risk factors increase the chances that the individual will experience a particular disease or dysfunction. While the statement, “Now that you know the possibility is there, you can take steps to prevent it” is not incorrect; it does not address the impact of the risk factor on the patient’s health. It is not always true that a risk factor can be managed by making lifestyle changes. The strategy of telling the patient that he or she is lucky and has the benefit of being able to do something about a risk factor minimizes the patient’s concern, and does not address the effect of the risk factor on the patient’s health. DIF: Apply REF: 3 OBJ: Describe key characteristics of medical, behavioural, and socioenvironmental approaches to health. TOP: Implementation MSC: CPNRE: Foundations of Practice 20. Classifications of health conceptualizations occur in which following way? a. Health as stability and actualization. b. Health as individual and wellness. c. Health as universal and being free from illness. d. Health as empowerment and absence of disease. e. Health and wellness. ANS: A
Health can be conceptualized (Pender, 2006) in three ways: health as stability, as actualization, and health as both of these together.
Canadian Fundamentals of Nursing 6th Edition Potter Test Bank DIF: Understand REF: 2 OBJ: Discuss ways that definitions of health have been conceptualized. TOP: Assessment MSC: CPNRE: Foundations of Practice
NURSINGTB.COM
Canadian Fundamentals of Nursing 6th Edition Potter Test Bank
Chapter 02: The Canadian Health Care Delivery System Potter et al: Canadian Fundamentals of Nursing, 6th Edition MULTIPLE CHOICE 1. What is the fastest growing component of the health care system? a. Primary care. b. Respite care. c. Home care. d. Palliative care. ANS: C
Canadian health care is shifting from an institution-based system to one in which community care is playing a greater role. Home care is one of the fastest growing components of the health care system, partly because patients are sent home from hospital sooner than they used to be. Primary care, respite care, and palliative care are not the fastest growing components of the health care system. DIF: Apply REF: 27 OBJ: Describe five levels of health care and associated types of services. TOP: Assessment MSC: CPNRE: Foundations of Practice 2. Which of the following is a contributing factor to the shortage of nurses in Canada? a. Increased birth rate, which creates a greater demand for nursing services. b. New technology, which replaces nurses at the bedside. c. Fewer people wishing to practise as nurses because health trends are focusing on
NU RSs.INGTB.C OM natural and alternative ther apie d. Retirement of a large percentage of nurses. ANS: D
The nursing workforce is currently challenged by the aging of workers, high retirement rates, ethical problems in international recruitment, and lack of full-time positions. The average age of RNs in Canada is 44.6 years; for LPNs, it is 41, and for RPNs, it is 46. The aging population is creating the greater demand for nursing services. Neither the replacement of nurses at the bedside by new technology nor the focus of health trends on natural and alternative therapies is a factor identified as contributing to the shortage of nurses. DIF: Apply REF: 19, Box 2-1 OBJ: Discuss principal factors influencing health care reform and the current health care delivery system. TOP: Planning MSC: CPNRE: Foundations of Practice 3. A nurse working with a group of Indigenous parents refers to Treaty 6, which includes a
provision for health care services and is often referred to as which of the following? a. The Truth and Reconciliation Act. b. The “medicine chest” clause. c. The Indigenous band agreements. d. The land and relationship understanding. ANS: B
Canadian Fundamentals of Nursing 6th Edition Potter Test Bank Treaty 6 was often referred to as the “medicine chest” clause, as it included a provision for health care services to be provided to Indigenous communities. The Indian Act, the Indian band agreements (signed with the British government before Confederation), and relationship agreements do not refer to Treaty 6. DIF: Understand REF: 20 OBJ: Identify initiatives related to enhancing quality of the Canadian health care delivery system. TOP: Planning MSC: CPNRE: Foundations of Practice 4. Which of the following is an example of a true health promotion service provided by a nurse? a. An immunization clinic. b. A diabetic support group. c. A prenatal nutrition class. d. A smoking cessation clinic. ANS: C
Community clinics offer prenatal nutrition classes that promote the health of the woman, fetus, and infant. Building healthy public policy, creating supportive environments, strengthening community action, developing personal skills, and reorienting health services are part of health promotion service. An immunization clinic is an example of disease and injury prevention. A diabetic support group may be an example of a rehabilitation service, to help patients adapt to a change in lifestyle. A smoking cessation clinic may be offered as part of a rehabilitation service or as a disease and injury prevention intervention. DIF: Apply REF: 28 OBJ: Describe five levels of health care and associated types of services. TOP: Implementation MSC: CPNRE: Foundations of Practice
NURSINGTB.COM
5. When conducting a health care system class for immigrants to Canada, the nurse informs them
that the federal government is responsible for which of the following? a. Providing health care insurance plans. b. Managing and planning insurable health services. c. Delivering health services to targeted groups. d. Providing long-term care services. ANS: C
The federal government is charged with delivering, or co-delivering, health care services for targeted groups. Health care insurance plans, managing and planning insurable health services, and providing long-term care services are the jurisdiction of the provinces or territories. DIF: Apply REF: 22 OBJ: Identify and define the principles of the Canada Health Act and significant legislation related to the Canadian health care system. TOP: Implementation MSC: CPNRE: Foundations of Practice 6. The nurse initiated a support group for adolescent parents and is teaching them about
chlorinated water and child immunizations, which are all examples of which of the following? a. Health promotion. b. Disease and injury prevention. c. Supportive care.
Canadian Fundamentals of Nursing 6th Edition Potter Test Bank d. Rehabilitation. ANS: B
Disease and injury prevention includes illness prevention (chlorinated water, immunizations) and support groups. Health promotion services include antismoking services, advocacy for healthy public policy, and provision of wellness services. Supportive care describes services provided over a prolonged period to people who are disabled, who have never been able to function independently, or who have a terminal disease. Rehabilitation, such as physiotherapy, involves restoring optimal health. DIF: Apply REF: 28 OBJ: Describe five levels of health care and associated types of services. TOP: Implementation MSC: CPNRE: Foundations of Practice 7. Primary care is best described as which of the following? a. Early detection and routine care, as well as prevention. b. Provision of a specialized medical service. c. Treatment of all patients with a minimum level of health insurance. d. Provision of medical services in a patient’s home. ANS: A
Primary care is the first contact of a patient with the health care system that leads to a decision regarding a course of action to resolve any actual or potential health problem. The focus is on early detection and routine care, with emphasis on education to prevent recurrences. Provision of a specialized medical service by a specialist or through referral is an example of secondary care. No patient is refused treatment on the basis of level of insurance. Medical services provided in the patient’s home, although not very common, could be categorized under primary care but would not bN esU tR deS scIriN beGpTriB m. arCyOcaMre. DIF: Understand REF: 28 OBJ: Identify various settings and models of care delivery in the Canadian health care delivery system. TOP: Assessment MSC: CPNRE: Foundations of Practice 8. A patient comes to the ambulatory care clinic for management of a chronic condition and asks
the nurse for an explanation of the Medicare system. The nurse’s response is based on the knowledge that it is which of the following? a. Canada’s national health insurance system. b. A fee-for-service insurance plan. c. A plan in which monies from provincial taxes are used for nursing homes. d. A social insurance program for low-income earners. ANS: A
Medicare is a key element of Canada’s social safety net. It is Canada’s national health insurance system, in which taxes are used to finance medically necessary services for all citizens, thus providing “free” health care to all. Medicare is not a fee-for-service insurance plan, a plan in which monies from provincial taxes are used for nursing homes, or a social insurance program for low-income earners. DIF: Understand REF: 19 OBJ: Discuss the evolution of Canada's social safety net and Medicare. TOP: Implementation MSC: CPNRE: Foundations of Practice
Canadian Fundamentals of Nursing 6th Edition Potter Test Bank 9. In teaching a group of older persons about health care spending, a nurse tells them that about
14% of the Canadian population is within their age group and that this group accounts for what percentage of health care spending? a. 25%. b. 35%. c. 45%. d. 55%. ANS: C
Older persons account for 45% of health care spending in Canada, not 25%, not 35%, and not 55%. DIF: Apply REF: 22 OBJ: Discuss principal factors influencing health care reform and the current health care delivery system. TOP: Implementation MSC: CPNRE: Foundations of Practice 10. To whom do the principles of the Canada Health Act of 1984 apply? a. Members of the Canadian Armed Forces. b. Insured residents of Canada. c. Inmates of federal penitentiaries. d. Royal Canadian Mounted Police (RCMP). ANS: B
The principles of the Canada Health Act apply to all insured residents of Canada (i.e., eligible residents) but exclude members of the Canadian Armed Forces, Royal Canadian Mounted Police (RCMP), eligible veterans, inmates of federal penitentiaries, and some refugee claimants. N R I G B.C M
U S N T
O
DIF: Understand REF: 20 OBJ: Identify and define the principles of the Canada Health Act and significant legislation related to the Canadian health care system. TOP: Assessment MSC: CPNRE: Foundations of Practice 11. When a mother brings her child to be seen in a nurse-managed setting, over concerns that her
child’s rash could be measles, what type of care is rendered? a. Respite care. b. Primary care. c. Supportive care. d. Secondary care. ANS: B
Primary care focuses on early detection and routine care and can be offered in nurse-managed clinics. Respite care is a component of supportive care. Supportive care describes services provided to disabled and terminally ill patients over a prolonged time period. Secondary care is the provision of specialized medical services by a physician specialist or through referral from a primary care provider. DIF: Understand REF: 28 OBJ: Describe five levels of health care and associated types of services. TOP: Assessment MSC: CPNRE: Foundations of Practice
Canadian Fundamentals of Nursing 6th Edition Potter Test Bank 12. Which of the following is one of the four pillars of primary health care, as described by the
National Primary Health Care Awareness Strategy (2006)? a. Teams. b. Universality. c. Health promotion. d. Immediate access to primary care. ANS: A
Teams are one of the four pillars, along with access, information, and healthy living. The four pillars do not include universality, health promotion, or immediate access to primary health care. DIF: Understand REF: 25, Box 2-3 OBJ: Identify various settings and models of care delivery in the Canadian health care delivery system. TOP: Planning MSC: CPNRE: Foundations of Practice 13. Mr. Jones is 72 years old. He is ambulatory and is to be discharged from hospital after a total
hip replacement, with requirements for daily physiotherapy and respiratory therapy. What is the most appropriate facility referral for the nurse to make? a. A rehabilitation centre. b. An assisted-living facility. c. A community health clinic. d. A home care agency. ANS: D
A home care agency provides health services to individuals and families in their homes. Home care was created to provide individualized care for people after hospital discharge but has increasingly included patientN s in ag. esCfro URa SraInge NGofTB OMm very young to very old; those with mental, physical, or developmental challenges; and those requiring recovery to end-stage care. A rehabilitation centre is not required, as Mr. Jones is ambulatory. Assisted-living facilities are community-based residential facilities in which adults live in their own apartments and are provided with a range of support services such as meals, social and recreational programs, and laundry A community health clinic is the first point of contact, offering a range of primary services that emphasize prevention, health promotion, health education, community development, and partnerships to develop a healthy local community. DIF: Apply REF: 26 | 27 OBJ: Identify various settings and models of care delivery in the Canadian health care delivery system. TOP: Implementation MSC: CPNRE: Foundations of Practice 14. What is an appropriate referral for an older patient who requires some assistance with daily
activities within a partially protective environment? a. Respite care. b. Rehabilitative care. c. Assisted living. d. Extended care. ANS: C
Canadian Fundamentals of Nursing 6th Edition Potter Test Bank Such a patient requires assisted living. Assisted living allows adults to receive a range of support services, including personalized assistance in achieving a level of independence. Personal assistance services are “designed to promote maximum dignity and independence,” including meal preparation, personal hygiene practice, mobility, and socialization. Respite care is a service that provides short-term relief for family caregivers or persons providing home care to the ill or disabled. Rehabilitative care provides therapy and training, with the goal of decreasing the patient’s dependence on care. An extended, or long-term, care facility provides intermediate medical, nursing, or custodial care for patients recovering from acute or chronic illness or disabilities. DIF: Apply REF: 26 | 27 OBJ: Identify various settings and models of care delivery in the Canadian health care delivery system. TOP: Implementation MSC: CPNRE: Foundations of Practice 15. The Canadian Patient Safety Institute (CPSI) provides leadership in building and advancing a
safer health care system. According to the CPSI, about two thirds of preventable incidents that occur during hospitalization are related to what? a. Procedures. b. Care or medication. c. Patient accidents. d. Infection control. ANS: B
According to the CPSI, about two thirds of preventable events are either care related or medication related. About one third of the events are related to procedures or patient accidents. Infections fall under the category of care.
NUR I G B.C M S N T O
DIF: Remember REF: 30 OBJ: Discuss principal factors influencing health care reform and the current health care delivery system. TOP: Planning MSC: CPNRE: Foundations of Practice 16. “Residents temporarily absent from their home province are to have their incurred services
paid for by the home province, in the same amount that would have been paid by the home province.” This statement reflects which principle of the Canada Health Act (1984)? a. Portability. b. Universality. c. Public administration. d. Accessibility. ANS: A
This statement describes the principle of portability. The principle of universality means that 100% of the insured persons of a province are covered by the health care plan. The principle of public administration means that the health care plan must be administered and operated on a nonprofit basis by a public authority, responsible to the provincial government and subject to audits of its accounts and financing transactions. The principle of accessibility means that the health care plan of a province must provide for (1) insured health services and reasonable access by insured persons; (2) reasonable compensation to physicians and dentists for all insured services rendered; and (3) payments to hospitals in regard to the cost of insured health services.
Canadian Fundamentals of Nursing 6th Edition Potter Test Bank DIF: Understand REF: 21, Table 2-1 OBJ: Identify and define the principles of the Canada Health Act and significant legislation related to the Canadian health care system. TOP: Planning MSC: CPNRE: Foundations of Practice 17. A Canadian is explaining to a non-Canadian friend the benefits of the Canadian health care
system. The Canadian discusses being able to access surgical or medical hospital-based care with no out-of-pocket charges; mentions that if he or she must travel for work to another province within Canada, he or she is still covered; and explains that it is easy to access health care services and the care received is respectful and embraces diversity. Which of the following principles of the Canada Health Act of 1984 is the Canadian describing? a. Universality, public administration, and protection. b. Comprehensiveness, public administration, and portability. c. Universality, accessibility, and protection. d. Accessibility, portability, and comprehensiveness. ANS: D
The principles described by the Canadian include universality, portability, accessibility, and comprehensiveness. The ability to access surgical or medical hospital-based care with no out-of-pocket charges refers to comprehensiveness; the ability to reasonably access health care services refers to accessibility; and respectful care that embraces diversity refers to universality. Portability refers to the ability to access health care services in another province. Public administration is another principle of the Canadian Health Act but is not described by the Canadian. Protection is not a principle of the Canadian Health Act. DIF: Apply REF: 21 OBJ: Identify and define the principles of the Canada Health Act and significant legislation related to the Canadian health care systemN. URSINGTT OB P:.C AsO seM ssment MSC: CPNRE: Foundations of Practice
Canadian Fundamentals of Nursing 6th Edition Potter Test Bank
Chapter 03: The Development of Nursing in Canada Potter et al: Canadian Fundamentals of Nursing, 6th Edition MULTIPLE CHOICE 1. Which of the following was the main health reason why health care facilities in the British
colonies were increasingly needed during the eighteenth-century British regime? a. Scurvy. b. Bubonic plague. c. Increased birth rate. d. Infectious diseases. ANS: D
Infectious diseases carried by immigrants and travellers spread rapidly in the British colonies, creating a need for more health care facilities. Neither scurvy nor bubonic plague was the main health reason for increased health care facilities. The increasing birth rate had nothing to do with an increased need for health care facilities. DIF: Understand REF: 35 | 36 OBJ: Discuss the historical development of nursing practice in Canada. TOP: Assessment MSC: CPNRE: Foundations of Practice 2. What was the main driving force for the modernization of nursing? a. Globalization. b. The Crimean War. c. Missionary nursing. N R I G B.C M d. The women’s suffrage movU emS ent.N
T
O
ANS: A
The history of modern nursing, at its heart, is a story of globalization. Nurses often travelled across geographic and national boundaries to bring nursing service and training to communities in need. Although Florence Nightingale reduced morbidity and mortality rates by applying modern principles of cleanliness and comfort during the Crimean War, this war was not the main driving force for the modernization of nursing. Missionary nursing was the result of a series of sociopolitical factors converging at the same time but was not the main driving force for the modernization of nursing. The women’s suffrage movement was also not the main driving force for the modernization of nursing, although nursing did become an instrument of women’s emancipation from restrictions of middle-class women. DIF: Understand REF: 37 OBJ: Discuss the historical development of nursing practice in Canada. TOP: Planning MSC: CPNRE: Foundations of Practice 3. Where did the first hospital diploma school in Canada open? a. Manitoba. b. Ontario. c. Quebec. d. Newfoundland. ANS: B
Canadian Fundamentals of Nursing 6th Edition Potter Test Bank Ontario opened the first hospital diploma school in Canada, the St. Catharines Training School, in 1874, at St. Catharines General and Marine Hospital. Manitoba, Quebec, and Newfoundland opened theirs later. DIF: Remember REF: 39 OBJ: Discuss the historical development of nursing education in Canada. TOP: Planning MSC: CPNRE: Foundations of Practice 4. What prompted the formation of the Victorian Order of Nurses (VON) in 1898? a. Flu epidemic. b. Measles outbreak. c. Labour and birth difficulties. d. War casualties. ANS: C
Lady Ishbel Aberdeen, wife of the governor general of Canada, conceived of the idea of establishing the VON after learning the plight of women in western Canada who had to give birth in remote locations with no assistance. A flu epidemic, a measles outbreak, and war casualties were not the health care issue responsible for the creation of the VON. DIF: Understand REF: 39 OBJ: Discuss the historical development of professional nursing. TOP: Planning MSC: CPNRE: Foundations of Practice 5. Which was the first province in Canada to gain legislation related to nursing registration? a. Ontario. b. Nova Scotia. c. New Brunswick. NURSINGTB.COM d. British Columbia. ANS: B
Nova Scotia was the first province in Canada to pass legislation related to nursing registration, with the passing of a voluntary registration act in 1910. Ontario, New Brunswick, and British Columbia passed legislation related to nursing registration later. DIF: Understand REF: 40 OBJ: Discuss the historical development of professional nursing. TOP: Planning MSC: CPNRE: Foundations of Practice 6. What is a fundamental and guiding principle of French-Canadian hospitals that survived into
the twentieth century? a. Universal precautions are fundamental in preventing nosocomial infections. b. Nursing is primarily focused on the tertiary level of care. c. Nurses need to recognize the effect of their health, values, and beliefs on practice. d. Care is accessible to all, regardless of their background, status in life, or ability to pay. ANS: D
Canadian Fundamentals of Nursing 6th Edition Potter Test Bank A fundamental and guiding principle of the French-Canadian hospitals that survived largely intact into the twentieth century was that care is available to all people, regardless of background, status in life, or ability to pay. This continues to be a principle for which nurses, through their professional organizations, have argued for determinedly in national debates on the nature and continuing direction of Canada’s national health care insurance program. The statement “Universal precautions are fundamental in preventing nosocomial infections” was not a guiding principle of French-Canadian hospitals. The statements “Nursing is primarily focused on the tertiary level of care” and “Nurses need to recognize the effect of their health, values, and beliefs on practice” do not reflect a contribution from the past to current nursing beliefs and practices. DIF: Apply REF: 45 OBJ: Discuss the historical development of nursing practice in Canada. TOP: Implementation MSC: CPNRE: Foundations of Practice 7. What is the primary motivator of nursing? a. Money. b. Altruism. c. Self-efficacy. d. A curative model of care. ANS: B
Nursing is practical and theoretical, is motivated by altruism, and is based on professional and ethical standards. Money, self-efficacy, and the curative model of care are not the primary motivators of nursing. DIF: Apply REF: 45 OBJ: Discuss the historical deN veU loR pm rs. inC g pOraMctice in Canada. SeIntNoGf nTuB TOP: Assessment MSC: CPNRE: Foundations of Practice 8. The proportion of male nurses in Canada in 2012 was
, in comparison with
in the year 2000. a. 2.1%; 0.9%. b. 6.4%; 4.7%. c. 12.4%; 8.6%. d. 18.2%; 12.4%. ANS: B
According to the most recent statistics, the proportion of male nurses as only 6.4% of the total nurse population in 2012, a slight increase over the 4.7% it represented in 2000. DIF: Understand REF: 39 OBJ: Discuss the historical development of nursing practice in Canada. TOP: Assessment MSC: CPNRE: Foundations of Practice 9. The Canadian Nurses Association (CNA) has made available to all nurses across Canada a
nursing portal database; what is it called? a. The Nurse. b. NurseOne. c. Nursing Initiatives. d. Evidence-informed Nursing.
Canadian Fundamentals of Nursing 6th Edition Potter Test Bank ANS: B
NurseOne is the CNA’s nursing portal database and is available to all nurses across Canada. The Nurse, Nursing Initiatives, and Evidence-based Nursing are not names by which the CNA’s nursing portal database is known. DIF: Apply REF: 43 OBJ: Discuss the historical development of nursing practice in Canada. TOP: Planning MSC: CPNRE: Foundations of Practice 10. Who was the most influential woman in the advancement of nursing in Canada? a. Marie Rollet Hébert. b. Florence Nightingale. c. Mary Agnes Snively. d. Jeanne Mance. ANS: D
Jeanne Mance (1606–1673) came to Ville Marie (New France) in 1642. As well as founding and managing Hôtel-Dieu, Mance assisted Maisonneuve in running the colony as confidant, advisor, and accountant. She is hailed as a founder of the city of Montreal. Today, the CNA awards its highest honour in the name of this courageous pioneer. Marie Rollet Hébert (1580– 1649) was the first laywoman to provide nursing care in New France in 1617. Florence Nightingale (1820–1910), who is considered the founder of modern nursing, was a nineteenth-century British nurse. Mary Agnes Snively (1847–1933) was a teacher before becoming a nurse. Upon graduation from the school of nursing at Bellevue Hospital in New York, she became superintendent of nurses at Toronto General Hospital. Toronto General Hospital subsequently became the largest school of nursing in Canada and a model to others. DIF: Remember REF: 36N ,B UoRxS3-I1 NGTB.COM OBJ: Discuss the historical development of professional nursing. TOP: Planning MSC: CPNRE: Foundations of Practice 11. What is the primary purpose of licensure laws for the nursing profession? a. To protect the public against unqualified and incompetent practitioners. b. To enhance the quality of nursing care and improve Canadians’ health outcomes. c. To ensure that nurses demonstrate knowledge and skills in a variety of professional
roles. d. To provide an opportunity for practitioners to validate their expertise in a specialty. ANS: A
Licensure laws are designed to protect the public against unqualified and incompetent practitioners. Because constitutional responsibility for education and health falls under the purview of the provinces and territories, each has a nursing practice act to regulate the licensure and practice of nursing. The other statements—“To enhance the quality of nursing care and improve Canadians’ health outcomes”; “To ensure that nurses demonstrate knowledge and skills in a variety of professional roles”; and “To provide an opportunity for practitioners to validate their expertise in a specialty”—do not reflect the primary purpose of licensure laws. DIF: Apply REF: 40 | 41 OBJ: Discuss the historical development of professional nursing. TOP: Implementation MSC: CPNRE: Foundations of Practice
Canadian Fundamentals of Nursing 6th Edition Potter Test Bank
12. Where did the traditions of good nursing practice that anchor nursing in Canada originate? a. England. b. United States. c. Germany. d. New France. ANS: D
Nursing in Canada is rooted in the traditions of good nursing practice that developed in New France, not in England, the United States, or Germany. DIF: Understand REF: 45 OBJ: Discuss the historical development of professional nursing. TOP: Assessment MSC: CPNRE: Foundations of Practice
NURSINGTB.COM
Canadian Fundamentals of Nursing 6th Edition Potter Test Bank
Chapter 04: Community Health Nursing Practice Potter et al: Canadian Fundamentals of Nursing, 6th Edition MULTIPLE CHOICE 1. How does public health nursing differ from community health nursing? a. Public health nurses focus on individuals and families. b. Public health nurses understand the needs of a population. c. Public health nurses ignore political processes. d. Public health nurses consider the individual as one member of a group. ANS: B
Public health nursing requires understanding the needs of a population. A public health nurse understands factors that influence the political processes used to affect public policy. The primary focus of community health nursing is the care of individuals, families, and groups in the community. By focusing on subpopulations, the community health nurse cares for the community as a whole and considers the individual or family as only one member of a group at risk. DIF: Understand REF: 51 | 52 OBJ: Differentiate between public health nursing and home health nursing. TOP: Assessment MSC: CPNRE: Foundations of Practice 2. When discussing social justice, how does the nurse describe it? a. The focus of social justice is on disease prevention. b. Social justice is used only with people in precarious situations. R talIrNesp GT B.CtyMand fairness. c. Social justice is rooted inNsU ocieS onsibiliO d. Social justice is a process to exercise the ability to enhance control. ANS: C
Social justice is rooted in notions of societal (social) responsibility and fairness (justice). “Social justice is the equitable, or fair, distribution of society's benefits, responsibilities and their consequences. It focuses on the relative position of social advantage of one individual or social group in relation to others in society as well as on the root causes of inequities and what can be done to eliminate them” (CNA, 2010, p. 13). A focus of primary health care is disease prevention, not social justice. Social justice applies to all people, not just those in precarious situations, and it is not to be used as a means to enhance control. DIF: Apply REF: 49 OBJ: Describe the standards, competencies, roles, and activities important for success in community health nursing practice. TOP: Implementation MSC: CPNRE: Foundations of Practice 3. How does the community health nurse differ from the community-based nurse? a. The community health nurse understands the needs of the population. b. The community health nurse focuses on the needs of the individual. c. The community health nurse is the first level of contact in the health care system. d. The community health nurse involves the family in decision making. ANS: A
Canadian Fundamentals of Nursing 6th Edition Potter Test Bank The community health nurse understands the needs of a population or community through experience with individual families in working through their social and health care issues. The community-based nurse (home health nurse) focuses on the needs of the individual or family. Community-based nursing centres function as the first point of contact between members of a community and the health care system. The community-based nurse learns to partner with patients and families so that ultimately the patient and the family become involved in planning, decision making, implementation, and evaluation of health care approaches. DIF: Understand REF: 50 OBJ: Differentiate between public health nursing and home health nursing. TOP: Assessment MSC: CPNRE: Foundations of Practice 4. The type of nursing that focuses on acute and chronic care of individuals and families while
enhancing patient autonomy is known as what kind of nursing? a. Public health nursing b. Community health nursing c. Home health nursing d. Community-focused nursing ANS: C
Home health nursing involves acute and chronic care of individuals and families and enhances their capacity for self-care while promoting autonomy in decision making. Public health nursing focuses on the needs of a population. Community health nursing involves caring for the community as a whole, and the individual or the family is considered only one member of a group at risk. Community-focused nursing entails an understanding of the needs of a population or community. DIF: Understand REF: 53NURSINGTB.COM OBJ: Differentiate between public health nursing and home health nursing. TOP: Assessment MSC: CPNRE: Foundations of Practice 5. The community health nurse is administering flu shots to children at a local playground. In
doing so, the nurse’s focus is on what? a. Preventing individual illness. b. Preventing community outbreak of illness. c. Preventing outbreak of illness in the family. d. The needs of the individual or family. ANS: B
In administering flu shots to members of a subpopulation, the community health nurse attempts to prevent outbreaks of illness in the community as a whole, not necessarily on preventing illness in or meeting the needs of a specific individual or family. DIF: Understand REF: 50 OBJ: Discuss the roles and functions of the community health nurse. TOP: Implementation MSC: CPNRE: Foundations of Practice 6. A group of teenage girls is receiving counselling related to birth control and disease
prevention. Why does the community health nurse provide such counselling? a. Focusing on subpopulations leads to community health. b. Community health nursing focuses on individuals only.
Canadian Fundamentals of Nursing 6th Edition Potter Test Bank c. Community health nursing excludes direct care to subpopulations. d. The focus is on preventing illness and unwanted pregnancy. ANS: A
By focusing on subpopulations, the community health nurse cares for the community as a whole and considers the individual or the family as only one member of a group at risk. In community health nursing, the primary focus is on the health care of individuals, families, and groups in a community. Subpopulations are often a clinical focus. The goal is to protect, promote, or maintain health, not to prevent illness or unwanted pregnancy. DIF: Understand REF: 50 OBJ: Discuss the roles and functions of the community health nurse. TOP: Implementation MSC: CPNRE: Foundations of Practice 7. Community-based nursing care takes place in community settings such as the home or a
clinic. Why is this an ideal setting? a. To exert greater control over individual or family decisions. b. To provide services close to where patients live. c. To isolate patients and prevent the spread of disease. d. To reduce the need for self-care. ANS: B
The ideal is to provide health care services close to where patients live. This lessens the cost of care, as well as the stress associated with the financial burdens of care. The focus is on the needs of the individual or family, not on controlling their decisions or making patients dependent on nurses. The nurse learns to partner with patients and families so that they assume responsibility for their health care decisions. This form of nursing is not concerned with preventing the spread ofNdU isR eaSseI. NGTB.COM DIF: Understand REF: 53 OBJ: Discuss the roles and functions of the community health nurse. TOP: Planning MSC: CPNRE: Foundations of Practice 8. The community-based nurse is caring for a patient who is home bound by arthritis and chronic
lung problems. The patient, however, receives many visitors from the neighbourhood and from former coworkers, as well as frequent phone calls from extended family. When concerned about how the large number of visitors may be fatiguing the patient, what should the nurse do? a. Restrict the number of visitors for the patient’s welfare. b. Voice concerns to the patient and proceed according to the patient’s wishes. c. Allow visitors to come and go freely as they have been. d. Create visiting hours when the patient may see non–family members. ANS: B
The nurse learns to partner with patients and families, so that ultimately the patient and the family assume responsibility for their health care decisions, such as the number and frequency of visits by family and friends. DIF: Apply REF: 53 OBJ: Discuss the roles and functions of the community health nurse. TOP: Implementation MSC: CPNRE: Foundations of Practice
Canadian Fundamentals of Nursing 6th Edition Potter Test Bank 9. The student nurse is trying to determine what type of nurse she wants to be after graduation.
In class, she states that community health nursing is probably not for her because community nursing focuses only on community issues such as preventing epidemics. What would be the instructor’s most appropriate response? a. “Community health nursing focuses on the health care of individuals, families, and groups in a community.” b. “Community health nursing focuses only on the health of a specific subgroup in a community.” c. “Community health nursing requires an advanced nursing degree, and so the student need not worry.” d. “Community health nursing focuses only on maintaining the health of the community.” ANS: A
The primary focus of community health nursing is on the health care of individuals, families, and groups in a community. Community health nursing includes public health nursing, home health (community-based) nursing, and community mental health nursing, as well as a variety of other areas of specialization such as street health and parish nursing. The goals are to preserve, protect, promote, and maintain health. Not all hiring agencies require community health nurses to have an advanced degree. DIF: Apply REF: 49 | 50 OBJ: Describe the standards, competencies, roles, and activities important for success in community health nursing practice. TOP: Implementation MSC: CPNRE: Foundations of Practice 10. Vulnerable populations include patients who are more likely than others to develop health
problems as a result of what?NURSINGTB.C OM a. Pregnancy. b. Nontraditional healing practices. c. Exposure to excessive risk. d. Unlimited access to health care. ANS: C
Vulnerable populations are those patients who are more likely to develop health problems as a result of exposure to excessive risk, who experience barriers when trying to access health care services, or who are dependent on others for care. Pregnancy is not a cause of vulnerability, except for mothers who are adolescents, are addicted to drugs, or are at high risk for other reasons. Nontraditional healing practices are part of the culture of many immigrant and Indigenous populations. Many of these healing practices are effective and complement traditional therapies. DIF: Knowledge REF: 54 OBJ: Explain the characteristics of patients from vulnerable populations that influence a nurse's approach to care. TOP: Assessment MSC: CPNRE: Foundations of Practice 11. The instructor is teaching student nurses about identifying members of vulnerable populations
when the nursing student asks, “Why is it that not all poor people are considered members of vulnerable populations?” What would be the instructor’s best answer? a. “All poor people are members of a vulnerable population.” b. “Poor people are members of a vulnerable population only if they take drugs.”
Canadian Fundamentals of Nursing 6th Edition Potter Test Bank c. “Poor people are members of a vulnerable population only if they are homeless.” d. “Members of vulnerable groups frequently have a combination of risk factors.” ANS: D
Members of vulnerable groups frequently have many risk factors or a combination of risk factors that increase their vulnerability to disease and disability. Individual risk factors are not always overwhelming, depending on the patient’s beliefs and values and sources of social support. DIF: Understand REF: 54 OBJ: Explain the characteristics of patients from vulnerable populations that influence a nurse's approach to care. TOP: Assessment MSC: CPNRE: Foundations of Practice 12. The nurse is making a home visit to a Korean family whose daughter gave birth 6 weeks
earlier. She finds the daughter in bed with a severe headache. The daughter’s father is holding her hand and is pressing different parts of the hand and lower arm. The mother explains that the father is trying to cure the headache by using pressure points. What is the nurse’s best response? a. Tell the father to stop and give the daughter Tylenol. b. Ask the mother or father, or both, to explain the procedure. c. Explain to the father that what he is doing will not work. d. Let the father finish and then give the daughter Tylenol. ANS: B
The nurse should not judge the patient’s/family’s beliefs and values about health. The nurse needs to create a comfortable, nonthreatening environment and to learn as much as possible about the patient’s culture and values that influence his or her health care practices. Tylenol may not be an acceptable alteNrna tive am . Criticizing the family’s beliefs and Cily UR SIfor NGthis TBf. OM practices will only create a barrier to care. DIF: Apply REF: 54, Box 4-5 OBJ: Explain the characteristics of patients from vulnerable populations that influence a nurse's approach to care. TOP: Implementation MSC: CPNRE: Foundations of Practice 13. The nurse is working in a community clinic when a man and woman bring a 12-year-old boy
in, stating that the child fell down a flight of stairs and hurt his arm. The nurse notices several other bruises on the child’s body at varying stages of healing. The boy is placed on the stretcher. When asked how he hurt himself, he states that he does not remember. However, the nurse notices that the boy continuously avoids looking at the man, while the man stares at him constantly. What should the nurse do? a. Ask the boy if the man hurt him. b. Confront the man directly. c. Ask the man and woman to step out of the room. d. Ask the woman if the man hurt the boy. ANS: C
Canadian Fundamentals of Nursing 6th Edition Potter Test Bank The nurse should ask the man and woman to step out. In dealing with patients at risk for or who have suffered abuse, it is important to provide protection and to interview the patient at a time when he or she has privacy and the individual suspected of being the abuser is not present. The boy may be less likely to be forthcoming with his attacker in the room. Confronting either adult directly may lead to violence. The other adult may also be a victim of abuse and may fear retribution if their problems are discussed with health care providers. DIF: Apply REF: 55 OBJ: Explain the characteristics of patients from vulnerable populations that influence a nurse's approach to care. TOP: Implementation MSC: CPNRE: Foundations of Practice 14. The nurse is working with a 16-year-old pregnant girl who tells the nurse that she needs an
abortion. The nurse provides the patient with information on alternatives to abortion, but after several sessions, the patient still insists on having the abortion. The competency of the nurse as a consultant requires what of the nurse? a. Insist that the patient speak with a “right-to-life” advocate. b. Provide a referral to an abortion service. c. Refuse to provide referral to an abortion service. d. Delay referral to an abortion service. ANS: B
As a consultant, the nurse is responsible for providing information, listening objectively, and being supportive, caring, and trustworthy. The nurse does not make decisions but rather helps the patient reach decisions that are best for him or her. To refuse to provide a referral, to delay referral, or to insist that the patient speak with a “right-to-life” advocate would not be supportive of the patient’s decision. Counsellors usually suggest and rarely insist.
NUR I G B.C M S N T O
DIF: Apply REF: 57 OBJ: Describe the standards, competencies, roles, and activities important for success in community health nursing practice. TOP: Implementation MSC: CPNRE: Foundations of Practice 15. The patient is in the hospital with the diagnosis of early-onset Alzheimer’s disease. Before the
patient is discharged, the community-based nurse is making a visit to the patient’s home, where he lives with his daughter and her family. What would be a major focus of this visit? a. Demonstrate to the caregiver techniques for providing care. b. Stress to the family how difficult it will be to provide care at home. c. Encourage the family to send the patient to an extended-care facility. d. Teach the family how to have the patient declared incompetent. ANS: A
The role of the community-based nurse, in dealing with patients with Alzheimer’s disease, is to maintain the best possible functioning, protection, and safety for the patient. The nurse should demonstrate to the primary family caregiver the techniques for dressing, feeding, and toileting the patient while providing encouragement and emotional support to the caregiver. The nurse should protect the patient’s rights and maintain family stability. DIF: Apply REF: 55 OBJ: Describe the standards, competencies, roles, and activities important for success in community health nursing practice. TOP: Implementation
Canadian Fundamentals of Nursing 6th Edition Potter Test Bank MSC: CPNRE: Foundations of Practice 16. The community has three components: structure or locale, the people, and the social systems.
While conducting a community assessment, the nurse seeks data on the average household income and the number of residents on public assistance. In doing so, the nurse is evaluating which of the following? a. Structure b. Population c. Welfare system d. Social system ANS: A
Economic status is part of the community structure. Population would involve age and gender distribution, growth trends, density, education level, and ethnic or religious groups. The welfare system is part of the social system that also includes the education, government, communication, and health systems. DIF: Understand REF: 58 | 59 TOP: Implementation
OBJ: Describe elements of a community assessment. MSC: CPNRE: Foundations of Practice
17. The patient is being readmitted to an inner-city hospital for chest pain 3 months after having a
heart attack. The patient was referred to the hospital’s cardiac rehabilitation program after her previous admission. The patient states that she began going to program and liked it but stopped. When asked why, she states that, at the beginning, the classes were at 9 a.m., but then got switched to 7 p.m., when it is dark out. The cardiac rehabilitation program was within walking distance of the patient’s home. What is the most likely cause of the patient’s unwillingness to go to the program? G B.C M N R I a. Lack of transportation U S N T O b. Fear of walking at night c. Reimbursement issues d. Noncompliance ANS: B
A community assessment should be done to determine the level of community violence at night in the patient’s neighbourhood. She claimed that she liked the program when it was at 9 a.m. She did not mention finances as a reason for not going, and the program was within walking distance to her house. Noncompliance is a label given unfairly to patients. Most “noncompliance” has a cause. The cause should be identified and dealt with, so that the therapy will be successful. DIF: Apply REF: 58 | 59 TOP: Implementation
OBJ: Describe elements of a community assessment. MSC: CPNRE: Foundations of Practice
18. Community-based nursing requires a strong knowledge base in communication, cultural
diversity, group dynamics, and which other following element? a. Family theory b. Nursing research designs. c. Tertiary care. d. Focus on the individual ANS: A
Canadian Fundamentals of Nursing 6th Edition Potter Test Bank With the individual and family as the patients, the context of community-based nursing is family-centred care within the community. This focus requires a strong knowledge base in family theory, principles of communication, group dynamics, and cultural diversity. The nurse leans to partner with patients and families, not just with individuals. Nursing research designs and tertiary care are not the main focus of community-based nursing. DIF: Knowledge REF: 49-51 OBJ: Discuss the roles and functions of the community health nurse. TOP: Assessment MSC: CPNRE: Foundations of Practice 19. Of the following list of patients, which one would be considered at low risk for being a
member of a vulnerable population? a. An immigrant who speaks only Chinese. b. A Guatemalan truck driver who speaks limited English. c. A 22-year-old pregnant woman. d. A 15-year-old rape victim. e. A 40-year-old schizophrenic. ANS: C
Immigrant populations face multiple diverse health issues that cities, counties, and states need to address. These health care needs pose significant legal and policy issues. For some immigrants, access to health care is limited because of language barriers and lack of benefits, resources, and transportation. Low-risk mothers and babies usually are not considered vulnerable populations unless other factors are noted. Physical, emotional, and sexual abuse (such as rape), as well as neglect, are major public health problems affecting older persons, women, and children. When a patient has a severe mental illness such as schizophrenia, multiple health and socioeconomic problems must be explored.
N R I G B.C M U S N T O
DIF: Analyze REF: 53-56 OBJ: Explain the characteristics of patients from vulnerable populations that influence a nurse's approach to care. TOP: Planning MSC: CPNRE: Foundations of Practice
Canadian Fundamentals of Nursing 6th Edition Potter Test Bank
Chapter 05: Theoretical Foundations of Nursing Practice Potter et al: Canadian Fundamentals of Nursing, 6th Edition MULTIPLE CHOICE 1. How is the idea of a theory best explained? a. “Mental maps” that make sense of information and decisional processes. b. Mental formulations of objects or events. c. Aspects of reality that can be consciously sensed. d. A purposeful set of assumptions that identify relationships between concepts. ANS: D
A theory is a purposeful set of assumptions that identify the relationships between concepts. Theories are useful because they provide a systematic view of explaining, predicting, and prescribing phenomena. The description “‘Mental maps’ that make sense of information and decisional processes” explains the idea of conceptual frameworks, which link ideas together. The description “Mental formulations of objects or events” explains the idea of concepts. The description “Aspects of reality that can be consciously sensed” explains the idea of phenomena. DIF: Understand REF: 63 OBJ: Describe selected theories of nursing practice and differentiate between them. TOP: Assessment MSC: CPNRE: Foundations of Practice 2. Different types of theories may be used by nurses seeking to study the basis of nursing
practice. The theory about why phenomena occur is which of the following? N R I G B.C M a. Prescriptive. U S N T O b. Descriptive. c. Grand. d. Middle-range. ANS: B
Descriptive theories are descriptions of phenomena, speculations on why phenomena occur, and explanations of the consequences of phenomena. Prescriptive theories address nursing interventions and help predict the consequences of a specific nursing intervention. Grand theories provide the structural framework for broad, abstract ideas about nursing. Middle-range theories address specific phenomena or concepts and reflect practice. DIF: Understand REF: 65, Table 5-2 OBJ: Describe selected theories of nursing practice and differentiate between them. TOP: Assessment MSC: CPNRE: Foundations of Practice 3. A body of knowledge that encompasses definitions of person, environment, health, and
nursing is referred to as which of the following? a. Family nursing practice. b. Prescriptive nursing theory. c. Advanced nursing practice. d. Metaparadigm of nursing. ANS: D
Canadian Fundamentals of Nursing 6th Edition Potter Test Bank The major components of nursing theory, sometimes called the metaparadigm concepts, are person, environment, health, and nursing. Family nursing practice, although perhaps focusing on all of these concepts, is not a body of knowledge. Prescriptive nursing theory addresses nursing interventions and helps predict consequences of a specific intervention. Advanced nursing practice may focus on these concepts but is not a body of knowledge specific to these concepts. DIF: Understand REF: 65 | 66 OBJ: Describe selected theories of nursing practice and differentiate between them. TOP: Assessment MSC: CPNRE: Foundations of Practice 4. Within a health care system that is based on Betty Neuman’s theory, what is the nurse’s goal
in caring for a patient who is having difficulty breathing and requires oxygen and medication? a. Strengthen the line of defences and focus on prevention. b. Promote attainment of biological self-care requisites. c. Assist in physiological adaptation to internal changes. d. Achieve the 14 basic needs. ANS: A
Neuman’s framework for practice includes nursing actions that focus on actual or potential stressors, and thus on prevention. Dorothea Orem’s theory focuses on the attainment of self-care. Sister Callista Roy’s theory focuses on adaptation. Virginia Henderson’s theory focuses on helping the patient to achieve 14 basic needs. DIF: Apply REF: 69 | 70 OBJ: Recognize selected conceptual frameworks associated with nursing practice. TOP: Implementation MSC: CPNRE: Foundations of Practice
NURSINGTB.COM
5. Although the different nursing theories have similarities, key elements distinguish one from
another. What is the emphasis of Jean Watson’s conceptual model? a. Self-care maintains wholeness. b. Subsystems exist in dynamic stability. c. Stimuli disrupt an adaptive system. d. Caring is central to the essence of nursing. ANS: D
Watson believed that nurses must do far more than deal with physical illness; they must attend to their primary function, which is caring. From Watson’s perspective, caring infuses all aspects of a nurse’s role and draws attention to nursing acts as embodying an esthetic that facilitates both healing and growth. Self-care is central to Dorothea Orem’s theory. The key emphasis of Dorothy Johnson’s theory is that subsystems exist in dynamic stability. The key emphasis of Sister Callista Roy’s theory is that stimuli disrupt an adaptive system. DIF: Understand REF: 71 OBJ: Recognize selected conceptual frameworks associated with nursing practice. TOP: Planning MSC: CPNRE: Foundations of Practice 6. A community health nurse is working with a variety of patients and decides to use a systems
theory approach to help them to meet their health care needs. When using systems theory, the nurse focuses on which of the following? a. The patient’s interaction with his or her environment.
Canadian Fundamentals of Nursing 6th Edition Potter Test Bank b. The hierarchy of the patient’s human needs. c. The patient’s attitudes toward health behaviours. d. The response of the patient to the process of growth and development. ANS: A
According to systems theory, a system is made up of parts that depend on one another, are interrelated, share a common purpose, and together form a whole. A patient’s interaction with the environment is an example of an open system. The nurse understands that factors that change the environment also can have an effect on the system. Abraham Maslow’s hierarchy of human needs is an interdisciplinary theory useful in planning individualized care. Determining a patient’s attitudes toward health behaviours follows a health and wellness theoretical model. Focusing on the response of a patient to the process of growth and development is consistent with developmental theories. DIF: Understand REF: 70 OBJ: Describe selected theories of nursing practice and differentiate between them. TOP: Planning MSC: CPNRE: Foundations of Practice 7. While working on a postoperative unit, the nurse is applying elements of self-care theory and
is assisting a patient to attain and manage self-care in wound management. Who was the nursing pioneer who developed this theory? a. Florence Nightingale. b. Virginia Henderson. c. Dorothea Orem. d. Hildegard Peplau. ANS: C
The goal of Orem’s theory isNtoUhRelp ie. ntCper form self-care. The goal of Nightingale’s SIthe NGpat TB OM theory is to shift the focus from the disease process toward an environment conducive to healing. Henderson defined nursing practice as assisting the individual, sick or well, in the performance of activities that will contribute to health, recovery, or a peaceful death. Peplau’s theory defined the core of nursing as the interpersonal relationship between the nurse and the patient. DIF: Understand REF: 68-70 OBJ: Describe selected theories of nursing practice and differentiate between them. TOP: Implementation MSC: CPNRE: Foundations of Practice 8. What is an element that is key to the framework for practice in Martha Rogers’s theory? a. Human becoming. b. Manipulation of the patient’s environment. c. Seven categories of behaviour and behavioural balance. d. Focus upon the life process of a human being along a time–space continuum. ANS: D
Canadian Fundamentals of Nursing 6th Edition Potter Test Bank The framework for practice, according to Rogers’s theory, consisted of presenting the patient of nursing not simply as a person but as an energy field in constant interaction with the environment, which itself was also an irreducible energy field, coextensive with the universe. Nursing’s role was to focus upon the life process of a human being along a time–space continuum. Rosemarie Parse, in her theory of human becoming (1997), viewed the individual as a unitary being who is “indivisible, unpredictable, and ever-changing” and “a freely choosing being who can be recognized through paradoxical patterns co-created all-at-once in mutual process with the universe.” Nightingale’s theory includes manipulation of the patient’s environment (i.e., appropriate noise, nutrition, hygiene, light, comfort, socialization, and hope) in the framework for practice. Dorothy Johnson’s theory includes seven categories of behaviour and behavioural balance in the framework for practice. DIF: Understand REF: 71 OBJ: Describe selected theories of nursing practice and differentiate between them. TOP: Planning MSC: CPNRE: Foundations of Practice 9. Whose theory is most pertinent in assisting the nurse who is helping a patient focus on stress
reduction? a. Hildegard Peplau’s. b. Virginia Henderson’s. c. Betty Neuman’s. d. Rosemarie Parse’s. ANS: C
Neuman’s theory focuses on stress reduction as the goal of the systems model of nursing practice. Peplau’s theory focuses on interpersonal relationships between nurses and patients and the interactive, therapeutic nature of the nursing process. Henderson’s theory focuses on N R theo INryG T B.C onM the notion of nurses’ engagement with, the 14 basic human needs. ParsUe’s S focusesO and care of, people in a continuous process of making choices and changing health priorities. DIF: Apply REF: 69-71 OBJ: Describe selected theories of nursing practice and differentiate between them. TOP: Implementation MSC: CPNRE: Foundations of Practice 10. What does the theorizing term proposition mean? a. A purposeful set of assumptions. b. A declarative assertion. c. The process of formulating concepts. d. The structure that links concepts together. ANS: B
A proposition is a declarative assertion. A purposeful set of assumptions is a theory. The process of formulating concepts is conceptualization. The structure that links concepts together is a conceptual framework. DIF: Understand REF: 64, Table 5-1 OBJ: Describe relationships between theorizing and other forms of nursing knowledge. TOP: Planning MSC: CPNRE: Foundations of Practice 11. What is a characteristic of complexity science? a. Patterns of knowledge application.
Canadian Fundamentals of Nursing 6th Edition Potter Test Bank b. A rigid approach for describing experiences. c. Reducing phenomena to smallest properties. d. Orientation to studying the nature of people’s needs. ANS: C
Complexity science consists of dynamic and interactive phenomena reduced to the smallest properties that can be observed within their natural context so that their interactions can be interpreted with as little interference as possible from prior assumptions. Patterns of knowledge application are not characteristic of complexity science. Complexity science is not a rigid approach for describing experiences, and it is not characterized by an orientation to studying the nature of people’s needs. DIF: Understand REF: 67 OBJ: Interpret current debates surrounding various theories of nursing practice. TOP: Planning MSC: CPNRE: Foundations of Practice 12. What is a current concern with the use of the nursing diagnoses approved by the North
American Nursing Diagnoses Association (NANDA)? a. NANDA’s list stifles critical thinking. b. NANDA’s list should replace conceptual nursing models. c. NANDA’s list is an inherent barrier to individualized care. d. NANDA’s list does not reflect practical diagnoses. ANS: C
NANDA’s list of nursing diagnoses is recognized by many nurses as a system that relies entirely upon an agreement about what constitutes average wellness and illness experiences. As such, it can create worrisome barriers to the individualized care of patients. NANDA’s list of nursing diagnoses may stifNleUcR ritic hink SIalNtG TBin.gC, bu OMt this has not been recognized as a concern; it is often considered to be less thorough than conceptual models of nursing, as it involves only one aspect of caring for patients; and it does reflect practical diagnoses. DIF: Understand REF: 67 OBJ: Describe challenges inherent in theorizing about nursing practice. TOP: Planning MSC: CPNRE: Foundations of Practice 13. In the late 1980s, the shift in scholarly work regarding nursing theories challenged nurses
when theorizing to include consideration of what type of knowledge? a. Clinical. b. Intuitive. c. Theoretical. d. Substantive. ANS: D
Afaf Meleis (1987) challenged nurses to direct their theorizing away from the processes by which nurses use knowledge and toward the equally challenging issues associated with the substance of that knowledge—that is, to include substantive knowledge. Neither clinical knowledge nor intuitive knowledge was related to this challenge. Theoretical knowledge had been the focus of theorizing for many years before the late 1980s. DIF: Understand REF: 71 OBJ: Appreciate the role of “theorizing” about the essence of nursing. TOP: Assessment MSC: CPNRE: Foundations of Practice
Canadian Fundamentals of Nursing 6th Edition Potter Test Bank
14. Interactionist theories focused on the relationships between nurses and their patients. Which
of the following was an interactionist theorist? a. Hildegard Peplau b. Dorothea Orem c. Florence Nightingale d. Betty Neuman ANS: A
Hildegard Peplau was an interactionist theorist. Dorothea Orem was a needs theorist, Florence Nightingale was a practice-based theorist, and Betty Neuman was a systems theorist. DIF: Apply REF: 69 | 70 OBJ: Describe selected theories of nursing practice and differentiate between them. TOP: Assessment MSC: CPNRE: Foundations of Practice
NURSINGTB.COM
Canadian Fundamentals of Nursing 6th Edition Potter Test Bank
Chapter 06: Evidence-Informed Practice Potter et al: Canadian Fundamentals of Nursing, 6th Edition MULTIPLE CHOICE 1. The extent to which the findings of a qualitative study are thought to be meaningful and
applicable to similar cases or other situations is known as which of the following? a. Credibility. b. Transferability. c. Validity. d. Reliability. ANS: B
Transferability refers to the extent to which the findings of a qualitative study are thought to be meaningful and applicable to similar cases or other situations. Credibility concerns whether the research and results are trustworthy. Validity refers to whether the study measured what it was intended to. Reliability concerns whether the study results are repeatable. DIF: Understand TOP: Evaluate
REF: 83 OBJ: Define the key terms listed. MSC: CPNRE: Foundations of Practice
2. In caring for patients, it is important for the nurse to realize that evidence-informed decision
making is which of the following? a. The only valid source of knowledge that should be used. b. Secondary to traditional or standard care knowledge. c. Dependent on patient values and expectations. N RSINGTB.COM d. Not related to quality improUvem ent studies ANS: C
Even when the best evidence available is used, application and outcomes will differ according to patients’ values, preferences, concerns, and expectations. Nurses often care for patients on the basis of tradition or convenience, or in the standard “It has always been done this way.” Although these sources have value, it is important to learn to rely more on research evidence than on nonresearch evidence. Evidence-informed decision making is closely related to quality improvement. DIF: Understand REF: 76 | 77, Box 6-1 OBJ: Explain the need for evidence to inform nurses’ decision making. TOP: Evaluate MSC: CPNRE: Foundations of Practice 3. The first step in evidence-informed practice is to ask a clinical question. In doing so, what
does the nurse need to realize with regard to researching interventions? a. The question is more important than its format. b. The question will lead the researcher to hundreds of articles that must be read. c. The question may be easier if it is in PICOT format. d. The question may be more useful the more general it is. ANS: C
Canadian Fundamentals of Nursing 6th Edition Potter Test Bank The PICOT format allows the nurse to ask questions that are intervention focused. Inappropriately formed questions will probably lead to irrelevant sources of information. It is not beneficial to read hundreds of articles. It is more beneficial to read the best four to six articles that specifically address the question. The more focused the question asked is, the easier it will become to search for evidence in the scientific literature. DIF: Understand TOP: Evaluate
REF: 77 OBJ: Discuss the steps of the research process. MSC: CPNRE: Foundations of Practice
4. When the best evidence is collected, what is the “gold standard” for research? a. The randomized controlled trial (RCT). b. The peer-reviewed article. c. Qualitative research. d. The opinion of expert committees. ANS: A
Individual RCTs are the “gold standard” for research. A peer-reviewed article is one that has been reviewed by a panel of experts; this is not a research method. Qualitative research is valuable in identifying information about how patients cope with or manage various health problems and their perceptions of illness. It does not usually have the robustness of an RCT. Expert opinion is on the bottom of the hierarchical pyramid of evidence. DIF: Knowledge REF: 83 OBJ: Discuss methods for developing new nursing knowledge. TOP: Assessment MSC: CPNRE: Foundations of Practice 5. The nursing team is writing a research article on a patient care topic. Which section will get
the reader to read the article bNeU caRuS seIoN f tG hT e vBa. luC eO ofMthe topic for the reader? a. Abstract. b. Introduction. c. Literature review or background. d. Results. ANS: B
The introduction contains information about its purpose and the importance of the topic to the audience who reads the article. The abstract is a brief summary of the article. The literature review or background offers a detailed background of the level of science or clinical information that is available about the topic of the article. The results section is the summary section of the article. DIF: Understand TOP: Assessment
REF: 78 OBJ: Discuss the steps of the research process. MSC: CPNRE: Collaborative Practice
6. The nurse is caring for a patient with chronic low back pain. In providing care for this patient,
the nurse wonders whether there is literature on nursing interventions for chronic low back pain. What are the best-known databases for nursing literature? a. MEDLINE and CINAHL b. EMBASE and PsycInfo c. PsycINFO and CINAHL d. MEDLINE and EMBASE ANS: A
Canadian Fundamentals of Nursing 6th Edition Potter Test Bank The best-known databases for nursing literature are MEDLINE and CINAHL. EMBASE includes biomedical and pharmaceutical studies. PsycINFO deals with psychology and related health care disciplines. DIF: Understand REF: 78 OBJ: Discuss methods for developing new nursing knowledge. TOP: Implementation MSC: CPNRE: Foundations of Practice 7. The nurse is developing a PICOT question related to sucrose administration before
phlebotomy in infants: “Is the pain score lower in infants who are administered sucrose one minute before phlebotomy than in infants who do not receive sucrose at all?” With a PICOT question, P is the population of interest, I is the intervention of interest, C is the comparison of interest, O is the outcome, and T is the time; is this a true PICO question? a. Yes, because the outcome always comes before the intervention. b. Yes, regardless of placement of elements. c. No, because the comparison comes after the intervention. d. No, because the outcome comes after the population. ANS: B
A well-designed PICOT question does not have to follow the sequence of P, I, C, O, and T. The aim is to ask a question that contains as many of the PICOT elements as possible. DIF: Understand REF: 77 OBJ: Discuss methods for developing new nursing knowledge. TOP: Implementation MSC: CPNRE: Foundations of Practice 8. In a review of literature for an evidence-informed practice study, what is the most reliable
NURSINGTB.COM level of evidence? a. Systematic review and meta-analysis. b. Randomized control trial (RCT). c. Case control study. d. Control trial without randomization. ANS: A
In a systematic review or meta-analysis, an independent researcher reviews all the RCTs conducted on the same clinical question and reports whether the evidence is conclusive or whether further study is needed. A single RCT is not as conclusive as a review of several RCTs on the same question. Control trials without randomization may involve bias in how the study is conducted. Case-control studies also have room for bias. DIF: Analyze REF: 83 OBJ: Discuss methods for developing new nursing knowledge. TOP: Assessment MSC: CPNRE: Collaborative Practice 9. Why is qualitative nursing research valuable? a. It excludes all bias. b. It entails the use of randomization in structure. c. It help determine associations between variables and conditions. d. It entails the study of phenomena that are difficult to quantify. ANS: D
Canadian Fundamentals of Nursing 6th Edition Potter Test Bank Qualitative nursing research is the study of phenomena that are difficult to quantify or categorize, such as patients’ perceptions of illness. No study can totally exclude bias. However, randomization, such as that used in randomized control studies, helps, but in qualitative nursing research, randomization is not usually used. Control studies determine whether there is an association between one or more predictor variables and the condition. DIF: Knowledge TOP: Assessment
REF: 83 OBJ: Define the key terms listed. MSC: CPNRE: Foundations of Practice
10. The nurses on the unit have used a PICOT question to develop an evidence-informed change
in protocol for a certain nursing procedure. However, to make these changes throughout the entire institution would require more support staff than is available at this time. What is the nurses’ best option? a. Drop the idea of making the change at this time. b. Insist that management hire the needed staff to facilitate the change. c. Seek employment in another institution that may have the staff needed. d. Conduct a pilot study to develop evidence to support the change. ANS: D
When evidence is not strong enough to apply in practice, or if resources are limited, the next option is to conduct a pilot study to investigate the PICO question. Dropping the idea would be counterproductive; insisting that management hire staff could be seen as a mandate and could produce negative results. Seeking employment at another institution most likely would not be the answer because most institutions operate under similar established guidelines. DIF: Apply REF: 79 OBJ: Discuss methods for developing new nursing knowledge. TOP: Implementation MSC: CPNRE: Collaborative PNraU ctR icS e INGTB.COM 11. According to hospital policy, when starting an intravenous (IV) catheter, the nurse must first
prepare the potential IV site with alcohol and dress it with a gauze dressing. The nurse has done a literature review and believes that evidence-informed practice dictates the use of a transparent dressing to prevent catheter dislodgement. What should the nurse do? a. Begin to use transparent dressing instead of gauze dressings. b. Bring findings to the policy and procedure committee. c. Use transparent dressings on half of her IV starts and gauze on the other. d. Continue following hospital policy without saying anything. ANS: B
As a result of her finding, the nurse should meet with the policy and procedure committee to recommend routine use of transparent dressings. However, until the policy is changed, or the nurse receives approval to conduct a pilot study, the nurse is obligated to follow hospital procedure. If the nurse has information that can lead to better patient care, he or she has an obligation (moral and professional) to bring it to the attention of policy makers. DIF: Apply REF: 79 OBJ: Discuss the steps of evidence-informed decision making. MSC: CPNRE: Collaborative Practice
TOP: Implementation
Canadian Fundamentals of Nursing 6th Edition Potter Test Bank 12. The nursing team is trying to identify common general themes relative to the effectiveness of
cardiac rehabilitation for patients who have had heart attacks and have gone through cardiac rehabilitation programs. The nurses conduct interviews and focus groups. What type of research is being implemented? a. Evaluation research b. Experimental research c. Qualitative research d. Nonexperimental research ANS: C
Qualitative research involves using inductive reasoning to develop generalizations or theories from specific observations or interviews. Evaluation and experimental research are forms of quantitative research. Nonexperimental descriptive studies describe, explain, or predict phenomena, such as factors that lead to an adolescent’s decision to smoke cigarettes. DIF: Understand REF: 83 OBJ: Discuss methods for developing new nursing knowledge. TOP: Assessment MSC: CPNRE: Collaborative Practice 13. In conducting a research study, the researcher must inform the participants, in lay language,
about the risks and benefits of participating. This is an important aspect of what? a. Anonymity. b. Confidentiality. c. Informed consent. d. The research process. ANS: C
Informed consent means thatNres earc suGbje ct. s (C 1)OaM re given full and complete information UR SIh N TB about the purpose of the study, procedures, data collection, potential harm and benefits, and alternative methods of treatment; (2) are capable of fully understanding the research; (3) have the power to voluntarily consent or decline participation; and (4) understand how confidentiality or anonymity is maintained. Confidentiality guarantees that any information the subject provides will not be reported in any manner that identifies the subject and will not be accessible to people outside the research team. Anonymity is the condition when even the researcher cannot link the subject to the data. The research process is a broader concept that provides an orderly series of steps that allow the researcher to move from asking a question to finding the answer. DIF: Knowledge TOP: Assessment
REF: 84 | 85 OBJ: Discuss the steps of the research process. MSC: CPNRE: Foundations of Practice
14. When evaluating quality improvement (QI) programs in relation to evidence-informed
decision making, what should the nurse note? a. Both are designed to improve performance. b. When evidence-informed practice (EIP) projects are implemented, it is important to review QI data. c. EIP is not at all related to QI. d. Evaluation of processes is the realm of performance improvement, not QI. ANS: B
Canadian Fundamentals of Nursing 6th Edition Potter Test Bank EIP and QI go hand in hand. When an EIP project is implemented, it is important to review available QI data. Reliable QI data improve the relevance and scope of an EIP project. Performance improvement is concerned with performance. QI is concerned with processes. DIF: Understand REF: 76 OBJ: Discuss methods for developing new nursing knowledge. TOP: Planning MSC: CPNRE: Collaborative Practice 15. The quality improvement committee has been alerted to an increased number of falls in the
hospital. Most of these falls have occurred at night and have involved patients who were trying to crawl over bed rails. A literature review revealed that most falls occur because patients are trying to go to the bathroom. The committee created a practice change that bed rails should be left in the down position, and hourly nursing rounds should be conducted. What is the committee’s next step? a. Evaluate the changes in 1 month. b. Wait a month before implementing the changes. c. Implement the changes as a pilot study. d. Communicate to staff the results of this inquiry. ANS: D
Quality improvement combined with evidence-informed practice is the foundation for excellent patient care and outcomes. Once a QI committee makes a practice change, it is important to communicate the results to staff. Practice changes will probably not last when QI committees fail to report findings and results of interventions. Once communicated, changes should be put in place as the committee deems reasonable (i.e., either fully or as a pilot study) and as soon as practical; this should be followed by re-evaluation. DIF: Apply REF: 78N| U 79RSINGTB.COM OBJ: Discuss methods for developing new nursing knowledge. TOP: Implementation MSC: CPNRE: Collaborative Practice 16. As nurses move forward in their education, different roles may be assumed with regard to
research. What is the expected research role for the entry-level nurse? a. To assume the role of a clinical expert. b. To incorporate evidence-informed practice activities into nursing practice. c. To develop methods of inquiry relevant to nursing. d. To acquire funding for research projects. ANS: B
Nurses can make links between research finding and nursing care by identifying appropriate clinical problems, reading peer-reviewed literature from numerous sources, and incorporating evidence-informed practice activities into the nursing practice of their unit or agency. Nurses, with experience, will develop clinical expertise, but this is not their expected research role. Nurses with advanced education or those that have a research focus will be responsible for developing methods of inquiry relevant to nursing and for acquiring funding for research projects. DIF: Understand REF: 85 OBJ: Explain the need for evidence to inform nurses’ decision making. TOP: Implementation MSC: CPNRE: Collaborative Practice
Canadian Fundamentals of Nursing 6th Edition Potter Test Bank 17. A nurse researcher is conducting a research project on optimal time frames for postoperative
ambulation of patients. After the researcher identifies the problem, what is the next step in the research process? a. Selecting the population. b. Reviewing the literature. c. Identifying the instrument to use for data analysis. d. Obtaining approval to conduct the study. ANS: B
After the problem is identified, the next step in the research process is reviewing the literature in order to determine what is known about the problem. After identification of the problem and review of the literature, the researcher will design the study protocol. Selecting the population is a component of this phase of the research process. The instrument to use for data analysis is identified during the process of designing the study protocol. This step occurs during the study design phase of the research process after problem identification and literature review have taken place. Obtaining necessary approvals is part of conducting the study, a part that follows the design phase in the research process. DIF: Analyze REF: 80 | 81 TOP: Implementation
OBJ: Define nursing research. MSC: CPNRE: Collaborative Practice
18. The nurse researcher is preparing to conduct research that will allow precise measurement of a
phenomenon. Which of the following methods will provide the nurse with the right kind of data? a. Experimental research. b. Phenomenology. c. Grounded theory. NURSINGTB.COM d. Ethnography. ANS: A
Experimental research is a type of quantitative research. Phenomenology, grounded theory, and ethnography are all types of qualitative research. DIF: Understand TOP: Assessment
REF: 81-83 OBJ: Define the key terms listed. MSC: CPNRE: Collaborative Practice
19. The nurse researcher is reviewing literature related to a potential problem that has been
identified on the nursing unit. The nurse realizes that nursing research is important because it is designed to do which of the following? a. Enhance the nurse’s chance at promotion. b. Maintain public funding c. Improve professional practice. d. Lead to decreases in budget expenditures. ANS: C
Nursing research is a way to identify new knowledge, improve professional education and practice, and use resources effectively. The research does not always result in lower budget expenditures; rather, it enhances more effective use of resources. Maintaining public funding is not a direct result of research. DIF: Understand
REF: 75-77
OBJ: Discuss the steps of the research process.
Canadian Fundamentals of Nursing 6th Edition Potter Test Bank TOP: Assessment
MSC: CPNRE: Foundations of Practice
NURSINGTB.COM
Canadian Fundamentals of Nursing 6th Edition Potter Test Bank
Chapter 07: Nursing Values and Ethics Potter et al: Canadian Fundamentals of Nursing, 6th Edition MULTIPLE CHOICE 1. Four patients in labour all request epidural analgesia to manage their pain at the same time.
Which ethical principle is compromised when only one nurse anaesthetist is on call? a. Justice. b. Nonmaleficence. c. Beneficence. d. Fidelity. ANS: A
Justice refers to fairness and is used frequently in discussion regarding access to health care resources. In this situation, the just distribution of resources—in this case pain management—cannot be justly apportioned. Nonmaleficence means “to do no harm,” beneficence means “to do good,” and fidelity means “to be true to” or “to be honest.” Each of these principles is partially alluded to in the question; however, justice is the principle most comprised because not all labouring patients have equal access to pain management owing to lack of personnel resources. DIF: Understand REF: 91 | 92 OBJ: Explain the relationship between ethics and professional nursing practice. TOP: Implementation MSC: CPNRE: Professional, Ethical, and Legal Practice 2. The patient tells the nurse that she is afraid to speak up regarding her desire to end care for
N R I G B.C M
fear of upsetting her husband aU nd cShildNrenT . WhichOprinciple in the nursing code of ethics ensures that the nurse will promote the patient’s cause? a. Responsibility. b. Advocacy. c. Confidentiality. d. Accountability. ANS: B
Nurses advocate for patients by supporting the patient’s cause. A nurse’s ability to advocate adequately for a patient is based on the unique relationship that develops between nurse and patient and on the opportunity to better understand the patient’s point of view. Responsibility refers to respecting one’s professional obligations and following through on promises; confidentiality deals with privacy issues; and accountability refers to owning one’s actions. DIF: Understand REF: 89 | 90 OBJ: Explain the relationship between ethics and professional nursing practice. TOP: Diagnosis MSC: CPNRE: Professional, Ethical, and Legal Practice 3. The patient’s son requests to view the documentation in his mother’s medical record. What is
the nurse’s best response to this request? a. “I’ll be happy to get that for you.” b. “You will have to talk to the physician about that.” c. “You will need your mother’s permission.” d. “You are not allowed to see it.”
Canadian Fundamentals of Nursing 6th Edition Potter Test Bank
ANS: C
Nurses should protect a patient's right to privacy and confidentiality by helping the patient access his or her health records (subject to legal requirements), intervening if other members of the health care team fail to respect the patient's privacy, and following policies that protect the patient's privacy. Private health information cannot be shared without the patient’s specific permission. The other three responses either are outright false or use poor communication techniques. DIF: Apply REF: 90 OBJ: Explain the relationship between ethics and professional nursing practice. TOP: Evaluate MSC: CPNRE: Professional, Ethical, and Legal Practice 4. When individuals work together to solve ethical dilemmas, they must examine their own
values. This step is crucial to ensure what? a. The group identifies the one correct solution. b. Fact is separated from opinion. c. Judgmental attitudes are not provoked. d. Different perspectives are respected. ANS: D
Values are personal beliefs that influence opinions; to be able to negotiate differences in opinions, the nurse must first be clear about personal values, which will influence behaviours, decisions, and actions. Ethical dilemmas are a problem in that no single correct solution exists. DIF: Remember TOP: Evaluate
REF: 88 OBJ: Discuss how values influence patient care. MSC: CP NR E: P rofe ssio ical, and Legal Practice N R I G Bna.l,CEthM
U S N T
O
5. Ethical dilemmas are common occurrences when caring for patients. The nurse understands
that dilemmas are a result of which of the following? a. Presence of conflicting values. b. Hierarchical systems. c. Judgemental perceptions of patients. d. Poor communication with the patient. ANS: A
The primary underlying reason that ethical dilemmas occur is that there are no clear-cut, universally accepted solutions to a problem when participating individuals do not share the same values. Poor communication and the hierarchical systems that exist in health care, such as reporting structures within the hospital, or the historically unequal relationship between physicians and nurses, may complicate dilemmas. Without clarification of values, the nurse may not be able to distinguish fact from opinion or value, and this can lead to judgemental attitudes. DIF: Understand TOP: Assessment
REF: 88 OBJ: Discuss the role of values in the study of ethics. MSC: CPNRE: Professional, Ethical, and Legal Practice
6. The nurse questions a physician’s order to administer a placebo to the patient. The nurse’s
action is based on which ethical principle? a. Autonomy. b. Beneficence.
Canadian Fundamentals of Nursing 6th Edition Potter Test Bank c. Justice. d. Fidelity. ANS: A
Autonomy is the freedom to make decisions without external control. In this case, the nurse questions the physician’s order for a placebo because such a decision was made without consultation with the patient. Beneficence refers to taking a positive action for others, and although it has implications in this situation, it is not the primary operating principle. Justice refers to fairness and is most often used in discussions about access to health care resources. Fidelity refers to the agreement to keep promises. DIF: Understand REF: 91 | 92 TOP: Implementation
OBJ: Examine and clarify personal values. MSC: CPNRE: Professional, Ethical, and Legal Practice
7. The nurse finds it difficult to care for a patient whose advance directive states that no
extraordinary resuscitation measures should be taken. Which step may help the nurse to find resolution in this assignment? a. Calling for an ethical committee consult. b. Declining the assignment on religious grounds. c. Clarifying the nurse’s own personal values. d. Persuading the family to challenge the directive. ANS: C
Values develop over time and are influenced by family, schools, religious traditions, and life experiences. The nurse must recognize that no two individuals have the same set of experiences, and so differences in values are more likely to be the norm than the exception. Closer inspection of one’s values may be a step in gaining an understanding of another person’s perspective. CallingNfU orRaS coInN suGltT , dBe. clC inO inM g the assignment, and persuading the family to challenge the patient’s directive are not ideal resolutions because they do not address the reason for the nurse’s discomfort, which is the conflict between the nurse’s values and those of the patient. DIF: Apply TOP: Assessment
REF: 88 OBJ: Examine and clarify personal values. MSC: CPNRE: Professional, Ethical, and Legal Practice
8. The nurse values autonomy above all other principles. Which patient assignment will the
nurse find most difficult to accept? a. Teenager in labour who requests epidural anaesthesia. b. Middle-aged father of three with an advance directive declining life support. c. Elderly patient who requires dialysis. d. Family elder who is making the decisions for a 30-year-old female member. ANS: D
Autonomy is the freedom to make decisions without external control. A nurse who values autonomy highly may find it difficult to accept situations in which the patient is not the primary decision maker regarding his or her care. A teenager requesting an epidural anaesthetic, a father with an advanced directive, and an elderly patient requiring dialysis are all patients who are making their own decisions and choices regarding care. DIF: Analyze TOP: Evaluate
REF: 91 OBJ: Examine and clarify personal values. MSC: CPNRE: Professional, Ethical, and Legal Practice
Canadian Fundamentals of Nursing 6th Edition Potter Test Bank 9. Which philosophy of health care ethics would be particularly useful for making ethical
decisions about vulnerable populations? a. Feminist ethics. b. Deontology. c. Bioethics. d. Utilitarianism. ANS: A
Feminist ethics focuses particularly on the nature of relationships, especially those in which there is a power imbalance or in which a point of view is not routinely accepted. Examples of populations that are considered vulnerable include children, pregnant women, incarcerated persons, and minority groups. Deontology refers to making decisions or “right-making characteristics”; bioethics focuses on consensus building; and utilitarianism refers to the greatest good for the greatest number. DIF: Remember REF: 92 OBJ: Describe some basic ethical philosophies relevant to health care. TOP: Assessment MSC: CPNRE: Professional, Ethical, and Legal Practice 10. A nurse argues that the health care system needs reform because a large number of patients
are uninsured and end up needing expensive emergency care when low-cost measures covered by insurance could have prevented their illnesses. What ethical framework is she using to make this case? a. Deontology. b. Ethics of care. c. Feminist ethics. d. Utilitarianism. ANS: D
NURSINGTB.COM
Utilitarianism is a system of ethics wherein value is determined by usefulness. This system of ethics focuses on the outcome of the greatest good for the greatest number of people. Deontology would not account for consequences of actions. The ethics of care would not be helpful because consensus on this issue is not achievable. Feminist ethics, which focus on relationships, are not addressed in this case. DIF: Evaluate REF: 91 OBJ: Describe some basic ethical philosophies relevant to health care. TOP: Assessment MSC: CPNRE: Professional, Ethical, and Legal Practice 11. The nurse has become aware that narcotics are missing in the patient care area. Which ethical
principle obligates the nurse to report the missing medications? a. Advocacy. b. Responsibility. c. Confidentiality. d. Accountability. ANS: B
Canadian Fundamentals of Nursing 6th Edition Potter Test Bank Responsibility refers to one’s willingness to respect and adhere to one’s professional obligations. One of the obligations nursing has is to protect patients and communities, including other nurses. If narcotics are missing, this may indicate that patients have not received medications ordered for their care, or it may suggest that a health care professional may be working under the influence of these drugs. Accountability refers to the ability to answer for one’s actions. Advocacy refers to the support of a particular cause. Confidentiality involves protecting patients’ personal health information and privacy. DIF: Understand REF: 89 | 90 OBJ: Explain the relationship between ethics and professional nursing practice. TOP: Evaluate MSC: CPNRE: Professional, Ethical, and Legal Practice 12. A young pregnant woman whose fetus has been exposed to multiple teratogens consents to
undergo serial percutaneous umbilical blood sampling (PUBS) to examine how exposure affects the fetus over time. Although these tests will not improve the fetus’s outcomes and will expose it to some risks, the information gathered may help infants in the future. Which ethical principle is at greatest risk? a. Autonomy. b. Fidelity. c. Nonmaleficence. d. Beneficence. ANS: C
Nonmaleficence is the ethical principle that focuses on avoidance of harm or hurt. The nurse must balance risks and benefits of care. Repeated PUBS may place the mother and fetus at risk for infection and increased pain, and the mother may be at risk for increased emotional health stress. Fidelity refers to the agreement to keep promises, autonomy refers to freedom to N R Iol,GaTndB.C M make decisions without externaUl coSntrN beneOficence refers to taking positive actions to help others. DIF: Apply REF: 91 | 92 OBJ: Describe some basic ethical philosophies relevant to health care. TOP: Evaluate MSC: CPNRE: Professional, Ethical, and Legal Practice 13. A high school teacher with advanced multiple sclerosis teaches from her wheelchair but
insists on being treated the same as other colleagues. Which of the following is the teacher demonstrating? a. Preserving dignity. b. Choosing from alternatives. c. Considering all consequences. d. Acting with a pattern of consistency. ANS: A
The teacher’s choice is preserving dignity, She cherishes her choice of being treated like everyone else despite her medical condition and publicly affirms the choice by teaching from her wheelchair. At this point, having already made a choice, the teacher is not choosing from alternatives—she could have chosen to quit teaching, but she did not—and is not demonstrating that she is still considering all consequences. The situation does not reflect that her actions have a pattern of consistency; she is not repeating a particular behaviour. DIF: Analyze
REF: 89 | 90
Canadian Fundamentals of Nursing 6th Edition Potter Test Bank OBJ: Describe some basic ethical philosophies relevant to health care. TOP: Assessment MSC: CPNRE: Professional, Ethical, and Legal Practice 14. Which of the following is an example of ethical responsibility? a. Delivery of competent care. b. Formation of interpersonal relationships. c. Application of the nursing process. d. Evaluation of new computerized technologies. ANS: A
Providing competent care is one of the values that nurses must uphold. Formation of interpersonal relationships, application of the nursing process, and evaluation of new computerized technologies are not ethical responsibilities. DIF: Understand REF: 89 OBJ: Explain the relationship between ethics and professional nursing practice. TOP: Assessment MSC: CPNRE: Professional, Ethical, and Legal Practice 15. The nurse is caring for a severely ill patient with acquired immune deficiency syndrome
(AIDS) who now requires ventilator support. Which intervention is considered futile? a. Administering the influenza vaccine. b. Providing oral care every 5 hours. c. Applying fentanyl patches prn for pain. d. Supporting the patient’s lower extremities with pillows. ANS: A
Futile refers to something that is hopeless or serves no useful purpose; in nursing, it refers to interventions that are unlikelN y toR proI ducG e benefit foM r the patient. A vaccine is administered to N TB.anCO prevent or lessen the likelihoodUof S contracting infectious disease at some time in the future; at this point, the patient is dying. Care delivered to a patient at the end of life is focused on pain management and comfort measures, such as providing oral care, applying fentanyl patches, and supporting the patient’s lower extremities. DIF: Understand REF: 95 OBJ: Identify contemporary ethical issues in nursing practice. MSC: CPNRE: Professional, Ethical, and Legal Practice
TOP: Implementation
16. During a severe respiratory epidemic, the local health care organizations decide to give health
care providers priority access to ventilators over other members of the community who also need that resource. Which philosophy would give the strongest support for this decision? a. Feminist ethics. b. Utilitarianism. c. Deontology. d. Ethics of care. ANS: B
Focusing on the greatest good for the most people, the organizations decide to ensure that as many health care workers as possible will survive to care for other members of the community. DIF: Understand REF: 91 OBJ: Identify contemporary ethical issues in nursing practice.
TOP: Implementation
Canadian Fundamentals of Nursing 6th Edition Potter Test Bank MSC: CPNRE: Professional, Ethical, and Legal Practice 17. How are determinations regarding quality of life characterized? a. They are based on a person’s ability to act according to ethical principles. b. They are based on a patient’s self-determination. c. They are value judgements that can vary from person to person. d. They are consistent and stable over the course of one’s lifetime. ANS: C
Determinations regarding quality of life are value judgements, which are based on what individuals believe is desirable. Beliefs about what people find desirable vary from person to person. Determinations regarding quality of life are not based on a person’s ability to act according to ethical principles or on a patient’s self-determination, and they may change over the course of one’s lifetime. DIF: Understand REF: 95 OBJ: Identify contemporary ethical issues in nursing practice. MSC: CPNRE: Professional, Ethical, and Legal Practice
TOP: Implementation
18. The nurse is caring for a patient who has been unresponsive since arrival via ambulance 8
days ago. The patient has not been identified, and no family members have been found. The nurse is concerned about the plan of care regarding maintenance or withdrawal of life support measures. The nurse determines that this is an ethical dilemma not resolved by scientific data. To resolve this ethical dilemma, which sequence of the following steps is correct? 1. The nurse identifies possible solutions or actions to resolve the dilemma. 2. The nurse reviews the medical record, including entries by all health care disciplines, to gather information relevant to this patient’s situation. 3. Health care providers useNnUeR goS tiI atN ioG nT toBre.dC efO inM e the patient’s plan of care. 4. The nurse evaluates the plan and revises it with input from other health care providers as necessary. 5. The nurse arranges a meeting with health care team members to clarify opinions, values, and facts. 6. The nurse states the problem. a. 2, 5, 6, 1, 3, 4. b. 5, 3, 2, 1, 6, 4. c. 6, 5, 2, 1, 3, 4. d. 2, 6, 5, 3, 1, 4. ANS: A
Once the nurse determines that an ethical dilemma exists, the nurse then uses the steps of processing an ethical dilemma to gather information relevant to the case; meets with members of the health care team to clarify values and distinguishes between fact, opinion, and values; and verbalizes the problem. Then the nurse identifies possible solutions or actions, works with the health care team to negotiate a plan, and evaluates the plan over time. DIF: Apply REF: 92-95 OBJ: Apply a method of ethical analysis to a clinical situation. TOP: Assessment | Diagnosis | Evaluate | Implementation | Planning MSC: CPNRE: Professional, Ethical, and Legal Practice
Canadian Fundamentals of Nursing 6th Edition Potter Test Bank
Chapter 08: Legal Implications in Nursing Practice Potter et al: Canadian Fundamentals of Nursing, 6th Edition MULTIPLE CHOICE 1. A newly hired experienced nurse is preparing to change a patient’s abdominal dressing and
hasn’t done it before at this hospital. Which action by the nurse is best? a. Ask another nurse to do it so the correct method can be viewed. b. Check the policy and procedure manual for the agency’s method. c. Change the dressing using the method taught in nursing school. d. Ask the patient how the dressing change has been recently done. ANS: B
The Canadian Council on Health Services requires accredited hospitals to have written nursing policies and procedures. These internal standards of care are specific and need to be accessible on all nursing units. For example, a policy/procedure outlining the steps to follow in changing a dressing or administering medication provides specific information about how nurses are to do it. The nurse being observed may not be doing the procedure according to the agency’s policy or procedure. The procedure taught in nursing school may not be consistent with the policy or procedure for this agency. The patient is not responsible for maintaining the standards of practice; patient input is important, but it’s not what directs nursing practice. DIF: Apply TOP: Planning
REF: 103 OBJ: List sources for standards of care for nurses. MSC: CPNRE: Professional, Ethical, and Legal Practice
2. A new nurse at a health care unit notes that a listing of patient names is kept in a closed book
N R I G B.C M
behind the front desk of the nuU rsinS g staNtionTso thaO t patients can be located easily. What action is most appropriate for the nurse to take? a. Move the book to the upper ledge of the nursing station for easier access. b. Talk with the nurse manager about the listing being a violation of confidentiality. c. Use the book as needed while keeping it away from individuals not involved in patient care. d. Ask the nurse manager to move the book to a more secluded area. ANS: C
Patients are entitled to confidentiality in health care. Nursing standards for what constitutes confidential information are based on professional ethics and the common law. The nurse’s judgement must be guided by the ideals of privacy and sensitivity to the needs and rights of patients who may not choose to have nurses intrude on their lives but who depend on nurses for their care. The nurse's fiduciary duty requires that confidential information not be shared with anyone else except on a need-to-know basis. The book is located where only staff would have access. It is not the responsibility of the new nurse to move items used by others on the patient unit. The listing’s confidentiality is protected as long as it is used appropriately as needed to provide care. There is no need to move the book to a more secluded area. DIF: Apply REF: 103 | 104 OBJ: Describe the legal responsibilities and obligations of nurses. TOP: Implementation MSC: CPNRE: Professional, Ethical, and Legal Practice
Canadian Fundamentals of Nursing 6th Edition Potter Test Bank 3. The law that would best deal with a nurse accidentally administering an incorrect dose of
morphine sulphate to a patient is which of the following? a. Civil law. b. Criminal law. c. Human rights law. d. Statute law. ANS: A
Civil laws protect the rights of individuals. Accidental administration of an incorrect dose of morphine sulphate would fall under civil law because it could cause harm to an individual. Criminal law aims to protect society from harm and provide punishment for intentional crimes (often imprisonment). Human rights law relates to public matters, not individual matters. Statute law is created by parliamentary, provincial, and territorial legislatures. DIF: Apply REF: 101 | 102 OBJ: Define legal aspects of nurse–patient, nurse–physician, nurse–nurse, and nurse–employer relationships. TOP: Implementation MSC: CPNRE: Professional, Ethical, and Legal Practice 4. The nurse has just obtained the licence to practise and is determining whether individual
malpractice insurance is necessary. Which of the following is the most important factor in the nurse’s decision to carry malpractice insurance? a. The amount of the malpractice insurance provided by the employer. b. Working in a critical area of nursing in which morbidity and mortality rates are high. c. Employment status and professional liability coverage. d. The nurse’s knowledge level of Good Samaritan laws. ANS: C
NURSINGTB.COM
All nurses should have clear knowledge of their employment status and professional liability coverage. Publicly funded health care facilities carry malpractice insurance. The facility is considered the employer and is liable for the negligent acts of its employees as long as their actions were within their scope of practice. The amount of the malpractice insurance provided by the employer is not the most important factor in deciding about private insurance. In general, the employer’s malpractice insurance coverage is much greater than private insurance coverage. The area of nursing is not the most important factor in deciding whether to carry malpractice insurance; lawsuits can occur anywhere. The nurse should be aware of Good Samaritan laws, but this would not ensure sufficient coverage for most nursing practice; therefore, it is not the most important factor in determining whether to purchase private malpractice insurance. DIF: Apply TOP: Planning
REF: 107 OBJ: Explain legal concepts that apply to nurses. MSC: CPNRE: Professional, Ethical, and Legal Practice
5. A nurse performs cardiopulmonary resuscitation (CPR) on a 92-year-old patient with brittle
bones and breaks a rib during the procedure, which then punctures a lung. The patient recovers completely without any residual problems and sues the nurse for pain and suffering and for malpractice. What key point will the prosecution attempt to prove? a. The CPR procedure was done incorrectly. b. The patient would have died if nothing was done. c. The patient was resuscitated according to policy.
Canadian Fundamentals of Nursing 6th Edition Potter Test Bank d. Patients with brittle bones might sustain fractures when chest compressions are
done. ANS: A
Certain criteria are necessary to establish nursing malpractice. In this situation, although harm was caused, it was not because of failure of the nurse to perform a duty according to standards the way other nurses would have performed in the same situation. If the nurse had done the procedure incorrectly, the patient probably would not have survived without any residual problems such as brain damage. The fact that the patient sustained injury as a result of age and physical status does not mean the nurse breached any duty to the patient. The nurse would need to make sure the defence attorney knew that the cardiopulmonary resuscitation (CPR) was done correctly and that without intervention, the patient probably would not have survived. The prosecution would try to prove that a breach of duty had occurred, which had caused injury, not that cardiopulmonary resuscitation was done correctly. The defence team, not the prosecution, would explain the correlation between brittle bones and rib fractures during CPR. DIF: Understand REF: 104 | 105 TOP: Implementation
OBJ: List the elements needed to prove negligence. MSC: CPNRE: Professional, Ethical, and Legal Practice
6. A recent immigrant who does not speak English is alert and requires hospitalization. What is
the initial action that the nurse must take to obtain informed consent? a. Ask a family member to translate what the nurse is saying. b. Notify the health care provider that the patient doesn’t speak English. c. Request an official interpreter to explain the terms of consent. d. Use hand gestures and medical equipment while explaining in English. ANS: C
NURSINGTB.COM
An official interpreter must be present to explain the terms of consent to a patient who speaks only a foreign language. A family member or acquaintance who speaks a patient’s language should not interpret health information. Family members can tell those caring for the patient what the patient is saying, but privacy regarding the patient’s condition, assessment, and other medical matters must be protected. There is no way to confirm that the family member is translating exactly what the nurse is saying. Privacy must be ensured and accurate information must be provided to the patient. After consent is obtained for treatment, the health care provider would be notified because little can be done without consent. The health care provider needs to have the translator available during the history and physical examination, as well as at other times, but the first step is to get a translator to obtain informed consent because this is not an emergency situation. Using hand gestures and medical equipment is inappropriate when communicating with a patient who does not understand the language spoken because (1) certain hand gestures may be acceptable in one culture and not appropriate in another, (2) the medical equipment may be unknown and frightening to the patient, and (3) the patient still does not understand what is being said. DIF: Apply REF: 106 OBJ: Give examples of legal issues that arise in nursing practice. TOP: Implementation MSC: CPNRE: Professional, Ethical, and Legal Practice 7. A pediatric oncology nurse is floated to an orthopedic trauma unit. What actions should the
nurse manager of the orthopedic unit take to enable this floated nurse to give safe care? a. Provide a complete orientation to the functioning of the entire unit.
Canadian Fundamentals of Nursing 6th Edition Potter Test Bank b. Determine acuity of patients’ conditions and the care the nurse can safely provide. c. Allow the nurse to choose which mealtime she would like. d. Assign unregulated care providers to assist her with care. ANS: B
Nurses who float need to inform the supervisor of any lack of experience in caring for the type of patients on the nursing unit. They also need to request and receive a basic orientation to the unit. Supervisors are liable if they give a staff nurse an assignment that he or she cannot safely handle. Before accepting employment, the nurse must learn the policies of the institution with regard to floating and must have an understanding of what is expected of a floating nurse. A complete orientation of the functioning of the entire unit would take a period of time that would exceed what the nurse has to spend on orientation. Allowing the nurse to choose which mealtime she would like is a nice gesture of thanks for the nurse, but it does not enable safe care. Unregulated care providers may help the nurse complete basic tasks such as hygiene and turning, but their help does not enable safe nursing care for which the nurse is ultimately responsible. DIF: Apply REF: 108 OBJ: Give examples of legal issues that arise in nursing practice. TOP: Implementation MSC: CPNRE: Professional, Ethical, and Legal Practice 8. An unconscious patient with a head injury needs immediate life-saving surgery. His wife
speaks only French, and the health care providers, who are not fluent in French, are having a difficult time explaining his condition to her. In this situation, what must the nurse know? a. Two licensed health care workers should witness and sign the preoperative consent form indicating that they heard an explanation of the procedure given in English. b. An ethical review board must be contacted to give its emergency advice on the N R I G B.C M U S N T O situation. c. A friend of the family may act as an interpreter, but the explanation cannot contain details of the patient’s accident because of confidentiality laws. d. The health care team should continue with the surgery after providing information in the best manner possible. ANS: D
In emergency situations, if it is impossible to obtain consent from the patient or an authorized person, the beneficial or life-saving procedure may be undertaken without liability for failure to obtain consent. In such cases, according to the law, the health care team would assume that the patient would wish to be given the treatment. This is referred to as the emergency doctrine. Two witnesses are required usually when telephone consents are involved. This is not the case in this situation. In an emergency, it is not necessary to contact the institutional review board; doing so would take up valuable time. A family member or acquaintance who is able to speak the patient’s language should not be used to interpret health care information. An official interpreter must be available to explain the terms of consent (except in an emergency situation). DIF: Analyze REF: 106 | 107 OBJ: Give examples of legal issues that arise in nursing practice. TOP: Implementation MSC: CPNRE: Professional, Ethical, and Legal Practice
Canadian Fundamentals of Nursing 6th Edition Potter Test Bank 9. Because of an influenza epidemic among the nursing staff, a nurse has been moved from the
ophthalmology unit to the general surgery unit. The supervisor recognizes that the nurse is inexperienced in this specialty. What should the nurse do? a. Politely refuse to move, take the day off, and go home. b. Ask to work with an experienced general surgery nurse. c. Submit a report noting their dissatisfaction. d. Notify the nursing regulatory body of the issue. ANS: B
Nurses who are temporarily reassigned to another unit (i.e., float) should inform the supervisor of their lack of experience in caring for patients on a particular nursing unit. They also should request, and be given, basic orientation to the unit. Asking to work with an experienced general surgery nurse would be an appropriate action. Refusing to accept an assignment may be considered insubordination, and patients will suffer if the number of available staff drops. The nurse can make a written protest to nursing administrators, but it should not be the nurse’s initial recourse. Notifying the nursing regulatory body should not be the nurse’s initial recourse. The nurse should first notify the supervisor and request appropriate orientation and training. DIF: Apply REF: 108 OBJ: Give examples of legal issues that arise in nursing practice. TOP: Implementation MSC: CPNRE: Professional, Ethical, and Legal Practice 10. A confused patient with a urinary catheter, nasogastric tube, and intravenous line keeps
touching these items, which are needed for care. The nurse has tried to explain to the patient that he should not touch them, but the patient continues. What is the best action by the nurse at this time? NURSINGTB.COM a. Apply restraints loosely on the patient’s dominant wrist. b. Try other approaches to prevent the patient from touching these care items. c. Notify the health care provider that restraints are needed immediately to maintain the patient’s safety. d. Allow the patient to pull out lines to prove that the patient needs to be restrained. ANS: B
The risks associated with the use of restraints are serious. With regard to restraints, a restraint-free environment is the first goal of care for all patients. Many alternatives to the use of restraints are available, and the nurse should try all of them before notifying the patient’s health care provider. In this situation, the patient is touching the items, not trying to pull them out; therefore, at this time, the patient’s well-being is not at risk. The nurse will have to check on the patient frequently and then will determine whether the health care provider needs to be informed of the situation. The inappropriate or unjustified use of restraints (e.g., by confining a person to an area, or by using physical or chemical restraints) may also be viewed as false imprisonment. The health care provider needs to know the situation but also needs to know that all approaches possible have been used before writing an order for restraints. Allowing the patient to pull out any of these items could cause harm to the patient. DIF: Apply REF: 104 OBJ: Give examples of legal issues that arise in nursing practice. TOP: Implementation MSC: CPNRE: Professional, Ethical, and Legal Practice
Canadian Fundamentals of Nursing 6th Edition Potter Test Bank 11. A nurse is working with a physician who provides medical assistance in dying (MAID) in
Canada. What must the nurse be aware of? a. The nurse can provide MAID to a patient by administering a substance prescribed by the physician. b. The patient requesting MAID must be at least 16 years of age and capable of making health care decisions. c. The patient’s natural death must be unforeseeable to be able to request MAID. d. The patient requesting MAID must be eligible for publicly funded health services in Canada. ANS: D
The patient requesting MAID must be eligible for publicly funded health services in Canada. Only a physician or nurse practitioner can provide a person with MAID by administering a substance that causes death. The patient requesting MAID must be at least 18 years of age and capable of making health care decisions. The patient’s natural death must be reasonably foreseeable, in view of all of the person’s medical circumstances. DIF: Apply REF: 110, Box 8-2 OBJ: Describe the legal responsibilities and obligations of nurses. TOP: Planning MSC: CPNRE: Professional, Ethical, and Legal Practice 12. The nurse is aware that when caring for patients with communicable diseases, such as severe
acute respiratory syndrome (SARS) and acquired immunodeficiency syndrome (AIDS), of which of the following? a. Every health care worker who comes into contact with a patient has the right to know the patient’s HIV infection status. b. They can refuse to care for a patient with SARS or AIDS if there is a lack of NsR I son GT B.Cbeing. M resources or ongoing threatU toSperN al well-O c. Nurses are responsible for providing their own personal protective equipment to care for patients with communicable diseases. d. In known cases of AIDS or SARS, information can be disclosed without the patient’s consent. ANS: B
Nurses are able to refuse to provide care when they experience an unreasonable burden. An unreasonable burden exists when the nurse’s ability to provide safe care and meet the standards of practice is compromised by unreasonable expectations, lack of resources, or ongoing threats to personal well-being. Not every health care worker who comes in contact with a patient needs to know the patient’s HIV status. Confidential information must be protected. Strict compliance with standard precautions or routine practices and the use of transmission-based (e.g., airborne or droplet) precautions for patients known to have or suspected of having serious communicable illnesses is the nurse’s wisest strategy; however, it is not the nurse’s responsibility to provide the personal protective equipment. The employer has an obligation to provide its employees with necessary protective gear. Whenever information about a patient is requested by any third parties, including insurance companies or employers, nurses must obtain a signed release from the patient before releasing confidential information. DIF: Apply REF: 109 TOP: Implementation
OBJ: List sources for standards of care for nurses. MSC: CPNRE: Professional, Ethical, and Legal Practice
Canadian Fundamentals of Nursing 6th Edition Potter Test Bank 13. A nursing student has been written up several times for being late with providing patient care
and for omitting aspects of patient care and not knowing basic procedures that were taught in the skills course one term earlier. The nursing student says, “I don’t understand what the big deal is. As my instructor, you are there to protect me and make sure I don’t make mistakes.” What is the best response from the nursing instructor? a. “You are expected to perform at the level of a professional nurse.” b. “You are expected to perform at the level of a nursing student.” c. “You are practicing under the licence of the nurse assigned to the patient.” d. “You are expected to perform at the level of a skilled nursing assistant.” ANS: A
Although nursing students are not employees of the health care agency where they are having their clinical experience, they are expected to perform as professional nurses would in providing safe patient care. Different levels of standards do not apply. Nursing students, like nurses, must provide safe, complete patient care. No standard is used for nursing students, other than that they must meet the standards of a professional nurse. The nursing instructor, not the nurse assigned to the patient, is responsible for the actions of the nursing student. DIF: Apply REF: 107 OBJ: Describe the legal responsibilities and obligations of nurses. TOP: Implementation MSC: CPNRE: Professional, Ethical, and Legal Practice 14. Which of the following is an example of care in which a nurse may be liable for actions that
constitute an unintentional tort? a. Physical restraining a patient who refuses care. b. Taking photos of a patient’s surgical wounds without the patient’s permission. c. The patient falling and being injured as a result of side rails being left down. Nistor RSyIoN G B.C M d. Talking about a patient’s hU f sexTually trOansmitted infections. ANS: C
An unintentional tort is an unintended wrongful act against another person that produces injury or harm. An example of an unintentional tort would be leaving the side rails down, which causes the patient’s fall and injury. Physically restraining a patient who refuses care would be an example of assault and battery. Taking photos of a patient’s surgical wounds without the patient’s permission and talking about a patient’s history of sexually transmitted infections are examples of invasion of privacy. Personal information about the patient should be kept confidential. DIF: Analyze REF: 104 | 105 OBJ: Describe the legal responsibilities and obligations of nurses. TOP: Assessment MSC: CPNRE: Professional, Ethical, and Legal Practice 15. A nursing student in the final term of nursing school is overheard by a nursing faculty
member telling another student that she got to insert a nasogastric tube in the emergency department while she was working as a nursing assistant. What advice is best for the nursing faculty member to give to the nursing student? a. “Just be careful when you are doing new procedures and make sure you are following directions by the nurse.” b. “Review your procedures before you go to work, so you will be prepared to do them if you have a chance.” c. “The nurse should not have allowed you to insert the nasogastric tube because
Canadian Fundamentals of Nursing 6th Edition Potter Test Bank something bad could have happened.” d. “You are not allowed to perform any procedures other than those in your job
description as a nursing assistant, even with the nurse’s permission.” ANS: D
When nursing students work as nursing assistants or nurse’s aides when not attending classes, they should not perform tasks that do not appear in a job description for a nurse’s aide or assistant. The nursing student should always follow the directions of the nurse, unless doing so violates the institution’s guidelines or job description under which the nursing student was hired. The nursing student should be able to safely complete the procedures delegated as a nursing assistant, and reviewing those not done recently is a good idea, but it has nothing to do with the situation. This option does not address the situation that the nursing student acted outside the job description for the nursing assistant position. The focus of the discussion between the nursing faculty member and the nursing student should be on following the job description under which the nursing student is working. DIF: Apply REF: 107 TOP: Implementation
OBJ: List sources for standards of care for nurses. MSC: CPNRE: Professional, Ethical, and Legal Practice
16. The nurse calculates the medication dose for an infant on the pediatric unit and determines
that the dose is twice what it should be. The pediatrician is contacted and says to administer the medication as ordered. What is the next action that the nurse should take? a. Notify the nursing supervisor. b. Give the medication as ordered. c. Give the amount calculated to be correct. d. Contact the pharmacy for clarification. ANS: A
NURSINGTB.COM
Nurses follow health care providers’ orders unless they believe the orders are in error or may harm patients. Therefore, the nurse needs to assess all orders. If an order seems to be erroneous or harmful, further clarification from the health care provider is necessary. If the health care provider confirms an order and the nurse still believes that it is inappropriate, the nurse should inform the supervising nurse. The supervising nurse should be able to help resolve the questionable order, but only the health care provider who wrote the order or a health care provider covering for the one who wrote the order can change the order. Harm to the infant could occur if the medication dosage is too high. The nurse cannot change an order. Giving the amount calculated to be correct would not be what another nurse would do in the same situation. Although the pharmacy is an excellent resource, only the health care provider can change the order. DIF: Apply REF: 108 OBJ: Define legal aspects of nurse–patient, nurse–physician, nurse–nurse, and nurse–employer relationships. TOP: Implementation MSC: CPNRE: Professional, Ethical, and Legal Practice 17. A nurse gives an incorrect medication to a patient without doing all of the mandatory checks,
but the patient has no ill effects from the medication. What actions should the nurse take after reassessing the patient and completing an incident report? a. Notify the health care provider of the situation. b. Document in the patient’s medical record that an incident report was filed. c. Document in the patient’s medical record why the omission occurred.
Canadian Fundamentals of Nursing 6th Edition Potter Test Bank d. Discuss what happened with all of the other nurses and staff on the unit. ANS: A
Examples of an incident include an error in technique or procedure such as failing to properly identify a patient. Institutions generally have specific guidelines to direct health care providers how to complete the incident or adverse occurrence report. The report is confidential and separate from the medical record. The nurse is responsible for providing information in the medical record about the occurrence. It is also best for the nurse to discuss the occurrence with nursing management only. The risk management department of the institution also requires complete documentation. The fact that an incident report was completed is not documented in the patient’s medical record. No discussion of why the omission in procedure occurred should be documented in the patient’s medical record. Errors should be discussed only with those who need to know, such as the health care provider, appropriate administrative personnel, and risk management. DIF: Apply REF: 111 OBJ: Give examples of legal issues that arise in nursing practice. TOP: Implementation MSC: CPNRE: Professional, Ethical, and Legal Practice 18. The nurse is providing care to a female patient who has received a diagnosis of terminal
cancer and who has a poor prognosis. When the patient’s husband comes in to the hospital cafeteria during a busy period, the nurse approaches him and openly expresses her sympathy to the man for his wife’s terminal illness. The patient’s husband bursts into tears and says that he did not know about his wife’s diagnosis. The nurse then takes him to a private area to discuss his feelings. This nurse’s action exemplifies which of the following violations? (Select all that apply.) a. Intentional tort. NURSINGTB.COM b. Unintentional tort. c. Negligence. d. Assault. ANS: A
This can be considered an intentional tort because the nurse is aware that he or she should not have breached confidentiality and that the patient’s rights were violated. The nurse can be held liable for slander as the nurse discusses private information about a patient that is overheard by others in the cafeteria. This action is also an example of invasion of privacy and occurs if a patient’s medical information is discussed without the consent of the patient. This action is not an example of negligence or of assault. DIF: Analyze REF: 103-105 OBJ: Give examples of legal issues that arise in nursing practice. TOP: Implementation MSC: CPNRE: Professional, Ethical, and Legal Practice
Canadian Fundamentals of Nursing 6th Edition Potter Test Bank
Chapter 09: Global Health Potter et al: Canadian Fundamentals of Nursing, 6th Edition MULTIPLE CHOICE 1. The nurse recognizes the terminology that applies to culture and ethnicity. Ethnicity is defined
as what? a. An appreciation for differences within another group and the promotion of respect for those differences. b. A common identity with members sharing social and cultural heritage. c. Many cultures coexisting and maintaining cultural differences. d. The belief that one’s own race or culture is more valuable than those of others. ANS: B
Ethnicity refers to a shared identity related to social and cultural heritage, including values, language, geographic space, and racial characteristics. The most important characteristic of an ethnic group is that its members feel a sense of common identity. Cultural pluralism is a perspective that appreciates another group for being different and “promotes respect for the right of others to have different beliefs, values, behaviours, and ways of life” (Racher & Annis, 2012, p. 159). Multiculturalism is regarded as a fundamental characteristic of Canadian society; many cultures coexist in Canadian society and maintain their cultural differences. Ethnocentrism is a tendency to hold one’s own race or culture as more valuable than those of others. DIF: Understand REF: 122 | 123 OBJ: Define key concepts related to health, illness, and diversity. N R: Pro INfess GTional, B.CEO M TOP: Assessment MSC: CPNURES thical, and Legal Practice 2. Prevention programs for populations is the main focus of which of the following? a. Global health. b. Public health. c. International health. d. Transcultural health. ANS: B
The main focus of public health is prevention programs for populations. Global health and international health embrace both prevention in populations and clinical care of individuals. Transcultural health is not a domain of medical health. DIF: Understand REF: 116, Table 9-1 OBJ: Differentiate between global health, international health, and public health in the context of professional nursing. TOP: Assessment MSC: CPNRE: Foundations of Practice 3. In understanding health inequity, what does the nurse know? a. Health differences are unavoidable. b. Poverty is not a root cause of health inequity. c. Health inequity is the absence of systematic disparities in health. d. Health inequity refers to unnecessary and unfair differences in health. ANS: D
Canadian Fundamentals of Nursing 6th Edition Potter Test Bank Health inequities are differences in health that are not only unnecessary and avoidable but also are considered unfair and unjust. Health differences are seen as avoidable. Poverty is often a root cause of health inequity. The absence of systematic disparities in health is characteristic of health equity. DIF: Remember REF: 117 OBJ: Define key concepts related to health, illness, and diversity. TOP: Assessment MSC: CPNRE: Professional, Ethical, and Legal Practice 4. When asked to describe the differences between ethnicity and race, what should the student
nurse explain? a. Ethnicity refers to a shared identity, whereas race is limited to biological attributes. b. Ethnicity and race are actually the same and are based in cultural norms. c. Ethnicity can be understood only through an etic world view. d. Race refers to a shared identity, whereas ethnicity is limited to biological attributes. ANS: A
Ethnicity refers to a shared identity related to social and cultural heritage such as values, language, geographical space, and racial characteristics. Ethnicity is different from race, which is limited to the common biological attributes shared by a group such as skin colour or blood type. In any intercultural encounter, there is an insider or native perspective (emic world view) and an outsider’s perspective (etic world view). Ethnicity is best understood by those who are a part of that ethnicity and have an emic worldview. DIF: Understand REF: 122 | 123 OBJ: Define key concepts related to health, illness, and diversity. TOP: Assessment
NNURRES MSC: CP : PI roN feG ssT ioB na. l,C EO thM ical, and Legal Practice
5. The nurse learns about cultural issues involved in the patient’s health care belief system and
enables patients and families to achieve meaningful and supportive care. Such care is known as what? a. Ethnocentrism. b. Culturally competent care. c. Cultural imposition. d. Culturally congruent care. ANS: B
Culturally competent care reflects the ability of a nurse to bridge cultural gaps in caring and enables patients and families to achieve meaningful and supportive caring. It is a step toward reaching culturally congruent care. Culturally congruent care, or care that fits the person’s valued life patterns and set of meanings, is the goal of transcultural nursing. Ethnocentrism is a tendency to hold one’s own way of life as superior to those of others. It is the cause of biases and prejudices. Cultural imposition is the use of one’s own values and lifestyles as the absolute guide in dealing with patients and interpreting behaviours. DIF: Remember REF: 123 OBJ: Describe the historical development of the concept of culture, cultural competence, cultural safety, and cultural humility in relation to nursing practice. TOP: Assessment MSC: CPNRE: Professional, Ethical, and Legal Practice
Canadian Fundamentals of Nursing 6th Edition Potter Test Bank 6. The nurse is caring for an Indigenous patient who has had recent surgery. In the patient’s
culture, it is a sign of weakness to complain of pain. In the nurse’s culture, people who are having pain ask for pain medicine. The nurse has assumed that the patient has not been having pain and does not need medication because he has not complained of pain. What is the nurse doing? a. Utilizing cultural imposition by not asking the patient about his pain. b. Striving to provide culturally congruent care by allowing the patient to suffer. c. Operating from an emic worldview of the patient’s cultural beliefs. d. Practising discrimination by not giving the patient pain medicine. ANS: A
Health care practitioners who have cultural ignorance or cultural blindness about differences generally resort to cultural imposition and use their own values and lifestyles as the absolute guide in dealing with patients and interpreting their behaviours. Culturally competent care is the care provided by the nurse who attempts to bridge cultural gaps in caring, work with cultural differences, and enable patients and families to achieve meaningful and supportive caring. The nurse in this case has not been able to do this. Any intercultural encounter consists of an inside or native perspective (emic worldview) and an outsider’s perspective (etic worldview). The nurse is obviously utilizing an etic worldview. The nurse did not purposefully ignore the patient’s need. DIF: Apply REF: 123 OBJ: Analyze components of cultural assessment critical to understanding the values, beliefs, and practices critical in the nursing care of people experiencing cultural transitions. TOP: Implementation MSC: CPNRE: Professional, Ethical, and Legal Practice 7. When a cultural assessment is performed, knowledge of a patient’s country of origin and its
NUisRkSnow INnGaTs B.C history and ecological contexts what?O M a. Ethnohistory. b. Biocultural history. c. Social organization. d. Religious and spiritual beliefs. ANS: A
A patient’s country of origin and its history and ecological contexts is known as ethnic heritage and ethnohistory, and knowledge about it is significant in health care. Biocultural history can help identify a patient’s health risks in relation to the ecological context of the culture. Social organization refers to units of organization in a cultural group defined by kinship status and appropriate roles for their members. Religious and spiritual beliefs are major influences in the patient’s world view about health and illness, pain and suffering, and life and death. Nurses need to understand the emic perspective of their patients. DIF: Remember REF: 126 | 127 OBJ: Analyze components of cultural assessment critical to understanding the values, beliefs, and practices critical in the nursing care of people experiencing cultural transitions. TOP: Assessment MSC: CPNRE: Professional, Ethical, and Legal Practice 8. A nursing student is caring for a patient who has just immigrated to Canada from Ghana. The
student just had a class on cultural safety and wants to practise doing a cultural assessment. What should the nursing student know? a. Patients who come from the same region or country share similar values, beliefs,
Canadian Fundamentals of Nursing 6th Edition Potter Test Bank attitudes, and experiences. b. Cultural assessment is a systematic and comprehensive examination of the cultural
care values, beliefs, and practices of individuals, families and communities. c. The patient should be discouraged from sharing personal stories because it will
take up too much time. d. The student should ask only open-ended questions. ANS: B
Cultural assessment is a systematic and comprehensive examination of the cultural care values, beliefs, and practices of individuals, families and communities. Not all patients who come from the same region or country share similar values, beliefs, attitudes, and experiences. The patient should be encouraged to share personal stories to reveal how he or she thinks and the cultural lifestyle that he or she embraces. The student should use open-ended questions, focused questions, and contrast questions. DIF: Apply REF: 126 OBJ: Analyze components of cultural assessment critical to understanding the values, beliefs, and practices critical in the nursing care of people experiencing cultural transitions. TOP: Implementation MSC: CPNRE: Professional, Ethical, and Legal Practice 9. The nurse is caring for a patient who has emigrated from another country. The patient needs
abdominal surgery but seems reluctant to sign the surgical permits. What is one tactic that the nurse should use? a. Determine the family social hierarchy. b. Encourage the patient to sign the permits. c. Call the physician so that surgery can be cancelled. d. Impress on the patient that her life is in jeopardy. ANS: A
NURSINGTB.COM
Nurses should determine the family social hierarchy as soon as possible to prevent offending patients and their families. Working with established family hierarchy prevents delays and achieves better patient outcomes. Encouraging the patient to sign against her social beliefs can cause familial strife. Explaining the level of jeopardy may create undue stress. Nurses should be able to determine the correct hierarchy and should not involve the physician at this time. DIF: Apply REF: 126 OBJ: Analyze components of cultural assessment critical to understanding the values, beliefs, and practices critical in the nursing care of people experiencing cultural transitions. TOP: Implementation MSC: CPNRE: Professional, Ethical, and Legal Practice 10. Which of the following is an outcome of nursing education that enables safe service to be
defined by those who receive the service? a. Cultural awareness. b. Cultural safety. c. Cultural sensitivity. d. Self-awareness. ANS: B
Canadian Fundamentals of Nursing 6th Edition Potter Test Bank Cultural safety is an outcome of nursing education that enables safe service to be defined by those who receive the service. Cultural safety involves considering the redistribution of power and resources in a relationship. Cultural awareness is a beginning step toward understanding that there are differences between cultures. Cultural sensitivity alerts nurses to the legitimacy of difference and begins a process of self-exploration. Self-awareness is the result of reflection. DIF: Understand REF: 124 OBJ: Define key concepts related to health, illness, and diversity. TOP: Assessment MSC: CPNRE: Professional, Ethical, and Legal Practice 11. A nursing student is doing a cultural assessment of a patient. When the student asks, “Who
makes the decisions for you or your family?” what is the nurse assessing? a. Language and communication. b. Caring beliefs and practices. c. Socioeconomic status. d. Social organization. ANS: D
The question “Who makes the decisions for you or your family?” would be used to assess social organization during the cultural assessment. The question does not address caring beliefs and practices, socioeconomic status, or language and communication. DIF: Apply REF: 127 OBJ: Analyze components of cultural assessment critical to understanding the values, beliefs, and practices critical in the nursing care of people experiencing cultural transitions. TOP: Assessment MSC: CPNRE: Professional, Ethical, and Legal Practice
NURSINGTB.COM
12. Members of a Chinese family that has been in Canada for 5 years have learned to speak
English and have adopted certain Western customs but have continued to adhere to their values, beliefs, and traditions. What is this process referred to as? a. Assimilation. b. Enculturation. c. Acculturation. d. Multiculturalism. ANS: C
Acculturation is the process of adapting to or adopting the characteristics of a new culture. Assimilation is a process whereby a minority group gradually acquires the attitudes and customs of the mainstream culture. Socialization into one’s primary culture as a child is known as enculturation. Multiculturalism—the coexistence of many cultures and the maintenance of cultural differences—is prevalent in Canadian society. DIF: Understand REF: 123 OBJ: Analyze components of cultural assessment critical to understanding the values, beliefs, and practices critical in the nursing care of people experiencing cultural transitions. TOP: Assessment MSC: CPNRE: Professional, Ethical, and Legal Practice 13. When caring for a patient of a different culture, it is important for the nurse to understand
which of the following? a. The nurse should protect the patient from family intrusion in her health care
Canadian Fundamentals of Nursing 6th Edition Potter Test Bank decisions. b. Working within the established family hierarchy produces better outcomes. c. Women as primary caregivers make independent health decisions. d. Gender is not a factor when it comes to role expectations. ANS: B
Working with established family hierarchy prevents delays and achieves better patient outcomes. Nurses need to determine who has authority for making decisions within the family and how to communicate with the proper individuals. The nurse must not assume that just because a woman is the primary caregiver, she will make decisions independently. The nurse should determine the family social hierarchy as soon as possible. Gender may also differentiate role expectations. DIF: Apply REF: 126 OBJ: Analyze components of cultural assessment critical to understanding the values, beliefs, and practices critical in the nursing care of people experiencing cultural transitions. TOP: Planning MSC: CPNRE: Professional, Ethical, and Legal Practice 14. Time takes on different meanings from one culture to another. Understanding this, when
planning nursing interventions, what should the nurse do? a. Avoid using set times for procedures. b. Mutually negotiate time schedules with patients. c. Encourage patients to set their own times for care, regardless of the schedule. d. Maintain the set times for treatments and inform patients of the schedule. ANS: B
Differences exist in the dimensions of time that cultures emphasize and how time is expressed. Patients’ access toNhe alth sm .aCyObMe achieved through time schedules that are UR SIserv NGice TB mutually negotiated, so as to allow for cultural patterns to be respected. For organizational purposes, the nurse should seek the patient’s input, and together the nurse and the patient may set a time to perform procedures. Although the patient’s input should be sought, it is not realistic to have patients set their own times for nursing care activities regardless of the schedule. Some procedures may be required more frequently than the patient would set, or the nurse may be unable to meet the needs of several patients on the unit at the same time. Maintaining set times for treatments and informing the patient of the schedule does not take into consideration the patient’s time orientation. DIF: Apply REF: 127 | 128 OBJ: Analyze components of cultural assessment critical to understanding the values, beliefs, and practices critical in the nursing care of people experiencing cultural transitions. TOP: Implementation MSC: CPNRE: Professional, Ethical, and Legal Practice 15. What characterizes culturally congruent care? a. It fits the patient’s valued life patterns and set of meanings. b. It is based on meanings generated by predetermined criteria. c. It is the same as the values of the professional health care system. d. It is based on the assumption that a person’s own way of life as superior to those of
other people. ANS: A
Canadian Fundamentals of Nursing 6th Edition Potter Test Bank The goal of transcultural nursing is culturally congruent care, or care that fits the person’s valued life patterns and set of meanings. Patterns and meanings are generated from people themselves, rather than from predetermined criteria. Culturally congruent care is sometimes different from the values and meanings of the professional health care system. Ethnocentrism is a tendency to hold one’s own way of life as superior to those of others, and it is not part of culturally congruent care. DIF: Understand REF: 123 OBJ: Apply research findings to the provision of culturally competent care with considerations for cultural safety and relational practice. TOP: Assessment MSC: CPNRE: Professional, Ethical, and Legal Practice 16. The nurse may work with patients from many different cultural backgrounds. Nurses,
unfortunately and inadvertently, may impose their own cultural beliefs on patients. Which of the following is an example of a nurse imposing personal perspectives on a patient? a. Adapting the patient’s room to accommodate extra family members who are visiting. b. Seeking information on gender-congruent care for an Egyptian patient. c. Directing an older Chinese patient to do rehabilitation exercises after she has refused to do them until her daughter arrives. d. Encouraging family members to assist with the patient’s care when it is appropriate for them to do so. ANS: C
In collectivistic cultures that value group reliance and interdependence, such as traditional South Asian cultures, caring behaviours are manifested by actively providing physical and psychological support for kin members. The nurse may perceive the patient’s refusal of INcare G T,B.C M case the nurse is imposing her own exercise as lack of motivationNfUorRsS elfand inOthis belief system on the patient. Adaptation of the patient’s room to accommodate extra family members is not an example of cultural imposition on a patient but rather is meeting the patient’s need by providing culturally congruent care. Seeking information on gender-congruent care for an Egyptian patient is an example of the desire to provide culturally congruent care. Encouraging family members to assist with the patient’s care is not an example of cultural imposition on a patient. Western culture tends to follow a pattern of caring that focuses on self-care and self-determination, whereas in non-Western cultures, people typically have care provided by others. DIF: Apply REF: 123 OBJ: Apply research findings to the provision of culturally competent care with considerations for cultural safety and relational practice. TOP: Implementation MSC: CPNRE: Professional, Ethical, and Legal Practice 17. Which of the following is considered a communicable disease? a. Diabetes. b. Cardiovascular disease. c. Influenza. d. Chronic respiratory disease. ANS: C
Canadian Fundamentals of Nursing 6th Edition Potter Test Bank Influenza is contagious and can be transmitted from one person to another; therefore, it is communicable. Noncommunicable diseases are not contagious, meaning they are not passed from one individual to another. The four main types of noncommunicable diseases are diabetes, cardiovascular diseases, chronic respiratory diseases, and cancers. DIF: Understand REF: 118 OBJ: Describe key challenges in the prevention and control of communicable disease, noncommunicable disease (NCDs), and neglected tropical disease (NTDs). TOP: Planning MSC: CPNRE: Foundations of Practice
NURSINGTB.COM
Canadian Fundamentals of Nursing 6th Edition Potter Test Bank
Chapter 10: Indigenous Health Potter et al: Canadian Fundamentals of Nursing, 6th Edition MULTIPLE CHOICE 1. The nurse is aware that, historically, Indigenous communities experienced health and
well-being through which of the following? a. A holistic view. b. A biomedical view. c. A spiritual view. d. A physical view. ANS: A
Historically, Indigenous communities experienced healing and well-being through a holistic view of health, in which illness and treatment consisted of physical, emotional, mental and spiritual dimensions. DIF: Understand REF: 136 OBJ: Differentiate between Indigenous health from a global perspective and Indigenous health from a Canadian perspective TOP: Planning MSC: CPNRE: Professional, Ethical, and Legal Practice 2. The significant educational, income, health and social disparities between Indigenous people
and other Canadians is a reflection of what? a. The geographical spread of Canada’s population. b. Genetic diseases that are passed through generations. c. Physical and psychologicN alUaR buSsI e fN roGmTtB he.IC ndOiaMn residential school system. d. Infectious diseases processes. ANS: C
Over several generations, the Indian residential school system left a legacy of physical and psychological abuse that “is reflected in the significant educational, income, health and social disparities between Aboriginal people and other Canadians” (TRC, 2015b, p. 135). DIF: Understand REF: 137 OBJ: Examine the legacy of residential schools, which has had devastating consequences for Indigenous communities across Canada. TOP: Diagnosis MSC: CPNRE: Professional, Ethical, and Legal Practice 3. According to the records kept, it is estimated that at least how many children died from
malnourishment, diseases such as tuberculosis (TB), and abuse at residential schools in Canada? a. 150. b. 1500. c. 3200. d. 5000. ANS: C
Canadian Fundamentals of Nursing 6th Edition Potter Test Bank It is estimated that at least 3200 children died from malnourishment, diseases such as TB, and abuse; this number includes those who ran away and those who froze to death. The number of children who died could be 5 to 10 times higher; however, because of poor record-keeping, the full number may never be known. DIF: Remember REF: 137 OBJ: Examine the legacy of residential schools, which has had devastating consequences for Indigenous communities across Canada. TOP: Evaluate MSC: CPNRE: Professional, Ethical, and Legal Practice 4. Which of the following can be defined as the pathways by which trauma is transmitted from
one generation to the next? a. Birth trauma. b. Post-traumatic stress disorder. c. Intergenerational trauma. d. Generational traumatic disorder. ANS: C
Intergenerational trauma is defined as the pathways by which the nature of trauma is understood and experienced by Indigenous “survivors of the residential school system and their descendants,” as well as “the pathways by which this trauma is transmitted from one generation to the next” (Aguiar & Halseth, 2015, p. 23). DIF: Understand REF: 137 OBJ: Examine the legacy of residential schools, which has had devastating consequences for Indigenous communities across Canada. TOP: Evaluate MSC: CPNRE: Professional, Ethical, and Legal Practice
NURSINGTB.COM
5. The Truth and Reconciliation Commission issued its final report in 2015. It stated that Canada
was guilty of the destruction of those structures and practices that allow a group to continue as a group. Such destruction is known as which of the following? a. Structural racism. b. Cultural genocide. c. Intergenerational trauma. d. Colonialism. ANS: B
The Truth and Reconciliation Commission stated that Canada was guilty of cultural genocide with regard to Indigenous people in Canada, which is the “destruction of those structures and practices that allow the group to continue as a group” (Truth and Reconciliation Commission, 2015a, p. 3). Such destruction of practices and structures included occupying and seizing land; forcing relocation of Indigenous peoples and confining them to reserves; disempowering them through replacement of existing forms of Indigenous government; and denying them basic rights, such as the right to practise their faith, the right to assemble, and the right to legal counsel. DIF: Understand REF: 138 OBJ: Examine the Canadian history of colonization that includes pre-European and post-European contact. TOP: Assessment MSC: CPNRE: Professional, Ethical, and Legal Practice
Canadian Fundamentals of Nursing 6th Edition Potter Test Bank 6. Health care for Indigenous children living on reserves is funded through the federal
government but is regulated through provincial/territorial systems and policies. This complex funding system, resulting in denial, delay, or disruption of services, is an example of what? a. Holistic health care. b. Cultural genocide. c. Intergenerational trauma. d. Structural racism. ANS: D
The complex funding system for Indigenous children living on reserves is one example of structural racism. Structural racism is the legitimized and “normalized” spectrum of attitudes, practices, and policies that consistently result in chronic and continuous substandard outcomes for Indigenous peoples. DIF: Apply REF: 138 OBJ: Describe the concepts of structural racism, child welfare, poverty, and the justice system in relation to nursing practice. TOP: Evaluate MSC: CPNRE: Professional, Ethical, and Legal Practice 7. In comparison with non-Indigenous children, Indigenous children are what in the Canadian
child welfare system? a. Overrepresented. b. Underrepresented. c. Equally represented. d. Nonexistent. ANS: A
Across Canada, Indigenous cN hild epCrese Rren Iare Gove Brr. Mnted in the child welfare system. In some U S N T O provinces, the rate of out-of-home care for Indigenous children is more than 10 times that of non-Indigenous children. DIF: Remember REF: 140 OBJ: Describe the concepts of structural racism, child welfare, poverty, and the justice system in relation to nursing practice. TOP: Assessment MSC: CPNRE: Professional, Ethical, and Legal Practice 8. Which of the following plays a role in the incarceration rates of Indigenous people? a. Violence. b. Fetal alcohol spectrum disorder. c. Schizophrenia. d. Depression. ANS: B
Fetal alcohol spectrum disorder (FASD) plays a role in the increasing incarceration rates of Indigenous people. It is estimated that 10 to 25% of all Canadians who are incarcerated suffer from FASD. For Indigenous people, there is often a connection between residential school experience, addiction, and FASD. DIF: Apply REF: 141 OBJ: Describe the concepts of structural racism, child welfare, poverty, and the justice system in relation to nursing practice. TOP: Evaluate MSC: CPNRE: Professional, Ethical, and Legal Practice
Canadian Fundamentals of Nursing 6th Edition Potter Test Bank
9. A nurse caring for an Indigenous family is aware that globally, Indigenous peoples have much
in common in terms of world views. How is the concept of time considered? a. It is focused on minutes rather than days. b. It is focused on years rather than months. c. It is focused on seasons rather than hours. d. It is focused on lunar cycles rather than months. ANS: C
The sense of time in Indigenous world views is elastic, focused on seasons rather than hours. It is not focused on minutes rather than days, years rather than months, or lunar cycles rather than months. DIF: Understand REF: 141 OBJ: Describe components of Indigenous cultural orientations in relation to nursing practice. TOP: Planning MSC: CPNRE: Professional, Ethical, and Legal Practice 10. Indigenous youth who were adopted have shown extremely poor self-esteem as they struggle
with reconfiguring their identity. This has been expressed in high rates of which problem? a. Eating disorders. b. Suicidal ideation. c. Schizophrenia. d. Dissociative identity disorder. ANS: B
Indigenous youth who were adopted have shown extremely poor self-esteem and high rates of suicidal ideation as they struggle with reconfiguring their identity. Mental health disorders in general are more prevalent inNthi s po anOinMothers; however, suicidal ideation is R Ipula Gtion Bt.hC U S N T directly linked to the reconfiguration of identity. DIF: Apply REF: 142 OBJ: Examine selected chronic illness experiences relevant to the Indigenous peoples of Canada. TOP: Diagnosis MSC: CPNRE: Professional, Ethical, and Legal Practice 11. A process that is reciprocal, in which people are viewed and treated as inherently worthy and
equal, is a fundamental aspect of nursing practice known as which of the following? a. Respect. b. Dignity. c. Justice. d. Acceptance. ANS: A
Respect is a process that is reciprocal, in which people are viewed and treated as inherently worthy and equal in principle. Respect is demonstrated by a willingness to be accepting and to listen actively, as well as genuinely trying to understand the situations patients found themselves in. Respect addresses dignity and justice. DIF: Apply REF: 143 OBJ: Differentiate between respect, trust, and spirituality in the context of caring for Indigenous people. TOP: Planning MSC: CPNRE: Professional, Ethical, and Legal Practice 12. A nurse who is caring for an Indigenous family must be aware of which of the following?
Canadian Fundamentals of Nursing 6th Edition Potter Test Bank a. b. c. d.
All Indigenous peoples practise burning sweetgrass as a cleansing activity. Many Indigenous peoples practise cultural knowledge because of colonization. Not all Indigenous people follow traditional Indigenous faiths. Items placed in the bed or pinned to the patient’s gown may be moved as needed.
ANS: C
Not all Indigenous people follow traditional Indigenous faiths. Many Indigenous peoples have lost their cultural knowledge because of colonization and cultural genocide. Some Indigenous people burn sweetgrass as a cleansing activity, but the nurse cannot assume that all Indigenous peoples practise this activity. Items placed in the bed or pinned to the patient’s gown should not be moved as needed, as they may hold spiritual and healing significance for the patient and family. DIF: Analyze REF: 143 OBJ: Describe components of Indigenous cultural orientations in relation to nursing practice. TOP: Implementation MSC: CPNRE: Professional, Ethical, and Legal Practice 13. The nurse is documenting an admission history for an Indigenous youth. Which of the
following questions might the nurse ask, in order to assess intermediate determinants of health? a. “What did you eat in the last 24 hours?” b. “What grade are you in?” c. “Do you smoke?” d. “How much physical activity do you do each week?” ANS: B
Intermediate determinants of health include education, community infrastructure, resources and systems. Asking the youN thU abRout SIacc NGess TBto.eCduc OMation would help the nurse assess intermediate determinants of health. Diet, physical activity, and smoking are all considered proximal determinants of health. DIF: Analyze REF: 144 OBJ: Examine selected chronic illness experiences relevant to the Indigenous peoples of Canada. TOP: Assessment MSC: CPNRE: Foundations of Practice 14. Which of the following would be considered a distal determinant of health? a. Effective policing services. b. Access to walking trails. c. Eating nutritious meals. d. Exercising daily. ANS: A
Distal determinants of health have the most influence on a population’s health and encompass the historical, political, social, and economic contexts from which intermediate and proximal determinants of health are constructed (such as stable government structures, competent leadership, and effective policing services). Access to walking trails is considered an intermediate determinant of health. Eating nutritious meals and exercising daily are considered proximal determinants of health. DIF: Apply REF: 144 OBJ: Examine selected chronic illness experiences relevant to the Indigenous peoples of Canada. TOP: Evaluate MSC: CPNRE: Foundations of Practice
Canadian Fundamentals of Nursing 6th Edition Potter Test Bank
15. Which of the following groups has the highest rate of type 2 diabetes mellitus? a. Indigenous people living in an urban setting. b. Indigenous people living in a rural setting. c. Non-Indigenous people living in an urban setting. d. Non-Indigenous people living in a rural setting. ANS: B
Indigenous people living in rural areas have a higher rate of diabetes than do those living in an urban setting. Indigenous men and women have a greater risk of diabetes at a much younger age than do non-Indigenous people. DIF: Understand REF: 145 OBJ: Examine selected chronic illness experiences relevant to the Indigenous peoples of Canada. TOP: Diagnosis MSC: CPNRE: Foundations of Practice 16. In comparison with their non-Indigenous counterparts, Indigenous persons with a diagnosis of
HIV infection are more likely to be what? a. Older. b. Male. c. Infected through intravenous drug use. d. Transgender. ANS: C
In comparison with their non-Indigenous counterparts, Indigenous persons with a diagnosis of HIV infection are more likely to be younger, female, and infected through intravenous drug use. There is currently no research on the effect of HIV/AIDS on Two Spirit people (the term Two Spirit is used by IndigenNous tify URpeo SIple NGtoTiBde.nC OMa range of roles and identities that include gender, sex, and sexual identity). DIF: Understand REF: 147 OBJ: Examine selected chronic illness experiences relevant to the Indigenous peoples of Canada. TOP: Assessment MSC: CPNRE: Foundations of Practice 17. Which of the following populations has the highest rate of lung cancer in the world? a. Indigenous people who live on reserves. b. Métis who live in Quebec. c. Indigenous elders. d. The circumpolar Inuit population. ANS: D
The circumpolar Inuit population has the highest rate of lung cancer in the world. Smoking is highly prevalent among the Inuit; almost 63% of Canadian Inuit are daily smokers. The effects of colonization, residential schools, diet changes, and alterations in lifestyle have all been cited as factors in the increase of cancers among Indigenous people in Canada. DIF: Remember REF: 147 OBJ: Examine selected chronic illness experiences relevant to the Indigenous peoples of Canada. TOP: Evaluate MSC: CPNRE: Foundations of Practice
Canadian Fundamentals of Nursing 6th Edition Potter Test Bank 18. The nurse is caring for an Indigenous woman who describes having been taken away from her
family and placed in a non-Indigenous home when she was a child. What is the term for this phenomenon? a. The Child Care Act. b. Forced adoption. c. The “Sixties Scoop.” d. The Protection Act. ANS: C
The Sixties Scoop was a phenomenon that continued into the 1990s, in which children were apprehended from reservations and Indigenous families on the slightest pretext in order to “save them from the effects of crushing poverty, unsanitary health conditions, poor housing and malnutrition” (Johnson, 1983, p. 23). Many of the children who were removed from their homes and placed with non-Indigenous families did not fare well. This phenomenon is not termed the Child Care Act, forced adoption, or the Protection Act. DIF: Apply REF: 139 OBJ: Describe the concepts of structural racism, child welfare, poverty, and the justice system in relation to nursing practice. TOP: Assessment MSC: CPNRE: Professional, Ethical, and Legal Practice 19. Which of the following is an intermediate determinant of health that has contributed to health
problems in Indigenous communities? a. Change in diet. b. Sedentary lifestyle. c. Contamination of wildlife, vegetation, and water. d. Smoking. ANS: C
NURSINGTB.COM
Poor environmental stewardship, disempowerment, and contamination of wildlife, vegetation, and water have contributed to health problems. Diet changes, sedentary lifestyle, and smoking would be considered proximal determinants of health, not intermediate. DIF: Understand REF: 144 | 145 OBJ: Examine selected chronic illness experiences relevant to the Indigenous peoples of Canada. TOP: Evaluate MSC: CPNRE: Foundations of Practice
Canadian Fundamentals of Nursing 6th Edition Potter Test Bank
Chapter 11: Nursing Leadership, Management, and Collaborative Practice Potter et al: Canadian Fundamentals of Nursing, 6th Edition MULTIPLE CHOICE 1. Which of the following is a requirement for a new nurse in delegating tasks to an unregulated
care provider (UCP)? a. Obtaining the UCP’s voluntary acceptance of the task. b. Communicating the task in understandable terms. c. Functioning with a laissez-faire style of leadership. d. Always supervising the UCP. ANS: B
When delegating, the nurse should always provide unambiguous and clear directions by describing the task, the desired outcome, and the period within which the task should be completed. Tasks should be delegated to UCPs who are capable, not necessarily to those who are willing. A laissez-faire style of leadership is not a requirement for delegation. The nurse does not necessarily have to supervise the UCP, unless it is required by policy or unless the nurse is unsure of the UCP’s ability to perform the task. DIF: Understand REF: 166 | 167 OBJ: Discuss principles for the appropriate delegation of patient care activities. TOP: Implementation MSC: CPNRE: Collaborative Practice 2. To be able to meet the needs of assigned patients and the responsibilities associated with the
position, nurses must be aware of time management techniques. A time management skill for N R INGTB.COM the nurse is which of the followUingS ? a. Meeting all of the patients’ needs in the early-morning hours. b. Planning effectively and being aware of competing priorities. c. Conducting patient assessments individually at separate times throughout the day. d. Leaving each day unplanned to allow for adaptations in treatments. ANS: B
Time management skills for nurses include reflecting on how they use their time, planning effectively, and being aware of competing priorities. Learning new technologies, juggling various priorities, and responding to multiple demands are issues to which nurses are required to respond in a timely manner. Meeting all of the patients’ needs in the early-morning hours would be an unrealistic goal because (1) some activities have specific time limits in terms of addressing patient needs, and some activities follow scheduled routines according to hospital policy and (2) the nurse may also have to work around changes in schedules, such as a test that was ordered for the morning. Time management involves using patient goals as a way to identify priorities. The nurse, in reviewing the care requirements, organizes his or her time so the activities of care and patient goals can be achieved. The nurse should complete the activities started with one patient before moving on to another. Because the nurse has a limited amount of time with patients, it is essential to remain goal-oriented and make a plan for using time wisely, which might include conducting patient assessments together than at separate times of the day. DIF: Apply REF: 166 OBJ: Discuss ways to apply skills of clinical care coordination in nursing practice.
Canadian Fundamentals of Nursing 6th Edition Potter Test Bank TOP: Implementation
MSC: CPNRE: Professional, Ethical, and Legal Practice
3. Nursing management in a facility is considering a nursing care delivery model that involves
the division of tasks, with each nurse assuming the responsibility for particular tasks. What is this model referred to as? a. Total patient care. b. Functional nursing. c. Team nursing. d. Primary nursing. ANS: B
Functional nursing is task focused, not patient focused. In this model, tasks are divided, with one nurse assuming responsibility for specific tasks. In the total patient care model, a registered nurse (RN) is responsible for all aspects of care for one or more patients. The RN may delegate aspects of care but retains accountability for care of all assigned patients. In team nursing, an RN leads a team that is composed of other RNs, registered psychiatric nurses or licensed practical nurses, and unregulated care providers (UCPs). The team members provide direct patient care to groups of patients, under the direction of the RN team leader. UCPs are given patient assignments rather than being assigned particular tasks. In the primary nursing model, an RN assumes responsibility for a caseload of patients over time. Typically, the RN selects the patients for his or her caseload and cares for the same patients during their hospitalization or stay in the health care setting. DIF: Understand REF: 162 OBJ: Describe the purpose, elements, and models for quality practice environments and patient safety. TOP: Assessment MSC: CPNRE: Collaborative Practice
Ba.nC Ment philosophy from centralized to 4. The medical centre has changNeU dR itsSoIvN erG alT lm agO em decentralized management. What is one advantage of a decentralized management structure over a centralized structure for the nursing units? a. Staff members are not responsible for defining their roles. b. Nurse managers handle the difficult decisions. c. Communication pathways are simplified. d. Each staff member is actively involved. ANS: D
In decentralized management, decision making is moved down to the level of staff. The advantage of this structure is that managers and staff are actively involved in shaping an organization’s identity and determining its success. If decentralized decision making is in place, professional staff members have a voice in identifying the LPN/RPN role. Each LPN/RPN on the work team is responsible for knowing his or her role and how it is to be implemented on the nursing unit. Decentralized management is characterized by autonomy (freedom to decide and act). The nurse manager does not necessarily handle the difficult decisions; the staff members who are best informed about a problem or issue make decisions on the basis of their knowledge. Communication pathways are not necessarily simplified in a decentralized management structure. DIF: Understand REF: 163 OBJ: Describe the relationships between nursing leadership and healthy practice environments, patient safety, and quality patient care outcomes. TOP: Implementation MSC: CPNRE: Professional, Ethical, and Legal Practice
Canadian Fundamentals of Nursing 6th Edition Potter Test Bank
5. Which of the following is one of the four elements of decision making? a. Justice. b. Responsibility. c. Accessibility. d. Resource allocation. ANS: B
Responsibility is one of the four elements of decision making. The other three elements are autonomy, authority, and accountability. Justice and resource allocation are ethical principles and are not among the four elements of decision making. Accessibility is one of the five principles of the Canada Health Act. DIF: Remember REF: 164 | 165 OBJ: Identify strategies for developing entry-level competencies related to management and leadership in nursing. TOP: Planning MSC: CPNRE: Professional, Ethical, and Legal Practice 6. The charge nurse asks an experienced nurse to admit a palliative patient who arrives on the
pediatric unit. What does this action exemplify? a. Evaluation. b. Supervision. c. Assignment. d. Delegation. ANS: C
Assignment is the action taken when the patient or interventions are within the nurse’s scope of practice. The experienced N nursRe isIcapGablB ofCtakM U S N Te . Oing on this patient assignment. Delegation is transferring the responsibility to perform a function or intervention to someone who would not otherwise have the authority to perform it (i.e., the function or intervention is not within the scope of practice or scope of employment of the care provider to whom the care is being delegated). This is not an example of evaluation or supervision. DIF: Understand REF: 167 OBJ: Discuss principles for the appropriate delegation of patient care activities. TOP: Planning MSC: CPNRE: Professional, Ethical, and Legal Practice 7. The nursing management team in a facility is investigating a nursing care delivery model that
involves staff members working under the direction of an RN leader. What does this model reflect? a. Team nursing. b. Primary nursing. c. Functional nursing. d. Total patient care. ANS: A
Canadian Fundamentals of Nursing 6th Edition Potter Test Bank In team nursing, an RN leads a team that is composed of other RNs, registered psychiatric nurses or licensed practical nurses, and UCPs. The team members provide direct patient care to groups of patients, under the direction of the RN team leader. Primary nursing is a model of care delivery whereby an RN assumes responsibility for a caseload of patients over time. Typically, the RN selects the patients for his or her caseload and cares for those patients during their hospitalization or stay in the health care setting. Functional nursing is task focused, not patient focused; in a functional nursing model, tasks are divided, with each nurse assuming responsibility for specific tasks. In a total patient care model, an RN is responsible for all aspects of care for one or more patients. The RN may delegate aspects of care but retains accountability for care of all assigned patients. DIF: Understand REF: 162 OBJ: Describe the relationships between nursing leadership and healthy practice environments, patient safety, and quality patient care outcomes. TOP: Implementation MSC: CPNRE: Collaborative Practice 8. Accountability is a critical aspect of nursing care. Which of the following is an example of a
specific decision-making process of accountability? a. Selecting the medication schedule for the patient. b. Implementing discharge teaching plans that meet individual patients’ needs. c. Evaluating the patient’s outcomes after implementation of care. d. Promoting participation of all staff members in unit meetings. ANS: C
Accountability refers to being answerable for actions. It involves follow-up and a reflective analysis of a nurse’s decisions to evaluate their effectiveness. Selecting the medication schedule for the patient is an example of taking responsibility. Implementing discharge Ndua RS I ient GTs’B.C M teaching plans that meet indiviU l patN needO s is an example of autonomy. Promoting participation of all staff members in unit meetings is an example both of decentralized management and of promoting authority. DIF: Understand REF: 165 OBJ: Describe entry-level professional nurse competencies related to leadership, management, and collaborative practice. TOP: Evaluate MSC: CPNRE: Professional, Ethical, and Legal Practice 9. The student nurse is seeking to learn skills associated with priority setting. Among the
different priorities of care, which of the following is an example of an intermediate priority? a. An obstructed airway. b. The need to urinate. c. Loss of consciousness. d. Activities of daily living in the home environment. ANS: B
The need to urinate is an intermediate priority because it is a nonemergency, non–life-threatening actual or potential need. An obstructed airway and loss of consciousness are of high priority because both are an immediate threat to a patient’s survival or safety. Activities of daily living in the home environment are a fourth-order priority; they represent an actual or potential problem with which the patient or family members may need help in the future.
Canadian Fundamentals of Nursing 6th Edition Potter Test Bank DIF: Apply REF: 166 OBJ: Discuss ways to apply skills of clinical care coordination in nursing practice. TOP: Assessment MSC: CPNRE: Foundations of Practice 10. The nurse on the unit is determining which activities may be delegated to an unregulated care
provider (UCP). A number of factors are included in the nurse’s decision. Assuming that the UCP is competent, the nurse may safely delegate which of the following activities? a. Ambulation of stable patients. b. Documenting an admission history for a new patient. c. Performance of the initial transfer of a postoperative patient. d. Administering medications prepared by the nurse. ANS: A
An institution’s policies and procedures and job description for a UCP provide specific guidelines with regard to which tasks or activities can be delegated. The nurse should match tasks to the delegate’s skills, such as ambulating stable patients. An admission history for a new patient should not be prepared by a UCP; the nurse should perform this task. The initial transfer of a postoperative patient should not be delegated to a UCP, as the patient would be considered unstable; the nurse should perform this task. The nurse should not delegate medication administration to anyone, if the nurse prepared it; the UCP is not licensed to administer medication. DIF: Apply REF: 166 | 167 OBJ: Discuss principles for the appropriate delegation of patient care activities. TOP: Implementation MSC: CPNRE: Collaborative Practice 11. UCPs are allowed to perform tasks with patients in which situation? a. When tasks are delegatedNbU yR anSoI thN erGUTCBP. . COM b. When asked by the patient. c. With another UCP. d. When tasks are delegated by a nurse. ANS: D
An institution’s policies, procedures, and job descriptions for UCPs provide specific guidelines regarding which tasks or activities can be delegated. UCPs are not allowed to perform acts authorized for nurses unless these tasks have been properly delegated by a nurse, and they can perform such actions only if they are within the UCP’s job description and employer policy. It is not appropriate to perform tasks that are delegated by another UCP, when asked by the patient, or with another UCP. DIF: Understand REF: 166 | 167 OBJ: Discuss principles for the appropriate delegation of patient care activities. TOP: Implementation MSC: CPNRE: Collaborative Practice 12. One type of nursing model is an emerging model among intraprofessional nursing teams and
other health care providers who are members of the interprofessional team. What is this model called? a. Primary nursing model. b. Functional nursing model. c. Team nursing model. d. Collaborative practice model.
Canadian Fundamentals of Nursing 6th Edition Potter Test Bank ANS: D
The Registered Nurses Association of Ontario (2016) has developed evidence-informed Best Practice Guidelines for collaborative practice among nurses, interprofessional health teams, and health professionals. The other models are not considered emerging models. The primary nursing model was popular in the 1970s and 1980s; the functional nursing model was popular during World War II in response to a nursing shortage; and the team nursing model was developed in response to a nursing shortage after World War II. DIF: Understand REF: 162 | 163 OBJ: Discuss how a nurse leader can contribute to collaborative practice and best practices implementation. TOP: Implementation MSC: CPNRE: Collaborative Practice 13. A management structure in which traditional units are reorganized into business units is called
what? a. Matrix. b. Centralized. c. Decentralized. d. Participatory. ANS: A
In a matrix management structure, traditional units are reorganized into business units. Staff may report to several managers who may be from a variety of professional practice backgrounds. Traditional units are not reorganized into business units in centralized, decentralized, or participatory management structures. DIF: Understand REF: 164, Table 11-1 OBJ: Discuss how a nurse leaN derUcR anScI ontr toC coll NGibut TBe . OMaborative practice and best practices implementation. TOP: Implementation MSC: CPNRE: Professional, Ethical, and Legal Practice 14. Which term characterizes the duties and activities that an individual is employed to perform? a. Autonomy. b. Authority. c. Responsibility. d. Accountability. ANS: C
The duties and activities that an individual is employed to perform reflect responsibility. Autonomy is the freedom of choice and responsibility for choices. Authority is the right to act in areas in which a nurse has been given and accepts responsibility according to legislation, standards, and the code of ethics. Accountability refers to being answerable for one’s actions. DIF: Understand REF: 164 | 165 OBJ: Describe entry-level professional nurse competencies related to leadership, management, and collaborative practice. TOP: Implementation MSC: CPNRE: Professional, Ethical, and Legal Practice 15. What is a nursing manager’s greatest challenge? a. Delegation. b. Communication. c. Time management.
Canadian Fundamentals of Nursing 6th Edition Potter Test Bank d. Clinical decision making. ANS: B
Communication with staff is one of the nursing manager’s greatest challenges, especially in a large work group in which change is constant. The nursing manager’s greatest challenge is not delegation, time management, or clinical decision making. DIF: Understand REF: 165 OBJ: Discuss how a nurse leader can contribute to collaborative practice and best practices implementation. TOP: Planning MSC: CPNRE: Collaborative Practice 16. A nurse has delegated a task to an unregulated care provider (UCP). What is the nurse
responsible for? a. Performing the task safely. b. Maintaining accountability for the task. c. Weekly performance evaluations of the UCP. d. Reporting any mistakes made by the UCP to all staff on the unit. ANS: B
When delegating a task to a UCP, the nurse is responsible for maintaining accountability for the task. The UCP is responsible for performing the task safely. Frequency of evaluation of the UCP’s performance would vary. If the UCP’s performance is unsatisfactory, the nurse must give constructive feedback, specifically discussing mistakes and how they could have been avoided. DIF: Understand REF: 166 OBJ: Discuss principles for the appropriate delegation of patient care activities. TOP: Planning MSC: CP NNRRE: CIollaGborati B.veCPraMctice
U S N T
O
Canadian Fundamentals of Nursing 6th Edition Potter Test Bank
Chapter 12: Critical Thinking in Nursing Practice Potter et al: Canadian Fundamentals of Nursing, 6th Edition MULTIPLE CHOICE 1. Critical thinking characteristics include which of the following? a. Considering what is important in a given situation. b. Accepting one, established way to provide patient care. c. Making decisions on the basis of intuition. d. Being able to read and follow prescriber orders. ANS: A
Critical thinking involves being able to decipher what is relevant and important in a given situation and to make a clinical decision on the basis of that importance. Patient care can be provided in many ways, not just one. Clinical decisions should be based on evidence and research. Following prescriber orders is not considered a critical thinking skill. DIF: Remember TOP: Assessment
REF: 174 | 175 OBJ: Describe characteristics of a critical thinker. MSC: CPNRE: Professional, Ethical, and Legal Practice
2. Which of these patient scenarios is most indicative of critical thinking? a. Administering pain relief medication according to what was given last shift. b. Asking a patient what pain relief methods, pharmacological and
nonpharmacological, have worked in the past. c. Offering pain relief medication on the basis of prescriber orders. d. Explaining to the patient that his reports of severe pain are not consistent with the
N R I G B.COM
minor procedure that was pU erfoSrmeNd. T
ANS: B
Asking the patient what pain relief methods have worked in the past is an example of exploring many options for pain relief. Administering medication on the basis of a previous assessment is not practising according to standards of care. The nurse is to assess a patient during each shift and intervene accordingly. Nonpharmacological pain relief methods are available, as are medications for pain. Pain is subjective; the nurse should offer pain relief methods on the basis of the patient’s reports without being judgemental. DIF: Apply TOP: Assessment
REF: 174 175 OBJ: Describe characteristics of a critical thinker. MSC: CPNRE: Professional, Ethical, and Legal Practice
3. Professional nurses are responsible for making clinical decisions in order to do what? a. Prove the efficacy of traditional methods of providing nursing care to patients. b. Take immediate action when a patient’s condition worsens. c. Apply clear textbook solutions to patients’ problems. d. Formulate standardized care plans for groups of patients. ANS: B
Professional nurses are responsible for making clinical decisions to take immediate action when a patient’s condition worsens. Patient care should be based on evidence-informed practice, not on tradition. Clear textbook solutions to patient problems are not always available. Care plans should be individualized.
Canadian Fundamentals of Nursing 6th Edition Potter Test Bank
DIF: Understand REF: 176 | 177 | 180 OBJ: Discuss the nurse’s responsibility in making clinical decisions. TOP: Implementation MSC: CPNRE: Professional, Ethical, and Legal Practice 4. The nurse needs a reminder of professional responsibility when performing which of these
actions? a. Making an informed clinical decision. b. Making an ethical clinical decision. c. Making a clinical decision in the patient’s best interest. d. Making a clinical decision that is based on previous shift assessments. ANS: D
The professional nurse is responsible for assessing patients each shift. Making informed, ethical decisions in the patient’s best interest is practising responsibly. DIF: Understand REF: 180 | 181 OBJ: Discuss the nurse’s responsibility in making clinical decisions. TOP: Evaluate MSC: CPNRE: Professional, Ethical, and Legal Practice 5. Which of the following demonstrates a nurse utilizing self-reflection to improve clinical
decision making? a. Using an objective approach in all situations. b. Obtaining data in an orderly manner. c. Improving a plan of care while thinking back on interventions performed. d. Providing evidence-informed explanations for all nursing interventions. ANS: C
NURSINGTB.COM
In self-reflection, the nurse utilizes critical thinking in evaluating the effectiveness of interventions and how they were performed. The other options do represent good nursing practice but are not the best examples of self-reflection. Using an objective approach and obtaining data in an orderly manner does not involve purposefully thinking back to discover the meaning or purpose of a situation. Providing evidence-informed explanations for nursing interventions does not always involve thinking back to discover the meaning of a situation. DIF: Understand REF: 182 | 183 OBJ: Discuss critical thinking competencies used in nursing practice. TOP: Evaluate MSC: CPNRE: Professional, Ethical, and Legal Practice 6. A nursing instructor needs to evaluate students’ abilities to synthesize data and identify
relationships between nursing diagnoses. Which learning assignment is best suited for this instructor’s needs? a. Concept mapping. b. Reflective journalling. c. Reading assignment with a written summary. d. Lecture and discussion. ANS: A
Concept maps challenge the student to synthesize data and identify relationships between nursing diagnoses. Reflective journalling involves thinking back to clarify concepts. Reading assignments and lecture do not best provide an instructor the ability to evaluate students’ abilities to synthesize data.
Canadian Fundamentals of Nursing 6th Edition Potter Test Bank
DIF: Understand REF: 182 | 183 OBJ: Discuss how concept maps can improve a nurse’s ability to think critically. TOP: Implementation MSC: CPNRE: Professional, Ethical, and Legal Practice 7. What is the first component of the critical thinking model for clinical decision making? a. Experience. b. Nursing process. c. Attitude. d. A scientific knowledge base. ANS: D
A scientific knowledge base is the first component for clinical decision making. After acquiring a sound knowledge base, the nurse can then apply knowledge to different clinical situations and use the nursing process to gain valuable experience. A critical thinking attitude is a guideline for how to approach a problem and apply knowledge to make a clinical decision. DIF: Remember REF: 177, Box 12-2 OBJ: Describe the components of a critical thinking model for clinical decision making. TOP: Implementation MSC: CPNRE: Professional, Ethical, and Legal Practice 8. A new graduate nurse will make the best clinical decisions by applying the components of the
nursing critical thinking model and which of the following? a. Drawing on past clinical experiences to formulate standardized care plans. b. Relying on recall of information from past lectures and textbooks. c. Depending on the charge nurse to determine priorities of care. d. Using the nursing procesN s. URSINGTB.COM ANS: D
Using the nursing process along with applying components of the nursing critical thinking model will help the new graduate nurse make the most appropriate clinical decisions. Care plans should be individualized, and recalling facts is not a use of critical thinking skills to make clinical decisions. The new nurse should not rely on the charge nurse to determine priorities of care. DIF: Apply REF: 181 OBJ: Describe the components of a critical thinking model for clinical decision making. TOP: Implementation MSC: CPNRE: Professional, Ethical, and Legal Practice 9. The critical thinking skill of evaluation in nursing practice can be best described as which of
the following? a. Examining the meaning of data. b. Reviewing the effectiveness of nursing actions. c. Supporting findings and conclusions. d. Searching for links between data and the nurse’s assumptions. ANS: B
Reviewing the effectiveness of interventions best describes evaluation. Examining the meaning of data is a form of inference. Supporting findings and conclusions is a way of providing explanations. Searching for links between the data and the nurse’s assumptions is a type of analysis.
Canadian Fundamentals of Nursing 6th Edition Potter Test Bank
DIF: Understand REF: 176 | 182, Table 12-1 OBJ: Discuss critical thinking competencies used in nursing practice. TOP: Evaluate MSC: CPNRE: Professional, Ethical, and Legal Practice 10. The patient appears to be in no apparent distress, but vital signs measured by an unregulated
care provider reveal that the pulse is extremely low. The nurse then auscultates an apical pulse and asks the patient whether he has any complaints or a history of heart problems. The nurse is utilizing which critical thinking skill? a. Interpretation. b. Evaluation. c. Self-regulation. d. Explanation. ANS: A
Interpretation involves collecting data in an orderly manner, looking for patterns to categorize data, and clarifying uncertain data. This nurse is clarifying the data in this situation. Evaluation involves determining the effectiveness of interventions. The nurse in this scenario is assessing the patient, not evaluating interventions. Self-regulation involves reflecting on experiences. Explanation entails supporting findings and conclusions. The nurse in this question is clarifying uncertain data (determining cause of the low pulse), not supporting the finding of a low pulse. DIF: Apply REF: 176, Table 12-1 OBJ: Discuss critical thinking competencies used in nursing practice. TOP: Assessment MSC: CPNRE: Professional, Ethical, and Legal Practice 11. A patient continues to reportN poUsR tsS urI giN caG lT inB ci. sioCnOpM ain at a level of 9 out of 10 after pain
medicine is given. The next dose of pain medicine is not due for another hour. What should the critically thinking nurse do first? a. Explain to the patient that nothing else has been ordered. b. Explore other options for pain relief. c. Offer to notify the health care provider after morning rounds are completed. d. Discuss the surgical procedure and reason for the pain. ANS: B
The critically thinking nurse should explore all options for pain relief first. The nurse should use critical thinking to determine the cause of the pain and determine various options for pain, in addition to the ordered pain medications. The nurse can act independently to determine all options for pain relief and does not have to wait until after the health care provider rounds are completed. Explaining the cause of the pain does not address options for pain relief. DIF: Apply REF: 177 OBJ: Discuss critical thinking competencies used in nursing practice. TOP: Implementation MSC: CPNRE: Professional, Ethical, and Legal Practice 12. The nursing student can best develop critical thinking skills by doing which of the following? a. Studying 3 hours more each night. b. Actively participating in all clinical experiences. c. Interviewing staff nurses about their nursing experiences. d. Attending all open skills laboratory opportunities.
Canadian Fundamentals of Nursing 6th Edition Potter Test Bank ANS: B
Nursing is an applied science, and to apply knowledge learned and develop critical thinking skills to make clinical decisions, the student should value and actively participate in all clinical experiences. Studying for longer hours, interviewing nurses, and attending skills laboratory sessions do not provide opportunities for clinical decision making, as do actual clinical experiences. DIF: Understand REF: 178 OBJ: Explain the relationship between clinical experience and critical thinking. TOP: Implementation MSC: CPNRE: Professional, Ethical, and Legal Practice 13. While caring for a hospitalized older woman after hip surgery, the new graduate nurse is faced
with the task of inserting an indwelling urinary catheter, which involves rotating the hip into a contraindicated position. The nurse exhibits critical thinking to perform this task by doing what? a. Following textbook procedure. b. Notifying the physician of the need for a urologist consult. c. Adapting the positioning technique to the situation. d. Postponing catheter insertion until the next shift. ANS: C
The nurse must use critical thinking skills and problem solving in this situation to adapt positioning technique. In practice, patient procedures are not always presented as in a textbook, but they are individualized. A urology consult is not warranted for position, but it may be warranted if the nurse encounters difficulty in insertion. Postponing insertion of the catheter is not an appropriate action. DIF: Apply REF: 17N 9URSINGTB.COM OBJ: Explain the relationship between clinical experience and critical thinking. TOP: Implementation MSC: CPNRE: Professional, Ethical, and Legal Practice 14. The nurse enters a room to find the patient sitting up in bed crying. How would the nurse
display a critical thinking attitude in this situation? a. Tell the patient she’ll be back in 30 minutes. b. Set a box of tissues at the patient’s bedside before leaving the room. c. Ask the patient why she is crying. d. Limit visitors while the patient is upset. ANS: C
The nurse should try to find out why the patient is crying. By using interpretation skills, the nurse can collect information to be able to make appropriate interventions. Telling the patient that she will return, providing tissues, and limiting visitors may be appropriate actions but do not address the reason why the patient is crying. DIF: Apply REF: 176 OBJ: Discuss critical thinking competencies used in clinical decision making. TOP: Assessment MSC: CPNRE: Professional, Ethical, and Legal Practice 15. Which of the following is a strategy used by learners to explore complex problems and engage
in decision making without the risk of harming a patient? a. Lectures and reading.
Canadian Fundamentals of Nursing 6th Edition Potter Test Bank b. Concept mapping. c. Reflective writing. d. Case-based learning. ANS: D
Case-based learning is a strategy that allows learners to explore complex problems and engage in decision making without the risk of harming a patient. Lectures and readings do not always engage the learner in decision making. Concept mapping is a visual representation of patient problems and interventions that depicts their relationship to one another. Reflective writing is used when a learner explains a clinical experience and explores his or her understanding and perception of the situation. DIF: Understand REF: 182 OBJ: Describe how case-based learning fosters critical thinking. TOP: Implementation MSC: CPNRE: Professional, Ethical, and Legal Practice 16. How do professional standards influence a nurse’s clinical decisions? a. They bypass the patient’s feelings to promote ethical standards. b. They set minimal passing standards for testing. c. They require the nurse to use critical thinking for the highest level of quality
nursing care. d. They require the nurse to use evidence-informed practice that is based on nurses’
needs. ANS: C
Upholding professional standards requires nurses to use critical thinking for the highest level of quality nursing care. Bypassing the patient’s feelings is not practising according to professional standards. The pN rim pose fessional standards is not to establish URary SIpur NG TBo.fCpro OM minimal passing standards for testing. Patient care should be based on patients’ needs, not on nurses’ needs. DIF: Remember REF: 181 OBJ: Explain how professional standards influence a nurse’s clinical decisions. TOP: Planning MSC: CPNRE: Professional, Ethical, and Legal Practice 17. A nurse who is caring for a patient with a pressure ulcer fails to apply the recommended
dressing according to hospital policy. If the patient is harmed, the nurse could be subject to legal action for not adhering to which of the following? a. Fairness. b. Intellectual standards. c. Independent reasoning. d. Institutional practice guidelines. ANS: D
Institutional practice guidelines are established standards and policies that can be used in court to make judgements about nursing actions. Intellectual standards are guidelines or principles for rational thought. Fairness and independent reasoning are two examples of critical thinking attitudes that are designed to help nurses make clinical decisions. DIF: Apply REF: 181 OBJ: Explain how professional standards influence a nurse’s clinical decisions. TOP: Implementation MSC: CPNRE: Professional, Ethical, and Legal Practice
Canadian Fundamentals of Nursing 6th Edition Potter Test Bank
18. Which of these findings, if identified in a plan of care, should the nurse revise because it is not
characteristic of critical thinking and the nursing process? a. Patient’s reactions to diagnostic testing. b. Nurse’s assumptions about hospital discharge. c. Identification of five different nursing diagnoses. d. Documentation of patient’s ability to cope with loss. ANS: B
The nurse should not assume when a patient is going to be discharged and document this information in a plan of care. Making assumptions is not an example of a critical thinking skill. The patient’s reactions to testing, having several nursing diagnoses, and a description of the patient’s coping abilities are all appropriate to document in the nursing plan of care. DIF: Analyze REF: 174-176 OBJ: Discuss the relationship of the nursing process to critical thinking. TOP: Evaluate MSC: CPNRE: Professional, Ethical, and Legal Practice 19. The nursing process involves which of the following steps in the clinical decision-making
process? a. Diagnosing the disease process. b. Performing nursing interventions. c. Setting professional nursing learning goals. d. Evaluating effectiveness of medical treatments. ANS: B
Implementing nursing interventions is one step in the clinical decision-making process. Diagnosing disease and evaluNatin ffec URgSeI NGtive TBne.ssCofOMmedical treatments are not nursing actions. Nurses are to use the nursing process to set patient goals, not the nurse’s learning goals. DIF: Understand REF: 181 OBJ: Discuss the relationship of the nursing process to critical thinking. TOP: Implementation MSC: CPNRE: Professional, Ethical, and Legal Practice
Canadian Fundamentals of Nursing 6th Edition Potter Test Bank
Chapter 13: Nursing Assessment, Diagnosis, and Planning Potter et al: Canadian Fundamentals of Nursing, 6th Edition MULTIPLE CHOICE 1. After completing a thorough database review and analyzing the data to identify any problems,
the nurse should proceed to what step of the nursing process? a. Assessment. b. Planning. c. Implementation. d. Evaluation. ANS: B
In the five-step nursing process, the nurse should establish mutual goals with the patient and prioritize care in the planning phase, which follows the diagnosis phase. The assessment phase of the nursing process involves gathering data. The implementation phase involves carrying out appropriate nursing interventions. During the evaluation phase, the nurse assesses the effectiveness of interventions. DIF: Understand REF: 188 OBJ: Identify and discuss the steps of nursing assessment. MSC: CPNRE: Foundations of Practice
TOP: Planning
2. A patient’s plan of care includes the goal of increasing mobility this shift. As the patient is
ambulating to the bathroom at the beginning of the shift, the patient suffers a fall. How should the nurse first revise the plan of care? G B.C M R the INpatiTent becO a. Asking physiotherapy to N asU sistS ause of the new injuries. b. Disregarding all previous diagnoses and establishing a new plan of care. c. Reassessing the patient. d. Setting new priorities for the patient. ANS: C
The nurse needs to reassess the patient after any type of change in health status. The nursing process is dynamic and ongoing. Asking physiotherapy to assist the patient is premature before the patient is reassessed and before prescriber orders have been made. The nurse may not need to disregard all previous diagnoses. Some diagnoses may still apply, but the patient needs to be reassessed first. Setting new priorities is not recommended before assessment and establishing diagnoses. DIF: Apply TOP: Planning
REF: 188 OBJ: Discuss criteria used in priority setting. MSC: CPNRE: Foundations of Practice
3. In the planning of patient care, a goal can be described as which of the following? a. A statement describing the patient’s accomplishments without a time restriction. b. A realistic statement predicting any negative responses to treatments. c. A broad statement describing a desired change in patient behaviour. d. An identified long-term nursing diagnosis. ANS: C
Canadian Fundamentals of Nursing 6th Edition Potter Test Bank A goal is a broad statement that describes a desired change in a patient’s condition or behaviour. A goal is mutually set with the patient and is time-limited, patient-centred, measurable, and realistic. DIF: Remember REF: 204 | 205 OBJ: Describe goal setting and discuss the difference between a goal and an expected outcome. TOP: Planning MSC: CPNRE: Foundations of Practice 4. When evaluating a plan of care, the nurse reviews the goals for the patient. Which goal
statement is realistic to assign to a patient with a pelvic fracture who is on bed rest? a. The patient will increase mobility by ambulating in the hallway two times this shift. b. The patient will increase mobility by turning side to back to side with assistance every 2 hours. c. The patient will increase mobility by using the walker correctly to ambulate to the bathroom as needed. d. The patient will increase mobility by using a sliding board correctly to transfer to the bedside commode as needed. ANS: B
The patient is ordered to be on bed rest; therefore, turning the patient in bed is the only option that is appropriate. When determining goals, the nurse needs to ensure that the goal is individualized and realistic for the patient. Goals must reflect the highest possible level of wellness and independence in function, which for this patient would be turning in bed. DIF: Evaluate REF: 205 OBJ: Describe goal setting and discuss the difference between a goal and an expected outcome. NNURRES TOP: Planning MSC: CP : FI oN unG daT tiB on. sC ofO PM ractice 5. The following statements are on a patient’s nursing care plan. Which of the following
statements represents an expected outcome? a. The patient will verbalize a decreased pain level less than 3 on a 0-to-10 scale by the end of this shift. b. The patient will demonstrate increased mobility in 2 days. c. The patient will demonstrate increased tolerance to activity over the next month. d. The patient will understand needed dietary changes by discharge. ANS: A
An expected outcome is a specific and measurable change that is expected as a result of nursing care. The other three options in this question are goals. Demonstrating increased mobility in 2 days and understanding necessary dietary changes by discharge are short-term goals because they are expected to occur in less than a week. Demonstrating increased tolerance to activity over a month-long period is a long-term goal because it is expected to occur over a longer period of time. DIF: Understand REF: 205 | 206 OBJ: Describe goal setting and discuss the difference between a goal and an expected outcome. TOP: Planning MSC: CPNRE: Foundations of Practice 6. Which patient outcome statement includes all seven guidelines for writing goal and outcome
statements?
Canadian Fundamentals of Nursing 6th Edition Potter Test Bank a. The patient will ambulate in hallways. b. The nurse will administer pain medication every 4 hours to keep the patient free
from discomfort. c. The nurse will monitor the patient’s heart rhythm continuously this shift. d. The patient will feed self at all mealtimes today without complaints of shortness of
breath. ANS: D
A goal or outcome statement should be patient-centred; should address one patient response; should be observable, measurable, and time-limited; should be mutually set by nurse and patient; and should be realistic. The statement “The patient will feed self at all mealtimes today without complaints of shortness of breath” includes all seven criteria for goal writing. “The patient will ambulate in hallways” is missing a time limit. Administering pain medication and monitoring the patient’s heart rhythm are nursing interventions; they are not patient behaviours or actions. DIF: Understand REF: 206 OBJ: Describe goal setting and discuss the difference between a goal and an expected outcome. TOP: Planning MSC: CPNRE: Foundations of Practice 7. A nursing assessment for a patient with a spinal cord injury reveals several pertinent problems
that a nurse can treat. While the plan of care is developed, which nursing diagnosis is the highest priority for this patient? a. Risk for impaired skin integrity. b. Risk for infection. c. Spiritual distress. d. Reflex urinary incontinence. I G B.C M ANS: D
N R U S N T
O
Reflex urinary incontinence is highest priority. If a patient’s incontinence is not addressed, then the patient is at higher risk for impaired skin integrity and infection. Risk for diagnoses are potential problems. They may be prioritized higher in some cases, but not in this situation. Spiritual distress is an actual diagnosis, but the adverse effects that could result from not assisting the patient with urinary elimination take priority in this case. Physiological problems do not always take priority, but urinary incontinence could cause the greatest harm if it is not addressed. DIF: Apply TOP: Planning
REF: 203 OBJ: Develop a plan of care from a nursing assessment. MSC: CPNRE: Foundations of Practice
8. The nurse is caring for seven patients this shift. After completing their assessments, the nurse
states that he does not know where to begin in developing care plans for these patients. Which of the following is an appropriate suggestion by another nurse? a. “Choose all the interventions and perform them in order of time needed for each one.” b. “Make sure you identify the scientific rationale for each intervention first.” c. “Decide on goals and outcomes you have chosen for the patients.” d. “Begin with the highest priority diagnoses, and then select appropriate interventions.” ANS: D
Canadian Fundamentals of Nursing 6th Edition Potter Test Bank When developing a plan of care, the nurse needs to rank the nursing diagnoses in order of priority and then select appropriate interventions. Choosing all the interventions should take place after the diagnoses are ranked, and interventions should be prioritized by patient needs, not just by time. The chosen interventions should be evidence informed with scientific rationales, but the diagnoses need to be prioritized first in order to prioritize interventions. Goals for a patient should be mutually set, not chosen just by the nurse. DIF: Apply TOP: Planning
REF: 203 | 204 OBJ: Develop a plan of care from a nursing assessment. MSC: CPNRE: Foundations of Practice
9. A patient’s son decides to stay at the bedside while his father is confused. When the nurse
develops the plan of care for this patient, what should the nurse do? a. Individualize the care plan only according to the patient’s needs. b. Request that the son leave at bedtime, so the patient can rest. c. Suggest that a female member of the family stay with the patient. d. Involve the son in the plan of care as much as possible. ANS: D
Family should be included in a patient’s plan of care as much as possible. The family can help patients meet health care goals. Meeting some of the family’s need as well as the patient’s needs may improve the patient’s level of wellness. The son should not be asked to leave if at all possible. In some situations, it may be best that family members not remain in the room, but this question stem does not indicate that this is the case in this situation. Suggesting that a female member of the family stay is not justified without a legitimate reason, and no reason is given in this question stem for such a suggestion. DIF: Apply TOP: Planning 10.
REF: 203 OBJ: De velop a plan of care from a nursing assessment. N R I G B.ofC M MSC: CPNURES : FoN undaT tions OPractice
Which of these outcomes would be most appropriate for a patient with a nursing diagnosis of Constipation related to slowed gastrointestinal motility secondary to pain medications? a. Patient will have one soft, formed bowel movement by end of shift. b. Patient will not take any pain medications this shift. c. Patient will walk unassisted to bathroom by the end of shift. d. Patient will not take laxatives or stool softeners this shift. ANS: A
The identified problem, or nursing diagnosis, is constipation. Therefore, the outcome should be that the constipation is relieved. To measure constipation relief, the nurse will be observing for the patient to have a bowel movement. Not taking pain medications may or may not relieve the constipation. Although not taking pain medicines might be an intervention, the patient should not have to be in pain to relieve constipation. Other measures, such as administering laxatives or stool softeners, might be appropriate interventions, but they are not outcomes. The patient walking unassisted to the bathroom addresses mobility, not constipation. The patient may need to walk to the bathroom to have a bowel movement, but the appropriate outcome for constipation is that the constipation is relieved as evidenced by a bowel movement—something that the nurse can observe. DIF: Apply TOP: Planning
REF: 204-206 OBJ: Develop a plan of care from a nursing assessment. MSC: CPNRE: Foundations of Practice
Canadian Fundamentals of Nursing 6th Edition Potter Test Bank 11. The nurse is working with a patient who is being prepared for a diagnostic test this afternoon.
The patient tells the nurse that she wants to have her hair shampooed. Which of the following is the most appropriate label with regard to assigning a priority for the patient’s request? a. Low priority. b. An unmet need. c. Intermediate priority. d. A safety and security need. ANS: A
The patient’s request would be of low priority because it is not directly related to a specific illness or prognosis. The patient’s request does not represent “an unmet need” and is not an intermediate priority. An intermediate priority is one that involves the nonemergency, non–life-threatening needs of the patient. The patient’s request does not reflect a safety and security need; the outcome does not threaten her well-being. DIF: Analyze TOP: Planning
REF: 203 OBJ: Discuss criteria used in priority setting. MSC: CPNRE: Foundations of Practice
12. Which of the following options correctly explains what the nurse should do with the plan of
care for a patient after it is developed? a. Place the original copy in the chart, so it cannot be tampered with or revised. b. Communicate the plan of care to all health care professionals involved in the patient’s care. c. Send the plan of care to the administration office to be filed. d. Send the plan of care to quality assurance for review. ANS: B
Bi. The patient’s nursing plan ofNcU arR e is dyGnT am cC piO ecM e of work that needs to be updated and SIa N revised as the patient’s condition changes. All health care providers involved in the patient’s care need to be informed of the plan of care. The plan of care is not placed on the chart and then never looked at again. The plan of care is not sent to the administrative office or the quality assurance office. DIF: Understand TOP: Planning
REF: 206 | 207 OBJ: Develop a plan of care from a nursing assessment. MSC: CPNRE: Foundations of Practice
13. What is the purpose and distinction of using a concept map when a plan of care is
implemented? a. Quality assurance in the health care facility. b. Multidisciplinary communication. c. Provision of a standardized format for patient problems. d. Identification of the relation of patient problems and interventions. ANS: D
A concept map is a diagram of patient problems and interventions that shows their relations to one another. The use of a concept map promotes critical thinking and helps the nurse organize complex patient data, process complex relationships, and achieve a holistic view of the patient’s situation. The purpose is not quality assurance in the health care facility. Multidisciplinary communication is enhanced with the use of critical pathways, not concept maps. Standardized or computerized care plans, not concept maps, provide a standardized format for patient problems. A concept map is highly individualized.
Canadian Fundamentals of Nursing 6th Edition Potter Test Bank
DIF: Apply REF: 195 | 196 OBJ: Explain the relationship of critical thinking to assessment, diagnosis, and planning. TOP: Planning MSC: CPNRE: Foundations of Practice 14. The use of critical thinking skills during the assessment phase of the nursing process ensures
that the nurse does which of the following? a. Completes a comprehensive database. b. Identifies pertinent nursing diagnoses. c. Intervenes on the basis of patient goals and priorities of care. d. Determines whether outcomes have been achieved. ANS: A
The assessment phase of the nursing process involves data collection to complete a thorough patient database. Identifying nursing diagnoses occurs during the diagnosis phase. The nurse carries out interventions during the implementation phase, and determines whether outcomes have been achieved during the evaluation phase. DIF: Understand REF: 188 OBJ: Explain the relationship of critical thinking to assessment, diagnosis, and planning. TOP: Assessment MSC: CPNRE: Foundations of Practice 15. Subjective data include which of the following? a. A patient’s feelings, perceptions, and reported symptoms. b. A description of the patient’s behaviour. c. Observations of a patient’s health status. d. Measurements of a patient’s health status. ANS: A
NURSINGTB.COM
Subjective data include the patient’s feelings, perceptions, and reported symptoms. Only patients provide subjective data relevant to their health condition. Data sometimes reflect physiological changes, which the nurse explores further through objective data collection. Describing the patient’s behaviour, observations made, and measurements of a patient’s health status are all examples of objective data. DIF: Remember REF: 191 OBJ: Differentiate between subjective and objective data. MSC: CPNRE: Foundations of Practice
TOP: Evaluate
16. A patient expresses fear of going home and being alone. Her vital signs are stable and her
incision is nearly completely healed. What can the nurse can infer from the subjective data? a. The patient can now perform the dressing changes herself. b. The patient can begin retaking all her previous medications. c. The patient is apprehensive about discharge. d. Surgery was not successful. ANS: C
Subjective data include expressions of fear of going home and being alone. These data indicate that the patient is apprehensive about discharge. Expressing fear is not an appropriate sign that a patient is able to perform dressing changes independently. An order from a health care provider is required before a patient is taught to resume previous medications. The nurse cannot infer that surgery was not successful if the incision is nearly completely healed.
Canadian Fundamentals of Nursing 6th Edition Potter Test Bank
DIF: Apply REF: 191 OBJ: Differentiate between subjective and objective data. MSC: CPNRE: Foundations of Practice
TOP: Assessment
17. Which of the following methods of data collection is utilized to establish a patient’s nursing
database? a. Reviewing the current literature to determine evidence-informed nursing actions. b. Orders for diagnostic and laboratory tests. c. Physical examination. d. Anticipated medications to be ordered. ANS: C
A nursing database includes results of a physical examination. Orders are included in the order section of the patient’s chart. The nurse reviews the current literature in the implementation phase of the nursing process to determine evidence-informed actions, and the health care provider is responsible for ordering medications. Medication orders are usually written after the database is completed. DIF: Remember REF: 188 OBJ: Identify and discuss the steps of nursing assessment. MSC: CPNRE: Foundations of Practice
TOP: Assessment
18. To gather information about a patient’s home and work surroundings, the nurse will need to
utilize which method of data collection? a. Carefully reviewing laboratory results. b. Conducting the physical assessment before collecting subjective information. c. Performing a thorough nuNrsUinRgShI eaN ltG hT hiB st. orC y.OM d. Prolonging the termination phase of the interview. ANS: C
A thorough nursing history includes information about the patient’s home and work surroundings. Neither laboratory results nor the physical assessment will reveal much about the home and work surroundings. Collecting data is part of the working phase of the interview. DIF: Understand REF: 189 OBJ: Describe the relationship between data collection and data analysis. TOP: Assessment MSC: CPNRE: Foundations of Practice 19. While interviewing an older female patient of Asian descent, the nurse notices that the patient
looks at the ground when answering questions. What should the nurse do? a. Notify the physician to recommend a psychological evaluation. b. Consider cultural differences during this assessment. c. Ask the patient to make eye contact to determine her affect. d. Continue with the interview and document that the patient is depressed. ANS: B
Older women of Asian descent consider it rude to look an authority figure, such as a health care provider, in the eye. This nurse needs to practise culturally competent care and appreciate the cultural differences. Assuming that the patient is depressed or in need of a psychological evaluation is inappropriate, as is asking the patient to make eye contact.
Canadian Fundamentals of Nursing 6th Edition Potter Test Bank
DIF: Apply REF: 193 OBJ: Describe the relationship between data collection and data analysis. TOP: Assessment MSC: CPNRE: Foundations of Practice 20. After reviewing the interview process and objectives during a patient-centred interview, what
will the nurse do? a. Begin by introducing himself or herself. b. Document a nursing health history. c. Explain that the interview will be over in a few more minutes. d. Tell the patient that he’ll be back to administer medications in 1 hour. ANS: B
After beginning with an introduction and reviewing the interview process and objectives, the nurse should conduct the actual interview and proceed with data collection. The termination phase includes telling the patient when the interview is nearing an end. Telling the patient that medications will be given later when the nurse returns would typically take place during the termination phase of the interview. DIF: Understand REF: 193 OBJ: Identify and discuss the steps of nursing assessment. MSC: CPNRE: Foundations of Practice
TOP: Assessment
21. The nurse is attempting to prompt the patient to elaborate on her complaints of daytime
fatigue. Which question should the nurse ask? a. “Is there anything that you are stressed about right now?” b. “What reasons do you think are contributing to your fatigue?” c. “What are your normal wN oU rkRhS ouIrsN?G ” TB.COM d. “Are you sleeping 8 hours a night?” ANS: B
The question about what factors might be contributing to the patient’s fatigue will elicit the best open-ended response. Asking whether the patient is stressed and asking whether the patient is sleeping 8 hours a night are closed-ended questions eliciting simple yes or no responses. Asking about normal work hours will elicit a matter-of-fact response and does not prompt the patient to elaborate on her complaints of daytime fatigue nor elicit the contributing reasons. DIF: Apply REF: 193 OBJ: Identify and discuss the steps of nursing assessment. MSC: CPNRE: Foundations of Practice 22. Components of a nursing health history include a. Current treatment orders. b. Nurse’s concerns. c. Nurse’s goals for the patient. d. Patient expectations. ANS: D
TOP: Assessment
Canadian Fundamentals of Nursing 6th Edition Potter Test Bank Components of a nursing health history include physical examination findings, patient expectations, environmental history, and diagnostic data. Current treatment orders are located in the order section in the patient’s chart and are not a part of the nursing health history. Patient concerns, not nurse’s concerns, are included in the database. Goals that are mutually established, not nurse’s goals, are part of the nursing care plan. DIF: Remember TOP: Assessment
REF: 194 OBJ: Describe the components of a nursing history. MSC: CPNRE: Foundations of Practice
23. While the patient’s lower extremity, which is in a cast, is assessed, the patient tells the nurse
about an inability to rest at night. The nurse disregards this complaint, thinking that no correlation has been noted between having a leg cast and developing restless sleep. What would be a more theoretically sound approach? a. Document the sleep patterns and complaint in the patient’s chart. b. Tell the patient you are just focused on the leg right now. c. Explain that a more thorough assessment will be needed next shift. d. Ask the patient about his usual sleep patterns and the onset of having difficulty resting. ANS: D
The nurse must use critical thinking skills to assess this situation first. The best response is to gather more assessment data by asking the patient about usual sleep patterns and the onset of having difficulty resting. The nurse should assess before documenting and should not ignore the patient’s complaints. DIF: Apply REF: 191 OBJ: Explain the relationship of critical thinking to assessment, diagnosis, and planning. NNURRES TOP: Assessment MSC: CP : FI oN unG daT tiB on. sC ofO PM ractice 24. A nurse comparing data validation and data interpretation correctly explains the difference
with which statement? a. “Validation involves looking for patterns in professional standards.” b. “Data interpretation involves discovering patterns in professional standards.” c. “Validation involves comparing data with other sources for accuracy.” d. “Data interpretation occurs before data validation.” ANS: C
Validation, by definition, involves comparing data with other sources for accuracy. Data interpretation involves identifying abnormal findings, clarifying information, and identifying patient problems. The nurse should validate data before interpreting the data and making inferences. The nurse is interpreting and validating patient data, not professional standards. DIF: Understand REF: 194 | 195 OBJ: Explain the relationship between data interpretation, validation, and clustering. TOP: Assessment MSC: CPNRE: Foundations of Practice 25. While completing an admission database, the nurse is interviewing a patient who states that he
is allergic to latex. What is the most appropriate first nursing action? a. Leave the room and place the patient in isolation. b. Ask the patient to describe the type of reaction. c. Proceed to the termination phase of the interview.
Canadian Fundamentals of Nursing 6th Edition Potter Test Bank d. Document the latex allergy on the medication administration record. ANS: B
The nurse should further assess and ask the patient to describe the type of reaction. The patient will not need to be placed in isolation; before terminating the interview or documenting the allergy, health care personnel need to be aware of what type of response the patient suffered. DIF: Apply REF: 189-191 OBJ: Identify and discuss the steps of nursing assessment. MSC: CPNRE: Foundations of Practice
TOP: Assessment
26. A patient verbalizes a low pain level of 2 out of 10 but exhibits extreme facial grimacing
while moving around in bed. What is the nurse’s best action in response to her observation? a. Proceed to the next patient’s room while making rounds. b. Offer a massage because the patient does not want any more pain medicine. c. Administer the pain medication ordered for moderate to severe pain. d. Ask the patient about the facial grimacing with movement. ANS: D
The nurse needs to clarify what she observes with what the patient states. Proceeding to the next room is ignoring this visual cue. The nurse cannot assume the patient does not want pain medicine just because the pain level is rated at 2 out of 10. The nurse should not administer medication for moderate to severe pain if it is not necessary. DIF: Apply REF: 189 | 191 OBJ: Identify and discuss the steps of nursing assessment. MSC: CPNRE: Foundations ofNPraR cticeI G B.C M
U S N T
TOP: Assessment
O
27. The nurse is assessing a patient with a hearing deficit. Where is the best place to conduct this
interview? a. The patient’s room with the door closed. b. The waiting area with the television turned off. c. The patient’s room before administration of pain medication. d. The patient’s room while the occupational therapist is working on leg exercises. ANS: A
Distractions should be eliminated as much as possible during an interview with a patient with a hearing deficit. The best place to conduct this interview is in the patient’s room with the door closed. The waiting area does not provide privacy. Pain can sometimes inhibit someone’s ability to concentrate, so interviewing before pain medication is administered is not advisable. It is best for the patient to be as comfortable as possible during an interview. Assessing a patient while another member of the health care team is working would be distracting and is not the best time for assessment to take place. DIF: Apply REF: 192 | 193 OBJ: Discuss the purposes of a patient interview and the use of interview techniques in obtaining a health history. TOP: Assessment MSC: CPNRE: Foundations of Practice 28. A nursing student is completing an assessment on an 80-year-old patient who is alert and
oriented. The patient’s daughter is present in the room. Which of the following actions made by the nursing student requires the nursing professor to intervene?
Canadian Fundamentals of Nursing 6th Edition Potter Test Bank a. b. c. d.
The nursing student is making eye contact with the patient. The nursing student is speaking only to the patient’s daughter. The nursing student nods periodically while the patient is speaking. The nursing student leans forward while talking with the patient.
ANS: B
When assessing an older person, nurses need to listen carefully and allow the patient to speak. Positive nonverbal communication, such as making eye contact, nodding, and leaning forward, shows interest in the patient. Gathering data from family members is acceptable, but when a patient is able to interact, nurses need to include information from the patient to complete the assessment. DIF: Evaluate REF: 192 | 193 OBJ: Identify and discuss the steps of nursing assessment. MSC: CPNRE: Foundations of Practice
TOP: Assessment
29. Which of the following is an example of subjective data? a. Patient’s wound appearance. b. Patient’s expression of fear regarding upcoming surgery. c. Patient pacing the floor while awaiting test results. d. Patient’s temperature. ANS: B
Subjective data include patient’s feelings, perceptions, and reported symptoms. Expressing feelings such as fear is an example of subjective data. Objective data are observations or measurements of a patient’s health status. In this question, the appearance of the wound and the patient’s temperature are objective data. Pacing is an observable patient behaviour and is also considered objective datN a.URSINGTB.COM DIF: Understand REF: 191 OBJ: Differentiate between subjective and objective data. MSC: CPNRE: Foundations of Practice
TOP: Assessment
Canadian Fundamentals of Nursing 6th Edition Potter Test Bank
Chapter 14: Implementing and Evaluating Nursing Care Potter et al: Canadian Fundamentals of Nursing, 6th Edition MULTIPLE CHOICE 1. In which step of the nursing process does the nurse determine whether the patient’s condition
has improved and whether the patient has met expected outcomes? a. Assessment. b. Planning. c. Implementation. d. Evaluation. ANS: D
In the five-step nursing process, the evaluation phase is the final step involving conducting evaluative measures to determine whether nursing interventions have been effective and whether the patient has met expected outcomes. Assessment, the first step of the process, includes data collection, validation, sorting, and documentation. Planning, the third step of the process, involves setting priorities, identifying patient goals and outcomes, and prescribing nursing interventions. During implementation, nurses initiate nursing care, which is necessary to help patients achieve their goals. DIF: Remember REF: 222 | 223 OBJ: Discuss the relationship between critical thinking and evaluation. TOP: Evaluate MSC: CPNRE: Foundations of Practice 2. After assembling a thorough database and carrying out nursing interventions based on priority
N R I G B.C M
diagnoses, the nurse proceeds tU o wS hichNsteT p of theOnursing process? a. Assessment. b. Planning. c. Implementation. d. Evaluation. ANS: D
In the five-step nursing process, evaluation is the last step, after assessment, diagnosis, planning, and implementation. Assessment involves gathering information about the patient. Next, nursing diagnoses are determined. During the planning phase, patient outcomes are determined. Implementation involves carrying out appropriate nursing interventions. DIF: Understand REF: 222 | 223 OBJ: Discuss the relationship between critical thinking and evaluation. TOP: Evaluate MSC: CPNRE: Foundations of Practice 3. A nursing student asks her nursing instructor to describe the primary purpose of evaluation.
Which of the following statements made by the nursing instructor is most accurate? a. “During evaluation, you determine whether all nursing interventions were completed.” b. “During evaluation, you determine when to downsize staffing on nursing units.” c. “Nurses use evaluation to determine the effectiveness of nursing care.” d. “Evaluation eliminates unnecessary paperwork and care planning.”
Canadian Fundamentals of Nursing 6th Edition Potter Test Bank ANS: C
The purpose of evaluation is to determine the effectiveness of nursing care. The other options are not true statements. During evaluation, the nurse does not simply determine whether nursing interventions were completed. The evaluation process is not used to determine when to downsize staffing or how to eliminate paperwork and planning. DIF: Understand REF: 222 | 223 OBJ: Discuss the relationship between critical thinking and evaluation. TOP: Evaluate MSC: CPNRE: Foundations of Practice 4. After assessing the patient and identifying the need for headache relief, the nurse administers
acetaminophen (Tylenol) for the patient’s headache. What is the nurse’s next priority action for this patient? a. Eliminate Acute pain from the nursing care plan. b. Direct the nursing assistant to ask whether the patient’s headache is relieved. c. Reassess the patient’s pain level in 30 minutes. d. Revise the plan of care. ANS: C
The nurse’s next priority action for this patient is to evaluate whether the nursing intervention of administering acetaminophen was effective. The nurse does not have enough evaluative data at this point to determine whether the nursing diagnosis of Acute pain needs to be deleted. Assessment is the nurse’s responsibility and is not to be delegated to an unregulated care provider. The nurse does not have enough evaluative data to determine whether the patient’s plan of care needs to be revised. DIF: Apply REF: 225 ReS OBJ: Explain the relationship N beU tw enIgN oaG lsToB f c. arC e,OeM xpected outcomes, and evaluative measures in evaluating nursing care. TOP: Evaluate MSC: CPNRE: Foundations of Practice 5. A nurse is getting ready to discharge to home a patient who has a nursing diagnosis of
Impaired physical mobility. Before discontinuing the patient’s plan of care, what does the nurse need to do? a. Determine whether the patient has transportation to get home. b. Evaluate whether patient goals and outcomes have been met. c. Establish whether the patient has a follow-up appointment scheduled. d. Ensure that the patient’s prescriptions have been filled. ANS: B
The nurse needs to evaluate whether goals and outcomes have been met before revising, continuing, or discontinuing a plan of care. The patient needs transportation, but that does not address the patient’s mobility status. Whether the patient has a follow-up appointment and ensuring that prescriptions are filled are not immediately relevant to the problem of mobility. DIF: Apply REF: 224 | 225 OBJ: Describe how evaluation leads to discontinuation, review, or modification of a plan of care. TOP: Evaluate MSC: CPNRE: Foundations of Practice 6. The nurse is evaluating whether patient goals and outcomes have been met. Of the following,
which is an expected outcome for a patient with a diagnosis of Impaired physical mobility?
Canadian Fundamentals of Nursing 6th Edition Potter Test Bank a. b. c. d.
The patient is able to ambulate in the hallway with crutches. The patient’s level of mobility will improve. The nurse provides assistance while the patient is walking in the hallways. The patient will deny pain while walking in the hallway.
ANS: A
An outcome is an expected, favourable, and measurable result of nursing care. The patient’s being able to ambulate in the hallway with crutches is an expected outcome of nursing care. Improvement in the patient’s level of mobility is a broader goal. The nurse’s assisting a patient to ambulate is an intervention. The patient’s denial of pain is an expected outcome for Acute pain, not for Impaired physical mobility. DIF: Apply REF: 224 | 225 OBJ: Explain the relationship between goals of care, expected outcomes, and evaluative measures in evaluating nursing care. TOP: Evaluate MSC: CPNRE: Foundations of Practice 7. The nurse is evaluating whether a patient’s turning schedule was effective in preventing the
formation of pressure ulcers. Which finding indicates success of the turning schedule? a. Staff documentation of turning the patient every 2 hours. b. Absence of skin breakdown. c. Presence of redness only on the heels of the patient. d. Patient’s eating 100% of all meals. ANS: B
To determine whether a turning schedule is successful, the nurse needs to assess for the presence of skin breakdown. Redness on any part of the body, even if only on the patient’s heels, indicates that the turniN ngUsR che dule noCt O suM ccessful. Documentation of interventions I Gwa Bs . S N T does not evaluate whether patient outcomes were met. The patient’s eating 100% of meals does not indicate the effectiveness of a turning schedule. DIF: Evaluate REF: 224 | 225 OBJ: Explain the relationship between goals of care, expected outcomes, and evaluative measures in evaluating nursing care. TOP: Evaluate MSC: CPNRE: Foundations of Practice 8. A new nurse states that she is confused about using evaluative measures when caring for
patients and asks the charge nurse for examples and an explanation. Which of the following is the most accurate response from the charge nurse? a. “Evaluative measures are multiple-page documents used to evaluate nurse performance.” b. “Evaluative measures include assessment data used to determine whether patients have met their expected outcomes and goals.” c. “Evaluative measures are used by quality assurance nurses to determine the progress a nurse is making from novice to expert nurse.” d. “Evaluative measures are objective views of incident reports.” ANS: B
Evaluative measures are used to determine whether patients have met their goals and outcomes. Evaluative measures are not multiple-page documents, and they are used to assess the patient’s status, not the nurse’s performance. Evaluative measures are not used for completing an incident report.
Canadian Fundamentals of Nursing 6th Edition Potter Test Bank
DIF: Understand REF: 224-226 OBJ: Give an example of evaluative measures for determining patients’ progress toward an outcome. TOP: Evaluate MSC: CPNRE: Foundations of Practice 9. The nurse is caring for a patient who has an open wound. For evaluating the progress of
wound healing, what is the nurse’s priority action? a. Ask the unregulated care providers whether the wound looks better. b. Documenting the progress of wound healing as “better” in the patient’s chart. c. Measuring the wound and observe for redness, swelling, or drainage. d. Leaving the dressing off the wound for easier access and more frequent assessments. ANS: C
The nurse performs evaluative measures, such as completing a wound assessment, to evaluate wound healing. Nurses do not delegate assessment to unregulated care providers. Documenting “better” is subjective and does not objectively describe the wound. Leaving the dressing off for the nurse’s benefit of easier access is not a part of the evaluation process. DIF: Apply REF: 225 | 226 OBJ: Give an example of evaluative measures for determining patients’ progress toward an outcome. TOP: Evaluate MSC: CPNRE: Foundations of Practice 10. The nurse is caring for a patient who has an order to change a dressing twice a day, at 0600
and 1800 hours. At 1400 hours, the nurse notices that the dressing is saturated. What is the nurse’s next action? a. Wait and change the dressing at 1800 as ordered. b. Revise the plan of care anNdUcR haS nI geNtG heTdB re. ssC inO gM now. c. Reassess the dressing and the wound in 1 hour. d. Discontinue the plan of care. ANS: B
On the basis of evaluative data, the nurse revises, discontinues, or continues a patient’s plan of care. Because the dressing is saturated, the nurse needs to revise the plan of care and change the dressing now. Waiting until 1800 hours or for another hour is not appropriate because assessment data—that the dressing is saturated—indicate that the dressing needs to be changed now. Data are insufficient to support discontinuing the plan of care. DIF: Apply REF: 226-228 OBJ: Describe how evaluation leads to discontinuation, review, or modification of a plan of care. TOP: Evaluate MSC: CPNRE: Foundations of Practice 11. A goal for a patient with a nursing diagnosis of Ineffective coping is to demonstrate effective
coping skills. Which of these patient behaviours indicates that interventions performed to meet this outcome have been successful? a. Stating he feels better after talking with his family and friends. b. Continuing to consume several alcoholic beverages a day. c. Disliking the support group meetings. d. Spending most of the day in bed. ANS: A
Canadian Fundamentals of Nursing 6th Edition Potter Test Bank Evaluative data indicative of effective coping help the nurse determine whether the patient has met the outcome. Among the options listed, talking to family and friends is the only positive one. The other behaviours indicate lack of progress toward meeting the patient’s goal. DIF: Apply REF: 225-227 OBJ: Give an example of evaluative measures for determining patients’ progress toward an outcome. TOP: Evaluate MSC: CPNRE: Foundations of Practice 12. A nurse is providing education to a patient about self-administering subcutaneous injections.
Which of these patient statements indicates that the patient understands the instructions? a. “I need to use a needle 1/2 inch (1.3 cm) longer than my thumb.” b. “I will give the medicine deep into my deltoid.” c. “My belly is a good place to give my injection.” d. “I need to throw the syringe and needle into the garbage when I am done giving myself my shot.” ANS: C
The skin is made up of several layers. The outer layer is called the epidermis. The second layer of skin is the dermis. The connective tissue under the dermis is called the subcutaneous tissue. This is where subcutaneous injections are given. The abdomen is a good site for subcutaneous injections because this is an area that has a lot of subcutaneous tissue. Using a needle 1/2 inch (1.3 cm) longer than a person’s thumb is not an evidence-informed method for measuring needle length needed for subcutaneous injection. The deltoid is a muscle, not a subcutaneous site. Disposing of needles and syringes into a garbage can creates a biomedical hazard and therefore is not appropriate. DIF: Evaluate REF: 226 OBJ: Discuss the process of seNleU ctR inS gI nuN rsG inT gB in. terC veOnM tions. MSC: CPNRE: Foundations of Practice
TOP: Evaluate
13. Which of these statements made by a patient who has a nursing diagnosis of Disturbed body
image is the best indicator of the patient’s early acceptance of body image? a. “I just won’t go to the pool this summer.” b. “I’m worried about what those other girls will think of me.” c. “I can’t wear that colour. It makes my hips stick out.” d. “I’ll wear the blue dress. It matches my eyes.” ANS: D
The nurse evaluating interventions for the diagnosis Disturbed body image is assessing for positive comments made by the patient that indicate acceptance of the patient’s looks and body image. The only positive comment among the options is that the patient is wearing the blue dress to match her eyes. The other comments do not reflect positive changes in body image. DIF: Evaluate REF: 216 | 217, Table 14-2 OBJ: Give an example of evaluative measures for determining patients’ progress toward an outcome. TOP: Evaluate MSC: CPNRE: Foundations of Practice 14. Which of these options is a patient outcome indicating positive progress toward resolving the
nursing diagnosis of Acute confusion? a. Side rails are up with bed alarm activated.
Canadian Fundamentals of Nursing 6th Edition Potter Test Bank b. Patient denies pain while ambulating with assistance. c. Patient wanders halls at night. d. Patient correctly states names of family members in the room. ANS: D
The outcome for the identified nursing diagnosis Acute confusion would address a decrease in or absence of confusion. One sign of orientation is when a patient responds to questions appropriately. Thus one possible sign that a patient’s confusion is improving is that a patient can correctly state the names of family members in the room. Keeping the side rails up and using a bed alarm are interventions to promote patient safety and prevent falls. The patient’s denying pain indicates positive progress toward resolving a diagnosis of Acute pain or Chronic pain. The patient’s wandering the halls is a sign of confusion. DIF: Evaluate REF: 216 | 217, Table 14-2 OBJ: Give an example of evaluative measures for determining patients’ progress toward an outcome. TOP: Evaluate MSC: CPNRE: Foundations of Practice 15. A nurse identifies a nursing diagnosis of Risk for falls when assessing a patient upon
admission. The nurse and the patient agree that the goal is for the patient to remain free from falls. However, the patient fell just before shift change. What is the nurse’s priority action when evaluating the patient’s plan of care? a. Counsel the unregulated care provider on duty when the patient fell. b. Identify factors interfering with goal achievement. c. Remove the “Fall Risk” sign from the patient’s door because the patient has suffered a fall. d. Request that the more experienced charge nurse complete the documentation about the fall. ANS: B
NURSINGTB.COM
After a change in the patient’s condition or an untoward event, the nurse attempts to identify factors interfering with goal achievement. In this case, the nurse identifies factors that interfered with goal achievement to determine the cause of the fall. The fall may not have been due to an error by the unregulated care provider; therefore, counselling should be reserved until after the cause has been determined. The patient remains at risk for falls, so the “Fall Risk” sign should remain on the door. The nurse witnessing the fall or the nurse assigned to the patient needs to complete the documentation. The charge nurse can be consulted to review the documentation. DIF: Understand REF: 227 OBJ: Describe how evaluation leads to discontinuation, review, or modification of a plan of care. TOP: Evaluate MSC: CPNRE: Foundations of Practice 16. A patient recently received a diagnosis of pneumonia. The nurse and the patient have
established a goal that the patient will not experience shortness of breath with activity in 3 days, with an expected outcome of having no secretions present in the lungs in 48 hours. Which of the following is an appropriate evaluative measure demonstrating progress toward this goal? a. Nonproductive cough present in 4 days. b. Scattered rhonchi throughout all lung fields in 2 days. c. Respirations 30/minute in 1 day. d. Lungs clear to auscultation after use of inhaler.
Canadian Fundamentals of Nursing 6th Edition Potter Test Bank
ANS: D
Goals are broad statements that describe changes in a patient’s condition or behaviour. Expected outcomes are shorter-term measurable criteria used to evaluate goal achievement. When an outcome is met, the patient is making progress toward goal achievement. In this case, the patient’s goal is to not experience shortness of breath with activity in 3 days. One way to achieve this goal is to eliminate respiratory secretions in the airway. The nurse can evaluate this expected outcome by assessing the patient’s lung sounds. If the lung sounds are clear, at least periodically throughout the day, the nurse can determine that the patient is making progress toward achieving the expected outcome. The time frame of 4 days in the first option is not appropriate because this time frame exceeds the time frame stated in the goal. Scattered rhonchi indicate fluid in the lungs, and a respiratory rate of 30 per minute is higher than normal. Either of these indicates that the patient is still probably experiencing respiratory distress. DIF: Apply REF: 224 | 225 OBJ: Explain the relationship between goals of care, expected outcomes, and evaluative measures in evaluating nursing care. TOP: Evaluate MSC: CPNRE: Foundations of Practice 17. A nurse administrator is at a meeting with nurses on the quality council. Several new
members are sitting on the council. They ask the nurse administrator to clarify what a nursing-sensitive outcome is. Which response by the nurse administrator best defines nursing-sensitive outcomes? a. “Nursing-sensitive outcomes determine the patient’s progress as a result of prescribed treatments, such as medications.” b. “Patient falls is an example of a nursing-sensitive outcome because they are directly affected by nursiN ngUiR ntS erI veNnG tioTnB s.. ” COM c. “Nursing-sensitive outcomes promote universal health care.” d. “We use nursing-sensitive outcomes at this hospital to evaluate nursing tasks and to determine safe staffing ratios.” ANS: B
A nursing-sensitive outcome is a measurable patient or family state, behaviour, or perception that is largely influenced by and sensitive to nursing interventions. Patient falls are one nursing-sensitive outcome because they are a direct measure of nursing care. Because the prescriber, not the nurse, prescribes treatments, the progress of the patient’s condition as a result of prescribed treatments is not an evaluation of a nursing-sensitive outcome. Promotion of universal health care and determining staffing ratios are not components of nursing-sensitive outcomes. DIF: Apply REF: 225 OBJ: Give an example of evaluative measures for determining patients’ progress toward an outcome. TOP: Evaluate MSC: CPNRE: Foundations of Practice 18. Which scenario best illustrates the use of data validation when an independent nursing clinical
decision is made? a. The nurse determines that she needs to remove a wound dressing when the patient reveals the time of the last dressing change, and she notices that the present dressing is saturated with fresh and old blood. b. The nurse administers pain medicine due at 1700 hours at 1600 hours because the
Canadian Fundamentals of Nursing 6th Edition Potter Test Bank patient complains of increased pain. c. The nurse removes a leg cast when the patient complains of decreased mobility. d. The nurse administers potassium when a patient complains of leg cramps. ANS: A
Changing the wound dressing is the only independent nursing action given. The nurse validates what the patient says by observing the dressing. In addition, this option is the only assessment option that involves data validation. Administering pain medicine or potassium and removing a leg cast are examples of nursing interventions. DIF: Apply REF: 194 | 195 OBJ: Describe and compare direct and indirect nursing interventions. TOP: Assessment MSC: CPNRE: Foundations of Practice 19. Another term for a collaborative nursing intervention is which of the following? a. Dependent intervention. b. Independent intervention. c. Interdependent intervention. d. Physician-initiated intervention. ANS: C
A collaborative, or interdependent, intervention requires the combined knowledge, skill, and expertise of multiple health care providers. A dependent intervention requires an order from a health care provider. An independent intervention is an action that the nurse initiates. DIF: Remember REF: 214 OBJ: Describe and compare direct and indirect nursing interventions. TOP: Planning MSC: CP NNRRE: CIollaGborati B.veCPraMctice
U S N T
O
20. A registered nurse administers pain medication to a patient suffering from fractured ribs.
What type of nursing intervention is this nurse implementing? a. Collaborative. b. Independent. c. Interdependent. d. Dependent. ANS: D
The nurse does not have prescriptive authority to order pain medications, unless the nurse is an advanced practice nurse. The intervention is therefore dependent. A collaborative, or interdependent, intervention involves therapies that require combined knowledge, skill, and expertise from multiple health care providers. An independent intervention does not require an order or collaboration with other health care providers. DIF: Apply REF: 214 OBJ: Describe and compare direct and indirect nursing interventions. TOP: Planning MSC: CPNRE: Collaborative Practice 21. Which intervention is most appropriate for the nursing diagnosis Impaired verbal
communication related to loss of facial motor control and decreased sensation? a. Obtain an interpreter for the patient as soon as possible. b. Assist the patient in performing swallowing exercises each shift. c. Ask the family to provide a sitter to remain with the patient at all times.
Canadian Fundamentals of Nursing 6th Edition Potter Test Bank d. Provide the patient with a writing board each shift. ANS: D
The cause of the patient’s problem will help guide the nurse to the proper nursing intervention. If the patient has a problem with verbal communication, then the nurse should choose an intervention that will address the problem. Providing the patient with a writing board will allow the patient to communicate by writing because the patient is unable to communicate verbally at this time. Obtaining an interpreter might be an appropriate intervention if the patient spoke a foreign language. Assisting with swallowing exercises will help the patient with swallowing, the diagnosis for which is different from Impaired verbal communication. Asking the family to provide a sitter at all times is often unrealistic and does not promote the patient’s independence, as does use of a writing board. DIF: Apply REF: 215-218 OBJ: Discuss the process of selecting nursing interventions. MSC: CPNRE: Foundations of Practice
TOP: Planning
22. Which intervention is most appropriate for the nursing diagnosis Impaired skin integrity
related to shearing forces? a. Administer pain medication every 4 hours as needed. b. Perform the ordered dressing change twice daily. c. Do not document the wound appearance in the chart. d. Keep the bed side rails up at all times. ANS: B
The most appropriate intervention for the diagnosis of Impaired skin integrity is to perform the ordered dressing change. The other options do not directly address the skin integrity. The patient may need pain medicN atio n be hanges, but Acute pain would be another UR SIfore NGdre TBss.inCgOcM nursing diagnosis. Documenting all objective findings is the nurse’s responsibility, even if a wound or infection is a health care–associated problem. Keeping the side rails up addresses safety, not skin integrity. DIF: Apply REF: 215-218 OBJ: Discuss the process of selecting nursing interventions. MSC: CPNRE: Foundations of Practice
TOP: Planning
23. A patient has reduced muscle strength after a left-sided stroke and is at risk for falling. Which
intervention is most appropriate for the nursing diagnosis Risk for falls? a. Encouraging the patient to remain in bed most of the shift. b. Keeping all side rails down at all times. c. Placing the patient in a room away from the nurses’ station if possible. d. Assisting patient into and out of bed every 6 hours or as tolerated. ANS: D
Canadian Fundamentals of Nursing 6th Edition Potter Test Bank Risk for falls is a potential nursing diagnosis; therefore, the nurse needs to implement actions that will prevent a fall. Assisting the patient into and out of bed is the most appropriate intervention to prevent the patient from falling. Encouraging the patient to stay in bed will not help the patient increase muscle strength; decreased muscle strength is the risk factor placing the patient in jeopardy of falling. The side rails should be up, not down, according to agency policy. This will remind the patent to ask for help to get up and will keep the patient from rolling out of bed. The patient should be placed near the nurses’ station, so a staff member can quickly get to the room and assist the patient if necessary. DIF: Apply REF: 215-218 OBJ: Discuss the process of selecting nursing interventions. MSC: CPNRE: Foundations of Practice
TOP: Planning
24. A nurse is planning care for a patient with a nursing diagnosis of Impaired skin integrity. The
patient needs many nursing interventions, including a dressing change, several intravenous antibiotics, and a walk. Which factor does the nurse consider when prioritizing interventions? a. Putting all of the patient’s physician-initiated interventions as first priority. b. Considering time as an influencing factor. c. Setting priorities based solely on physiological factors. d. Not changing priorities once they’ve been established. ANS: B
Prioritizing is based on the patient needs. Time is a factor to be included in planning before continuing on to the implementation phase. The nurse avoids setting priorities solely on the basis of physiological factors and considers psychosocial factors as well. Priorities can change according to the patient’s needs and responses to treatments. The nurse avoids setting priorities on the basis of the type of nursing intervention.
N R I G B.C M U S N T O
DIF: Remember REF: 215-218 OBJ: Discuss the process of selecting nursing interventions. MSC: CPNRE: Foundations of Practice
TOP: Planning
25. Which of the following is an element of the evaluation process? a. Setting priorities for patient care. b. Collecting subjective and objective data to determine whether criteria or standards
are met. c. Ambulating 7.6 m (25 feet) in the hallway. d. Administering oxygen as ordered. ANS: B
During the evaluation process, the nurse gathers and documents objective and subjective data to determine whether the patient is meeting expected outcomes and is working toward achievement of goals. The evaluation process requires the use of critical thinking about attitudes and standards to analyze the nurse’s findings and to determine whether a plan of care needs to be terminated, continued, or revised. Setting priorities is part of planning. Ambulating with a patient in the hallway and administering oxygen are interventions, so they are included in the implementation step of the nursing process. DIF: Understand REF: 224-228 OBJ: Discuss the relationship between critical thinking and evaluation. TOP: Evaluate MSC: CPNRE: Foundations of Practice
Canadian Fundamentals of Nursing 6th Edition Potter Test Bank
Chapter 15: Documenting and Reporting Potter et al: Canadian Fundamentals of Nursing, 6th Edition MULTIPLE CHOICE 1. A nurse preceptor is working with a student nurse. Which behaviour by the student nurse will
prompt the nurse preceptor to intervene? a. The student nurse reviews the patient’s medical record. b. The student nurse reads the patient’s plan of care. c. The student nurse shares patient information with a friend. d. The student nurse documents medication administered to the patient. ANS: C
The Personal Information Protection and Electronic Documents Act (PIPEDA) is federal legislation that protects personal information, including health information, and is part of professional practice. Confidentiality and compliance with PIPEDA are mandatory. When a student nurse shares patient information with a friend, confidentiality and PIPEDA standards have been violated. A student can review patients’ medical records only to seek information needed to provide safe and effective patient care. For example, when nurses are assigned to care for a patient, they need to review the patient’s medical record and plan of care. They do not share this information with classmates, and they do not access the medical records of other patients on the unit. DIF: Apply REF: 237 | 238 OBJ: Identify ways to maintain confidentiality of records and reports. TOP: Evaluate MSC: CPNRE: Professional, Ethical, and Legal Practice
N R I G B.C M U S N T O
2. A nurse prepared an audio-recorded exchange with another nurse of information about a
patient. Which action did the nurse complete? a. Report. b. Record. c. Consultation. d. Referral. ANS: A
Reports are oral, written, or audio-recorded exchanges of information among caregivers. A patient’s record or chart is a confidential, permanent legal document consisting of information relevant to his or her health care. Consultations are another form of discussion in which one professional caregiver gives formal advice about the care of a patient to another caregiver. Nurses document referrals (arrangements for the services of another care provider). DIF: Remember REF: 238 OBJ: Describe the purpose and content of a change-of-shift report. TOP: Implementation MSC: CPNRE: Professional, Ethical, and Legal Practice 3. Which situation best indicates that the nurse has a good understanding of auditing and
monitoring of patients’ health records? a. The nurse determines the degree to which standards of care are met by reviewing patients’ health records. b. The nurse realizes that care not documented in patients’ health records still
Canadian Fundamentals of Nursing 6th Edition Potter Test Bank qualifies as care provided. c. The nurse knows that reimbursement is based on the diagnosis-related groups
documented in patients’ records. d. The nurse compares data in patients’ records to determine whether a new treatment
had better outcomes than the standard treatment. ANS: A
The patient record is a valuable source of data for all members of the health care team. Its purposes include communication, legal documentation, financial billing, education, research, and auditing/monitoring. The auditing/monitoring purpose involves nurses’ auditing records throughout the year to determine the degree to which standards of care are met and to identify areas needing improvement and staff development. The legal documentation purpose involves the concept that even though nursing care may have been excellent, in a court of law, “care not documented is care not provided.” The financial billing or reimbursement purpose involves diagnosis-related groups (DRGs) as the basis for establishing reimbursement for patient care. For research purposes, the researcher compares the patient’s recorded findings to determine whether the new method was more effective than the standard protocol. Analysis of data from research contributes to evidence-informed nursing practice and quality health care. DIF: Analyze TOP: Evaluate
REF: 234 OBJ: Identify purposes of a health care record. MSC: CPNRE: Professional, Ethical, and Legal Practice
4. After providing care, a nurse charts in the patient’s record. Which entry should the nurse
document? a. “Appears restless when sitting in the chair.” b. “Drank adequate amounts of water.” c. “Apparently is asleep with eyes closed.” N R I G B.C M U S N T O d. “Skin pale and cool.” ANS: D
A factual record contains descriptive, objective information about what a nurse sees, hears, feels, and smells. An objective description is the result of direct observation and measurement: for example, “BP 80/50, patient diaphoretic, heart rate 102 and regular.” Vague terms such as appears, seems, or apparently should be avoided because these words are suggestive of an opinion, do not accurately communicate facts, and do not inform another caregiver of details regarding behaviours exhibited by the patient. Use of exact measurements establishes accuracy. For example, a description such as “Intake, 360 mL of water” is more accurate than “Drank adequate amounts of water.” DIF: Apply REF: 239-241 OBJ: Describe six quality guidelines for documentation and reporting. TOP: Evaluate MSC: CPNRE: Professional, Ethical, and Legal Practice 5. A nurse has provided care to a patient. Which entry should the nurse document in the patient’s
record? a. “Patient seems to be in pain and states, ‘I feel uncomfortable.’” b. “Status unchanged, doing well.” c. “Left abdominal incision 5 cm in length without redness, drainage, or edema.” d. “Patient is hard to care for and refuses all treatments and medications. Family present.”
Canadian Fundamentals of Nursing 6th Edition Potter Test Bank ANS: C
Use of exact measurements establishes accuracy. Charting that an abdominal wound is “5 cm in length without redness, drainage, or edema” is more descriptive than “Large wound healing well.” The patient’s record should include objective data to support subjective data, so that charting is as descriptive as possible. Vague terms such as appears, seems, or apparently convey opinion rather than fact, do not accurately communicate facts, and do not inform another caregiver of details regarding behaviours exhibited by the patient. The nurse should avoid using generalized, uninformative phrases such as “Status unchanged” or “Had a good day.” It is essential to avoid the use of unnecessary words and irrelevant details or personal opinions. A statement such as “Patient is hard to care for” is a personal opinion and should be avoided. It is also a critical comment that can be used as evidence of nonprofessional behaviour or poor quality of care. The only statement to chart would be “Refuses all treatments and medications.” DIF: Apply REF: 239-241 OBJ: Describe six quality guidelines for documentation and reporting. TOP: Evaluate MSC: CPNRE: Professional, Ethical, and Legal Practice 6. A preceptor is working with a new nurse on documentation. Which situation will cause the
preceptor to intervene? a. The new nurse uses a black ink pen to chart. b. The new nurse charts consecutively on every other line. c. The new nurse ends each entry with signature and title. d. The new nurse keeps the password secure. ANS: B
Chart should be consecutive, line by line (not every other line); if space is left, a line should N it,RaSndItNheGnTurse’s B.CO M be drawn horizontally through U name should be signed at the end. No lines should be left blank. All entries should be written legibly and in black ink. Each entry should end with the nurse’s signature and title. For computer documentation, the nurse should keep the password secure. DIF: Apply REF: 235, Table 15-1 OBJ: Discuss legal guidelines for documentation. MSC: CPNRE: Professional, Ethical, and Legal Practice
TOP: Evaluate
7. A nurse is charting on a patient’s record. Which action is most accurate legally? a. Charting legibly. b. Stating that the patient is belligerent. c. Using correction fluid to correct error. d. Writing entry for another nurse. ANS: A
The nurse should record all entries legibly. Personal opinions should be avoided; only objective and factual observations of patient’s behaviour should be entered. All patient comments should be noted; for example, “Patient refuses to cough and deep breathe, saying, ‘I don’t care what you say, I will not do it.’” Entries should not be erased, covered with correction fluid, or scratched out, even if they are in error. Charting should be done only by the nurse who performed the action or made the observation. DIF: Understand
REF: 239-241
OBJ: Discuss legal guidelines for documentation.
Canadian Fundamentals of Nursing 6th Edition Potter Test Bank TOP: Planning
MSC: CPNRE: Professional, Ethical, and Legal Practice
8. A nurse wants to integrate all pertinent patient information into one record, regardless of the
number of times a patient enters the health care system. Which term should the nurse use to describe this system? a. Electronic medical record. b. Electronic health record. c. Electronic charting record. d. Electronic problem record. ANS: B
A unique feature of an electronic health record (EHR) is its ability to integrate all pertinent patient information into one record, regardless of the number of times a patient enters a health care system. The electronic medical record (EMR) contains patient data gathered in a health care setting at a specific time and place and is a part of the EHR; the two terms are frequently (and mistakenly) used interchangeably. There are no such terms as “electronic charting record” or “electronic problem record.” DIF: Understand REF: 236 OBJ: Discuss the use of electronic health records in documentation. TOP: Diagnosis MSC: CPNRE: Professional, Ethical, and Legal Practice 9. A nurse has taught the patient how to use crutches. The patient went up and down the stairs
using crutches with no difficulties. Which information will the nurse use for the “I” in PIE charting? a. “Patient went up and down stairs.” b. “Deficient knowledge regarding crutches.” c. “Demonstrated use of cruN tcU hR esS .”INGTB.COM d. “Used crutches with no difficulties.” ANS: C
A second progress note method is the PIE format. The narrative note includes a statement of the problem (P), the intervention (I), and the evaluation (E). The “I” is “Demonstrated use of crutches.” “Patient went up and down stairs” and “Used crutches with no difficulties” are examples of the “E.” “Deficient knowledge regarding crutches” is the “P.” DIF: Apply REF: 242 OBJ: Describe the different methods used in record keeping. MSC: CPNRE: Professional, Ethical, and Legal Practice
TOP: Implementation
10. A nurse is using the source record and wants to find the patient’s daily weights. Where should
the nurse look? a. Database. b. Medical history and examination. c. Progress notes. d. Graphic sheet and flow sheet. ANS: D
Canadian Fundamentals of Nursing 6th Edition Potter Test Bank In a source record, the patient’s chart has a separate section for each discipline (e.g., nursing, medicine, social work, respiratory therapy) in which to record data. Graphic sheets and flow sheets are records of repeated observations and measurements such as vital signs, daily weights, and intake and output. In the problem-oriented medical record, the database section contains all available assessment information pertaining to the patient (e.g., history and physical examination, the nurse’s admission history and ongoing assessment, the dietitian’s assessment, laboratory reports, radiological test results). In the source record, the medical history and examination section contains results of the initial examination performed by the physician, including findings, family history, confirmed diagnoses, and medical plan of care. Also in the source record, the progress notes constitute an ongoing record of the patient’s progress and response to medical therapy and a review of the disease process; these notes are often interdisciplinary and include documentation from health-related disciplines (e.g., health care providers, physiotherapy, social work). DIF: Apply REF: 243, Table 15-3 OBJ: Describe the different methods used in record keeping. MSC: CPNRE: Professional, Ethical, and Legal Practice
TOP: Planning
11. A nurse is a member of an interdisciplinary team that uses critical pathways. According to the
critical pathway, on day 2 of the hospital stay, the patient should be sitting in the chair. It is day 3, and the patient cannot sit in the chair. What should the nurse do? a. Focus charting, using the DAR format. b. Add this observation to the problem list. c. Document the variance in the patient’s record. d. Report a positive variance in the next interdisciplinary team meeting. ANS: C
NURSINGTB.COM
A variance is the situation when the activities on the critical pathway are not completed as predicted or the patient does not meet expected outcomes. An example of a variance is when a patient develops pulmonary complications after surgery, and oxygen therapy and monitoring with pulse oximetry are needed. A positive variance is a situation when a patient progresses more rapidly than expected (e.g., use of a Foley catheter may be discontinued a day early). When a nurse is using the problem-oriented medical record, after analyzing data, health care team members identify problems and make a single problem list. A type of narrative format charting is focus charting. It involves the use of DAR notes, which include data (both subjective and objective), action or nursing intervention, and response of the patient (i.e., evaluation of effectiveness). DIF: Apply REF: 243 | 244 OBJ: Describe the role of critical pathways in multidisciplinary documentation. TOP: Implementation MSC: CPNRE: Professional, Ethical, and Legal Practice 12. A nurse needs to begin discharge planning for a patient admitted with pneumonia and a
productive cough. When is the best time for the nurse to start discharge planning for this patient? a. Upon the patient’s admission. b. Right before the patient’s discharge. c. After the congestion is treated. d. When the primary care provider writes the order. ANS: A
Canadian Fundamentals of Nursing 6th Edition Potter Test Bank Ideally, discharge planning begins at admission. Right before discharge is too late for discharge planning. After the congestion is treated is also too late for discharge planning. Usually the primary care provider writes the order too close to discharge, and nurses do not need an order to begin the teaching that will be needed for discharge. By identifying discharge needs early, nursing and other health care providers can begin planning for home care, support services, and any equipment needs at the patient’s home. DIF: Apply REF: 245 OBJ: Identify elements to include when documenting a patient discharge plan. TOP: Planning MSC: CPNRE: Foundations of Practice 13. A patient is being discharged home. Which information should the nurse include? a. Acuity level. b. Community resources. c. Standardized care plan. d. Kardex. ANS: B
Discharge documentation includes information about medications, diet, community resources, follow-up care, and whom to contact in case of an emergency or for questions. A patient’s level of medical acuity, usually determined by a computer program, is based on the type and number of nursing interventions (e.g., intravenous [IV] therapy, wound care, ambulation assistance) required over a 24-hour period. Acuity level can be used for staffing and billing. Some institutions use standardized care plans to make documentation more efficient. The plans, based on the institution’s standards of nursing practice, are preprinted, established guidelines used to care for patients who have similar health problems. In some settings, a Kardex, a portable “flip-over” file or notebook, is kept at the nurses’ station. Most Kardex N tme RSntIsN GonB.C M forms have an activity and treaU ectiT and aOnursing care plan section, which organize information for quick reference. DIF: Apply REF: 245 OBJ: Identify elements to include when documenting a patient discharge plan. TOP: Planning MSC: CPNRE: Professional, Ethical, and Legal Practice 14. A nurse developed the following discharge summary sheet. Which critical information should
be added? Topic: Discharge Summary Medication Diet Activity level Follow-up care Wound care Phone numbers When to call the doctor Time of discharge a. Kardex form. b. Admission nursing history. c. Mode of transportation. d. SOAP notes. ANS: C
Canadian Fundamentals of Nursing 6th Edition Potter Test Bank In discharge summary information, the nurse lists actual time of discharge, mode of transportation, and who accompanied the patient. In some settings, a Kardex, a portable “flip-over” file or notebook, is kept at the nurses’ station. A Kardex is for nurses, not for patients to take upon discharge. A nurse completes a nursing history form when a patient is admitted to a nursing unit, not when the patient is discharged. SOAP notes are not given to patients who are being discharged. SOAP notes are a type of documentation style. DIF: Evaluate REF: 245, Box 15-5 OBJ: Identify elements to include when documenting a patient discharge plan. TOP: Evaluate MSC: CPNRE: Professional, Ethical, and Legal Practice 15. According to documentation guidelines, which notation is the most appropriate? a. “1230 hours: Patient’s vital signs taken.” b. “0700 hours: Patient drank adequate amount of fluids.” c. “0900 hours: Morphine given for lower abdominal pain.” d. “0830 hours: Increased IV fluid rate to 100 mL per hour.” ANS: D
Information within a recorded entry must be complete, containing appropriate and essential information. The notation “0830 hours: Increased IV fluid rate to 100 mL per hour” provides the time and action taken by the nurse, including the reason for doing so. The entry “1230 hours: Patient’s vital signs taken” does not indicate what the patient’s vital signs were. The entry “0700 hours: Patient drank adequate amount of fluids” does not provide the specific amount that the patient drank. Stating “adequate” is subjective, not objective. The notation “0900 hours: Morphine given for lower abdominal pain” does not have the patient describe his or her pain, or rate it according to a pain scale for comparison later. It also does not indicate whether the patient’s pain was in the lower left or lower right quadrant, or both.
N R I G B.C M U S N T O
DIF: Apply REF: 239-241 OBJ: Describe six quality guidelines for documentation and reporting. TOP: Implementation MSC: CPNRE: Professional, Ethical, and Legal Practice 16. The final “R” when using the I-SBAR-R communication technique represents which of the
following? a. Recovery. b. Repeat back. c. Reorganization. d. Reintegration. ANS: B
The final “R” in I-SBAR-R represents “repeat back,” which includes repeating back orders that have been given or clarifying any questions. The final “R” in I-SBAR-R does not stand for “recovery,” “reorganization,” or “reintegration.” DIF: Apply REF: 248, Box 15-9 OBJ: Discuss the advantages of standardized documentation forms. TOP: Implementation MSC: CPNRE: Professional, Ethical, and Legal Practice 17. A nurse is giving a hand-off report to the nurse on the next shift. Which information is critical
for the nurse to report? a. The patient had a good day with no complaints.
Canadian Fundamentals of Nursing 6th Edition Potter Test Bank b. The family is demanding and argumentative. c. The patient has a new pain medication, hydrocodone bitartrate and acetaminophen
(Lortab). d. The family is poor and had to go on welfare. ANS: C
Significant changes in the way therapies are to be given (e.g., “Different position for pain relief, new medication”) should be described to staff. Results should not be described simply as “good” or “poor”; they should be specific. Critical comments about patient’s or family’s behaviour are considered idle gossip and should not be mentioned. DIF: Apply REF: 246-248 OBJ: Describe the purpose and content of a change-of-shift report. TOP: Implementation MSC: CPNRE: Professional, Ethical, and Legal Practice 18. A new nurse asks the preceptor why a change-of-shift report is important, inasmuch as care is
documented in the chart. What is the preceptor’s best response? a. “A change-of-shift report provides an opportunity to share essential information to ensure patient safety and continuity of care.” b. “A change-of-shift report provides the oncoming nurse with data to help set priorities and establish reimbursement costs.” c. “A change-of-shift report provides an opportunity for the oncoming nurse to ask questions and determine research priorities.” d. “A change-of-shift report provides important information to caregivers and develops relationships within the health care team.” ANS: A
Properly performed, a changeN-of -shi ft N reG por t p.roCvid UR SI TB OMes an opportunity to share essential information to ensure patient safety and continuity of care. Reimbursement costs and research priorities/opportunities are functions of the medical record. Establishing relationships is not the purpose of the change-of-shift report. DIF: Remember REF: 246-248 OBJ: Describe the purpose and content of a change-of-shift report. TOP: Implementation MSC: CPNRE: Professional, Ethical, and Legal Practice 19. A nurse is preparing a change-of-shift report for a patient who had chest pain. Which
information is critical for the nurse to include? a. “Pupils equal and reactive to light.” b. “The family is a ‘pain.’” c. “Had poor results from the pain medication.” d. “Sharp pain of 8 on a scale of 1 to 10.” ANS: D
Elements in a change-of-shift report include identification of significant changes in measurable terms (e.g., pain scale) and by observation. Report elements do not include normal findings or routine information retrievable from other sources or derogatory or inappropriate comments about the patient or family, which could possibly lead to legal charges if overheard by the patient or family. This kind of language contributes to prejudicial opinions about the patient. Results should not be described simply as “good” or “poor”; they should be specific. DIF: Understand
REF: 246-248
Canadian Fundamentals of Nursing 6th Edition Potter Test Bank OBJ: Describe the purpose and content of a change-of-shift report. TOP: Planning MSC: CPNRE: Professional, Ethical, and Legal Practice 20. Which situation will require the nurse to obtain a telephone order? a. As the nurse and primary care provider leave a patient’s room, the primary care
provider gives the nurse an order. b. At 0100 hours, a patient’s blood pressure drops from 120/80 to 90/50 and the incision dressing is saturated with blood. c. At 0800, the nurse and primary care provider make rounds and the primary care provider tells the nurse a diet order. d. A nurse reads an order correctly as written by the primary care provider in the patient’s medical record. ANS: B
A nurse makes a telephone report when significant events or changes in a patient’s condition have occurred. Telephone orders and verbal orders usually occur at night or during emergencies. Because the time is 0100 hours (1 a.m.) and the primary care provider is not present, the nurse will need to call the primary care provider for a telephone order. A verbal order involves the health care provider giving orders to a nurse while they are standing near each other. Just reading an order that is correctly written in the chart does not require a telephone order. DIF: Apply TOP: Assessment
REF: 246 OBJ: Explain how to verify telephone orders. MSC: CPNRE: Professional, Ethical, and Legal Practice
21. A nurse obtained a telephone order (TO) from a primary care provider for a patient in pain.
Which chart entry should the nurse document? a. “12/16/20 0915 Tylenol 3N, U 2R taS blI etN s,G evTeB ry.6ChO ouMrs for incisional pain. VO Dr. Day/J. Winds, LPN, read back.” b. “12/16/20 0915 Tylenol 3, 2 tablets, every 6 hours for incisional pain. TO J. Winds, LPN, read back.” c. “12/16/20 0915 Tylenol 3, 2 tablets, every 6 hours for incisional pain. TO Dr. Day/J. Winds, LPN, read back.” d. “12/16/20 0915 Tylenol 3, 2 tablets, every 6 hours for incisional pain. TO J. Winds, LPN.” ANS: C
The nurse receiving a TO writes down the complete order or enters it into the computer as it is being given. Then he or she reads the order back to the health care provider, a procedure called read back, and the person who gave the order confirms that the read back is correct. “10/16/2011: 0815, Tylenol 3, 2 tablets, every 6 hours for incisional pain. TO Dr. Knight/J. Woods, LPN, read back” is such an example. VO stands for verbal order, not telephone order. The doctor’s name and read back must be included in the chart entry. DIF: Apply REF: 246 TOP: Implementation
OBJ: Explain how to verify telephone orders. MSC: CPNRE: Professional, Ethical, and Legal Practice
22. A nurse records the following: “Patient is wheezing and experiencing some dyspnea on
exertion.” This represents which of the following? a. The “S” in SOAP documentation. b. Focus documentation.
Canadian Fundamentals of Nursing 6th Edition Potter Test Bank c. The “P” of PIE documentation. d. The “R” in DAR documentation. ANS: C
The statement is an example of the “P” of PIE documentation because it describes the patient’s problem. The “S” in SOAP documentation represents subjective data (verbalizations of the patient). Focus charting does not concentrate only on problems. The “R” in DAR documentation is the response of the patient. The situation described concerns the patient’s problem, not the patient’s response. DIF: Analyze REF: 241, Box 15-1 OBJ: Describe the different methods used in record keeping. MSC: CPNRE: Professional, Ethical, and Legal Practice
TOP: Implementation
23. A hospital is using computer software that allows all health care providers to use a protocol
system to document the care they provide. Which type of system/design will the nurse be using? a. Clinical decision support system. b. Nursing process design. c. Critical pathway design. d. Computerized provider order entry system. ANS: C
One design model for nursing information systems (NIS) is the protocol or critical pathway design. With this design, all health care providers use a protocol system to document the care they provide, allowing them to select one or more appropriate protocols for a patient. A clinical decision support system is based on “rules” and “if-then” statements, linking information or producing aleN rts, war ning otC her information for the user. The nursing UR SI NGs,ToBr . OM process design is the most traditional design for an NIS. This design organizes documentation within well-established formats such as admission and postoperative assessments, problem lists, care plans, discharge planning instructions, and intervention lists or notes. Computerized provider order entry (CPOE) is a process by which the health care provider directly enters orders for patient care into the hospital information system. DIF: Understand REF: 249 | 250 OBJ: Describe the role of critical pathways in interdisciplinary documentation. TOP: Evaluate MSC: CPNRE: Professional, Ethical, and Legal Practice 24. A nurse wants to reduce data entry errors on the computer system. Which behaviour should
the nurse implement? a. Use the same password all the time. b. Share password with only one other staff member. c. Print out and review computer nursing notes at home. d. Chart on the computer immediately after care is provided. ANS: D
Canadian Fundamentals of Nursing 6th Edition Potter Test Bank To increase accuracy and decrease unnecessary duplication, many health care agencies keep records or computers near a patient’s bedside to facilitate immediate documentation of information as it is collected from a patient. A good system requires frequent and random changes in personal passwords to prevent unauthorized persons from tampering with records. When a nurse uses a health care agency computer system, it is essential that his or her computer password not be shared with anyone under any circumstances. The nurse destroys (e.g., shreds) anything that is printed when the information is no longer needed. Taking nursing notes home is a violation of the Health Insurance Portability and Accountability Act (HIPAA) and confidentiality. DIF: Apply REF: 240 | 249 | 250 OBJ: Discuss the use of electronic health records in documentation. TOP: Implementation MSC: CPNRE: Professional, Ethical, and Legal Practice 25. Which entry necessitates follow-up by the nurse manager? a. 0800 Patient states, “Fell out of bed.” Patient found lying by bed on the floor. Legs
equal in length bilaterally with no distortion, pedal pulses strong, leg strength equal and strong, no bruising or bleeding. Neuro checks within normal limits. States, “Did not pass out.” Assisted back to bed. Call bell within reach. Bed monitor on.—Jane More, LPN b. 0810 Notified primary care provider of patient’s status. New orders received. —Jane More, LPN c. 0815 Portable x-ray of L hip taken in room. States, “I feel fine.” —Jane More, LPN d. 0830 Incident report completed and placed on chart.—Jane More, LPN ANS: D
NURSINGTB.COM
The incident report is not mentioned in the patient’s medical record. Instead the nurse documents in the patient’s medical record an objective description of what happened, what the nurse observed, and follow-up actions taken. It is important to evaluate and document the patient’s response to the error or incident. The patient’s health care provider should be contacted whenever an incident happens. DIF: Analyze TOP: Evaluate
REF: 248 | 249 OBJ: Discuss legal guidelines for documentation. MSC: CPNRE: Professional, Ethical, and Legal Practice
26. The action that a nurse would take when documenting on the patient’s record and notes that
he or she has made an error is which of the following? a. Drawing a line through the error and initialling and dating it. b. Erasing the error and writing over the material in the same spot. c. Using a dark-coloured marker to cover the error and continuing immediately after that point. d. Footnoting the error at the bottom of the page, including initials and the date. ANS: A
Canadian Fundamentals of Nursing 6th Edition Potter Test Bank If the nurse has made an error in documentation, he or she should draw a single line through the error, write the word “error” above it, and sign his or her name or initials. Then the nurse should record the note correctly. The nurse should not erase, apply correction fluid to, or scratch out errors made while recording because charting then becomes illegible. Entries should be made only in ink so that they cannot be erased. Using a dark-coloured marker to cover the error and continuing immediately after that point appears as if the nurse were attempting to hide something or deface the record. Footnotes are not used in nursing documentation. DIF: Apply REF: 235, Table 15-1 OBJ: Discuss legal guidelines for documentation. MSC: CPNRE: Professional, Ethical, and Legal Practice
TOP: Implementation
27. A slight hematoma has developed on the patient’s left forearm. The nurse labels the problem
as an infiltrated intravenous (IV) line. The nurse elevates the forearm. The patient states, “My arm feels better.” When using the DAR notes of focus charting, the nurse would document the “R” as which of the following? a. “My arm feels better.” b. “Slight hematoma on left forearm.” c. “Infiltrated IV line.” d. “Elevation of left forearm.” ANS: A
The “R” in the data–action–response (DAR) documentation of focus charting is the response by the patient. In this case, the nurse would document, “Patient states, ‘My arm feels better.’ “Slight hematoma on left forearm” and “Infiltrated IV line” are examples of the “D” in a DAR note, referring to data in focus charting. “Elevation of left forearm” is the “A” in a DAR note, N R erv IN G onB.C M describing the action or nursingUintS entiT in foO cus charting. DIF: Apply REF: 242 OBJ: Describe the different methods used in record keeping. MSC: CPNRE: Professional, Ethical, and Legal Practice
TOP: Implementation
28. A nurse is discussing the advantages of standardized documentation forms in the nursing
information system. Which advantage should the nurse describe? a. Varied clinical databases. b. Reduced errors of omission. c. Increased hospital costs. d. More time to read charts. ANS: B
Advantages associated with the nursing information system include increased time to spend with patients (not to read charts); better access to information; enhanced quality of documentation; reduction in errors of omission; reduced, not increased, hospital costs; increased nurse job satisfaction; compliance with requirements of accrediting agencies (e.g., TJC); and development of a common, not varied, clinical database. DIF: Understand REF: 250 OBJ: Discuss the advantages of standardized documentation forms. TOP: Implementation MSC: CPNRE: Professional, Ethical, and Legal Practice
Canadian Fundamentals of Nursing 6th Edition Potter Test Bank 29. A nurse is creating a plan to reduce data entry errors and maintain confidentiality. Which
guidelines should the nurse include? a. Create a password with just letters. b. Bypass the firewall. c. Avoid use of a programmed speed-dial key when faxing. d. Shred papers containing personal health information. ANS: D
All papers containing private health information must be destroyed. Most agencies have shredders or locked receptacles for shredding and later incineration. Strong passwords have combinations of letters, numbers, and symbols that are difficult to guess. A firewall is a combination of hardware and software that protects private network resources (e.g., the information system of the hospital) from outside hackers, network damage, and theft or misuse of information and should not be bypassed. When faxing, the nurse should use programmed speed-dial keys to eliminate the chance of a dialling error and misdirected information. DIF: Create REF: 237 | 238 OBJ: Identify ways to maintain confidentiality of electronic and written records. TOP: Evaluate MSC: CPNRE: Professional, Ethical, and Legal Practice
NURSINGTB.COM
Canadian Fundamentals of Nursing 6th Edition Potter Test Bank
Chapter 16: Nursing Informatics and Canadian Nursing Practice Potter et al: Canadian Fundamentals of Nursing, 6th Edition MULTIPLE CHOICE 1. Of the following, which is a current technological application that is critical for supporting
clinical judgement, decision making, and optimal patient outcomes? a. Patient documentation systems. b. Laboratory reporting software. c. Diagnostic imaging systems. d. Billing and financial management databases. ANS: A
Patient documentation systems inform nursing practice through supporting clinical judgement and decision making, as well as supporting optimal nursing care. In addition, effective documentation supports aggregation with documentation from other nurses and ensuring optimal patient outcomes. Laboratory reporting does not necessarily inform effective nursing practice or support optimal patient outcomes; it is an example of a current technological application. Diagnostic imaging is a current technological application tool. Billing and financial management do not optimize nursing outcomes. DIF: Remember REF: 254 OBJ: Identify and compare Canadian strategies for identifying and documenting key nursing data. TOP: Implementation MSC: CPNRE: Professional, Ethical, and Legal Practice 2. As nurses continue to contextualize technology within the scope of their professional practice,
N R I G B.C M
what is the main nursing issue U withStecNhnoTlogy thO at requires consideration? a. Computers. b. Management information systems. c. Information and information management. d. Order entry applications. ANS: C
The issues for nurses are no longer computers or management information systems but rather information and information management. The computer and its associated software are merely tools to support nurses as they practise their profession. Computers and management information systems are no longer issues for nurses. Order entry applications are current technological applications. DIF: Understand REF: 255 OBJ: Identify and compare Canadian strategies for identifying and documenting key nursing data. TOP: Implementation MSC: CPNRE: Professional, Ethical, and Legal Practice 3. Nursing informatics is a specialty area of nursing practice dedicated to the optimal use of
technology to support professional practice and optimal patient outcomes. Which of the following is a significant obstacle to the collection of nursing data? a. Canada Health Infoway. b. International Classification for Nursing Practice (ICNP), which lacks unified terminology for recording nursing practice. c. Lack of evidence-informed practices for health information.
Canadian Fundamentals of Nursing 6th Edition Potter Test Bank d. Absence of universally accepted methods for defining and coding nursing
contributions to health outcomes. ANS: D
Despite the escalation of the use of technology in health care and the recognition of nursing informatics, no methods for defining and coding nursing contributions to health outcomes are universally accepted, and this persistent absence is a significant obstacle to the collection of nursing data. The other options are not obstacles to collecting nursing data. The Canada Health Infoway is an incorporated national body with mandates to generate consensus on health information standards, to drive forward a national agenda of creating an electronic health record (EHR), and to act as the liaison to international standards and development organizations. The ICNP was developed by the International Council of Nurses; it comprises seven axes with associated terms for describing nursing practice and is the only unified international terminology for recording nursing practice. As health care systems respond to an increasingly complex technological environment, long-standing routines and tools are being superseded by strategic, evidence-informed practices that mandate high-quality, timely health information. DIF: Understand REF: 255 OBJ: Discuss why using standardized nursing data is important for acknowledging the professional contributions of nursing to health outcomes of Canadians. TOP: Planning MSC: CPNRE: Professional, Ethical, and Legal Practice 4. According to the Canadian Organization for Advancement of Computers in Health (COACH),
what is health informatics? a. The intersection of clinical, information management/information technology, and management practices to achieve better health. NUs R I gGscTience, B.CcOomputer M b. The specialty that integrate nuS rsinN science, and information science. c. The application of computer science and information science to nursing. d. A clinical term used to describe multidisciplinary clinical practice. ANS: A
COACH defines health informatics as the “intersection of clinical, IM/IT [information management/information technology] and management practices to achieve better health” (COACH, 2009, p. 7). Health informatics covers all disciplines. Role-centred/role-based nursing informatics was a step in the evolution of health informatics that integrates nursing science, computer science, and information science, according to the American Nurses Association. The application of computer science and information science to nursing is the definition of nursing informatics by the Canadian Nurses Association (CNA). A clinical term used to describe multidisciplinary clinical practice is Systematized NOmenclature of MEDicine—Clinical Terms (SNOMED CT); this is not how COACH defines health informatics. DIF: Understand REF: 255 | 256 OBJ: Differentiate how nursing informatics differs from routine use of technologies in nursing practice. TOP: Assessment MSC: CPNRE: Professional, Ethical, and Legal Practice 5. The Management Information System (MIS) Group was an important organization in
Canadian health care information management but this group did not collect which of the following?
Canadian Fundamentals of Nursing 6th Edition Potter Test Bank a. b. c. d.
Demographic data. Clinical nursing data. Statistical medical data. Resource utilization data.
ANS: B
Clinical nursing data were not collected by the MIS Group because the information was restricted to physician-driven data. The MIS Group did collect demographic data, statistical medical data, and resource utilization data. DIF: Understand REF: 256 OBJ: Discuss how health information data standards influence Canadian nursing practice. TOP: Assessment MSC: CPNRE: Professional, Ethical, and Legal Practice 6. Which institute was developed as a result of the Wilk Report? a. Hospital Medical Records Institute (HMRI). b. Statistics Canada and Health and Welfare Institute. c. Canadian Nursing Informatics Association. d. Canadian Institute for Health Information (CIHI). ANS: D
The Wilk Report (National Task Force on Health Information, 1991) had a significant effect on Canadian health information, triggering the 1993 merger of the MIS Group, HMRI, portions of Statistics Canada, and Health and Welfare Canada to create the CIHI. The Canadian Nursing Informatics Association was not developed as a result of the Wilk Report. DIF: Understand REF: 256 OBJ: Discuss how health infoN rmatR ion I dataGstanB dards inM fluence Canadian nursing practice. U SProfessional, N T .CEthical, O and Legal Practice TOP: Assessment MSC: CPNRE: 7. The CIHI disseminates essential data and analysis on Canada’s health care system and the
health of Canadians. The CIHI influenced issues directly related to nursing by including issues related to which of the following? a. Nursing science and information science to support patients. b. Nursing workforce recruitment and retention. c. Clinical practice, administration, research, and education. d. Advocating for health policy. ANS: B
The CIHI is the national, independent, and not-for-profit body that records, analyzes, and disseminates data and analysis on Canada’s health care system and the health of Canadians. Although not initially attentive to nursing data, the CIHI later became more important to several issues directly influencing nursing, including issues related to nursing workforce recruitment and retention. Nursing science, information science, and supporting patients is part of the definition of role-centred nursing informatics. Clinical practice, administration, research, and education are part of the definition of nursing informatics related to information technology. Advocating for health policy is the mandate of the ICN, not the CIHI. DIF: Understand REF: 256 OBJ: Discuss how health information data standards influence Canadian nursing practice. TOP: Assessment MSC: CPNRE: Professional, Ethical, and Legal Practice
Canadian Fundamentals of Nursing 6th Edition Potter Test Bank 8. Canada Health Infoway was a key outcome of the federal/provincial/territorial partnership.
What is its mandate? a. To create an electronic health record system. b. To unify international terminology for recording nursing practice. c. To advocate for effective health policy. d. To implement provincial/territorial standards. ANS: A
Infoway has a mandate to drive forward a national agenda to create an electronic health record system. Unifying international terminology for recording nursing practice is associated with the ICNP, not Canada Health Infoway. Advocating for effective health policy is associated with the International Council of Nurses, not Canada Health Infoway. Canada Health Infoway does have a mandate to generate consensus on health information standards but not to implement provincial/territorial standards. DIF: Understand REF: 256 OBJ: Identify and compare Canadian strategies for identifying and documenting key nursing data. TOP: Implementation MSC: CPNRE: Professional, Ethical, and Legal Practice 9. A patient has had several admissions to his local hospital for asthma and was recently
transferred to another hospital to have hip replacement surgery. What is the benefit of a standardized EHR? a. Timely access to health data. b. Improved coordination of care. c. Enhanced ability to work collaboratively with other organizations. d. Enhanced ability to extract best practice data from published sources. ANS: B
NURSINGTB.COM
The patient’s health information could be transferred from the local hospital to the hospital where hip replacement surgery will be done. The patient would benefit from a standardized EHR by improved coordination of care, reduced repetition of health information, and reduced duplication of tests and procedures. Health care providers benefit from timely access to health data benefits. Service delivery organizations benefit from the enhanced ability to work collaboratively with other organizations. Educators benefit from the enhanced ability to extract best practice data from published sources. DIF: Analyze REF: 255 | 256 OBJ: Discuss the relationship between national privacy legislation and nursing practice in a digital practice environment. TOP: Implementation MSC: CPNRE: Professional, Ethical, and Legal Practice 10. Which of the following Health Information: Nursing Component (HI:NC) can be described as
the amount and type of nursing resource used to provide care? a. Patient status. b. Nursing interventions. c. Nursing intensity. d. Patient outcome. ANS: C
NURSINGTB.COM
Canadian Fundamentals of Nursing 6th Edition Potter Test Bank Nursing intensity is the amount and type of nursing resource used to provide care. Patient status is the health status of patients. Nursing interventions are the purposeful and deliberate health-affecting interventions. Patient outcome is the patient status at a defined point after health care intervention. DIF: Understand REF: 260 OBJ: Identify and compare Canadian strategies for identifying and documenting key nursing data. TOP: Planning MSC: CPNRE: Professional, Ethical, and Legal Practice 11. Which of the following is a Health Information: Nursing Component (HI:NC)? a. Patient diagnosis. b. Educational background. c. Primary nurse identifier. d. Independent nursing judgement. ANS: C
One of the five HI:NCs is primary nurse identifier. The remaining four are patient status, nursing intervention, patient outcome, and nursing intensity. Patient diagnosis, educational background, and independent nursing are not among the five HI:NCs. DIF: Understand REF: 259 | 260 OBJ: Identify and compare Canadian strategies for identifying and documenting key nursing data. TOP: Planning MSC: CPNRE: Professional, Ethical, and Legal Practice 12. The seven axes of the ICNP are which of the following? a. Assessment, Analysis, Intervention, Action, Outcome, and Evaluation. b. Focus, Judgement, Action, Means, Location, Client, and Time. c. Status, Intervention, OutcN oU mR e,SInIteNnG sity , I. deCnO tifM ier, Client, and Time. TB d. Focus, Standards, Delivery, Outcomes, Intensity, Evaluation, and Time. ANS: B
ICNP has a model with seven axes: Focus, Judgement, Action, Means, Location, Client, and Time. The INCP is a standardized terminology for nursing practice and is built with the Web Ontology Language (OWL). DIF: Remember REF: 260 OBJ: Develop a beginning understanding of the scope of nursing informatics concepts and the ways in which nurses can be involved in nursing informatics. TOP: Planning MSC: CPNRE: Professional, Ethical, and Legal Practice 13. Canadian privacy legislation indicates which of the following? a. All privacy legislation is developed at the provincial/territorial level. b. Privacy legislation only addresses protection of personal health information. c. Privacy legislation cannot be violated if a nurse is practising within his or her
standards of practice. d. An understanding of the privacy legislation can assist nurses in upholding the
Code of Ethics. ANS: D
Canadian Fundamentals of Nursing 6th Edition Potter Test Bank An understanding of both the Privacy Act and the Personal Information Protection and Electronic Documents Act can assist nurses in upholding the Code of Ethics, as well as the standards for nursing practice. Some privacy legislation, but not all, is developed provincially and territorially; there are also federal privacy laws. Privacy legislation addresses the protection of personal health information and other personal information. It is possible for a nurse to violate privacy legislation even though the nurse is fulfilling his or her standards of practice. DIF: Understand REF: 262 OBJ: Discuss the relationship between national privacy legislation and nursing practice in a digital practice environment. TOP: Planning MSC: CPNRE: Professional, Ethical, and Legal Practice 14. Which of the following is one of the three fundamental directions for the CNA’s e-Nursing
Strategy? a. Assessment. b. Participation. c. Collaborative care. d. Nursing practice integration. ANS: B
Participation is one of the three fundamental directions for the CNA’s e-Nursing Strategy. The other two directions are access and competencies. Assessment, collaborative care, and nursing practice integration are not among the three fundamental directions for the CNA’s e-Nursing Strategy. DIF: Understand REF: 264-266 OBJ: Discuss how the CanadiN anUNR urS seI sN AG ssT ocB ia. tioC n'O s nMational e-Nursing Strategy will influence current and future nursing practice. TOP: Implementation MSC: CPNRE: Professional, Ethical, and Legal Practice 15. Nurses can create discussion groups online with which current technology? a. Skype. b. NurseONE. c. NurseConnect. d. COACH. ANS: C
NurseConnect is a portal that enables nurses to create discussion groups among its subscribers. It is part of NurseONE. Skype is not a means for nurses to create discussion groups online. NurseONE is the Canadian Nurses Portal, and it provides professional links, professional development, Library, and NurseConnect. COACH is the acronym for the Canadian Organization for the Advancement of Computers in Health and is not a means for nurses to create discussion groups online. DIF: Remember REF: 265 OBJ: Discuss how the Canadian Nurses Association's national e-Nursing Strategy will influence current and future nursing practice. TOP: Implementation MSC: CPNRE: Professional, Ethical, and Legal Practice
Canadian Fundamentals of Nursing 6th Edition Potter Test Bank
Chapter 17: Communication and Relational Practice Potter et al: Canadian Fundamentals of Nursing, 6th Edition MULTIPLE CHOICE 1. What characterizes nurses who make the best communicators? a. Developing critical thinking skills. b. Liking different kinds of people. c. Learning effective psychomotor skills. d. Maintaining perceptual biases. ANS: A
Nurses who develop critical thinking skills make the best communicators. Critical thinking helps the nurse overcome perceptual biases or human tendencies that interfere with accurately perceiving and interpreting messages from others. Just liking people does not make for effective communication because it is important to apply critical thinking standards to ensure sound effective communication. Just learning psychomotor skills does not ensure that the nurse will use those techniques, and communication involves more than psychomotor skills. Nurses who maintain perceptual biases do not make good communicators. DIF: Remember REF: 272 | 273 OBJ: Describe aspects of critical thinking that are important to the communication process. TOP: Implementation MSC: CPNRE: Professional, Ethical, and Legal Practice 2. A nurse believes that the nurse–patient relationship is a partnership and that both are equal
participants. Which term should the nurse use to describe this belief? N R I G B.C M a. Critical thinking. U S N T O b. Authentic. c. Mutuality. d. Attending. ANS: C
Effective interpersonal communication requires a sense of mutuality, a belief that the nurse–patient relationship is a partnership and that both are equal participants. Critical thinking in nursing, based on established standards of nursing care and ethical standards, promotes effective communication and uses such standards as humility, self-confidence, independent attitude, and fairness. To be authentic (one’s self) and to respond appropriately to the other person are important for interpersonal relationships but do not mean mutuality. Attending is giving all of one’s attention to the patient. DIF: Understand REF: 272 OBJ: Identify significant features and therapeutic outcomes of nurse–patient helping relationships. TOP: Planning MSC: CPNRE: Professional, Ethical, and Legal Practice 3. A nurse wants to present information about flu immunizations to older persons in the
community. Which type of communication should the nurse use? a. Interpersonal. b. Public. c. Transpersonal. d. Small group.
Canadian Fundamentals of Nursing 6th Edition Potter Test Bank
ANS: B
Public communication is interaction with an audience. Nurses have opportunities to speak with groups of consumers about health-related topics, present scholarly work to colleagues at conferences, or lead classroom discussions with peers or students. Intrapersonal communication is a powerful form of communication that occurs within an individual. Transpersonal communication is interaction that occurs within a person’s spiritual domain. When nurses work on committees, lead patient support groups, form research teams, or participate in patient care conferences, they use a small group communication process. DIF: Apply REF: 274 OBJ: Describe the five levels of communication and their uses in nursing. TOP: Planning MSC: CPNRE: Professional, Ethical, and Legal Practice 4. Which communication technique is being used most effectively with which scenario? a. Interpersonal communication to change negative self-talk to positive self-talk. b. Small group communication to present information to an audience. c. Intrapersonal communication to build strong teams. d. Transpersonal communication to enhance meditation. ANS: D
Transpersonal communication is interaction that occurs within a person’s spiritual domain. Many people use prayer, meditation, guided reflection, religious rituals, or other means to communicate with their “higher power.” Interpersonal communication is one-on-one interaction between the nurse and another person that often occurs face to face. Meaningful interpersonal communication results in exchange of ideas, problem solving, expression of feelings, decision making, goal accomplishment, team building, and personal growth. Small group communication is inteN ract iR onSI that Gocc Bur.s Cwhe Mn a small number of persons meet. This U N T O type of communication is usually goal directed and requires an understanding of group dynamics. When nurses work on committees, lead patient support groups, form research teams, or participate in patient care conferences, they use a small group communication process. Intrapersonal communication is a powerful form of communication that occurs within an individual. For example, people improve their health and self-esteem through positive self-talk by replacing negative thoughts with positive assertions. DIF: Evaluate REF: 273 | 274 OBJ: Describe the five levels of communication and their uses in nursing. TOP: Evaluate MSC: CPNRE: Professional, Ethical, and Legal Practice 5. A nurse is standing beside the patient’s bed. The nurse asks, “How are you doing?” The
patient responds, “I don’t feel good.” In this situation, which element is the feedback? a. The nurse. b. The patient. c. “How are you doing?” d. “I don’t feel good.” ANS: D
“I don’t feel good” is the feedback because the feedback is the message the receiver returns to the sender. The sender is the person who encodes and delivers the message, and the receiver is the person who receives and decodes the message. The nurse is the sender. The patient is the receiver. “How are you doing?” is the message.
Canadian Fundamentals of Nursing 6th Edition Potter Test Bank DIF: Apply REF: 274 OBJ: Describe the basic elements of the communication process. TOP: Assessment MSC: CPNRE: Professional, Ethical, and Legal Practice 6. A nurse is sitting at the patient’s bedside documenting a nursing history. Which zone of
personal space is the nurse using? a. Intimate. b. Personal. c. Social. d. Public. ANS: B
Personal space is 45 cm to 1 m (~18 inches to ~4 feet) away from a person and is used when the nurse is sitting at a patient’s bedside, taking a patient’s nursing history, or teaching an individual patient. Intimate space is 0 to 45 cm (0 to 18 inches) and is used in such activities as performing a physical assessment, bathing, grooming, dressing, feeding, and toileting a patient. The social zone is 1 to 4 m (4 to 12 feet) and is used in such activities as making rounds with a physician, sitting at the head of a conference table, or teaching a class for patients with diabetes. The public zone is 4 m (12 feet) and greater and is used in such activities as speaking at a community forum, testifying at a legislative hearing, or lecturing. DIF: Remember REF: 276, Box 17-2 OBJ: Describe the basic elements of the communication process. TOP: Implementation MSC: CPNRE: Professional, Ethical, and Legal Practice 7. A smiling patient angrily states, “I will not cough and deep breathe.” How will the nurse
interpret this finding? a. The patient’s personal spN acU eR wS asIvN ioG laT teB d..COM b. The patient’s affect is inappropriate. c. The patient’s vocabulary is poor. d. The patient’s denotative meaning is wrong. ANS: B
An inappropriate affect is a facial expression that does not match the content of a verbal message (e.g., smiling when describing a sad situation). The patient is smiling but is angry, which indicates an inappropriate affect. The patient’s personal space was not violated. The patient’s vocabulary is not poor. Individuals who use a common language share denotative meaning: baseball has the same meaning for everyone who speaks English, but code denotes cardiac arrest primarily to health care providers. The patient’s denotative meaning is correct for cough and deep breathe. DIF: Evaluate REF: 275 | 276 OBJ: Describe the basic elements of the communication process. TOP: Evaluate MSC: CPNRE: Professional, Ethical, and Legal Practice 8. The nurse asks a patient where the pain is, and the patient responds by pointing to the area of
pain. Which form of communication did the patient use? a. Verbal. b. Nonverbal. c. Intonation. d. Vocabulary.
Canadian Fundamentals of Nursing 6th Edition Potter Test Bank ANS: B
The patient gestured (pointed), which is a type of nonverbal communication. Gestures emphasize, punctuate, and clarify the spoken word. Pointing to an area of pain is sometimes more accurate than describing its location. Verbal is the spoken word or message. Intonation or tone of voice dramatically affects the meaning of a message. Vocabulary consists of words used for verbal communication. DIF: Understand REF: 275 | 276 OBJ: Describe the basic elements of the communication process. TOP: Assessment MSC: CPNRE: Professional, Ethical, and Legal Practice 9. A patient has been admitted to the hospital numerous times. The nurse asks the patient to
share a personal story about the care that has been received. Which interaction is the nurse using? a. Narrative. b. Socializing. c. Nonjudgemental. d. SBAR. ANS: A
In a therapeutic relationship, nurses often encourage patients to share personal stories. Sharing stories is called narrative interaction. Socializing is an important initial component of interpersonal communication. It helps people get to know one another and relax. It is easy, superficial, and not deeply personal. Nonjudgemental acceptance of the patient, an important characteristic of the relationship, conveys a willingness to hear a message or acknowledge feelings; it is not a technique that involves personal stories. SBAR (situation, background, assessment, and recommendation) is a popular communication tool that helps standardize N R I G B.C M communication among health cUareSproN videTrs. O DIF: Remember REF: 278 OBJ: Identify significant features and therapeutic outcomes of nurse–patient helping relationships. TOP: Assessment MSC: CPNRE: Professional, Ethical, and Legal Practice 10. Before meeting the patient, a nurse talks to other caregivers about the patient. The nurse is in
which phase of the helping relationship? a. Preinteraction. b. Orientation. c. Working. d. Termination. ANS: A
Canadian Fundamentals of Nursing 6th Edition Potter Test Bank The time before the nurse meets the patient is called the preinteraction phase. This phase can involve such things as reviewing available data, including the medical and nursing history; talking to other caregivers who have information about the patient; and anticipating health concerns or issues that can arise. During the orientation phase, the nurse and the patient meet and get to know one another. This phase can involve such things as setting the tone for the relationship by adopting a warm, empathetic, caring manner; recognizing that the initial relationship is often superficial, uncertain, and tentative; or expecting the patient to test the nurse’s competence and commitment. In the working phase, the nurse and the patient work together to solve problems and accomplish goals. This phase can involve such things as encouraging and helping the patient express feelings about his or her health, encouraging and helping the patient with self-exploration, or providing information needed to understand and change behaviour. The termination phase occurs during the ending of the relationship. This phase can involve such things as reminding the patient that the relationship is about to end, evaluating goal achievement with the patient, or reminiscing about the relationship with the patient. DIF: Understand REF: 277, Box 17-3 OBJ: List nursing focus areas within the four phases of a nurse–patient helping relationship. TOP: Assessment MSC: CPNRE: Professional, Ethical, and Legal Practice 11. During the initial home visit, a home health nurse lets the patient know that the visits are
expected to end in about a month. The nurse is in which phase of the helping relationship? a. Preinteraction. b. Orientation. c. Working. d. Termination.
NURSINGTB.COM
ANS: B
Letting the patient know when to expect the relationship to be terminated occurs in the orientation phase. Preinteraction occurs before the nurse meets the patient. Working occurs when the nurse and the patient work together to solve problems and accomplish goals. Termination occurs during the ending of the relationship. DIF: Apply REF: 277, Box 17-3 OBJ: List nursing focus areas within the four phases of a nurse–patient helping relationship. TOP: Assessment MSC: CPNRE: Professional, Ethical, and Legal Practice 12. A nurse and patient take action to meet health-related goals. The nurse is in which phase of
the helping relationship? a. Preinteraction. b. Orientation. c. Working. d. Termination. ANS: C
The working phase is when the nurse and the patient work together to solve problems and accomplish goals. Preinteraction occurs before the nurse meets the patient. Orientation occurs when the nurse and the patient meet and get to know each other. Termination occurs during the ending of the relationship. DIF: Remember
REF: 277, Box 17-3
Canadian Fundamentals of Nursing 6th Edition Potter Test Bank OBJ: List nursing focus areas within the four phases of a nurse–patient helping relationship. TOP: Implementation MSC: CPNRE: Professional, Ethical, and Legal Practice 13. The nurse is in the process of conducting an admission interview with the patient. At one
point in the discussion, the patient has provided information that the nurse would like to clarify. Using the technique of clarification, how does the nurse respond? a. “I’m not sure that I understand what you mean by that statement.” b. “The electrocardiogram records information about your heart’s electrical activity.” c. “Let’s look at the problem you have had with your medication at home.” d. “What’s your biggest concern at the moment?” ANS: A
In clarifying, the nurse checks whether understanding is accurate by restating an unclear message to clarify the sender’s meaning or by asking the other person to restate the message, explain further, or give an example of what the person means. The response “I’m not sure that I understand what you mean by that statement” indicates that the nurse wants to clarify what the patient is saying so that he or she can have an accurate understanding of what the patient means. The statement “The electrocardiogram records information about your heart’s electrical activity” is an example of providing information, not clarification. The statement “Let’s look at the problem you have had with your medication at home” is an example of focusing, not clarification. The question “What’s your biggest concern at the moment?” is an example of asking relevant questions, not of clarification. DIF: Apply REF: 284 OBJ: Discuss effective approaches to communicating with patients at various developmental levels. TOP: Planning MSC: CPNRE: Professional, Ethical, and Legal Practice 14. The patient draws back whenNtU heRnS urIseNrG eaTcB he. sC ovOeM r the side rails to take his blood pressure.
What should the nurse do first to promote effective communication? a. Tell the patient that the blood pressure can be taken at a later time. b. Rotate the nurses who are assigned to take the patient’s blood pressure. c. Continue to perform the procedure quickly and quietly. d. Apologize for startling the patient and explain the need for contact. ANS: D
Nurses often have to enter a patient’s personal space to provide care. The nurse should convey confidence, gentleness, and respect for privacy. Apologizing for startling the patient and explaining the need for contact demonstrates respect and provides information so the patient can understand the need for personal contact. Telling the patient that the blood pressure can be taken at a later time does not promote effective communication. Rotating the nurses who are assigned to take the patient’s blood pressure impedes the nurse’s ability to form a therapeutic, helping relationship. Continuing to perform the procedure quickly and quietly may send a negative nonverbal message to the patient. It also does not promote effective communication. DIF: Apply REF: 282-285 OBJ: Identify the practices important for relational inquiry. MSC: CPNRE: Professional, Ethical, and Legal Practice
TOP: Implementation
15. A patient has trouble speaking words, and the patient’s speech is garbled. Which nursing
diagnosis is most appropriate for this patient? a. Hopelessness.
Canadian Fundamentals of Nursing 6th Edition Potter Test Bank b. Impaired verbal communication. c. Hearing loss. d. Self-care deficit. ANS: B
A patient with impaired verbal communication has defining characteristics such as an inability to articulate words, inappropriate verbalization, difficulty forming words, and difficulty comprehending. Hopelessness implies that the patient has no hope for the future. Hearing loss is not a nursing diagnosis. Just because a patient has garbled speech does not mean that a hearing loss has occurred; a physical problem such as a stroke could cause the speech to be garbled. Self-care deficit does not apply in this situation because this usually relates to bathing and grooming. DIF: Analyze REF: 286 OBJ: Identify patient health states and conditions that contribute to impaired communication. TOP: Diagnosis MSC: CPNRE: Professional, Ethical, and Legal Practice 16. Which person is the best referral for a patient who speaks a foreign language? a. A family member. b. A speech therapist. c. An interpreter. d. A mental health nurse specialist. ANS: C
The services of interpreters are often necessary for patients who speak a foreign language. Use of a family member to interpret can lead to legal issues; speech therapists help patients with aphasia; and mental health nurse specialists help angry or highly anxious patients to communicate more effectivelN yU . RSINGTB.COM DIF: Understand REF: 275 OBJ: Discuss nursing care measures for patients with special communication needs. TOP: Planning MSC: CPNRE: Professional, Ethical, and Legal Practice 17. A nurse is using SOLER to facilitate active listening. Which technique should the nurse use
for R? a. Relax. b. Respect. c. Reminisce. d. Reassure. ANS: A
In SOLER, the R stands for “relax.” It is important to communicate a sense of being relaxed and comfortable with the patient. Active listening enhances trust because the nurse communicates acceptance and respect for the patient, but it is not the R in SOLER. Reminiscence is a therapeutic communication technique, especially when used with the elderly. Reassurance can be therapeutic if the nurse helps patients understand that there are many kinds of hope and that meaning and personal growth can come from illness experiences. However, false reassurance can block communication. DIF: Understand REF: 282 OBJ: Describe qualities, behaviours, and approaches that affect interprofessional communication TOP: Implementation MSC: CPNRE: Professional, Ethical, and Legal Practice
Canadian Fundamentals of Nursing 6th Edition Potter Test Bank
18. An older patient is wearing a hearing aid. Which technique should the nurse use to facilitate
communication? a. Speak clearly and loudly. b. Turn off the television. c. Chew gum. d. Use at least 14-point print. ANS: B
Patients who are hearing impaired benefit when nurses use the following techniques: checking for hearing aids and glasses, reducing environmental noise, getting the patient’s attention before speaking, not chewing gum, and speaking at normal volume (not shouting). Using at least 14-point print is for patients who are sight/visually impaired, not hearing impaired. DIF: Apply REF: 281, Box 17-6 OBJ: Discuss effective approaches to communicating with patients at various developmental levels. TOP: Implementation MSC: CPNRE: Professional, Ethical, and Legal Practice 19. When making rounds, the nurse finds a patient who is not able to sleep because of anticipation
of surgery in the morning. Which therapeutic response is most appropriate? a. “It will be okay. Your surgeon will talk to you in the morning.” b. “Why can’t you sleep? You have the best surgeon in the hospital.” c. “Don’t worry. The surgeon ordered a sleeping pill to help you sleep.” d. “It must be difficult not to know what the surgeon will find. What can I do to help?” ANS: D
“It must be difficult not to knN ow suB rg.eC onOw Rwha It NthGe T Mill find. What can I do to help?” is an U S example of therapeutic communication techniques of empathy and offering of self. False reassurances (“It will be okay” and “Don’t worry”) tend to block communication. Patients frequently interpret “why” questions as accusations or think the nurse knows the reason and is simply testing them. DIF: Apply REF: 282-285 OBJ: Describe qualities, behaviours, and approaches that affect interprofessional communication TOP: Implementation MSC: CPNRE: Professional, Ethical, and Legal Practice 20. Which situation will prompt the nurse to intervene and follow up on the unregulated care
provider’s behaviour? a. The unregulated care provider is calling the older patient “honey.” b. The unregulated care provider is facing the older patient when talking. c. The unregulated care provider cleans the older patient’s glasses. d. The unregulated care provider allows time for the older patient to respond. ANS: A
Health care providers should communicate with older persons on an adult level and avoid patronizing or speaking in a condescending manner. Terms of endearment such as “honey,” “dear,” “grandma,” or “sweetheart” should be avoided. Facing an older patient, making sure the older person has clean glasses, and allowing time to respond facilitate communication with older patients and should be encouraged, not stopped. DIF: Apply
REF: 279
Canadian Fundamentals of Nursing 6th Edition Potter Test Bank OBJ: Discuss effective approaches to communicating with patients at various developmental levels. TOP: Implementation MSC: CPNRE: Professional, Ethical, and Legal Practice 21. A confused older person is wearing thick glasses and a hearing aid. Which intervention is a
priority to facilitate communication? a. Focusing on tasks to be completed. b. Allowing time for the patient to respond. c. Limiting conversations with the patient. d. Using gestures and other nonverbal cues. ANS: B
Allowing time for the patient to respond will facilitate communication, especially for an older confused patient. Focusing on tasks to be completed and limiting conversations do not facilitate communication; in fact, they block communication. Using gestures and other nonverbal cues is not effective for visually impaired or cognitively impaired patients. DIF: Apply REF: 281, Box 17-6 OBJ: Discuss effective approaches to communicating with patients at various developmental levels. TOP: Implementation MSC: CPNRE: Professional, Ethical, and Legal Practice 22. The staff is having a hard time getting an older person to communicate. Which technique
should the nurse suggest the staff use? a. Allowing the patient to reminisce. b. Trying to change topics often. c. Involving only the patient in conversations. d. Asking the patient for explanations. ANS: A
NURSINGTB.COM
Encouraging older persons to share life stories and reminisce about the past has a therapeutic effect and increases their sense of well-being. Sudden shifts from subject to subject should be avoided. It is helpful to include the patient’s family and friends and to become familiar with the patient’s favourite topics for conversation. Asking for explanations is a nontherapeutic technique. DIF: Apply REF: 278-281 OBJ: Discuss effective approaches to communicating with patients at various developmental levels. TOP: Planning MSC: CPNRE: Professional, Ethical, and Legal Practice 23. Which of the following patients will cause the greatest communication concerns for a nurse? a. A patient who is alert, has strong self-esteem, and is hungry. b. A patient who is oriented, pain free, and blind. c. A patient who is cooperative, depressed, and hard of hearing. d. A patient who is dyspneic, has a tracheostomy, and is anxious. ANS: D
Facial trauma, laryngeal cancer, or endotracheal intubation often prevents movement of air past vocal cords or mobility of the tongue, which results in inability to articulate words. A patient extremely short of breath needs to use oxygen to breathe rather than speak. Persons with severe anxiety are sometimes unable to perceive environmental stimuli or hear explanations. People who are alert, have strong self-esteem, and are cooperative and pain free do not cause communication concerns. Hunger, blindness, and difficulty hearing individually can cause communication concerns, but do not necessarily affect communication.
Canadian Fundamentals of Nursing 6th Edition Potter Test Bank
DIF: Evaluate REF: 279 | 280 OBJ: Identify patient health states and conditions that contribute to impaired communication. TOP: Evaluate MSC: CPNRE: Professional, Ethical, and Legal Practice 24. A patient is aphasic, and the nurse notices that the patient’s hands shake intermittently. Which
nursing action is most appropriate to facilitate communication? a. Use a picture board. b. Use pen and paper. c. Use an interpreter. d. Use a hearing aid. ANS: A
Using a pen and paper can be frustrating for a patient who is nonverbal (aphasic) and whose handwriting is shaky; the nurse can revise the care plan to include use of a picture board instead. An interpreter is used for a patient who speaks a foreign language. A hearing aid is used for patients who are hard of hearing, not for a patient with aphasia. DIF: Apply REF: 286 OBJ: Discuss nursing care measures for patients with special communication needs. TOP: Implementation MSC: CPNRE: Professional, Ethical, and Legal Practice 25. What is characteristic of a nurse using critical thinking to enhance communication with
patients? a. Showing sympathy appropriately. b. Using automatic responses fluently. c. Self-examining personal communication skills. d. Demonstrating passive reN mU arR ksSaIcN cuG raT teB ly. . COM ANS: C
Nurses who use critical thinking skills interpret messages received from others, analyze their content, make inferences about their meaning, evaluate their effects, explain rationales for communication techniques used, and self-examine personal communication skills. Sympathy is concern, sorrow, or pity felt for the patient and is nontherapeutic. Clichés and stereotyped remarks are automatic phrases that communicate that the nurse is not taking concerns seriously or responding thoughtfully. Passive responses serve to avoid conflict or to sidestep issues. DIF: Understand REF: 272 OBJ: Describe aspects of critical thinking that are important to the communication process. TOP: Implementation MSC: CPNRE: Professional, Ethical, and Legal Practice 26. A patient says, “You are the worst nurse I have ever had.” Which response by the nurse is the
most assertive? a. “If I were you, I’d feel grateful for a nurse like me.” b. “I feel uncomfortable hearing that statement.” c. “How can you say that when I have been checking on you regularly?” d. “You shouldn’t say things like that, it is not right.” ANS: B
Canadian Fundamentals of Nursing 6th Edition Potter Test Bank Assertive responses contain “I” messages such as “I want,” “I need,” “I think,” or “I feel.” Giving personal opinions (“If I were you”), arguing (“How can you say that?”), and showing disapproval (using words like should, good, bad, right) are not assertive or therapeutic. DIF: Apply REF: 279 OBJ: Describe qualities, behaviours, and approaches that affect interprofessional communication TOP: Implementation MSC: CPNRE: Professional, Ethical, and Legal Practice 27. Which critical thinking standards should the nurse use to ensure sound effective
communication with patients? a. Faith, spiritual expression, and humility. b. Supportiveness, independent attitude, and self-confidence. c. Self-confidence, spiritual expression, and independent attitude. d. Self-confidence, humility, and independent attitude. ANS: D
A self-confident attitude is important because the nurse who conveys confidence and comfort while communicating more readily establishes an interpersonal helping-trusting relationship. In addition, an independent attitude encourages the nurse to communicate with colleagues and share ideas about nursing interventions. An attitude of humility is necessary to recognize and communicate the need for more information before making a decision. Faith, supportiveness, and spiritual expression are attributes of caring, not critical thinking standards. DIF: Understand REF: 272 OBJ: Describe aspects of critical thinking that are important to the communication process. TOP: Implementation MSC: CPNRE: Professional, Ethical, and Legal Practice
NURSINGTB.COM
Canadian Fundamentals of Nursing 6th Edition Potter Test Bank
Chapter 18: Patient-Centred Care: Interprofessional Collaborative Practice Potter et al: Canadian Fundamentals of Nursing, 6th Edition MULTIPLE CHOICE 1. A nurse is describing the antibiotic she is about to administer through a patient’s central line.
The patient accepts what the nurse is saying, and is able to repeat it back. However, the patient does not internalize this information or ask questions. This is an example of which of the five ways of knowing? a. Constructed knowing. b. Received knowing. c. Subjective knowing. d. Perceived knowing. ANS: B
Received knowing refers to when a patient defers to the nurse and accepts without debate what the nurse teaches him or her. Constructive knowing refers to when knowledge is received and the patient compares this knowledge against his or her assumptions, challenging his or her own assumptions and those of others. Subjective knowing refers to when the patient listens to what the nurse is explaining and internalizes the meaning. Perceived knowing refers to when the patient receives the knowledge and considers how it relates to what he or she already understands. DIF: Apply REF: 292 | 293 OBJ: Explore your own ways of learning and how these can be integrated to support your learning of how to be a professional nurse and provide care to patients. TOP: Assessment N R I G B.C M O MSC: CPNRE: Foundations of PU ractS ice N T 2. Which of the following patterns of knowing relates to how a nurse, as a unique individual,
chooses to respond in a situation involving a patient? a. Empirical knowing. b. Personal knowing. c. Aesthetic knowing. d. Ethical knowing. ANS: C
Aesthetic knowing relates to how a nurse, as a unique individual, chooses to respond in a situation involving a patient. Empirical knowing is related to the knowledge that is applied to such situations. Personal knowing reflects the nursing understanding accumulated from previous experiences. Ethical knowing is shaped by the nurse’s own values. DIF: Understand REF: 293 OBJ: Explore your own ways of learning and how these can be integrated to support your learning of how to be a professional nurse and provide care to patients. TOP: Assessment MSC: CPNRE: Foundations of Practice 3. A nurse is planning their care for a patient based on Orlando’s nursing process. The nursing
process occurs in which order? a. Perception, feeling, thought, and action. b. Feeling, thought, perception, and action.
Canadian Fundamentals of Nursing 6th Edition Potter Test Bank c. Thought, feeling, action, and perception. d. Perception, thought, feeling, and action. ANS: D
The nursing process occurs in the following order: perception, thought, feeling, and action. DIF: Understand REF: 295 | 296 OBJ: Gain an understanding of the frameworks that provide guidance to what nursing professional practice entails. TOP: Planning MSC: CPNRE: Foundations of Practice 4. A nurse determines that a patient is in pain and decides to give an analgesic medication the
patient has been ordered. The nurse then goes back to assess the patient’s pain after 30 minutes. The act of administering the analgesic medication is best described as which stage of the nursing process? a. Assessment. b. Planning. c. Intervention. d. Evaluation. ANS: C
The act of administering the analgesic medication is best described as the intervention stage. The nurse’s determining that the patient is in pain would be described as assessment; the nurse’s decision to give the analgesic reflects planning; and evaluation refers to the nurse’s returning to reassess the patient’s pain. DIF: Apply REF: 296 OBJ: Gain an understanding of the frameworks that provide guidance to what nursing professional practice entails. TOP: PlN anniR ng I GMSB C:.C CPNM RE: Foundations of Practice
U S N T
O
5. What would be the best question to ask a patient in the noticing phase of the Clinical
Judgement Model? a. “Where is the pain the worst?” b. “How are you feeling today?” c. “How was your night?” d. “What are your goals for today?” ANS: D
The most critical information to obtain from a patient is what his or her goals are. This information will help the nurse focus further exploration of the patient’s goals and plan around the patient’s goals. The other questions will elicit important information but are not the considered the most critical questions in this phase. DIF: Apply REF: 296 OBJ: Gain an understanding of the frameworks that provide guidance to what nursing professional practice entails. TOP: Assessment MSC: CPNRE: Foundations of Practice 6. A nurse gathers information through the noticing phase of the Clinical Judgement Model. The
nurse is interpreting this information by using experience, expertise, and knowledge, along with personality and environment. This is best described as which of the following? a. Intuitive interpreting. b. Narrative interpreting. c. Experiential interpreting.
Canadian Fundamentals of Nursing 6th Edition Potter Test Bank d. Analytic interpreting. ANS: A
Intuitive interpreting is described when a nurse applies his or her experience, expertise, and knowledge, along with personality and environment, to understand information. Narrative interpreting involves trying to understand the particular care and is viewed as human beings’ primary way of making sense of experience, through an interpretation of human concerns, intents, and motive. Analytic interpreting involves the integration of information gathered through the noticing phase along with the patient’s goals. Experiential interpreting is not listed as a type of interpretation in this model. DIF: Understand REF: 297 OBJ: Gain an understanding of the frameworks that provide guidance to what nursing professional practice entails. TOP: Planning MSC: CPNRE: Foundations of Practice 7. Who is the expert in the patient’s management of illness? a. The nurse. b. The physician. c. The patient. d. The interprofessional team. ANS: C
Patients are the experts in their management of illness. The nurse, physician, and interprofessional team are all important partners in care. DIF: Understand REF: 298 OBJ: Gain an understanding of the frameworks that provide guidance to what nursing professional practice entails. TOP: AN ssessRmenIt GMSB C:.C CPNM RE: Foundations of Practice
U S N T
O
8. What is the cause of internalized myths that nurses may have about their own profession and
those of other health care providers? a. Role ambiguity. b. Media portrayal. c. Familial experiences. d. Socialization. ANS: D
Socialization is the most encompassing cause of internalized myths nurses may have of their own profession and those of other health care providers. Role ambiguity may contribute to these myths but is not a cause. Media portrayal and familial experiences are parts of socialization, contributing to the myths. DIF: Understand REF: 298 | 299 OBJ: Explore how to integrate your growing nursing professional identity with that of an interprofessional identity. TOP: Evaluate MSC: CPNRE: Professional, Ethical, and Legal Practice 9. To clarify the role of a nurse in patient-centred care, a nurse is responsible for which of the
following? a. Asking other interprofessional team members about their roles. b. Shadowing other interprofessional team members to experience their roles. c. Being able to explain own role and the knowledge and skills that the nurse brings
Canadian Fundamentals of Nursing 6th Edition Potter Test Bank to patient-centred care. d. Being able to explain the roles of all interprofessional team members. ANS: C
Role clarification requires a nurse to be able to explain what a professional nurse’s role is and what knowledge and skills the nurse brings to patient-centred care. Asking other interprofessional team members about their roles, shadowing other interprofessional team members, and being able to explain the roles of all interprofessional team members contribute to role clarification but are not the nurse’s main responsibility when clarifying his or her own role. DIF: Understand REF: 299 OBJ: Explore how as a beginning nurse professional you can work within interprofessional teams. TOP: Implementation MSC: CPNRE: Collaborative Practice 10. A nurse is caring for six patients on a postsurgical unit. She hears a code blue being called on
one of her patients, and she runs to the patient’s room. When the nurse arrives, a code team leader has been established. The first thing the nurse should do is which of the following? a. Introduce herself as the patient’s nurse, offering her specific knowledge and skills. b. Immediately jump in and assist with the airway. c. Stand back, observe the situation, and wait to be given a role by the code leader. d. Acknowledge that the situation is under control and go back to her other five patients. ANS: A
The nurse should introduce herself, provide relevant information, and assist in role clarification by explaining the skills she can contribute, such as initiation of intravenous (IV) treatment or medication admN inis trati on.NG ToTfBu. ncC tion UR SI OMbest as a group, the team members need to be aware of each other’s roles. If the nurse were to jump in and assist with the airway without introducing herself, the team may not know who she is or what her role is. The nurse is not being an active team member by standing back and waiting for a role or by leaving the room during the code. It is her responsibility as the patient’s primary nurse to provide the information and skills she has to offer to this situation. DIF: Apply REF: 300 OBJ: Explore how as a beginning nurse professional you can work within interprofessional teams. TOP: Implementation MSC: CPNRE: Professional, Ethical, and Legal Practice 11. At morning rounds, a nurse explains the change in status her patient had overnight. The
interprofessional team members all recognize that this nurse has been working in the unit for more than 10 years and is extremely competent at assessment and interpretation. This recognition influences how the team is interpreting the nurse’s report. Which aspect is being described? a. Affinity. b. Immediacy. c. Respect. d. Control. ANS: A
Canadian Fundamentals of Nursing 6th Edition Potter Test Bank Affinity describes how the nurse is viewed on the basis of perceived competence, which influences how the team interprets the nurse’s report. Immediacy refers to the urgency of the message the nurse is relaying and was not alluded to in the question. Respect refers to trust gained over time through experience interacting with the team. Control refers to power differentials and was not alluded to in the question. DIF: Apply REF: 301 OBJ: Explore how as a beginning nurse professional you can work within interprofessional teams. TOP: Implementation MSC: CPNRE: Collaborative Practice 12. When communicating with other members of the interprofessional team, the nurse should
explain which of the following? a. Situation, background, assessment, and reassessment. b. Situation, background, assessment, and recommendations. c. Subjective information, behaviour, assessment, and recommendations. d. Socialization, behaviour, action, and reaction. ANS: B
The acronym SBAR stands for situation, background, assessment, and recommendations, all of which should be explained by the nurse when interacting with the interprofessional team. DIF: Understand REF: 301 OBJ: Explore how as a beginning nurse professional you can work within interprofessional teams. TOP: Implementation MSC: CPNRE: Collaborative Practice 13. A patient undergoing chemotherapy visits with his interprofessional team once a month. He
observes the oncologist encouraging other team members, such as the nurse, the pharmacist, or the social-worker, to take N onUtR heSlI eaN dG roT leBd. urCinOgMthe meetings, on the basis of the patient’s concerns and goals. Which leadership element is being demonstrated by the oncologist? a. Inspiring a shared vision. b. Enabling others to act. c. Challenging the process. d. Encouraging the heart. ANS: B
The oncologist is demonstrating the leadership element of enabling others to act by encouraging other members to take on the leadership role and support patients in their decision making with the team. Inspiring a shared vision would be demonstrated by having the members focus on patient-specific goals and helping bring the ideas together in an agreed-upon plan with the patient. Challenging the process would be demonstrated by reflecting on how the team is working together and making needed changes. Encouraging the heart would be demonstrated by recognizing the positive work of all team members, including the patient, toward meeting patient-set goals. DIF: Apply REF: 303, Box 18-4 OBJ: Explore how as a beginning nurse professional you can work within interprofessional teams. TOP: Evaluate MSC: CPNRE: Collaborative Practice 14. The Canadian Interprofessional Health Collaborative (CIHC) has four key competencies for
interprofessional collaboration. They are role clarification, team functioning, collaborative leadership, and which of the following?
Canadian Fundamentals of Nursing 6th Edition Potter Test Bank a. b. c. d.
Patient-centred care. Quality improvement. Dealing with interprofessional conflict. Reflection.
ANS: C
The four key competencies for the CIHC National Interprofessional Competency Framework are role clarification, team functioning, collaborative leadership, and dealing with interprofessional conflict. Patient-centered care, quality improvement, and reflection are not key competencies in this framework. DIF: Understand REF: 300, Fig. 18-4 OBJ: Explore how as a beginning nurse professional you can work within interprofessional teams. TOP: Assessment MSC: CPNRE: Collaborative Practice 15. Which way of knowing is primarily affected by the ideals that are significant or important to
the person, the family, or the community? a. Empirical knowing. b. Aesthetic knowing. c. Personal knowing. d. Ethical knowing. ANS: D
Ethical knowing is shaped by one’s values, which are the ideals that have significant meaning or importance to the person, the family, or the community. Aesthetic knowing concerns how the person, as a unique individual, chooses to respond in a situation. Empirical knowing is related to the knowledge gained that can be applied to specific situations. Personal knowing reflects the understanding acN cuU mR ulS atI edNfG roTmBp.reCvO ioM us experiences. DIF: Understand REF: 293 OBJ: Explore your own ways of learning and how these can be integrated to support your learning of how to be a professional nurse and provide care to patients. TOP: Assessment MSC: CPNRE: Professional, Ethical, and Legal Practice 16. A nurse calls the physician and says, “Dr. Green, the patient LT is complaining of increased
pain to his left leg, rating it 8/10. LT is the 8-year-old boy in room 12 who broke his femur in a snowboarding accident yesterday. His pain has been well controlled at 2 to 4/10 for the past day, but his analgesic dose was weaned after rounds this morning. His foot does not appear to be swollen below the cast, and his perfusion is okay with palpable pedal pulses and a capillary refill of 2 to 3 seconds to the toes on the left foot.” Which elements of SBAR communication did the nurse include? a. Situation, background, and assessment. b. Subjective information, assessment, and recommendation. c. Background, assessment, and reassessment. d. Assessment, recommendation, and reassessment. ANS: A
Canadian Fundamentals of Nursing 6th Edition Potter Test Bank The communication elements of SBAR are situation, background, assessment, and recommendation. The nurse explained the situation by saying, “The patient LT is complaining of increased pain to his left leg.” The background was described by “LT is the 8-year-old boy in room 12 who broke his femur in a snowboarding accident yesterday. His pain has been well controlled at 2 to 4/10 for the past day, but his analgesic dose was weaned after rounds this morning.” Assessment was described by “His foot does not appear to be swollen below the cast, and his perfusion is okay with palpable pedal pulses and a capillary refill of 2 to 3 seconds to the toes on the left foot.” The nurse did not include a recommendation. Subjective information and reassessment are not elements of SBAR. DIF: Analyze REF: 301 OBJ: Explore how as a beginning nurse professional you can work within interprofessional teams. TOP: Assessment MSC: CPNRE: Foundations of Practice
NURSINGTB.COM
Canadian Fundamentals of Nursing 6th Edition Potter Test Bank
Chapter 19: Family Nursing Potter et al: Canadian Fundamentals of Nursing, 6th Edition MULTIPLE CHOICE 1. What is an example of circular communication patterns between father and child? a. The father and child are engaged in a game of catch. b. The father uses rewards to encourage the child to go to bed. c. The father comforts the crying child. d. The child disobeys his father. ANS: C
Circular communication refers to communication between family members in which each person influences the behaviour of the other. An example is when a parent comforts a child because the child cries; because the parent responds to the child, the child feels safe and secure. The example of the father and child being engaged in a game of catch is not an example of circular communication. It reflects a subsystem (structural assessment) of father and child; it reflects a relationship within the family. The example of the father using rewards to encourage the child to go to bed is not an example of circular communication. It illustrates how influence (expressive functioning) is used to affect or control another person’s behaviour. Influence can be classified as instrumental (e.g., using privileges to reward good behaviour), psychological (e.g., giving praise or admonishment), or corporal (e.g., hugging or hitting). The example of the child disobeying his father is not an example of circular communication. Rather, it is an example of a boundary. DIF: Analyze TOP: Assessment
REF: 31N 7 R I GOBB J:.DefC iM ne the key terms listed. MSC: CPNURES : ProN fessT ional, EO thical, and Legal Practice
2. Balancing employment and family life creates a variety of challenges in terms of child care
and household work for both parents. Why does this have major implications in health care? a. Maternal employment has been demonstrated to be harmful for children. b. Maternal employment has shifted the majority of household tasks to the male. c. Fathers now participate more fully in day-to-day parenting responsibilities. d. The number of single-parent families has decreased since 1970. ANS: C
There is no proof that maternal employment is damaging for children. However, finding quality child care is a major issue. Managing household tasks is another challenge. Although equal division of labour receives verbal approval, most household tasks remain “women’s work.” Evidence suggests that the fathering role is changing. Fathers now participate more fully in day-to-day parenting responsibilities. The number of single-parent families, which doubled from the 1940s to the 1990s, seems to be stabilizing. DIF: Understand REF: 309 OBJ: Discuss how family members influence one another’s health. TOP: Diagnosis MSC: CPNRE: Foundations of Practice
Canadian Fundamentals of Nursing 6th Edition Potter Test Bank 3. A husband brings his children in to visit their mother in the hospital. The nurse asks how the
family is getting along at home without their mom around. The husband states, “None of her jobs are getting done, and I don’t do those jobs, so the house and the kids are falling apart.” The nurse suspects that this family structure is which of the following? a. Very flexible. b. Quite rigid. c. Extremely open. d. Hardy. ANS: B
A rigid structure specifically dictates who is able to accomplish a task and may limit the number of people outside the immediate family who assume these tasks. A crisis-proof, or effective, family can integrate the need for stability with the need for growth and change; has a flexible structure for task performances; and is receptive to help from outside the family system while maintaining a sense of strength and stability. An effective family exerts influence on the immediate environment of home, neighbourhood, and school, whereas a crisis-prone family may lack, or believe it lacks, control over these environments. An extremely open or very flexible structure also presents problems for the family in that consistent patterns of behaviour do not exist, and enactment of roles is overly flexible. Hardiness is the internal strength and durability of the family unit characterized by a sense of control over the outcome of life and an active rather than passive orientation in adapting to stressful events. DIF: Apply REF: 314 OBJ: Discuss how family members influence one another’s health. TOP: Implementation MSC: CPNRE: Foundations of Practice
N R I G B.C M
4. It is essential for family membU ers tS o reN alizT e that aOfamily’s beliefs, values, and practices
strongly influence the health-promoting behaviours of its members, and to understand which of the following? a. Canadian families are part of the same culture with the same values and beliefs. b. Economic status has little effect on a family’s ability to access adequate health care. c. Family environment in early life has a strong influence on later health practices. d. All families place a high value on good health and health practices. ANS: C
Family environment is crucial because health behaviour reinforced early in life has a strong influence on later health practices. Some families do not place a high value on good health. In fact, some families accept harmful practices. Although Canadian families exist within the same culture, they live in very different ways as a result of race, values, social class, and ethnicity. Economic stability increases a family’s access to adequate health care. DIF: Understand REF: 310 OBJ: Discuss how family members influence one another’s health. TOP: Assessment MSC: CPNRE: Foundations of Practice 5. In an assessment for signs of a healthy family, what should the nurse expect to find? a. Change is viewed as detrimental to family processes. b. The response to stressors is passive. c. The structure is flexible enough to adapt to crises.
Canadian Fundamentals of Nursing 6th Edition Potter Test Bank d. Minimal influence is exerted on the environment. ANS: C
A healthy family has a flexible structure that allows adaptable performance of tasks and acceptance of help from outside the family system. The structure is flexible enough to allow adaptability but not so flexible that the family lacks cohesiveness and a sense of stability. The healthy family is able to integrate the need for stability with the need for growth and change. It does not view change as detrimental to family processes. The healthy family demonstrates control over the environment and does not passively respond to stressors. The healthy family exerts influence on the immediate environment of home, neighbourhood, and school. DIF: Evaluate REF: 310 OBJ: Discuss how family members influence one another’s health. TOP: Evaluate MSC: CPNRE: Foundations of Practice 6. An 86-year-old patient with diabetes who requires daily insulin injections lives with his
daughter and her husband. To assess instrumental functioning, what would the nurse ask? a. “Mr. Casey, how do you think your daughter feels about your refusing to take your insulin?” b. “What do you feel has aggravated your difficulty in controlling the diabetes?” c. “Mr. Casey, who is responsible for administering your insulin injections?” d. “How will you deal with hypoglycemic reactions?” ANS: C
Instrumental functioning includes all normal activities of daily living, such as preparing meals, eating, sleeping, and attending to health needs. For families with health problems, these activities may include administration of medications. The question “How do you think your daughter feels about youNrUreRfus ing toTtaBk. eC you r insulin?” explores expressive SI NG OM functioning, particularly emotional communication. The question “What do you feel has aggravated your difficulty in controlling the diabetes?” explores the patient’s beliefs within the category of expressive functioning. The question “How will you deal with hypoglycemic reactions?” evaluates the family’s ability to solve problems; problem solving is in the category of expressive functioning. DIF: Apply REF: 316 OBJ: Ask assessment questions to learn relevant information about family functioning in the context of health or illness. TOP: Implementation MSC: CPNRE: Foundations of Practice 7. When the nurse is initiating the care of families, what is one factor that helps organize the
family approach to the nursing process? a. The view of all patients as unique individuals. b. The realization that families have little effect on individuals. c. The realization that individuals have little effect on families. d. The realization that individuals have an effect on families. ANS: D
A change in one family member, such as an illness or health condition, affects the other family members. Family nursing promotes, supports, and provides for the well-being and health of the family and individual family members. DIF: Understand
REF: 311 | 312
Canadian Fundamentals of Nursing 6th Edition Potter Test Bank OBJ: Compare family as context, family as patient and family in context, explaining how these different perspectives influence nursing practice with families. TOP: Assessment MSC: CPNRE: Foundations of Practice 8. The nurse is interviewing a patient who is being admitted to the hospital. The patient’s family
went home before the nurse’s interview. The nurse asks the patient, “Who decides where to go on vacation?” In asking this, what is the nurse trying to do? a. Assess the family structure. b. Assess the family form. c. Assess the family function. d. Make a categorical generalization. ANS: A
To assess the family structure, the nurse asks questions that determine the power structure and patterning of roles and tasks (e.g., “Who decides where to go on vacation?”). When focusing on family form, the nurse should begin the family assessment by determining the patient’s definition of family. Family function is the ability of the family to provide emotional support and to cope with health problems or situations. The question asked above will not help assess that. Categorical generalizations are misleading and should be avoided. DIF: Apply REF: 313-315 OBJ: Ask assessment questions to learn relevant information about family functioning in the context of health or illness. TOP: Implementation MSC: CPNRE: Foundations of Practice 9. When focusing on older persons, the nurse must be aware of which of the following? a. Elder abuse happens only in lower socioeconomic classes. b. Older persons have the saN mUeR soScI iaN l nGeT twBo. rkC sO asMdo younger people. c. Caregivers may be spouses or middle-aged children. d. Caregiver stress is minimal when caring for a parent. ANS: C
Caregivers are typically spouses, who may be older persons with declining physical stamina, or middle-aged children, who often have other responsibilities. Abuse of older persons in families occurs across all social classes. Later-life families have a different social network than younger families because many friends and same-generation family members have died or have been ill themselves. The nurse should assess for caregiver stress such as tension in relationships between family and care recipient, changes in level of health, changes in mood, and anxiety and depression. DIF: Remember REF: 310 OBJ: Compare family as context, family as patient and family in context, explaining how these different perspectives influence nursing practice with families. TOP: Assessment MSC: CPNRE: Foundations of Practice 10. When the nurse views the family as context, the primary focus is on the health and
development of an individual member existing within a specific environment (i.e., the patient’s family). Although the focus is on the individual’s health status, the nurse should do which of the following? a. Assess how much the family provides the patient’s basic needs. b. Assess family patterns versus individual characteristics.
Canadian Fundamentals of Nursing 6th Edition Potter Test Bank c. Maintain distinctions between “family as patient” and “family as context.” d. Plan care to meet not only the patient’s needs but also those of the patient’s family. ANS: A
When the nurse views the family as context, the primary focus is on the health and development of an individual member existing within a specific environment (i.e., the patient’s family). Although the focus is on the individual’s health status, the nurse assesses how much the family provides the individual’s basic needs. Family patterns are in the realm of “family as patient.” It is important to understand that although the nurse is able to make theoretical and practical distinctions between “family as context” and “family as patient,” they are not necessarily mutually exclusive. Nurses often use the two simultaneously, as with the perspective of “family as system.” “Family as patient” involves planning to meet the needs of the patient and those of the patient’s family as well. DIF: Understand REF: 311 OBJ: Compare family as context, family as patient and family in context, explaining how these different perspectives influence nursing practice with families. TOP: Assessment MSC: CPNRE: Foundations of Practice 11. Mrs. Lily is talking about her recent chronic diagnosis and how it has affected her
emotionally, intellectually, socially, and spiritually. The nurse knows that this is referred to as which of the following? a. Reflection. b. Illness narrative. c. Comprehensive honesty. d. Complete life review. ANS: B
NURSINGTB.COM
An illness narrative is a patient’s story of how illness affects his or her whole being, including emotional, intellectual, social, and spiritual components. Reflection, comprehensive honesty, and a complete life review do not refer to these components. DIF: Understand REF: 320 TOP: Implementation
OBJ: Discuss family nursing as relational inquiry. MSC: CPNRE: Foundations of Practice
12. The nurse is caring for an older patient who apparently has no family. When questioned about
his family and his definition of family, the patient states, “I have no family. They’re all gone.” When asked, “Who prepares your meals?” he states, “I do, or I go out.” Given the three different approaches to family nursing practice, which would be most appropriate for this patient? a. Family as context. b. Family as patient. c. Family as system. d. Combination of “family as context” and “family as patient.” ANS: A
If only one family member receives nursing care, it is realistic and practical to view the “family as context.” Although family nursing is based on the assumption that all people regardless of age are a member of some type of family form, the patient insists that he has no family. The nurse should investigate further. However, at this time, family as patient or as system, or a combination of these, is not appropriate.
Canadian Fundamentals of Nursing 6th Edition Potter Test Bank
DIF: Analyze REF: 311 OBJ: Compare family as context, family as patient and family in context, explaining how these different perspectives influence nursing practice with families. TOP: Evaluate MSC: CPNRE: Foundations of Practice 13. The nurse is caring for an older woman and notices that she is not using her cane properly.
Which of the following statements by the nurse would most likely elicit a positive response from the patient? a. “You’re doing that all wrong. Let me show you how to do it.” b. “I don’t know who showed you how to use the cane like that, but you’re not doing it right. Let me show you again.” c. “You use the cane the way I did before I was shown a way to keep from tripping over it; do you mind if I show you?” d. “I used to use the cane the same way you are using it: the wrong way. I’ll show you the right way to do it.” ANS: C
When the nurse is respectful instead of coming across as an authority on the subject, the patient’s defences will be down, making the patient more willing to listen without feeling embarrassed. DIF: Understand REF: 319 OBJ: Discuss the relational approaches and practices needed to conduct a family interview and effectively intervene with the family in nursing practice. TOP: Implementation MSC: CPNRE: Foundations of Practice 14. The nurse is providing dischaNrgeRteaI chinG g foBr . anColdMer woman who will need dressing changes
U S N T
O
at home. Her husband, who is also an older person, is her only source of care. The husband states that he will not be able to perform the dressing changes. What does the nurse need to arrange for? a. Home care service referrals. b. Extra dressing supplies. c. Cancellation of the discharge. d. An order for antibiotics. ANS: A
Discharge planning with a family involves an accurate assessment of what will be needed for care at the time of discharge, along with any shortcomings in the home setting. If no one can change the dressings properly, the nurse must arrange for a home care service referral. Extra dressing supplies will not help the situation if the husband is afraid to use them. Only under extreme situations or in an unsafe situation will the discharge be cancelled. An order for antibiotics is not a replacement for good dressing change technique. DIF: Apply REF: 311 | 316 OBJ: Discuss the relational approaches and practices needed to conduct a family interview and effectively intervene with the family in nursing practice. TOP: Implementation MSC: CPNRE: Foundations of Practice 15. Although the family as a whole differs from individual members, the measure of family health
is more than a summary of the health of all members. Of the following, what areas are unique to family assessment in the Calgary Family Assessment Model (CFAM)?
Canadian Fundamentals of Nursing 6th Edition Potter Test Bank a. b. c. d.
Development, family health, and functionality. Development, structure, and functionality. Individual health, family health, and functionality. Development, individual health, and family health.
ANS: B
Although the family as a whole differs from individual members, the measure of family health is more than a summary of the health of all members. With the CFAM, family assessment is completed in the developmental, structural, and functional areas. Although individual health is important, it varies from the family focus. DIF: Remember REF: 313 OBJ: State the three major categories of the Calgary Family Assessment Model (CFAM) and understand subcategories important to consider in a family assessment. TOP: Assessment MSC: CPNRE: Foundations of Practice
NURSINGTB.COM
Canadian Fundamentals of Nursing 6th Edition Potter Test Bank
Chapter 20: Patient Education Potter et al: Canadian Fundamentals of Nursing, 6th Edition MULTIPLE CHOICE 1. A nurse is asked about the goal of patient education. What is the nurse’s best response? a. “The goal of educating others is to help people meet standards of the Nurse
Practice Act.” b. “The goal of educating others is to help people achieve optimal levels of health.” c. “The goal of educating others is to help people become dependent on the health care team.” d. “The goal of educating others is to help people provide self-care only while they are in the hospital.” ANS: B
The goal of educating people about their health is to help individuals, families, and communities achieve optimal levels of health. Although all provincial and territorial Nurse Practice Acts acknowledge that patient teaching falls within the scope of nursing practice, this is the nurse’s standard, not the goal of education. Patient education helps patients make informed decisions about their care and become healthier and more independent, not dependent. Nurses provide patients with information needed for self-care to ensure continuity of care from the hospital to the home, not only in the hospital. DIF: Remember REF: 325 TOP: Implementation
OBJ: Describe the purposes of patient education. MSC: CPNRE: Foundations of Practice
N R I G B.C M
2. A nurse is teaching a group of U healS thyN aduT lts abouOt the benefits of flu immunizations. Which
purpose of patient education is the nurse fulfilling? a. Restoration of health. b. Coping with impaired functions. c. Promotion of health and illness prevention. d. Health analogies. ANS: C
A nurse is a visible, competent resource for patients who want to improve their physical and psychological well-being. In the school, home, clinic, or workplace, nurses promote health and prevent illness by providing information and skills that enable patients to practise healthier behaviours. Injured and ill patients need information and skills to help them regain or maintain their level of health; this is referred to as restoration of health. Not all patients recover fully from illness or injury. Many have to learn to cope with permanent health alterations; this is known as coping with impaired functions. Analogies supplement verbal instruction with familiar images that make complex information more real and understandable; for example, when explaining arterial blood pressure, the nurse can use an analogy of the flow of water through a hose. DIF: Understand TOP: Planning
REF: 325 OBJ: Describe the purposes of patient education. MSC: CPNRE: Foundations of Practice
3. A nurse provides teaching about coping with long-term impairment of functions. Which
situation serves as the best example?
Canadian Fundamentals of Nursing 6th Edition Potter Test Bank a. Teaching a family member to give medications through the patient’s permanent
gastric tube. b. Teaching a woman who recently had a hysterectomy about her pathology reports. c. Teaching expectant parents about physical and psychological changes in
childbearing women. d. Teaching a teenager with a broken leg how to use crutches. ANS: A
Not all patients recover fully from illness or injury. Many have to learn to cope with permanent health alterations. New knowledge and skills are often necessary for patients or family members, or both, to continue activities of daily living. Teaching family members to help the patient with health care management (e.g., giving medications through gastric tubes, doing passive range-of-motion exercises) is an example of teaching them how to cope with long-term impairment of functions. Injured and ill patients need information and skills to help them regain or maintain their levels of health. Some examples of this include teaching a woman who recently had a hysterectomy about her pathology reports and expected length of recovery and teaching a teenager with a broken leg how to use crutches. In childbearing classes, expectant parents are taught about physical and psychological changes in the woman and about fetal development; this is part of health maintenance. DIF: Understand REF: 326 OBJ: Identify appropriate topics that address a patient’s health education needs. TOP: Evaluate MSC: CPNRE: Foundations of Practice 4. Which statement indicates that the nurse has a good understanding of teaching/learning? a. “Teaching and learning can be separated.” b. “Learning is an interactive process that promotes teaching.” N scio RSus, INdeli GTberate B.CsOetMof actions designed to help the c. “Learning consists of a conU
teacher.”
d. “Teaching is most effective when it responds to the learner’s needs.” ANS: D
Teaching is most effective when it responds to the learner’s needs. It is impossible to separate teaching from learning. Teaching is an interactive process that promotes learning. Teaching consists of a conscious, deliberate set of actions that help individuals gain new knowledge, change attitudes, adopt new behaviours, or perform new skills. DIF: Understand TOP: Evaluate
REF: 326 OBJ: Describe the purposes of patient education. MSC: CPNRE: Foundations of Practice
5. Which action best indicates that learning has occurred? a. A nurse presents information about diabetes. b. A patient demonstrates how to inject insulin. c. A family member listens to a lecture on diabetes. d. A primary care provider hands a diabetes pamphlet to the patient. ANS: B
Canadian Fundamentals of Nursing 6th Edition Potter Test Bank Learning is the purposeful acquisition of new knowledge, attitudes, behaviours, and skills. Complex patterns are involved in learning new skills, changing existing attitudes, transferring learning to new situations, and solving problems. A new mother exhibits learning when she demonstrates how to bathe her newborn. A nurse presenting information and a primary care provider handing a pamphlet to a patient are examples of teaching. A family member listening to a lecture does not indicate that learning occurred; a change in knowledge, attitudes, behaviours, or skills must be evident. DIF: Understand TOP: Evaluate
REF: 327 OBJ: Use appropriate methods to evaluate learning. MSC: CPNRE: Foundations of Practice
6. The nurse is demonstrating to the patient how to put on antiembolism stockings. In the middle
of the lesson, the patient asks, “Why have my feet been swelling?” The nurse stops and responds to the patient, adhering to which teaching principle? a. Timing. b. Setting priorities. c. Building on existing knowledge. d. Organizing teaching materials. ANS: A
The nurse who stops a demonstration of applying antiembolism stockings to answer a patient’s question is following the teaching principle of timing. If the patient has a question, it is important to answer the question right away, so that the focus can return to the task being taught. Setting priorities is important for conserving the time and energy of the patient and nurse. A patient learns best on the basis of preexisting cognitive abilities and knowledge. This situation is not an example of building on existing knowledge. Organizing teaching material means that the nurse consideN rs thR e orIderGin w Bh.icCh toMpresent the information.
U S N T
DIF: Apply REF: 333 | 334 TOP: Implementation
O
OBJ: Explain the role of the nurse in patient education. MSC: CPNRE: Foundations of Practice
7. A nurse teaches a patient with heart failure about healthy food choices. The patient states that
eating yogurt is better than eating cake. In this situation, which element represents feedback? a. The nurse. b. The patient. c. The nurse’s teaching about healthy food choices. d. The patient’s statement that eating yogurt is better than eating cake. ANS: D
Feedback should show the success of the learner in achieving objectives (i.e., the learner verbalizes information or provides a return demonstration of skills learned). The nurse is the sender. The patient is the receiver. The teaching is the message. DIF: Apply REF: 335 OBJ: Determine appropriate communication principles when patient education is provided. TOP: Evaluate MSC: CPNRE: Foundations of Practice 8. While preparing a teaching plan, the nurse describes what the learner will be able to
accomplish after the teaching session. Which action is the nurse completing? a. Developing learning objectives. b. Providing positive reinforcement.
Canadian Fundamentals of Nursing 6th Edition Potter Test Bank c. Implementing interpersonal communication. d. Presenting facts and knowledge. ANS: A
Learning objectives describe what the learner will be able to do after successful instruction. Positive reinforcement follows feedback and involves the use of praise and acknowledgement of new attitudes, behaviours, or knowledge. Interpersonal communication in general is necessary for the teaching/learning process and should be used in teaching. Facts and knowledge will be presented in the teaching session. DIF: Understand TOP: Planning
REF: 333 OBJ: Write learning objectives for a teaching plan. MSC: CPNRE: Foundations of Practice
9. A student nurse learns that a normal adult heartbeat is 60 to 100 beats/minute. In which
domain did learning take place? a. Kinesthetic. b. Cognitive. c. Affective. d. Psychomotor. ANS: B
Cognitive learning includes all intellectual behaviours and requires thinking. In the hierarchy of cognitive behaviours, the simplest behaviour is acquiring knowledge. The student nurse acquired knowledge, which is cognitive. A kinesthetic learner is a type of learner who learns best with a hands-on approach. Affective learning deals with expression of feelings and acceptance of attitudes, opinions, or values. Psychomotor learning involves acquiring skills that require integration of mental and muscular activities, such as the ability to walk or to use an eating utensil. NURSINGTB.COM DIF: Apply TOP: Assessment
REF: 327 | 328 OBJ: Describe the domains of learning. MSC: CPNRE: Foundations of Practice
10. A nurse wants the patient to begin to accept the chronic nature of diabetes. Which teaching
technique should the nurse use to enhance learning? a. Lecture. b. Demonstration. c. Role play. d. Question and answer session. ANS: C
Affective learning deals with expression of feelings and acceptance of attitudes, opinions, or values. Role play and discussion (one-on-one and group) are effective teaching methods for the affective domain. Lecture and question-and-answer sessions are effective teaching methods for the cognitive domain. Demonstration is an effective teaching method for the psychomotor domain. DIF: Analyze REF: 328, Box 20-4 OBJ: Describe the domains of learning. TOP: Implementation MSC: CPNRE: Foundations of Practice 11. When the nurse describes a patient’s perceived ability to successfully complete a task, which
term should the nurse use?
Canadian Fundamentals of Nursing 6th Edition Potter Test Bank a. b. c. d.
Self-efficacy. Motivation. Attentional set. Active participation.
ANS: A
Self-efficacy, a concept included in social learning theory, refers to a person’s perceived ability to successfully complete a task. Motivation is a force that acts on or within a person (e.g., an idea, an emotion, a physical need) to cause the person to behave in a particular way. An attentional set is the mental state that allows the learner to focus on and comprehend a learning activity. Learning occurs when the patient is actively involved in the educational session. DIF: Remember TOP: Diagnosis
REF: 330 | 331 OBJ: Identify basic learning principles. MSC: CPNRE: Foundations of Practice
12. A toddler is going to have surgery on the right ear. Which teaching method is most
appropriate for this developmental stage? a. Encourage independent learning. b. Use discussion throughout the teaching session. c. Apply a bandage to a doll’s ear. d. Develop a problem-solving scenario. ANS: C
Nurses should use play to teach a procedure or activity (e.g., handling examination equipment, applying a bandage to a doll) to toddlers. Independent learning is for adults. Use of discussion is for older children, adolescents, and adults, not for toddlers. Problem solving helps adolescents make choices; it N isUtoRoSaI dvNanGcT edBf. orCaOtM oddler. DIF: Analyze REF: 329, Box 20-5 OBJ: Discuss how to integrate health education into patient-centred care. TOP: Planning MSC: CPNRE: Foundations of Practice 13. Which factors should the nurse assess to determine a patient’s ability to learn? a. Developmental capabilities and physical capabilities b. Sociocultural background and motivation c. Psychosocial adaptation to illness and active participation d. Stage of grieving and overall physical health ANS: A
Developmental and physical capabilities reflect a person’s ability to learn. Sociocultural background, motivation, psychosocial adaptation to illness, and active participation are factors in readiness to learn. Readiness to learn is also related to the stage of grieving. Overall physical health does reflect ability to learn; however, because it is paired here with stage of grieving (which is a readiness to learn factor), this is a wrong answer. DIF: Understand REF: 328-330 OBJ: Differentiate factors that determine readiness to learn from those that determine the ability to learn. TOP: Assessment MSC: CPNRE: Foundations of Practice 14. A nurse is preparing to teach a patient about heart failure. Which environment is best for
patient learning?
Canadian Fundamentals of Nursing 6th Edition Potter Test Bank a. b. c. d.
A darkened, quiet room A well-lit, ventilated room A private room at 30°C (85°F) temperature A group room for 10 to 12 patients with heart failure
ANS: B
The ideal environment for learning is a room that is well lit and has good ventilation, appropriate furniture, and a comfortable temperature. Although quiet is appropriate, a darkened room interferes with the patient’s ability to watch the nurse’s actions, especially when the nurse is demonstrating a skill or using visual aids such as posters or pamphlets. A room that is cold, hot, or stuffy makes the patient too uncomfortable to focus on the information being presented. Learning in a group of six or fewer is more effective than in larger groups and tends to prevent outburst behaviours. DIF: Apply TOP: Planning
REF: 328 OBJ: Establish an environment that promotes learning. MSC: CPNRE: Foundations of Practice
15. Which situation indicates to the nurse that the patient is ready to learn? a. A patient has sufficient upper body strength to move from a bed to a wheelchair. b. A patient has the ability to grasp and apply the elastic bandage. c. A patient with a below-the-knee amputation is motivated about how to walk with
assistive devices. d. A patient has normal eyesight to identify the markings on a syringe and
coordination to handle a syringe. ANS: C
Motivation or readiness to learn sometimes results from social task mastery, or physical motives may be involved. OfN teU nR paStiI enNtsG ’T mB ot. ivC esOaM re physical. Some patients are motivated to return to a level of self-sufficiency; for example, a patient with a below-the-knee amputation is motivated to learn how to walk with assistive devices. Readiness to learn should not be confused with ability to learn. All the other answers are examples of ability to learn because the capabilities mentioned often depend on the patient’s level of physical development and overall physical health. To learn psychomotor skills, a patient needs to possess a certain level of strength, coordination, and sensory acuity. For example, it is useless to teach a patient to transfer from a bed to a wheelchair if he or she has insufficient upper body strength. An older patient with poor eyesight or an inability to grasp objects tightly cannot learn to apply an elastic bandage or handle a syringe. DIF: Evaluate REF: 330 OBJ: Differentiate factors that determine readiness to learn from those that determine the ability to learn. TOP: Evaluate MSC: CPNRE: Foundations of Practice 16. A nurse is teaching the staff about nursing and teaching processes. During the teaching
process, what should the nurse do? a. Assess all sources of data. b. Identify it as the same as the nursing process. c. Perform nursing care therapies. d. Focus on a patient’s learning needs. ANS: D
Canadian Fundamentals of Nursing 6th Edition Potter Test Bank The teaching process focuses on the patient’s learning needs and willingness and capability to learn. Nursing and teaching processes are not the same. Assessing all sources of data and performing nursing care therapies are components of the nursing process. DIF: Remember REF: 326 OBJ: Compare and contrast the nursing and teaching processes. TOP: Planning MSC: CPNRE: Foundations of Practice 17. Which nursing action is most appropriate for assessing a patient’s learning needs? a. Assess the patient’s total health care needs. b. Assess the patient’s health literacy. c. Assess all sources of patient data. d. Assess the goals of patient care. ANS: B
Because health literacy influences how you deliver teaching strategies, the nurse must assess a patient’s health literacy before providing instruction. The nursing process requires assessment of all sources of data to determine a patient’s total health care needs. Evaluation of the teaching process involves determining outcomes of the teaching/learning process and the achievement of learning objectives, not patient care. Assessing the goal of meeting patient care is the evaluation component of the nursing process. DIF: Analyze REF: 331 | 338 OBJ: Compare and contrast the nursing and teaching processes. TOP: Assessment MSC: CPNRE: Foundations of Practice 18. A nurse is going to teach a patient about hypertension. Which action should the nurse
NURSINGTB.COM implement first? a. Set mutual goals for knowledge of hypertension. b. Teach what the patient wants to know about hypertension. c. Assess what the patient already knows about hypertension. d. Evaluate the outcomes of patient education for hypertension. ANS: C
Assessment is the first step of any teaching session, followed by diagnosing, planning, implementation, and evaluation. An effective assessment provides the basis for individualized patient teaching. Assessing what the patient currently knows improves the outcomes of patient education. DIF: Apply REF: 331, Table 20-1 OBJ: Identify appropriate topics that address a patient’s health education needs. TOP: Assessment MSC: CPNRE: Foundations of Practice 19. A patient had a stroke and must use a cane for support. A nurse is preparing to teach the
patient about the cane. Which learning objective/outcome is most appropriate? a. The patient will walk to the bathroom and back to bed using a cane. b. The patient will understand the importance of using a cane. c. The patient will learn how to use a cane. d. The patient will know the correct use of a cane. ANS: A
Canadian Fundamentals of Nursing 6th Edition Potter Test Bank Outcomes refer to the patient’s ability to do something on completion of teaching such as will empty colostomy bag, or will administer an injection. Understanding, learning, and knowing are not behaviours that can be observed or evaluated. DIF: Apply TOP: Planning
REF: 333 OBJ: Write learning objectives for a teaching plan. MSC: CPNRE: Foundations of Practice
20. Which learning objective/outcome has the highest priority for a patient with life-threatening,
severe food allergies that necessitate use of an EpiPen (epinephrine)? a. The patient will demonstrate the correct way to administer epinephrine. b. The patient will identify the main ingredients in several foods. c. The patient will list the side effects of epinephrine. d. The patient will learn about food labels. ANS: A
Once the nurse assists in meeting patient needs related to basic survival (how to give epinephrine), the nurse can discuss other topics, such as nutritional needs and side effects of medications. For example, a patient with a recent diagnosis of coronary artery disease may have deficient knowledge related to the illness and its implications. The patient benefits most by first learning about the correct way to take nitroglycerine and how long to wait before calling for help when chest pain occurs. Thus in this situation, the patient benefits most by first learning about the correct way to take epinephrine. “The patient will learn about food labels” is not objective or measurable and is not specific enough. DIF: Evaluate TOP: Evaluate
REF: 333 OBJ: Write learning objectives for a teaching plan. MSC: CPNRE: Foundations of Practice
21. After a teaching session on taNkU inR gS blI ooNdGpT reB ss. urC esO, M the nurse tells the patient, “You took that
blood pressure like an experienced nurse.” What type of reinforcement did the nurse use? a. Material. b. Activity. c. Social. d. Entrusting. ANS: C
Three types of reinforcers are social, material, and activity. When a nurse works with a patient, most reinforcers are social and are used to acknowledge a learned behaviour (e.g., smiles, compliments, words of encouragement). Examples of material reinforcers include food, toys, and music. Activity reinforcers rely on the principle that a person is motivated to complete a task if he or she has the opportunity to engage in a more desirable activity afterwards. The entrusting approach is a teaching approach that provides the patient the opportunity to manage self-care. It is not a type of reinforcement. DIF: Understand REF: 336 OBJ: Determine appropriate communication principles when patient education is provided. TOP: Implementation MSC: CPNRE: Foundations of Practice 22. A patient with heart failure is learning to reduce salt in the diet. When would be the best time
for the nurse to address this topic? a. At bedtime, when the patient is relaxed. b. At lunchtime while the nurse is preparing the food tray.
Canadian Fundamentals of Nursing 6th Edition Potter Test Bank c. At bath time, when the nurse is cleaning the patient. d. At medication time, when the nurse is administering medication. ANS: B
Appropriate times to talk about food/diet changes during routine nursing care are at breakfast, lunch, and dinner times or when the patient is filling out the menu. Many nurses find that they are able to teach more effectively while delivering nursing care. For example, while hanging blood, you explain to the patient why the blood is necessary and the symptoms of a transfusion reaction that need to be reported immediately. In this situation, because the teaching is about food, coordinating it with routine nursing care that involves food can be effective. Bedtime would be a good time to discuss routines that enhance sleep. Bath time would be a good time to describe skin care and how to prevent pressure ulcers. Medication time would be a good time to explain the purposes and side effects of the medication. DIF: Apply REF: 333 OBJ: Include patient teaching while performing routine nursing care. TOP: Implementation MSC: CPNRE: Foundations of Practice 23. A patient has been taught how to cough and deep breathe. Which evaluation method is most
appropriate? a. Return demonstration. b. Computer instruction. c. Verbalization of steps. d. Cloze test. ANS: A
To demonstrate mastery of the skill, the patient should perform a return demonstration under the same conditions that willN beUeRxpe hom Irien Gced Ba.t C Me or in the place where the skill is to be S N T O performed. Computer instruction is a programmed instruction format in which computers store response patterns for learners and select further lessons on the basis of these patterns (programs can be individualized). Computer instruction is a teaching tool rather than an evaluation tool. Verbalization of steps can be an evaluation tool, but it is not as effective for evaluating a psychomotor skill as a return demonstration is. The Cloze test is a test of reading comprehension in which patients fill in the blanks in a written paragraph. DIF: Evaluate TOP: Evaluate
REF: 336 OBJ: Use appropriate methods to evaluate learning. MSC: CPNRE: Foundations of Practice
24. A patient has been taught how to change a colostomy bag but is having trouble measuring and
manipulating the equipment and has many questions. What is the nurse’s next action? a. Refer to a mental health specialist. b. Refer to an ostomy specialist. c. Refer to a dietitian. d. Refer to a wound care specialist. ANS: B
Resources that specialize in a particular health need (e.g., wound care or ostomy specialists) are integral to successful patient education. A mental health specialist is helpful for emotional issues rather than for physical problems. A dietitian is a resource for nutritional needs. A wound care specialist provides complex wound care.
Canadian Fundamentals of Nursing 6th Edition Potter Test Bank DIF: Understand REF: 334 TOP: Implementation
OBJ: Use appropriate methods to evaluate learning. MSC: CPNRE: Foundations of Practice
25. A nurse has taught a patient about healthy eating habits. Which learning objective/outcome is
most appropriate for the affective domain? a. The patient will state three facts about healthy eating. b. The patient will identify two foods for a healthy snack. c. The patient will verbalize the value of eating healthy. d. The patient will cook a meal with low-fat oil. ANS: C
Affective learning deals with expression of feelings and acceptance of attitudes, opinions, or values. Having the patient value healthy eating habits falls within the affective domain. Stating three facts or identifying two foods for a healthy snack falls within the cognitive domain. Cooking falls within the psychomotor domain. DIF: Analyze TOP: Planning
REF: 327 OBJ: Write learning objectives for a teaching plan. MSC: CPNRE: Foundations of Practice
26. A nurse is assessing the ability to learn of a patient who has recently suffered a stroke. Which
question/statement will best assess the patient’s ability to learn? a. “What do you want to know about strokes?” b. “On a scale from 1 to 10, tell me where you rank your desire to learn.” c. “Do you feel strong enough to perform the tasks I will teach you?” d. “Please read this handout and tell me what it means.” ANS: D
A patient’s reading level affeN cts R ity tG o leB CRea Mding level is often difficult to assess Uabil SI N Tar.n.are Ooften because patients who are functionally illiterate able to conceal it by using excuses such as not having the time or not being able to see. One way to assess a patient’s reading level and level of understanding is to ask the patient to read instructions from an educational handout and then explain their meaning. Asking patients what they want to know helps identify previous learning and learning needs and preferences; it does not help assess ability to learn. Motivation is related to readiness to learn, not ability to learn. Just asking a patient whether he or she feels strong is not as effective as actually assessing the patient’s strength. DIF: Evaluate REF: 329 | 338 | 339 OBJ: Differentiate factors that determine readiness to learn from those that determine the ability to learn. TOP: Assessment MSC: CPNRE: Foundations of Practice 27. A nurse is preparing to teach a kinesthetic learner about exercise. Which technique should the
nurse use? a. Let the patient touch and use the exercise equipment. b. Provide the patient with pictures of the exercise equipment. c. Let the patient listen to a podcast about the exercise equipment. d. Provide the patient with a case study about the exercise equipment. ANS: A
Canadian Fundamentals of Nursing 6th Edition Potter Test Bank Kinesthetic learners learn best while they are moving and participating in hands-on activities. Demonstrations and role-playing work well with these learners. Patients who are visual learners learn best from pictures and diagrams that explain information. Patients who prefer auditory learning are distracted by pictures and prefer listening to information (e.g., podcasts). Patients who learn best by reasoning logically and intuitively learn better if presented with a case study that requires careful analysis and discussion with others to arrive at conclusions. DIF: Apply REF: 330 OBJ: Identify appropriate topics that address a patient’s health education needs. TOP: Planning MSC: CPNRE: Foundations of Practice 28. Which statements by the nurse indicate a good understanding of patient education/teaching? a. “Patient education is an essential component of safe, patient-centred care.” b. “Patient education is not effective with children.” c. “Patient teaching can increase health care costs.” d. “Patient teaching does not need to be documented in the chart.” ANS: A
Patient education is an essential component of providing safe, patient-centred care. It is important to document evidence of successful patient education in patients’ medical records. Patient education is effective for children. Different techniques must be used with children. Creating a well-designed, comprehensive teaching plan that fits a patient’s unique learning needs reduces health care costs, improves the quality of care, and ultimately changes behaviours to improve patient outcomes. DIF: Understand REF: 324-326 | 340 OBJ: Identify the role of the nurse in patient education. MSC: CPNRE: Foundations ofNPU raR ctiS ceINGTB.COM
TOP: Evaluate
29. Which following situation would cause the nurse to postpone a teaching session? a. The patient is mildly anxious. b. The patient is asking questions. c. The patient is afebrile. d. The patient is in the bargaining phase. ANS: D
When patients enter the stage of bargaining, they are still unready to learn, and the teaching session should be postponed. The nurse should postpone teaching when an illness becomes aggravated by complications such as a high fever or respiratory difficulty; however, if a patient is afebrile, this should not affect learning. A mild level of anxiety motivates learning. When patients are ready to learn, they frequently ask questions. DIF: Apply REF: 338 | 339 OBJ: Include patient teaching while performing routine nursing care. TOP: Implementation MSC: CPNRE: Foundations of Practice
Canadian Fundamentals of Nursing 6th Edition Potter Test Bank
Chapter 21: Developmental Theories Potter et al: Canadian Fundamentals of Nursing, 6th Edition MULTIPLE CHOICE 1. A child is assessed as being extremely active, requiring a highly structured environment, and
irregular in her habits. According to Chess and Thomas (1995), an accurate interpretation of the child’s category of temperament is which of the following? a. Easy child. b. Difficult child. c. Hyperactive child. d. Slow–to–warm up child. ANS: B
According to Chess and Thomas, “difficult child” would accurately describe this child’s behaviour. The “easy child” is even-tempered, regular, and predictable. The temperament category “hyperactive child” does not exist within Chess and Thomas’s common behaviour categories. The “slow–to–warm up child” typically reacts negatively and with mild intensity to new stimuli. DIF: Analyze REF: 346 OBJ: Discuss factors influencing growth and development. MSC: CPNRE: Foundations of Practice
TOP: Assessment
2. The nursing instructor needs to provide further instruction to the student who makes which
statement? N R I G B.C M a. “Intellectual development iU s afS fectN ed bT y cogniO tive processes.” b. “Socioemotional processes can influence an individual’s growth and development.” c. “Breast development is an example of a change resulting from biological processes.” d. “An individual’s biological processes determine physical characteristics and do not affect growth and development.” ANS: D
Human growth and development are now seen as processes in which sociocultural, biological, and psychological forces interact with the individual over time. Cognitive processes comprise changes in intelligence, use of language, and development of thinking. Socioemotional processes consist of variations in personality, emotions, and relationships with others. Height and weight, development of gross and fine motor skills, and sexual maturation resulting from hormonal changes during puberty are examples of changes resulting from biological processes. DIF: Understand REF: 344 OBJ: Discuss factors influencing growth and development. MSC: CPNRE: Foundations of Practice
TOP: Assessment
3. Which of these statements would be most appropriate for a nurse to state when assessing an
adult patient for growth and developmental delays? a. “How many times per week do you exercise?”
Canadian Fundamentals of Nursing 6th Edition Potter Test Bank b. “Are you able to stand on one foot for 5 seconds?” c. “Would you please describe your usual activities during the day?” d. “How many hours a day do you spend watching television or sitting in front of a
computer?” ANS: C
Understanding normal growth and development helps nurses predict, prevent, and detect deviations from patients’ own expected patterns. The nurse can then compare expected patterns of activity on the basis of age with the patient’s stated activity patterns to determine deviations from the patient’s own expected patterns. Asking the patient to describe his or her usual daily activities will elicit useful information about the patient’s own expected patterns. How many hours are spent watching television or in front of a computer and how many times the patient exercises in a week are closed-ended questions. These questions would not provide the nurse with as much information about the patient’s expected patterns when his or her stated patterns are compared with expected patterns for the patient’s age group to detect delays. DIF: Apply REF: 343 OBJ: Discuss factors influencing growth and development. MSC: CPNRE: Foundations of Practice
TOP: Assessment
4. Which of the following is a priority reason for being knowledgeable about biophysical
developmental theories? a. Understanding how the physical body grows. b. Predicting definite patterns of cognitive development. c. Anticipating how patients’ social behaviours develop. d. Describing the process of psychological development. ANS: A
N R I G B.C M U S N T O
Biophysical development concerns how people’s physical bodies grow and change. Nurses and other health care providers are able to quantify the changes that occur as a newborn grows into adulthood and compare them with established norms to detect abnormalities. Biophysical development is concerned with physical growth, not cognitive development, social behaviours, or psychological development. DIF: Knowledge REF: 344 | 345 OBJ: Name and describe the major developmental theories associated with each tradition. TOP: Assessment MSC: CPNRE: Foundations of Practice 5. While assessing an 18-month-old toddler, the nurse distinguishes normal from abnormal
findings by remembering that Gesell’s theory of development accounts for which of the following statements? a. “The developmental stage of the toddler is affected solely by environmental influence.” b. “Developmental patterns are not affected by gene activity.” c. “Skill development should be identical to that of other toddlers in the playroom.” d. “Environmental influence does not affect the sequence of development.” ANS: D
Canadian Fundamentals of Nursing 6th Edition Potter Test Bank According to Gesell’s theory of development, environment plays a part in child development, but it does not have any part in the sequence of development. Other factors influencing growth and development include biological, cognitive, and socioemotional processes. Environmental factors support, change, and modify the pattern of development, but they do not generate progressions of development. Each child’s pattern of growth and development is unique and is directed by gene activity. Not every child develops certain skills at the same time. Children grow according to their own genetic blueprint. DIF: Understand REF: 345 | 346 OBJ: Name and describe the major developmental theories associated with each tradition. TOP: Assessment MSC: CPNRE: Foundations of Practice 6. Which statement is in accordance with Freud’s psychoanalytical/psychosocial theory? a. Adult personality is the result of resolved conflicts between sources of sexual
pleasure and the mandates of reality. b. Development occurs throughout the life span and focuses on psychosocial stages. c. The genital stage precedes the phallic stage of development. d. Problems evident in adult life are due to early successes and resolution of earlier
developmental stages. ANS: A
Freud believed that adult personality is the result of how an individual resolved conflicts between sources of sexual pleasure and the mandates of reality. Freud had a strong influence on Erik Erikson, but Erikson’s theory differed from Freud’s in that it focused on psychosocial stages rather than psychosexual stages. Freud’s five stages of psychoanalytical development are, in sequential order, oral, anal, phallic, latency, and genital. The phallic stage precedes the genital stage. In theory, problems in adult life would be due to unresolved conflicts and N R I G B.C M U S N T O failures. DIF: Knowledge REF: 349 | 350 OBJ: Name and describe the major developmental theories associated with each tradition. TOP: Assessment MSC: CPNRE: Foundations of Practice 7. The nurse is teaching a young adult couple about promoting the health of their 8-year-old
child. The nurse knows that the parents understand the developmental stage their child is in according to Erikson when they state which of the following? a. “We should provide proper support for learning new skills.” b. “We should encourage devoted relationships with others.” c. “We should limit choices and provide harsh punishment for mistakes.” d. “We should not leave our child at school for longer than 3 hours at a time.” ANS: A
An 8-year-child would be in the industry versus inferiority stage of development. During this stage, the child needs to be praised for accomplishments such as learning new skills. Developing devoted relationships is part of the identity versus role confusion stage, usually occurring during puberty. During the autonomy versus shame and doubt stage, limiting choices and harsh punishment lead to feelings of shame and doubt. Separation anxiety is usually a part of the trust versus mistrust stage. DIF: Understand REF: 350-352 OBJ: Name and describe the major developmental theories associated with each tradition.
Canadian Fundamentals of Nursing 6th Edition Potter Test Bank TOP: Evaluate
MSC: CPNRE: Foundations of Practice
8. Jean Piaget’s cognitive developmental theory focuses on four stages of development,
including which of the following? a. Formal operations. b. Intimacy versus isolation. c. Latency. d. The postconventional level. ANS: A
Jean Piaget’s theory includes four stages in sequential order: sensorimotor, preoperational, concrete operations, and formal operations. Intimacy versus isolation is part of Erik Erikson’s psychosocial theory of development. Latency is stage 4 of Freud’s five-stage psychosexual theory of development. The postconventional level of reasoning is part of Kohlberg’s theory of moral development. DIF: Knowledge REF: 347, Table 21-2 OBJ: Name and describe the major developmental theories associated with each tradition. TOP: Assessment MSC: CPNRE: Foundations of Practice 9. According to Piaget’s formal operations level, what is a 13-year-old likely to do? a. Hit other students to deal with environmental change. b. Use play to understand her surroundings. c. Question her parents about an upcoming presidential election. d. Question where the ice is hiding when ice has melted in her drink. ANS: C
In the formal operations perioNd, aRdolI esceGntsBa. ndCyoM ng adults begin to think about such U Sfinding N Tjustice, Ouand subjects as achieving world peace, seeking meaning in life. Asking about a presidential election demonstrates that the adolescent is concerned about political issues that affect others besides her. Hitting would be a common schema during the sensorimotor stage of development. Using play to learn about the environment is indicative of the preoperational stage. During the concrete operations stage (ages 6 to 12 years), children are able to coordinate two concrete perspectives in social and scientific thinking, such as understanding the difference between “hiding” and “melting.” DIF: Understand REF: 347, Table 21-2 OBJ: Name and describe the major developmental theories associated with each tradition. TOP: Assessment MSC: CPNRE: Foundations of Practice 10. According to Piaget’s theory of cognitive development, the nurse should allow a hospitalized
4-year-old patient to safely play with which item? a. The pump administering intravenous fluids. b. The blood pressure cuff. c. A baseball bat. d. A book to read alone in a quiet place. ANS: B
Canadian Fundamentals of Nursing 6th Edition Potter Test Bank A 4-year-old child would be in the preoperational period. Children at this stage are still egocentric. Play is very important for fostering cognitive development. Children should be allowed to play with any equipment that is safe, such as a blood pressure cuff, and should be allowed to communicate feelings about their health care. The intravenous pump is not a safe piece of equipment for a 4-year-old child to play with. A baseball bat is not typically found in a hospital setting and is a potentially dangerous toy to play with in the hospital. A 4-year-old child is of preschool age and may not be able to read yet. Also, the book does not allow for any human interaction and communication if it is read alone. DIF: Apply REF: 347, Table 21-2 OBJ: Name and describe the major developmental theories associated with each tradition. TOP: Planning MSC: CPNRE: Foundations of Practice 11. Which of these manifestations, if identified in a 6-year-old patient, should the nurse associate
with a possible developmental delay according to Piaget’s theory? a. The child speaks in complete sentences but often talks only about himself. b. The child still plays with a favourite doll that he has had since he was a toddler. c. The child continues to suck his thumb. d. The child describes an event from his own perspective, even though the entire family was present. ANS: C
Thumb sucking is a characteristic of the sensorimotor stage (birth to 2 years), in which schemas become self-initiated activities. For example, the infant who learns that sucking achieves a pleasing result generalizes the action to suck fingers, blanket, or clothing. Successful achievement leads to greater exploration. By age 6, the child is in the preoperational stage of development. The child is expected to be egocentric, even though N. Pla RSyIbN GTB.C M language ability is progressingU ecom es a pO rimary means by which children foster their cognitive development; therefore, playing with a doll is considered normal at this age. Children see objects and persons from only one point of view—their own—at this stage. DIF: Analyze REF: 347, Table 21-2 OBJ: Name and describe the major developmental theories associated with each tradition. TOP: Assessment MSC: CPNRE: Foundations of Practice 12. An 18-month-old patient is brought into the clinic for evaluation because the mother is
concerned. The 18-month-old child hits her siblings and says only “No” when communicating verbally. According to Piaget’s theory, what recommendation should the nurse make a priority? a. Consult the social worker because the child is hitting other children. b. Reassure the mother that the child is developmentally within specified norms. c. Encourage the mother to seek psychological counselling for the child. d. Remove all toys from the child’s room until this behaviour ceases. ANS: B
Canadian Fundamentals of Nursing 6th Edition Potter Test Bank At 18 months, the child is in the sensorimotor period of development. Piaget described hitting, looking, grasping, and kicking as normal schemas to deal with the environment. The social worker does not need to be consulted in this case, and psychological counselling is not warranted, because the child is exhibiting normal behaviours. Play is an important part of all children’s development. Removing the toys is not necessary because this child is exhibiting normal behaviours; removing toys and the opportunity to play with them may actually hinder the child’s development. DIF: Evaluate REF: 347, Table 21-2 OBJ: Name and describe the major developmental theories associated with each tradition. TOP: Implementation MSC: CPNRE: Foundations of Practice 13. A nursing student overhears a 72-year-old patient reflecting on whether his or her life was
meaningful. The nursing student acknowledges that the patient is in which stage of Erikson’s eight stages of life? a. Initiative versus guilt. b. Industry versus inferiority. c. Integrity versus despair. d. Intimacy versus isolation. ANS: C
During the integrity versus despair stage (65 years and older), older persons reflect on their life and feel satisfaction or disappointment. The question answered during this stage is “Has my life been worthwhile?” The older person reflecting on his or her life is not in the initiative versus guilt stage, the industry versus inferiority stage, or the intimacy versus isolation stage. DIF: Apply REF: 351, Table 21-5 NaUjoR OBJ: Name and describe the m r dSeI veN loG pm TeBnt.alCthOeoMries associated with each tradition. TOP: Planning MSC: CPNRE: Foundations of Practice 14. The parents of a 14-year-old boy express concern over their child’s rebellious behaviour. The
nurse should plan to respond to the parents’ concern by informing them of which of the following? a. Their child should be referred to a juvenile correctional facility. b. Their child’s behaviour is normal because the adolescent is trying to adjust to his emerging identity. c. Their child’s behaviour is a matter of concern because he is probably conflicted about establishing companionship with a partner. d. Their child’s behaviour is expected because he is expressing his need to support future generations. ANS: B
According to Erikson, a 14-year-old adolescent is developing his identity in relation to role confusion. A teenager is very concerned with self and is often preoccupied with body image. Frequently, teenagers express themselves rebelliously as they struggle to discover their own identities. Rebellious behaviour is very common and normal at this stage of development. A juvenile correctional facility usually is not necessary. Establishing companionship occurs during the young adult period. Feeling the need to support future generations is usually experienced by middle-aged adults. DIF: Apply
REF: 351, Table 21-5
Canadian Fundamentals of Nursing 6th Edition Potter Test Bank OBJ: Name and describe the major developmental theories associated with each tradition. TOP: Planning MSC: CPNRE: Foundations of Practice 15. The teaching plan for a 3-year-old child who is at risk for developmental delay should include
which of these instructions for the parents? a. “Encourage play as your child is exploring his or her surroundings.” b. “Insist that your child discuss various points of view, not just his or her own.” c. “Discuss world events with your child to foster language development.” d. “Actively encourage your child to read lengthy books to expedite reading and writing abilities.” ANS: A
A 3-year-old child uses play to learn and discover the surrounding environment. Children at this age are egocentric and often are unable to see the world from any perspective other than their own. Very young children are not able to understand and comment on world events because their thinking has not advanced to abstract reasoning yet. A 3-year-old child is probably unable to read. Asking a child to perform an activity that is beyond his or her developmental abilities will probably result in frustration at not being able to complete the task. DIF: Plan REF: 347, Table 21-2 OBJ: Name and describe the major developmental theories associated with each tradition. TOP: Application MSC: CPNRE: Foundations of Practice 16. A nurse should instruct the parents of a 10-year-old child to keep which of the following
theoretical principles in mind when dealing with a behavioural problem at home? a. Strategies that worked well with the first child will be equally as effective for the NURSINGTB.COM second child. b. Encourage the child to volunteer some time at a local hospital to instill a sense of fulfillment. c. Bargaining about chores in exchange for privileges may be an effective method of encouraging helpful activities. d. Do not offer praise for accomplishments and punishment for behavioural issues. ANS: C
In the concrete operations period, children begin to cooperate and share new information about the acts they perform. Parents can adjust their approaches to guide the child into helpful activities within the home, such as bargaining about chores in exchange for privileges. With the birth of a second child, most parents find that the strategies that worked well with the first child no longer work at all. After birth, children grow according to their genetic blueprint and gain skills in an orderly manner, but at each individual’s own pace. The need for a sense of fulfillment is usually experienced by middle-aged adults, not children. School-aged children need praise to discourage a sense of inferiority; providing praise is the best choice for encouraging positive behaviours while growth and development are nurtured. DIF: Understand REF: 347, Table 21-2 OBJ: Discuss nursing implications associated with the application of developmental principles to patient care. TOP: Implementation MSC: CPNRE: Foundations of Practice
Canadian Fundamentals of Nursing 6th Edition Potter Test Bank 17. The parents of a 15-month-old child express concern to the nurse about their child’s
thumb-sucking habit. Which of these explanations related to the child’s age and developmental level would be most appropriate for the nurse to give the parents? a. “Thumb sucking at this age indicates a developmental delay and should be further assessed.” b. “Sucking achieves a pleasing result for infants, and generalizing that action by thumb sucking is normal.” c. “Thumb sucking at this age demonstrates a transition away from egocentric thinking.” d. “At this age, thumb sucking will enhance language development.” ANS: B
Action patterns are used by infants and toddlers to deal with the environment. For example, the infant who learns that sucking achieves a pleasing result generalizes that action to suck fingers, blankets, or clothing. Children remain egocentric into the preoperational period. Thumb sucking does not indicate transition away from egocentric thinking. No statements have supported thumb sucking as enhancing language development. DIF: Apply REF: 347, Table 21-2 OBJ: Discuss nursing implications associated with the application of developmental principles to patient care. TOP: Implementation MSC: CPNRE: Foundations of Practice 18. Which of these approaches would be most appropriate for the nurse to use when teaching a
4-year-old patient about a scheduled surgery? a. Give the parents a book to read about the procedure and do not discuss the procedure with the child to decrease anxiety. NUing RSbyINtelli GTngBthe .CcOhild M that she can ask only three b. Set boundaries before teach questions because time is limited. c. Insist that the parents wait outside the room to ensure privacy of the child. d. Allow the child to touch and hold medical equipment such as thermometers and empty syringes. ANS: D
Nursing interventions during the preoperational period (age 2 to 7 years) should recognize the use of play (such as handling equipment) to help the child understand the events taking place. Giving the parents a book and not involving the child is not the best option, because the nurse should explain all procedures to children and their parents. Children tend to ask a lot of questions; therefore, limiting questions may increase anxiety. Parents and the child all should be involved in preoperative teaching because the parents will be the primary caregivers upon discharge. DIF: Apply REF: 347, Table 21-2 OBJ: Discuss nursing implications associated with the application of developmental principles to patient care. TOP: Implementation MSC: CPNRE: Foundations of Practice 19. The nurse is planning playroom activities for a hospitalized 6-year-old patient. The playroom
has children’s books and building blocks. Which one of the following additional age-appropriate items should the nurse ensure is available for the child? a. Crayons and paper.
Canadian Fundamentals of Nursing 6th Edition Potter Test Bank b. 500-piece puzzle. c. A hockey stick. d. Magazines and newspapers. ANS: A
A school-aged child thrives on feelings of accomplishment. Drawing pictures, looking at children’s books, and building blocks are all ways that a child this age could play while developing a sense of accomplishment. A 500-piece puzzle would be too difficult for a 6-year-old child to complete without the possibility of getting frustrated. Most magazines and newspapers would be written at too high a reading level for a 6-year-old child. A hockey stick typically is not found in a hospital setting and is a potentially dangerous toy to play with in the hospital. If play items offered to the child are too difficult, the child may become frustrated and may experience a feeling of inferiority. DIF: Apply REF: 351 OBJ: Discuss nursing implications associated with the application of developmental principles to patient care. TOP: Planning MSC: CPNRE: Foundations of Practice
NURSINGTB.COM
Canadian Fundamentals of Nursing 6th Edition Potter Test Bank
Chapter 22: Conception Through Adolescence Potter et al: Canadian Fundamentals of Nursing, 6th Edition MULTIPLE CHOICE 1. To promote parent–child attachment with a healthy newborn, what should the nurse do? a. Encourage close physical contact as soon as possible after birth. b. Do not allow the newborn to remain with parents until the second hour after
delivery. c. Never leave the newborn alone with the mother during the first 8 hours after
delivery. d. Isolate the newborn in the nursery during the first hour after delivery. ANS: A
After immediate physical evaluation and application of identification bracelets, the nurse promotes the parents’ and newborn’s need for close physical contact. Early parent–child interaction encourages parent–child attachment. Most healthy newborns are awake and alert for the first half-hour after birth. This is a good time for parent–child interaction to begin. No evidence in the question stem suggests that the baby cannot be left alone with the parents during the first 8 hours or that the baby should be isolated in the nursery during the first hour. DIF: Knowledge REF: 362 OBJ: Discuss physiological and psychosocial health concerns during the transition of the child from intrauterine to extrauterine life. TOP: Assessment MSC: CPNRE: Foundations of Practice
thU erRoS fI a nNeG wT boBrn.uCnO deMrstands associated health risks to her baby 2. The nurse knows that the moN when she states which of the following? a. “I need to moisten the umbilical cord every hour during the day until the cord falls off.” b. “I need to remind anyone who wants to hold the baby to wash their hands.” c. “I need to leave the blankets off the baby to prevent smothering.” d. “I can throw away the bulb syringe now because my baby is breathing on her own.” ANS: B
Prevention of infection is a major concern in the care of the newborn. Good hand hygiene technique is the most important factor in protecting the newborn from infection. The umbilical stump should be kept clean and dry. Newborns are susceptible to heat loss and cold stress. The nurse should place the newborn directly on the mother’s abdomen, and cover him or her in warm blankets, making sure to keep the head well covered, or place the infant unclothed in an infant warmer with a temperature probe in place. Removal of nasopharyngeal and oropharyngeal secretions remains a priority of care to keep the airway patent. DIF: Understand REF: 362 | 363 OBJ: Discuss physiological and psychosocial health concerns during the transition of the child from intrauterine to extrauterine life. TOP: Assessment MSC: CPNRE: Foundations of Practice 3. What is the priority assessment immediately after and infant’s birth?
Canadian Fundamentals of Nursing 6th Edition Potter Test Bank a. b. c. d.
Assess infant–parent interactions. Promote parent–newborn physical contact. Open the infant’s airway. Assess gestational age.
ANS: C
The most extreme physiological change occurs when the newborn leaves the in utero environment and develops independent respiratory functioning. Direct nursing care includes maintaining an open airway, stabilizing and maintaining the newborn’s body temperature, and protecting the newborn from infection. After immediate physical evaluation and application of identification bracelets, the nurse promotes the parents’ and newborn’s need for close physical contact. After a comprehensive physical assessment, the nurse assesses gestational age and interactions between infant and parents. DIF: Apply REF: 362 | 363 OBJ: Discuss physiological and psychosocial health concerns during the transition of the child from intrauterine to extrauterine life. TOP: Assessment MSC: CPNRE: Foundations of Practice 4. Immediate intervention is needed when the newborn exhibits which of the following? a. A soft, protuberant abdomen. b. Moulding. c. Lack of reflexes. d. Cyanotic hands and feet. ANS: C
Normal reflexes include blinking in response to bright lights, startling in response to sudden loud noises, and sucking, rooN ting aspi UR, gr SI NGng,TBya.wCnin OMg, coughing, sneezing, and hiccupping. Assessment of these reflexes is vital because the newborn depends largely on reflexes for survival and in response to its environment. Normal physical characteristics include the continued presence of lanugo on the skin of the back; cyanosis of the hands and feet for the first 24 hours; and a soft, protuberant abdomen. Moulding, or overlapping of the soft skull bones, allows the fetal head to adjust to various diameters of the maternal pelvis and is a common occurrence with vaginal births. DIF: Apply REF: 364 | 365 OBJ: Discuss physiological and psychosocial health concerns during the transition of the child from intrauterine to extrauterine life. TOP: Assessment MSC: CPNRE: Foundations of Practice 5. Normal physical findings in a healthy newborn include which of the following? a. Sporadic motor movements. b. Cyanosis of the feet and hands for the first 48 hours. c. Triangle-shaped anterior fontanel. d. Weight of 4800 g. ANS: A
Movements in the newborn are generally sporadic, but they are symmetric and involve all four extremities. Cyanosis of the hands and feet is normal for the first 24 hours, not 48 hours. The anterior fontanel is diamond shaped, and the posterior fontanel is triangular; both are found between the unfused bones of the skull. The expected, normal weight of a healthy newborn is between 2700 and 4000 g (6 to 9 pounds).
Canadian Fundamentals of Nursing 6th Edition Potter Test Bank
DIF: Knowledge REF: 364 | 365 OBJ: Describe characteristics of physical growth of the fetus and of the child from birth to adolescence. TOP: Assessment MSC: CPNRE: Foundations of Practice 6. When comparing physical growth patterns between school-aged children and adolescents,
what does the nurse note? a. Physical growth usually slows during the adolescent period. b. Boys usually exceed girls in height and weight by the end of the middle-school years. c. Secondary sex characteristics usually develop during the adolescent years. d. The distribution of muscle and fat remains constant during the adolescent years. ANS: C
Hormonal changes during adolescence contribute to the development of secondary sex characteristics such as pubic and axillary hair growth and voice changes. Physical growth usually slows during the school-aged period, and then a growth spurt occurs during adolescence. Girls usually exceed boys in height and weight by the end of the middle school years. As height and weight increase during adolescence, the distribution of muscle and fat changes. DIF: Understand REF: 381 | 382 | 386 | 387 OBJ: Describe characteristics of physical growth of the fetus and of the child from birth to adolescence. TOP: Assessment MSC: CPNRE: Foundations of Practice 7. Which of the following is characteristic of the cognitive changes in a preschooler? a. The ability to think in a logical manner about the here and now. b. The ability to think abstraNcU tlyRaSnI dN deG alTeB ff. ecC tiO veM ly with hypothetical problems. c. The inability to assume the view of another person and to use symbols to represent
others. d. The ability to classify objects by size or colour. ANS: D
Preschoolers demonstrate their ability to think more complexly by classifying objects according to size or colour. Cognitive changes that provide the ability to think in a logical manner about the here and now and to understand the relationships between things and ideas occur during the school-age years. It is during the teenage years when the individual thinks abstractly and deals effectively with hypothetical problems. The preschooler is unable to assume the view of another person. DIF: Knowledge REF: 377 OBJ: Describe cognitive and psychosocial development from birth to adolescence. TOP: Assessment MSC: CPNRE: Foundations of Practice 8. The nursing instructor will need to provide further instruction to the student who uses which
of these statements when describing the differences between cognitive and psychosocial development in children? a. “The preschooler develops the ability to play in small groups.” b. “The toddler may participate in parallel play.” c. “The school-aged child still requires total assistance in all activities for safety.” d. “The toddler period is a time of potential frustration manifested by temper
Canadian Fundamentals of Nursing 6th Edition Potter Test Bank tantrums.” ANS: C
The care provider should promote independence within safe limits for the school-aged child. School-aged children, according to Erikson, are in the industry versus inferiority stage of development. School-aged children like to perform tasks by themselves when possible and need to be praised for those tasks. Children continue to engage in solitary play during toddlerhood but also begin to participate in parallel play, which is playing beside rather than with another child. Toddlers’ strong will is frequently exhibited in negative behaviour when caregivers attempt to direct their actions. Temper tantrums result when parental restrictions frustrate toddlers. DIF: Understand REF: 383 OBJ: Describe cognitive and psychosocial development from birth to adolescence. TOP: Assessment MSC: CPNRE: Foundations of Practice 9. The nurse is observing his 2-year-old hospitalized patient in the playroom. What is the nurse
most likely to observe the child doing? a. Participating as the leader of a small group activity. b. Sitting beside another child while playing with blocks. c. Separating building blocks into groups by size and colour. d. Seeking out same-sex children to play with. ANS: B
The child beside another child and playing is exhibiting parallel play, characteristic of a toddler. Participating as a group leader does not usually occur until approximately age 5. Preschoolers (ages 3 to 5) demonstrate their ability to think more complexly by classifying objects according to size or cN olou year ild does not have this ability yet. The play URr.SAI2NG TB-o.ldCch OM of preschool children becomes more social after the third birthday as it shifts from parallel to associative play. However, gender does not become a factor until the child reaches school age, when he or she prefers same-sex peers to opposite-sex peers. DIF: Apply REF: 375 OBJ: Describe cognitive and psychosocial development from birth to adolescence. TOP: Assessment MSC: CPNRE: Foundations of Practice 10. A patient in her second trimester of pregnancy comes to the prenatal clinic for a checkup.
What is the most important for the nurse to assess in caring for a woman at this stage of pregnancy? a. Detection of fetal movement. b. Observation that the uterus is below the symphysis pubis. c. Confirmation of the desire to breastfeed or bottle-feed. d. Determination of the presence of morning sickness. ANS: A
During the second trimester, between 16 and 20 weeks’ gestation, the prospective mother begins to feel fetal movement. During the second trimester, the uterus should be above the level of the symphysis pubis. The desire to breastfeed or bottle-feed is more likely to be confirmed during the third trimester. Morning sickness is most likely to occur during the first trimester. DIF: Apply
REF: 362
Canadian Fundamentals of Nursing 6th Edition Potter Test Bank OBJ: Discuss physiological and psychosocial health concerns during the transition of the child from intrauterine to extrauterine life. TOP: Assessment MSC: CPNRE: Foundations of Practice 11. During infant/child development, play is best recognized as which of the following? a. A means to interact with the environment and relate to others. b. Independent of cognitive and social development. c. Nonexploratory and simply play. d. Too soon to achieve milestones. ANS: A
During infancy, play is a meaningful set of activities through which the child interacts with the environment and relates to others. Play provides opportunities for development of cognitive, social, and motor skills. Much of infant play is exploratory as infants use their senses to observe and examine their own bodies and objects of interest in their surroundings. Adults facilitate infant learning by planning activities that promote the development of milestones and by providing toys that are safe for the infant to explore with the mouth and manipulate with the hands. DIF: Knowledge REF: 367 OBJ: Explain the role of play in the development of a child. MSC: CPNRE: Foundations of Practice
TOP: Assessment
12. Which statement by the nurse best explains the importance of play during the toddler stage of
development? a. “Exploration can suppress the toddler’s curiosity to promote safety.” b. “Parental control during play will eliminate the frustration of learning NURSINGTB.COM self-control.” c. “Play can enhance cognitive and psychosocial development.” d. “Play will enhance the toddler’s ability to explore the environment safely without supervision.” ANS: C
Play can enhance cognitive and psychosocial development. The toddler’s curiosity is evident in his or her exploration of the environment. Children strive for independence. Their strong wills are frequently exhibited in negative behaviour when caregivers attempt to direct their actions. Temper tantrums result when parental restrictions frustrate toddlers. Parents need to provide toddlers with graded independence, allowing them to do things that do not result in harm to themselves or others. DIF: Understand REF: 375 OBJ: Explain the role of play in the development of a child. MSC: CPNRE: Foundations of Practice
TOP: Assessment
13. After comparing appropriate play activities for infants and preschool children, the nurse
should offer which of the following activities to an infant? a. Set of cards to organize and separate into groups. b. Set of plastic stacking rings. c. Paperback book. d. Set of sock puppets with movable eyes. ANS: B
Canadian Fundamentals of Nursing 6th Edition Potter Test Bank Play becomes manipulative as the child learns control of the hands. Adults facilitate infant learning by planning activities that promote the development of milestones, and by providing toys that are safe for the infant to explore with the mouth and manipulate with the hands, such as rattles, wooden blocks, plastic stacking rings, squeezable stuffed animals, and busy boxes. Preschoolers demonstrate their ability to think more complexly by classifying objects according to size or colour, and thus the cards are more appropriate for them. Neither group is ready for paperback books. The sock puppet with movable eyes could create a choking hazard if one of the eyes comes off. DIF: Understand REF: 367 OBJ: Explain the role of play in the development of a child. MSC: CPNRE: Foundations of Practice
TOP: Implementation
14. A mother expresses concern because her 5-year-old child frequently talks about friends who
don’t exist. What is the nurse’s best response to this mother’s concern? a. “Have you considered a child psychological evaluation?” b. “It’s very normal for a 5-year-old child to have imaginary playmates.” c. “You should stop your child from playing electronic games.” d. “Pretend play is a sign your child watches too much television.” ANS: B
At age 5, some children have imaginary playmates. Imaginary playmates are a sign of health and allow the child to distinguish between reality and fantasy. The child does not need a psychological evaluation on the basis of this information. Television, videos, electronic games, and computer programs help support development and the learning of basic skills; however, these should be only one part of the child’s total play activities. DIF: Apply REF: 37N 9URSINGTB.COM OBJ: Explain the role of play in the development of a child. MSC: CPNRE: Foundations of Practice
TOP: Implementation
15. Encouraging children to play a game of kickball would be best suited for which age group? a. Infant. b. Toddler. c. Preschool. d. School-aged. ANS: D
A game of kickball would be best suited for school-aged children because in this age group, play involves peers and the pursuit of group goals. Although solitary activities are not eliminated, group play overshadows them. Younger children typically are not able to participate cooperatively in groups yet. Infants begin to play simple social games such as patty-cake and peek-a-boo. Toddlers engage in solitary play but also begin to participate in parallel play. Preschoolers playing together engage in similar if not identical activities; however, no division of labour, rigid organization, or rules are observed. By the age of 5, the group has a temporary leader for each activity. DIF: Apply REF: 381 | 383 OBJ: Explain the role of play in the development of a child. MSC: CPNRE: Foundations of Practice
TOP: Implementation
Canadian Fundamentals of Nursing 6th Edition Potter Test Bank 16. Which of these manifestations, if identified in a school-aged child during a routine
assessment, should a nurse associate with a possible developmental delay or problem? a. Withdrawn demeanour and verbalizes that he has no friends. b. Absence of secondary sex characteristics. c. Lack of peer relationships. d. Curiosity about his or her sexuality. ANS: A
School-aged children should begin to develop friendships and to socialize with others. Interaction with peers allows them to define their own accomplishments in relation to others as they work to develop a positive self-image. The absence of secondary sex characteristics is a major concern of adolescents, not school-aged children, because physical evidence of maturity encourages the development of adult masculine and feminine behaviours in the adolescent. Lack of peer relationships is also a concern of adolescents, not of school-aged children, because adolescents seek a group identity to fulfill their esteem and acceptance needs. Today many researchers believe that school-aged children have a great deal of curiosity about their sexuality. Some experiment, but this play is usually transitory. DIF: Analyze REF: 383 | 384 OBJ: Explain the role of play in the development of a child. MSC: CPNRE: Foundations of Practice
TOP: Assessment
17. The nurse who is teaching a parent about developmental needs of the infant knows that the
parent understands an infant’s developmental needs when he or she states which of the following? a. “My child is too young to understand words.” b. “My child will begin to speak in sentences by 1 year of age.” N joy RSplay INing GTpeek-a B.CO M c. “My child will probably enU -boo.” d. “While my child is in the hospital, I should let the nurses provide most of the care.” ANS: C
By 9 months of age, infants play simple social games such as patty-cake and peek-a-boo. By 1 year of age, not only do infants recognize their own names but many are able to say three to five words and understand almost 100 words. Extended separations from parents complicate the attachment process and increase the number of caregivers with whom they must interact. Ideally, the parents provide most of the care during hospitalizations. DIF: Apply REF: 366 | 367 OBJ: Discuss ways in which you can help parents meet their children's developmental needs. TOP: Assessment MSC: CPNRE: Foundations of Practice 18. During hospitalization, what should the nurse encourage the parents of an 8-month-old infant
to do? a. Provide as much care as possible. b. Not worry about attachments because the infant is too young to develop them. c. Remember that infants cannot differentiate a stranger from a familiar person. d. Relax and allow nursing staff to care for the child at all times. ANS: A
Canadian Fundamentals of Nursing 6th Edition Potter Test Bank Extended separations from parents complicate the attachment process and increase the number of caregivers with whom the infant must interact. Ideally, the parents provide most of the care during hospitalizations. Close attachment to the primary caregivers, most often parents, is usually established by the age of 8 months. Infants seek out these persons for support and comfort during times of stress. By this age, most infants are able to differentiate a stranger from a familiar person and respond differently to the two. DIF: Apply REF: 367 OBJ: Discuss ways in which you can help parents meet their children's developmental needs. TOP: Implementation MSC: CPNRE: Foundations of Practice 19. The nursing student correctly explains health promotion teaching points for parents of
toddlers when she makes which statement? a. “Setting consistent, firm limits will help the child cope with the frustration of learning self-control.” b. “Slower development of motor skills prevents the child from participating in self-care activities.” c. “Toddlers have a natural sense of right and wrong and know when they do something wrong.” d. “Temper tantrums should never be tolerated, and toddlers need to do what they are told.” ANS: A
Patience, support, and firm, consistent limits allow toddlers to develop socially acceptable behaviours and to cope with the frustration of learning self-control. Rapid development of motor skills allows the child to participate in self-care activities such as feeding, dressing, and toileting. Because children’s moral development is closely associated with their cognitive N R dler INsGisTonly B.C M abilities, moral development inUtodS beOginning. Toddlers do not understand concepts of right and wrong. Temper tantrums result when restrictions frustrate toddlers. Parents need to provide toddlers with graded independence, allowing them to do things that do not result in harm to themselves or others. DIF: Apply REF: 372 OBJ: Discuss ways in which you can help parents meet their children's developmental needs. TOP: Implementation MSC: CPNRE: Foundations of Practice 20. A 5-year-old boy is admitted for surgery to have his tonsils removed. On the basis of
knowledge about children of this age, what would the nurse plan to do? a. Allow the child to take responsibility for his own preoperative hygienic care. b. Leave the child alone to relax before the procedure. c. Allow the child to handle and look at the equipment when the nurse takes his blood pressure. d. Provide magazines and puzzles for diversion. ANS: C
Preschool children may cooperate if they are allowed to manipulate the equipment. A preschool child is unable to take responsibility for his or her own preoperative hygienic care. Leaving the preschooler alone may increase the child’s anxiety. Magazines and puzzles would be more appropriate activities for the older child. The preschool child likes to engage in pretend play by using the imagination and imitating adult behaviour.
Canadian Fundamentals of Nursing 6th Edition Potter Test Bank DIF: Apply REF: 376-379 OBJ: Describe cognitive and psychosocial development from birth to adolescence. TOP: Implementation MSC: CPNRE: Foundations of Practice 21. Which of these statements, if made by a parent, would necessitate further instruction? a. “I should not be surprised that my teenager has so many friends.” b. “I get worried because my teenager thinks he’s indestructible. He takes a lot of
risks.” c. “I should cover for my school-aged child when he makes a mistake until he learns
the ropes.” d. “My 10-year-old child is always hungry right after school, so I usually fix him a
nutritious snack.” ANS: C
School and home influence growth and development, which requires adjustment by parents and by the child. The child learns to cope with rules and expectations presented by the school and by peers. Parents have to learn to allow their child to make decisions, accept responsibility, and learn from life’s experiences. Teenagers typically are very social and have many friends. Adolescents seek a group identity because they need esteem and acceptance. By mid-adolescence, children believe that they are unique and the exception to all rules, which gives rise to their risk-taking behaviours. Obesity occurs because children often rush into the home after school or play and eat the most easily obtainable and appealing foods. Providing nutritious snacks is often the best way to ensure good nutritional intake. DIF: Analyze REF: 380, Table 22-5 OBJ: Discuss ways in which you can help parents meet their children's developmental needs. TOP: Implementation MSC: CPNRE: Foundations of Practice
NURSINGTB.COM
22. Which of these toys, if selected by the parent of a 10-month-old child, would indicate that the
parent has a correct understanding of infant growth and development? a. A game requiring two to four players. b. Electronic games. c. Small, plastic alphabet letters and magnets. d. Plastic stacking rings. ANS: D
Adults facilitate infant learning by planning activities that promote the development of milestones and by providing toys that are safe for the infant to explore with the mouth and manipulate with the hands, such as rattles, wooden blocks, plastic stacking rings, squeezable stuffed animals, and busy boxes. Infants are not capable of participating in small group activities. By age 4, children play in groups of two or three. For the toddler (not the infant), television, videos, electronic games, and computer programs help support development and learning of basic skills. Adults should provide toys that are safe for the infant to explore with the mouth. Small, plastic letters and magnets could be choking hazards for an infant. DIF: Evaluate REF: 367 OBJ: Discuss ways in which you can help parents meet their children's developmental needs. TOP: Implementation MSC: CPNRE: Foundations of Practice
Canadian Fundamentals of Nursing 6th Edition Potter Test Bank
Chapter 23: Young to Middle Adulthood Potter et al: Canadian Fundamentals of Nursing, 6th Edition MULTIPLE CHOICE 1. The nurse is performing a physical examination in a 40-year-old patient. The nurse will
probably find that at this age, the patient is experiencing which of the following physiological changes related to normal aging? a. Decreased heart rate. b. Decreased sense of smell. c. Decreased strength of abdominal muscles. d. Decreased function of the cranial nerves. ANS: C
A physiological change related to normal aging in the middle-aged adult would be decreased strength of abdominal muscles. The middle-aged adult should not have a decreased heart rate, should have a normal sense of smell, and should have normal functioning of the cranial nerves. DIF: Apply REF: 404, Table 23-3 OBJ: Describe normal physiological changes in young and middle adulthood, including pregnancy. TOP: Assessment MSC: CPNRE: Foundations of Practice 2. The nurse is caring for a hospitalized young man who is uninsured even though he works as a
dishwasher at a local restaurant. He states that he would like to get a better job, but he has no education. How can the nurse best assist this patient psychosocially? N ndRS I rrals GT. B.C OM a. By providing information aU refeN b. By telling the patient that he needs to go back to school. c. By focusing on the patient’s medical diagnoses. d. By expecting the patient to be flexible in his decision making. ANS: A
Support from the nurse, access to information, and appropriate referrals provide opportunities for achievement of a patient’s potential. Many young adults lack the necessary resources or support systems to facilitate further education or development of skills necessary for many positions in the workplace. As a result, some young adults have limited occupational choices. Health is not merely the absence of disease but involves wellness in all human dimensions. Insecure persons tend to be more rigid in making decisions. DIF: Analyze REF: 397 | 398 OBJ: Discuss major life events and developmental tasks of young and middle-aged adults. TOP: Implementation MSC: CPNRE: Foundations of Practice 3. A nursing student is asked to compare major life events of young adult, middle adult, and
childbearing families. Which statement by the student demonstrates understanding? a. “Young adults have gained sexual experience and do not need sexual education.” b. “Once a woman has her baby, stress levels decrease, as does health risk.” c. “The social pressure to get married is greater now than it ever was.” d. “When married people both work, income is increased, but so is stress.”
Canadian Fundamentals of Nursing 6th Edition Potter Test Bank ANS: D
The two-career family has benefits and liabilities. Stressors result from transfer to a new city; increased expenditures of physical, mental, or emotional energy; child care demands; and household needs. To avoid stress, partners should share all responsibilities. Young adults are at risk for sexually transmitted infections; therefore, there is an increased need for education regarding mode of transmission, prevention, and symptom recognition and management of sexually transmitted infections. The stress that many women experience after childbirth has a significant effect on their health. Social pressure to get married is not as great as it once was, and many young adults do not marry until their late 20s, early 30s, or later or do not marry at all. DIF: Understand REF: 399 OBJ: Discuss major life events and developmental tasks of young and middle-aged adults. TOP: Assessment MSC: CPNRE: Foundations of Practice 4. The nurse knows that the young adult patient understands the health risks that affect his or her
age group when the patient makes which statement? a. “It’s probably safe for me to start smoking. At my age, there’s not enough time for cancer to develop.” b. “I am sure that I am going to get emphysema. Both my mother and my aunt had it. It’s genetic.” c. “Controlling the amount of stress in my life may decrease the risk of illness.” d. “I don’t do drugs. I do drink coffee, but caffeine is not a drug. It is perfectly safe and has no side effects.” ANS: C
Lifestyle habits that activate the stress response increase the risk of illness. Smoking is a N r pu RSlmo INnar GT B.Cac,Mand vascular disease, not only in smokers well-documented risk factor foU y, cardiO but also in individuals who are exposed to second-hand smoke. The presence of certain chronic illnesses in the family increases the family member’s risk for developing a disease. Family risk is distinct from hereditary disease. Caffeine is a drug; it is a naturally occurring legal stimulant that is readily available. Caffeine stimulates catecholamine release, which, in turn, stimulates the central nervous system; it also increases gastric acid secretion, heart rate, and basal metabolic rate. DIF: Understand REF: 400 | 401 OBJ: Discuss major life events and developmental tasks of young and middle-aged adults. TOP: Evaluate MSC: CPNRE: Foundations of Practice 5. When choosing an appropriate topic for a young adult health fair, the nurse ranks which topic
as least relevant? a. Unplanned pregnancies. b. Menopause and climacteric factors. c. Smoking cessation. d. Alcohol and drug use. ANS: B
Canadian Fundamentals of Nursing 6th Edition Potter Test Bank The onset of menopause and the climacteric affect the sexual health of the middle-aged adult, not the young adult. Unplanned pregnancies are a continued source of stress that can result in adverse health outcomes for the mother (young adult), infant, and family. Smoking is a well-documented risk factor for pulmonary, cardiac, and vascular diseases in smokers and in individuals who are exposed to second-hand smoke, and it constitutes a health risk for young adults. Substance abuse directly or indirectly contributes to mortality and morbidity in young adults. DIF: Understand REF: 401 | 405 OBJ: Discuss major life events and developmental tasks of young and middle-aged adults. TOP: Assessment MSC: CPNRE: Foundations of Practice 6. During middle adulthood, the 50-year-old patient is likely to adapt favourably to a changing
body image if he or she does what? a. Decreases the amount of physical exercise. b. Eats a diet composed of 40% fat. c. Gets less than 5 hours of sleep per night. d. Engages in good hygiene practices. ANS: D
High self-esteem, a favourable body image, and a positive attitude toward physiological changes occur when adults engage in physical exercise, balanced diets, adequate sleep, and good hygiene practices that promote vigor and health. DIF: Knowledge REF: 404 OBJ: Discuss major life events and developmental tasks of young and middle-aged adults. TOP: Assessment MSC: CPNRE: Foundations of Practice
NURSINGTB.COM
7. A patient states that she is pregnant, is concerned because she does not know what to expect,
and wants her husband to play an active part in the birthing process. What should the nurse tell the patient? a. Childbirth classes can prepare pregnant women and their partners for what is coming. b. The frequency of sexual intercourse is key to helping the husband feel valued. c. After the birth, the stress of pregnancy will disappear and will be replaced by relief. d. After the baby is born, the wife should accept the extra responsibilities of motherhood. ANS: A
Childbirth education can prepare pregnant women, their partners, and other support persons to participate in the birthing process. The psychodynamic aspect of sexual activity is as important as the type or frequency of sexual intercourse to young adults. The stress that many women experience after childbirth has a significant effect on their health. To avoid stress in a two-career family, partners should share all responsibilities. DIF: Apply REF: 402-404 OBJ: Discuss major life events and developmental tasks of young and middle-aged adults. TOP: Assessment MSC: CPNRE: Foundations of Practice 8. What do changing norms and values about family life in Canada reveal?
Canadian Fundamentals of Nursing 6th Edition Potter Test Bank a. b. c. d.
Basic shifts in attitudes in Canadian society. Greater resistance to cohabitation without marriage. Decreased numbers of infants born to unmarried women. Greater support and acceptance from the health care system.
ANS: A
Changing norms and values about family life in Canada reveal basic shifts in attitudes in Canadian society. The trend toward greater acceptance of cohabitation without marriage is a factor in the increase in numbers of infants being born to unmarried women. Many parents from family structures that do not conform to the norm feel lack of support and even bias from the health care system. DIF: Knowledge REF: 399 OBJ: Discuss the significance of family in the life of the adult. MSC: CPNRE: Foundations of Practice
TOP: Assessment
9. When describing relevant family psychosocial factors in middle adulthood that cause stress,
the nurse would not include which of the following? a. Singlehood and feeling isolated. b. Choices stemming from marital changes. c. Financial security and certainty. d. Planning for the future when children leave home. ANS: C
In the middle adult years, as children depart from the household, the family enters the postparental family stage. Time and financial demands on parents decrease, and the couple faces the task of redefining their own relationship. Psychosocial factors involving the family include the stresses of singlehNood alT cB ha. ngCes, UR, SmIarit NG OMtransition of the family as children leave home, and the care of aging parents. DIF: Understand REF: 405 | 406 OBJ: Discuss the significance of family in the life of the adult. MSC: CPNRE: Foundations of Practice
TOP: Assessment
10. What should the nurse recognize when comparing the physical changes in young and middle
adulthood? a. Fertility issues do not occur in young adulthood. b. Young adults are quite active but are at risk for illness in later years. c. Young adults tend to suffer more from severe illness. d. Exercise is less important in young adulthood than in middle adulthood. ANS: B
Young adults are generally active and have a minimum of major health problems. However, their lifestyles may put them at risk for illnesses or disabilities during their middle or older years. An estimated 10% to 15% of reproductive couples are infertile, and many are young adults. Exercise in young adulthood is increasingly important to prevent or decrease the development of chronic health conditions such as high blood pressure, obesity, and diabetes that develop later in life. DIF: Understand REF: 404 OBJ: Describe normal physiological changes in young and middle adulthood, including pregnancy. TOP: Assessment MSC: CPNRE: Foundations of Practice
Canadian Fundamentals of Nursing 6th Edition Potter Test Bank
11. During a routine physical assessment, the nurse obtaining a health history notes that a
50-year-old woman reports pain and redness in the right breast. What is the nurse’s best action in response to this finding? a. Explain to the patient that breast tenderness is normal at her age. b. Tell the patient that redness is not a cause for concern and is quite common. c. Assess the patient as thoroughly as possible. d. Inform her that redness is the precursor to normal unilateral breast enlargement. ANS: C
A comprehensive assessment offers direction for health promotion recommendations, as well as for planning and implementing any acutely needed intervention. Redness or pain in breasts and increased size of one breast are abnormal physical assessment findings in middle adulthood. DIF: Apply REF: 401 | 402 OBJ: Describe normal physiological changes in young and middle adulthood, including pregnancy. TOP: Assessment MSC: CPNRE: Foundations of Practice 12. A 55-year-old woman presents to the outpatient clinic, describing irregular menstrual periods
and hot flashes. What should the nurse explain? a. Those symptoms are normal when a woman undergoes the climacteric. b. An assessment is not really needed because these problems are normal for older women. c. The patient’s age and symptoms point toward normal menopause. d. The patient should stop regular exercise because that is probably causing her symptoms. ANS: C
NURSINGTB.COM
The most significant physiological changes during middle age are menopause in women and the climacteric in men. The nurse should continue with the examination because a comprehensive assessment offers direction for health promotion recommendations, as well as for planning and implementing any acutely needed interventions. High self-esteem, a favourable body image, and a positive attitude toward physiological changes occur when adults engage in physical exercise, balanced diets, adequate sleep, and good hygiene practices that promote vigorous, healthy bodies. DIF: Apply REF: 405 OBJ: Describe normal physiological changes in young and middle adulthood, including pregnancy. TOP: Assessment MSC: CPNRE: Foundations of Practice 13. The nurse is teaching a class to pregnant women about common physiological changes during
pregnancy. Which statement by the nurse accurately describes these changes? a. “Pregnancy enhances your ability to cope with stress.” b. “Being nauseated and feeling tired will not affect your physical body image.” c. “You and your partner may experience feelings of uncertainty about assuming the roles of parents.” d. “Returning home after delivery will rejuvenate you and foster independence.” ANS: C
Canadian Fundamentals of Nursing 6th Edition Potter Test Bank Expectant parents think about and have feelings of uncertainty about impending role changes. Parents need reassurance that childbirth and childrearing are natural and positive experiences but are also stressful. Parents often are unable to cope with particular stressors. Morning sickness and fatigue contribute to poor body image. New mothers often return home from the hospital fatigued and unfamiliar with infant care. DIF: Understand REF: 402-404 OBJ: Discuss cognitive and psychosocial changes that occur during the adult years. TOP: Implementation MSC: CPNRE: Foundations of Practice 14. A nurse is about to discuss the risks of repeated sun exposure with a young adult patient.
Which of these statements would be most expected from this patient before the discussion? a. “I’ll make an appointment with my doctor right away for a full skin check.” b. “I should consider participating in a health fair about safe sun practices.” c. “I have a mole that has been bothering me. I’ll call my family doctor for an appointment to get it checked.” d. “I’ve had this mole my whole life. So what if it changed colour? My skin is fine.” ANS: D
Young adults often ignore physical symptoms and often postpone seeking health care. Making an appointment with the doctor and participating in health fairs are not typical behaviours of young adults. DIF: Apply REF: 398 OBJ: Discuss cognitive and psychosocial changes that occur during the adult years. TOP: Implementation MSC: CPNRE: Foundations of Practice 15. When performing a thoroughNpU syRcS hoIsoNcG iaT l aBss.eC ssO mM ent on a young adult, what must the nurse
realize? a. Having a job is the best way to relieve stress. b. Although psychologically disturbing, stress does not lead to physical illness. c. Change is inevitable and is not a factor in stress-related illness. d. Psychosocial health is often related to job and family stress. ANS: D
The psychosocial health concerns of young adults are often related to job and family stressors. If stress is prolonged and the patient is unable to adapt to the stressor, health problems will develop. Job assessment also includes conditions and hours, duration of employment, changes in sleep or eating habits, and evidence of increased irritability or nervousness. When a patient seeks health care and presents stress-related symptoms, the nurse needs to assess for the occurrence of a life change event. DIF: Apply REF: 398 | 399 OBJ: Describe health concerns of young and middle-aged adults. TOP: Assessment MSC: CPNRE: Foundations of Practice 16. A 25-year-old patient is brought to the hospital by police after crashing his car in a high-speed
chase when trying to avoid arrest for spousal abuse. What should the nurse do? a. Question the patient about drug use. b. Offer the patient a cup of coffee to calm his nerves. c. Be aware that substance abuse is usually obvious.
Canadian Fundamentals of Nursing 6th Edition Potter Test Bank d. Deal with the issue at hand, and put off asking about previous illnesses. ANS: A
Reports of arrests because of driving while intoxicated, wife or child abuse, or disorderly conduct are reasons for the nurse to investigate the possibility of drug abuse more carefully. Caffeine is a legal drug, naturally occurring in coffee, that stimulates the central nervous system and is not the choice for calming nerves. Substance abuse is not always diagnosable, particularly in its early stages. The nurse may obtain important information by making specific inquiries about past medical problems, changes in food intake or sleep patterns, and problems of emotional lability. DIF: Apply REF: 401 OBJ: Describe health concerns of young and middle-aged adults. TOP: Implementation MSC: CPNRE: Foundations of Practice 17. A 61-year-old obese patient receives diagnoses of type 2 diabetes and high blood pressure.
The patient states that he is upset about the diet restrictions imposed by the treatment regimen. What is the nurse’s best approach? a. Tell the patient that he must do what the doctor tells him. b. Offer counselling on nutrition and exercise. c. Tell the patient about what happened to other patients who did not change their lifestyle. d. Explain that he needs to accept the care provider’s advice without question if he wants to get better. ANS: B
Counselling related to physical activity and nutrition is an important component of the plan of care for overweight and obesN eU paR tien SIts.NTGoThBel.pCthe OMpatients develop positive health habits, the nurse becomes a teacher and a facilitator, providing information and positive reinforcement. Ultimately, however, the patient decides which behaviours will become habits of daily living. Scare tactics do not usually work. By providing information about how the body works and how patients form and change habits, the nurse raises the patient’s level of knowledge regarding the potential effect of behaviour on health. The nurse should encourage patients to express their feelings to promote problem solving and recognition of risk factors by patients themselves. DIF: Apply REF: 406 | 407 OBJ: Describe health concerns of young and middle-aged adults. TOP: Implementation MSC: CPNRE: Foundations of Practice 18. Nurses must be knowledgeable about developmental theories in order to provide competent
care to young and middle-aged adult patients. Which statement by the student nurse demonstrates a need for further instruction by the nursing instructor? a. “These theories provide nurses with a basis for understanding the life events and developmental tasks of young and middle-aged adults.” b. “It is important to understand societal structures and roles because they have not changed in the past 20 or 30 years.” c. “Patients present challenges to nurses, many of whom are themselves young or middle-aged adults coping with the demands of their respective developmental period.” d. “Nurses need to recognize the needs of their patients even if they are not
Canadian Fundamentals of Nursing 6th Edition Potter Test Bank experiencing the same challenges and events.” ANS: B
Faced with a societal structure that differs greatly from the norms of 20 or 30 years ago, both men and women are assuming different roles in today’s society. Developmental theories provide nurses with a basis for understanding the life events and developmental tasks of young and middle adults. Patients present challenges to nurses who themselves are often young or middle-aged adults coping with the demands of their respective developmental period. Nurses need to recognize the needs of their patients even if they are not experiencing the same challenges and events. DIF: Knowledge REF: 397 | 398 OBJ: Discuss major life events and developmental tasks of young and middle-aged adults. TOP: Assessment MSC: CPNRE: Foundations of Practice 19. When providing prenatal care, the nurse provides information related to exercise patterns and
proper diet. What additional information does the nurse expect to provide? a. Protecting against urinary infection. b. No longer needing condoms. c. Discussing care of the child once the infant is born. d. Physical assessments only during the last trimester. ANS: C
Pregnancy affects the entire child-bearing family, and all members benefit from education. Prenatal care includes a thorough physical assessment of the pregnant woman during regularly scheduled intervals (not just the last trimester). Information regarding sexually transmitted, other vaginal, and urinary infections that will adversely affect the fetus is important for a pregnant woman, as is counsN elU linRgSaI boN uG t eT xB er. ciC seOpM atterns, diet, and child care. DIF: Knowledge REF: 402-404 OBJ: Describe health concerns of young and middle-aged adults. TOP: Implementation MSC: CPNRE: Foundations of Practice
Canadian Fundamentals of Nursing 6th Edition Potter Test Bank
Chapter 24: Older Persons Potter et al: Canadian Fundamentals of Nursing, 6th Edition MULTIPLE CHOICE 1. As the aging population in Canada increases, which of the following is true? a. The baby boomer generation accounts for a very small percentage of this group. b. The average lifespan has also increased. c. The population segment of people older than 85 is decreasing. d. Diversity of this age group will certainly decrease. ANS: B
Part of that increase is a result of the extension of the average lifespan. In 2012, the average life expectancy at birth for Canadians was 81.1 years, in comparison with 70.9 in 1991. Two other factors that contribute to the projected increase in the number of older persons are the aging of the baby boom generation and the growth of the population segment over age 85. The baby boomers are the large group of persons born between 1946 and 1964. The diversity of the group older than 65 will also possibly increase. DIF: Remember REF: 411 OBJ: Discuss demographic trends related to older persons in Canada. TOP: Assessment MSC: CPNRE: Foundations of Practice 2. As a patient ages, how should the nursing plan of care change? a. It should be standardized because all geriatric patients have the same needs. b. It needs to be individualized to the patient’s unique needs. NUolo RS IlG B.C. M c. It should be based on chron gicaN agT e aloneO d. It focuses on the disabilities that all aging persons face. ANS: B
Nurses must approach every older person as a unique individual. The nursing care of older persons poses special challenges because of great variation in their physiological, cognitive, and psychosocial health. Nurses need to take into account the cultural, ethnic, and racial diversity of this population (not just age) as they care for older persons from these groups. Aging does not inevitably lead to disability and dependence. DIF: Understand REF: 412 OBJ: Identify common myths and stereotypes about older persons. TOP: Assessment MSC: CPNRE: Foundations of Practice 3. Which of these findings, if identified in a patient on a gerontological unit, would be most
surprising to a culturally sensitive nurse? a. The older person’s not being functionally independent. b. Preferences in food, music, and religion. c. Use of conventions of the handshake, silence, and eye contact. d. Personal health practices and spiritual resources. ANS: A
Canadian Fundamentals of Nursing 6th Edition Potter Test Bank Most older people remain functionally independent despite the increasing prevalence of chronic disease. Examples of culturally competent nursing approaches to older persons include respect for preferences in food, music, and religion; appropriate use of conventions of the handshake, silence, and eye contact; use of interpreters; use of physical assessment norms appropriate for the ethnic group; and asking about personal health practices, family customs, lifestyle preferences, and spiritual resources. DIF: Analyze REF: 412 OBJ: Identify common myths and stereotypes about older persons. TOP: Assessment MSC: CPNRE: Foundations of Practice 4. Which of the following statements by a new graduate nurse should be corrected by an
experienced nurse? a. “Most older patients are ill and disabled. That’s why we care for so many of them in the hospital.” b. “Many older persons are still interested in sexual relations.” c. “Patients older than 65 are still lifelong learners.” d. “Many older patients remain independent enough to live alone.” ANS: A
Older persons are sometimes stereotyped as ill and disabled. However, although many experience chronic conditions or have at least one disability that limits performance of activities of daily living (ADLs), only 23% of older person describe their health as poor or fair. Older persons do report continued enjoyment of sexual relationships. Although changes in vision or hearing and reduced energy and endurance sometimes affect the process of learning, older persons are lifelong learners. Most older persons live in noninstitutional settings with family members or alone. B.C M
N R I G U S N T
O
DIF: Remember REF: 412 | 413 OBJ: Identify common myths and stereotypes about older persons. TOP: Assessment MSC: CPNRE: Foundations of Practice 5. Which teaching strategy is best to utilize with older persons? a. Provide several topics of discussion at once to promote independence and making
choices. b. Avoid uncomfortable silences after questions by helping patients complete their
statements. c. Ask patients to recall past experiences that correspond with their interests. d. Speak in a high pitch to help patients hear better. ANS: C
Teaching strategies include the use of past experiences to connect new learning with previous knowledge, focusing on a single topic to help the patient concentrate, giving the patient enough time in which to respond because older persons’ reaction times are longer than those of younger persons, and keeping the tone of voice low; older persons are able to hear low sounds better than high-frequency sounds. DIF: Understand REF: 412 | 413 OBJ: Identify nursing interventions related to the physiological, cognitive, and psychosocial changes of aging. TOP: Implementation MSC: CPNRE: Foundations of Practice
Canadian Fundamentals of Nursing 6th Edition Potter Test Bank
6. An older patient has fallen and broken his hip. As a consequence, the patient’s family is
concerned about his ability to care for himself, especially during his convalescence. What should the nurse do? a. Stress that older patients usually ask for help when needed. b. Inform the family that placement in a nursing centre is a permanent solution. c. Tell the family to enrol the patient in a ceramics class to maintain his quality of life. d. Provide information and answer questions as family members make choices among care options. ANS: D
Nurses assist older persons and their families by providing information and answering questions as they make choices among care options. Some older persons deny functional declines and refuse to ask for assistance with tasks that place their safety at great risk. The decision to enter a nursing centre is never final, and sometimes a resident of a nursing centre is discharged to home or to another residence for less acute care. What defines quality of life varies from person to person. Nurses must listen to what the older individual considers to be most important rather than making assumptions about the person’s priorities. DIF: Remember REF: 414 OBJ: Identify nursing interventions related to the physiological, cognitive, and psychosocial changes of aging. TOP: Implementation MSC: CPNRE: Foundations of Practice 7. What is the best suggestion a nurse could make to a family requesting help in selecting a local
nursing centre? N R I G B.C M a. Suggest choosing a nursingUcenStre N thatTis as saO nitary as possible. The closer the centre is to hospital standards, the better. b. Have family members evaluate nursing home staff according to their ability to get tasks done efficiently. c. Make sure that nursing home staff members get patients out of bed every day for the entire day. d. Explain that it is probably best for the family to visit the centre and inspect it personally. ANS: D
An important step in the process of selecting a nursing home is to visit the nursing home. The nursing home should not feel like a hospital. It is a home, a place where people live. Members of the nursing home staff should focus on the person, not the task. Residents should be out of bed and dressed according to their preferences, not staff preferences. DIF: Apply REF: 414 OBJ: Discuss common developmental tasks of older persons. MSC: CPNRE: Foundations of Practice
TOP: Implementation
8. A 70-year-old patient who suffers from worsening dementia is no longer able to live alone. In
discussing health care services and possible long-term living arrangements with the patient’s only son, what should the nurse suggest? a. An apartment setting with neighbours close by. b. Weekly home health visits.
Canadian Fundamentals of Nursing 6th Edition Potter Test Bank c. A nursing centre because home care is no longer safe. d. That placement is irrelevant because the patient is retreating to a place of
inactivity. ANS: C
Some family caregivers consider nursing centre placement when in-home care becomes increasingly difficult or when convalescence from hospitalization requires more assistance than the family is able to provide. An apartment setting and home health visits are not appropriate because older persons who cannot take care of themselves are not safe living alone at other times. Others are unable to engage in activities designed to benefit older persons such as senior health promotion activities (such as some health visits) and thus do not receive the benefits that these programs offer. Worsening dementia does not necessarily mean a retreat into inactivity, but it does require a realistic review of strengths and limitations. DIF: Apply REF: 414 OBJ: List the types of community-based and institutional health care services available to older persons. TOP: Implementation MSC: CPNRE: Foundations of Practice 9. Several theories on aging have been put forth; how should the nurse use these theories? a. To guide nursing care. b. To explain the stochastic view of genetically programmed physiological changes. c. To select one theory to guide nursing care for all geriatric patients. d. To understand the nonstochastic views of aging as the result of cellular damage. ANS: A
Although theories on aging are in various stages of development and have limitations, the nurse should use them to incrN eas e un stan di. ngCof UR SIder NG TB OMthe phenomena affecting the health and well-being of older persons and to guide nursing care. According to stochastic theories, aging is the result of random cellular damage occurring over time. No single universally accepted theory predicts and explains the complexities of the aging process. According to nonstochastic theories, aging is the result of genetically programmed physiological mechanisms within the body. DIF: Apply REF: 413 OBJ: Describe the concepts of aging well and quality of life. MSC: CPNRE: Foundations of Practice
TOP: Implementation
10. How would the nurse correctly describe psychosocial theories on aging? a. As theories that describe changes in roles and relationships of older persons. b. As theories that emphasize that all adults age in similar ways. c. As theories that stress the need for older people to discontinue activities as they
age. d. As theories that describe behaviour patterns for all aging persons as unpredictable. ANS: A
Canadian Fundamentals of Nursing 6th Edition Potter Test Bank Psychosocial theories of aging explain changes in behaviours, roles, and relationships that come with aging. Although some theories generalize about aging, each individual ages uniquely both biologically and psychosocially. According to the activity theory, the continuation of activities performed during middle age is necessary for successful aging. According to the continuity theory, personality remains the same and behaviour becomes more predictable as people age. DIF: Remember REF: 419 OBJ: Discuss issues related to psychosocial changes connected with aging. TOP: Assessment MSC: CPNRE: Foundations of Practice 11. When comparing developmental tasks of middle-aged persons versus older persons, what
should the nurse infer? a. Learning to cope with loss is most common during the middle adult years. b. After age 65, most older persons age both biologically and psychologically the same way. c. Older persons will need nursing assistance to deal with loss. d. Older persons fear and resent retirement as a disruption of their lifestyle. ANS: C
Some older persons deny their own aging in ways that are potentially problematic. For example, some older persons deny functional declines and refuse to ask for assistance with tasks that place their safety at great risk. The need to cope with loss is much greater in the older population. The majority of older persons cope with the death of a spouse. Some must cope with the death of adult children and grandchildren. All experience the death of friends. The ways that older persons adjust to the changes of aging are highly individualized. Many older persons welcome retirement as a time to pursue new interests and hobbies, participate in N R edu INcati GT B.CtartM a new business career. volunteer activities, continue thUeir S on, or sO DIF: Understand REF: 415 | 419 OBJ: Discuss common developmental tasks of older persons. MSC: CPNRE: Foundations of Practice
TOP: Assessment
12. An 80-year-old man is brought to the emergency department with an exacerbation of chronic
obstructive pulmonary disease (COPD). He states that he quit smoking 30 years ago, so it cannot be COPD. He argues, “It’s just these colds I’ve been getting. They’re just getting worse and worse.” The nurse understands which of the following? a. These symptoms are more associated with normal aging than with disease. b. Older persons should be encouraged to maintain physical exercise and activity. c. The patient’s age will require adjustment of lifestyle to one of inactivity. d. Older persons usually are aware and accepting of the aging process. ANS: B
Older persons should be encouraged to maintain physical exercise and activity, as tolerated. The primary benefits of exercise include maintaining and strengthening functional ability and promoting a sense of enhanced well-being. The COPD exacerbation is associated with the disease, not normal aging. The presence of disease sometimes alters the timing of the changes or their effect on daily life. Acceptance of personal aging does not mean retreat into inactivity, but it does require a realistic review of strengths and limitations. Some older persons find it difficult to accept that they are aging.
Canadian Fundamentals of Nursing 6th Edition Potter Test Bank DIF: Apply REF: 424 OBJ: Describe common physiological changes associated with aging. TOP: Planning MSC: CPNRE: Foundations of Practice 13. During assessment of an older person’s skin integrity, expected findings include which of the
following? a. Decreased elasticity. b. Oily skin. c. Increased facial hair in men. d. Faster nail growth. ANS: A
Loss of skin elasticity is a common finding in the older person. Other common findings include pigmentation changes, glandular atrophy (oil, moisture, sweat glands), thinning hair (facial hair: decreased in men, increased in women), slower nail growth, and atrophy of epidermal arterioles. DIF: Remember REF: 416, Table 24-1 OBJ: Describe common physiological changes associated with aging. TOP: Assessment MSC: CPNRE: Foundations of Practice 14. An older patient in no acute distress reports being less able to taste and smell. What is the
nurse’s best response to this information? a. Notify the physician immediately to rule out cranial nerve damage. b. Perform testing on the vestibulocochlear nerve and a hearing test. c. Schedule the patient for an appointment at a smell and taste disorders clinic. d. Explain to the patient that diminished senses are normal in older persons. ANS: D
NURSINGTB.COM
Diminished taste and smell senses are common findings in older persons. Scheduling an appointment at a smell and taste disorders clinic, testing the vestibulocochlear nerve, or an attempt to rule out cranial nerve damage is unnecessary at this time per the information provided. DIF: Apply REF: 416, Table 24-1 OBJ: Describe common physiological changes associated with aging. TOP: Assessment MSC: CPNRE: Foundations of Practice 15. Which symptom is an expected cognitive change in the older person? a. Disorientation. b. Slower reaction time. c. Poor judgement. d. Loss of language skills. ANS: B
Slower reaction time is a common change in older persons as a result of degeneration of nerve cells, decreased numbers of neurotransmitters, and decreased rate of conduction of impulses. Symptoms of cognitive impairment, such as disorientation, loss of language skills, loss of the ability to calculate, and poor judgement are not normal aging changes and necessitate further investigation of underlying causes. DIF: Understand
REF: 416, Table 24-1
Canadian Fundamentals of Nursing 6th Edition Potter Test Bank OBJ: Describe common physiological changes associated with aging. TOP: Assessment MSC: CPNRE: Foundations of Practice 16. A patient with gradual, progressive cognitive impairment (dementia) is admitted to the
nursing unit after hip replacement surgery. Which of the following is a nursing care principle for care of cognitively impaired older persons? a. Maintain physical health. b. Evaluate the patient’s manifestations of standard symptoms. c. Assist the patient with all ADLs. d. Isolate the patient to protect others. ANS: A
The nurse works to monitor and maintain physical health. The nurse should also assess the person’s unique manifestations of the disease as it progresses while facilitating independent performance of ADLs. Social interaction based on the patient’s abilities is to be promoted. DIF: Apply REF: 418 | 419 OBJ: Identify nursing interventions related to the physiological, cognitive, and psychosocial changes of aging. TOP: Planning MSC: CPNRE: Foundations of Practice 17. To promote physical well-being and socialization in an older person, what should the nurse
realize? a. Social isolationism is always a chosen behaviour. b. Body image plays no role in decision making by the older person. c. No community resources are focused on the older person. d. Older persons may have a functional purpose in social arenas. ANS: D
NURSINGTB.COM
Social service agencies in most communities welcome older persons as volunteers and provide the opportunity for older persons to serve while meeting their socialization or other needs. Although some older persons choose isolation or a lifelong pattern of reduced interaction with others, other older persons do not choose isolation but are vulnerable to its consequences. Some older persons withdraw from social interaction because of feelings of rejection. These older persons see themselves as unattractive and rejected because of changes in their personal appearance resulting from normal aging or because of body image changes. Many communities have outreach programs designed to make contact with isolated older persons. DIF: Understand REF: 420 OBJ: Identify nursing interventions related to the physiological, cognitive, and psychosocial changes of aging. TOP: Implementation MSC: CPNRE: Foundations of Practice 18. A male older patient expresses his concern and anxiety about decreased penile firmness
during erection. What is the nurse’s best response? a. Explaining that over time, his libido will decrease, as will the frequency of sexual activity. b. Telling the patient to double his antidepressant medication to increase his libido. c. Telling the patient that this change is expected in aging persons. d. Telling the patient that touching should be avoided unless intercourse is planned. ANS: C
Canadian Fundamentals of Nursing 6th Edition Potter Test Bank Decreased firmness during erection is an expected change in aging persons. Libido does not necessarily decrease as one ages. Many older persons use prescription medications that depress sexual activity, such as antihypertensives, antidepressants, sedatives, and hypnotics. Touch complements traditional sexual methods or serves as an alternative sexual expression when physical intercourse is not desired or possible. DIF: Apply REF: 420 OBJ: Identify nursing interventions related to the physiological, cognitive, and psychosocial changes of aging. TOP: Implementation MSC: CPNRE: Foundations of Practice 19. A patient asks the nurse what the term polypharmacy means. The nurse defines this term as
which of the following? a. Multiple side effects experienced when a medication is taken. b. The concurrent use of many medications. c. The many adverse drug effects reported to the pharmacy. d. The risks of medication effects that are due to aging. ANS: B
Polypharmacy refers to the concurrent use of many medications. It does not have anything to do with side effects, adverse drug effects, or risks of medication use that are due to aging. DIF: Remember REF: 426 TOP: Implementation
OBJ: Describe selected health concerns of older persons. MSC: CPNRE: Foundations of Practice
20. An outcome for an older patient living alone is to be free from falls. Which of these
statements by a patient indicates that teaching on safety concerns has been effective? a. “I’ll leave my throw rugsNinUpRlaScI .yCfOeeMt won’t touch the cold tile.” eN soGthTaB tm b. “I’ll take my time getting up from the bed or chair.” c. “I should wear my favourite smooth bottom socks to protect my feet when walking around.” d. “I will have my son dim the lighting outside to decrease the glare in my eyes.” ANS: B
Older persons taking medications with adverse effects such as postural hypotension, dizziness, or sedation need to be aware of these potential effects and to take precautions such as changing position slowly or ambulating with assistance if unsteady. Household items that are easy to trip over, such as throw rugs, are a risk factor for falls. Other risk factors include wearing shoes in poor repair or slippery soles. Impaired vision and poor lighting are other risk factors. DIF: Evaluate REF: 425 TOP: Implementation
OBJ: Describe selected health concerns of older persons. MSC: CPNRE: Foundations of Practice
21. One of the greatest challenges for the nurse caring for older persons is ensuring safe
medication use. What is one way to reduce the risks associated with medication usage? a. Periodically reviewing the patient’s list of medications. b. Informing the patient that polypharmacy is to be avoided at all cost. c. Being aware that medication is absorbed the same way regardless of patient age. d. Focusing only on prescribed medications. ANS: A
Canadian Fundamentals of Nursing 6th Edition Potter Test Bank Periodic and thorough review of all medications is important: the number of medications used should be the fewest necessary to ensure the greatest therapeutic benefit with the least amount of harm. Although polypharmacy reflects inappropriate prescribing, the concurrent use of multiple medications is necessary in situations in which an older person has multiple acute and chronic conditions. Older persons are at risk for adverse drug effects because of age-related changes in the absorption, distribution, metabolism, and excretion of drugs. Nurses should work collaboratively with older persons to ensure safe and appropriate use of all medications, both prescribed medications and over-the-counter medications. DIF: Apply TOP: Assessment
REF: 426 OBJ: Describe selected health concerns of older persons. MSC: CPNRE: Foundations of Practice
22. An older patient has developed acute confusion. The patient has been taking tranquilizers for
the past week. The patient’s vital signs are normal. What should the nurse do? a. Take into account age-related changes in body systems that affect pharmacokinetic activity. b. Increase the dose of tranquilizer if the cause of the confusion is an infection. c. Note when the confusion occurs and medicate before that time. d. Restrict telephone usage to prevent further confusion. ANS: A
Sedatives and tranquilizers sometimes prescribed for acutely confused older persons sometimes cause or exacerbate confusion. Drugs used to manage confused behaviours should be administered carefully and must account for age-related changes in body systems that affect pharmacokinetic activity. When confusion has a physiological cause (such as an infection), the nurse should specifically treat that cause, rather than the confused behaviour. When confusion varies by time of day or is related to environmental factors, N R hIasNma GTking B.C M nonpharmacological measures,UsucS thO e environment more meaningful and providing adequate light, should be used. Making telephone calls to friends or family members allows older persons to hear reassuring voices, which may be beneficial. DIF: Apply REF: 426 TOP: Implementation
OBJ: Describe selected health concerns of older persons. MSC: CPNRE: Foundations of Practice
23. Which of these assessments of an older adult, who has a urinary tract infection, necessitates an
immediate nursing intervention? a. Presbycusis. b. Confusion. c. Death of a spouse 3 months ago. d. Temperature of 36.4°C (97.6°F). ANS: B
Confusion is a common manifestation in older persons with urinary tract infection; however, the cause requires further assessment. There may be another reason for the confusion. Confusion is not a normal finding in older persons, even though it is common with concurrent infections. Difficulty hearing (presbycusis) is an expected finding in older persons. Coping with the death of a spouse is a psychosocial concern to be addressed after the acute physiological concern in this case. Older persons tend to have lower temperatures, so the nurse needs to assess for slight elevations. A temperature of 36.4°C (97.6°F) is within normal limits.
Canadian Fundamentals of Nursing 6th Edition Potter Test Bank DIF: Apply REF: 415 OBJ: Identify nursing interventions related to the physiological, cognitive, and psychosocial changes of aging. TOP: Planning MSC: CPNRE: Foundations of Practice 24. Which of these patient statements is the most reliable indicator that an older person has the
correct understanding of health promotion activities? a. “I need to increase my fat intake and limit protein.” b. “I should discontinue my fitness club membership for safety reasons.” c. “I’m up to date on my immunizations, but at my age, I don’t need the tetanus vaccine.” d. “I still keep my dentist appointments even though I have partials now.” ANS: D
General preventive measures for the nurse to recommend to older persons include keeping periodic dental appointments to promote good oral hygiene; eating a low-fat, well-balanced diet; exercising regularly; and maintaining immunizations for influenza, pneumococcal pneumonia, and tetanus. DIF: Evaluate REF: 422, Box 24-8 OBJ: Describe selected health concerns of older persons. MSC: CPNRE: Foundations of Practice
TOP: Evaluate
25. A 72-year-old woman was recently widowed. She worked as a teller at a bank for 40 years
and has been retired for the past 5 years. She never learned how to drive. She lives in a rural area that does not have public transportation. Which of the following psychosocial changes does the nurse focus on as a priority? a. Sexuality. b. Housing and environmenN t. URSINGTB.COM c. Retirement. d. Social isolation. ANS: D
The highest priority at this time is the potential for social isolation. This woman does not know how to drive and lives in a rural community that does not have public transportation. All of these factors contribute to her social isolation. Other possible changes she may be going through right now include sexuality related to her advanced age and recent death of her spouse; however, this is not the priority at this time. She has been retired for 5 years, so this is also not an immediate concern. She may eventually experience needs related to housing and environment, but the question does not indicate this as an issue at this time. DIF: Analyze REF: 419 | 420 OBJ: Describe the concepts of aging well and quality of life. MSC: CPNRE: Foundations of Practice
TOP: Assessment
26. A recently widowed 80-year-old man is dehydrated and is admitted to the hospital for
intravenous fluid replacement. During the evening shift, the patient becomes acutely confused, and the student nurse plans to assess the patient for a reversible cause. The nursing instructor will need to provide further instruction to the student who plans to assess for which of the following? a. Electrolyte imbalance. b. Hypoglycemia.
Canadian Fundamentals of Nursing 6th Edition Potter Test Bank c. Drug effects. d. Dementia. ANS: D
Delirium, or acute confusional state, is a potentially reversible cognitive impairment that is often due to a physiological event. Physiological causes of delirium can include electrolyte imbalances, cerebral anoxia, hypoglycemia, medications, drug effects, tumours, subdural hematomas, and cerebrovascular infection, infarction, or hemorrhage. Unlike delirium, dementia is a gradual, progressive, irreversible cerebral dysfunction. DIF: Apply REF: 418 OBJ: Differentiate among delirium, dementia, and depression. MSC: CPNRE: Foundations of Practice
NURSINGTB.COM
TOP: Implementation
Canadian Fundamentals of Nursing 6th Edition Potter Test Bank
Chapter 25: The Experience of Loss, Death, and Grief Potter et al: Canadian Fundamentals of Nursing, 6th Edition MULTIPLE CHOICE 1. A nurse encounters a family whose adult child died last year. The parents are talking about the
upcoming anniversary of their child’s death. The nurse spends time with them discussing their child’s life and death. The nurse’s action best demonstrates which nursing principle? a. Pain management technique. b. Facilitating normal mourning. c. Grief evaluation. d. Palliative care. ANS: B
Anniversaries can reopen grief processes. A nurse should openly acknowledge the loss and talk about the common renewal of grief around the anniversary of the individual’s death. This facilitates normal mourning. The nurse is not attempting to alleviate a physical pain. The actions are of open communication, not evaluation. Palliative care refers to comfort measures for symptom relief. DIF: Apply REF: 447 OBJ: Identify your role in assisting patients who have experienced loss, death, and grief. TOP: Implementation MSC: CPNRE: Foundations of Practice 2. A patient with cancer asks the nurse when he or she will be able to access palliative care. On
the basis of the knowledge about palliative care, what will the nurse’s response be? Ne th RS I G a B.C M a. It is only available for thosU at haNve T terminOal illness. b. It can help patients achieve optimal pain management. c. It is offered when patients have less than 6 to 12 more months. d. It is available if indicated in an advance directive. ANS: B
Palliative care can help patients achieve optimal pain management. It is for any age, any diagnosis, and at any time, not just during the last few months of life. Hospice care is more appropriate when the patient is expected to live less than 6 to 12 more months. An advance directive can include information about palliative care but is not required for such care to be initiated. DIF: Remember REF: 447 | 448 TOP: Implementation
OBJ: Discuss principles of palliative care. MSC: CPNRE: Foundations of Practice
3. A terminally ill patient is experiencing constipation secondary to pain medication. What is the
best way for the nurse to improve the patient’s constipation problem? a. Massage the patient’s abdomen. b. Contact the provider to discontinue pain medication. c. Administer enemas twice daily for 7 days. d. Use a stimulant laxative and increase fluid intake. ANS: D
Canadian Fundamentals of Nursing 6th Edition Potter Test Bank Opioid medication is known to slow gastrointestinal transit time, which places the patient at high risk for constipation. Stimulant laxatives are indicated for opioid-induced constipation. Added water to the diet will allow water to be pulled into the gastrointestinal tract, which helps soften stool. Massaging the patient’s abdomen may cause further discomfort. Discontinuing pain medication is inappropriate for a terminally ill patient. Enema administration is not the first step in the treatment of opioid-induced constipation. DIF: Apply REF: 449 TOP: Implementation
OBJ: Discuss principles of palliative care. MSC: CPNRE: Foundations of Practice
4. A severely depressed patient cannot state any positive attributes to his or her life. The nurse
sits with this patient and helps identify several activities that the patient is actually looking forward to in life. The nurse is helping the patient demonstrate which spiritual concept? a. Time management. b. Hope. c. Charity. d. Faith. ANS: B
The concept of hope is vital to human beings; it enables a person to anticipate positive experiences. Being patient and friendly and creating positive relationships are key concepts in all areas of nursing, but especially with depressed patients. The nurse’s actions do not address time management, charity, or faith. DIF: Understand REF: 437 OBJ: Develop a nursing care plan for a patient or family experiencing loss and grief. TOP: Implementation MSC: CPNRE: Foundations of Practice
NURSINGTB.COM
5. The nurse is discussing future treatments with a patient who has a terminal illness. The nurse
notes that the patient has not been eating and responds to the nurse’s information by stating, “What does it matter?” What is the most appropriate nursing diagnosis for this patient? a. Social isolation. b. Hopelessness. c. Denial. d. Powerlessness. ANS: D
A defining characteristic for the nursing diagnosis of Powerlessness may include a statement by the patient such as “What does it matter?” when offered choices or information concerning him or her. The patient’s behaviour and verbalization is not an example of social isolation. The patient is not avoiding others or being restricted from seeing others. Hopelessness is more reflected by the comment “I have no future” than “What does it matter?” The patient’s behaviour and verbalization do not indicate denial. DIF: Apply REF: 442, Box 25-7 OBJ: Develop a nursing care plan for a patient or family experiencing loss and grief. TOP: Diagnosis MSC: CPNRE: Foundations of Practice
Canadian Fundamentals of Nursing 6th Edition Potter Test Bank 6. Family members gather in the emergency department after learning that a family member was
involved in a motor vehicle accident. After learning of the family member’s unexpected death, the surviving family members begin to cry and scream in despair. The nurse recognizes this as which stage of the Bowlby Attachment Theory? a. Numbing. b. Disorganization and despair. c. Bargaining. d. Yearning and searching. ANS: D
Yearning and searching characterize the second bereavement phase in Bowlby’s Attachment Theory. Emotional outbursts are common in this phase. During the numbing phase, the family may feel a sense of unreality. During disorganization and despair, the reason why the loss occurred is constantly questioned. Bargaining is part of the Kübler-Ross stages, not of the Bowlby Attachment Theory. DIF: Apply REF: 434 OBJ: Describe and compare the phases of grieving from Kübler-Ross (1969), Bowlby (1980), and Worden (1991). TOP: Implementation MSC: CPNRE: Foundations of Practice 7. After the anticipated demise of a chronically ill patient, the unit nurse is found crying in the
staff lounge. What would be the best response to her crying? a. “It is normal to feel this way. Give yourself some time to mourn.” b. “Your other patients still need you, so hurry back to them.” c. “You’re being a bad role model to the unit’s nursing students.” d. “Why don’t you take a sedative to cope?” ANS: A
NURSINGTB.COM
Nurses often witness suffering on a daily basis. Nurses, as humans, also experience grief and loss when they have been intensely involved in the patient’s suffering and death. Their colleagues should offer comfort and understanding and maintain a stable patient care environment. It is inappropriate to create guilt by telling a grieving nurse to hurry back to his or her patients or by indicating that he or she is a bad role model. Suggesting that a colleague take sedative during a shift is dangerous for the safety of patients in his or her care. DIF: Apply REF: 453 OBJ: Discuss your own experience of loss when caring for dying patients. TOP: Implementation MSC: CPNRE: Foundations of Practice 8. A family is grieving after learning of a family member’s accidental death. The transplant
coordinator requests to talk with the family about possible organ and tissue donation. The nurse recognizes which of the following? a. All religions allow for organ donation. b. Life support must be removed before organ and tissue retrieval occurs. c. The best time for organ and tissue donation is immediately after the autopsy. d. The transplant coordinator will need a private place to talk with the family. ANS: D
Canadian Fundamentals of Nursing 6th Edition Potter Test Bank The transplant coordinator should always meet with family members in a private, quiet area. Not all religions allow for organ donation. A patient may be on life support during organ removal to preserve organ tissues. Autopsy compromises organ integrity; removal should occur before that. DIF: Apply REF: 452 OBJ: Describe the procedure for care of the body after death. MSC: CPNRE: Professional, Ethical, and Legal Practice
TOP: Implementation
9. The newly graduated nurse is assigned to his or her first dying patient. How can the nurse best
prepare to care for this patient? a. Completing a course dealing with death and dying. b. Controlling his or her own emotions about death. c. Drawing on the experience of the death of a loved one. d. Developing an understanding of his or her own feelings about death. ANS: D
When caring for patients experiencing grief, it is important for the nurse to assess his or her own emotional well-being and to understand his or her own feelings about death. The nurse who is aware of his or her own feelings will be less likely to place personal situations and values before those of the patient. Although coursework on death and dying may add to the nurse’s knowledge base, it does not best prepare the nurse for caring for a dying patient. The death of a patient can raise many emotions. Being able to control one’s own emotions is important; however, it is unlikely that the nurse would be able to do so if he or she has not first acknowledged his or her own feelings about death. The death of a loved one is not a prerequisite to caring for a dying patient. The experience of caring for a dying patient may help a nurse mature in dealing with loss, or it may bring up many negative emotions if N R I G B.C M O complicated grief is present. U S N T DIF: Apply REF: 453 OBJ: Discuss your own experience of loss when caring for dying patients. TOP: Implementation MSC: CPNRE: Professional, Ethical, and Legal Practice 10. The palliative team’s primary obligation to a patient in severe pain includes which of the
following? a. Supporting the patient’s nurse in her grief. b. Providing post-mortem care for the patient. c. Teaching the patient the stages of grief. d. Enhancing the patient’s quality of life. ANS: D
The primary goal of palliative care is to help patients and families achieve the best quality of life. Providing support for the patient’s nurse is not the primary obligation when the patient is experiencing severe pain. Not all collaborative team members would be able to provide post-mortem care; nor would nutritionists, social workers, or pharmacists. Teaching about stages of grief should not be the focus when a patient is in severe pain. DIF: Understand REF: 447 | 448 TOP: Implementation
OBJ: Discuss principles of palliative care. MSC: CPNRE: Foundations of Practice
Canadian Fundamentals of Nursing 6th Edition Potter Test Bank 11. A man is hospitalized after surgery in which both lower extremities are amputated because of
injuries sustained during military service. The nurse should recognize his need to grieve for what type of loss? a. Maturational loss. b. Situational loss. c. Perceived loss. d. Uncomplicated loss. ANS: B
Loss of a body part from injury is a situational loss. Maturational losses occur as part of normal life transitions. Perceived loss is not obvious to other people. Uncomplicated is not a type of loss; it is a description of normal grief. DIF: Understand REF: 433 TOP: Implementation
OBJ: List and discuss the five categories of loss. MSC: CPNRE: Foundations of Practice
12. “I know it seems strange, but I feel guilty being pregnant after the death of my son last year,”
said a woman during her routine obstetrical examination. The nurse spends extra time with this woman, helping her to better bond with her unborn child. This demonstrates which nursing technique? a. Facilitating mourning. b. Providing curative therapy. c. Promoting spirituality. d. Eradicating grief. ANS: A
The nurse facilitates mourning in family members who are still surviving. By acknowledging the pregnant woman’s emotioNnU s,RthSeInN urG seThB el. psCtO heMmother bond with her fetus and recognize the emotions that still exist for the deceased child. The nurse is not attempting to help the patient eradicate grief, which would be unrealistic. Curative therapy and spiritual promotion are not addressed by the nurse’s statement. DIF: Understand REF: 434 TOP: Implementation
OBJ: List and discuss the five categories of loss. MSC: CPNRE: Professional, Ethical, and Legal Practice
13. Three of the nurse’s patients have died during the past 2 days. Which approach is most
appropriate to manage the nurse’s sadness? a. Telling the next patients why the nurse is sad. b. Talking with a colleague or writing in a journal. c. Exercising vigorously rather than sleeping. d. Avoiding friends until the nurse feels better. ANS: B
Self-care strategies for nurses include talking with a close colleague and reflecting on feelings by writing in a journal. It is inappropriate for a nurse to talk with patients to resolve the nurse’s grief. Although exercise is important for self-care, sleep is also important. Shutting oneself away from friends is not self-care; the nurse should spend time with people who are nurturing. DIF: Understand REF: 453 OBJ: Discuss your own experience of loss when caring for dying patients. TOP: Implementation MSC: CPNRE: Professional, Ethical, and Legal Practice
Canadian Fundamentals of Nursing 6th Edition Potter Test Bank
14. A woman is called into her supervisor’s office regarding her deteriorating work performance
since the loss of her husband 2 years ago. The woman begins sobbing and saying that she is “falling apart” at home as well. The woman is escorted to the nurse’s office, where the nurse recognizes the woman’s symptoms as which of the following? a. Normal grief. b. Complicated grief. c. Disenfranchised grief. d. Perceived grief. ANS: B
An individual undergoing a complicated grieving process that interferes with common routines of life for excessively long periods of time is experiencing complicated (dysfunctional) grief. Normal grief is the most common reaction to death; it involves a complex range of normal coping strategies. Disenfranchised grief involves a relationship that is not socially sanctioned. Perceived grief is not a type of grief; perceived loss is a loss that is not obvious to other people. DIF: Understand TOP: Assessment
REF: 434 | 435 OBJ: Describe the types of grief. MSC: CPNRE: Professional, Ethical, and Legal Practice
15. The father has recently begun to attend his children’s school functions since the death of his
wife. This would best be described as which task in the Worden’s grief tasks model? a. Task I. b. Task II. c. Task III. d. Task IV. ANS: C
NURSINGTB.COM
The Worden grief tasks model consists of four tasks. In Task III, the surviving family member begins to adjust to life without the deceased. Task I is accepting the reality of the loss, Task II is working through the pain of grief, and Task IV is emotionally relocating the deceased and moving on with life. DIF: Understand REF: 434 OBJ: Describe and compare the phases of grieving from Kübler-Ross (1969), Bowlby (1980), and Worden (1991). TOP: Assessment MSC: CPNRE: Foundations of Practice 16. An identified outcome for the family of the patient with a terminal illness is that family
members will be able to provide psychological support to the dying patient. To help the family to meet this outcome, the nurse will develop a teaching plan that will include teaching about which of the following? a. Bathing techniques. b. Application of oxygen devices. c. Recognition of patient needs and fears. d. Hospice care. ANS: C
Canadian Fundamentals of Nursing 6th Edition Potter Test Bank A dying patient’s family is better prepared to provide psychological support if the nurse discusses with them ways to listen and respond to needs and fears of the dying person. Demonstration of bathing techniques and application of oxygen devices may help the family meet the patient’s physical needs but would not provide psychological support for the patient. Information on when to contact the hospice nurse is important for the family to have and may help them feel they are being supported in caring for the dying patient. However, contact information does not help the family provide psychological support to the dying patient. DIF: Apply REF: 450, Box 25-9 OBJ: Describe how to involve family members in palliative care. TOP: Planning MSC: CPNRE: Foundations of Practice 17. Validation of a dying person’s life would be demonstrated by which nursing action? a. Taking pictures of visitors. b. Calling the organ donation coordinator. c. Listening to family stories about the person. d. Providing quiet visiting time. ANS: C
Listening to family members’ stories validates the importance of the dying patient’s life and reinforces the patient’s dignity. Taking pictures of visitors does not address the value of a person’s life. Calling an organ donation coordinator and providing private visiting time are components of the dying process, but they do not validate a dying person’s life. DIF: Apply REF: 450 OBJ: Develop a nursing care plan for a patient or family experiencing loss and grief. TOP: Implementation MSC: CPNRE: Foundations of Practice
NURSINGTB.COM
18. A couple is informed that their fetus’s condition is incompatible with life after birth. Nurses
can best help the couple with their end-of-life decision making by offering them which of the following? a. An advance directive to complete. b. Brief discussion and funeral guidance. c. Time and careful explanations. d. Instructions on how to proceed. ANS: C
Families can have limited knowledge when asked to make important ethical decisions. Nurses have the time, patience, and knowledge base to help the family understand their ethical situation and to help them make their own educated decision, rather than giving them instructions. Advance directives are completed by the person who is dying. Funeral guidance is best provided by a chaplain or a caretaker. DIF: Apply REF: 439-441 OBJ: Develop a nursing care plan for a patient or family experiencing loss and grief. TOP: Implementation MSC: CPNRE: Professional, Ethical, and Legal Practice 19. A correctional facility nurse is called to the scene of an inmate who died under suspect
circumstances. The correction officer wants to move the body to the funeral home quickly because he is not comfortable with death. The inmate’s body will need to be transported where?
Canadian Fundamentals of Nursing 6th Edition Potter Test Bank a. b. c. d.
Coroner’s office for an autopsy. Police department for an investigation. Directly to the inmate’s family. Warden for inspection.
ANS: A
Physicians often request that an autopsy be performed if death occurred under unusual circumstances; as the result of foul play, homicide, or suicide; or as an accidental death, as occurs in car accidents. The nurse must understand the policies that are applied in cases of foul play death and must ensure that the decedent’s body is properly cared for after death, despite the emotional feelings of individuals in close contact with the decedent. DIF: Apply REF: 451, Box 25-10 OBJ: Describe the procedure for care of the body after death. MSC: CPNRE: Foundations of Practice
TOP: Implementation
20. When teaching a group of older persons about cultural beliefs and death, the nurse indicates
which of the following? a. The ethical decisions surrounding a patient’s death should be based on hospital policy and not culture. b. Maintaining rituals and practices allows a sense of acceptance of the dying process. c. The nurse must decide which cultural practices will be incorporated in care of the dying. d. Regardless of culture, following hospital practices will help focus patient and family on the dying process. ANS: B
NURSINGTB.COM
Maintaining the integrity of rituals and mourning practices gives families a sense of acceptance of the patient’s death and promotes inner peace. The nurse should be familiar with policies and procedures, but ethical decisions should be made with an understanding and appreciation of the patient’s culture. The nurse must assess the terminally ill patient’s and family’s wishes for end-of-life care and develop a plan of care that integrates patient culture and spiritual beliefs with nursing actions. DIF: Apply REF: 436, Box 25-2 OBJ: Identify your role in assisting patients who have experienced loss, death, and grief. TOP: Implementation MSC: CPNRE: Foundations of Practice 21. The nurse is preparing to assist the patient in the end stage of her life. The patient is showing
signs of fatigue. What should the nurse do to provide comfort for this patient? a. Spend more time with the patient. b. Limit the use of analgesics. c. Provide larger meals with more seasoning. d. Determine valued activities, and schedule rest periods. ANS: D
Canadian Fundamentals of Nursing 6th Edition Potter Test Bank To promote comfort in the terminally ill patient, the nurse should help the patient identify values or desired tasks and then help the patient to conserve energy for those tasks. Spending more time with the patient conveys caring and allows verbalization, but it is not the best way to promote comfort for a fatigued patient. The use of analgesics should not be limited. Controlling the terminally ill patient’s level of pain is a primary concern in promoting comfort. Nausea, vomiting, and anorexia may increase the terminally ill patient’s likelihood of inadequate nutrition. The nurse should serve smaller portions and bland foods, which may be more palatable. DIF: Apply REF: 447 | 448 OBJ: Explain reasons for the need for improved end-of-life care for patients. TOP: Implementation MSC: CPNRE: Professional, Ethical, and Legal Practice 22. During a follow-up visit, a woman is describing new onset of marital discord with her
terminally ill spouse. Using the Kübler-Ross behavioural theory, the nurse recognizes that the spouse is in which stage of grief? a. Denial. b. Bargaining. c. Anger. d. Depression. ANS: C
Kübler-Ross’ traditional theory involves five stages of grief. The anger stage of adjustment to an impending death can involve resistance, anger at a deity, anger at people, and anger at the situation. Denial involves failure to accept a death. Bargaining is an action to delay acceptance of the inevitability of death. Depression would manifest as withdrawal from others. DIF: Apply REF: 43N 4URSINGTB.COM OBJ: Describe and compare the phases of grieving from Kübler-Ross (1969), Bowlby (1980), and Worden (1991). TOP: Implementation MSC: CPNRE: Foundations of Practice 23. Mrs. Harrison’s father died a week ago. Mr. Harrison is experiencing headaches and fatigue
and keeps shouting at his wife to turn down the television, although he has not done so in the past. Mrs. Harrison is having trouble sleeping and has no appetite. How should the nurse interpret these assessment findings as the basis for a follow-up assessment? a. Mrs. Harrison is grieving and Mr. Harrison is angry. b. Mrs. Harrison is ill and Mr. Harrison is grieving. c. Both Mr. and Mrs. Harrison are probably in denial. d. Both Mr. and Mrs. Harrison are probably grieving. ANS: D
Each individual responds to loss differently. Some people may demonstrate anger or denial as a response to grief; Mr. Harrison’s anger is most likely a manifestation of grief. Common physical symptoms of grief include tightness in the chest and throat, shortness of breath, feelings of weakness and lethargy, insomnia, and loss of appetite. Denial is the situation when the person indicates that he or she is not accepting that the loss happened. DIF: Apply REF: 433-435 OBJ: Describe the characteristics of a person experiencing grief. TOP: Assessment MSC: CPNRE: Foundations of Practice
Canadian Fundamentals of Nursing 6th Edition Potter Test Bank
Chapter 26: Self-Concept Potter et al: Canadian Fundamentals of Nursing, 6th Edition MULTIPLE CHOICE 1. The nursing student can help geriatric patients’ self-concept by using which technique? a. Discussing current weather. b. Reviewing old photos with patients. c. Encouraging patients to sing. d. Allowing patients extra computer time. ANS: B
Nurses can improve older patients’ self-image by reviewing old photographs with them. This form of life review is helpful to older persons in remembering positive life events and people. Discussing weather does not involve personal reflection. Singing improves global cognition, not self-concept. Giving patients extra computer time is not applicable to improving self-concept. DIF: Apply REF: 465, Box 26-4 OBJ: Apply the nursing process to promote a client’s self-concept. TOP: Implementation MSC: CPNRE: Foundations of Practice 2. While documenting an adolescent’s health history, the nurse recognizes that the patient began
to act out behaviourally and engage in risky behaviour when her parents divorced. In considering a nursing diagnosis of Altered self-concept, the nurse would gather what information? N R I G B.C M a. How long the parents wereU O maS rriedN. T b. How the patient views her behaviours. c. Why the parents are divorcing. d. Why she is acting out of control. ANS: B
A nurse can identify situational life stressors that can affect a person’s self-concept. By openly exploring a patient’s thoughts and feelings, the nurse will be able to use communication skills in a therapeutic manner. This facilitates the patient’s insight into behaviours and enables the nurse to make referrals or provide needed health teaching. The length of time the parents were married and the reason for the parents’ divorce do not explain the patient’s behaviours. Why the patient is out of control is not as important as how the patient views her actions when out of control. DIF: Evaluate REF: 461 | 462 OBJ: Apply the nursing process to promote a client’s self-concept. TOP: Implementation MSC: CPNRE: Foundations of Practice 3. The developmental self-concept task known as initiative versus guilt would occur in which
person? a. A 3-week-old neonate. b. A 5-year-old kindergarten student. c. An 11-year-old student. d. A 15-year-old high school student.
Canadian Fundamentals of Nursing 6th Edition Potter Test Bank
ANS: B
The initiative versus guilt developmental stage occurs between the ages of 3 and 6 years. If a child shows initiative, the outcome of this developmental task is to develop purpose. A neonate developmental task is to develop trust. An 11-year-old is in the stage of new skill mastery, and a 15-year-old is struggling with identity versus role confusion. DIF: Remember REF: 459, Box 26-1 OBJ: Describe the components of self-concept as related to psychosocial and cognitive developmental stages. TOP: Assessment MSC: CPNRE: Foundations of Practice 4. A verbally abusive partner has made many negative comments to his significant other, the
patient, over the years. In the crisis centre, the nurse would anticipate that the patient may have which of the following self-concept deficits? a. Negative body image. b. Role confusion. c. Rigidity. d. Yearning. ANS: A
Consistent negative comments can cause devaluation of an individual’s self-concept. The effect of negative self-esteem can invade all areas of a person’s life, including body image. Role confusion is part of a developmental task (identity versus role confusion). Rigidity and yearning are not components of self-concept. DIF: Remember REF: 460 OBJ: Discuss factors that influN encR e theIfolG lowiB ng.cC ompMonents of self-concept: identity, body image, U S N TTOP: AsOsessment and role performance. MSC: CPNRE: Foundations of Practice 5. Two 50-year-old men are discussing their Saturday activities. The first man describes tutoring
children as a volunteer at a community centre. The other man says that he would never work with children and that he prefers to work out at the gym to meet young women to date. The second man’s statement reflects which developmental stage? a. Inferiority. b. Role confusion. c. Self-absorption. d. Mistrust. ANS: C
In the generativity versus self-absorption developmental task, a self-absorbed person is concerned about his or her own personal wants and desires in a self-centred manner. Mistrust versus trust is encountered in the first year of life. Industry versus inferiority commonly occurs in school-aged children. Identity versus role confusion commonly occurs at the start of adolescence into young adulthood. DIF: Understand REF: 459 OBJ: Describe the components of self-concept as related to psychosocial and cognitive developmental stages. TOP: Assessment MSC: CPNRE: Foundations of Practice
Canadian Fundamentals of Nursing 6th Edition Potter Test Bank 6. An adult is adjusting to the idea that his chronically ill parent will move into the family home.
The community health nurse would assess the adult son for which potential stressor secondary to the new family living arrangement? a. Role confusion. b. Role ambiguity. c. Role performance. d. Role overload. ANS: D
Role overload involves having more roles, or more responsibilities within a role, than are manageable. Role overload is common in individuals who unsuccessfully attempt to meet the demands of work and family while trying to find some personal time. Role confusion is an aspect of the developmental task of adolescence and young adulthood (identity versus role confusion). Role ambiguity involves unclear role expectations. Role performance itself is not a stressor unless it is judged ineffective. DIF: Understand REF: 463 OBJ: Identify stressors that affect self-concept and self-esteem. TOP: Assessment MSC: CPNRE: Foundations of Practice 7. A nurse grimaces when seeing a patient’s colostomy opening while changing the colostomy
bag. This expression is most likely to have what effect on the patient? a. Assisting recovery by using honest communication. b. Motivating the patient to increase physical activity. c. Promoting development of a negative body image. d. Developing a kind nickname for the colostomy opening. ANS: C
NURSINGTB.COM
Negative nonverbal reactions by a nurse to a patient’s scar or surgical alterations contribute to the patient’s developing a negative body image. Honest expressions of distaste by the nurse will not facilitate recovery or ongoing communication. Encouraging physical activity or adopting a positive nickname will not promote acceptance of the colostomy opening. DIF: Understand REF: 465 | 466 OBJ: Explore ways in which a nurse’s self-concept and nursing actions can affect a client’s self-concept and self-esteem. TOP: Implementation MSC: CPNRE: Foundations of Practice 8. “I’m such a loser. I only had that job for a month.” Identify appropriate nursing outcome
criteria for this individual. a. The patient will verbalize three life areas in which he or she functions well. b. The patient will find new employment before the next clinic visit. c. The patient will confront his or her former boss about work problems. d. The patient will identify why he or she is considered a bad employee. ANS: A
Canadian Fundamentals of Nursing 6th Edition Potter Test Bank Verbalizing three life areas in which a person functions well is an individualized measurable outcome that is realistic. Confronting a former boss could have physical and emotional repercussions for the patient. If the person is voicing concerns that he or she has problems obtaining employment, then putting extra pressure on the person to obtain employment would be detrimental to the patient and does not reflect a supportive and caring nursing outcome. Any implication that the patient is a bad employee is inappropriate. DIF: Analyze REF: 472, Box 26-11 OBJ: Apply the nursing process to promote a client’s self-concept. TOP: Planning MSC: CPNRE: Foundations of Practice 9. Children learn to live an authentic lifestyle through culturally accepted behaviours, values,
and role modelling. A child who does this is attempting to create what? a. His or her own body image. b. His or her own self-esteem. c. His or her own identity. d. His or her own role performance. ANS: C
Identity versus role confusion is a stage that occurs between the ages of 12 and 20 years. Body image and self-esteem are attitudes related to personal reflection and attitudes. Role performance is the result of creating an identity. DIF: Understand REF: 459 OBJ: Describe the components of self-concept as related to psychosocial and cognitive developmental stages. TOP: Planning MSC: CPNRE: Foundations of Practice
NURSINGTB.COM
10. Which of the following individuals is most likely to need the nurse’s assistance because of the
presence of identity confusion? a. A 49-year-old man with stable employment. b. A 35-year-old recently divorced mother of twins. c. A 22-year-old in the third year of college. d. A 50-year-old self-employed woman. ANS: B
Identity confusion can occur when people do not maintain a clear, consistent, and continuous consciousness of personal identity. A newly divorced woman would be trying to adapt to a new lifestyle of being single while being a single parent of twins. This situation could lead to identity confusion. A college sophomore would have had at least 2 years to adjust to the new life setting, and a self-employed woman would probably be content with creating her own employment opportunity. There is no indication that the middle-aged man with stable employment should have identity confusion. DIF: Analyze REF: 463 OBJ: Incorporate research findings to promote evidence-informed practice for identity confusion, disturbed body image, low self-esteem, and role conflict. TOP: Planning MSC: CPNRE: Foundations of Practice 11. Identify the assessments suggestive of an altered self-concept. a. Limping gait and large smile.
Canadian Fundamentals of Nursing 6th Edition Potter Test Bank b. Slumped posture and poor personal hygiene. c. Verbally responds when asked a question. d. Appropriately dressed with clean clothes. ANS: B
A self-concept is created by an individual’s identity, body image, and role performance. Poor personal hygiene and slumping posture best describe a person with an impaired self-concept. Smiling, appropriate responses, and appropriate appearance are all signs of normal self-concept. DIF: Understand REF: 467, Box 26-7 OBJ: Apply the nursing process to promote a client’s self-concept. TOP: Planning MSC: CPNRE: Foundations of Practice 12. A priority nursing intervention for a patient who has undergone a mastectomy is which of the
following? a. Using therapeutic silence to encourage the patient to talk. b. Using communication skills to clarify family and patient expectations. c. Telling her that the nurse knows she will do fine because many other women have. d. Rotating nursing personnel in the patient’s care, so the patient can talk to many people. ANS: B
The nurse recognizes the need to use therapeutic communication skills, allowing the patient the opportunity to talk openly about issues that are important to a person who has undergone body-altering surgery. When a nurse does not allow for the development of a patient–nurse therapeutic relationship, open and honest conversation is impossible. Silence can be useful, but building rapport and openNing onOaM re necessary first. Reassurance that a URthe SIcon NGver TBsa.tiC person will do fine dismisses any potential concerns the patient may have. Rotating nursing personnel does not allow time for the patient to build rapport with any one nurse. DIF: Understand REF: 469 OBJ: Apply the nursing process to promote a client’s self-concept. TOP: Planning MSC: CPNRE: Foundations of Practice 13. The nurse in an addictions clinic is working with a patient on priority setting before the
patient’s discharge from residential treatment. Which of the following would be an appropriate priority for a patient at this clinic? a. Identifying local self-help groups before being discharged from the program. b. Staying away from all triggers that cause substance abuse. c. Stating a plan to never be tempted by illicit substances after discharge. d. Identifying personal areas of weakness to grow stronger. ANS: A
Canadian Fundamentals of Nursing 6th Edition Potter Test Bank Providing the patient with resources such as local self-help groups can help to turn limitations into strengths. Trying to avoid all triggers that can result in addictive behaviours is not realistic. It is also unrealistic to believe that the patient will never be tempted because temptation can arise from multiple sources. On the other hand, an appropriate priority would be to recognize that triggers will arise and that the patient should learn how to handle being confronted in the post-discharge setting. Having a person talk about his or her weaknesses without recognizing a person’s strengths could be a trigger to return to an addictive lifestyle, so this would not be the most appropriate priority. DIF: Apply REF: 467 OBJ: Apply the nursing process to promote a client’s self-concept. TOP: Planning MSC: CPNRE: Foundations of Practice 14. The nurse can best assess the patient’s self-concept by evaluating which of the following? a. The patient’s drug abuse history. b. The patient’s nonverbal behaviour. c. The patient’s personal journal. d. The patient’s social networking site. ANS: B
Nonverbal behaviours are key indicators of a patient’s self-concept. A history of drug abuse does not necessarily indicate current self-concept, and people who do not have a drug abuse history may have a low self-concept. It would be an invasion of privacy and trust for a nurse to read a patient’s personal journal or social networking site. DIF: Apply REF: 466 | 467 OBJ: Apply the nursing process to promote a client’s self-concept. NNURRES TOP: Assessment MSC: CP : FI oN unG daT tiB on. sC ofO PM ractice 15. A newly hired nurse is struggling with night shift work and caring for multiple family
members at home. The nurse manager calls the new nurse in to talk about how the nurse is negatively affecting patient self-concept by ignoring patients’ concerns. What should the nurse manager should focus on? a. The new nurse’s role overload. b. The new nurse’s self-esteem. c. The new nurse’s ego integrity. d. The new nurse’s ethics and morals. ANS: A
Role overload involves having more roles or responsibilities than are manageable. Self-esteem is the overall feeling of self-worth. The nurse is not involved in an ethical or moral dilemma. Ego integrity is part of a developmental task associated with older persons. DIF: Apply REF: 463 OBJ: Explore ways in which a nurse’s self-concept and nursing actions can affect a client’s self-concept and self-esteem. TOP: Implementation MSC: CPNRE: Foundations of Practice 16. A 9-year-old is proudly telling everyone about mastering the yellow belt in her martial arts
class. What is the appropriate developmental task achieved? a. Initiative versus guilt.
Canadian Fundamentals of Nursing 6th Edition Potter Test Bank b. Industry versus inferiority. c. Identity versus role confusion. d. Autonomy versus shame and doubt. ANS: B
Industry versus inferiority is a stage that occurs between the ages of 6 and 12 years. It is during this developmental task that a person gains self-esteem through new skill mastery. The other self-concept development tasks occur at other stages of life and would not apply to this situation. DIF: Apply REF: 459, Box 26-1 OBJ: Describe the components of self-concept as related to psychosocial and cognitive developmental stages. TOP: Planning MSC: CPNRE: Foundations of Practice 17. The nurse can assist the patient in becoming more self-aware by using which technique? a. Setting up an appointment to allow the patient to vent. b. Allowing the patient to openly explore thoughts and feelings. c. Assisting the patient to physically punch a pillow when upset. d. Providing materials for the patient to write complaint letters. ANS: B
Increasing the patient’s self-awareness is achieved through establishing a trusting nurse– patient relationship that allows the patient to openly explore thoughts and feelings. A priority nursing intervention is the expert use of communication skills to clarify the patient’sand family’s expectations. Open exploration can make the situation less threatening and encourages behaviours that expand self-awareness. Punching pillows does not create self-awareness. Writing comN plai nt S leI tter s an R G Bd .veCntin Mg can create more internal turmoil in a U N T O person and do not promote self-awareness. DIF: Apply REF: 465 OBJ: Apply the nursing process to promote a client’s self-concept. TOP: Planning MSC: CPNRE: Foundations of Practice 18. Which of the following is a behaviour suggestive of altered self-concept? a. Unkempt appearance. b. Interest in what is happening. c. Being independent. d. Easily making decisions. ANS: A
Behaviours that are suggestive of altered self-concept include avoidance of eye contact, hesitant speech, unkempt appearance, being excessively dependent, demonstrating a difficulty in making decisions, and a lack of interest in what is happening. DIF: Understand REF: 467 OBJ: Apply the nursing process to promote a client’s self-concept. TOP: Assessment MSC: CPNRE: Foundations of Practice
Canadian Fundamentals of Nursing 6th Edition Potter Test Bank
Chapter 27: Sexuality Potter et al: Canadian Fundamentals of Nursing, 6th Edition MULTIPLE CHOICE 1. A nurse is caring for a 15-year-old who in the past 6 months has had multiple male and female
sexual partners. The nurse knows that which therapeutic statement would be most effective? a. “I know you feel invincible, but STIs [sexually transmitted infections] and unwanted pregnancy are a real risk. Let’s discuss what you think is the best method of protecting yourself.” b. “Having sexual interaction with both males and females places you at higher risk for STIs. To protect yourself you need to decide which orientation you are.” c. “Your current friends are leading you to make poor choices. You should find new friends to hang out with.” d. “I think it’s best to notify your parents. They know what’s best for you and can help make sure you practise safe sex.” ANS: A
Most teenagers feel invincible, and for this reason, some participate in risky behaviours. The nurse should acknowledge this feeling to the patient and offer education and alternatives, while giving the patient the autonomy to make his or her own decisions. The nurse should not force the patient to make a choice of orientation and should not pass judgement on a patient’s sexual orientation or social network; this would make the patient feel defensive and would eliminate the trust in the relationship. Involving parents is not the first line of action; parents should be notified only if the child is in a life-or-death situation.
N RSINGTB.COM
DIF: Apply REF: 478U TOP: Implementation
OBJ: Identify high-risk and safer sex behaviours. MSC: CPNRE: Professional, Ethical, and Legal Practice
2. A nurse is caring for a patient who expresses a desire to have an elective abortion. The nurse’s
religious and ethical values are strongly opposed. How should the nurse best handle the situation? a. Continue to care for the patient, and limit conversation as much as possible. b. Refer the patient to a family planning centre or health care provider. c. Attempt to educate the patient about the consequences of abortion. d. Inform the patient that because of moral issues, another nurse will have to care for her. ANS: B
The nurse must be aware of personal beliefs and values and is not required to participate in counselling or procedures that compromise those values. However, the patient is entitled to nonjudgemental care and should be referred to someone who can create a trusting environment. The nurse should not care for a patient if the quality of care could be jeopardized. The nurse should not attempt to project personal values onto a patient. The nurse also should not create tension by informing the patient that she does not have the same morals; this could cause the patient to feel guilty or defensive when receiving care from any health care provider. DIF: Apply REF: 483 OBJ: Discuss the nurse's role in maintaining or enhancing a patient's sexual health.
Canadian Fundamentals of Nursing 6th Edition Potter Test Bank TOP: Implementation
MSC: CPNRE: Professional, Ethical, and Legal Practice
3. When responding to an adolescent girl’s inquiry related to emergency contraception, the nurse
tells her that the most effective method is which of the following? a. Levonorgestrel for emergency contraception (LNG-EC). b. Ulipristal acetate for emergency contraception (UPA-EC). c. Hormonal intrauterine device. d. Copper intrauterine device. ANS: D
The most effective method for emergency contraception is the use of a copper intrauterine device, which can be inserted and is effective within 7 days after unprotected intercourse. Hormonal intrauterine devices are not used as emergency contraception. Both LNG-EC and UPA-EC are used as emergency contraception but are not as effective as the copper intrauterine device. DIF: Apply REF: 482 OBJ: Use critical thinking skills to assist patients in meeting their sexual needs. TOP: Implementation MSC: CPNRE: Foundations of Practice 4. An 18-year-old male patient informs the nurse that he isn’t sure whether he is homosexual
because he is attracted to both genders. With which statement can the nurse establish a trusting relationship with the patient? a. “Don’t worry. It’s just a phase you will grow out of.” b. “Those are abnormal impulses. You should seek therapy.” c. “At your age, it is normal to be curious about both genders.” d. “Having questions about sexuality is normal. Tell me about your sexual experience.”
NURSINGTB.COM
ANS: C
Young adults have questions about sexuality. The patient will feel most comfortable discussing his sexual concerns further if the nurse establishes that it is normal to ask questions about sexuality. The nurse can then discuss in greater detail. Although it is normal for young adults to be curious about sexuality, the nurse should use caution in giving advice on taking sexual action. The nurse should promote safe sex practices. Telling the patient not to worry dismisses his concern. Telling the patient that he is abnormal might offend the patient and prevent him from establishing an open relationship. Asking the patient about his sexual experience is also inappropriate in this situation and is not likely to assist in building a trusting relationship with the patient. DIF: Understand REF: 438 OBJ: Describe key concepts of sexual development across the lifespan. TOP: Education | Teaching MSC: CPNRE: Foundations of Practice 5. A nurse is caring for a 35-year-old female patient who recently started taking antidepressants
after repeated attempts at fertility treatment. The patient tells the nurse, “I feel happier, but my sex drive is gone.” Which nursing diagnosis has the highest priority? a. Sexual dysfunction. b. Ineffective coping. c. Deficient knowledge about contraception. d. Risk for self-directed violence.
Canadian Fundamentals of Nursing 6th Edition Potter Test Bank
ANS: A
Antidepressants have adverse effects on sexual desire and response. The nurse should be sure to educate the patient on the potential for these side effects and how to correct for them (e.g., using lubricant to ease discomfort). The patient has taken steps toward effective coping by seeking therapy. The patient has not expressed a reason for the nurse to be concerned about contraceptives. The nurse should always assess for concerns about violence in a patient’s life. Although some antidepressants have been related to self-directed violence, this patient states that she is feeling happy. DIF: Apply REF: 484 OBJ: Define appropriate nursing diagnoses for patients with alterations in sexuality. TOP: Planning MSC: CPNRE: Foundations of Practice 6. An adolescent female student who is sexually active visits the office of the school nurse. The
nurse knows that teaching has been effective when the student states which of the following? a. “My boyfriend is able to withdraw before ejaculation, and that prevents me from getting pregnant.” b. “I take my temperature every morning, and when it goes down for at least two days, we have unprotected sex.” c. “We use ‘foam’ before each time we have sex, and I haven’t gotten pregnant yet.” d. “We use a condom with a water-based lubricant.” ANS: D
As a more effective contraceptive method, the water-based lubricant reduces the risk of the condom’s breaking. The patient is verbalizing understanding. Any act of unprotected intercourse can result in pregnancy. Withdrawal is not an effective contraceptive method; any act of unprotected intercoursN e ca nS reI sult nan cy. The statement “I take my temperature R GinTpBre.gC M U N O every morning, and when it goes down for at least two days, we have unprotected sex,” also does not demonstrate understanding of the basal body temperature method of contraception. Using spermicidal foam alone is not recommended. The patient should use a condom and foam to be more effective in preventing pregnancy. DIF: Apply REF: 482, Table 27-1 OBJ: Identify and describe nursing interventions to promote sexual health. TOP: Education | Teaching MSC: CPNRE: Foundations of Practice 7. A patient who has had several sexual partners in the past month expresses a desire to use a
contraceptive. Which contraceptive method should the nurse recommend? a. Spermicide. b. Condom. c. Diaphragm. d. Oral contraceptive. ANS: B
Condoms are both a contraceptive and a barrier against STIs and human immunodeficiency virus (HIV) infection; proper use greatly reduces the risk of infection. Spermicides, diaphragms, and oral contraceptives all protect against pregnancy; however, they do not prevent bodily fluids from coming in contact with the patient during sexual intercourse. DIF: Understand REF: 481 OBJ: Use critical thinking skills to assist patients in meeting their sexual needs.
Canadian Fundamentals of Nursing 6th Edition Potter Test Bank TOP: Education | Teaching
MSC: CPNRE: Foundations of Practice
8. A woman who has been in a monogamous relationship for the past 6 months presents to clinic
with herpes on her labia. The patient is distraught because she believes her partner must have cheated on her. Which response by the nurse is most effective in establishing an open rapport with a patient? a. Sharing an anecdote. b. Informing the patient that all encounters are confidential. c. Telling the patient that she must be honest about every sexual experience she has had. d. Asking the patient what concerns or fears she has related to the visit. ANS: B
If effective open communication is to be established with the patient, the patient must know that she can trust her health care provider. By telling the patient that all discussions with the nurse are confidential, the nurse establishes trust. After establishing that trust, the nurse can further build the relationships by asking the patient open questions about her fear. Forcing the patient to confide by sharing every sexual encounter may hinder the development of a trusting relationship. DIF: Apply TOP: Assessment
REF: 483 OBJ: Assess a patient’s sexuality. MSC: CPNRE: Professional, Ethical, and Legal Practice
9. A patient admits that he knew for several months that he had an STI but did not report it. Why
did he behave this way? a. Many patients with STIs do not acknowledge the importance of STI testing and treatment. N R I G B.C M b. Many patients with STIs arU e heSsitaN nt tT o admitO they contracted an STI. c. Many patients with STIs have lifestyles that subject them to repeated exposure. d. Many patients with STIs are uncomfortable discussing sexual health issues with a same-sex health care provider. ANS: B
Many patients feel guilt, embarrassment, and anxiety about contracting an STI; this can cause them to defer care until the problem becomes severe. Although knowledge about the severity of STIs may be insufficient, this should not deter seeking care. Regardless of lifestyle, STIs should be treated immediately to prevent further transmission. The role of the health care provider is to establish a trusting relationship. A person may be open to discussing health promotion behaviours but may still be hesitant to discuss STIs. DIF: Understand REF: 480 OBJ: Identify and describe nursing interventions to promote sexual health. TOP: Implementation MSC: CPNRE: Foundations of Practice 10. The nurse is leading a seminar about menopause and age-related changes. The nurse knows
that a patient does not fully understand the changes of aging when the patient makes which statement? a. “I will no longer ovulate after menopause.” b. “Orgasms are no longer achievable after menopause.” c. “Hormones of sexual regulation such as estrogen decrease with age.” d. “As men age, their ability to sexually perform may decrease.”
Canadian Fundamentals of Nursing 6th Edition Potter Test Bank
ANS: B
Orgasms are achievable at any age. Estrogen levels decrease, and this may impede sexual desire. The nurse should ask the patient whether she has any questions on action that will assist in healthy sexual patterns. All other statements indicate that the patient does have an understanding of age-related changes. DIF: Understand REF: 478 | 479 OBJ: Use critical thinking skills to assist patients in meeting their sexual needs. TOP: Evaluate MSC: CPNRE: Foundations of Practice 11. A patient who had an ostomy placed 1 month ago states that he is feeling depressed and does
not want to participate in sexual activities anymore because he is afraid that his partner is not physically attracted to him. Which nursing intervention will be most effective in helping this patient resume sexual activity? a. Reassure the patient that lots of people resume sex the same week the ostomy is placed. b. Inform the patient about a support group for people with ostomies. c. Teach the patient about intimate activities that can be done to incorporate the ostomy. d. Discuss ways to adapt to new body image so the patient will be comfortable in resuming intimacy. ANS: D
The nurse should address the patient’s need to be comfortable with his own body image; once this is done, the patient can resume sexual activity. Reassuring the patient that others manage to have sexual intercourse with an ostomy may help to decrease anxiety but may have the unintended effect of making N the pati ent feel R I G Bas.tChou Mgh he is abnormal because he has not yet U S N T O resumed sexual activity. Support groups may be helpful for the patient, but this is not the most effective intervention a nurse can provide. The patient should be discouraged from manipulating the ostomy during sexual activity; this can cause infection, trauma, or prolapse to the stoma. DIF: Apply REF: 485 OBJ: Use critical thinking skills to assist patients in meeting their sexual needs. TOP: Implementation MSC: CPNRE: Foundations of Practice 12. A mother brings her 10-year-old daughter into a clinic and inquires about getting her a human
papillomavirus (HPV) vaccine that day. What information does the nurse give the mother? a. The HPV vaccine is safe for children over the age of 9. b. The HPV vaccine is recommended only after a girl or woman becomes sexually active. c. The HPV vaccine will prevent a girl or woman from ever getting cervical cancer. d. The HPV vaccine is not currently available for 10-year-old girls. ANS: A
The HPV vaccine is effective against the four most common types of HPV that can cause cervical cancer. The recommendation is that girls and women between the ages of 9 and 26 be offered this vaccine; it is important that the vaccine be given before the initiation of sexual activity. The vaccine is most effective if administered before sexual activity or exposure. DIF: Understand
REF: 480
Canadian Fundamentals of Nursing 6th Edition Potter Test Bank OBJ: Identify and describe nursing interventions to promote sexual health. TOP: Implementation MSC: CPNRE: Foundations of Practice 13. A nursing student is providing education to a group of older persons who are in an
independent living retirement village. Which of the following statements made by the nursing student prompts the nursing professor to intervene? a. “Avoiding alcohol use will enhance your sexual functioning.” b. “You do not need to worry about getting a sexually transmitted infection at this point in your life.” c. “You need to tell your partner how you feel about sex and any fears you may have.” d. “Using pillows and taking pain medication if needed before having sexual intercourse often helps alleviate pain associated with intercourse and improves sexual functioning.” ANS: B
Any person who has sexual intercourse can develop an STI at any age. People with multiple sex partners are at greater risk. Therefore, the instructor needs to intervene when the student tells the older persons that they are not at risk for developing an STI. Sexual functioning is enhanced by avoiding the use of alcohol; using pillows; taking pain medications before intercourse if needed; and communicating thoughts, fears, and feelings about sex. DIF: Evaluate REF: 480 | 481 OBJ: Identify and describe nursing interventions to promote sexual health. TOP: Implementation MSC: CPNRE: Foundations of Practice 14. While teaching adolescents about chlamydia, what does the school nurse tells the group? a. Chlamydia is a viral infecNtiU onRtS haIt N caGnT noBt . beCcO urMed. b. Chlamydia can be treated with a full course of antibiotics. c. Chlamydia can be contracted via bloodborne exchange. d. Chlamydia can be prevented with the use of a spermicide. ANS: B
Chlamydia is the most common of all bacterial STIs and is treated with an antibiotic. Chlamydia is caused by bacteria, not by a virus, and can be treated with an antibiotic. STIs such as chlamydia are transmitted from infected individuals to partners during intimate sexual contact. STIs are contracted not via bloodborne exchange but rather through body fluids. Chlamydia is not prevented with the use of a spermicide. DIF: Apply REF: 480 OBJ: Identify common sexually transmitted infections (STIs). MSC: CPNRE: Foundations of Practice
TOP: Implementation
15. A patient asks the nurse what signs and symptoms are associated with chlamydia. How should
the nurse respond? a. The first signs that chlamydia presents are frequency and burning upon urination. b. Symptoms of chlamydia usually affect only women. c. Small red blisters appear first and then multiply. d. Dementia results if chlamydia goes untreated too long. ANS: A
Canadian Fundamentals of Nursing 6th Edition Potter Test Bank Urinary problems and abnormal discharge are often the first signs of chlamydia. Both men and women can experience these symptoms, although it is known as the “silent STI” because symptoms may not manifest for some time after the disease is contracted. Small red blisters are associated with genital warts and herpes. Chlamydia does not cause dementia. DIF: Remember REF: 480 OBJ: Identify common sexually transmitted infections (STIs). MSC: CPNRE: Foundations of Practice
TOP: Assessment
16. What is an example of a quality in the critical thinking model for sexuality assessment? a. Displaying curiosity. b. Applying an ethic of care. c. Accepting the sexual orientation of the patient. d. Determining the patient’s sexual concerns. ANS: A
One of the three qualities in the critical thinking model for sexuality assessment is displaying curiosity. The other two qualities are displaying integrity and taking risks if necessary. Applying an ethic of care is an example of a standard in the model. Accepting a patient’s sexual orientation and determining the patient’s sexual concerns are examples of knowledge in the model. DIF: Apply REF: 486 OBJ: Use critical thinking skills to assist patients in meeting their sexual needs. TOP: Implementation MSC: CPNRE: Foundations of Practice 17. A patient states that she is afraid that she and her husband will not be able to maintain a
healthy sexual relationship nN owUtR hS atI thN eyGhTaB ve.aCbO abMy in the house. To assist these patients, what should the nurse know? a. Whether they have similar parenting beliefs. b. How long they have been married. c. That effective communication about sexuality requires a nondiscriminatory attitude. d. The level of knowledge the patients have regarding healthy sexual relationships. ANS: C
In response to identified concerns, the nurse may initiate discussion. Effective communication about sexuality requires caring, sensitivity, tact, compassion, the use of appropriate language, and a nondiscriminatory attitude. It is important not to have preconceived notions about the patients’ sexual identity or activity. Having similar parenting beliefs will have less effect on their sexual relationship than the use of effective communication; to assist the patients in adjusting to the change of becoming a family, the nurse should explore their communication patterns. How long they have been married would be less significant. The level of knowledge the patients have regarding healthy sexual relationships would not affect their sexual relationship as much as their ability to discuss their feelings with one another. DIF: Apply REF: 483 | 486 | 487 OBJ: Identify and describe nursing interventions to promote sexual health. TOP: Implementation MSC: CPNRE: Foundations of Practice
Canadian Fundamentals of Nursing 6th Edition Potter Test Bank 18. A 68-year-old woman asks the nurse whether a contraceptive is still necessary. What is the
best response by the nurse? a. “A barrier such as a condom is recommended if you are unsure whether your sexual partners have been tested for STIs.” b. “No, because you have gone through menopause and can no longer become pregnant.” c. “Let’s discuss this further; what is your current level of sexual activity?” d. “Yes, as long as using a barrier does not decrease sexual performance.” ANS: C
The nurse should first assess the patient’s risk for contracting an STI. If the patient has been in a monogamous relationship, and both parties are STI free, there is no need for a barrier. However, even though a patient has gone through menopause, there is still risk for an STI. The best way for a nurse to be sensitive to a patient’s needs is to find out more about the patient. Discussing sexual performance in relation to using a barrier method does not answer the patient’s question. DIF: Analyze REF: 486 | 487 OBJ: Describe key concepts of sexual development across the lifespan. TOP: Implementation MSC: CPNRE: Foundations of Practice 19. When conducting an education session on the use of contraceptives, the nurse informs the
group that which method has the lowest initial rate of effectiveness? a. Use of a male condom. b. Tubal ligation. c. Use of a female condom. d. Use of a vaginal contraceptive ring. B.C M ANS: C
N R I G U S N T
O
Of the answer choices, the female condom has the lowest rate of effectiveness, beginning at 79%. The initial rate of effectiveness of the male condom is 86%. The initial rate of effectiveness of a tubal ligation is 99.5%. The initial rate of effectiveness of the vaginal contraceptive ring is 98%. DIF: Apply REF: 482 OBJ: Identify and describe nursing interventions to promote sexual health. TOP: Implementation MSC: CPNRE: Foundations of Practice 20. A patient expresses concern that her partner no longer finds her attractive and is considering
having three-way sex to spice up the relationship. Which response is the best option for the nurse in this situation? a. “Please help me understand how you are feeling about your relationship right now.” b. “If your partner truly loves you, he doesn’t need to have sex with another person.” c. “Nonmonogamous sex is immoral; here is the number for the local church.” d. “Have you considered the consequences of adding a third party to a sexual relationship?” ANS: A
Canadian Fundamentals of Nursing 6th Edition Potter Test Bank The nurse should be nonjudgemental, and asking for clarification of the patient’s feelings will help the nurse better understand the situation. Making assumptions about the patient’s partner is not appropriate. Passing judgement or imposing the nurse’s morals on the patient is unethical. Asking the patient to consider the consequences of a three-way is judgemental and implies that nontraditional sex is inappropriate. DIF: Apply REF: 483 | 486 | 487 OBJ: Identify and describe nursing interventions to promote sexual health. TOP: Implementation MSC: CPNRE: Foundations of Practice
NURSINGTB.COM
Canadian Fundamentals of Nursing 6th Edition Potter Test Bank
Chapter 28: Spirituality in Health and Health Care Potter et al: Canadian Fundamentals of Nursing, 6th Edition MULTIPLE CHOICE 1. The word spirituality derives from the Latin word spiritus, which refers to breath or wind.
Today, spirituality can be described as which of the following? a. Transcendence beyond self, everyday living, and suffering. b. Less important than coping with the patient’s illness. c. Patient centred and having no bearing on the nurse’s belief patterns. d. Equated to formal religious practice and having a minor effect on health care. ANS: A
Today, spirituality is often defined as a connectedness with self, others, a higher power or nature, or a combination of these; a sense of meaning in life; and transcendence beyond self, everyday living, and suffering. Spirituality is an important factor that helps individuals achieve the balance needed to maintain health and well-being and to cope with illness. It positively affects and enhances health, quality of life, health promotion behaviours, and disease prevention activities. Nurses need an awareness of their own spirituality to provide appropriate and relevant spiritual care to others. The concepts of spirituality and religion are often discussed interchangeably, but spirituality is a much broader and more unifying concept than religion. The human spirit is powerful, and spirituality has different meanings for different people. DIF: Understand REF: 494 OBJ: Compare and contrast the concepts of religion and spirituality. N R: Fo IN G tions B.C M TOP: Assessment MSC: CPNURES undaT ofO Practice 2. The nurse is caring for a patient who claims that he does not believe in God and does not
believe in an “ultimate reality.” The nurse realizes what about this patient? a. The patient is devoid of spirituality. b. The patient is an atheist/agnostic. c. The patient finds no meaning through relationships with others. d. The patient believes that what he does is meaningless. ANS: B
Some individuals do not believe in the existence of God (atheist) or do not know whether God or another ultimate reality exists (agnostic). This does not mean that spirituality is not an important concept for the atheist or the agnostic. Atheists search for meaning in life through their work and their relationships with others. Agnostics discover meaning in what they do or how they live because they find no ultimate meaning for the way things are. They believe that people bring meaning to what they do. DIF: Remember REF: 496 OBJ: Compare and contrast the concepts of religion and spirituality. TOP: Assessment MSC: CPNRE: Foundations of Practice 3. The nurse is caring for a patient who is terminally ill with very little time left to live. The
patient states, “I always believed that there was life after death. Now, I’m not so sure. Do you think there is?” The nurse states, “I believe there is.” What has the nurse attempted to do?
Canadian Fundamentals of Nursing 6th Edition Potter Test Bank a. b. c. d.
Strengthen the patient’s religion. Provide hope. Support the patient’s agnostic beliefs. Support the horizontal dimension of spiritual well-being.
ANS: B
When a person has the attitude of something to look forward to, hope is present. Religion is the system of organized beliefs and worship that a person practises to outwardly express spirituality. This is not evident here. Agnostics do not know whether God or another ultimate reality exists and are uncertain whether there is life after death. The horizontal dimension of spiritual well-being describes positive relationships and connections people have with others. In this case, the patient is more concerned with the vertical dimension, which supports the transcendent relationship with God or some other higher power. DIF: Apply REF: 497 | 503 OBJ: Discuss research findings that suggest a relationship between spiritual practices and patients' health status. TOP: Implementation MSC: CPNRE: Foundations of Practice 4. In discussing spirituality with adolescent patients, of what should the nurse be aware? a. That many adolescents have a good concept of a supreme being. b. That many adolescents question religious practices and values. c. That adolescents fully accept the higher meaning of their faith. d. That adolescents do good deeds for others as a spiritual task. ANS: B
Adolescents often reconsider their childlike concept of a spiritual power, and in the search for an identity, they may either qN ues tion es.aC ndOvMalues or find the spiritual power as the UR SIpra NGctic TB motivation to seek a clearer meaning to life. Adolescents do not necessarily have a good concept of a supreme being; this conceptualization occurs as people mature. Adolescents do not necessarily fully accept the higher meaning of their faith. Older persons, not adolescents, often turn to important relationships and doing good deeds for others as spiritual tasks. DIF: Apply REF: 496 OBJ: Discuss the role of the nurse in providing spiritual nursing care. TOP: Implementation MSC: CPNRE: Foundations of Practice 5. The nurse is admitting a patient to the hospital. The patient states that he is a very spiritual
person but does not practise any specific religion. What should the nurse understand about these statements? a. That they are contradictory. b. That they indicate a strong religious affiliation. c. That they indicate a lack of faith. d. That they are reasonable. ANS: D
These statements are reasonable and are not contradictory. Many people tend to use the terms spirituality and religion interchangeably. Although closely associated, these terms are not synonymous. Religious practices encompass spirituality, but spirituality does not need to include religious practice. When a person has the attitude of something to live for and look forward to, hope is present.
Canadian Fundamentals of Nursing 6th Edition Potter Test Bank DIF: Understand REF: 496 OBJ: Compare and contrast the concepts of religion and spirituality. TOP: Assessment MSC: CPNRE: Foundations of Practice 6. Which of the following is an intervention that will facilitate connectedness? a. Restricting visits from pets. b. Placing memorabilia in the patient’s view. c. Avoiding physical touch and massage. d. Implementing relevant rituals and traditions. ANS: B
An example of an intervention to facilitate connectedness is to place memorabilia in view of the patient. Visits from pets should be encouraged, not restricted. Physical touch and massage are interventions to facilitate connectedness and therefore should be implemented, not avoided. Implementing relevant rituals and traditions is an example of an intervention to facilitate hope, not connectedness. DIF: Understand REF: 497 | 498 OBJ: Discuss the role of the nurse in providing spiritual nursing care. TOP: Planning MSC: CPNRE: Foundations of Practice 7. Which statement about religion and spirituality is true? a. Religion is a unifying theme in people’s lives. b. Spirituality is unique to the individual. c. Spirituality encompasses religion. d. Religion and spirituality are synonymous. ANS: B
NURSINGTB.COM
Spirituality is a complex concept that is unique to each individual. Religion is the system of organized beliefs and worship that a person practises to outwardly express spirituality. People from different religions view spirituality differently. Although closely associated, spirituality and religion are not synonymous. Religious practices encompass spirituality, but spirituality does not need to include religious practice. DIF: Remember REF: 495 | 496 OBJ: Compare and contrast the concepts of religion and spirituality. TOP: Assessment MSC: CPNRE: Foundations of Practice 8. What is the best statement by a nurse who is assessing the spiritual needs of a 76-year-old
patient who was just admitted to the nursing unit with terminal cancer of the liver? a. “I notice you have a Bible. Is that a source of spiritual strength for you?” b. “What do you believe happens to your spirit when you die?” c. “We would allow members of your church to visit you whenever you desire.” d. “Has your terminal condition made you lose your faith or beliefs?” ANS: A
Canadian Fundamentals of Nursing 6th Edition Potter Test Bank Stating the observation of the patient having a Bible opens communication regarding the patient’s source of strength. Assessing a patient’s sources of strength and faith can direct interaction with him or her, including medical treatment plans. “What do you believe happens to your spirit when you die?” is not the best response, as it does not provide information that would assist the nurse in meeting the patient’s spiritual needs. “We would allow members of your church to visit you whenever you desire” is not the best response, as it implies both that the patient goes, or should go, to church and the assumption that church members are a source of strength for the patient. It does not provide assessment information to determine the patient’s spiritual needs. “Has your terminal condition made you lose your faith or beliefs?” has a negative connotation and does not help assess the patient’s sources of strength or beliefs. DIF: Understand REF: 499 OBJ: Describe a process for understanding patients' spirituality. TOP: Assessment MSC: CPNRE: Foundations of Practice 9. What is the nurse’s initial action while working with a patient to assess and support the
patient’s spirituality? a. Referring the patient to the agency chaplain. b. Helping the patient to use faith to get well. c. Providing a variety of religious literature. d. Determining the patient’s perceptions and belief system. ANS: D
To assess and support the patient’s spirituality, the nurse should first determine the patient’s perceptions and belief system. Exploring the patient’s spirituality may reveal responses to health problems that necessitate nursing intervention, or it may reveal the existence of a strong set of resources that enable the patient to cope effectively. Although the agency chaplain may N refe RSrral INisGnTotB.C M be a source for referral, such a U the fO irst action the nurse should take in assessing and supporting a patient’s spirituality. The nurse needs first to assess a patient’s spirituality to determine the patient’s perceptions and belief system before attempting to assist the patient to use faith to get well. Providing a variety of religious literature may be ineffective because it does not address the patient as an individual and does not help assess the patient’s personal spiritual needs. DIF: Apply REF: 499 OBJ: Describe a process for understanding patients' spirituality. TOP: Implementation MSC: CPNRE: Foundations of Practice 10. If a patient is identified as following the traditional health care beliefs of Judaism, while
planning patient care, which of the following should be incorporated in nursing care? a. Observance of the Sabbath. b. Faith healing. c. The removal of religious symbols. d. Sacraments of Holy Communion. ANS: A
Observance of the Sabbath is important to a patient who follows the traditional health care beliefs of Judaism. This patient may refuse treatments scheduled on the Sabbath. Followers of the Islamic or Christian faith may use faith healing in response to illness. Religious symbols should not be removed from individuals who practise any religion. Sacraments of Holy Communion are practised by followers of the Christian faith.
Canadian Fundamentals of Nursing 6th Edition Potter Test Bank
DIF: Analyze REF: 502, Table 28-1 OBJ: Discuss the role of the nurse in providing spiritual nursing care. TOP: Planning MSC: CPNRE: Foundations of Practice 11. Which of the following attributes of spiritual care in nursing practice is described as a
direction to place patients’ needs first and protect vulnerability? a. Altruism. b. Interpersonalism. c. Intuitiveness. d. Integrativeness. ANS: A
Altruism is one attribute of spiritual care, meaning direction to place patients’ needs first and protect vulnerability. Interpersonalism is the willingness to be present and engage matters of spirituality without fear. Intuitiveness is the discernment of spiritual matters relevant to the patient. Integrativeness is an ethic of care that influences all aspects of nursing. DIF: Understand REF: 503, Figure 28-2 OBJ: Describe the relational attributes that facilitate spiritual nursing care. TOP: Planning MSC: CPNRE: Foundations of Practice 12. The patient is admitted with chronic back pain. The nurse who is caring for this patient should
do which of the following? a. Focus on finding quick remedies for the back pain. b. Look at how pain influences the patient’s ability to function. c. Realize that the patient’s only goal is relief of the back pain. d. Help the patient realize thNaU t tR heSreIiN sG litT tleBh.oC peOoM f relief from chronic pain. ANS: B
The nurse should not view the patient’s back pain as merely a problem to solve with quick remedies; rather, the nurse should examine how the pain influences the patient’s ability to function and achieve goals established in life (not just pain relief). Mobilizing the patient’s hope is central to a healing relationship. DIF: Apply REF: 497 | 504 OBJ: Describe a process for understanding patients' spirituality. TOP: Implementation MSC: CPNRE: Foundations of Practice 13. In caring for the patient’s spiritual needs, what should the nurse understand? a. Establishing presence is part of the art of nursing. b. Presence involves “doing for” the patient. c. A caring presence involves listening to the patient’s wishes only. d. The nurse must use her expertise to make decisions for the patient. ANS: A
Establishing presence is part of the art of nursing. Presence involves “being with” a patient versus “doing for” a patient. The nurse should demonstrate a caring presence by listening to the patient’s concerns and willingly involving family in discussions about the patient’s health. The nurse should also show self-confidence when providing health instruction and support patients as they make decisions about their health.
Canadian Fundamentals of Nursing 6th Edition Potter Test Bank DIF: Understand REF: 496 | 499 OBJ: Describe the relational attributes that facilitate spiritual nursing care. TOP: Planning MSC: CPNRE: Foundations of Practice 14. When asked about his religious affiliation, a patient responds, “That’s personal; why do you
want to know?” Which of the following is the most appropriate nursing response? a. “You need not answer my question if you prefer not to share that information.” b. “By knowing your religious preferences, I can best meet your spiritual needs.” c. “All information that you provide will be kept in strict confidence.” d. “I did not mean to offend you; we ask that question of all of our new admissions.” ANS: B
Information about religious and spiritual beliefs and practices should be obtained from patients when they enter health care facilities. Nursing care should take into account the biological, psychological, social, cultural, and spiritual needs of patients. Applying knowledge of spiritual concepts and using therapeutic communication skills will help the nurse recognize and understand the patient’s spiritual beliefs, so “You need not answer my question if you prefer not to share that information” is not an appropriate response. Although it is true that patient information is kept confidential, it is important to understand the patient’s belief system to treat the patient with dignity and respect. In the assessment of spiritual caregiving, it is important for the nurse to inform the patient that he or she is obtaining information to ensure sensitivity to the patient’s needs and give the patient the opportunity to communicate his spiritual needs. DIF: Apply REF: 499 | 500 OBJ: Discuss the role of the nurse in providing spiritual nursing care. TOP: Implementation MSC: CPNRE: Foundations of Practice
NURSINGTB.COM
15. Which of the following can be used to develop the nurse’s own spiritual awareness while
facilitating the spiritual practices of others? a. Hosting a church group on the medical unit. b. Engaging in mindfulness. c. Taking part in other religions’ practices. d. Taking a university-level course on religion. ANS: B
Mindfulness can be an important strategy for developing spiritual practice, both for oneself and others. Nurses should learn about various religious practices to be able to support their patients’ spiritual health, but they are not required to host a church group, take a university-level course on religion, or take part in other religions’ practices. DIF: Understand REF: 497 OBJ: Discuss the role of the nurse in providing spiritual nursing care. TOP: Implementation MSC: CPNRE: Foundations of Practice 16. The patient is having a difficult time dealing with his diagnosis of acquired immune
deficiency syndrome (AIDS). He states, “It’s not fair. I’m totally isolated from my family because of this. Even my father hates me for this. He won’t even speak to me.” What does the nurse need to do? a. Assure the patient that his father will accept his situation soon. b. Use therapeutic communication to establish trust and caring.
Canadian Fundamentals of Nursing 6th Edition Potter Test Bank c. Point out that the patient has no control and that he has to face the consequences. d. Tell the patient, “If your father can’t get over it, forget it. You have to move on.” ANS: B
The nurse needs to use therapeutic communication to establish trust and a caring presence because providing spiritual care requires caring, compassion, and respect. The nurse should not offer false hope. The nurse should help the patient maintain feelings of control. The nurse should encourage renewing relationships if possible and establishing connections with self, significant others, and a higher being. DIF: Apply REF: 500 | 501 OBJ: Describe the relational attributes that facilitate spiritual nursing care. TOP: Implementation MSC: CPNRE: Foundations of Practice 17. The nurse is caring for a patient who is in the final stages of a terminal disease. The patient is
very weak but refuses to use a bedpan and wants to get up to use the bedside commode. What should the nurse do? a. Explain to the patient that he is too weak and needs to use the bedpan. b. Insert a rectal tube so that the patient no longer needs to actively defecate. c. Enlist assistance from family members if possible and assist the patient to get up. d. Put the patient on a bedpan and stay with him until he is finished. ANS: C
Establishing presence is part of the art of nursing. Presence involves “being with” a patient versus “doing for” a patient. The nurse should demonstrate a caring presence by listening to the patient’s concerns and willingly involving family in discussions about the patient’s health. The nurse should support patients as they make decisions about their health. If at all possible, the nurse should encourage thNeUpR atie ntNtoGma SI TBin.taCinOaMs much independence as possible. Inserting a rectal tube involves “doing for” instead of “being with.” Placing the patient on the bedpan is against the patient’s wishes and is another form of “doing for.” DIF: Apply REF: 496 | 499 OBJ: Describe the relational attributes that facilitate spiritual nursing care. TOP: Implementation MSC: CPNRE: Foundations of Practice 18. In assessing the spiritual health of her patients, what should the nurse understand? a. Spiritual beliefs change as patients grow and develop. b. Spiritual health in older persons does not lead to peace and acceptance of others. c. Older persons often express spirituality by focusing on themselves. d. The basis of beliefs among older people is focused on one or two factors. ANS: A
Spiritual beliefs change as patients grow and develop. Health spirituality in older persons leads to peace and acceptance of self. However, older persons often express their spirituality by turning to important relationships and giving of themselves to others. Beliefs among older people vary on the basis of many factors, such as gender, past experience, religion, economic status, and ethnic background. DIF: Understand REF: 496 OBJ: Discuss appropriate ways to facilitate patients' spiritual practices. TOP: Assessment MSC: CPNRE: Foundations of Practice
Canadian Fundamentals of Nursing 6th Edition Potter Test Bank 19.
Which of the following are included in the five Cs of relational practice? Compassion, curiosity, commitment, competence, and corresponding. Compassion, communication, curiosity, candour, and corresponding. Commitment, caring, candour, communication, and competence. Commitment, communication, curiosity, caring, and competence.
a. b. c. d.
ANS: A
The five Cs of relational practice are compassion, curiosity, commitment, competence and corresponding. Communication, caring, and candour are not included in this framework. DIF: Understand REF: 503, Box 28-7 OBJ: Discuss the role of the nurse in providing spiritual nursing care. TOP: Planning MSC: CPNRE: Foundations of Practice
NURSINGTB.COM
Canadian Fundamentals of Nursing 6th Edition Potter Test Bank
Chapter 29: Stress and Adaptation Potter et al: Canadian Fundamentals of Nursing, 6th Edition MULTIPLE CHOICE 1. In a natural disaster relief facility, the nurse observes that an elderly man has a recovery plan,
whereas a 25-year-old man is still overwhelmed by the disaster situation. These different reactions to the same situation would be explained best by which of the following? a. Restorative care. b. Strong financial resources. c. Maturational and sociocultural factors. d. Immaturity and intelligence factors. ANS: C
Maturational factors and sociocultural factors can affect people differently, depending on their life experiences. An older individual would have more life experiences to draw from and determine which strategies were successful, whereas a younger individual would have had fewer such. Nothing in the scenario implies that either man is in restorative care, has strong financial resources, or is immature or intelligent. DIF: Apply REF: 514 OBJ: Discuss the integration of stress theory with nursing theories. TOP: Planning MSC: CPNRE: Foundations of Practice 2. A woman who was sexually assaulted more than a month ago presents to the health clinic
with complaints of recurrent nightmares, fear of going to sleep, repeated vivid memories of N Rfeel INmu GT B.Ction. M The nurse recognizes the signs and the sexual assault, and inabilityUto S ch emoO symptoms of which medical problem? a. General adaptation syndrome. b. Post-traumatic stress disorder (PTSD). c. Developmental crisis. d. Alarm reaction. ANS: B
PTSD is characterized by vivid recollections of the traumatic event and emotional numbing and often is accompanied by nightmares. Symptoms of PTSD are more persistent than those of acute stress disorder, having endured for at least 1 month. General adaptation syndrome is the expected reaction to a major stressor. Developmental crisis occurs as a person moves through life stages rather than in response to a trauma. Alarm reaction involves physiological events such as increased activation of the sympathetic nervous system that would have occurred at the time of the sexual assault. DIF: Apply REF: 512 OBJ: Differentiate acute stress disorder and post-traumatic stress disorder. TOP: Assessment MSC: CPNRE: Foundations of Practice 3. The nurse teaches stress reduction and relaxation training to a health education group of
patients after cardiac bypass surgery. The nurse is performing which level of intervention? a. Primary. b. Secondary.
Canadian Fundamentals of Nursing 6th Edition Potter Test Bank c. Tertiary. d. Quad level. ANS: C
Tertiary-level interventions assist patients in adapting to life with an illness. Tertiary prevention focuses on the person who already has the disease and is recovering or rehabilitating, and the goals are to slow down the disease process, prevent further damage or pain from the disease, and prevent the current disease from creating other health problems. Primary interventions include stress prevention, promotion of wellness, and risk factor reduction before illness occurs. Secondary interventions are implemented after symptoms appear and help the person develop resources to manage illness and stress. “Quad level” interventions do not exist. DIF: Understand REF: 513 OBJ: Discuss the integration of stress theory with nursing theories. TOP: Assessment MSC: CPNRE: Foundations of Practice 4. A nurse is teaching guided imagery to a prenatal class. Which of the following is an example
of guided imagery? a. Singing. b. Back massage. c. Peaceful sensory words. d. Listening to music. ANS: C
Guided imagery is used as a means to create a relaxed state through the person’s imagination, often with the use of sensory words. Imagination allows the person to create a soothing and peaceful environment. SinginNg,UbRack , aCnd listening to music are other types of stress SIma NGssag TBe. OM management techniques. DIF: Understand REF: 520 OBJ: Describe stress management techniques beneficial for coping with stress. TOP: Assessment MSC: CPNRE: Foundations of Practice 5. After a natural disaster occurred, an emergency worker referred a family for crisis intervention
services. One family member refused to attend the services, stating, “No way, I’m not crazy.” The best response the nurse can give is which of the following? a. “Many times, disasters can create mental health problems, so you really should participate with your family.” b. “Crisis intervention is a short-term problem-solving type of help, and seeking this help does not mean that you have a mental illness.” c. “Don’t worry now. The psychiatrists are well trained to help.” d. “Crisis intervention will help your family communicate better.” ANS: B
Crisis intervention is a type of brief therapy that is more directive than typical psychotherapy or counselling. It focuses on problem solving and involves only the problem created by the crisis. The goal of crisis intervention is to create stability for the person involved in the crisis while promoting self-reliance. The other options do not properly reassure the patient and build trust. DIF: Apply
REF: 521
OBJ: Discuss the process of crisis intervention.
Canadian Fundamentals of Nursing 6th Edition Potter Test Bank TOP: Implementation
MSC: CPNRE: Foundations of Practice
6. Pediatric stressors related to self-esteem and changes in family structure reflect which
maturational school age category? a. Elementary school age. b. Preadolescence. c. Adolescence. d. Early adulthood. ANS: B
Children of preadolescent age experience stress related to self-esteem issues, changing family structure due to divorce or death of a parent, or hospitalization. Adolescent stressors include identity issues with peer groups and separation from parents. Elementary school age stressors include friends, family, and school relations. Adult stressors centre on life events. DIF: Apply REF: 514 TOP: Implementation
OBJ: Describe how stress is conceptualized. MSC: CPNRE: Foundations of Practice
7. During the evaluation stage of the critical thinking model applied to a patient coping with
stress, what will the nurse do? a. Select nursing interventions to promote the patient’s adaptation to stress. b. Establish short- and long-term goals with the patient experiencing stress. c. Identify stress management interventions for achieving expected outcomes. d. Reassess patient’s stress-related symptoms and compare with expected outcomes. ANS: D
During the evaluation stage, the nurse compares current stress-related symptoms against established measurable outcoN mes luat e t. hC e eff URtoSIeva NG TB OMectiveness of the intervention. Identifying and selecting appropriate interventions and establishing goals are part of the planning process. DIF: Understand REF: 522 OBJ: Develop a care plan for patients experiencing stress. MSC: CPNRE: Foundations of Practice
TOP: Implementation
8. The nurse is involved in crisis intervention with a family in which the father has just lost his
job and is experiencing periods of depression. The mother has a chronic debilitating illness that has put added responsibilities on the adolescent child, who is having behavioural problems. How does the nurse help the family focus on their feelings? a. Pointing out the connection between the situation and their responses. b. Encouraging the use of the family’s usual coping skills. c. Working on time management skills. d. Discussing past experiences. ANS: A
When using a crisis intervention approach, the nurse helps the patient make the mental connection between the stressful event and the patient’s reaction to it. Because an individual’s or family’s usual coping strategies are ineffective in managing the stress of the precipitating event in a crisis situation, the use of new coping mechanisms is required. Time management skills will not help reduce the stress of the precipitating event in a crisis situation. Past experiences may not be helpful in managing the stress of the precipitating event in a crisis situation.
Canadian Fundamentals of Nursing 6th Edition Potter Test Bank DIF: Analyze REF: 521 TOP: Implementation
OBJ: Discuss the process of crisis intervention. MSC: CPNRE: Foundations of Practice
9. A senior university student contacts the campus health clinic about a first-year student living
on the same dormitory floor. The senior student reports that the first-year student is crying and is not adjusting to university life. The clinic nurse recognizes this as a combination of situational and maturational stress factors. What would be the best comment to the senior student? a. “I’d better call 911 because your friend is suicidal.” b. “Give her this list of university and community resources.” c. “You must make an appointment for the student to obtain medications.” d. “I’d recommend you help the student pack her bags to go home.” ANS: B
A health care provider can help to reduce situational stress factors for individuals. Providing the student with a list of resources is one way to begin this process, as part of secondary prevention strategies. This is not a medical or psychiatric emergency, so calling 911 is not necessary. Not everyone who has sadness needs medications; some need only counselling. Not enough information is given to indicate whether leaving the university is the best solution for the student. DIF: Apply REF: 513 OBJ: Develop a care plan for patients experiencing stress. MSC: CPNRE: Foundations of Practice
TOP: Implementation
10. When working in a highly stressful nursing unit and accepting additional shifts, a new nursing
graduate should develop a strategy to prevent burnout. What would the best strategy for the NURSINGTB.COM new nurse be? a. Identify limits and scope of work responsibilities. b. Write for 10 minutes in a journal every day. c. Use progressive muscle relaxation. d. Delegate complex nursing tasks to licensed professional nurses. ANS: A
An important step in preventing burnout is acknowledging one’s own limitations, as well as what one’s scope of work is while on the job. By doing this, the nurse will help to prevent emotional exhaustion and will limit the effects of chronic stress. Journalling and muscle relaxation are good stress-relieving techniques but are not directed at the cause of the workplace stress. Delegating, if not applicable, is an inappropriate coping mechanism. DIF: Apply REF: 521 OBJ: Discuss how stress in the workplace can affect health care providers. TOP: Implementation MSC: CPNRE: Professional, Ethical, and Legal Practice 11. A teenager with celiac disease continues to eat food she knows will make her ill several hours
after ingestion. In view of the appropriate tertiary-level interventions, what would be the nursing intervention? a. Teach the patient about Canada’s Food Guide to Healthy Eating. b. Administer antidiarrheal medications with meals. c. Assist the teenager in meeting dietary restrictions while eating foods similar to those eaten by her friends.
Canadian Fundamentals of Nursing 6th Edition Potter Test Bank d. Admonish the teenager and her parents regarding her consistently poor diet
choices. ANS: C
Tertiary interventions help the patient adapt to life with an illness. By adjusting the diet to meet dietary guidelines and also addressing adolescent emotional needs, the nurse will help the teenager to follow an appropriate diet without health complications and see herself as a “typical and normal” teenager. Teaching about Canada’s Food Guide to Healthy Eating will not address the real issue, which is that the teenager is still eating what she knows will make her ill. Administering antidiarrheal medications may help but is not a tertiary intervention. Admonishing the teenager and parents is not a tertiary intervention, and because this approach is nontherapeutic, it may cause communication problems. DIF: Apply REF: 513 OBJ: Develop a care plan for patients experiencing stress. MSC: CPNRE: Foundations of Practice
TOP: Implementation
12. A trauma survivor is requesting sleep medication because of “bad dreams.” Concerned about
PTSD, the nurse would ask which of the following? a. “Are you reliving your trauma?” b. “Are you having chest pain?” c. “Can you describe your phobias?” d. “Can you tell me when you wake up?” ANS: A
People who have PTSD often have flashbacks, re-experiencing the trauma. The other answers involve assessment of problems not specific to PTSD.
NURSINGTB.COM
DIF: Apply REF: 512 OBJ: Differentiate acute stress disorder and post-traumatic stress disorder. TOP: Implementation MSC: CPNRE: Foundations of Practice 13. A 23-year-old patient who recently suffered a head injury in a motor vehicle accident is
unconscious. What physiological component is primarily responsible for the patient’s level of consciousness? a. Medulla oblongata. b. Reticular formation. c. Pituitary gland. d. External stress response. ANS: B
The reticular formation is primarily responsible for an individual’s level of consciousness. The medulla oblongata controls vital functions such as heart rate, blood pressure, and respiration. The pituitary gland supplies hormones that control vital functions. The pituitary gland produces hormones necessary for adaptation to stress. The external stress response is not primarily responsible for a person’s level of consciousness. DIF: Analyze REF: 510 OBJ: Define the key biological systems involved in stress responses. TOP: Assessment MSC: CPNRE: Foundations of Practice 14. An example of an assessment finding of caregiver strain would be which of the following?
Canadian Fundamentals of Nursing 6th Edition Potter Test Bank a. b. c. d.
Caregiver routinely creates a weekly menu plan. Caregiver has not received medical care when ill. Caregiver can identify respite care provider. Caregiver attends religious services.
ANS: B
A nurse will identify a caregiver’s lack of self-care as a potential example of caregiver role strain. Sacrificing their own health to care for the identified patient places caregivers at risk for becoming ill themselves. If caregivers jeopardize their own health, they may not be able to care for the actual patient. In all of the other options, the caregiver is handling caregiver stress appropriately. DIF: Apply REF: 513, Box 29-3 OBJ: Develop a care plan for patients experiencing stress. MSC: CPNRE: Foundations of Practice
TOP: Implementation
15. Nurses in the medical centre are working with patients experiencing PTSD after losing family
members from severe acute respiratory syndrome. Which of the following approaches would be appropriate to incorporate into the plan of care? a. Suppression of anxiety-producing memories. b. Reinforcement that the PTSD is short term. c. Promotion of relaxation strategies. d. Focusing on physical needs. ANS: C
Teaching the patient relaxation strategies can help reduce the stress of anxiety-provoking thoughts and events, as seen in PTSD, and reinforces an adaptive coping strategy. In the presence of anxiety-provokinN gU thRoug veC nts, SIhts NGand TBe. OMa common physiological symptom is muscle tension. Physiological tension is diminished through a systematic approach to releasing tension in major muscle groups. Suppression would be a maladaptive coping mechanism. PTSD is not a short-term problem but a long-term one; the focus should be on developing adaptive coping mechanisms and lowering the individual’s anxiety level. The primary focus for the patient who is experiencing PTSD is not on physical needs. DIF: Apply REF: 520 OBJ: Differentiate acute stress disorder and post-traumatic stress disorder. TOP: Implementation MSC: CPNRE: Foundations of Practice 16. The nursing student gave a wellness lecture on the importance of accurate assessment and
intervention from a personal, family, and community perspective. The other nursing students enjoyed the lecture about which nursing theory? a. Ego defence model. b. Situational model. c. Evidence-informed practice model. d. Neuman’s systems model. ANS: D
Canadian Fundamentals of Nursing 6th Edition Potter Test Bank Neuman’s systems model is based on an individual’s/family’s/community’s relationship to stress and the reaction to stress. This model promotes wellness on primary, secondary, and tertiary levels. The other items listed as models are not nursing theories. Ego defence mechanisms are unconscious coping mechanisms. Situational factors include stressors such as relocation or family job changes. Evidence-informed practice consists of relying on data or other reputable information sources to guide nursing care. DIF: Apply REF: 513 OBJ: Discuss the integration of stress theory with nursing theories. TOP: Implementation MSC: CPNRE: Foundations of Practice 17. An adult who was in a motor vehicle accident is brought into the emergency department by
paramedics, who report the following in-transit vital signs: Oral temperature: 37.2°C (99.0°F) Pulse: 102 beats per minute Respiratory rate: 26 breaths per minute Blood pressure: 140/106 The nurse can identify that which hormones are the likely causes of the abnormal vital signs? a. Antidiuretic hormone (ADH) and adrenocorticotropic hormone (ACTH). b. ACTH and epinephrine. c. ADH and norepinephrine. d. Epinephrine and norepinephrine. ANS: D
Epinephrine and norepinephrine are catecholamine hormones secreted by the adrenal medulla that rapidly elevate heart rate and blood pressure. ACTH originates from the anterior pituitary gland and stimulates cortisolN releR ; AD igC inatM U ase SI NGHToBr. Oes from the posterior pituitary gland and increases renal reabsorption of water. ACTH, cortisol, and ADH do not increase heart rate. DIF: Apply REF: 510 | 511 OBJ: Discuss the integration of stress theory with nursing theories. TOP: Implementation MSC: CPNRE: Foundations of Practice 18. The nurse is working with patients in an outpatient health care setting. One of the patients is
experiencing job-related stress. Which of the following is a factor that may indicate chronic stress? a. Avoiding discussion of job problems. b. Saying negative things about other colleagues. c. Experiencing chronic pain and irritable bowel syndrome. d. Blaming the boss for being unreasonably demanding. ANS: C
Chronic pain disorders, irritable bowel syndrome, somatization disorder, and clinical depression have all been linked to chronic stress. A patient’s avoiding discussion of the problem, negative comments about his or her colleagues, and blaming of the boss may not be related to chronic stress. DIF: Understand TOP: Assessment
REF: 510-512 OBJ: Describe how stress is conceptualized. MSC: CPNRE: Foundations of Practice
Canadian Fundamentals of Nursing 6th Edition Potter Test Bank 19. A 72-year-old patient is in a long-term care facility after having had a cerebrovascular
accident. The patient is noncommunicative, enteral feedings are not being absorbed, and respirations are becoming laboured. What stage of the general adaptation syndrome is this patient experiencing? a. Resistance stage. b. Exhaustion stage. c. Reflex pain stage. d. Alarm reaction stage. ANS: B
The exhaustion stage occurs when the body can no longer resist the effects of the stressor and when the energy necessary to maintain adaptation is depleted. During the resistance or adaptation stage, the body stabilizes. Reflex pain response is not a stage of the general adaptation syndrome. During the alarm reaction, increasing hormone levels result in increased blood volume, epinephrine and norepinephrine amounts, heart rate, blood flow to muscles, oxygen intake, and mental alertness. DIF: Understand REF: 509 OBJ: Discuss the integration of stress theory with nursing theories. TOP: Implementation MSC: CPNRE: Foundations of Practice 20. While giving a lecture on attention-deficit–hyperactivity disorder, the nurse encourages which
of the following to reduce children’s stress regarding homework assignments? a. Time management skills. b. Prevention of iron deficiency anemia. c. Routine preventive health visits. d. Speech articulation skills. ANS: A
NURSINGTB.COM
Time management skills are most helpful in reducing homework-related stress. Anemia prevention will improve energy levels but not alleviate stress. Routine health visits are important but do not directly affect ability to complete homework. Speech and other developmental aspects need to be developed if the child is to be successful, but skill development will not directly reduce homework-related stress. DIF: Apply REF: 520 OBJ: Describe stress management techniques beneficial for coping with stress. TOP: Implementation MSC: CPNRE: Foundations of Practice
Canadian Fundamentals of Nursing 6th Edition Potter Test Bank
Chapter 30: Vital Signs Potter et al: Canadian Fundamentals of Nursing, 6th Edition MULTIPLE CHOICE 1. How does the posterior hypothalamus help control temperature? a. By causing vasoconstriction. b. By shunting blood to the skin and extremities. c. By increasing sweat production. d. By causing vasodilation. ANS: A
If the posterior hypothalamus senses that the body’s temperature is lower than the set point, the body initiates heat conservation mechanisms. Vasoconstriction of blood vessels reduces blood flow to the skin and extremities. The anterior hypothalamus controls heat loss by inducing sweating, vasodilation of blood vessels, and inhibition of heat production. DIF: Remember REF: 528 OBJ: Explain the physiology of normal regulation of blood pressure, pulse, oxygen saturation, and respirations. TOP: Assessment MSC: CPNRE: Foundations of Practice 2. Of the following mechanisms of heat loss by the body, what is the mechanism that transfers
heat away through the use of air movement? a. Radiation. b. Conduction. c. Convection. NURSINGTB.COM d. Evaporation. ANS: C
Convection is the transfer of heat away from the body by air movement. Conduction is the transfer of heat from one object to another with direct contact. Radiation is the transfer of heat from the surface of one object to the surface of another without direct contact between the two. Evaporation is the transfer of heat energy when a liquid is changed to a gas. DIF: Understand REF: 529 OBJ: Explain the physiology of normal regulation of blood pressure, pulse, oxygen saturation, and respirations. TOP: Assessment MSC: CPNRE: Foundations of Practice 3. The patient has a temperature of 40.7°C (105.2°F). The nurse is attempting to lower his
temperature by providing tepid sponge baths and placing cool compresses in strategic body locations. The nurse is attempting to lower the patient’s temperature through the use of which of the following? a. Radiation. b. Conduction. c. Convection. d. Evaporation. ANS: B
Canadian Fundamentals of Nursing 6th Edition Potter Test Bank Applying an ice pack or bathing a patient with a cool cloth increases conductive heat loss. Radiation is the transfer of heat from the surface of one object to the surface of another without direct contact between the two. Evaporation is the transfer of heat energy when a liquid is changed to a gas. Convection is the transfer of heat away from the body by air movement. DIF: Apply REF: 529 OBJ: Explain the physiology of normal regulation of blood pressure, pulse, oxygen saturation, and respirations. TOP: Implementation MSC: CPNRE: Foundations of Practice 4. In focusing on temperature regulation of newborns and infants, what should the nurse know? a. Temperatures are basically the same for infants and older persons. b. Infants have well-developed temperature-regulating mechanisms. c. The normal temperature range gradually increases as the person ages. d. Newborns need to wear a cap to prevent heat loss. ANS: D
A newborn loses up to 30% of body heat through the head and therefore needs to wear a cap to prevent heat loss. Temperature control mechanisms in newborns are immature and respond drastically to changes in the environment. The normal temperature range gradually drops with age. DIF: Understand REF: 529 OBJ: Describe factors that cause variations in body temperature, pulse, oxygen saturation, respirations, and blood pressure. TOP: Assessment MSC: CPNRE: Foundations of Practice
NURSINGTB.COM
5. The nurse is working the night shift on a surgical unit and is making rounds at 0400 hours.
She notices that the patient’s temperature is 36°C (96.8°F), whereas at 1600 hours the preceding day, it was 37°C (98.6°F). What should the nurse do? a. Call the physician immediately to report a possible infection. b. Realize that this is a normal temperature variation. c. Provide another blanket to conserve body temperature. d. Provide medication to lower the temperature further. ANS: B
Body temperature normally changes 0.5°C to 1°C (0.9°F to 1.8°F) during a 24-hour period. Unless the patient is complaining of being cold, there is no physiological need for an extra blanket or for medication to lower the body temperature further. There is also no need to call a physician to report a normal temperature variation. DIF: Apply REF: 530 OBJ: Describe factors that cause variations in body temperature, pulse, oxygen saturation, respirations, and blood pressure. TOP: Implementation MSC: CPNRE: Foundations of Practice 6. The nurse is caring for a patient who has a temperature reading of 38° C (100.4°F). His last
two temperature readings were 37°C (98.6°F) and 36°C (96.8°F). What should the nurse do? a. Call the physician and anticipate an order to treat the fever. b. Assume that the patient has an infection and order blood cultures. c. Wait an hour and recheck the patient’s temperature.
Canadian Fundamentals of Nursing 6th Edition Potter Test Bank d. Be aware that temperatures this high are harmful and affect patient safety. ANS: C
Waiting an hour and rechecking the patient’s temperature would be the most appropriate action in this case. A fever usually is not harmful if it stays below 39°C (102.2°F), and a single temperature reading does not always indicate a fever. In addition to physical signs and symptoms of infection, a fever determination is based on comparison between several temperature readings at different times of the day and the usual value for that person at that time. Mild temperature elevations enhance the body’s immune system by stimulating white blood cell production. Staff nurses usually do not order blood cultures, and nurses should base actions on knowledge, not on assumptions. DIF: Apply REF: 530 OBJ: Describe factors that cause variations in body temperature, pulse, oxygen saturation, respirations, and blood pressure. TOP: Implementation MSC: CPNRE: Foundations of Practice 7. When heat loss mechanisms of the body are unable to keep pace with excess heat production,
the result is known as which of the following? a. Pyrexia. b. The plateau phase. c. The set point. d. Becoming afebrile. ANS: A
Pyrexia, or fever, occurs because heat loss mechanisms are unable to keep pace with excess heat production, resulting in an abnormal rise in body temperature. The set point is the temperature point determinedNby al. am Cus. URthe SIhyp NGoth TB OMWhen pyrogens trigger immune system responses, the hypothalamus reacts to raise the set point, and the body produces and conserves heat. During the plateau phase, chills subside and the person feels warm and dry as heat production and loss equilibrate at the new level. When the fever “breaks,” the patient becomes afebrile. DIF: Remember REF: 530 OBJ: Identify ranges of acceptable vital sign values for an infant, a child, and an adult. TOP: Assessment MSC: CPNRE: Foundations of Practice 8. The nurse is caring for a patient who has an elevated temperature. What should the nurse
know? a. Fever and hyperthermia are the same thing. b. Hyperthermia occurs when the body cannot reduce heat loss. c. Hyperthermia is an upward shift in the set point. d. Hyperthermia occurs when the body cannot reduce heat production. ANS: D
Elevation in body temperature that is related to the body’s inability to promote heat loss or reduce heat production is hyperthermia. Fever and hyperthermia are not the same thing. Fever, not hyperthermia, is an upward shift in the set point. DIF: Understand REF: 530 | 531 OBJ: Describe factors that cause variations in body temperature, pulse, oxygen saturation, respirations, and blood pressure. TOP: Assessment
Canadian Fundamentals of Nursing 6th Edition Potter Test Bank MSC: CPNRE: Foundations of Practice 9. The patient is restless and has a temperature of 39°C (102.2°F). What is one of the first things
the nurse should do? a. Administer oxygen to the patient. b. Restrict the patient’s fluid intake. c. Increase the patient’s activity. d. Increase the patient’s metabolic rate. ANS: A
During a fever, cellular metabolism increases and oxygen consumption rises. Myocardial hypoxia produces angina. Cerebral hypoxia produces confusion. Interventions during a fever include oxygen therapy. Dehydration is a serious problem resulting from increased respiration and diaphoresis, and a dehydrated patient is at risk for fluid volume deficit. Fluids should not be restricted. Increasing activity would increase the metabolic rate further, which would not be advisable. DIF: Apply REF: 531 OBJ: Describe factors that cause variations in body temperature, pulse, oxygen saturation, respirations, and blood pressure. TOP: Implementation MSC: CPNRE: Foundations of Practice 10. A patient’s temperature must be measured every 2 hours. Which of the following tasks cannot
be delegated to an unregulated care provider? a. Selecting appropriate route and device. b. Obtaining temperature measurement at ordered frequency. c. Being aware of the usual values for the patient. d. Assessing changes in bodN yU teR mS pI erN atG urT e.B.COM ANS: D
The nurse is responsible for assessing changes in body temperature. The nurse instructs an unregulated care provider to select the appropriate route and device to measure temperature, to obtain temperature measurement at ordered frequency, and to be aware of the usual values for the patient. DIF: Apply REF: 532 OBJ: Appropriately delegate vital sign measurement to unregulated care providers. TOP: Implementation MSC: CPNRE: Collaborative Practice 11. The patient requires routine temperature assessment but is confused and easily agitated and
has a history of seizures. What would be the best way to take his temperature? a. Orally. b. Tympanically. c. Rectally. d. By the axillary method. ANS: B
Canadian Fundamentals of Nursing 6th Edition Potter Test Bank The tympanic route is easily accessible, necessitates minimal patient repositioning, and often can be used without disturbing the patient. Its measurement time is also very rapid. Oral temperature measurements require patient cooperation and are not recommended for patients with a history of epilepsy. Rectal temperature measurements require positioning and may increase patient agitation. Axillary temperature measurements take a long time and continuous positioning by the nurse. The patient’s agitation state may not allow for long periods of immobility. DIF: Apply REF: 537, Box 30-4 OBJ: Explain variations in technique used to assess an infant’s, a child’s, and an adult’s vital signs. TOP: Implementation MSC: CPNRE: Foundations of Practice 12. The patient is being admitted to the surgical unit after a motor vehicle accident. His jaw is
broken, and he has several broken teeth. His skin is cool and diaphoretic. To obtain an accurate temperature, the nurse uses which of the following routes? a. Oral. b. Axillary. c. Rectal. d. Temporal. ANS: C
The rectal route is argued to be more reliable when oral temperature cannot be obtained. Temperatures are not measured orally in patients who have had oral surgery or trauma, a history of epilepsy, or shaking chills. Axillary temperature measurement is affected by exposure to the environment, including time to place the thermometer; it also takes a long time. Temporal artery temperature is affected by skin moisture such as diaphoresis or sweating.
NURSINGTB.COM
DIF: Apply REF: 537, Box 30-4 OBJ: Accurately assess pulse, respirations, oxygen saturation, and blood pressure. TOP: Implementation MSC: CPNRE: Foundations of Practice 13. The nurse is caring for an infant and is measuring the patient’s vital signs. The best site for the
nurse to measure the infant’s pulse would be which artery? a. Radial. b. Brachial. c. Femoral. d. Popliteal. ANS: B
The brachial or apical pulse is the best site for assessing an infant’s or a young child’s pulse because other peripheral pulses such as the radial, femoral, and popliteal arteries are deep and are difficult to palpate accurately. DIF: Apply REF: 542 OBJ: Explain variations in technique used to assess an infant’s, a child’s, and an adult’s vital signs. TOP: Implementation MSC: CPNRE: Foundations of Practice 14. The patient is found to be unresponsive and not breathing. To determine the presence of
central blood circulation and circulation of blood to the brain, the nurse checks which pulse? a. Radial.
Canadian Fundamentals of Nursing 6th Edition Potter Test Bank b. Brachial. c. Posterior tibial. d. Carotid. ANS: D
The heart continues to deliver blood through the carotid artery to the brain as long as possible. The carotid pulse is easily accessible during physiological shock or cardiac arrest. The radial pulse is used to assess peripheral circulation or to assess the status of circulation to the hand. The brachial site is used to assess the status of circulation to lower arm. The posterior tibial pulse is used to assess the status of circulation to the foot. DIF: Apply REF: 542 OBJ: Accurately assess pulse, respirations, oxygen saturation, and blood pressure. TOP: Implementation MSC: CPNRE: Foundations of Practice 15. The nurse needs to measure the radial pulse from a patient. For accuracy, what must the nurse
do? a. Place the tips of the nurse’s first two fingers over the groove along the thumb side
of the patient’s wrist. b. Place the thumb over the groove along the thumb side of the patient’s wrist. c. Apply a very light touch so that the pulse is not obliterated. d. Apply very strong pressure to detect the pulse. ANS: A
The nurse places the tips of the first two or middle three fingers over the groove along the radial or thumb side of the patient’s inner wrist. Fingertips are the most sensitive parts of the hand for palpating arterial pulsation. The thumb has a pulsation that interferes with accuracy. The pulse is more accuratelyNass esse wG ithTm Bo.dCerat UR SId N OMe pressure. Too much pressure occludes the pulse and impairs blood flow. DIF: Apply REF: 544 OBJ: Accurately assess pulse, respirations, oxygen saturation, and blood pressure. TOP: Implementation MSC: CPNRE: Foundations of Practice 16. While the nurse is assessing the patient’s respirations, it is important for the patient to do
what? a. Be aware of the procedure being done. b. Not know that respirations are being assessed. c. Understand that respirations are estimated to save time. d. Not be touched until the entire process is finished. ANS: B
Do not let a patient know that respirations are being assessed. A patient who is aware of the assessment can alter the rate and depth of breathing. Respirations are the easiest of all vital signs to assess, but they are often the most haphazardly measured. Do not estimate respirations. Accurate measurement requires observation and palpation of chest wall movement. DIF: Apply REF: 549 OBJ: Accurately assess pulse, respirations, oxygen saturation, and blood pressure. TOP: Implementation MSC: CPNRE: Foundations of Practice
Canadian Fundamentals of Nursing 6th Edition Potter Test Bank 17. The patient’s blood pressure is 140/60. How would the nurse record the pulse pressure? a. As 140. b. As 60. c. As 80. d. As 200. ANS: C
The difference between the systolic pressure and the diastolic pressure is the pulse pressure. For a blood pressure of 140/60, the pulse pressure is 80 (140 – 60). The systolic pressure is 140. The diastolic pressure is 60. The systolic pressure (140) added to the diastolic pressure (60) is 200, but this has no clinical significance. DIF: Understand REF: 557 OBJ: Accurately record and report vital sign measurements. MSC: CPNRE: Foundations of Practice
TOP: Implementation
18. The thickness or viscosity of the blood affects the ease with which blood flows through small
vessels. Examining what value might help determine the amount of blood viscosity? a. Hematocrit. b. Cardiac output. c. Arterial size. d. Blood volume. ANS: A
The hematocrit, or the percentage of red blood cells in the blood, determines blood viscosity. Blood pressure also depends on the cardiac output or volume pumped by the heart, but cardiac output does not affect viscosity. Differences in arterial size help modify blood pressure; the smaller lumen of a vessel incN reas ular esC istan UResSvIasc NG TBr. OMce but does not affect viscosity. Blood volume also affects blood pressure, but it does not directly affect viscosity. DIF: Understand REF: 557 OBJ: Explain the physiology of normal regulation of blood pressure, pulse, oxygen saturation, and respirations. TOP: Assessment MSC: CPNRE: Foundations of Practice 19. The patient is being admitted to the medical unit with complaints of shortness of breath. The
patient has had chronic lung disease for many years but still smokes. What should the nurse do? a. Administer high levels of oxygen. b. Use oxygen cautiously in this patient. c. Place a paper bag over the patient’s face to allow rebreathing of carbon dioxide. d. Administer CO2 via mask. ANS: B
Because low levels of arterial O2 provide the stimulus that allows the patient to breathe, administration of high oxygen levels will be fatal for patients with chronic lung disease. Oxygen must be used cautiously in such patients. Patients with chronic lung disease have ongoing hypercarbia (elevated CO2 levels) and do not need to have CO2 administered or “rebreathed.” DIF: Apply REF: 549 OBJ: Explain the physiology of normal regulation of blood pressure, pulse, oxygen saturation, and respirations. TOP: Implementation
Canadian Fundamentals of Nursing 6th Edition Potter Test Bank MSC: CPNRE: Foundations of Practice 20. Which statement is true of the ovulation phase? a. Progesterone levels are below normal. b. Body temperature is below baseline levels. c. Body temperature is at previous baseline levels or higher. d. Intense body heat and sweating occur. ANS: C
Progesterone levels rise and fall cyclically during the menstrual cycle. When progesterone levels are low, the body temperature is a few tenths of a degree below the baseline. The lower temperature persists until ovulation occurs. During ovulation, greater amounts of progesterone enter the circulatory system and raise the body temperature to previous baseline levels or higher. These temperature variations help to predict a woman’s most fertile time, in order to achieve pregnancy. Women who undergo menopause (cessation of menstruation) often experience periods of intense body heat and sweating (hot flashes) lasting from 30 seconds to 5 minutes. DIF: Remember REF: 530 OBJ: Describe factors that cause variations in body temperature, pulse, oxygen saturation, respirations, and blood pressure. TOP: Assessment MSC: CPNRE: Foundations of Practice 21. The nurse is caring for a patient who has a pulse rate of 44. His blood pressure is within
normal limits. In trying to determine the cause of the patient’s low heart rate, what would the nurse suspect? a. That the patient would have a fever. b. Possible hemorrhage or bN leU edRinSgI . NGTB.COM c. That the patient is taking calcium channel blockers or digitalis medications. d. Chronic obstructive pulmonary disease (COPD). ANS: C
Negative chronotropic drugs such as digitalis, beta-adrenergic agents, and calcium channel blockers can slow down pulse rate. Fever, bleeding, hemorrhage, and COPD all increase the body’s need for oxygen, which leads to an increased heart rate. DIF: Understand REF: 546 OBJ: Describe factors that cause variations in body temperature, pulse, oxygen saturation, respirations, and blood pressure. TOP: Assessment MSC: CPNRE: Foundations of Practice 22. The patient is being admitted to the medical unit. She has arm, hand, and leg edema; her
temperature is 35.3°C (95.6°F); and her hands are cold secondary to peripheral vascular disease, of which she has a history. It is reported that she has a latex allergy. To quickly measure the patient’s oxygen saturation, what should the nurse do? a. Attach a finger probe to the patient’s index finger. b. Place a nonadhesive sensor on the patient’s ear lobe. c. Attach a disposable adhesive sensor to the bridge of the patient’s nose. d. Place the sensor on the same arm that the electronic blood pressure cuff is on. ANS: B
Canadian Fundamentals of Nursing 6th Edition Potter Test Bank The nurse should measure oxygen saturation on the ear or bridge of the nose in an adult patient has a history of peripheral vascular disease. Sensors should not be attached to fingers, ears, or the bridge of the nose if area is edematous or skin integrity is compromised. Disposable adhesive probes should be avoided if the patient has latex allergy. The probe should not be attached to fingers that are hypothermic. The sensor should not be placed on the same extremity as the electronic blood pressure cuff because blood flow to finger will be temporarily interrupted when cuff inflates. DIF: Apply REF: 555 OBJ: Describe factors that cause variations in body temperature, pulse, oxygen saturation, respirations, and blood pressure. TOP: Planning MSC: CPNRE: Foundations of Practice 23. The patient is admitted with shortness of breath and chest discomfort. Which of the following
laboratory values could account for the patient’s symptoms? a. Hemoglobin level of 80 g/L. b. Hematocrit level of 45%. c. Red blood cell count of 5.0 million/mm3. d. Pulse oximetry value of 90%. ANS: A
The concentration of hemoglobin reflects the patient’s capacity to carry oxygen. Normal hemoglobin levels range from 10 to 18 g/100 mL (135 to 180 g/L) in men and from 12 to 16 g/100 mL (120 to 160 g/L) in women. A hemoglobin level of 8.0 (80 g/L) is low and indicates a decreased ability to deliver oxygen to meet bodily needs. All other values listed are considered normal. DIF: Analyze REF: 55N 0U | 5R 5S 1 INGTB.COM OBJ: Describe factors that cause variations in body temperature, pulse, oxygen saturation, respirations, and blood pressure. TOP: Implementation MSC: CPNRE: Foundations of Practice 24. Of the following blood pressure values, which would be considered as a high normal reading? a. 98/50 in a 7-year-old child. b. 115/70 in an infant. c. 135/85 in an older person. d. 120/80 in a middle-aged adult. ANS: C
An adult’s blood pressure tends to rise with advancing age. In most people, blood pressure should be lower than 140/90. Blood pressure greater than 140/90 is defined as hypertension. Blood pressure of 98/50 is normal for a child, and 115/70 can be normal for an infant. DIF: Evaluate REF: 557 OBJ: Describe factors that cause variations in body temperature, pulse, oxygen saturation, respirations, and blood pressure. TOP: Evaluate MSC: CPNRE: Foundations of Practice 25. A nurse is caring for a patient who smokes and drinks caffeine. What is important for the
nurse to understand before she assesses the patient’s blood pressure (BP)? a. Neither caffeine nor smoking affects blood pressure. b. That the patient must stop smoking for at least 3 hours.
Canadian Fundamentals of Nursing 6th Edition Potter Test Bank c. That the patient should perform mild exercises. d. Caffeine and smoking can cause false BP elevations. ANS: D
Smoking immediately increases BP, and this increase lasts up to 15 minutes. Caffeine increases BP for up to 3 hours. Both affect a patient’s blood pressure. The patient should rest at least 5 minutes before BP is measured. DIF: Understand REF: 561 TOP: Implementation
OBJ: Describe ethnic variations in blood pressure. MSC: CPNRE: Foundations of Practice
26. When taking the pulse of an infant, the nurse notices that the rate is 145 beats/min and the
rhythm is regular. The nurse realizes that this rate is which of the following? a. Normal for an infant. b. The proper rate for a toddler. c. Too slow for an infant. d. The same as that of a normal adult. ANS: A
The normal rate for an infant is 120 to 160 beats/min. The rate obtained (145 beats/min) is within the normal range for an infant. The normal rate for a toddler is between 90 and 140 beats/min. The normal rate for an adult is between 60 and 100 beats/min. DIF: Remember REF: 547, Table 30-3 OBJ: Identify ranges of acceptable vital sign values for an infant, a child, and an adult. TOP: Assessment MSC: CPNRE: Foundations of Practice 27. The nurse is caring for an eldN erlyRpatien that his temperature is 36°C (96.8°F). This I tGandBn.oCtes M
O patient is in which condition? U S N T a. Suffering from hypothermia. b. Expressing a normal temperature. c. Hyperthermic in relation to his age. d. Demonstrating the increased metabolism that accompanies aging. ANS: B
The average body temperature of older persons is approximately 36°C (96.8°F). This is not hypothermia or hyperthermia. Older persons have poor vasomotor control, reduced amounts of subcutaneous tissue, and reduced metabolism. The end result is lowered body temperature. DIF: Understand REF: 529 | 530 OBJ: Identify ranges of acceptable vital sign values for an infant, a child, and an adult. TOP: Assessment MSC: CPNRE: Foundations of Practice 28. When assessing the temperature of newborns and children, the nurse decides to utilize a
temporal artery thermometer. Why is this preferable to methods used for adults? a. It is accurate even when the forehead is covered with hair. b. It is not affected by skin moisture. c. It reflects rapid changes in radiant temperature. d. There is no risk of injury to patient or nurse. ANS: D
Canadian Fundamentals of Nursing 6th Edition Potter Test Bank The temporal artery thermometer is especially beneficial when used in premature infants, newborns, and children because there is no risk of injury to the patient or nurse. However, it is inaccurate when the head is covered or when hair is on the forehead, and measurement is affected by skin moisture such as diaphoresis or sweating. The temporal artery thermometer provides very rapid measurement and reflects rapid changes in core temperature, not radiant temperature. DIF: Understand REF: 537, Box 30-4 OBJ: Explain variations in technique used to assess an infant’s, a child’s, and an adult’s vital signs. TOP: Assessment MSC: CPNRE: Foundations of Practice 29. Which artery is the most appropriate for assessing the pulse of a small child? a. Radial. b. Femoral. c. Brachial. d. Ulnar. ANS: C
The brachial or apical pulse is the best site for assessing an infant’s or a young child’s pulse because other peripheral pulses are deep and difficult to palpate accurately. DIF: Apply REF: 542 OBJ: Explain variations in technique used to assess an infant’s, a child’s, and an adult’s vital signs. TOP: Implementation MSC: CPNRE: Foundations of Practice 30. The nurse is caring for a newborn in the hospital nursery. She notices that the infant is
breathing rapidly but is pink, warm, and dry. The nurse knows that the normal respiratory rate GiTnuBt. for a newborn is how many bN reU atR hsSpIeN rm e?COM a. 30 to 60. b. 25 to 32. c. 16 to 19. d. 12 to 20. ANS: A
The acceptable respiratory rate range for a newborn is 30 to 60 breaths per minute. An infant (6 months) is expected to have a rate between 30 and 50 breaths per minute. A toddler’s respiratory range is 25 to 32 breaths per minute. A child should breathe 20 to 30 times a minute. An adolescent should breathe 16 to 19 times a minute. An adult should breathe 12 to 20 times a minute. DIF: Remember REF: 553 OBJ: Explain variations in technique used to assess an infant’s, a child’s, and an adult’s vital signs. TOP: Assessment MSC: CPNRE: Foundations of Practice 31. In attempting to obtain oxygen saturation readings in a toddler, what should the nurse do? a. Place the sensor on the earlobe. b. Place the sensor on the bridge of the nose. c. Determine whether the toddler has a tape allergy. d. Ignore any variation between the oximeter pulse rate and the patient’s apical pulse
rate. ANS: C
Canadian Fundamentals of Nursing 6th Edition Potter Test Bank The nurse should determine whether the patient has latex allergy because disposable adhesive probes should not be used on patients with latex allergies. Sensors should not be placed on the earlobe or bridge of the nose in infants and toddlers because of skin fragility. Oximeter pulse rate and the patient’s apical pulse rate should be the same. Any difference necessitates re-evaluation of oximeter sensor probe placement and reassessment of pulse rates. DIF: Apply REF: 555 OBJ: Explain variations in technique used to assess an infant’s, a child’s, and an adult’s vital signs. TOP: Implementation MSC: CPNRE: Foundations of Practice 32. The nurse is preparing to assess the blood pressure of a 3-year-old. How should the nurse
proceed? a. Choose the cuff that says “Child” instead of “Infant.” b. Obtain the reading before the child has a chance to “settle down.” c. Use the diaphragm portion of the stethoscope to detect Korotkoff sounds. d. Explain to the child what the procedure will be. ANS: D
Preparing the child for the blood pressure cuff’s unusual sensation increases cooperation. Most children will understand the analogy of a “tight hug on your arm.” Different arm sizes require careful and appropriate cuff size selection. Do not choose a cuff on the basis of the name of the cuff. An “Infant” cuff is too small for some infants. Readings are difficult to obtain in restless or anxious infants and children. Allow at least 15 minutes for children to recover from recent activities and become less apprehensive. Korotkoff sounds are difficult to hear in children because of low frequency and amplitude. A pediatric stethoscope bell is often helpful. DIF: Apply REF: 56N 6URSINGTB.COM OBJ: Explain variations in technique used to assess an infant’s, a child’s, and an adult’s vital signs. TOP: Implementation MSC: CPNRE: Foundations of Practice 33. What is one benefit of using a stationary automatic blood pressure device? a. It fits over clothing. b. It is extremely reliable. c. It is the method of choice for irregular heart rhythms. d. It is more reliable when pressure is less than 90 mm Hg systolic. ANS: A
The cuff fits over clothing. However, the reliability of stationary machines is limited. Electronic blood pressure measurement is not recommended with irregular heart rates or when systolic blood pressure is less than 90 mm Hg. DIF: Remember REF: 568 OBJ: Describe the benefits and precautions involving self-measurement of blood pressure. TOP: Assessment MSC: CPNRE: Foundations of Practice 34. The nurse is caring for a patient who is being discharged from the hospital after being treated
for hypertension. The patient is instructed to measure his blood pressure three times a day and to keep a record of the readings. The nurse recommends that the patient purchase a portable electronic blood pressure device. What other instruction should the nurse give the patient? a. The patient can apply the cuff in any manner he chooses because the machine is
Canadian Fundamentals of Nursing 6th Edition Potter Test Bank designed to be used by nonprofessionals. b. The patient must not adjust his medication dosages without consulting the health
care provider. c. The cuff can be placed over clothing if necessary. d. The machine is accurate when blood pressures are low. ANS: B
Patients must be advised that because of possible inaccuracies in the blood pressure devices, they must not adjust their medication regimens without consulting their health care provider. Because of their sensitivity, improper cuff placement or movement of the arm causes electronic devices to give incorrect readings. Stationary blood pressure devices are often found in public places, and the cuff fits over clothing. The same is not true for portable devices. Electronic blood pressure measurement is not recommended when systolic blood pressure is less than 90 mm Hg. DIF: Understand REF: 568 | 569 OBJ: Describe the benefits and precautions involving self-measurement of blood pressure. TOP: Assessment MSC: CPNRE: Foundations of Practice 35. The nurse is caring for a patient who complains of feeling light-headed and “woozy.” The
nurse checks the patient’s pulse and finds that it is irregular. The patient’s blood pressure is 100/72. It was 113/80 an hour earlier. What should the nurse do? a. Call the physician immediately. b. Perform an apical/radial pulse assessment. c. Apply more pressure to the radial artery to assess the pulse. d. Use the thumb to detect the patient’s pulse. ANS: B
NURSINGTB.COM
If the pulse is irregular, the nurse should assess the apical/radial pulse to detect a pulse deficit. If pulse count differs by more than 2, a pulse deficit exists, which sometimes indicates alterations in cardiac output. The nurse needs to gather as much information as possible before calling the physician. The radial pulse is more accurately assessed with moderate pressure. Too much pressure occludes the pulse and impairs blood flow. Fingertips are the most sensitive parts of the hand to palpate arterial pulsations; the thumb has a pulsation of its own that interferes with accurate measurement. DIF: Apply REF: 544 | 547 | 548 OBJ: Identify when vital signs should be measured. MSC: CPNRE: Foundations of Practice
TOP: Implementation
36. Of the following patients, which one is the best candidate to have his temperature measured
orally? a. A 27-year-old postoperative patient with an elevated temperature. b. A teenaged boy who has just returned from outside “for a smoke.” c. An 87-year-old confused man with suspected hypothermia. d. A 20-year-old man with a history of epilepsy. ANS: A
Canadian Fundamentals of Nursing 6th Edition Potter Test Bank An elevated temperature needs to be evaluated, and no contraindication is described for this patient. Ingestion of hot/cold fluids or foods, smoking, or receiving oxygen by mask/cannula can necessitate delays in measuring oral temperature. Temperatures are not measured orally in patients who have just had oral surgery or trauma, have a history of epilepsy, or have shaking chills, and they should not be measured orally in infants, small children, or confused patients. DIF: Evaluate REF: 537, Box 30-4 OBJ: Identify when vital signs should be measured. MSC: CPNRE: Foundations of Practice
TOP: Assessment
37. The physician order reads “Lopressor (metoprolol), 50 mg PO daily. Do not give if systolic
blood pressure is less than 100 mm Hg.” The patient’s blood pressure is 92/66. The nurse does not give the medication; what else does the nurse do? a. Does not tell the patient what the blood pressure is. b. Documents only what the blood pressure was. c. Documents that the medication was not given owing to low blood pressure. d. Does not need to inform the health care provider that the medication was held. ANS: C
The nurse must document any interventions initiated as a result of vital sign measurement, such as withholding an antihypertensive drug. The nurse should inform the patient of the blood pressure value and the need for periodic reassessment of the blood pressure. Documenting the blood pressure only is not sufficient; any intervention must be documented as well. Abnormal findings must be reported to the nurse in charge or to the health care provider. DIF: Apply REF: 570 | 571 OBJ: Accurately record and reN poUrtRvS itaIl N sigGnT mB ea.suCreOmMents. MSC: CPNRE: Foundations of Practice
TOP: Implementation
38. After taking the patient’s temperature, the nurse documents the value and the route used to
obtain the reading. Why is this done? a. Temperatures are the same regardless of the route used. b. Temperatures vary depending on the route used. c. Temperatures are cooler when taken rectally than when taken orally. d. Axillary temperatures are higher than oral temperatures. ANS: B
Temperatures obtained vary depending on the site used. Rectal temperatures are usually 0.5°C (0.9°F) higher than oral temperatures, and axillary temperatures are usually 0.5°C (0.9°F) lower than oral temperatures. DIF: Understand REF: 532 OBJ: Accurately record and report vital sign measurements. MSC: CPNRE: Foundations of Practice
TOP: Implementation
39. When temperature assessment is required, which of the following cannot be delegated to an
unregulated care provider? a. Temperature measurement. b. Assessment of changes in body temperature. c. Selection of appropriate route and device.
Canadian Fundamentals of Nursing 6th Edition Potter Test Bank d. Consideration of factors that falsely raise temperature. ANS: B
The nurse is responsible for assessing changes in body temperature. The task of temperature measurement can be delegated. The nurse instructs an unregulated care provider to select the appropriate route and device to measure temperature and to consider specific factors that falsely raise or lower temperature. DIF: Understand REF: 527 | 532 OBJ: Appropriately delegate vital sign measurement to unregulated care providers. TOP: Implementation MSC: CPNRE: Collaborative Practice 40. The unregulated care provider is measuring vital signs and reports that a patient’s blood
pressure is abnormally low. What should the nurse do? a. Have the unregulated care provider retake the blood pressure. b. Ignore the report and have it rechecked at the next scheduled time. c. Remeasure the blood pressure herself and assess the patient’s condition. d. Have the unregulated care provider assess the patient’s other vital signs. ANS: C
The unregulated care provider should report abnormalities to the nurse, who should further assess the patient. The unregulated care provider should not remeasure the blood pressure or other vital signs because the nurse needs to assess the patient. The report cannot be ignored. DIF: Apply REF: 527 OBJ: Appropriately delegate vital sign measurement to unregulated care providers. TOP: Implementation MSC: CPNRE: Collaborative Practice
N R I G B.C M
41. The patient has new-onset restlU essnSessNandTconfusO ion. His pulse rate is elevated, as is his
respiratory rate. His oxygen saturation, however, is 94% according to the portable pulse oximeter. The nurse ignores the oximeter reading and calls the physician to obtain an order for an arterial blood gas (ABG) measurement. The nursing instructor will provide further education to the practical nursing student who states that which of the following can cause inaccurate pulse oximetry readings? a. O2 saturations (SaO2) >70%. b. Carbon monoxide inhalation. c. Nail polish. d. Hypothermia at the assessment site. ANS: A
Inaccurate pulse oximetry readings can be caused by outside light sources, carbon monoxide (caused by smoke inhalation or poisoning), patient motion, jaundice, intravascular dyes (methylene blue), nail polish, artificial nails, metal studs, or dark skin. Other factors include peripheral vascular disease (atherosclerosis), hypothermia at the assessment site, pharmacological vasoconstrictors (e.g., epinephrine), low cardiac output, hypotension, peripheral edema, and tight probes. DIF: Understand REF: 556, Box 30-11 OBJ: Accurately assess tympanic, oral, rectal, and axillary temperatures. TOP: Assessment MSC: CPNRE: Foundations of Practice
Canadian Fundamentals of Nursing 6th Edition Potter Test Bank 42. The nurse is assessing the patient and his family for probable familial causes of the patient’s
hypertension. The nurse begins by analyzing the patient’s personal history, as well as family history and current lifestyle situation. Which of the following issues would be considered risk factors? a. Low blood cholesterol levels. b. European descent. c. Recent weight loss. d. Heavy alcohol consumption. ANS: D
Obesity, cigarette smoking, heavy alcohol consumption, high blood cholesterol and triglyceride levels, and continued exposure to stress are risk factors linked to hypertension. Weight loss and low blood cholesterol levels are not risk factors for hypertension. The incidence of hypertension is higher among ethnic groups such as South Asian, First Nations, Métis, and Inuit, and in people of African descent, not those of European descent. DIF: Analyze TOP: Evaluate
REF: 560 OBJ: Describe ethnic variations in blood pressure. MSC: CPNRE: Foundations of Practice
43. When recording the patient’s respiratory status, the nurse records the respiratory rate,
character of respirations, and which of the following? a. Only normal findings. b. Only in the graphic section. c. Amount of oxygen therapy. d. Korotkoff phase. ANS: C
Respiratory rate and characteN rU shR ouSldIbNeGrec or. deCdOinMnurses’ notes or on a vital sign flow TB sheet. Type and amount of oxygen therapy, if used during assessment, should be noted. After administration of specific therapies, respiratory assessment should be documented in narrative form in nurses’ notes. The nurse should document both normal and abnormal findings. Korotkoff phase is related to blood pressure assessment. DIF: Remember REF: 552 OBJ: Accurately record and report vital sign measurements. MSC: CPNRE: Professional, Ethical, and Legal Practice
TOP: Implementation
Canadian Fundamentals of Nursing 6th Edition Potter Test Bank
Chapter 31: Pain Assessment and Management Potter et al: Canadian Fundamentals of Nursing, 6th Edition MULTIPLE CHOICE 1. What is the most appropriate way to assess the pain of a patient who is oriented and has
recently had surgery? a. Assessing the patient’s body language. b. Observing the cardiac monitor for increased heart rate. c. Asking the patient to rate the level of pain. d. Asking the patient to describe the effect of pain on the ability to cope. ANS: C
Pain is a subjective measure. Therefore, the best way to assess a patient’s pain is to ask the patient to rate the pain. Nonverbal communication, such as body language, is not as informative for assessing pain, especially when the patient is oriented. Heart rate sometimes increases when a patient is in pain, but this sign is not specific to pain. Pain sometimes affects a patient’s ability to cope, but assessing the effect of pain on coping assesses the patient’s ability to cope; it does not assess the patient’s pain. DIF: Understand REF: 575 | 585 OBJ: Perform an assessment of a patient experiencing pain. MSC: CPNRE: Foundations of Practice
TOP: Assessment
2. A nurse is caring for a patient who recently had an abdominal hysterectomy and states that she
is experiencing severe pain. The patient’s blood pressure is 110/60, and her heart rate is 60. In N ppe RS I G inB.C M addition, the patient does not aU ar toNbeT any dOistress. Which response by the nurse is most therapeutic? a. “Your vital signs do not show that you are having pain; can you describe your pain?” b. “You do not look like you are in pain.” c. “OK, I will go get you some narcotic pain relievers immediately.” d. “What would you like to try to alleviate your pain?” ANS: D
The nurse must believe that a patient is in pain whenever the patient reports that he or she is in pain, even if the patient does not appear to be in pain. Whenever the patient reports pain, the nurse needs to collaborate with the patient to determine the best method of pain relief, whether it be medication, meditation, or repositioning. The nurse must be careful to not judge the patient based on the basis of vital signs or nonverbal communication and must not assume that the patient is seeking narcotics. The patient is a partner in pain management, so going to get narcotics to treat the pain without consulting with the patient first is not appropriate. DIF: Understand REF: 585 | 590 | 591 OBJ: Describe guidelines for selecting and individualizing pain interventions. TOP: Implementation MSC: CPNRE: Foundations of Practice 3. Which of the following statements made by a patient reflects that the patient understands the
relationship between the gate control theory of pain and the use of meditation to relieve pain? a. “Meditation controls pain by blocking pain impulses from coming through the
Canadian Fundamentals of Nursing 6th Edition Potter Test Bank gate.” b. “Meditation will help me sleep through the pain because it opens the gate.” c. “Meditation stops the occurrence of pain stimuli.” d. “Meditation alters the chemical composition of pain neuroregulators, which closes
the gate.” ANS: A
According to the gate theory, pain impulses cause pain when they get through gates that are open. Pain is blocked when the gates are closed. Nonpharmacological pain relief measures, such as meditation, work by closing the gates, which keeps pain impulses from coming through. Meditation does not open pain gates or stop pain from occurring. Meditation also does not have an effect on pain neuroregulators. DIF: Evaluate TOP: Evaluate
REF: 578 OBJ: Describe the physiology of pain. MSC: CPNRE: Foundations of Practice
4. A nursing student is planning care for an older patient who is experiencing pain. Which of the
following statements made by the nursing student prompts the nursing professor to clarify the nursing student’s knowledge? a. “Older patients often have difficulty determining what is causing their pain.” b. “It is safe to administer opioids to older persons as long as you start with small doses and frequently assess the patient’s response to the medication.” c. “As adults age, their ability to perceive pain decreases.” d. “Patients who have dementia probably experience pain, and their pain is not always well controlled.” ANS: C
Aging does not affect the abiN litU yR toSpI erN ceGivTeBp. aiC n.OSM ometimes older persons have difficulty interpreting their pain and determining its cause because multiple diseases and vague symptoms affect similar parts of the body. Opioids are safe to use in older persons as long as they are slowly titrated and the nurse frequently monitors the patient. Current evidence shows that patients with dementia probably experience unrelieved pain because their pain is difficult to assess. DIF: Evaluate REF: 581, Box 31-4 OBJ: Identify barriers to effective pain management. MSC: CPNRE: Foundations of Practice
TOP: Planning
5. The nurse is caring for two patients; both are having a hysterectomy. The first patient is
having the hysterectomy after a complicated birth. The second patient has uterine cancer. What will most likely influence the experience of pain for these two patients? a. Neurological factors. b. Competency of the surgeon. c. Meaning of pain. d. Postoperative support personnel. ANS: C
Canadian Fundamentals of Nursing 6th Edition Potter Test Bank The patient’s perception of pain is influenced by psychological factors, such as anxiety and coping, which in turn influence the patient’s experience of pain. Each patient’s experience is different. The degree and quality of pain perceived by a patient are related to the meaning of the pain. Neurological factors can interrupt or influence pain perception, but the question does not indicate that either of these patients is experiencing alterations in neurological function. The knowledge, attitudes, and beliefs of nurses, physicians, and other health care personnel about pain affect pain management but do not necessarily influence a patient’s pain perceptions. DIF: Understand TOP: Assessment
REF: 577 OBJ: Identify components of the pain experience. MSC: CPNRE: Foundations of Practice
6. The nurse anticipates administering an opioid fentanyl patch to which patient? a. A 15-year-old adolescent with a broken femur. b. A 30-year-old adult with cellulitis. c. A 50-year-old patient with prostate cancer. d. An 80-year-old patient with a broken hip. ANS: C
A fentanyl patch is an extended-relief opioid that provides pain relief for 24 hours a day. This is ideal for patients who have chronic severe pain, such as those who have cancer. The other patients are expected to experience acute pain. Therefore, they will probably benefit more from oral or intravenous (IV) opioids for short-term pain relief. DIF: Apply REF: 602 OBJ: Explain the various pharmacological approaches to treating pain. TOP: Implementation MSC: CPNRE: Foundations of Practice
NURSINGTB.COM
7. Which of the following statements made by the patient indicates to the nurse that teaching
about a patient-controlled analgesia (PCA) device has been effective? a. “This is the only pain medication I will need to be on.” b. “I can administer the pain medication as frequently as I need to.” c. “I feel less anxiety about the possibility of overdosing.” d. “I will need the nurse to notify me when it is time for another dose.” ANS: C
A PCA device allows the patient to determine the level of pain relief delivered, which reduces the risk of oversedation. Its use often eases anxiety because the patient is not reliant on the nurse for pain relief. Other medications, such as oral analgesics, can be given in addition to the PCA device. The PCA device does have a minimum time limit to prevent overdose, but the patient can lengthen the amount of time between doses. One benefit of PCA is that the patient does not need to rely on the nurse to administer pain medication; the patient determines when to take the medication. DIF: Evaluate REF: 598 | 599 OBJ: Explain the various pharmacological approaches to treating pain. TOP: Implementation MSC: CPNRE: Foundations of Practice 8. A nurse is caring for a patient who is experiencing pain after abdominal surgery. What
information is important for the nurse to tell the patient when providing patient education about effective pain management?
Canadian Fundamentals of Nursing 6th Edition Potter Test Bank a. “To prevent overdose, you need to wait to ask for pain medication until you begin
to experience pain.” b. “You should take your medication after you walk to make sure you do not fall
while you are walking.” c. “We should work together to create a regular schedule of medications that does not allow for breakthrough pain.” d. “You need to take oral pain medications when you experience severe pain.” ANS: C
The best way to manage pain is to develop a schedule of medications that are given around the clock to prevent breakthrough pain. The nurse should not wait until the patient is in pain because it takes medications 10 to 30 minutes to begin to relieve pain. The nurse administers pain medications before painful activities, such as walking, and administers them intravenously when pain is severe. DIF: Apply REF: 591 OBJ: Discuss nursing implications for administering analgesics. TOP: Implementation MSC: CPNRE: Foundations of Practice 9. A nurse is caring for a patient who recently had spinal surgery. The nurse knows that patients
usually experience acute pain after this type of surgery. The patient refuses to get up and walk and is not moving around in the bed. However, the patient is stoic and denies experiencing pain at this time. What most likely explains this patient’s behaviour and response to surgery? a. The surgery successfully cured the patient’s pain. b. The patient’s culture is possibly influencing the patient’s experience of pain. c. The patient is experiencing urinary retention because of manipulation of the spine during surgery; this is preventing the patient from experiencing pain. N R bel IN G about B.CpOainM to influence pain management at d. The nurse is allowing persoUnalS iefs T this time. ANS: B
A patient’s culture often influences the patient’s expression of pain. In this case, the patient has just had surgery, and the nurse knows that this surgical procedure usually causes pain. It is important at this time for the nurse to examine cultural and ethnic factors that are possibly affecting the patient’s denial of pain at this time. Even if surgery corrects neurological factors that create chronic pain, surgery causes pain in the acute period. Urinary retention usually creates pain and does not mask surgical pain. The nurse is not allowing personal beliefs to influence pain management because the nurse is attempting to determine the reason why the patient is not verbalizing the experience of pain. DIF: Apply REF: 583 | 584 OBJ: Explain how cultural factors influence the pain experience. TOP: Assessment MSC: CPNRE: Foundations of Practice 10. A nurse is providing discharge teaching for a patient with a fractured humerus. The patient is
going home with oxycodone–acetaminophen (Percocet). What important patient education does the nurse provide? a. “Be sure to eat a meal high in fat before taking the medication, to avoid a stomach ulcer.” b. “Narcotics can be addictive, so do not take them unless you are in severe pain.” c. “You need to drink plenty of fluids and eat a diet high in fibre.”
Canadian Fundamentals of Nursing 6th Edition Potter Test Bank d. “As your pain severity lessens, you will begin to give yourself once-daily
intramuscular injections.” ANS: C
A common side effect of opioid analgesics is constipation. Therefore, the nurse encourages the patient to drink fluids and eat fibre to prevent constipation. Although medications can be irritating to the stomach, a diet high in fat does not prevent gastric ulcers. To best manage pain, the patient needs to take pain medication before painful procedures or activities or before pain becomes severe. As the patient’s pain gets better, the strength of the medications will decrease. Intramuscular, IV, and topical analgesics are used for more severe and chronic pain. DIF: Analyze REF: 592 | 597 | 598 OBJ: Discuss nursing implications for administering analgesics. TOP: Implementation MSC: CPNRE: Foundations of Practice 11. A patient arrives at the emergency department experiencing a headache and rates the pain as 7
on a 0-to-10 pain scale. What nonpharmacological intervention does the nurse implement for this patient while awaiting orders for pain medication from the health care provider? a. Frequently reassessing the patient’s pain scores. b. Reassuring the patient that the provider will come to the emergency department soon. c. Softly playing music that the patient finds relaxing. d. Teaching the patient how to do yoga. ANS: C
The appropriate nonpharmacological pain management intervention is to quietly play music that the patient finds relaxingN .U MR usic SIdiv NGerts TBa.pCerso OMn’s attention away from pain and creates relaxation. The patient’s pain scores are reassessed during evaluation. Building the patient’s expectation of the provider’s arrival does not address the patient’s pain. Although yoga promotes relaxation, nurses teach relaxation techniques only when a patient is not experiencing acute pain. Because the patient is having acute pain, this is not an appropriate time to provide patient teaching. DIF: Apply REF: 595 OBJ: Describe applications for the use of nonpharmacological pain interventions. TOP: Implementation MSC: CPNRE: Foundations of Practice 12. A patient who has had type 2 diabetes for 26 years is beginning to experience peripheral
neuropathy in the feet and lower legs, which is causing the patient to have a decreased ability to feel pain in the lower extremities. The nurse is providing education to the patient to prevent injury to the feet. The nurse tells the patient to always wear shoes or slippers when walking. Which of the following statements made by the nurse best explains the rationale for this instruction? a. “Wearing shoes blocks pain perception and helps you adapt to pain, which ends up protecting your feet.” b. “Shoes provide nonpharmacological pain relief to people with diabetes and peripheral neuropathy.” c. “Since you cannot feel pain as much in your feet, you need to open your neurological gates to allow pain sensations to come through. Wearing shoes helps to open those gates, which protects your feet.”
Canadian Fundamentals of Nursing 6th Edition Potter Test Bank d. “You have lost the ability to withdraw from pain because of your peripheral
neuropathy. If you step on something and are not wearing shoes, you will not feel it; this could possibly cause injury to your foot.” ANS: D
This patient is losing the ability to feel pain because of peripheral neuropathy. The patient will no longer have protective reflexes to prevent injury to the feet. Wearing shoes prevents the patient from injuring the feet because they protect the feet. Shoes do not block pain perception or help people adapt to pain. Shoes are not a form of nonpharmacological pain relief. Wearing shoes will not have an effect on opening or closing the pain gates. DIF: Apply REF: 576-578 TOP: Implementation
OBJ: Describe the physiology of pain. MSC: CPNRE: Foundations of Practice
13. A nurse is assessing a patient who started to have severe pain 3 days ago. When the nurse asks
the patient to describe the pain, the patient states, “The pain feels like it is in my stomach. It is a burning pain, and it spreads out in a circle around the spot where it hurts the most.” What type of pain does the nurse document that the patient is having at this time? a. Superficial pain. b. Idiopathic pain. c. Chronic pain. d. Visceral pain. ANS: D
Visceral pain comes from visceral organs, such as those from the gastrointestinal tract. Visceral pain is diffuse and radiates in several directions. Superficial pain has a short duration and is usually a sharp pain. Pain of an unknown cause is idiopathic pain. Chronic pain lasts NURSINGTB.COM longer than 6 months. DIF: Analyze REF: 587, Table 31-5 OBJ: Perform an assessment of a patient experiencing pain. MSC: CPNRE: Foundations of Practice
TOP: Assessment
14. A patient who had a motor vehicle accident 2 days ago is experiencing pain and is receiving
patient-controlled analgesia (PCA). How does the nurse know that the patient is experiencing effective pain management with the PCA? a. The patient is sleeping and is difficult to arouse. b. The patient rates pain at an acceptable level of 3 on a 0-to-10 scale. c. Sufficient medication is left in the PCA syringe. d. The patient presses the control button to deliver pain medication. ANS: B
The effectiveness of pain relief measures is determined by the patient’s generally rating pain at the level of 3 or less on a 0-to-10 pain scale. If the patient is satisfied with the amount of pain relief, then pain measures are effective. A patient who is sleeping and is difficult to arouse is possibly oversedated; the nurse needs to assess this patient further. The amount of medication left in the PCA syringe does not indicate whether pain management is effective. Pressing the button shows that the patient knows how to use the PCA but does not help evaluate pain management. DIF: Evaluate TOP: Evaluate
REF: 598 | 599 OBJ: Evaluate a patient’s response to pain interventions. MSC: CPNRE: Foundations of Practice
Canadian Fundamentals of Nursing 6th Edition Potter Test Bank
15. The nurse recognizes that which of the following is a modifiable contributor to a patient’s
perception of pain? a. Age and gender. b. Anxiety and fear. c. Culture. d. Previous pain experience. ANS: B
The nurse can take measures to ease the patient’s anxiety and fear related to pain. Age, gender, culture, and previous pain experience are all nonmodifiable factors that the nurse can help the patient understand, but the nurse cannot alter them. DIF: Understand TOP: Assessment
REF: 583 OBJ: Identify components of the pain experience. MSC: CPNRE: Foundations of Practice
16. The nurse is evaluating the effectiveness of guided imagery for pain management as used for a
patient who has second- and third-degree burns and needs extensive dressing changes. Which situation best indicates that guided imagery is controlling the patient’s pain effectively during dressing changes? a. The patient’s need for analgesic medication decreases during the dressing changes. b. The patient rates pain during the dressing change as a 6 on a scale of 0 to 10. c. The patient’s facial expressions are stoic during the procedure. d. The patient can tolerate more pain, so dressing changes can be performed more frequently. ANS: A
The purpose of guided imageN ryUiR s to he.pC atie SIallo NGwTtB OMnt to alter the perception of pain. Guided imagery works in conjunction with analgesic medications, potentiating their effects. If the patient needs less pain medication during dressing changes, then guided imagery is helping to manage the patient’s pain. A rating of 6 on a 0-to-10 pain scale indicates that the patient is having moderate pain and is not experiencing pain relief at this time. A person who is stoic is not showing feelings, which makes it difficult to know whether the patient is experiencing pain. The ability to change dressings more frequently is not a way to evaluate the effectiveness of guided imagery. DIF: Evaluate TOP: Evaluate
REF: 594 OBJ: Evaluate a patient’s response to pain interventions. MSC: CPNRE: Foundations of Practice
17. A nurse is providing medication education to a patient who just started taking ibuprofen, a
nonselective nonsteroidal anti-inflammatory drug (NSAID). Which statement made by the nurse best indicates how ibuprofen works? a. “Ibuprofen helps to remove factors that cause or stimulate pain.” b. “Ibuprofen reduces anxiety, which will help you better cope with your pain.” c. “Ibuprofen helps to decrease the production of prostaglandins.” d. “Ibuprofen binds with opiate receptors to reduce your pain.” ANS: C
Canadian Fundamentals of Nursing 6th Edition Potter Test Bank NSAIDs such as ibuprofen probably work by decreasing the synthesis of prostaglandins to inhibit cellular responses to inflammation. Ibuprofen does not remove factors that cause pain, nor does it enhance coping with pain. Opioids bind with opiate receptors to modify perceptions of pain. DIF: Understand REF: 597 OBJ: Explain how the physiology of pain relates to selecting interventions for pain relief. TOP: Implementation MSC: CPNRE: Foundations of Practice 18. A nurse has brought the patient his scheduled pain medication. The patient asks the nurse to
wait to give pain medication until the time for the dressing change, which is 2 hours away. Which response by the nurse is most therapeutic? a. “This medication will still be providing you relief at the time of your dressing change.” b. “OK, swallow this pain pill, and I will return in a minute to fill your wound.” c. “Would you like medication to be given for dressing changes on top of your regularly scheduled medication?” d. “Your medication is scheduled for this time, and I can’t adjust the time for you. I’m sorry, but you must take your pill right now.” ANS: C
As-needed (prn) doses of medication can be given to patients in certain circumstances, as with an extensive dressing change. By asking to hold off on the dose, the patient is indicating that the dressing changes are extremely painful. The regularly scheduled dose might not be as effective for the patient. Oral medications take 30 to 60 minutes to take effect. If the nurse began the dressing change right then, the medication would not have been absorbed yet. The patient has the right to refuse to take a medication. It is the nurse’s responsibility to N Rd I G heB.C M communicate with the providerUanS wiN th tT patieO nt about a pain control plan that works for both. DIF: Evaluate REF: 598 OBJ: Explain various pharmacological approaches to treating pain. TOP: Implementation MSC: CPNRE: Foundations of Practice 19. A nurse receives an order from a health care provider to administer acetaminophen, 500 mg,
to a patient with moderate arthritis pain. The order is to give 2 tablets every 4 hours by mouth as needed for pain. The patient has a history of gastric ulcer. What is the nurse’s best next action? a. Give the acetaminophen to the patient immediately because the patient is experiencing pain. b. Ask the health care provider to verify the dosage and frequency of the medication. c. Ask the health care provider for an order for a nonsteroidal anti-inflammatory drug (NSAID). d. Ask the health care provider for an order to play music for the patient, in addition to providing the pain medication. ANS: B
Canadian Fundamentals of Nursing 6th Edition Potter Test Bank The maximum safe 24-hour dosage for acetaminophen is 4 g. If the patient took 2 500-mg tablets of acetaminophen every 4 hours, the patient would be ingesting 6 g of acetaminophen in 24 hours. This exceeds the safe dosage of acetaminophen, so the best action is to question this order. Giving the medication as ordered would possibly result in the patient taking more acetaminophen than what is considered a safe dose. Acetaminophen overdose can result in liver failure. NSAIDs can cause bleeding, especially in the gastrointestinal tract; therefore, NSAIDs are probably contraindicated in this patient. Implementing music therapy is a nursing intervention and is an independent nursing action; thus an order to start music therapy is not needed. DIF: Evaluate REF: 597 OBJ: Discuss nursing implications for administering analgesics. TOP: Implementation MSC: CPNRE: Foundations of Practice 20. The nurse knows that which technique is best for assessing pain in a child who is 4 years of
age? a. Asking the parents whether they think their child is in pain. b. Using the FACES scale. c. Asking the child to rate the level of pain on a 0-to-10 pain scale. d. Checking to see what previous nurses have charted. ANS: B
Assessing pain intensity in children requires special techniques. Young children often have difficulty expressing their pain. The FACES scale is used to assess pain in children who are verbal. Because most 4-year-olds are verbal, this is an appropriate scale to use with this child. Parents’ statement of pain is not an effective way to assess pain in children because children’s statements are the most important. The 0-to-10 pain scale is too difficult for a 4-year-old child N RSINGTB.COM to understand. Previous documU entation by nurses will indicate what the child’s pain has been but will not indicate the child’s current pain intensity. DIF: Understand TOP: Assessment
REF: 586-588 OBJ: Describe the components of pain assessment. MSC: CPNRE: Foundations of Practice
21. Which statement made by a nursing educator best explains why it is important for nurses to
determine a patient’s medical history and recent drug use? a. “Health care providers have a responsibility to prevent drug seekers from gaining access to drugs.” b. “This information is useful in determining what type of pain interventions will most likely be effective in providing pain relief.” c. “Some recreational drugs have pharmaceutical counterparts that may be more effective in managing pain.” d. “Getting this information gives the nurse an opportunity to provide patient teaching about drug abstinence.” ANS: B
Canadian Fundamentals of Nursing 6th Edition Potter Test Bank In providing effective pain management, it is important to understand the patient’s history, what drugs the patient has already tried, and what interventions work best or have negative actions. It is not the nurse’s responsibility to judge or question a patient’s pain or label the patient as a “drug seeker.” Nurses need to avoid labelling patients as drug seekers because this term is poorly defined and creates bias and prejudice among other health care providers. Although certain recreational drugs do have pharmaceutical counterparts, this is not the sole purpose of assessing drug use. The nurse needs more information beyond a patient’s medical and medication history to determine whether a patient needs teaching about drug abstinence. DIF: Evaluate REF: 585 | 603 | 604 OBJ: Identify barriers to effective pain management. MSC: CPNRE: Foundations of Practice
TOP: Assessment
22. A nurse is supervising a student who is caring for a patient with chronic pain. Which
statement by the student indicates an understanding of pain management? a. “This patient says her pain is a 5, but she is not acting like it. I am not going to give her any pain medication.” b. “The patient is sleeping, so I pushed her PCA button for her.” c. “I need to reassess the patient’s pain 1 hour after administering oral pain medication.” d. “It wasn’t time for the patient’s medication, so when she requested it, I gave her a placebo.” ANS: C
Because the action of oral medications usually peaks in about an hour, the nurse needs to reassess the patient’s pain within an hour of administration. Nurses must believe any patient report of pain, even if nonverbal communication is not consistent with pain ratings. The Nshou RSldIpNush GTthe B.C M patient is the only person who U PCOA button. Pushing the PCA when a patient is sleeping is dangerous and may lead to narcotic overdose or respiratory depression. Giving the patient a placebo and telling her it is her medication is unethical. DIF: Evaluate REF: 598 OBJ: Discuss nursing implications for administering analgesics. TOP: Evaluate MSC: CPNRE: Foundations of Practice 23. The nurse is assessing how a patient’s pain is affecting mobility. Which assessment question
is most appropriate? a. “Have you considered working with a physiotherapist?” b. “What activities, if any, has your pain prevented you from doing?” c. “Would you please rate your pain on a scale from 0 to 10 for me?” d. “What effect does your pain medication typically have on your pain?” ANS: B
Because the nurse is interested in knowing whether the patient’s pain is affecting mobility, the priority assessment question is to ask the patient how the pain affects his or her ability to participate in normal activities of daily living. Although a physical therapist is a good resource to have, especially if pain is severely affecting mobility, considering working with a physical therapist does not describe the effect of pain on the patient’s mobility. Assessing quality of pain and effectiveness of pain medication does not help the nurse to understand how it is affecting the patient’s mobility.
Canadian Fundamentals of Nursing 6th Edition Potter Test Bank DIF: Understand REF: 584 | 585 OBJ: Perform an assessment of a patient experiencing pain. MSC: CPNRE: Foundations of Practice
TOP: Assessment
24. The nurse is teaching a student nurse about pain assessment scales. Which statement by the
student indicates correct understanding? a. “You cannot use a pain scale to compare the pain of my patient with the pain of your patient.” b. “When patients say they don’t need pain medication, they aren’t in pain.” c. “Pain assessment scales determine the quality of a patient’s pain.” d. “A patient’s behaviour is more reliable than the patient’s report of pain.” ANS: C
To gain a better understanding of a patient’s current pain status and to determine what interventions are needed, the nurse should assess both current and previous pain scores. A patient who rates pain at 4 might find the pain manageable if 24 hours earlier he had rated his pain at 10. Some patients do not express their pain or do not wish to take medications to relieve the pain. This does not mean they aren’t in pain; the nurse can try nonpharmacological therapies for this patient. DIF: Evaluate REF: 586 OBJ: Perform an assessment of a patient experiencing pain. MSC: CPNRE: Foundations of Practice
TOP: Assessment
25. The nurse is administering pain medication for several patients. Which patient does the nurse
administer medication to first? a. The patient who needs to take a scheduled dose of maintenance pain medication. IeNdG b. The patient who needs toNbU eR prS em icaTteBd.bC efO orM e walking. c. The patient with a PCA running who needs to have the syringe replaced. d. The patient who is experiencing pain rated 8 out of 10 and has a STAT order for pain medication. ANS: D
STAT medications need to be given as soon as possible. In addition, this patient is the priority because of the report of severe pain. The other patients need pain medication, but their situations are not as high a priority as that of the patient with the STAT medication order. DIF: Analyze REF: 591 OBJ: Discuss nursing implications for administering analgesics. TOP: Implementation MSC: CPNRE: Foundations of Practice 26. The nurse is assessing a patient for opioid tolerance. Which finding supports the such a
finding? a. Increasingly higher doses of opioid are needed to control pain. b. The patient needed a substantial dose of naloxone (Narcan). c. The patient asks for pain medication close to the time it is due around the clock. d. The patient no longer experiences sedation from the usual dose of opioid. ANS: A
Canadian Fundamentals of Nursing 6th Edition Potter Test Bank In opioid tolerance, increasing doses of an opioid are needed to control pain. Naloxone (Narcan) is an opioid antagonist that is given to reverse the effects of opioid overdose. Taking pain medications regularly around the clock is an effective way to control pain. The pain medication for this patient is probably managing the patient’s pain effectively because the patient is not asking for the medication before it is due. A patient no longer experiencing a side effect of an opioid does not indicate opioid tolerance. DIF: Understand TOP: Assessment
REF: 602 OBJ: Evaluate a patient’s response to pain interventions. MSC: CPNRE: Foundations of Practice
27. A nurse is caring for a patient with rheumatoid arthritis who is now going to be taking 2
acetaminophen (Tylenol) tablets every 6 hours to control pain. Which part of the patient’s social history is the nurse most concerned about? a. Patient drinks 1 to 2 glasses of wine every night. b. Patient smokes 2 packs of cigarettes a day. c. Patient occasionally smokes marijuana. d. Patient takes antianxiety medications. ANS: A
The major adverse effect of acetaminophen is hepatotoxicity. Both alcohol and acetaminophen are metabolized by the liver; when taken together, they can cause liver damage. Smoking cigarettes and smoking marijuana are not healthy behaviours, but their effects on health are not affected by acetaminophen. Antianxiety medications can be taken with acetaminophen. DIF: Apply REF: 597 OBJ: Discuss nursing implications for administering analgesics. TOP: Assessment MSC: CPNRE: Foundations of Practice
NURSINGTB.COM
28. The nurse is caring for a patient who suddenly experiences chest pain. What is the nurse’s first
priority? a. Call the rapid response team. b. Ask the patient to rate and describe the pain. c. Raise the head of the bed. d. Administer pain relief medications. ANS: B
The nurse’s ability to establish a nursing diagnosis, plan and implement care, and evaluate the effectiveness of care depends on an accurate and timely assessment. The other responses are all interventions; the nurse cannot know which intervention is appropriate until the nurse completes the assessment, makes a nursing diagnosis, and plans care. DIF: Apply REF: 584 | 585 OBJ: Describe guidelines for selecting and individualizing pain interventions. TOP: Assessment MSC: CPNRE: Foundations of Practice 29. The nurse is caring for a patient who recently had surgery to repair a hernia. The patient’s pain
was rated 7 on a scale of 0 to 10 before pain medication was administered. One hour after receiving an oral opioid, the patient ranks his pain at 3. The patient asks the nurse why he isn’t receiving more pain medication. Which is the nurse’s best response? a. “This medication can be given only every 4 hours. It is not time for you to have any other pain medication right now.”
Canadian Fundamentals of Nursing 6th Edition Potter Test Bank b. “I will notify the health care provider to come perform an assessment if your pain
doesn’t improve in 30 minutes.” c. “If the pain becomes severe, we may need to transfer you to an intensive care
unit.” d. “It can take longer for oral pain medication to work, and your pain is going down. Let’s try boosting you up in bed and putting an ice pack on the incision to see if that helps.” ANS: D
The patient is responding well to the oral pain medication and it can take longer for oral medications to relieve pain. Trying nonpharmacological interventions as an addition to opioid medications is appropriate at this time. If nonpharmacological interventions combined with the oral opioid are ineffective, the nurse needs to notify the health care provider and ask for a change in the medication or for additional pain medication. Saying that the patient has to wait 4 hours for additional pain medication is inaccurate because the nurse needs to provide further nursing interventions if pain is not relieved at an acceptable level for the patient. Admission to an intensive care unit is not typically necessary to manage pain after surgery for a hernia. DIF: Apply REF: 598 OBJ: Describe guidelines for selecting and individualizing pain interventions. TOP: Implementation MSC: CPNRE: Foundations of Practice 30. Which of the following is the best way for the nurse to manage pain for a patient with chronic
pain from arthritis? a. Administering pain medication before any activity. b. Providing intravascular bolus as needed for breakthrough pain. c. Giving medications around the clock. B.C M N R I G d. Administering pain medicaU tionSonlN y wT hen nonOpharmacological measures have failed. ANS: C
When a patient with arthritis has chronic pain, the best way to manage pain is to take medication regularly throughout the day to maintain constant pain relief. “Before activity” is nonspecific, and the medication may not have time to work before activity. If the patient waits until having pain to take the medication, pain relief takes longer. Nonpharmacological measures are used in conjunction with medications unless requested otherwise by the patient. DIF: Understand REF: 579 | 598 OBJ: Describe guidelines for selecting and individualizing pain interventions. TOP: Implementation MSC: CPNRE: Foundations of Practice 31. A nurse is caring for a patient who fell on the ice and has connective tissue damage in the
wrist and hand. What type of pain is the patient experiencing? a. Visceral pain. b. Somatic pain. c. Peripherally generated pain. d. Centrally generated pain. ANS: B
Canadian Fundamentals of Nursing 6th Edition Potter Test Bank Somatic pain comes from bone, joint, or muscle. Visceral pain arises from the visceral organs such as the gastrointestinal tract and pancreas. Peripherally generated pain can be caused by polyneuropathies or mononeuropathies. Centrally generated pain results from injury to the central or peripheral nervous system. DIF: Remember REF: 579 | 580 OBJ: Perform an assessment of a patient experiencing pain. MSC: CPNRE: Foundations of Practice
TOP: Assessment
32. The nurse is caring for an infant in the pediatric unit. Which of the following is the most
accurate description of factors that will influence the perception and management of pain for this patient? a. Infants cannot tolerate analgesics because of an underdeveloped metabolism. b. Infants have an increased sensitivity to pain in comparison with older children. c. Pain cannot be accurately assessed in infants. d. Infants respond behaviourally and physiologically to painful stimuli. ANS: D
Infants cannot verbally express their pain, but they do express pain with behavioural cues and physiological indicators. Infants can tolerate analgesics, but proper dosing and close monitoring are essential. Infants and older children have the same sensitivity to pain. Pain can be assessed even though the neonate cannot verbalize; the nurse can observe behavioural clues. Nurses observe behavioural cues and physiological responses to assess pain in infants. DIF: Understand TOP: Assessment
REF: 581 | 589 OBJ: Identify components of the pain experience. MSC: CPNRE: Foundations of Practice
33. The nurse is administering ibN upUrR ofS enI(N AG dvTilB ). toC anOoMlder patient. Which of the following
assessment data would cause the nurse to withhold the medication? a. Patient states that last bowel movement was 4 days ago. b. Stated allergy to aspirin. c. Patient rates intermittent joint pain as 2 out of 10. d. Patient experienced respiratory depression after administration of an opioid medication. ANS: B
Patients with an allergy to aspirin are sometimes also allergic to other NSAIDs. The nurse needs to verify that the health care provider is aware of the allergy to aspirin before administering ibuprofen. NSAIDs do not interfere with bowel function and are used for the treatment of mild to moderate acute intermittent pain. NSAIDs also do not suppress the central nervous system. DIF: Understand REF: 597 OBJ: Explain the various pharmacological approaches to treating pain. TOP: Assessment MSC: CPNRE: Foundations of Practice
Canadian Fundamentals of Nursing 6th Edition Potter Test Bank
Chapter 32: Health Assessment and Physical Examination Potter et al: Canadian Fundamentals of Nursing, 6th Edition MULTIPLE CHOICE 1. A nurse is a preceptor for a nurse who just graduated from nursing school. When caring for a
patient, the new graduate nurse begins to explain to the patient the purpose of completing a physical assessment. Which of the following statements made by the new graduate nurse prompts the preceptor to intervene? a. “I will use the information from my assessment to figure out if your antihypertensive medication is working effectively.” b. “Nursing assessment data are used only to provide information about the effectiveness of your medical care.” c. “Nurses use data from their patient’s physical assessment to determine a patient’s educational needs.” d. “Information gained from physical assessment helps nurses better understand their patients’ emotional needs.” ANS: B
Nursing assessment data are used to evaluate the effectiveness of all aspects of a patient’s care, not just the patient’s medical care. Assessment data help the nurse evaluate the effectiveness of medications and determine a patient’s health care needs, including the need for patient education. Nurses also use assessment data to identify patients’ psychosocial and cultural needs. DIF: Evaluate REF: 61N 2 R I GOBB J:.DiC scM uss the purposes of physical assessment. U S N T O TOP: Communication and Documentation MSC: CPNRE: Foundations of Practice 2. For a weak patient with bilateral basilar pneumonia, which is the best position for a complete
geriatric physical examination? a. Prone position. b. Sims’s position. c. Supine position. d. Lateral recumbent position. ANS: C
The supine position is the most normally relaxed position. It will not further compromise the patient’s breathing. If the patient becomes short of breath easily, the head of the bed can be raised. This position would be easiest for weak older person to get into for an examination. Lateral recumbent and prone positions cause respiratory difficulty for any patient with respiratory difficulties. Sims’s position is used for assessment of the rectum and the vagina. DIF: Understand REF: 617, Table 32-3 OBJ: List techniques for preparing a patient physically and psychologically before and during an examination. TOP: Planning MSC: CPNRE: Foundations of Practice
Canadian Fundamentals of Nursing 6th Edition Potter Test Bank 3. During an annual gynecological examination, a college student discusses her upcoming
college break at a tropical location. After the student receives an oral contraceptive prescription, the nurse identifies the importance of skin cancer prevention education by discussing which evidence-informed prevention technique? a. Applying water-based sunscreen only before swimming. b. Using tanning bed daily for 7 days before college break trip. c. Applying broad-spectrum sunscreen of SPF 5. d. Taking extra precautions in the sun secondary to the prescription. ANS: D
Oral contraceptives can make the skin more sensitive to the sun. For this reason, the patient should be educated about the need for sun protection with such techniques as the use of wide-brimmed hats, use of broad-spectrum sunscreen of SPF 15 or greater, not tanning during midday, and not using tanning beds. Broad-spectrum sunscreens should be applied 15 minutes before a person goes out into the sun and after swimming or perspiring. DIF: Understand REF: 628, Box 32-9 OBJ: Discuss ways to incorporate health promotion and health teaching into the examination. TOP: Planning MSC: CPNRE: Foundations of Practice 4. A head and neck physical examination is completed on a 50-year-old woman. All physical
findings are normal except that she has fine, brittle hair. On the basis of the physical findings, which of the following laboratory tests would the nurse expect to be ordered? a. Liver function test. b. Lead level. c. Thyroid-stimulating hormone test. d. Complete blood cell count (CBC). G B.C M ANS: C
N R I U S N T
O
Thyroid disease can make hair thin and brittle. Liver function testing is indicated for a patient who has jaundice. Lead levels and a CBC are not indicated for the presence of brittle hair. DIF: Understand REF: 627 OBJ: Identify how nurses use physical assessment skills during routine nursing care. TOP: Planning MSC: CPNRE: Foundations of Practice 5. A febrile preschool-aged child presents to the after-hours clinic. Varicella is diagnosed on the
basis of the illness history and the presence of small, circumscribed skin lesions filled with serous fluid. The nurse documents the varicella lesions as which type of skin lesion? a. Vesicle. b. Wheal. c. Papule. d. Pustule. ANS: A
Vesicles are circumscribed, elevated skin lesions filled with serous fluid that are smaller than 1 cm in diameter. Wheals are irregularly shaped, elevated areas of superficial localized edema that vary in size. They are common with bug bites and hives. Papules are palpable, circumscribed, solid elevations in the skin that are smaller than 1 cm in diameter. Pustules are elevations of skin similar to vesicles, but they are filled with pus. DIF: Understand
REF: 626, Box 32-7
Canadian Fundamentals of Nursing 6th Edition Potter Test Bank OBJ: Describe physical measurements made in assessing each body system. TOP: Assessment MSC: CPNRE: Foundations of Practice 6. A school nurse recognizes a belt buckle–shaped ecchymosis on a 7-year-old student. When
privately asked about how the injury occurred, the student described falling on the playground. Upon suspecting abuse, the school nurse’s best next action is which of the following? a. Interviewing the patient in the presence of his/her teacher. b. Ignoring the findings because child abuse is a declining problem. c. Realizing that abuse victims usually report abusive situations. d. Contacting Social Services and reporting suspected abuse. ANS: D
Most provinces and territories mandate a report to a social service centre if nurses suspect abuse or neglect. When abuse is suspected, the nurse interviews the patient in private. Abuse of children, women, and older persons is a growing health problem. It is difficult to detect abuse because victims often will not complain or report that they are in an abusive situation. DIF: Apply REF: 620 OBJ: Identify how nurses use physical assessment skills during routine nursing care. TOP: Implementation MSC: CPNRE: Foundations of Practice 7. A nurse identifies Pediculosis humanus capitis. Considering the possible complications of
treatment, the nurse knows to not use which of the following treatment products? a. Fine-toothed comb. b. Pediculicide. c. Lindane-based shampoo. NURSINGTB.COM d. Vinegar hair rinse. ANS: C
Products containing lindane, a toxic ingredient, often cause adverse reactions and neurotoxic effects. Patients who have head lice are instructed to shampoo thoroughly with pediculicide (shampoo available at drugstores) in cold water, comb thoroughly with a fine-toothed comb, and discard the comb. A dilute solution of vinegar and water helps loosen nits. DIF: Apply REF: 629, Box 32-10 OBJ: Describe physical measurements made in assessing each body system. TOP: Implementation MSC: CPNRE: Foundations of Practice 8. A parent calls the school nurse with questions regarding the recent school vision screening.
Snellen chart examination revealed 20/60 for both eyes in the child. Considering the visual acuity results, what does the nurse informs the parent? a. That the child should have an optometric examination. b. That the child is suffering from strabismus. c. That the child may have presbyopia. d. That the child has vision issues probably because of cataracts. ANS: A
Canadian Fundamentals of Nursing 6th Edition Potter Test Bank Normal vision is 20/20. The larger the denominator, the poorer the patient’s visual acuity. For example, a value of 20/60 means that the patient, when standing 20 feet away, can read a line that a person with normal vision can read from 60 feet away. Strabismus is a (congenital) condition in which both eyes do not focus on an object simultaneously: these eyes appear crossed. Acuity may not be affected. Presbyopia is impaired near vision that occurs in middle-aged and older persons and is caused by loss of elasticity of the lens. Cataracts develop slowly and progressively after age 35 or suddenly after trauma. DIF: Apply REF: 634 OBJ: Identify preventive screenings and the appropriate age(s) for each screening to occur. TOP: Implementation MSC: CPNRE: Foundations of Practice 9. During a routine pediatric history documentation and physical examination, the parents report
that their child was a premature infant and was so small that he had to stay in the neonatal intensive care unit longer than usual. They state that the infant was yellow when born, and that he developed an infection that required “every antibiotic under the sun” to cure him. Considering the neonatal history, the nurse determines that it is especially important to perform which type of a focused examination? a. Cardiac. b. Respiratory. c. Ophthalmic. d. Hearing acuity. ANS: D
Risk factors for hearing problems include low birth weight, nonbacterial intrauterine infection, and excessively high bilirubin levels. Hearing loss caused by ototoxicity (injury to auditory nerves) can result from high maintenance doses of antibiotics. Cardiac, respiratory, and eye N essm RSen INtsGbT B.CotMrelevant to this child’s condition. examinations are important assU ut are nO DIF: Apply REF: 636, Table 32-13 OBJ: Identify data to collect from the nursing history before an examination. TOP: Implementation MSC: CPNRE: Foundations of Practice 10. During a presentation about sexually transmitted infections to high school students, the nurse
recommends the human papillomavirus (HPV) vaccine series to prevent which of the following? a. Cervical cancer. b. Genital lesions. c. Vaginal discharge. d. Swollen perianal tissues. ANS: A
HPV infection increases the risk for cervical cancer. HPV vaccine is recommended by the Public Health Agency of Canada for female patients aged 9 to 26 years. Vaginal discharge, painful or swollen perianal tissues, and genital lesions are signs and symptoms that may indicate a sexually transmitted infection. DIF: Understand REF: 664, Table 32-26 OBJ: Identify preventive screenings and the appropriate age(s) for each screening to occur. TOP: Implementation MSC: CPNRE: Foundations of Practice
Canadian Fundamentals of Nursing 6th Edition Potter Test Bank 11. A male student comes to the college health clinic. He hesitantly describes that his testis has a
pea-sized hard lump. The nurse recognizes this as a potential sign of which of the following? a. Inguinal hernia. b. Sexually transmitted infection. c. Testicular cancer. d. Diuretic use. ANS: C
Irregular lumps of the testes may indicate testicular cancer. Testicular cancer is cancer that begins in the testicles. Testicular cancer is the most common form of cancer in men between the ages of 15 and 35 years. A hernia manifests with bulging in the scrotum. Sexually transmitted infections often manifest with genital lesions. Use of diuretics, sedatives, or antihypertensives can cause difficulty in achieving erection or ejaculation but does not usually cause lumps. DIF: Understand REF: 667 OBJ: Identify self-screening examinations commonly performed by patients. TOP: Implementation MSC: CPNRE: Foundations of Practice 12. The nurse is urgently called to the gymnasium regarding an injured student. The student is
crying in severe pain with a malformed fractured lower leg. The proper sequence for the nurse’s initial assessment is a. Deep palpation, light palpation, inspection. b. Light palpation, deep palpation, inspection. c. Inspection, light palpation. d. Auscultation, deep palpation, light palpation. ANS: C
NURSINGTB.COM
Inspection is the use of vision and hearing to distinguish normal from abnormal findings. Light palpation determines areas of tenderness and skin temperature, moisture, and texture. Deep palpation is used to examine the condition of organs, such as those in the abdomen; it must be done with caution and is done after light palpation. Auscultation is used to evaluate sound. DIF: Apply REF: 613 | 614 OBJ: Demonstrate the techniques used with each physical assessment skill. TOP: Implementation MSC: CPNRE: Foundations of Practice 13. On admission, a patient weighs 113.4 kg (250 pounds). The weight is recorded as 116.1 kg
(256 pounds) on the second inpatient day. The nurse should evaluate the patient for which of the following? a. Fluid retention. b. Fluid loss. c. Decreased nutritional reserves. d. Anorexia. ANS: A
This patient has gained 2.7 kg (6 pounds) in a 24-hour period. A weight gain of 2.3 kg (5 pounds) or more in a day indicates fluid retention problems. A downward trend may indicate a reduction in nutritional reserves that may be caused by decreased intake such as anorexia or by fluid loss.
Canadian Fundamentals of Nursing 6th Edition Potter Test Bank
DIF: Apply REF: 620 OBJ: Identify how nurses use physical assessment skills during routine nursing care. TOP: Implementation MSC: CPNRE: Foundations of Practice 14. The patient is a 50-year-old African Canadian man who has come in for his routine annual
physical. Which of the following preventive screenings does the nurse recommend? a. Digital rectal examination of the prostate (DRE) annually. b. Blood test for cancer antigen 125 (CA-125) once a year. c. Complete eye examination every year. d. Colonoscopy every 3 years. ANS: A
Men need to have a digital rectal examination of the prostate every year beginning at 50 years of age. CA-125 blood tests are indicated for women at high risk for ovarian cancer. Because this patient is a man, the CA-125 test is not needed. Patients older than 65 need to have complete eye examinations yearly. Colonoscopy every 10 years is recommended in patients 50 years of age and older. DIF: Apply REF: 619, Table 32-4 OBJ: Discuss ways to incorporate health promotion and health teaching into the examination. TOP: Implementation MSC: CPNRE: Foundations of Practice 15. An older patient is being seen for a chronic entropion. The nurse realizes that entropion places
the patient at risk for which of the following? a. Ectropion. b. Infection. c. Exophthalmos. NURSINGTB.COM d. Strabismus. ANS: B
Entropion can cause the lashes of the lids to irritate the conjunctiva and cornea, and irritation can lead to infection. In ectropion, eyelid margins turn outward so that the lashes do not irritate the conjunctiva. Exophthalmos is a bulging of the eyes and usually indicates hyperthyroidism. Strabismus, or crossing of the eyes, results from neuromuscular injury or inherited abnormalities. DIF: Apply REF: 632 OBJ: Identify how nurses use physical assessment skills during routine nursing care. TOP: Implementation MSC: CPNRE: Foundations of Practice 16. During a school physical examination, the nurse reviews the patient’s current medical history,
which is positive for asthma, eczema, and allergic rhinitis. Which physical findings should the nurse expect on nasal examination? a. Polyp. b. Yellow discharge. c. Pale nasal mucosa. d. Puffiness of nasal mucosa. ANS: C
Canadian Fundamentals of Nursing 6th Edition Potter Test Bank Pale nasal mucosa with clear mucoid discharge indicates allergic rhinitis. Polyps are tumour-like growths. Yellow discharge would be seen with infection. Habitual use of intranasal cocaine and opioids causes puffiness and increased vascularity of the nasal mucosa. DIF: Analyze REF: 637 OBJ: Identify data to collect from the nursing history before an examination. TOP: Assessment MSC: CPNRE: Foundations of Practice 17. Objective physical data describe air moving through small airways, auscultated over the
lung’s periphery. The expected inspiratory-to-expiratory phase of this normal vesicular breath sound is which of the following? a. The inspiratory phase lasts exactly as long as the expiratory phase. b. The expiration phase is longer than the inspiration phase. c. The expiration phase is two times longer than the inspiration phase. d. The inspiratory phase is three times longer than the expiratory phase. ANS: D
Vesicular breath sounds are normal breath sounds heard over the lung’s periphery, caused by air moving through smaller airways; the inspiratory phase is three times longer than the expiratory phase. In bronchovesicular breath sounds, the inspiratory phase is equally as long as the expiratory phase. In bronchial breath sounds, the expiration phase longer than the inspiration phase at a 3:2 ratio. DIF: Analyze REF: 646, Table 32-18 OBJ: Discuss normal physical findings in young, middle-aged, and older persons. TOP: Assessment MSC: CPNRE: Foundations of Practice 18. A teenaged female patient reN poU rtR sS inI teN rmGitT teB nt.aC bdOoM minal pain that has lasted for 12 hours. No
dysuria is present. When performing an abdominal assessment, what should the nurse do? a. Recommend that the patient take more laxatives. b. Ask the patient about the colour of her stools. c. Avoid sexual references such as possible pregnancy. d. Palpate first the spots that are most tender. ANS: B
Black or tarry stools (melena) indicate gastrointestinal alteration. The nurse should caution patients about the dangers of excessive use of laxatives or enemas. The nurse should also determine whether the patient is pregnant and note her last menstrual period. Pregnancy causes changes in abdominal shape and contour. Painful areas should be assessed last, to minimize discomfort and anxiety. DIF: Apply REF: 661, Table 32-25 OBJ: Discuss normal physical findings in young, middle-aged, and older persons. TOP: Implementation MSC: CPNRE: Foundations of Practice 19. During a genitourinary examination of a 30-year-old man, the nurse identifies a small amount
of a white, thick substance on the patient’s uncircumcised glans penis. What is the nurse’s next step? a. Notify his provider about a suspected sexually transmitted infection (STI). b. Recognize this as a normal finding. c. Tell the patient to avoid doing self-examinations until symptoms clear.
Canadian Fundamentals of Nursing 6th Edition Potter Test Bank d. Avoid embarrassing questions about sexual activity. ANS: B
A small amount of thick, white smegma sometimes collects under the foreskin in uncircumcised boys and men. Penile pain or swelling, genital lesions, and urethral discharge are signs and symptoms that may indicate STI. All men 15 years and older need to perform a male-genital self-examination monthly. The nurse needs to assess a patient’s sexual history and use of safe sex habits. Sexual history reveals risks for STI and human immunodeficiency virus (HIV) infection. DIF: Apply REF: 666 OBJ: Discuss normal physical findings in young, middle-aged, and older persons. TOP: Implementation MSC: CPNRE: Foundations of Practice 20. In preparation for a rectal examination, a nonambulatory male patient is informed of the need
to be placed in which position? a. Sims’s position. b. Forward bending with flexed hips. c. Knee-chest. d. Dorsal recumbent. ANS: A
Nonambulatory patients are best examined in a side-lying Sims’s position. Forward bending would require the patient to be able to stand upright. Knees to chest would be difficult to maintain in a nonambulatory male and is embarrassing and uncomfortable. The dorsal recumbent position does not provide adequate access for a rectal examination and is used for abdominal assessment because it promotes relaxation of abdominal muscles.
NURSINGTB.COM
DIF: Understand REF: 617, Table 32-3 OBJ: List techniques used to prepare a patient physically and psychologically before and during an examination. TOP: Planning MSC: CPNRE: Foundations of Practice 21. The patient is asked what the statement “A stitch in time saves nine” means to him (a mental
status examination technique); what is it used to assess? a. Knowledge. b. Long-term memory. c. Abstract thinking. d. Recent memory. ANS: C
For an individual to explain common phrases such as “A stitch in time saves nine” requires a higher level of intellectual function. Knowledge-based assessment is factual. The nurse assesses knowledge by asking how much the patient knows about his illness or the reason for seeking health care. To assess memory, the patient is asked to recall the maiden name of the patient’s mother, a birthday, or a special date in history. It is best to ask open-ended questions rather than simple yes/no questions. Patients demonstrate immediate recall by repeating a series of numbers in the order in which they are presented or in reverse order. DIF: Understand REF: 675 OBJ: Describe physical measurements made in assessing each body system. TOP: Planning MSC: CPNRE: Foundations of Practice
Canadian Fundamentals of Nursing 6th Edition Potter Test Bank 22. During a routine physical examination of a 70-year-old patient, a blowing sound is auscultated
over the carotid artery. The nurse notifies the medical provider of the unexpected physical finding, which is known as which of the following? a. Clubbing. b. Bruit. c. Right-sided heart failure. d. Phlebitis. ANS: B
A bruit is the sound of turbulence of blood passing through a narrowed blood vessel. A bruit can reflect cardiovascular disease in the carotid artery of middle-aged to older persons. Clubbing of the fingers is due to insufficient oxygenation at the periphery as a result of conditions such as chronic emphysema and congenital heart disease. Jugular venous distension, not bruit, is a possible sign of right-sided heart failure. In some patients with heart disease, the jugular veins are distended when the patients sit. Phlebitis is an inflammation of a vein that occurs commonly after trauma to the vessel wall, infection, immobilization, and the need for prolonged insertion of intravenous catheters. It affects predominantly peripheral veins. DIF: Understand REF: 652 OBJ: Describe physical measurements made in assessing each body system. TOP: Planning MSC: CPNRE: Foundations of Practice 23. The nurse believes that several new female patients should receive additional health education
about the need for more frequent Papanicolaou (Pap) smears and gynecological examinations. Which of the following assessment findings reveals the patient at highest risk for cervical cancer and thus having the greatest need for patient education? NUe,RiSs I G xually B.CaOctive. M a. 13 years old, does not smok noNt seT b. 15 years old, is a social smoker, is celibate. c. 22 years old, smokes 1 pack of cigarettes per day, has multiple sexual partners. d. 50 years old, stopped smoking 30 years ago, has history of hysterectomy. ANS: C
Female patients considered to be at higher risk for cervical cancer include those who smoke and are older than 21 with weak immune systems, multiple sex partners, and a history of sexually transmitted infections. Of all the assessment findings listed, those of the 22-year-old smoker with multiple sexual partners include the greatest number of risk factors for cervical cancer. The other patients are at lower risk. DIF: Apply REF: 664, Table 32-26 OBJ: Discuss ways to incorporate health promotion and health teaching into the examination. TOP: Planning MSC: CPNRE: Foundations of Practice 24. A patient has been admitted to the medical unit after being involved in a motor vehicle
accident. On physical examination, the patient’s level of consciousness is reported as opening eyes to pain and responding with inappropriate words and flexion withdrawal to painful stimuli. The nurse correctly identifies the patient’s Glasgow Coma Scale score as which of the following? a. 5. b. 7. c. 9.
Canadian Fundamentals of Nursing 6th Edition Potter Test Bank d. 11. ANS: C
According to the guidelines of the Glasgow Coma Scale, the patient has a score of 9. Opening eyes to pain is 2 points; inappropriate word use is 3 points; and flexion withdrawal is 4 points. The total for this patient is 2 + 3 + 4 = 9. DIF: Apply REF: 674, Table 32-33 OBJ: Identify how nurses use physical assessment skills during routine nursing care. TOP: Assessment MSC: CPNRE: Foundations of Practice 25. While assessing the skin of an 82-year-old male patient, a nurse discovers nonpainful ruby red
papules on the patient’s trunk. What is the nurse’s next action? a. Explaining that the patient has basal cell carcinoma and should watch for spread. b. Documenting cherry angiomas as a normal geriatric skin finding. c. Telling the patient that he has a benign squamous cell carcinoma. d. Documenting the presence of edema. ANS: B
The skin is normally free of lesions, except for common freckles or age-related changes such as skin tags, senile keratosis (thickening of skin), cherry angiomas (ruby red papules), and atrophic warts. Basal cell carcinoma is most common in sun-exposed areas and frequently occurs in a background of sun-damaged skin; it almost never spreads to other parts of the body. Squamous cell carcinoma is more serious than basal cell and develops on the outer layers of sun-exposed skin; these cells may travel to lymph nodes and throughout the body. The nurse should report abnormal lesions to the health care provider for further examination. Edema is an area of skin that becomes swollen or edematous from a buildup of fluid in the tissues. This has nothing to dN oU wR ithScIhN erG ryTaB ng.ioCmOaM s. DIF: Apply REF: 625 OBJ: Discuss normal physical findings in young, middle-aged, and older persons. TOP: Assessment MSC: CPNRE: Foundations of Practice 26. During a preschool readiness examination, the nurse prepares to perform visual acuity
screenings. In view of the children’s age, the best equipment to test central vision is which of the following? a. Snellen test. b. E chart. c. Reading test. d. Penlight. ANS: B
The E chart is used when an individual is unable to read, as would be the case for a preschool-aged child. A Snellen chart and a reading test are too advanced for a preschooler’s education level. A penlight is used to check light perception; it shines a light into the eye, and then it is turned off. If the patient notes when the light is turned on or off, light perception is intact. DIF: Apply REF: 634 OBJ: Identify preventive screenings and the appropriate age(s) for each screening to occur. TOP: Assessment MSC: CPNRE: Foundations of Practice
Canadian Fundamentals of Nursing 6th Edition Potter Test Bank 27. A nurse suspects an abnormal thyroid shape during the physical examination. Why does the
nurse offer the patient a glass of water and watch her drinking as she drinks? a. To visualize an enlarged thyroid gland. b. To evaluate for exostosis. c. To test the patient’s gag reflex. d. To visualize the uvula and soft palate. ANS: A
This technique is used to visual an abnormally large thyroid gland. Normally, the thyroid cannot be visualized. An exostosis is a bony growth between the two palates that is noted when the oral cavity is examined. The patient’s gag reflex is tested by placing a tongue depressor on the posterior tongue. The uvula and soft palate are visualized with the use of a penlight. Both structures should rise centrally as the patient says, “Ah.” DIF: Understand REF: 642 OBJ: Demonstrate the techniques used with each physical assessment skill. TOP: Assessment MSC: CPNRE: Foundations of Practice 28. The patient is a 54-year-old man with a medium frame. He weighs 67.1 kg (148 pounds) and
is 173 cm (5 feet 8 inches) tall, with a body mass index (BMI) of 22 kg/m2. The nurse realizes that this patient is which of the following? a. Overweight. b. Underweight. c. At his desired weight. d. Obese. ANS: C
2 A normal BMI is 18.5 to 24.9Nkg . Th is T pB ati. enCt O isMat his desired weight. He is not UR/mSI NG overweight, underweight, or obese.
DIF: Remember REF: 622, Figure 32-1 OBJ: Discuss normal physical findings in young, middle-aged, and older persons. TOP: Assessment MSC: CPNRE: Foundations of Practice 29. A patient in the emergency department is complaining of left lower abdominal pain. The
comprehensive abdominal examination would include, in proper order, which of the following? a. Inspection, palpation, and auscultation. b. Percussion, inspection, and auscultation. c. Inspection, palpation, and percussion. d. Inspection, auscultation, and palpation. ANS: D
The order of an abdominal examination differs slightly from that of other assessments. It begins with inspection, followed by auscultation. Performing auscultation before palpation lessens the chance of altering the frequency and character of bowel sounds. DIF: Apply REF: 660 | 661 OBJ: Demonstrate the techniques used with each physical assessment skill. TOP: Assessment MSC: CPNRE: Foundations of Practice 30. What is the best term for breath sounds created by air moving through large lung airways?
Canadian Fundamentals of Nursing 6th Edition Potter Test Bank a. b. c. d.
Bronchovesicular. Rhonchi. Bronchial. Vesicular.
ANS: A
Bronchovesicular breath sounds are created by air moving through large airways. Vesicular sounds are created by air moving through smaller airways. Bronchial sounds are created by air moving through the trachea close to the chest wall. Rhonchi are abnormal lung sounds that are loud, low-pitched, rumbling coarse sounds heard during inspiration or expiration that sometimes clear by coughing. DIF: Remember REF: 646, Table 32-18 OBJ: Describe physical measurements made in assessing each body system. TOP: Assessment MSC: CPNRE: Foundations of Practice 31. The patient presents to the clinic with dysuria and hematuria. How does the nurse proceed to
assess for kidney inflammation? a. Lightly palpates each abdominal quadrant. b. Inspects abdomen for abnormal movement or shadows using indirect lighting. c. Uses deep palpation posteriorly. d. Percusses posteriorly the costovertebral angle at the scapular line. ANS: D
With the patient sitting or standing erect, direct or indirect percussion is used to assess for kidney inflammation. With the ulnar surface of the partially closed fist, the nurse percusses posteriorly the patient’s costovertebral angle at the scapular line. If the kidneys are inflamed, the patient feels tenderness dN urin syM stematic palpation approach for each U RgSpeIrcu N Gssio T Bn..ACO quadrant of the abdomen is used to assess for muscular resistance, distension, abdominal tenderness, and superficial organs or masses. Light palpation would not reveal kidney tenderness because the kidneys sit deep within the abdominal cavity. The lower ribs and heavy back muscles protect the kidneys, so the kidneys cannot be palpated posteriorly. Kidney inflammation does not cause abdominal movement; however, to inspect the abdomen for abnormal movement or shadows, the nurse should stand on the patient’s right side and inspect from above the abdomen, using direct light over the abdomen. DIF: Apply REF: 662 OBJ: Demonstrate the techniques used with each physical assessment skill. TOP: Assessment MSC: CPNRE: Foundations of Practice 32. The advanced practice nurse is conducting a comprehensive eye examination on an
80-year-old African Canadian woman. Which of the following findings requires the nurse to contact the patient’s physician for further examination? a. A thin white ring along the margin of the iris. b. A black pupil. c. Dilated pupils. d. A black fundus of the eye. ANS: C
Canadian Fundamentals of Nursing 6th Edition Potter Test Bank Dilatation of the pupils result from glaucoma, trauma, neurological disorders, eye medication, or withdrawal from opioids. Shining a beam of light through the pupil and onto the retina stimulates the third cranial nerve and causes the muscles of the iris to constrict. Any abnormality along the nerve pathways from the retina to the iris alters the ability of the pupils to react to light. A thin white ring along the margin of the iris, called an arcus senilis, is common with aging but is abnormal in anyone younger than 40. The pupils are normally black, round, regular, and equal in size. The fundus of African Canadian patients can be black. DIF: Apply REF: 633 OBJ: Discuss cultural diversity, cultural competency, and cultural safety as these relate to the provision of culturally competent health and physical assessment and improved patient health outcomes. TOP: Assessment MSC: CPNRE: Foundations of Practice 33. An elderly patient has been taking high doses of antibiotics and is experiencing a sudden loss
of hearing. The nurse should contact the health care provider and do what else? a. Stop antibiotic use until the physician responds. b. Tell the patient that older patients often lose low-frequency hearing. c. Explain that hearing loss usually occurs with thinning of the eardrum. d. Assure the patient that rapid hearing loss is normal in older people. ANS: A
Older persons are especially at risk for hearing loss caused by ototoxicity (injury to auditory nerve) resulting from high maintenance doses of antibiotics (e.g., aminoglycosides). Continuation of the medications is a physician responsibility. Older persons experience an inability to hear high-frequency sounds and consonants. Deterioration of the cochlea and thickening of the tympanic membrane cause older persons to gradually lose hearing acuity. DIF: Apply REF: 63N 6URSINGTB.COM OBJ: Identify how nurses use physical assessment skills during routine nursing care. TOP: Assessment MSC: CPNRE: Foundations of Practice 34. The patient has had a stroke that has affected her ability to speak, and she becomes extremely
frustrated when she tries to speak. She responds correctly to questions and instructions but cannot form words coherently. This patient is showing signs of which type of aphasia? a. Expressive. b. Receptive. c. Sensory. d. Combination. ANS: A
The two types of aphasia are sensory (or receptive) and motor (or expressive). The patient cannot express herself in words and is showing signs of expressive aphasia. She responds correctly to questions and instructions, indicating that she does not have receptive or sensory aphasia. Patients sometimes suffer a combination of receptive and expressive aphasia, but that is not the case with this patient. DIF: Understand REF: 675 OBJ: Describe physical measurements made in assessing each body system. TOP: Assessment MSC: CPNRE: Foundations of Practice
Canadian Fundamentals of Nursing 6th Edition Potter Test Bank 35. The school nurse is assessing the tympanic membranes of a 3-year-old. Which of the
following is a normal assessment finding? a. Presence of a yellow waxy substance. b. Swollen meatus. c. Discomfort with palpation of the auricle. d. Tenderness in the mastoid area. ANS: A
A yellow, waxy substance called cerumen is commonly present. Tenderness in the mastoid area indicates mastoiditis. The nurse should inspect the opening of the ear canal for size and presence of discharge. If discharge is present, the nurse should wear clean gloves during the examination. Swelling or occlusion of the meatus is not normal. Yellow or green, foul-smelling discharge indicates the presence of infection or a foreign body. DIF: Apply REF: 635 | 636 OBJ: Discuss normal physical findings in young, middle-aged, and older persons. TOP: Assessment MSC: CPNRE: Foundations of Practice
NURSINGTB.COM
Canadian Fundamentals of Nursing 6th Edition Potter Test Bank
Chapter 33: Infection Control Potter et al: Canadian Fundamentals of Nursing, 6th Edition MULTIPLE CHOICE 1. The patient and the nurse are discussing Rickettsia rickettsii, the cause of Rocky Mountain
spotted fever. Which patient statement to the nurse indicates understanding regarding the mode of transmission of this disease? a. “When I go camping, I will be sure to wear sunscreen.” b. “When I go camping, I will drink bottled water.” c. “When I go camping, I will be sure to wear insect repellent.” d. “When I go camping, I will be sure to use hand gel on my hands.” ANS: C
Each infectious disease has a specific mode of transmission: a component of the chain of infection. Rocky Mountain spotted fever is caused by bacteria transmitted by the bite of ticks. Wearing a repellent that is designed for repelling ticks, mosquitoes, and other insects can help in preventing transmission of this disease. Drinking plenty of uncontaminated water, wearing sunscreen, and using alcohol-based hand gels for cleaning hands are all important tasks to perform while camping, but they do not contribute to or prevent transmission of this disease. DIF: Understand REF: 685 OBJ: Explain the relationship between the chain of infection and transmission of infection. TOP: Evaluate MSC: CPNRE: Foundations of Practice 2. The nurse is providing an educational session for a group of preschool workers. The nurse
N R I G B.C M
reminds the group that the mosU t imS porN tantTthing toOdo to prevent the spread of infection is which of the following? a. Encourage preschool children to eat a nutritious diet. b. Encourage parents to provide a multivitamin to the children. c. Clean the toys every afternoon before putting them away. d. Perform hand hygiene between each interaction with children. ANS: D
The single most important thing that individuals can do to prevent the spread of infection is to perform hand hygiene before and after eating, going to the bathroom, changing a diaper, and wiping a nose, as well as after cleaning toys or tables, after picking up after the children, and between touching each individual child. It is important for preschool children to have a nutritious diet; a healthy individual can fight infection more effectively. A physician, along with the parent, makes decisions about dietary supplements. Cleaning the toys can decrease the number of pathogens but is not the most important thing to do in this scenario. DIF: Remember REF: 700 OBJ: Give an example of preventing infection for each element of the infection chain. TOP: Implementation MSC: CPNRE: Foundations of Practice 3. The nurse is admitting a patient with an infectious disease process. What question would be
appropriate for a nurse to ask this patient? a. “Do you have a chronic disease, and how long have you had it?” b. “Do you have any children living in the home?”
Canadian Fundamentals of Nursing 6th Edition Potter Test Bank c. “What is your marital status—single, married, or divorced?” d. “Do you have any cultural or religious beliefs that will influence your care?” ANS: A
Some factors increase the susceptibility of an individual to infection. These include age, nutritional status, presence of chronic disease, trauma, and smoking. The other questions are part of an admission assessment process but are not pertinent to the infectious disease process. DIF: Understand REF: 692 | 693 OBJ: Give an example of preventing infection for each element of the infection chain. TOP: Assessment MSC: CPNRE: Foundations of Practice 4. The patient underwent a surgical procedure, and povidone-iodine (Betadine) was utilized as
the surgical preparation. Two days postoperatively, the nurse’s assessment indicates that the incision is red and has a small amount of purulent drainage. The patient reports tenderness at the incision site. The patient’s temperature is 38.1°C (100.5°F), and the white blood cell (WBC) count is 10,500/mm3 (10.5 109/L). Which nursing action should the nurse take? a. Plan to change the surgical dressing during the shift. b. Check to see what solution was used for skin preparation in surgery. c. Collect supplies to culture the surgical incision. d. Communicate the patient’s needs to the health care provider. ANS: D
Organisms enter the body in several different ways. Proper skin preparation for surgery is essential for decreasing the chance of infection. The nursing assessment indicates signs and symptoms of infection, and the health care provider should be notified. Changing the dressing may be a need during the shift but is not a first priority. Checking the skin preparation used 2 days ago may or may not revN eaU lR usS efI ulNiG nfT orB m. atC ioO nM at this time. Collecting supplies for culture may be necessary after the nurse talks to the physician. DIF: Apply REF: 691 | 712 OBJ: Give an example of preventing infection for each element of the infection chain. TOP: Implementation MSC: CPNRE: Foundations of Practice 5. The nurse is providing an education session to an adult community group about the effects of
smoking. Which of the following is the most important point to be included in the educational session? a. Smoke from tobacco products clings to your clothing and hair. b. Smoking affects the cilia lining the upper airways in the lungs. c. Smoking tobacco products can be very expensive. d. Smoking can affect the colour of the patient’s fingernails. ANS: B
A normal defence mechanism against infection in the respiratory tract is the cilia lining the upper airways of the lungs and normal mucus. When a patient inhales a microbe, the cilia and mucus trap the microbe and sweep them up and out to be expectorated or swallowed. Smoking may alter this defence mechanism and increase the patient’s potential for infection. Smoking can be expensive, the smell does cling to hair and clothing, and the tar within the smoke can alter the colour of a patient’s nails. These points can be included in the education but do not constitute the most important information.
Canadian Fundamentals of Nursing 6th Edition Potter Test Bank DIF: Understand REF: 688 OBJ: Identify the body’s normal defences against infection. MSC: CPNRE: Foundations of Practice
TOP: Implementation
6. A woman presents to the clinic with reports of a white discharge and itching in the vaginal
area. During the health history, which of these questions should the nurse prioritize? a. “When was the last time you visited the physician?” b. “Has this condition affected your eating habits?” c. “What medications are you currently taking?” d. “Are you able to sleep at night?” ANS: C
The body contains normal flora (microorganisms) that live on the surface of skin, saliva, oral mucosa, gastrointestinal tract, and genitourinary tract. The normal flora of the vagina causes vaginal secretions to have a low pH. This inhibits the growth of many microorganisms. Antibiotics and oral contraceptives can disrupt normal flora in the vagina, which can lead to an overgrowth of Candida albicans in that area. It is important to ask the patient about current medications to obtain information that may assist with diagnosis. Visiting the physician is important for the patient’s health maintenance. Learning about the patient’s eating and sleeping habits will assist in the plan of care. DIF: Apply REF: 688 OBJ: Identify the body’s normal defences against infection. MSC: CPNRE: Foundations of Practice
TOP: Assessment
7. The nurse is caring for a school-aged child who has injured his leg after a bicycle accident. To
determine whether the child is experiencing a localized inflammatory response, the nurse .pCtoOmMs? should assess for which of thN esU eR siS gnIs N anGdTsB ym a. Fever, malaise, anorexia, and nausea and vomiting. b. Chest pain, shortness of breath, and nausea and vomiting. c. Dizziness and disorientation to time, date, and place. d. Edema, redness, tenderness, and loss of function. ANS: D
The body’s cellular response to an injury is seen as inflammation. Inflammation can be triggered by physical agents, chemical agents, or microorganisms. Signs of localized inflammation include swelling, redness, heat, pain or tenderness, and loss of function in the affected body part. Systemic signs of inflammation include fever, malaise, and anorexia, as well as nausea and vomiting. Chest pain, shortness of breath, and nausea and vomiting are signs and symptoms of a cardiac alteration. Dizziness and disorientation to time, date, and place may indicate a neurologic alteration. DIF: Remember TOP: Assessment
REF: 688 | 689 OBJ: Discuss the events in the inflammatory response. MSC: CPNRE: Foundations of Practice
8. Which interventions utilized by the nurse would indicate the ability to recognize the
inflammatory response? a. Rest, ice, compression, and elevation. b. Turn, cough, and deep breathe. c. Orient to date, time, and place. d. Passive range-of-motion exercises.
Canadian Fundamentals of Nursing 6th Edition Potter Test Bank ANS: A
One sign of the inflammatory response, particularly after an injury, is swelling or edema. Resting the affected injured area, using ice as ordered, wrapping the area to provide support—particularly if it is an extremity—and elevating the injured area will help decrease swelling or edema. Turning, coughing, and deep breathing are utilized for postoperative patients and for immobilized patients to help prevent an infectious process such as pneumonia. Orientation to date, time, and place is assessed in many patients who may be confused. Passive range of motion is utilized for individuals who need to improve movement of their extremities, including immobilized patients. DIF: Understand REF: 689 TOP: Implementation
OBJ: Discuss the events in the inflammatory response. MSC: CPNRE: Foundations of Practice
9. The nurse is caring for a group of medical-surgical patients. Which patient is most at risk for
developing an infection? a. The patient who is in observation for chest pain. b. The patient who is recovering from a right total hip arthroplasty. c. The patient who has been admitted with dehydration. d. The patient who has been admitted for stabilization of atrial fibrillation. ANS: B
The patient who is recovering from a right total hip arthroplasty has had a surgical procedure wherein bone was removed from the body and an implant was placed within the body. The patient has a large incision from surgery. The patient also has an intravenous infusion to provide fluids and medication. All these breaks in the skin increase the likelihood of infection. The patient has been given anaesthetics and medication for pain. Both of these depress the respiratory system and have the potential to decrease the expansion of alveoli and to increase Nespi RS I ryGsT B.CThe M other patients may have one break in the the chance of infection in the rU ratoN ystem. O skin when an intravenous infusion is used. DIF: Apply TOP: Assessment
REF: 688 | 692 OBJ: Identify patients most at risk for infection. MSC: CPNRE: Foundations of Practice
10. The nurse is caring for a patient with leukemia and is preparing to provide fluids through a
vascular access device. Which nursing intervention is priority in this procedure? a. Position the patient comfortably. b. Maintain aseptic technique. c. Gather available supplies. d. Review the procedure with the patient. ANS: B
Patients with disease processes of the immune system are at particular risk for infection. These diseases include leukemia, acquired immune deficiency syndrome (AIDS), lymphoma, and aplastic anemia. These disease processes weaken the defences against an infectious organism. Any time an intravenous device is accessed, aseptic technique must be maintained with wearing of appropriate personal protective equipment, preparation of the skin, and use of sterile gloves, sterile supplies, appropriate flushing, and appropriate discontinuation. Reviewing the procedure with the patient, positioning the patient, and gathering the supplies are all important steps in the procedure but are not the priority in the procedure. DIF: Apply
REF: 692 | 693
OBJ: Identify patients most at risk for infection.
Canadian Fundamentals of Nursing 6th Edition Potter Test Bank TOP: Implementation
MSC: CPNRE: Foundations of Practice
11. The nurse is caring for an adult patient in the clinic who is a victim of flooding and was
evacuated. The patient presents with signs and symptoms of a urinary tract infection. Along with needed education surrounding this diagnosis, the nurse teaches the patient about rest, exercise, eating properly, and how to utilize deep breathing and visualization. Which of these explanations would best support these nursing interventions? a. Urinary tract infections are painful, and these techniques would help with managing the pain. b. Interventions listed are standard topics taught during health care visits. c. Stress for long periods of time can lead to exhaustion and decreased resistance to infection. d. The patient requested this information to teach to extended family at home. ANS: C
The body responds to emotional or physical stress by the general adaptation syndrome. If stress extends for long periods of time, this can lead to exhaustion, whereby energy stores are depleted and the body has no defences against invading organisms. Techniques of deep breathing and visualization may be helpful with pain, but the interventions listed are not all standard interventions taught at every health care visit. DIF: Analyze TOP: Evaluate
REF: 692 OBJ: Identify patients most at risk for infection. MSC: CPNRE: Foundations of Practice
12. The nurse is caring for a patient who is susceptible to infection. Which of the following
nursing interventions will assist in decreasing the risk of infection? a. Teaching the patient about fall prevention. NUctRnSutri INtiou GTs B .CO b. Teaching the patient to sele foods . M c. Teaching the patient to take a temperature. d. Teaching the patient about the effects of alcohol. ANS: B
When protein intake is inadequate as a result of poor diet, the rate of protein breakdown exceeds that of tissue synthesis. A reduction in the intake of protein and other nutrients such as carbohydrates and fats reduces the body’s defences against infection and impairs wound healing. Teaching the patient about fall prevention, how to measure temperature, or about the effects of alcohol does not decrease the risk of infection. DIF: Apply REF: 692 TOP: Implementation
OBJ: Identify patients most at risk for infection. MSC: CPNRE: Foundations of Practice
13. A patient with diabetes presents to the clinic for a dressing change. The wound is on the right
foot and has purulent yellow drainage. Which of these interventions would be most appropriate for the nurse to provide? a. Position the patient comfortably on the stretcher. b. Explain the procedure for dressing change to the patient. c. Don gloves and other appropriate personal protective equipment. d. Review the medication list that the patient brought from home. ANS: C
Canadian Fundamentals of Nursing 6th Edition Potter Test Bank Localized infections are most common in the skin or with mucous membrane breakdown. The nurse should wear gloves and other personal protective equipment as appropriate when examining or providing treatment to localized infected areas. Positioning the patient, explaining the procedure, and reviewing the medication list are all tasks that need to be completed, but preventing the spread of infection takes precedence. DIF: Apply REF: 687 OBJ: Describe the signs and symptoms of a localized and a systemic infection. TOP: Implementation MSC: CPNRE: Foundations of Practice 14. Which of these interventions would take priority and should be included in a plan of care for a
patient who presents with pneumonia? a. Observe the patient for decreased activity tolerance. b. Assume that the patient is in pain and treat accordingly. c. Maintain the temperature at 18.3°C (65°F). d. Provide the patient ice chips as requested. ANS: A
Systemic infection causes more generalized symptoms than does local infection. This type of infection can result in fever, fatigue, nausea and vomiting, and malaise. The nurse should be alert for changes in the patient’s level of activity and responsiveness. Respiratory infection may result in a productive cough with purulent sputum, shortness of breath, and activity intolerance. Nurses do not assume but assess and communicate with the patient about pain, temperature, and ice chips. Asking these questions would not be a priority as much as assessing the patient and determining the effect that the systemic infection is having on the patient. DIF: Understand REF: 69N 4URSINGTB.COM OBJ: Describe the signs and symptoms of a localized and a systemic infection. TOP: Implementation MSC: CPNRE: Foundations of Practice 15. The infection control nurse is reviewing data for the medical-surgical unit. The nurse notices a
spike in postoperative infections on this unit and categorizes these health care–associated infection as which type? a. Iatrogenic. b. Exogenous. c. Endogenous. d. Nosocomial. ANS: B
An exogenous organism is one that is present outside the patient. A postoperative infection is an exogenous infection because the organism that has caused the infection originates from outside the body. An example is Staphylococcus aureus. An endogenous organism is part of the normal flora of residing virulent organisms that could cause infection. An endogenous infection can occur when part of the patient’s flora becomes altered, and overgrowth results. Iatrogenic infection results from a diagnostic or therapeutic procedure, such as a colonoscopy, that is conducted in a health care setting. Nosocomial infection is the term formerly used for health care–acquired infection. DIF: Remember REF: 689 OBJ: Explain conditions that promote the transmission of health care–associated infection.
Canadian Fundamentals of Nursing 6th Edition Potter Test Bank TOP: Evaluate
MSC: CPNRE: Foundations of Practice
16. Which of the following nursing actions would most increase a patient’s risk for developing a
health care–associated infection? a. Use of surgical aseptic technique to suction an airway. b. Urinary catheter drainage bag placed below the level of the bladder. c. Clean technique for inserting a urinary catheter. d. Use of a sterile bottled solution more than once within a 24-hour period. ANS: C
Using clean technique (medical asepsis) to insert a urinary catheter would place the patient at risk for a health care–associated infection. Urinary catheters need to be inserted with sterile technique, also referred to as surgical asepsis. This involves eliminating all microorganisms, including pathogens and spores, from an object or area. Placing a catheter into a sterile body cavity such as the bladder necessitates sterile technique. Sterile technique should also be used when an airway is suctioned because the airway is considered a sterile body cavity. Keeping the urinary catheter drainage bag below the bladder helps decrease the risk of developing a health care–associated infection because it prevents reflux of urine from the bag back into the bladder. Bottled solutions may be used repeatedly during a 24-hour period; however, special care is needed to ensure that the solution in the bottle remains sterile. After 24 hours, the solution should be discarded. DIF: Analyze REF: 697 | 712 OBJ: Explain conditions that promote the transmission of health care–associated infection. TOP: Evaluate MSC: CPNRE: Foundations of Practice 17. The nurse is dressed and is preparing to care for a patient in the perioperative area. The nurse
Mn and gloves. Which action would has scrubbed her hands and hN asUdRoS nnIeN dG aT steBri. leCgO ow indicate a break in sterile technique? a. Touching protective eyewear. b. Standing with hands folded on chest. c. Accepting sterile supplies from the surgeon. d. Staying with the sterile table once it is open. ANS: A
Once a nurse is gowned and gloved with sterile gown and gloves, touching nonsterile protective eyewear would indicated a break in sterile technique. Sterile objects remain sterile only when touched by another sterile object. Standing with hands folded on chest is common practice and prevents arms and hands from touching unsterile objects. Accepting sterile supplies from the surgeon who has opened them with the appropriate technique is acceptable. Staying with a sterile table once opened is a common practice to ascertain that no one or nothing has contaminated the table. DIF: Understand REF: 712 | 713 OBJ: Explain the difference between medical and surgical asepsis. TOP: Implementation MSC: CPNRE: Foundations of Practice 18. The nurse is caring for a patient with an incision. Which of the following actions would best
indicate an understanding of medical and surgical asepsis? a. Donning sterile gown and gloves to remove the wound dressing. b. Utilizing clean gloves to remove the dressing and sterile supplies for the new
Canadian Fundamentals of Nursing 6th Edition Potter Test Bank dressing. c. Donning clean goggles, gown, and gloves to dress the wound. d. Utilizing clean gloves to remove the dressing and clean supplies for the new
dressing. ANS: B
Clean gloves (medical asepsis) are used to remove contaminated dressings and sterile supplies, including gloves and dressings (surgical asepsis–sterile technique) to reapply sterile dressings. Wearing sterile gowns and gloves is not necessary when removing soiled dressings. Donning clean gloves to dress a sterile wound would contaminate the sterile supplies. Utilizing clean supplies for a sterile dressing would not help in decreasing the number of microbes at the incision site. DIF: Understand REF: 697 | 712 OBJ: Explain the difference between medical and surgical asepsis. TOP: Implementation MSC: CPNRE: Foundations of Practice 19. The nurse is caring for a patient in the endoscopy area. The nurse observes the technician
performing the following tasks. Which of these observations would require the nurse to intervene? a. Hand hygiene after removing gloves. b. Placing the endoscope in a container for transfer. c. Removing gloves to transfer the endoscope. d. Disinfecting endoscopes in the workroom. ANS: C
Routine practices are used to prevent and control the spread of infection. Transferring contaminated equipment withNout URthe SIpro NGtect TBio.nCofOgMloves can help spread microbes to inanimate objects and to the person doing the transfer. Utilizing gloves, washing hands, covering contaminated supplies during transfer, and disinfecting equipment in the appropriate way in the appropriate places reflect the principles of basic medical asepsis and routine practices and can break the chain of infection. DIF: Apply REF: 697 | 712 OBJ: Explain the rationale and components of routine practices. TOP: Implementation MSC: CPNRE: Foundations of Practice 20. The nurse is caring for a patient with a nursing diagnosis of Risk for infection. Aware of the
need for routine practices, what is the nurse careful to do? a. Teach the patient about good nutrition. b. Wear eyewear when emptying a urinary drainage bag. c. Avoid contact with intact skin without wearing gloves. d. Don gloves when wearing artificial nails. ANS: B
Routine practices include the wearing of eyewear whenever there is a possibility of a splash or splatter. Teaching the patient about good nutrition is positive but does not apply to routine practices. The term routine practices applies to all blood and body fluids except sweat, even if blood is not present. It also applies to nonintact skin and mucous membranes. DIF: Understand REF: 705 OBJ: Explain the rationale and components of routine practices.
Canadian Fundamentals of Nursing 6th Edition Potter Test Bank TOP: Implementation
MSC: CPNRE: Foundations of Practice
21. The nurse is caring for a patient who has just given birth. The nurse is checking the patient for
excessive vaginal drainage. It is important for the nurse to utilize which type of precautions? a. Contact precautions. b. Protective precautions. c. Droplet precautions. d. Routine practices. ANS: D
Routine practices apply to contact with blood, body fluid, nonintact skin, and mucous membranes of all patients. Contact precautions apply to individuals with colonization of infection such as methicillin-resistant S. aureus (MRSA). Protective precautions apply to individuals who have undergone transplantations. Droplet precautions focus on diseases that are transmitted by large droplets. DIF: Remember REF: 705 OBJ: Explain the rationale and components of routine practices. TOP: Implementation MSC: CPNRE: Foundations of Practice 22. The nurse is caring for a patient in the hospital. The nurse observes the unregulated care
provider turning off the handle faucet with his hands. What professional practice supports the need for follow-up with the unregulated care provider? a. The nurse is responsible for providing a safe environment for the patient. b. This is a key step in the procedure for hand hygiene. c. Allowing the water to run is a waste of resources and money. d. Different scopes of practice allow modification of procedures. ANS: A
NURSINGTB.COM
The nurse is responsible for providing a safe environment for the patient. The effectiveness of infection control practices depends on conscientiousness and consistency in using effective aseptic technique. It is human nature to forget key procedural steps or to take shortcuts. However, failure to comply with basic procedures places the patient at risk for infection that can impair recovery or lead to death. After hand hygiene, the health care provider should turn off a handle faucet with a dry paper towel and avoid touching the handles with his or her own hands to assist in preventing the transfer of microorganisms. Wet towels and hands allow the transfer of pathogens from faucet to hands. The principles and procedures for hand hygiene are universal and apply to all members of health care teams. Being resourceful and aware of the cost of health care is important, but taking shortcuts that may endanger an individual’s health is not prudent. DIF: Analyze TOP: Evaluate
REF: 701-703 OBJ: Perform proper procedures for hand hygiene. MSC: CPNRE: Foundations of Practice
23. The nurse is caring for a patient who becomes nauseated and vomits without warning. The
nurse has contaminated hands. What is the nurse’s best next step? a. Cleaning hands with wipes from the bedside table. b. Hand hygiene with an antimicrobial soap and water. c. Using an alcohol-based waterless hand gel. d. Instructing the patient to wash his face and hands.
Canadian Fundamentals of Nursing 6th Edition Potter Test Bank ANS: B
The U.S. Centers for Disease Control and Prevention recommend that when hands are visibly soiled, the health care provider should wash with a plain soap or with antimicrobial soap. Cleaning hands with wipes or using waterless hand gel does not meet this standard. If hands are not visibly soiled, an alcohol-based waterless antiseptic agent should be used for routinely decontaminating hands. The patient may very well need to wash his face and hands, but this is not the best next step. DIF: Apply TOP: Planning
REF: 700 OBJ: Perform proper procedures for hand hygiene. MSC: CPNRE: Foundations of Practice
24. The nurse is performing hand hygiene before assisting a physician with insertion of a chest
tube. While washing hands, the nurse touches the sink. What is the next action the nurse should take? a. Inform the physician and recruit another nurse to assist. b. Rinse and dry hands, and begin assisting the physician. c. Repeat hand hygiene with antiseptic soap. d. Extend the hand hygiene procedure to 5 minutes. ANS: C
The inside of the sink and the counter at the edges of the sink, faucet, and handles are considered contaminated areas. If the hands touch any of these areas during hand hygiene, the nurse should repeat the hand hygiene procedure with antiseptic soap. There is no need to inform the physician or be relieved of this assignment. If the hands are contaminated when touching the sink, drying hands and proceeding with the procedure could possibly contaminate materials, contribute to increased microbial counts during the procedure, and result in infection in the patient. Extending the time for washing the hands (although this is N RSINGTB.COM what will happen when the proU cedure is repeated) is not the focus. The focus is to repeat the whole hand hygiene procedure utilizing antiseptic soap. DIF: Apply REF: 701-703 TOP: Implementation
OBJ: Perform proper procedures for hand hygiene. MSC: CPNRE: Foundations of Practice
25. The nurse is observing a family member changing a dressing for a patient in the home
environment. Which of these observations would indicate that the family member has a correct understanding of how to manage contaminated dressings? a. The family member removes gloves and gathers items for disposal. b. The family member places the used dressings in a plastic bag. c. The family member saves part of the dressing because it is clean. d. The family member wraps the used dressing in toilet tissue before placing in the garbage. ANS: B
Contaminated dressings and other infectious items should be placed in impervious plastic or brown paper bags and then disposed of properly in garbage containers. Gloves should be worn during this process. Parts of the dressing should not be saved, even though they may seem clean, because microbes may be present. DIF: Evaluate REF: 711, Box 33-18 OBJ: Explain how infection-control measures in the home may differ from those in the hospital. TOP: Evaluate MSC: CPNRE: Foundations of Practice
Canadian Fundamentals of Nursing 6th Edition Potter Test Bank
26. The home health nurse is teaching a patient and family about hand hygiene in the home. The
nurse is sure to emphasize hand hygiene at what time? a. Before and after shaking hands. b. Before and after treatments. c. Before opening the refrigerator. d. Before and after using a computer. ANS: B
Patients should perform hand hygiene before and after treatments and when coming in contact with body fluids. Depending on the patient, holding hands does not necessitate washing of hands before, but hand hygiene afterwards is good practice, especially before touching eyes, nose, or mouth. Hand hygiene before and after opening the refrigerator and using the computer is not required, but during cold and flu season, it might be advisable. DIF: Apply REF: 711, Box 33-18 OBJ: Explain how infection-control measures in the home may differ from those in the hospital. TOP: Implementation MSC: CPNRE: Foundations of Practice 27. The nurse has been caring for a patient in the perioperative area for several hours. The
surgical mask the nurse is wearing has become moist. The nurse’s best next step is what? a. Change the mask when relieved. b. Air-dry the mask while at lunch, and reapply. c. Ask for relief, step out of the surgical area, and apply a new mask. d. Not change the mask, if the nurse is comfortable. ANS: C
A mask should fit snugly aroN und e an Rthe Ifac G Bd .noCse. MAfter the mask is worn for several hours, U S N T O it can become moist. The mask should be changed as soon as possible because moisture encourages the growth of microorganisms. Waiting to change the mask, air-drying it, and wearing it because it is comfortable do not support the principles of infection control. DIF: Apply REF: 709 OBJ: Properly don a surgical mask, sterile gown, and sterile gloves. TOP: Implementation MSC: CPNRE: Foundations of Practice 28. The nurse is caring for a patient for whom contact precautions are ordered. Which of the
following actions would be appropriate to prevent the spread of disease? a. Wearing a gown, gloves, facemask, and goggles for interactions with the patient. b. Using a dedicated blood pressure cuff that stays in the room and is used for that patient only. c. Placing the patient in a room with negative airflow. d. Transporting the patient quickly to the radiology department. ANS: B
Canadian Fundamentals of Nursing 6th Edition Potter Test Bank Contact precautions are a type of isolation precaution used for patients with illness that can be transmitted through direct or indirect contact. A patient is placed on contact precautions if a disease is present that can be transmitted through direct or indirect contact. Patients who are on contact precautions should have dedicated equipment wherever possible. This would mean, for example, that one blood pressure cuff and one stethoscope would stay in the room with the patient and would be used for that patient only. A gown and gloves may be required for interactions with a patient who is on contact precautions, but a facemask and goggles are not part of contact precautions. A room with negative airflow is needed for patients placed on airborne precautions; it is not necessary for a patient on contact precautions. When a patient on contact precautions needs to be transported, he or she should wear clean gowns, and wheelchairs or gurneys should be covered with an extra layer of sheets. Anyone who might come in contact with the patient needs to be protected, and equipment must be cleaned with an approved germicide after patient use and before another patient uses the shared equipment. DIF: Apply REF: 697 | 705 OBJ: Explain the rationale and components of routine practices. TOP: Implementation MSC: CPNRE: Foundations of Practice 29. The nurse is caring for a patient whose cultures are positive for Clostridium difficile. Which of
the following nursing actions would be appropriate in view of the presence of this organism? a. Instructing assistive personnel to use soap and water rather than sanitizer to clean hands. b. Placing the patient on droplet precautions. c. Wearing an N95 respirator when entering the patient room. d. Teaching the patient cough etiquette. ANS: A
NURSINGTB.COM
C. difficile is a spore-forming organism that can be transmitted through direct and indirect patient contact. Because C. difficile is a spore-forming organism, hand sanitizer is not effective in preventing its transmission. Hands must be washed with soap and water to prevent transmission. This organism is not transmitted via the droplet route; therefore, droplet precautions are not needed. An N95 respirator is used primarily for providing care to patients with airborne illness. All patients should be taught cough etiquette; this action is not specific to patients who have C. difficile infection. DIF: Apply REF: 689 OBJ: Explain the rationale and practices for additional (isolation) precautions. TOP: Implementation MSC: CPNRE: Foundations of Practice 30. The nurse is changing linens for a postoperative patient and feels a stick in her hand. A
nonactivated safe needle is noted in the linens. In this scenario, the nurse may be at risk for which of the following infections? a. Hepatitis B. b. C. difficile. c. Methicillin-resistant S. aureus (MRSA). d. Diphtheria. ANS: A
Canadian Fundamentals of Nursing 6th Edition Potter Test Bank Bloodborne pathogens such as those associated with hepatitis B and C are most commonly transmitted by contaminated needles. C. difficile is spread by contact with and ingestion of this microbe, and MRSA is spread by contact. Diphtheria is spread by droplets to anyone within 1 m (3 feet) of the patient. DIF: Remember REF: 700 | 704 OBJ: Explain conditions that promote the transmission of health care–associated infection. TOP: Diagnosis MSC: CPNRE: Foundations of Practice 31. The nurse is caring for a patient who has a bloodborne pathogen. The nurse splashes blood
above the glove onto intact skin while discontinuing an intravenous infusion. The nurse’s best next step is which of the following? a. To obtain an alcohol swab, remove the blood with an alcohol swab, and continue care. b. To immediately wash the site with soap and running water, and seek guidance from the manager. c. To delay washing of the site until the nurse is finished providing care to the patient. d. To do nothing; accidentally getting splashed with blood happens frequently and is part of the job. ANS: B
After getting splashed with blood from a patient who has a known bloodborne pathogen, it is important to cleanse the site immediately and thoroughly with soap and running water and notify the manager and employee health for guidance on next steps in the process. Removing the blood with an alcohol swab, delaying washing, and doing nothing because the splash was to intact skin could possibly spread the blood within the room and could spread the infection. N RSI NG TB.C OM Contamination should be contaU ined immediately to prevent spread of infection through contact. DIF: Apply REF: 706, Table 33-7 OBJ: Explain the rationale and components of routine practices. TOP: Implementation MSC: CPNRE: Foundations of Practice 32. The nurse is caring for a patient with an order for routine practices. The nurse includes hand
hygiene as part of the plan of care to which of the following? a. Provide an uninterrupted chain of infection. b. Decrease the incidence of health care–associated infection. c. Maintain a sterile environment. d. Decrease the drying effects of soap. ANS: B
Hand hygiene is part of routine practices and assists in interrupting the chain of infection. Hand hygiene can assist in decreasing the incidence of health care–associated infection, protect the nurse from the transfer of microorganisms, decrease the transmission of microbes to other patients, and prevent contamination of clean supplies. Hands are a common means of transmission of bacteria from one place to another. Proper hand hygiene does not decrease the drying effects of soap—in fact, it increases the drying effects of soap. Proper hand hygiene assists in maintaining a clean environment. DIF: Remember
REF: 700 | 705
OBJ: Perform proper procedures for hand hygiene.
Canadian Fundamentals of Nursing 6th Edition Potter Test Bank TOP: Planning
MSC: CPNRE: Foundations of Practice
33. The nurse is preparing to insert a urinary catheter. The nurse is using open gloving to don the
sterile gloves. Which four steps below will the nurse include in this process? 1. Lay glove package on clean flat surface above waistline. 2. Remove outer glove package by tearing the package open. 3. Glove the dominant hand of the nurse first. 4. While putting on the first glove, touch only the outside surface of the glove. 5. With gloved dominant hand, slip fingers underneath second glove cuff. 6. After second glove is on, interlock hands. a. 2, 3, 5, 6. b. 1, 2, 3, 6. c. 1, 3, 5, 6. d. 3, 4, 5, 6. ANS: C
Sterile objects held below the waist are considered contaminated. Gloving the dominant hand helps to improve dexterity. Slipping the fingers underneath the second glove cuff helps to protect the gloved fingers. Sterile touching sterile prevents glove contamination. Interlocking fingers ensures a smooth fit over the fingers. To open sterile supplies, the sides of the package are carefully separated and peeled apart; this presents the sterile contents from accidentally opening and touching contaminated objects. Touching the outside of the glove surface will contaminate the sterile item; only the inside of the glove—the piece that will be against the skin—should be touched. DIF: Apply REF: 723 | 724 OBJ: Properly don a surgical mask, sterile gown, and sterile gloves. N R I G B.C M TOP: Implementation U S N MTSC: CPONRE: Foundations of Practice 34. The nurse and the student nurse are caring for two different patients on the medical-surgical
unit. One patient is on airborne precautions, and one is on contact precautions. The nurse explains to the student the different interventions for care. The nurse would provide additional instruction if the student makes which of the following statements? a. “Be consistent in nursing interventions; there is only one difference in the precautions.” b. “Wash hands before entering and leaving both of the patients’ rooms.” c. “Apply the knowledge the nurse has of the disease process to prevent the spread of microorganisms.” d. “Patients with an order of airborne precautions wear a mask during transportation to departments.” ANS: A
Hand hygiene, properly disposing of supplies, applying knowledge of the disease process, and having patients with orders of airborne precautions wear a mask during transfer are all principles to follow when caring for patients in isolation. Multiple differences are evident between these types of isolation, including the type of room used for the patient and what the nurse wears while caring for the patient. DIF: Apply REF: 705 | 706 OBJ: Explain the rationale and practices for additional (isolation) precautions. TOP: Planning MSC: CPNRE: Foundations of Practice
Canadian Fundamentals of Nursing 6th Edition Potter Test Bank
35. The nurse is caring for a patient who needs a protective environment. The nurse has provided
the care needed and is now leaving the room. Which sequence of steps is in the correct order for removal of the personal protective equipment and associated tasks? 1. Remove eyewear/face shield and goggles. 2. Perform hand hygiene. 3. Remove gloves. 4. Untie gown, allow gown to fall from shoulders, and do not touch outside of gown; dispose of properly. 5. Remove mask by strings; do not touch outside of mask. 6. Dispose of all contaminated supplies and equipment in designated receptacles. 7. Leave room and close the door. a. 2, 1, 3, 5, 2, 6, 7. b. 3, 1, 4, 5, 2, 7, 6. c. 3, 1, 4, 5, 2, 6, 7. d. 4, 2, 1, 3, 5, 7, 6. ANS: B
The correct order for removing personal protective equipment for a patient in a protective environment and for performing associated tasks is to remove gloves, remove eyewear, remove gown, remove mask, perform hand hygiene, leave room and close doors, and dispose of all contaminated supplies and equipment in a manner that prevents the spread of microorganisms. DIF: Remember REF: 706-708 OBJ: Explain the rationale and practices for additional (isolation) precautions. TOP: Implementation N R I MSC: CPNRE: Foundations of Practice
G B.C M U S N T O
36. The nurse manager is evaluating current infection control data for the medical unit. The nurse
compares past patient data with current data to look for trends. The nurse manager examines the chain of infection for possible solutions. In which sequence are these items in the proper order? 1. A mode of transmission 2. An infectious agent or pathogen 3. A susceptible host 4. A reservoir or source for pathogen growth 5. A portal of entry to a host 6. A portal of exit from the reservoir a. 3, 4, 2, 6, 1, 5. b. 5, 4, 3, 1, 2, 6. c. 1, 2, 4, 6, 5, 3. d. 2, 4, 6, 1, 5, 3. ANS: D
The nurse manager is evaluating the chain of infection to determine actions that could be implemented to influence the spread of infection in the intensive care unit. Understanding the spread of infection and directing actions toward those steps have the potential to decrease infection in the setting. For spread of infection, the chain has to be uninterrupted with an infectious agent, a reservoir and portal of exit, a mode of transmission, a portal of entry, and a susceptible host.
Canadian Fundamentals of Nursing 6th Edition Potter Test Bank DIF: Remember REF: 684-687 OBJ: Explain the relationship between the chain of infection and transmission of infection. TOP: Evaluate MSC: CPNRE: Foundations of Practice
NURSINGTB.COM
Canadian Fundamentals of Nursing 6th Edition Potter Test Bank
Chapter 34: Medication Administration Potter et al: Canadian Fundamentals of Nursing, 6th Edition MULTIPLE CHOICE 1. A nurse knows that patient education has been effective when the patient makes which
statement? a. “I must take my parenteral medication with food.” b. “If I am 30 minutes late taking my medication, I should skip that dose.” c. “I will rotate the location where I give myself injections.” d. “Once I start feeling better, I will stop taking my medication.” ANS: C
Rotating injection sites provides greater consistency in absorption of medication. Parenteral medication absorption is not affected by the timing of meals. Taking a medication 30 minutes late is within the 60-minute window of the time medications should be taken. Medications should be stopped in accordance with the provider’s orders. With some medications, such as antibiotics, it is crucial that the full course of medication is taken to avoid relapse of infection. DIF: Apply REF: 752 | 791-792 OBJ: Examine the nurse’s role and responsibilities regarding medication administration. TOP: Evaluate MSC: CPNRE: Foundations of Practice 2. Which statement by the patient is an indication to use the Z-track method? a. “I’m really afraid that a big needle will hurt.” b. “The last shot like that turned my skin colours.” NUicat RSions IN.”GTB.COM c. “I am allergic to many med d. “My legs are too obese for the needle to go through.” ANS: B
The Z-track is indicated when the medication being administered has the potential to irritate sensitive tissues. The Z-track method is not meant to reduce discomfort from the procedure. If a patient is allergic to a medication, it should not be administered. If a patient has additional subcutaneous tissue to go through, a needle of a different size may be selected. DIF: Understand REF: 794 OBJ: Describe factors to consider when choosing routes of medication administration. TOP: Planning MSC: CPNRE: Foundations of Practice 3. The physician orders that a 2-year-old child have ear irrigation performed daily. How does the
nurse correctly perform the procedure? a. Pulling the auricle down and back to straighten the ear canal. b. Pulling the auricle upward and outward to straighten the ear canal. c. Instilling the irrigation solution by holding the syringe just inside the ear canal. d. Holding the fluid in the canal for 2 to 3 minutes with a cotton swab. ANS: A
Canadian Fundamentals of Nursing 6th Edition Potter Test Bank Children up to 3 years of age should have the auricle pulled down and back, children 4 years of age and up and adults should have the auricle pulled upward and outward. Irrigation solution should be instilled 1 cm (0.4 in) above the opening of the ear canal. Irrigation solution should be allowed to drain freely during instillation. DIF: Understand REF: 769, Box 34-21 OBJ: Prepare and administer subcutaneous, intramuscular, and intradermal injections; intravenous medications; hypodermoclysis infusions; oral and topical skin preparations; eye, ear, and nose drops; vaginal instillations; rectal suppositories; and inhalants. TOP: Planning MSC: CPNRE: Foundations of Practice 4. The patient is to receive phenytoin (Dilantin) at 0900. The nurse knows that the ideal time to
draw a trough level is when? a. 0800 hours. b. 0830 hours. c. 0900 hours. d. 0930 hours. ANS: B
Trough levels are generally measured 30 minutes before the drug is administered. If the medication is to be administered at 0900 hours, the trough should be measured at 0830 hours. DIF: Understand REF: 733 TOP: Implementation
OBJ: Discuss factors that influence medication actions. MSC: CPNRE: Foundations of Practice
5. A physician orders 1000 mL of normal saline to be infused at a rate of 50 mL/hr. The nurse
plans on hanging a new bag at what time? a. 2 hours. NURSINGTB.COM b. 5 hours. c. 10 hours. d. 20 hours. ANS: D
It will take 20 hours for a litre (1000 mL) of fluid to infuse at a rate of 50 mL/hr. After 2 hours, only 100 mL would have infused. After 5 hours, only 250 mL would have infused. At 10 hours, 500 mL would have infused. DIF: Remember REF: 738 TOP: Implementation
OBJ: Calculate a prescribed medication dose. MSC: CPNRE: Foundations of Practice
6. The nurse is preparing to administer a 0.5-mL rabies vaccine into the deltoid muscle of a
pediatric patient. Which needle size is best for the procedure? a. 20 gauge 3.8 cm (11/2 inch) b. 23 gauge 1.3 cm (1/2 inch) c. 25 gauge 1.6 cm (5/8 inch) d. 27 gauge 1 cm (3/8 inch) ANS: C
For an intramuscular injection into the deltoid, a 25-gauge, 1.6-cm (5/8-inch) needle (depending upon the size of the child) is recommended. The other choices did not include the correct gauge of 25.
Canadian Fundamentals of Nursing 6th Edition Potter Test Bank DIF: Remember REF: 786 OBJ: Describe factors to consider when choosing routes of medication administration. TOP: Implementation MSC: CPNRE: Foundations of Practice 7. The nurse is giving an intramuscular (IM) injection. The nurse notices blood return in the
syringe. What should the nurse do? a. Administer the injection at a slower rate. b. Withdraw the needle and prepare the injection again. c. Pull the needle back slightly and inject the medication. d. Give the injection and hold pressure over the site for 3 minutes. ANS: B
Blood return indicates improper placement, and the injection should not be given. Instead, the nurse should withdraw the needle, dispose of the syringe and needle properly, and prepare the medication again. Administering IM medication into a blood vessel could have dangerous adverse effects, and the medication will be absorbed faster than intended owing to increased blood flow. Holding pressure is not an appropriate intervention. Pulling back the needle slightly does not guarantee proper placement of the needle and medication administration. DIF: Apply REF: 789 OBJ: Describe the importance of safe medication techniques. MSC: CPNRE: Foundations of Practice
TOP: Implementation
8. The nurse is planning to administer a tuberculin test with a 27-gauge, 1-cm (3/8-inch) needle.
The nurse should insert the needle at what angle? a. 15 degrees. b. 45 degrees. NURSINGTB.COM c. 90 degrees. d. 180 degrees. ANS: A
A 27-gauge, 1-cm (3/8-inch) needle is used for intradermal injections such as a tuberculin test, which should be inserted at a 5- to 15-degree angle, just under the dermis of the skin, about 3 mm. Placing the needle at 45 degrees, 90 degrees, or 180 degrees will place the medication too deep. DIF: Understand REF: 794 OBJ: Describe factors to consider when choosing routes of medication administration. TOP: Implementation MSC: CPNRE: Foundations of Practice 9. The nurse knows to assess for signs of medication toxicity within older persons because of
which physiological change? a. Reduced blood albumin level. b. Delayed esophageal clearance. c. Decreased gastric peristalsis. d. Decreased cognitive function. ANS: A
Canadian Fundamentals of Nursing 6th Edition Potter Test Bank The degree to which medications bind to serum proteins, such as albumin, affects the medication's distribution. Medications bound to albumin cannot exert pharmacological activity. The unbound, or free, medication is the active form of the medication. Older persons have a decrease in albumin in their bloodstream, which is probably the result of a change in liver function. The same is true for patients who have liver disease or malnutrition, who, along with older persons, have the potential for more medication to be unbound and thus may be at risk for an increase in medication activity or toxicity, or both. DIF: Understand REF: 731 TOP: Implementation
OBJ: Discuss factors that influence medication actions. MSC: CPNRE: Foundations of Practice
10. A nurse interprets that a scribbled medication order reads 25 mg. The nurse administers 25 mg
of the medication to a patient, and then discovers that the dose was incorrectly interpreted and should have been 15 mg. Who is ultimately responsible for the error? a. Physician. b. Pharmacist. c. Nurse. d. No fault. ANS: C
Ultimately, the person administering the medication is responsible for ensuring that it is correct. The nurse administered the medication, so in this case it is the nurse. This is the importance of verifying the Ten Rights of Medication Administration. DIF: Understand REF: 743 | 744 OBJ: Compare and contrast the roles of the physician, the pharmacist, and the nurse in medication administration. TOP: Implementation MSC: CPNRE: Professional, EN thU icR al,SaI ndNLG egTaB l P. raC ctO icM e 11. A patient is to receive medication through a nasogastric tube. What is the most important
nursing action to ensure effective absorption? a. Thoroughly shake the medication before administering. b. After all medications are administered, flush tube with 15 to 30 mL of water. c. Position patient in the supine position for 30 minutes. d. Clamp suction for 30 to 60 minutes after medication administration. ANS: D
Absorption time for a medication administered through a nasogastric tube is the same as for an oral medication: 30 to 60 minutes. Therefore, the nurse would need to hold the suction for that amount of time to let the medication absorb. Thoroughly shaking the medication mixes the medication before administration but does not affect absorption. Flushing the medications ensures that all were administered. Patients with nasogastric tubes should never be positioned supine but instead should be positioned at a 30- to 90-degree angle to prevent aspiration, provided no contraindication to such positioning is known. DIF: Analyze REF: 762, Box 34-19 OBJ: Describe the importance of safe medication techniques. MSC: CPNRE: Foundations of Practice
TOP: Implementation
Canadian Fundamentals of Nursing 6th Edition Potter Test Bank 12. Aspirin is an analgesic, antipyretic, antiplatelet, and anti-inflammatory agent. A physician
writes for aspirin 650 mg every 4 to 6 hours, “prn [as needed]: febrile.” For which patient would this order be appropriate? a. 7-year-old with hemophilia. b. 21-year-old with a sprained ankle. c. 35-year-old with a severe headache. d. 62-year-old female with pneumonia. ANS: D
The provider wrote for the medication to be given for a fever. Hemophilia is a bleeding disorder; therefore, antiplatelet agents would be contraindicated. Although it can be used for inflammatory problems and pain, this is not what the order was written for. DIF: Evaluate REF: 732 OBJ: Discuss factors to use when assessing a patient’s needs for and response to medication therapy. TOP: Implementation MSC: CPNRE: Foundations of Practice 13. A patient is in need of immediate pain relief for a severe headache. The nurse knows that
which medication will be absorbed the quickest? a. Tylenol, 650 mg PO. b. Morphine, 4 mg SQ. c. Ketorolac (Toradol), 8 mg IM. d. Hydromorphone (Dilaudid), 4 mg IV. ANS: D
The intravenous (IV) route is the fastest route for absorption because of the increase in blood flow. Oral (PO), subcutaneous (SQ), and IM are other ways to deliver medication but with less blood flow. N R I G B.C M
U S N T
DIF: Understand REF: 731 TOP: Implementation
O
OBJ: Discuss factors that influence medication actions. MSC: CPNRE: Foundations of Practice
14. A drug requires a low pH to be metabolized. Knowing this, the nurse anticipates that the
medication will be administered by which route? a. Oral. b. Parenteral. c. Buccal. d. Inhalation. ANS: A
An oral medication would pass through to the stomach, which is an area of low pH. The nurse would question an order for a medication for which an acidic environment is needed to be metabolized. Buccal, inhalation, and parenteral routes provide neutral or alkaline environments. DIF: Analyze REF: 731 OBJ: Describe factors to consider when choosing routes of medication administration. TOP: Implementation MSC: CPNRE: Foundations of Practice 15. The nurse knows that a patient is having an idiosyncratic reaction with the stimulant
pseudoephedrine (Sudafed) when what happens? a. The patient experiences blurred vision while driving.
Canadian Fundamentals of Nursing 6th Edition Potter Test Bank b. The patient falls asleep during daily activities. c. The patient presents with a pruritic rash. d. The patient develops xerostomia. ANS: B
An idiosyncratic reaction is a reaction opposite to what the side effects of the medication normally are, or an overreaction or underreaction to the medication. Blurred vision is a toxic effect. A rash could indicate an allergic reaction. Dry mouth is a typical response to a stimulant. DIF: Apply REF: 732 OBJ: Differentiate among different types of medication actions. TOP: Assessment MSC: CPNRE: Foundations of Practice 16. An order is written for phenytoin (Dilantin) 500 mg IM q3-4h prn for pain. The nurse
recognizes that treatment of pain is not a standard therapeutic indication for this drug. The nurse believes that the prescriber meant to write for hydromorphone (Dilaudid). What should the nurse do? a. Give the patient Dilaudid, as it was meant to be written. b. Call the prescriber to clarify and justify the order. c. Administer the medication and monitor the patient frequently. d. Refuse to give the medication and notify the nurse supervisor. ANS: B
If the nurse is apprehensive about the drug, dose, route, or reason for a medication, the nurse should first call the prescriber and clarify. The nurse should not change the order without the prescriber’s consent. Ultimately, the nurse can be held responsible for administering an incorrect medication. If the pN resc lliC ngOtM o change the order and does not justify the URribe SIr NisGunw TBi. order in a reasonable and evidence-based manner, the nurse may refuse to give the medication and notify her supervisor. DIF: Apply REF: 743-746 OBJ: Compare and contrast the roles of the physician, the pharmacist, and the nurse in medication administration. TOP: Implementation MSC: CPNRE: Professional, Ethical, and Legal Practice 17. A patient needs assistance excreting a gaseous medication. What is the correct nursing action? a. Encourage the patient to cough and deep-breathe. b. Suction the patient’s respiratory secretions. c. Administer the antidote via inhalation. d. Administer 100% fraction of inspired oxygen (FiO2) via simple face mask. ANS: A
Gaseous and volatile medications are excreted through gas exchange. Deep breathing and coughing will assist in clearing the medication more quickly. DIF: Understand REF: 731 OBJ: Describe the physiological mechanisms of medication action, including absorption, distribution, metabolism, and excretion of medications. TOP: Implementation MSC: CPNRE: Foundations of Practice
Canadian Fundamentals of Nursing 6th Edition Potter Test Bank 18. The nurse knows that patient education about a buccal medication has been effective when the
patient states a. “I should let the medication dissolve completely.” b. “I can only drink water, not juice, with this medication.” c. “For faster distribution, I should chew my medication first.” d. “I should place the medication in the same location.” ANS: A
Buccal medications should be placed in the side of the cheek and allowed to dissolve completely. Buccal medications act with the patient’s saliva and mucosa. The patient should not chew or swallow the medication. Gastric secretions may destroy some medications. The patient should rotate sides of the check to avoid irritating the mucosal lining. DIF: Understand REF: 734 OBJ: Discuss methods of educating a patient about prescribed medications. TOP: Evaluate MSC: CPNRE: Foundations of Practice 19. What is the nurse’s priority action to protect a patient from medication error? a. Requesting that the prescriber write out an order, rather than giving a verbal order b. Asking anxious family members to leave the room before giving a medication c. Checking the patient’s room number against the medication administration record d. Administering as many of the medications as possible at one time ANS: A
Verbal orders should be limited to urgent situations where written communication is unavailable. The nurse should explain the reasons and logistics of a procedure to calm anxious family members and should ask family members not to distract the person administering medication, for the patient’s N safe prB op.eC r ed ucation, if the family members are creating Ifte URty.SA NGr T OM an unsafe environment, the nurse may ask them to step out of the room. The medication administration record should be checked against the patient’s hospital identification band; a room number is not an acceptable identifier. Medications should be given when scheduled, and medications with special assessment indications should be separated. DIF: Analyze REF: 740, Box 34-5 OBJ: Implement nursing actions to prevent medication errors. MSC: CPNRE: Professional, Ethical, and Legal Practice
TOP: Implementation
20. The patient is in severe pain and is requesting a prn medication before the prn time interval
has elapsed. What is the nurse’s priority? a. Give the medication early for any pain score greater than 8. b. Call the prescriber and request a stat order. c. Explain to the patient why he will have to wait for the medication. d. Document the patient’s request and pain score. ANS: B
The nurse should use clinical judgement to advocate for the patient by requesting a stat order for the patient’s breakthrough pain. The nurse cannot give a medication without an order because this violates the “Right Time” portion of the Ten Rights of Medication Administration. If a nurse determines that a patient is in severe pain, she must use clinical judgement to find that patient a means of pain relief. Although the nurse should document the patient’s request and pain score, this is not the priority.
Canadian Fundamentals of Nursing 6th Edition Potter Test Bank
DIF: Analyze REF: 747 TOP: Implementation
OBJ: Explain the rights of medication administration. MSC: CPNRE: Foundations of Practice
21. A patient is at risk for aspiration. What nursing action is most appropriate? a. Hold the patient’s cup for him so he can concentrate on taking pills. b. Thin out liquids so they are easier to swallow. c. Give the patient a straw to control the flow of liquids. d. Have the patient self-administer the medication. ANS: D
Aspiration occurs when food, fluid, or medication intended for gastrointestinal administration inadvertently enters the respiratory tract. To minimize aspiration risk, allow the patient, if capable, to self-administer medication. Patients should also hold their own cup to control how quickly they take in fluid. Liquids should be thickened to reduce the risk of aspiration. Patients at risk for aspiration should not be given straws because use of a straw decreases the control the patient has over volume intake. DIF: Understand REF: 756, Box 34-18 OBJ: Discuss factors to use when assessing a patient’s needs for and response to medication therapy. TOP: Implementation MSC: CPNRE: Foundations of Practice 22. A confused patient refuses his medication. What is the nurse’s first response? a. Agree with the patient’s decision and document it in his chart. b. Educate the patient about the importance of the medication. c. Discreetly hide the medication in the patient’s favourite Jell-O. d. Inform the patient that he must take the medication whether he wants to or not. ANS: B
NURSINGTB.COM
Much of a patient’s apprehension about medication comes from lack of understanding, and educating the patient may lead to better compliance. Ultimately, the patient does have the right to refuse the medication; however, the nurse should first try to educate the patient. Hiding, deceiving, or forcing a patient into taking a medication is unethical and violates his right to autonomy. DIF: Apply REF: 748 OBJ: Discuss methods of educating a patient about prescribed medications. TOP: Planning MSC: CPNRE: Foundations of Practice 23. A patient who is being discharged today is going home with an inhaler. The patient is to
administer 2 puffs of his inhaler twice daily. The inhaler contains 200 puffs. When should the nurse appropriately advise the patient to refill his medication? a. As soon as he leaves the hospital b. When the inhaler is half empty c. Six weeks after the patient starts using the inhaler d. Fifty days after discharge ANS: C
The inhaler should last the patient 50 days; the nurse should advise the patient to refill the prescription when he has 7 to 10 days of medication remaining. Refilling it as soon as he leaves the hospital or when the inhaler is half empty is too early. If the patient waits 50 days, the patient will run out of medication before it can be refilled.
Canadian Fundamentals of Nursing 6th Edition Potter Test Bank
DIF: Understand REF: 775 TOP: Implementation
OBJ: Calculate a prescribed medication dose. MSC: CPNRE: Foundations of Practice
24. Why does a subcutaneous injection takes longer to absorb than an IV injection? a. Fewer blood vessels are found under the subcutaneous level. b. Adipose tissue takes longer to metabolize medication. c. Connective tissue holds medication in place longer. d. Some medication leaks out after instillation. ANS: A
How quickly a medication is absorbed is dependent on blood flow to the site. Locations with less blood supply take longer to absorb. Absorption is not based on adipose tissue; however, excessive adipose tissue may cause the medication to take longer before reaching the blood supply. The connective tissue is not part of medication absorption. If a medication is properly administered, none of it should be wasted. DIF: Understand TOP: Planning
REF: 785-792 OBJ: Discuss factors that influence medication actions. MSC: CPNRE: Foundations of Practice
25. The nurse realizes which patient is at greatest risk for an unintended synergistic effect? a. A 72-year-old who is seeing four different specialists. b. A 4-year-old who has mistakenly taken the entire packet of his mother’s birth
control pills. c. A 50-year-old who was prescribed a second blood pressure medication. d. A 35-year-old drug addict who has ingested meth mixed with several household chemicals. ANS: A
NURSINGTB.COM
A synergistic effect occurs when two medications potentiate each other, creating a greater effect than a single medication on its own. Polypharmacy is likely to occur when the 72-year-old is seeing four different health care providers. Polypharmacy places the patient at risk for unintended mixing of medications that potentiate each other. The child taking too much of a medication by mistake could experience overdose or toxicity. The 50-year-old is prescribed two different blood pressure medications for their synergistic effect, but this is a desired event. A drug addict mixing chemicals can be toxic. DIF: Analyze REF: 733 | 754 | 755 OBJ: Differentiate among different types of medication actions. TOP: Assessment MSC: CPNRE: Foundations of Practice 26. Which patient using an inhaler would benefit most from using a spacer? a. A 3-year-old with a cleft palate. b. A 25-year-old with multiple sclerosis. c. A 50-year-old with hearing impairment. d. A 72-year-old with left-sided hemiparesis. ANS: B
Canadian Fundamentals of Nursing 6th Edition Potter Test Bank A spacer is indicated for a patient who has limited coordination or function. Individuals with multiple sclerosis often lose motor control and function and have difficulty seeing. Children often have difficulty using a spacer, so a simple face mask is preferred for infants and children younger than 4. Hearing impairment may make teaching the patient to use the inhaler difficult, but it does not indicate the need for a spacer. A patient with one-sided weakness would have a difficult time assembling and administering an inhaler by using a spacer, but the patient could use the inhaler single-handedly. DIF: Analyze REF: 768-775 OBJ: Describe factors to consider when choosing routes of medication administration. TOP: Implementation MSC: CPNRE: Foundations of Practice 27. The prescriber wrote an order for a 40-kg child to receive 25 mg of medication four times a
day. The therapeutic range is 5 to 10 mg/kg/day. What is the nurse’s priority? a. Administer the medication because it is within the therapeutic range. b. Notify the physician that the prescribed dose is in the toxic range. c. Notify the physician that the prescribed dose is below the therapeutic range. d. Change the dose to one that is within range. ANS: C
The dosage range for a 40-kg patient is 200 to 400 mg a day. The prescribed dose is 100 mg/day, which is below therapeutic range. The nurse should notify the physician first and ask for clarification on the order. The dose is not above the therapeutic range and is not at a toxic level. The nurse should never alter an order without the prescriber’s approval and consent. DIF: Apply REF: 737-739 TOP: Implementation
OBJ: Calculate a prescribed medication dose. MSC: CPNRE: Professional, Ethical, and Legal Practice
NURSINGTB.COM
28. The nurse is administering an IV medication that is to be infused over 10 minutes. Which
method should the nurse choose to efficiently administer the medication? a. Place the medication in a large-volume catheter-tipped syringe. b. Mix the medication into the patient’s maintenance fluids. c. Attach separate tubing and set the medication syringe in a mini-infusion pump. d. Stand at the patient’s bedside and carefully watch the clock while pushing the medication. ANS: C
To administer this medication efficiently, the nurse should use an infusion pump to run the medication in over a prolonged time. This method is more accurate and is more time efficient than other methods because the nurse can leave the room. The nurse should not mix medication into the maintenance bag without pharmacist and physician approval. Pushing the medication is not a time-efficient method for the nurse. A catheter-tipped syringe is an inappropriate device for administration of a medication. DIF: Analyze REF: 796 OBJ: Describe factors to consider when choosing routes of medication administration. TOP: Implementation MSC: CPNRE: Foundations of Practice
Canadian Fundamentals of Nursing 6th Edition Potter Test Bank 29. The nurse is preparing to administer medications to two patients with the same last name.
After the first administration, the nurse realizes that she did not check the identification of the patient before administering medication. Which of the following actions should the nurse complete first? a. Return to the room to check and assess the patient. b. Administer the antidote to the patient immediately. c. Alert the charge nurse that a medication error has occurred. d. Complete proper documentation of the medication error in the patient’s chart. ANS: A
The nurse’s first priority is to establish the safety of the patient by assessing the patient. Second, the nurse should notify the charge nurse and the physician. The antidote should be administered if required. Finally, the nurse needs to complete proper documentation. DIF: Evaluate REF: 743 | 747 OBJ: Implement nursing actions to prevent medication errors. MSC: CPNRE: Foundations of Practice
TOP: Implementation
30. The nurse knows that caring for two patients with the same last name can lead to a medication
error involving which right of medication administration? a. Right medication. b. Right patient. c. Right dose. d. Right route. ANS: B
The nurse should ask the patient to verify his or her identity and should check the patient’s ID bracelet against the medicatioNnUrR ecor reCpro IdNtoGens Bu. Mper patient. Acceptable patient identifiers S T O include the patient’s name, an identification number assigned by a health care agency, or a telephone number. The patient’s room number should not be used as an identifier. To identify a patient correctly in an acute care setting, the nurse should compare the patient identifiers on the medication administration record with the patient’s identification bracelet while at the patient’s bedside. Right medication, right dose, and right route are equally as important, but this example outlines right patient. DIF: Remember REF: 746-749 TOP: Implementation
OBJ: Explain the rights of medication administration. MSC: CPNRE: Foundations of Practice
31. A patient states that she would prefer not to take her daily allergy pill this morning because it
makes her too drowsy throughout the day. How may the nurse respond therapeutically? a. “The physician ordered it; therefore, you must take your medication every morning at the same time whether you’re drowsy or not.” b. “Let’s change the time you take your pill to 9 p.m., so the drowsiness occurs when you would normally be sleeping.” c. “You can skip this medication on days when you need to be awake and alert.” d. “Try to get as much done as you can before you take your pill, so you can sleep in the afternoon.” ANS: B
Canadian Fundamentals of Nursing 6th Edition Potter Test Bank The nurse should use knowledge about the medication to educate the patient about potential response to medications. Then the medication schedule can be altered based on that knowledge, after the physician has been notified. It is the patient’s right to refuse her medication; however, the nurse should educate the patient on the importance and effects of her medication. Asking a patient to change her entire life schedule around a medication is unreasonable and will decrease compliance. The nurse should be supportive and should offer solutions to manage medication effects. DIF: Apply REF: 747 OBJ: Discuss factors to use when assessing a patient’s needs for and response to medication therapy. TOP: Implementation MSC: CPNRE: Foundations of Practice 32. A provider has ordered a STAT medication to be administered. The nurse knows that the best
route of administration is a. IV. b. IM. c. SQ. d. PO. ANS: A
IV medications have the quickest effect because they receive the most blood flow. A STAT order is to be carried out as quickly as possible, so the effect should be as immediate as possible. Oral, subcutaneous (SQ), and IM are other ways to deliver medication but with less blood flow. DIF: Understand REF: 731 OBJ: Describe the physiological mechanisms of medication action, including absorption, TOP: Implementation distribution, metabolism, and exNcU reR tioS nI ofNmGeT dicBa. tioC nsO .M MSC: CPNRE: Foundations of Practice 33. A nurse is attempting to administer medication to a child, but the child refuses to take the
medication. The nurse asks for the parent’s cooperation by saying a. “Please hold your child’s arms down at her sides, so I can get the full dose of medication into her mouth.” b. “I will prepare the medication for you and observe if you would like to try to administer the medication.” c. “Let’s turn the lights off and give the child a moment to fall asleep before administering the medication.” d. “Since your child loves applesauce, let’s add the medication to it, so your child doesn’t resist.” ANS: B
Children often have difficulties taking medication, but it is less traumatic for the child if the parent administers the medication. Holding down the child is not the best option because it may further upset the child. Never administer an oral medication to a sleeping child. Don’t mix medications into the child’s favourite foods, because the child might start to refuse the food. DIF: Evaluate REF: 754 TOP: Implementation
OBJ: Discuss factors that influence medication actions. MSC: CPNRE: Foundations of Practice
Canadian Fundamentals of Nursing 6th Edition Potter Test Bank 34. A 64-year-old quadriplegic patient needs an IM injection of antibiotic. What is the best site
for the administration? a. Deltoid. b. Dorsal gluteal. c. Ventrogluteal. d. Vastus lateralis. ANS: D
Vastus lateralis is a large muscle that is easily accessible from the supine position. Because the patient does not walk, the ventrogluteal muscle atrophies and is not the ideal location. The dorsal gluteal site is a location for a subcutaneous injection, and this patient requires an IM injection. The deltoid is easily accessible, but this muscle is not well developed in many adults. DIF: Apply REF: 793, Box 34-27 OBJ: Discuss factors that influence medication actions. MSC: CPNRE: Foundations of Practice
TOP: Implementation
35. Which nursing action is the number one priority for ensuring that medication stays in the
target therapeutic range? a. Measuring the peak and trough levels at the same time each day. b. Administering a double dose after a dose was missed. c. Delivering the same amount of the drug at the same time each day. d. Increasing absorption by holding all other medications 1 hour before administration. ANS: A
The quantity and distributionNofUR a me ferent body compartments change SIdic NGatio TBn .inCdif OM constantly. Measuring peak and trough levels allows health care providers to see whether the current medication dosage is effective for the patient, or if it needs to be adjusted. Administering a double dose is dangerous and could cause the medication levels to cross the toxic threshold. Delivering the same amount each day may not be therapeutic or may be toxic for the patient. Holding all other medications should not affect the peak or half-life of the medications, assuming that they are compatible. DIF: Apply REF: 733 OBJ: Discuss factors to use when assessing a patient’s needs for and response to medication therapy. TOP: Implementation MSC: CPNRE: Foundations of Practice 36. Which of the following demonstrates proper oral medication administration? a. Removing the medication from the wrapper and placing it in a cup labelled with
the patient’s information. b. Using the edge of the medicine cup to fill with 0.5 mL of liquid medication. c. Placing all of the patient’s medications in the same cup, except medications with
assessments. d. Combining liquid medications from 2 single dose cups into 1 medicine cup. ANS: C
Canadian Fundamentals of Nursing 6th Edition Potter Test Bank Placing medications that require preadministration assessment in a separate cup serves as a reminder to check before the medication is given, which makes it easier for the nurse to withhold medication if necessary. Medications should not be removed from their package until they are in the patient’s room because this makes identification of the pill easier and reduces contamination. When measuring a liquid, the nurse should use the meniscus level to measure, not the edge. In addition, liquid medications measuring less than 10 mL should be drawn up in a needleless syringe. Single-dose medications should not be transferred to medicine cups to reduce unnecessary manipulation of the dose. DIF: Understand REF: 758 OBJ: Describe the importance of safe medication techniques. MSC: CPNRE: Foundations of Practice
TOP: Implementation
37. A patient who is receiving IV fluids notifies the nurse that his arm feels tight. Upon
assessment, the nurse notes that the arm is swollen and cool to the touch. What should the nurse’s first action be? a. Discontinue the IV site, and apply a warm compress. b. Attach a syringe, and pull back on the plunger to aspirate the IV fluid. c. Start a new IV site distal from the site. d. Stop the IV fluids, and notify the physician immediately. ANS: A
An IV site that is puffy, swollen, and cool to the touch indicates infiltration. The IV site should be discontinued immediately because it is no longer a viable access point. Pulling back on the syringe will not result in fluid return because there is no longer venous access. A new IV site should be established in the opposite arm after the old IV line has been removed. The IV line should be removed; it is not sufficient to only stop the fluids.
N R I G B.C M U S N T O
DIF: Apply REF: 729 OBJ: Describe the importance of safe medication techniques. MSC: CPNRE: Foundations of Practice
TOP: Implementation
38. The physician orders 4 mg of oxycodone to be delivered every 6 hours. After 4 hours, the
patient is complaining that she is in more pain. The nurse advises the physician to make which medication adjustment? a. Add an additional narcotic on top of the oxycodone. b. Divide the dose in half and administer 2 mg every 3 hours. c. Give another 4 mg of oxycodone after 4 hours. d. Change the medication being administered for pain relief. ANS: B
The patient’s metabolism causes the peak effect to occur when the medicine is at its highest concentration. After reaching its peak, the serum concentration of the medication falls progressively. Spreading out the dose ensure that the patient will receive constant pain relief. Changing the medication, increasing the dose, or adding another medication is not the best course of action. DIF: Apply REF: 804 OBJ: Discuss factors to use when assessing a patient’s needs for and response to medication therapy. TOP: Implementation MSC: CPNRE: Foundations of Practice
Canadian Fundamentals of Nursing 6th Edition Potter Test Bank 39. The nursing instructor asks the student to explain methods to reduce the risk of needlestick
injury. The nursing instructor would provide further instruction to the student who makes which following statement? a. Recap the needle after giving an injection. b. Use needleless systems when available c. Never force a needle into the sharps disposal. d. Clearly mark sharps disposal containers. ANS: A
To prevent the risk of needlesticks, the nurse should never recap needles. Needleless systems or sharps with engineered sharps injury protections (SESIP) safety devices should be used when available. Needles should not be forced into the box. Receptacles should be marked clearly to warn of danger. DIF: Remember REF: 796, Box 34-28 OBJ: Describe the importance of safe medication techniques. MSC: CPNRE: Foundations of Practice
NURSINGTB.COM
TOP: Implementation
Canadian Fundamentals of Nursing 6th Edition Potter Test Bank
Chapter 35: Complementary and Alternative Approaches in Health Care Potter et al: Canadian Fundamentals of Nursing, 6th Edition MULTIPLE CHOICE 1. A patient describes practising a complementary and alternative therapy involving
concentrating and controlling his respiratory rate and pattern, recognizing that breath work is to yoga as which of the following? a. The “zone” is to acupressure. b. Massage therapy is to Ayurveda. c. Reiki therapy is to therapeutic touch. d. Prayer is to tai chi. ANS: C
This is an analogy in which different therapies within specific categories are compared. Both yoga and breath work are mind–body therapies, whereas both reiki and therapeutic touch therapies are energy field therapies. The other options have different design structures; thus they do not fit the analogy. DIF: Apply REF: 816 OBJ: Describe the various treatments associated with complementary and alternative medicine. TOP: Planning MSC: CPNRE: Foundations of Practice 2. A teenager with an anxiety disorder is referred for biofeedback because her parents do not
want her on anxiolytics. The nurse recognizes that the teenager understands her health education on biofeedback when she makes which statement? Ne R I GTyB.C M a. “Biofeedback will allow mU toSdireNct m enerO gies in an intentional way when stressed.” b. “Biofeedback will allow me to manipulate my stressed-out joints.” c. “Biofeedback will help me with my thoughts, feelings, and physiological responses to stress.” d. “Biofeedback will let me assess and redirect my energy fields.” ANS: C
By using electromechanical instruments, a person can receive information or feedback on his or her stress level. Having this knowledge allows the patient to develop awareness and voluntary control over his or her physiological symptoms. Biofeedback does not address energy fields. Directing energies is therapeutic touch. Manipulation of body alignment and joints is done by a chiropractor. DIF: Apply REF: 818 OBJ: Understand how complementary and alternative approaches assist in health promotion and disease prevention. TOP: Planning MSC: CPNRE: Foundations of Practice 3. A 70-year-old patient is newly admitted to a skilled nursing facility with the diagnoses of
Alzheimer’s dementia, lipidemia, and hypertension and a history of pulmonary embolism. Medications brought on admission included lisinopril, hydrochlorothiazide, warfarin, low-dose aspirin, ginkgo biloba, and echinacea. The nurse contacts the patient’s medical provider over which potential drug-drug interaction? a. Lisinopril and Echinacea.
Canadian Fundamentals of Nursing 6th Edition Potter Test Bank b. Warfarin and ginkgo biloba. c. Echinacea and warfarin. d. Lisinopril and hydrochlorothiazide. ANS: B
Warfarin and blood thinners interact with ginkgo biloba as designed to improve memory. All herbal supplements should be evaluated with current pharmacological medications. The other options do not have drug interactions with each other. DIF: Analyze REF: 826 OBJ: Recognize botanical treatments as an approach to complementary and alternative medicine. TOP: Planning MSC: CPNRE: Foundations of Practice 4. An acquaintance of a nurse asks for a nonmedical approach for excessive worry and work
stress. The most appropriate complementary and alternative medicine (CAM) therapy that the nurse can recommend is which of the following? a. Meditation. b. Ayurvedic herbs. c. Acupuncture. d. Chiropractic therapy. ANS: A
Meditation is indicated for stress-related illness. A person can learn to calm down and cope with stress through the use of meditation. Ayurvedic herbs have been used for centuries to treat illness. Acupuncture focuses on redirecting qi via the body’s meridian energy lines to influence deeper internal organs. Chiropractic therapy involves manipulation of the spinal column and includes physiotherapy and diet therapy.
NURSINGTB.COM
DIF: Analyze REF: 821 OBJ: Describe the various treatments associated with complementary and alternative medicine. TOP: Planning MSC: CPNRE: Foundations of Practice 5. The therapy that is more effective in treating physical ailments than in preventing disease or
managing chronic illness is which medicine? a. Allopathic. b. Complementary. c. Alternative. d. Mind-body. ANS: A
Allopathic medicine is synonymous with traditional Western medicine, which is highly effective in treating numerous physical ailments, but it is in general less effective in preventing disease, decreasing stress-induced illness, managing chronic disease, and caring for the emotional and spiritual needs of individuals. Complementary, alternative, and mind–body types of medicine can be used in tandem with allopathic medicine but are distinctly different. DIF: Understand REF: 815 OBJ: Identify the differences between complementary and alternative approaches in health care. TOP: Planning MSC: CPNRE: Foundations of Practice
Canadian Fundamentals of Nursing 6th Edition Potter Test Bank 6. Which of the following is considered a healing practice that promotes harmony within a
community and in the physical and spiritual worlds through sweating and purging, herbal remedies, and shamanic healing? a. Latin American practices. b. Traditional Chinese medicine. c. Ayurveda. d. Traditional Indigenous medicine. ANS: D
Traditional Indigenous medicine is a healing practice that promotes harmony within a community and in the physical and spiritual worlds through sweating and purging, herbal remedies, and shamanic healing. Latin American practice includes a humoral model for classifying food, activity, drugs, illnesses, and a series of folk illnesses. Traditional Chinese medicine promotes health and treats disease through acupuncture, herbal remedies, massage, acupressure, qigong, and moxibustion. Ayurveda incorporates a combination of remedies, such as herbs, purgatives, and oils, to treat disease. DIF: Understand REF: 817, Table 35-1 OBJ: Describe the various treatments associated with complementary and alternative medicine. TOP: Assessment MSC: CPNRE: Foundations of Practice 7. A patient asks about the new clinic in town that is staffed by allopathic and complementary
practitioners. What type of clinic would this be? a. Integrative medical clinic. b. Ayurvedic clinic. c. Naturopathic medical clinic. d. Healing intention clinic. ANS: A
NURSING TB.COM
An integrative medical program allows health care consumers to be treated by a team of providers consisting of both allopathic and complementary practitioners. The other options are solely complementary clinics. DIF: Understand REF: 817 OBJ: Identify the differences between complementary and alternative approaches in health care. TOP: Planning MSC: CPNRE: Foundations of Practice 8. Which of the following can be described as a range of practices that relax the body and calm
the mind? a. Mindfulness. b. Imagery. c. Meditation. d. Biofeedback. ANS: C
Meditation comprises a range of practices that relax the body and calm the mind. Mindfulness helps build awareness of the present moment with an attitude of openness. With imagery, patients treat pathological conditions by concentrating on an image or series of images. Biofeedback involves the use of instruments to provide a person with visual or auditory information about autonomic physiological functions. DIF: Understand
REF: 817, Table 35-1
Canadian Fundamentals of Nursing 6th Edition Potter Test Bank OBJ: Describe the various treatments associated with complementary and alternative medicine. TOP: Planning MSC: CPNRE: Foundations of Practice 9. A therapeutic touch practitioner scans the patient’s body to identify what? a. Blocked chakra. b. Accumulated tension. c. The flow of qi. d. Structural and functional imbalance. ANS: B
The therapeutic touch practitioner scans the body to identify areas of accumulated tension. The practitioner will then attempt to redirect these accumulated energies back into balance. Chiropractic therapy involves balancing structural and functional imbalance through spinal manipulation. Qi is the vital life force in traditional Chinese medicine. Chakras are the centres of spiritual power involved in reiki therapy. DIF: Understand REF: 822 OBJ: Describe the various treatments associated with complementary and alternative medicine. TOP: Planning MSC: CPNRE: Foundations of Practice 10. Which of the following diets is believed to have anticancer properties, and consists of 40 to
60% whole cereal grain, 20 to 30% vegetables, and 5 to 10% beans? a. Macrobiotic diet. b. Gerson therapy. c. Mediterranean diet. d. Paleo diet. ANS: A
NURSINGTB.COM
The macrobiotic diet is a predominantly vegan diet (but it also includes white meat fish, occasional fruits, seeds, and nuts) believed to have anticancer properties. The diet consists of 40 to 60% whole cereal grain, 20 to 30% vegetables, and 5 to 10% beans. The Gerson therapy advocates a low-salt, high-potassium organic diet of fruit juices, raw vegetables, and nutritional supplements and is primarily used in the treatment of cancer. The Mediterranean diet is high in whole grains, nuts, fruits, vegetables, and omega-3 essential fatty acids, which provide a protective benefit for cardiac health and various inflammatory diseases. The Paleo diet does not include grains or legumes. DIF: Understand REF: 817, Table 35-1 OBJ: Describe the various treatments associated with complementary and alternative medicine. TOP: Planning MSC: CPNRE: Foundations of Practice 11. What is a basic foundational principle of chiropractic care? a. Structure and function coexist. b. Chiropractic care is risk free. c. Human beings need external hands-on care to be healthiest. d. Subluxation will cause permanently restricted joint movement. ANS: A
Canadian Fundamentals of Nursing 6th Edition Potter Test Bank Chiropractors use their hands as instruments to restore structural and functional balance. Practitioners of chiropractic care believe that general health is affected via the nervous system. Chiropractic care is not risk free, just as allopathic medical care is not risk free. Subluxation eventually can cause permanently restricted joint movement, but this is not a basic foundational principle of chiropractic care. Although hands-on care is used in chiropractic care, a natural diet and regular exercise are critical components for the body to function properly. DIF: Understand REF: 822 OBJ: Understand how complementary and alternative approaches assist in health promotion and disease prevention. TOP: Planning MSC: CPNRE: Collaborative Practice 12. A holistic nurse is a nurse who does what? a. Recommends a vegan diet for all patients. b. Recognizes the mind–body–spirit connection. c. Provides spiritual literature to patients. d. Knows about resources for fresh herbs. ANS: B
The mind–body–spirit connection is important to a nurse with a holistic style of nursing. Nursing involves caring for the entire patient. A vegan diet is an aspect of dietary treatment, but it does not allow for alternative viewpoints or well-rounded care. Spiritual literature and knowing about resources are excellent alternative aspects of allopathic medicine, but they are not specific to holistic nurses. DIF: Understand REF: 819 OBJ: Identify the differences between complementary and alternative approaches in health care. NNURRES TOP: Planning MSC: CP : FI oN unG daT tiB on. sC ofO PM ractice 13. Drawbacks of complementary and alternative therapies would be all of the following, except
which one? a. Lack of evidence-informed guidelines. b. Minimal supportive research studies. c. Strong support by allopathic medical providers. d. Lack of a long tradition of therapies taught in health care education. ANS: C
Allopathic medical providers have not been traditionally educated in complementary and alternative therapies; however, that is beginning to change with courses included in medical and nursing curricula. These providers often feel uncomfortable recommending CAM because of that lack of knowledge and training. In addition, research on CAM is limited, which makes it difficult to create guidelines; this is another obstacle to the use of CAM, but the research base is beginning to expand. DIF: Understand REF: 824 OBJ: Identify the differences between complementary and alternative approaches in health care. TOP: Planning MSC: CPNRE: Foundations of Practice 14. Physiological symptoms of a stress response include all of the following except which one? a. Constricted pupils. b. Tachycardia.
Canadian Fundamentals of Nursing 6th Edition Potter Test Bank c. Tachypnea. d. Elevated blood pressure. ANS: A
Pupils dilate during stressful situations to increase visual capacity and sight, especially in darkened conditions; this is a survival mechanism. The physiological cascade of changes associated with the stress response includes increased heart and respiratory rates, elevated blood pressure, muscle tightening, increased metabolic rate, a sense of foreboding, fear, nervousness, irritability, and a negative mood. DIF: Understand REF: 821 OBJ: Explain the scope of naturopathic and chiropractic medicine. TOP: Planning MSC: CPNRE: Foundations of Practice 15. Which of the following is a long-term outcome for an individual who is learning relaxation
therapy? a. Identifying tension in the body and consciously releasing the tension. b. Having no tension in his or her life. c. Increasing delta brain activity. d. Increasing the focus on himself or herself. ANS: A
Long-term relaxation therapy focuses on active recognition and release of stress. It is not realistic to expect a tension-free life. Delta brain waves are high-amplitude brain waves associated with the deepest stages of sleep. The outcome of relaxation therapy is not to put a person to sleep or to increase the focus on self. DIF: Understand REF: 82N 1URSINGTB.COM OBJ: Understand how complementary and alternative approaches assist in health promotion and disease prevention. TOP: Planning MSC: CPNRE: Foundations of Practice 16. What is one benefit of meditation over other forms of behavioural therapy? a. Meditation improves communication skills. b. Meditation cures hypertension. c. Meditation does not require memorization. d. Meditation balances insulin and other body hormones. ANS: C
Meditation involves relaxing the body and stilling the mind, which anyone can do through a variety of measures. Meditation does not improve a person’s communication skills, cure any illness, or balance any bodily hormones. DIF: Understand REF: 821 OBJ: Understand how complementary and alternative approaches assist in health promotion and disease prevention. TOP: Planning MSC: CPNRE: Foundations of Practice 17. Which one of the following meals would be high in phytochemicals? a. Sandwich with whole grain bread and lean turkey. b. Grilled salmon and whole-grain rice. c. Spinach salad with roasted beets. d. Chicken breast and mashed potatoes.
Canadian Fundamentals of Nursing 6th Edition Potter Test Bank ANS: C
Phytochemicals can be found in several plant-based foods, such as spinach, carrots, oranges, beets, red peppers, blueberries, and more. The Canadian Cancer Society advises choosing a variety of colourful foods each day. DIF: Apply REF: 824 OBJ: Identify the differences between complementary and alternative approaches in health care. TOP: Planning MSC: CPNRE: Foundations of Practice 18. Which substance could cause an abnormal drug interaction in a patient taking an
antidepressant medication? a. Digoxin. b. Aspirin. c. Chamomile. d. Ginger. ANS: C
Chamomile is known to cause drowsiness. Other herbal supplements can affect serotonin levels in the brain and can affect antidepressant medication. A patient should check with a provider before combining herbal supplements with medications. Antidepressants do not interact with digoxin, aspirin, or ginger. DIF: Understand REF: 825 OBJ: Recognize botanical treatments as an approach to complementary and alternative medicine. TOP: Planning MSC: CPNRE: Foundations of Practice 19. Which of the following statements is true concerning the regulation of herbal therapies in
Canada? NURSINGTB.COM a. Herbal remedies are harmless and do not cause side effects because they are natural plants. b. A natural health product that has been approved under the Canadian Food and Drugs Act will carry a Drug Identification Number (DIN) on its label. c. All herbal therapy manufacturers must follow strict quality control and manufacturing guidelines. d. There is no need to be concerned about concurrent use of herbal therapies and prescription or over-the-counter medications. ANS: B
According to the federal Food and Drugs Act, all drugs must be proven safe and effective before they can be sold to the public. Natural health products that have been approved for sale under the new regulations have been assigned a DIN (DIN-HM for homeopathic medicines) or Natural Product Number (NPN). These numbers certify that the product has passed a review of their formulation, labelling, and instructions for use. Health Canada advises Canadians to use only health products that carry a DIN, DIN-HM, or NPN on the label. Many people believe, incorrectly, that because herbs are natural plants, they will not cause harm or side effects. Not all companies follow strict quality control and manufacturing guidelines, which set standards for acceptable levels of pesticides, residual solvents, bacterial levels, and heavy metals. For this reason, herbal medicine should be purchased from reputable manufacturers. Concurrent use of herbal or other natural products with prescription or over-the-counter medications should be monitored. Herbs can inhibit or enhance a particular medication’s site of action.
Canadian Fundamentals of Nursing 6th Edition Potter Test Bank
DIF: Apply REF: 825 OBJ: Identify the differences between complementary and alternative approaches in health care. TOP: Assessment MSC: CPNRE: Foundations of Practice 20. The complementary and alternative therapy that is known to alter immune function is which
of the following? a. Biofeedback. b. Imagery. c. Breath work. d. Acupuncture. ANS: B
Imagery and visualization techniques produce a powerful psychophysiological response. Imagery is commonly used by patients with cancer to improve their immune system despite radical treatments such as chemotherapy. Biofeedback teaches the patient to analyze triggers of stress. Acupuncture involves puncture of the skin, which may increase risk of infection and should be used with caution in someone who is immunocompromised. Breath work involves using a variety of breathing patterns to relax, invigorate, or open emotional channels. DIF: Understand REF: 821 OBJ: Describe the various treatments associated with complementary and alternative medicine. TOP: Planning MSC: CPNRE: Foundations of Practice
NURSINGTB.COM
Canadian Fundamentals of Nursing 6th Edition Potter Test Bank
Chapter 36: Activity and Exercise Potter et al: Canadian Fundamentals of Nursing, 6th Edition MULTIPLE CHOICE 1. The coordinated efforts of the musculoskeletal and nervous system maintain balance, posture,
and body alignment. What does body alignment refer to? a. A low centre of gravity balanced over a wide base of support. b. The result of weight, centre of gravity, and balance. c. The relationship of one body part to another. d. The force that occurs in a direction to oppose movement. ANS: C
Body alignment refers to the relationship of one body part to another body part along a horizontal or vertical line. Body balance is the state in which a relatively low centre of gravity is balanced over a wide, stable base of support. Coordinated body movement is a result of weight, centre of gravity, and balance. Friction is a force that occurs in a direction to oppose movement. DIF: Remember REF: 833 OBJ: Describe the role of the musculoskeletal and nervous systems in the regulation of activity and exercise. TOP: Assessment MSC: CPNRE: Foundations of Practice 2. A structural curvature of the spine associated with vertebral rotation is known as which of the
following? a. Scoliosis. b. Osteogenesis. c. Osteomalacia. d. Arthritis.
NURSINGTB.COM
ANS: A
Scoliosis is a structural curvature of the spine associated with vertebral rotation. Osteogenesis imperfecta is an inherited disorder in which bones are porous, short, bowed, and deformed. Osteomalacia is an uncommon metabolic disease characterized by inadequate and delayed mineralization; as a result, bone is compact and spongy. Arthritis is an inflammatory joint disease characterized by inflammation or destruction of the synovial membrane and articular cartilage and by systemic signs of inflammation. DIF: Remember REF: 837 OBJ: Discuss physiological and pathological influences on body alignment and joint mobility. TOP: Assessment MSC: CPNRE: Foundations of Practice 3. How is joint degeneration unlike arthritis? a. Joint degeneration results only from noninflammatory disease. b. Joint degeneration results only from inflammatory disease. c. Joint degeneration involves overgrowth of bone at the articular ends. d. Joint degeneration affects mostly non–weight-bearing joints. ANS: C
Canadian Fundamentals of Nursing 6th Edition Potter Test Bank Joint degeneration, which can occur with inflammatory and noninflammatory disease, is marked by changes in articular cartilage combined with overgrowth of bone at the articular ends. Degenerative changes commonly affect weight-bearing joints. DIF: Understand REF: 837 OBJ: Discuss physiological and pathological influences on body alignment and joint mobility. TOP: Assessment MSC: CPNRE: Foundations of Practice 4. The nurse is providing care to a patient who is bedridden. To avoid becoming fatigued, the
nurse raises the height of the patient’s bed. The nurse understands that raising the bed helps maintain balance in what way? a. It prevents a shift in the nurse’s base of support. b. It narrows the nurse’s base of support. c. It allows the nurse to bring his or her feet close together. d. It shifts the centre of gravity farther away from the nurse’s base of support. ANS: A
By raising the height of the bed when performing a procedure, the nurse avoids bending too far at the waist and causing a shift in the base of support. Balance is maintained by maintaining proper body alignment and posture through two simple techniques: First, the base of support is widened by separating the feet to a comfortable distance; second, balance is increased by bringing the centre of gravity closer to the base of support. DIF: Understand REF: 841 OBJ: Discuss physiological and pathological influences on body alignment and joint mobility. TOP: Assessment MSC: CPNRE: Foundations of Practice 5. Approximately what percentaNgU eR ofSaI ll N baGcT kB pa. inCiO s aMssociated with manual lifting tasks? a. More than 10%. b. More than 20%. c. More than 40%. d. More than 50%. ANS: D
More than half of all cases of back pain are associated with manual lifting tasks. DIF: Remember REF: 859 OBJ: Describe how to maintain and use proper body mechanics. TOP: Assessment MSC: CPNRE: Foundations of Practice 6. The nurse is preparing to position an immobile patient. Before doing so, the nurse must
understand which of the following? a. Manual lifting is the easier method and should be tried first. b. Following body mechanics principles alone will prevent back injury. c. Body mechanics can be ignored when patient-handling equipment is used. d. Body mechanics alone are not sufficient to prevent injuries. ANS: D
Canadian Fundamentals of Nursing 6th Edition Potter Test Bank Body mechanics alone are not sufficient to prevent musculoskeletal injuries when positioning or transferring patients. The use of patient-handling equipment in combination with proper body mechanics is more effective than either one in isolation. Body mechanics cannot be ignored even when patient-handling equipment is being used. Manual lifting is the last resort, and it is used only when it does not involve lifting most or all of the patient’s weight. DIF: Understand REF: 859 OBJ: Describe how to use proper body mechanics and ergonomics to prevent musculoskeletal injuries. TOP: Assessment MSC: CPNRE: Foundations of Practice 7. The nurse is preparing to reposition a patient. Before doing so, what must the nurse do? a. Assess the weight to be lifted and the assistance needed. b. Attempt to manually lift the patient alone before asking for assistance. c. Attempt a manual lift only when lifting most or all of the patient’s weight. d. Not use the agency lift team if a mechanical lift is available. ANS: A
Before lifting, the nurse assesses the weight to be lifted and determines the assistance needed and the resources available. Manual lifting is the last resort, and it is used when the task at hand does not involve lifting most or all of the patient’s weight. Safe patient-handling equipment should be used in conjunction with agency lift teams to reduce the risk of injury to the patient and members of the health care team. DIF: Apply REF: 859 OBJ: Describe how to maintain and use proper body mechanics. TOP: Implementation MSC: CPNRE: Foundations of Practice
IeNtrGicTaBre.thCrO 8. Isotonic, isometric, and resisN tivUeR isS om eeMcategories of exercise. They are classified according to the type of muscle contraction involved. Of the following exercises, which are considered isotonic? a. Bicycling, swimming, walking, jogging, dancing. b. Tightening or tensing of muscles without moving body parts. c. Push-ups, hip lifting, pushing feet against a footboard on the bed. d. Quadriceps set exercises and contraction of the gluteal muscles. ANS: A
Examples of isotonic exercises are walking, swimming, dance aerobics, jogging, bicycling, and moving arms and legs with light resistance. Isometric exercises involve tightening or tensing of muscles without moving body parts. Examples include quadriceps set exercises and contraction of the gluteal muscles. Examples of resistive isometric exercises are push-ups and hip lifting, as well as pushing against a footboard at the foot of the bed with the feet. DIF: Remember REF: 834 OBJ: Describe the benefits of implementing an exercise program for the purpose of health promotion. TOP: Assessment MSC: CPNRE: Foundations of Practice 9. In planning a physical activity program for a patient, what must the nurse understand? a. Isotonic exercises cause contraction without changing muscle length. b. The best program includes a combination of exercises. c. Isometric contraction involves the movement of body parts. d. Resistive isometric exercises can lead to bone wasting.
Canadian Fundamentals of Nursing 6th Edition Potter Test Bank ANS: B
The best program of physical activity includes a combination of exercises that produce different physiological and psychological benefits. Isotonic exercises cause muscle contractions and changes in muscle length. Isometric exercises involve tightening or tensing of muscles without moving body parts. Resistive isometric exercises help promote muscle strength and provide sufficient stress against bone to promote osteoblastic activity. DIF: Understand REF: 834 OBJ: Describe the benefits of implementing an exercise program for the purpose of health promotion. TOP: Assessment MSC: CPNRE: Foundations of Practice 10. An active lifestyle is important for maintaining and promoting health. In developing an
exercise program, what should the nurse know? a. Physical exercise is contraindicated for patients with chronic illnesses. b. Regular physical activity is beneficial only for the body part that is exercised. c. Physical exercise has no effect on psychological well-being. d. Physical activity enhances functioning of all body systems. ANS: D
Regular physical activity and exercise enhance the functioning of all body systems, including cardiopulmonary functioning, musculoskeletal fitness, weight control and maintenance, and psychological well-being. It is also essential in treatment of chronic illness. DIF: Understand REF: 834 OBJ: Describe the benefits of implementing an exercise program for the purpose of health promotion. TOP: Assessment MSC: CPNRE: Foundations of Practice
xerc prog r old 11. The nurse is developing an eN URiseSI NGram TBf.oC OMer patients living in a nursing home. To
develop a beneficial health promotion program, what must the nurse need to understand about older people? a. Exercise is of very little benefit because the patients are old. b. It is important to disregard their current interests in favour of exercise. c. No physical benefit can be gained without a formal exercise program. d. Adjustments to exercise programs may have to be made to prevent problems. ANS: D
Exercise is extremely beneficial for older persons, but adjustments to an exercise program may have to be made for those of advanced age to prevent problems. When developing an exercise program for any older person, the nurse should consider not only the person’s current activity level, range of motion, muscle strength and tone, and response to physical activity, but also the person’s interests, capacities, and limitations. Older persons who are unable to participate in a formal exercise program are able to achieve the benefits of improved joint mobility and enhanced circulation by simply stretching and exaggerating movements during performance of routine activities of daily living. DIF: Understand REF: 849 | 860 OBJ: Describe important factors to consider when planning an exercise program for patients across the lifespan and for those with specific chronic illnesses. TOP: Assessment MSC: CPNRE: Foundations of Practice
Canadian Fundamentals of Nursing 6th Edition Potter Test Bank 12. The nurse is attempting to start an exercise program in a local community as a health
promotion project. In explaining the purpose of the project, what does the nurse explain to community leaders? a. A sedentary lifestyle contributes to the development of health-related problems. b. The recommended frequency of workouts should be twice a day. c. An exercise prescription should incorporate aerobic exercise only. d. The purpose of weight training is to bulk up muscles. ANS: A
A sedentary lifestyle contributes to the development of health-related problems. A holistic approach is taken to develop overall fitness and includes warm-ups, aerobic exercise, resistance training, weight training, and so forth. The recommended frequency of aerobic exercise is three to five times per week or every other day for approximately 30 minutes. Cross-training is recommended for the patient who prefers to exercise every day. Some patients use weight training to bulk up their muscles. However, the purposes of weight training from a health perspective are to develop tone and strength and to simulate and maintain healthy bone. DIF: Understand REF: 856 OBJ: Describe the benefits of implementing an exercise program for the purpose of health promotion. TOP: Assessment MSC: CPNRE: Foundations of Practice 13. The patient is eager to begin his exercise program with a 3.2-km (2-mile) jog. The nurse
instructs the patient to warm up with stretching exercises. The patient states that he is ready and does not want to waste time with a “warm-up.” What does the nurse explain about the warm-up? a. It allows the body to readjust gradually to baseline functioning. N crea RSses INthe GTpotenti B.CO M b. It prepares the body and deU al for injury. c. It should not involve stretching exercises because they can lead to injury. d. It should be performed with high intensity to prepare for the coming challenge. ANS: B
The warm-up activity prepares the body for activity and decreases the potential for injury. It usually lasts about 5 to 10 minutes and may include stretching, calisthenics, or aerobic activity, or a combination of these, performed at a low intensity. The cool-down period allows the body to readjust gradually to baseline functioning and provides an opportunity to combine movement such as stretching with relaxation-enhancing mind–body awareness. DIF: Understand REF: 856 OBJ: Describe the benefits of implementing an exercise program for the purpose of health promotion. TOP: Assessment MSC: CPNRE: Foundations of Practice 14. Many patients find it difficult to incorporate an exercise program into their daily lives because
of time constraints. For these patients, it is beneficial to reinforce that many activities of daily living (ADLs) are used to accumulate the recommended 30 minutes or more per day of moderate-intensity physical activity. When instructing these patients, what should the nurse explain? a. Housework is not considered an aerobic exercise. b. To strengthen back muscles, the patient should bend using back muscles. c. Daily chores should begin with gentle stretches. d. The patient should stick to one chore until it is done before beginning a new one.
Canadian Fundamentals of Nursing 6th Edition Potter Test Bank
ANS: C
Daily chores should begin with gentle stretches. Housework is considered aerobic exercise. To make it more aerobic, the person can work faster and scrub harder. To prevent back injury, people should bend their legs rather than their back. Cleaning activities should be alternated to prevent overworking the same muscle groups. DIF: Understand REF: 856 OBJ: Describe important factors to consider when planning an exercise program for patients across the lifespan and for those with specific chronic illnesses. TOP: Assessment MSC: CPNRE: Foundations of Practice 15. The nurse is developing an exercise plan for someone with a diagnosis of heart failure and
exercise intolerance. In doing so, what should the nurse include? a. Plan for 20 minutes of continuous aerobic activity and increase as tolerated. b. Perform 5-minute walks at the patient’s pace at least 2 times a day. c. Instruct the patient that he should not take his beta blocker medication on exercise days. d. Encourage a high-calorie diet to plan for extra calorie expenditure. ANS: B
A patient with the diagnosis of exercise intolerance should begin by performing 5-minute walks at his own pace at least twice a day. Such a patient would not be able to tolerate 20 minutes of continuous aerobic activity. Patients should be instructed to take medications as ordered. Low-calorie, low-sodium, and high-protein diets are best for patients with exercise intolerance. DIF: Apply REF: 857 NURSINGTB.C M O an exercise program for patients across OBJ: Describe important factors to consider when planning the lifespan and for those with specific chronic illnesses. TOP: Implementation MSC: CPNRE: Foundations of Practice 16. Which of the following exercise activities would probably provide the opportunity for
mind–body awareness? a. Warm-up activity. b. Resistance training. c. Aerobic exercise. d. Cool-down activity. ANS: D
The cool-down period allows the body to readjust gradually to baseline functioning and provides an opportunity to combine movement such as stretching with relaxation-enhancing mind–body awareness. The warm-up activity prepares the body and decreases the potential for injury. Aerobic exercise includes running, bicycling, and jumping rope and is the main portion of exercise activity; it precedes the cool-down period. Resistance training increases muscle strength and endurance and is associated with improved performance of daily activities but not with enhancing mind–body awareness. DIF: Analyze REF: 860 OBJ: Describe important factors to consider when planning an exercise program for patients across the lifespan and for those with specific chronic illnesses. TOP: Evaluate MSC: CPNRE: Foundations of Practice
Canadian Fundamentals of Nursing 6th Edition Potter Test Bank
17. The patient is brought to the emergency department with possible injury to his shoulder. To
help determine the degree of injury, what should the nurse evaluate? a. The patient’s gait. b. The patient’s range of motion. c. Fine motor coordination. d. Activity tolerance. ANS: B
Evaluating range of motion is one assessment technique used to determine the degree of damage or injury to a joint. Gait is the manner or style of walking; it may have little bearing on the shoulder damage. Assessing fine motor coordination would be beneficial in helping to assess the patient’s ability to perform tasks but would not help in evaluating the shoulder. Activity tolerance refers to the type and amount of exercise or activity a person is able to perform. Damage to the shoulder would affect activity tolerance, but the degree of activity intolerance would not help assess the damage done to the shoulder. DIF: Apply REF: 854 OBJ: Describe how to assess patients for activity intolerance. MSC: CPNRE: Foundations of Practice
TOP: Implementation
18. The nurse is examining a patient who is at the clinic with severe elbow pain. Of the following
situations, which would cause the nurse to suspect a ligament tear or joint fracture? a. Range of motion of the elbow is limited. b. Joint motion is greater than normal. c. The patient has arthritis. d. The elbow cannot be moved (frozen). B.C M ANS: B
NURSING T
O
Increased mobility (beyond normal) of a joint may indicate connective tissue disorders, ligament tears, or possible joint fractures. Limited range of motion often indicates inflammation such as arthritis, fluid in the joint, altered nerve supply, or contractures (frozen joints). DIF: Analyze REF: 854 OBJ: Describe how to assess patients for activity intolerance. MSC: CPNRE: Foundations of Practice
TOP: Evaluate
19. The patient has been bedridden for several months owing to severe heart disease. In
determining a plan of care for this patient that will address his activity level, the nurse formulates which of the following nursing diagnoses? a. Fatigue related to poor physical condition. b. Impaired gas exchange related to decreased cardiac output. c. Decreased cardiac output related to decreased myocardial contractility. d. Activity intolerance related to physical deconditioning. ANS: D
Canadian Fundamentals of Nursing 6th Edition Potter Test Bank When activity and exercise are problems for a patient, nursing diagnoses often focus on the individual’s ability to move. The diagnostic label directs nursing interventions. In this case, physical deconditioning must be addressed in relation to activity level; perhaps an early goal is 6-minute walks twice a day. Physical deconditioning is the cause of fatigue as well, so it would take priority over the diagnosis of Fatigue related to poor physical condition. Decreased cardiac output and myocardial contractility are serious concerns that must be addressed before activity intolerance to keep the patient safe and to help determine the level of exercise that the patient can tolerate, but reconditioning of the patient’s body will help improve contractility and cardiac output. DIF: Apply REF: 833 | 837 OBJ: Develop a nursing care plan for a patient with impaired mobility and activity intolerance. TOP: Implementation MSC: CPNRE: Foundations of Practice 20. The patient weighs 204.5 kg (450 pounds) and complains of shortness of breath with any
exertion. His health care provider has recommended that he begin an exercise program. He states that he can hardly get out of bed and just cannot do anything around the house. To focus on the cause of the patient’s problems, the nurse devises which of the following nursing diagnoses? a. Activity intolerance related to excessive weight. b. Activity intolerance related to bed rest. c. Impaired gas exchange related to shortness of breath. d. Imbalanced nutrition: less than body requirements. ANS: A
The diagnostic label directs nursing interventions. This requires the correct selection of related factors. For example, Activity intolerance related to excess weight necessitates very different N R torIN Grolonge B.CO M interventions than if the relatedUfacS is pT d bed rest. In this case, the intolerance is related to the patient’s excessive weight. He is not on bed rest, although it is difficult for him to get out of bed. Shortness of breath is a symptom, not a cause, of Impaired gas exchange, and so this nursing diagnosis is not applicable. The patient does have an imbalance of nutrition, but it is more than body requirements. DIF: Apply REF: 855 OBJ: Develop a nursing care plan for a patient with impaired mobility and activity intolerance. TOP: Implementation MSC: CPNRE: Foundations of Practice 21. To promote resistive isometric exercise, the nurse helps the patient on bed rest by encouraging
which one of the following? a. Quadriceps setting. b. Gluteal muscle contraction. c. Moving the arms and legs in circles. d. Pushing against a footboard. ANS: D
In resistive isometric exercises, the individual contracts the muscle while pushing against a stationary object or resisting the movement of an object. An example of a resistive isometric exercise is pushing against a footboard. Quadriceps setting and gluteal muscle contraction are examples of isometric exercise. Moving the arms and legs in a circle is an example of isotonic exercise.
Canadian Fundamentals of Nursing 6th Edition Potter Test Bank DIF: Apply REF: 834 OBJ: Develop a nursing care plan for a patient with impaired mobility and activity intolerance. TOP: Implementation MSC: CPNRE: Foundations of Practice 22. The patient is admitted with a stroke. The prognosis is uncertain, but the patient is unable to
move his right arm and leg. What should the nurse keep in mind? a. Active range of motion is the only thing that will prevent contractures from forming. b. Passive range of motion must be instituted to help prevent contracture formation. c. Range-of-motion exercises should be started 2 days after the patient is stable. d. Range-of-motion exercises should be done on major joints only. ANS: B
When patients cannot participate in active range of motion, passive range of motion must be instituted to maintain joint mobility and prevent contractures. Passive range-of-motion exercises can be substituted for active exercises when needed. For the patient who does not have voluntary motor control, passive range-of-motion exercises are the exercises of choice. Unless contraindicated, the nursing care plan includes exercising each joint (not just major joints) through as nearly a full range of motion as possible. The nurse should initiate passive range-of-motion exercises as soon as the patient loses the ability to move the extremity or joint. DIF: Understand REF: 838 | 861 OBJ: Describe the interventions for maintaining activity tolerance and mobility during the acute, restorative, and continuing care of patients. TOP: Assessment MSC: CPNRE: Foundations of Practice 23. A 40-year-old healthy womaN nU haR sS prI esNeG ntT edBt. oC thO e cMlinic for a physical examination. When
the patient asks the nurse to recommend the amount of time she should spend per week doing moderate-intensity physical activity, what should the nurse recommend? a. One hour per week or 15 minutes a day for four days. b. Thirty minutes or more a day, for a weekly total of 150 minutes. c. One hour per day, for a weekly total of seven hours. d. Ten minutes a day, for a total of 60 minutes per week. ANS: B
According to the Canadian Diabetes Association, adults should accumulate 150 minutes or more per week of moderate-intensity (brisk) physical activity. Neither 1 hour per week, or 15 minutes a day for four days, nor 10 minutes a day for a total of 1 hour per week is considered an adequate amount of time for exercise for adults. One hour per day, for a weekly total of 7 hours exceeds the recommended amount of time for exercise for adults, but the statement does not specify the intensity of the physical activity. DIF: Apply REF: 862, Table 36-3 OBJ: Describe the interventions for maintaining activity tolerance and mobility during the acute, restorative, and continuing care of patients. TOP: Implementation MSC: CPNRE: Foundations of Practice 24. The nurse is ambulating a patient in the hall when she notices that he is beginning to fall.
What should the nurse do? a. Grab the patient and hold him tight to prevent the fall.
Canadian Fundamentals of Nursing 6th Edition Potter Test Bank b. Gently lower the patient to the floor. c. Jump back and let the patient fall naturally. d. Push the patient against the wall and guide him to the floor. ANS: B
If the patient has a fainting episode or begins to fall, the nurse should assume a wide base of support with one foot in front of the other, thus supporting the patient’s body weight. Then the nurse extends one leg, lets the patient slide against the leg, and gently lowers the patient to the floor, protecting the patient’s head. Grabbing the patient will shift the nurse’s centre of gravity and may lead to a back injury. Allowing the patient to fall could lead to head injury for the patient. Pushing the patient against the wall could also cause the patient to hit his head and cause injury. DIF: Apply REF: 862 OBJ: Describe how to use proper body mechanics and ergonomics to prevent musculoskeletal injuries. TOP: Implementation MSC: CPNRE: Foundations of Practice 25. In assisting the patient to exercise, which of the following should the nurse do? a. Expect that pain will occur with exercise of unused muscle groups. b. Set the pace for the exercise class. c. Force muscles or joints to go just beyond resistance. d. Stop the exercise if pain is experienced. ANS: D
The nurse should assess for pain, shortness of breath, or a change in vital signs. If any of these is present, the exercise should be stopped. Each patient should be allowed to exercise at his or her own pace. The nurse asseN sses ntT liB m. itC atio nMs and must not force a muscle or a joint Rfor Ijoi G U S N O during exercise. DIF: Apply REF: 860 OBJ: Describe the interventions for maintaining activity tolerance and mobility during the acute, restorative, and continuing care of patients. TOP: Implementation MSC: CPNRE: Foundations of Practice 26. The nurse is developing a plan of care for a patient with a diagnosis of activity intolerance. Of
the following strategies, which has the best chance of maintaining patient compliance? a. Performing 20 minutes of aerobic exercise daily with 10-minute warm-up and cool-down periods. b. Instructing the patient to use an exercise log to record day, time, duration, and responses to exercise activity. c. Instructing the patient on the evils of not exercising, and getting her to take responsibility for her current health status. d. Arranging for the patient to join a gym for which she will have to pay, so that she does not need to depend on insurance. ANS: B
Canadian Fundamentals of Nursing 6th Edition Potter Test Bank Keeping a log may increase adherence to an exercise prescription. Cross-training (combination of exercise activities) provides variety to combat boredom and increases the potential for total body conditioning as opposed to daily aerobic exercise. “Blaming” a patient for his or her health status is usually counterproductive. Instead, the nurse should instruct the patient about the physiological benefits of a regular exercise program. Developing a plan of exercise that the patient may perform at home may improve compliance. DIF: Evaluate REF: 858 OBJ: Evaluate the nursing care plan for maintaining activity and exercise for patients across the lifespan and with specific chronic illnesses. TOP: Evaluate MSC: CPNRE: Foundations of Practice 27. The nurse is working with the patient in developing an exercise plan. The patient tells the
nurse that she just will not participate in a formal exercise program. The nurse then suggests that exercise activities can be incorporated into activities of daily living (ADLs). The patient seems to be agreeable to that concept. Of the following activities, which would be considered a formal resistance training activity? a. Doing laundry. b. Making the bed. c. Ironing. d. Kneading bread. ANS: D
Formal resistance training includes weight training, but the same benefits can be obtained by performing ADLs such as pushing a vacuum cleaner, raking leaves, shovelling snow, and kneading bread. DIF: Evaluate REF: 85N 6URSINGTB.COM OBJ: Evaluate the nursing care plan for maintaining activity and exercise for patients across the lifespan and with specific chronic illnesses. TOP: Evaluate MSC: CPNRE: Foundations of Practice 28. Bones perform five functions in the body: support, protection, movement, mineral storage,
and hematopoiesis. In body mechanics, which functions are the most important? a. Support and movement. b. Protection and mineral storage. c. Support and hematopoiesis. d. Support and protection. ANS: A
Bones perform five functions in the body: support, protection, movement, mineral storage, and hematopoiesis. In body mechanics, two of these functions—support and movement—are most important. In support, bones serve as the framework and contribute to the shape, alignment, and positioning of body parts. In movement, bones, together with their joints, constitute levers for muscle attachment. As muscles contract and shorten, they pull on bones, producing joint movement. Protection involves encasing the soft tissue organs in a protective cage. Mineral storage helps to strengthen bones but also helps regulate blood levels of certain nutrients. Hematopoiesis is the formation of blood cells. DIF: Remember REF: 834 OBJ: Describe the role of the musculoskeletal and nervous systems in the regulation of activity and
Canadian Fundamentals of Nursing 6th Edition Potter Test Bank exercise.
TOP: Assessment
MSC: CPNRE: Foundations of Practice
29. The student is planning to assess the activity tolerance of a patient. The nursing instructor
would need to provide further instruction to the student who plans evaluate which one of the following? a. Emotional factors. b. Age. c. Pregnancy status. d. Race. ANS: D
Factors influencing activity tolerance include physiological factors such as skeletal abnormalities, emotional factors such as anxiety/depression, developmental factors such as age and gender, and pregnancy status. Race is not a factor because people of all races are faced with similar factors that affect their activity tolerance. DIF: Apply REF: 854, Box 36-7 OBJ: Describe how to assess patients for activity intolerance. MSC: CPNRE: Foundations of Practice
NURSINGTB.COM
TOP: Implementation
Canadian Fundamentals of Nursing 6th Edition Potter Test Bank
Chapter 37: Quality and Patient Safety Potter et al: Canadian Fundamentals of Nursing, 6th Edition MULTIPLE CHOICE 1. A home health nurse is performing a home assessment for safety. Which of the following
comments by the patient would indicate a need for further education? a. “I will schedule an appointment with a chimney inspector next week.” b. “Daylight savings is the time to change batteries on the carbon monoxide detector.” c. “If I feel dizzy when using the heater, I need to have it inspected.” d. “If I’m cooking for only myself, I don’t need to wash my hands.” ANS: D
Nurses should teach basic techniques for safe food handling, such as handwashing and checking spoilage. Handwashing should be done before all food preparation. Checking the chimney and heater, changing the batteries on the detector, and following up on symptoms such as dizziness, nausea, and fatigue are all statements that would indicate that the individual has understood the education. DIF: Apply REF: 875 OBJ: Discuss methods to prevent and reduce safety risks. MSC: CPNRE: Foundations of Practice
TOP: Evaluate
2. The nurse is caring for an elderly patient admitted with nausea, vomiting, and diarrhea. Upon
completing the health history, which priority concern would require collaboration with social Ns he RSalth INcar GT B.C? M services to address the patient’U e needsO a. The electricity was turned off 2 days ago. b. The water comes from the county water supply. c. A son and family recently moved into the home. d. The home is not furnished with a microwave oven. ANS: A
Electricity is needed for refrigeration of food, and lack of electricity could have contributed to the nausea, vomiting, and diarrhea by potentially leading to food poisoning. This discussion about the patient’s electrical needs can be referred to social services. The water supply, the increased number of individuals in the home, and not having a microwave may or may not be concerns but do not pertain to the current health care needs of this patient. DIF: Understand REF: 888 OBJ: Discuss methods to prevent and reduce safety risks. MSC: CPNRE: Foundations of Practice
TOP: Implementation
3. A homeless adult patient presents to the emergency department. The nurse obtains the
following vital signs: temperature 34.9°C (94.8°F), blood pressure 100/56, apical pulse 56, respiratory rate 12. Which of the vital signs should be addressed immediately? a. Respiratory rate b. Temperature c. Apical pulse d. Blood pressure
Canadian Fundamentals of Nursing 6th Edition Potter Test Bank
ANS: B
Hypothermia is defined as a core body temperature of 35°C (95°F) or below. Homeless individuals are more at risk for hypothermia owing to exposure to the elements. DIF: Apply REF: 888 OBJ: Describe assessment activities designed to identify patients’ physical, psychosocial, and cognitive statuses as they pertain to his or her safety. TOP: Assessment MSC: CPNRE: Foundations of Practice 4. An ambulatory patient is admitted to the extended care facility with a diagnosis of
Alzheimer’s disease. In the Hendrich II Fall Risk Model, what is the most significant indicator of risk for falls? a. Confusion/disorientation/impulsivity b. Dizziness/vertigo c. Symptomatic depression d. Altered elimination ANS: A
According to the Hendrich II Fall Risk Model, the most significant indicator of risk for falls is confusion/disorientation/impulsivity. Dizziness/vertigo, symptomatic depression, and altered elimination are also risk factors; however, they are not the most significant indicators. DIF: Remember REF: 879, Table 37-1 OBJ: Describe nursing interventions specific to patients’ age for reducing safety risks. TOP: Implementation MSC: CPNRE: Foundations of Practice 5. A 1-year-old child is scheduled to receive an intravenous (IV) line. The most appropriate type
N R I G B.C M
of restraint to use for this patieU nt toSpreNvenTt removOal of the IV line is which of the following? a. A wrist restraint. b. A jacket restraint. c. An elbow restraint. d. A mummy restraint. ANS: D
A mummy restraint is used in the short term for a small child or infant for examination or treatment involving the head and neck. This would be the most appropriate type of restraint to use for a 1-year-old who is going to receive an IV line. The wrist restraint maintains immobility of an extremity to prevent the patient from removing a therapeutic device, such as an IV tube. It would not be the best choice for starting an IV on a 1-year-old. The jacket restraint is often used to prevent a patient from getting up and falling. This, too, is not the best choice for starting an IV line. An elbow restraint is commonly used with infants and children to prevent elbow flexion, as in cases in which an IV line is already in place. DIF: Apply REF: 894 OBJ: Describe nursing interventions specific to patients’ age for reducing safety risks. TOP: Implementation MSC: CPNRE: Foundations of Practice 6. A visiting nurse completes an assessment of the ambulatory patient in the home and
determines the nursing diagnosis of Risk for injury related to decreased vision. On the basis of this assessment, the patient will benefit the most from which of the following actions? a. Installing fluorescent lighting throughout the house
Canadian Fundamentals of Nursing 6th Edition Potter Test Bank b. Evaluating the need to reposition furniture c. Maintaining complete bed rest in a hospital bed with side rails d. Applying physical restraints ANS: B
Evaluating the positioning of furniture in the room and stairways is the best intervention to help prevent falls for the patient with decreased vision. Attempts should be made to reduce glare. Light bulbs that are 60 W or less may be increased to 75 W nonglare to help improve visibility. The best intervention to prevent falls is first to orient the patient to the surroundings. Maintaining complete bed rest is not the best option. Complete bed rest can cause other health problems because of a lack of mobility. The patient should not be restrained for poor vision. Attempts should be made to help compensate for the decreased vision to prevent falls. DIF: Analyze REF: 879 OBJ: Describe nursing interventions specific to patients’ age for reducing safety risks. TOP: Diagnosis MSC: CPNRE: Foundations of Practice 7. When teaching a parent about interventions for accidental poisoning, what instruction should
be included regarding flushing a child’s eye, in relation to the water temperature? a. Cold b. Lukewarm c. Room temperature d. Above room temperature ANS: C
The nurse should teach the parent that an eye flush is to be done with water at room temperature. The water for an eye flush is not to be cold, lukewarm, or above room temperature. N R I G B.C M
U S N T
O
DIF: Apply REF: 885, Box 37-8 OBJ: Describe nursing interventions specific to patients’ age for reducing safety risks. TOP: Implementation MSC: CPNRE: Foundations of Practice 8. The nurse knows that children in late infancy and toddlerhood are at risk for injury from
which of the following? a. Learning to walk. b. Trying to pull up on furniture. c. Being dropped by a caregiver. d. Growing ability to explore and oral activity. ANS: D
Injury is a leading cause of death in children over age 1, which is closely related to normal growth and development because of the child’s increased oral activity and growing ability to explore the environment. DIF: Understand REF: 883 OBJ: Discuss methods to prevent and reduce safety risks. MSC: CPNRE: Foundations of Practice
TOP: Evaluate
9. A nurse is teaching a community group of school-aged parents about safety. The most
important item to prioritize and explain is how to check the proper fit of which one of the following?
Canadian Fundamentals of Nursing 6th Edition Potter Test Bank a. b. c. d.
A bicycle helmet. Swimming goggles. Soccer shin guards. Baseball sliding shorts.
ANS: A
Bicycle-related injuries are a major cause of death and disability among children. Proper fit of the helmet helps to decrease head injuries resulting from bicycle accidents. Goggles, shin guards, and sliding shorts are important sports safety equipment and should fit properly, but they do not protect from this leading cause of death. DIF: Understand REF: 886 OBJ: Discuss methods to prevent and reduce safety risks. TOP: Implementation | Teaching | Learning MSC: CPNRE: Foundations of Practice 10. The nurse is presenting an educational session on safety for parents of adolescents. The nurse
should include which of the following teaching points? a. Adolescents need unsupervised time with friends two to three times a week. b. Parents and friends should teach adolescents how to drive. c. Adolescents need information about the effects of beer on the liver. d. Adolescents need to be reminded to use seatbelts on long trips. ANS: C
Providing information about drugs and alcohol is important because adolescents may choose to participate in risk-taking behaviours. Adolescents need to socialize but need supervision. Parents can encourage and support learning processes associated with driving, but organized classes can help to decrease m ehicG le aB cc. idC entM s. Seatbelts should be used all the time. NotoRr vI
U S N T
O
DIF: Understand REF: 877 OBJ: Discuss methods to prevent and reduce safety risks. MSC: CPNRE: Foundations of Practice
TOP: Implementation
11. The nurse discussed threats to adult safety with a college group. Which of the following
statements would indicate understanding of the topic? a. “Our campus is safe; we leave our dorms unlocked all the time.” b. “As long as I have only two drinks, I can still be the designated driver.” c. “I am young, so I can work nights and go to school with 2 hours’ sleep.” d. “I guess smoking even at parties is not good for my body.” ANS: D
Lifestyle choices frequently affect adult safety. Smoking conveys great risk for pulmonary and cardiovascular disease. It is prudent to secure belongings. When an individual has been determined to be the designated driver, that individual must not consume alcohol, beer, or wine. Sleep is important, regardless of the age of the individual, for rest and integration of learning. The average young adult needs 61/2 to 8 hours of sleep each night. DIF: Understand REF: 877 OBJ: Describe nursing interventions specific to patients’ age for reducing safety risks. TOP: Evaluate MSC: CPNRE: Foundations of Practice
Canadian Fundamentals of Nursing 6th Edition Potter Test Bank 12. The nurse is caring for a hospitalized patient. Which of the following behaviours alerts the
nurse to consider the need for restraint? a. The patient refuses to call for help to go to the bathroom. b. The patient continues to remove the nasogastric tube. c. The patient gets confused regarding the time at night. d. The patient does not sleep and continues to ask for items. ANS: B
Restraints are utilized only when alternatives have been exhausted, the patient continues a behaviour that can be harmful to himself or herself or to others, and the restraint is clinically justified. In this circumstance, continuing to remove a needed nasogastric tube would meet these criteria. Refusing to call for help, although unsafe, is not a reason for restraint. Getting confused at night regarding the time is not a reason for restraint; nor is not sleeping and bothering the staff to ask for items. DIF: Understand REF: 888 OBJ: Describe methods to evaluate interventions designed to maintain or promote safety. TOP: Assessment MSC: CPNRE: Foundations of Practice 13. The nurse is discussing with a patient’s physician the need for restraint. The nurse indicates
that alternatives have been utilized. What behaviours would indicate that the alternatives are working? a. The patient continues to get up from the chair at the nurses’ station. b. The patient apologizes for being “such a bother.” c. The patient folds three washcloths over and over. d. The sitter leaves the patient alone to go to lunch. ANS: C
NURSINGTB.COM
Offering diversionary activities such as something to hold is a way to keep the hands busy and provides an alternative to restraints. Assigning a room near the nurses’ station or a chair at the desk can be an alternative for continuous monitoring. Getting up constantly can be cause for concern. Apologizing is not an alternative to restraints. Having a sitter sit with the patient to keep him occupied can be an alternative to restraints, but the sitter needs to be with the patient continuously. DIF: Understand REF: 890, Box 37-11 OBJ: Describe assessment activities designed to identify patients’ physical, psychosocial, and cognitive statuses as they pertain to his or her safety. TOP: Evaluate MSC: CPNRE: Foundations of Practice 14. The nurse is caring for a patient who suddenly becomes confused and tries to remove an
intravenous infusion. The nurse begins to develop a plan to care for the patient. Which nursing intervention should take priority? a. Gather restraint supplies. b. Try alternatives to restraint. c. Assess the patient. d. Call the physician for a restraint order. ANS: C
Canadian Fundamentals of Nursing 6th Edition Potter Test Bank When a patient becomes suddenly confused, the priority is to assess the patient, including checking laboratory test and oxygen status, and treating and eliminating the cause of the change in mental status. If interventions and alternatives are exhausted, the nurse, working with the physician, may determine the need for restraints. DIF: Apply REF: 892 OBJ: Describe assessment activities designed to identify patients’ physical, psychosocial, and cognitive statuses as they pertain to his or her safety. TOP: Planning MSC: CPNRE: Foundations of Practice 15. Equipment-related accidents are risks in the health care agency. The nurse assesses for this
risk when using which of the following? a. Intravenous (IV) pumps. b. A device that measures urine. c. Computer-based documentation. d. A manual medication-dispensing device. ANS: A
A dysfunctional IV pump can cause an equipment-related accident, such as too-rapid infusion of IV fluids. Devices used by the nurse to measure urine, computer documentation, and manual dispensing devices can break or malfunction but are not used directly on a patient. DIF: Remember REF: 876 OBJ: Describe the most common safety risks in health care. MSC: CPNRE: Foundations of Practice
TOP: Assessment
16. The nurse is discussing measures to minimize the risk of injury from an automobile accident
with an 84-year-old independNent ien.tC who URfem SIale NGpat TB OMlives alone and says she drives only to church, to the doctor’s office, and for groceries. What change has the greatest potential for affecting the patient’s safety? a. Taking public transportation whenever it is available. b. Planning all trips around church and doctor appointments. c. Planning to drive for short trips and only during daylight hours. d. Arranging for family and friends to drive the patient whenever possible. ANS: C
The nurse should educate the patient regarding safe driving tips (e.g., driving shorter distances and only during daylight hours, using side and rearview mirrors carefully, and looking toward the “blind spot” before changing lanes). Taking public transportation may not meet the needs of an independent patient. Planning all trips around church and doctor appointments may not be realistic. Arranging for family and friends to drive the patient may not meet her needs. DIF: Analyze REF: 887 OBJ: Describe nursing interventions specific to patients’ age for reducing safety risks. TOP: Implementation MSC: CPNRE: Foundations of Practice 17. A patient with an intravenous infusion requests a new gown after bathing. Which of the
following actions is most appropriate? a. Disconnect the intravenous tubing, thread the end through the sleeve of the old gown and through the sleeve of the new gown, and reconnect. b. Thread the intravenous bag and tubing through the sleeve of the old gown and
Canadian Fundamentals of Nursing 6th Edition Potter Test Bank through the sleeve of the new gown without disconnecting. c. Inform the patient that a new gown is not an option while receiving an intravenous
infusion in the hospital. d. Call the charge nurse for assistance because linen use is monitored and this is not a
common procedure. ANS: B
Procedure-related accidents such as contamination of sterile items can occur in the health care setting. Keeping the intravenous tubing intact without breaks in the system is imperative to decrease the risk of infection while changing a patient’s gown and satisfying the patient’s request. DIF: Apply REF: 877, Box 37-3 OBJ: Describe the most common safety risks in health care. MSC: CPNRE: Foundations of Practice
TOP: Implementation
18. A confused patient needs to have restraints applied to prevent him from pulling out his Foley
catheter. Which of the following options can the nurse delegate to an unregulated care provider (UCP)? a. Applying restraints. b. Obtaining a physician’s order to restrain the patient. c. Documenting the events that led to restraining the patient. d. Evaluating the effectiveness of the restraints. ANS: A
The nurse is always responsible for assessment of patients’ safety needs. Although the UCP can apply the restraints under the nurse’s direction, he or she cannot take physicians’ orders, document the events that ledN toUrR estr aini ngTtB he.pC atie SI NG OMnt, or evaluate the effectiveness of the restraints. DIF: Apply REF: 892 OBJ: Develop a nursing care plan for patients whose safety is threatened. TOP: Implementation MSC: CPNRE: Collaborative Practice 19. According to the Hendrich II Fall Risk Model, a patient with a risk score of 6 is considered to
be at which risk level? a. No risk. b. Low risk. c. Medium risk. d. High risk. ANS: D
According to the Hendrich II Fall Risk Model, a score of 5 or greater indicates that the patient is at high risk for a fall, not “no risk,” “low risk,” or “medium risk.” DIF: Analyze REF: 879, Table 37-1 OBJ: Describe nursing interventions specific to patients’ age for reducing safety risks. TOP: Assessment MSC: CPNRE: Foundations of Practice 20. An older patient presents to the emergency department after stepping in front of a car at a
crosswalk. After the patient has been examined in triage, the nurse interviews the patient. Which of the following comments would necessitate follow-up by the nurse?
Canadian Fundamentals of Nursing 6th Edition Potter Test Bank a. b. c. d.
“I try to exercise, so I walk that block almost every day.” “I waited and stepped out when the traffic sign said go.” “The car was going too fast; the speed limit is 20.” “I was so surprised; I didn’t see or hear the car coming.”
ANS: D
As patients age, sensory impairment can increase the risk for injury. The statement that the patient did not see or hear the car coming necessitates follow-up by the nurse; the patient needs hearing and eye examinations. Exercise is important at every stage of development. The patient seemed to comprehend how to cross an intersection correctly and was able to determine the speed of the car. DIF: Apply REF: 877, Box 37-4 OBJ: Describe assessment activities designed to identify patients’ physical, psychosocial, and cognitive statuses as they pertain to his or her safety. TOP: Diagnosis MSC: CPNRE: Foundations of Practice 21. An age-related musculoskeletal change that predisposes the older person to accidents is which
of the following? a. Increase in muscle function. b. Increase in joint mobility. c. Increase in nocturia. d. Decrease in muscle strength. ANS: D
Musculoskeletal system changes associated with aging that increase the risk of accidents in the older people include decreases in muscle strength and function, decreases in joint mobility, increasing brittleneN ssUoR f bo , po stu.rC al O chManges, and limited range of motion. Ines G B S N T Rather than increasing with age, muscle function decreases. Joints become less mobile with age, not more mobile. Nocturia is common in older persons and increases their risk for injury; however, it is a genitourinary change, not a musculoskeletal system change. DIF: Understand REF: 877, Box 37-4 OBJ: Describe nursing interventions specific to patients’ age for reducing safety risks. TOP: Assessment MSC: CPNRE: Foundations of Practice 22. The patient is confused, is trying to get out of bed, and is pulling at the IV infusion tubing.
These data would help to support which nursing diagnosis? a. Risk for poisoning. b. Knowledge deficit. c. Impaired home maintenance. d. Risk for injury. ANS: D
The patient’s behaviours support the nursing diagnosis of Risk for injury. The patient is confused, is pulling at the IV line, and is trying to climb out of bed. Injury could result if the patient falls out of bed or begins to bleed as a result of a pulled line. Nothing in the scenario indicates that this patient lacks knowledge or is at risk for poisoning. Nothing in the scenario refers to the patient’s home maintenance. DIF: Understand REF: 880 OBJ: Identify relevant nursing diagnoses associated with risks to safety.
Canadian Fundamentals of Nursing 6th Edition Potter Test Bank TOP: Diagnosis
MSC: CPNRE: Foundations of Practice
23. A confused patient is restless and continues to try to remove his oxygen and urinary catheter.
What are the priority nursing diagnosis and intervention to implement for this patient? a. Risk for injury: Prevent harm to patient, and use restraints if alternatives fail. b. Deficient knowledge: Explain the purpose of oxygen therapy and the urinary catheter. c. Disturbed body image: Encourage patient to express concerns about body. d. Caregiver role strain: Identify resources to assist with care. ANS: A
The priority nursing diagnosis is Risk for injury. This patient could cause harm to himself by interrupting the oxygen therapy or by damaging the urethra in pulling the urinary catheter out. Before restraining a patient, it is important to implement and exhaust alternatives to restraint. Alternatives can include distraction and providing companionship or supervision. Patients may be moved to a location closer to the nurses’ station; trained sitters or family members may be involved. Nurses need to ensure that patients are provided adequate food, liquid, toileting, and relief from pain. If these and other strategies fail, the patient may need restraints; in this case, an order would need to be obtained for the restraint. This patient may have deficient knowledge, and educating the patient about treatments could be considered as an alternative to restraints; however, the highest priority is preventing injury. This scenario does not indicate that the patient has a disturbed body image or that the patient’s caregiver is strained. DIF: Apply REF: 881 | 882 OBJ: Identify relevant nursing diagnoses associated with risks to safety. TOP: Diagnosis MSC: CPNRE: Foundations of Practice
NURSINGTB.COM
24. Which of the following assessment findings is most critical for a patient who is currently
being restrained with mechanical wrist restraints? a. Angry, loud crying. b. Urinary incontinence. c. Reddened areas on wrists. d. Hands cool to the touch. ANS: D
Although the use of any restraint may be associated with serious complications, including pressure ulcers, constipation, pneumonia, urinary and fecal incontinence, and urinary retention, the most serious concerns are contractures, nerve damage, and circulatory impairment. Coolness of the patient’s hands would indicate poor circulation, which can result in permanent damage. Angry, loud crying, urinary incontinence, and reddened areas on the wrist are concerns, but they are not the most critical concerns for the patient with wrist restraints. DIF: Analyze REF: 890 OBJ: Describe methods to evaluate interventions designed to maintain or promote safety. TOP: Assessment MSC: CPNRE: Foundations of Practice 25. The nurse is providing information regarding safety and accidental poisoning to a
grandmother who will be taking custody of a 1-year-old grandchild. Which of the following comments would indicate that the grandmother needs further instruction?
Canadian Fundamentals of Nursing 6th Edition Potter Test Bank a. If I think my grandchild has come in contact with a poison, I will call my local
poison control centre.” b. “Never induce vomiting if my grandchild drinks bleach.” c. “I should call 9-1-1 if my grandchild loses consciousness.” d. “If my grandchild eats a plant, I should provide syrup of ipecac.” ANS: D
Syrup of ipecac to induce vomiting after ingestion of a poison has not proved effective in preventing poisoning. This medication should not be administered to the child. Phone numbers for poison control centres across Canada can be accessed at http://www.aboutkidshealth.ca/En/HealthAZ/ TestsAndTreatments/Resources/Pages/Poison-Information-Centres-in-Canada.aspx. After a caustic substance such as bleach has been drunk, vomiting should not be induced; vomiting can cause further burning and injury as the medication is eliminated. Loss of consciousness in association with poisoning necessitates calling 9-1-1. DIF: Understand REF: 885, Box 37-8 OBJ: Describe nursing interventions specific to patients’ age for reducing safety risks. TOP: Evaluate MSC: CPNRE: Foundations of Practice 26. An elderly patient presents to the hospital with a history of falls, confusion, and stroke. The
nurse determines that the patient is at high risk for falls. Which of the following interventions is most appropriate for the nurse to take? a. Place the patient in restraints. b. Lock beds and wheelchairs when transferring. c. Place a bath mat outside the tub. d. Silence fall alert alarm upon request of family. ANS: B
N R I G B.C M U S N T O
Locking the bed and wheelchairs when the patient is transferred helps prevent these pieces of equipment from moving during transfer and will assist in the prevention of falls. Patients are not automatically placed in restraints. The restraint process consists of many steps, including thorough assessment and exhausting of alternative strategies. All mats and rugs should be secured to help prevent falls. Silencing alarms upon the request of family is not appropriate and could endanger the patient. DIF: Understand REF: 888 OBJ: Describe nursing interventions specific to patients’ age for reducing safety risks. TOP: Implementation MSC: CPNRE: Foundations of Practice 27. The nurse has been called to a hospital room where a patient is using a hair dryer from home.
The patient received an electrical shock from the dryer and is now unconscious and is not breathing. What is the best next step? a. Ask the family to leave the room. b. Check for a pulse. c. Begin chest compressions. d. Defibrillate the patient. ANS: B
Canadian Fundamentals of Nursing 6th Edition Potter Test Bank In this scenario, the patient is in a hospital setting, and it has been determined that the patient is not conscious and is not breathing. The next step is to check the pulse. An electrical shock can interfere with the heart’s normal electrical impulses and can cause arrhythmias. Checking the pulse helps to determine the need for cardiopulmonary resuscitation (CPR) and defibrillation. DIF: Apply REF: 898 OBJ: Describe nursing interventions specific to patients’ age for reducing safety risks. TOP: Implementation MSC: CPNRE: Foundations of Practice 28. The nurse determines that the patient may need a restraint and recognizes which one of the
following? a. An order for a restraint may be implemented indefinitely until it is no longer required by the patient. b. Restraints may be ordered on an as-needed basis. c. No order or consent is necessary for restraints in long-term care facilities. d. Restraints are to be periodically removed so that the patient can be re-evaluated. ANS: D
Restraints must be periodically removed, and the nurse must reassess the patient to determine whether the restraints continue to be needed. It is not true that an order for a restraint may be implemented indefinitely until it is no longer required by the patient. A physician’s order for restraints must have a limited time frame. If the orders are to be renewed, this should be done within a specified time frame according to the agency’s policy. Restraints are not to be ordered as needed. The use of restraints must be part of the patient’s medical treatment, and an order for restraints must conform to provincial or territorial legislation and agency policy. In some settings, physicians and nurses may order restraints.
NUR I G B.C M S N T O
DIF: Evaluate REF: 891 OBJ: Discuss methods to prevent and reduce safety risks. MSC: CPNRE: Foundations of Practice
TOP: Evaluate
29. The student nurse is caring for a patient in restraints. The nursing instructor has asked the
student to document information about the restraints in the medical record. Which following piece of information that the student wishes to include would prompt the nursing instructor to provide further direction to the student? a. The patient states that her gown is soiled and needs changing. b. Attempts to distract the patient with television are unsuccessful. c. The patient has been placed in bilateral wrist restraints at 0815 hours. d. Released from restraints, active range-of-motion exercises complete. ANS: A
Attempts at alternatives are documented in the medical record, as are type of restraint and time restrained. Assessments related to oxygenation, orientation, skin integrity, circulation, and position are documented, along with release from restraints and patient response. Comments about hygiene are not necessarily required in nursing documentation of restraints. DIF: Apply REF: 892, Skill 37-1 OBJ: Describe nursing interventions specific to patients’ age for reducing safety risks. TOP: Implementation MSC: CPNRE: Professional, Ethical, and Legal Practice
Canadian Fundamentals of Nursing 6th Edition Potter Test Bank
Chapter 38: Hygiene Potter et al: Canadian Fundamentals of Nursing, 6th Edition MULTIPLE CHOICE 1. A number of factors influence a patient’s personal preferences for hygiene. Because of this, it
is important for the nurse to realize which of the following? a. No two individuals perform hygiene in the same manner. b. It is important to standardize a patient’s hygienic practices. c. Hygiene care is always routine and expected. d. Hygiene is not the time to learn about patient needs. ANS: A
No two individuals perform hygiene in the same manner; it is important to individualize the patient’s care according to knowledge about the patient’s unique hygiene practices and preferences. Hygiene care by nurses is never routine; this care requires intimate contact with the patient and communication skills to promote the therapeutic relationship. In addition, during hygiene, the nurse should take time to learn about the patient’s health promotion practices and needs, emotional needs, and health care education needs. DIF: Remember REF: 904 OBJ: Discuss common factors that influence personal hygiene practices. TOP: Assessment MSC: CPNRE: Professional, Ethical, and Legal Practice 2. Social groups influence hygiene preferences and practices, including the type of hygienic
products used and the nature and frequency of personal care. Which of the following Nikel RS I G nfluenc B.COedMby family customs? developmental stages is most lU y toNbe iT a. Adolescent. b. Toddler. c. Adult. d. Older person. ANS: B
During childhood, family customs influence hygiene. As children enter their adolescent years, peer group behaviour often influences personal hygiene. During the adult years, involvement with friends and work groups shapes the expectations people have about their personal appearance. Some older persons’ hygiene practices change because of living conditions and available resources. DIF: Analyze REF: 904 OBJ: Discuss common factors that influence personal hygiene practices. TOP: Assessment MSC: CPNRE: Foundations of Practice 3. The patient received a diagnosis of diabetes 12 years ago. When admitted today, the patient is
unkempt and is in need of a bath and foot care. When questioned about his hygiene habits, the patient tells the nurse that baths are taken once a week where he comes from, although he takes a sponge bath every other day. To provide ultimate care for this patient, what must the nurse understand? a. Personal preferences determine hygiene practices and are unchangeable. b. Patients who appear unkempt place little importance on hygiene practices.
Canadian Fundamentals of Nursing 6th Edition Potter Test Bank c. The patient’s illness may necessitate teaching of new hygiene practices. d. All cultures value cleanliness with the same degree of importance. ANS: C
Each patient has individual desires and preferences about when to bathe, shave, and perform hair care. However, they are not unchangeable. In addition, the nurse must assist the patient in developing new hygiene practices when indicated by an illness or condition. For example, the nurse must teach a patient with diabetes proper foot hygiene. Patients who appear unkempt often need further assessment regarding their ability to participate in daily hygiene. Patients with certain types of physical limitations or disabilities often lack the physical energy and dexterity, or both, to perform hygienic care. Culturally, maintaining cleanliness does not hold the same importance for some ethnic groups as it does for others. DIF: Understand REF: 904 | 905 OBJ: Discuss common factors that influence personal hygiene practices. TOP: Assessment MSC: CPNRE: Foundations of Practice 4. The nurse is caring for a patient who refuses “A.M. care.” When asked why, the patient tells
the nurse that she always bathes in the evening. What should the nurse do? a. Defer the bath until evening and pass on the information to the next shift. b. Tell the patient that she must bathe in the morning because that is the “normal” routine. c. Explain to the patient the importance of maintaining morning hygiene practices. d. Cancel hygiene for the day and attempt again in the morning. ANS: A
Each patient has individual desires and preferences about when to bathe, shave, and perform hair care. Knowing the patienNt’s ef. erC enc es assists the nurse in providing URpers SIona NGl pr TB OM individualized care for the patient. Hygiene care is never routine. Maintaining individual personal preferences is important unless new hygiene practices are indicated by an illness or condition. Cancelling hygiene care is not an option. Practices must be adapted to meet individual needs. No evidence demonstrates greater benefit with morning or evening hygiene care. DIF: Apply REF: 912 OBJ: Discuss common factors that influence personal hygiene practices. TOP: Implementation MSC: CPNRE: Professional, Ethical, and Legal Practice 5. Successful critical thinking requires synthesis of knowledge, experience, information gathered
from patients, critical thinking qualities, and intellectual and professional standards. Once the assessment has been done, what is it important for the nurse to understand? a. The nursing diagnoses never change. b. The patient’s condition never changes. c. Critical thinking is ongoing. d. Hygiene needs to become a simple routine. ANS: C
A patient’s condition is always changing, which necessitates ongoing critical thinking and changing of nursing diagnoses. Because hygienic care is so important for a patient to feel comfortable, refreshed, and renewed, the nurse should avoid making hygiene care a simple routine.
Canadian Fundamentals of Nursing 6th Edition Potter Test Bank
DIF: Understand REF: 905 OBJ: Discuss the role that the nursing process and critical thinking play in the provision of hygiene care. TOP: Assessment MSC: CPNRE: Professional, Ethical, and Legal Practice 6. When the nurse provides hygiene for an older patient, why is it important for the nurse to
closely assess the skin? a. As people age, skin becomes more resilient. b. As people age, sweat glands become more active. c. As people age, skin becomes less subject to bruising. d. As people age, less frequent bathing may be required. ANS: D
Daily bathing, as well as bathing with water that is too hot or soap that is harsh, causes the skin to become excessively dry. As the patient ages, the skin loses its resiliency and moisture, and sebaceous and sweat glands become less active. The epithelium thins, and elastic collagen fibres shrink, which makes the skin fragile and prone to bruising and breaking. DIF: Understand REF: 911 OBJ: Describe how hygiene care for the older patient may differ from that for the younger patient. TOP: Assessment MSC: CPNRE: Foundations of Practice 7. The nurse is bathing a patient and notices movement in the patient’s hair. What should the
nurse do? a. Ignore the movement and continue. b. Use gloves or a tongue blade to inspect the hair. c. Examine the hair without gloves to make picking lice easier. d. Shave the hair off of the p NaUtieRnSt’I s hNeG adT. B.COM ANS: B
In community health and home care settings, it is particularly important to inspect the hair for lice so that appropriate hygienic treatment can be provided. Suspicions cannot be ignored. If pediculosis capitis (head lice) is suspected, the nurse must protect himself or herself against self-infestations by hand hygiene and by using gloves or tongue blades to inspect the patient’s hair. Shaving hair off affected areas is the treatment for pediculosis pubis (crab lice) and is rarely used for head lice. DIF: Apply REF: 908 OBJ: Conduct a comprehensive assessment of a patient’s total hygiene needs. TOP: Implementation MSC: CPNRE: Foundations of Practice 8. When assessing a patient’s skin, what does the nurse need to know? a. Restricted movement can increase blood circulation. b. Paralyzed patients have normal sensory function. c. Loss of subcutaneous tissue may increase the rate of wound healing. d. Moisture on the skin can lead to skin maceration. ANS: D
Canadian Fundamentals of Nursing 6th Edition Potter Test Bank Moisture on the surface of the skin serves as a medium for bacterial growth and causes irritation, softens epidermal cells, and leads to skin maceration. When restricted from moving freely, dependent body parts are exposed to pressure that reduces circulation to affected tissues. Nurses should know which patients require help to turn and change positions. Patients with paralysis, circulatory insufficiency, or local nerve damage are unable to sense an injury to the skin. In patients with limited caloric and protein intake, the skin becomes thinner and less elastic, with loss of subcutaneous tissue, which results in impairment of or delay in wound healing. DIF: Remember REF: 908, Box 38-2 OBJ: Describe conditions that place the patient at risk for problems related to the hair, scalp, integument, feet, nails, eyes, ears, nose, mouth, and throat, and discuss their related interventions. TOP: Assessment MSC: CPNRE: Foundations of Practice 9. The nurse is caring for a patient who is immobile. Why is the nurse aware that the patient is at
risk for impaired skin integrity? a. Pressure reduces circulation to affected tissue. b. Patients with limited caloric intake develop thicker skin. c. Inadequate blood flow leads to decreased tissue ischemia. d. Local nerve damage leads to pain sensation. ANS: A
Body parts exposed to pressure have reduced circulation to affected tissue. In patients with limited caloric and protein intake, the skin becomes thinner and less elastic, with loss of subcutaneous tissue. Inadequate blood flow causes ischemia and breakdown. Patients with paralysis, circulatory insufficiency, or local nerve damage are unable to sense an injury to affected parts of the skin.
NURSINGTB.COM
DIF: Understand REF: 908, Box 38-2 OBJ: Describe conditions that place the patient at risk for problems related to the hair, scalp, integument, feet, nails, eyes, ears, nose, mouth, and throat, and discuss their related interventions. TOP: Assessment MSC: CPNRE: Foundations of Practice 10. The nurse is caring for a patient who has diabetes mellitus, circulatory insufficiency,
peripheral neuropathy, and urinary incontinence. What does the nurse know about patients with these conditions? a. They have decreased pain sensation and are at increased risk of skin impairment. b. They are at decreased risk of developing infection because of urinary pH level. c. They have decreased caloric intake, which results in accelerated wound healing. d. They have impaired venous return, which allows for greater circulation and less breakdown. ANS: A
Patients with paralysis, circulatory insufficiency, or local nerve damage are unable to sense an injury to affected parts of the skin. The presence of perspiration, urine, watery fecal material, and wound drainage on the skin results in breakdown and infection. In patients with limited caloric and protein intake, the skin becomes thinner and less elastic, with loss of subcutaneous tissue. This results in impairment or delay of wound healing. Impaired venous return decreases circulation to the extremities. Inadequate blood flow causes ischemia and breakdown.
Canadian Fundamentals of Nursing 6th Edition Potter Test Bank DIF: Understand REF: 908, Box 38-2 OBJ: Describe conditions that place the patient at risk for problems related to the hair, scalp, integument, feet, nails, eyes, ears, nose, mouth, and throat, and discuss their related interventions. TOP: Assessment MSC: CPNRE: Foundations of Practice 11. The nurse is caring for a patient who has undergone external fixation of a broken leg and has a
cast in place. To prevent skin impairment, what should the nurse do? a. Not allow the patient to turn in bed because that may lead to redislocation of the leg. b. Restrict the patient’s dietary intake to reduce the number of times on the bedpan. c. Assess all surfaces exposed to the cast for pressure areas. d. Keep the patient’s blood pressure low to prevent overperfusion of tissue. ANS: C
Assess all surfaces exposed to casts, cloth restraints, bandages and dressings, tubing, or orthopedic braces. An external device applied to or around the skin exerts pressure or friction on the skin, which leads to skin impairment. When restricted from moving, dependent body parts are exposed to pressure that reduces circulation to affected tissues. Nurses should know which patients require assistance to turn and change positions. In patients with limited caloric and protein intake, would healing becomes impaired or delayed. Keeping the blood pressure artificially low may decrease arterial blood supply, leading to ischemia and breakdown. DIF: Apply REF: 908, Box 38-2 OBJ: Describe conditions that place the patient at risk for problems related to the hair, scalp, integument, feet, nails, eyes, ears, nose, mouth, and throat, and discuss their related interventions. TOP: Implementation MSC: CPNRE: Foundations of Practice
s,UwRhic hN wG ould 12. Of the following interventionN SI TBb.eCthe OMmost important for preventing skin impairment in a mobile patient with local nerve damage? a. Turn the patient every 2 hours. b. Limit caloric and protein intake. c. Insert an indwelling urinary catheter. d. During a bath, assess for pain. ANS: D
While a patient is bathing, the nurse should assess the status of sensory nerve function by checking for pain, tactile sensation, and temperature sensation. When restricted from moving freely, dependent body parts are exposed to pressure that reduces circulation; however, a mobile patient is able to change positions. Limiting caloric and protein intake may cause wound healing to become impaired or delayed. The presence of perspiration, urine, watery fecal material, and wound drainage on the skin also causes wound healing to become impaired or delayed. However, a mobile patient can use bathroom facilities or a urinal. DIF: Analyze REF: 908, Box 38-2 OBJ: Describe conditions that place the patient at risk for problems related to the hair, scalp, integument, feet, nails, eyes, ears, nose, mouth, and throat, and discuss their related interventions. TOP: Assessment MSC: CPNRE: Foundations of Practice 13. Of the following disorders, which is caused by a virus? a. Corns. b. Plantar warts.
Canadian Fundamentals of Nursing 6th Edition Potter Test Bank c. Athlete’s foot. d. Callus. ANS: B
Plantar warts appear on the sole of the foot and are caused by the papillomavirus. Corns are caused by friction and pressure from ill-fitting or loose shoes. Athlete’s foot (tinea pedis) is a fungal infection. A callus is caused by local friction or pressure. DIF: Remember REF: 909, Table 38-3 OBJ: Describe conditions that place the patient at risk for problems related to the hair, scalp, integument, feet, nails, eyes, ears, nose, mouth, and throat, and discuss their related interventions. TOP: Assessment MSC: CPNRE: Foundations of Practice 14. The nurse is caring for a patient who is complaining of severe foot pain caused by corns. The
patient states that she has been using oval corn pads to self-treat the corns, but they seem to be getting worse. What does the nurse explain? a. Corn pads are an adequate treatment and should be continued. b. The patient should avoid soaking her feet before using a pumice stone. c. Tighter shoes would help to compress the corns and make them smaller. d. Depending on severity, surgery may be needed to remove the corns. ANS: D
Depending on severity of pain and the size of the corn, surgical removal may be necessary. Oval corn pads should be avoided because they increase pressure on the toes and reduce circulation. Warm water soaks help soften corns before gentle rubbing with a callus file or pumice stone. Wider and softer shoes, especially shoes with a wider toe box, are helpful. DIF: Understand REF: 90N 9, TR ableI38-G 3 M U the S patient N TB.C OBJ: Describe conditions that place at riskOfor problems related to the hair, scalp, integument, feet, nails, eyes, ears, nose, mouth, and throat, and discuss their related interventions. TOP: Planning MSC: CPNRE: Foundations of Practice 15. The patient receives a diagnosis of athlete’s foot (tinea pedis). The patient says that she is
relieved because it is “only athlete’s foot” and it can be treated easily. What does the nurse explain about athlete’s foot? a. It is generally isolated to the feet and never recurs. b. It is contagious and frequently recurs. c. It is caused by the papillomavirus. d. It is treated with salicylic acid or electrodesiccation. ANS: B
Athlete’s foot spreads to other body parts, especially the hands. It is contagious and frequently recurs. It is caused by a fungus, not the papillomavirus. It is not treated with salicylic acid or electrodesiccation; those are treatments for plantar warts. DIF: Understand REF: 909, Table 38-3 OBJ: Describe conditions that place the patient at risk for problems related to the hair, scalp, integument, feet, nails, eyes, ears, nose, mouth, and throat, and discuss their related interventions. TOP: Planning MSC: CPNRE: Foundations of Practice 16. When assessing a patient’s feet, the nurse notices that the toenails are thick and separated
from the nail bed. What causes this condition?
Canadian Fundamentals of Nursing 6th Edition Potter Test Bank a. b. c. d.
Fungi. Nail polish. Friction. Nail polish remover.
ANS: A
Inflammatory lesions and fungus of the nail bed cause nails to become thickened and horny and to separate from the nail bed. If a female patient is affected, the nurse should ask whether she frequently polishes her nails and uses polish remover because chemicals in these products cause excessive nail dryness, but they do not actually cause thickening and separation of nails from the nail bed. Friction and pressure from ill-fitting or loose shoes causes keratosis (corns). It is seen mainly on or between toes, over bony prominences. DIF: Understand REF: 911 OBJ: Describe conditions that place the patient at risk for problems related to the hair, scalp, integument, feet, nails, eyes, ears, nose, mouth, and throat, and discuss their related interventions. TOP: Assessment MSC: CPNRE: Foundations of Practice 17. The nurse is providing education about proper foot care to a patient who has diabetes mellitus.
Why is this important? a. Plantar warts can develop from foot fungi. b. Poor foot care leads to neuropathy. c. A strong dorsalis pedis pulse indicates poor blood flow. d. Foot ulcers are the most common precursor to amputation. ANS: D
Foot ulceration is the most common single indication for lower extremity amputations among persons with diabetes. PlantaN r wa caus dC byOtM he papillomavirus, not a fungus. Palpable URrtsSIareNG TBe. dorsalis pedis and posterior tibial pulses indicate that adequate blood flow is reaching peripheral tissues. Neuropathy is a degeneration of the peripheral nerves usually resulting from poor control of blood glucose levels; it is not a direct result of poor foot care. DIF: Understand REF: 927 OBJ: Understand the importance of foot care for the diabetic patient. TOP: Assessment MSC: CPNRE: Foundations of Practice 18. The nurse is providing oral care to an unconscious patient and notes that the patient has
extremely bad breath. What is the term for “bad breath”? a. Alopecia. b. Halitosis. c. Dental caries. d. Neuropathy. ANS: B
Halitosis is the term for “bad breath.” Alopecia is hair loss. Dental caries is tooth decay. Neuropathy is a degeneration of peripheral nerves that leads to loss of sensation in the extremities. DIF: Remember REF: 908 | 932 OBJ: Describe conditions that place the patient at risk for problems related to the hair, scalp, integument, feet, nails, eyes, ears, nose, mouth, and throat, and discuss their related interventions. TOP: Assessment MSC: CPNRE: Foundations of Practice
Canadian Fundamentals of Nursing 6th Edition Potter Test Bank
19. The nurse is caring for an unresponsive patient who has a nasogastric tube in place for
continuous tube feedings. Why does the nurse assess the patient’s oral hygiene? a. Oral hygiene helps prevent gingivitis. b. Oral hygiene may cause glossitis. c. Oral hygiene may lead to halitosis. d. Oral hygiene causes tongue coating. ANS: A
Early identification of poor oral hygiene practices and common oral problems reduces the risk for gum disease and dental caries. Patients frequently develop common oral problems as a result of inadequate oral care or as a consequence of disease (e.g., oral malignancy) or as a side effect of treatments such as radiation and chemotherapy. These problems include recession of gum tissue, inflammation of gums (gingivitis), coating of the tongue, glossitis (inflammation of the tongue), discoloration of teeth (particularly along gum margins), dental caries, loss of teeth, and halitosis (foul-smelling breath). DIF: Understand REF: 908 OBJ: Describe conditions that place the patient at risk for problems related to the hair, scalp, integument, feet, nails, eyes, ears, nose, mouth, and throat, and discuss their related interventions. TOP: Assessment MSC: CPNRE: Foundations of Practice 20. The patient is being treated for cancer with weekly radiation and chemotherapy treatments.
The nurse is aware that the patient’s oral mucosa needs to be assessed because radiation therapy and chemotherapy can have what effects? a. Increase in saliva production. b. Decrease in the risk of oral inflammation. c. Decrease in drying of oraN lm ucS osIaN . GTB.COM UR d. Increase in likelihood of oral problems. ANS: D
Patients frequently develop common oral problems as a result of inadequate oral care, as a consequence of disease (e.g., oral malignancy), or as a side effect of treatments such as radiation and chemotherapy. DIF: Understand REF: 908 OBJ: Describe conditions that place the patient at risk for problems related to the hair, scalp, integument, feet, nails, eyes, ears, nose, mouth, and throat, and discuss their related interventions. TOP: Assessment MSC: CPNRE: Foundations of Practice 21. In providing oral care to an unconscious patient, what is an important action by the nurse? a. Moistening the patient’s mouth with lemon-glycerine sponges. b. Holding the patient’s mouth open with his or her fingers. c. Rinsing the mouth and immediately suctioning the oral cavity. d. Using foam swabs to help remove plaque. ANS: C
Canadian Fundamentals of Nursing 6th Edition Potter Test Bank When providing oral hygiene to an unconscious patient, the nurse needs to protect him or her from choking and aspiration. Two nurses should provide care: One nurse does the actual cleaning, and the other removes secretions with suction equipment. The nurse can delegate some tasks to unregulated care providers. Some agencies use equipment that combines a mouth swab with the suction device. This device can be used safely by one nurse to provide oral care. Commercially made foam swabs are ineffective in removing plaque. Lemon-glycerine sponges should not be used because they dry mucous membranes and erode tooth enamel. While cleansing the oral cavity, the nurse should use a small oral airway or a padded tongue blade to hold the patient’s mouth open. The nurse should never use his or her own fingers to hold the patient’s mouth open: A human bite contains multiple pathogenic microorganisms. DIF: Apply REF: 931 OBJ: Describe conditions that place the patient at risk for problems related to the hair, scalp, integument, feet, nails, eyes, ears, nose, mouth, and throat, and discuss their related interventions. TOP: Implementation MSC: CPNRE: Foundations of Practice 22. The nurse is teaching the patient about flossing and oral hygiene. Which of the following is
the nurse’s instruction? a. Flossing needs to be done at least three times a day. b. To prevent bleeding, the patient should use waxed floss. c. Flossing removes plaque and bacteria from the teeth. d. Applying toothpaste to the teeth before flossing is harmful. ANS: C
Dental flossing removes plaque and bacteria between teeth. To prevent bleeding, the patient should use unwaxed floss. Flossing once a day is sufficient. If toothpaste is applied to the INeGinTdirect B.COcontact M with tooth surfaces, aiding in cavity teeth before flossing, fluorideNw UillRScom prevention. DIF: Apply REF: 931 OBJ: Describe conditions that place the patient at risk for problems related to the hair, scalp, integument, feet, nails, eyes, ears, nose, mouth, and throat, and discuss their related interventions. TOP: Implementation MSC: CPNRE: Foundations of Practice 23. The nurse is caring for a patient who has head lice (pediculosis capitis). What should the nurse
know about treating this condition? a. Products containing lindane are most effective. b. Head lice may spread to furniture and other people. c. Treatment must be repeated in 7 to 10 days. d. Manual removal is not a realistic option as treatment. ANS: B
Head lice are difficult to remove and spread to furniture and other people if not treated. Products containing lindane should not be used because the ingredient is toxic and is known to cause adverse reactions. Treatments need to be repeated 12 to 24 hours after the initial treatment. Manual removal is the best option when treatment has failed. DIF: Understand REF: 910, Table 38-4 OBJ: Describe conditions that place the patient at risk for problems related to the hair, scalp, integument, feet, nails, eyes, ears, nose, mouth, and throat, and discuss their related interventions.
Canadian Fundamentals of Nursing 6th Edition Potter Test Bank TOP: Assessment
MSC: CPNRE: Foundations of Practice
24. Scaling of the scalp accompanied by itching is known as which of the following? a. Dandruff. b. Pediculosis. c. Alopecia. d. Ticks. ANS: A
Dandruff is scaling of the scalp that is accompanied by itching. Pediculosis (lice) is an infestation of tiny, greyish-white parasitic insects in mammals. Alopecia is hair loss or balding. Ticks are small, grey-brown parasites that burrow into the skin and suck blood. DIF: Remember REF: 910, Table 38-4 OBJ: Describe conditions that place the patient at risk for problems related to the hair, scalp, integument, feet, nails, eyes, ears, nose, mouth, and throat, and discuss their related interventions. TOP: Assessment MSC: CPNRE: Foundations of Practice 25. In finding pediculosis capitis (head lice) in a patient, what would the nurse expect to observe? a. Greyish-white parasites with red legs. b. Pustules or bites behind ears and at the hairline. c. Balding patches in periphery of the hairline. d. Brittle and broken hair. ANS: B
Head lice are on the scalp, attached to hair stands. Bites or pustules may be observed behind the ears and at the hairline. Greyish-white parasites with red legs are pediculosis pubis (crab lice), not head lice, and are foNundRin I pubG ic hB ai. r. C AloM pecia (hair loss) is found in all ethnicities U S N T and is characterized by brittle and broken hair andObalding patchiness in the periphery of the hairline. DIF: Remember REF: 910, Table 38-4 OBJ: Describe conditions that place the patient at risk for problems related to the hair, scalp, integument, feet, nails, eyes, ears, nose, mouth, and throat, and discuss their related interventions. TOP: Assessment MSC: CPNRE: Foundations of Practice 26. The nurse is caring for a patient who has multiple ticks on her legs and body. To rid the
patient of ticks, what should the nurse do? a. Burn the ticks in an ashtray once removed. b. Use blunt tweezers and pull upward with steady pressure. c. Allow the ticks to drop off by themselves. d. Use products containing lindane to kill the ticks. ANS: B
Using blunt tweezers, the nurse grasps the tick as close to its head as possible and pulls upward with even, steady pressure. The nurse holds the tick until it pulls out, usually for about 3 to 4 minutes. The tick should be saved in a plastic bag and put it in the freezer if necessary in order to identify the type of tick. Because ticks transmit several diseases to people, they must be removed. Allowing them to drop off by themselves is not an option. Lindane is an ingredient that was traditionally used in treatment for pediculosis capitis (head lice); it should no longer be used because the ingredient is toxic and is known to cause adverse reactions.
Canadian Fundamentals of Nursing 6th Edition Potter Test Bank DIF: Apply REF: 910, Table 38-4 OBJ: Describe conditions that place the patient at risk for problems related to the hair, scalp, integument, feet, nails, eyes, ears, nose, mouth, and throat, and discuss their related interventions. TOP: Implementation MSC: CPNRE: Foundations of Practice 27. The patient received a diagnosis of pediculosis capitis (head lice), was treated upon
admission, and was re-treated 24 hours later, and yet the patient is still has an infestation. What should be the nurse’s next action? a. Re-treat the patient with a medicated shampoo for eliminating lice. b. Use a product containing lindane to get rid of the lice. c. Manually remove the lice using a fine-toothed comb. d. Have the patient bathe or shower thoroughly. ANS: C
Manual removal is the best option when treatment has failed. Re-treating with a medicated shampoo may lead to adverse reactions and should not be done without consultation with the care provider. Products containing lindane should not be used because the ingredient is toxic and is known to cause adverse reactions. Although bathing or showering is a good idea, this is usually considered a treatment for pediculosis corporis (body lice), not pediculosis capitis (head lice). DIF: Apply REF: 910, Table 38-4 OBJ: Describe conditions that place the patient at risk for problems related to the hair, scalp, integument, feet, nails, eyes, ears, nose, mouth, and throat, and discuss their related interventions. TOP: Implementation MSC: CPNRE: Foundations of Practice 28. The nurse is caring for an older patient with Alzheimer’s disease who is ambulatory but
requires total assistance withNhis esT oB f d.aC ilyOlM iving (ADLs). The nurse notices that his URacti SIviti NG skin is dry and wrinkled. What should the nurse do? a. Make sure that the patient is receiving daily baths. b. Reduce the number of baths per week if possible. c. Be aware that sweat glands become more active with aging. d. Be sure that the patient is using soap with his bath. ANS: B
Decreasing the number of baths per week may help prevent further drying of the skin. As people age, the skin loses its resiliency and moisture, and sebaceous and sweat glands become less active. Daily bathing, as well as bathing with water that is too hot or soap that is harsh, causes the skin to become excessively dry. DIF: Apply REF: 918, Box 38-9 OBJ: Describe how hygiene care for the older patient may differ from that for the younger patient. TOP: Implementation MSC: CPNRE: Foundations of Practice 29. A self-sufficient bedridden patient unable to reach all body parts needs which type of bath? a. Complete bed bath. b. Bag bath. c. Sponge bath. d. Partial bed bath. ANS: D
Canadian Fundamentals of Nursing 6th Edition Potter Test Bank A partial bath consists of washing body parts that the patient cannot reach, including the back, and providing a back rub. Dependent patients in need of partial hygiene or self-sufficient bedridden patients who are unable to reach all body parts receive a partial bed bath. Complete bed baths are administered to totally dependent patients in bed. The bag bath contains several soft, nonwoven cotton cloths that are premoistened in a solution of no-rinse surfactant cleanser and emollient. It can be used for any patient. The sponge bath involves bathing from a bath basin or a sink with the patient sitting in a chair. DIF: Remember REF: 918 OBJ: Discuss the different approaches used in maintaining a patient’s comfort and safety during hygiene care. TOP: Assessment MSC: CPNRE: Foundations of Practice 30. The nurse is preparing to provide a complete bed bath to an unconscious patient. The nurse
decides to use a bag bath. She does this for which of the following reasons? a. Washbasins can harbour Gram-negative organisms. b. Bag baths involve the use of soaps that enhance cleansing. c. Bag baths do not contain emollients. d. Bag baths increase skin flaking and scaling. ANS: A
When washbasins are not cleaned and dried completely after use, they are more likely to be contaminated by Gram-negative organisms. Successive uses of the washbasin cause the patient’s skin to harbour more Gram-negative organisms. Bag baths do not contain soap. Instead, they contain a no-rinse surfactant, a humectant to trap moisture, and an emollient that significantly reduces overall skin dryness, especially skin flaking and scaling. DIF: Understand REF: 926 OBJ: Discuss the different appN roU acRhS esIuN seG d iT nB m. aiC ntO ainMing a patient’s comfort and safety during hygiene care. TOP: Assessment MSC: CPNRE: Foundations of Practice 31. The female nurse is caring for a male patient who is uncircumcised and not ambulatory,
although he has full function of arms and hands. The nurse is providing the patient with a partial bed bath. What statement is true about perineal care for this patient? a. It is not necessary because he is not circumcised. b. It should be postponed because it may cause him embarrassment. c. It should be done by the patient. d. It should be done by the nurse. ANS: C
Patients most in need of perineal care are those at greatest risk for acquiring an infection, such as uncircumcised boys and men. If a patient is able to perform perineal self-care, this independence should be encouraged. Embarrassment should not cause the nurse to overlook the patient’s hygiene needs. The nurse should provide this care only if the patient is unable to do so. DIF: Apply REF: 926 OBJ: Discuss the different approaches used in maintaining a patient’s comfort and safety during hygiene care. TOP: Implementation MSC: CPNRE: Foundations of Practice 32. After the patient’s bath, what should the nurse do?
Canadian Fundamentals of Nursing 6th Edition Potter Test Bank a. b. c. d.
Not offer a backrub because it is not therapeutic. Routinely give backrubs of 2 minutes or less. Assume that all patients need backrubs after their bath. Not offer a backrub for 48 hours after the patient undergoes coronary artery bypass surgery.
ANS: D
The nurse should consult the medical record for any contraindications to a massage (e.g., fractured ribs, burns, heart surgery). A backrub of 3 minutes’ duration actually enhances patient comfort and relaxation and thus is very therapeutic. It is important to ask whether a patient would like a backrub because some individuals dislike physical contact. DIF: Apply REF: 927, Box 38-10 OBJ: Discuss the different approaches used in maintaining a patient’s comfort and safety during hygiene care. TOP: Implementation MSC: CPNRE: Foundations of Practice 33. The nurse is providing a complete bed bath to a patient, using a commercial bath cleansing
pack (bag bath). What should the nurse do? a. Use one towel for the entire bath. b. Dry the skin with a towel. c. Allow the skin to air dry. d. Not use a bath blanket or towel. ANS: C
The nurse should allow the skin to air dry for 30 seconds. Drying the skin with a towel removes the emollient that is left behind after the water/cleanser solution evaporates. It is permissible to lightly cover tN heUpR atie SIntNwGithTBa .baCthObMlanket or towel to prevent chilling. The cleansing pack contains 8 to 10 premoistened towels for cleansing. A single towel is used for each general body part cleansed. DIF: Apply REF: 926 OBJ: Successfully perform hygiene for care of the integument; perineum; feet, hands, and nails; mouth; and eyes, ears, and nose. TOP: Implementation MSC: CPNRE: Foundations of Practice 34. In providing perineal care to a female patient, how should the nurse wash? a. Upward from rectum to pubic area. b. From back to front. c. From pubic area to rectum. d. In a circular motion. ANS: C
Cleansing from pubic area to rectum (front to back) reduces the transfer of microorganisms to the urinary meatus and decreases the risk of urinary tract infection. Cleansing from rectum to pubic area or back to front increases the risk of urinary tract infection. Circular motions are used in male perineal care. DIF: Apply REF: 923 OBJ: Successfully perform hygiene for care of the integument; perineum; feet, hands, and nails; mouth; and eyes, ears, and nose. TOP: Implementation MSC: CPNRE: Foundations of Practice
Canadian Fundamentals of Nursing 6th Edition Potter Test Bank
35. The nurse is providing perineal care to an uncircumcised male patient. When providing such
care, how should the nurse treat the foreskin? a. Leave the foreskin alone because there is little chance of infection. b. Retract the foreskin for cleansing and allow it to return on its own. c. Retract the foreskin and keep retracted. d. Retract the foreskin and return it to its natural position when done. ANS: D
Return the foreskin to its natural position. Keeping the foreskin retracted leads to tightening of the foreskin around the shaft of the penis, causing local edema and discomfort. The foreskin may not return to its natural position on its own. Patients at greatest risk for infection are uncircumcised boys and men. DIF: Apply REF: 923 OBJ: Successfully perform hygiene for care of the integument; perineum; feet, hands, and nails; mouth; and eyes, ears, and nose. TOP: Implementation MSC: CPNRE: Foundations of Practice 36. Patients with diabetes mellitus need special foot care to prevent the development of ulcers.
Knowing this, what is the nurse’s action? a. Trimming the patient’s toenails daily. b. Having the patient soak his or her feet twice a day. c. Requesting a consult with a nail care specialist. d. Assessing the brachial artery. ANS: C
Patients with peripheral vascN ularRdisI or B abCetesMmellitus often require nail care from a Sease NGSome Tdi. O allow cutting of nails with a specialist to reduce the risk of U infection. agencies provider’s order; however, most do not. Patients with diabetes do not soak hands and feet; soaking increases the risk of infection because of maceration of the skin. When assessing the patient’s feet, the nurse palpates the dorsalis pedis of the foot, not the brachial artery. DIF: Apply REF: 926 | 927 OBJ: Successfully perform hygiene for care of the integument; perineum; feet, hands, and nails; mouth; and eyes, ears, and nose. TOP: Implementation MSC: CPNRE: Foundations of Practice 37. The uncooperative patient is resisting attempts by the nurse to provide oral hygiene. To
provide the needed care, what may the nurse do? a. Use a padded tongue blade. b. Hold the patient’s mouth open with her fingers. c. Position the patient on his back. d. Use undiluted hydrogen peroxide as a cleaner. ANS: A
Canadian Fundamentals of Nursing 6th Edition Potter Test Bank If the patient is unconscious or uncooperative, or is having difficulty keeping the mouth open, a padded tongue blade can help. It is inserted over the tongue to keep the teeth apart when the patient is relaxed. The nurse must not use force. The nurse must never place fingers into the mouth of an unconscious or debilitated patient; the normal response of the patient is to bite down. The patient should be positioned on his side, or the head turned, to allow for drainage. Placing the patient on his back could lead to aspiration. Hydrogen peroxide and sodium bicarbonate effectively remove debris but, if not diluted carefully, may cause superficial burns. DIF: Apply REF: 934 OBJ: Successfully perform hygiene for care of the integument; perineum; feet, hands, and nails; mouth; and eyes, ears, and nose. TOP: Implementation MSC: CPNRE: Foundations of Practice 38. How is basic eye care provided? a. The nurse cleanses the eye with soap and water. b. The nurse applies pressure directly to the eyeball. c. The nurse cleanses from inner canthus to outer canthus. d. The nurse provides less frequent care to unconscious patients. ANS: C
When cleansing the patient’s eyes, the nurse should obtain a clean washcloth and cleanse from inner canthus to outer canthus. A different section of the washcloth should be used for each eye. Cleansing simply involves washing with a clean washcloth moistened in water. Soap causes burning and irritation. Direct pressure must never be applied over the eyeball because this causes serious injury. Unconscious patients often require more frequent eye care than do conscious patients.
NURSINGTB.COM
DIF: Apply REF: 939 OBJ: Successfully perform hygiene for care of the integument; perineum; feet, hands, and nails; mouth; and eyes, ears, and nose. TOP: Implementation MSC: CPNRE: Foundations of Practice 39. The nurse is teaching a patient about contact lens care. The patient has plastic lenses, so what
does the nurse instruct the patient to do? a. Use tap water to clean lenses. b. Keep the lenses is a cool, dry place when not being used. c. Reuse storage solution for up to a week. d. Wash and rinse lens storage case daily. ANS: D
The lens storage case should be thoroughly washed and rinsed on a daily basis. It should be cleaned periodically with soap or liquid detergent, rinsed thoroughly with warm water, and allowed to air dry. Tap water should not be used to clean lenses because tap water is not sterile and can introduce microorganisms. Lenses should be kept moist or wet when not worn. Fresh solution is used daily when lenses are stored and disinfected. DIF: Apply REF: 939, Box 38-14 OBJ: Successfully perform hygiene for care of the integument; perineum; feet, hands, and nails; mouth; and eyes, ears, and nose. TOP: Implementation MSC: CPNRE: Foundations of Practice
Canadian Fundamentals of Nursing 6th Edition Potter Test Bank 40. The patient complains to the nurse about a perceived decrease in hearing. When the nurse
examines the patient’s ear, she notices a large amount of cerumen (ear wax) buildup at the entrance to the ear canal. What should the nurse do? a. Apply gentle, downward retraction of the ear canal. b. Tell the patient to use a bobby pin to extract earwax. c. Teach the patient how to use cotton-tipped applicators. d. Instill hot water into the ear canal to melt the wax. ANS: A
When cerumen is visible, gentle, downward retraction at the entrance to the ear canal causes the wax to loosen and slip out. Instruct the patient never to use sharp objects such as bobby pins or paper clips to remove earwax because they can traumatize the ear canal and rupture the tympanic membrane. Cotton-tipped applicators should be avoided as well because they cause earwax to become impacted within the canal. Instilling cold or hot water in to the ear causes nausea, vomiting, or both. DIF: Apply REF: 940 OBJ: Successfully perform hygiene for care of the integument; perineum; feet, hands, and nails; mouth; and eyes, ears, and nose. TOP: Implementation MSC: CPNRE: Foundations of Practice 41. The patient is being fitted with a hearing aid. In teaching the patient how to care for the
hearing aid, the nurse instructs the patient to do which of the following? a. Wear the hearing aid 24 hours per day except when sleeping. b. Change the battery every day or as needed. c. Avoid the use of hairspray, but aerosol perfumes are allowed. d. Adjust the volume for a talking distance of 1 m. ANS: D
NURSINGTB.COM
The volume is adjusted to a comfortable level for talking at a distance of 1 m. Initially, the patient should wear a hearing aid for 15 to 20 minutes; then wear time is gradually increased to 10 to 12 hours per day. Batteries last 1 week with daily wear of 10 to 12 hours. People who wear hearing aids should avoid the use of hairspray and perfume; residue from the spray can cause the hearing aid to become oily and greasy. DIF: Apply REF: 941, Box 38-15 OBJ: Successfully perform hygiene for care of the integument; perineum; feet, hands, and nails; mouth; and eyes, ears, and nose. TOP: Implementation MSC: CPNRE: Foundations of Practice 42. The patient is complaining of an inability to clear his nasal passages. How should the nurse
instruct the patient? a. To blow his nose forcefully to clear the passage. b. To insert a cotton-tipped applicator as far as possible. c. To apply gentle suction with a pediatric bulb suction device. d. To use a dry washcloth to absorb secretions. ANS: C
Canadian Fundamentals of Nursing 6th Edition Potter Test Bank Excessive nasal secretions can be removed using gentle suctioning. However, patients usually remove secretions from the nose by gentle blowing into a soft tissue. The patient should be cautioned against harsh blowing, which creates pressure capable of injuring the eardrum, the nasal mucosa, and even sensitive eye structures. If the patient is unable to remove nasal secretions, the nurse can assist by using a wet washcloth or a cotton-tipped applicator moistened in water or saline. The applicator must never be inserted beyond the length of the cotton tip. DIF: Apply REF: 934 OBJ: Successfully perform hygiene for care of the integument; perineum; feet, hands, and nails; mouth; and eyes, ears, and nose. TOP: Implementation MSC: CPNRE: Foundations of Practice 43. Of the following hearing aids, which interferes the most with wearing eyeglasses and using a
phone? a. In-the-canal hearing aid. b. In-the-ear hearing aid. c. Behind-the-ear hearing aid. d. They are all equally useful. ANS: C
The behind-the-ear aid hooks around and behind the ear and is connected by a short, clear, hollow plastic tube to an ear mould inserted into the external auditory canal. It is useful for patients with rapidly progressive hearing loss or manual dexterity difficulties, but it is more visible and interferes with wearing eyeglasses and using a phone. An in-the-canal aid is the newest, smallest, and least visible type of hearing aid; it fits entirely in the ear canal. It does not interfere with wearing eyeglasses or using the telephone, but it does not accommodate N R I G erity B.C M progressive hearing loss, and mUanuSal dNextT is O needed to operate. An in-the-ear aid does not interfere with wearing of eyeglasses or phone usage, but it is more noticeable than the in-the-canal aid and is not useful for persons with skin problems in the ear canal. DIF: Evaluate REF: 940 OBJ: Successfully perform hygiene for care of the integument; perineum; feet, hands, and nails; mouth; and eyes, ears, and nose. TOP: Assessment MSC: CPNRE: Foundations of Practice 44. The use of critical thinking attitudes is necessary to design a plan of care to meet the patient’s
hygiene needs. Which of the following is considered to be a critical thinking attribute? a. Curiosity. b. Communication principles. c. Prior experience. d. Knowledge of cultural variations. ANS: A
Use of critical thinking attitudes, such as curiosity and humility, is necessary to design a plan of care to meet the patient’s hygiene needs. Communication principles and knowledge of cultural variations in hygiene are considered knowledge elements, and prior experience is part of the experience elements of the critical thinking model for hygiene assessment. DIF: Remember REF: 906 OBJ: Discuss the role that the nursing process and critical thinking play in the provision of hygiene
Canadian Fundamentals of Nursing 6th Edition Potter Test Bank care. TOP:
Implementation
MSC: CPNRE: Professional, Ethical, and Legal Practice
45. Of the following patients, which one does the nurse expect to perform perineal care
independently? a. A patient with urinary and fecal incontinence. b. A circumcised male patient who is ambulatory. c. A patient with rectal and perineal surgical dressings. d. A patient with an indwelling catheter. ANS: B
Patients at greatest risk for skin breakdown in the perineal area are those with urinary or fecal incontinence, rectal and perineal surgical dressings, or indwelling urinary catheters, along with the morbidly obese. Circumcised boys and men are not at high risk for acquiring infection, and ambulatory patients can usually provide self-perineal care. DIF: Evaluate REF: 926 OBJ: Successfully perform hygiene for care of the integument; perineum; feet, hands, and nails; mouth; and eyes, ears, and nose. TOP: Assessment MSC: CPNRE: Foundations of Practice
NURSINGTB.COM
Canadian Fundamentals of Nursing 6th Edition Potter Test Bank
Chapter 39: Cardiopulmonary Functioning and Oxygenation Potter et al: Canadian Fundamentals of Nursing, 6th Edition MULTIPLE CHOICE 1. What is the structure that is responsible for returning oxygenated blood to the heart? a. Pulmonary artery. b. Pulmonary vein. c. Superior vena cava. d. Inferior vena cava. ANS: B
The pulmonary vein carries oxygenated blood to the heart. The pulmonary artery carries deoxygenated blood from the heart to the lungs. Both venae cavae return blood to the right atrium of the heart. DIF: Remember REF: 958 OBJ: Describe the structure and function of the cardiopulmonary system. TOP: Planning MSC: CPNRE: Foundations of Practice 2. Where are chemical receptors that stimulate inspiration? a. Brain. b. Lungs. c. Aorta. d. Heart. ANS: C
NURSINGTB.COM
Chemical receptors in the aorta send signals to begin the inspiration process. The brain, lungs, and heart all are affected by this chemical reaction. DIF: Remember REF: 957 OBJ: Describe the neural and chemical regulation of respiration. TOP: Planning MSC: CPNRE: Foundations of Practice 3. The nurse knows that the primary function of the alveoli is which of the following? a. Carry out gas exchange. b. Store oxygen. c. Regulate tidal volume. d. Produce hemoglobin. ANS: A
The alveolus is a capillary membrane that allows gas exchange of oxygen and carbon dioxide during respiration. The alveoli do not store oxygen, regulate tidal volume, or produce hemoglobin. DIF: Remember REF: 956 OBJ: Describe the structure and function of the cardiopulmonary system. TOP: Knowledge MSC: CPNRE: Foundations of Practice 4. What will anemia result in? a. Hypoxemia.
Canadian Fundamentals of Nursing 6th Edition Potter Test Bank b. Impaired ventilation. c. Hypovolemia. d. Decreased lung compliance. ANS: A
Patients who are anemic do not have normal oxygen-carrying capacity. As a result, oxygen cannot properly perfuse the tissues, which results in hypoxemia. Ventilation is impaired when oxygen/carbon dioxide exchange occurs at the alveolar level. Hypovolemia is a decrease in circulating blood volume. Lung compliance refers to the elasticity of the lung tissue. DIF: Understand REF: 959 OBJ: Identify and describe clinical outcomes as a result of disturbances in conduction, altered cardiac output, impaired valvular function, myocardial ischemia, and impaired tissue perfusion. TOP: Assessment MSC: CPNRE: Foundations of Practice 5. The process of exchanging gases through the alveolar capillary membrane is known as which
of the following? a. Disassociation. b. Diffusion. c. Perfusion. d. Ventilation. ANS: B
Diffusion is the process of gases exchanging across the alveoli and capillaries of body tissues. Disassociation is not related to oxygenation. Perfusion is the ability of the cardiovascular system to carry oxygenated blood to tissues and return deoxygenated blood to the heart. Ventilation is the process of moving gases into and out of the lungs.
N R I G B.C M
U S N T O DIF: Remember REF: 958 OBJ: Identify the physiological processes involved in ventilation, perfusion, and exchange of respiratory gases. TOP: Assessment MSC: CPNRE: Foundations of Practice 6. In caring for a patient who was in a motor vehicle accident that resulted in trauma to C4, what
would the nurse expect to find? a. Decreased tidal volumes. b. Increased perfusion. c. Increased use of accessory muscles. d. Decreased hemoglobin. ANS: A
A C4 injury results in damage to the phrenic nerve and causes a decrease in inspiratory lung expansion. Accessory muscles are also damaged by a C4 injury. The patient may exhibit decreased perfusion and increased hemoglobin to compensate for hypoxemia. DIF: Understand REF: 970, Table 39-3 OBJ: Describe the neural and chemical regulation of respiration. TOP: Assessment MSC: CPNRE: Foundations of Practice 7. In which condition would the nurse expect to see increased ventilations? a. Increased oxygen saturation. b. Decreased carbon dioxide levels. c. Decreased pH.
Canadian Fundamentals of Nursing 6th Edition Potter Test Bank d. Increased hemoglobin levels. ANS: C
Retained CO2 creates H+ byproducts that lower pH. This sends a chemical signal to increase respiratory rate and would result in increased ventilation. All other options would cause the ventilation rate to normalize or decrease in order to increase carbon dioxide retention or as the result of delivery of higher levels of oxygen to tissues. DIF: Understand REF: 970, Table 39-3 OBJ: Describe the neural and chemical regulation of respiration. TOP: Assessment MSC: CPNRE: Foundations of Practice 8. Why does the nurse recommend that a patient install a carbon monoxide detector in the home? a. It is required by law. b. Carbon monoxide tightly bonds to hemoglobin, causing hypoxia. c. Carbon monoxide signals the cerebral cortex to cease ventilations. d. Carbon monoxide combines with oxygen in the body and produces a deadly toxin. ANS: B
Carbon monoxide has a high affinity for hemoglobin; it prevents oxygen from bonding to hemoglobin, and therefore oxygen cannot be transported to tissues. A carbon monoxide detector is not required by law; carbon monoxide does not signal the cerebral cortex to cease ventilations, and it combines not with oxygen but with hemoglobin to produce a toxin. DIF: Remember REF: 968 OBJ: Identify nursing care interventions in the primary care, acute care, and restorative and continuing care settings that promote oxygenation. TOP: Planning MSC: CPNRE: Foundations ofNPraR cticeI G B.C M
U S N T
O
9. While performing an assessment, the nurse hears crackles in the patient’s lung fields. The
nurse also learns that the patient is sleeping on three pillows. What do these symptoms probably indicate? a. Left-sided heart failure. b. Right-sided heart failure. c. Atrial fibrillation. d. Myocardial ischemia. ANS: A
Left-sided heart failure results in pulmonary congestion, the signs and symptoms of which include shortness of breath, crackles, and discomfort in the supine position. Right-sided heart failure is systemic and results in peripheral edema and hepatojugular distension. Atrial fibrillation is an irregular heart rate. Myocardial ischemia most often results in chest pain, along with shortness of breath, nausea, and fatigue. DIF: Understand REF: 961 OBJ: Identify the physiological processes involved in ventilation, perfusion, and exchange of respiratory gases. TOP: Assessment MSC: CPNRE: Foundations of Practice 10. The nurse knows that a myocardial infarction is an occlusion of what blood vessel? a. Pulmonary artery. b. Ascending aorta. c. Coronary artery.
Canadian Fundamentals of Nursing 6th Edition Potter Test Bank d. Carotid artery. ANS: C
A myocardial infarction is the lack of blood flow as a result of obstruction to the coronary artery, which supplies the heart with blood. The ascending aorta is a vessel that leads from the heart to perfuse the brain. The pulmonary artery supplies blood to the lungs. The carotid artery supplies blood to the brain. DIF: Remember REF: 963 OBJ: Identify the physiological processes of cardiac output, myocardial blood flow, and coronary artery circulation. TOP: Assessment MSC: CPNRE: Foundations of Practice 11. Myocardial blood flow is unidirectional; the nurse knows that the correct pathway is which of
the following? a. Right atrium, right ventricle, left ventricle, left atrium. b. Right atrium, left atrium, right ventricle, left ventricle. c. Right atrium, right ventricle, left atrium, left ventricle. d. Right atrium, left atrium, left ventricle, right ventricle. ANS: C
Unoxygenated blood flows through the venae cavae into the right atrium, where it is pumped down to the right ventricle; the blood is then pumped out the pulmonary artery and is returned oxygenated via the pulmonary vein to the left atrium, where it flows to the left ventricle and is pumped out to the rest of the body via the aorta. DIF: Remember REF: 954 OBJ: Describe the structure and function of the cardiopulmonary system. TOP: Pathology MSC: CP on. sC of PM ractice NNRRE: FIounGdatiB
U S N T
O
12. The nurse caring for a patient with ischemia of the left coronary artery would expect to find
which sign? a. Increased ventricular diastole. b. Increased stroke volume. c. Decreased preload. d. Decreased afterload. ANS: D
The left coronary artery supplies the muscles of the left ventricle; the strength of the muscle affects the contractility of the heart. The other options are not affected by the muscles of the left ventricle. DIF: Understand REF: 963 OBJ: Identify the physiological processes of cardiac output, myocardial blood flow, and coronary artery circulation. TOP: Assessment MSC: CPNRE: Foundations of Practice 13. Normal cardiac output is 2.5 to 4 L/minute/m3 in a healthy adult at rest. Which of the
following is the correct formula to calculate cardiac output? a. Stroke volume heart rate. b. Stroke volume/body surface area. c. Body surface area cardiac index. d. Heart rate/stroke volume.
Canadian Fundamentals of Nursing 6th Edition Potter Test Bank ANS: A
Cardiac output can be calculated by multiplying the stroke volume and the heart rate. The other options are not measures of cardiac functioning. DIF: Remember REF: 955 OBJ: Identify the physiological processes of cardiac output, myocardial blood flow, and coronary artery circulation. TOP: Assessment MSC: CPNRE: Foundations of Practice 14. The nurse is careful to monitor a patient’s cardiac output because this helps the nurse to
determine which of the following? a. Peripheral extremity circulation. b. Oxygenation requirements. c. Cardiac arrhythmias. d. Ventilation status. ANS: A
Cardiac output indicates how much blood is being circulated systemically. Oxygen status would be determined by pulse oximetry and the presence of cyanosis. Cardiac arrhythmias are irregular rhythms whose electrical impulse monitored through five-lead electrocardiography (ECG). Ventilation status is not dependent solely on cardiac output. DIF: Apply REF: 955 OBJ: Describe the relationship among cardiac output, preload, afterload, contractility, and heart rate. TOP: Assessment MSC: CPNRE: Foundations of Practice 15. A nurse is assisting a patient with ambulation. The patient becomes short of breath and begins
to complain of sharp chest pain. Which action by the nurse is the first priority? a. Call for the emergency reN spo ngOtM he defibrillator. URnse SIteam NGTtoBb.riC b. Have the patient sit down in the nearest chair. c. Return the patient to the room and apply 100% oxygen. d. Ask a co-worker to get the ECG machine STAT. ANS: B
The patient is experiencing cardiac distress for reasons unknown. The nurse should first secure the safety of the patient and decrease the workload on the patient’s heart by putting him in a resting position; this will increase cardiac output by decreasing afterload. Once the patient is stable, the nurse can obtain oxygen to put on the patient. Next, the nurse can begin to monitor the patient’s oxygen and cardiac status. If necessary, the emergency team may be activated to defibrillate. DIF: Apply REF: 964 | 967 OBJ: Identify nursing care interventions in the primary care, acute care, and restorative and continuing care settings that promote oxygenation. TOP: Implementation MSC: CPNRE: Foundations of Practice 16. A patient has inadequate stroke volume in relation to decreased preload. What action does the
nurse anticipate? a. Placing the patient on oxygen monitoring. b. Administering vasodilators. c. Verifying that the blood consent form has been signed. d. Preparing the patient for dialysis.
Canadian Fundamentals of Nursing 6th Edition Potter Test Bank ANS: C
Preload is affected by the circulating volume; if the patient has decreased fluid, it will need to be replaced with fluid or blood therapy. Before blood products are administered, typing and matching should be performed. Monitoring the patient’s oxygenation status will not affect preload. Administering vasodilators affects afterload. Dialysis would further remove fluid from the patient, thus decreasing preload. DIF: Apply REF: 955 OBJ: Describe the relationship among cardiac output, preload, afterload, contractility, and heart rate. TOP: Planning MSC: CPNRE: Foundations of Practice 17. When caring for a patient with atrial fibrillation, the nurse is most concerned with which vital
sign? a. Heart rate. b. Pain. c. Oxygen saturation. d. Blood pressure. ANS: C
Atrial fibrillation results in pooling of blood in the atria, forming emboli that can be pumped out to the rest of the body. The most common manifestations are stroke, myocardial infarction, and pulmonary embolus. A sudden and drastic drop in oxygenation and blood pressure can indicate both pulmonary embolus and myocardial infarction. DIF: Apply REF: 960 OBJ: Identify and describe clinical outcomes as a result of disturbances in conduction, altered cardiac output, impaired valvular function, myocardial ischemia, and impaired tissue perfusion. TOP: Assessment MSC: CP : FI oN unG daT tiB on. sC ofO PM ractice NNURRES 18. The nurse would expect a patient with right-sided heart failure to have which of the
following? a. Peripheral edema. b. Basilar crackles. c. Chest pain. d. Cyanosis. ANS: A
Right-sided heart failure results from inability of the right side of the heart to pump effectively, which leads to a systemic backup. Peripheral edema and hepatojugular distension are signs of right-sided failure. Basilar crackles can indicate pulmonary congestion from left-sided heart failure. Cyanosis and chest pain result from inadequate tissue perfusion. DIF: Remember REF: 970, Table 39-4 OBJ: Identify nursing care interventions in the primary care, acute care, and restorative and continuing care settings that promote oxygenation. TOP: Assessment MSC: CPNRE: Foundations of Practice 19. Which statement by the patient indicates an understanding of atelectasis? a. “It is important to do breathing exercises every hour to prevent atelectasis.” b. “If I develop atelectasis, I will need a chest tube to drain excess fluid.” c. “Atelectasis affects only those with chronic conditions such as emphysema.” d. “Hyperventilation will open up my alveoli, preventing atelectasis.”
Canadian Fundamentals of Nursing 6th Edition Potter Test Bank
ANS: A
Atelectasis develops when alveoli do not expand. Breathing exercises increase lung volume and open the airways. Deep breathing opens the pores of Kohn between the alveoli to allow sharing of oxygen between alveoli. This prevents atelectasis from developing. DIF: Analyze REF: 957 OBJ: Identify and describe clinical outcomes of hyperventilation, hypoventilation, and hypoxemia. TOP: Evaluate MSC: CPNRE: Foundations of Practice 20. A nurse is caring for a patient whose temperature is 37.9°C (100.2°F). The nurse expects this
patient to hyperventilate for which reason? a. Increased metabolic demands. b. Anxiety over illness. c. Decreased drive to breathe. d. Destruction of lung tissues by infection. ANS: A
Fever increases the metabolic demands of the body, increasing production of carbon dioxide. The body hyperventilates to get rid of excess carbon dioxide. Anxiety can cause hyperventilation, but fever would be the primary cause. Hyperventilation decreases the drive to breathe. The cause of the fever in this question is unknown. DIF: Apply REF: 964 OBJ: Identify and describe clinical outcomes of hyperventilation, hypoventilation, and hypoxemia. TOP: Assessment MSC: CPNRE: Foundations of Practice 21. What assessment finding is the earliest sign of hypoxia? N R I G B.C M U S N T a. Restlessness. O b. Decreased blood pressure. c. Cardiac dysrhythmias. d. Cyanosis. ANS: A
Hypoxia results from inadequate tissue oxygen at the cellular level. The earliest sign of hypoxia is restlessness; as it progresses, mental status changes, cardiac changes, and cyanosis can occur. Early hypoxia results in elevated blood pressure. In later hypoxia, changes in vital sign occur, such as increased heart and respiratory rates. Cyanosis is a late sign of hypoxia. DIF: Understand REF: 964 OBJ: Identify and describe clinical outcomes of hyperventilation, hypoventilation, and hypoxemia. TOP: Assessment MSC: CPNRE: Foundations of Practice 22. A nurse is caring for a patient who suffered a myocardial infarction in the left coronary artery.
Upon assessment, what would the nurse expect to find? a. Blood in the sputum. b. Distended jugular vein. c. Peripheral edema. d. Crackles in the lungs. ANS: D
Canadian Fundamentals of Nursing 6th Edition Potter Test Bank The left coronary artery supplies the left ventricle of the heart; damage to the muscle in the left ventricle leads to pulmonary congestion and frothy sputum, and crackles may be heard. A distended jugular vein and peripheral edema are associated with damage to the right side of the heart. Blood in the sputum is indicative of an infection such as tuberculosis. DIF: Analyze REF: 961 OBJ: Identify and describe clinical outcomes as a result of disturbances in conduction, altered cardiac output, impaired valvular function, myocardial ischemia, and impaired tissue perfusion. TOP: Assessment MSC: CPNRE: Foundations of Practice 23. A nurse is caring for a patient who has poor tissue perfusion as the result of hypertension.
When the patient asks what he should eat for breakfast, what should the nurse recommend? a. A bowl of cereal with whole milk and a banana. b. A cup of nonfat yogurt with granola, and a handful of dried apricots. c. Whole wheat toast with butter, a side of cottage cheese. d. Omelette with sausage, cheese, and onions. ANS: B
Diets high in potassium, fibre, and calcium and low in fat are best for someone who is managing hypertension. Nonfat yogurt with granola is a good source of calcium, fibre, and potassium; dried apricots are a second source of potassium. Although cereal and a banana provide fibre and potassium, skim milk should be substituted for whole milk to decrease fat. An omelette with sausage and cheese is high in fat, as is butter. DIF: Apply REF: 966 OBJ: Identify nursing care interventions in the primary care, acute care, and restorative and continuing care settings that promote oxygenation. TOP: Planning MSC: CPNRE: Foundations ofNPU raR ctice SINGTB.COM 24. A nurse caring for a patient with chronic obstructive pulmonary disease (COPD) knows that
which oxygen delivery device is most appropriate? a. Nasal cannula. b. Simple facemask. c. Partial non-rebreathing mask. d. Non-rebreathing mask. ANS: A
Nasal cannulas deliver oxygen from 1 to 6 L/min. A patient with COPD should never receive more than 3 L/min because this decreases the drive to breathe, which results in hypoventilation. All other devices are intended for flow rates greater than 6 L/min. DIF: Analyze REF: 979 | 1002 OBJ: Identify nursing care interventions in the primary care, acute care, and restorative and continuing care settings that promote oxygenation. TOP: Planning MSC: CPNRE: Foundations of Practice 25. The nurse determines that an older patient is at risk for infection because of decreased
immunity. Which plan of care best addresses the prevention of infection for the patient? a. Encourage the patient to stay up to date on all vaccinations. b. Inform the patient of the importance of finishing the entire dose of antibiotics. c. Schedule the patient for annual tuberculosis skin testing. d. Create an exercise routine to run 30 minutes every day.
Canadian Fundamentals of Nursing 6th Edition Potter Test Bank
ANS: A
A nursing care plan for preventive health measures should be reasonable and feasible. Keeping up to date on vaccinations is important because vaccine reduces the severity of illnesses and serious complications. Although it is important to finish the full course of antibiotics, it is not a preventive health measure. Scheduling annual tuberculosis skin tests does not address prevention. The exercise routine should be reasonable to increase compliance. DIF: Apply REF: 977 OBJ: Identify nursing care interventions in the primary care, acute care, and restorative and continuing care settings that promote oxygenation. TOP: Planning MSC: CPNRE: Foundations of Practice 26. The nurse would expect which change in cardiac output for a patient with fluid volume
overload? a. Increased preload. b. Decreased afterload. c. Decreased tissue perfusion. d. Increased heart rate. ANS: A
Preload refers to the stretch of the ventricle in relation to the volume of blood; an increase in circulating volume would increase the preload of the heart. Afterload refers to resistance; increased pressure would lead to increased resistance, and afterload would increase. A decrease in tissue perfusion would be seen with hypovolemia. A decrease in fluid volume would cause an increase in heart rate because the body is attempting to increase cardiac output. N R I G B.C M
U S N T
O
DIF: Understand REF: 955 OBJ: Describe the relationship among cardiac output, preload, afterload, contractility, and heart rate. TOP: Planning MSC: CPNRE: Foundations of Practice 27. The nurse expects to find which of the following in a patient with angina pectoris? a. Experience feelings of indigestion after eating a heavy meal. b. Have decreased oxygen saturation during rest. c. Hypoventilate during periods of acute stress. d. Complain of tingling in the left arm that lasts throughout the morning. ANS: A
Angina pectoris is chest pain that results from limited oxygen supply. Often pain is precipitated by activities such as exercise, stress, and eating a heavy meal and lasts from 1 to 15 minutes. Hyperventilation may occur to compensate for decreased oxygen perfusion and during periods of acute stress. Symptoms of angina pectoris are relieved by rest, nitroglycerine, or both. Oxygen saturation, pain, and tingling in the arm should be relieved by rest. Pain or arm tingling that persists could be a sign of myocardial infarction. DIF: Understand REF: 963 OBJ: Identify and describe clinical outcomes as a result of disturbances in conduction, altered cardiac output, impaired valvular function, myocardial ischemia, and impaired tissue perfusion. TOP: Planning MSC: CPNRE: Foundations of Practice
Canadian Fundamentals of Nursing 6th Edition Potter Test Bank 28. Which of the following is a nonmodifiable risk factor for lung disease? a. Allergies. b. Smoking. c. Stress. d. Asbestos exposure. ANS: A
A nonmodifiable risk factor is one over which the patient has no control. Allergies can be managed, but immune-mediated responses cannot be controlled. Smoking, stress, and asbestos exposure are all modifiable risk factors. DIF: Understand REF: 969 OBJ: Describe the effect of a patient’s level of health, age, lifestyle, and environment on cardiopulmonary oxygenation. TOP: Assessment MSC: CPNRE: Foundations of Practice 29. The nurse is creating a plan of care for an obese patient who is suffering from fatigue related
to ineffective breathing. Which intervention best addresses a short-term goal that the patient could achieve? a. Running 30 minutes every morning. b. Stopping smoking immediately. c. Sleeping on two to three pillows at night. d. Limiting the diet to 1500 calories a day. ANS: C
To achieve a short-term goal, the nurse should plan a lifestyle change that the patient can make immediately that will have a quick effect. Sleeping on several pillows at night would immediately relieve orthopneNaU anRdSoI pen atC ient ’s airway, thereby reducing sleep apnea NGthe TBp. OM and reducing fatigue. Running 30 minutes a day would improve cardiopulmonary health, but a patient needs to build up exercise tolerance. Smoking cessation is another process, but many people have difficulty quitting immediately; quitting often occurs as a slow progression, beginning with reduction of frequency. A more realistic short-term goal would be to gradually reduce the number of cigarettes smoked. Limiting caloric intake can help a patient lose weight, but this is a gradual process and is not reasonable for a short-term goal. DIF: Analyze REF: 968 OBJ: Identify nursing care interventions in the primary care, acute care, and restorative and continuing care settings that promote oxygenation. TOP: Planning MSC: CPNRE: Foundations of Practice 30. A nurse is caring for a patient with left-sided hemiparesis who has developed bronchitis and
has a heart rate of 105, blood pressure of 156/90, and a respiration rate of 30. Which nursing diagnosis is the priority for this patient? a. Activity intolerance. b. Risk for skin breakdown. c. Impaired gas exchange. d. Risk for infection. ANS: C
Canadian Fundamentals of Nursing 6th Edition Potter Test Bank The most important nursing intervention is to maintain airway and circulation for this patient; therefore, Impaired gas exchange is the first nursing priority. Activity intolerance is a concern but is not the priority in this case. Risk for skin breakdown and Risk for infection are also important but do not address an immediate impairment with physiological integrity. DIF: Analyze REF: 973 OBJ: Identify nursing care interventions in the primary care, acute care, and restorative and continuing care settings that promote oxygenation. TOP: Planning MSC: CPNRE: Foundations of Practice 31. Which nursing intervention is most effective in preventing hospital-acquired pneumonia in an
older patient? a. Assisting the patient to cough, turn, and breathe deeply every 2 hours. b. Encouraging patient to drink through a straw to prevent aspiration. c. Discontinuing humidification delivery device to keep excess fluid from lungs. d. Monitoring oxygen saturation and frequently assessing lung bases. ANS: A
The goal of the nursing action should be the prevention of pneumonia; the intervention that best addresses this is for the patient to cough, turn, and breathe deeply to keep secretions from pooling at the base of the lungs. Drinking through a straw increases the risk of aspiration. Humidification thins respiratory secretions, which makes them easier to expel. Monitoring oxygen status is important but is not a method of prevention. DIF: Analyze REF: 978 | 979 OBJ: Identify nursing care interventions in the primary care, acute care, and restorative and continuing care settings that promote oxygenation. TOP: Implementation MSC: CPNRE: Foundations ofNPU raR ctice SINGTB.COM 32. The nurse is assessing a patient with emphysema. Which assessment finding necessitates
further follow-up with the physician? a. Clubbing of the fingers. b. Increased anterior-posterior diameter of the chest. c. Hemoptysis. d. Tachypnea. ANS: C
Hemoptysis (blood in the sputum) is an abnormal occurrence in emphysema, and further diagnostic studies are needed to determine the cause. Clubbing of the fingers, barrel chest, and tachypnea are all typical findings in a patient with emphysema. DIF: Apply REF: 968 OBJ: Identify nursing care interventions in the primary care, acute care, and restorative and continuing care settings that promote oxygenation. TOP: Assessment MSC: CPNRE: Foundations of Practice 33. A patient with COPD asks the nurse why he is having increased difficulty with his fine motor
skills, such as buttoning his shirt. Which response by the nurse is most therapeutic? a. “Your body isn’t receiving enough oxygen to send down to your fingers; this causes them to club and makes dexterity difficult.” b. “Your disease process makes even the smallest tasks seem exhausting. Try taking a nap before getting dressed.”
Canadian Fundamentals of Nursing 6th Edition Potter Test Bank c. “Often patients with your disease lose mental status and forget how to perform
daily tasks.” d. “Your disease affects both your lungs and your heart, and not enough blood is
being pumped, so you are losing sensory feedback in your extremities.” ANS: A
Clubbing of the nail bed is a frequent symptom of COPD and can make activities of daily living difficult. Taking a nap decreases fatigue but does not help the patient perform fine motor skills. Loss of mental status is not a normal finding with COPD. Low oxygen level, not low circulating blood volume, is the problem in COPD. DIF: Apply REF: 969 OBJ: Describe the effect of a patient’s level of health, age, lifestyle, and environment on cardiopulmonary oxygenation. TOP: Patient Teaching MSC: CPNRE: Foundations of Practice 34. The nurse is caring for a patient with a tracheostomy tube. Which nursing intervention is most
effective in promoting effective airway clearance? a. Suctioning respiratory secretions several times every hour. b. Administering humidified oxygen through a tracheostomy collar. c. Instilling normal saline into the tracheostomy to thin secretions before suctioning. d. Deflating the tracheostomy cuff before allowing the patient to cough up secretions. ANS: B
Humidification of air will help keep the mucous membranes moist and will make secretions easier to expel. Suctioning should be done only as needed; too frequent suctioning can damage the mucosal lining, which would result in thicker secretions. Normal saline should never be instilled into a trachN eost y be se.thCisOcMould lead to infection. The purpose of the URom SI NGcau TB tracheostomy cuff is to keep secretions from entering the lungs; the nurse should not deflate the tracheostomy cuff unless instructed to do so by the physician. DIF: Apply REF: 1002 OBJ: Identify nursing care interventions in the primary care, acute care, and restorative and continuing care settings that promote oxygenation. TOP: Implementation MSC: CPNRE: Foundations of Practice 35. The nurse is educating a student nurse on caring for a patient with a chest tube. The nurse
knows that teaching has been effective when the student makes which statement? a. “I should strip the drains on the chest tube every hour to promote drainage.” b. “If the chest tube becomes dislodged, the first thing I should do is notify the physician.” c. “I should clamp the chest tube when giving the patient a bed bath.” d. “I should report if I do not see continuous bubbling in the water-seal chamber.” ANS: D
Correct care of a chest tube involves knowing normal and abnormal functioning of the tube. Bubbling in the water-seal chamber is expected. Stripping the drain requires a prescriber order. If the chest tube becomes dislodged, the nurse should immediately apply occlusive pressure over the insertion site. The chest tube should not be clamped unless necessary; if so, the length of time clamped must be minimal to reduce the risk of pneumothorax. DIF: Analyze
REF: 998
Canadian Fundamentals of Nursing 6th Edition Potter Test Bank OBJ: Identify nursing care interventions in the primary care, acute care, and restorative and continuing care settings that promote oxygenation. TOP: Planning MSC: CPNRE: Foundations of Practice 36. Which nursing diagnosis is the priority when the nurse is caring for a patient with a traumatic
brain injury who had a tracheostomy placed? a. Risk for skin breakdown. b. Impaired gas exchange. c. Ineffective airway clearance. d. Risk for infection. ANS: C
Patients with tracheotomies rely on the tracheostomy to provide a stable open airway. The nurse is also concerned that the patient would not be able to cough up his own secretions, which could occlude the tracheostomy and put the patient at risk for ineffective airway clearance. Nursing priorities are airway, breathing, and circulation. Nursing diagnoses of actual conditions should be addressed before “Risk” diagnoses. Skin breakdown and infection are not immediately life threatening. DIF: Analyze REF: 980 | 1005 OBJ: Identify nursing care interventions in the primary care, acute care, and restorative and continuing care settings that promote oxygenation. TOP: Planning MSC: CPNRE: Foundations of Practice 37. The nurse knows that the most effective method for suctioning a patient with a tracheostomy
tube is which of the following? a. Set suction regulator at 150 to 200 mm Hg. b. Liberally lubricate the enN dU ofRthSeIsN ucG tiT onBc. atC heOteMr with a water-soluble solution. c. Limit the length of suctioning to 10 to 15 seconds. d. Apply suction while gently rotating and inserting the catheter. ANS: C
Suctioning passes should be limited to 10 to 15 seconds to avoid oxygen desaturation. Suction for a tracheostomy should be set at 100 to 150 mm Hg. Excessive lubrication can clog the catheter or occlude the airway. Suction should not be applied until after the catheter has been inserted. DIF: Apply REF: 983-986 OBJ: Identify nursing care interventions in the primary care, acute care, and restorative and continuing care settings that promote oxygenation. TOP: Implementation MSC: CPNRE: Foundations of Practice 38. The nurse is assessing a patient with a right pneumothorax. Which finding would the nurse
expect? a. Bilateral expiratory crackles. b. Absence of breath sounds on the right side. c. Right-sided wheezes on inspiration. d. Trachea deviation to the right. ANS: B
Canadian Fundamentals of Nursing 6th Edition Potter Test Bank A right pneumothorax is a collection of air in the pleural space, which causes the lung to collapse; therefore, no breath sounds should be heard on that side. Crackles indicate pneumonia. Wheezes are asthma related. A collapsed right lung would cause the trachea to deviate to the left. DIF: Apply REF: 998 OBJ: Identify the physiological processes involved in ventilation, perfusion, and exchange of respiratory gases. TOP: Assessment MSC: CPNRE: Foundations of Practice 39. While the nurse is changing the ties on a tracheostomy collar, the patient coughs, dislodging
the tracheostomy tube. What is the nurse’s first nursing action? a. Press the emergency response button. b. Place the patient on a facemask delivering 100% oxygen. c. Insert a spare tracheostomy. d. Manually occlude the tracheostomy with sterile gauze. ANS: C
The nurse’s first priority is to establish a stable airway by inserting a spare tracheostomy tube into the patient’s airway. The nurse could activate the emergency response team if the patient is still unstable after the tracheostomy tube is placed. A patient with a tracheostomy has an impairment that causes him or her not to be able to breathe normally; a facemask would not be an effective method of getting air into the lungs. Manually occluding pressure over the tracheostomy site is not appropriate. DIF: Apply REF: 997 OBJ: Identify nursing care interventions in the primary care, acute care, and restorative and continuing care settings that promote oxygenation. TOP: Implementation I G B.C M MSC: CPNRE: Foundations ofNPU raR ctice S N T O
Canadian Fundamentals of Nursing 6th Edition Potter Test Bank
Chapter 40: Fluid, Electrolyte, and Acid–Base Balances Potter et al: Canadian Fundamentals of Nursing, 6th Edition MULTIPLE CHOICE 1. Approximately two-thirds of the body’s total water volume exists in which fluid? a. Intracellular. b. Interstitial. c. Intravascular. d. Transcellular. ANS: A
Intracellular fluid accounts for approximately two-thirds of the fluids in the body and about 42% of total body weight. Interstitial fluid, intravascular fluid, and transcellular fluid constitute extracellular fluid, which is the fluid outside a cell. DIF: Remember REF: 1016 OBJ: Describe the distribution, composition, movement, and regulation of body fluids. TOP: Assessment MSC: CPNRE: Foundations of Practice 2. What is the process in which water moves passively from an area of lower particle
concentration to an area of higher particle concentration? a. Hydrolysis. b. Osmosis. c. Filtration. d. Active transport. ANS: B
NURSINGTB.COM
The process in which water moves passively from an area of low particle concentration to an area of higher particle concentration is known as osmosis. Hydrolysis is a term not related to fluid and electrolyte balance. Filtration is mediated by fluid pressure from an area of higher pressure to an area of lower pressure. Active transport requires metabolic activity and is not passive. DIF: Remember REF: 1017 OBJ: Describe the distribution, composition, movement, and regulation of body fluids. TOP: Assessment MSC: CPNRE: Foundations of Practice 3. The nurse knows that in a patient who has venous congestion from right-sided heart failure,
edema develops as a result of an imbalance with regard to which pressure? a. Hydrostatic. b. Osmotic. c. Oncotic. d. Concentration. ANS: A
Venous congestion increases capillary hydrostatic pressure. Increased hydrostatic pressure causes increases in movement of fluid into the interstitial area, which results in edema. Osmotic and oncotic pressures involve the concentrations of solutes and can contribute to edema in other situations. Concentration pressure is not a nursing term.
Canadian Fundamentals of Nursing 6th Edition Potter Test Bank DIF: Remember REF: 1018 OBJ: Describe common fluid, electrolyte, and acid–base imbalances, and identify related risk factors. TOP: Assessment MSC: CPNRE: Foundations of Practice 4. The nurse understands that administering a hypertonic solution to a patient will shift water
from which space to which space? a. Intracellular; extracellular. b. Extracellular; intracellular. c. Intravascular; intracellular. d. Intravascular; interstitial. ANS: A
The concentration of a hypertonic solution is greater than those of normal body fluids, and so water will shift out of cells because of the osmotic pull of the extra particles. Movement of water into cells occurs when hypotonic fluids are administered. Distribution of fluid between intravascular and interstitial spaces occurs by filtration, the net sum of hydrostatic and osmotic pressures. DIF: Understand REF: 1052 OBJ: Describe the distribution, composition, movement, and regulation of body fluids. TOP: Assessment MSC: CPNRE: Foundations of Practice 5. Which patient is most at risk for sensible water loss? a. A 7-year-old child with asthma. b. A 24-year-old adult with constipation. c. A 56-year-old patient with gastroenteritis. d. An 80-year-old patient with pneumonia. ANS: D
NURSINGTB.COM
Sensible water loss is the loss of fluids from the skin through visible perspiration, such as that occurring with a resolving fever related to pneumonia. Asthma would be insensible water loss through respiration. Gastroenteritis causes diarrhea with its large volume loss. Constipation does not affect fluid loss. DIF: Apply REF: 1018 OBJ: Describe common fluid, electrolyte, and acid–base imbalances, and identify related risk factors. TOP: Assessment MSC: CPNRE: Foundations of Practice 6. What is the most abundant cation in the blood? a. Sodium. b. Potassium. c. Chloride. d. Magnesium. ANS: A
Sodium is the most abundant cation in the blood. Potassium is the predominant intracellular cation. Chloride is an anion (negatively charged) rather than a cation (positively charged). Magnesium is found predominantly inside cells and in bone. DIF: Remember REF: 1016 OBJ: Describe the processes involved in regulating acid–base balance. TOP: Assessment MSC: CPNRE: Foundations of Practice
Canadian Fundamentals of Nursing 6th Edition Potter Test Bank
7. The nurse receives the patient’s most recent blood work results. Which laboratory value is of
greatest concern? a. Sodium, 145 mmol/L (145 mEq/L). b. Calcium, 4.5 mmol/L (17.5 mg/dL). c. Potassium, 3.5 mmol/L (3.5 mEq/L). d. Chloride, 100 mmol/L (100 mEq/L). ANS: B
Normal calcium levels range from 2.25 to 2.75 mmol/L (8.5 mg/dL to 10.5 mg/dL); therefore, a value of 4.5 mmol/L (17.5 mg/dL) is abnormally high and of concern. The rest of the laboratory values are within their normal ranges: Normal sodium levels range from 135 to 145 mmol/L (135 to 145 mEq/L); normal potassium levels range from 3.5 to 5.0 mmol/L (3.5 to 5.0 mEq/L); and normal chloride levels range from 97 to 107 mmol/L (98 to 106 mEq/L). DIF: Remember REF: 1020 OBJ: Choose appropriate clinical assessments for fluid, electrolyte, and acid–base balances. TOP: Assessment MSC: CPNRE: Foundations of Practice 8. The nurse would expect a patient with increased levels of serum calcium to also have which
abnormal levels? a. Increased potassium levels. b. Decreased phosphate levels. c. Decreased sodium levels. d. Increased magnesium levels. ANS: B
Serum calcium and phosphatN eU haR veSaInNinGver elat ionship. When one level is elevated, the Bse.rC M T O other is decreased, except in some patients with end-stage renal disease. Increased level of serum calcium would not necessarily cause changes in levels of potassium, sodium, or magnesium. DIF: Remember REF: 1020 OBJ: Choose appropriate clinical assessments for fluid, electrolyte, and acid–base balances. TOP: Assessment MSC: CPNRE: Foundations of Practice 9. The nurse knows that an imbalance of which ion causes acid–base impairment? a. Hydrogen. b. Calcium. c. Magnesium. d. Sodium. ANS: A
The concentration of hydrogen ions determines pH. Low pH reflects an acidic environment; high pH reflects an alkaline environment. Calcium, magnesium, and sodium are ions, but imbalances of those ions are not direct acid–base impairments. DIF: Remember REF: 1020 OBJ: Choose appropriate clinical assessments for fluid, electrolyte, and acid–base balances. TOP: Assessment MSC: CPNRE: Foundations of Practice
Canadian Fundamentals of Nursing 6th Edition Potter Test Bank 10. The nurse would expect a patient with respiratory acidosis to have an excessive amount of
which of the following? a. Carbon dioxide. b. Bicarbonate. c. Oxygen. d. Phosphate. ANS: A
In respiratory acidosis, the lungs are not able to excrete enough carbon dioxide. Carbon dioxide and water create carbonic acid. A buildup of carbonic acid causes the extracellular fluid to become more acidic, which lowers the pH. Bicarbonate levels are normal with uncompensated respiratory acidosis or elevated with compensated respiratory acidosis. Excessive oxygen and phosphate levels are not characteristic of respiratory acidosis. DIF: Remember REF: 1021 OBJ: Describe the processes involved in acid–base balance. MSC: CPNRE: Foundations of Practice
TOP: Assessment
11. A 2-year-old was brought into the emergency department after ingesting several morphine
tablets from a bottle in his mother’s purse. The nurse knows that the child is at greatest risk for which acid–base imbalance? a. Respiratory acidosis. b. Respiratory alkalosis. c. Metabolic acidosis. d. Metabolic alkalosis. ANS: A
Morphine overdose can causN e res y de ssio n and hypoventilation. Hypoventilation URpira SItorNG TBpr.eC OM results in retention of CO2 and respiratory acidosis. Respiratory alkalosis would result from hyperventilation, causing a decrease in CO2 levels. Metabolic acid–base imbalance would be a result of kidney dysfunction, vomiting, diarrhea, or other conditions that affect metabolic acids. DIF: Analyze REF: 1025 OBJ: Describe common fluid, electrolyte, and acid–base imbalances, and identify related risk factors. TOP: Assessment MSC: CPNRE: Foundations of Practice 12. A patient was admitted for a bowel obstruction and has had a nasogastric tube set to low
intermittent suction for the past 3 days. The patient’s respiratory rate has decreased to 12 breaths per minute. The nurse would expect the patient to have which of the following arterial blood gas values? a. pH, 7.78; PaCO2, 40 mm Hg; HCO3–, 30 mmol/L (30 mEq/L). b. pH, 7.52; PaCO2, 48 mm Hg; HCO3–, 28 mmol/L (28 mEq/L). c. pH, 7.35; PaCO2, 35 mm Hg; HCO3–, 26 mmol/L (26 mEq/L). d. pH, 7.25; PaCO2, 47 mm Hg; HCO3–, 29 mmol/L (29 mEq/L). ANS: B
Canadian Fundamentals of Nursing 6th Edition Potter Test Bank In compensated metabolic alkalosis, the pH and bicarbonate (HCO3–) values (metabolic values) are alkaline, and CO2 level is slightly acidic (compensatory respiratory acidosis). In this case, the pH of 7.52 is alkaline (normal = 7.35 to 7.45), partial pressure of arterial carbon dioxide (PaCO2) is acidic (normal = 35 to 45 mm Hg), and HCO3– level is elevated (normal = 22 to 26 mmol/L [22 to 26 mEq/L]). When the pH is 7.78, PaCO2 is 40 mm Hg, and HCO3– level is 30 mmol/L (30 mEq/L), the condition is uncompensated metabolic alkalosis. A pH of 7.35, a PaCO2 35 mm Hg, and a HCO3– level of 26 mmol/L (26 mEq/L) are within normal limits. When the pH is 7.25, PaCO2 is 47 mm Hg, and HCO3– level is 29 mmol/L (29 mEq/L), the condition is compensated respiratory acidosis. DIF: Analyze REF: 1025, Table 40-5 OBJ: Interpret basic fluid, electrolyte, and acid–base laboratory values. TOP: Assessment MSC: CPNRE: Foundations of Practice 13. The nurse would not expect full compensation to occur for which acid–base imbalance? a. Respiratory acidosis. b. Respiratory alkalosis. c. Metabolic acidosis. d. Metabolic alkalosis. ANS: B
Usually the cause of respiratory alkalosis is a temporary event (e.g., an asthma or anxiety attack). The kidneys take about 24 hours to compensate for an event, so it is unlikely that much if any compensation for respiratory alkalosis is observable. Respiratory acidosis usually results from longer term conditions such as chronic lung disease, narcotic overdose, or another event that causes respiratory depression. The kidneys still do not respond for about 24 hours of the event. For both metabolic imbalances, the respiratory system is quick to attempt to N ave RSdif IN G ltyB.C M compensate; however, it may hU ficuT sustaOining that compensation. DIF: Understand REF: 1025 OBJ: Describe the processes involved in regulating acid–base balance. TOP: Assessment MSC: CPNRE: Foundations of Practice 14. A nurse is caring for a patient whose electrocardiogram displays changes characteristic of
hypokalemia. Which assessment finding would the nurse expect? a. Thready peripheral pulses. b. Abdominal distension. c. Dry mucous membranes. d. Flushed skin. ANS: B
Signs and symptoms of hypokalemia are muscle weakness and fatigue, abdominal distension, decreased bowel sounds, and cardiac dysrhythmias. Thready peripheral pulses indicate hypovolemia. Dry mucous membranes and flushed skin are indicative of dehydration and hypernatremia. DIF: Apply REF: 1023, Table 40-4 OBJ: Choose appropriate clinical assessments for fluid, electrolyte, and acid–base balances. TOP: Assessment MSC: CPNRE: Foundations of Practice 15. In which patient would the nurse expect to see a positive Chvostek sign?
Canadian Fundamentals of Nursing 6th Edition Potter Test Bank a. b. c. d.
A 7-year-old child admitted for severe burns. A 24-year-old adult admitted for chronic alcohol abuse. A 50-year-old patient admitted for an acute exacerbation of hyperparathyroidism. A 75-year-old patient admitted for a broken hip in relation to osteoporosis.
ANS: B
A positive Chvostek sign is representative of hypocalcemia or hypomagnesemia. Hypomagnesemia is common with alcohol abuse. Hypocalcemia can be brought on by alcohol abuse and pancreatitis (which also can be affected by alcohol consumption). Patients with burns frequently experience extracellular fluid volume (ECV) deficit. Hyperparathyroidism causes hypercalcemia. Immobility is associated with hypercalcemia. DIF: Apply REF: 1023, Table 40-4 OBJ: Choose appropriate clinical assessments for fluid, electrolyte, and acid–base balances. TOP: Assessment MSC: CPNRE: Foundations of Practice 16. Which organ system is responsible for compensation of respiratory acidosis? a. Respiratory. b. Renal. c. Gastrointestinal. d. Endocrine. ANS: B
The kidneys are responsible for respiratory acidosis compensation. A problem with the respiratory system causes respiratory acidosis, and so another organ system (renal) needs to compensate. Problems with the gastrointestinal and endocrine systems can cause acid–base imbalances, but these systems cannot compensate for an existing imbalance.
NURSINGTB.COM
DIF: Understand REF: 1025 OBJ: Describe the processes involved in regulating acid–base balance. TOP: Assessment MSC: CPNRE: Foundations of Practice 17. Which laboratory value should the nurse examine when evaluating a patient with
uncompensated respiratory alkalosis? a. Partial pressure of arterial oxygen (PaO2). b. Anion gap. c. PaCO2. d. HCO3–. ANS: C
Uncompensated respiratory imbalances are revealed by the PaCO2 levels. PaO2 indicates oxygen status. High anion gap is indicative of metabolic acidosis. HCO3– levels are studied to evaluate compensation for respiratory imbalances or uncompensated metabolic imbalances. DIF: Remember REF: 1026, Table 40-6 OBJ: Choose appropriate clinical assessments for fluid, electrolyte, and acid–base balances. TOP: Assessment MSC: CPNRE: Foundations of Practice 18. The nurse is caring for a patient who has diabetes and is in renal failure. Which laboratory
findings would the nurse expect? a. pH, 7.3; PaCO2, 36 mm Hg; HCO3–, 19 mEq/L (19 mmol/L). b. pH, 7.5; PaCO2, 35 mm Hg; HCO3–, 35 mEq/L (35 mmol/L).
Canadian Fundamentals of Nursing 6th Edition Potter Test Bank c. pH, 7.3; PaCO2, 47 mm Hg; HCO3–, 23 mEq/L (23 mmol/L). d. pH, 7.35; PaCO2, 40 mm Hg; HCO3–, 25 mEq/L (25 mmol/L). ANS: A
Patients in renal failure develop metabolic acidosis. The laboratory values that reflect this are pH of 7.3, PaCO2 of 36 mm Hg, and HCO3– level of 19 mmol/L (19 mEq/L). Laboratory findings of pH of 7.5, PaCO2 of 35 mm Hg, and HCO3– level of 35 mmol/L (35 mEq/L) reflect metabolic alkalosis. Laboratory findings of pH of 7.3, PaCO2 of 47 mm Hg, and HCO3– level of 23 mmol/L (23 mEq/L) reflect respiratory acidosis. Laboratory findings of pH of 7.35, PaCO2 of 40 mm Hg, and HCO3– level of 25 mmol/L (25 mEq/L) are within normal ranges. DIF: Analyze REF: 1031 OBJ: Choose appropriate clinical assessments for fluid, electrolyte, and acid–base balances. TOP: Assessment MSC: CPNRE: Foundations of Practice 19. The nurse is assessing a patient and finds crackles in the lung bases and neck vein distension.
The nurse gives the patient a diuretic. What electrolyte imbalance is the nurse most concerned about? a. Potassium imbalance. b. Sodium imbalance. c. Calcium imbalance. d. Phosphate imbalance. ANS: A
A diuretic can cause excess excretion of potassium, unless it is a potassium-sparing diuretic. The other electrolytes are not excreted in the same way with diuretics.
NURSINGTB.COM
DIF: Understand REF: 1023 OBJ: Describe common fluid, electrolyte, and acid–base imbalances, and identify related risk factors. TOP: Assessment MSC: CPNRE: Foundations of Practice 20. A patient receiving chemotherapy has gained 2 kg (5 pounds) in 2 days. Which assessment
question by the nurse is most appropriate? a. “Are you having difficulty sleeping at night?” b. “How many calories a day do you consume?” c. “Do you have dry mouth or feel thirsty?” d. “How many times a day do you urinate?” ANS: D
A rapid gain in weight usually indicates ECV excess if ECV was initially normal. The patient’s description of urination habits will indicate whether the body is trying to excrete the excess fluid or whether renal dysfunction is contributing to ECV excess. Difficulty sleeping at night can occur if the body builds up excessive fluid in the lungs; however, it could also mean that the patient is getting up frequently to urinate, so the question is not specific enough. Caloric intake does not account for rapid weight changes. Dry mouth and thirst accompany ECV deficit, which would be associated with rapid weight loss. DIF: Apply REF: 1032, Table 40-8 OBJ: Choose appropriate clinical assessments for fluid, electrolyte, and acid–base balances. TOP: Implementation MSC: CPNRE: Foundations of Practice
Canadian Fundamentals of Nursing 6th Edition Potter Test Bank 21. Which fluid order should the nurse question for a patient with a traumatic brain injury? a. 0.45% sodium chloride. b. 0.9% sodium chloride. c. Lactated Ringer’s solution. d. Dextrose 5% in 0.9% sodium chloride. ANS: A
A fluid of 0.45% sodium chloride is a hypotonic solution, and hypotonic solutions cause cells to swell, which can cause increased intracranial pressure. This can be life-threatening for a patient with a traumatic brain injury. The other solutions are physiologically isotonic sodium-containing solutions that will expand ECV but do not cause cell swelling. In the fluid container, dextrose 5% in 0.9% sodium chloride is hypertonic, but the dextrose enters cells rapidly, leaving isotonic 0.9% sodium chloride. DIF: Analyze REF: 1039, Table 40-9 OBJ: Discuss the purpose of, and procedure for, initiating, maintaining, and discontinuing intravenous therapy and peripheral vascular access devices. TOP: Implementation MSC: CPNRE: Foundations of Practice 22. The physician asks the nurse to monitor the fluid volume statuses of a patient with congestive
heart failure and a patient at risk for clinical dehydration. What is the most effective nursing intervention for monitoring both of these patients? a. Weigh the patients every morning before breakfast. b. Ask the patients to record their intake and output. c. Measure the patients’ blood pressure every 4 hours. d. Assess the patients for edema in extremities. ANS: A
NURSINGTB.COM
An effective measure of fluid retention or loss is daily weights; each kilogram (2.2 pounds) gained or lost is equivalent to 1 litre of fluid gained or lost. This measurement should be performed at the same time every day using the same scale and the same amount of clothing. Although intake and output records are important assessment measures, some patients are not able to keep their own records themselves. Blood pressure can decrease with ECV deficit but will not necessarily increase with recent ECV excess (heart failure patient). Edema occurs with ECV excess but not with clinical dehydration. DIF: Apply REF: 1042 OBJ: Identify and discuss nursing interventions for patients with fluid, electrolyte, and acid–base imbalances. TOP: Implementation MSC: CPNRE: Foundations of Practice 23. A nurse is caring for a cancer patient who presents with anorexia, blood pressure of 100/60,
elevated white blood cell count, and oral candidiasis. The nurse knows that the purpose of starting total parenteral nutrition (TPN) is to a. Replace fluid, electrolytes, and nutrients in the patient. b. Stimulate the patient’s appetite to eat. c. Provide medication to raise the patient’s blood pressure. d. Deliver antibiotics to fight off infection. ANS: A
Canadian Fundamentals of Nursing 6th Edition Potter Test Bank TPN is an intravenous (IV) solution composed of nutrients and electrolytes to replace the ones the patient is not eating. TPN does not stimulate the appetite, and it does not contain blood pressure medication or antibiotics. DIF: Apply REF: 1038 OBJ: Discuss the purpose of, and procedure for, initiating, maintaining, and discontinuing intravenous therapy and peripheral vascular access devices. TOP: Implementation MSC: CPNRE: Foundations of Practice 24. A patient presents to the emergency department with the complaint of vomiting and diarrhea
for the past 48 hours. The nurse anticipates which fluid therapy initially? a. 0.9% sodium chloride. b. Dextrose 10% in water. c. Dextrose 5% in water. d. 0.45% sodium chloride. ANS: A
Patients with prolonged vomiting and diarrhea become hypovolemic. The best solution to replace ECV is 0.9% sodium chloride, which is an isotonic solution. Dextrose 10% in water, dextrose 5% in water, and 0.45% sodium chloride act as hypotonic solutions in the body. The first consideration is replacing ECV to oxygenate tissues. DIF: Apply REF: 1039, Table 40-9 OBJ: Discuss the purpose of, and procedure for, initiating, maintaining, and discontinuing intravenous therapy and peripheral vascular access devices. TOP: Implementation MSC: CPNRE: Foundations of Practice 25. A patient with a lower respirN ator y in fect ionTB ha.s C pHOM of 7.25, a PaCO2 of 55 mm Hg, and a UR SI NG
HCO3– level of 20 mmol/L (20 mEq/L). The physician has been notified. Which is the priority nursing intervention for this patient? a. Check the colour of the patient’s urine output. b. Place the patient in Trendelenburg position. c. Encourage the patient to increase respirations. d. Place the patient in high Fowler’s position. ANS: C
The patient has respiratory acidosis from CO2 retention. Increasing rate and depth of respiration will allow the patient to blow off excess carbon dioxide, and this will begin to correct the imbalance. Checking the urine colour is not a necessary assessment. The Trendelenburg position would probably make it more difficult for the patient to breathe and should be avoided. High Fowler’s position may be more comfortable for the patient, but it is not necessary. DIF: Apply REF: 1025 OBJ: Identify and discuss nursing interventions for patients with fluid, electrolyte, and acid–base imbalances. TOP: Implementation MSC: CPNRE: Foundations of Practice 26. The nurse knows that intravenous fluid therapy has been effective for a patient with
hypernatremia when what happens? a. Serum sodium concentration returns to normal. b. Systolic and diastolic blood pressure decrease.
Canadian Fundamentals of Nursing 6th Edition Potter Test Bank c. Large amounts of emesis and diarrhea decrease. d. Urine output increases to 150 mL/hr. ANS: A
Hypernatremia is diagnosed by elevated serum sodium concentration. Blood pressure is not an accurate indicator of hypernatremia. Emesis and diarrhea will not stop because of IV therapy. Urine output is influenced by many factors, including ECV. A large dilute urine output can cause further hypernatremia. DIF: Apply REF: 1023 OBJ: Identify and discuss nursing interventions for patients with fluid, electrolyte, and acid–base imbalances. TOP: Implementation MSC: CPNRE: Foundations of Practice 27. The nurse would select the dorsal venous plexus of the foot as an IV site for which patient? a. A 2-year-old child. b. A 22-year-old adult. c. A 50-year-old patient. d. An 80-year-old patient. ANS: A
Use of the foot as an IV site is common in children but is avoided in adults because of the risk for thrombophlebitis. DIF: Understand REF: 1053, Figure 40-17 OBJ: Discuss the purpose of, and procedure for, initiating, maintaining, and discontinuing intravenous therapy and peripheral vascular access devices. TOP: Implementation MSC: CPNRE: Foundations of Practice G B.C M
N R I U S N T
O
28. Which assessment finding should cause a nurse to question administering a sodium-containing
isotonic IV fluid? a. Blood pressure 102/58. b. Dry mucous membranes. c. Poor skin turgor. d. Pitting edema. ANS: D
Pitting edema indicates that the patient may be retaining excess extracellular fluid, and the nurse should question the order for a solution meant to rehydrate the patient. All other options are consistent with ECV deficit, and the patient would benefit from a sodium-containing isotonic solution that expands ECV. DIF: Understand REF: 1052 OBJ: Discuss the purpose of, and procedure for, initiating, maintaining, and discontinuing intravenous therapy and peripheral vascular access devices. TOP: Implementation MSC: CPNRE: Foundations of Practice 29. A patient is to receive 1500 mL of 0.9% sodium chloride intravenously at a rate of 125 mL/hr.
The nurse is using microdrip gravity drip tubing. What is the minute flow rate (drops per minute)? a. 12 gtt/min. b. 24 gtt/min.
Canadian Fundamentals of Nursing 6th Edition Potter Test Bank c. 125 gtt/min. d. 150 gtt/min. ANS: C
Microdrip tubing delivers 60 gtt/mL. To obtain a rate of 125 mL/hr with microdrip tubing, the calculation is 125 mL/hr 60 gtt/mL 60 min = 125 gtt/min. DIF: Apply REF: 1054 OBJ: Discuss the purpose of, and procedure for, initiating, maintaining, and discontinuing intravenous therapy and peripheral vascular access devices. TOP: Implementation MSC: CPNRE: Foundations of Practice 30. A nurse begins infusing a 250-mL bag of IV fluid at 1845 hours on Monday and programs the
pump to infuse at 20 mL/hr. At what time should the infusion be completed? a. 0645 Tuesday. b. 0675 Tuesday. c. 0715 Tuesday. d. 0735 Tuesday. ANS: C
This answer would be calculated as follows: 250 mL ÷ 20 mL/hr = 12.5 hr 0.5 hr 60 min = 30 min 1845 hours + 12 hr 30 min = 3115, which would be 0715 hours on Tuesday, the following day. DIF: Apply REF: 1055 OBJ: Discuss the purpose of, aNnd R rocI foB r, . inC itiatM and discontinuing Upvascular Sedur NGeaccess T Oing, maintaining, intravenous therapy and peripheral devices. TOP: Implementation MSC: CPNRE: Foundations of Practice 31. A nurse is caring for a patient with diabetes who has a bowel obstruction and has orders to
ensure that the volume of intake matches the output. In the past 4 hours, the patient received dextrose 5% with 0.9% sodium chloride through a 22-gauge catheter, infusing at 150 mL/hr, and has eaten 200 mL of ice chips. The patient also has a nasogastric suction tube set to low continuous suction, which yielded an output of 300 mL. The patient has voided 400 mL of urine. After reporting these values to the physician, what orders does the nurse anticipate? a. Add a potassium supplement to replace loss from output. b. Decrease the rate of intravenous fluids to 100 mL/hr. c. Discontinue the nasogastric suctioning. d. Administer a diuretic to prevent fluid volume excess. ANS: A
The total fluid intake and output equals 700 mL, which meets the provider goals. The nurse should record half the volume of ice chips when calculating intake. Patients with nasogastric suctioning are at risk for potassium deficit, so the nurse would anticipate a potassium supplement to correct this condition. The other measures would be unnecessary because the net fluid volume is equal. DIF: Analyze REF: 1018 OBJ: Identify and discuss nursing interventions for patients with fluid, electrolyte, and acid–base imbalances. TOP: Implementation
Canadian Fundamentals of Nursing 6th Edition Potter Test Bank MSC: CPNRE: Foundations of Practice 32. A patient was admitted for hypovolemia and has IV fluid running at 250 mL/hr. The patient
complains of burning at the IV insertion site. Upon assessment, the nurse does not find redness, swelling, heat, or coolness. The nurse suspects which of the following? a. IV has infiltrated. b. IV has caused phlebitis. c. Fluid is infusing too quickly. d. Patient is allergic to the fluid. ANS: C
The infusion may be flowing faster than the vein can handle, which causes discomfort. The nurse should slow down the infusion. With infiltration, the skin around the IV insertion site becomes blanched, cool, and edematous. Pain, warmth, erythema, and a palpable venous cord are all symptoms of phlebitis. Allergic response to the fluid could involve a combination of itching, flushing, hypotension, and dyspnea, depending on the severity. DIF: Apply REF: 1041 OBJ: Discuss complications associated with intravenous therapy. TOP: Assessment MSC: CPNRE: Foundations of Practice 33. The nurse is caring for a patient with sepsis. The plan of care for the patient is to administer
antibiotics three times a day for 4 weeks. What device will be used to administer these antibiotics? a. A continuous infusion. b. A peripheral catheter with a heparin lock. c. A peripherally inserted central catheter (PICC) line. d. An implanted port cathetN erU . RSINGTB.COM ANS: C
A PICC line is a type of central venous device that can be introduced into a peripheral vein for administration of IV antibiotics for an extended period, over the course of several weeks. A continuous infusion would not be appropriate if the patient were to receive antibiotics only three times daily. A peripheral catheter would not be necessary or have a heparin lock. An implanted port catheter is intended for long-term use of venous access over months, or even years. DIF: Apply REF: 1051 OBJ: Discuss the purpose of, and procedure for, initiating, maintaining, and discontinuing intravenous therapy and peripheral vascular access devices. TOP: Implementation MSC: CPNRE: Foundations of Practice 34. A patient had an acute intravascular hemolytic reaction during a blood transfusion. After
discontinuing of the blood transfusion, what is the nurse’s next action? a. Run normal saline through the existing tubing. b. Start normal saline at to-keep-open (TKO) rate with new tubing. c. Discontinue the IV catheter. d. Return the blood to the blood bank. ANS: B
Canadian Fundamentals of Nursing 6th Edition Potter Test Bank The nurse should first attach new tubing and begin running in normal saline at a rate to keep the vein open, in case any sorts of medications need to be delivered through that IV site. The existing tubing should not be used because that would infuse the blood in the tubing into the patient. It is necessary to preserve the IV catheter in place for IV access to treat the patient. After the patient has been assessed and stabilized, the blood can be returned to the blood bank. DIF: Understand REF: 1058 OBJ: Discuss the purpose of, and procedure for, initiating a blood transfusion and interventions to manage transfusion reaction. TOP: Implementation MSC: CPNRE: Foundations of Practice 35. A nurse is assessing a patient who is receiving a blood transfusion and finds that the patient is
anxiously fidgeting in bed. The patient is afebrile and dyspneic. The nurse auscultates crackles in both lung bases and sees jugular vein distension. The nurse recognizes that the patient is experiencing which transfusion complication? a. Anaphylactic shock. b. Septicemia. c. Fluid volume overload. d. Hemolytic reaction. ANS: C
The signs and symptoms are concurrent with fluid volume overload. Anaphylactic shock would have presented with urticaria, dyspnea, and hypotension. Septicemia would include a fever. A hemolytic reaction would consist of flank pain, chills, and fever. DIF: Understand REF: 1064 | 1065 OBJ: Discuss the purpose of, and procedure for, initiating a blood transfusion and interventions to COplMementation manage transfusion reaction. NURSINGTT OB P:.Im MSC: CPNRE: Foundations of Practice 36. The nurse selects appropriate tubing for a blood transfusion by ensuring that the tubing has
which of the following? a. Two-way valves to allow the patient’s blood to mix and warm the incoming transfused blood. b. An injection port to mix additional electrolytes into the blood. c. An air vent to let bubbles in the blood escape. d. A filter to ensure that clots do not enter the patient. ANS: D
All blood transfusions must have a filter to prevent microemboli from being administered to the patient. The patient’s blood should not be aspirated to mix with the infusion blood. The blood should not have air bubbles to vent; if a bag of blood does have bubbles, the nurse should promptly return the blood to the blood bank. The only substance compatible with blood is normal saline; no additives should be mixed with the infusing blood. DIF: Understand REF: 1065 OBJ: Discuss the purpose of, and procedure for, initiating a blood transfusion and interventions to manage transfusion reaction. TOP: Implementation MSC: CPNRE: Foundations of Practice 37. The nurse is caring for a patient with hyperkalemia. Which body system would be most
important for the nurse to monitor closely?
Canadian Fundamentals of Nursing 6th Edition Potter Test Bank a. b. c. d.
Gastrointestinal. Neurological. Cardiac. Respiratory.
ANS: C
Potassium balance is necessary for cardiac function. Hyperkalemia places the patient at risk for potentially serious dysrhythmias. Monitoring of gastrointestinal, neurological, and respiratory systems would be indicated for other electrolyte imbalances. DIF: Apply REF: 1024 OBJ: Identify and discuss nursing interventions for patients with fluid, electrolyte, and acid–base imbalances. TOP: Assessment MSC: CPNRE: Foundations of Practice 38. A patient has the following laboratory values: sodium level, 145 mmol/L (145 mEq/L);
potassium level, 4.5 mmol/L (4.5 mEq/L); and calcium level, 1.05 mmol/L (4.5 mg/dL). What would the nurse expect to find in the assessment? a. Lightheadedness when the patient stands up. b. Weak quadriceps muscles. c. Tingling of the extremities and tetany. d. Decreased deep tendon reflexes. ANS: C
This patient has hypocalcemia; the normal calcium range is 2.25 to 2.75 mmol/L (8.4 to 10.5 mg/dL). Sodium and potassium values are within their normal ranges (sodium, 135 to 145 mmol/L [135 to 145 mEq/L]; potassium, 3.5 to 5.0 mmol/L [3.5 to 5.0 mEq/L]). Hypocalcemia causes muscle tetany, Trousseau’s sign, and tingling of the extremities. Lightheadedness when a patiN ent stan UR SIdsNuGpTisBa.mCani OMfestation of ECV deficit or sometimes hypokalemia. Weak quadriceps muscles are associated with potassium imbalances. Decreased deep tendon reflexes are related to hypercalcemia or hypermagnesemia. DIF: Apply REF: 1026 | 1027 OBJ: Choose appropriate clinical assessments for fluid, electrolyte, and acid–base balances. TOP: Assessment MSC: CPNRE: Foundations of Practice 39. A patient informs the nurse that he has the type of diabetes that does not have to do with blood
sugar. The nurse advises the patient to make which dietary change? a. Drink plenty of fluids throughout the day to stay hydrated. b. Avoid food high in acid to avoid metabolic acidosis. c. Reduce the quantity of carbohydrates ingested to lower blood sugar. d. Include a serving of dairy in each meal to elevate calcium levels. ANS: A
The patient is indicating that he has diabetes insipidus, which places him at risk for dehydration and hypernatremia. To prevent dehydration, the patient should drink plenty of fluids to replace the extra water excreted in the urine. Foods high in acid should be avoided by a patient with gastroesophageal reflux disease. A reduction in carbohydrates applies to patients with type 2 diabetes mellitus. Calcium-rich dairy products would be recommended for a patient with hypocalcemia. DIF: Analyze REF: 1030 OBJ: Identify and discuss nursing interventions for patients with fluid, electrolyte, and acid–base
Canadian Fundamentals of Nursing 6th Edition Potter Test Bank imbalances.
TOP: Planning
MSC: CPNRE: Foundations of Practice
40. The nurse assesses the patient’s IV and notes that there is edema of the extremity near the
insertion site and that the skin is in this area is discoloured and cool to the touch. Which of the following assessments would also indicate that a patient’s IV has infiltrated? a. Pain and warmth at the insertion site. b. Reddish streak proximal to the insertion site. c. Numbness or loss of sensation. d. Palpable venous cord. ANS: C
Infiltration results in skin that is edematous near the IV insertion site. Skin is cool to the touch and may be bruised or discoloured, and the patient may experience some numbness. Pain, warmth, erythema, a reddish streak, and a palpable venous cord are all symptoms of phlebitis. DIF: Remember REF: 1049 | 1051, Table 40-11 OBJ: Discuss complications associated with intravenous therapy. TOP: Assessment MSC: CPNRE: Foundations of Practice 41. When discontinuing a peripheral IV access, the nurse should do which of the following? a. Use scissors to remove the IV site dressing and tape. b. Apply firm pressure with sterile gauze during removal. c. Wear sterile gloves and a mask. d. Apply pressure to the site for 2 to 3 minutes after removal. ANS: D
The nurse should stop the infusion before removing the IV catheter, so that the fluid does not drip on the patient’s skin; keeNp thRe cI pa. raC llelM U Sathe NGterTB O to the skin while removing it, to reduce trauma to the vein; and apply pressure to the site for 2 to 3 minutes after removal, to decrease bleeding from the site. Scissors should not be used because they may accidentally cut the catheter or tubing or may injure the patient. During removal of the IV catheter, light pressure, not firm pressure, is indicated to prevent trauma. Clean gloves are used for discontinuing a peripheral IV access because gloved hands will handle the external dressing, tubing, and tape, which are not sterile. DIF: Remember REF: 1061 OBJ: Discuss the purpose of, and procedure for, initiating, maintaining, and discontinuing intravenous therapy and peripheral vascular access devices. TOP: Implementation MSC: CPNRE: Foundations of Practice
Canadian Fundamentals of Nursing 6th Edition Potter Test Bank
Chapter 41: Sleep Potter et al: Canadian Fundamentals of Nursing, 6th Edition MULTIPLE CHOICE 1. The physiological processes of sleep are complex. Which of the following statements is the
most appropriate one regarding this process? a. Circadian rhythms occur in a cycle longer than 24 hours. b. Non–rapid eye movement (NREM) refers to the cycle that most people experience when in a high-stimulus environment. c. The reticular activating system is partly responsible for the level of consciousness of a person. d. The bulbar synchronizing region causes the rapid-eye-movement (REM) sleep in most normal adults. ANS: C
The ascending reticular activating system located in the upper brainstem is believed to contain special cells that maintain alertness and wakefulness. Circadian rhythms are cyclical rhythms that are part of everyday life. The most familiar rhythm is the 24-hour day–night cycle known as the diurnal or circadian rhythm. It is not longer than 24 hours. The NREM stage is part of the sleep cycle that most people experience in a low-stimulus environment. The bulbar synchronizing region is the area of the brain where serotonin is released to produce sleep; it is not responsible for REM sleep. DIF: Apply REF: 1073 OBJ: Explain the effect of the 24-hour sleep–wake cycle on biological functions. N R: Fo IN G tions B.C M TOP: Assessment MSC: CPNURES undaT ofO Practice 2. The nurse is alert to patients who may be predisposed to obstructive sleep apnea. This
includes which of the following individuals? a. Patients with heart disease. b. Patients with respiratory infections. c. Patients with nasal polyps. d. Patients who are obese. ANS: C
Structural abnormalities such as nasal polyps, certain jaw configurations, enlarged tonsils, or a deviated septum predispose a patient to obstructive sleep apnea. Individuals with heart disease may have sleep disorders; women with heart disease have the majority of sleep disturbances, but not apnea in particular. Respiratory infections do not predispose a patient to obstructive sleep apnea. Obesity, smoking and alcohol use, and a history of obstructive sleep apnea greatly increase the risk of developing sleep apnea, but they do not necessarily predispose the person to sleep apnea. DIF: Understand REF: 1077 OBJ: Discuss the characteristics of common sleep disorders. MSC: CPNRE: Foundations of Practice
TOP: Assessment
3. When a patient is deprived of sleep, the nurse might assess which of the following symptoms? a. Elevated blood pressure.
Canadian Fundamentals of Nursing 6th Edition Potter Test Bank b. Confusion and irritability. c. Inappropriateness and rapid respirations. d. Decreased temperature and talkativeness. ANS: B
Psychological symptoms of sleep deprivation include confusion and irritability. A decrease in capacity for reasoning and judgement could lead to inappropriate behaviour. Elevated blood pressure, rapid respirations, and decreased temperature are not signs of sleep deprivation. Psychological symptoms of sleep deprivation include confusion and irritability. Many patients with sleep deprivation is withdrawn, not talkative. DIF: Understand REF: 1079, Box 41-7 OBJ: Discuss the characteristics of common sleep disorders. MSC: CPNRE: Foundations of Practice
TOP: Assessment
4. The parents of a newborn wonder when she should start to sleep through the night. The
nurse’s response should be that in infants, a nighttime pattern of sleep usually develops by which age? a. 1 month. b. 2 months. c. 3 months. d. 6 months. ANS: D
Infants do not usually develop a nighttime pattern of sleep by 1 month, 2 months, or 3 months of age. Infants do usually develop a nighttime pattern of sleep before 6 months of age. DIF: Understand REF: 10N 79 R I G B.C M U S N T O OBJ: Compare the sleep requirements of different age groups. MSC: CPNRE: Foundations of Practice
TOP: Planning
5. The mother of a preschooler tells the nurse that the child has started crying and resisting going
to sleep at the scheduled bedtime. Which of the following actions should the nurse advise the parent to take? a. Offer the child a bedtime snack. b. Eliminate one of the naps during the day. c. Allow the child to sleep longer in the mornings. d. Maintain consistency in the same bedtime ritual. ANS: D
It is most important that the parent maintain a consistent bedtime routine. If a bedtime snack is already part of that routine, then this is allowable. If it is not, then the child might use having a snack only as a measure of procrastination. After 3 years of age, the child may give up daytime naps. A bedtime routine used consistently will be more effective in helping the child resist going to sleep during the day. Allowing the child to sleep longer in the mornings will not aid in establishing a consistent sleep pattern. The same regular bedtime and wake-up schedule should be maintained. DIF: Understand REF: 1088 OBJ: Compare the sleep requirements of different age groups. MSC: CPNRE: Foundations of Practice
TOP: Implementation
Canadian Fundamentals of Nursing 6th Edition Potter Test Bank 6. An 11-year-old child in middle school is currently experiencing fatigue during classes in
relation to sleep deprivation. Which of the following responses by the school nurse is the most appropriate one when counselling the child’s parents regarding this assessment? a. “What are the child’s usual sleep patterns?” b. “Establish bedtimes for the child, and withhold his allowance whenever he does not adhere to those bedtimes.” c. “We need to explore other health-related problems, as sleep problems are probably not the cause of the child’s fatigue.” d. “The bulbar synchronizing region of the child’s central nervous system (CNS) is causing these insomniac problems.” ANS: A
A school-age child will be tired the following day if allowed to stay up later than usual. The nurse should first ask a question to assess the child’s usual sleep patterns. The response “Establish bedtimes for the child, and withhold his allowance whenever he does not adhere to those bedtimes” is not appropriate because the nurse is assuming that the boy is not adhering to a bedtime. A sleep problem, not a health-related problem, is often the cause of fatigue. The response about the bulbar synchronizing region is incorrect because the nurse is assuming the child is experiencing insomnia. DIF: Apply REF: 1088 OBJ: Compare the sleep requirements of different age groups. MSC: CPNRE: Foundations of Practice
TOP: Assessment
7. In describing the sleep patterns of older persons, the nurse recognizes that which of the
following statements is true? a. Older persons are more difficult to arouse. N R I G B.C M b. Older persons require moreUsleS ep tN hanT middleO -aged adults. c. Older persons take less time to fall asleep. d. Older persons have a decline in slow-wave or deep sleep. ANS: D
As people age, they do not become more difficult to arouse. Older people do not require more sleep than do middle-aged adults. Older people awaken more often during the night, and it may take more time for an older person to fall asleep. As people age, slow-wave (deep) sleep declines progressively. DIF: Understand REF: 1080 OBJ: Compare the sleep requirements of different age groups. MSC: CPNRE: Foundations of Practice
TOP: Assessment
8. A patient who is currently taking a diuretic should be informed by the nurse that he or she
may experience which of the following effects? a. Nocturia. b. Nightmares. c. Increased daytime sleepiness. d. Reduced REM sleep. ANS: A
Canadian Fundamentals of Nursing 6th Edition Potter Test Bank The nurse should inform the patient who is currently taking a diuretic that he or she might experience nighttime awakening because of nocturia. Diuretic use does not cause nightmares, increase daytime sleepiness, or reduce REM sleep. DIF: Apply REF: 1080 OBJ: Identify factors that normally promote sleep and factors that normally disrupt sleep. TOP: Assessment MSC: CPNRE: Foundations of Practice 9. As a result of studies regarding infant safety during sleep, which of the following instructions
does the nurse give the parents? a. Provide a stuffed toy for comfort. b. Apply a loose-fitting plastic mattress cover. c. Place the infant on his or her back. d. Use small pillows in the crib. ANS: C
To reduce the chance of suffocation, pillows, stuffed toys, or the ends of loose blankets should not be placed in cribs. Loose-fitting plastic mattress covers can cause suffocation. Infants are usually placed on their backs to prevent suffocation or on their sides to prevent aspiration of stomach contents. DIF: Apply REF: 1090 OBJ: Identify nursing interventions designed to promote normal sleep cycles for individuals of all ages. TOP: Implementation MSC: CPNRE: Foundations of Practice 10. A 74-year-old patient reports having sleeping difficulties. To have a better idea of the
patient’s problem, how should the nurse respond? a. “What do you do just befN orU eR goSinIgNtoGbTeB d?.”COM b. “Let’s make sure that your bedroom is completely darkened at night.” c. “Why don’t you try napping more during the daytime?” d. “You should always eat something just before bedtime.” ANS: A
To assess the patient’s sleeping problem, the nurse should inquire about predisposing factors, such as by asking, “What do you do just before going to bed?” Assessment is aimed at understanding the characteristics of any sleep problem and the patient’s usual sleep habits so that ways for promoting sleep can be incorporated into nursing care. Older persons may prefer to sleep in softly lit rooms. Napping more during the daytime is often not the best solution. The patient does not always have to eat something before going to bed. DIF: Apply TOP: Assessment
REF: 1080 OBJ: Conduct a sleep history for a patient. MSC: CPNRE: Foundations of Practice
11. Which of the following information provided by the patient’s bed partner is most associated
with sleep apnea? a. Restlessness. b. Talking during sleep. c. Sleepwalking. d. Excessive snoring. ANS: D
Canadian Fundamentals of Nursing 6th Edition Potter Test Bank Partners of patients with sleep apnea often complain that the patient’s snoring disturbs their sleep. Restlessness is not the symptom most associated with sleep apnea. Sleep talking is associated with sleep–wake transition disorders, not sleep apnea. Sleepwalking is associated with parasomnias (specifically arousal disorders and sleep–wake transition disorders). DIF: Apply REF: 1084, Box 41-10 OBJ: Discuss the characteristics of common sleep disorders. MSC: CPNRE: Foundations of Practice
TOP: Assessment
12. Which of the following instructions does the nurse give the patient in teaching methods to
promote positive sleep habits at home? a. Use the bedroom only for sleep or sexual activity. b. Eat a large meal 1 to 2 hours before bedtime. c. Exercise vigorously before bedtime. d. Stay in bed if sleep does not come after 30 minutes. ANS: A
The nurse should explain that if possible, the bedroom should not be used for intensive studying, snacking, TV watching, or other nonsleep activity, other than sexual activity. The nurse should instruct the patient to avoid heavy meals for 3 hours before bedtime; a light snack may help. The nurse should instruct the patient to try to exercise daily, preferably in the morning or afternoon, and to avoid vigorous exercise in the evening within 2 hours of bedtime. The patient who does not fall asleep within 30 minutes of going to bed should be advised by the nurse to rise and do some quiet activity until he or she feels sleepy enough to return to bed. DIF: Apply REF: 1084 OBJ: Identify nursing intervenN tiU onR sS apI prN opGriT atB e f. orCpO atM ients with sleep alterations. TOP: Implementation MSC: CPNRE: Foundations of Practice 13. The nurse is discussing sleep habits with the patient in the sleep assessment clinic. Of the
following activities performed before sleeping, the nurse informs the patient that which one of the following can potentially interfere with the patient’s sleep? a. Listening to classical music. b. Finishing office work. c. Reading novels. d. Drinking warm milk. ANS: B
Noise should be kept to a minimum. Soft music may be used to mask noise if necessary. At home, a patient should not try to finish office work or resolve family problems before bedtime. Reading a light novel, watching an enjoyable TV program, or listening to music can help a person to relax. Relaxation exercises can be useful at bedtime. A dairy product snack, such as warm milk, contains L-tryptophan and may be helpful in promoting sleep. DIF: Understand REF: 1084 OBJ: Identify nursing interventions appropriate for patients with sleep alterations. TOP: Implementation MSC: CPNRE: Foundations of Practice
Canadian Fundamentals of Nursing 6th Edition Potter Test Bank 14. The patient needs pharmacological treatment to assist with his sleep patterns. The nurse
anticipates that treatment with an anxiety-reducing, relaxation-promoting medication will include the use of which one of the following? a. Barbiturates. b. Amphetamines. c. Benzodiazepines. d. Tricyclic antidepressants. ANS: C
The benzodiazepines cause relaxation, reduce anxiety, and have hypnotic effects by facilitating the action of neurons in the CNS that suppress responsiveness to stimulation, thereby decreasing levels of arousal. Withdrawal from CNS depressants such as barbiturates can cause insomnia and must be managed carefully. Patients can also develop tolerance of and dependence on barbiturates. CNS stimulants, such as amphetamines, should be used sparingly and only under medical management. Amphetamine sulphate may be used to treat narcolepsy; prolonged use may lead to drug dependence. Tricyclic antidepressants can cause insomnia when discontinued and should be managed carefully. They are used primarily to treat depression. DIF: Understand REF: 1092 OBJ: Identify nursing interventions appropriate for patients with sleep alterations. TOP: Planning MSC: CPNRE: Foundations of Practice 15. The nurse is completing an assessment on the patient’s sleep patterns. Which of the following
is a specific question that the nurse should ask to determine the potential presence of sleep apnea? a. “How easily do you fall asleep?” G B.C M N R I b. “Do you have vivid, lifelikU O e drS eamN s?”T c. “Do you ever experience loss of muscle control or falling?” d. “Do you snore loudly or experience headaches?” ANS: D
To assess for sleep apnea, the nurse may ask, “Do you snore loudly?” and “Do you experience headaches after awakening?” A positive response may indicate that the patient experiences sleep apnea. The question “How easily do you fall asleep?” helps assess for the potential presence of insomnia. The questions “Do you have vivid, lifelike dreams?” and “Do you ever experience loss of muscle control or falling?” helps determine the potential presence of narcolepsy. DIF: Understand REF: 1083, Box 41-9 OBJ: Conduct a sleep history for a patient. MSC: CPNRE: Foundations of Practice
TOP: Assessment
16. Which of the following is an effective brief method for assessing sleep quality? a. Sleep history. b. Sleep monitor. c. Visual analogue scale. d. Sleep questionnaire diary. ANS: C
Canadian Fundamentals of Nursing 6th Edition Potter Test Bank One effective brief method for assessing sleep quality is the use of a visual analogue scale. Documenting sleep history and keeping a sleep questionnaire diary are not brief and help assess more than just sleep quality. A sleep monitor is not necessarily used to measure sleep quality, and its use, too, is not brief. DIF: Apply TOP: Assessment
REF: 1083 OBJ: Describe ways to evaluate sleep therapies. MSC: CPNRE: Foundations of Practice
17. Which sleep stage begins the initial period of deep sleep? a. 1. b. 2. c. 3. d. 4. ANS: C
Stage 3 is the stage that begins the initial period of deep sleep. DIF: Understand TOP: Assessment
REF: 1074 OBJ: Describe the stages of a normal sleep cycle. MSC: CPNRE: Foundations of Practice
18. Which stage of sleep accounts for the greatest proportion of total sleep time in adults? a. 1. b. 2. c. 3. d. 4. ANS: B
Stage 2 accounts for the greaN testR proI portG ionB (4.5Cto 5M5%) of total sleep time in adults.
U S N T
DIF: Remember TOP: Assessment
O
REF: 1074 OBJ: Describe the stages of a normal sleep cycle. MSC: CPNRE: Foundations of Practice
19. Which of the following is a determinant of dreams? a. Gender. b. Age. c. Height. d. Weight. ANS: B
Age is a determinant of dreams, along with personality, physiological conditions, psychological conditions, pharmacological therapies, and situational factors. Height, weight, and gender are not identified determinants of dreams. DIF: Understand TOP: Assessment
REF: 1075 OBJ: Compare the characteristics of rest and sleep. MSC: CPNRE: Foundations of Practice
20. Which of the following sleep disorders is a parasomnia disorder of arousal? a. Sleep apnea. b. Narcolepsy. c. Sleepwalking. d. Early-morning awakening.
Canadian Fundamentals of Nursing 6th Edition Potter Test Bank ANS: C
Sleepwalking is an example of a parasomnia disorder of arousal. Sleep apnea is a sleep-related breathing disorder. Narcolepsy is an example of excessive sleepiness. Early-morning awakening is an example of an insomnia sleep disorder. DIF: Understand REF: 1077, Box 41-4 OBJ: Discuss the characteristics of common sleep disorders. MSC: CPNRE: Foundations of Practice
NURSINGTB.COM
TOP: Assessment
Canadian Fundamentals of Nursing 6th Edition Potter Test Bank
Chapter 42: Nutrition Potter et al: Canadian Fundamentals of Nursing, 6th Edition MULTIPLE CHOICE 1. The energy needed to maintain life-sustaining activities for a specific period of time at rest is
known as which of the following? a. Basal metabolic rate. b. Resting energy expenditure. c. Nutrients. d. Nutrient density. ANS: A
The basal metabolic rate is the energy needed to maintain life-sustaining activities for a specific period of time at rest. The resting energy expenditure, or resting metabolic rate, is the amount of energy that an individual needs to consume over a 24-hour period for the body to maintain all of its internal working activities while at rest. Nutrients are the elements necessary for body processes and function. Nutrient density is the proportion of essential nutrients to the number of kilocalories. Foods with high nutrient density provide a large number of nutrients in relation to kilocalories. DIF: Remember REF: 1098 OBJ: Explain the importance of maintaining a balance between energy intake and expenditure. TOP: Assessment MSC: CPNRE: Foundations of Practice 2. In general, when energy requirements are completely met by kilocalorie intake in food, what
NURSINGTB.COM happens? a. Weight increases. b. Weight decreases. c. Weight does not change. d. Kilocalories are not a factor in energy requirements. ANS: C
In general, when energy requirements are completely met by kilocalorie intake in food, weight does not change. When kilocalories ingested exceed a person’s energy demands, the individual gains weight. If kilocalories ingested fail to meet a person’s energy requirement, the individual loses weight; therefore, kilocalories are a factor in energy requirements. DIF: Understand REF: 1099 OBJ: Explain the importance of maintaining a balance between energy intake and expenditure. TOP: Assessment MSC: CPNRE: Foundations of Practice 3. In determining kilocalorie expenditure, the nurse knows that carbohydrates and proteins
provide 4 kcal of energy per gram ingested. The nurse also knows that fats provide how many kilocalories per gram? a. 3. b. 4. c. 6. d. 9.
Canadian Fundamentals of Nursing 6th Edition Potter Test Bank ANS: D
Fats (lipids) are the most calorie-dense nutrient, providing 9 kcal per gram. DIF: Remember REF: 1100 OBJ: List the end products of carbohydrate, protein, and fat metabolism. TOP: Assessment MSC: CPNRE: Foundations of Practice 4. Some proteins are manufactured in the body, but others are not. Those that must be obtained
through diet are known as which of the following? a. Amino acids. b. Dispensable amino acids. c. Triglycerides. d. Essential amino acids. ANS: D
The simplest form of protein is the amino acid. The body does not synthesize essential amino acids, so these must be provided in the diet. The body synthesizes nonessential (dispensable) amino acids. Triglycerides are made up of three fatty acids attached to a glycerol. DIF: Remember REF: 1099 OBJ: List the end products of carbohydrate, protein, and fat metabolism. TOP: Assessment MSC: CPNRE: Foundations of Practice 5. Knowing that protein is required for tissue growth, maintenance, and repair, the nurse must
understand that for optimal tissue healing to occur, the patient must be in which state? a. Negative nitrogen balance. b. Positive nitrogen balance. c. Total dependence on protN ein kca l pro ucCtion URforSI NG TBd. OM. d. Neutral nitrogen balance. ANS: B
When intake of nitrogen is greater than output, the body is in positive nitrogen balance. Positive nitrogen balance is required for growth, normal pregnancy, maintenance of lean muscle mass and vital organs, and wound healing. In negative nitrogen balance, the body loses more nitrogen than it gains. In neutral nitrogen balance, nitrogen gain equals nitrogen loss; this state is not optimal for tissue healing. Protein can provide energy (4 kcal/g), but because of its essential role in growth, maintenance, and repair, the diet should provide adequate kilocalories from nonprotein sources. Protein is spared as an energy source when carbohydrate in the diet is sufficient to meet the energy needs of the body. DIF: Understand REF: 1100 OBJ: Explain the significance of saturated, unsaturated, polyunsaturated and trans fats. TOP: Assessment MSC: CPNRE: Foundations of Practice 6. In providing diet education for a patient on a low-fat diet, it is important for the nurse to
understand which of the following? a. Saturated fats are found mostly in vegetable sources. b. Saturated fats are found mostly in animal sources. c. Unsaturated fats are found mostly in animal sources. d. Linoleic acid is a saturated fatty acid. ANS: B
Canadian Fundamentals of Nursing 6th Edition Potter Test Bank Most animal fats have high proportions of saturated fatty acids, whereas vegetable fats have higher amounts of unsaturated and polyunsaturated fatty acids. Linoleic acid, an unsaturated fatty acid, is the only essential fatty acid in humans. DIF: Understand REF: 1100 OBJ: Explain the significance of saturated, unsaturated, polyunsaturated and trans fats. TOP: Assessment MSC: CPNRE: Foundations of Practice 7. Fats are composed of triglycerides and fatty acids. Which of the following statements is true? a. Triglycerides are made up of three fatty acids. b. Triglycerides can be saturated. c. Triglycerides can be monounsaturated. d. Triglycerides can be polyunsaturated. ANS: A
Triglycerides circulate in the blood and are made up of three fatty acids attached to a glycerol. Fatty acids (not triglycerides) can be saturated or unsaturated (monounsaturated or polyunsaturated). DIF: Remember REF: 1100 OBJ: Explain the significance of saturated, unsaturated, polyunsaturated and trans fats. TOP: Assessment MSC: CPNRE: Foundations of Practice 8. The patient has received a diagnosis of cardiovascular disease and placed on a low-fat diet.
The patient asks the nurse, “How much fat should I have? I guess the less fat, the better.” What does the nurse need to explain? a. Fats have no significance in health and the incidence of disease. b. All fats come from externNaU l sR ouSrI ceN s,GaT ndBs. oC thO eyMcan be easily controlled. c. Deficiencies occur when fat intake falls below 20% of daily total fat intake. d. Vegetable fats are the major source of saturated fats and should be avoided. ANS: C
The acceptable macronutrient distribution range, the range associated with reduced risk of chronic illness while essential intakes of total fat are provided, is 20 to 35% for adults. Various types of fatty acids have significance for health and for the incidence of disease and are referred to in dietary guidelines. Linoleic acid and arachidonic acid, which are important for metabolic processes, are manufactured by the body when linoleic acid is available. Most animal fats have high proportions of saturated fatty acids, whereas vegetable fats have higher amounts of unsaturated and polyunsaturated fatty acids. DIF: Understand REF: 1100 OBJ: Explain the significance of saturated, unsaturated, polyunsaturated and trans fats. TOP: Assessment MSC: CPNRE: Foundations of Practice 9. When inserting a nasoenteric tube, the nurse will rotate the tube how much? a. 45 degrees. b. 90 degrees. c. 180 degrees. d. 360 degrees. ANS: C
The tube is to be rotated 180 degrees while being inserted.
Canadian Fundamentals of Nursing 6th Edition Potter Test Bank
DIF: Apply REF: 1138 OBJ: Describe the procedure for initiating and maintaining tube feedings and avoiding related complications. TOP: Implementation MSC: CPNRE: Foundations of Practice 10. The patient states that she is an lacto-ovo-vegetarian. Which of the following types of food
does an lacto-ovo-vegetarian consume? a. Fish and poultry. b. Only plant foods. c. Milk and fish. d. Eggs and milk. ANS: D
Ovo-lacto-vegetarians avoid meat, fish, and poultry but consume eggs and milk. Vegans consume only plant-based food. DIF: Understand REF: 1112 OBJ: Identify the potential nutritional deficits associated with vegetarian diets, with special consideration to vegan and ovo-lactate diets. TOP: Assessment MSC: CPNRE: Foundations of Practice 11. The nurse is providing nutrition teaching to a Korean patient. In doing so, the nurse must
understand that the focus of the teaching should be on which of the following? a. Changing the patient’s diet to a more conventional Canadian diet. b. Discouraging the patient’s ethnic food choices. c. Food preferences of the patient, including racial and ethnic choices. d. Comparing the patient’s eNthnR ic pI refeG rencBe. sw CithMCanadian dietary choices.
U S N T
O
ANS: C
The nurse needs to make sure to consider the food preferences of patients from different racial and ethnic groups, vegetarians, and others when planning diets. Initiation of a balanced diet is more important than conversion to what may be considered a Canadian diet. Ethnic food choices may be just as nutritious as “Canadian” choices. Foods should be chosen for their nutritive value and should not be compared with the “Canadian” diet. DIF: Understand REF: 1111 OBJ: Describe Eating Well with Canada's Food Guide and its value in planning nutritious meals. TOP: Assessment MSC: CPNRE: Foundations of Practice 12. When teaching a patient about current dietary guidelines for the general population, the nurse
explains referenced daily intakes and daily reference values, otherwise known as daily values. In providing this information, the nurse understands what about daily values? a. They have replaced recommended daily allowances (RDAs). b. They have provided a more understandable calculation of RDAs for the public. c. They are based on percentages of a diet consisting of 1200 kcal/day. d. They are not usually easy to find, and computer experience is required. ANS: B
Canadian Fundamentals of Nursing 6th Edition Potter Test Bank Daily values did not replace RDAs but provided a separate, more understandable calculation for the public. Daily values are based on percentages of a diet consisting of 2000 kcal/day; these values constitute the daily values used on food labels, which are easy for anyone to find. Computer experience is not required. DIF: Understand REF: 1106 OBJ: Specify recommended dietary intake for age and sex groups in Canada to ensure that patients meet the varied essential vitamins, minerals, and nutritional requirements throughout their growth and development. TOP: Assessment MSC: CPNRE: Foundations of Practice 13. The nurse is teaching the patient about dietary guidelines. In discussing the four components
of dietary reference intakes (DRIs), it is important to understand which of the following? a. The estimated average requirement is appropriate for 100% of the population. b. The recommended dietary allowance (RDA) meets the needs of the individual. c. Adequate intake determines the nutrient requirements of the RDA. d. The tolerable upper intake level is not a recommended level of intake. ANS: D
The tolerable upper intake level is the highest level that probably poses no risk of adverse health events. It is not a recommended level of intake. The estimated average requirement is the recommended amount of a nutrient that appears sufficient to maintain a specific body function for 50% of the population based on age and gender. The RDA reflects the average needs of 98% of the population, not the exact needs of the individual. Adequate intake is the intake for individuals suggested on the basis of observed or experimentally determined estimates of nutrient intakes and is used when evidence is insufficient for setting of the RDA. DIF: Understand REF: 1103 OBJ: Specify recommended dN ietU arR yS inI taN keGfoTr B ag. eC anO dM sex groups in Canada to ensure that patients meet the varied essential vitamins, minerals, and nutritional requirements throughout their growth and development. TOP: Assessment MSC: CPNRE: Foundations of Practice 14. In teaching mothers-to-be about infant nutrition, what does the nurse instruct them to do? a. Give cow’s milk during the first year of life. b. Supplement breast milk with corn syrup. c. Add honey to infant formulas for increased energy. d. Remember that breast milk or formula is sufficient for the first 6 months. ANS: D
Breast milk or formula provides sufficient nutrition for the first 6 months of life. Infants should not have regular cow’s milk during the first year of life. Cow’s milk causes gastrointestinal bleeding, is too concentrated for the infant’s kidneys to manage, increases the risk for developing milk product allergies, and is a poor source of iron and vitamins C and E. Honey and corn syrup are potential sources of botulism toxin and should not be used in the infant diet. DIF: Remember REF: 1107 OBJ: Specify recommended dietary intake for age and sex groups in Canada to ensure that patients meet the varied essential vitamins, minerals, and nutritional requirements throughout their growth and development. TOP: Assessment MSC: CPNRE: Foundations of Practice
Canadian Fundamentals of Nursing 6th Edition Potter Test Bank 15. To counter obesity in adolescents, increasing physical activity is often more important than
curbing intake. Sports and regular, moderate to intense exercise necessitate dietary modifications to meet increased energy needs for adolescents. The nurse understands that these modifications include which of the following? a. Decreasing carbohydrates to 25 to 30% of total intake. b. Decreasing protein intake to 0.75 g/kg/day. c. Drinking water before and after exercise. d. Providing vitamin and mineral supplements. ANS: C
Adequate hydration is very important for all athletes. They need to drink water before and after exercise to prevent dehydration, especially in hot, humid environments. Carbohydrates, both simple and complex, are the main source of energy, providing 55 to 60% of total daily kilocalories. Protein needs increase to 1.0 to 1.5 g/kg/day. Vitamin and mineral supplements are not required, but intake of iron-rich foods is necessary to prevent anemia. DIF: Understand REF: 1109 OBJ: Specify recommended dietary intake for age and sex groups in Canada to ensure that patients meet the varied essential vitamins, minerals, and nutritional requirements throughout their growth and development. TOP: Assessment MSC: CPNRE: Foundations of Practice 16. In providing prenatal care to a patient, what does the nurse teach the expectant mother? a. Protein intake needs to decrease to preserve kidney function. b. Calcium intake is especially important in the first trimester. c. Folic acid is needed to help prevent birth defects and anemia. d. The mother should take in as many extra vitamins and minerals as possible. ANS: C
NURSINGTB.COM
Folic acid intake is particularly important for DNA synthesis and growth of red blood cells. Inadequate intake may lead to fetal neural tube defects (such as anencephaly) or maternal megaloblastic anemia. Protein intake throughout pregnancy needs to increase to 60 g daily. Calcium intake is especially critical in the third trimester, when fetal bones are mineralized. Prenatal care usually includes vitamin and mineral supplementation to ensure daily intakes; however, pregnant women should not take additional supplements beyond prescribed amounts. DIF: Understand REF: 1109 OBJ: Specify recommended dietary intake for age and sex groups in Canada to ensure that patients meet the varied essential vitamins, minerals, and nutritional requirements throughout their growth and development. TOP: Assessment MSC: CPNRE: Foundations of Practice 17. The patient is an 80-year-old man who is visiting the clinic today for his routine physical
examination. The patient’s skin turgor is fair, but he has been complaining of fatigue and weakness. The skin is warm and dry, pulse rate is 126 beats per minute, and urinary sodium level is slightly elevated. After assessment, the nurse should recommend that the patient do which of the following? a. Decrease his intake of milk and dairy products to decrease the risk of osteoporosis. b. Drink more grapefruit juice to enhance vitamin C intake and medication absorption. c. Drink more water to prevent further dehydration. d. Eat more meat because meat is the only source of usable protein.
Canadian Fundamentals of Nursing 6th Edition Potter Test Bank
ANS: C
Thirst sensation diminishes with age, which leads to inadequate fluid intake or dehydration. Symptoms of dehydration in older persons include confusion, weakness, hot dry skin, furrowed tongue, and high urinary sodium. Milk continues to be an important food for older women and men, who need adequate calcium to protect against osteoporosis. After age 70, osteoporosis affects men and women equally. Older persons should be cautioned to avoid grapefruit and grapefruit juice because these will decrease absorption of many drugs. Some older persons avoid meats because of cost or because they are difficult to chew. Cream soups and meat-based vegetable soups are nutrient-dense sources of protein. DIF: Analyze REF: 1111 OBJ: Specify recommended dietary intake for age and sex groups in Canada to ensure that patients meet the varied essential vitamins, minerals, and nutritional requirements throughout their growth and development. TOP: Assessment MSC: CPNRE: Foundations of Practice 18. The nurse is assessing a patient for nutritional status. In doing so, what must the nurse do? a. Choose a single objective tool that fits the patient’s condition. b. Combine multiple objective measures with subjective measures. c. Forego the assessment in the presence of chronic disease. d. Use the Mini Nutritional Assessment for pediatric patients. ANS: B
Using a single objective measure is ineffective in predicting risk of nutritional problems. Multiple objective measures should be combined with subjective measures related to nutrition to adequately screen for nutritional problems. Chronic disease and increased metabolic requirements are risk factors for the development of nutritional problems; affected patients may be in critical need of thiN s asR menGt. TB i Nutritional Assessment was developed to Cin Usess SI N T he.M OM use for screening older persons in home care programs, nursing homes, and hospitals. DIF: Apply REF: 1113-1116 OBJ: Discuss the major methods of nutritional assessment. MSC: CPNRE: Foundations of Practice
TOP: Assessment
19. The patient has a calculated body mass index (BMI) of 34. This would the patient be
classified? a. As unclassifiable. b. As being of normal weight. c. As being overweight. d. As being obese. ANS: D
BMI greater than 30 characterizes obesity. BMI between 25 and 30 is classified as overweight. BMI less than 25 is considered normal or underweight. All BMIs can be classified; they are calculated as weight in kilograms by their height in metres squared. DIF: Analyze REF: 1109, Table 42-2 OBJ: Discuss the major methods of nutritional assessment. MSC: CPNRE: Foundations of Practice
TOP: Assessment
Canadian Fundamentals of Nursing 6th Edition Potter Test Bank 20. A patient is seen in the outpatient clinic for follow-up of a nutritional deficiency. In planning
for the patient’s dietary intake, the nurse includes a complete protein. Which one of the following is an example of a complete protein? a. Cheese. b. Oats. c. Legumes. d. Vegetables. ANS: A
A complete protein contains all essential amino acids in sufficient quantity to support growth and maintain nitrogen balance. Cheese, chicken, fish, and soybeans are examples of complete proteins. Incomplete proteins lack a sufficient quantity of one or more of the nine essential amino acids and include cereals, legumes, and vegetables. DIF: Remember REF: 1100 OBJ: List the end products of carbohydrate, protein, and fat metabolism. TOP: Assessment MSC: CPNRE: Foundations of Practice 21. The patient is an older woman and has been given a nursing diagnosis of Imbalanced
nutrition: less than body requirements. What role should the nurse play in her treatment regimen? a. Encourage weight gain as rapidly as possible. b. Encourage large meals three times a day. c. Decrease fluid intake to prevent feeling full. d. Encourage fibre intake. ANS: D
Increasing fibre intake detersNcon nd.eC nha URstip SIatio NGn TaB OMnces appetite. Weight gain should be slow and progressive. The patient should be encouraged to eat frequent small meals should to increase dietary intake and to help offset anorexia. Older persons need eight 8-ounce glasses of fluid per day from beverage and food sources. DIF: Remember REF: 1121-1123 OBJ: Identify three major nutritional problems, the patients who are at risk, and related nutrition therapy. TOP: Assessment MSC: CPNRE: Foundations of Practice 22. In determining the nutritional status of a patient and developing a plan of care, it is important
to evaluate the patient according to what data? a. Published standards. b. Nursing professional standards. c. Absence of family input. d. Patient input only. ANS: A
Published standards are based on scientific findings and are important references to use when a plan of care is developed. Nursing standards cannot be used alone. Other health care providers must be consulted to adopt interventions that reflect the patient’s needs. Family should be involved in evaluation and design of interventions. Although patient input is important, synthesis of patient information from multiple sources is necessary to devising an individualized approach to care that is relevant to the patient’s needs. DIF: Apply
REF: 1098 | 1112
Canadian Fundamentals of Nursing 6th Edition Potter Test Bank OBJ: Formulate a plan of care to help meet the specific nutritional needs of infants, toddlers, preschoolers, school-aged children, adolescents, adults, and older persons. TOP: Implementation MSC: CPNRE: Foundations of Practice 23. In creating a plan of care to meet the nutritional needs of the patient, the nurse needs to
explore the patient’s feelings about weight and food. Why must the nurse do this? a. To determine which category of plan to use. b. To set realistic goals for the patient. c. To mutually plan goals with patient and team. d. To prevent the need for a dietitian consult. ANS: C
Mutually planned goals negotiated by patient, registered dietitian, and nurse ensure success. Individualized planning cannot be overemphasized. Preplanned and categorical care plans are not effective unless they are individualized to meet patient needs. It is important to explore patients’ feelings about weight and food to help them set realistic and achievable goals. The nurse does not set goals for the patient. The plan should reflect the combined effort of patient, nurse, and dietitian, and so a dietitian consult is required. DIF: Apply REF: 1119 OBJ: Formulate a plan of care to help meet the specific nutritional needs of infants, toddlers, preschoolers, school-aged children, adolescents, adults, and older persons. TOP: Implementation MSC: CPNRE: Foundations of Practice 24. The patient is admitted with facial trauma, including a broken nose, and has a history of
esophageal reflux and of aspiration pneumonia. In view of this information, which of the following tubes is appropriate for this patient? a. Nasogastric tube. NURSINGTB.COM b. Percutaneous endoscopic gastrostomy (PEG) tube. c. Nasointestinal tube. d. Jejunostomy tube. ANS: D
Patients with gastroparesis or esophageal reflux or with a history of aspiration pneumonia may require placement of tubes beyond the stomach into the intestine. The nasogastric tube and the PEG tube are placed in the stomach, and placement could lead to aspiration. The nasointestinal tube and the nasogastric tube may be contraindicated by the facial trauma and the broken nose. The jejunostomy tube is the only tube in the list that is placed beyond the stomach and is not contraindicated by facial trauma. DIF: Understand REF: 1135 OBJ: Describe the procedure for initiating and maintaining tube feedings and avoiding related complications. TOP: Assessment MSC: CPNRE: Foundations of Practice 25. The nurse is preparing to insert a nasogastric tube in a patient who is semiconscious. To
determine the needed length of the tube to be inserted, the nurse measures what length? a. From the tip of the nose to the xiphoid process of the sternum. b. From the earlobe to the xiphoid process of the sternum. c. From the tip of the nose to the earlobe. d. From the tip of the nose to the earlobe to the xiphoid process. ANS: D
Canadian Fundamentals of Nursing 6th Edition Potter Test Bank The nurse measures the distance from the tip of the nose to the earlobe to the xiphoid process of the sternum. This approximates the distance from the nose to the stomach in 98% of patients. For duodenal or jejunal placement, an additional 20 to 30 centimetres is required. DIF: Apply REF: 1138, Skill 42-3 OBJ: Describe the procedure for initiating and maintaining tube feedings and avoiding related complications. TOP: Implementation MSC: CPNRE: Foundations of Practice 26. Before the patient is given an intermittent tube feeding, what should the nurse do? a. Make sure that the tube is secured to the gown with a safety pin. b. Have the tube feeding at room temperature. c. Inject air into the stomach via the tube and auscultate. d. Place the patient in a supine position. ANS: B
Cold formula causes gastric cramping and discomfort because the mouth and the esophagus cannot warm the liquid. Safety pins should not be used; they can become unfastened and may cause harm to the patient. Auscultation is no longer considered a reliable method for verification of tube placement because a tube inadvertently placed in the lungs, pharynx, or esophagus transmits sound similar to that of air entering the stomach. The patient should be placed in high-Fowler’s position, or the head of the bed should be elevated at least 30 degrees, to help prevent aspiration. DIF: Apply REF: 1142, Skill 42-4 OBJ: Describe the procedure for initiating and maintaining tube feedings and avoiding related complications. TOP: Implementation MSC: CPNRE: Foundations ofNPU raR ctice SINGTB.COM 27. At present, what is the most reliable method for verification of placement of small-bore
feeding tubes? a. Auscultation. b. Aspiration of contents. c. Radiography. d. pH testing. ANS: C
At present, the most reliable method for verification of placement of small-bore feeding tubes is x-ray examination. Aspiration of contents and pH testing are not infallible. The nurse would need a more precise indicator to help differentiate the source of tube feeding aspirate. Auscultation is no longer considered a reliable method for verification of tube placement because a tube inadvertently placed in the lungs, pharynx, or esophagus transmits sound similar to that of air entering the stomach. DIF: Understand REF: 1148 OBJ: Describe the procedure for initiating and maintaining tube feedings and avoiding related complications. TOP: Assessment MSC: CPNRE: Foundations of Practice 28. The nurse is concerned about pulmonary aspiration when providing her patient with tube
feedings. What should the nurse do? a. Verify tube placement before feeding.
Canadian Fundamentals of Nursing 6th Edition Potter Test Bank b. Lower the head of the bed to a supine position. c. Add blue food colouring to the enteral formula. d. Run the formula over 12 hours to decrease volume. ANS: A
A major cause of pulmonary aspiration is regurgitation of formula. The nurse needs to verify tube placement and elevate the head of the bed 30 to 45 degrees during feedings and for 2 hours afterward. Blue food colouring is no longer added to enteral formula to assist with detection of aspirate. The formula should not hang longer than 4 to 8 hours. After that length of time, formula becomes a medium for bacterial growth after that length of time. DIF: Apply REF: 1150-1155 OBJ: Describe the procedure for initiating and maintaining tube feedings and avoiding related complications. TOP: Implementation MSC: CPNRE: Foundations of Practice 29. The patient is to receive multiple medications via the nasogastric tube. The nurse is concerned
that the tube may become clogged. What can the nurse do to prevent this? a. Irrigate the tube with 60 mL of water after all medications are given. b. Check with the pharmacy to find out if liquid forms of the medications are available. c. Instill nonliquid medications without diluting. d. Mix all medications together to decrease the number of administrations. ANS: B
Crushed medication should be avoided if liquid formulation is available. Each tube is irrigated with 30 mL of water before and after each medication. Crushed medications, if used, should be diluted. The nurse should N read arm lo. giC calOM information on compatibility of drugs and Rph I Gaco B U S N T formula before mixing medications. DIF: Apply REF: 1141, Skill 42-3 OBJ: Describe the procedure for initiating and maintaining tube feedings and avoiding related complications. TOP: Implementation MSC: CPNRE: Foundations of Practice 30. The patient has just started on enteral feedings but is complaining of abdominal cramping.
What should the nurse do? a. Slow the rate of tube feeding. b. Instill cold formula to “numb” the stomach. c. Place the patient in a supine position. d. Change the tube feeding to a high-fat formula. ANS: A
One possible cause of abdominal cramping is a rapid increase in rate or volume. Lowering the rate of delivery may increase tolerance. Another possible cause of abdominal cramping is use of cold formula. The nurse should warm the formula to room temperature. The nurse should maintain the head of the bed at least 30 degrees. High-fat formulas are also a cause of abdominal cramping. DIF: Apply REF: 1152, Table 42-9 OBJ: Describe the procedure for initiating and maintaining tube feedings and avoiding related complications. TOP: Implementation
Canadian Fundamentals of Nursing 6th Edition Potter Test Bank MSC: CPNRE: Foundations of Practice 31. The patient has just started on an enteral feeding and has developed diarrhea after being on the
feeding for 2 hours. Which is the most likely cause of the diarrhea? a. Clostridium difficile infection. b. Antibiotic therapy. c. Formula intolerance. d. Bacterial contamination. ANS: C
Hyperosmolar formulas can cause diarrhea. If that is the case, the solution is to lower the rate, dilute the formula, or change to an isotonic formula. Antibiotics destroy normal intestinal flora and disturb the internal ecology, allowing for C. difficile toxin buildup. However, this takes time, and the description does not suggest that this patient was taking antibiotics. Proximity to the start of the enteral feedings is more suspect. Bacterial contamination of the feeding usually occurs when feedings are left hanging for longer than 8 hours. DIF: Apply REF: 1151, Table 42-9 OBJ: Describe the procedure for initiating and maintaining tube feedings and avoiding related complications. TOP: Assessment MSC: CPNRE: Foundations of Practice 32. In providing diabetic teaching for a patient with type 1 diabetes mellitus, the nurse gives the
patient which instruction? a. Insulin is the only consideration that must be taken into account. b. Saturated fat should be limited to less than 30% of total calories. c. Cholesterol intake should be greater than 200 mg/day. d. Nonnutritive sweeteners can be used without restriction. ANS: B
NURSINGTB.COM
The patient with diabetes should limit daily fat to less than 30% of total calories and cholesterol intake to less than 200 mg/day. Patients with type 1 diabetes require both insulin and dietary restrictions for optimal control. Nonnutritive sweeteners can be eaten as long as the recommended daily intake levels are followed. DIF: Apply REF: 1127, Box 42-9 OBJ: Identify three major nutritional problems, the patients who are at risk, and related nutrition therapy. TOP: Implementation MSC: CPNRE: Foundations of Practice 33. The parent of an 8-year-old asks the nurse about any special nutritional needs for children in
this age group. The nurse mentions that children in this age group need to do which of the following? a. Increase their intake of B vitamins. b. Significantly increase iron intake. c. Maintain sufficient intake of protein and vitamins A and C. d. Increase carbohydrates to meet increased energy needs. ANS: C
Canadian Fundamentals of Nursing 6th Edition Potter Test Bank School-aged children’s diets should be carefully assessed for adequate protein and vitamins A and C. School-aged children frequently fail to eat a proper breakfast, and their intake at school is unsupervised. An increase in B-complex vitamins is needed to support heightened metabolic activity of the adolescent, but not the school-aged child. The pregnant woman needs to increase iron intake significantly, but the school-aged child does not. Increased energy needs are expected in the adolescent period, not in the school-aged group; therefore, an 8-year-old child does not need to increase carbohydrates to meet increased energy needs. DIF: Analyze REF: 1108 OBJ: Specify recommended dietary intake for age and sex groups in Canada to ensure that patients meet the varied essential vitamins, minerals, and nutritional requirements throughout their growth and development. TOP: Assessment MSC: CPNRE: Foundations of Practice 34. The nurse is providing home care for a patient who has acquired immune deficiency
syndrome (AIDS). In preparing meals for this patient, what should the nurse do? a. Provide small, frequent nutrient-dense meals. b. Encourage intake of fatty foods to increase caloric intake. c. Prepare hot meals because they are more easily tolerated. d. Avoid salty foods and limit liquids to preserve electrolytes. ANS: A
Small, frequent, nutrient-dense meals in which fatty foods and overly sweet foods are limited are easier to tolerate. Patients benefit from eating cold foods and drier or saltier foods with fluid in between. DIF: Apply REF: 1128 OBJ: Identify three major nutritional problems, the patients who are at risk, and related nutrition therapy. TOP: ImNplem tatio URen SI NGn TB.COM MSC: CPNRE: Foundations of Practice 35. To provide successful nutritional therapies to patients, what must the nurse understand? a. Patients will have to change diet preferences drastically to be successful. b. The patient will tell the nurse when to change the plan of care. c. Expectations of nurses frequently differ from those of the patient. d. Nurses should never alter the plan of care regardless of outcome. ANS: C
Expectations and health care values held by nurses frequently differ from those held by patients. Successful interventions and outcomes depend on recognition of this fact, in addition to nursing knowledge and skill. If ongoing nutritional therapies are not resulting in successful outcomes, patients expect nurses to recognize this fact and alter the plan of care accordingly. Working closely with patients enables the nurse to redefine expectations that are realistically met within the limits of conditions and treatments and to identify their dietary preferences and cultural beliefs. DIF: Understand REF: 1129 OBJ: Discuss diet counselling and patient teaching in relation to patient expectations. TOP: Implementation MSC: CPNRE: Foundations of Practice 36. In measuring the effectiveness of nutritional interventions, what must the nurse remember? a. To expect results to occur rapidly.
Canadian Fundamentals of Nursing 6th Edition Potter Test Bank b. Not to be concerned with physical measures such as weight. c. To expect to maintain a course of action regardless of changes in condition. d. To evaluate outcomes according to the patient’s expectations and goals. ANS: D
The nurse should measure the effectiveness of nutritional interventions by evaluating the patient’s expected outcomes and goals of care. Nutrition therapy does not always produce rapid results. Ongoing comparisons need to be made with baseline measures of weight, serum albumin or prealbumin, and protein and kilocalorie intake. Changes in the patient’s condition may indicate a need to change the nutritional plan of care. DIF: Apply REF: 1129 OBJ: Discuss diet counselling and patient teaching in relation to patient expectations. TOP: Implementation MSC: CPNRE: Foundations of Practice 37. What action should the nurse take when expected nutritional outcomes are not being met? a. Revise the nurse measures or expected outcomes. b. Alter the outcomes on the basis of nursing standards. c. Ensure that patient expectations are congruent with the nurse’s expectations. d. Readjust the plan to exclude cultural beliefs. ANS: A
When expected outcomes are not met, the nurse should revise the nursing measures or expected outcomes according to the patient’s needs or preferences, not solely on the basis of nursing standards. Expectations and health care values held by nurses frequently differ from those held by patients. Working closely with patients enables the nurse to redefine expectations that are realistically met within the limits of conditions and treatments and to identify their dietary preferenNcU esRaS ndIcN uG ltuTraBl . beCliO efM s. DIF: Apply REF: 1129 OBJ: Discuss diet counselling and patient teaching in relation to patient expectations. TOP: Implementation MSC: CPNRE: Foundations of Practice 38. The patient is asking the nurse about the best way to stay healthy. The nurse will provide
further instruction if the patient states that he or she will do which of the following? a. Maintain body weight in a healthy range. b. Increase physical activity. c. Increase intake of meat and other high-protein foods. d. Choose and prepare foods with little salt. ANS: C
Canada's Food Guide notes the importance of physical activity in maintaining energy balance and recommends that adults spend 30 to 60 minutes per day carrying out some physical activity and children and youth spend at least 90 minutes per day. The guide provides advice on the use of vitamin and mineral supplementation when the recommended food intake pattern does not ensure adequate amounts and to maintain a healthy body weight. Although recommendations for salt, alcohol, and caffeine are not specified in the guide, Canadians are advised by nutrition experts to limit salt to “healthy” levels, alcohol to no more than 5% of total energy, and caffeine to no more than 400 mg for the general population. DIF: Apply REF: 1103-1106 OBJ: Describe Eating Well with Canada's Food Guide and its value in planning nutritious meals.
Canadian Fundamentals of Nursing 6th Edition Potter Test Bank TOP: Implementation
MSC: CPNRE: Foundations of Practice
NURSINGTB.COM
Canadian Fundamentals of Nursing 6th Edition Potter Test Bank
Chapter 43: Urinary Elimination Potter et al: Canadian Fundamentals of Nursing, 6th Edition MULTIPLE CHOICE 1. If obstructed, which component of the urination system would cause peristaltic waves? a. Kidneys. b. Ureters. c. Bladder. d. Urethra. ANS: B
Ureters drain urine from the kidneys into the bladder; if they become obstructed, peristaltic waves attempt to push the obstruction into the bladder. The kidneys, bladder, and urethra do not produce peristaltic waves. Obstruction of both bladder and urethra typically does not occur. DIF: Remember TOP: Evaluate
REF: 1167 OBJ: Describe the process of urination. MSC: CPNRE: Foundations of Practice
2. When reviewing laboratory results, the nurse should immediately notify the health care
provider about which finding? a. Glomerular filtration rate of 20 mL/min. b. Urine output of 280 mL/hr. c. pH of 6.4. d. Protein level of 2 mg/100 mL. I G B.C M ANS: A
N R U S N T
O
Normal glomerular filtration rate should be approximately 125 mL/min; a severe decrease in renal perfusion could indicate a life-threatening problem such as shock or dehydration. Normal urine output is 1000 to 2000 mL/day; an output of 30 mL/hr or less for 2 or more hours would be cause for concern. The normal pH of urine is between 4.6 and 8.0. Protein levels up to 8 mg/100 mL are acceptable; however, values in excess of this could indicate renal disease. DIF: Apply REF: 1172 OBJ: Describe the nursing implications of common diagnostic tests of the urinary system. TOP: Implementation MSC: CPNRE: Foundations of Practice 3. A patient is experiencing oliguria. Which action should the nurse perform first? a. Increase the patient’s intravenous fluid rate. b. Encourage the patient to drink caffeinated beverages. c. Assess for bladder distension. d. Request an order for diuretics. ANS: C
Canadian Fundamentals of Nursing 6th Edition Potter Test Bank The nurse first should gather all assessment data to determine the potential cause of oliguria. It could be that the patient does not have adequate intake, or it could be that the bladder sphincter is not functioning and the patient is retaining water. Increasing fluids is effective if the patient does not have adequate intake, or if dehydration occurs. Caffeine can work as a diuretic but is not helpful if an underlying pathological process is present. An order for diuretics can be obtained if the patient was retaining water, but this should not be the first action. DIF: Analyze REF: 1174 | 1175 OBJ: Describe characteristics of normal and abnormal urine. MSC: CPNRE: Foundations of Practice
TOP: Assessment
4. A patient requests the nurse’s assistance to the bedside commode and becomes frustrated
when unable to void in front of the nurse. The nurse understands that the patient is unable to void for which reason? a. Anxiety can make it difficult for abdominal and perineal muscles to relax enough to void. b. The patient does not recognize the physiological signals that indicate a need to void. c. The patient is lonely, and calling the nurse in under false pretenses is a way to get attention. d. The patient is not drinking enough fluids to produce adequate urine output. ANS: A
Attempting to void in the presence of another person can cause anxiety and tension in the muscles, which makes voiding difficult. The nurse should give the patient privacy and adequate time if appropriate. No evidence suggests that an underlying physiological or N R I G B.C M O psychological condition exists.U S N T DIF: Understand REF: 1168, Box 43-1 OBJ: Identify factors that commonly influence urinary elimination. TOP: Implementation MSC: CPNRE: Foundations of Practice 5. The nurse knows that urinary tract infection (UTI) is the most common health care–associated
infection for which reason? a. Catheterization procedures are performed more frequently than indicated. b. Escherichia coli pathogens are transmitted during surgical or catheterization procedures. c. Perineal care is often neglected by nursing staff. d. Bedpans and urinals are not stored properly and transmit infection. ANS: B
E. coli is the leading pathogenic cause of UTIs; this pathogen enters during invasive procedures. Sterile technique is imperative to prevent the spread of infection. Frequent catheterizations can place a patient at high risk for UTI; however, infection is caused by bacteria, not by the procedure itself. Perineal care is important, and buildup of bacteria can lead to infection, but this is not the primary cause. Bedpans and urinals may become bacteria ridden and should be cleaned frequently; however, bedpans and urinals are not inserted into the urinary tract, so they are unlikely to be the primary cause of UTI. DIF: Understand
REF: 1169
Canadian Fundamentals of Nursing 6th Edition Potter Test Bank OBJ: Compare common alterations in urinary elimination. MSC: CPNRE: Foundations of Practice
TOP: Implementation
6. An 86-year-old patient tells the nurse that she is experiencing uncontrollable leakage of urine.
Which nursing diagnosis should the nurse include in the patient’s plan of care? a. Urinary retention. b. Hesitancy. c. Urgency. d. Urinary incontinence. ANS: D
Age-related changes such as loss of pelvic muscle tone can cause involuntary loss of urine, known as urinary incontinence. Urinary retention is the inability to empty the bladder. Hesitancy occurs as difficulty initiating urination. Urgency is the feeling of the need to void immediately. DIF: Apply REF: 1169 OBJ: Identify nursing diagnoses appropriate for patients with alterations in urinary elimination. TOP: Planning MSC: CPNRE: Foundations of Practice 7. A patient has fallen several times in the past week when attempting to get to the bathroom.
The patient informs the nurse that he gets up three or four times a night to urinate. Which recommendation by the nurse is most appropriate in correcting this urinary problem? a. Clear the path to the bathroom of all obstacles before bed. b. Leave the bathroom light on to illuminate a pathway. c. Limit fluid and caffeine intake before bed. d. Practise Kegel exercises to strengthen bladder muscles. ANS: C
NURSINGTB.COM
Reducing fluids, especially caffeine and alcohol, before bedtime can reduce nocturia. Clearing a path to the restroom or illuminating the path, or shortening the distance to the restroom, may reduce falls but will not correct the urination problem. Kegel exercises are useful if a patient is experiencing incontinence. DIF: Apply REF: 1169 OBJ: Discuss nursing measures to promote normal micturition and to reduce episodes of incontinence. TOP: Implementation MSC: CPNRE: Foundations of Practice 8. When caring for a patient with urinary retention, the nurse would anticipate an order for which
of the following? a. Limited fluid intake. b. A urinary catheter. c. Diuretic medication. d. Renal angiography. ANS: B
A urinary catheter would relieve urinary retention. Reducing fluids would reduce the amount of urine produced but would not alleviate the urine retention. Diuretic medication would increase urine production and may worsen the discomfort caused by urine retention. Renal angiography is not an appropriate diagnostic test for urinary retention.
Canadian Fundamentals of Nursing 6th Edition Potter Test Bank DIF: Apply REF: 1169, Table 43-1 OBJ: Discuss nursing measures to promote normal micturition and to reduce episodes of incontinence. TOP: Implementation MSC: CPNRE: Foundations of Practice 9. Upon palpation, the nurse notices that the bladder is firm and distended; the patient expresses
an urge to urinate. The nurse should follow up by asking which question? a. “When was the last time you voided?” b. “Do you lose urine when you cough or sneeze?” c. “Have you noticed any change in your urination patterns?” d. “Do you have a fever or chills?” ANS: A
To obtain an accurate assessment, the nurse should first determine the source of the discomfort. Urinary retention causes the bladder to be firm and distended. Further assessment to determine the pathological process of the condition can be performed later. Questions concerning fever and chills, changing urination patterns, and losing urine during coughing or sneezing focus on specific pathological conditions. DIF: Apply REF: 1171 | 1181 OBJ: Obtain a nursing history for a patient with urinary elimination problems. TOP: Implementation MSC: CPNRE: Foundations of Practice 10. Which of the following is the primary function of the kidneys? a. Metabolizing and excreting medications. b. Maintaining fluid and electrolyte balance. c. Storing and excreting urine. d. Filtering blood cells and N prU otR eiS nsI . NGTB.COM ANS: B
The main purpose of the kidney is to maintain fluid and electrolyte balance by filtering waste products and regulating pressures. The kidneys filter the by-products of medication metabolism. The bladder stores and excretes urine. The kidneys help to maintain red blood cell volume by producing erythropoietin. DIF: Understand TOP: Assessment
REF: 1166 OBJ: Describe the process of urination. MSC: CPNRE: Foundations of Practice
11. While receiving a shift report on a patient, the nurse is informed that the patient has urinary
incontinence. Upon assessment, what would the nurse expect to find? a. An in-dwelling Foley catheter. b. Reddened irritated skin on the buttocks. c. Tiny blood clots in the patient’s urine. d. Foul-smelling discharge indicative of a UTI. ANS: B
Urinary incontinence is uncontrolled urinary elimination; if the urine has prolonged contact with the skin, skin breakdown can occur. An indwelling Foley catheter is a solution for urine retention. Blood clots and foul-smelling discharge are often signs of infection. DIF: Apply REF: 1169 | 1171 OBJ: Compare common alterations in urinary elimination.
TOP: Assessment
Canadian Fundamentals of Nursing 6th Edition Potter Test Bank MSC: CPNRE: Foundations of Practice 12. Which nursing diagnosis related to alterations in urinary function in an older person should be
a nurse’s first priority for action? a. Self-care deficit related to decreased mobility. b. Risk of infection. c. Anxiety related to urinary frequency. d. Impaired self-esteem related to lack of independence. ANS: B
In many older persons, muscle tone is poor, which leads to an inability of the bladder to empty completely. Residual urine greatly increases the risk of infection. According to Maslow’s hierarchy of needs, physical health risks should be addressed before emotional/cognitive risks such as anxiety and self-esteem. Decreased mobility can lead to self-care deficit; the nurse’s priority concern for this diagnosis would be infection because this patient must rely on others for basic hygiene. DIF: Apply REF: 1173 OBJ: Identify nursing diagnoses appropriate for patients with alterations in urinary elimination. TOP: Planning MSC: CPNRE: Foundations of Practice 13. A patient asks about treatment for urge urinary incontinence. The nurse’s best response is
which advice to the patient? a. Perform pelvic floor exercises. b. Drink cranberry juice. c. Avoid voiding frequently. d. Wear an adult diaper. ANS: A
NURSINGTB.COM
Poor muscle tone leads to an inability to control urine flow. The nurse should recommend pelvic muscle strengthening exercises such as Kegel exercises; this solution best addresses the patient’s problem. Drinking cranberry juice is a preventive measure for urinary tract infection. The nurse should not encourage the patient to reduce voiding; residual urine in the bladder increases the risk of infection. Wearing an adult diaper could be considered if attempts to correct the root of the problem fail. DIF: Analyze REF: 1183 OBJ: Identify nursing diagnoses appropriate for patients with alterations in urinary elimination. TOP: Planning MSC: CPNRE: Foundations of Practice 14. The nurse suspects that a urinary tract infection has progressed to cystitis when the patient
complains of which symptom? a. Dysuria. b. Flank pain. c. Frequency. d. Fever and chills. ANS: C
Cystitis is inflammation of the bladder; associated symptoms include hematuria and urgency/frequency. Dysuria is a common symptom of a lower urinary tract infection. Flank pain, fever, and chills are all signs of pyelonephritis.
Canadian Fundamentals of Nursing 6th Edition Potter Test Bank DIF: Understand REF: 1168 | 1169 OBJ: Compare common alterations in urinary elimination. MSC: CPNRE: Foundations of Practice
TOP: Assessment
15. Which assessment question should the nurse ask if stress incontinence is suspected? a. “Does your bladder feel distended?” b. “Do you empty your bladder completely when you void?” c. “Do you experience urine leakage when you cough or sneeze?” d. “Do your symptoms increase with consumption of alcohol or caffeine?” ANS: C
Stress incontinence can be related to intra-abdominal pressure that causes urine leakage, as would happen during coughing or sneezing. The patient’s response about the fullness of the bladder would rule out retention and overflow. An inability to void completely can refer to urge incontinence. Physiological causes and medications can affect elimination, but this is not related to stress incontinence. DIF: Apply REF: 1170, Table 43-2 OBJ: Compare common alterations in urinary elimination. MSC: CPNRE: Foundations of Practice
TOP: Planning
16. To obtain a clean-voided urine specimen from a female patient, the nurse should teach the
patient to do which of the following? a. Cleanse the urethral meatus from the area of most contamination to least. b. Initiate the first part of the urine stream directly into the collection cup. c. Hold the labia apart while voiding into the specimen cup. d. Drink fluids 5 minutes before collecting the urine specimen. ANS: C
NURSINGTB.COM
The patient should hold the labia apart to reduce bacterial levels in the specimen. The urethral meatus should be cleansed from the area of least contamination to greatest contamination (front-to-back). The initial stream flushes out microorganisms in the urethra and prevents bacterial transmission in the specimen. Patients should drink fluids 30 to 60 minutes before giving a specimen. DIF: Understand TOP: Planning
REF: 1178-1180, Skill 43-1 MSC: CPNRE: Foundations of Practice
OBJ: Obtain urine specimens.
17. When viewing a urine specimen under a microscope, what would the nurse expect to see if the
specimen is from a patient with a urinary tract infection? a. Bacteria. b. Casts. c. Crystals. d. Protein. ANS: A
Bacteria indicate a urinary tract infection. Crystals would be seen with renal stone formation. Casts indicate renal alterations. Protein is not visible under a microscope, but its presence indicates renal disease. DIF: Remember REF: 1180 | 1181, Table 43-5 OBJ: Describe characteristics of normal and abnormal urine.
TOP: Assessment
Canadian Fundamentals of Nursing 6th Edition Potter Test Bank MSC: CPNRE: Foundations of Practice 18. The nurse would expect the urine of a patient with uncontrolled diabetes mellitus to be which
of the following? a. Cloudy. b. Discoloured. c. Sweet smelling. d. Painful. ANS: C
Incomplete fat metabolism and buildup of ketones give urine a sweet or fruity odour. Cloudy urine may indicate infection or renal failure. Discolouration of urine may result from various medications. Painful urination indicates an alteration in urinary elimination. DIF: Understand REF: 1177 OBJ: Describe characteristics of normal and abnormal urine. MSC: CPNRE: Foundations of Practice
TOP: Assessment
19. What signs and symptoms would the nurse expect to observe in a patient with excessive
numbers of white blood cells present in the urine? a. Fever and chills. b. Difficulty with urinary elimination. c. Increased blood pressure. d. Abnormal blood sugar. ANS: A
The presence of white blood cells in urine indicates a urinary tract infection. Difficulty with urinary elimination indicatesN bloR Uckag SIeNoGr ren TBa.l dCam OMage. Increase in blood pressure is associated with renal disease or damage and some medications. Blood glucose levels would be abnormal in someone with ketones in the urine, as this finding indicates diabetes. DIF: Understand REF: 1204 OBJ: Describe characteristics of normal and abnormal urine. MSC: CPNRE: Foundations of Practice
TOP: Assessment
20. For a patient who has severe flank pain and calcium phosphate crystals revealed on urinalysis,
the nurse would anticipate an order for which diagnostic test? a. Renal ultrasonography. b. Bladder scan. c. Kidney, ureter, and bladder (KUB) radiography. d. Intravenous pyelography. ANS: D
Flank pain and calcium phosphate crystals are associated with renal calculi. Intravenous pyelography allows the health care provider to observe pathological problems such as obstruction of the ureter. Renal ultrasonography is performed to identify gross structures. A bladder scan helps measure the amount of urine in the bladder. A KUB radiograph shows size, shape, symmetry, and location of the kidneys. DIF: Understand REF: 1161 | 1182, Table 43-6 OBJ: Describe the nursing implications of common diagnostic tests of the urinary system. TOP: Planning MSC: CPNRE: Foundations of Practice
Canadian Fundamentals of Nursing 6th Edition Potter Test Bank
21. A nurse is caring for a patient who just underwent intravenous pyelography that revealed a
renal calculus obstructing the left ureter. What is the nurse’s first priority in caring for this patient? a. Turn the patient onto the right side to alleviate pressure on the left kidney. b. Encourage the patient to increase fluid intake to flush the obstruction. c. Administer narcotic medications to alleviate pain. d. Monitor the patient for fever, rash, and difficulty breathing. ANS: D
To perform intravenous pyelography, iodine-based dye is administered to reveal functionality of the urinary system. Many individuals are allergic to shellfish; therefore, the first nursing priority is to assess the patient for an allergic reaction that could be life-threatening. The nurse should then encourage the patient to drink fluids to flush dye used in the procedure. Narcotics can be administered but are not the first priority. Turning the patient on the side will not affect patient safety. DIF: Analyze REF: 1182, Table 43-6 OBJ: Describe the nursing implications of common diagnostic tests of the urinary system. TOP: Planning MSC: CPNRE: Foundations of Practice 22. Which statement by the patient about upcoming computed tomography (CT) indicates a need
for further teaching? a. “I’m allergic to shrimp, so I should monitor myself for an allergic reaction.” b. “I will complete my bowel prep program the night before the scan.” c. “I will be anaesthetized so that I lie perfectly still during the procedure.” d. “I will ask the technician to play music to ease my anxiety.” ANS: C
NURSINGTB.COM
Patients are not anaesthetized for a CT scan; instead, the nurse should educate patients about the need to lie perfectly still and about possible methods of overcoming feelings of claustrophobia. The other options are correct. Patients need to be assessed for an allergy to shellfish if receiving contrast material for the CT scan. Bowel cleansing is often performed before CT scanning. Listening to music will help the patient relax and remain still during the examination. DIF: Apply REF: 1182, Table 43-6 OBJ: Describe the nursing implications of common diagnostic tests of the urinary system. TOP: Planning MSC: CPNRE: Foundations of Practice 23. The nurse is visiting the patient who has a nursing diagnosis of Impaired urinary elimination:
retention. On assessment, the nurse anticipates that this patient will exhibit which of the following? a. Severe flank pain and hematuria. b. Pain and burning on urination. c. Loss of the urge to void. d. Sensation of urgency and voiding of small amounts. ANS: D
Canadian Fundamentals of Nursing 6th Edition Potter Test Bank With urinary retention, urine continues to collect in the bladder, stretching its walls and causing feelings of pressure, discomfort, tenderness over the symphysis pubis, restlessness, and diaphoresis. The sphincter temporarily opens to allow a small volume of urine (<75 mL) to escape, with no real relief of discomfort. Severe flank pain and hematuria are data indicating an upper UTI (pyelonephritis). Pain and burning on urination are symptoms of a lower UTI (such as a bladder infection). Data indicating reflex incontinence would include a loss of the urge to void. DIF: Apply REF: 1171 OBJ: Describe the nursing implications of common diagnostic tests of the urinary system. TOP: Planning MSC: CPNRE: Foundations of Practice 24. A nurse anticipates urodynamic testing for a patient with which symptom? a. Involuntary urine leakage. b. Severe flank pain. c. Presence of blood in urine. d. Dysuria. ANS: A
Urodynamic testing evaluates the muscle function of the bladder and is used to look for the cause of urinary incontinence. Severe flank pain indicates renal calculi; CT scan or intravenous pyelography would be a more efficient diagnostic test. Blood indicates trauma to the urethral or bladder mucosa. Pain on elimination may warrant cultures to check for infection. DIF: Understand REF: 1183, Table 43-6 OBJ: Describe the nursing implications of common diagnostic tests of the urinary system. NNURRES TOP: Planning MSC: CP : FI oN unG daT tiB on. sC ofO PM ractice 25. A patient is having difficulty voiding in a bedpan but states that she feels her bladder is full.
To stimulation micturition, which nursing intervention should the nurse try first? a. Exiting the room and informing the patient that the nurse will return in 30 minutes to check on the patient’s progress. b. Utilizing the power of suggestion by turning on the faucet and letting the water run. c. Obtaining an order for a Foley catheter. d. Administering diuretic medication. ANS: B
To stimulate micturition, the nurse should attempt noninvasive procedures first. Running warm water or stroking the inner aspect of the upper thigh promotes sensory perception that leads to urination. A patient should not be left alone on a bedpan for 30 minutes because this could cause skin breakdown. Catheterization places the patient at increased risk of infection and should not be the first intervention attempted. Diuretics are useful if the patient is not producing urine, but they do not stimulate micturition. DIF: Apply REF: 1186 OBJ: Discuss nursing measures to promote normal micturition and to reduce episodes of incontinence. TOP: Planning MSC: CPNRE: Foundations of Practice
Canadian Fundamentals of Nursing 6th Edition Potter Test Bank 26. A nurse is caring for an 8-year-old patient who is embarrassed about urinating in his bed at
night. Which intervention should the nurse suggest to reduce the frequency of this occurrence? a. “Drink your nightly glass of milk earlier in the evening.” b. “Set your alarm clock to wake you every 2 hours, so you can get up to void.” c. “Line your bedding with plastic sheets to protect your mattress.” d. “Empty your bladder completely before going to bed.” ANS: A
Nightly incontinence and nocturia are often resolved by limiting of fluid intake 2 hours before bedtime. Setting the alarm clock to wake does not correct the physiological problem, nor does lining the bedding with plastic sheets. Emptying the bladder may help with early nighttime urination, but will not affect urine produced throughout the night from late-night fluid intake. DIF: Apply REF: 1186 OBJ: Discuss nursing measures to promote normal micturition and to reduce episodes of incontinence. TOP: Implementation MSC: CPNRE: Foundations of Practice 27. Many individuals have difficulty voiding in a bedpan or urinal while lying in bed for which
reason? a. They are embarrassed that they will urinate on the bedding. b. They would feel more comfortable assuming a normal voiding position. c. They feel they are losing their independence by asking the nursing staff to help. d. They are worried about acquiring a urinary tract infection. ANS: B
Assuming a normal voiding pNosi tion relax and be able to void; lying in bed is R Ihel GpsTpBat.ieCntsOM U S N not the typical position in which people void. Men usually are most comfortable when standing; women are more comfortable when sitting and squatting. Embarrassment at using the bedpan and worrying about a urinary tract infection are not related to the lying-in-bed position. Fear of loss of independence is not related to use of the bedpan or urinal. DIF: Understand REF: 1186 OBJ: Discuss nursing measures to promote normal micturition and to reduce episodes of incontinence. TOP: Planning MSC: CPNRE: Foundations of Practice 28. The nurse would anticipate inserting a coudé catheter for which patient? a. An 8-year-old boy undergoing anaesthesia for a tonsillectomy. b. A 24-year-old woman who is going into labour. c. A 56-year-old man admitted for bladder irrigation. d. An 86-year-old woman admitted for a urinary tract infection. ANS: C
A coudé-tip catheter has a curved tip that is used for patients with enlarged prostates. This would be indicated for a middle-aged man who needs bladder irrigation. Coudé-tip catheters are not indicated for children or women. DIF: Apply REF: 1189 TOP: Implementation
OBJ: Insert a urinary catheter. MSC: CPNRE: Foundations of Practice
Canadian Fundamentals of Nursing 6th Edition Potter Test Bank 29. The nurse knows that which in-dwelling catheter procedure places the patient at greatest risk
for acquiring a urinary tract infection? a. Emptying the drainage bag every 8 hours or when half full. b. Kinking the catheter tubing to obtain a urine specimen. c. Placing the drainage bag on the side rail of the patient’s bed. d. Failing to secure the catheter tubing to the patient’s thigh. ANS: C
Placing the drainage bag on the side rail of the bed could allow the bag to be raised above the level of the bladder and urine to flow back into the bladder. The urine in the drainage bag is a medium for bacteria; allowing it to reenter the bladder can cause infection. The drainage bag should be emptied and output recorded every 8 hours or when needed. Urine specimens are obtained by temporarily kinking the tubing; a prolonged kink could lead to bladder distension. Failure to secure the catheter to the patient’s thigh places the patient at risk for tissue injury from catheter dislodgement. DIF: Apply REF: 1189 OBJ: Discuss nursing measures to reduce urinary tract infection. TOP: Implementation MSC: CPNRE: Foundations of Practice 30. A nurse notifies the provider immediately if a patient with an in-dwelling catheter does which
of the following? a. Complains of discomfort upon insertion of the catheter. b. Places the drainage bag higher than the waist while ambulating. c. Has not collected any urine in the drainage bag for 2 hours. d. Is incontinent of stool and contaminates the external portion of the catheter. ANS: C
NURSINGTB.COM
If the patient has not produced urine in 2 hours, the physician needs to be notified immediately because this could indicate renal failure. Discomfort upon catheter insertion is unpleasant but unavoidable. The nurse, not the provider, is responsible for maintaining the integrity of the catheter by ensuring that the drainage bag is below the patient’s bladder. Stool left on the catheter can cause infection and should be removed as soon as it is noticed. The nurse should ensure that frequent perineal care is being provided. DIF: Apply TOP: Evaluate
REF: 1169 OBJ: Insert a urinary catheter. MSC: CPNRE: Foundations of Practice
31. The nurse would question an order to insert a urinary catheter in which patient? a. A 26-year-old patient with a recent spinal cord injury at T2. b. A 30-year-old patient requiring drug screening for employment. c. A 40-year-old patient undergoing bladder repair surgery. d. An 86-year-old patient requiring monitoring of urinary output for renal failure. ANS: B
Urinary catheterization places the patient at increased risk for infection and should be performed only when necessary. Urine can be obtained via clean-catch technique for a drug screening or urinalysis. Spinal cord injury, surgery, and renal failure with critical intake and output monitoring are all appropriate reasons for catheterization. DIF: Apply REF: 1169, Table 43-1 OBJ: Insert a urinary catheter. TOP: Implementation MSC: CPNRE: Foundations of Practice
Canadian Fundamentals of Nursing 6th Edition Potter Test Bank
32. When caring for a hospitalized patient with a urinary catheter, which nursing action best
prevents the patient from acquiring an infection? a. Inserting the catheter by using strict clean technique. b. Performing hand hygiene before and after providing perineal care. c. Fully inflating the catheter’s balloon according to the manufacturer’s recommendation. d. Disconnecting and replacing the catheter drainage bag once per shift. ANS: B
Hand hygiene helps prevent infection in patients with a urinary catheter. A catheter should be inserted in the hospital setting with sterile technique. Inflating the balloon fully prevents dislodgement and trauma, not infection. Disconnecting the drainage bag from the catheter creates a break in the system and an open portal of entry for pathogens and increases risk of infection. DIF: Apply REF: 1172 OBJ: Discuss nursing measures to reduce urinary tract infection. TOP: Implementation MSC: CPNRE: Foundations of Practice 33. An 86-year-old patient asks the nurse what lifestyle changes will reduce the chance of a
urinary tract infection. Which response is accurate? a. Urinary tract infections are unavoidable in older people because of a weakened immune system. b. Decreasing fluid intake will decrease the amount of urine with bacteria produced. c. Making sure to cleanse the perineal area from back to front after voiding will reduce the chance of infection. G B.C M N R I d. Increasing consumption of U acidSic fN oodT s such aOs cranberry juice will reduce the chance of infection. ANS: D
Cranberry juice and other acidic foods decrease adherence of bacteria to the bladder wall, and increasing fluid intake will help flush bacteria, thus preventing them from residing in the bladder for prolonged periods of time. Urinary tract infections are avoidable in older people with proper knowledge and hygiene. Perineal skin should be cleansed from front to back to avoid spreading fecal matter to the urethra. DIF: Apply REF: 1173, Box 43-2 OBJ: Discuss nursing measures to reduce urinary tract infection. TOP: Implementation MSC: CPNRE: Foundations of Practice 34. A nurse is providing education to a patient being treated for a urinary tract infection. Which of
the following statements by the patient indicates an understanding? a. “Since I’m taking medication, I do not need to worry about proper hygiene.” b. “I should drink 15 to 20 glasses of fluid a day to help flush the bacteria out.” c. “My medication may discolour my urine; this should resolve once the medication is stopped.” d. “I should not have sexual intercourse until the infection has resolved.” ANS: C
Canadian Fundamentals of Nursing 6th Edition Potter Test Bank Some medications cause discoloration of urine; this is normal and will dissipate as the medication leaves the system. Even if the patient is on medication, hygiene is important to prevent spread or reinfection. Fluid intake should be increased to help flush out bacteria; however, 15 to 20 glasses is too much. Sexual intercourse is allowed with a urinary tract infection, as long as good hygiene and safe practices are used. DIF: Apply REF: 1176 OBJ: Discuss nursing measures to reduce urinary tract infection. TOP: Evaluate MSC: CPNRE: Foundations of Practice 35. To reduce patient discomfort during closed-catheter irrigation, what should the nurse do? a. Use room temperature irrigation solution. b. Administer the solution as quickly as possible. c. Allow the solution to sit in the bladder for at least 1 hour. d. Raise the bag of irrigation solution at least 30 centimetres above the bladder. ANS: A
Using cold solutions, instilling solutions too quickly, and prolonging filling of the bladder can cause discomfort and cramping. To reduce this, the nurse should ensure that the solution is at room temperature, lower the solution bag so that the fluid instills slowly, and drain the bladder fully after an ordered amount of time. DIF: Apply REF: 1201, Skill 43-4 OBJ: Irrigate a urinary catheter. TOP: Implementation MSC: CPNRE: Foundations of Practice 36. Which observation by the nurse best indicates that bladder irrigation for urinary retention has
been effective? a. An output that is larger thNaU nR thS eI amNoG unTtB in. stC ilO ledM. b. The presence of blood clots or sediment in the drainage bag. c. Reduction in discomfort from bladder distension. d. Visualizing clear urinary catheter tubing. ANS: A
An output that is greater than what was irrigated into the bladder shows progress that the bladder is draining urine. The other observations do not objectively help measure the increase in urine output. DIF: Analyze REF: 1176 TOP: Implementation
OBJ: Irrigate a urinary catheter. MSC: CPNRE: Foundations of Practice
37. The nurse anticipates urinary diversion from the kidneys to a site other than the bladder for
which patient? a. A 12-year-old girl with severe abdominal trauma. b. A 24-year-old man with severe genital warts around the urethra. c. A 50-year-old man who has recently undergone prostatectomy. d. A 75-year-old woman with end-stage renal disease. ANS: A
Urinary diversion would be needed in a patient with abdominal trauma who might have injury to the urinary system. Genital warts are not a reason for urinary diversion. Patients with a prostatectomy may require intermittent catheterization after the procedure. End-stage renal disease would not be affected by rerouting the flow of urine.
Canadian Fundamentals of Nursing 6th Edition Potter Test Bank
DIF: Apply REF: 1171 OBJ: Compare common alterations in urinary elimination. MSC: CPNRE: Foundations of Practice
TOP: Implementation
38. Which of the following symptom is most closely associated with uremic syndrome? a. Fever. b. Headache. c. Clay-coloured stools. d. Dysuria. ANS: B
Uremic syndrome is associated with end-stage renal disease. Signs and symptoms include headache, altered mental status, coma, seizures, nausea, vomiting, and pericarditis. DIF: Remember REF: 1171 | 1172 OBJ: Compare common alterations in urinary elimination. MSC: CPNRE: Foundations of Practice
TOP: Implementation
39. The nurse understands that peritoneal dialysis and hemodialysis involve which of the
following processes to clean the patient’s blood? a. Gravity and osmosis. b. Osmosis and diffusion. c. Diffusion and gravity. d. Filtration and osmosis. ANS: B
Osmosis and diffusion are theNtwo used to R processes Is G B.C M clean the patient’s blood in both types of dialysis. In peritoneal dialysis,U osmSosiN anT d dialysOis occur across the semi-permeable peritoneal membrane. In hemodialysis, osmosis and dialysis occur through the filter membrane on the artificial kidney. In peritoneal dialysis, the dialysate flows by gravity out of the abdomen. Gravity has no effect on cleansing of the blood. Filtration is the process that occurs in the glomerulus as blood flows through the kidney. DIF: Understand REF: 1172 OBJ: Identify two modalities of renal replacement therapy. MSC: CPNRE: Foundations of Practice
TOP: Implementation
Canadian Fundamentals of Nursing 6th Edition Potter Test Bank
Chapter 44: Bowel Elimination Potter et al: Canadian Fundamentals of Nursing, 6th Edition MULTIPLE CHOICE 1. The nurse knows that most nutrients are absorbed in which portion of the digestive tract? a. Stomach. b. Duodenum. c. Ileum. d. Cecum. ANS: B
Most nutrients are absorbed in the duodenum; the exceptions are certain vitamins, iron, and salt (which are absorbed in the ileum). Food is broken down in the stomach. The cecum is the beginning of the large intestine. DIF: Remember REF: 1212 OBJ: Discuss the role of gastrointestinal organs in digestion and elimination. TOP: Implementation MSC: CPNRE: Foundations of Practice 2. The nurse would expect the least formed stool to be present in which portion of the digestive
tract? a. Ascending colon. b. Descending colon. c. Transverse colon. d. Sigmoid colon. ANS: A
NURSINGTB.COM
The path of digestion goes from the ascending colon, across the transverse colon, to the descending colon, and into the sigmoid colon; therefore, the least formed stool would be in the ascending colon. DIF: Understand REF: 1212 OBJ: Explain the physiological aspects of normal defecation. MSC: CPNRE: Foundations of Practice
TOP: Assessment
3. Which of the following is not a function of the large intestine? a. Absorbing nutrients. b. Absorbing water. c. Secreting bicarbonate. d. Eliminating waste. ANS: A
Nutrient absorption takes place in the small intestine. The other options are all functions of the large intestine. DIF: Remember REF: 1213 | 1214 TOP: Implementation
OBJ: Describe the functions of the large intestine. MSC: CPNRE: Foundations of Practice
4. The nurse is caring for a patient who is bedridden. The nurse asks the patient whether he
needs to have a bowel movement 30 minutes after eating a meal, for what reason?
Canadian Fundamentals of Nursing 6th Edition Potter Test Bank a. b. c. d.
The digested food needs to make room for recently ingested food. Ingestion of food triggers the digestive system to begin peristalsis. The smell of bowel elimination in the room would deter the patient from eating. More ancillary staff members are available after mealtimes.
ANS: B
Peristalsis occurs only a few times a day; the strongest peristaltic waves are triggered by ingestion of food. The intestine can hold a great deal of food. A patient’s voiding schedule should not be based on the staff’s convenience. DIF: Understand REF: 1214 | 1216 OBJ: List the nursing measures included in bowel training. MSC: CPNRE: Foundations of Practice
TOP: Implementation
5. A nurse is assisting a patient in making dietary choices that promote healthy bowel
elimination. Which menu option should the nurse recommend? a. Grape and walnut chicken salad sandwich on whole wheat bread. b. Broccoli and cheese soup with potato bread. c. Dinner salad topped with hard-boiled eggs, cheese, and fat-free dressing. d. Turkey and mashed potatoes with brown gravy. ANS: A
A healthy diet for the bowel should include foods high in bulk-forming fibre. Whole grains, fresh fruit, and fresh vegetables are excellent sources. Foods without much fibre and with high levels of fat can slow down peristalsis, causing constipation. DIF: Apply REF: 1216 OBJ: List the nursing measureN s thR at promote nB ormal elM imination. U SINGMSC: T .CCPNRE: O Foundations of Practice TOP: Implementation 6. A patient informs the nurse that she was using laxatives three times daily to lose weight. After
stopping use of the laxative, the patient had difficulty with constipation and wonders if she needs to take laxatives again. With which statement does the nurse educate the patient? a. “Long-term laxative use causes the bowel to become less responsive to stimuli, and constipation may occur.” b. “Laxatives can cause trauma to the intestinal lining and scarring may result, leading to decreased peristalsis.” c. “Natural laxatives such as mineral oil are safer than chemical laxatives for relieving constipation.” d. “Laxatives cause the body to become malnourished, so when the patient begins eating again, the body absorbs all of the food, and no waste products are produced.” ANS: A
Long-term laxative use can lead to constipation. Increasing fluid and fibre intake can help with this problem. Laxatives do not cause scarring. Natural laxatives like mineral oil come with their own set of risks, such as rendering the body unable to absorb fat-soluble vitamins. Even if malnourished, the body will produce waste if substance is consumed. DIF: Understand REF: 1225, Box 44-5 OBJ: Use critical thinking in the provision of care to patients with alterations in bowel elimination. TOP: Implementation MSC: CPNRE: Foundations of Practice
Canadian Fundamentals of Nursing 6th Edition Potter Test Bank
7. A patient with a hip fracture is having difficulty defecating into a bed pan while lying in bed.
Which action by the nurse would assist the patient in having a successful bowel movement? a. Administering laxatives to the patient. b. Raising the head of the bed. c. Preparing to administer a barium enema. d. Withholding narcotic pain medication. ANS: B
Lying in bed is an unnatural position for defecation; raising the head of the bed assists the patient into a more normal position that allows proper contraction of muscles for elimination. Laxatives would not give the patient control over bowel movements. A barium enema is used for a diagnostic test; it is not used as an intervention to promote defecation. Pain relief measures should be given; however, action should be taken to prevent constipation. DIF: Apply REF: 1215 OBJ: Discuss the psychological and physiological factors that influence the elimination process. TOP: Implementation MSC: CPNRE: Foundations of Practice 8. Which patient is most at risk for increased peristalsis? a. A 5-year-old child who ignores the urge to defecate owing to embarrassment. b. A 21-year-old patient with three final examinations on the same day. c. A 40-year-old woman with major depressive disorder. d. An 80-year-old man in an assisted-living environment. ANS: B
Stress can stimulate digestion and increase peristalsis. Depression, ignoring the urge to defecate, and age-related chaN ngeR s inI oldG er pB eo. plC e arM e causes of constipation.
U S N T
O
DIF: Understand REF: 1224 | 1227 OBJ: Describe the common physiological alterations in elimination. TOP: Implementation MSC: CPNRE: Foundations of Practice 9. A patient expresses concerns over having black stool. The fecal occult test is negative. Which
response by the nurse is most appropriate? a. “This is probably a false-negative result; we should rerun the test.” b. “Do you take iron supplements?” c. “You should schedule a colonoscopy as soon as possible.” d. “Sometimes severe stress can alter stool colour.” ANS: B
Certain medications and supplements, such as iron, can alter the colour of stool. The fecal occult test involves three separate samples obtained over a period of time. A colonoscopy is health prevention screening that should be done every 5 to 10 years; it is not the nurse’s initial priority. Stress alters gastrointestinal (GI) motility and stool consistency, not colour. DIF: Apply REF: 1221, Table 44-2 | 1223, Table 44-4 OBJ: Use critical thinking in the provision of care to patients with alterations in bowel elimination. TOP: Implementation MSC: CPNRE: Foundations of Practice 10. Which physiological change can cause a paralytic ileus? a. Chronic cathartic abuse.
Canadian Fundamentals of Nursing 6th Edition Potter Test Bank b. Surgery with anaesthesia for Crohn’s disease. c. Suppression of hydrochloric acid from medication. d. Fecal impaction. ANS: B
Surgical manipulation of the bowel can cause a paralytic ileus. DIF: Remember REF: 1220 OBJ: Describe the common physiological alterations in elimination. TOP: Implementation MSC: CPNRE: Foundations of Practice 11. Fecal impactions occur in which portion of the colon? a. Ascending. b. Descending. c. Transverse. d. Rectum. ANS: D
A fecal impaction is a collection of hardened feces wedged in the rectum that cannot be expelled. It results from unrelieved constipation. Feces at this point in the colon contain the least amount of moisture. Feces found in the ascending, transverse, and descending colon still consist mostly of liquid and do not form a hardened mass. DIF: Remember TOP: Assessment
REF: 1225 OBJ: Assess a patient’s pattern of elimination. MSC: CPNRE: Foundations of Practice
12. The nurse knows that a bowel elimination schedule would be most beneficial in the plan of
care for which patient? NURSINGTB.COM a. A 40-year-old patient with an ileostomy. b. A 25-year-old patient with Crohn’s disease. c. A 30-year-old patient with C. difficile. d. A 70-year-old patient with stool incontinence. ANS: D
A bowel retraining program is helpful for a patient with incontinence. It helps the person who still has neuromuscular control defecate normally. An ileostomy, Crohn’s disease, and C. difficile infection all relate to uncontrollable bowel movements, for which a schedule of elimination is of no use. DIF: Analyze REF: 1229 OBJ: List the nursing measures included in bowel training. MSC: CPNRE: Foundations of Practice
TOP: Planning
13. Which nursing intervention is most effective in promoting normal defecation for a patient who
has muscle weakness in the legs that prevents ambulation? a. Elevating the head of the bed 45 degrees 60 minutes after breakfast. b. Using a mobility device to place the patient on a bedside commode. c. Giving the patient a pillow to brace against the abdomen while bearing down. d. Administering a soapsuds enema every 2 hours. ANS: B
Canadian Fundamentals of Nursing 6th Edition Potter Test Bank The best way to promote normal defecation is to assist the patient into a posture that is as normal as possible while defecating. The use of a mobility device promotes nurse and patient safety. Elevating the head of the bed would be appropriate if the patient were to void with a bedpan. However, the patient’s condition does not necessitate use of a bedpan. Giving the patient a pillow may reduce discomfort, but this is not the best way to promote defecation. A soapsuds enema is indicated for a patient who needs assistance to stimulate peristalsis. It promotes nonnatural defecation. DIF: Apply REF: 1218 OBJ: List the nursing measures included in bowel training. MSC: CPNRE: Foundations of Practice
TOP: Implementation
14. The nurse is devising a plan of care for a patient with the nursing diagnosis of Constipation
related to opioid use. Which of the following outcomes would the nurse evaluate as successful for the patient to establish normal defecation? a. The patient reports eliminating a soft, formed stool. b. The patient has quit taking opioid pain medication. c. The patient’s lower left quadrant is tender to the touch. d. The nurse hears bowel sounds present in all four quadrants. ANS: A
The nurse’s goal is for the patient to continue taking opioid medication and to have normal bowel elimination. Normal stools are soft and formed. Discontinuing pain medication is not a desired outcome for the patient. Tenderness in the left lower quadrant indicates constipation and does not further address bowel elimination. The presence of bowel sounds indicates that the bowels are moving; however, they are not an indication of defecation. DIF: Apply REF: 12N 14URSINGTB.COM OBJ: List the nursing diagnoses related to alterations in elimination. TOP: Evaluate MSC: CPNRE: Foundations of Practice 15. The nurse is emptying an ileostomy pouch for a patient. Which assessment finding would the
nurse report immediately? a. Liquid consistency of stool. b. Presence of blood in the stool. c. Noxious odour from the stool. d. Continuous output from the stoma. ANS: B
Blood in the stool may indicate a problem with the surgical procedure, and the physician should be notified. All other options are expected findings for an ileostomy. DIF: Apply REF: 1221 OBJ: Use critical thinking in the provision of care to patients with alterations in bowel elimination. TOP: Assessment MSC: CPNRE: Foundations of Practice 16. The nurse would anticipate which diagnostic examination for a patient with black tarry stools? a. Ultrasonography. b. Barium enema study. c. Upper endoscopy. d. Flexible sigmoidoscopy.
Canadian Fundamentals of Nursing 6th Edition Potter Test Bank ANS: C
Black tarry stools are an indication of ulceration or bleeding in the upper portion of the GI tract; upper endoscopy would allow visualization of the bleeding. No other option would allow upper GI visualization. DIF: Understand REF: 1221, Table 44-3 OBJ: Use critical thinking in the provision of care to patients with alterations in bowel elimination. TOP: Planning MSC: CPNRE: Foundations of Practice 17. The nurse has attempted to administer a tap water enema for a patient with fecal impaction
with no success. What is the next priority nursing action? a. Preparing the patient for a second tap water enema. b. Donning gloves for digital removal of the stool. c. Positioning the patient on the left side. d. Inserting a rectal tube. ANS: B
When enemas are not successful, digital removal of the stool is occasionally necessary to break up pieces of the stool or to stimulate the anus to defecate. Tap water enemas should not be repeated because of risk of fluid imbalance. Positioning the patient on the left side does not promote defecation. A rectal tube is indicated for a patient with liquid stool incontinence but would not be applicable or effective for this patient. DIF: Apply REF: 1235, Box 44-10 OBJ: Use critical thinking in the provision of care to patients with alterations in bowel elimination. TOP: Implementation MSC: CPNRE: Foundations of Practice
hich 18. The nurse should question wN URord SIer? NGTB.COM a. A normal saline enema to be repeated every 4 hours until stool is produced. b. A hypertonic solution enema with a patient with fluid volume excess. c. A sodium polystyrene sulfonate (Kayexalate) enema for a patient with hypokalemia. d. An oil retention enema for a patient using mineral oil laxatives. ANS: C
Sodium polystyrene sulfonate binds to and helps excrete potassium, so it would be contraindicated in patients who are hypokalemic (have low potassium). Normal saline enemas can be repeated without risk of fluid or electrolyte imbalance. Hypertonic solutions are intended for patients who cannot handle large fluid volume, and they are contraindicated in dehydrated patients. Because mineral oil laxatives and an oil retention enema have the same intended effect of lubricating the colon and rectum, an oil retention enema is not needed. DIF: Apply REF: 1232 OBJ: Use critical thinking in the provision of care to patients with alterations in bowel elimination. TOP: Planning MSC: CPNRE: Foundations of Practice 19. The nurse is preparing to perform a fecal occult blood test. The nurse plans to properly
perform the examination by doing which of the following? a. Applying liberal amounts of stool to the guaiac paper. b. Not allowing patients to collect their own specimens. c. Reporting any abnormal findings to the care provider.
Canadian Fundamentals of Nursing 6th Edition Potter Test Bank d. Applying sterile disposable gloves. ANS: C
Abnormal findings such as a positive test result should be reported to the care provider. A fecal occult blood test is a clean procedure; sterile gloves are not needed. A thin specimen smear is all that is required. Patients can collect their own specimens if possible. DIF: Understand REF: 1221 | 1222, Box 44-1 OBJ: Describe the nursing implications for common diagnostic examinations of the gastrointestinal tract. TOP: Planning MSC: CPNRE: Foundations of Practice 20. A nurse is preparing a patient for magnetic resonance imaging (MRI). Which nursing action is
most important? a. Ensuring that the patient does not eat or drink 2 hours before the examination. b. Removing all of the patient’s metallic jewellery. c. Administering a colon-cleansing product 12 hours before the examination. d. Obtaining an order for a pain medication before the test is performed. ANS: B
No jewellery or metal products should be in the same room as an MRI machine because of the high-power magnet used in the machine. The patient needs to be on NPO status (nothing by mouth) 4 to 6 hours before the examination. Colon cleansing products are not necessary for MRIs. Pain medication is not needed before the examination is performed. DIF: Apply REF: 1223, Box 44-3 OBJ: Describe nursing implications for common diagnostic examinations of the gastrointestinal tract. TOP: Planning MSC: CPNRE: Foundations of Practice
N R I G B.C M
21. After a patient returns from a bU ariuS m sN walT low stuO dy, what is the nurse’s priority? a. Encourage the patient to increase fluids to flush out the barium. b. Monitor stools closely for bright red blood or mucus, which indicates trauma from
the procedure. c. Inform the patient that the bowel movements are radioactive and that the patient
should be sure to flush the toilet three times. d. Thicken all patient drinks to prevent aspiration. ANS: A
The nurse should encourage the patient to increase fluid intake to flush and remove excess barium from the body. Barium swallow studies are noninvasive procedures that do not involve trauma, and so no blood or mucus would result, and aspiration risk would not be increased. Barium is not a radioactive substance, so multiple flushes are not needed. DIF: Apply REF: 1223, Box 44-3 OBJ: Describe the nursing implications for common diagnostic examinations of the gastrointestinal tract. TOP: Planning MSC: CPNRE: Foundations of Practice 22. A nurse is educating a patient on how to irrigate an ostomy bag. Which statement by the
patient indicates the need for further instruction? a. “I can use a Fleet enema to save money because it contains the same irrigation solution.” b. “Sitting on the toilet lets the irrigation sleeve eliminate into the bowl.” c. “I should never attempt to reach into my stoma to remove fecal material.”
Canadian Fundamentals of Nursing 6th Edition Potter Test Bank d. “Using warm tap water will reduce cramping and discomfort during the
procedure.” ANS: A
Enema applicators should never be used in the stoma because they can cause damage. A special coned irrigation device is used for ostomies. Irrigating a stoma into the toilet is an effective management technique. Fingers and other objects should not be placed into the stoma because they may cause trauma to the intestinal wall. Warm tap water will reduce cramping during irrigation. DIF: Apply REF: 1237 | 1238 OBJ: Discuss the nursing measures required for patients with a bowel diversion. TOP: Implementation MSC: CPNRE: Foundations of Practice 23. A patient had an ileostomy surgically placed 2 days ago. Which diet would the nurse
recommend to the patient to ease the transition to use of the new ostomy? a. Eggs over easy, whole wheat toast, and orange juice with pulp. b. Chicken fried rice with stir-fried vegetables and iced tea. c. Turkey meatloaf with white rice and apple juice. d. Fish sticks with macaroni and cheese and soda. ANS: C
During the first week or so after ostomy placement, the patient should consume easy-to-digest, low-fibre foods such as poultry, rice and noodles, and strained fruit juices. Fried foods can irritate digestion and can cause blockage. Foods high in fibre will be useful later in the recovery process but can cause blockage if the GI tract is not accustomed to digesting with an ileostomy.
NURSINGTB.COM
DIF: Apply REF: 1239 OBJ: Discuss the nursing measures required for patients with a bowel diversion. TOP: Implementation MSC: CPNRE: Foundations of Practice 24. The nurse knows that the ideal time to change an ostomy pouch is when? a. Before the patient eats a meal, when the patient is comfortable. b. When the patient feels that he or she needs to have a bowel movement. c. When ordered in the patient’s chart. d. After the patient has ambulated the length of the hallway. ANS: A
The nurse wants to change the ostomy appliance when output is as low as possible to ensure a smooth procedure. Patients with ostomies do not feel the urge to defecate because the sensory portion of the anus is not stimulated. The decision to change the ostomy pouch involves nursing judgement. After a patient ambulates, stool output is increased. DIF: Apply REF: 1240, Skill 44-2 OBJ: Discuss the nursing measures required for patients with a bowel diversion. TOP: Assessment MSC: CPNRE: Foundations of Practice 25. The nurse administers a cathartic to a patient. The nurse determines that the cathartic has had
a therapeutic effect when what happens? a. The patient has a decreased level of anxiety. b. The patient experiences pain relief.
Canadian Fundamentals of Nursing 6th Edition Potter Test Bank c. The patient has a bowel movement. d. The patient passes flatulence. ANS: C
A cathartic is a laxative that stimulates a bowel movement. It would be considered effective if the patient experiences a bowel movement. The other options are not outcomes of administration of a cathartic. DIF: Apply REF: 1230 OBJ: Use critical thinking in the provision of care to patients with alterations in bowel elimination. TOP: Planning MSC: CPNRE: Foundations of Practice 26. An older person’s perineal skin appears to be dry and thin with mild excoriation. When
hygiene is provided after a bowel movement, what should the nurse do? a. Thoroughly scrub the skin with a washcloth and hypoallergenic soap. b. Apply a skin protective lotion after perineal care. c. Tape an occlusive moisture barrier pad to the patient’s skin. d. Massage the skin with deep kneading pressure. ANS: B
The nurse gently cleans the skin and applies a moistening barrier cream. Tape and occlusive dressings can damage skin. Excessive pressure and force are inappropriate and may cause skin breakdown. DIF: Apply REF: 1230 OBJ: List the nursing measures that promote normal elimination. TOP: Planning MSC: CPNRE: Foundations of Practice
N R I G B.C M
27. The patient is seen in the gastroUentS erolN ogyTclinic aOfter having experienced changes in his
bowel elimination. A colonoscopy is ordered, and the patient has questions about the examination. What information should the nurse give the patient before the colonoscopy? a. No special preparation is required. b. Light sedation is normally used. c. No metallic objects are allowed. d. Swallowing of an opaque liquid is required. ANS: B
Light sedation is required for a colonoscopy, as is special preparation. Clear liquids are given the day before, to be followed by some form of bowel cleanser. Enemas until only clear fluid is passed also may be ordered. No restriction of metallic objects is made for a colonoscopy. A colonoscopy does not require the swallowing of an opaque liquid. DIF: Apply REF: 1223, Box 44-3 OBJ: Describe the nursing implications for common diagnostic examinations of the gastrointestinal tract. TOP: Implementation MSC: CPNRE: Foundations of Practice 28. A nurse is providing discharge teaching for a patient who is going home with a guaiac test.
Which statement by the patient indicates the need for further education? a. “If I get a positive result, I have gastrointestinal bleeding.” b. “I should not eat red meat before my examination.” c. “I should schedule to perform the examination when I am not menstruating.” d. “I will need to perform this test three times if I have a positive result.”
Canadian Fundamentals of Nursing 6th Edition Potter Test Bank
ANS: A
A positive result does not mean GI bleeding; it could be a false-positive finding caused by consumption of red meat, some raw vegetables, or nonsteroidal anti-inflammatory drugs. Proper patient education is important for viable results. The patient needs to avoid certain foods to rule out a false-positive finding. If the test result is positive, the patient will need to repeat the test at least three times. Menses and hemorrhoids can also lead to false-positive results. DIF: Apply REF: 1221 | 1222 OBJ: Describe nursing implications for common diagnostic examinations of the gastrointestinal tract. TOP: Evaluate MSC: CPNRE: Foundations of Practice 29. A nurse is caring for an older patient with fecal incontinence caused by cathartic use. The
nurse is most concerned about which complication that carries the greatest risk for severe injury? a. Rectal skin breakdown. b. Contamination of existing wounds. c. Falls from attempts to reach the bathroom. d. Cross-contamination into the upper GI tract. ANS: C
The nurse is most concerned about the worst injury the patient could receive, which involves falling while attempting to get to the bathroom. To reduce injury, the nurse should clear the path to the bathroom and reinforce use of the call light. The question concerns the greatest risk of injury, not the most frequent occurrence or the event most likely to occur. DIF: Apply REF: 12N 30 R I G B.C M S N ofTcare to O OBJ: Use critical thinking in the U provision patients with alterations in bowel elimination. TOP: Planning MSC: CPNRE: Foundations of Practice 30. The nurse is caring for a patient with Clostridium difficile infection. Which of the following
nursing actions will be most effective in preventing the spread of bacteria? a. Monthly in-service education about contact precautions. b. Placing all contaminated items in biohazard bags. c. Mandatory cultures for all patients. d. Proper hand hygiene techniques. ANS: D
Proper hand hygiene is the best way to prevent the spread of bacteria. Monthly in-service education places emphasis on education, not on action. Biohazard bags are appropriate but cannot be used on every item that C. difficile comes in contact with, such as a human. Mandatory cultures are expensive and unnecessary and would not prevent the spread of bacteria. DIF: Apply REF: 1219 OBJ: Discuss the psychological and physiological factors that influence the elimination process. TOP: Planning MSC: CPNRE: Foundations of Practice 31. A nurse is caring for a patient who has had diarrhea for the past week. Which additional
assessment finding would the nurse expect? a. Increased energy levels.
Canadian Fundamentals of Nursing 6th Edition Potter Test Bank b. Distended abdomen. c. Decreased serum bicarbonate level. d. Increased blood pressure. ANS: C
Chronic diarrhea can result in metabolic acidosis, which is diagnostic of low serum bicarbonate. Patients with chronic diarrhea are dehydrated with decreased blood pressure. Diarrhea also causes loss of electrolytes, nutrients, and fluid, which decreases energy levels. A distended abdomen would indicate constipation. DIF: Apply TOP: Planning
REF: 1226 OBJ: Assess a patient’s pattern of elimination. MSC: CPNRE: Foundations of Practice
32. The nurse is caring for a patient who had a colostomy placed yesterday. The nurse should
report which assessment finding immediately? a. Stoma is protruding from the abdomen. b. Stoma is moist. c. Stool is discharging from the stoma. d. Stoma is purple. ANS: D
A purple stoma may indicate strangulation or poor circulation to the stoma and may require surgical intervention. A stoma should be reddish-pink and moist in appearance. It can be flush with the skin, or it can protrude. Stool is an expected outcome of stoma placement. DIF: Apply REF: 1242, Skill 44-2 OBJ: Discuss the nursing measures required for patients with a bowel diversion. TOP: Assessment MSC: CP on. sC of PM ractice NNRRE: FIounGdatiB
U S N T
O
33. A patient has constipation and hypernatremia. The nurse prepares to administer which type of
enema? a. Oil retention. b. Carminative. c. Saline. d. Tap water. ANS: D
Tap water enema would draw fluid into the system, inasmuch as it is hypotonic and would help flush out excess sodium. Oil retention would not address sodium problems. Carminative enemas are used to provide relief from distension caused by gas. A saline enema would worsen hypernatremia. DIF: Apply REF: 1232 OBJ: List the nursing measures that promote normal elimination. TOP: Implementation MSC: CPNRE: Foundations of Practice 34. A guaiac test has been ordered. The nurse knows that this is a test for which of the following? a. Bright red blood. b. Dark black blood. c. Blood that contains mucus. d. Blood that cannot be seen.
Canadian Fundamentals of Nursing 6th Edition Potter Test Bank ANS: D
Fecal occult blood tests are used to test for blood that may be present in stool that cannot be seen by the naked eye. The presence of such blood is usually indicative of GI bleeding. DIF: Understand REF: 1221 | 1222 OBJ: Describe the nursing implications for common diagnostic examinations of the gastrointestinal tract. TOP: Assessment MSC: CPNRE: Foundations of Practice 35. The nurse should place the patient in which position when preparing to administer an enema? a. Left Sims’s position. b. Fowler’s. c. Supine. d. Semi-Fowler’s. ANS: A
Side-lying Sims’s position allows the enema solution to flow downward by gravity along the natural curve of the sigmoid colon. This helps improve retention of the enema. Administering an enema in a sitting position may cause the curved rectal tube to scrape the rectal wall. DIF: Remember REF: 1233, Skill 44-1 OBJ: List the nursing measures that promote normal elimination. TOP: Implementation MSC: CPNRE: Foundations of Practice 36. The nurse is assessing a patient 2 hours after a colonoscopy. For this procedure, what focused
assessment will the nurse include? a. Bowel sounds. b. Presence of flatulence. c. Bowel movements. NURSINGTB.COM d. Nausea. ANS: A
The nurse does want to hear the presence of bowel sounds; absence of bowel sounds may indicate a complication from the surgery. Bowel movements and flatulence are not expected in the hours after surgery. Nausea is not a problem after colonoscopy. DIF: Apply REF: 1224 OBJ: Describe the nursing implications for common diagnostic examinations of the gastrointestinal tract. TOP: Assessment MSC: CPNRE: Foundations of Practice
Canadian Fundamentals of Nursing 6th Edition Potter Test Bank
Chapter 45: Mobility and Immobility Potter et al: Canadian Fundamentals of Nursing, 6th Edition MULTIPLE CHOICE 1. A patient has been on prolonged bed rest, and the nurse is observing for signs associated with
immobility. While assessing the patient, the nurse is alert to which of the following signs? a. Increased blood pressure. b. Decreased heart rate. c. Increased urinary output. d. Decreased peristalsis. ANS: D
Immobility disrupts the normal functioning of the gastrointestinal system, which results in decreased appetite and slowed peristalsis. In an immobilized patient, circulating fluid volume decreases, blood pools in the lower extremities, and autonomic response decreases. These factors result in decreased venous return, followed by a decrease in cardiac output, which is reflected by a decline in blood pressure. Recumbency increases cardiac workload and results in an increased pulse rate. Fluid intake can diminish with immobility, and this, in combination with other causes, such as fever, increases the risk of dehydration. Urinary output may decline on or about the fifth or sixth day after immobilization, and the urine is often highly concentrated. DIF: Remember REF: 1249 OBJ: Identify changes in physiological and psychosocial function associated with immobility. TOP: Assessment MSC: CPNRE: Foundations of Practice
N R I G B.C M U S N T O
2. A 61-year-old patient recently had left-sided paralysis from a cerebrovascular accident (CVA;
also known as stroke). In planning care for this patient, the nurse would do which of the following? a. Encourage an even gait when walking in place. b. Assess the extremities for unilateral swelling and muscle atrophy. c. Encourage holding the breath frequently to hyperinflate the lungs. d. Teach the use of a two-point crutch technique for ambulation. ANS: B
Because edema moves to dependent body regions, assessment of the immobilized patient should include observation of the sacrum, legs, and feet. Unilateral increases in calf diameter can be an early indication of thrombosis. The patient who has suffered a CVA with left-sided paralysis may not be capable of an even gait. Having the patient hold his or her breath frequently is not an appropriate nursing intervention. To prevent stasis of pulmonary secretions, the patient’s position should be changed every 2 hours, and fluid intake should be increased to 2000 mL, if not contraindicated. The patient should breathe deeply and cough every 1 to 2 hours to promote chest expansion. Two-point crutch technique would not be appropriate for the patient with left-sided paralysis. The patient would be more likely to ambulate safely with a walker or a cane. If crutches are used, the patient should use a three-point support. DIF: Analyze REF: 1272, Skill 45-1 OBJ: Assess for correct and impaired body alignment and mobility.
Canadian Fundamentals of Nursing 6th Edition Potter Test Bank TOP: Implementation
MSC: CPNRE: Foundations of Practice
3. When a patient with impaired physical mobility is in the recumbent position, what angle of
lateral position is recommended? a. 15 degrees. b. 30 degrees. c. 45 degrees. d. 90 degrees. ANS: B
When a patient with impaired physical mobility is in the recumbent position, the 30-degree lateral position reduces pressure from the sacral area and reduces the risk of skin breakdown. DIF: Apply REF: 1265, Box 45-7 | 1272 OBJ: Assess for correct and impaired body alignment and mobility. TOP: Assessment MSC: CPNRE: Foundations of Practice 4. The patient has sequential compression stockings in place. The nurse evaluates that the
stockings have been implemented appropriately by the new staff nurse when the nurse observes what? a. Intermittent pressure is set at 40 mm Hg. b. Initial measurement is made around the patient’s calves. c. Stockings are wrapped directly over the leg from ankle to knee. d. Stockings are removed every hour during application. ANS: A
Inflation pressure averages 40 mm Hg. Initial measurement is made around the largest part of the patient’s thigh. A protectN ive sRtocI is . plC aceM U Skine NGtteTB Od over the patient’s leg. Then the stocking is wrapped around the leg, starting at the ankle, with the opening over the patella. Stockings are not removed every hour. For optimal results, sequential compression or intermittent pneumatic compression devices are used as soon as possible and maintained until the patient becomes fully ambulatory. The stockings should be removed periodically to assess the condition of the patient’s skin. DIF: Apply REF: 1268 OBJ: Develop individualized nursing care plans for patients with impaired mobility. TOP: Evaluate MSC: CPNRE: Foundations of Practice 5. The patient with torticollis would exhibit which of the following? a. Exaggeration of the lumbar spine curvature. b. Increased convexity of the thoracic spine. c. Abnormal anteroposterior and lateral curvature of the spine. d. Contracture of the sternocleidomastoid muscle with a head incline. ANS: D
Torticollis is the inclining of the head to the affected side, in which the sternocleidomastoid muscle is contracted. Lordosis is an exaggeration of the lumbar spine curvature. Kyphosis is an increased convexity in the curvature of the thoracic spine. Kyphoscoliosis is an abnormal anteroposterior and lateral curvature of the spine. DIF: Understand REF: 1248, Table 45-1 OBJ: Discuss physiological and pathological influences on body alignment and joint mobility.
Canadian Fundamentals of Nursing 6th Edition Potter Test Bank TOP: Assessment
MSC: CPNRE: Foundations of Practice
6. The nurse expects to maintain the patient’s legs in abduction after total hip replacement
surgery with the use of which of the following? a. Foot boot. b. Wedge pillow. c. Trochanter roll. d. Sandbag. ANS: B
A wedge pillow is a triangular pillow made of heavy foam used to maintain the legs in abduction after total hip replacement surgery. A foot boot maintains feet in dorsiflexion. A trochanter roll prevents external rotation of the hips. Sandbags can be used to shape body contours and immobilize an extremity. DIF: Apply REF: 1271 OBJ: Develop individualized nursing care plans for patients with impaired mobility. TOP: Implementation MSC: CPNRE: Foundations of Practice 7. The patient is about to get up for the first time after a period of bed rest. What is the initial
nursing action? a. Assess respiratory function. b. Obtain a baseline blood pressure. c. Assist the patient to sit at the edge of the bed. d. Ask the patient whether he or she feels lightheaded. ANS: B
When getting the patient up fN or tR he fI irst G ftC er aMperiod of bed rest, the nurse should N time TBa.obtains O a baseline blood pressure. Assessing the document orthostatic changes. U TheSnurse first patient’s respiratory function is not the nurse’s first intervention when getting a patient up for the first time after prolonged bed rest. After the nurse assesses the patient’s blood pressure, the nurse can assist the patient to a sitting position at the side of the bed. After the patient is in the sitting position at the side of the bed, the nurse should ask the patient whether he or she feels lightheaded. DIF: Apply REF: 1261 | 1272, Skill 45-1 OBJ: Develop individualized nursing care plans for patients with impaired mobility. TOP: Implementation MSC: CPNRE: Foundations of Practice 8. Immobilized patients frequently have hypercalcemia, which places them at risk for what? a. Osteoporosis. b. Renal calculi. c. Pressure ulcers. d. Thrombus formation. ANS: B
Canadian Fundamentals of Nursing 6th Edition Potter Test Bank Renal calculi are calcium stones that lodge in the renal pelvis or pass through the ureters. Immobilized patients are at risk for calculi because they frequently have hypercalcemia. Osteoporosis is caused by accelerated bone loss. A pressure ulcer is an impairment of the skin that results from prolonged ischemia (decreased blood supply) within tissues. A thrombus is an accumulation of platelets, fibrin, clotting factors, and cellular elements of the blood attached to the interior wall of a vein or artery, which sometimes occludes the lumen of the vessel. DIF: Remember REF: 1252 OBJ: Identify changes in physiological and psychosocial function associated with immobility. TOP: Assessment MSC: CPNRE: Foundations of Practice 9. Patients on bed rest or otherwise immobile are at risk for what condition? a. Increased metabolic rate. b. Increased diarrhea (peristalsis). c. Altered metabolic function. d. Increased appetite. ANS: C
Immobility disrupts normal metabolic functioning: decreasing the metabolic rate; altering the metabolism of carbohydrates, fats, and proteins (nutritional function); causing fluid, electrolyte, and calcium imbalances; and causing gastrointestinal disturbances such as decreased appetite and slowing of peristalsis. DIF: Remember REF: 1250 OBJ: Identify changes in physiological and psychosocial function associated with immobility. TOP: Assessment MSC: CPNRE: Foundations of Practice
NURSINGTB.COM
10. In caring for a patient who is immobile, what is important for the nurse to understand? a. The effects of immobility are the same for everyone. b. Immobility helps maintain sleep–wake patterns. c. Changes in role and self-concept may lead to depression. d. Immobile patients are often eager to help in their own care. ANS: C
Many immobilized patients become depressed because of changes in role and self-concept. Every patient responds to immobility differently. Immobility or bed rest frequently affects coping and creates sleep–wake alterations because of changes in routine or in the environment. Because immobilization removes the patient from a daily routine, he or she has more time to worry about disability. Worrying increases the patient’s depression quickly, causing withdrawal. Many withdrawn patients do not want to participate in their own care. DIF: Understand REF: 1252 OBJ: Identify changes in physiological and psychosocial function associated with immobility. TOP: Assessment MSC: CPNRE: Foundations of Practice 11. Immobility is a major risk factor for pressure ulcers. In caring for the patient who is
immobilized, the nurse needs to be aware of which of the following? a. Breaks in skin integrity are easy to heal. b. Preventing a pressure ulcer is more expensive than treating one. c. A 30-degree lateral position is recommended.
Canadian Fundamentals of Nursing 6th Edition Potter Test Bank d. Pressure ulcers are caused by a sudden influx of oxygen to the tissue. ANS: C
Immobility is a major risk factor for pressure ulcers. A 30-degree lateral position is recommended for patients at risk for pressure ulcers. Any break in the integrity of the skin is difficult to heal. Preventing a pressure ulcer is much less expensive than treating one; therefore, preventive nursing interventions are imperative. Tissue metabolism depends on the supply of oxygen and nutrients to and the elimination of metabolic wastes from the blood. Pressure affects cellular metabolism by decreasing or totally eliminating tissue circulation. DIF: Understand REF: 1265, Box 45-7 | 1272 OBJ: Identify changes in physiological and psychosocial function associated with immobility. TOP: Assessment MSC: CPNRE: Foundations of Practice 12. The nurse is caring for a patient who has suffered a stroke. As part of the patient’s ongoing
care, what should the nurse do? a. Encourage the patient to perform as many self-care activities as possible. b. Provide a complete bed bath to promote patient comfort. c. Place the patient on bed rest to prevent fatigue. d. Understand that the patient will not eat owing to a decreased energy need. ANS: A
Nurses should encourage the patient to perform as many self-care activities as possible, thereby maintaining the highest level of mobility. Sometimes nurses inadvertently contribute to a patient’s immobility by providing unnecessary help with activities such as bathing and transferring. Placing the patient on bed rest without sufficient ambulation leads to loss of mobility and functional decline, resulting in weakness, fatigue, and increased risk for falls. Anorexia and insufficient assN isU taR ncSeI wN itG hT eaB tin.gCleOaM d to malnutrition. DIF: Apply REF: 1247 | 1253 OBJ: Identify changes in physiological and psychosocial function associated with immobility. TOP: Implementation MSC: CPNRE: Foundations of Practice 13. The nurse is assessing the way the patient walks. The manner of walking is known as what? a. Activity tolerance. b. Body alignment. c. Range of motion. d. Gait. ANS: D
The term gait describes a particular manner or style of walking. Activity tolerance is the type and amount of exercise or work that a person is able to perform. Body alignment refers to the position of the joints, tendons, ligaments, and muscles while the person is standing, sitting, and lying. Range of motion is the maximum amount of movement available at a joint in one of the three planes of the body: sagittal, frontal, or transverse. DIF: Remember REF: 1254-1259 OBJ: Assess for correct and impaired body alignment and mobility. TOP: Assessment MSC: CPNRE: Foundations of Practice 14. When assessing the body alignment of a patient while he or she is standing, the nurse is aware
of which of the following?
Canadian Fundamentals of Nursing 6th Edition Potter Test Bank a. b. c. d.
When observed posteriorly, the hips and shoulders form an S pattern. When observed laterally, the spinal curves align in a reversed S pattern. The arms should be crossed over the chest or in the lap. The feet should be close together with toes pointed out.
ANS: B
When the patient is observed laterally, the head is erect and the spinal curves are aligned in a reversed S pattern. When observed posteriorly, the shoulders and hips are straight and parallel. The arms hang comfortably at the sides. The feet are slightly apart to achieve a base of support, and the toes are pointed forward. DIF: Understand REF: 1259 OBJ: Assess for correct and impaired body alignment and mobility. TOP: Assessment MSC: CPNRE: Foundations of Practice 15. The nurse is evaluating the body alignment of a patient in the sitting position. In this position,
how is the body aligned? a. The body weight is directly on the buttocks only. b. Both feet are supported on the floor with ankles flexed. c. The edge of the seat is in contact with the popliteal space. d. The arms hang comfortably at the sides. ANS: B
Both feet are supported on the floor, and the ankles are comfortably flexed. With patients of short stature, a footstool is used to ensure that the ankles are comfortably flexed. Body weight is evenly distributed on the buttocks and thighs. A 2.5- to 5.0-cm (1- to 2-inch) space is maintained between the edge of the seat and the popliteal space on the posterior surface of the knee to ensure that no pressuN reUisRpla SIced NGonTBth.e Cpop OMliteal artery or nerve. The patient’s forearms are supported on the armrest, in the lap, or on a table in front of the chair. DIF: Understand REF: 1259 | 1260 OBJ: Assess for correct and impaired body alignment and mobility. TOP: Assessment MSC: CPNRE: Foundations of Practice 16. The nurse is assessing body alignment for a patient who is immobilized. What must the nurse
do? a. b. c. d.
Place the patient in the supine position. Remove the pillow from under the patient’s head. Insert positioning supports to help the patient. Place the patient in a lateral position.
ANS: D
For a patient who is immobilized or bedridden, body alignment is assessed with the patient in the lateral position, not supine. The nurse should remove all positioning support from the bed, except for the pillow under the patient’s head. DIF: Apply REF: 1260 OBJ: Assess for correct and impaired body alignment and mobility. TOP: Implementation MSC: CPNRE: Foundations of Practice 17. The nurse must assess the patient for hazards of immobility by performing a head-to-toe
physical assessment. When assessing the respiratory system, what should the nurse do?
Canadian Fundamentals of Nursing 6th Edition Potter Test Bank a. b. c. d.
Assess the patient at least every 4 hours. Inspect chest wall movements during the expiratory cycle only. Auscultate the entire lung region to assess lung sounds. Focus auscultation on the upper lung fields.
ANS: C
The nurse auscultates the entire lung region to identify diminished breath sounds, crackles, or wheezes. A respiratory assessment is performed at least every 2 hours for patients with restricted activity. The nurse inspects chest wall movements during the full inspiratory– expiratory cycle. Auscultation is focused on the dependent lung fields because pulmonary secretions tend to collect in these lower regions. DIF: Apply REF: 1260 OBJ: Assess for correct and impaired body alignment and mobility. TOP: Implementation MSC: CPNRE: Foundations of Practice 18. The nurse is aware that patients who are immobile are at increased risk of developing deep
vein thromboses (DVTs). Because of this, what action does the nurse take? a. Make sure that elastic stockings are not removed. b. Measure the calf circumference of both legs. c. Dorsiflex the foot while assessing for patient discomfort. d. Measure both ankles to determine size. ANS: B
The nurse measures bilateral calf circumference and records it daily as an assessment for deep vein thrombosis (DVT). Homans’s sign, or calf pain on dorsiflexion of the foot, should not be assessed in patients when a DVT is suspected; it is no longer considered a reliable indicator in assessing for DVT, and it is pNres entSI inN oG ther onCditi UR TBc. OMons. The patient’s elastic stockings or sequential compression devices, or both, should be removed every 8 hours and the calves observed for redness, warmth, and tenderness. Bilateral calf (not ankle) circumferences should be measured daily to detect unilateral increases that may be an early indication of thrombosis. DIF: Apply REF: 1261 OBJ: Assess for correct and impaired body alignment and mobility. TOP: Implementation MSC: CPNRE: Foundations of Practice 19. A patient is admitted to the medical unit after a CVA. There is evidence of left-sided
hemiparesis, and the nurse will be following up on range-of-motion (ROM) and other exercises performed in physiotherapy. Which of the following principles of ROM exercises does the nurse correctly teach the patient and family members? a. Flex the joint to the point of discomfort. b. Work from proximal to distal joints. c. Move the joints quickly. d. Provide support to the extremity. ANS: D
While the patient is performing ROM exercises, support should be provided to the extremity. The joint should be flexed to the point of resistance, not to the point of discomfort. When performing ROM exercises, the patient should begin at distal joints and work toward proximal joints. Joints should be moved slowly through the ROM. Quick movement could cause injury. DIF: Apply
REF: 1282-1284
Canadian Fundamentals of Nursing 6th Edition Potter Test Bank OBJ: Describe essential techniques when assisting with active and passive range-of-motion (ROM) exercises, assisting a patient to move up in bed, repositioning a patient, assisting a patient to a sitting position, and transferring a patient from a bed to a chair or from a bed to a stretcher. TOP: Implementation MSC: CPNRE: Foundations of Practice 20. The nurse is caring for an older patient with the diagnosis of urinary tract infection (UTI). The
patient is confused and agitated. It is important for the nurse to realize that confusion in older people is which of the following? a. Not a normal expectation. b. Purely psychological in origin. c. Not a common manifestation with UTIs. d. Acceptable and needs no further assessment. ANS: A
Acute confusion in older persons is not normal; a thorough nursing assessment is the priority. Abrupt changes in personality often have a physiological cause such as surgery, a medication reaction, a pulmonary embolus, or an acute infection. For example, the primary symptom of compromised older patients with an acute urinary tract infection or fever is confusion. Identifying confusion is an important component of the nurse’s assessment. DIF: Understand REF: 1262 OBJ: Identify changes in physiological and psychosocial function associated with immobility. TOP: Assessment MSC: CPNRE: Foundations of Practice 21. In preparing to create a nursing diagnosis for a patient who is immobile, what is important for
the nurse to understand? a. Physiological issues should be the major focus. b. Psychosocial issues shouN ldUbR eS thI em ajoTrBfo.cC usO .M NG c. Developmental issues should be the major focus. d. All dimensions are important to health. ANS: D
Often the physiological dimension is the major focus of nursing care for patients with impaired mobility. Thus the psychosocial and developmental dimensions are neglected. However, all dimensions are important to health. DIF: Understand REF: 1263 OBJ: Formulate appropriate nursing diagnoses for impaired mobility. TOP: Assessment MSC: CPNRE: Foundations of Practice 22. Many patients who experience an alteration in mobility have one or more nursing diagnoses.
For whom would the nurse use the diagnosis Impaired physical mobility? a. A patient who is not completely immobile. b. A patient who is completely immobile. c. A patient who is at risk for multisystem problems. d. A patient who is at risk for single-system involvement. ANS: A
Canadian Fundamentals of Nursing 6th Edition Potter Test Bank The diagnosis of Impaired physical mobility applies to the patient who has some limitation but is not completely immobile. The diagnosis of Risk for disuse syndrome applies to the patient who is immobile and at risk for multisystem problems because of inactivity. Beyond these diagnoses, the list of potential diagnoses is extensive because immobility affects multiple body systems. DIF: Apply REF: 1263 OBJ: Formulate appropriate nursing diagnoses for impaired mobility. TOP: Implementation MSC: CPNRE: Foundations of Practice 23. The patient has the nursing diagnosis of Impaired physical mobility related to pain in the left
shoulder. This diagnosis means that the nurse should do what? a. Encourage the patient to do self-care. b. Keep the patient as mobile as possible. c. Encourage the patient to perform ROM exercises. d. Assist the patient with comfort measures. ANS: D
The diagnosis related to pain requires the nurse to assist the patient with comfort measures so that the patient is then willing and better able to move. Pain must be controlled before the patient will be willing to initiate movement. The diagnosis of Reluctance to initiate movement necessitates interventions aimed at keeping the patient as mobile as possible and encouraging the patient to perform self-care and ROM exercises. This cannot be accomplished until comfort is achieved. DIF: Apply REF: 1263 OBJ: Formulate appropriate nursing diagnoses for impaired mobility. NURSINGMTSBC. TOP: Implementation : C CPONM RE: Foundations of Practice 24. In developing an individualized plan of care for a patient, what is important for the nurse to
do? a. b. c. d.
Set goals that are a little beyond the capabilities of the patient. Use his or her judgement and not be swayed by family desires. Establish goals that are measurable and realistic. Explain that without taking alignment risks, there can be no progress.
ANS: C
The nurse must develop an individualized plan of care for each nursing diagnosis and must set goals that are individualized, realistic, and measurable. The nurse should set realistic expectations for care and should include the patient and family when possible. The goals focus on preventing problems or risks to body alignment and mobility. DIF: Apply REF: 1264 OBJ: Develop individualized nursing care plans for patients with impaired mobility. TOP: Implementation MSC: CPNRE: Foundations of Practice 25. When the nurse creates a plan of care for a patient who is experiencing alterations in mobility,
which of the following is true? a. The nurse cannot delegate interventions to nursing assistive personnel. b. The nurse is solely responsible for modifying activities of daily living (ADLs). c. The nurse consults other health care team members to help plan therapy.
Canadian Fundamentals of Nursing 6th Edition Potter Test Bank d. The nurse consults wound care specialists only when wounds are apparent. ANS: C
The nurse should collaborate with other health care team members such as physiotherapists or occupational therapists when considering mobility needs. Nurses often delegate some interventions to unregulated care providers. Unregulated care providers may perform such tasks as turning and positioning patients, applying elastic stockings, and helping patient use the incentive spirometer. Occupational therapists are a resource for planning ADLs that patients need to modify or relearn. It is especially important in priority setting to account for potential complications. Many times, actual problems such as pressure ulcers are addressed only after they develop. They should be addressed before they develop. DIF: Apply REF: 1282 OBJ: Develop individualized nursing care plans for patients with impaired mobility. TOP: Implementation MSC: CPNRE: Collaborative Practice 26. The patient is being admitted to the neurological unit with the diagnosis of stroke. When
should the nurse begin discharge planning? a. At the time of admission. b. The day before the patient is to be discharged. c. As soon as the patient’s discharge destination is known. d. When outpatient therapy will no longer be needed. ANS: A
Discharge planning begins when a patient enters the health care system. In anticipation of the patient’s discharge from an institution, the nurse makes appropriate referrals or consults a case manager or a discharge planner to ensure that the patient’s needs will be met at home. Referrals to home care or outN paUtiR enSt I thN erG apTyBa. reCoO ftM en needed. DIF: Apply REF: 1264 OBJ: Develop individualized nursing care plans for patients with impaired mobility. TOP: Implementation MSC: CPNRE: Foundations of Practice 27. Of the following nursing goals, which is the most appropriate for a patient who has had a total
hip replacement? a. The patient will walk 305 m (1000 feet), using her walker, by the time of discharge. b. The patient will ambulate by the time of discharge. c. The patient will ambulate briskly on the treadmill by the time of discharge. d. The nurse will assist the patient to ambulate in the hall. ANS: A
A goal of walking 305 m (1000 feet) with the use of her walker by the time of discharge” is individualized, realistic, and measurable. “The patient will ambulate by the time of discharge” is not measurable because it does not specify the distance. Even though the patient will ambulate, this does not quantify how far. “Ambulating briskly on a treadmill” is not realistic for this patient. The last option focuses on the nurse, not the patient, and is not measurable. DIF: Analyze REF: 1264 OBJ: Develop individualized nursing care plans for patients with impaired mobility. TOP: Assessment MSC: CPNRE: Foundations of Practice
Canadian Fundamentals of Nursing 6th Edition Potter Test Bank 28. Prevention of plantar flexion through the use of pillows to support the lower legs and elevate
the toes is a priority intervention for which patient? a. A 54-year-old with osteoarthritis in all lower extremity joints. b. A 25-year-old with a fractured pelvis as a result of a motorcycle accident. c. A 78-year-old with left-sided paralysis caused by a CVA. d. A 15-year-old who has been comatose for 2 years as a result of a head injury sustained from a fall off a roof. ANS: C
The patient who has suffered a CVA with resulting left-sided paralysis (hemiplegia) is at risk for footdrop. Neither the 54-year-old patient with osteoarthritis in all lower extremity joints nor the 25-year-old patient with a fractured pelvis as a result of a motorcycle accident would have the kind of nerve damage that causes footdrop. There is little chance that the 15-year-old patient who has been comatose for 2 years as a result of a head injury sustained from a fall off a roof will ever be capable of mobility. DIF: Analyze REF: 1251 | 1252 OBJ: Develop individualized nursing care plans for patients with impaired mobility. TOP: Implementation MSC: CPNRE: Foundations of Practice 29. The patient is immobilized after undergoing hip replacement surgery. Which of the following
would place the patient at risk for hemorrhage? a. Thick, tenacious pulmonary secretions. b. Low-molecular-weight heparin doses to prevent DVT. c. Sequential compression devices wrapped around the legs to prevent DVT formation. d. Elastic stockings to promote venous return. ANS: B
N R I G B.C M U S N T O
Heparin and low-molecular-weight heparin are the drugs most widely used in the prophylaxis of DVT. Because bleeding is a potential side effect of these medications, the nurse must assess the patient continually for signs of bleeding. Pulmonary secretions that become thick and tenacious are difficult to remove and are a sign of inadequate hydration, but not of bleeding. Sequential compression devices consist of sleeves or stockings made of fabric or plastic that are wrapped around the leg and are secured with Velcro. They decrease venous stasis by increasing venous return through the deep veins of the legs. They do not usually cause bleeding. Elastic stockings also aid in maintaining external pressure on the muscles of the lower extremities and in promoting venous return. They do not usually cause bleeding. DIF: Analyze REF: 1265, Box 45-7 | 1269 OBJ: Identify changes in physiological and psychosocial function associated with immobility. TOP: Implementation MSC: CPNRE: Foundations of Practice 30. The nurse needs to transfer the patient from the bed to the chair. What should the nurse
remember? a. To avoid using a transfer or gait belt around the patient’s waist before transfer. b. Not to allow the patient to help in any way because resistance can lead to injury. c. To assess for the need of a mechanical lift and help. d. To ensure that the patient has stockings on his feet for transfer. ANS: C
Canadian Fundamentals of Nursing 6th Edition Potter Test Bank Careful assessment of the patient’s ability to assist in the positioning technique to be used is extremely important. The use of a mechanical lift should be considered. The nurse’s role in assisting the patient to a sitting position is to guide and instruct. If the patient can bear weight and move to a sitting position independently, he or she should be allowed to do so, and the nurse may offer assistance. A transfer belt maintains stability of the patient during transfer and reduces risk for falls. The nurse must ensure that the patient has stable nonskid shoes on the feet. DIF: Apply REF: 1281 OBJ: Describe essential techniques when assisting with active and passive range-of-motion (ROM) exercises, assisting a patient to move up in bed, repositioning a patient, assisting a patient to a sitting position, and transferring a patient from a bed to a chair or from a bed to a stretcher. TOP: Implementation MSC: CPNRE: Foundations of Practice 31. The nurse is caring for a patient with a spinal cord injury and notices that the patient’s hips
have a tendency to rotate externally when the patient is supine. To help prevent injury secondary to this rotation, what can the nurse use? a. A trochanter roll. b. The trapeze bar. c. Hand rolls. d. Hand–wrist splints. ANS: A
A trochanter roll prevents external rotation of the hips when the patient is in a supine position. Hand rolls maintain the thumb in slight adduction and in opposition to the fingers. Hand–wrist splints are individually moulded for the patient to maintain proper alignment of the thumb and the wrist. The trapeze bar is a triangular device that hangs down from a securely fastened overhead bar that is attachedN toUtR heSbI edNfG raT mBe. . IC t aOllM ows the patient to pull with the upper extremities to raise the trunk off the bed, assists in transfer from bed to wheelchair, or helps the patient perform upper arm exercises. DIF: Remember REF: 1281 OBJ: Describe essential techniques when assisting with active and passive range-of-motion (ROM) exercises, assisting a patient to move up in bed, repositioning a patient, assisting a patient to a sitting position, and transferring a patient from a bed to a chair or from a bed to a stretcher. TOP: Implementation MSC: CPNRE: Foundations of Practice 32. The patient is unable to move himself and needs to be pulled up in bed. For this repositioning
to be done safely, what must the nurse understand? a. The procedure can be done by one person if the bed is in the flat position. b. Side rails should be in the up position to prevent the patient from falling out. c. The pillow should be placed under the patient’s head and shoulders. d. Assistive devices or additional nurses should be used. ANS: D
Canadian Fundamentals of Nursing 6th Edition Potter Test Bank Helping a patient move up in bed is not a one-person task. Doing so without help from other coworkers or without the aid of an assistive device (e.g., friction-reducing pad) is not recommended and is not considered safe for the patient or the nurse. When a patient is pulled up in bed, the bed should be flat or in a Trendelenburg position (when tolerated) for gravity assistance, and the side rails should be down. The pillow should be removed from under the patient’s head and shoulders and placed at the head of the bed to keep the patient’s head from striking the head of the bed. DIF: Understand REF: 1272-1280, Skill 45-1 OBJ: Describe essential techniques when assisting with active and passive range-of-motion (ROM) exercises, assisting a patient to move up in bed, repositioning a patient, assisting a patient to a sitting position, and transferring a patient from a bed to a chair or from a bed to a stretcher. TOP: Implementation MSC: CPNRE: Foundations of Practice 33. The nurse is caring for a patient who is immobile and needs to be turned every 2 hours. The
patient has poor lower extremity circulation, and the nurse is concerned about irritation of the patient’s toes. What is one strategy that the nurse could use? a. A foot cradle. b. A trochanter roll. c. The trapeze bar. d. Hand rolls. ANS: A
A foot cradle may be used in patients with poor peripheral circulation as a means of reducing pressure on the tips of a patient’s toes. A trochanter roll prevents external rotation of the hips when the patient is in a supine position. Hand rolls maintain the thumb in slight adduction and in opposition to the fingers. The trapeze bar is a triangular device that hangs down from a securely fastened overhead bN arUthRaS t iI sN atG tacThB ed.tC oO thM e bed frame. It allows the patient to pull with the upper extremities to raise the trunk off the bed, assists in transfer from bed to wheelchair, or helps the patient perform upper arm exercises. DIF: Apply REF: 1275, Skill 45-1 OBJ: Describe essential techniques when assisting with active and passive range-of-motion (ROM) exercises, assisting a patient to move up in bed, repositioning a patient, assisting a patient to a sitting position, and transferring a patient from a bed to a chair or from a bed to a stretcher. TOP: Implementation MSC: CPNRE: Foundations of Practice 34. In applying for a job on a nursing unit that requires frequent patient positioning, of what
should the nurse be aware? a. That nurses are at low risk for back injury. b. That nurses are especially at risk for high back injuries. c. That nurses should be aware of agency policies. d. That nurses should not need to use assistive devices. ANS: C
Nurses and other health care staff are especially at risk for injury to lumbar muscles when lifting, transferring, or positioning immobilized patients. Therefore, the nurse should be aware of agency policies and protocols that protect staff and patients from injury. According to current evidence, using mechanical or other ergonomic assistive devices is the safest way to reposition and lift patients who are unable to perform these activities themselves.
Canadian Fundamentals of Nursing 6th Edition Potter Test Bank DIF: Understand REF: 1247 OBJ: Describe essential techniques when assisting with active and passive range-of-motion (ROM) exercises, assisting a patient to move up in bed, repositioning a patient, assisting a patient to a sitting position, and transferring a patient from a bed to a chair or from a bed to a stretcher. TOP: Implementation MSC: CPNRE: Foundations of Practice 35. When the nurse is preparing a plan of care for an immobilized patient, what should the nurse
keep in mind? a. To use established expected outcomes to evaluate the patient’s response to care. b. To display an air of professional superiority when interventions are not successful. c. Never to vary from interventions that have been successful for other patients. d. To use only objective data in determining whether interventions have been successful. ANS: A
The nurse should use established expected outcomes to evaluate the patient’s response to care. The nurse should use creativity when designing new interventions to improve the patient’s mobility status and should display humility when identifying interventions that were not successful. The nurse should ask the patient whether his or her expectations of care are being met and should use objective data to determine the success of interventions. DIF: Apply REF: 1295, Figure 45-22 OBJ: Evaluate the nursing care plan for maintaining body alignment and mobility. TOP: Implementation MSC: CPNRE: Foundations of Practice 36. It has been determined that each of the following patients is at risk for falling. Which one
requires the nurse’s priority for ambulation? a. A 16-year-old with a spraN ine d an UR SIkleNGbein TBg.dCisch OMarged from the emergency department. b. A 54-year-old who has taken the initial dose of an antihypertensive medication. c. A 45-year-old postoperative patient up for the first time since knee surgery. d. An 81-year-old who is asthmatic and had a hip replaced 18 months ago. ANS: C
Disease, injury, pain, physical development (e.g., age), and life changes (e.g., pregnancy) compromise the ability to remain balanced. Medications that cause dizziness and prolonged immobility also affect balance. Although all of the options represent a potential risk for falling, the postoperative patient has both prolonged immobility and physical injury (surgery), and so is at greatest risk. The 16-year-old with a sprained ankle being discharged from the emergency department, the 54-year-old who has taken the initial dose of an antihypertensive medication, and the 81-year-old who is asthmatic and had a hip replaced 18 months ago do not receive priority for ambulation. DIF: Analyze REF: 1251 OBJ: Develop individualized nursing care plans for patients with impaired mobility. TOP: Implementation MSC: CPNRE: Foundations of Practice 37. The nurse needs to reposition a 136.1-kg (300-pound) patient. Which of the following
strategies is most likely to prevent back injury? a. Turning the patient alone, using the lift pad and applying pillows. b. Putting the bed in Trendelenburg and pulling from the head of the bed.
Canadian Fundamentals of Nursing 6th Edition Potter Test Bank c. Assessing and obtaining the number of people needed to help. d. Bending at the waist and pulling the lift pad, using the arms. ANS: C
The nurse must assess and obtain the number of people needed; to prevent injury, the task should not be started until it can be completed safely. The nurse should assess the situation and not turn the patient alone if this cannot be done safely. The nurse’s trunk should be erect and the knees bent, so that multiple muscle groups (not just the arms) work together in a coordinated manner. This is not a one-person task: the nurse must not pull from the head of the bed. DIF: Analyze REF: 1273, Skill 45-1 OBJ: Describe essential techniques when assisting with active and passive range-of-motion (ROM) exercises, assisting a patient to move up in bed, repositioning a patient, assisting a patient to a sitting position, and transferring a patient from a bed to a chair or from a bed to a stretcher. TOP: Implementation MSC: CPNRE: Foundations of Practice 38. The nurse is caring for a patient who had a stroke that caused total paralysis of the right side.
To help maintain joint function and to prevent contractures, passive ROM exercises will be initiated. When should therapy begin? a. After the acute phase of the disease has passed. b. As soon as the ability to move is lost. c. Once the patient enters the rehabilitation unit. d. No ROM exercise is needed. ANS: B
Passive ROM exercises should begin as soon as the patient’s ability to move the extremity or joint is lost. The nurse shouldNnoR U t wa SIitNfoGrTthBe.acCute OMphase to end. It may be some time before the patient enters the rehabilitation unit, and contractures could form by then. ROM exercise is certainly needed by this patient. DIF: Understand REF: 1254 | 1284 OBJ: Develop individualized nursing care plans for patients with impaired mobility. TOP: Implementation MSC: CPNRE: Foundations of Practice 39. The nurse is admitting a patient who has had a stroke. The physician writes orders for “ROM
as needed.” What does the nurse understand about this situation? a. The nurse will have to move all the patient’s extremities. b. The patient is unable to move his extremities. c. Further assessment of the patient is needed. d. The patient needs to restrict his mobility as much as possible. ANS: C
The patient must be assessed further. Some patients are able to move some joints actively, whereas the nurse must help other joints with passive movement. With a weak patient, the nurse may have to support an extremity while the patient performs the movement. In general, exercises need to be as active as the patient’s health and mobility allow. DIF: Understand REF: 1253, Figure 45-5 OBJ: Develop individualized nursing care plans for patients with impaired mobility. TOP: Implementation MSC: CPNRE: Foundations of Practice
Canadian Fundamentals of Nursing 6th Edition Potter Test Bank 40. While performing passive ROM exercises, the nurse stands at the side of the bed closest to the
joint being exercised and does what else? a. Forces the joint just a bit beyond the point of resistance. b. Moves the joint until the patient complains of pain. c. Repeats each movement twice. d. Carries out movements slowly and smoothly. ANS: D
The nurse carries out movements slowly and smoothly, just to the point of resistance. ROM exercises should not cause pain. A joint should never be forced beyond its capacity. Each movement needs to be repeated five times during the session. DIF: Apply REF: 1284 OBJ: Develop individualized nursing care plans for patients with impaired mobility. TOP: Implementation MSC: CPNRE: Foundations of Practice 41. Two nurses are standing on opposite sides of the bed to move the patient up in bed with a
drawsheet. Which of the following describes the correct position for the nurses to take to safely position the patient? a. The nurses should face the patient. b. The nurses should face the direction of movement. c. The nurses should face each other. d. The nurses should face opposite the direction of movement. ANS: B
Facing the direction of movement ensures proper balance and prevents twisting of the nurses’ bodies while moving the patient. Facing the patient, facing each other, and facing opposite the direction of movement are noNt U thR eS coIrrN ecGt T poBs. itiC onOsM for the nurses to take. DIF: Apply REF: 1274, Skill 45-1 OBJ: Develop individualized nursing care plans for patients with impaired mobility. TOP: Implementation MSC: CPNRE: Foundations of Practice 42. The patient has suffered a spinal cord injury and needs to be repositioned through the
log-rolling technique to keep the spinal column in straight alignment. Which of the following is the proper technique for nurses to perform log-rolling? a. Obtaining assistance from at least two or three other people. b. Having the patient reach for the opposite side rail when turning. c. Moving the top part of the patient’s torso, then the bottom part. d. Not using pillows after turning because the softness causes misalignment. ANS: A
At least three to four people are needed to perform this skill safely. The patient crosses arms on chest to prevent injury to the arms. The patient should be moved as one unit in a smooth, continuous motion on the count of three. The nurses gently let the patient as a unit lean back toward pillows for support. DIF: Apply REF: 1274 | 1279, Skill 45-1 OBJ: Describe essential techniques when assisting with active and passive range-of-motion (ROM) exercises, assisting a patient to move up in bed, repositioning a patient, assisting a patient to a sitting position, and transferring a patient from a bed to a chair or from a bed to a stretcher. TOP: Implementation MSC: CPNRE: Foundations of Practice
Canadian Fundamentals of Nursing 6th Edition Potter Test Bank
43. The nurse is caring for a patient with the nursing diagnosis of Impaired physical mobility. The
nurse needs to be alert for which one of the following potential complications? a. Pulmonary emboli. b. Diarrhea. c. Somnolence. d. Increased socialization. ANS: A
Immobility leads to complications such as pulmonary emboli or pneumonia. Other possible diagnoses include impaired skin integrity. Constipation, insomnia, and social isolation are complications more common than somnolence or increased socialization. DIF: Apply REF: 1263 OBJ: Formulate appropriate nursing diagnoses for impaired mobility. TOP: Implementation MSC: CPNRE: Foundations of Practice
NURSINGTB.COM
Canadian Fundamentals of Nursing 6th Edition Potter Test Bank
Chapter 46: Skin Integrity and Wound Care Potter et al: Canadian Fundamentals of Nursing, 6th Edition MULTIPLE CHOICE 1. The nurse is working on a medical-surgical unit that has been participating in a research
project associated with pressure injuries. The nurse recognizes that the risk factors that predispose a patient to pressure injury development include which of the following? a. A diet low in calories and fat. b. Alteration in level of consciousness. c. Shortness of breath. d. Muscular pain. ANS: B
Patients who are confused or disoriented or who have changing levels of consciousness are unable to protect themselves. The patient may feel the pressure but may not understand what to do to relieve the discomfort or to communicate that he or she is feeling discomfort. Impaired sensory perception, impaired mobility, shear, friction, and moisture are other predisposing factors. Shortness of breath, muscular pain, and a diet low in calories and fat are not included among the predisposing factors. DIF: Remember REF: 1291 | 1292 OBJ: Discuss the risk factors that contribute to pressure injury formation. TOP: Assessment MSC: CPNRE: Foundations of Practice 2. The nurse is caring for a patient who was involved in an automobile accident 2 weeks ago.
N R I G B.C M
The patient sustained a head inU jurySandNis T unconscOious. What is the major element involved in the development of a pressure injury? a. Pressure. b. Resistance. c. Stress. d. Weight. ANS: A
Pressure is the main element that causes pressure injury. Three pressure-related factors contribute to pressure injury development: pressure intensity, pressure duration, and tissue tolerance. When the intensity of the pressure exerted on the capillary exceeds 12 to 32 mm Hg, this occludes the vessel, causing ischemic injury to the tissues it normally feeds. High pressure over a short time and low pressure over a long time cause skin breakdown. Resistance (the ability to remain unaltered by the damaging effect of something), stress (worry or anxiety), and weight (individuals of all sizes, shapes, and ages acquire skin breakdown) are not major causes of pressure injury. DIF: Remember REF: 1290 OBJ: Discuss the risk factors that contribute to pressure injury formation. TOP: Assessment MSC: CPNRE: Foundations of Practice 3. Which nursing observation would indicate that the patient was at risk for pressure injury
formation? a. The patient ate two thirds of breakfast.
Canadian Fundamentals of Nursing 6th Edition Potter Test Bank b. The patient has fecal incontinence. c. The patient has a raised red rash on the right shin. d. The patient’s capillary refill is less than 2 seconds. ANS: B
The presence and duration of moisture on the skin increase the risk of pressure injury formation by making it susceptible to injury. Moisture can originate from wound drainage, excessive perspiration, and fecal or urinary incontinence. Bacteria and enzymes in the stool can enhance the opportunity for skin breakdown because the skin is moistened and softened, which causes maceration. Eating a balanced diet is important for nutrition, but eating just two thirds of the meal does not indicate that the individual is at risk. A raised red rash on the leg again is a concern and can affect the integrity of the skin, but it is located on the shin, which is not an area at high risk for skin breakdown. Pressure can influence capillary refill, leading to skin breakdown, but a capillary response of less than 2 seconds is within normal limits. DIF: Understand REF: 1292 OBJ: Discuss the risk factors that contribute to pressure injury formation. TOP: Implementation MSC: CPNRE: Foundations of Practice 4. The wound care nurse visits a patient in the long-term care unit. The patient has a stage 3
pressure injury. The wound seems to be healing, and healthy tissue is observed. How would the nurse stage this pressure injury? a. Stage 1 pressure injury. b. Healing stage 2 pressure injury. c. Healing stage 3 pressure injury. d. Stage 3 pressure injury. ANS: C
NURSINGTB.COM
When a pressure injury has been staged and is beginning to heal, the pressure injury keeps the same stage and is labelled with the words “healing stage.” Once a pressure injury has been staged, the stage endures even as the pressure injury heals. This pressure injury was labelled stage 3; it cannot return to a previous stage such as stage 1 or 2. This pressure injury is healing, so it is no longer labelled simply stage 3. DIF: Remember TOP: Assessment
REF: 1299 OBJ: Recognize the stages of pressure injury. MSC: CPNRE: Foundations of Practice
5. The nurse is admitting an older patient from a nursing home. During the assessment, the nurse
notes a shallow open pressure injury without slough on the right heel of the patient. How should this pressure injury be staged? a. 1. b. 2. c. 3. d. 4. ANS: B
Canadian Fundamentals of Nursing 6th Edition Potter Test Bank This would be a stage 2 pressure injury because it manifests as partial-thickness skin loss involving epidermis, dermis, or both. The pressure injury is superficial and manifests clinically as an abrasion, blister, or shallow crater. In a stage 1 injury, skin is intact with nonblanchable redness over a bony prominence. With a stage 3 pressure injury, subcutaneous fat may be visible, but bone, tendon, and muscles are not exposed. Stage 4 involves full-thickness tissue loss with exposed bone, tendon, or muscle. DIF: Remember REF: 1299-1301, Table 46-3 OBJ: Recognize the stages of pressure injury. MSC: CPNRE: Foundations of Practice
TOP: Assessment
6. The nurse is completing a skin assessment on a patient with darkly pigmented skin. Which of
the following would be used first to assist in staging a pressure injury on this patient? a. Cotton-tipped applicator. b. Disposable measuring tape. c. Sterile gloves. d. Halogen light. ANS: D
When a patient with darkly pigmented skin undergoes a skin assessment, proper lighting is essential to accurately complete the first step in assessment—inspection—and the whole assessment process. Natural light or a halogen light is recommended. Fluorescent light sources can produce blue tones on darkly pigmented skin and can interfere with an accurate assessment. Other items that could possibly be used during the assessment include gloves for infection control, a disposable measuring device to measure the size of the wound, and a cotton-tipped applicator to measure the depth of the wound, but these items not the first item used.
NURSINGTB.COM
DIF: Apply REF: 1291, Box 46-2 OBJ: Recognize the stages of pressure injury. MSC: CPNRE: Foundations of Practice
TOP: Assessment
7. The nurse is caring for a patient with a stage 4 pressure injury. The nurse recalls that a
pressure injury takes time to heal and that the healing process is an example of which of the following? a. Primary intention. b. Partial-thickness wound repair. c. Full-thickness wound repair. d. Tertiary intention. ANS: C
Pressure injuries are full-thickness wounds that extend into the dermis and heal by scar formation because the deeper structures do not regenerate; hence the need for full-thickness repair. The full-thickness repair has three phases: inflammatory, proliferative, and remodelling. A wound heals by primary intention when wounds such as surgical wounds have little tissue loss; the skin edges are approximated or closed; and the risk for infection is low. Partial-thickness repairs are done on partial-thickness wounds that are shallow, involving loss of the epidermis and maybe partial loss of the dermis. These wounds heal by regeneration because the epidermis regenerates. In tertiary intention, a wound is left open for several days, and then the wound edges are approximated; wound closure is delayed until risk of infection is resolved.
Canadian Fundamentals of Nursing 6th Edition Potter Test Bank
DIF: Understand TOP: Planning
REF: 1302 | 1303 OBJ: Discuss the normal process of wound healing. MSC: CPNRE: Foundations of Practice
8. The nurse is caring for a patient with a large abrasion from a motorcycle accident. The nurse
recalls that if the wound is kept moist, it can resurface in how long? a. 4 days. b. 2 days. c. 1 day. d. 7 days. ANS: A
A partial-thickness wound repair has three compartments: the inflammatory response, epithelial proliferation and migration, and re-establishment of the epidermal layers. Epithelial proliferation and migration start at all edges of the wound, allowing for quick resurfacing. Epithelial cells begin to migrate across the wound bed soon after the wound occurs. A wound left open to air resurfaces within 6 to 7 days, whereas a wound that is kept moist can resurface in 4 days. One or 2 days is too soon for this process to occur, moist or dry. DIF: Remember TOP: Planning
REF: 1303 OBJ: Discuss the normal process of wound healing. MSC: CPNRE: Foundations of Practice
9. The nurse is caring for a patient who is experiencing a full-thickness repair. The nurse would
expect to see which of the following in this type of repair? a. Eschar. b. Slough. c. Granulation. NURSINGTB.COM d. Purulent drainage. ANS: C
Granulation tissue is red, moist tissue composed of new blood vessels, the presence of which indicates progression toward healing. Soft yellow or white tissue is characteristic of slough, which must be removed for the wound to heal. Black or brown necrotic tissue is called eschar and also must be removed for a wound to heal. Purulent drainage is indicative of an infection and must be resolved for the wound to heal. DIF: Understand TOP: Assessment
REF: 1303 OBJ: Discuss the normal process of wound healing. MSC: CPNRE: Foundations of Practice
10. The nurse is caring for a patient who has undergone a laparoscopic appendectomy. The nurse
recalls that this type of wound heals by which process? a. Tertiary intention. b. Secondary intention. c. Partial-thickness repair. d. Primary intention. ANS: D
Canadian Fundamentals of Nursing 6th Edition Potter Test Bank A clean surgical incision is an example of a wound with little loss of tissue that heals with primary intention. The skin edges are approximated or closed, and the risk for infection is low. Partial-thickness repairs are done on partial-thickness wounds that are shallow, involving loss of the epidermis and maybe partial loss of the dermis. These wounds heal by regeneration because the epidermis regenerates. In tertiary intention, a wound is left open for several days, and then the wound edges are approximated; wound closure is delayed until the risk of infection is resolved. A wound involving loss of tissue such as a burn or a pressure injury or laceration heals by secondary intention. The wound is left open until it becomes filled with scar tissue. It takes longer for a wound to heal by secondary intention; thus the chance of infection is greater. DIF: Remember REF: 1301-1303 OBJ: Describe the differences between wounds that heal by primary and secondary intention. TOP: Planning MSC: CPNRE: Foundations of Practice 11. The nurse is caring for a patient in the burn unit. The nurse recalls that this type of wound
heals by which process? a. Tertiary intention. b. Secondary intention. c. Partial-thickness repair. d. Primary intention. ANS: B
A wound involving loss of tissue such as a burn or a pressure injury or laceration heals by secondary intention. The wound is left open until it becomes filled with scar tissue. It takes longer for a wound to heal by secondary intention; thus, the chance of infection is greater. A clean surgical incision is an example of a wound with little loss of tissue that heals by primary N R I Gd B.C M intention. The skin edges are apUproSximNateT or cloO sed, and the risk for infection is low. Partial-thickness repairs are done on partial-thickness wounds that are shallow, involving loss of the epidermis and maybe partial loss of the dermis. These wounds heal by regeneration because the epidermis regenerates. In tertiary intention, a wound is left open for several days, and then the wound edges are approximated; wound closure is delayed until the risk of infection is resolved. DIF: Remember REF: 1301-1303 OBJ: Describe the differences between wounds that heal by primary and secondary intention. TOP: Planning MSC: CPNRE: Foundations of Practice 12. Which nursing observation would indicate that a wound healed by secondary intention? a. Minimal scar tissue. b. Minimal loss of tissue function. c. Permanent dark redness at site. d. Scarring can be severe. ANS: D
A wound healing by secondary intention takes longer than one healing by primary intention. The wound is left open until it becomes filled with scar tissue. If the scarring is severe, loss of function is often permanent. Wounds that heal by primary intention heal quickly with minimal scarring. Scar tissue contains few pigmented cells and has a lighter colour than normal skin. DIF: Understand
REF: 1301-1303
Canadian Fundamentals of Nursing 6th Edition Potter Test Bank OBJ: Describe the differences between wounds that heal by primary and secondary intention. TOP: Assessment MSC: CPNRE: Foundations of Practice 13. The nurse is caring for a patient who has undergone a total hysterectomy. Which nursing
observation would indicate that the patient was experiencing a complication of wound healing? a. The incision site has started to itch. b. The incision site is approximated. c. The patient has pain at the incision site. d. The incision has a mass, bluish in colour. ANS: D
A hematoma is a localized collection of blood underneath the tissues. It appears as swelling; a change in colour, sensation, or warmth; or a mass that often takes on a bluish discoloration. A hematoma near a major artery or vein is dangerous because it can put pressure on the vein or artery and obstruct blood flow. Itching of an incision site can be associated with clipping of hair, dressings, or possibly the healing process. Incisions should be approximated with edges together. After surgery, when nerves in the skin and tissues have been traumatized by the surgical procedure, it is expected that the patient would experience pain. DIF: Understand TOP: Assessment
REF: 1306 | 1308 OBJ: Describe the complications of wound healing. MSC: CPNRE: Foundations of Practice
14. Which of these findings if seen in a postoperative patient should the nurse associate with
dehiscence? a. Complaint by patient that something has given way. b. Protrusion of visceral organs through a wound opening. c. Chronic drainage of fluidNthUrR ouSgI hN thG eT inB ci. sioCnOsM ite. d. Drainage that is malodorous and purulent. ANS: A
Dehiscence is a development in which a wound fails to heal properly and the layers of skin and tissue separate. It often involves abdominal surgical wounds and occurs after a sudden strain such as coughing, vomiting, or sitting up in bed. Affected patients often report feeling as though something has given way. In evisceration, vital organs protrude through a wound opening. A fistula is an abnormal passage between two organs or between an organ and the outside of the body that can be characterized by chronic drainage of fluid. Infection is characterized by drainage that is malodorous and purulent. DIF: Understand TOP: Assessment
REF: 1309 OBJ: Describe the complications of wound healing. MSC: CPNRE: Foundations of Practice
15. A patient has developed a pressure injury. What laboratory data would be important to gather? a. Serum albumin level. b. Creatine kinase level. c. Vitamin E level. d. Potassium level. ANS: A
Canadian Fundamentals of Nursing 6th Edition Potter Test Bank Normal wound healing necessitates proper nutrition. Serum proteins are biochemical indicators of malnutrition, and serum albumin is probably the most frequently measured of these parameters. The best measurement of nutritional status is prealbumin level because it reflects not only what the patient has ingested but also what the body has absorbed, digested, and metabolized. Measurement of creatine kinase helps in the diagnosis of myocardial infarctions and has no known role in wound healing. Potassium is a major electrolyte that helps to regulate metabolic activities, cardiac muscle contraction, skeletal and smooth muscle contraction, and transmission and conduction of nerve impulses. Vitamin E is a fat-soluble vitamin that prevents the oxidation of unsaturated fatty acids. It is believed to reduce the risk of coronary artery disease and cancer. Vitamin E has no known role in wound healing. DIF: Understand REF: 1292 OBJ: Explain the factors that impede or promote wound healing. TOP: Assessment MSC: CPNRE: Foundations of Practice 16. Which of the following would be the most important piece of assessment data to gather with
regard to wound healing? a. Muscular strength assessment. b. Sleep assessment. c. Pulse oximetry assessment. d. Sensation assessment. ANS: C
Oxygen fuels the cellular functions essential to the healing process; the ability to perfuse tissues with adequate amounts of oxygenated blood is critical in wound healing. Blood flow through the pulmonary capillaries provides red blood cells for oxygen attachment. Oxygen diffuses from the alveoli into the pulmonary blood; most of the oxygen attaches to N heRrSedIbNloo GTdB.C M hemoglobin molecules within tU cells.O Red blood cells carry oxygenated hemoglobin molecules through the left side of the heart and out to the peripheral capillaries, where the oxygen detaches, depending on the needs of the tissues. Pulse oximetry measures the oxygen saturation of blood. Assessment of muscular strength and sensation, although useful for fitness and mobility testing, does not provide any data with regard to wound healing. Sleep, although important for rest and for integration of learning and restoration of cognitive function, does not provide any data with regard to wound healing. DIF: Apply REF: 1292 OBJ: Explain the factors that impede or promote wound healing. TOP: Assessment MSC: CPNRE: Foundations of Practice 17. The nurse is caring for a patient with a healing stage 3 pressure injury. Upon entering the
room, the nurse notices an odour and observes a purulent discharge, along with increased redness at the wound site. What is the next best step for the nurse? a. Complete the head-to-toe assessment, and include current treatment, vital signs, and laboratory results. b. Notify the charge nurse about the change in status and the potential for infection. c. Notify the physician by utilizing Situation, Background, Assessment, and Recommendation (SBAR). d. Notify the wound care nurse about the change in status and the potential for infection. ANS: A
Canadian Fundamentals of Nursing 6th Edition Potter Test Bank The patient is showing signs and symptoms associated with infection in the wound. It is serious and needs treatment but is not a life-threatening emergency, for which care would be needed immediately to prevent long-term consequences. The nurse should complete the assessment; gather all data such as current treatment modalities, medications, vital signs (including temperature), and laboratory results such as the most recent complete blood cell count or white cell count. The nurse can then notify the physician and receive treatment orders for the patient. It is important to notify the charge nurse and consult the wound nurse on the patient’s status and on any new orders. DIF: Apply REF: 1306 | 1307 OBJ: Explain the factors that impede or promote wound healing. TOP: Implementation MSC: CPNRE: Foundations of Practice 18. The nurse is collaborating with the dietitian in treatment of a patient with a stage 3 pressure
injury. After the collaboration, the nurse orders a meal plan that includes increased levels of what? a. Fat. b. Carbohydrates. c. Protein. d. Vitamin E. ANS: C
Protein needs are especially increased in supporting the activity of wound healing. The physiological processes of wound healing depend on the availability of protein, vitamins (especially A and C), and the trace minerals of zinc and copper. A balanced diet of fat and carbohydrates, along with protein, vitamins, and minerals, is needed in any diet. Wound healing does not require increased amounts of fats or carbohydrates. Vitamin E has no known N R I G B.C M role in wound healing. U S N T O DIF: Apply REF: 1317 OBJ: Explain the factors that impede or promote wound healing. TOP: Implementation MSC: CPNRE: Foundations of Practice 19. The nurse is completing an assessment on an individual who has a stage 4 pressure injury. The
wound is malodorous, and a drain is currently in place. The nurse determines that the patient is experiencing problems with self-concept when the patient states which of the following? a. “I think I will be ready to go home early next week.” b. “I am so weak and tired; I want to feel better.” c. “I am ready for my bath and linen change as soon as possible.” d. “I am hoping there will be something good for dinner tonight.” ANS: C
The patient’s psychological response to any wound is part of the nurse’s assessment. Body image changes can influence self-concept. Factors that affect the patient’s perception of the wound include the presence of scars, drains, odour from drainage, and temporary or permanent prosthetic devices. This patient’s wound is malodorous, and a drain is in place. By asking for a bath and change in linens, the patient is indicating that he or she may be concerned about the smell in the room. Statements about wanting to feel better, going home, and wondering about what is for dinner could be interpreted as positive statements that indicate progress along the health journey.
Canadian Fundamentals of Nursing 6th Edition Potter Test Bank DIF: Analyze REF: 1292 | 1293 OBJ: Explain the factors that impede or promote wound healing. TOP: Implementation MSC: CPNRE: Foundations of Practice 20. A patient presents to the emergency department with a laceration of the right forearm caused
by a fall. After determining that the patient is stable, what is the next best step? a. Inspect the wound for bleeding. b. Inspect the wound for foreign bodies. c. Determine the size of the wound. d. Determine the need for a tetanus antitoxin injection. ANS: A
After determining that a patient’s condition is stable, the nurse should inspect the wound for bleeding. An abrasion will have limited bleeding, a laceration can bleed more profusely, and a puncture wound bleeds in relation to the size and depth of the wound. The nurse should first address any bleeding issues. Then the nurse inspects the wound for foreign bodies; traumatic wounds are dirty and may need to be addressed. A large open wound may expose bone or tissue and be protected, or the wound may need suturing. When the wound is caused by a dirty penetrating object, the nurse should determine the need for a tetanus vaccination. DIF: Apply REF: 1308 | 1309 OBJ: Describe the differences in nursing care for acute and chronic wounds. TOP: Implementation MSC: CPNRE: Foundations of Practice 21. The nurse is caring for a patient on the medical-surgical unit with a wound that has a drain
and a dressing that needs changing. Which of these actions should the nurse take first? a. Don sterile gloves. b. Provide analgesic medicaN tiU onRs S asIoN rdGeT reB d..COM c. Avoid accidentally removing the drain. d. Gather supplies. ANS: B
Because removal of dressings is painful, it often helps to give the patient an analgesic at least 30 minutes before the wound is exposed and the dressing is changed. The sequence of the next events includes gathering supplies for the dressing change, donning gloves, and avoiding the accidental removal of the drain during the procedure. DIF: Apply REF: 1319, Skill 46-3 | 1326 OBJ: Describe the differences in nursing care for acute and chronic wounds. TOP: Implementation MSC: CPNRE: Foundations of Practice 22. The nurse is caring for a patient who has a wound drain with a collection device. The nurse
notices that the collection device has a sudden decrease in drainage. What would be the nurse’s next best step? a. Remove the drain; a drain is no longer needed. b. Call the physician; a blockage is present in the tubing. c. Call the charge nurse to look at the drain. d. As long as the evacuator is compressed, do nothing. ANS: B
Canadian Fundamentals of Nursing 6th Edition Potter Test Bank Because a drainage system needs to be patent, look for drainage flow through the tubing, as well as around the tubing. A sudden decrease in drainage through the tubing may indicate drain blockage, and the nurse must notify the physician. The health care provider determines the need for drain removal and removes drains. Notifying the charge nurse, although important for communication, is not the next step in providing care for this patient. The evacuator may be compressed when a blockage is present. DIF: Apply REF: 1307 OBJ: Describe the differences in nursing care for acute and chronic wounds. TOP: Implementation MSC: CPNRE: Collaborative Practice 23. The nurse is caring for a patient who has a stage 4 pressure injury and is awaiting plastic
surgery consultation. Which of the following specialty beds would be most appropriate? a. Standard mattress. b. Nonpowered redistribution air mattress. c. Low-air-loss therapy unit. d. Lateral rotation. ANS: B
A static air mattress, or nonpowered redistribution mattress, is utilized for the patient at high risk for skin breakdown, as in this scenario. A low-air-loss therapy unit is utilized for stage 4 pressure injury and when prevention or treatment of skin breakdown is needed. If the patient has a stage 3 or stage 4 pressure injury or a postoperative myocutaneous flap, the low-air-loss therapy unit would be an appropriate selection. A standard mattress is utilized for an individual who does not have actual or potential altered or impaired skin integrity. Lateral rotation is used for treatment and prevention of pulmonary complications associated with mobility.
NURSINGTB.COM
DIF: Understand REF: 1313 OBJ: Describe the differences in nursing care for acute and chronic wounds. TOP: Assessment MSC: CPNRE: Foundations of Practice 24. The nurse is caring for a patient with a pressure injury on the left hip. The pressure injury is
black. The nurse recognizes that the next step in caring for this patient includes which of the following? a. Monitoring of the wound. b. Irrigation of the wound. c. Debridement of the wound. d. Management of drainage. ANS: C
Débridement is the removal of nonviable necrotic tissue. Removal of necrotic tissue is necessary to rid the pressure injury of a source of infection, to enable visualization of the wound bed, and to provide a clean base for healing. A wound will not move through the phases of healing if it is infected. Irrigating the wound with noncytotoxic cleaners will not damage or kill fibroblasts and healing tissue and will help to keep the wound clean after debridment. When the nurse is treating a pressure injury, it is important to monitor and reassess the wound at least every 8 hours. Management of drainage helps keep the wound clean. DIF: Understand
REF: 1317
Canadian Fundamentals of Nursing 6th Edition Potter Test Bank OBJ: Describe the differences in nursing care for acute and chronic wounds. TOP: Planning MSC: CPNRE: Foundations of Practice 25. The nurse is caring for a patient with a healing stage 3 pressure injury. The wound is clean
and granulating. Which of the following orders would the nurse question? a. Use a low-air-loss therapy unit. b. Consult a dietitian. c. Irrigate with hydrogen peroxide. d. Utilize hydrogel dressing. ANS: C
Pressure injuries should be cleaned with noncytotoxic cleansers such as normal saline, which will not kill fibroblasts and healing tissue. Cytotoxic cleansers such as Dakin’s solution, acetic acid, povidone-iodine, and hydrogen peroxide can hinder the healing process and should not be utilized on clean, granulating wounds. Consulting a dietitian for the nutritional needs of the patient, utilizing a low-air-loss therapy unit to decrease pressure, and applying hydrogel dressings to provide a moist environment for healing are all appropriate orders. DIF: Analyze REF: 1316 OBJ: Describe the differences in nursing care for acute and chronic wounds. TOP: Implementation MSC: CPNRE: Foundations of Practice 26. The nurse is completing an assessment of the skin’s integrity, which includes which of the
following? a. Pressure points. b. All pulses. c. Breath sounds. d. Bowel sounds.
NURSINGTB.COM
ANS: A
The nurse continually assesses the skin for signs of pressure injury development. Assessment of tissue pressure damage includes visual and tactile inspection of the skin. The nurse should observe pressure points such as bony prominences and areas next to invasive treatments such as a binasal cannula and the nares. Assessment of pulses, breath sounds, and bowel sounds is part of a head-to-toe assessment and could influence the function of the body and ultimately skin integrity; however, this assessment is not a specific part of a skin assessment. DIF: Understand REF: 1296, Skill 46-1 OBJ: Complete an assessment for a patient with impaired skin integrity. TOP: Assessment MSC: CPNRE: Foundations of Practice 27. The nurse is using the Braden scale to complete a skin risk assessment. The patient has some
sensory impairment and skin that is rarely moist, walks occasionally, and has slightly limited mobility, along with excellent intake of meals and no apparent problem with friction and shear. What would be the patient’s Braden scale total score? a. 15. b. 17. c. 21. d. 23. ANS: C
Canadian Fundamentals of Nursing 6th Edition Potter Test Bank The Braden scale is composed of six subscales: sensory perception, moisture, activity, mobility, nutrition, and friction and shear. The total score ranges from 6 to 23, and a lower total score indicates a higher risk for pressure injury development. On the Braden scale, the patient receives 3 for slight sensory impairment, 4 for skin’s rarely being moist, 3 for walking occasionally, 3 for slightly limited mobility, 4 for excellent intake of meals, and 4 for no problem with friction and shear. The total score is 21. DIF: Apply REF: 1298, Table 46-2 OBJ: Complete an assessment for a patient with impaired skin integrity. TOP: Assessment MSC: CPNRE: Foundations of Practice 28. The nurse is caring for a patient with a stage 4 pressure injury. The nurse assigns which of the
following nursing diagnoses? a. Readiness for enhanced nutrition. b. Impaired physical mobility. c. Impaired skin integrity. d. Chronic pain. ANS: C
After the assessment is completed and the information that the patient has a stage 4 pressure injury is gathered, a diagnosis of Impaired skin integrity is selected. Readiness for enhanced nutrition would be selected for an individual with an adequate diet that could be improved. Impaired physical mobility and Chronic pain, as well as the nutrition nursing diagnosis, could well be the nursing diagnoses selected for this patient, but current data in the question strongly support Impaired skin integrity. DIF: Understand REF: 1309 | 1310 Bp.aiC OBJ: List nursing diagnoses aN ssU ocR iaS tedIw thT im reO dM skin integrity. NiG TOP: Planning MSC: CPNRE: Foundations of Practice 29. The nurse has collected the following assessment data: right heel with reddened area that does
not blanch. What nursing diagnosis would the nurse assign? a. Ineffective tissue perfusion. b. Risk for infection. c. Imbalanced nutrition: less than body requirements. d. Acute pain. ANS: A
The area on the heel has a decreased supply of blood and oxygen (tissue perfusion), which has resulted in tissue damage. The most appropriate nursing diagnosis with this information is Ineffective tissue perfusion. Risk for infection, Acute pain, and Imbalanced nutrition may be other nursing diagnoses, but the data provided do not support them. DIF: Understand REF: 1309 | 1310 OBJ: List nursing diagnoses associated with impaired skin integrity. TOP: Planning MSC: CPNRE: Foundations of Practice 30. The nurse is caring for a patient with a stage 3 pressure injury. The nurse has assigned a
nursing diagnosis of Risk for infection. Which intervention would be most important for this patient? a. Teach the family how to manage the odour associated with the wound.
Canadian Fundamentals of Nursing 6th Edition Potter Test Bank b. Discuss with the family how to prepare for care of the patient in the home. c. Encourage thorough hand hygiene of all individuals caring for the patient. d. Encourage increased quantities of carbohydrates and fats. ANS: C
The number one way to decrease the risk of infection by breaking the chain of infection is to wash hands. Encouraging fluid and food intake helps with overall wellness and wound healing, especially protein, but an increase in carbohydrates and fats does not affect the risk of infection. If the patient will be discharged before the wound is healed, the family will certainly need education on how to care for the patient. Teaching the family how to manage the odour associated with a wound is certainly important, but those interventions do not directly relate to the risk of infection and breaking the chain of the infectious process. DIF: Apply REF: 1326 OBJ: List nursing diagnoses associated with impaired skin integrity. TOP: Implementation MSC: CPNRE: Foundations of Practice 31. The patient in medical-surgical acute care has received a nursing diagnosis of Impaired skin
integrity. Which health care provider does the nurse consult? a. Respiratory therapist. b. Registered dietitian. c. Chaplain. d. Case manager. ANS: B
Adequate calories, protein, vitamins, and minerals promote wound healing. Assessment and a plan for the patient to optimize the diet are essential. The nurse is the coordinator of care, and collaborating with the dietitiaNnUwRoul dN reG sult ning the best meals for the patient. The SI TBin.pClan OM respiratory therapist can be consulted when a patient has issues with the respiratory system. Case management can be consulted when the patient has a discharge need. A chaplain can be consulted when the patient has a spiritual need. DIF: Apply REF: 1311 | 1331 OBJ: List nursing diagnoses associated with impaired skin integrity. TOP: Implementation MSC: CPNRE: Foundations of Practice 32. The nurse is caring for a patient with a stage 2 pressure injury and has assigned a nursing
diagnosis of Risk for infection. The patient is unconscious and bedridden. The nurse is completing the plan of care and is writing goals for the patient. What is the best goal for this patient? a. The patient’s family will demonstrate specific care of the wound site. b. The patient will state what to look for with regard to an infection. c. The patient will remain free of an increase in temperature and of malodorous or purulent drainage from the wound. d. The patient’s family members will wash their hands when visiting the patient. ANS: C
Canadian Fundamentals of Nursing 6th Edition Potter Test Bank Because the patient has an open wound and the skin is no longer intact to protect the tissue, the patient is at increased risk for infection. The nurse will be assessing the patient for signs and symptoms of infection, including an increase in temperature, an increase in white blood cell count, and malodorous and purulent drainage from the wound. The patient is unconscious and is unable to communicate the signs and symptoms of infection; also, enabling the patient to communicate is an intervention, not a goal for this diagnosis. It is important for the patient’s family to be able to demonstrate how to care for the wound and wash their hands, but these statements are interventions, not goals or outcomes for this nursing diagnosis. DIF: Apply REF: 1314-1316 OBJ: List nursing diagnoses associated with impaired skin integrity. TOP: Planning MSC: CPNRE: Foundations of Practice 33. On inspection of the patient’s wound, the nurse notes that it has a large amount of exudate.
Which of the following is an appropriate dressing for the nurse to select? a. Foam. b. Hydrogel. c. Hydrocolloid. d. Transparent film. ANS: A
Foam dressings are useful for wounds with large amounts of drainage or to assist with autolytic debridement because they can be left in place for as long as 7 days. This would be the most appropriate type of dressing for this wound. A hydrogel dressing donates moisture to the wound bed. This type of dressing hydrates the wound and absorbs some smaller amounts of exudates, but it would not be appropriate for a wound with a large amount of exudate. A hydrocolloid dressing is adhesive and occlusive. This type of dressing supports healing in N isRmSost INuse GTfulB.C M clean, granulating wounds andU on sO hallow to moderately deep wounds. It should not be used in heavily draining wounds, full-thickness wounds, or infected wounds. Transparent film traps the wound’s moisture over the wound, which provides a moist environment. It is used for small, superficial wounds; it is not appropriate for a wound with a large amount of exudate. DIF: Apply REF: 1323 | 1324, Table 46-9 OBJ: List nursing diagnoses associated with impaired skin integrity. TOP: Implementation MSC: CPNRE: Foundations of Practice 34. The home health nurse is caring for a patient with impaired skin integrity in the home. The
nurse is reviewing dressing changes with the caregiver. Which intervention assists in managing the expenses associated with long-term wound care? a. Sterile technique. b. No-touch technique. c. Double bagging of contaminated dressings. d. Ability of the caregiver. ANS: B
Canadian Fundamentals of Nursing 6th Edition Potter Test Bank The caregiver can use the same no-touch technique for dressing changes that is used for changing surface dressings without touching the wound or the surface that might come in contact with the wound. Double bagging is required for the disposal of contaminated dressings. The dressings go in a bag, which is fastened and then placed in the household garbage bag. The ability of the caregiver certainly is a component of the success of home treatment, but it does not influence the cost of supplies. DIF: Apply REF: 1312, Box 46-10 OBJ: Develop a nursing care plan for a patient with impaired skin integrity. TOP: Implementation MSC: CPNRE: Foundations of Practice 35. The nurse is caring for a patient who has suffered a stroke and has residual mobility problems.
The patient is at risk for skin impairment. Which initial interventions should the nurse select to decrease this risk? a. Gentle cleaners and thorough drying of the skin. b. Absorbent pads and garments. c. Positioning with use of pillows. d. Therapeutic beds and mattresses. ANS: A
Assessment and skin hygiene are two initial defences for preventing skin breakdown. The nurse should avoid soaps and hot water when cleansing the skin and should instead use gentle cleansers with nonionic surfactants. After the patient is bathed, the nurse dries the skin completely and applies moisturizer to keep the epidermis well lubricated. The use of absorbent pads and garments is controversial and should be considered only when other alternatives have been exhausted. Positioning the patient reduces pressure and shearing force to the skin and is part of the plan of care but is not one of the initial components. Depending N Ralty INbed GTmay B.CbeOneeded, M on the needs of the patient, a spUeciS but again, this does not provide the initial defence against skin breakdown. DIF: Apply REF: 1311 | 1312 OBJ: Develop a nursing care plan for a patient with impaired skin integrity. TOP: Implementation MSC: CPNRE: Foundations of Practice 36. The nurse is caring for a patient who is at risk for skin impairment. The patient is able to sit up
in a chair. The nurse includes this intervention in the plan of care. How long should the nurse schedule the patient to sit in the chair? a. At least 3 hours. b. Not longer than 30 minutes. c. Less than 2 hours. d. As long as the patient remains comfortable. ANS: C
When patients are able to sit up in a chair, the nurse should make sure to limit the amount of time to 2 hours or less. The chair sitting time should be individualized. Pressure on the ischial tuberosities is greater in the sitting position than in a supine position. Foam, gel, or an air cushion may be used to distribute weight. Sitting longer than 2 hours can increase the chance of ischemia. DIF: Apply REF: 1313 OBJ: Develop a nursing care plan for a patient with impaired skin integrity.
Canadian Fundamentals of Nursing 6th Edition Potter Test Bank TOP: Implementation
MSC: CPNRE: Foundations of Practice
37. The nurse is staffing a medical-surgical unit that is assigned most of the patients with pressure
injuries. The nurse has become competent in the care of pressure wounds and recognizes that which of the following is a staged pressure injury that does not require a dressing? a. Stage 1. b. Stage 2. c. Stage 3. d. Stage 4. ANS: A
Stage I intact pressure injuries that resolve slowly without epidermal loss over 7 to 14 days do not require a dressing. This allows visual inspection and monitoring. A transparent dressing could be used to protect the patient from shear but cannot be used in the presence of excessive moisture. A composite film, hydrocolloid, or hydrogel can be utilized on a clean stage 2 pressure injury. A hydrocolloid, hydrogel covered with foam, calcium alginate, gauze, and growth factors can be utilized with a clean stage 3 injury. Hydrogel, calcium alginate, gauze, and growth factors can be utilized with a clean stage 4 injury. For an unstageable wound covered with eschar, the nurse should utilize a dressing of adherent film or gauze with an ordered solution of enzymes. In rare cases when eschar is dry and intact, no dressing is used, but this is an unstaged pressure injury. DIF: Understand REF: 1300, Table 46-3 OBJ: List appropriate nursing interventions for a patient with impaired skin integrity. TOP: Diagnosis MSC: CPNRE: Foundations of Practice 38. The nurse is caring for a patient with a wound. The patient appears anxious as the nurse is
preparing to change the dressN inU gR .W ouBld.tC heOnMurse do to decrease the patient’s anxiety? ShIaNt sGhT a. Tell the patient to close his or her eyes. b. Explain the procedure. c. Turn on the television. d. Ask the family to leave the room. ANS: B
Explaining the procedure educates the patient regarding the dressing change and involves him or her in care, thereby allowing the patient some control in decreasing anxiety. Telling the patient to close the eyes and turning on the television are distractions that do not usually decrease a patient’s anxiety. If the family is a support system, asking support systems to leave the room can actually increase a patient’s anxiety. DIF: Understand REF: 1319, Skill 46-3 OBJ: List appropriate nursing interventions for a patient with impaired skin integrity. TOP: Intervention MSC: CPNRE: Foundations of Practice 39. The nurse is cleansing a wound site. As the nurse is doing so, what intervention should be
included? a. Allowing the solution to flow from the most contaminated to the least contaminated. b. Scrubbing vigorously when applying solutions to the skin. c. Cleansing in a direction from the least contaminated area. d. Utilizing clean gauze and clean gloves to cleanse a site.
Canadian Fundamentals of Nursing 6th Edition Potter Test Bank
ANS: C
The nurse cleanses surgical or traumatic wounds by applying noncytotoxic solution with sterile gauze or irrigations. Cleansing proceeds in a direction from the least contaminated area. The nurse should use gentle friction when applying solutions to the skin, and allow irrigation to flow from the least to the most contaminated area. DIF: Apply REF: 1327 OBJ: List appropriate nursing interventions for a patient with impaired skin integrity. TOP: Intervention MSC: CPNRE: Foundations of Practice 40. The nurse is caring for a patient after an open abdominal aortic aneurysm repair. The nurse
requests an abdominal binder and carefully applies the binder. What is the best explanation for the nurse to use when teaching the patient the reason for the binder? a. The binder creates pressure over the abdomen. b. The binder supports the abdomen. c. The binder reduces edema at the surgical site. d. The binder secures the dressing in place. ANS: B
The patient has a large abdominal incision. This incision will need support to remain closed, and an abdominal binder will support this wound, especially during movement, as well as during deep breathing and coughing. A binder can be used to create pressure over a body part (e.g., over an artery after it has been punctured), but this is not the best explanation for using the binder in this example. A binder can be used to prevent edema (e.g., in an extremity) but is not used to reduce edema at a surgical site. A binder can be used to secure dressings such as elastic webbing applied around a leg after vein stripping, but this is not the best explanation for using the binder in this exNamR ple.I G B.C M
U S N T
O
DIF: Apply REF: 1330 | 1331 OBJ: List appropriate nursing interventions for a patient with impaired skin integrity. TOP: Intervention MSC: CPNRE: Foundations of Practice 41. The nurse determines that the patient’s wound may be infected. In order to perform a
quantitative swab for wound culture, which of the following actions should the nurse take? a. Collect the superficial drainage. b. Collect the culture before cleansing the wound. c. Obtain a Culturette tube and use sterile technique. d. Use the same technique as for collecting an anaerobic culture. ANS: C
To collect a quantitative swab for a wound culture, the nurse uses a sterile swab from a Culturette tube and sterile technique. The nurse never collects a wound culture sample from old or superficial drainage. Resident colonies of bacteria from the skin grow in superficial drainage and may not be the true causative organisms of a wound infection. The nurse should clean the wound first with normal saline to remove skin flora, before obtaining the culture. The nurse uses different methods of specimen collection for aerobic or anaerobic organisms. DIF: Apply REF: 1308, Box 46-7 OBJ: List appropriate nursing interventions for a patient with impaired skin integrity. TOP: Intervention MSC: CPNRE: Foundations of Practice
Canadian Fundamentals of Nursing 6th Edition Potter Test Bank 42. The patient has a nursing diagnosis of Risk for skin impairment and has a score of 15 on the
Braden scale upon admission. The nurse has implemented interventions for this nursing diagnosis. Upon reassessment, which Braden score would be the best sign that the risk for skin breakdown is decreasing? a. 12. b. 13. c. 20. d. 23. ANS: D
The Braden scale is composed of six subscales: sensory perception, moisture, activity, mobility, nutrition, and friction and shear. The total score ranges from 6 to 23, and a lower total score indicates a higher risk for pressure injury development. The cutoff score for onset of pressure injury risk with the Braden scale in the general adult population is 18. The best sign is a perfect score of 23. DIF: Analyze REF: 1295 | 1298, Table 46-2 OBJ: List appropriate nursing interventions for a patient with impaired skin integrity. TOP: Evaluate MSC: CPNRE: Foundations of Practice 43. The nurse is caring for a patient with potential skin breakdown. Which of the following
components would the nurse include in the skin assessment? a. Mobility, hyperemia, induration, and blanching. b. Hyperemia, induration, blanching, and temperature of skin. c. Induration, nutritional status, mobility, and hyperemia. d. Nutritional status, induration, blanching, and temperature of skin. ANS: B
NURSINGTB.COM
Assessment of the skin includes both visual and tactile inspection. The nurse assesses for hyperemia and abnormal reactive hyperemia (when the skin turns red after an obstruction of blood flow returns and vasodilatation causes the tissue to turn red). The nurse also assesses for indurated (hardened) areas on the skin and palpate reddened areas for blanching. Changes in temperature can indicate changes in blood flow to that area of the skin. Mobility and nutritional status are certainly part of the overall assessment for pressure injury risk but are not part of the actual skin assessment. DIF: Apply REF: 1295-1299 OBJ: Complete an assessment for a patient with impaired skin integrity. TOP: Assessment MSC: CPNRE: Foundations of Practice 44. The student nurse is caring for a patient who will have both a large abdominal bandage and an
abdominal binder. The nursing instructor asks the student what the nursing responsibilities and activities will be before applying the bandage and binder. The nursing instructor will provide further instruction to the student if the student states that the nurse will need to do which of the following? a. Inspect the skin for abrasions and edema. b. Cover exposed wounds. c. Assess the skin at underlying areas for circulatory impairment. d. Cleanse the area with hydrogen peroxide. ANS: D
Canadian Fundamentals of Nursing 6th Edition Potter Test Bank Before applying a bandage or a binder, the nurse has several responsibilities. The nurse would need to inspect the skin for abrasions, edema, and discoloration or exposed wound edges. The nurse also is responsible for covering exposed wounds or open abrasions with a sterile dressing and assessing the condition of underlying dressings and changing them if they are soiled. To check for signs of circulatory impairment, the nurse assesses the skin of underlying areas that will be distal to the bandage so that a comparison can be made after bandages are applied. Marking the sites of all abrasions is not necessary. Although it is important for the skin to be clean, and even though it may need to be cleaned with a noncytotoxic cleanser, cleansing with hydrogen peroxide can interfere with wound healing. DIF: Apply REF: 1330 | 1331 OBJ: List appropriate nursing interventions for a patient with impaired skin integrity. TOP: Intervention MSC: CPNRE: Foundations of Practice
NURSINGTB.COM
Canadian Fundamentals of Nursing 6th Edition Potter Test Bank
Chapter 47: Sensory Alterations Potter et al: Canadian Fundamentals of Nursing, 6th Edition MULTIPLE CHOICE 1. A nurse is administering a vaccine to a 4-year-old child who is visually impaired. After the
needle enters the arm, the child says, “Ow, that was sharp!” The nurse knows that the ability to recognize and interpret stimuli is known as which of the following? a. Sensation. b. Reception. c. Perception. d. Reaction. ANS: C
Perception is the awareness of stimuli and interpretation of information signals. Reception refers to receiving stimuli and creating a nerve impulse. Reaction is how a person responds to a perceived stimulus. Sensation is a general term that refers to awareness of sensory stimuli through the body’s sense mechanisms. DIF: Understand REF: 1341 OBJ: Differentiate among the processes of reception, perception, and reaction to sensory stimuli. TOP: Assessment MSC: CPNRE: Foundations of Practice 2. What is the involuntary motion of retracting the body from painful stimuli? a. Sensation. b. Reception. NURSINGTB.COM c. Perception. d. Reaction. ANS: D
Reaction is how a person responds to a perceived stimulus. Perception is the consciousness of stimuli and interpretation of information signals. Reception refers to receiving stimuli and creating a nerve impulse. Sensation is the combination of all three. DIF: Understand REF: 1340 | 1341 OBJ: Differentiate among the processes of reception, perception, and reaction to sensory stimuli. TOP: Assessment MSC: CPNRE: Foundations of Practice 3. A nurse is caring for a patient with a nursing diagnosis of Hearing deficit related to
presbycusis. Which assessment of the patient would indicate an adaptation to the sensory deficit? a. The patient frequently cleans out his ears with a cotton swab. b. The patient turns one ear toward the nurse during conversation. c. The patient isolates himself or herself from social situations. d. The patient asks the nurse to speak loudly during conversations. ANS: B
Canadian Fundamentals of Nursing 6th Edition Potter Test Bank Adaptation for a sensory deficit indicates that the patient alters his or her behaviour to accommodate for his sensory deficit, such as turning the less affected ear toward the speaker. Cleaning the ear would not have an effect for a patient with presbycusis. Avoiding others because of a sensory deficit is maladaptive. Asking the nurse to speak loud alters the environment but is not an adaptation of the patient’s behaviour. DIF: Understand REF: 1356 | 1357 OBJ: Conduct an assessment of an individual’s sensory status. MSC: CPNRE: Foundations of Practice
TOP: Assessment
4. The nurse would be most concerned about the risk of malnutrition for a patient with which
sensory deficit? a. Xerostomia. b. Disequilibrium. c. Cataracts. d. Peripheral neuropathy. ANS: A
Xerostomia is a decrease in production of saliva; this decreases the ability and desire to eat and can lead to nutritional problems. The other options do not address taste- or nutrition-related concerns. DIF: Remember REF: 1343 OBJ: Discuss common causes and effects of sensory alterations. TOP: Assessment MSC: CPNRE: Foundations of Practice 5. Which of the following sensory changes are normal with aging? a. Impairment of night visioN nU . RSINGTB.COM b. Difficulty hearing low pitch. c. Increase in taste discrimination. d. Heightening of the sense of smell. ANS: A
Night vision becomes impaired as physiological changes in the eye occur. Older persons lose the ability to distinguish high-pitched noises and consonants. Senses of smell and taste are also decreased with aging. DIF: Remember REF: 1345 OBJ: Discuss common sensory changes that normally take place as individuals age. TOP: Assessment MSC: CPNRE: Foundations of Practice 6. A nurse is caring for an older patient who was in a motor vehicle accident because he thought
the traffic light was green. The patient asks the nurse if he should no longer drive. Which response by the nurse is most therapeutic? a. “Yes, you should stop driving. As you age, your cognitive function declines, and becoming confused puts everyone else on the road at risk.” b. “Yes, you should ask family members to drive you around from now on. Your reflex skills have declined so much you can’t avoid an accident.” c. “No, as you age, you lose the ability to see colours. You need to think about traffic lights in a new way. If the top is illuminated, it means stop, and if the bottom is illuminated, it means go.”
Canadian Fundamentals of Nursing 6th Edition Potter Test Bank d. “No, instead you should see your ophthalmologist and get some glasses to help
you see better.” ANS: C
Part of the normal aging process is an inability to distinguish colours. Much as with a younger adult who is colour blind, the nurse should teach the patient new ways to adapt to his deficit. This patient’s accident was not due to impaired cognitive function or reflexes. Glasses will not assist the patient in colour discrimination. DIF: Apply REF: 1345 OBJ: Discuss common sensory changes that normally take place as individuals age. TOP: Assessment MSC: CPNRE: Foundations of Practice 7. A nurse is caring for a patient who recently had a stroke and is going to be discharged at the
end of the week. The nurse notices that the patient is having difficulty with eating his meal and is becoming tearful. The nurse includes which intervention in the patient’s plan of care? a. Teach the patient about special devices used to assist patients with eating meals. b. Order the patient food that does not require utensils. c. Place a consult for a home health nurse. d. Obtain an order for antidepressant medications. ANS: A
The nurse should include implementations that help the patient adapt to his deficit while maintaining independence. Teaching the patient to use assistive devices allows the patient to care for himself. Changing the type of food that the patient eats may not work for every culture in which touching food with fingers is unacceptable, or the patient may not enjoy eating foods that do not require utensils. The services of a home health nurse are not necessary as long as the patient is able N toU caRreSfI orNhGims lf.CIns tead of placing the patient on TBe. OM antidepressants, the nurse should assist the patient in attempting to adapt behaviour to the sensory deficit. DIF: Analyze REF: 1346 | 1358 OBJ: Develop a plan of care for individuals with sensory alterations. TOP: Planning MSC: CPNRE: Foundations of Practice 8. Which nursing diagnosis addresses psychological concerns for a patient with both hearing and
visual sensory impairment? a. Self-care deficit. b. Risk for falls. c. Social isolation. d. Impaired physical mobility. ANS: C
In focusing on the psychological aspect of care, the nurse is most concerned about social isolation for a patient who may have difficulty communicating because of visual and hearing impairment. Both self-care deficit and fall risk are physiological risks for the patient. Impaired physical mobility would not apply to this patient. DIF: Understand REF: 1351 | 1352 OBJ: Develop a plan of care for individuals with sensory alterations. TOP: Planning MSC: CPNRE: Foundations of Practice
Canadian Fundamentals of Nursing 6th Edition Potter Test Bank 9. A patient informs the nurse that she often becomes nauseated when riding in motor vehicles.
The nurse knows that this is related to which sensory deficit? a. Neurological deficit. b. Visual deficit. c. Hearing deficit. d. Balance deficit. ANS: D
Vertigo is a result of vestibular dysfunction and often is precipitated by a change in head position. This disequilibrium can cause nausea and vomiting. The other options would not result in nausea that follows movement. DIF: Understand REF: 1343 OBJ: Discuss common causes and effects of sensory alterations. TOP: Assessment MSC: CPNRE: Foundations of Practice 10. A home health nurse is assembling a puzzle with an older patient and notices that the patient
is having difficulty connecting two puzzle pieces. The nurse knows that this is most likely related to which aspect of sensory deprivation? a. Perceptual. b. Cognitive. c. Affective. d. Social. ANS: A
Alterations in spatial orientation and in visual-motor coordination are signs of perceptual dysfunction. Cognitive function is the ability to think and the capacity to learn; the patient is not disoriented or unable to lN earn AI ffec tive roC blem UR. S NG TBp. OMs include boredom and restlessness; the patient is participating in an activity. The patient is socializing with the home health nurse, so isolation is not a problem. DIF: Understand REF: 1344, Box 47-1 OBJ: Discuss common causes and effects of sensory alterations. TOP: Assessment MSC: CPNRE: Foundations of Practice 11. Which assessment question should the nurse ask to best understand how visual alterations are
affecting the patient’s self-care ability? a. “Have you stopped reading books or switched to books on audiotape?” b. “Are you able to prepare a meal or write a cheque?” c. “How do you protect yourself from injury at work?” d. “How does your vision impairment make you feel?” ANS: B
To best understand how vision is affecting self-care ability, the nurse wants to target questions to encompass what self-care tasks the patient has difficulty doing, such as preparing meals and writing cheques. Switching from reading to books on audiotape gives the nurse an idea of the severity of the deficit but not its effect on activity of daily living. Assessing whether the patient is taking measures to protect himself is important, but this does not address self-care activities. Emotional assessment of a patient is also important but does not properly address the goal of determining the effect of visual alterations on self-care ability. DIF: Analyze
REF: 1344-1346
Canadian Fundamentals of Nursing 6th Edition Potter Test Bank OBJ: Discuss the relationship of sensory function to an individual’s level of wellness. TOP: Assessment MSC: CPNRE: Foundations of Practice 12. Which nursing assessment best measures cognitive functioning? a. The Mini-Mental Status Exam (MMSE). b. Questions about the patient’s name, where he is, and what month it is. c. Questions to the patient’s family about whether the patient is behaving normally. d. Evaluations of the patient’s ability to read the newspaper. ANS: A
The MMSE is a formal diagnostic tool that is used to assess a patient’s level of cognitive functioning. Asking the patient orientation questions evaluates only the patient’s orientation to self and surroundings, not abstract reasoning or critical thinking ability. Family members are not the most reliable source of information about the patient, although information received from the family should be considered. Reading a paper is not a means of comprehensive assessment; in addition, a patient may have high cognitive functioning and not know how to read English. DIF: Understand REF: 1349 OBJ: Conduct an assessment of an individual’s sensory status. MSC: CPNRE: Foundations of Practice
TOP: Assessment
13. During a community screening, the nurse notes that a 50-year-old patient is currently taking
steroid medications. How often does the nurse recommend that this patient have an eye examination? a. Every 3 to 4 months. b. Every 6 months. NURSINGTB.COM c. Every 1 to 2 years. d. Every 4 years. ANS: C
Patients between the ages of 40 and 64 years should have an eye examination every 1 to 2 years if there is a family history of glaucoma, if the patient has had a serious eye injury in the past, or if the patient is taking steroid medication. DIF: Understand REF: 1348, Box 47-6 OBJ: Conduct an assessment of an individual’s sensory status. MSC: CPNRE: Foundations of Practice
TOP: Implementation
14. A new nurse is caring for a patient who is undergoing chemotherapy for cancer. The patient is
becoming malnourished because nothing tastes good. Which recommendation by the nurse would be most appropriate for this patient? a. “Practice good oral hygiene to keep your taste buds well hydrated.” b. “Blend foods together in interesting flavour combinations.” c. “Eat soft foods that are easy to chew and swallow.” d. “Avoid adding spices or aromatic ingredients to food to prevent nausea.” ANS: A
Good oral hygiene is important for stimulating and hydrating taste buds. Having an unpleasant taste in the mouth discourages the patient from eating. Foods should not be blended together because this confuses the ability to discriminate flavours and taste. Texturized, spicy, and aromatic foods stimulate the appetite and make eating more enjoyable.
Canadian Fundamentals of Nursing 6th Edition Potter Test Bank
DIF: Apply REF: 1357 OBJ: Develop a plan of care for individuals with sensory alterations. TOP: Implementation MSC: CPNRE: Foundations of Practice 15. The nurse is creating a plan of care for a patient with glaucoma. Which nursing diagnosis
addresses the complication of the sensory deficit that places the patient at greatest risk for injury? a. Risk for falls. b. Body image disturbance. c. Social isolation. d. Fear. ANS: A
Impaired depth perception and inability to see obstacles can lead to falls, and so visual disturbance poses great risk for injury. Body image disturbance, Social isolation, and Fear are all valid nursing diagnoses that apply to a patient with vision deficit; however, they do not address the greatest risk for injury. DIF: Analyze REF: 1351 | 1352 OBJ: Develop a plan of care for individuals with sensory alterations. TOP: Planning MSC: CPNRE: Foundations of Practice 16. A patient has hyperesthesia associated with a neurological trauma. Which of the following is
an appropriate nursing intervention in regard to the patient’s sense of touch? a. Reminding the patient of the need to have frequent tactile contact. b. Keeping the patient loosely covered with sheets and blankets. c. Allowing the patient to liN eU mR otS ioI nlN esG s.TB.COM d. Using touch as a form of therapy. ANS: B
If a patient is overly sensitive to tactile stimuli (hyperesthesia), the nurse must minimize irritating stimuli. Keeping bed linens loose to minimize direct contact with the patient and protecting the patient’s skin from exposure to irritants are helpful measures. Frequent tactile contact and allowing the patient to lie motionless are not appropriate interventions for the patient with hyperesthesia. Using touch as a form of therapy would not be an appropriate nursing intervention for the patient with hyperesthesia. DIF: Apply REF: 1357 OBJ: Identify interventions for preventing sensory deprivation and controlling sensory overload. TOP: Implementation MSC: CPNRE: Foundations of Practice 17. The nurse is caring for a patient with conductive hearing loss in one ear resulting from
prolonged cerumen impaction. Which intervention by the nurse is most important in establishing effective communication with the patient? a. Speaking in a loud voice, enunciating every syllable. b. Having direct conversation with the patient in his affected ear. c. If the patient does not understand what the nurse is saying, repeating the phrase again. d. Speaking with hands, face, and expressions. ANS: D
Canadian Fundamentals of Nursing 6th Edition Potter Test Bank Using gestures other than just speaking helps the patient understand what you are saying and makes it a meaningful stimulus. Speaking in loud tones can distort a patient’s ability to hear; the nurse should speak in normal low tones. If the patient does not understand the first time, the nurse should try rephrasing instead of repeating the message. The nurse can direct conversation toward the patient’s unaffected ear. DIF: Understand REF: 1357, Box 47-11 OBJ: Describe conditions in the health care setting or a patient’s home that you can adjust to promote meaningful sensory stimulation. TOP: Implementation MSC: CPNRE: Foundations of Practice 18. The home health nurse is caring for a patient with a tactile deficit; the nurse is concerned
about injury related to inability to feel harmful stimuli. The nurse determines that the patient is able to safely care for himself when the patient demonstrates which action? a. Places coloured stickers on faucet handles to indicate temperature and keeps a thermometer near the tub. b. Asks the nurse to test the temperature of the water before entering the bath. c. Replaces all lace-up shoes with Velcro ones and purchases shampoo caps. d. Dispenses all medications onto a plate for easy access in the morning. ANS: A
By placing colour-coded stickers and other reminders about dangerous stimuli, the patient is able to safely keep up hygiene. Asking the nurse to test the water does not promote independence, although it does promote safety. Zipper and Velcro clothing is easier for a patient with a tactile deficit to wear, and shower caps allow the patient to stay well-groomed with minimal effort, but neither promotes safety. Leaving the lids off of medications can be dangerous, as can placing all medications out at once. It may be difficult for the patient to sort N Rlect INthe GTcorrect B.COtypes M and numbers of pills. through mixed medications andUseS DIF: Evaluate REF: 1357 OBJ: Discuss ways to maintain a safe environment for patients with sensory deficits. TOP: Evaluate MSC: CPNRE: Foundations of Practice 19. The nurse is caring for a patient in acute respiratory distress. The patient has multiple
monitoring systems on that constantly beep and make noise. The patient is becoming agitated and frustrated over inability to sleep. Which action by the nurse is most appropriate for this patient? a. Providing the patient with a therapeutic back rub. b. Turning off the alarms on the monitoring devices. c. Administering an opioid medication to help the patient sleep. d. Providing the patient with earplugs. ANS: D
Giving the patient control over stimuli helps to decrease the frustration that results from sensory overload. Adding more stimuli such as a back rub can increase sensory overload. Turning off monitors and alarms is unsafe; the nurse needs to be aware of critical situations. Opioid medications should not be the first option; however, antianxiety medications and sleep aids may be considered. DIF: Apply REF: 1351 | 1355 OBJ: Identify interventions for preventing sensory deprivation and controlling sensory overload.
Canadian Fundamentals of Nursing 6th Edition Potter Test Bank TOP: Implementation
MSC: CPNRE: Foundations of Practice
20. The nurse is caring for a patient with expressive aphasia caused by a traumatic brain injury.
Which desired outcome should be included in the plan of care? a. Patient will recover full use of speech vocabulary in 1 week. b. Patient will carry a pen and a pad of paper around for communication. c. Patient will thicken drinks to prevent aspiration. d. Patient will communicate nonverbally. ANS: D
Patients with expressive aphasia may take a prolonged time to regain speech function, depending on the cause of the incident. To adapt to expressive aphasia, the nurse and the patient need to work on ways to communicate nonverbally through means such as pointing and gestures. A patient who has expressive aphasia may not be able to speak or write words. Thickening drinks prevents aspiration risk and is not included in a plan of care for this patient. DIF: Apply REF: 1347 OBJ: Develop a plan of care for individuals with sensory alterations. TOP: Planning MSC: CPNRE: Foundations of Practice 21. The nurse is aware that which patient is most at risk for sensory deprivation? a. A patient in the critical care unit (CCU) under constant monitoring after a
myocardial infarction. b. A patient on the unit with tuberculosis on airborne precautions. c. A patient who recently had a stroke and has left-sided weakness. d. A patient receiving hospice care for end-stage brain cancer. ANS: B
NURSINGTB.COM
Sensory deprivation occurs when a person has decreased stimulation and limited sensory input. A patient in isolation is at risk for sensory deprivation because he or she has limited exposure to meaningful stimuli. A patient in the CCU would be at risk for sensory overload with all the monitors and visitors. A patient with a stroke or with brain cancer may have difficulty with tactile sensation and may have sensory deficits but is not at risk for sensory deprivation. DIF: Evaluate REF: 1358 OBJ: Discuss common causes and effects of sensory alterations. TOP: Planning MSC: CPNRE: Foundations of Practice 22. What nursing action can the nurse implement to comfort an older patient with sensory
deprivation to improve meaningful stimuli? a. Placing a “Do not disturb” sign on the patient’s door. b. Offering the patient a back rub. c. Asking the patient whether he would like a newspaper to read. d. Placing the patient in the room farthest from the nurses’ station. ANS: B
The patient with sensory deprivation needs meaningful stimuli, and therapeutic massage helps establish a humanistic relationship that the patient is missing. All of the other options do not promote interaction and promote further social isolation. DIF: Understand
REF: 1358
Canadian Fundamentals of Nursing 6th Edition Potter Test Bank OBJ: Identify interventions for preventing sensory deprivation and controlling sensory overload. TOP: Implementation MSC: CPNRE: Foundations of Practice 23. The nurse is caring for a patient who is a well-known surgeon at the hospital. Because of his
status, all the hospital’s physicians want to be sure to pay him a visit. The nurse notices the patient becoming more agitated and withdrawn with each group of visitors. The nurse asks the patient if he would like a “Do not disturb” sign placed on the door. A few hours later, the nurse notices a physician who is not involved in the patient’s care attempting to enter the room. Which response by the nurse is most appropriate? a. Allowing the physician to enter because he has higher authority than the nurse. b. Calling for security to remove the visitor. c. Firmly explaining that the patient does not wish to have visitors at this time, so do not enter the room. d. Scolding the physician for not obeying the signs on the door and respecting the patient’s wishes. ANS: C
The nurse acts as an advocate for the patient (who is experiencing sensory overload and would benefit from a quiet environment) by firmly and politely asking the visitor to leave, regardless of position in the hospital. The nurse should not allow anyone to enter unless the patient approves it. Security is not a necessary measure at this time. The nurse should act with professionalism when addressing the visitor; scolding the visitor is not appropriate. DIF: Understand REF: 1340 OBJ: Describe conditions in the health care setting or a patient’s home that you can adjust to promote meaningful sensory stimulation. TOP: Implementation MSC: CPNRE: Foundations of Practice
NURSINGTB.COM
24. The nurse is caring for a patient who is recovering from a traumatic brain injury and
frequently becomes disoriented about everything except her location. Which nursing intervention would be effective in orienting a patient with neurological deficit? a. Assessing the patient’s level of consciousness and documenting every 4 hours. b. Keeping a day-by-day calendar at the patient’s bedside and having the patient manage it. c. Placing a patient observer in the patient’s room for safety. d. Informing the patient that she cannot be discharged unless she is awake, alert, and oriented. ANS: B
Keeping a calendar in the patient’s room helps to orient the patient to the dates and gives the patient a sense of control over her environment. Assessing the patient’s level of consciousness is not an action that will directly affect the patient’s confusion. A patient observer is unnecessary unless the patient is in danger from the confusion. The nurse should encourage the patient toward recovery but should be sensitive to the time it takes for progression. DIF: Apply REF: 1344 OBJ: Discuss ways to maintain a safe environment for patients with sensory deficits. TOP: Implementation MSC: CPNRE: Foundations of Practice 25. A nurse is establishing a relationship with the patient who is visually impaired. Which is the
most appropriate method to teach the patient how to contact the nurse for assistance?
Canadian Fundamentals of Nursing 6th Edition Potter Test Bank a. Placing a raised Braille sticker on the call button, and instructing the patient to
press for assistance. b. Instructing the patient to yell at the top of his lungs to get the attention of the staff. c. Explaining to the patient that a staff person will stop by once an hour to see if the
patient needs anything. d. Sharing cell phone numbers with the patient so he can call the nurse if he needs
her. ANS: A
The nurse should devise a plan of care that accommodates the patient’s visual deficit. Placing a raised Braille sticker on the call light allows the patient to find it and page the nurse for assistance as needed. Yelling at the top of the lungs is stressful for the patient and for surrounding patients. Making hourly rounds is not sufficient; the nurse needs to ensure that the patient can get in touch with her at any time. Sharing personal phone numbers with the patient is inappropriate. DIF: Apply REF: 1355 | 1356 OBJ: Develop a plan of care for individuals with sensory alterations. TOP: Implementation MSC: CPNRE: Foundations of Practice 26. The nurse is developing a plan of care for a patient who is having a prosthetic eye placed.
Which nursing diagnosis related to patient safety is the priority for the nurse to include in the plan of care? a. Self-care deficit. b. Risk for injury. c. Anxiety. d. Body image disturbance. ANS: B
NURSING TB.COM
The patient with a prosthetic eye will require a period of adjustment to new depth perception and visual sensation. Until the patient adapts, preventing injury should be the nurse’s priority. The other options are not directly related to the safety of the patient for eye surgery. DIF: Analyze REF: 1351 | 1352 OBJ: Develop a plan of care for individuals with sensory alterations. TOP: Planning MSC: CPNRE: Foundations of Practice
Canadian Fundamentals of Nursing 6th Edition Potter Test Bank
Chapter 48: Care of Surgical Patients Potter et al: Canadian Fundamentals of Nursing, 6th Edition MULTIPLE CHOICE 1. The nurse is serving as preceptor for a student nurse and explains that perioperative nursing
care occurs when and where? a. Before, during, and after surgery. b. In preadmission testing. c. During the surgical procedure. d. In the postanaesthesia care unit (PACU). ANS: A
Perioperative nursing care occurs before, during, and after a surgery. Preadmission testing occurs before surgery and is considered preoperative. Nursing care provided during the surgical procedure is considered intraoperative, and in the PACU, it is considered postoperative. All of these are parts of the perioperative phase, but each individual phase does not explain the term completely. DIF: Remember REF: 1363 OBJ: Understand the aspects of perioperative nursing care. MSC: CPNRE: Foundations of Practice
TOP: Implementation
2. The nurse is caring for a patient who is scheduled to undergo a surgical procedure. The nurse
is completing an assessment and reviews the patient’s laboratory test results and allergies. In which perioperative nursing phase would this work be completed? N R I G B.C M a. Perioperative. U S N T O b. Preoperative. c. Intraoperative. d. Postoperative. ANS: B
Reviewing the patient’s laboratory tests and allergies is done before surgery in the preoperative phase. Perioperative means before, during, and after surgery. Intraoperative means during the surgical procedure in the operating suite; postoperative means after the surgery and could occur in the PACU, in the ambulatory surgical area, or on the hospital unit. DIF: Understand REF: 1374 OBJ: Understand the aspects of perioperative nursing care. MSC: CPNRE: Foundations of Practice
TOP: Assessment
3. The nurse is caring for a patient in the PACU. The patient has developed profuse bleeding
from the surgical site, and the surgeon has determined the need to return to the operating area. How would this procedure be classified? a. Elective. b. Urgent. c. Emergency. d. Major. ANS: C
Canadian Fundamentals of Nursing 6th Edition Potter Test Bank An emergency procedure must be done immediately to save life or preserve function of a body part. An example would be repair of a perforated appendix, repair of a traumatic amputation, or control of internal hemorrhaging. An urgent procedure is necessary for a patient’s health and often prevents additional problems from developing. An example would be excision of a cancerous tumour, removal of a gallbladder for stones, or vascular repair for obstructed artery. An elective procedure is performed on the basis of the patient’s choice; it is not essential and is not always necessary for health. An example would be a bunionectomy, plastic surgery, or hernia reconstruction. A major procedure involves extensive reconstruction or alteration in body parts; it poses great risks to well-being. An example would be a coronary artery bypass or colon resection. DIF: Remember REF: 1366, Table 48-1 OBJ: Differentiate between classifications of surgery and types of anaesthesia. TOP: Assessment MSC: CPNRE: Foundations of Practice 4. The nurse is caring for a patient in preadmission testing. The patient has been assigned a
physical status classification of PS-III by the American Society of Anesthesiologists. Which of the following assessments would support this classification? a. Denial of any major illnesses or conditions. b. Normal, healthy patient. c. History of hypertension, 36 kg (80 pounds) overweight, history of asthma. d. History of myocardial infarction that limits activity. ANS: C
A classification PS-III means that a patient has a severe systemic disease. Patients with hypertension, obesity, diabetes mellitus, and asthma have a physical status that fits into this category. PS-I is the classification for a normal healthy patient. PS-II is the classification for a N ase RS I -IV GTisB.C M patient with mild systemic diseU . PSN the cO lassification for a patient with severe systemic disease that is a constant threat to life. DIF: Understand REF: 1366, Table 48-2 OBJ: Differentiate between classifications of surgery and types of anaesthesia. TOP: Assessment MSC: CPNRE: Foundations of Practice 5. The patient has arrived at the ambulatory surgery centre for a colonoscopy. The patient is
scheduled to receive moderate sedation (conscious sedation) during the procedure. Moderate sedation is used routinely for procedures that require which of the following? a. An outpatient setting. b. A depressed level of consciousness. c. Loss of sensation in an area of the body. d. Immobility of the patient. ANS: B
Moderate sedation (conscious sedation) is used routinely for procedures that necessitate not complete anaesthesia but rather a depressed level of consciousness. Not all patients who are treated on an outpatient basis receive moderate sedation. Regional anaesthesia such as local anaesthesia provides loss of sensation in an area of the body. General anaesthesia is used for patients who need to be immobile and to not remember the surgical procedure. DIF: Remember REF: 1397 OBJ: Differentiate between classifications of surgery and types of anaesthesia.
Canadian Fundamentals of Nursing 6th Edition Potter Test Bank TOP: Assessment
MSC: CPNRE: Foundations of Practice
6. The nurse is caring for a patient in the PACU who has undergone a left total knee
arthroplasty. The anaesthesia provider has indicated that the patient received a left femoral peripheral nerve block. Which assessment immediately after surgery would be an expected finding for a patient with this type of regional block? a. Decreased pulse at the left posterior tibia. b. Left toes cool to touch and slightly cyanotic. c. Sensation decreased in the left leg. d. Patient report of pain in the left foot. ANS: C
Induction of regional anaesthesia results in loss of sensation in an area of the body. The peripheral nerve block influences the portion of sensory pathways that are anaesthetized in the targeted area of the body. Decreased pulse, coolness of toes to touch, and cyanosis are indications of decreased blood flow and are not expected findings. Reports of pain the in the left foot may indicate that the block is not working or is subsiding and is not an expected finding in the immediate postoperative period. DIF: Understand REF: 1397 OBJ: Differentiate between classifications of surgery and types of anaesthesia. TOP: Assessment MSC: CPNRE: Foundations of Practice 7. The nurse is preparing a patient for surgery. Aims of assessment before surgery include which
of the following? a. Establishing a patient’s baseline of normal function. b. Planning for care after the procedure. RiS c. Educating the patient andNfU am lyIaN boG uT t tB he.pC roOcM edure. d. Gathering appropriate equipment for the patient’s needs. ANS: A
The aim of assessment of the patient before surgery is to establish the patient’s normal preoperative function to prevent and minimize possible postoperative complications. Gathering appropriate equipment, planning care, and educating the patient and family are all important interventions that must be provided for the surgical patient; they are part of the nursing process but are not the reason for completing an assessment of the surgical patient. DIF: Understand REF: 1400 OBJ: List factors to include in the preoperative, intraoperative, and postoperative assessment of a surgical patient. TOP: Assessment MSC: CPNRE: Foundations of Practice 8. The nurse is documenting a medication history for the surgical patient in preadmission testing.
Which of the following medications should the nurse instruct the patient to withhold in preparation for surgery? a. Ibuprofen. b. Acetaminophen. c. Vitamin C. d. Miconazole. ANS: A
Canadian Fundamentals of Nursing 6th Edition Potter Test Bank Nonsteroidal anti-inflammatory drugs (NSAIDs) such as ibuprofen inhibit platelet aggregation and prolong bleeding time, thereby increasing susceptibility to postoperative bleeding. Acetaminophen is a pain reliever, vitamin C actually assists in wound healing, and miconazole is an antifungal. None of these has any special implications for surgery. DIF: Apply REF: 1371, Table 48-5 OBJ: List factors to include in the preoperative, intraoperative, and postoperative assessment of a surgical patient. TOP: Assessment MSC: CPNRE: Foundations of Practice 9. The nurse is caring for a potential surgical patient in the preadmission testing unit. The
medication history indicates that the patient is currently taking warfarin (Coumadin). Which of the following actions should the nurse take? a. Consult with the physician regarding a radiological examination of the chest. b. Consult with the physician regarding an international normalized ratio (INR). c. Consult with the physician regarding blood urea nitrogen (BUN). d. Consult with the physician regarding a complete blood cell count (CBC). ANS: B
Warfarin is an anticoagulant that is utilized for different maladies, but its action is to increase the time it takes for the blood to clot. This action can put the surgical patient at risk for bleeding. Typically, if at all possible, this medication is withheld for several days before a surgical procedure to decrease this risk. INR, prothrombin time, activated partial thromboplastin time, and platelet counts reveal the clotting ability of the blood. Chest radiography, BUN measurement, and CBC are diagnostic screening tools for surgery but are not specific to warfarin. DIF: Apply REF: 1374, Table 48-6 OBJ: List factors to include inNthU eR prS eoIpN erG atiT veB ,. intC raO opMerative, and postoperative assessment of a surgical patient. TOP: Assessment MSC: CPNRE: Foundations of Practice 10. The nurse is encouraging the postoperative patient to utilize diaphragmatic breathing. Reasons
for this intervention include which of the following? a. Management of pain. b. Decreased healing time. c. Prevention of atelectasis. d. Decreased thrombus formation. ANS: C
During general anaesthesia, the lungs are not fully inflated during surgery and the cough reflex is suppressed, and so mucus collects within airway passages. After surgery, patients may have reduced lung volume and may need to expend greater effort to cough and deep breathe; inadequate lung expansion can lead to atelectasis and pneumonia. Purposely utilizing diaphragmatic breathing can decrease this risk. Diaphragmatic breathing, except for the components of distraction, minimal increased level of oxygen, and minimal chest wall movement, does not influence pain, healing time, or thrombus formation. More effective interventions are available for these situations. DIF: Understand REF: 1404 | 1405, Table 48-10 OBJ: Demonstrate postoperative exercises: diaphragmatic breathing, coughing, turning, and leg exercises. TOP: Intervention MSC: CPNRE: Foundations of Practice
Canadian Fundamentals of Nursing 6th Edition Potter Test Bank 11. The nurse is caring for a postoperative patient on the medical-surgical floor. To prevent
venous stasis and the formation of thrombus after general anaesthesia, what does the nurse encourage the patient to do? a. Coughing. b. Diaphragmatic breathing. c. Incentive spirometry. d. Leg exercises. ANS: D
After general anaesthesia, circulation slows, and when the rate of blood slows, a greater tendency for clot formation is noted. Immobilization results in decreased muscular contractions in the lower extremities; these promote venous stasis. Coughing, diaphragmatic breathing, and incentive spirometry are utilized to decrease the risk of atelectasis. DIF: Remember REF: 1385, Skill 48-1 | 1404 | 1405, Table 48-10 OBJ: Demonstrate postoperative exercises: diaphragmatic breathing, coughing, turning, and leg exercises. TOP: Intervention MSC: CPNRE: Foundations of Practice 12. The nurse is caring for a surgical patient preoperatively. The nurse teaches the principles and
demonstrates leg exercises for the patient. The patient is unable to perform leg exercises correctly. What is the nurse’s best next step? a. Assess for the presence of anxiety, pain, or fatigue. b. Ask the patient why he or she does not want to do the exercises. c. Encourage the patient to practise at a later date. d. Assess the educational methods used to educate the patient. ANS: A
If the patient is unable to perN form afte r sound educational principles and URleg SIex NGerci TBse.s C OM demonstration are provided, the nurse should look for circumstances that may be affecting the patient’s ability to learn. In this case, the patient can be anticipating the upcoming surgery and may be experiencing anxiety. The patient may also be in pain or may be fatigued; both of these can affect the ability to learn. Assessment of educational methods may be needed, but in this case, sound principles and demonstration are being utilized. Asking anyone why he or she is not cooperating can cause defensiveness and may not help in attaining the answer. In this case, the patient really may want to participate and may not know why he or she is unable to learn. The nurse is aware that the patient is unable to do the exercises; encouraging the patient to practise later will not help the patient meet goals. DIF: Apply REF: 1389, Skill 48-1 OBJ: Demonstrate postoperative exercises: diaphragmatic breathing, coughing, turning, and leg exercises. TOP: Intervention MSC: CPNRE: Foundations of Practice 13. Which nursing assessment would indicate that the patient is performing diaphragmatic
breathing correctly? a. Hands placed on border of rib cage with fingers extended will touch as chest wall contracts. b. Hands placed on chest wall with fingers extended will separate as chest wall contracts. c. The patient will feel upward movement of the diaphragm during inspiration. d. The patient will feel downward movement of the diaphragm during expiration.
Canadian Fundamentals of Nursing 6th Edition Potter Test Bank ANS: A
Positioning the hands along the borders of the rib cage allows the patient to feel movement of the chest and abdomen as the diaphragm descends and the lungs expand. As the patient takes a deep breath and slowly exhales, the middle fingers will touch while the chest wall contracts. The fingers will separate as the chest wall expands. The patient will feel normal downward movement of the diaphragm during inspiration and normal upward movement during expiration. DIF: Apply REF: 1385-1387, Skill 48-1 OBJ: Demonstrate postoperative exercises: diaphragmatic breathing, coughing, turning, and leg exercises. TOP: Assessment MSC: CPNRE: Foundations of Practice 14. The nurse is caring for a postoperative patient with an abdominal incision. A pillow is used
during coughing to provide which of the following? a. Pain relief. b. Splinting. c. Distraction. d. Anxiety reduction. ANS: B
Surgical incisions cut through muscles, tissues, and nerve endings. Deep breathing and coughing exercises place additional stress on the suture line and cause discomfort. Splinting incisions with hands and a pillow provides firm support and reduces incisional pull. Providing a pillow during coughing does not provide distraction, reduce anxiety, or relieve pain. Coughing can increase anxiety because it can cause pain. DIF: Understand REF: 1387, Skill 48-1 OBJ: Demonstrate postoperatiN veUeR xeS rcI isN esG : dT iaB ph.raCgO mM atic breathing, coughing, turning, and leg exercises. TOP: Intervention MSC: CPNRE: Foundations of Practice 15. The nurse is encouraging a reluctant postoperative patient to breathe deeply and cough. What
explanation can the nurse provide that may encourage the patient to cough more effectively? a. “If you don’t breathe deeply and cough, you will get pneumonia.” b. “Deep breathing and coughing will clear out the anaesthesia.” c. “Coughing will not harm the incision if done correctly.” d. “You will need to cough only a few times during this shift.” ANS: C
If coughing is done correctly with proper support of the incision, it will not harm the incision. Deep breathing and coughing help clear out mucus in the respiratory system that has accumulated because of the anaesthetic. Although it is correct that a patient may experience atelectasis and pneumonia if deep breathing and coughing are not performed, the statement sounds threatening; the information could be communicated in a more therapeutic manner. Deep breathing and coughing is encouraged every 2 hours while the patient is awake. DIF: Apply REF: 1387, Skill 48-1 OBJ: Demonstrate postoperative exercises: diaphragmatic breathing, coughing, turning, and leg exercises. TOP: Intervention MSC: CPNRE: Foundations of Practice
Canadian Fundamentals of Nursing 6th Edition Potter Test Bank 16. The nurse and the unregulated care provider are helping a postoperative patient turn in the
bed. To assist in minimizing discomfort, which instruction should the nurse provide to the patient? a. “Close your eyes and think about something pleasant.” b. “Hold your breath and count to three.” c. “Hold my shoulders with your hands.” d. “Place your hand over your incision.” ANS: D
The nurse should instruct the patient to place one hand over the incisional area to splint it, providing support and minimizing pulling during turning. Closing one’s eyes, holding one’s breath, and holding the nurse’s shoulders do not help support the incision during a turn. DIF: Apply REF: 1387, Skill 48-1 OBJ: Demonstrate postoperative exercises: diaphragmatic breathing, coughing, turning, and leg exercises. TOP: Intervention MSC: CPNRE: Foundations of Practice 17. An obese patient admitted for abdominal surgery is more susceptible to the postoperative
complication of which of the following? a. Anemia. b. Seizures. c. Protein loss. d. Dehiscence. ANS: D
An obese patient is susceptible to poor wound healing and wound infection because of the structure of fatty tissue, which contains a poor blood supply. This increases the risk for dehiscence. A patient who isN mal URnou SIrish NGed,TBno.tCanOoMbese patient, is more susceptible to being anemic. An obese patient is not more susceptible than nonobese patients to seizures or to protein loss. A patient with liver disease may have altered protein metabolism. DIF: Apply REF: 1369 | 1370 OBJ: Describe the rationales for nursing interventions designed to prevent postoperative complications. TOP: Intervention MSC: CPNRE: Foundations of Practice 18. The nurse is providing preoperative teaching for the ambulatory surgical patient who will
have a cyst removed from the right arm. Which would be the best explanation for diet progression after surgery? a. “Start with clear liquids, soup, and crackers. Advance to a normal diet as you tolerate.” b. “There is no limitation on your diet. You can have whatever you want.” c. “Stay on clear liquids for 24 hours. Then you can progress to a normal diet.” d. “Start with clear liquids for 2 hours, then full liquids for 2 hours. Then progress to a normal diet.” ANS: A
Canadian Fundamentals of Nursing 6th Edition Potter Test Bank The type of surgery that patients undergo determines how quickly they can resume normal physical activity and regular eating habits. It is normal to progress gradually in activity and eating, and if the patient tolerates activity and diet well, he or she can progress more quickly. A common complication after surgery is nausea and vomiting. This can be caused by the anaesthetic, fluid imbalance from being on NPO (nothing by mouth) status, and pain. The gastrointestinal tract may be hypoactive as a result of anaesthesia. It is best to start with a clear liquid to see whether the patient can tolerate the liquid without vomiting. If so, it is appropriate for the patient to progress to soup and crackers and to advance as tolerated. Starting with a heavy, greasy meal could cause nausea and vomiting. There is no need to stay on clear liquids for 24 hours after this procedure. Putting a time frame on the progression is too prescriptive. Progression should be adjusted for the patient’s needs. DIF: Apply TOP: Intervention
REF: 1407 OBJ: Design a preoperative teaching plan. MSC: CPNRE: Foundations of Practice
19. The nurse explains the pain relief measures available after surgery during preoperative
teaching for a surgical patient. Which of the following comments from the patient indicates the need for additional education on this topic? a. “I will take the pain medication as the physician prescribes it.” b. “I will be asked to rate my pain on a pain scale.” c. “I will have minimal pain because of the anaesthesia.” d. “I will take my pain medications before doing postoperative exercises.” ANS: C
Pain after surgery is expected and is one of the patient’s fears. Anaesthesia will be provided during the procedure itself, and the patient should not experience pain during the procedure. Pain management is utilized after the postoperative phase. The nurse should inform the patient NainRS I G B.C M of interventions available for pU relieNf, inTcludingOmedication, relaxation, and distraction. The patient needs to know and understand how to take the medications that the physician will prescribe postoperatively. During the stay in the facility, the level of pain is frequently assessed by the nurses. Coordinating pain medication with postoperative exercises helps minimize discomfort and allows the exercises to be more effective. DIF: Analyze TOP: Evaluate
REF: 1391 OBJ: Design a preoperative teaching plan. MSC: CPNRE: Foundations of Practice
20. The nurse is making a preoperative education appointment with a patient. The patient asks
whether he should bring family with him to the appointment. What is the best response by the nurse? a. “There is no need for an additional person at the appointment.” b. “Your family can come and wait with you in the waiting room.” c. “We recommend including family in this appointment to ease everyone’s anxiety.” d. “It is required that you have a family member at this appointment.” ANS: C
Canadian Fundamentals of Nursing 6th Edition Potter Test Bank It is ideal to attempt perioperative education before admission, during the hospital stay, and after discharge. Including family members in perioperative education is advisable. Often a family member is a coach for postoperative exercises when the patient returns from surgery. If anxious relatives do not understand routine postoperative events, it is likely that their anxiety will heighten the patient’s fears and concerns. Preparation of family members before surgery helps to minimize anxiety and misunderstanding. An additional person is needed at the appointment if at all possible, and he or she needs to be involved in the process, not just waiting in the waiting room; however, it is certainly not a requirement for actually completing the surgery that someone comes to this appointment. DIF: Apply TOP: Intervention
REF: 1365 OBJ: Design a preoperative teaching plan. MSC: CPNRE: Foundations of Practice
21. The nurse is reviewing the surgical consent with the patient during preoperative education.
The patient indicates that he does not understand what procedure will be completed. What is the nurse’s best next step? a. Notify the physician about the patient’s question. b. Explain the procedure that will be completed. c. Ask the patient to sign the form. d. Continue with preoperative education. ANS: A
Surgery cannot be legally or ethically performed until the patient understands the need for a procedure, the steps involved, the risks, expected results, and alternative treatments. It is the surgeon’s responsibility to explain the procedure and obtain informed consent. It is important for the nurse to pause to notify the physician of the patient’s questions. It is not within the nurse’s scope to explain the procedure for the first time. The nurse can certainly reinforce Ned,RS ItheGT B.C tion M needs to come from the physician. It is what the physician has explainU but N informaO not prudent to ask a patient to sign a form for a procedure that he or she does not understand. DIF: Apply TOP: Intervention
REF: 1378 OBJ: Design a preoperative teaching plan. MSC: CPNRE: Professional, Ethical, and Legal Practice
22. During preoperative assessment for an operation at 0730 hours, the patient indicates to the
nurse that he had a cup of coffee at 0700 hours. The nurse reports this information to the anaesthesia provider, anticipating what? a. A delay in or cancellation of surgery. b. Questions regarding components of the coffee. c. Additional questions about why the patient had coffee. d. Instructions to determine what education was provided in the preoperative visit. ANS: A
A patient traditionally takes nothing by mouth (NPO) after midnight on the morning of surgery to keep the stomach empty and thus reduce the risk of vomiting and aspiration. It is recommended that patients fast from clear liquids at least 2 hours before elective procedures that necessitate general anaesthesia, regional anaesthesia, or sedation/analgesia. A delay in or cancellation of surgery will be in order for this case. Questions regarding components of the coffee (e.g., milk; can determine the length of time for a delay), asking why, and evaluating the preoperative education may all be items to be addressed, especially from a performance improvement perspective, but at this time in caring for this patient, the surgery must be delayed or cancelled.
Canadian Fundamentals of Nursing 6th Edition Potter Test Bank
DIF: Understand TOP: Intervention
REF: 1391 OBJ: Prepare a patient for surgery. MSC: CPNRE: Foundations of Practice
23. An appendectomy is appropriately documented by the nurse as which type of surgery? a. Diagnostic surgery. b. Palliative surgery. c. Ablative surgery. d. Reconstructive surgery. ANS: C
Ablative surgery is the excision or removal of a diseased body part, as in an appendectomy. Diagnostic surgery is surgical exploration that allows the physician to confirm a diagnosis. This type of surgery may involve removal of tissue for further diagnostic testing. An example would be a breast mass biopsy. Palliative surgery relieves or reduces the intensity of disease symptoms. It will not produce a cure. An example is resection of nerve roots. Reconstructive surgery restores function or appearance to traumatized or malfunctioning tissues. An example is internal fixation of a hip fracture. DIF: Apply REF: 1366, Table 48-1 OBJ: Differentiate between classifications of surgery and types of anaesthesia. TOP: Intervention MSC: CPNRE: Foundations of Practice 24. The nurse has completed a preoperative assessment for a patient going to surgery and gathers
assessment data. Of the following, which would be the most important next step? a. Notify the operating suite that the patient has a latex allergy. b. Document that the patient had a bath at home this morning. c. Ask the nursing assistantN toUoRbS taI inNvG itaTl B sig.nC s.OM d. Administer the ordered preoperative intravenous antibiotic. ANS: A
Innumerable products that contain latex are used in the operating suite and the PACU. For a patient with this allergy, special considerations are required for preparation of the room to the types of tubes, gloves, drapes, and instruments utilized. Ensuring that the patient has a safe environment takes time, and if the correct supplies are not available, a delay in their arrival may necessitate cancelling or delaying the surgery. Obtaining vital signs, documenting, and administering medications are all part of the process and should be done—with the latex allergy in mind. However, making sure that the operating suite and PACU are safe environments is the first step. DIF: Apply TOP: Assessment
REF: 1371 OBJ: Prepare a patient for surgery. MSC: CPNRE: Foundations of Practice
25. The nurse is preparing a patient for a surgical procedure on the right great toe. Which of the
following actions would be most important to include in this patient’s preparation? a. Ascertain that the surgical site has been correctly marked. b. Ascertain where the family will be located during the procedure. c. Place the patient in a clean surgical gown. d. Ask the patient to remove all hairpins and cosmetics. ANS: A
Canadian Fundamentals of Nursing 6th Edition Potter Test Bank Because errors have occurred, with patients undergoing the wrong surgery on the wrong site, the universal protocol has been implemented and is used with all invasive procedures. Part of this protocol includes marking the operative site with indelible ink. Knowing where the family is during a procedure, placing the patient in a clean gown, and asking the patient to remove all hairpins and cosmetics are important but are not most important in this list of items. DIF: Apply TOP: Assessment
REF: 1394 OBJ: Prepare a patient for surgery. MSC: CPNRE: Foundations of Practice
26. The nurse is caring for a patient intraoperatively. Primary roles of the circulating nurse
include which of the following? a. Establishing and implementing the plan of care. b. Maintaining a sterile field. c. Assisting with applying sterile drapes. d. Handing sterile instruments and supplies to the surgeon. ANS: A
The circulating nurse must be a registered nurse and has the responsibilities of preoperative assessment, establishing and implementing the plan of care, evaluating the care provided, and ensuring continuity of care postoperatively. The scrub nurse, who can be a registered nurse, a licensed practical nurse, or a surgical technologist, maintains the sterile field, assists with applying the sterile drapes, and hands sterile instruments and supplies to the surgeon. DIF: Understand TOP: Assessment
REF: 1395 OBJ: Explain the nurse’s role in the operating room. MSC: CPNRE: Foundations of Practice
27. The nurse is caring for a patient in the preoperative holding area of an ambulatory surgery
GoTstBa.ppCrO centre. Which nursing actionNwUoR ulS dI beNm opM riate for this area? a. Monitor vital signs every 15 minutes. b. Empty the urinary drainage bag. c. Apply a warm blanket. d. Check the surgical dressing. ANS: C
The temperature in the preoperative holding area and in adjacent operating suites is usually cool. The nurse should offer the patient an extra warm blanket. The main activities in this area include verification of the patient’s identity, the surgery to be performed, and the patient’s physical and emotional readiness for the procedure. The intravenous catheter is usually inserted, and the preoperative checklist is reviewed. Vital signs are not normally monitored unless there is a specific reason, such as a medication being administered. Ambulatory surgical patients do not typically come to the holding area with a urinary drainage bag or a surgical dressing. Taking care of those, if appropriate, is performed in the PACU. DIF: Apply TOP: Intervention
REF: 1396 OBJ: Explain the nurse’s role in the operating room. MSC: CPNRE: Foundations of Practice
28. The nurse is caring for a patient in the operating suite. Which of the following outcomes
would be most appropriate for this patient? a. At the end of the intraoperative phase, the patient will be free of burns at the grounding pad. b. At the end of the intraoperative phase, the patient will be free of infection.
Canadian Fundamentals of Nursing 6th Edition Potter Test Bank c. At the end of the intraoperative phase, the patient will be free of nausea and
vomiting. d. At the end of the intraoperative phase, the patient will be free of pain. ANS: A
A primary focus of intraoperative care is to prevent injury and complications related to anaesthesia, surgery, positioning, and equipment use, including use of the electrical cautery grounding pad for prevention of burns. The perioperative nurse is an advocate for the patient during surgery and protects the patient’s dignity and rights at all times. Evaluation of many goals and outcomes does not occur until after surgery. Signs and symptoms of infection do not have the time to manifest during the intraoperative phase. During the intraoperative phase, the patient is anaesthetized and unconscious, and an endotracheal tube that prevents conversation and complaints is in place. Nausea, vomiting, and pain typically begin in the postoperative phase of the experience. DIF: Understand TOP: Planning
REF: 1396 OBJ: Explain the nurse’s role in the operating room. MSC: CPNRE: Foundations of Practice
29. The nurse is caring postoperatively for a patient with a history of obstructive sleep apnea. The
nurse monitors for which of the following? a. Choking and noisy, irregular respirations. b. Shallow respirations. c. Moaning and reports of pain. d. Disorientation. ANS: A
One of the greatest concerns after general anaesthesia is airway obstruction. Choking and noisy, irregular respirations aN reUcR laS ssI icNsG igT nsBa. ndCsOyM mptoms of airway obstruction. A number of factors contribute to obstruction, including a history of obstructive sleep apnea; weak pharyngeal/laryngeal muscle tone from anaesthetics; secretions in the pharynx, bronchial tree, or trachea; and laryngeal or subglottic edema. In the postanaesthetic patient, the tongue is a major cause of airway obstruction. Shallow respirations are indicative of respiratory depression. Moaning and reports of pain are common in all surgical patients and are expected. Disorientation is common when the patient first awakens from anaesthesia, but it can be a sign of hypoxia. DIF: Understand REF: 1399 | 1400 OBJ: Describe the rationales for nursing interventions designed to prevent postoperative complications. TOP: Planning MSC: CPNRE: Foundations of Practice 30. The nurse is caring for a patient in the operating suite who is experiencing tachypnea,
tachycardia, premature ventricular contractions, and muscle rigidity. The nurse suspects that this patient may be experiencing which of the following? a. Hypoxia. b. Malignant hyperthermia. c. Fluid imbalance. d. Hemorrhage. ANS: B
Canadian Fundamentals of Nursing 6th Edition Potter Test Bank A life-threatening, rare complication of anaesthesia is malignant hyperthermia. Malignant hyperthermia causes tachycardia, tachypnea, premature ventricular contractions, unstable blood pressure, cyanosis, skin mottling, and muscular rigidity. It often occurs during induction of anaesthesia. Hypoxia would manifest with decreased oxygen saturation. Fluid imbalance would be assessed with intake and output and can manifest with tachycardia and blood pressure fluctuations but is not characterized by muscle rigidity. Hemorrhage can manifest with tachycardia and decreased blood pressure, along with a thready pulse; usually some sign or symptom of blood loss is noted (e.g., from drains incision, orifice, and abdomen). DIF: Analyze REF: 1401 OBJ: Describe the rationales for nursing interventions designed to prevent postoperative complications. TOP: Evaluate MSC: CPNRE: Foundations of Practice 31. The nurse is caring for a postoperative patient who has undergone a carpal tunnel repair. The
patient has a temperature of 36.1°C and is shivering. Which of the following is the best reason for this condition? a. The patient is dressed only in a gown. b. Anaesthesia lowers metabolism. c. The surgical suite has laminar flow. d. The open body cavity contributed to heat loss. ANS: B
The operating suite and recovery room environments are cool. The patient’s anaesthetically depressed level of body function results in lowering of metabolism and a fall in body temperature. The patient’s being dressed in a gown and laminar flow in the surgical suite can contribute to a decrease in temperature, but the length of time required for this procedure would minimize this effect. Also, the patient in this type of case does not have a large open N RSINGTB.COM body cavity to contribute to heU at loss. DIF: Analyze REF: 1401 | 1406 OBJ: Describe the rationales for nursing interventions designed to prevent postoperative complications. TOP: Evaluate MSC: CPNRE: Foundations of Practice 32. The nurse is monitoring a patient in PACU for postoperative fluid and electrolyte imbalance.
Which of the following actions would be most appropriate for this patient? a. Encouraging copious amounts of water. b. Weighing the patient and comparing with preoperative weight. c. Measuring and recording all intake and output. d. Starting an additional intravenous (IV) line. ANS: C
Accurate recording of intake and output reflects renal and circulatory function. The nurse should measure and record all sources of intake and output. Copious amounts of water in a postoperative patient might produce nausea and vomiting. In the PACU, it is impractical to weigh the patient who is waking from surgery, but in the days afterward, it is a good assessment parameter for fluid imbalance. Starting an additional IV line is not necessary and is not important at this juncture. DIF: Apply REF: 1407 OBJ: Describe the rationales for nursing interventions designed to prevent postoperative complications. TOP: Intervention MSC: CPNRE: Foundations of Practice
Canadian Fundamentals of Nursing 6th Edition Potter Test Bank
33. The nurse is caring for a patient in the PACU. The patient asks for a bedpan and states to the
nurse, “I feel as if I need to go to the bathroom, but I can’t.” Which of the following nursing interventions would be most appropriate? a. Encourage the patient to wait a minute and try again. b. Call the physician and obtain an order for catheterization. c. Assess the patient’s intake and the patient for bladder distension. d. Inform the patient that everyone feels this way after surgery. ANS: C
Depending on the surgery, some patients do not regain voluntary control over urinary function for 6 to 8 hours after anaesthesia. The nurse should assess the amount of fluid that the patient obtained while in surgery and palpate the lower abdomen just above the symphysis pubis for bladder distension. If fluid intake is not excessive and the bladder is nondistended, allowing some time might be appropriate. Not all postsurgical patients feel as if they need to urinate but cannot. If the bladder is distended and the patient is unable to void, catheterization might be in order. DIF: Apply REF: 1407 OBJ: Describe the rationales for nursing interventions designed to prevent postoperative complications. TOP: Intervention MSC: CPNRE: Foundations of Practice 34. The ambulatory surgical nurse calls to check on the patient at home the morning after surgery.
The patient is reporting continued nausea and vomiting. Which of the following discharge education points should be reviewed with the patient? a. Instruct the patient to take deep breaths. b. Instruct the patient to drink ginger ale and eat crackers. IN GT M to connect the patient with the c. Instruct the patient to callNtU heRpS hys icia n,B.C and aO ttempt physician. d. Instruct the patient to go to the emergency department. ANS: C
Postoperative nausea and vomiting sometimes occur once the patient is at home even if symptoms were not present in the surgery centre. Options for therapy include medications. Instructing the patient to call the physician and connecting the patient with the physician can help the patient to obtain relief. Taking deep breaths, drinking ginger ale, and eating crackers are interventions that may be helpful, but this patient needs additional help. Instructing the patient to go to the emergency department is an option with continued nausea and vomiting. DIF: Understand REF: 1365 | 1408 OBJ: Explain the differences and similarities in caring for ambulatory (day) surgical patients versus inpatient surgical patients. TOP: Intervention MSC: CPNRE: Foundations of Practice 35. The nurse is caring for an ambulatory surgical patient. To be discharged home, which one of
the following criteria must the patient meet? a. Ability to eat crackers. b. Manageable pain. c. Ability to pass gas. d. Ability to dress self. ANS: B
Canadian Fundamentals of Nursing 6th Edition Potter Test Bank To be discharged home, patients need to meet certain criteria. These criteria include meeting phase 1 criteria of activity, circulation, respiration, consciousness, and oxygen saturation, as well as phase 2 criteria of dry and intact dressing, manageable pain, ambulation, ability to drink fluids, and voiding. Passing gas, eating, and the ability to dress oneself are not included in these criteria. DIF: Remember REF: 1399, Table 48-9 OBJ: Describe the rationales for nursing interventions designed to prevent postoperative complications. TOP: Assessment MSC: CPNRE: Foundations of Practice 36. The nurse is preparing for a patient who will be going to surgery. The nurse screens for risk
factors that can increase a person’s risks in surgery. Which following risk factor is included in the nurse’s screening? a. Ethnicity. b. Obesity. c. Family support. d. Ambulatory status. ANS: B
Very young and old patients are at risk during surgery because of immature or declining physiological status. Normal tissue repair and resistance to infection depend on adequate nutrients. Obesity increases surgical risk by reducing respiratory and cardiac function. During pregnancy, the concern is for the mother and the developing fetus. Because all major systems of the mother are affected during pregnancy, risks for operative complications are increased. Ethnicity, family support, and ambulatory status are not risks associated with a surgical procedure. DIF: Remember REF: 14N 02U|R 1S 40I 3 NGTB.COM OBJ: List factors to include in the preoperative, intraoperative, and postoperative assessment of a surgical patient. TOP: Assessment MSC: CPNRE: Foundations of Practice 37. The nurse is providing preoperative education and reviews with the patient what it will be like
to be in the surgical environment. What one of the following points should the nurse include? a. The operative suite will be very dark. b. Families are not allowed in the operating suite. c. The operating table or bed will be comfortable and soft. d. The surgical area is very warm. ANS: B
Families are not allowed in the operating suite for several reasons, which include infection control and the emotional effect of seeing a loved one in that condition. The rooms are very bright so that everyone can see, and the operating table is very uncomfortable for the patient. The operating suite itself is kept cool to decrease microbial growth, so it can be very cold to patients as they enter the suite, particularly with limited clothing. DIF: Apply TOP: Intervention
REF: 1395 | 1396 OBJ: Design a preoperative teaching plan. MSC: CPNRE: Foundations of Practice
Canadian Fundamentals of Nursing 6th Edition Potter Test Bank
Chapter 01: Health and Wellness Potter et al: Canadian Fundamentals of Nursing, 6th Edition MULTIPLE CHOICE 1. The nurse is using the population health promotion model to develop actions for improving
health. After asking, “On what should we take action?”; “How should we take action?”; and “Why should we take action?” the nurse will ask which of the following questions? a. “With whom should we act?” b. “When should we take action?” c. “Which government should take action?” d. “Where should we first act?” ANS: A
The next question to ask when using the population health model approach is “With whom should we act?” The other choices are not questions included in this model. DIF: Apply REF: 13 (Figure 1-5) OBJ: Contrast distinguishing features of health promotion and disease prevention. TOP: Implementation MSC: NCLEX: Health Promotion and Maintenance 2. The principle “Health promotion is multisectoral” means which of the following? a. Relationships between individual, social, and environmental factors must be
recognized. b. Physical, mental, social, ecological, cultural, and spiritual aspects of health must
be recognized. N R I G B.C M c. In order to change unhealthU y liS vingNanT d workiO ng conditions, areas other than health must also be involved. d. Health promotion uses knowledge from disciplines such as social, economic, political, environmental, medical, and nursing sciences, as well as from first-hand experience. ANS: C
The statement “Health promotion is multisectoral” is the principle explained by the necessity to involve areas other than health in order to change unhealthy living and working conditions. DIF: Understand REF: 11 OBJ: Contrast distinguishing features of health promotion and disease prevention. TOP: Planning MSC: NCLEX: Health Promotion and Maintenance 3. According to the World Health Organization, what is the best description of “health”? a. Simply the absence of disease. b. Involving the total person and environment. c. Strictly personal in nature. d. Status of pathological state. ANS: B
Canadian Fundamentals of Nursing 6th Edition Potter Test Bank WHO defines health as “. . .the extent to which an individual or group is able, on the one hand, to realize aspirations and satisfy needs; and, on the other hand, to change or cope with the environment. Health is, therefore, seen as a resource for everyday life, not the objective of living; it is a positive concept emphasizing social and personal resources, as well as physical capacities.” Nurses’ attitudes toward health and illness should consider the total person, as well as the environment in which the person lives. People free of disease are not equally healthy. Views of health have broadened to include mental, social, and spiritual well-being, as well as a focus on health at family and community levels. Conditions of life, rather than pathological states, are what determine health. DIF: Knowledge REF: 2 OBJ: Discuss ways that definitions of health have been conceptualized. TOP: Evaluate MSC: NCLEX: Health Promotion and Maintenance 4. What priority strategy for health promotion in Canada is optional but seen as important to
incorporate in nursing education curricula? a. Knowledge of disease prevention. b. Strategies for health promotion. c. Policy advocacy. d. Concepts of determinants of health. ANS: C
Increasingly, policy advocacy is incorporated into nursing role statements and nursing education curricula. Nurses should think about policies that have contributed to health problems, policies that would help alleviate health problems, and how nurses champion public policies. Disease prevention, health promotion, and concepts of determinants of health are integral parts of nursing curricula. I G B.C M
N R U S N T
O
DIF: Understand REF: 11| 12 OBJ: Analyze how the nature and scope of nursing practice are influenced by different conceptualizations of health and health determinants. TOP: Planning MSC: NCLEX: Health Promotion and Maintenance 5. Which of the following is a prerequisite for health, as identified by the Ottawa Charter for
Health Promotion? a. Education. b. Social support. c. Self-esteem. d. Physical environment. ANS: A
Education is one of the nine prerequisites for health that were identified in the Ottawa Charter for Health Promotion. Lack of social support and low self-esteem were identified as psychosocial risk factors by Labonte (1993). Dangerous physical environments were identified as socioenvironmental risk factors by Labonte (1993). DIF: Understand REF: 4 OBJ: Discuss contributions of the following Canadian publications to conceptualizations of health and health determinants: Lalonde Report, Ottawa Charter, Epp Report, Strategies for Population Health, Jakarta Declaration, Bangkok Charter, Toronto Charter. TOP: Planning MSC: NCLEX: Health Promotion and Maintenance
Canadian Fundamentals of Nursing 6th Edition Potter Test Bank 6. The determinant of health with the greatest effect on the health of Canadians is which of the
following? a. Education. b. Health services. c. Social support networks. d. Income and social status. ANS: D
Income, income distribution, and social status constitute the greatest determinant of health because they influence most other determinants. Some investigators suggest that literacy and education are important influences on health status because they affect many other health determinants. Approximately 25% of a population’s health status is attributed to the quality of its health care services. Social support affects health, health behaviours, and health care utilization but is not the greatest determinant of health. DIF: Understand REF: 6 OBJ: Discuss key health determinants and their interrelationships and how they influence health. TOP: Planning MSC: NCLEX: Health Promotion and Maintenance 7. A paraplegic patient in the hospital for an electrolyte imbalance is receiving care at which
prevention level? a. Primary prevention level. b. Secondary prevention level. c. Tertiary prevention level. d. Health promotion level. ANS: B
The secondary prevention levNelUfR ocu onTeBa. rlyCde tection of disease once pathogenesis has SIsesNG OM occurred, so that prompt treatment can be initiated to halt disease and limit disability. The primary prevention level focuses on health promotion, specific protection measures such as immunizations, and the reduction of risk factors such as smoking. The tertiary prevention level focuses on minimizing residual disability. DIF: Apply REF: 11 OBJ: Contrast distinguishing features of health promotion and disease prevention. TOP: Implementation MSC: NCLEX: Health Promotion and Maintenance 8. The nurse incorporates levels of prevention on the basis of patient needs and the type of
nursing care provided. Which of the following is an example of tertiary level preventive caregiving? a. Teaching a patient how to irrigate a new temporary colostomy. b. Providing a lesson on hygiene for an elementary school class. c. Informing a patient that immunizations for her infant are available through the health department. d. Arranging for a hospice nurse to visit with the family of a patient with cancer. ANS: D
Canadian Fundamentals of Nursing 6th Edition Potter Test Bank Tertiary prevention is provided when a defect or disability is permanent and irreversible. At this level, the hospice nurse aims to help the patient and his or her family to achieve a high level of function, despite the limitations caused by the patient’s illness. Teaching a patient how to irrigate a new colostomy is an example of secondary prevention. If the colostomy is to be permanent, care may later move to the tertiary level of prevention. Providing a lesson on hygiene for an elementary school class and informing a patient about available immunizations are examples of primary prevention. DIF: Apply REF: 11 TOP: Implementation
OBJ: Discuss the three levels of disease prevention. MSC: NCLEX: Health Promotion and Maintenance
9. The nurse is working on a committee to evaluate the need for increasing the levels of fluoride
in the drinking water of the community. In doing so, the nurse is fostering which concept? a. Anticipatory prevention. b. Primary prevention. c. Secondary prevention. d. Tertiary prevention. ANS: B
Fluoridation of municipal drinking water and fortification of homogenized milk with vitamin D are examples of primary prevention strategies. With active strategies of health promotion, individuals are motivated to adopt specific health programs such as weight reduction and smoking cessation programs. “Anticipatory prevention” is not a known concept. Secondary prevention promotes early detection of disease (e.g., screening). Tertiary prevention activities are initiated in the convalescence phase of disease. DIF: Apply REF: 11 OBJ: Di scuss the three levels of disease prevention. NURSINGMTSC: B.NCC M Health Promotion and Maintenance TOP: Implementation OLEX: 10. The nurse is working in a clinic that is designed to provide health education and
immunizations. As such, this clinic focuses on which type of prevention? a. Primary prevention. b. Secondary prevention. c. Tertiary prevention. d. Diagnosis and prompt intervention. ANS: A
Primary prevention precedes disease or dysfunction and is applied to people considered physically and emotionally healthy. Health promotion includes health education programs, immunizations, and physical and nutritional fitness activities. Secondary prevention focuses on individuals who are experiencing health problems or illnesses and who are at risk for developing complications or worsening conditions; activities are directed at diagnosis and prompt intervention. Tertiary prevention is provided when a defect or disability is permanent and irreversible. It involves minimizing the effects of long-term disease or disability through interventions directed at preventing complications and deterioration. DIF: Understand REF: 11 TOP: Implementation
OBJ: Discuss the three levels of disease prevention. MSC: NCLEX: Health Promotion and Maintenance
Canadian Fundamentals of Nursing 6th Edition Potter Test Bank 11. The patient is admitted to the emergency department of the local hospital from home with
reports of chest discomfort and shortness of breath. She is administered oxygen and breathing treatments, laboratory tests and blood gas measurements are performed, and electrocardiography is conducted. What level of preventive care is this patient receiving? a. Primary prevention. b. Secondary prevention. c. Tertiary prevention. d. Health promotion. ANS: B
Secondary prevention focuses on individuals who are experiencing health problems or illnesses and who are at risk for developing complications or worsening conditions. Activities are directed at diagnosis and prompt intervention. Primary prevention precedes disease or dysfunction and is applied to people considered physically and emotionally healthy. Health promotion includes health education programs, immunizations, and physical and nutritional fitness activities. Tertiary prevention is provided when a defect or disability is permanent and irreversible. It involves minimizing the effects of long-term disease or disability through interventions directed at preventing complications and deterioration. DIF: Apply REF: 11 TOP: Implementation
OBJ: Discuss the three levels of disease prevention. MSC: NCLEX: Health Promotion and Maintenance
12. A patient is admitted to a rehabilitation facility after a stroke. The patient has right-sided
paralysis and is unable to speak. The patient will be receiving physiotherapy and speech therapy. What are these examples of? a. Primary prevention. b. Secondary prevention. NURSINGTB.COM c. Tertiary prevention. d. Health promotion. ANS: C
Tertiary prevention is provided when a defect or disability is permanent and irreversible. It involves minimizing the effects of long-term disease or disability through interventions directed at preventing complications and deterioration. Secondary prevention focuses on individuals who are experiencing health problems or illnesses, and who are at risk for developing complications or worsening conditions. Activities are directed at diagnosis and prompt intervention. Primary prevention precedes disease or dysfunction and is applied to people considered physically and emotionally healthy. Health promotion includes health education programs, immunizations, and physical and nutritional fitness activities. DIF: Apply REF: 11 TOP: Implementation
OBJ: Discuss the three levels of disease prevention. MSC: NCLEX: Health Promotion and Maintenance
13. Risk factors can be placed in the following interrelated categories: genetic and physiological
factors, age, physical environment, and lifestyle. The presence of any of these risk factors means which of the following? a. A person with the risk factor will get the disease. b. The chances of getting the disease are increased. c. The disease is guaranteed not to develop if the risk factor is controlled. d. Risk modification will have no effect on disease prevention.
Canadian Fundamentals of Nursing 6th Edition Potter Test Bank ANS: B
The presence of risk factors does not mean that a disease will develop, but risk factors increase the chances that the individual will experience a particular disease or dysfunction. Control of risk factors does not guarantee that a disease will not develop. However, risk factor identification assists patients in visualizing those areas in life that can be modified or even eliminated to promote wellness and prevent illness. DIF: Knowledge REF: 3 OBJ: Identify factors that have led to each approach to health. MSC: NCLEX: Health Promotion and Maintenance
TOP: Assessment
14. Since the early 1990s, which group has had the highest amount of absenteeism of all workers
in Canada? a. “White collar sector” workers. b. Nurses. c. Workers in the trades. d. Transport and equipment operators. ANS: B
There is considerable concern regarding negative workplace conditions in the health care sector. Nurses have had the highest or second-highest rate of absenteeism of all workers in Canada since the early 1990s. Rates of absenteeism for the “white collar sector,” for workers in the trades, and for transport and equipment operators are not available. DIF: Analyze REF: 7 OBJ: Analyze how the nature and scope of nursing practice are influenced by different conceptualizations of health and health determinants. TOP: Planning MSC: NCLEX: Health PromotN ioU n aRnS dI MN aiG nteTnB an. ceCOM 15. Which of the following is a true statement about nutrition in Canada, according to the
research? a. Canadians have increased their total fat and salt consumption. b. Canadians report that their children eat the recommended daily number of fruits and vegetables. c. Fifty percent of children aged 2 to 17 years are overweight or obese. d. The 2004 Canadian Community Health Survey (CCHS) revealed that 40% of adult Canadians were obese (body mass index of 30 or more) and 50% were overweight. ANS: A
One quarter of Canadians overall, and one third of teenagers aged 14 to 18 years, reported eating at a fast-food outlet the previous day; such foods are high in fats and salts. Seventy percent of children aged 4 to 8 ate fewer than the minimum servings of fruits and vegetables daily. Of children aged 2 to 17 years, 26% were overweight or obese, not 50%. The 2004 Canadian Community Health Survey (CCHS) revealed that 23% of adult Canadians were obese (body mass index of 30 or more), not 40%, and 36% were overweight, not 50%. DIF: Apply REF: 8| 9 OBJ: Discuss key health determinants and their interrelationships and how they influence health. TOP: Planning MSC: NCLEX: Health Promotion and Maintenance
Canadian Fundamentals of Nursing 6th Edition Potter Test Bank 16. One of the five health promotion strategies, as identified by the Ottawa Charter for Health
Promotion, is which of the following? a. Creating supportive environments. b. Strengthening educational opportunities. c. Developing a medical approach. d. Minimizing stressful situations. ANS: A
“Creating supportive environments” is one of the five broad health promotion strategies identified by the Ottawa Charter for Health Promotion. The other strategies are building healthy public policy, strengthening community action, developing personal skills, and reorienting health services. “Strengthening educational opportunities,” “Developing a medical approach,” and “Minimizing stressful situations” are not among the five strategies. DIF: Apply REF: 11| 12 OBJ: Discuss contributions of the following Canadian publications to conceptualizations of health and health determinants: Lalonde Report, Ottawa Charter, Epp Report, Strategies for Population Health, Jakarta Declaration, Bangkok Charter, Toronto Charter. TOP: Planning MSC: NCLEX: Health Promotion and Maintenance 17. Which of the following is an example of tertiary prevention? a. Reduction of risk factors, such as smoking. b. Breast self-examination and testicular self-examination. c. Cardiac rehabilitation programs. d. Blood pressure screening to detect hypertension. ANS: C
Tertiary prevention activitiesNoccR thG e coBn. vaC lescMence stage of disease and are directed U urSiIn N toward minimizing residual disability andT helpingOpeople to live productively with limitations. An example is a cardiac rehabilitation program after a myocardial infarction. Breast self-examination and testicular self-examination are examples of secondary prevention, as is blood pressure screening to detect hypertension. Reducing risk factors, such as smoking, is an example of primary prevention. DIF: Understand REF: 11 TOP: Implementation
OBJ: Describe the three levels of disease prevention. MSC: NCLEX: Health Promotion and Maintenance
18. When the nurse is educating an adult patient about health promotion activities, which of the
following is the most important internal patient factor for the nurse to consider? a. Emotional wellness. b. Developmental stage. c. Professed spirituality. d. Levels of education and literacy. ANS: D
Canadian Fundamentals of Nursing 6th Edition Potter Test Bank Levels of education and literacy are important influences to consider when the nurse is educating an adult patient about health promotion activities. Literacy can influence health both directly (e.g., with regard to medication use, safety practices) as well as indirectly (through use of services, lifestyles, income, work environments, and stress levels). Spirituality is reflected in a person’s values and beliefs, the relationships established with family and friends, and the ability to find hope and meaning in life; however, these can change throughout life; however, it is not the most important factor to consider. The nurse must consider the patient’s level of growth and development when using the patient’s health beliefs and practices as a basis for planning care, but this is not the most important patient factor to consider. In this case, the patient is at the adult developmental stage. The patient’s emotional wellness—degree of stress, depression, or fear, for example—can influence health beliefs and practices. The manner in which a person handles stress throughout each phase of life will influence the way he or she reacts to illness. However, this is not the best available option. DIF: Analyze REF: 7 OBJ: Discuss key health determinants and their interrelationships and how they influence health. TOP: Implementation MSC: NCLEX: Health Promotion and Maintenance 19. When discussing the effect of a known risk factor on a patient’s health, what would the nurse
would say? a. “It doesn’t mean that you’ll get the disease, just that the odds are greater for you.” b. “Now that you know the possibility is there, you can take steps to prevent it.” c. “This risk factor can be managed by making a change to your lifestyle.” d. “You’re lucky because you have the benefit of being able to do something about it.” ANS: A
NURSINGTB.COM
The presence of risk factors does not mean that a disease will develop, but risk factors increase the chances that the individual will experience a particular disease or dysfunction. Although the statement “Now that you know the possibility is there, you can take steps to prevent it” is not incorrect, it does not address the effect of the risk factor on the patient’s health. It is not always true that a risk factor can be managed by making lifestyle changes. The strategy of telling the patient that he or she is lucky and has the benefit of being able to do something about a risk factor minimizes the patient’s concern, and does not address the effect of the risk factor on the patient’s health. DIF: Apply REF: 3 OBJ: Describe key characteristics of medical, behavioural, and socioenvironmental approaches to health. TOP: Implementation MSC: NCLEX: Health Promotion and Maintenance MULTIPLE RESPONSE 1. Classifications of health conceptualizations occur in which ways? (Select all that apply.) a. Health as stability. b. Health as free from illness. c. Health as universal. d. Health as actualization. e. Health as individual. ANS: A, D
Canadian Fundamentals of Nursing 6th Edition Potter Test Bank Health can be conceptualized (Pender, 2006) in three ways: health as stability, health as actualization, and health as both of these together. DIF: Understand REF: 2 OBJ: Discuss ways that definitions of health have been conceptualized. TOP: Assessment MSC: NCLEX: Health Promotion and Maintenance 2. Health challenges, as identified by Achieving Health for All: A Framework for Health
Promotion (Epp, 1986), include which of the following? (Select all that apply.) a. Mutual aid b. Reducing inequities c. Increasing prevention d. Enhancing coping ANS: B, C, D
Health challenges, as identified by Epp (1986), include reducing inequities, increasing prevention, and enhancing coping. Mutual aid is a health promotion mechanism in the framework; it does not refer to a health challenge. DIF: Understand REF: 4 OBJ: Discuss contributions of the following Canadian publications to conceptualizations of health and health determinants: Lalonde Report, Ottawa Charter, Epp Report, Strategies for Population Health, Jakarta Declaration, Bangkok Charter, Toronto Charter. TOP: Assessment MSC: NCLEX: Health Promotion and Maintenance
NURSINGTB.COM
Canadian Fundamentals of Nursing 6th Edition Potter Test Bank
Chapter 02: The Canadian Health Care Delivery System Potter et al: Canadian Fundamentals of Nursing, 6th Edition MULTIPLE CHOICE 1. What is the fastest growing component of the health care system? a. Primary care. b. Respite care. c. Home care. d. Palliative care. ANS: C
Canadian health care is shifting from an institution-based system to one in which community care is playing a greater role. Home care is one of the fastest growing components of the health care system, partly because patients are sent home from hospital sooner than they used to be. Primary care, respite care, and palliative care are not the fastest growing components of the health care system. DIF: Apply REF: 27 OBJ: Describe five levels of health care and associated types of services. TOP: Assessment MSC: NCLEX: Health Promotion and Maintenance 2. Which of the following is a contributing factor to the shortage of nurses in Canada? a. Increased birth rate, which creates a greater demand for nursing services. b. New technology, which replaces nurses at the bedside. c. Fewer people wishing to practise as nurses because health trends are focusing on
NU RSs.INGTB.C OM natural and alternative ther apie d. Retirement of a large percentage of nurses. ANS: D
The nursing workforce is currently challenged by the aging of workers, high retirement rates, ethical problems in international recruitment, and lack of full-time positions. The average age of registered nurses in Canada is 44.6 years. The aging of the population is creating the greater demand for nursing services. Neither the replacement of nurses at the bedside by new technology nor the focus of health trends on natural and alternative therapies is a factor identified as contributing to the shortage of nurses. DIF: Apply REF: 19 (Box 2-1) OBJ: Discuss principal factors influencing health care reform and the current health care delivery system. TOP: Planning MSC: NCLEX: Safe and Effective Care Environment 3. A nurse working with a group of Indigenous parents refers to Treaty 6, which includes a
provision for health care services and is often referred to as a. The Truth and Reconciliation Act b. The “medicine chest” clause. c. The Indigenous band agreements. d. The land and relationship understanding. ANS: B
Canadian Fundamentals of Nursing 6th Edition Potter Test Bank Treaty 6 was often referred to as the “medicine chest” clause, as it included a provision for health care services to be provided to Indigenous communities. The Indian Act, the Indigenous band agreements (signed with the British government before Confederation), and land and relationship agreements do not refer to Treaty 6. DIF: Understand REF: 20 OBJ: Identify initiatives related to enhancing quality of the Canadian health care delivery system. TOP: Planning MSC: NCLEX: Health Promotion and Maintenance 4. Which of the following is an example of a true health promotion service provided by a nurse? a. An immunization clinic. b. A diabetic support group. c. A prenatal nutrition class. d. A smoking cessation clinic. ANS: C
Community clinics offer prenatal nutrition classes that promote the health of the woman, fetus, and infant. Building healthy public policy, creating supportive environments, strengthening community action, developing personal skills, and reorienting health services are part of health promotion service. An immunization clinic is an example of disease and injury prevention. A diabetic support group may be an example of a rehabilitation service, to help patients adapt to a change in lifestyle. A smoking cessation clinic may be offered as part of a rehabilitation service or as a disease and injury prevention intervention. DIF: Apply REF: 28 OBJ: Describe five levels of health care and associated types of services. TOP: Implementation MSC: NCLEX: Health Promotion and Maintenance
NURSINGTB.COM
5. When conducting a health care system class for immigrants to Canada, the nurse informs them
that the federal government is responsible for which of the following? a. Providing health care insurance plans. b. Managing and planning insurable health services. c. Delivering health care services to targeted groups. d. Providing long-term care services. ANS: C
The federal government is charged with delivering, or co-delivering, health care services for targeted groups. Health care insurance plans, managing and planning insurable health services, and providing long-term care services are the jurisdiction of the provinces or territories. DIF: Apply REF: 22 OBJ: Identify and define the principles of the Canada Health Act and significant legislation related to the Canadian health care system. TOP: Implementation MSC: NCLEX: Health Promotion and Maintenance 6. The nurse initiated a support group for adolescent parents and is teaching them about
chlorinated water and child immunizations, which are all examples of which of the following? a. Health promotion. b. Disease and injury prevention. c. Supportive care.
Canadian Fundamentals of Nursing 6th Edition Potter Test Bank d. Rehabilitation. ANS: B
Disease and injury prevention includes illness prevention (chlorinated water, immunizations) and support groups. Health promotion services include antismoking services, advocacy for healthy public policy, and provision of wellness services. Supportive care describes services provided over a prolonged period to people who are disabled, who have never been able to function independently, or who have a terminal disease. Rehabilitation, such as physiotherapy, involves restoring optimal health. DIF: Apply REF: 28 OBJ: Describe five levels of health care and associated types of services. TOP: Implementation MSC: NCLEX: Health Promotion and Maintenance 7. Primary care is best described as what? a. Early detection and routine care, as well as prevention. b. Provision of a specialized medical service. c. Treatment of all patients with a minimum level of health insurance. d. Provision of medical services in a patient’s home. ANS: A
Primary care is the first contact of a patient with the health care system that leads to a decision regarding a course of action to resolve any actual or potential health problem. The focus is on early detection and routine care, with emphasis on education to prevent recurrences. Provision of a specialized medical service by a specialist or through referral is an example of secondary care. No patient is refused treatment on the basis of level of insurance. Medical services provided in the patient’s home, although not very common, could be categorized under primary care but would not bN esU tR deS scIriN beGpTriB m. arCyOcaMre. DIF: Understand REF: 28 OBJ: Identify various settings and models of care delivery in the Canadian health care delivery system. TOP: Assessment MSC: NCLEX: Health Promotion and Maintenance 8. A patient comes to the ambulatory care clinic for management of a chronic condition and asks
the nurse for an explanation of the Medicare system. The nurse’s response is based on the knowledge that it is a. Canada’s national health insurance system. b. A fee-for-service insurance plan. c. A plan in which monies from provincial taxes are used for nursing homes. d. A social insurance program for low-income earners. ANS: A
Medicare is a key element of Canada’s social safety net. It is Canada’s national health insurance system, in which taxes are used to finance medically necessary services for all citizens, thus providing “free” health care to all. Medicare is not a fee-for-service insurance plan, a plan in which monies from provincial taxes are used for nursing homes, or a social insurance program for low-income earners. DIF: Understand REF: 19 OBJ: Discuss the evolution of Canada's social safety net and Medicare. TOP: Implementation MSC: NCLEX: Health Promotion and Maintenance
Canadian Fundamentals of Nursing 6th Edition Potter Test Bank 9. In teaching a group of older persons about health care spending, a nurse tells them that about
14% of the Canadian population is in their age group and that this group accounts for what percentage of health care spending? a. 25%. b. 35%. c. 45%. d. 55%. ANS: C
Older persons account for 45% of health care spending in Canada, not 25, not 35, and not 55%. DIF: Apply REF: 22 OBJ: Discuss principal factors influencing health care reform and the current health care delivery system. TOP: Implementation MSC: NCLEX: Health Promotion and Maintenance 10. To whom do the principles of the Canada Health Act of 1984 apply? a. Members of the Canadian Armed Forces. b. Insured residents of Canada. c. Inmates of federal penitentiaries. d. Royal Canadian Mounted Police (RCMP). ANS: B
The principles of the Canada Health Act apply to all insured residents of Canada (i.e., eligible residents) but exclude members of the Canadian Armed Forces, Royal Canadian Mounted Police (RCMP), eligible veterans, inmates of federal penitentiaries, and some refugee claimants. N R I G B.C M
U S N T
O
DIF: Understand REF: 20 OBJ: Identify and define the principles of the Canada Health Act and significant legislation related to the Canadian health care system. TOP: Assessment MSC: NCLEX: Health Promotion and Maintenance 11. When a mother brings her child to be seen in a nurse-managed setting, over concerns that her
child’s rash could be measles, what type of care is rendered? a. Respite care. b. Primary care. c. Supportive care. d. Secondary care. ANS: B
Primary care focuses on early detection and routine care, and can be offered in nurse-managed clinics. Respite care is a component of supportive care. Supportive care describes services provided to disabled and terminally ill patients over a prolonged time period. Secondary care is the provision of specialized medical services by a physician specialist or through referral from a primary care provider. DIF: Understand REF: 28 OBJ: Describe five levels of health care and associated types of services. TOP: Assessment MSC: NCLEX: Health Promotion and Maintenance
Canadian Fundamentals of Nursing 6th Edition Potter Test Bank 12. Which of the following is one of the four pillars of primary health care, as described by the
National Primary Health Care Awareness Strategy (2006)? a. Teams. b. Universality. c. Health promotion. d. Immediate access to primary care. ANS: A
Teams are one of the four pillars, along with access, information, and healthy living. The four pillars do not include universality, health promotion, or immediate access to primary health care. DIF: Understand REF: 25 (Box 2-3) OBJ: Identify various settings and models of care delivery in the Canadian health care delivery system. TOP: Planning MSC: NCLEX: Health Promotion and Maintenance 13. Mr. Jones is 72 years old. He is ambulatory and is to be discharged from hospital after a total
hip replacement, with requirements for daily physiotherapy and respiratory therapy. What is the most appropriate facility referral for the nurse to make? a. A rehabilitation centre. b. An assisted-living facility. c. A community health clinic. d. A home care agency. ANS: D
A home care agency provides health services to individuals and families in their homes. Home care was created to provide individualized care for people after hospital discharge but has increasingly included patientN s in ag. esCfro URa SraInge NGofTB OMm very young to very old; those with mental, physical, or developmental challenges; and those requiring recovery to end-stage care. A rehabilitation centre is not required, as Mr. Jones is ambulatory. Assisted-living facilities are community-based residential facilities in which adults live in their own apartments and are provided with a range of support services such as meals, social and recreational programs, and laundry. A community health clinic is the first point of contact, offering a range of primary services that emphasize prevention, health promotion, health education, community development, and partnerships to develop a healthy local community. DIF: Apply REF: 26| 27 OBJ: Identify various settings and models of care delivery in the Canadian health care delivery system. TOP: Implementation MSC: NCLEX: Health Promotion and Maintenance 14. What is an appropriate referral for an older patient who requires some assistance with daily
activities within a partially protective environment? a. Respite care. b. Rehabilitative care. c. Assisted living. d. Extended care. ANS: C
Canadian Fundamentals of Nursing 6th Edition Potter Test Bank Such a patient requires assisted living. Assisted living allows adults to receive a range of support services, including personalized assistance in achieving a level of independence. Personal assistance services are “designed to promote maximum dignity and independence,” including meal preparation, personal hygiene practice, mobility, and socialization. Respite care is a service that provides short-term relief for family caregivers or persons providing home care to the ill or disabled. Rehabilitative care provides therapy and training, with the goal of decreasing the patient’s dependence on care. An extended, or long-term, care facility provides intermediate medical, nursing, or custodial care for patients recovering from acute or chronic illness or disabilities. DIF: Apply REF: 26| 27 OBJ: Identify various settings and models of care delivery in the Canadian health care delivery system. TOP: Implementation MSC: NCLEX: Health Promotion and Maintenance 15. The Canadian Patient Safety Institute (CPSI) provides leadership in building and advancing a
safer health care system. According to the CPSI, about two thirds of preventable incidents that occur during hospitalization are related to what? a. Procedures. b. Care or medication. c. Patient accidents. d. Infection control. ANS: B
According to the CPSI, about two thirds of preventable events are either care related or medication related. About one third of the events are related to procedures or patient accidents. Infections fall under the category of care.
NUR I G B.C M S N T O
DIF: Remember REF: 30 OBJ: Discuss principal factors influencing health care reform and the current health care delivery system. TOP: Planning MSC: NCLEX: Health Promotion and Maintenance 16. “Residents temporarily absent from their home province are to have their incurred services
paid for by the home province, in the same amount that would have been paid by the home province.” This statement reflects which principle of the Canada Health Act (1984)? a. Portability. b. Universality. c. Public administration. d. Accessibility. ANS: A
This statement describes the principle of portability. The principle of universality means that 100% of the insured persons of a province are covered by the health care plan. The principle of public administration means that the health care plan must be administered and operated on a nonprofit basis by a public authority, responsible to the provincial government and subject to audits of its accounts and financing transactions. The principle of accessibility means that the health care plan of a province must provide for (1) insured health services and reasonable access by insured persons; (2) reasonable compensation of physicians and dentists for all insured services rendered; and (3) payments to hospitals in regard to the cost of insured health services.
Canadian Fundamentals of Nursing 6th Edition Potter Test Bank DIF: Understand REF: 21 (Table 2-1) OBJ: Identify and define the principles of the Canada Health Act and significant legislation related to the Canadian health care system. TOP: Planning MSC: NCLEX: Health Promotion and Maintenance MULTIPLE RESPONSE 1. Canadian health care workers’ rights include which of the following? (Select all that apply.) a. Reasonable working conditions. b. To refuse dangerous work. c. To receive appropriate compensation. d. Access to proactive measures. e. Access to their own health care information. ANS: A, B, D
Canadian health care workers’ rights include the right to reasonable working conditions; absence of discrimination, abuse, and harassment of any type; the right to refuse dangerous work; and access to proactive measures (such as personal protective equipment). Receiving appropriate compensation and access to their own health care information are not included in Canadian health care workers’ rights. DIF: Understand REF: 23 OBJ: Discuss multiple roles and challenges for all categories of regulated nurses in different health care settings. TOP: Implementation MSC: NCLEX: Health Promotion and Maintenance 2. A Canadian is explaining to a non-Canadian friend the benefits of the Canadian health care
N R I G B.C M
system. The Canadian discusseU s beSingNablT e to accO ess surgical or medical hospital-based care with no out-of-pocket charges; mentions that if he or she must travel for work to another province within Canada, he or she is still covered; and explains that it is easy to access health care services and that the care received is respectful and embraces diversity. Which of the following principles of the Canada Health Act of 1984 is the Canadian describing? (Select all that apply.) a. Public administration. b. Comprehensiveness. c. Universality. d. Portability. e. Accessibility. f. Protection. ANS: B, C, D, E
The principles described by the Canadian include comprehensiveness, universality, portability, and accessibility. The ability to access surgical or medical hospital-based care with no out-of-pocket charges refers to comprehensiveness; the ability to reasonably access health care services refers to accessibility; and respectful care that embraces diversity refers to universality. Portability refers to the ability to access healthcare services in another province. Public administration is another principle of the Canadian Health Act but is not described by the Canadian. Protection is not a principle of the Canadian Health Act. DIF: Apply REF: 21 OBJ: Identify and define the principles of the Canada Health Act and significant legislation related to
Canadian Fundamentals of Nursing 6th Edition Potter Test Bank the Canadian health care system. TOP: Assessment MSC: NCLEX: Health Promotion and Maintenance
NURSINGTB.COM
Canadian Fundamentals of Nursing 6th Edition Potter Test Bank
Chapter 03: The Development of Nursing in Canada Potter et al: Canadian Fundamentals of Nursing, 6th Edition MULTIPLE CHOICE 1. Which of the following was the main reason why health care facilities in the British colonies
were increasingly needed during the eighteenth-century British regime? a. Scurvy. b. Bubonic plague. c. Increased birth rate. d. Infectious diseases. ANS: D
Infectious diseases carried by immigrants and travellers spread rapidly in the British colonies, which created a need for more health care facilities. Neither scurvy nor bubonic plague was the main health reason for increased health care facilities. The increasing birth rate had nothing to do with an increased need for health care facilities. DIF: Understand REF: 35| 36 OBJ: Discuss the historical development of nursing practice in Canada. TOP: Assessment MSC: NCLEX: Health Promotion and Maintenance 2. What was the main driving force for the modernization of nursing? a. Globalization. b. The Crimean War. c. Missionary nursing. N R I G B.C M d. The women’s suffrage movU O emS ent.N T ANS: A
The history of modern nursing, at its heart, is a story of globalization. Nurses often travelled across geographic and national boundaries to bring nursing service and training to communities in need. Although Florence Nightingale reduced morbidity and mortality rates by applying modern principles of cleanliness and comfort during the Crimean War, this war was not the main driving force for the modernization of nursing. Missionary nursing was the result of a series of sociopolitical factors converging at the same time but was not the main driving force for the modernization of nursing. The women’s suffrage movement was also not the main driving force for the modernization of nursing, although nursing did become an instrument of women’s emancipation from restrictions of middle-class women. DIF: Understand REF: 37 OBJ: Discuss the historical development of nursing practice in Canada. TOP: Planning MSC: NCLEX: Health Promotion and Maintenance 3. Where did the first hospital diploma school in Canada open? a. Manitoba. b. Ontario. c. Quebec. d. Newfoundland. ANS: B
Canadian Fundamentals of Nursing 6th Edition Potter Test Bank Ontario opened the first hospital diploma school in Canada, the St. Catharines Training School, in 1874, at St. Catharines General and Marine Hospital. Manitoba, Quebec, and Newfoundland opened theirs later. DIF: Remember REF: 39 OBJ: Discuss the historical development of nursing education in Canada. TOP: Planning MSC: NCLEX: Health Promotion and Maintenance 4. The formation of the Victorian Order of Nurses (VON) in 1898 was prompted by what? a. Flu epidemic. b. Measles outbreak. c. Labour and birth difficulties. d. War casualties. ANS: C
Lady Ishbel Aberdeen, wife of the governor general of Canada, conceived of the idea of establishing the VON after learning of the plight of women in western Canada who had to give birth in remote locations with no assistance. A flu epidemic, a measles outbreak, and war casualties were not the health care issue responsible for the creation of the VON. DIF: Understand REF: 39 OBJ: Discuss the historical development of professional nursing. TOP: Planning MSC: NCLEX: Health Promotion and Maintenance 5. Which was the first province in Canada to pass legislation related to nursing registration? a. Ontario. b. Nova Scotia. c. New Brunswick. NURSINGTB.COM d. British Columbia. ANS: B
Nova Scotia was the first province in Canada to pass legislation related to nursing registration, with the passing of a voluntary registration act in 1910. Ontario, New Brunswick, and British Columbia passed legislation related to nursing registration later. DIF: Understand REF: 40 OBJ: Discuss the historical development of professional nursing. TOP: Planning MSC: NCLEX: Health Promotion and Maintenance 6. The first undergraduate nursing degree program was established where? a. Ontario. b. Nova Scotia. c. New Brunswick. d. British Columbia. ANS: D
British Columbia was the first province to establish an undergraduate nursing degree program, at the University of British Columbia in 1919. Ontario, Nova Scotia, and New Brunswick establish undergraduate nursing degree programs later. DIF: Remember REF: 41 OBJ: Discuss the historical development of professional nursing.
Canadian Fundamentals of Nursing 6th Edition Potter Test Bank TOP: Planning
MSC: NCLEX: Health Promotion and Maintenance
7. What is a fundamental and guiding principle of French-Canadian hospitals that survived into
the twentieth century? a. Universal precautions are fundamental in preventing nosocomial infections. b. Nursing is primarily focused on the tertiary level of care. c. Nurses need to recognize the effects of their health, values, and beliefs on practice. d. Care is accessible to all, regardless of their background, status in life, or ability to pay. ANS: D
A fundamental and guiding principle of the French-Canadian hospitals that survived largely intact into the twentieth century was that care should be available to all people, regardless of background, status in life, or ability to pay. This continues to be a principle for which nurses, through their professional organizations, have argued doggedly in national debates on the nature and continuing direction of Canada’s national health care insurance program. The statement “Universal precautions are fundamental in preventing nosocomial infections” was not a guiding principle of French-Canadian hospitals. The statements “Nursing is primarily focused on the tertiary level of care” and “Nurses need to recognize the effect of their health, values, and beliefs on practice” do not reflect a contribution from the past to current nursing beliefs and practices. DIF: Apply REF: 45 OBJ: Discuss the historical development of nursing practice in Canada. TOP: Implementation MSC: NCLEX: Health Promotion and Maintenance 8. What is the primary motivator of nursing? a. b. c. d.
NURSINGTB.COM Money. Altruism. Self-efficacy. A curative model of care.
ANS: B
Nursing is practical and theoretical, is motivated by altruism, and is based on professional and ethical standards. Money, self-efficacy, and the curative model of care are not the primary motivators of nursing. DIF: Apply REF: 45 OBJ: Discuss the historical development of nursing practice in Canada. TOP: Assessment MSC: NCLEX: Health Promotion and Maintenance 9. The act that was passed in Ontario in 1997, allowing registered nurses who hold an extended
certificate to provide primary health care services, is called which of the following? a. Ontario Nurse Practitioners Act b. Extended Class Nursing Act c. Expanded Nursing Services for Patients Act d. Primary Care Services Act ANS: C
Canadian Fundamentals of Nursing 6th Edition Potter Test Bank In 1997, Ontario passed the Expanded Nursing Services for Patients Act, which allows registered nurses who hold an extended certificate to provide primary health care services. The act is not called the Ontario Nurse Practitioners Act, the Extended Class Nursing Act, or the Primary Care Services Act. DIF: Remember REF: 43 OBJ: Discuss the historical development of nursing education in Canada. TOP: Assessment MSC: NCLEX: Health Promotion and Maintenance 10. The proportion of male nurses in Canada in 2012 was
, in comparison with
in the year 2000. a. 2.1%; 0.9% b. 6.4%; 4.7% c. 12.4%; 8.6% d. 18.2%; 12.4% ANS: B
According to the most recent statistics, the proportion of male nurses was only 6.4% of the total nurse population in 2012, a slight increase over the 4.7% it represented in 2000. DIF: Understand REF: 39 OBJ: Discuss the historical development of nursing practice in Canada. TOP: Assessment MSC: NCLEX: Health Promotion and Maintenance 11. The Canadian Nurses Association (CNA) has made available to all nurses across Canada a
nursing portal database; what is it called? a. The Nurse. b. NurseOne. NURSINGTB.COM c. Nursing Initiatives. d. Evidence-informed Nursing. ANS: B
NurseOne is the CNA’s nursing portal database and is available to all nurses across Canada. The Nurse, Nursing Initiatives, and Evidence-based Nursing are not names by which the CNA’s nursing portal database is known. DIF: Apply REF: 43 OBJ: Discuss the historical development of nursing practice in Canada. TOP: Planning MSC: NCLEX: Health Promotion and Maintenance 12. Who was the most influential woman in the advancement of nursing in Canada? a. Marie Rollet Hébert. b. Florence Nightingale. c. Mary Agnes Snively. d. Jeanne Mance. ANS: D
Canadian Fundamentals of Nursing 6th Edition Potter Test Bank Jeanne Mance (1606–1673) came to Ville Marie (New France) in 1642. As well as founding and managing Hôtel-Dieu, Mance assisted Sieur de Maisonneuve in running the colony as confidant, advisor, and accountant. She is hailed as a founder of the city of Montreal. Today, the CNA awards its highest honour in the name of this courageous pioneer. Marie Rollet Hébert (1580–1649) was the first laywoman to provide nursing care in New France in 1617. Florence Nightingale (1820–1910), who is considered the founder of modern nursing, was a nineteenth-century British nurse. Mary Agnes Snively (1847–1933) was a teacher before becoming a nurse. Upon graduation from the school of nursing at Bellevue Hospital in New York, she became superintendent of nurses at Toronto General Hospital. Toronto General Hospital subsequently became the largest school of nursing in Canada and a model to others. DIF: Remember REF: 36 (Box 3-1) OBJ: Discuss the historical development of professional nursing. TOP: Planning MSC: NCLEX: Health Promotion and Maintenance 13. What is the primary purpose of licensure laws for the nursing profession? a. To protect the public against unqualified and incompetent practitioners. b. To enhance the quality of nursing care and improve Canadians’ health outcomes. c. To ensure that nurses demonstrate knowledge and skills in a variety of professional
roles. d. To provide an opportunity for practitioners to validate their expertise in a specialty. ANS: A
Licensure laws are designed to protect the public against unqualified and incompetent practitioners. Because constitutional responsibility for education and health falls under the purview of the provinces and territories, each has a nursing practice act to regulate the licensure and practice of nursing. The other statements—“To enhance the quality of nursing N ealt RSh IouNtco GT B.C ToMensure that nurses demonstrate care and improve Canadians’ hU mes”; “O knowledge and skills in a variety of professional roles”; and “To provide an opportunity for practitioners to validate their expertise in a specialty”—do not reflect the primary purpose of licensure laws. DIF: Apply REF: 40| 41 OBJ: Discuss the historical development of professional nursing. TOP: Implementation MSC: NCLEX: Health Promotion and Maintenance 14. Where did the traditions of good nursing practice that anchor nursing in Canada originate? a. England. b. United States. c. Germany. d. New France. ANS: D
Nursing in Canada is rooted in the traditions of good nursing practice that developed in New France, not in England, the United States, or Germany. DIF: Understand REF: 45 OBJ: Discuss the historical development of professional nursing. TOP: Assessment MSC: NCLEX: Health Promotion and Maintenance
Canadian Fundamentals of Nursing 6th Edition Potter Test Bank 15. Basic entry to nursing practice as the baccalaureate degree was first implemented in Canada in
which province? a. Ontario. b. Alberta. c. British Columbia. d. Prince Edward Island. ANS: B
Prince Edward Island was the first province to implement the baccalaureate degree as the requirement for entry to nursing practice, in 1992. In 1975, the Alberta Task Force on Nursing Education proposed their entry-to-practice position, which was that all new nursing graduates be qualified at the baccalaureate level; however, Alberta was not the first to implement this requirement., as was neither Ontario nor British Columbia. DIF: Understand REF: 41 OBJ: Discuss the historical development of nursing education in Canada. TOP: Planning MSC: NCLEX: Health Promotion and Maintenance MULTIPLE RESPONSE 1. Nurses working at Red Cross outpost stations in Ontario between 1922 and 1984 acquired
diverse skills including which of the following? (Select all that apply.) a. Diagnosing illnesses. b. Performing Caesarean sections. c. Suturing wounds. d. Administering anaesthetics. e. Inserting central lines NURSINGTB.COM f. Taking radiographs. ANS: A, C, D, F
Nurses working at Red Cross outpost stations in Ontario between 1922 and 1984 acquired diverse skills that today are considered to be more in the purview of physicians, including suturing wounds, administering anaesthetics, taking radiographs, and diagnosing illnesses. The nurses working at these outpost stations did not perform Caesarean sections or insert central lines. DIF: Understand REF: 38 OBJ: Discuss the historical development of nursing education in Canada. TOP: Planning MSC: NCLEX: Health Promotion and Maintenance
Canadian Fundamentals of Nursing 6th Edition Potter Test Bank
Chapter 04: Community Health Nursing Practice Potter et al: Canadian Fundamentals of Nursing, 6th Edition MULTIPLE CHOICE 1. How does public health nursing differ from community health nursing? a. Public health nurses focus on individuals and families. b. Public health nurses understand the needs of a population. c. Public health nurses ignore political processes. d. Public health nurses consider the individual as one member of a group. ANS: B
Public health nursing requires understanding the needs of a population. A public health nurse understands factors that influence the political processes used to affect public policy. The primary focus of community health nursing is the care of individuals, families, and groups in the community. By focusing on subpopulations, the community health nurse cares for the community as a whole and considers the individual or family as only one member of a group at risk. DIF: Understand REF: 51| 52 OBJ: Differentiate between public health nursing and home health nursing. TOP: Assessment MSC: NCLEX: Health Promotion and Maintenance 2. When discussing social justice, how does the nurse describe it? a. The focus of social justice is on disease prevention. b. Social justice is used only with people in precarious situations. R talIrNesp GT B.CtyMand fairness. c. Social justice is rooted inNsU ocieS onsibiliO d. Social justice is a process to exercise the ability to enhance control. ANS: C
Social justice is rooted in notions of societal (social) responsibility and fairness (justice). “Social justice is the equitable, or fair, distribution of society's benefits, responsibilities and their consequences. It focuses on the relative position of social advantage of one individual or social group in relation to others in society as well as on the root causes of inequities and what can be done to eliminate them” (CNA, 2010, p. 13). A focus of primary health care is disease prevention, not social justice. Social justice applies to all people, not just those in precarious situations, and it is not to be used as a means to enhance control. DIF: Apply REF: 49 OBJ: Describe the standards, competencies, roles, and activities important for success in community health nursing practice. TOP: Implementation MSC: NCLEX: Health Promotion and Maintenance 3. How does a community health nurse differ from the community-based nurse? a. The community health nurse understands the needs of the population. b. The community health nurse focuses on the needs of the individual. c. The community health nurse is the first point of contact in the health care system. d. The community health nurse involves the family in decision making. ANS: A
Canadian Fundamentals of Nursing 6th Edition Potter Test Bank The community health nurse understands the needs of a population or community through experience with individual families in working through their social and health care issues. The community-based nurse (home health nurse) focuses on the needs of the individual or family. Community-based nursing centres function as the first point of contact between members of a community and the health care system. The community-based nurse learns to partner with patients and families so that ultimately the patient and the family become involved in planning, decision making, implementation, and evaluation of health care approaches. DIF: Understand REF: 50 OBJ: Differentiate between public health nursing and home health nursing. TOP: Assessment MSC: NCLEX: Health Promotion and Maintenance 4. The type of nursing that focuses on acute and chronic care of individuals and families while
enhancing patient autonomy is known as what kind of nursing? a. Public health nursing. b. Community health nursing. c. Home health nursing. d. Community-focused nursing. ANS: C
Home health nursing involves acute and chronic care of individuals and families and enhances their capacity for self-care while promoting autonomy in decision making. Public health nursing focuses on the needs of a population. Community health nursing involves caring for the community as a whole, and the individual or the family is considered only one member of a group at risk. Community-focused nursing entails an understanding of the needs of a population or community. DIF: Understand REF: 53NURSINGTB.COM OBJ: Differentiate between public health nursing and home health nursing. TOP: Assessment MSC: NCLEX: Health Promotion and Maintenance 5. The community health nurse is administering flu shots to children at a local playground. In
doing so, the nurse’s focus is on what? a. Preventing individual illness. b. Preventing community outbreak of illness. c. Preventing outbreak of illness in the family. d. The needs of the individual or family. ANS: B
In administering flu shots to members of a subpopulation, the community health nurse attempts to prevent outbreaks of illness in the community as a whole, not necessarily on preventing illness in or meeting the needs of a specific individual or family. DIF: Understand REF: 50 OBJ: Discuss the roles and functions of the community health nurse. TOP: Implementation MSC: NCLEX: Health Promotion and Maintenance 6. A group of teenage girls is receiving counselling about birth control and disease prevention.
Why does the community health nurse provide such counselling? a. Focusing on subpopulations leads to community health. b. Community health nursing focuses on individuals only.
Canadian Fundamentals of Nursing 6th Edition Potter Test Bank c. Community health nursing excludes direct care to subpopulations. d. The focus is on preventing illness and unwanted pregnancy. ANS: A
By focusing on subpopulations, the community health nurse cares for the community as a whole and considers the individual or the family as only one member of a group at risk. In community health nursing, the primary focus is on the health care of individuals, families, and groups in a community. Subpopulations are often a clinical focus. The goal is to protect, promote, and maintain health, not to prevent illness or unwanted pregnancy. DIF: Understand REF: 50 OBJ: Discuss the roles and functions of the community health nurse. TOP: Implementation MSC: NCLEX: Health Promotion and Maintenance 7. Community-based nursing care takes place in community settings such as the home or a
clinic. Why is this an ideal setting? a. To exert greater control over individual or family decisions. b. To provide services close to where patients live. c. To isolate patients and prevent the spread of disease. d. To reduce the need for self-care. ANS: B
The ideal is to provide health care services close to where patients live. This lessens the cost of care, as well as the stress associated with the financial burdens of care. The focus is on the needs of the individual or family, not on controlling their decisions or making patients dependent on nurses. The nurse learns to partner with patients and families so that they assume responsibility for their health care decisions. This form of nursing is not concerned with preventing the spread ofNdU isR eaSseI. NGTB.COM DIF: Understand REF: 53 OBJ: Discuss the roles and functions of the community health nurse. TOP: Planning MSC: NCLEX: Health Promotion and Maintenance 8. The community-based nurse is caring for a patient who is home bound by arthritis and chronic
lung problems. The patient, however, receives many visitors from the neighbourhood and from former co-workers, as well as frequent phone calls from extended family. When concerned about how the large number of visitors may be fatiguing the patient, what should the nurse do? a. Restrict the number of visitors for the patient’s welfare. b. Voice concerns to the patient and proceed according to the patient’s wishes. c. Allow visitors to come and go freely as they have been. d. Create visiting hours when the patient may see non–family members. ANS: B
The nurse learns to partner with patients and families, so that ultimately the patient and the family assume responsibility for their health care decisions, such as the number and frequency of visits by family and friends. DIF: Apply REF: 53 OBJ: Discuss the roles and functions of the community health nurse. TOP: Implementation MSC: NCLEX: Health Promotion and Maintenance
Canadian Fundamentals of Nursing 6th Edition Potter Test Bank 9. The student nurse is trying to determine what type of nurse she wants to be after graduation.
In class, she states that community health nursing is probably not for her because community nursing focuses only on community issues such as preventing epidemics. What would be the instructor’s most appropriate response? a. Community health nursing focuses on the health care of individuals, families, and groups in a community. b. Community health nursing focuses only on the health of a specific subgroup in a community. c. Community health nursing requires an advanced nursing degree, and so the student need not worry. d. Community health nursing focuses only on maintaining the health of the community. ANS: A
The primary focus of community health nursing is on the health care of individuals, families, and groups in a community. Community health nursing includes public health nursing, home health (community-based) nursing, and community mental health nursing, as well as a variety of other areas of specialization such as street health and parish nursing. The goals are to preserve, protect, promote, and maintain health. Not all hiring agencies require community health nurses to have an advanced degree. DIF: Apply REF: 49| 50 OBJ: Describe the standards, competencies, roles, and activities important for success in community health nursing practice. TOP: Implementation MSC: NCLEX: Health Promotion and Maintenance 10. Vulnerable populations include patients who are more likely than others to develop health
problems as a result of what?NURSINGTB.C OM a. Pregnancy. b. Nontraditional healing practices. c. Exposure to excessive risk. d. Unlimited access to health care. ANS: C
Vulnerable populations are patients who are more likely than others to develop health problems as a result of exposure to excessive risk, who experience barriers when trying to access health care services, or who are dependent on others for care. Pregnancy is not a cause of vulnerability, except for mothers who are adolescents, are addicted to drugs, or are at high risk for other reasons. Nontraditional healing practices are part of the culture of many immigrant and Indigenous populations. Many of these healing practices are effective and complement traditional medical therapies. DIF: Knowledge REF: 54 OBJ: Explain the characteristics of patients from vulnerable populations that influence a nurse's approach to care. TOP: Assessment MSC: NCLEX: Health Promotion and Maintenance 11. The instructor is teaching student nurses about identifying members of vulnerable populations
when one nursing student asks, “Why is it that not all poor people are considered members of vulnerable populations?” What would be the instructor’s best answer? a. “All poor people are members of a vulnerable population.” b. “Poor people are members of a vulnerable population only if they take drugs.”
Canadian Fundamentals of Nursing 6th Edition Potter Test Bank c. “Poor people are members of a vulnerable population only if they are homeless.” d. “Members of vulnerable groups frequently have a combination of risk factors.” ANS: D
Members of vulnerable groups frequently have many risk factors or a combination of risk factors that increase their vulnerability to disease and disability. Individual risk factors are not necessarily overwhelming, depending on the patient’s beliefs and values and sources of social support. DIF: Understand REF: 54 OBJ: Explain the characteristics of patients from vulnerable populations that influence a nurse's approach to care. TOP: Assessment MSC: NCLEX: Health Promotion and Maintenance 12. The nurse is making a home visit to a Korean family whose daughter gave birth 6 weeks
earlier. She finds the daughter in bed with a severe headache. The daughter’s father is holding her hand and is pressing different parts of the hand and lower arm. The mother explains that the father is trying to cure the headache by using pressure points. What would be the nurse’s best response? a. Tell the father to stop and give the daughter Tylenol. b. Ask the mother or father, or both, to explain the procedure. c. Explain to the father that what he is doing will not work. d. Let the father finish and then give the daughter Tylenol. ANS: B
The nurse should not judge the patient’s or family’s beliefs and values about health. The nurse needs to create a comfortable, nonthreatening environment and to learn as much as possible about the patient’s culture and values that influence his or her health care practices. Tylenol may not be an acceptable alteNrna tive am . Criticizing the family’s beliefs and Cily UR SIfor NGthis TBf. OM practices will only create a barrier to care. DIF: Apply REF: 54 (Box 4-5) OBJ: Explain the characteristics of patients from vulnerable populations that influence a nurse's approach to care. TOP: Implementation MSC: NCLEX: Health Promotion and Maintenance 13. The nurse is working in a community clinic when a man and woman bring a 12-year-old boy
in, stating that the child fell down a flight of stairs and hurt his arm. The nurse notices several other bruises on the child’s body at varying stages of healing. The boy is placed on the stretcher. When asked how he hurt himself, he states that he does not remember. However, the nurse notices that the boy continuously avoids looking at the man, while the man stares at him constantly. What should the nurse do? a. Ask the boy if the man hurt him. b. Confront the man directly. c. Ask the man and woman to step out of the room. d. Ask the woman if the man hurt the boy. ANS: C
Canadian Fundamentals of Nursing 6th Edition Potter Test Bank Ask the man and woman to step out of the room. When dealing with patients at risk for or who have suffered abuse, it is important to provide protection and to interview the patient at a time when he or she has privacy and the individual suspected of being the abuser is not present. The boy may be less likely to be forthcoming with his attacker in the room. Confronting either adult directly may lead to violence. The other adult may also be a victim of abuse and may fear retribution if their problems are discussed with health care providers. DIF: Apply REF: 55 OBJ: Explain the characteristics of patients from vulnerable populations that influence a nurse's approach to care. TOP: Implementation MSC: NCLEX: Health Promotion and Maintenance 14. The nurse is working with a 16-year-old pregnant girl who tells the nurse that she needs an
abortion. The nurse provides the patient with information on alternatives to abortion, but after several sessions, the patient still insists on having the abortion. The competency of the nurse as a consultant requires what of the nurse? a. Insist that the patient speak with a “right-to-life” advocate. b. Provide a referral to an abortion service. c. Refuse to provide a referral to an abortion service. d. Delay providing a referral to an abortion service. ANS: B
As a consultant, the nurse is responsible for providing information, listening objectively, and being supportive, caring, and trustworthy. The nurse does not make decisions but rather helps the patient reach decisions that are best for him or her. To refuse to provide a referral, to delay referral, or to insist that the patient speak with a “right-to-life” advocate would not be supportive of the patient’s decision. Counsellors usually suggest and rarely insist.
NUR I G B.C M S N T O
DIF: Apply REF: 57 OBJ: Describe the standards, competencies, roles, and activities important for success in community health nursing practice. TOP: Implementation MSC: NCLEX: Health Promotion and Maintenance 15. The patient is in the hospital with the diagnosis of early-onset Alzheimer’s disease. Before the
patient is discharged, the community-based nurse is making a visit to the patient’s home, where he lives with his daughter and her family. What would be a major focus of this visit? a. Demonstrate to the caregiver techniques for providing care. b. Stress to the family how difficult it will be to provide care at home. c. Encourage the family to send the patient to an extended-care facility. d. Teach the family how to have the patient declared incompetent. ANS: A
The role of the community-based nurse, when dealing with patients with Alzheimer’s disease, is to maintain the best possible functioning, protection, and safety for the patient. The nurse should demonstrate to the primary family caregiver the techniques for dressing, feeding, and toileting the patient while providing encouragement and emotional support to the caregiver. The nurse should protect the patient’s rights and maintain family stability. DIF: Apply REF: 55 OBJ: Describe the standards, competencies, roles, and activities important for success in community health nursing practice. TOP: Implementation
Canadian Fundamentals of Nursing 6th Edition Potter Test Bank MSC: NCLEX: Health Promotion and Maintenance 16. The community has three components: structure or locale, the people, and the social systems.
While conducting a community assessment, the nurse seeks data on the average household income and the number of residents on public assistance. In doing so, the nurse is evaluating which of the following? a. Structure. b. Population. c. Welfare system. d. Social system. ANS: A
Economic status is part of the community structure. Population would involve age and gender distribution, growth trends, density, education level, and ethnic or religious groups. The welfare system is part of the social system, which also includes the education, government, communication and health systems. DIF: Understand REF: 58| 59 TOP: Implementation
OBJ: Describe elements of a community assessment. MSC: NCLEX: Health Promotion and Maintenance
17. The patient is being readmitted to an inner-city hospital for chest pain 3 months after having a
heart attack. The patient was referred to the hospital’s cardiac rehabilitation program after her previous admission. The patient states that she began going to the program and liked it but stopped. When asked why, she states that, at the beginning, the classes were at 9 a.m. but then were switched to 7 p.m., when it is dark out. The cardiac rehabilitation program was within walking distance of the patient’s home. What is the most likely cause of the patient’s unwillingness to go to the program? G B.C M N R I a. Lack of transportation U S N T O b. Fear of walking at night c. Reimbursement issues d. Noncompliance ANS: B
A community assessment should be done to determine the level of community violence at night in the patient’s neighbourhood. She claimed that she liked the program when it was at 9 a.m. She did not mention finances as a reason for not going, and the program was within walking distance of her home. Noncompliance is a label given unfairly to patients. Most “noncompliance” has a cause. The cause should be identified and dealt with, so that the therapy will be successful. DIF: Apply REF: 58| 59 TOP: Implementation
OBJ: Describe elements of a community assessment. MSC: NCLEX: Health Promotion and Maintenance
MULTIPLE RESPONSE 1. Community-based nursing requires a strong knowledge base in which of the following?
(Select all that apply.) a. Family theory b. Communication c. Group dynamics
Canadian Fundamentals of Nursing 6th Edition Potter Test Bank d. Focus on the individual e. Cultural diversity ANS: A, B, C, E
With the individual and family as the patients, the context of community-based nursing is family-centred care within the community. This focus requires a strong knowledge base in family theory, principles of communication, group dynamics, and cultural diversity. The nurse leans to partner with patients and families, not just with individuals. DIF: Knowledge REF: 49-51 OBJ: Discuss the roles and functions of the community health nurse. TOP: Assessment MSC: NCLEX: Health Promotion and Maintenance 2. Community-based nursing centres function as the first level of contact between members of a
community and the health care delivery system. Which situation is ideal for health care services? (Select all that apply.) a. They are provided where patients live. b. They reduce the cost of health care for the patient. c. They provide direct access to nurses. d. They exclude interference from family or friends. ANS: A, B, C
Community-based nursing centres function as the first level of contact between members of a community and the health care delivery system. Ideally, health care services are provided near where patients live. Such centres help reduce the cost of health care for the patient and the stress associated with the financial burdens of care. In addition, these centres offer direct access to nurses and patient-centred health services and readily include the patient and the patient’s family or friends inN aU plR anSoIfN caGreT. B.COM DIF: Understand REF: 53 OBJ: Discuss the roles and functions of the community health nurse. TOP: Planning MSC: NCLEX: Health Promotion and Maintenance 3. Of the following list of patients, which would be considered at high risk to be members of a
vulnerable population? (Select all that apply.) a. An immigrant who speaks only Chinese. b. A truck driver from Central America who speaks limited English. c. A 22-year-old pregnant woman. d. A 15-year-old rape victim. e. A 40-year-old schizophrenic. ANS: A, B, D, E
Immigrant populations face multiple diverse health care needs that cities, counties, and states need to address. These health care needs pose significant legal and policy issues. For some immigrants, access to health care is limited because of language barriers and lack of benefits, resources, and transportation. Mothers and babies usually are not considered vulnerable populations unless other risk factors are noted. Physical, emotional, and sexual abuse (such as rape), as well as neglect, is a major public health problem affecting older persons, women, and children. When a patient has a severe mental illness such as schizophrenia, multiple health and socioeconomic problems must be recognized. DIF: Analyze
REF: 53-56
Canadian Fundamentals of Nursing 6th Edition Potter Test Bank OBJ: Explain the characteristics of patients from vulnerable populations that influence a nurse's approach to care. TOP: Planning MSC: NCLEX: Health Promotion and Maintenance
NURSINGTB.COM
Canadian Fundamentals of Nursing 6th Edition Potter Test Bank
Chapter 05: Theoretical Foundations of Nursing Practice Potter et al: Canadian Fundamentals of Nursing, 6th Edition MULTIPLE CHOICE 1. How is the idea of a theory best explained? a. “Mental maps” that make sense of information and decisional processes. b. Mental formulations of objects or events. c. Aspects of reality that can be consciously sensed. d. A purposeful set of assumptions that identify relationships between concepts. ANS: D
A theory is a purposeful set of assumptions that identify the relationships between concepts. Theories are useful because they provide a systematic view of explaining, predicting, and prescribing phenomena. The description “Mental maps that make sense of information and decisional processes” explains the idea of conceptual frameworks, which link ideas together. The description “Mental formulations of objects or events” explains the idea of concepts. The description “Aspects of reality that can be consciously sensed” explains the idea of phenomena. DIF: Understand REF: 63 OBJ: Describe selected theories of nursing practice and differentiate between them. TOP: Assessment MSC: NCLEX: Safe and Effective Care Environment 2. Different types of theories may be used by nurses seeking to study the basis of nursing
practice. The theory about why phenomena occur is which of the following? N R I G B.C M a. Prescriptive. U S N T O b. Descriptive. c. Grand. d. Middle-range. ANS: B
Descriptive theories are descriptions of phenomena, speculations on why phenomena occur, and explanations of the consequences of phenomena. Prescriptive theories address nursing interventions and help predict the consequences of a specific nursing intervention. Grand theories provide the structural framework for broad, abstract ideas about nursing. Middle-range theories address specific phenomena or concepts and reflect practice. DIF: Understand REF: 65 (Table 5-2) OBJ: Describe selected theories of nursing practice and differentiate between them. TOP: Assessment MSC: NCLEX: Safe and Effective Care Environment 3. A body of knowledge that encompasses definitions of person, environment, health, and
nursing is referred to as which of the following? a. Family nursing practice. b. Prescriptive nursing theory. c. Advanced nursing practice. d. Metaparadigm of nursing. ANS: D
Canadian Fundamentals of Nursing 6th Edition Potter Test Bank The major components of nursing theory, sometimes called the metaparadigm concepts, are person, environment, health, and nursing. Family nursing practice, although perhaps focusing on all of these concepts, is not a body of knowledge. Prescriptive nursing theory addresses nursing interventions and helps predict consequences of a specific intervention. Advanced nursing practice may focus on these concepts but is not a body of knowledge specific to these concepts. DIF: Understand REF: 65| 66 OBJ: Describe selected theories of nursing practice and differentiate between them. TOP: Assessment MSC: NCLEX: Safe and Effective Care Environment 4. Within a health care system that is based on Betty Neuman’s theory, what is the nurse’s goal
in caring for a patient who is having difficulty breathing and requires oxygen and medication? a. Strengthen the line of defences and focus on prevention. b. Promote attainment of biological self-care requisites. c. Assist in physiological adaptation to internal changes. d. Achieve the 14 basic needs. ANS: A
Neuman’s framework for practice includes nursing actions that focus on actual or potential stressors and thus on prevention. Dorothea Orem’s theory focuses on the attainment of self-care. Sister Callista Roy’s theory focuses on adaptation. Virginia Henderson’s theory focuses on helping the patient to achieve 14 basic needs. DIF: Apply REF: 69| 70 OBJ: Recognize selected conceptual frameworks associated with nursing practice. TOP: Implementation MSC: NCLEX: Health Promotion and Maintenance
NURSINGTB.COM
5. Although the different nursing theories have similarities, key elements distinguish one from
another. What is the emphasis of Jean Watson’s conceptual model? a. Self-care maintains wholeness. b. Subsystems exist in dynamic stability. c. Stimuli disrupt an adaptive system. d. Caring is central to the essence of nursing. ANS: D
Watson believed that nurses must do far more than deal with physical illness; they must attend to their primary function, which is caring. From Watson’s perspective, caring infuses all aspects of a nurse’s role and draws attention to nursing acts as embodying an esthetic that facilitates both healing and growth. Self-care is central to Dorothea Orem’s theory. The key emphasis of Dorothy Johnson’s theory is that subsystems exist in dynamic stability. The key emphasis of Sister Callista Roy’s theory is that stimuli disrupt an adaptive system. DIF: Understand REF: 71 OBJ: Recognize selected conceptual frameworks associated with nursing practice. TOP: Planning MSC: NCLEX: Health Promotion and Maintenance 6. A community health nurse is working with a variety of patients and decides to use a systems
theory approach to help them to meet their health care needs. When using systems theory, the nurse focuses on which of the following? a. The patient’s interaction with his or her environment.
Canadian Fundamentals of Nursing 6th Edition Potter Test Bank b. The hierarchy of the patient’s human needs. c. The patient’s attitudes toward health behaviours. d. The response of the patient to the process of growth and development. ANS: A
According to systems theory, a system is made up of parts that depend on one another, are interrelated, share a common purpose, and together form a whole. A patient’s interaction with the environment is an example of an open system. The nurse understands that factors that change the environment also can have an effect on the system. Abraham Maslow’s hierarchy of human needs is an interdisciplinary theory useful in planning individualized care. Determining a patient’s attitudes toward health behaviours follows a health and wellness theoretical model. Focusing on the response of a patient to the process of growth and development is consistent with developmental theories. DIF: Understand REF: 70 OBJ: Describe selected theories of nursing practice and differentiate between them. TOP: Planning MSC: NCLEX: Health Promotion and Maintenance 7. While working on a postoperative unit, the nurse is applying elements of self-care theory and
is assisting a patient to attain and manage self-care in wound management. Who was the nursing pioneer who developed this theory? a. Florence Nightingale. b. Virginia Henderson. c. Dorothea Orem. d. Hildegard Peplau. ANS: C
The goal of Orem’s theory isNtoUhRelp ie. ntCper form self-care. The goal of Nightingale’s SIthe NGpat TB OM theory is to shift the focus from the disease process toward an environment conducive to healing. Henderson defined nursing practice as assisting the individual, sick or well, in the performance of activities that will contribute to health, recovery, or a peaceful death. Peplau’s theory defined the core of nursing as the interpersonal relationship between the nurse and the patient. DIF: Understand REF: 68-70 OBJ: Describe selected theories of nursing practice and differentiate between them. TOP: Implementation MSC: NCLEX: Health Promotion and Maintenance 8. What is an element that is key to the framework for practice in Martha Rogers’s theory? a. Human becoming. b. Manipulation of the patient’s environment. c. Seven categories of behaviour and behavioural balance. d. Focus upon the life process of a human being along a time–space continuum. ANS: D
Canadian Fundamentals of Nursing 6th Edition Potter Test Bank The framework for practice, according to Rogers’s theory, consisted of presenting the patient of nursing not simply as a person but as an energy field in constant interaction with the environment, which itself was also an irreducible energy field, coextensive with the universe. Nursing’s role was to focus upon the life process of a human being along a time–space continuum. Rosemarie Parse, in her theory of human becoming (1997), viewed the individual as a unitary being who is “indivisible, unpredictable, and ever-changing” and “a freely choosing being who can be recognized through paradoxical patterns co-created all-at-once in mutual process with the universe.” Nightingale’s theory includes manipulation of the patient’s environment (i.e., appropriate noise, nutrition, hygiene, light, comfort, socialization, and hope) in the framework for practice. Dorothy Johnson’s theory includes seven categories of behaviour and behavioural balance in the framework for practice. DIF: Understand REF: 71 OBJ: Describe selected theories of nursing practice and differentiate between them. TOP: Planning MSC: NCLEX: Health Promotion and Maintenance 9. Whose theory is most pertinent in assisting the nurse who is helping a patient focus on stress
reduction? a. Hildegard Peplau’s. b. Virginia Henderson’s. c. Betty Neuman’s. d. Rosemarie Parse’s. ANS: C
Neuman’s theory focuses on stress reduction as the goal of the systems model of nursing practice. Peplau’s theory focuses on interpersonal relationships between nurses and patients and the interactive, therapeutic nature of the nursing process. Henderson’s theory focuses on N R theo INryG T B.C onM the notion of nurses’ engagement with, the 14 basic human needs. ParsUe’s S focusesO and care of, people in a continuous process of making choices and changing health priorities. DIF: Apply REF: 69-71 OBJ: Describe selected theories of nursing practice and differentiate between them. TOP: Implementation MSC: NCLEX: Health Promotion and Maintenance 10. What does the theorizing term proposition mean? a. A purposeful set of assumptions. b. A declarative assertion. c. The process of formulating concepts. d. The structure that links concepts together. ANS: B
A proposition is a declarative assertion. A purposeful set of assumptions is a theory. The process of formulating concepts is conceptualization. The structure that links concepts together is a conceptual framework. DIF: Understand REF: 64 (Table 5-1) OBJ: Describe relationships between theorizing and other forms of nursing knowledge. TOP: Planning MSC: NCLEX: Safe and Effective Care Environment 11. What is a characteristic of complexity science? a. Patterns of knowledge application.
Canadian Fundamentals of Nursing 6th Edition Potter Test Bank b. A rigid approach for describing experiences. c. Reducing phenomena to smallest properties. d. Orientation toward studying the nature of people’s needs. ANS: C
Complexity science consists of dynamic and interactive phenomena reduced to the smallest properties that can be observed within their natural context so that their interactions can be interpreted with as little interference as possible from prior assumptions. Patterns of knowledge application are not characteristic of complexity science. Complexity science is not a rigid approach for describing experiences, and it is not characterized by an orientation toward studying the nature of people’s needs. DIF: Understand REF: 67 OBJ: Interpret current debates surrounding various theories of nursing practice. TOP: Planning MSC: NCLEX: Safe and Effective Care Environment 12. What is a current concern with the use of the nursing diagnoses approved by the North
American Nursing Diagnoses Association (NANDA)? a. NANDA’s list stifles critical thinking. b. NANDA’s list should replace conceptual nursing models. c. NANDA’s list is an inherent barrier to individualized care. d. NANDA’s list does not reflect practical diagnoses. ANS: C
NANDA’s list of nursing diagnoses is recognized by many nurses as a system that relies entirely upon an agreement about what constitutes average wellness and illness experiences. As such, it can create worrisome barriers to the individualized care of patients. NANDA’s list of nursing diagnoses may stifNleUcR ritic hink SIalNtG TBin.gC, bu OMt this has not been recognized as a concern; it is often considered to be less thorough than conceptual models of nursing, as it involves only one aspect of caring for patients; and it does reflect practical diagnoses. DIF: Understand REF: 67 OBJ: Describe challenges inherent in theorizing about nursing practice. TOP: Planning MSC: NCLEX: Safe and Effective Care Environment 13. In the late 1980s, the shift in scholarly work regarding nursing theories challenged nurses
when theorizing to include consider what type of knowledge? a. Clinical. b. Intuitive. c. Theoretical. d. Substantive. ANS: D
Afaf Meleis (1987) challenged nurses to direct their theorizing away from the processes by which nurses use knowledge and toward the equally challenging issues associated with the substance of that knowledge—that is, to include substantive knowledge. Neither clinical knowledge nor intuitive knowledge was related to this challenge. Theoretical knowledge had been the focus of theorizing for many years before the late 1980s. DIF: Understand REF: 71 OBJ: Appreciate the role of “theorizing” about the essence of nursing. TOP: Assessment MSC: NCLEX: Safe and Effective Care Environment
Canadian Fundamentals of Nursing 6th Edition Potter Test Bank
MULTIPLE RESPONSE 1. Interactionist theories focused on the relationships between nurses and their patients. Which
of the following were interactionist theorists? (Select all that apply.) a. Hildegard Peplau b. Dorothea Orem c. Joyce Travelbee d. Evelyn Adam e. Betty Neuman ANS: A, C, D
Hildegard Peplau, Joyce Travelbee, and Evelyn Adam were all interactionist theorists. Dorthea Orem was a needs theorist, and Betty Neuman was a systems theorist. DIF: Apply REF: 69-70 OBJ: Describe selected theories of nursing practice and differentiate between them. TOP: Assessment MSC: NCLEX: Health Promotion and Maintenance
NURSINGTB.COM
Canadian Fundamentals of Nursing 6th Edition Potter Test Bank
Chapter 06: Evidence-Informed Practice Potter et al: Canadian Fundamentals of Nursing, 6th Edition MULTIPLE CHOICE 1. The extent to which the findings of a qualitative study are thought to be meaningful and
applicable to similar cases or other situations is known as which of the following? a. Credibility. b. Transferability. c. Validity. d. Reliability. ANS: B
Transferability refers to the extent to which the findings of a qualitative study are thought to be meaningful and applicable to similar cases or other situations. Credibility concerns whether the research and results are trustworthy. Validity refers to whether the study measured what it intended to. Reliability concerns whether the study results are repeatable. DIF: Understand TOP: Evaluate
REF: 83 OBJ: Define the key terms listed. MSC: NCLEX: Safe and Effective Care Environment
2. In caring for patients, it is important for the nurse to realize that evidence-informed decision
making is which of the following? a. The only valid source of knowledge that should be used. b. Secondary to traditional or standard care knowledge. c. Dependent on patient values and expectations. N RSINGTB.COM d. Not related to quality improUvem ent studies ANS: C
Even when the best evidence available is used, application and outcomes will differ according to patients’ values, preferences, concerns, and expectations. Nurses often care for patients on the basis of tradition or convenience, as in the standard “It has always been done this way.” Although these sources have value, it is important to learn to rely more on research evidence than on nonresearch evidence. Evidence-informed decision making is closely related to quality improvement. DIF: Understand REF: 76| 77 (Box 6-1) OBJ: Explain the need for evidence to inform nurses’ decision making. TOP: Evaluate MSC: NCLEX: Safe and Effective Care Environment 3. The first step in evidence-informed practice is to ask a clinical question. In doing so, what
does the nurse need to realize with regard to researching interventions? a. The question is more important than its format. b. The question will lead the researcher to hundreds of articles that must be read. c. The question may be easier if it is in PICOT format. d. The question may be more useful the more general it is. ANS: C
Canadian Fundamentals of Nursing 6th Edition Potter Test Bank The PICOT format enables the nurse to ask questions that are intervention focused. Inappropriately formulated questions will probably lead to irrelevant sources of information. It is not beneficial to read hundreds of articles. It is more beneficial to read the best four to six articles that specifically address the question. The more focused the question asked is, the easier it will become to search for evidence in the scientific literature. DIF: Understand TOP: Evaluate
REF: 77 OBJ: Discuss the steps of the research process. MSC: NCLEX: Safe and Effective Care Environment
4. When the best evidence is collected, what is the “gold standard” for research? a. The randomized controlled trial (RCT). b. The peer-reviewed article. c. Qualitative research. d. The opinion of expert committees. ANS: A
Individual RCTs are the “gold standard” for research. A peer-reviewed article is one that has been reviewed by a panel of experts; this is not a research method. Qualitative research is valuable in identifying information about how patients cope with or manage various health problems and their perceptions of illness. It does not usually have the robustness of an RCT. Expert opinion is on the bottom of the hierarchical pyramid of evidence. DIF: Knowledge REF: 83 OBJ: Discuss methods for developing new nursing knowledge. TOP: Assessment MSC: NCLEX: Safe and Effective Care Environment 5. The nurse is writing a research article on a patient care topic. Which section will get the reader
to read the article because ofN thU eR vaSluIeNoG f tT hB e t. opCicOfM or the reader? a. Abstract. b. Introduction. c. Literature review or background. d. Results. ANS: B
The introduction contains information about its purpose and the importance of the topic to the readers of the article. The abstract is a brief summary of the article. The literature review or background offers a detailed background of the level of science or clinical information that is available about the topic of the article. The results section is the summary section of the article. DIF: Understand TOP: Assessment
REF: 78 OBJ: Discuss the steps of the research process. MSC: NCLEX: Safe and Effective Care Environment
6. The nurse is caring for a patient with chronic low back pain. In providing care for this patient,
the nurse wonders whether there is literature on nursing interventions for chronic low back pain. What are the best-known databases for nursing literature? a. MEDLINE and CINAHL. b. EMBASE and PsycInfo. c. PsycINFO and CINAHL. d. MEDLINE and EMBASE. ANS: A
Canadian Fundamentals of Nursing 6th Edition Potter Test Bank The best-known databases for nursing literature are MEDLINE and CINAHL. EMBASE includes biomedical and pharmaceutical studies. PsycINFO deals with psychology and related health care disciplines. DIF: Understand REF: 78 OBJ: Discuss methods for developing new nursing knowledge. TOP: Implementation MSC: NCLEX: Health Promotion and Maintenance 7. The nurse is developing a PICOT question related to sucrose administration before
phlebotomy in infants: “Is the pain score lower in infants who are administered sucrose one minute before phlebotomy than in infants who do not receive sucrose at all?” With a PICOT question, P is the population of interest, I is the intervention of interest, C is the comparison of interest, O is the outcome, and T is the time; is this a true PICOT question? a. Yes, because the outcome always comes before the intervention. b. Yes, regardless of placement of elements. c. No, because the comparison comes after the intervention. d. No, because the outcome comes after the population. ANS: B
A well-designed PICOT question does not have to follow the sequence of P, I, C, O, and T. The aim is to ask a question that contains as many of the PICOT elements as possible. DIF: Understand REF: 77 OBJ: Discuss methods for developing new nursing knowledge. TOP: Implementation MSC: NCLEX: Safe and Effective Care Environment 8. In a review of literature for an evidence-informed practice study, what is the most reliable
NURSINGTB.COM level of evidence? a. Systematic review and meta-analysis. b. Randomized control trial (RCT). c. Case-control study. d. Control trial without randomization. ANS: A
In a systematic review or meta-analysis, an independent researcher reviews all the RCTs conducted on the same clinical question and reports whether the evidence is conclusive or whether further study is needed. A single RCT is not as conclusive as a review of several RCTs on the same question. Control trials without randomization may involve bias in how the study is conducted. Case-control studies also have room for bias. DIF: Analyze REF: 83 OBJ: Discuss methods for developing new nursing knowledge. TOP: Assessment MSC: NCLEX: Safe and Effective Care Environment 9. Why is qualitative nursing research valuable? a. It excludes all bias. b. It entails the use of randomization in structure. c. It helps determine associations between variables and conditions. d. It entails the study of phenomena that are difficult to quantify. ANS: D
Canadian Fundamentals of Nursing 6th Edition Potter Test Bank Qualitative nursing research is the study of phenomena that are difficult to quantify or categorize, such as patients’ perceptions of illness. No study can totally exclude bias. However, randomization, such as that used in randomized control studies, helps, but in qualitative nursing research, randomization is not usually used. Control studies determine whether there is an association between one or more predictor variables and the condition. DIF: Knowledge TOP: Assessment
REF: 83 OBJ: Define the key terms listed. MSC: NCLEX: Safe and Effective Care Environment
10. The nurse has used her PICOT question to develop an evidence-informed change in protocol
for a certain nursing procedure. However, to make these changes throughout the entire institution would require more support staff than is available at this time. What is the nurse’s best option? a. Drop the idea of making the change at this time. b. Insist that management hire the needed staff to facilitate the change. c. Seek employment in another institution that may have the staff needed. d. Conduct a pilot study to develop evidence to support the change. ANS: D
When evidence is not strong enough to apply in practice, or if resources are limited, the next option is to conduct a pilot study to investigate the PICOT question. Dropping the idea would be counterproductive; insisting that management hire staff could be seen as a mandate and could produce negative results. Seeking employment at another institution probably would not be the answer because most institutions operate under similar established guidelines. DIF: Apply REF: 79 OBJ: Discuss methods for developing new nursing knowledge. TOP: Implementation MSC: NCLEX: Safe and EffecN tivU eR CS arI eE NnGviTroBnm.eCntOM 11. According to hospital policy, when starting an intravenous (IV) catheter, the nurse must first
prepare the potential IV site with alcohol and dress it with a gauze dressing. The nurse has done a literature review and believes that evidence-informed practice dictates the use of a transparent dressing to prevent catheter dislodgement. What should the nurse do? a. Begin to use transparent dressing instead of gauze dressings. b. Bring findings to the policy and procedure committee. c. Use transparent dressings on half of her IV starts and gauze on the other. d. Continue following hospital policy without saying anything. ANS: B
As a result of her finding, the nurse should meet with the policy and procedure committee to recommend routine use of transparent dressings. However, until the policy is changed, or until the nurse receives approval to conduct a pilot study, the nurse is obligated to follow hospital procedure. If the nurse has information that can lead to better patient care, he or she has an obligation (moral and professional) to bring it to the attention of policymakers. DIF: Apply REF: 79 OBJ: Discuss the steps of evidence-informed decision making. MSC: NCLEX: Safe and Effective Care Environment
TOP: Implementation
Canadian Fundamentals of Nursing 6th Edition Potter Test Bank 12. The nurse is trying to identify common general themes relative to the effectiveness of cardiac
rehabilitation for patients who have had heart attacks and have gone through cardiac rehabilitation programs. The nurse conducts interviews and focus groups. What type of research is the nurse conducting? a. Evaluation research. b. Experimental research. c. Qualitative research. d. Nonexperimental research. ANS: C
Qualitative research involves using inductive reasoning to develop generalizations or theories from specific observations or interviews. Evaluation and experimental research are forms of quantitative research. Nonexperimental descriptive studies describe, explain, or predict phenomena, such as factors that lead to an adolescent’s decision to smoke cigarettes. DIF: Understand REF: 83 OBJ: Discuss methods for developing new nursing knowledge. TOP: Assessment MSC: NCLEX: Health Promotion and Maintenance 13. In conducting a research study, the researcher must inform the participants, in lay language,
about the risks and benefits of participating. This is an important aspect of what? a. Anonymity. b. Confidentiality. c. Informed consent. d. The research process. ANS: C
Informed consent means thatNres earc suGbje ct. s (C 1)OaM re given full and complete information UR SIh N TB about the purpose of the study, procedures, data collection, potential harm and benefits, and alternative methods of treatment; (2) are capable of fully understanding the research; (3) have the power to voluntarily consent or decline participation; and (4) understand how confidentiality or anonymity is maintained. Confidentiality guarantees that any information the subject provides will not be reported in any manner that identifies the subject and will not be accessible to people outside the research team. Anonymity is the condition when even the researcher cannot link the subject to the data. The research process is a broader concept that provides an orderly series of steps that allow the researcher to move from asking a question to finding the answer. DIF: Knowledge TOP: Assessment
REF: 84| 85 OBJ: Discuss the steps of the research process. MSC: NCLEX: Safe and Effective Care Environment
14. When evaluating quality improvement (QI) programs in relation to evidence-informed
decision making, what should the nurse note? a. Both are designed to improve performance. b. When implementing evidence-informed practice (EIP) projects, it is important to review QI data. c. EIP is not at all related to QI. d. Evaluation of processes is the realm of performance improvement, not QI. ANS: B
Canadian Fundamentals of Nursing 6th Edition Potter Test Bank EIP and QI go hand in hand. When an EIP project is implemented, it is important to review available QI data. Reliable QI data improve the relevance and scope of an EIP project. Performance improvement is concerned with performance. QI is concerned with processes. DIF: Understand REF: 76 OBJ: Discuss methods for developing new nursing knowledge. TOP: Planning MSC: NCLEX: Safe and Effective Care Environment 15. The quality improvement committee has been alerted to an increased number of falls in the
hospital. Most of these falls have occurred at night and have involved patients who were trying to crawl over bed rails. A literature review revealed that most falls occur because patients are trying to go to the bathroom. The committee created a practice change that bed rails should be left in the down position, and hourly nursing rounds should be conducted. What is the committee’s next step? a. Evaluate the changes in 1 month. b. Wait a month before implementing the changes. c. Implement the changes as a pilot study. d. Communicate to staff the results of this inquiry. ANS: D
Quality improvement combined with evidence-informed practice is the foundation for excellent patient care and outcomes. Once a QI committee makes a practice change, it is important to communicate the results to staff. Practice changes will probably not last when QI committees fail to report findings and results of interventions. Once communicated, changes should be put in place as the committee deems reasonable (i.e., either fully or as a pilot study) and as soon as practical; this should be followed by re-evaluation. DIF: Apply REF: 78N | 7U9RSINGTB.COM OBJ: Discuss methods for developing new nursing knowledge. TOP: Implementation MSC: NCLEX: Safe and Effective Care Environment 16. As nurses move forward in their education, different roles may be assumed with regard to
research. What is the expected research role for the baccalaureate-prepared nurse? a. To assume the role of a clinical expert. b. To incorporate evidence-informed practice activities into nursing practice. c. To develop methods of inquiry relevant to nursing. d. To acquire funding for research projects. ANS: B
Nurses can make links between research findings and nursing care by identifying appropriate clinical problems, reading peer-reviewed literature from numerous sources, and incorporating evidence-informed practice activities into the nursing practice of their unit or agency. Nurses, with experience, will develop clinical expertise, but this is not their expected research role. Nurses with advanced degrees or those who have a research focus are responsible for developing methods of inquiry relevant to nursing and for acquiring funding for research projects. DIF: Understand REF: 85 OBJ: Explain the need for evidence to inform nurses’ decision making. TOP: Implementation MSC: NCLEX: Health Promotion and Maintenance
Canadian Fundamentals of Nursing 6th Edition Potter Test Bank 17. The nurse is conducting a research project on optimal time frames for postoperative
ambulation of patients. After the nurse identifies the problem, what is the next step in the research process? a. Selecting the population. b. Reviewing the literature. c. Identifying the instrument to use for data analysis. d. Obtaining approval to conduct the study. ANS: B
After the problem is identified, the next step in the research process is reviewing the literature in order to determine what is known about the problem. After identification of the problem and review of the literature, the researcher will design the study protocol. Selecting the population is a component of this phase of the research process. The instrument to use for data analysis is identified during the process of designing the study protocol. This step occurs during the study design phase of the research process after problem identification and literature review have taken place. Obtaining necessary approvals is part of conducting the study, a part that follows the design phase in the research process. DIF: Analyze REF: 80| 81 TOP: Implementation
OBJ: Define nursing research. MSC: NCLEX: Safe and Effective Care Environment
MULTIPLE RESPONSE 1. The nurse is preparing to conduct research that will allow precise measurement of a
phenomenon. Which of the following methods will provide the nurse with the right kind of data? (Select all that apply.) a. Experimental research. N R I G B.C M U S N T O b. Surveys. c. Evaluation research. d. Phenomenology. e. Grounded theory. ANS: A, B, C
Experimental research, surveys, and evaluation research are all forms of quantitative research. Phenomenology and grounded theory are forms of qualitative research. DIF: Understand TOP: Assessment
REF: 81-83 OBJ: Define the key terms listed. MSC: NCLEX: Safe and Effective Care Environment
2. Before conducting any study with human subjects, the researcher must obtain approval from
the agency’s human subjects committee or research ethics board (REB). The REB ensures that the researcher will do what? (Select all that apply.) a. Obtain informed consent. b. Minimize risk to subjects. c. Ensure confidentiality. d. Identify risks and benefits of participation. e. Ensure that subjects complete the study. ANS: A, B, C, D
Canadian Fundamentals of Nursing 6th Edition Potter Test Bank Researchers must protect the confidentiality of people who participate in the study, obtain informed consent, minimize risk to subjects, identify risks and benefits of participation, ensure that participation in the study is voluntary, and allow subjects to withdraw from studies at any time. DIF: Understand REF: 83-85 TOP: Implementation
OBJ: Discuss the steps of the research process. MSC: NCLEX: Safe and Effective Care Environment
3. The nurse is reviewing literature related to a potential problem that has been identified on the
nursing unit. The nurse realizes that nursing research is important in that it is designed to do which of the following? (Select all that apply.) a. Enhance the nurse’s chance at promotion. b. Identify new knowledge. c. Improve professional practice. d. Enhance effective use of resources. e. Lead to decreases in budget expenditures. ANS: B, C, D
Nursing research is a way to identify new knowledge, improve professional education and practice, and use resources effectively. The research does not always result in lower budget expenditures; rather, it enhances more effective use of resources. A promotion is not a direct result of research. DIF: Understand TOP: Assessment
REF: 75-77 OBJ: Discuss the steps of the research process. MSC: NCLEX: Safe and Effective Care Environment
NURSINGTB.COM
Canadian Fundamentals of Nursing 6th Edition Potter Test Bank
Chapter 07: Nursing Values and Ethics Potter et al: Canadian Fundamentals of Nursing, 6th Edition MULTIPLE CHOICE 1. Four patients in labour all request epidural analgesia to manage their pain at the same time.
Which ethical principle is compromised when only one nurse anaesthetist is on call? a. Justice. b. Nonmaleficence. c. Beneficence. d. Fidelity. ANS: A
Justice refers to fairness and is used frequently in discussion regarding access to health care resources. In this situation, the distribution of resources—in this case, pain management— cannot be justly apportioned. Nonmaleficence means “do no harm,” beneficence means “to do good,” and fidelity means “to be true to” or “to be honest.” Each of these principles is partially alluded to in the question; however, justice is the principle most compromised because not all labouring patients have equal access to pain management owingto lack of personnel resources. DIF: Understand REF: 91| 92 OBJ: Explain the relationship between ethics and professional nursing practice. TOP: Implementation MSC: NCLEX: Safe and Effective Care Environment 2. The patient tells the nurse that she is afraid to speak up regarding her desire to end care for
N R I G B.C M
fear of upsetting her husband aU nd cShildNrenT . WhichOprinciple in the nursing code of ethics ensures that the nurse will promote the patient’s cause? a. Responsibility. b. Advocacy. c. Confidentiality. d. Accountability. ANS: B
Nurses advocate for patients by supporting the patient’s cause. A nurse’s ability to advocate adequately for a patient is based on the unique relationship that develops between nurse and patient and on the opportunity to better understand the patient’s point of view. Responsibility refers to respecting one’s professional obligations and following through on promises. Confidentiality refers to privacy issues, and accountability refers to owning one’s actions. DIF: Understand REF: 89| 90 OBJ: Explain the relationship between ethics and professional nursing practice. TOP: Diagnosis MSC: NCLEX: Safe and Effective Care Environment 3. The patient’s son requests to view the documentation in his mother’s medical record. What is
the nurse’s best response to this request? a. “I’ll be happy to get that for you.” b. “You will have to talk to the physician about that.” c. “You will need your mother’s permission.” d. “You are not allowed to see it.”
Canadian Fundamentals of Nursing 6th Edition Potter Test Bank
ANS: C
Nurses should protect a patient's right to privacy and confidentiality by helping the patient access his or her health records (subject to legal requirements), intervening if other members of the health care team fail to respect the patient's privacy, and following policies that protect the patient's privacy. Private health information cannot be shared without the patient’s specific permission. The other three responses either are outright false or reflect poor communication techniques. DIF: Apply REF: 90 OBJ: Explain the relationship between ethics and professional nursing practice. TOP: Evaluate MSC: NCLEX: Safe and Effective Care Environment 4. When individuals work together to solve ethical dilemmas, they must examine their own
values. This step is crucial to ensure what? a. The group identifies the one correct solution. b. Fact is separated from opinion. c. Judgemental attitudes are not provoked. d. Different perspectives are respected. ANS: D
Values are personal beliefs that influence opinions; to be able to negotiate differences in opinions, the nurse must first be clear about personal values, which will influence behaviours, decisions, and actions. Ethical dilemmas are a problem in that no single correct solution exists. DIF: Remember TOP: Evaluate
REF: 88 OBJ: Discuss how values influence patient care. MSC: NN CLER X: SIafeG andB E. ffeCctivM e Care Environment
U S N T
O
5. Ethical dilemmas are common occurrences in the care of patients. The nurse understands that
dilemmas are a result of which of the following? a. Presence of conflicting values. b. Hierarchical systems. c. Judgemental perceptions of patients. d. Poor communication with the patient. ANS: A
The primary underlying reason that ethical dilemmas occur is that there are no clear-cut, universally accepted solutions to a problem when participating individuals do not share the same values. Poor communication and the hierarchical systems that exist in health care, such as reporting structures within the hospital or the historically unequal relationship between physicians and nurses, may complicate dilemmas. Without clarification of values, the nurse may not be able to distinguish fact from opinion or value, and this can lead to judgemental attitudes. DIF: Understand TOP: Assessment
REF: 88 OBJ: Discuss the role of values in the study of ethics. MSC: NCLEX: Safe and Effective Care Environment
6. The nurse questions a physician’s order to administer a placebo to the patient. The nurse’s
action is based on which ethical principle? a. Autonomy. b. Beneficence.
Canadian Fundamentals of Nursing 6th Edition Potter Test Bank c. Justice. d. Fidelity. ANS: A
Autonomy is the freedom to make decisions without external control. In this case, the nurse questions the physician’s order for a placebo because such a decision was made without consultation with the patient. Beneficence refers to taking a positive action for others, and although it has implications in this situation, it is not the primary operating principle. Justice refers to fairness and is most often used in discussions about access to health care resources. Fidelity refers to the agreement to keep promises. DIF: Understand REF: 91| 92 TOP: Implementation
OBJ: Examine and clarify personal values. MSC: NCLEX: Safe and Effective Care Environment
7. The nurse finds it difficult to care for a patient whose advance directive states that no
extraordinary resuscitation measures should be taken. Which step may help the nurse to find resolution in this assignment? a. Calling for an ethical committee consult. b. Declining the assignment on religious grounds. c. Clarifying the nurse’s own personal values. d. Persuading the family to challenge the directive. ANS: C
Values develop over time and are influenced by family, schools, religious traditions, and life experiences. The nurse must recognize that no two individuals have the same set of experiences, and so differences in values are more likely the norm than the exception. Closer inspection of one’s own values may be a step in gaining an understanding of another person’s perspective. Calling for a conNsuUlR t, S deI clN inG inT gB th. eC asOsiM gnment, and persuading the family to challenge the patient’s directive are not ideal resolutions because they do not address the reason for the nurse’s discomfort, which is the conflict between the nurse’s values and those of the patient. DIF: Apply TOP: Assessment
REF: 88 OBJ: Examine and clarify personal values. MSC: NCLEX: Safe and Effective Care Environment
8. The nurse values autonomy above all other principles. With which patient will the nurse find
most difficult to deal with the situation? a. Teenager in labour who requests epidural anaesthesia. b. Middle-aged father of three with an advance directive declining life support. c. Elderly patient who requires dialysis. d. Family elder who is making the decisions for a 30-year-old female member. ANS: D
Autonomy is the freedom to make decisions without external control. A nurse who values autonomy highly may find it difficult to accept situations in which the patient is not the primary decision maker regarding his or her care. A teenager requesting an epidural anaesthetic, a father with an advanced directive, and an elderly patient requiring dialysis all are patients who are making their own decisions and choices regarding care. DIF: Analyze TOP: Evaluate
REF: 91 OBJ: Examine and clarify personal values. MSC: NCLEX: Safe and Effective Care Environment
Canadian Fundamentals of Nursing 6th Edition Potter Test Bank 9. Which philosophy of health care ethics would be particularly useful for making ethical
decisions about vulnerable populations? a. Feminist ethics. b. Deontology. c. Bioethics. d. Utilitarianism. ANS: A
Feminist ethics focuses particularly on the nature of relationships, especially those in which there is a power imbalance or in which a point of view is not routinely accepted. Examples of populations that are considered vulnerable include children, pregnant women, incarcerated persons, and minority groups. Deontology refers to making decisions or “right-making characteristics”; bioethics refers to consensus building; and utilitarianism refers to the greatest good for the greatest number. DIF: Remember REF: 92 OBJ: Describe some basic ethical philosophies relevant to health care. TOP: Assessment MSC: NCLEX: Safe and Effective Care Environment 10. A nurse argues that the health care system needs reform because a large number of patients
are uninsured and end up needing expensive emergency care when low-cost measures covered by insurance could have prevented their illnesses. What ethical framework is she using to make this case? a. Deontology. b. Ethics of care. c. Feminist ethics. d. Utilitarianism. ANS: D
NURSINGTB.COM
Utilitarianism is a system of ethics wherein value is determined by usefulness. This system of ethics focuses on the outcome of the greatest good for the greatest number of people. Deontology would not account for consequences of actions. The ethics of care would not be helpful because consensus on this issue is not achievable. Feminist ethics, which focus on relationships, is not addressed in this case. DIF: Evaluate REF: 91 OBJ: Describe some basic ethical philosophies relevant to health care. TOP: Assessment MSC: NCLEX: Safe and Effective Care Environment 11. The nurse has become aware that narcotics are missing in the patient care area. Which ethical
principle obligates the nurse to report the missing medications? a. Advocacy. b. Responsibility. c. Confidentiality. d. Accountability. ANS: B
Canadian Fundamentals of Nursing 6th Edition Potter Test Bank Responsibility refers to one’s willingness to respect and adhere to one’s professional obligations. One of the obligations of nurses is to protect patients and communities, including other nurses. If narcotics are missing, this may indicate that patients have not received medications ordered for their care, or it may suggest that a health care provider may be working under the influence of these drugs. Accountability refers to the ability to answer for one’s actions. Advocacy refers to the support of a particular cause. Confidentiality involves protecting patients’ personal health information and privacy. DIF: Understand REF: 89| 90 OBJ: Explain the relationship between ethics and professional nursing practice. TOP: Evaluate MSC: NCLEX: Safe and Effective Care Environment 12. A young pregnant woman whose fetus has been exposed to multiple teratogens consents to
undergo serial percutaneous umbilical blood sampling (PUBS) to examine how exposure affects the fetus over time. Although these tests will not improve the fetus’s outcomes and will expose it to some risks, the information gathered may help infants in the future. Which ethical principle is at greatest risk? a. Autonomy. b. Fidelity. c. Nonmaleficence. d. Beneficence. ANS: C
Nonmaleficence is the ethical principle that focuses on avoidance of harm or hurt. The nurse must balance risks and benefits of care. Repeated PUBS may place the mother and fetus at risk for infection and increased pain, and the mother may be at risk for increased emotional stress. Fidelity refers to the agreement to keep promises, autonomy refers to freedom to make N rol, RSand INbe GT B.C M decisions without external contU neficencOe refers to taking positive actions to help others. DIF: Apply REF: 91| 92 OBJ: Describe some basic ethical philosophies relevant to health care. TOP: Evaluate MSC: NCLEX: Safe and Effective Care Environment 13. A high school teacher with advanced multiple sclerosis teaches from her wheelchair but
insists on being treated the same as other colleagues. Which of the following is the teacher demonstrating? a. Preserving dignity. b. Choosing from alternatives. c. Considering all consequences. d. Acting with a pattern of consistency. ANS: A
The teacher’s choice is preserving dignity, which is a value included in the Canadian Nurses Association (CNA) Code of Ethics. She cherishes her choice of being treated the same as her colleagues despite her medical condition, and she publicly affirms the choice by teaching from her wheelchair. At this point, having already made a choice, the teacher is not choosing from alternatives—she could have chosen to quit teaching, but she did not—and is not demonstrating that she is still considering all consequences. The situation does not reflect that her actions have a pattern of consistency; she is not repeating a particular behaviour.
Canadian Fundamentals of Nursing 6th Edition Potter Test Bank DIF: Analyze REF: 89| 90 OBJ: Describe some basic ethical philosophies relevant to health care. TOP: Assessment MSC: NCLEX: Safe and Effective Care Environment 14. Which of the following is an example of ethical responsibility? a. Delivery of competent care. b. Formation of interpersonal relationships. c. Application of the nursing process. d. Evaluation of new computerized technologies. ANS: A
Providing competent care is one of the values in the CNA Code of Ethics that nurses must uphold. Formation of interpersonal relationships, application of the nursing process, and evaluation of new computerized technologies are not ethical responsibilities. DIF: Understand REF: 89 OBJ: Explain the relationship between ethics and professional nursing practice. TOP: Assessment MSC: NCLEX: Safe and Effective Care Environment 15. The nurse is caring for a severely ill patient with acquired immune deficiency syndrome
(AIDS) who now requires ventilator support. Which intervention is considered futile? a. Administering the influenza vaccine. b. Providing oral care every 5 hours. c. Applying fentanyl patches as needed for pain. d. Supporting the patient’s lower extremities with pillows. ANS: A
Futile refers to something that servesMno useful purpose; in nursing, it refers to NUisRhopeless INGTorB.C S interventions that are unlikely to produce benefit O for the patient. A vaccine is administered to prevent or lessen the likelihood of contracting an infectious disease at some time in the future; at this point, the patient is dying. Care delivered to a patient at the end of life is focused on pain management and comfort measures, such as providing oral care, applying fentanyl patches, and supporting the patient’s lower extremities. DIF: Understand REF: 95 OBJ: Identify contemporary ethical issues in nursing practice. MSC: NCLEX: Health Promotion and Maintenance
TOP: Implementation
16. During a severe respiratory epidemic, the local health care organizations decide to give health
care providers priority access to ventilators over other members of the community who also need that resource. Which philosophy would give the strongest support for this decision? a. Feminist ethics. b. Utilitarianism. c. Deontology. d. Ethics of care. ANS: B
Focusing on the greatest good for the most people, the organizations decide to ensure that as many health care workers as possible will survive to care for other members of the community. DIF: Understand
REF: 91
Canadian Fundamentals of Nursing 6th Edition Potter Test Bank OBJ: Identify contemporary ethical issues in nursing practice. MSC: NCLEX: Safe and Effective Care Environment
TOP: Implementation
17. How are determinations regarding quality of life characterized? a. They are based on a person’s ability to act according to ethical principles. b. They are based on a patient’s self-determination. c. They are value judgements that can vary from person to person. d. They are consistent and stable over the course of one’s lifetime. ANS: C
Determinations regarding quality of life are value judgements, which are based on what individuals believe is desirable. Beliefs about what people find desirable vary from person to person. Determinations regarding quality of life are not based on a person’s ability to act according to ethical principles, or based on a patient’s self-determination, and they may change over the course of one’s lifetime. DIF: Understand REF: 95 OBJ: Identify contemporary ethical issues in nursing practice. MSC: NCLEX: Safe and Effective Care Environment
TOP: Implementation
MULTIPLE RESPONSE 1. The nurse is caring for a patient supported with a ventilator who has been unresponsive since
arrival via ambulance 8 days ago. The patient has not been identified, and no family members have been found. The nurse is concerned about the plan of care regarding maintenance or withdrawal of life support measures. The nurse determines that this is an ethical dilemma not resolved by scientific data. PN laceRtheIsteps urseMwill use to resolve this ethical dilemma in G thBe.nC U S N T O the correct order. a. The nurse identifies possible solutions or actions to resolve the dilemma. b. The nurse reviews the medical record, including entries by all health care disciplines, to gather information relevant to this patient’s situation. c. Health care providers use negotiation to redefine the patient’s plan of care. d. The nurse evaluates the plan and revises it with input from other health care providers as necessary. e. The nurse arranges a meeting with health care team members to clarify opinions, values, and facts. f. The nurse states the problem. ANS: A, B, C, D, E, F
Once the nurse determines that an ethical dilemma exists, the nurse then uses the steps of processing an ethical dilemma to gather information relevant to the case; meets with members of the health care team to clarify values and distinguish between fact, opinion, and values; and verbalizes the problem. Then the nurse identifies possible solutions or actions, works with the health care team to negotiate a plan, and evaluates the plan over time. DIF: Apply REF: 92-95 OBJ: Apply a method of ethical analysis to a clinical situation. TOP: Assessment | Diagnosis | Evaluate | Implementation | Planning MSC: NCLEX: Safe and Effective Care Environment
Canadian Fundamentals of Nursing 6th Edition Potter Test Bank
Chapter 08: Legal Implications in Nursing Practice Potter et al: Canadian Fundamentals of Nursing, 6th Edition MULTIPLE CHOICE 1. A newly hired experienced nurse is preparing to change a patient’s abdominal dressing and
has not done it before at this hospital. Which action by the nurse is best? a. Ask another nurse to do it so the correct method can be viewed. b. Check the policy and procedure manual for the agency’s method. c. Change the dressing using the method taught in nursing school. d. Ask the patient how the dressing change has been recently done. ANS: B
The Canadian Council on Health Services requires accredited hospitals to have written nursing policies and procedures. These internal standards of care are specific and need to be accessible on all nursing units. For example, a policy/procedure outlining the steps to follow in changing a dressing or administering medication provides specific information about how nurses are to do it. The nurse being observed may not be doing the procedure according to the agency’s policy or procedure. The procedure taught in nursing school may not be consistent with the policy or procedure for this agency. The patient is not responsible for maintaining the standards of practice; patient input is important, but it’s not what directs nursing practice. DIF: Apply TOP: Planning
REF: 103 OBJ: List sources for standards of care for nurses. MSC: NCLEX: Safe and Effective Care Environment
2. A new nurse at a health care unit notes that a listing of patient names is kept in a closed book
N R I G B.C M
behind the front desk of the nuU rsinS g staNtionTso thaO t patients can be located easily. What action is most appropriate for the nurse to take? a. Move the book to the upper ledge of the nursing station for easier access. b. Talk with the nurse manager about the listing being a violation of confidentiality. c. Use the book as needed while keeping it away from individuals not involved in patient care. d. Ask the nurse manager to move the book to a more secluded area. ANS: C
Patients are entitled to confidentiality in health care. Nursing standards for what constitutes confidential information are based on professional ethics and the common law. The nurse’s judgement must be guided by the ideals of privacy and sensitivity to the needs and rights of patients who may not choose to have nurses intrude on their lives but who depend on nurses for their care. The nurse's fiduciary duty requires that confidential information not be shared with anyone else except on a need-to-know basis. The book is located where only staff would have access. It is not the responsibility of the new nurse to move items used by others on the patient unit. The listing’s confidentiality is protected as long as it is used appropriately as needed to provide care. There is no need to move the book to a more secluded area. DIF: Apply REF: 103| 104 OBJ: Describe the legal responsibilities and obligations of nurses. TOP: Implementation MSC: NCLEX: Safe and Effective Care Environment
Canadian Fundamentals of Nursing 6th Edition Potter Test Bank 3. The law that would best deal with a nurse accidentally administering an incorrect dose of
morphine sulphate to a patient is which of the following? a. Civil law. b. Criminal law. c. Human rights law. d. Statute law. ANS: A
Civil laws protect the rights of individuals. Accidental administration of an incorrect dose of morphine sulphate would fall under civil law because it could cause harm to an individual. Criminal law aims to protect society from harm and provide punishment for intentional crimes (often imprisonment). Human rights law relates to public matters, not individual. Statute law is created by parliamentary, provincial, and territorial legislatures. DIF: Apply REF: 101| 102 OBJ: Define legal aspects of nurse–patient, nurse–physician, nurse–nurse, and nurse–employer relationships. TOP: Implementation MSC: NCLEX: Safe and Effective Care Environment 4. The nurse has just obtained the licence to practise and is determining whether individual
malpractice insurance is necessary. Which of the following is the most important factor in the nurse’s decision to carry malpractice insurance? a. The amount of the malpractice insurance provided by the employer. b. Working in a critical area of nursing in which morbidity and mortality rates are high. c. Employment status and professional liability coverage. d. The nurse’s knowledge level of Good Samaritan laws. ANS: C
NURSINGTB.COM
All nurses should have clear knowledge of their employment status and professional liability coverage. Publicly funded health care facilities carry malpractice insurance. The facility is considered the employer and is liable for the negligent acts of its employees as long as their actions were within their scope of practice. The amount of the malpractice insurance provided by the employer is not the most important factor in deciding about private insurance. In general, the employer’s malpractice insurance coverage is much greater than private insurance coverage. The area of nursing is not the most important factor in deciding whether to carry malpractice insurance; lawsuits can occur anywhere. The nurse should be aware of Good Samaritan laws, but this would not ensure sufficient coverage for most nursing practice; therefore, it is not the most important factor in determining whether to purchase private malpractice insurance. DIF: Apply TOP: Planning
REF: 107 OBJ: Explain legal concepts that apply to nurses. MSC: NCLEX: Safe and Effective Care Environment
5. A nurse performs cardiopulmonary resuscitation (CPR) on a 92-year-old patient with brittle
bones and breaks a rib during the procedure, which then punctures a lung. The patient recovers completely without any residual problems and sues the nurse for pain and suffering and for malpractice. What key point will the prosecution attempt to prove? a. The CPR procedure was done incorrectly. b. The patient would have died if nothing was done. c. The patient was resuscitated according to policy.
Canadian Fundamentals of Nursing 6th Edition Potter Test Bank d. Patients with brittle bones might sustain fractures when chest compressions are
done. ANS: A
Certain criteria are necessary to establish nursing malpractice. In this situation, although harm was caused, it was not because of failure of the nurse to perform a duty according to standards the way other nurses would have performed in the same situation. If the nurse had done the procedure incorrectly, the patient probably would not have survived without any residual problems such as brain damage. The fact that the patient sustained injury as a result of age and physical status does not mean the nurse breached any duty to the patient. The nurse would need to make sure the defence attorney knew that the cardiopulmonary resuscitation (CPR) was done correctly and that without intervention, the patient probably would not have survived. The prosecution would try to prove that a breach of duty had occurred, which had caused injury, not that cardiopulmonary resuscitation was done correctly. The defence team, not the prosecution, would explain the correlation between brittle bones and rib fractures during CPR. DIF: Understand REF: 104| 105 TOP: Implementation
OBJ: List the elements needed to prove negligence. MSC: NCLEX: Safe and Effective Care Environment
6. A recent immigrant who does not speak English is alert and requires hospitalization. What is
the initial action that the nurse must take to obtain informed consent? a. Ask a family member to translate what the nurse is saying. b. Notify the health care provider that the patient does not speak English. c. Request an official interpreter to explain the terms of consent. d. Use hand gestures and medical equipment while explaining in English. ANS: C
NURSINGTB.COM
An official interpreter must be present to explain the terms of consent to a patient who speaks only a foreign language. A family member or acquaintance who speaks a patient’s language should not interpret health information. Family members can tell those caring for the patient what the patient is saying, but privacy regarding the patient’s condition, assessment, and other medical matters must be protected. There is no way to confirm that the family member is translating exactly what the nurse is saying. Privacy must be ensured and accurate information must be provided to the patient. After consent is obtained for treatment, the health care provider would be notified because little can be done without consent. The health care provider needs to have the translator available during the history and physical examination, as well as at other times, but the first step is to get a translator to obtain informed consent because this is not an emergency situation. Using hand gestures and medical equipment is inappropriate when communicating with a patient who does not understand the language spoken because (1) certain hand gestures may be acceptable in one culture and not appropriate in another, (2) the medical equipment may be unknown and frightening to the patient, and (3) the patient still does not understand what is being said. DIF: Apply REF: 106 OBJ: Give examples of legal issues that arise in nursing practice. TOP: Implementation MSC: NCLEX: Safe and Effective Care Environment 7. A pediatric oncology nurse is floated to an orthopedic trauma unit. What actions should the
nurse manager of the orthopedic unit take to enable this floated nurse to give safe care? a. Provide a complete orientation to the functioning of the entire unit.
Canadian Fundamentals of Nursing 6th Edition Potter Test Bank b. Determine the acuity of patients’ and the care that the nurse can safely provide. c. Allow the nurse to choose which mealtime she would like. d. Assign unregulated care providers to assist her with care. ANS: B
Nurses who float need to inform the supervisor of any lack of experience in caring for the type of patients on the nursing unit. They also need to request and receive a basic orientation to the unit. Supervisors are liable if they give a staff nurse an assignment that he or she cannot safely handle. Before accepting employment, a nurse must learn the policies of the institution with regard to floating and must have an understanding of what is expected of a floating nurse. A complete orientation of the functioning of the entire unit would take a period of time that would exceed what the nurse has to spend on orientation. Allowing the nurse to choose which mealtime she would like is a nice gesture of thanks for the nurse, but it does not enable safe care. Unregulated care providers may help the nurse complete basic tasks such as hygiene and turning, but their help does not enable safe nursing care for which the nurse is ultimately responsible. DIF: Apply REF: 108 OBJ: Give examples of legal issues that arise in nursing practice. TOP: Implementation MSC: NCLEX: Safe and Effective Care Environment 8. An unconscious patient with a head injury needs immediate life-saving surgery. His wife
speaks only French, and the health care providers, who are not fluent in French, are having a difficult time explaining his condition to her. In this situation, what must the nurse know? a. Two licensed health care workers should witness and sign the preoperative consent form indicating that they heard an explanation of the procedure given in English. b. An ethical review board must be contacted to give its emergency advice on the N R I G B.C M U S N T O situation. c. A friend of the family may act as an interpreter, but the explanation cannot contain details of the patient’s accident because of confidentiality laws. d. The health care team should continue with the surgery after providing information in the best manner possible. ANS: D
In emergency situations, if it is impossible to obtain consent from the patient or an authorized person, the beneficial or life-saving procedure may be undertaken without liability for failure to obtain consent. In such cases, according to the law, the health care team would assume that the patient would wish to be given the treatment. This is referred to as the emergency doctrine. Two witnesses are required usually when telephone consents are involved. This is not the case in this situation. In an emergency, it is not necessary to contact the institutional review board; doing so would take up valuable time. A family member or acquaintance who is able to speak the patient’s language should not be used to interpret health care information. An official interpreter must be available to explain the terms of consent (except in an emergency situation). DIF: Analyze REF: 106| 107 OBJ: Give examples of legal issues that arise in nursing practice. TOP: Implementation MSC: NCLEX: Safe and Effective Care Environment
Canadian Fundamentals of Nursing 6th Edition Potter Test Bank 9. Because of an influenza epidemic among the nursing staff, a nurse has been moved from the
ophthalmology unit to the general surgery unit. The supervisor recognizes that the nurse is inexperienced in this specialty. What should the nurse do? a. Politely refuse to move, take the day off, and go home. b. Ask to work with an experienced general surgery nurse. c. Submit a report noting his or her dissatisfaction. d. Notify the Canadian Nurses Association (CNA) of the issue. ANS: B
Nurses who are temporarily reassigned to another unit (i.e., float) should inform the supervisor of their lack of experience in caring for patients on a particular nursing unit. They also should request, and be given, basic orientation to the unit. Asking to work with an experienced general surgery nurse would be an appropriate action. Refusing to accept an assignment may be considered insubordination, and patients will suffer if the number of available staff drops. The nurse can make a written protest to nursing administrators, but it should not be the nurse’s initial recourse. Notifying the CNA should also not be the nurse’s initial recourse. The nurse should first notify the supervisor and request appropriate orientation and training. DIF: Apply REF: 108 OBJ: Give examples of legal issues that arise in nursing practice. TOP: Implementation MSC: NCLEX: Safe and Effective Care Environment 10. A confused patient with a urinary catheter, a nasogastric tube, and an intravenous line keeps
touching these items, which are needed for care. The nurse has tried to explain to the patient that he should not touch them, but the patient continues. What is the best action by the nurse at this time? NURSINGTB.COM a. Apply restraints loosely on the patient’s dominant wrist. b. Try other approaches to prevent the patient from touching these care items. c. Notify the health care provider that restraints are needed immediately to maintain the patient’s safety. d. Allow the patient to pull out lines to prove that the patient needs to be restrained. ANS: B
The risks associated with the use of restraints are serious. With regard to restraints, a restraint-free environment is the first goal of care for all patients. Many alternatives to the use of restraints are available, and the nurse should try all of them before notifying the patient’s health care provider. In this situation, the patient is touching the items, not trying to pull them out; therefore, at this time, the patient’s well-being is not at risk. The nurse will have to check on the patient frequently and then will determine whether the health care provider needs to be informed of the situation. The inappropriate or unjustified use of restraints (e.g., by confining a person to an area or by using physical or chemical restraints) may also be viewed as false imprisonment. The health care provider needs to know the situation but also needs to ensure that all approaches possible are used before restraints are ordered. Allowing the patient to pull out any of these items could cause harm to the patient. DIF: Apply REF: 104 OBJ: Give examples of legal issues that arise in nursing practice. TOP: Implementation MSC: NCLEX: Safe and Effective Care Environment
Canadian Fundamentals of Nursing 6th Edition Potter Test Bank 11. A nurse is working with a physician who provides medical assistance in dying (MAID) in
Canada. What must the nurse be aware of? a. The nurse can provide MAID to a patient by administering a substance prescribed by the physician. b. The patient requesting MAID must be at least 16 years of age and capable of making health care decisions. c. The patient’s natural death must be unforeseeable in order to request MAID. d. The patient requesting MAID must be eligible for publicly funded health services in Canada. ANS: D
The patient requesting MAID must be eligible for publicly funded health services in Canada. Only a physician or nurse practitioner can provide a person with MAID by administering a substance that causes death. The patient requesting MAID must be at least 18 years of age and capable of making health care decisions. The patient’s natural death must be reasonably foreseeable, in view of all the person’s medical circumstances. DIF: Apply REF: 110 (Box 8-2) OBJ: Describe the legal responsibilities and obligations of nurses. TOP: Planning MSC: NCLEX: Safe and Effective Care Environment 12. The nurse is aware that when caring for patients with communicable diseases, such as severe
acute respiratory syndrome (SARS) and acquired immunodeficiency syndrome (AIDS), a. Every health care worker who comes into contact with a patient has the right to know the patient’s HIV infection status. b. They can refuse to care for a patient with SARS or AIDS if there are a lack of resources or ongoing threats to personal well-being. Nr pro RSvid INing GTtheir B.C c. The nurse is responsible foU OnMpersonal protective equipment to ow care for patients with communicable diseases. d. In known cases of AIDS or SARS, information can be disclosed without the patient’s consent. ANS: B
Nurses are able to refuse to provide care when they experience an unreasonable burden. An unreasonable burden exists when the nurse’s ability to provide safe care and meet the standards of practice is compromised by unreasonable expectations, lack of resources, or ongoing threats to personal well-being. Not every health care worker who comes in contact with a patient needs to know the patient’s HIV status. Confidential information must be protected. Strict compliance with standard precautions or routine practices and the use of transmission-based (e.g., airborne or droplet) precautions for patients known or suspected of having serious communicable illnesses is the nurse’s wisest strategy; however, it is not the nurse’s responsibility to provide the personal protective equipment. The employer has an obligation to provide their employees with necessary protective gear. Whenever information about a patient is requested by any third parties, including insurance companies or employers, nurses must obtain a signed release from the patient before releasing confidential information. DIF: Apply REF: 109 TOP: Implementation
OBJ: List sources for standards of care for nurses. MSC: NCLEX: Safe and Effective Care Environment
Canadian Fundamentals of Nursing 6th Edition Potter Test Bank 13. A nursing student has been written up several times for being late with providing patient care
and for omitting aspects of patient care and not knowing basic procedures that were taught in the skills course one term earlier. The nursing student says, “I don’t understand what the big deal is. As my instructor, you are there to protect me and make sure I don’t make mistakes.” What is the best response from the nursing instructor? a. “You are expected to perform at the level of a professional nurse.” b. “You are expected to perform at the level of a nursing student.” c. “You are practicing under the licence of the nurse assigned to the patient.” d. “You are expected to perform at the level of a skilled nursing assistant.” ANS: A
Although nursing students are not employees of the health care agency where they are having their clinical experience, they are expected to perform as professional nurses would in providing safe patient care. Different levels of standards do not apply. Nursing students, just as nurses, provide safe, complete patient care, or they do not. No standard is used for nursing students, other than that they must meet the standards of a professional nurse. The nursing instructor, not the nurse assigned to the patient, is responsible for the actions of the nursing student. DIF: Apply REF: 107 OBJ: Describe the legal responsibilities and obligations of nurses. TOP: Implementation MSC: NCLEX: Safe and Effective Care Environment 14. An example of care where a nurse may be liable for actions that constitute an unintentional
tort is a. Physical restraining a patient who refuses care. b. Taking photos of a patient’s surgical wounds without the patient’s permission. N R I G s B.C M c. The patient falling and beinUg inSjureNd aT a resuO lt of side rails being left down. d. Talking about a patient’s history of sexually transmitted infections. ANS: C
An unintentional tort is an unintended wrongful act against another person that produces injury or harm. An example of an unintentional tort would be leaving the side rails down, which causes the patient’s fall and injury. Physically restraining a patient who refuses care would be an example of assault and battery. Taking photos of a patient’s surgical wounds without the patient’s permission is an example of invasion of privacy. Talking about a patient’s history of sexually transmitted infections would fall under the category of invasion of privacy. Personal information about the patient should be kept confidential. DIF: Analyze REF: 104| 105 OBJ: Describe the legal responsibilities and obligations of nurses. TOP: Assessment MSC: NCLEX: Safe and Effective Care Environment 15. A nursing student in the final term of nursing school is overheard by a nursing faculty
member telling another student that she got to insert a nasogastric tube in the emergency department while she was working as a nursing assistant. What advice is best for the nursing faculty member to give to the nursing student? a. “Just be careful when you are doing new procedures and make sure you are following directions by the nurse.” b. “Review your procedures before you go to work, so you will be prepared to do them if you have a chance.”
Canadian Fundamentals of Nursing 6th Edition Potter Test Bank c. “The nurse should not have allowed you to insert the nasogastric tube because
something bad could have happened.” d. “You are not allowed to perform any procedures other than those in your job
description as a nursing assistant, even with the nurse’s permission.” ANS: D
When nursing students work as nursing assistants or nurse’s aides when not attending classes, they should not perform tasks that do not appear in a job description for a nurse’s aide or assistant. The nursing student should always follow the directions of the nurse, unless doing so violates the institution’s guidelines or job description under which the nursing student was hired. The nursing student should be able to safely complete the procedures delegated as a nursing assistant, and reviewing those not done recently is a good idea, but it has nothing to do with the situation. This option does not address the situation that the nursing student acted outside the job description for the nursing assistant position. The focus of the discussion between the nursing faculty member and the nursing student should be on following the job description under which the nursing student is working. DIF: Apply REF: 107 TOP: Implementation
OBJ: List sources for standards of care for nurses. MSC: NCLEX: Safe and Effective Care Environment
MULTIPLE RESPONSE 1. The nurse calculates the medication dose for an infant on the pediatric unit and determines
that the dose is twice what it should be. The pediatrician is contacted and says to administer the medication as ordered. What is the next action that the nurse should take? (Select all that apply.) a. Notify the nursing supervN isor UR. SINGTB.COM b. Check the chain of command policy for such situations. c. Give the medication as ordered. d. Give the amount calculated to be correct. e. Contact the pharmacy for clarification. ANS: A, B
Nurses follow health care providers’ orders unless they believe the orders are in error or may harm patients. Therefore, the nurse needs to assess all orders. If an order seems to be erroneous or harmful, further clarification from the health care provider is necessary. If the health care provider confirms an order and the nurse still believe that it is inappropriate, the nurse should inform the supervising nurse or follow the established chain of command. The supervising nurse should be able to help resolve the questionable order, but only the health care provider who wrote the order or a health care provider covering for the one who wrote the order can change the order. Harm to the infant could occur if the medication dosage was too high. The nurse cannot change an order. Giving the amount calculated to be correct would not be what another nurse would do in the same situation. Although the pharmacy is an excellent resource, only the health care provider can change the order. DIF: Apply REF: 108 OBJ: Define legal aspects of nurse–patient, nurse–physician, nurse–nurse, and nurse–employer relationships. TOP: Implementation MSC: NCLEX: Safe and Effective Care Environment
Canadian Fundamentals of Nursing 6th Edition Potter Test Bank 2. A nurse gives an incorrect medication to a patient without doing all of the mandatory checks,
but the patient has no ill effects from the medication. What actions should the nurse take after reassessing the patient? (Select all that apply.) a. Notify the health care provider of the situation. b. Document in the patient’s medical record that an incident report was filed. c. Document in the patient’s medical record why the omission occurred. d. Discuss what happened with all of the other nurses and staff on the unit. e. Continue to monitor the patient for any untoward effects from the medication. f. Send an incident report to risk management after completing it. ANS: A, E, F
Examples of an incident include an error in technique or procedure such as failing to properly identify a patient. Institutions generally have specific guidelines to direct health care providers how to complete the incident or adverse occurrence report. The report is confidential and separate from the medical record. The nurse is responsible for providing information in the medical record about the occurrence. It is also best for the nurse to discuss the occurrence with nursing management only. The risk management department of the institution also requires complete documentation. The fact that an incident report was completed is not documented in the patient’s medical record. No discussion of why the omission in procedure occurred should be documented in the patient’s medical record. Errors should be discussed only with those who need to know such as the health care provider, appropriate administrative personnel, and risk management. DIF: Apply REF: 111 OBJ: Give examples of legal issues that arise in nursing practice. TOP: Implementation MSC: NCLEX: Safe and Effective Care Environment
N R I G B.C M
SaleNpaT 3. The nurse is providing care to U a fem tient whOo has been diagnosed with terminal cancer and who has a poor prognosis. When the patient’s husband comes in to the hospital cafeteria during a busy period, the nurse approaches him and openly expresses her sympathy to the man for his wife’s terminal illness. The patient’s husband bursts into tears and says that he did not know about his wife’s diagnosis. The nurse then takes him to a private area to discuss his feelings. This nurse’s action exemplifies which of the following violations? (Select all that apply.) a. Intentional tort. b. Unintentional tort. c. Invasion of privacy. d. Negligence. e. Assault. f. Battery. ANS: A, C
This can be considered an intentional tort, because the nurse is aware that they should not be breaching confidentiality, and violated the patient’s rights. The nurse can be held liable for slander as the nurse discusses private information about a patient that is overheard by others in the cafeteria. This action is also an example of invasion of privacy and occurs if a patient’s medical information is discussed without the consent of the patient. This action is not an example of assault. This action is not an example of negligence. This action is not an example of battery. DIF: Analyze
REF: 103-105
Canadian Fundamentals of Nursing 6th Edition Potter Test Bank OBJ: Give examples of legal issues that arise in nursing practice. TOP: Implementation MSC: NCLEX: Safe and Effective Care Environment 4. A patient has just been told that he has approximately six months to live and asks about
advance directives. Which statements by the nurse give the patient correct information? (Select all that apply.) a. “You have the right to refuse treatment at any time.” b. “If you want certain procedures or actions taken or not taken, and you might not be able to tell anyone at the time, you need to complete documents ahead of time that give your health care provider this information.” c. “You will be resuscitated at any time to allow you the longest length of survival.” d. “You might want to think about choosing someone who will make medical decisions for you in the event that you are unable to make your desires known.” e. “We will get someone who knows the province’s guidelines to assist you in setting up your living will.” f. “If you travel to another province, your living will should cover your wishes.” ANS: A, B, D, E
The ethical doctrine of autonomy ensures the patient the right to refuse medical treatment. Living wills are written documents that direct treatment in accordance with a patient’s wishes in the event of a terminal illness or condition. With this legal document, the patient is able to declare which medical procedures he or she wants or does not want when terminally ill or in a persistent vegetative state. Each province/territory providing for living wills has its own requirements for executing the health care proxy or durable power of attorney for health care. This is a legal document that designates a person or persons of one’s choosing to make health care decisions when the patient is no longer able to make decisions on his or her own behalf. This agent makes health careNtreatment R I decisions G B.CObased M on the patient’s wishes. Cardiopulmonary resuscitationU(CPSR)N is aT n emergency treatment provided without patient consent. Health care providers perform CPR on an appropriate patient unless a do not resuscitate (DNR) order has been placed in the patient’s chart. The statutes assume that all patients will be resuscitated unless a written DNR order is found in the chart. Legally competent adult patients can consent to a DNR order verbally or in writing after receiving appropriate information from the health care provider. Differences among the provinces have been noted regarding advance directives, so the patient should check provincial/territorial laws to see if a province/territory will honour an advance directive that was originated in another province/territory. DIF: Apply REF: 110 OBJ: Describe the legal responsibilities and obligations of nurses. TOP: Implementation MSC: NCLEX: Safe and Effective Care Environment 5. The administration of medications in accordance with a prescriber’s prescription is a basic
nursing responsibility. The nurse is responsible for knowing: (Select all that apply). a. Purpose of the medication. b. Effect of the medication. c. Cost of the medication. d. Potential adverse effects of the medication. e. Contraindications of the medication. ANS: A, B, D, E
Canadian Fundamentals of Nursing 6th Edition Potter Test Bank When administering medications in accordance with a prescriber’s prescription, the nurse is responsible for knowing the purpose and effect of the medication, along with the potential adverse effects and contraindications of the medication. The nurse is not expected to know the cost of the medications they are giving. DIF: Apply REF: 108 OBJ: Describe the legal responsibilities and obligations of nurses. TOP: Implementation MSC: NCLEX: Safe and Effective Care Environment
NURSINGTB.COM
Canadian Fundamentals of Nursing 6th Edition Potter Test Bank
Chapter 09: Global Health Potter et al: Canadian Fundamentals of Nursing, 6th Edition MULTIPLE CHOICE 1. The nurse recognizes the terminology that applies to culture and ethnicity. Ethnicity is defined
as what? a. An appreciation for differences within another group and the promotion of respect for those differences. b. A common identity with members sharing social and cultural heritage. c. Many cultures coexisting and maintaining cultural differences. d. The belief that one’s own race or culture is more valuable than those of others. ANS: B
Ethnicity refers to a shared identity related to social and cultural heritage, including values, language, geographic space, and racial characteristics. The most important characteristic of an ethnic group is that its members feel a sense of common identity. Cultural pluralism is a perspective that appreciates another group for being different and “promotes respect for the right of others to have different beliefs, values, behaviours, and ways of life” (Racher & Annis, 2012, p. 159). Multiculturalism is regarded as a fundamental characteristic of Canadian society; many cultures coexist in Canadian society and maintain their cultural differences. Ethnocentrism is a tendency to hold one’s own race or culture as more valuable than those of others. DIF: Understand REF: 122| 123 OBJ: Define key concepts related to health, illness, and diversity. N R : Ps IN G social B.CInOtegrity M TOP: Assessment MSC: NCLUEXS ychoT 2. Prevention programs for populations is the main focus of which of the following? a. Global health. b. Public health. c. International health. d. Transcultural health. ANS: B
The main focus of public health is prevention programs for populations. Global health and international health are concerned with both prevention in populations and clinical care of individuals. Transcultural health is not a domain of medical health. DIF: Understand REF: 116 (Table 9-1) OBJ: Differentiate between global health, international health, and public health in the context of professional nursing. TOP: Assessment MSC: NCLEX: Psychosocial Integrity 3. In understanding health inequity, what does the nurse know? a. Health differences are unavoidable. b. Poverty is not a root cause of health inequity. c. Health inequity is the absence of systematic disparities in health. d. Health inequity refers to unnecessary and unfair differences in health. ANS: D
Canadian Fundamentals of Nursing 6th Edition Potter Test Bank Health inequities refer to differences in health that are not only unnecessary and avoidable but, in addition, are considered unfair and unjust. Health differences are seen as avoidable. Poverty is often a root cause of health inequity. The absence of systematic disparities in health is characteristic of health equity. DIF: Remember REF: 117 OBJ: Define key concepts related to health, illness, and diversity. TOP: Assessment MSC: NCLEX: Psychosocial Integrity 4. When asked to describe the differences between ethnicity and race, what should the student
nurse explain? a. Ethnicity refers to a shared identity, whereas race is limited to biological attributes. b. Ethnicity and race are actually the same and are based in cultural norms. c. Ethnicity can be understood only through an etic world view. d. Race refers to a shared identity, whereas ethnicity is limited to biological attributes. ANS: A
Ethnicity refers to a shared identity related to social and cultural heritage such as values, language, geographical space, and racial characteristics. Ethnicity is different from race, which is limited to the common biological attributes shared by a group such as skin colour or blood type. In any intercultural encounter, there is an insider or native perspective (emic world view) and an outsider’s perspective (etic world view). Ethnicity is best understood by those who are a part of that ethnicity and have an emic world view. DIF: Understand REF: 122| 123 OBJ: Define key concepts related to health, illness, and diversity. TOP: Assessment MSC: NN CLUER XS : PIsyNcG hoT soBc. ialCInOteM grity 5. The nurse learns about cultural issues involved in the patient’s health care belief system and
enables patients and families to achieve meaningful and supportive care. Such care is known as what? a. Ethnocentrism. b. Culturally competent care. c. Cultural imposition. d. Culturally congruent care. ANS: B
Culturally competent care reflects the ability of a nurse to bridge cultural gaps in caring and enables patients and families to achieve meaningful and supportive caring. It is a step toward reaching culturally congruent care. Culturally congruent care, or care that fits the person’s valued life patterns and set of meanings, is the goal of transcultural nursing. Ethnocentrism is a tendency to hold one’s own way of life as superior to those of others. It is the cause of biases and prejudices. Cultural imposition is the use of one’s own values and lifestyles as the absolute guide in dealing with patients and interpreting behaviours. DIF: Remember REF: 123 OBJ: Describe the historical development of the concept of culture, cultural competence, cultural safety, and cultural humility in relation to nursing practice. TOP: Assessment MSC: NCLEX: Psychosocial Integrity
Canadian Fundamentals of Nursing 6th Edition Potter Test Bank 6. The nurse is caring for an Indigenous patient who has had recent surgery. In the patient’s
culture, it is a sign of weakness to complain of pain. In the nurse’s culture, people who are having pain ask for pain medicine. The nurse has assumed that the patient has not been having pain and does not need medication because he has not complained of pain. What is the nurse doing? a. Utilizing cultural imposition by not asking the patient about his pain. b. Striving to provide culturally congruent care by allowing the patient to suffer. c. Operating from an emic world view of the patient’s cultural beliefs. d. Practising discrimination by not giving the patient pain medicine. ANS: A
Health care practitioners who have cultural ignorance or cultural blindness about differences generally resort to cultural imposition and use their own values and lifestyles as the absolute guide in dealing with patients and interpreting their behaviours. Culturally competent care is the care provided by the nurse who attempts to bridge cultural gaps in caring, work with cultural differences, and enable patients and families to achieve meaningful and supportive caring. The nurse in this case has not been able to do this. Any intercultural encounter consists of an inside or native perspective (emic world view) and an outsider’s perspective (etic world view). The nurse is obviously utilizing an etic world view. The nurse did not purposefully ignore the patient’s need. DIF: Apply REF: 123 OBJ: Analyze components of cultural assessment critical to understand the values, beliefs, and practices critical in the nursing care of people experiencing cultural transitions. TOP: Implementation MSC: NCLEX: Psychosocial Integrity 7. When a cultural assessment is performed, knowledge of a patient’s country of origin and its
NUisRkSnow INnGaTs B.C history and ecological contexts what?O M a. Ethnohistory. b. Biocultural history. c. Social organization. d. Religious and spiritual beliefs. ANS: A
A patient’s country of origin and its history and ecological contexts is known as ethnic heritage and ethnohistory, and knowledge about it is significant in health care. Biocultural history can help identify a patient’s health risks in relation to the ecological context of the culture. Social organization refers to units of organization in a cultural group defined by kinship status and appropriate roles for their members. Religious and spiritual beliefs are major influences in the patient’s world view about health and illness, pain and suffering, and life and death. Nurses need to understand the emic perspective of their patients. DIF: Remember REF: 126| 127 OBJ: Analyze components of cultural assessment critical to understand the values, beliefs, and practices critical in the nursing care of people experiencing cultural transitions. TOP: Assessment MSC: NCLEX: Psychosocial Integrity 8. A nursing student is caring for a patient who has just immigrated to Canada from Ghana. The
student just had a class on cultural safety and wants to practise doing a cultural assessment. What should the nursing student know? a. Patients who come from the same region or country share similar values, beliefs,
Canadian Fundamentals of Nursing 6th Edition Potter Test Bank attitudes, and experiences. b. Cultural assessment is a systematic and comprehensive examination of the cultural
care values, beliefs, and practices of individuals, families, and communities. c. The patient should be discouraged from sharing personal stories because it will
take up too much time. d. The student should ask only open-ended questions. ANS: B
Cultural assessment is a systematic and comprehensive examination of the cultural care values, beliefs, and practices of individuals, families, and communities. Not all patients who come from the same region or country share similar values, beliefs, attitudes, and experiences. The patient should be encouraged to share personal stories to reveal how he or she thinks and the cultural lifestyle that he or she embraces. The student should use open-ended questions, focused questions, and contrast questions. DIF: Apply REF: 126 OBJ: Analyze components of cultural assessment critical to understand the values, beliefs, and practices critical in the nursing care of people experiencing cultural transitions. TOP: Implementation MSC: NCLEX: Psychosocial Integrity 9. The nurse is caring for a patient who has emigrated from another country. The patient needs
abdominal surgery but seems reluctant to sign the surgical permits. What is one tactic that the nurse should use? a. Determine the family social hierarchy. b. Encourage the patient to sign the permits. c. Call the physician so that surgery can be cancelled. d. Impress on the patient that her life is in jeopardy. ANS: A
NURSINGTB.COM
Nurses should determine the family social hierarchy as soon as possible to prevent offending patients and their families. Working with established family hierarchy prevents delays and achieves better patient outcomes. Encouraging the patient to sign against her social beliefs can cause familial strife. Explaining the level of jeopardy may create undue stress. Nurses should be able to determine the correct hierarchy and should not involve the physician at this time. DIF: Apply REF: 126 OBJ: Analyze components of cultural assessment critical to understand the values, beliefs, and practices critical in the nursing care of people experiencing cultural transitions. TOP: Implementation MSC: NCLEX: Psychosocial Integrity 10. Which of the following is an outcome of nursing education that enables safe service to be
defined by those who receive the service? a. Cultural awareness. b. Cultural safety. c. Cultural sensitivity. d. Self-awareness. ANS: B
Canadian Fundamentals of Nursing 6th Edition Potter Test Bank Cultural safety is an outcome of nursing education that enables safe service to be defined by those who receive the service. Cultural safety involves considering the redistribution of power and resources in a relationship. Cultural awareness is a beginning step toward understanding that there are differences between cultures. Cultural sensitivity alerts nurses to the legitimacy of difference and begins a process of self-exploration. Self-awareness is the result of reflection. DIF: Understand REF: 124 OBJ: Define key concepts related to health, illness, and diversity. TOP: Assessment MSC: NCLEX: Psychosocial Integrity 11. A nursing student is performing a cultural assessment of their patient. When the student asks,
“Who makes the decisions for you or your family?” what is the nurse assessing? a. Language and communication. b. Caring beliefs and practices. c. Socioeconomic status. d. Social organization. ANS: D
The question “Who makes the decisions for you or your family?” would be used to assess social organization during the cultural assessment. The question does not address caring beliefs and practices, socioeconomic status, or language and communication. DIF: Apply REF: 127 OBJ: Analyze components of cultural assessment critical to understand the values, beliefs, and practices critical in the nursing care of people experiencing cultural transitions. TOP: Assessment MSC: NCLEX: Psychosocial Integrity
NURSINGTB.COM
12. Members of a Chinese family that has been in Canada for 5 years have learned to speak
English and have adopted certain Western customs but have continued to adhere to their values, beliefs, and traditions. What is this process referred to as? a. Assimilation. b. Enculturation. c. Acculturation. d. Multiculturalism. ANS: C
Acculturation is the process of adapting to or adopting the characteristics of a new culture. Assimilation is a process whereby a minority group gradually acquires the attitudes and customs of the mainstream culture. Socialization into one’s primary culture in childhood is known as enculturation. Multiculturalism—the coexistence of many cultures and the maintenance of cultural differences—is prevalent in Canadian society. DIF: Understand REF: 123 OBJ: Analyze components of cultural assessment critical to understand the values, beliefs, and practices critical in the nursing care of people experiencing cultural transitions. TOP: Assessment MSC: NCLEX: Psychosocial Integrity 13. When caring for a patient of a different culture, it is important for the nurse to understand
which of the following? a. The nurse should protect the patient from family intrusion in her health care
Canadian Fundamentals of Nursing 6th Edition Potter Test Bank decisions. b. Working within the established family hierarchy produces better outcomes. c. Women as primary caregivers make independent health decisions. d. Gender is not a factor with regard to role expectations. ANS: B
Working with established family hierarchy prevents delays and achieves better patient outcomes. Nurses need to determine who has authority for making decisions within the family and how to communicate with the proper individuals. The nurse must not assume that just because a woman is the primary caregiver, she will make decisions independently. The nurse should determine the family social hierarchy as soon as possible. Gender may also differentiate role expectations. DIF: Apply REF: 126 OBJ: Analyze components of cultural assessment critical to understand the values, beliefs, and practices critical in the nursing care of people experiencing cultural transitions. TOP: Planning MSC: NCLEX: Psychosocial Integrity 14. Time takes on different meanings from one culture to another. Understanding this, when
planning nursing interventions, the nurse should a. Avoid using set times for procedures. b. Mutually negotiate time schedules with patients. c. Encourage patients to set their own times for care, regardless of the schedule. d. Maintain the set times for treatments and inform patients of the schedule. ANS: B
Differences exist in the dimensions of time that cultures emphasize and how time is expressed. Patients’ access toNhe alth sm .aCyObMe improved through time schedules that UR SIserv NGice TB are mutually negotiated, so as to allow for cultural patterns to be respected. For organizational purposes, the nurse should seek the patient’s input, and together the nurse and the patient may set a time to perform procedures. Although the patient’s input should be sought, it is not realistic to have patients set their own times for nursing care activities regardless of the schedule. Some procedures may be required more frequently than the patient would set, or the nurse may be unable to meet the needs of several patients on the unit at the same time. Maintaining set times for treatments and informing the patient of the schedule does not take into consideration the patient’s time orientation. DIF: Apply REF: 127| 128 OBJ: Analyze components of cultural assessment critical to understand the values, beliefs, and practices critical in the nursing care of people experiencing cultural transitions. TOP: Implementation MSC: NCLEX: Psychosocial Integrity 15. What characterizes culturally congruent care? a. It fits the patient’s valued life patterns and set of meanings. b. It is based on meanings generated by predetermined criteria. c. It is the same as the values of the professional health care system. d. It is based on the assumption that a person’s own way of life is superior to those of
other people. ANS: A
Canadian Fundamentals of Nursing 6th Edition Potter Test Bank The goal of transcultural nursing is culturally congruent care, or care that fits the person’s valued life patterns and set of meanings. Patterns and meanings are generated from people themselves, rather than from predetermined criteria. Culturally congruent care is sometimes different from the values and meanings of the professional health care system. Ethnocentrism is the tendency to view one’s own way of life as superior to those of others, and it is not part of culturally congruent care. DIF: Understand REF: 123 OBJ: Apply research findings to the provision of culturally competent care with considerations for cultural safety and relational practice. TOP: Assessment MSC: NCLEX: Psychosocial Integrity 16. The nurse may work with patients from many different cultural backgrounds. Nurses,
unfortunately and inadvertently, may impose their own cultural beliefs on patients. Which of the following is an example of a nurse imposing personal perspectives on a patient? a. Adapting the patient’s room to accommodate extra family members who are visiting. b. Seeking information on gender-congruent care for an Egyptian patient. c. Directing an older Chinese patient to do rehabilitation exercises after she has refused to do them until her daughter arrives. d. Encouraging family members to assist with the patient’s care when it is appropriate for them to do so. ANS: C
In collectivistic cultures that value group reliance and interdependence, such as traditional South Asian cultures, caring behaviours are manifested by actively providing physical and psychological support for kin members. The nurse may perceive the patient’s refusal to INcare G T,B.C M case the nurse is imposing her own exercise as lack of motivationNfUorRsS elfand inOthis belief system on the patient. Adaptation of the patient’s room to accommodate extra family members is not an example of cultural imposition on a patient but rather is meeting the patient’s need by providing culturally congruent care. Seeking information on gender-congruent care for an Egyptian patient is an example of the desire to provide culturally congruent care. Encouraging family members to assist with the patient’s care is not an example of cultural imposition on a patient. Western culture tends to follow a pattern of caring that focuses on self-care and self-determination, whereas in non-Western cultures, people typically have care provided by others. DIF: Apply REF: 123 OBJ: Apply research findings to the provision of culturally competent care with considerations for cultural safety and relational practice. TOP: Implementation MSC: NCLEX: Psychosocial Integrity MULTIPLE RESPONSE 1. Which of the following is considered a noncommunicable disease? (Select all that apply.) a. Diabetes. b. Cardiovascular diseases. c. Influenza. d. Chronic respiratory diseases. e. Cancer.
Canadian Fundamentals of Nursing 6th Edition Potter Test Bank ANS: A, B, D, E
Noncommunicable diseases are not contagious, which means they are not passed from one individual to another. The four main types of noncommunicable diseases are cardiovascular diseases, chronic respiratory diseases, cancers, and diabetes. Influenza is contagious and can be transmitted from one person to another. DIF: Understand REF: 118 OBJ: Describe key challenges in the prevention and control of communicable disease, noncommunicable disease (NCDs), and neglected tropical disease (NTDs). TOP: Planning MSC: NCLEX: Psychosocial Integrity 2. Which of the following are characteristics of the Sustainable Development Goals (SDGs)?
(Select all that apply.) a. SDGs include 10 universal goals with targets planned for the next 12 years. b. The SDGs address the health and well-being of persons of all ages. c. SDGs represent an expectation that all countries must promote prosperity while providing protection to the planet. d. SDGs are legally binding. e. Governments globally are expected to establish national frameworks for achievement of SDGs. ANS: B, C, E
The SDGs address the health and well-being of persons of all ages, including newborns, children, adolescents, and middle-aged and older persons. The unique feature of the SDGs is the importance for all countries to promote prosperity while protecting the planet. Governments globally are expected to assume ownership and establish national frameworks for achievement of the 17 goals. SDGs include 17 universal goals with targets planned for the next 15 years. The SDGs areN nU otRleS gI alN lyG bT inB di. ngC. OM DIF: Analyze REF: 117| 118 OBJ: Describe the United Nations (UN) Sustainable Development Goals (SGDs) TOP: Evaluate MSC: NCLEX: Psychosocial Integrity
Canadian Fundamentals of Nursing 6th Edition Potter Test Bank
Chapter 10: Indigenous Health Potter et al: Canadian Fundamentals of Nursing, 6th Edition MULTIPLE CHOICE 1. The nurse is aware that historically, Indigenous communities experienced health and
well-being through which of the following? a. A holistic view. b. A biomedical view. c. A spiritual view. d. A physical view. ANS: A
Historically, Indigenous communities experienced healing and well being through a holistic view of health, in which illness and treatment consisted of physical, emotional, mental, and spiritual dimensions. DIF: Understand REF: 136 OBJ: Differentiate between Indigenous health from a global perspective and Indigenous health from a Canadian perspective TOP: Planning MSC: NCLEX: Health Promotion and Maintenance 2. The significant educational, income, health, and social disparities between Indigenous people
and other Canadians is a reflection of what? a. The geographical spread of Canada’s population. b. Genetic diseases that are passed through generations. c. Physical and psychologicN alUaR buSsI e fN roGmTtB he.IC ndOiaMn residential school system. d. Infectious diseases processes. ANS: C
Over several generations, the Indian residential school system left a legacy of physical and psychological abuse that “is reflected in the significant educational, income, health and social disparities between Aboriginal people and other Canadians” (Truth and Reconciliation Commission, 2015b, p. 135). DIF: Understand REF: 137 OBJ: Examine the legacy of residential schools, which has had devastating consequences for Indigenous communities across Canada. TOP: Diagnosis MSC: NCLEX: Psychosocial Integrity 3. According to the records kept, it is estimated that at least how many children died from
malnourishment, diseases such as tuberculosis (TB), and abuse at residential schools in Canada? a. 150. b. 1500. c. 3200. d. 5000. ANS: C
Canadian Fundamentals of Nursing 6th Edition Potter Test Bank It is estimated that at least 3200 children died from malnourishment, diseases such as TB, and abuse; this number includes those who ran away and those who froze to death. The number of children who died could be 5 to 10 times higher; however, because of poor record keeping, the full number may never be known. DIF: Remember REF: 137 OBJ: Examine the legacy of residential schools, which has had devastating consequences for Indigenous communities across Canada. TOP: Evaluate MSC: NCLEX: Psychosocial Integrity 4. Which of the following can be defined as the pathways by which trauma is transmitted from
one generation to the next? a. Birth trauma. b. Post-traumatic stress disorder. c. Intergenerational trauma. d. Generational traumatic disorder. ANS: C
Intergenerational trauma is defined as the pathways by which the nature of trauma is understood and experienced by Indigenous “survivors of the residential school system and their descendants” as well as “the pathways by which this trauma is transmitted from one generation to the next” (Aguiar & Halseth, 2015, p. 23). DIF: Understand REF: 137 OBJ: Examine the legacy of residential schools, which has had devastating consequences for Indigenous communities across Canada. TOP: Evaluate MSC: NCLEX: Psychosocial Integrity
NURSINGTB.COM
5. The Truth and Reconciliation Commission issued its final report in 2015. It stated that Canada
was guilty of the destruction of those structures and practices that allow a group to continue as a group. Such destruction is known as which of the following? a. Structural racism. b. Cultural genocide. c. Intergenerational trauma. d. Colonialism. ANS: B
The Truth and Reconciliation Commission stated that Canada was guilty of cultural genocide with regard to Indigenous people in Canada, which is the “destruction of those structures and practices that allow the group to continue as a group” (Truth and Reconciliation Commission, 2015a, p. 3). Such destruction of practices and structures included occupying and seizing land; forcing relocation of Indigenous peoples and confining them to reserves; disempowering them through replacement of existing forms of Indigenous government; and denying them basic rights, such as the right to practice their faith, the right to assemble, and the right to legal counsel. DIF: Understand REF: 138 OBJ: Examine the Canadian history of colonization that includes pre-European and post-European contact. TOP: Assessment MSC: NCLEX: Psychosocial Integrity
Canadian Fundamentals of Nursing 6th Edition Potter Test Bank 6. Health care for Indigenous children living on reserves is funded through the federal
government, but is regulated through provincial/territorial systems and policies. This complex funding system, which results in denial, delay, or disruption of services, is an example of what? a. Holistic health care. b. Cultural genocide. c. Intergenerational trauma. d. Structural racism. ANS: D
The complex funding system for Indigenous children living on reserves is one example of structural racism. Structural racism is the legitimized and “normalized” spectrum of attitudes, practices, and policies that consistently result in chronic and continuous substandard outcomes for Indigenous peoples. DIF: Apply REF: 138 OBJ: Describe the concepts of structural racism, child welfare, poverty, and the justice system in relation to nursing practice. TOP: Evaluate MSC: NCLEX: Safe and Effective Care Environment 7. In comparison with non-Indigenous children, Indigenous children are what in the Canadian
child welfare system? a. Overrepresented. b. Underrepresented. c. Equally represented. d. Nonexistent. ANS: A
NURSINGTB.COM
Across Canada, Indigenous children are overrepresented in the child welfare system. In some provinces, the rate of out-of-home care for Indigenous children is more than 10 times that of non-Indigenous children. DIF: Remember REF: 140 OBJ: Describe the concepts of structural racism, child welfare, poverty, and the justice system in relation to nursing practice. TOP: Assessment MSC: NCLEX: Safe and Effective Care Environment 8. Which of the following plays a role in the incarceration rates of Indigenous people? a. Violence. b. Fetal alcohol spectrum disorder. c. Schizophrenia. d. Depression. ANS: B
Fetal alcohol spectrum disorder (FASD) plays a role in the increasing incarceration rates of Indigenous people. It is estimated that 10 to 25% of all Canadians who are incarcerated suffer from FASD. For Indigenous people, there is often a connection among residential school experience, addiction, and FASD. DIF: Apply REF: 141 OBJ: Describe the concepts of structural racism, child welfare, poverty, and the justice system in relation to nursing practice. TOP: Evaluate
Canadian Fundamentals of Nursing 6th Edition Potter Test Bank MSC: NCLEX: Psychosocial Integrity 9. A nurse caring for an Indigenous family is aware that globally, Indigenous peoples have much
in common in terms of world views. How is the concept of time considered? a. It is focused on minutes rather than days. b. It is focused on years rather than months. c. It is focused on seasons rather than hours. d. It is focused on lunar cycles rather than months. ANS: C
The sense of time in Indigenous world views is elastic, focused on seasons rather than hours. It is not focused on minutes rather than days, years rather than months, or lunar cycles rather than months. DIF: Understand REF: 141 OBJ: Describe components of Indigenous cultural orientations in relation to nursing practice. TOP: Planning MSC: NCLEX: Safe and Effective Care Environment 10. Indigenous youth who were adopted have shown extremely poor self-esteem as they struggle
with reconfiguring their identity. This has been expressed in high rates of which problems? a. Eating disorders. b. Suicidal ideation. c. Schizophrenia. d. Dissociative identity disorder. ANS: B
Indigenous youth who were adopted have shown extremely poor self-esteem and high rates of suicidal ideation as they strugNgleRwitI reG conB guC ringMtheir identity. Mental health disorders in U Sh N Tfi. O general are more prevalent in this population than in others; however, suicidal ideation is directly linked to the reconfiguration of identity. DIF: Apply REF: 142 OBJ: Examine selected chronic illness experiences relevant to the Indigenous peoples of Canada. TOP: Diagnosis MSC: NCLEX: Psychosocial Integrity 11. A process that is reciprocal, in which people are viewed and treated as inherently worthy and
equal, is a fundamental aspect of nursing practice known as which of the following? a. Respect. b. Dignity. c. Justice. d. Acceptance. ANS: A
Respect is a process that is reciprocal, in which people are viewed and treated as inherently worthy and equal in principle. Respect is demonstrated by a willingness to be accepting and to listen actively, as well as genuinely trying to understand the situations patients found themselves in. Respect addresses dignity and justice. DIF: Apply REF: 143 OBJ: Differentiate between respect, trust, and spirituality in the context of caring for Indigenous people. TOP: Planning MSC: NCLEX: Safe and Effective Care Environment
Canadian Fundamentals of Nursing 6th Edition Potter Test Bank 12. A nurse who is caring for an Indigenous family must be aware of which of the following? a. All Indigenous peoples practise burning sweetgrass as a cleansing activity. b. Many Indigenous peoples practise cultural knowledge because of colonization. c. Not all Indigenous people follow traditional Indigenous faiths. d. Items placed in the bed or pinned to the patient’s gown may be moved as needed. ANS: C
Not all Indigenous people follow traditional Indigenous faiths. Many Indigenous peoples have lost their cultural knowledge because of colonization and cultural genocide. Some Indigenous people burn sweetgrass as a cleansing activity, but the nurse cannot assume that all Indigenous peoples practise this activity. Items placed in the bed or pinned to the patient’s gown should not be moved as needed, as they may hold spiritual and healing significance for the patient and family. DIF: Analyze REF: 143 OBJ: Describe components of Indigenous cultural orientations in relation to nursing practice. TOP: Implementation MSC: NCLEX: Health Promotion and Maintenance 13. The nurse is documenting an admission history for an Indigenous youth. Which of the
following questions might the nurse ask, in order to assess intermediate determinants of health? a. “What did you eat in the last 24 hours?” b. “What grade are you in?” c. “Do you smoke?” d. “How much physical activity do you do each week?” ANS: B
Intermediate determinants ofNhea incl ude on, community infrastructure, resources URlthSI NG TBed.uCcati OM and systems. Asking the youth about access to education would help the nurse assess intermediate determinants of health. Diet, physical activity, and smoking are all considered proximal determinants of health. DIF: Analyze REF: 144 OBJ: Examine selected chronic illness experiences relevant to the Indigenous peoples of Canada. TOP: Assessment MSC: NCLEX: Health Promotion and Maintenance 14. Which of the following would be considered a distal determinant of health? a. Effective policing services. b. Access to walking trails. c. Eating nutritious meals. d. Exercising daily. ANS: A
Distal determinants of health have the most influence on a population’s health and encompass the historical, political, social, and economic contexts from which intermediate and proximal determinants of health are constructed (such as stable government structures, competent leadership, and effective policing services). Access to walking trails is considered an intermediate determinant of health. Eating nutritious meals and exercising daily are considered proximal determinants of health. DIF: Apply REF: 144 OBJ: Examine selected chronic illness experiences relevant to the Indigenous peoples of Canada.
Canadian Fundamentals of Nursing 6th Edition Potter Test Bank TOP: Evaluate
MSC: NCLEX: Health Promotion and Maintenance
15. Which of the following groups has a higher rate of diabetes mellitus (type 2 diabetes)? a. Indigenous people living in an urban setting. b. Indigenous people living in a rural setting. c. Non-Indigenous people living in an urban setting. d. Non-Indigenous people living in a rural setting. ANS: B
Indigenous people living in rural areas have a higher rate of diabetes than those living in an urban setting. Indigenous men and women have a greater risk of diabetes at a much younger age than do non-Indigenous people. DIF: Understand REF: 145 OBJ: Examine selected chronic illness experiences relevant to the Indigenous peoples of Canada. TOP: Diagnosis MSC: NCLEX: Physiological Integrity 16. In comparison with their non-Indigenous counterparts, Indigenous persons with a diagnosis of
HIV infection are more likely to be what? a. Older. b. Male. c. Infected through intravenous drug use. d. Transgender. ANS: C
In comparison with their non-Indigenous counterparts, Indigenous persons with a diagnosis of HIV infection are more likely to be younger, female, and infected through intravenous drug use. There is currently no resN earcRh oI e efB ctCof H U Sn th NG Tfe. OMIV/AIDS on Two Spirit people (the term Two Spirit is used by Indigenous people to identify a range of roles and identities that include gender, sex, and sexual identity). DIF: Understand REF: 147 OBJ: Examine selected chronic illness experiences relevant to the Indigenous peoples of Canada. TOP: Assessment MSC: NCLEX: Health Promotion and Maintenance 17. Which of the following populations has the highest rate of lung cancer in the world? a. Indigenous people who live on reserves. b. Métis who live in Quebec. c. Indigenous elders. d. The circumpolar Inuit population. ANS: D
The circumpolar Inuit population has the highest rate of lung cancer in the world. Smoking is highly prevalent among the Inuit; almost 63% of Canadian Inuit are daily smokers. The effects of colonization, residential schools, diet changes, and alterations in lifestyle have all been cited as factors in the increase of cancers among Indigenous people in Canada. DIF: Remember REF: 147 OBJ: Examine selected chronic illness experiences relevant to the Indigenous peoples of Canada. TOP: Evaluate MSC: NCLEX: Health Promotion and Maintenance
Canadian Fundamentals of Nursing 6th Edition Potter Test Bank 18. The nurse is caring for an Indigenous woman who describes having been taken away from her
family and placed in a non-Indigenous home when she was a child. What is the term for this phenomenon? a. The Child Care Act. b. Forced adoption. c. The “Sixties Scoop.” d. The Protection Act. ANS: C
The Sixties Scoop was a phenomenon that continued into the 1990s, in which children were apprehended from reservations and Indigenous families on the slightest pretext in order to “save them from the effects of crushing poverty, unsanitary health conditions, poor housing and malnutrition” (Johnson, 1983, p. 23). Many of the children who were removed from their homes and placed with non-Indigenous families did not fare well. This phenomenon is not termed the Child Care Act, forced adoption, or the Protection Act. DIF: Apply REF: 139 OBJ: Describe the concepts of structural racism, child welfare, poverty, and the justice system in relation to nursing practice. TOP: Assessment MSC: NCLEX: Psychosocial Integrity MULTIPLE RESPONSE 1. Which of the following are steps to becoming a settler ally? (Select all that apply.) a. Self-awareness. b. Self-education. c. Create an open and suppoNrtivR U e SenIvir NoGnm TBen.t.COM d. Diagnosis. e. Action. ANS: A, B, C, E
A settler ally is someone who collectively considers how steps taken can advance the Truth and Reconciliation Commission process. The steps to becoming a settler ally include engaging in self-awareness, self-education, creating an open and supportive environment, and action. DIF: Apply REF: 139 (Box 10-2) OBJ: Differentiate between respect, trust, and spirituality in the context of caring for Indigenous people. TOP: Planning MSC: NCLEX: Safe and Effective Care Environment 2. Which of the following are intermediate determinants of health that have contributed to health
problems in Indigenous communities? (Select all that apply.) a. Poor environmental stewardship. b. Disempowerment. c. Contamination of wildlife, vegetation, and water. d. Change in diet. e. Sedentary lifestyle. ANS: A, B, C
Canadian Fundamentals of Nursing 6th Edition Potter Test Bank Community infrastructure in Indigenous communities has been limited, contributing to economic insecurity and marginalization. Insufficient social resources have led to decreased access to funding. Poor environmental stewardship, disempowerment, and contamination of wildlife, vegetation, and water have contributed to health problems. Diet changes and sedentary lifestyle would be considered proximal determinants of health, not intermediate. DIF: Understand REF: 144| 145 OBJ: Examine selected chronic illness experiences relevant to the Indigenous peoples of Canada. TOP: Evaluate MSC: NCLEX: Health Promotion and Maintenance
NURSINGTB.COM
Canadian Fundamentals of Nursing 6th Edition Potter Test Bank
Chapter 11: Nursing Leadership, Management, and Collaborative Practice Potter et al: Canadian Fundamentals of Nursing, 6th Edition MULTIPLE CHOICE 1. Which of the following is a requirement for a new nurse in delegating tasks to an unregulated
care provider (UCP)? a. Obtaining the UCP’s voluntary acceptance of the task. b. Communicating the task in understandable terms. c. Functioning with a laissez-faire style of leadership. d. Always supervising the UCP. ANS: B
When delegating, the nurse should always provide unambiguous and clear directions by describing the task, the desired outcome, and the period within which the task should be completed. Tasks should be delegated to UCPs who are capable, not necessarily to those who are willing. A laissez-faire style of leadership is not a requirement for delegation. The nurse does not necessarily have to supervise the UCP, unless it is required by policy or unless the nurse is unsure of the UCP’s ability to perform the task. DIF: Understand REF: 166| 167 OBJ: Discuss principles for the appropriate delegation of patient care activities. TOP: Implementation MSC: NCLEX: Safe and Effective Care Environment 2. To be able to meet the needs of assigned patients and the responsibilities associated with the
position, nurses must be aware of time management techniques. A time management skill for N R ? INGTB.COM the nurse is which of the followUingS a. Meeting all of the patients’ needs in the early-morning hours. b. Planning effectively and being aware of competing priorities. c. Conducting patient assessments individually at separate times throughout the day. d. Leaving each day unplanned to allow for adaptations in treatments. ANS: B
Time management skills for nurses include reflecting on how they use their time, planning effectively, and being aware of competing priorities. Learning new technologies, juggling various priorities, and responding to multiple demands are issues to which nurses are required to respond in a timely manner. Meeting all of the patients’ needs in the early-morning hours would be an unrealistic goal because (1) some activities have specific time limits in terms of addressing patient needs, and some activities follow scheduled routines according to hospital policy and (2) the nurse may also have to work around changes in schedules, such as a test that was ordered for the morning. Time management involves using patient goals as a way to identify priorities. The nurse, in reviewing the care requirements, organizes his or her time so the activities of care and patient goals can be achieved. The nurse should complete the activities started with one patient before moving on to another. Because the nurse has a limited amount of time with patients, it is essential to remain goal-oriented and make a plan for using time wisely, which might include conducting patient assessments together rather than at separate times of the day. DIF: Apply REF: 166 OBJ: Discuss ways to apply skills of clinical care coordination in nursing practice.
Canadian Fundamentals of Nursing 6th Edition Potter Test Bank TOP: Implementation
MSC: NCLEX: Safe and Effective Care Environment
3. Nursing management in a facility is considering a nursing care delivery model that involves
the division of tasks, with each nurse assuming the responsibility for particular tasks. What is this model referred to as? a. Total patient care. b. Functional nursing. c. Team nursing. d. Primary nursing. ANS: B
Functional nursing is task-focused, not patient-focused. In this model, tasks are divided, with each nurse assuming responsibility for specific tasks. In the total patient care model, a registered nurse (RN) is responsible for all aspects of care for one or more patients. The RN may delegate aspects of care but retains accountability for care of all assigned patients. In team nursing, an RN leads a team that is composed of other RNs, registered psychiatric nurses or licensed practical nurses, and unregulated care providers (UCPs). The team members provide direct patient care to groups of patients, under the direction of the RN team leader. UCPs are given patient assignments rather than being assigned particular tasks. In the primary nursing model, an RN assumes responsibility for a caseload of patients over time. Typically, the RN selects the patients for his or her caseload and cares for the same patients during their hospitalization or stay in the health care setting. DIF: Understand REF: 162 OBJ: Describe the purpose, elements, and models for quality practice environments and patient safety. TOP: Assessment MSC: NCLEX: Safe and Effective Care Environment
Ba.nC Ment philosophy from centralized to 4. The medical centre has changNeU dR itsSoIvN erG alT lm agO em decentralized management. What is one advantage of a decentralized management structure over a centralized structure for the nursing units? a. Staff members are not responsible for defining their roles. b. Nurse managers handle the difficult decisions. c. Communication pathways are simplified. d. Each staff member is actively involved. ANS: D
In decentralized management, decision making is moved down to the level of staff. The advantage of this structure is that managers and staff are actively involved in shaping an organization’s identity and determining its success. If decentralized decision making is in place, professional staff members have a voice in identifying the RN’s role. Each RN on the work team is responsible for knowing his or her role and how it is to be implemented on the nursing unit. Decentralized management is characterized by autonomy (freedom to decide and act). The nurse manager does not necessarily handle the difficult decisions; the staff members who are best informed about a problem or issue make decisions on the basis of their knowledge. Communication pathways are not necessarily simplified in a decentralized management structure. DIF: Understand REF: 163 OBJ: Describe the relationships between nursing leadership and healthy practice environments, patient safety, and quality patient care outcomes. TOP: Implementation MSC: NCLEX: Safe and Effective Care Environment
Canadian Fundamentals of Nursing 6th Edition Potter Test Bank
5. Which of the following is one of the four elements of decision making? a. Justice. b. Responsibility. c. Accessibility. d. Resource allocation. ANS: B
Responsibility is one of the four elements of decision making. The other three elements are autonomy, authority, and accountability. Justice and resource allocation are ethical principles and are not among the four elements of decision making. Accessibility is one of the five principles of the Canada Health Act. DIF: Remember REF: 164| 165 OBJ: Identify strategies for developing entry-level competencies related to management and leadership in nursing. TOP: Planning MSC: NCLEX: Safe and Effective Care Environment 6. The charge nurse asks an experienced nurse to admit a critically ill patient who arrives in the
emergency department. What does this action exemplify? a. Evaluation. b. Supervision. c. Assignment. d. Delegation. ANS: C
Assignment is the action taken when the patient or interventions are within a nurse’s scope of practice. The experienced nuN rse iR caIpabG taC kingMon this patient assignment. Delegation is U sS N leToBf . O transferring the responsibility to perform a function or intervention to someone who would not otherwise have the authority to perform it (i.e., the function or intervention is not within the scope of practice or scope of employment of the care provider to whom the care is being delegated). This is not an example of evaluation or supervision. DIF: Understand REF: 167 OBJ: Discuss principles for the appropriate delegation of patient care activities. TOP: Planning MSC: NCLEX: Safe and Effective Care Environment 7. The nursing management team in a facility is investigating a nursing care delivery model that
involves staff members working under the direction of an RN leader. What does this model reflect? a. Team nursing. b. Primary nursing. c. Functional nursing. d. Total patient care. ANS: A
Canadian Fundamentals of Nursing 6th Edition Potter Test Bank In team nursing, an RN leads a team that is composed of other RNs, registered psychiatric nurses or licensed practical nurses, and UCPs. The team members provide direct patient care to groups of patients, under the direction of the RN team leader. Primary nursing is a model of care delivery whereby an RN assumes responsibility for a caseload of patients over time. Typically, the RN selects the patients for his or her caseload and cares for those patients during their hospitalization or stay in the health care setting. Functional nursing is task focused, not patient focused; in a functional nursing model, tasks are divided, with each nurse assuming responsibility for specific tasks. In a total patient care model, an RN is responsible for all aspects of care for one or more patients. The RN may delegate aspects of care but retains accountability for care of all assigned patients. DIF: Understand REF: 162 OBJ: Describe the relationships between nursing leadership and healthy practice environments, patient safety, and quality patient care outcomes. TOP: Implementation MSC: NCLEX: Safe and Effective Care Environment 8. Accountability is a critical aspect of nursing care. Which of the following is an example of a
specific decision-making process of accountability? a. Selecting the medication schedule for the patient. b. Implementing discharge teaching plans that meet individual patients’ needs. c. Evaluating the patient’s outcomes after implementation of care. d. Promoting participation of all staff members in unit meetings. ANS: C
Accountability refers to being answerable for actions. It involves follow-up and a reflective analysis of a nurse’s decisions to evaluate their effectiveness. Selecting the medication schedule for the patient is an example of taking responsibility. Implementing discharge Ndua RS I ient GTs’B.C M teaching plans that meet indiviU l patN needO s is an example of autonomy. Promoting participation of all staff members in unit meetings is an example both of decentralized management and of promoting authority. DIF: Understand REF: 165 OBJ: Describe entry-level professional nurse competencies related to leadership, management, and collaborative practice. TOP: Evaluate MSC: NCLEX: Safe and Effective Care Environment 9. The student nurse is seeking to learn skills associated with priority setting. Among the
different priorities of care, which of the following is an example of an intermediate priority? a. An obstructed airway. b. The need to urinate. c. Loss of consciousness. d. Activities of daily living in the home environment. ANS: B
The need to urinate is an intermediate priority because it is a nonemergency, non–life-threatening actual or potential need. An obstructed airway is of high priority because it is an immediate threat to a patient’s survival or safety. Loss of consciousness is of high priority because it is an immediate threat to a patient’s survival or safety. Activities of daily living in the home environment are a fourth-order priority; they represent an actual or potential problem with which the patient or family members may need help in the future.
Canadian Fundamentals of Nursing 6th Edition Potter Test Bank DIF: Apply REF: 166 OBJ: Discuss ways to apply skills of clinical care coordination in nursing practice. TOP: Assessment MSC: NCLEX: Safe and Effective Care Environment 10. The nurse on the unit is determining which activities may be delegated to an unregulated care
provider (UCP). A number of factors are included in the nurse’s decision. Assuming that the UCP is competent, the RN may safely delegate which of the following activities? a. Ambulation of stable patients. b. Documenting an admission history for a new patient. c. Performance of the initial transfer of a postoperative patient. d. Administering medications prepared by the RN. ANS: A
An institution’s policies and procedures and job description for a UCP provide specific guidelines with regard to which tasks or activities can be delegated. The nurse should match tasks to the delegate’s skills, such as ambulating stable patients. An admission history for a new patient should not be prepared by a UCP; the RN should perform this task. The initial transfer of a postoperative patient should not be delegated to a UCP, as the patient would be considered unstable; the RN should perform this task. The nurse should not delegate medication administration to anyone, if the nurse prepared it; the UCP is not licensed to administer medication. DIF: Apply REF: 166| 167 OBJ: Discuss principles for the appropriate delegation of patient care activities. TOP: Implementation MSC: NCLEX: Safe and Effective Care Environment 11. UCPs are allowed to perform tasks with patients in which situation? a. When tasks are delegatedNbU yR anSoI thN erGUTCBP. . COM b. When asked by the patient. c. With another UCP. d. When tasks are delegated by an RN. ANS: D
An institution’s policies, procedures, and job descriptions for UCPs provide specific guidelines regarding which tasks or activities can be delegated. UCPs are not allowed to perform actions authorized for RNs unless these tasks have been properly delegated by an RN, and they can perform such actions only if they are within the UCP’s job description and employer policy. It is not appropriate to perform tasks that are delegated by another UCP, when asked by the patient, or with another UCP. DIF: Understand REF: 166| 167 OBJ: Discuss principles for the appropriate delegation of patient care activities. TOP: Implementation MSC: NCLEX: Safe and Effective Care Environment 12. One type of nursing model is an emerging model among intraprofessional nursing teams and
other health care providers who are members of the interprofessional team. What is this model called? a. Primary nursing model. b. Functional nursing model. c. Team nursing model. d. Collaborative practice model.
Canadian Fundamentals of Nursing 6th Edition Potter Test Bank ANS: D
The Registered Nurses Association of Ontario (2016) has developed evidence-informed Best Practice Guidelines for collaborative practice among nurses, interprofessional health teams, and health professionals. The other models are not considered emerging models: The primary nursing model was popular in the 1970s and 1980s; the functional nursing model was popular during World War II in response to a nursing shortage; and the team nursing model was developed in response to a nursing shortage after World War II. DIF: Understand REF: 162| 163 OBJ: Discuss how a nurse leader can contribute to collaborative practice and best practices implementation. TOP: Implementation MSC: NCLEX: Safe and Effective Care Environment 13. A management structure in which traditional units are reorganized into business units is called
what? a. Matrix. b. Centralized. c. Decentralized. d. Participatory. ANS: A
In a matrix management structure, traditional units are reorganized into business units. Staff may report to several managers who may be from a variety of professional practice backgrounds. Traditional units are not reorganized into business units in centralized, decentralized, or participatory management structures. DIF: Understand REF: 164 (Table 11-1) OBJ: Discuss how a nurse leaN derUcR anScI ontr toC coll NGibut TBe . OMaborative practice and best practices implementation. TOP: Implementation MSC: NCLEX: Safe and Effective Care Environment 14. Which term characterizes the duties and activities that an individual is employed to perform? a. Autonomy. b. Authority. c. Responsibility. d. Accountability. ANS: C
The duties and activities that an individual is employed to perform reflect responsibility. Autonomy is the freedom of choice and responsibility for choices. Authority is the right to act in areas in which a nurse has been given and accepts responsibility according to legislation, standards, and the code of ethics. Accountability refers to being answerable for one’s actions. DIF: Understand REF: 164| 165 OBJ: Describe entry-level professional nurse competencies related to leadership, management, and collaborative practice. TOP: Implementation MSC: NCLEX: Safe and Effective Care Environment 15. What is a nursing manager’s greatest challenge? a. Delegation. b. Communication. c. Time management.
Canadian Fundamentals of Nursing 6th Edition Potter Test Bank d. Clinical decision making. ANS: B
Communication with staff is one of the nursing manager’s greatest challenges, especially in a large work group in which change is constant. The nursing manager’s greatest challenge is not delegation, time management, or clinical decision making. DIF: Understand REF: 165 OBJ: Discuss how a nurse leader can contribute to collaborative practice and best practices implementation. TOP: Planning MSC: NCLEX: Safe and Effective Care Environment MULTIPLE RESPONSE 1. Creating a culture of patient safety includes which of the following? (Select all that apply.) a. Awareness of health care error. b. Ensuring feedback on safety issues. c. Commitment to a just, punitive environment. d. Promotion of a systems approach to care. e. Reporting of adverse events. ANS: A, B, D, E
Awareness of health care error, ensuring feedback on safety issues, promotion of a systems approach to care, and reporting of adverse events are characteristics of a culture of patient safety. Commitment to a just, nonpunitive (rather than punitive) culture is advocated in this culture. DIF: Understand REF: 168 OBJ: Describe the purpose, elN emUeR ntS s, I anN dG mT odB el. sC foO r qM uality practice environments and patient safety. TOP: Planning MSC: NCLEX: Safe and Effective Care Environment 2. A nurse has delegated a task to an unregulated care provider (UCP). Which of the following is
the nurse responsible for? (Select all that apply.) a. Performing the task safely. b. Maintaining accountability of the task. c. Ensuring the task is within the UCP’s job description. d. Supervising the UCP if required. e. Providing clear instructions when delegating a task. ANS: B, C, D, E
When delegating a task to a UCP, the nurse is responsible for maintaining accountability for the task, ensuring that the task is within the UCP’s job description, supervising the UCP if required by policy, and providing clear instructions about the task. The UCP is responsible for performing the task safely. DIF: Understand REF: 166 OBJ: Discuss principles for the appropriate delegation of patient care activities. TOP: Planning MSC: NCLEX: Safe and Effective Care Environment
Canadian Fundamentals of Nursing 6th Edition Potter Test Bank
Chapter 12: Critical Thinking in Nursing Practice Potter et al: Canadian Fundamentals of Nursing, 6th Edition MULTIPLE CHOICE 1. Critical thinking characteristics include which of the following? a. Considering what is important in a given situation. b. Accepting one, established way to provide patient care. c. Making decisions based on intuition. d. Being able to read and follow prescriber orders. ANS: A
Critical thinking involves being able to decipher what is relevant and important in a given situation and to make a clinical decision on the basis of that importance. Patient care can be provided in many ways, not just one. Clinical decisions should be based on evidence and research. Following prescriber orders is not considered a critical thinking skill. DIF: Remember TOP: Assessment
REF: 174| 175 OBJ: Describe characteristics of a critical thinker. MSC: NCLEX: Safe and Effective Care Environment
2. Which of these patient scenarios is most indicative of critical thinking? a. Administering pain relief medication according to what was given last shift. b. Asking a patient what pain relief methods, pharmacological and
nonpharmacological, have worked in the past. c. Offering pain relief medication based on prescriber orders. d. Explaining to the patient that his reports of severe pain are not consistent with the
N R I G B.COM
minor procedure that was pU erfoSrmeNd. T
ANS: B
Asking the patient what pain relief methods have worked in the past is an example of exploring many options for pain relief. Administering medication on the basis of a previous assessment is not practising according to standards of care. The nurse is to assess a patient during each shift and intervene accordingly. Nonpharmacological pain relief methods are available, as are medications for pain. Pain is subjective; the nurse should offer pain relief methods on the basis of the patient’s reports without being judgemental. DIF: Apply TOP: Assessment
REF: 174| 175 OBJ: Describe characteristics of a critical thinker. MSC: NCLEX: Safe and Effective Care Environment
3. Professional nurses are responsible for making clinical decisions in order to do what? a. Prove traditional methods of providing nursing care to patients. b. Take immediate action when a patient’s condition worsens. c. Apply clear textbook solutions to patients’ problems. d. Formulate standardized care plans for groups of patients. ANS: B
Professional nurses are responsible for making clinical decisions to take immediate action when a patient’s condition worsens. Patient care should be based on evidence-informed practice, not on tradition. Clear textbook solutions to patient problems are not always available. Care plans should be individualized.
Canadian Fundamentals of Nursing 6th Edition Potter Test Bank
DIF: Understand REF: 176| 177| 180 OBJ: Discuss the nurse’s responsibility in making clinical decisions. TOP: Implementation MSC: NCLEX: Safe and Effective Care Environment 4. The nurse needs a reminder of professional responsibility when performing which of these
actions? a. Making an informed clinical decision. b. Making an ethical clinical decision. c. Making a clinical decision in the patient’s best interest. d. Making a clinical decision based on previous shift assessments. ANS: D
The professional nurse is responsible for assessing patients each shift. Making informed, ethical decisions in the patient’s best interest is practising responsibly. DIF: Understand REF: 180| 181 OBJ: Discuss the nurse’s responsibility in making clinical decisions. TOP: Evaluate MSC: NCLEX: Safe and Effective Care Environment 5. Which of the following demonstrates a nurse utilizing self-reflection to improve clinical
decision making? a. Using an objective approach in all situations. b. Obtaining data in an orderly manner. c. Improving a plan of care while thinking back on interventions performed. d. Providing evidence-informed explanations for all nursing interventions. ANS: C
NURSINGTB.COM
In self-reflection, the nurse utilizes critical thinking in evaluating the effectiveness of interventions and how they were performed. The other options do represent good nursing practice but are not the best examples of self-reflection. Using an objective approach and obtaining data in an orderly manner does not involve purposefully thinking back to discover the meaning or purpose of a situation. Providing evidence-informed explanations for nursing interventions does not always involve thinking back to discover the meaning of a situation. DIF: Understand REF: 182| 183 OBJ: Discuss critical thinking competencies used in nursing practice. TOP: Evaluate MSC: NCLEX: Safe and Effective Care Environment 6. A nursing instructor needs to evaluate students’ abilities to synthesize data and identify
relationships between nursing diagnoses. Which learning assignment is best suited for this instructor’s needs? a. Concept mapping. b. Reflective journalling. c. Reading assignment with a written summary. d. Lecture and discussion. ANS: A
Concept maps challenge the student to synthesize data and identify relationships between nursing diagnoses. Reflective journalling involves thinking back to clarify concepts. Reading assignments and lecture do not best provide an instructor the ability to evaluate students’ abilities to synthesize data.
Canadian Fundamentals of Nursing 6th Edition Potter Test Bank
DIF: Understand REF: 182| 183 OBJ: Discuss how concept maps can improve a nurse's ability to think critically. TOP: Implementation MSC: NCLEX: Safe and Effective Care Environment 7. What is the first component of the critical thinking model for clinical decision making? a. Experience. b. Nursing process. c. Attitude. d. A scientific knowledge base. ANS: D
A scientific knowledge base is the first component of clinical decision making. After acquiring a sound knowledge base, the nurse can then apply the knowledge to different clinical situations and use the nursing process to gain valuable experience. A critical thinking attitude is a guideline for how to approach a problem and apply knowledge to make a clinical decision. DIF: Remember REF: 177 (Box 12-2) OBJ: Describe the components of a critical thinking model for clinical decision making. TOP: Implementation MSC: NCLEX: Safe and Effective Care Environment 8. A new graduate nurse will make the best clinical decisions by applying the components of the
nursing critical thinking model and which of the following? a. Drawing on past clinical experiences to formulate standardized care plans. b. Relying on recall of information from past lectures and textbooks. c. Depending on the charge nurse to determine priorities of care. d. Using the nursing procesN s. URSINGTB.COM ANS: D
Using the nursing process along with applying components of the nursing critical thinking model will help the new graduate nurse make the most appropriate clinical decisions. Care plans should be individualized, and recalling facts is not a use of critical thinking skills to make clinical decisions. The new nurse should not rely on the charge nurse to determine priorities of care. DIF: Apply REF: 181 OBJ: Describe the components of a critical thinking model for clinical decision making. TOP: Implementation MSC: NCLEX: Safe and Effective Care Environment 9. The critical thinking skill of evaluation in nursing practice can be best described as which of
the following? a. Examining the meaning of data. b. Reviewing the effectiveness of nursing actions. c. Supporting findings and conclusions. d. Searching for links between data and the nurse’s assumptions. ANS: B
Reviewing the effectiveness of interventions best describes evaluation. Examining the meaning of data is a form of inference. Supporting findings and conclusions is a way of providing explanations. Searching for links between the data and the nurse’s assumptions is a type of analysis.
Canadian Fundamentals of Nursing 6th Edition Potter Test Bank
DIF: Understand REF: 176 (Table 12-1)| 182 OBJ: Discuss critical thinking competencies used in nursing practice. TOP: Evaluate MSC: NCLEX: Safe and Effective Care Environment 10. The patient appears to be in no apparent distress, but vital signs measured by an unregulated
care provider reveal that the pulse is extremely low. The nurse then auscultates an apical pulse and asks the patient whether he has any complaints or a history of heart problems. The nurse is utilizing which critical thinking skill? a. Interpretation. b. Evaluation. c. Self-regulation. d. Explanation. ANS: A
Interpretation involves collecting data in an orderly manner, looking for patterns to categorize data, and clarifying uncertain data. This nurse is clarifying the data in this situation. Evaluation involves determining the effectiveness of interventions. The nurse in this scenario is assessing the patient, not evaluating interventions. Self-regulation involves reflecting on experiences. Explanation entails supporting findings and conclusions. The nurse in this question is clarifying uncertain data (determining cause of the low pulse), not supporting the finding of a low pulse. DIF: Apply REF: 176 (Table 12-1) OBJ: Discuss critical thinking competencies used in nursing practice. TOP: Assessment MSC: NCLEX: Safe and Effective Care Environment 11. A patient continues to reportN poUsR tsS urI giN caG lT inB ci. sioCnOpM ain at a level of 9 out of 10 after pain
medicine is given. The next dose of pain medicine is not due for another hour. What should the critically thinking nurse do first? a. Explain to the patient that nothing else has been ordered. b. Explore other options for pain relief. c. Offer to notify the health care provider after morning rounds are completed. d. Discuss the surgical procedure and reason for the pain. ANS: B
The critically thinking nurse should explore all options for pain relief first. The nurse should use critical thinking to determine the cause of the pain and determine various options for pain, in addition to the ordered pain medications. The nurse can act independently to determine all options for pain relief and does not have to wait until after the health care provider’s rounds are completed. Explaining the cause of the pain does not address options for pain relief. DIF: Apply REF: 177 OBJ: Discuss critical thinking competencies used in nursing practice. TOP: Implementation MSC: NCLEX: Safe and Effective Care Environment 12. The nursing student can best develop critical thinking skills by doing which of the following? a. Studying 3 hours more each night. b. Actively participating in all clinical experiences. c. Interviewing staff nurses about their nursing experiences. d. Attending all open skills laboratory opportunities.
Canadian Fundamentals of Nursing 6th Edition Potter Test Bank ANS: B
Nursing is an applied science, and to apply knowledge learned and to develop critical thinking skills to make clinical decisions, the student should value and actively participate in all clinical experiences. Studying for longer hours, interviewing nurses, and attending skills laboratory sessions do not provide opportunities for clinical decision making, as do actual clinical experiences. DIF: Understand REF: 178 OBJ: Explain the relationship between clinical experience and critical thinking. TOP: Implementation MSC: NCLEX: Safe and Effective Care Environment 13. While caring for a hospitalized older woman after hip surgery, the new graduate nurse is faced
with the task of inserting an indwelling urinary catheter, which involves rotating the hip into a contraindicated position. The nurse exhibits critical thinking to perform this task by doing what? a. Following textbook procedure. b. Notifying the physician of the need for a urologist consult. c. Adapting the positioning technique to the situation. d. Postponing catheter insertion until the next shift. ANS: C
The nurse must use critical thinking skills and problem solving in this situation to adapt positioning technique. In practice, patient procedures are not always presented as in a textbook, but they are individualized. A urology consult is not warranted for position, but it may be warranted if the nurse encounters difficulty in insertion. Postponing insertion of the catheter is not an appropriate action. DIF: Apply REF: 17N 9URSINGTB.COM OBJ: Explain the relationship between clinical experience and critical thinking. TOP: Implementation MSC: NCLEX: Safe and Effective Care Environment 14. The nurse enters a room to find the patient sitting up in bed crying. How would the nurse
display a critical thinking attitude in this situation? a. Tell the patient she’ll be back in 30 minutes. b. Set a box of tissues at the patient’s bedside before leaving the room. c. Ask the patient why she is crying. d. Limit visitors while the patient is upset. ANS: C
The nurse should try to find out why the patient is crying. By using interpretation skills, the nurse can collect information to be able to make appropriate interventions. Telling the patient that she will return, providing tissues, and limiting visitors may be appropriate actions but do not address the reason why the patient is crying. DIF: Apply REF: 176 OBJ: Discuss critical thinking competencies used in clinical decision making. TOP: Assessment MSC: NCLEX: Safe and Effective Care Environment 15. Which of the following is a strategy used by learners to explore complex problems and engage
in decision making without the risk of harming a patient? a. Lectures and reading.
Canadian Fundamentals of Nursing 6th Edition Potter Test Bank b. Concept mapping. c. Reflective writing. d. Case-based learning. ANS: D
Case-based learning is a strategy that allows learners to explore complex problems and engage in decision making without the risk of harming a patient. Lectures and readings do not always engage the learner in decision making. Concept mapping is a visual representation of patient problems and interventions that depicts their relationship to one another. Reflective writing is used when a learner explains a clinical experience and explores their understanding and perception of the situation. DIF: Understand REF: 182 OBJ: Describe how case-based learning fosters critical thinking. TOP: Implementation MSC: NCLEX: Safe and Effective Care Environment 16. How do professional standards influence a nurse’s clinical decisions? a. They bypass the patient’s feelings to promote ethical standards. b. They set minimal passing standards for testing. c. They require the nurse to use critical thinking for the highest level of quality
nursing care. d. They require the nurse to utilize evidence-informed practice that is based on
nurses’ needs. ANS: C
Upholding professional standards requires nurses to use critical thinking for the highest level of quality nursing care. Bypassing the patient’s feelings is not practising according to professional standards. The pN rim pose fessional standards is not the establishment URarySIpur NG TBo.fCpro OM of minimal passing standards for testing. Patient care should be based on patients’ needs, not on nurses’ needs. DIF: Remember REF: 181 OBJ: Explain how professional standards influence a nurse’s clinical decisions. TOP: Planning MSC: NCLEX: Safe and Effective Care Environment 17. A nurse who is caring for a patient with a pressure ulcer fails to apply the recommended
dressing according to hospital policy. If the patient is harmed, the nurse could be subject to legal action for not adhering to which of the following? a. Fairness. b. Intellectual standards. c. Independent reasoning. d. Institutional practice guidelines. ANS: D
Institutional practice guidelines are established standards and policies that can be used in court to make judgements about nursing actions. Intellectual standards are guidelines or principles for rational thought. Fairness and independent reasoning are two examples of critical thinking attitudes that are designed to help nurses make clinical decisions. DIF: Apply REF: 181 OBJ: Explain how professional standards influence a nurse’s clinical decisions. TOP: Implementation MSC: NCLEX: Safe and Effective Care Environment
Canadian Fundamentals of Nursing 6th Edition Potter Test Bank
18. Which of these findings, if identified in a plan of care, should the registered nurse revise
because it is not characteristic of critical thinking and the nursing process? a. Patient’s reactions to diagnostic testing. b. Nurse’s assumptions about hospital discharge. c. Identification of five different nursing diagnoses. d. Documentation of patient’s ability to cope with loss. ANS: B
The nurse should not assume when a patient is going to be discharged and document this information in a plan of care. Making assumptions is not an example of a critical thinking skill. The patient’s reactions to testing, having several nursing diagnoses, and a description of the patient’s coping abilities are all appropriate to document in the nursing plan of care. DIF: Analyze REF: 174-176 OBJ: Discuss the relationship of the nursing process to critical thinking. TOP: Evaluate MSC: NCLEX: Safe and Effective Care Environment MULTIPLE RESPONSE 1. The nursing process involves which of the following steps in the clinical decision-making
process? (Select all that apply.) a. Identifying patient needs. b. Diagnosing the disease process. c. Determining priorities of care. d. Setting goals. e. Performing nursing intervNentR U ions SI. NGTB.COM f. Evaluating effectiveness of medical treatments. ANS: A, C, D, E
Diagnosing disease is not a nursing action. Nurses are to use the nursing process to evaluate the effectiveness of nursing interventions, not medical treatments. Identifying patient needs, determining priorities of care, setting realistic goals, and implementing nursing interventions are all steps in the clinical decision-making process. DIF: Understand REF: 181 OBJ: Discuss the relationship of the nursing process to critical thinking. TOP: Implementation MSC: NCLEX: Safe and Effective Care Environment 2. Which of the following demonstrates appropriate clinical decision making for groups of
patients? (Select all that apply.) a. Identifying the nursing diagnosis of each patient. b. Prioritizing care based on urgency and complexity of problems. c. Delaying family-centred care until you have more time. d. Combining activities to resolve more than one patient problem at a time. e. Delegating basic cares to unregulated care providers. ANS: A, B, D, E
Canadian Fundamentals of Nursing 6th Edition Potter Test Bank Clinical decision making for groups of patients should begin with identifying the nursing diagnosis of each patient in order to prioritize care. Combining activities can be an effective use of time, if used appropriately. Basic care activities can be delegated to unregulated care providers so that the nurse can spend time on activities requiring professional nursing knowledge. Patient and family-centred care should not be delayed; patients and families should be involved in decision making and participants in care at all times. DIF: Understand REF: 181 (Box 12-3) OBJ: Discuss the nurse’s responsibility in making clinical decisions. TOP: Implementation MSC: NCLEX: Safe and Effective Care Environment
NURSINGTB.COM
Canadian Fundamentals of Nursing 6th Edition Potter Test Bank
Chapter 13: Nursing Assessment, Diagnosis, and Planning Potter et al: Canadian Fundamentals of Nursing, 6th Edition MULTIPLE CHOICE 1. After completing a thorough database review and analyzing the data to identify any problems,
the nurse should proceed to what step of the nursing process? a. Assessment. b. Planning. c. Implementation. d. Evaluation. ANS: B
In the five-step nursing process, the nurse should establish mutual goals with the patient and prioritize care in the planning phase, which follows the diagnosis phase. The assessment phase of the nursing process involves gathering data. The implementation phase involves carrying out appropriate nursing interventions. During the evaluation phase, the nurse assesses the effectiveness of interventions. DIF: Understand REF: 188 OBJ: Identify and discuss the steps of nursing assessment. MSC: NCLEX: Safe and Effective Care Environment
TOP: Planning
2. A patient’s plan of care includes the goal of increasing mobility this shift. As the patient is
ambulating to the bathroom at the beginning of the shift, the patient suffers a fall. How should the nurse first revise the plan of care? G B.C M R the INpatiTent becO a. Asking physiotherapy to N asU sistS ause of the new injuries. b. Disregarding all previous diagnoses and establishing a new plan of care. c. Reassessing the patient. d. Setting new priorities for the patient. ANS: C
The nurse needs to reassess the patient after any type of change in health status. The nursing process is dynamic and ongoing. Asking physiotherapy to assist the patient is premature before the patient is reassessed and before prescriber orders have been made. The nurse may not need to disregard all previous diagnoses. Some diagnoses may still apply, but the patient needs to be reassessed first. Setting new priorities is not recommended before assessment and establishing diagnoses. DIF: Apply TOP: Planning
REF: 188 OBJ: Discuss criteria used in priority setting. MSC: NCLEX: Safe and Effective Care Environment
3. When planning patient care, a goal can be described as which of the following? a. A statement describing the patient’s accomplishments without a time restriction. b. A realistic statement predicting any negative responses to treatments. c. A broad statement describing a desired change in patient behaviour. d. An identified long-term nursing diagnosis. ANS: C
Canadian Fundamentals of Nursing 6th Edition Potter Test Bank A goal is a broad statement that describes a desired change in a patient’s condition or behaviour. A goal is mutually set with the patient and is time-limited, patient-centred, measurable, and realistic. DIF: Remember REF: 204| 205 OBJ: Describe goal setting and discuss the difference between a goal and an expected outcome. TOP: Planning MSC: NCLEX: Safe and Effective Care Environment 4. When evaluating a plan of care, the nurse reviews the goals for the patient. Which goal
statement is realistic for a patient with a pelvic fracture on bed rest? a. The patient will increase mobility by ambulating in the hallway two times this shift. b. The patient will increase mobility by turning side to back to side with assistance every 2 hours. c. The patient will increase mobility by using the walker correctly to ambulate to the bathroom as needed. d. The patient will increase mobility by using a sliding board correctly to transfer to the bedside commode as needed. ANS: B
The patient is ordered to be on bed rest; therefore, turning the patient in bed is the only option that is appropriate. When determining goals, the nurse needs to ensure that the goal is individualized and realistic for the patient. Goals must reflect the highest possible level of wellness and independence in function, which for this patient would be turning in bed. DIF: Evaluate REF: 205 OBJ: Describe goal setting and discuss the difference between a goal and an expected outcome. TOP: Planning MSC: NN CLUER XS : SIaN feG anTdB E. ffeCcO tivM e Care Environment 5. The following statements are part of a patient’s nursing care plan. Which of the following
statements represents an expected outcome? a. The patient will verbalize a decreased pain level less than 3 on a 0-to-10 scale by the end of this shift. b. The patient will demonstrate increased mobility in 2 days. c. The patient will demonstrate increased tolerance of activity over the next month. d. The patient will understand needed dietary changes by discharge. ANS: A
An expected outcome is a specific and measurable change that is expected as a result of nursing care. The other three options in this question are goals. Demonstrating increased mobility in 2 days and understanding necessary dietary changes by discharge are short-term goals because they are expected to occur in less than a week. Demonstrating increased tolerance of activity over a month-long period is a long-term goal because it is expected to occur over a longer period of time. DIF: Understand REF: 205| 206 OBJ: Describe goal setting and discuss the difference between a goal and an expected outcome. TOP: Planning MSC: NCLEX: Safe and Effective Care Environment 6. Which patient outcome statement includes all seven guidelines for writing goal and outcome
statements?
Canadian Fundamentals of Nursing 6th Edition Potter Test Bank a. The patient will ambulate in hallways. b. The nurse will administer pain medication every 4 hours to keep the patient free
from discomfort. c. The nurse will monitor the patient’s heart rhythm continuously this shift. d. The patient will feed self at all mealtimes today without complaints of shortness of
breath. ANS: D
A goal or outcome statement should be patient-centred; should address one patient response; should be observable, measurable, and time-limited; should be mutually set by nurse and patient; and should be realistic. The statement “The patient will feed self at all mealtimes today without complaints of shortness of breath” includes all seven criteria for goal writing. “The patient will ambulate in hallways” is missing a time limit. Administering pain medication and monitoring the patient’s heart rhythm are nursing interventions; they are not patient behaviours or actions. DIF: Understand REF: 206 OBJ: Describe goal setting and discuss the difference between a goal and an expected outcome. TOP: Planning MSC: NCLEX: Safe and Effective Care Environment 7. A nursing assessment for a patient with a spinal cord injury reveals several pertinent problems
that a nurse can treat. While the plan of care is developed, which nursing diagnosis is the highest priority for this patient? a. Risk for impaired skin integrity. b. Risk for infection. c. Spiritual distress. d. Reflex urinary incontinence. I G B.C M ANS: D
N R U S N T
O
Reflex urinary incontinence is highest priority. If a patient’s incontinence is not addressed, then the patient is at higher risk for impaired skin integrity and infection. Remember that the Risk for diagnoses are potential problems. They may be prioritized higher in some cases, but not in this situation. Spiritual distress is an actual diagnosis, but the adverse effects that could result from not assisting the patient with urinary elimination take priority in this case. Physiological problems do not always take priority, but urinary incontinence could cause the greatest harm if it is not addressed. DIF: Apply TOP: Planning
REF: 203 OBJ: Develop a plan of care from a nursing assessment. MSC: NCLEX: Safe and Effective Care Environment
8. The nurse is caring for seven patients this shift. After completing their assessments, the nurse
states that he doesn’t know where to begin in developing care plans for these patients. Which of the following is an appropriate suggestion by another nurse? a. “Choose all the interventions and perform them in order of time needed for each one.” b. “Make sure you identify the scientific rationale for each intervention first.” c. “Decide on goals and outcomes you have chosen for the patients.” d. “Begin with the highest priority diagnoses, and then select appropriate interventions.” ANS: D
Canadian Fundamentals of Nursing 6th Edition Potter Test Bank When developing a plan of care, the nurse needs to rank the nursing diagnoses in order of priority and then select appropriate interventions. Choosing all the interventions should take place after the diagnoses are ranked, and interventions should be prioritized by patient needs, not just by time. The chosen interventions should be evidence informed with scientific rationales, but the diagnoses need to be prioritized first in order to prioritize interventions. Goals for a patient should be mutually set, not chosen just by the nurse. DIF: Apply TOP: Planning
REF: 203| 204 OBJ: Develop a plan of care from a nursing assessment. MSC: NCLEX: Safe and Effective Care Environment
9. A patient’s son decides to stay at the bedside while his father is confused. When developing
the plan of care for this patient, what should the nurse do? a. Individualize the care plan only according to the patient’s needs. b. Request that the son leave at bedtime, so the patient can rest. c. Suggest that a female member of the family stay with the patient. d. Involve the son in the plan of care as much as possible. ANS: D
Family should be included in a patient’s plan of care as much as possible. The family can help patients meet health care goals. Meeting some of the family’s need as well as the patient’s needs may improve the patient’s level of wellness. The son should not be asked to leave if at all possible. In some situations, it may be best that family members not remain in the room, but this question stem does not indicate that this is the case in this situation. Suggesting that a female member of the family stay is not justified without a legitimate reason, and no reason is given in this question stem for such a suggestion. DIF: Apply TOP: Planning 10.
REF: 203 OBJ: De velop a plan of care from a nursing assessment. N R I G . C MSC: NCLUEXS : SaN fe anTdB Effec tive M Care Environment O
Which of these outcomes would be most appropriate for a patient with a nursing diagnosis of Constipation related to slowed gastrointestinal motility secondary to pain medications? a. Patient will have one soft, formed bowel movement by end of shift. b. Patient will not take any pain medications this shift. c. Patient will walk unassisted to bathroom by the end of shift. d. Patient will not take laxatives or stool softeners this shift. ANS: A
The identified problem, or nursing diagnosis, is constipation. Therefore, the outcome should be that the constipation is relieved. To measure constipation relief, the nurse will be observing for the patient to have a bowel movement. Not taking pain medications may or may not relieve the constipation. Although not taking pain medicines might be an intervention, the patient should not have to be in pain to relieve constipation. Other measures, such as administering laxatives or stool softeners, might be appropriate interventions, but they are not outcomes. The patient walking unassisted to the bathroom addresses mobility, not constipation. The patient may need to walk to the bathroom to have a bowel movement, but the appropriate outcome for constipation is that the constipation is relieved, as evidenced by a bowel movement—something that the nurse can observe. DIF: Apply TOP: Planning
REF: 204-206 OBJ: Develop a plan of care from a nursing assessment. MSC: NCLEX: Safe and Effective Care Environment
Canadian Fundamentals of Nursing 6th Edition Potter Test Bank 11. The nurse is working with a patient who is being prepared for a diagnostic test this afternoon.
The patient tells the nurse that she wants to have her hair shampooed. Which of the following is the most appropriate label with regard to assigning a priority for the patient’s request? a. Low priority. b. An unmet need. c. Intermediate priority. d. A safety and security need. ANS: A
The patient’s request would be of low priority because it is not directly related to a specific illness or prognosis. The patient’s request does not represent “an unmet need” and is not an intermediate priority. An intermediate priority is one that involves the nonemergency, non–life-threatening needs of the patient. The patient’s request does not reflect a safety and security need; the outcome does not threaten her well-being. DIF: Analyze TOP: Planning
REF: 203 OBJ: Discuss criteria used in priority setting. MSC: NCLEX: Safe and Effective Care Environment
12. Which of the following options correctly explains what the nurse should do with the plan of
care for a patient after it is developed? a. Place the original copy in the chart, so it cannot be tampered with or revised. b. Communicate the plan of care to all health care providers involved in the patient’s care. c. Send the plan of care to the administration office to be filed. d. Send the plan of care to quality assurance for review. ANS: B
Bi. The patient’s nursing plan ofNcU arR e is dyGnT am cC piO ecM e of work that needs to be updated and SIa N revised as the patient’s condition changes. All health care providers involved in the patient’s care need to be informed of the plan of care. The plan of care is not placed on the chart and never looked at again. The plan of care is not sent to the administrative office or quality assurance office. DIF: Understand TOP: Planning
REF: 206| 207 OBJ: Develop a plan of care from a nursing assessment. MSC: NCLEX: Safe and Effective Care Environment
13. What is the purpose and distinction of using a concept map when a plan of care is
implemented? a. Quality assurance in the health care facility. b. Multidisciplinary communication. c. Provision of a standardized format for patient problems. d. Identification of the relation of patient problems and interventions. ANS: D
A concept map is a diagram of patient problems and interventions that shows their relations to one another. The use of a concept map promotes critical thinking and helps the nurse organize complex patient data, process complex relationships, and achieve a holistic view of the patient’s situation. The purpose is not quality assurance in the health care facility. Multidisciplinary communication is enhanced with the use of critical pathways, not concept maps. Standardized or computerized care plans, not concept maps, provide a standardized format for patient problems. A concept map is highly individualized.
Canadian Fundamentals of Nursing 6th Edition Potter Test Bank
DIF: Apply REF: 195| 196 OBJ: Explain the relationship of critical thinking to assessment, diagnosis, and planning. TOP: Planning MSC: NCLEX: Safe and Effective Care Environment 14. The use of critical thinking skills during the assessment phase of the nursing process ensures
that the nurse a. Completes a comprehensive database. b. Identifies pertinent nursing diagnoses. c. Intervenes on the basis of patient goals and priorities of care. d. Determines whether outcomes have been achieved. ANS: A
The assessment phase of the nursing process involves data collection to complete a thorough patient database. Identifying nursing diagnoses occurs during the diagnosis phase. The nurse carries out interventions during the implementation phase and determines whether outcomes have been achieved during the evaluation phase. DIF: Understand REF: 188 OBJ: Explain the relationship of critical thinking to assessment, diagnosis, and planning. TOP: Assessment MSC: NCLEX: Safe and Effective Care Environment 15. Subjective data include which of the following? a. A patient’s feelings, perceptions, and reported symptoms. b. A description of the patient’s behaviour. c. Observations of a patient’s health status. d. Measurements of a patient’s health status. ANS: A
NURSINGTB.COM
Subjective data include the patient’s feelings, perceptions, and reported symptoms. Only patients provide subjective data relevant to their health condition. Data sometimes reflect physiological changes, which the nurse explores further through objective data collection. Describing the patient’s behaviour, observations made, and measurements of a patient’s health status are all examples of objective data. DIF: Remember REF: 191 OBJ: Differentiate between subjective and objective data. MSC: NCLEX: Safe and Effective Care Environment
TOP: Evaluate
16. A patient expresses fear of going home and being alone. Her vital signs are stable and her
incision is nearly completely healed. The nurse can infer from the subjective data that a. The patient can now perform the dressing changes herself. b. The patient can begin retaking all her previous medications. c. The patient is apprehensive about discharge. d. Surgery was not successful. ANS: C
Subjective data include expressions of fear of going home and being alone. These data indicate that the patient is apprehensive about discharge. Expressing fear is not an appropriate sign that a patient is able to perform dressing changes independently. An order from a health care provider is required before a patient is taught to resume previous medications. The nurse cannot infer that surgery was not successful if the incision is nearly completely healed.
Canadian Fundamentals of Nursing 6th Edition Potter Test Bank
DIF: Apply REF: 191 OBJ: Differentiate between subjective and objective data. MSC: NCLEX: Safe and Effective Care Environment
TOP: Assessment
17. Which of the following methods of data collection is utilized to establish a patient’s nursing
database? a. Reviewing the current literature to determine evidence-informed nursing actions. b. Orders for diagnostic and laboratory tests. c. Physical examination. d. Anticipated medications to be ordered. ANS: C
A nursing database includes results of a physical examination. Orders are included in the order section of the patient’s chart. The nurse reviews the current literature in the implementation phase of the nursing process to determine evidence-informed actions, and the health care provider is responsible for ordering medications. Medication orders are usually written after the database is completed. DIF: Remember REF: 188 OBJ: Identify and discuss the steps of nursing assessment. MSC: NCLEX: Safe and Effective Care Environment
TOP: Assessment
18. To gather information about a patient’s home and work surroundings, the nurse will need to
utilize which method of data collection? a. Carefully review laboratory results. b. Conduct the physical assessment before collecting subjective information. c. Document a thorough nuN rsiU nR gS heI alN thGhT isB to. ryC . OM d. Prolong the termination phase of the interview. ANS: C
A thorough nursing history includes information about the patient’s home and work surroundings. Neither laboratory results nor the physical assessment will reveal much about the home and work surroundings. Collecting data is part of the working phase of the interview. DIF: Understand REF: 189 OBJ: Describe the relationship between data collection and data analysis. TOP: Assessment MSC: NCLEX: Safe and Effective Care Environment 19. While interviewing an older female patient of Asian descent, the nurse notices that the patient
looks at the ground when answering questions. What should this nurse do? a. Notify the physician to recommend a psychological evaluation. b. Consider cultural differences during this assessment. c. Ask the patient to make eye contact to determine her affect. d. Continue with the interview and document that the patient is depressed. ANS: B
Older women of Asian descent consider it rude to look an authority figure, such as a health care provider, in the eye. This nurse needs to practise culturally competent care and appreciate the cultural differences. Assuming that the patient is depressed or in need of a psychological evaluation is inappropriate, as is asking the patient to make eye contact.
NURSINGTB.COM
Canadian Fundamentals of Nursing 6th Edition Potter Test Bank
DIF: Apply REF: 193 OBJ: Describe the relationship between data collection and data analysis. TOP: Assessment MSC: NCLEX: Safe and Effective Care Environment 20. After reviewing the interview process and objectives during a patient-centred interview, what
will the nurse do? a. Begin by introducing himself or herself. b. Document a nursing health history. c. Explain that the interview will be over in a few more minutes. d. Tell the patient that he’ll be back to administer medications in 1 hour. ANS: B
After beginning with an introduction and reviewing the interview process and objectives, the nurse should conduct the actual interview and proceed with data collection. The termination phase includes telling the patient when the interview is nearing an end. Telling the patient medications will be given later when the nurse returns would typically take place during the termination phase of the interview. DIF: Understand REF: 193 OBJ: Identify and discuss the steps of nursing assessment. MSC: NCLEX: Safe and Effective Care Environment
TOP: Assessment
21. The nurse is attempting to prompt the patient to elaborate on her complaints of daytime
fatigue. Which question should the nurse ask? a. “Is there anything that you are stressed about right now?” b. “What reasons do you think are contributing to your fatigue?” c. “What are your normal wN oU rkRhS ouIrsN?G ” TB.COM d. “Are you sleeping 8 hours a night?” ANS: B
The question about what factors might be contributing to the patient’s fatigue will elicit the best open-ended response. Asking whether the patient is stressed and asking whether the patient is sleeping 8 hours a night are closed-ended questions, eliciting simple yes or no responses. Asking about normal work hours will elicit a matter-of-fact response and does not prompt the patient to elaborate on her complaints of daytime fatigue nor elicit the contributing reasons. DIF: Apply REF: 193 OBJ: Identify and discuss the steps of nursing assessment. MSC: NCLEX: Safe and Effective Care Environment 22. Components of a nursing health history include a. Current treatment orders. b. Nurse’s concerns. c. Nurse’s goals for the patient. d. Patient expectations. ANS: D
TOP: Assessment
Canadian Fundamentals of Nursing 6th Edition Potter Test Bank Components of a nursing health history include physical examination findings, patient expectations, environmental history, and diagnostic data. Current treatment orders are located under the Orders section in the patient’s chart and are not a part of the nursing health history. Patient concerns, not nurse’s concerns, are included in the database. Goals that are mutually established, not nurse’s goals, are part of the nursing care plan. DIF: Remember TOP: Assessment
REF: 194 OBJ: Describe the components of a nursing history. MSC: NCLEX: Safe and Effective Care Environment
23. While the patient’s lower extremity, which is in a cast, is assessed, the patient tells the nurse
about an inability to rest at night. The nurse disregards this complaint, thinking that no correlation has been noted between having a leg cast and developing restless sleep. What would be a more theoretically sound first approach? a. Document the sleep patterns and complaint in the patient’s chart. b. Tell the patient you are just focused on the leg right now. c. Explain that a more thorough assessment will be needed next shift. d. Ask the patient about his usual sleep patterns and when the difficulty with resting began. ANS: D
The nurse must use critical thinking skills to assess this situation first. The best response is to gather more assessment data by asking the patient about usual sleep patterns and when the difficulty with resting began. The nurse should assess before documenting and should not ignore the patient’s complaints. DIF: Apply REF: 191 OBJ: Explain the relationship of critical thinking to assessment, diagnosis, and planning. TOP: Assessment MSC: NN CLUER XS : SIaN feG anTdB E. ffeCcO tivM e Care Environment 24. A nurse comparing data validation and data interpretation correctly explains the difference
with which statement? a. “Validation involves looking for patterns in professional standards.” b. “Data interpretation involves discovering patterns in professional standards.” c. “Validation involves comparing data with other sources for accuracy.” d. “Data interpretation occurs before data validation.” ANS: C
Validation, by definition, involves comparing data with other sources for accuracy. Data interpretation involves identifying abnormal findings, clarifying information, and identifying patient problems. The nurse should validate data before interpreting the data and making inferences. The nurse is interpreting and validating patient data, not professional standards. DIF: Understand REF: 194| 195 OBJ: Explain the relationship between data interpretation, validation, and clustering. TOP: Assessment MSC: NCLEX: Safe and Effective Care Environment 25. While completing an admission database, the nurse is interviewing a patient who states that he
is allergic to latex. What is the most appropriate first nursing action? a. Leave the room and place the patient in isolation. b. Ask the patient to describe the type of reaction. c. Proceed to the termination phase of the interview.
Canadian Fundamentals of Nursing 6th Edition Potter Test Bank d. Document the latex allergy on the medication administration record. ANS: B
The nurse should further assess and ask the patient to describe the type of reaction. The patient will not need to be placed in isolation; before terminating the interview or documenting the allergy, health care personnel need to be aware of what type of response the patient suffered. DIF: Apply REF: 189-191 OBJ: Identify and discuss the steps of nursing assessment. MSC: NCLEX: Safe and Effective Care Environment
TOP: Assessment
26. A patient verbalizes a low pain level of 2 out of 10 but exhibits extreme facial grimacing
while moving around in bed. What is the nurse’s best action in response to her observation? a. Proceed to the next patient’s room while making rounds. b. Offer a massage because the patient does not want any more pain medicine. c. Administer the pain medication ordered for moderate to severe pain. d. Ask the patient about the facial grimacing with movement. ANS: D
The nurse needs to clarify what she observes with what the patient states. Proceeding to the next room is ignoring this visual cue. The nurse cannot assume the patient does not want pain medicine just because the pain level is rated 2 out of 10. The nurse should not administer medication for moderate to severe pain if it is not necessary. DIF: Apply REF: 189| 191 OBJ: Identify and discuss the steps of nursing assessment. MSC: NCLEX: Safe and EffecN tiveR CarI e EnG viroB nm .eCnt M
U S N T
TOP: Assessment
O
27. The nurse is assessing a patient with a hearing deficit. Where is the best place to conduct this
interview? a. The patient’s room with the door closed. b. The waiting area with the television turned off. c. The patient’s room before administration of pain medication. d. The patient’s room while the occupational therapist is working on leg exercises. ANS: A
Distractions should be eliminated as much as possible when interviewing a patient with a hearing deficit. The best place to conduct this interview is in the patient’s room with the door closed. The waiting area does not provide privacy. Pain can sometimes inhibit someone’s ability to concentrate, so interviewing before pain medication is administered is not advisable. It is best for the patient to be as comfortable as possible during an interview. Assessing a patient while another member of the health care team is working would be distracting and is not the best time for assessment to take place. DIF: Apply REF: 192| 193 OBJ: Discuss the purposes of a client interview and the use of interview techniques in documenting a health history. TOP: Assessment MSC: NCLEX: Safe and Effective Care Environment 28. A nursing student is completing an assessment on an 80-year-old patient who is alert and
oriented. The patient’s daughter is present in the room. Which of the following actions made by the nursing student requires the nursing professor to intervene?
Canadian Fundamentals of Nursing 6th Edition Potter Test Bank a. b. c. d.
The nursing student is making eye contact with the patient. The nursing student is speaking only to the patient’s daughter. The nursing student nods periodically while the patient is speaking. The nursing student leans forward while talking with the patient.
ANS: B
When assessing an older person, nurses need to listen carefully and allow the patient to speak. Positive nonverbal communication, such as making eye contact, nodding, and leaning forward, shows interest in the patient. Gathering data from family members is acceptable, but when a patient is able to interact, nurses need to include information from the patient on to complete the assessment. DIF: Evaluate REF: 192| 193 OBJ: Identify and discuss the steps of nursing assessment. MSC: NCLEX: Safe and Effective Care Environment
TOP: Assessment
MULTIPLE RESPONSE 1. Which of the following are examples of subjective data? (Select all that apply.) a. Patient describing excitement about discharge. b. Patient’s wound appearance. c. Patient’s expression of fear regarding upcoming surgery. d. Patient pacing the floor while awaiting test results. e. Patient’s temperature. ANS: A, C
Subjective data include patient’s feelings, perceptions, and reported symptoms. Expressing Nfear RSisIaNnGexTample B.CO M feelings such as excitement or U of subjective data. Objective data are observations or measurements of a patient’s health status. In this question, the appearance of the wound and the patient’s temperature are objective data. Pacing is an observable patient behaviour and is also considered objective data. DIF: Understand REF: 191 OBJ: Differentiate between subjective and objective data. MSC: NCLEX: Safe and Effective Care Environment
TOP: Assessment
Canadian Fundamentals of Nursing 6th Edition Potter Test Bank
Chapter 14: Implementing and Evaluating Nursing Care Potter et al: Canadian Fundamentals of Nursing, 6th Edition MULTIPLE CHOICE 1. In which step of the nursing process does the nurse determine whether the patient’s condition
has improved and whether the patient has met expected outcomes? a. Assessment. b. Planning. c. Implementation. d. Evaluation. ANS: D
In the five-step nursing process, the evaluation phase is the final step involving conducting evaluative measures to determine whether nursing interventions have been effective and whether the patient has met expected outcomes. Assessment, the first step of the process, includes data collection, validation, sorting, and documentation. Planning, the third step of the process, involves setting priorities, identifying patient goals and outcomes, and prescribing nursing interventions. During implementation, nurses initiate nursing care, which is necessary to help patients achieve their goals. DIF: Remember REF: 222| 223 OBJ: Discuss the relationship between critical thinking and evaluation. TOP: Evaluate MSC: NCLEX: Safe and Effective Care Environment 2. After assembling a thorough database and carrying out nursing interventions based on priority
N R I G B.C M
diagnoses, the nurse proceeds tU o wS hichNsteT p of theOnursing process? a. Assessment. b. Planning. c. Implementation. d. Evaluation. ANS: D
In the five-step nursing process, evaluation is the last step, following assessment, diagnosis, planning, and intervening. Assessment involves gathering information about the patient. Next, nursing diagnoses are determined. During the planning phase, patient outcomes are determined. Implementation involves carrying out appropriate nursing interventions. DIF: Understand REF: 222| 223 OBJ: Discuss the relationship between critical thinking and evaluation. TOP: Evaluate MSC: NCLEX: Safe and Effective Care Environment 3. A nursing student asks her nursing instructor to describe the primary purpose of evaluation.
Which of the following statements made by the nursing instructor is most accurate? a. “During evaluation, you determine whether all nursing interventions were completed.” b. “During evaluation, you determine when to downsize staffing on nursing units.” c. “Nurses use evaluation to determine the effectiveness of nursing care.” d. “Evaluation eliminates unnecessary paperwork and care planning.”
Canadian Fundamentals of Nursing 6th Edition Potter Test Bank ANS: C
The purpose of evaluation is to determine the effectiveness of nursing care. The other options are not true statements. Evaluation entails more than simply determining whether nursing interventions were completed. The evaluation process is not used to determine when to downsize staffing or how to eliminate paperwork and planning. DIF: Understand REF: 222| 223 OBJ: Discuss the relationship between critical thinking and evaluation. TOP: Evaluate MSC: NCLEX: Safe and Effective Care Environment 4. After assessing the patient and identifying the need for headache relief, the nurse administers
acetaminophen (Tylenol) for the patient’s headache. What is the nurse’s next priority action for this patient? a. Eliminate Acute pain from the nursing care plan. b. Direct the unregulated care provider to ask if the patient’s headache is relieved. c. Reassess the patient’s pain level in 30 minutes. d. Revise the plan of care. ANS: C
The nurse’s next priority action for this patient is to evaluate whether the nursing intervention of administering acetaminophen was effective. The nurse does not have enough evaluative data at this point to determine whether the nursing diagnosis of Acute pain needs to be deleted. Assessment is the nurse’s responsibility and is not to be delegated to an unregulated care provider. The nurse does not have enough evaluative data to determine whether the patient’s plan of care needs to be revised. DIF: Apply REF: 225 ReS OBJ: Explain the relationship N beU tw enIgN oaG lsToB f c. arC e,OeM xpected outcomes, and evaluative measures in evaluating nursing care. TOP: Evaluate MSC: NCLEX: Safe and Effective Care Environment 5. A nurse is getting ready to discharge to home a patient who has a nursing diagnosis of
Impaired physical mobility. Before discontinuing the patient’s plan of care, what does the nurse need to do? a. Determine whether the patient has transportation to get home. b. Evaluate whether patient goals and outcomes have been met. c. Establish whether the patient has a follow-up appointment scheduled. d. Ensure that the patient’s prescriptions have been filled. ANS: B
The nurse needs to evaluate whether goals and outcomes have been met before revising, continuing, or discontinuing a plan of care. The patient needs transportation, but that does not address the patient’s mobility status. Whether the patient has a follow-up appointment and ensuring that prescriptions are filled are not immediately relevant to the problem of mobility. DIF: Apply REF: 224| 225 OBJ: Describe how evaluation leads to discontinuation, review, or modification of a plan of care. TOP: Evaluate MSC: NCLEX: Safe and Effective Care Environment 6. The nurse is evaluating whether patient goals and outcomes have been met. Of the following,
which is an expected outcome for a patient with a diagnosis of Impaired physical mobility?
Canadian Fundamentals of Nursing 6th Edition Potter Test Bank a. b. c. d.
The patient is able to ambulate in the hallway with crutches. The patient’s level of mobility will improve. The nurse provides assistance while the patient is walking in the hallways. The patient will deny pain while walking in the hallway.
ANS: A
An outcome is an expected, favourable, and measurable result of nursing care. The patient’s being able to ambulate in the hallway with crutches is an expected outcome of nursing care. Improvement in the patient’s level of mobility is a broader goal. The nurse’s assisting a patient to ambulate is an intervention. The patient’s denial of pain is an expected outcome for Acute pain, not for Impaired physical mobility. DIF: Apply REF: 224| 225 OBJ: Explain the relationship between goals of care, expected outcomes, and evaluative measures in evaluating nursing care. TOP: Evaluate MSC: NCLEX: Safe and Effective Care Environment 7. The nurse is evaluating whether a patient’s turning schedule was effective in preventing the
formation of pressure ulcers. Which finding indicates success of the turning schedule? a. Staff documentation of turning the patient every 2 hours. b. Absence of skin breakdown. c. Presence of redness only on the heels of the patient. d. Patient’s eating 100% of all meals. ANS: B
To determine whether a turning schedule is successful, the nurse needs to assess for the presence of skin breakdown. Redness on any part of the body, including only the patient’s heels, indicates that the turniN ngUsR che dule noCt O suM ccessful. Documentation of interventions I Gwa Bs . S N T does not evaluate whether patient outcomes were met. The patient’s eating 100% of meals does not indicate the effectiveness of a turning schedule. DIF: Evaluate REF: 224| 225 OBJ: Explain the relationship between goals of care, expected outcomes, and evaluative measures in evaluating nursing care. TOP: Evaluate MSC: NCLEX: Safe and Effective Care Environment 8. A new nurse states that she is confused about using evaluative measures when caring for
patients and asks the charge nurse for examples and an explanation. Which of the following is the most accurate response from the charge nurse? a. “Evaluative measures are multiple-page documents used to evaluate nurse performance.” b. “Evaluative measures include assessment data used to determine whether patients have met their expected outcomes and goals.” c. “Evaluative measures are used by quality assurance nurses to determine the progress a nurse is making from novice to expert nurse.” d. “Evaluative measures are objective views of incident reports.” ANS: B
Evaluative measures are used to determine whether patients have met their goals and outcomes. Evaluative measures are not multiple-page documents, and they are used to assess the patient’s status, not the nurse’s performance. Evaluative measures are not used for completing an incident report.
Canadian Fundamentals of Nursing 6th Edition Potter Test Bank
DIF: Understand REF: 224-226 OBJ: Give an example of evaluative measures for determining clients’ progress toward an outcome. TOP: Evaluate MSC: NCLEX: Safe and Effective Care Environment 9. The nurse is caring for a patient who has an open wound. For evaluating the progress of
wound healing, what is the nurse’s priority action? a. Asking the unregulated care providers whether the wound looks better. b. Documenting the progress of wound healing as “better” in the patient’s chart. c. Measuring the wound and observe for redness, swelling, or drainage. d. Leaving the dressing off the wound for easier access and more frequent assessments. ANS: C
The nurse performs evaluative measures, such as completing a wound assessment, to evaluate wound healing. Nurses do not delegate assessment to unregulated care providers. Documenting “better” is subjective and does not objectively describe the wound. Leaving the dressing off for the nurse’s benefit of easier access is not a part of the evaluation process. DIF: Apply REF: 225| 226 OBJ: Give an example of evaluative measures for determining clients’ progress toward an outcome. TOP: Evaluate MSC: NCLEX: Safe and Effective Care Environment 10. The nurse is caring for a patient who has an order to change a dressing twice a day, at 0600
and 1800. At 1400, the nurse notices that the dressing is saturated. What is the nurse’s next action? a. Wait and change the dressing at 1800 as ordered. b. Revise the plan of care anNdUcR haS nI geNtG heTdB re. ssC inO gM now. c. Reassess the dressing and the wound in 1 hour. d. Discontinue the plan of care. ANS: B
Based on evaluative data, the nurse revises, discontinues, or continues a patient’s plan of care. Because the dressing is saturated, the nurse needs to revise the plan of care and change the dressing now. Waiting until 1800 or for another hour is not appropriate because assessment data—that the dressing is saturated—indicate that the dressing needs to be changed now. Data are insufficient to support discontinuing the plan of care. Instead, data at this time indicate the need for revision of the plan of care. DIF: Apply REF: 226-228 OBJ: Describe how evaluation leads to discontinuation, review, or modification of a plan of care. TOP: Evaluate MSC: NCLEX: Safe and Effective Care Environment 11. A goal for a patient with a nursing diagnosis of Ineffective coping is to demonstrate effective
coping skills. Which of these patient behaviours indicates that interventions performed to meet this outcome have been successful? a. Stating that he feels better after talking with his family and friends. b. Continuing to consume several alcoholic beverages a day. c. Disliking the support group meetings. d. Spending most of the day in bed. ANS: A
Canadian Fundamentals of Nursing 6th Edition Potter Test Bank Evaluative data that show signs of effective coping will help the nurse determine whether the patient has met the outcome. Among the options listed, talking to family and friends is the only positive one. The other behaviours indicate unsuccessful progress toward meeting the patient’s goal. DIF: Apply REF: 225-227 OBJ: Give an example of evaluative measures for determining clients’ progress toward an outcome. TOP: Evaluate MSC: NCLEX: Safe and Effective Care Environment 12. A nurse is providing education to a patient about self-administering subcutaneous injections.
Which of these patient statements indicates that the patient understands the instructions? a. “I need to use a needle 1/2 inch (1.3 cm) longer than my thumb.” b. “I will give the medicine deep into my deltoid.” c. “My belly is a good place to give my injection.” d. “I need to throw the syringe and needle into the garbage when I am done giving myself my shot.” ANS: C
The skin is made up of several layers. The outer layer is called the epidermis. The second layer of skin is the dermis. The connective tissue under the dermis is called the subcutaneous tissue. This is where subcutaneous injections are given. The abdomen is a good site for subcutaneous injections because this is an area that has a lot of subcutaneous tissue. Using a needle 1/2 inch (1.3 cm) longer than a person’s thumb is not an evidence-informed method for measuring needle length necessary for subcutaneous injection. The deltoid is a muscle, not a subcutaneous site. Disposing of needles and syringes into a garbage can creates a biomedical hazard and therefore is not appropriate. DIF: Evaluate REF: 22N 6URSINGTB.COM OBJ: Discuss the process of selecting nursing interventions. MSC: NCLEX: Health Promotion and Maintenance
TOP: Evaluate
13. Which of these statements made by a patient who has a nursing diagnosis of Disturbed body
image is the best indicator of the patient’s early acceptance of body image? a. “I just won’t go to the pool this summer.” b. “I’m worried about what those other girls will think of me.” c. “I can’t wear that colour. It makes my hips stick out.” d. “I’ll wear the blue dress. It matches my eyes.” ANS: D
The nurse evaluating interventions for the diagnosis Disturbed body image is assessing for positive comments made by the patient that indicate acceptance of the patient’s looks and body image. The only positive comment made is that the patient is wearing the blue dress to match her eyes. The other comments do not reflect positive changes in body image. DIF: Evaluate REF: 216| 217 (Table 14-2) OBJ: Give an example of evaluative measures for determining clients’ progress toward an outcome. TOP: Evaluate MSC: NCLEX: Health Promotion and Maintenance 14. Which of these options is a patient outcome indicating positive progress toward resolving the
nursing diagnosis of Acute confusion? a. Side rails are up with bed alarm activated.
Canadian Fundamentals of Nursing 6th Edition Potter Test Bank b. Patient denies pain while ambulating with assistance. c. Patient wanders halls at night. d. Patient correctly states names of family members in the room. ANS: D
The outcome for the identified nursing diagnosis Acute confusion would address a decrease in or absence of confusion. One sign of orientation is when a patient responds to questions appropriately. Thus, one possible sign that a patient’s confusion is improving is that a patient can correctly state the names of family members in the room. Keeping the side rails up and using a bed alarm are interventions to promote patient safety and prevent falls. The patient’s denying pain indicates positive progress toward resolving a diagnosis of Acute pain or Chronic pain. The patient’s wandering the halls is a sign of confusion. DIF: Evaluate REF: 216| 217 (Table 14-2) OBJ: Give an example of evaluative measures for determining clients’ progress toward an outcome. TOP: Evaluate MSC: NCLEX: Health Promotion and Maintenance 15. A nurse identifies a nursing diagnosis of Risk for falls when assessing a patient upon
admission. The nurse and the patient agree that the goal is for the patient to remain free from falls. However, the patient fell just before shift change. What is the nurse’s priority action when evaluating the patient’s plan of care? a. Counsel the unregulated care provider on duty when the patient fell. b. Identify factors interfering with goal achievement. c. Remove the “fall risk” sign from the patient’s door because the patient has suffered a fall. d. Request that the more experienced charge nurse complete the documentation about the fall. ANS: B
NURSINGTB.COM
After a change in the patient’s condition or an untoward event, the nurse attempts to identify factors interfering with goal achievement. In this case, the nurse identifies the factors that interfered with goal achievement to determine the cause of the fall. The fall may not have been due to an error by the unregulated care provider; therefore, counselling should be reserved until after the cause has been determined. The patient remains at risk for falls, so the “fall risk” sign should remain on the door. The nurse witnessing the fall or the nurse assigned to the patient needs to complete the documentation. The charge nurse can be consulted to review the documentation. DIF: Understand REF: 227 OBJ: Describe how evaluation leads to discontinuation, review, or modification of a plan of care. TOP: Evaluate MSC: NCLEX: Safe and Effective Care Environment 16. A patient recently received a diagnosis of pneumonia. The nurse and the patient have
established a goal that the patient will not experience shortness of breath with activity in 3 days, with an expected outcome of having no secretions present in the lungs in 48 hours. Which of the following is an appropriate evaluative measure demonstrating progress toward this goal? a. Nonproductive cough present in 4 days. b. Scattered rhonchi throughout all lung fields in 2 days. c. Respirations 30/minute in 1 day. d. Lungs clear to auscultation after use of inhaler.
Canadian Fundamentals of Nursing 6th Edition Potter Test Bank
ANS: D
Goals are broad statements that describe changes in a patient’s condition or behaviour. Expected outcomes are shorter term measurable criteria used to evaluate goal achievement. When an outcome is met, the patient is making progress toward goal achievement. In this case, the patient’s goal is to not experience shortness of breath with activity in 3 days. One way to achieve this goal is to eliminate respiratory secretions in the airway. The nurse can evaluate this expected outcome by assessing the patient’s lung sounds. If the lung sounds are clear, at least periodically throughout the day, the nurse can determine that the patient is making progress toward achieving the expected outcome. The time frame of 4 days in the first option is not appropriate because this time frame exceeds the time frame stated in the goal. Scattered rhonchi indicate fluid in the lungs, and a respiratory rate of 30 per minute is higher than normal. Either of these indicates that the patient is still probably experiencing respiratory distress. DIF: Apply REF: 224| 225 OBJ: Explain the relationship between goals of care, expected outcomes, and evaluative measures in evaluating nursing care. TOP: Evaluate MSC: NCLEX: Health Promotion and Maintenance 17. A nurse administrator is at a meeting with nurses on the quality council. Several new
members are sitting on the council. They ask the nurse administrator to clarify what a nursing-sensitive outcome is. Which response by the nurse administrator best defines nursing-sensitive outcomes? a. “Nursing-sensitive outcomes determine the patient’s progress as a result of prescribed treatments, such as medications.” b. “Patient falls are an example of a nursing-sensitive outcome because they are directly affected by nursiN ngUiR ntS erI veNnG tioTnB s.. ” COM c. “Nursing-sensitive outcomes promote universal health care.” d. “We use nursing-sensitive outcomes at this hospital to evaluate nursing tasks and to determine safe staffing ratios.” ANS: B
A nursing-sensitive outcome is a measurable patient or family statement, behaviour, or perception that is largely influenced by and sensitive to nursing interventions. Patient falls are one nursing-sensitive outcome because they are a direct measure of nursing care. Because the prescriber prescribes treatments, the progress of the patient’s condition as a result of prescribed treatments is not an evaluation of a nursing-sensitive outcome. Promotion of universal health care and determining staffing ratios are not components of nursing-sensitive outcomes. DIF: Apply REF: 225 OBJ: Give an example of evaluative measures for determining clients’ progress toward an outcome. TOP: Evaluate MSC: NCLEX: Health Promotion and Maintenance 18. Which scenario best illustrates the use of data validation when an independent nursing clinical
decision is made? a. The nurse determines that a wound dressing needs to be removed when the patient reveals the time of the last dressing change, and the nurse notices that the present dressing is saturated with fresh and old blood. b. The nurse administers pain medicine due at 1700 hours at 1600 hours because the
Canadian Fundamentals of Nursing 6th Edition Potter Test Bank patient complains of increased pain. c. The nurse removes a leg cast when the patient complains of decreased mobility. d. The nurse administers potassium when a patient complains of leg cramps. ANS: A
Changing the wound dressing is the only independent nursing action given. The nurse validates what the patient says by observing the dressing. In addition, this option is the only assessment option that involves data validation. Administering pain medicine or potassium and removing a leg cast are examples of nursing interventions. DIF: Apply REF: 194| 195 OBJ: Describe and compare direct and indirect nursing interventions. TOP: Assessment MSC: NCLEX: Safe and Effective Care Environment 19. Another term for a collaborative nursing intervention is which of the following? a. Dependent intervention. b. Independent intervention. c. Interdependent intervention. d. Physician-initiated intervention. ANS: C
A collaborative, or interdependent, intervention requires the combined knowledge, skill, and expertise of multiple health care professionals. A dependent intervention requires an order from a health care provider. An independent intervention is an action that the nurse initiates. DIF: Remember REF: 214 OBJ: Describe and compare direct and indirect nursing interventions. TOP: Planning MSC: NN CLER X: SIafeG andB E. ffeCctivM e Care Environment
U S N T
O
20. A registered nurse administers pain medication to a patient suffering from fractured ribs.
What type of nursing intervention is this nurse implementing? a. Collaborative. b. Independent. c. Interdependent. d. Dependent. ANS: D
The nurse does not have prescriptive authority to order pain medications, unless the nurse is an advanced practice nurse. The intervention is therefore dependent. A collaborative, or interdependent, intervention involves therapies that require combined knowledge, skill, and expertise from multiple health care providers. An independent intervention does not require an order or collaboration with other health care providers. DIF: Apply REF: 214 OBJ: Describe and compare direct and indirect nursing interventions. TOP: Planning MSC: NCLEX: Safe and Effective Care Environment 21. Which intervention is most appropriate for the nursing diagnosis Impaired verbal
communication related to loss of facial motor control and decreased sensation? a. Obtain an interpreter for the patient as soon as possible. b. Assist the patient in performing swallowing exercises each shift. c. Ask the family to provide a sitter to remain with the patient at all times.
Canadian Fundamentals of Nursing 6th Edition Potter Test Bank d. Provide the patient with a writing board each shift. ANS: D
The cause of the patient’s problem will help guide the nurse to the proper nursing intervention. If the patient has a problem with verbal communication, then the nurse should choose an intervention that will address the problem. Providing the patient with a writing board will allow the patient to communicate by writing because the patient is unable to communicate verbally at this time. Obtaining an interpreter might be an appropriate intervention if the patient spoke a foreign language. Assisting with swallowing exercises will help the patient with swallowing, the diagnosis for which is different from Impaired verbal communication. Asking the family to provide a sitter at all times is often unrealistic and does not promote the patient’s independence, as does providing a writing board. DIF: Apply REF: 215-218 OBJ: Discuss the process of selecting nursing interventions. MSC: NCLEX: Health Promotion and Maintenance
TOP: Planning
22. Which intervention is most appropriate for the nursing diagnosis Impaired skin integrity
related to shearing forces? a. Administer pain medication every 4 hours as needed. b. Perform the ordered dressing change twice daily. c. Do not document the wound appearance in the chart. d. Keep the bed’s side rails up at all times. ANS: B
The most appropriate intervention for the diagnosis of Impaired skin integrity is to perform the ordered dressing change. The other options do not directly address the skin integrity. The patient may need pain medicN atio n be hanges, but Acute pain would be another UR SIfore NGdre TBss.inCgOcM nursing diagnosis. Documenting all objective findings is the nurse’s responsibility, even if a wound or infection is a health care–associated problem. Keeping the side rails up addresses safety, not skin integrity. DIF: Apply REF: 215-218 OBJ: Discuss the process of selecting nursing interventions. MSC: NCLEX: Health Promotion and Maintenance
TOP: Planning
23. A patient has reduced muscle strength after a left-sided stroke and is at risk for falling. Which
intervention is most appropriate for the nursing diagnosis Risk for falls? a. Encourage patient to remain in bed most of the shift. b. Keep all side rails down at all times. c. Place patient in room away from the nurses’ station if possible. d. Assist patient into and out of bed every 6 hours, or as tolerated. ANS: D
Canadian Fundamentals of Nursing 6th Edition Potter Test Bank Risk for falls is a potential nursing diagnosis; therefore, the nurse needs to implement actions that will prevent a fall. Assisting the patient into and out of bed is the most appropriate intervention to prevent the patient from falling. Encouraging activity builds muscle strength; encouraging the patient to stay in bed will not promote muscle strength. Decreased muscle strength is the risk factor placing the patient in jeopardy of falling. The side rails should be up, not down, according to agency policy. This will remind the patent to ask for help to get up and will keep the patient from rolling out of bed. The patient should be placed near the nurses’ station so that a staff member can quickly get to the room and assist the patient if necessary. DIF: Apply REF: 215-218 OBJ: Discuss the process of selecting nursing interventions. MSC: NCLEX: Health Promotion and Maintenance
TOP: Planning
MULTIPLE RESPONSE 1. A nurse is planning care for a patient with a nursing diagnosis of Impaired skin integrity. The
patient needs many nursing interventions, including a dressing change, several intravenous antibiotics, and a walk. What actions does the nurse consider when prioritizing interventions? (Select all that apply.) a. Putting all the patient’s nursing diagnoses in order of priority. b. Considering time as an influencing factor. c. Setting priorities solely on the basis of physiological factors. d. Utilizing critical thinking. e. Not changing priorities once they’ve been established. ANS: A, B, D
The nurse avoids setting prioN ritie soI lely is of physiological factors; psychosocial Rs S Gon Bth.eCbas M U N T O factors must be considered as well. Prioritizing the problems, or nursing diagnoses, will help the nurse decide which problem to address first. Time is a factor to be included in planning before the nurse continues on to the implementation phase. Nurses use critical thinking throughout the entire nursing process. Priorities can change on the basis of patient needs and responses to treatments. DIF: Remember REF: 215-218 OBJ: Discuss the process of selecting nursing interventions. MSC: NCLEX: Health Promotion and Maintenance
TOP: Planning
2. Which of the following are examples of evaluative measures that a nurse should utilize when
determining the patient’s response to nursing care? (Select all that apply.) a. Observations of wound healing. b. Assessment of respiratory rate and depth. c. Blood pressure measurement. d. Implementation of nursing interventions. e. Patient’s subjective report of feelings about a new diagnosis of cancer. ANS: A, B, C, E
Evaluative measures require the nurse to use assessment skills and techniques to determine the patient’s response to nursing care. Examples of evaluative measures include assessment of wound healing and respiratory status, blood pressure measurement, and assessment of patient feelings. Determining whether nursing interventions were used is not an evaluative measure.
Canadian Fundamentals of Nursing 6th Edition Potter Test Bank DIF: Apply REF: 225| 226 OBJ: Give an example of evaluative measures for determining clients’ progress toward an outcome. TOP: Evaluate MSC: NCLEX: Health Promotion and Maintenance 3. Identify elements of the evaluation process. (Select all that apply.) a. Setting priorities for patient care. b. Collecting subjective and objective data to determine whether criteria or standards
are met. c. Helping the patient ambulate 7.6 m (25 feet) in the hallway. d. Documenting findings. e. Terminating, continuing, or revising the care plan. ANS: B, D, E
During the evaluation process, you gather and document objective and subjective data to determine whether the patient is meeting expected outcomes and is working toward achievement of goals. The evaluation process requires the use of critical thinking about attitudes and standards to analyze your findings and to determine whether a plan of care needs to be terminated, continued, or revised. Setting priorities is part of planning, and helping a patient ambulate in the hallway is an intervention, so it is included in the implementation step of the nursing process. DIF: Understand REF: 224-228 OBJ: Discuss the relationship between critical thinking and evaluation. TOP: Evaluate MSC: NCLEX: Health Promotion and Maintenance
NURSINGTB.COM
Canadian Fundamentals of Nursing 6th Edition Potter Test Bank
Chapter 15: Documenting and Reporting Potter et al: Canadian Fundamentals of Nursing, 6th Edition MULTIPLE CHOICE 1. A nurse preceptor is supervising a student nurse. Which behaviour by the student nurse will
require the nurse preceptor to intervene? a. The student nurse reviews the patient’s medical record. b. The student nurse reads the patient’s plan of care. c. The student nurse shares patient information with a friend. d. The student nurse documents medication administered to the patient. ANS: C
The Personal Information Protection and Electronic Documents Act (PIPEDA) is federal legislation that protects personal information, including health information, and is part of professional practice. Confidentiality and compliance with PIPEDA are mandatory. When a student nurse shares patient information with a friend, confidentiality and PIPEDA standards have been violated. A student can review patients’ medical records only to seek information needed to provide safe and effective patient care. For example, when nurses are assigned to care for a patient, they need to review the patient’s medical record and plan of care. They do not share this information with classmates, and they do not access the medical records of other patients on the unit. DIF: Apply REF: 237| 238 OBJ: Identify ways to maintain confidentiality of records and reports. TOP: Evaluate MSC: NCLEX: Safe and Effective Care Environment
N R I G B.C M U S N T O
2. A nurse prepared an audiotaped exchange with another nurse of information about a patient.
Which action did the nurse complete? a. A report. b. A record. c. A consultation. d. A referral. ANS: A
Reports are oral, written, or audiotaped exchanges of information among caregivers. A patient’s record or chart is a confidential, permanent legal document consisting of information relevant to his or her health care. Consultations are another form of discussion in which one professional caregiver gives formal advice about the care of a patient to another caregiver. Nurses document referrals (arrangements for the services of another care provider). DIF: Remember REF: 238 OBJ: Describe the purpose and content of a change-of-shift report. TOP: Implementation MSC: NCLEX: Safe and Effective Care Environment 3. Which situation best indicates that the nurse has a good understanding of auditing and
monitoring patients’ health records? a. The nurse determines the degree to which standards of care are met by reviewing patients’ health records. b. The nurse realizes that care not documented in patients’ health records still
Canadian Fundamentals of Nursing 6th Edition Potter Test Bank qualifies as care provided. c. The nurse knows that reimbursement is based on the diagnosis-related groups
documented in patients’ records. d. The nurse compares data in patients’ records to determine whether a new treatment
had better outcomes than the standard treatment. ANS: A
The patient record is a valuable source of data for all members of the health care team. Its purposes include communication, legal documentation, financial billing, education, research, and auditing/monitoring. The auditing/monitoring purpose involves nurses’ auditing records throughout the year to determine the degree to which standards of care are met and to identify areas needing improvement and staff development. The legal documentation purpose reflects the concept that even though nursing care may have been excellent, in a court of law, “care not documented is care not provided.” The financial billing or reimbursement purpose involves diagnosis-related groups (DRGs) as the basis for establishing reimbursement for patient care. For research purposes, the investigator compares patients’ recorded findings to determine whether the new method was more effective than the standard protocol. Analysis of data from research contributes to evidence-based nursing practice and quality health care. DIF: Analyze TOP: Evaluate
REF: 234 OBJ: Identify purposes of a health care record. MSC: NCLEX: Safe and Effective Care Environment
4. After providing care, a nurse charts in the patient’s record. Which entry should the nurse
document? a. “Appears restless when sitting in the chair” b. “Drank adequate amounts of water” c. “Apparently is asleep with eyes closed” N R I G B.C M U S N T O d. “Skin pale and cool” ANS: D
A factual record contains descriptive, objective information about what a nurse sees, hears, feels, and smells. An objective description is the result of direct observation and measurement: for example, “B/P 80/50, patient diaphoretic, heart rate 102 and regular.” Vague terms such as appears, seems, or apparently should be avoided because these words are suggestive of an opinion, do not accurately communicate facts, and do not inform another caregiver of details regarding behaviours exhibited by the patient. Use of exact measurements establishes accuracy. For example, a description such as “Intake, 360 mL of water” is more accurate than “Drank adequate amounts of water.” DIF: Apply REF: 239-241 OBJ: Describe six quality guidelines for documentation and reporting. TOP: Evaluate MSC: NCLEX: Safe and Effective Care Environment 5. A nurse has provided care to a patient. Which entry should the nurse document in the patient’s
record? a. “Patient seems to be in pain and states, ‘I feel uncomfortable.’” b. “Status unchanged, doing well.” c. “Left abdominal incision 5 cm in length without redness, drainage, or edema.” d. “Patient is hard to care for and refuses all treatments and medications. Family present.”
Canadian Fundamentals of Nursing 6th Edition Potter Test Bank ANS: C
Use of exact measurements establishes accuracy. Charting that an abdominal wound is “5 cm in length without redness, drainage, or edema” is more descriptive than “Large wound healing well.” The patient’s record should include objective data to support subjective data, so that charting is as descriptive as possible. Vague terms such as appears, seems, or apparently convey opinion rather than fact, do not accurately communicate facts, and do not inform another caregiver of details regarding behaviours exhibited by the patient. The nurse should avoid using generalized, uninformative phrases such as “Status unchanged” or “Had a good day.” It is essential to avoid the use of unnecessary words and irrelevant details or personal opinions. A statement such as “Patient is hard to care for” is a personal opinion and should be avoided. It is also a critical comment that can be used as evidence of nonprofessional behaviour or poor quality of care. The only statement to chart would be “Refuses all treatments and medications.” DIF: Apply REF: 239-241 OBJ: Describe six quality guidelines for documentation and reporting. TOP: Evaluate MSC: NCLEX: Safe and Effective Care Environment 6. A preceptor is supervising a new nurse on documentation. Which situation will cause the
preceptor to intervene? a. The new nurse uses a black ink pen to chart. b. The new nurse charts consecutively on every other line. c. The new nurse ends each entry with signature and title. d. The new nurse keeps the password secure. ANS: B
Charting should be consecutive, line by line (not every other line); if space is left, a line Nhrou RSghIN G ndB.C M should be drawn horizontally tU it, aT the nOurse’s name should be signed at the end. No lines should be left blank. All entries should be written legibly and in black ink. Each entry should end with the nurse’s signature and title. For computer documentation, the nurse should keep the password secure. DIF: Apply REF: 235 (Table 15-1) OBJ: Discuss legal guidelines for documentation. MSC: NCLEX: Safe and Effective Care Environment
TOP: Evaluate
7. A nurse is charting on a patient’s record. Which action is most accurate legally? a. Charting legibly. b. Stating that the patient is belligerent. c. Using correction fluid to correct error. d. Writing entry for another nurse. ANS: A
The nurse should record all entries legibly. Personal opinions should be avoided; only objective and factual observations of patient’s behaviour should be entered. All patient comments should be noted; for example, “Patient refuses to cough and deep breathe, saying, ‘I don’t care what you say, I will not do it.’” Entries should not be erased, covered with correction fluid, or scratched out, even if they are in error. Charting should be done only by the nurse who performed the action or made the observation. DIF: Understand
REF: 239-241
OBJ: Discuss legal guidelines for documentation.
Canadian Fundamentals of Nursing 6th Edition Potter Test Bank TOP: Planning
MSC: NCLEX: Safe and Effective Care Environment
8. A nurse wants to integrate all pertinent patient information into one record, regardless of the
number of times a patient enters the health care system. Which term should the nurse use to describe this system? a. Electronic medical record. b. Electronic health record. c. Electronic charting record. d. Electronic problem record. ANS: B
A unique feature of an electronic health record (EHR) is its ability to integrate all pertinent patient information into one record, regardless of the number of times a patient enters a health care system. The electronic medical record (EMR) contains patient data gathered in a health care setting at a specific time and place and is a part of the EHR; the two terms are frequently (and mistakenly) used interchangeably. There are no such terms as electronic charting record or electronic problem record. DIF: Understand REF: 236 OBJ: Discuss the use of electronic health records in documentation. TOP: Diagnosis MSC: NCLEX: Safe and Effective Care Environment 9. A nurse has taught the patient how to use crutches. The patient went up and down the stairs
using crutches with no difficulties. Which information will the nurse use for the “I” in PIE charting? a. “Patient went up and down stairs.” b. “Deficient knowledge regarding crutches.” c. “Patient demonstrated usN e oUfRcS ruI tcN heG s.T ” B.COM d. “Patient used crutches with no difficulties.” ANS: C
A second progress note method is the PIE format. The narrative note includes a statement of the problem (P), the intervention (I), and the evaluation (E). The “I” is “Demonstrated use of crutches.” “Patient went up and down stairs” and “Used crutches with no difficulties” are examples of the “E.” “Deficient knowledge regarding crutches” is the “P.” DIF: Apply REF: 242 OBJ: Describe the different methods used in record keeping. MSC: NCLEX: Health Promotion and Maintenance
TOP: Implementation
10. A nurse is using the source record and wants to find the patient’s daily weights. Where should
the nurse look? a. Database. b. Medical history and examination. c. Progress notes. d. Graphic sheet and flow sheet. ANS: D
Canadian Fundamentals of Nursing 6th Edition Potter Test Bank In a source record, the patient’s chart has a separate section for each discipline (e.g., nursing, medicine, social work, respiratory therapy) in which to record data. Graphic sheets and flow sheets are records of repeated observations and measurements such as vital signs, daily weights, and intake and output. In the problem-oriented medical record, the database section contains all available assessment information pertaining to the patient (e.g., history and physical examination, the nurse’s admission history and ongoing assessment, the dietitian’s assessment, laboratory reports, radiologic test results). In the source record, the medical history and examination section contains results of the initial examination performed by the physician, including findings, family history, confirmed diagnoses, and medical plan of care. Also in the source record, the progress notes constitute an ongoing record of the patient’s progress and response to medical therapy and a review of the disease process; these notes are often interdisciplinary and include documentation from health-related disciplines (e.g., health care providers, physiotherapy, social work). DIF: Apply REF: 243 (Table 15-3) OBJ: Describe the different methods used in record keeping. MSC: NCLEX: Safe and Effective Care Environment
TOP: Planning
11. A nurse is a member of an interdisciplinary team that uses critical pathways. According to the
critical pathway, on day 2 of the hospital stay, the patient should be sitting in the chair. It is day 3, and the patient cannot sit in the chair. What should the nurse do? a. Focus charting, using the DAR format. b. Add this observation to the problem list. c. Document the variance in the patient’s record. d. Report a positive variance in the next interdisciplinary team meeting. ANS: C
NURSINGTB.COM
A variance is the situation when the activities on the critical pathway are not completed as predicted or the patient does not meet expected outcomes. An example of a variance is when a patient develops pulmonary complications after surgery, and oxygen therapy and monitoring with pulse oximetry are needed. A positive variance is a situation when a patient progresses more rapidly than expected (e.g., use of a Foley catheter may be discontinued a day early). When a nurse is using the problem-oriented medical record, after analyzing data, health care team members identify problems and make a single problem list. A type of narrative format charting is focus charting. It involves the use of DAR notes, which include data (both subjective and objective), action (or nursing intervention), and response of the patient (i.e., evaluation of effectiveness). DIF: Apply REF: 243| 244 OBJ: Describe the role of critical pathways in multidisciplinary documentation. TOP: Implementation MSC: NCLEX: Health Promotion and Maintenance 12. A nurse needs to begin discharge planning for a patient admitted with pneumonia and a
productive cough. When is the best time for the nurse to start discharge planning for this patient? a. Upon admission. b. Right before discharge. c. After the congestion is treated. d. When the primary care provider writes the order. ANS: A
Canadian Fundamentals of Nursing 6th Edition Potter Test Bank Ideally, discharge planning begins at admission. Right before discharge is too late for discharge planning. After the congestion is treated is also too late for discharge planning. Usually the primary care provider writes the order too close to discharge, and nurses do not need an order to begin the teaching that will be needed for discharge. By identifying discharge needs early, nursing and other health care providers can begin planning for home care, support services, and any equipment needs at the patient’s home. DIF: Apply REF: 245 OBJ: Identify elements to include when documenting a patient discharge plan. TOP: Planning MSC: NCLEX: Health Promotion and Maintenance 13. A patient is being discharged home. Which information should the nurse include? a. Acuity level. b. Community resources. c. Standardized care plan. d. Kardex. ANS: B
Discharge documentation includes information about medications, diet, community resources, follow-up care, and whom to contact in case of an emergency or for questions. A patient’s level of medical acuity, usually determined by a computer program, is based on the type and number of nursing interventions (e.g., intravenous [IV] therapy, wound care, ambulation assistance) required over a 24-hour period. Acuity level can be used for staffing and billing. Some institutions use standardized care plans to make documentation more efficient. The plans, based on the institution’s standards of nursing practice, are preprinted, established guidelines used to care for patients who have similar health problems. In some settings, a Kardex, a portable “flip-over” file or notebook, is kept at the nurses’ station. Most Kardex N tme RSntIsN GonB.C M forms have an activity and treaU ectiT and aOnursing care plan section, which helps nurses organize information for quick reference. DIF: Apply REF: 245 OBJ: Identify elements to include when documenting a patient discharge plan. TOP: Planning MSC: NCLEX: Safe and Effective Care Environment 14. A nurse developed the following discharge summary sheet. Which critical information should
be added? TOPIC: DISCHARGE SUMMARY Medication Diet Activity level Follow-up care Wound care Phone numbers When to call the doctor Time of discharge a. Kardex form b. Admission nursing history c. Mode of transportation d. SOAP notes ANS: C
Canadian Fundamentals of Nursing 6th Edition Potter Test Bank In discharge summary information, the nurse lists actual time of discharge, mode of transportation, and who accompanied the patient. In some settings, a Kardex, a portable “flip-over” file or notebook, is kept at the nurses’ station. A Kardex is for nurses, not for patients to take upon discharge. A nurse completes a nursing history form when a patient is admitted to a nursing unit, not when the patient is discharged. SOAP notes are not given to patients who are being discharged. SOAP notes are a type of documentation style. DIF: Evaluate REF: 245 (Box 15-5) OBJ: Identify elements to include when documenting a patient discharge plan. TOP: Evaluate MSC: NCLEX: Health Promotion and Maintenance 15. According to documentation guidelines the most appropriate notation is a. “1230 hours: Patient’s vital signs taken.” b. “0700 hours: Patient drank adequate amount of fluids.” c. “0900 hours: Morphine given for lower abdominal pain.” d. “0830 hours: Increased IV fluid rate to 100 mL per hour.” ANS: D
Information within a recorded entry must be complete, containing appropriate and essential information. The notation “0830 hours: Increased IV fluid rate to 100 mL per hour” provides the time and action taken by the nurse, including the reason for doing so. The entry “1230 hours: Patient’s vital signs taken” does not indicate what the patient’s vital signs were. The entry “0700 hours: Patient drank adequate amount of fluids” does not provide the specific amount that the patient drank. Stating “adequate” is subjective, not objective. The notation “0900 hours: Morphine given for lower abdominal pain” does not indicate how the patient describes his or her pain or rates it on a pain scale for comparison later. It also does not indicate whether the patient’s pain was in the lower left or lower right quadrant, or both.
N R I G B.C M U S N T O
DIF: Apply REF: 239-241 OBJ: Describe six quality guidelines for documentation and reporting. TOP: Implementation MSC: NCLEX: Safe and Effective Care Environment 16. The final “R” when using the I-SBAR-R communication technique represents which of the
following? a. Recovery. b. Repeat back. c. Reorganization. d. Reintegration. ANS: B
The final “R” in I-SBAR-R represents “repeat back,” which includes repeating back orders that have been given or clarifying any questions. The final “R” in I-SBAR-R does not stand for “recovery,” “reorganization,” or “reintegration.” DIF: Apply REF: 248 (Box 15-9) OBJ: Discuss the advantages of standardized documentation forms. TOP: Implementation MSC: NCLEX: Safe and Effective Care Environment 17. A nurse is giving a hand-off report to the nurse on the next shift. Which information is critical
for the nurse to report? a. The patient had a good day with no complaints.
Canadian Fundamentals of Nursing 6th Edition Potter Test Bank b. The family is demanding and argumentative. c. The patient has a new pain medication, hydrocodone bitartrate and acetaminophen
(Lortab). d. The family is poor and had to go on welfare. ANS: C
Significant changes in the way therapies are to be given (e.g., “Different position for pain relief, new medication”) should be described to staff. Results should not be described simply as “good” or “poor”; they should be specific. Critical comments about patient’s or family’s behaviour are considered idle gossip and should not be mentioned. DIF: Apply REF: 246-248 OBJ: Describe the purpose and content of a change-of-shift report. TOP: Implementation MSC: NCLEX: Safe and Effective Care Environment 18. A new nurse asks the preceptor why a change-of-shift report is important, inasmuch as care is
documented in the chart. What is the preceptor’s best response? a. “A change-of-shift report provides an opportunity to share essential information to ensure patient safety and continuity of care.” b. “A change-of-shift report provides the oncoming nurse with data to help set priorities and establish reimbursement costs.” c. “A change-of-shift report provides an opportunity for the oncoming nurse to ask questions and determine research priorities.” d. “A change-of-shift report provides important information to caregivers and develops relationships within the health care team.” ANS: A
Properly performed, a changeN-of -shi ft N reG por t p.roCvid UR SI TB OMes an opportunity to share essential information to ensure patient safety and continuity of care. Reimbursement costs and research priorities/opportunities are functions of the medical record. The purpose of the change-of-shift report is not to develop relationships. DIF: Remember REF: 246-248 OBJ: Describe the purpose and content of a change-of-shift report. TOP: Implementation MSC: NCLEX: Safe and Effective Care Environment 19. A nurse is preparing a change-of-shift report for a patient who had chest pain. Which
information is critical for the nurse to include? a. “Pupils equal and reactive to light.” b. “The family is a ‘pain.’” c. “Had poor results from the pain medication.” d. “Sharp pain of 8 on a scale of 1 to 10.” ANS: D
Elements in a change-of-shift report include identification of significant changes in measurable terms (e.g., pain scale) and by observation. Report elements do not include normal findings or routine information retrievable from other sources or derogatory or inappropriate comments about the patient or family, which could possibly lead to legal charges if overheard by the patient or family. This kind of language contributes to prejudicial opinions about the patient. Results should not be described simply as “good” or “poor”; they should be specific. DIF: Understand
REF: 246-248
Canadian Fundamentals of Nursing 6th Edition Potter Test Bank OBJ: Describe the purpose and content of a change-of-shift report. TOP: Planning MSC: NCLEX: Safe and Effective Care Environment 20. Which situation will require the nurse to obtain a telephone order? a. As the nurse and primary care provider leave a patient’s room, the primary care
provider gives the nurse an order. b. At 0100, a patient’s blood pressure drops from 120/80 to 90/50 and the incision dressing is saturated with blood. c. At 0800, the nurse and primary care provider make rounds and the primary care provider tells the nurse a diet order. d. A nurse reads an order correctly as written by the primary care provider in the patient’s medical record. ANS: B
A registered nurse makes a telephone report when significant events or changes in a patient’s condition have occurred. Telephone orders and verbal orders usually occur at night or during emergencies. Because the time is 0100 (1 a.m.) and the primary care provider is not present, the nurse will need to call the primary care provider for a telephone order. A verbal order involves the health care provider giving orders to a nurse while they are standing near each other. Just reading an order that is correctly written in the chart does not require a telephone order. DIF: Apply TOP: Assessment
REF: 246 OBJ: Explain how to verify telephone orders. MSC: NCLEX: Safe and Effective Care Environment
21. A nurse obtained a telephone order (TO) from a primary care provider for a patient in pain.
Which chart entry should the nurse document? a. “12/16/20, 0915: TylenolN3U , 2RtS abIleNtsG ,T evB er. yC 6O hM ours for incisional pain. VO Dr. Day/J. Winds, RN, read back.” b. “12/16/20, 0915: Tylenol 3, 2 tablets, every 6 hours for incisional pain. TO J. Winds, RN, read back.” c. “12/16/20, 0915: Tylenol 3, 2 tablets, every 6 hours for incisional pain. TO Dr. Day/J. Winds, RN, read back.” d. “12/16/20, 0915: Tylenol 3, 2 tablets, every 6 hours for incisional pain. TO J. Winds, RN.” ANS: C
The nurse receiving a TO writes down the complete order or enters it into the computer as it is being given. Then he or she reads the order back to the health care provider, a procedure called read back, and receives confirmation from the person who gave the order that it is correct. Option C The entry ending “TO Dr. Day/J. Winds, RN, read back” is such an example. VO stands for verbal order, not telephone order. The doctor’s name and “read back” must be included in the chart entry. DIF: Apply REF: 246 TOP: Implementation
OBJ: Explain how to verify telephone orders. MSC: NCLEX: Safe and Effective Care Environment
22. A nurse records the following: “Patient is wheezing and experiencing some dyspnea on
exertion.” This represents which of the following? a. The “S” in SOAP documentation. b. Focus documentation.
Canadian Fundamentals of Nursing 6th Edition Potter Test Bank c. The “P” of PIE documentation. d. The “R” in DAR documentation. ANS: C
The statement is an example of the “P” of PIE documentation because it describes the patient’s problem. The “S” in SOAP documentation represents subjective data (verbalizations of the patient). Focus charting does not concentrate on problems only. The “R” in DAR documentation is the response of the patient. This situation describes the patient’s problem, not the patient’s response. DIF: Analyze REF: 241 (Box 15-1) OBJ: Describe the different methods used in record keeping. MSC: NCLEX: Safe and Effective Care Environment
TOP: Implementation
23. A hospital is using computer software that allows all health care providers to use a protocol
system to document the care they provide. Which type of system/design will the nurse be using? a. Clinical decision support system. b. Nursing process design. c. Critical pathway design. d. Computerized provider order entry system. ANS: C
One design model for nursing information systems (NIS) is the protocol or critical pathway design. With this design, all health care providers use a protocol system to document the care they provide, allowing them to select one or more appropriate protocols for a patient. A clinical decision support system is based on “rules” and “if-then” statements, linking information and producing alN erts r oCthe UR, wa SIrnin NGgs, TBo. OMr information for the user. The nursing process design is the most traditional design for an NIS. This design organizes documentation within well-established formats such as admission and postoperative assessments, problem lists, care plans, discharge planning instructions, and intervention lists or notes. Computerized provider order entry (CPOE) is a process by which the health care provider directly enters orders for patient care into the hospital information system. DIF: Understand REF: 249| 250 OBJ: Describe the role of critical pathways in interdisciplinary documentation. TOP: Evaluate MSC: NCLEX: Safe and Effective Care Environment 24. A nurse wants to reduce data entry errors on the computer system. Which behaviour should
the nurse implement? a. Use the same password all the time. b. Share password with only one other staff member. c. Print out and review computer nursing notes at home. d. Chart on the computer immediately after care is provided. ANS: D
Canadian Fundamentals of Nursing 6th Edition Potter Test Bank To increase accuracy and decrease unnecessary duplication, many health care agencies keep records or computers near a patient’s bedside to facilitate immediate documentation of information as it is collected from a patient. A good system requires frequent and random changes in personal passwords to prevent unauthorized persons from tampering with records. When a nurse uses a health care agency computer system, it is essential that his or her computer password not be shared with anyone under any circumstances. The nurse destroys (e.g., shreds) anything that is printed when the information is no longer needed. Taking nursing notes home is a violation of the Health Insurance Portability and Accountability Act (HIPAA) and confidentiality. DIF: Apply REF: 240| 249| 250 OBJ: Discuss the use of electronic health records in documentation. TOP: Implementation MSC: NCLEX: Safe and Effective Care Environment 25. Which entry will require follow-up by the nurse manager? a. 0800: Patient states, “Fell out of bed.” Patient found lying by bed on the floor.
Legs equal in length bilaterally with no distortion, pedal pulses strong, leg strength equal and strong, no bruising or bleeding. Neuro checks within normal limits. States, “Did not pass out.” Assisted back to bed. Call bell within reach. Bed monitor on. Jane More, RN b. 0810: Notified primary care provider of patient’s status. New orders received. Jane More, RN c. 0815: Portable x-ray of L hip taken in room. Patient states, “I feel fine.” Jane More, RN d. 0830: Incident report completed and placed on chart. -------------------Jane MorN e,URR NSINGTB.COM ANS: D
The incident report is not mentioned in the patient’s medical record. Instead, the nurse documents in the patient’s medical record an objective description of what happened, what the nurse observed, and follow-up actions taken. It is important to evaluate and document the patient’s response to the error or incident. The patient’s health care provider should be contacted whenever an incident happens. DIF: Analyze TOP: Evaluate
REF: 248| 249 OBJ: Discuss legal guidelines for documentation. MSC: NCLEX: Safe and Effective Care Environment
26. The action that a nurse would take when documenting on the patient’s record and notes that
he or she has made an error is which of the following? a. Drawing a line through the error and initialing and dating it. b. Erasing the error and writing over the material in the same spot. c. Using a dark-coloured marker to cover the error and continuing immediately after that point. d. Footnoting the error at the bottom of the page, including initials and the date. ANS: A
Canadian Fundamentals of Nursing 6th Edition Potter Test Bank If the nurse has made an error in documentation, he or she should draw a single line through the error, write the word “error” above it, and sign his or her name or initials. Then the nurse should record the note correctly. The nurse should not erase, apply correction fluid to, or scratch out errors made while recording because charting then becomes illegible. Entries should be made only in ink so that they cannot be erased. Using a dark-coloured marker to cover the error and continuing immediately after that point appears as if the nurse were attempting to hide something or deface the record. Footnotes are not used in nursing documentation. DIF: Apply REF: 235 ( Table 15-1) OBJ: Discuss legal guidelines for documentation. MSC: NCLEX: Safe and Effective Care Environment
TOP: Implementation
27. A slight hematoma has developed on the patient’s left forearm. The nurse labels the problem
as an infiltrated intravenous (IV) line. The nurse elevates the forearm. The patient states, “My arm feels better.” When using the DAR notes of focus charting, the nurse would document the “R” as which of the following? a. “My arm feels better.” b. “Slight hematoma on left forearm.” c. “Infiltrated IV line.” d. “Elevation of left forearm.” ANS: A
The “R” in the data–action–response (DAR) documentation of focus charting is the response by the patient. In this case, the nurse would document, “Patient states, ‘My arm feels better.’ Both “Slight hematoma on left forearm” and “Infiltrated IV line” are examples of the “D” in a DAR note, referring to data in focus charting. “Elevation of left forearm” is the “A” in a DAR N ursi RSngIiNnter GTvention B.COinMfocus charting. note, describing the action or nU DIF: Apply REF: 242 OBJ: Describe the different methods used in record keeping. MSC: NCLEX: Safe and Effective Care Environment
TOP: Implementation
28. A nurse is discussing the advantages of standardized documentation forms in the nursing
information system. Which advantage should the nurse describe? a. Varied clinical databases. b. Reduced errors of omission. c. Increased hospital costs. d. More time to read charts. ANS: B
Advantages associated with the nursing information system include increased time to spend with patients (not to read charts); better access to information; enhanced quality of documentation; reduced errors of omission; reduced, not increased, hospital costs; increased nurse job satisfaction; compliance with requirements of accrediting agencies (e.g., TJC); and development of a common, not varied, clinical database. DIF: Understand REF: 250 OBJ: Discuss the advantages of standardized documentation forms. TOP: Implementation MSC: NCLEX: Safe and Effective Care Environment
Canadian Fundamentals of Nursing 6th Edition Potter Test Bank MULTIPLE RESPONSE 1. Identify the purposes of a health care record. (Select all that apply.) a. Communication. b. Legal documentation. c. Reimbursement. d. Education. e. Research. f. Nursing process. ANS: A, B, D, E
The patient record is a valuable source of data for all members of the health care team. Its purposes include communication, legal documentation, education, research, and auditing/monitoring. Nursing process is a way of thinking and performing nursing care; it is not something to be documented in a health care record. DIF: Remember TOP: Planning
REF: 234| 236 OBJ: Identify purposes of a health care record. MSC: NCLEX: Safe and Effective Care Environment
2. A nurse is creating a plan to reduce data entry errors and maintain confidentiality. Which
guidelines should the nurse include? (Select all that apply.) a. Create a password with just letters. b. Bypass the firewall. c. Use a programmed speed-dial key when faxing. d. Implement an automatic sign-off. e. Impose disciplinary actions for inappropriate access. f. Shred papers containing personal health information. ANS: C, D, E, F
NURSINGTB.C OM
In faxing, programmed speed-dial keys should be used to eliminate the chance of a dialing error and misdirected information. An automatic sign-off is a safety mechanism that logs a user off the computer system after a specified period of inactivity; it is used in most patient care areas and other departments that handle sensitive data. Disciplinary action, including loss of employment, occurs when nurses or other health care personnel inappropriately access patient information. All papers containing private health information must be destroyed. Most agencies have shredders or locked receptacles for shredding and later incineration. Strong passwords are combinations of letters, numbers, and symbols that are difficult to guess. A firewall is a combination of hardware and software that protects private network resources (e.g., the information system of the hospital) from outside hackers, network damage, and theft or misuse of information and should not be bypassed. DIF: Create REF: 237| 238 OBJ: Identify ways to maintain confidentiality of electronic and written records. TOP: Evaluate MSC: NCLEX: Safe and Effective Care Environment
Canadian Fundamentals of Nursing 6th Edition Potter Test Bank
Chapter 16: Nursing Informatics and Canadian Nursing Practice Potter et al: Canadian Fundamentals of Nursing, 6th Edition MULTIPLE CHOICE 1. Of the following, which is a current technological application that is critical for supporting
clinical judgement, decision making, and optimal client outcomes? a. Client documentation systems. b. Laboratory reporting software. c. Diagnostic imaging systems. d. Billing and financial management databases. ANS: A
Client documentation systems inform effective nursing practice through supporting clinical judgement and decision making, as well as supporting optimal nursing care. Effective documentation is critical for supporting clinical decision making, as well as supporting aggregation with documentation from other nurses and ensuring optimal patient outcomes. Laboratory reporting does not necessarily inform effective nursing practice or support optimal patient outcomes; it is an example of a current technological application. Diagnostic imaging is a current technological application tool. Billing and financial management do not support effective nursing outcomes. DIF: Remember REF: 254 OBJ: Identify and compare Canadian strategies for identifying and documenting key nursing data. TOP: Implementation MSC: NCLEX: Safe and Effective Care Environment 2. As nurses continue to contexN tuU alR izS eI tecNhG noTloBg. yC wO ithMin the scope of their professional practice,
what is the main nursing issue with technology that requires consideration? a. Computers. b. Management information systems. c. Information and information management. d. Order entry applications. ANS: C
The issues for nurses are no longer computers or management information systems but rather information and information management. The computer and its associated software are merely tools to support nurses as they practise their profession. Computers are no longer an issue for nurses. Management information systems are no longer an issue for nurses. Order entry applications are a current technological application. DIF: Understand REF: 255 OBJ: Identify and compare Canadian strategies for identifying and documenting key nursing data. TOP: Implementation MSC: NCLEX: Safe and Effective Care Environment 3. Nursing informatics is a specialty area of nursing practice dedicated to the optimal use of
technology to support professional practice and optimal client outcomes. A significant obstacle to the collection of nursing data is the a. Canada Health Infoway. b. International Classification for Nursing Practice (ICNP), which lacks unified terminology for recording nursing practice.
Canadian Fundamentals of Nursing 6th Edition Potter Test Bank c. Lack of evidence-informed practices for health information. d. Absence of universally accepted methods for defining and coding nursing
contributions to health outcomes. ANS: D
Despite the escalation of the use of technology in health care and the recognition of nursing informatics, persistent absence of universally accepted methods for defining and coding nursing contributions to health outcomes is a significant obstacle to the collection of nursing data. The other options are not obstacles to collecting nursing data. The Canada Health Infoway is an incorporated national body with mandates to generate consensus on health information standards, to drive forward a national agenda of creating an electronic health record (EHR), and to act as the liaison to international standards and development organizations. The ICNP was developed by the International Council of Nurses; it comprises seven axes with associated terms for describing nursing practice and is the only unified international terminology for recording nursing practice. As health care systems respond to an increasingly complex technological environment, long-standing routines and tools are being superseded by strategic, evidence-informed practices that necessitate high-quality, timely health information. DIF: Understand REF: 255 OBJ: Discuss why using standardized nursing data is important for acknowledging the professional contributions of nursing to health outcomes of Canadians. TOP: Planning MSC: NCLEX: Safe and Effective Care Environment 4. According to the Canadian Organization for Advancement of Computers in Health (COACH),
what is health informatics? a. The intersection of clinical, information management–information technology, and N RveIbNette GTr B.C M management practices to acUhieS healthO . b. The specialty that integrates nursing science, computer science, and information science. c. The application of computer science and information science to nursing. d. A clinical term used to describe multidisciplinary clinical practice. ANS: A
COACH defines health informatics as the “intersection of clinical, IM/IT [information management/information technology] and management practices to achieve better health” (COACH, 2009, p. 7). Health informatics covers all disciplines. Role-centred/role-based nursing informatics was a step in the evolution of health informatics that integrates nursing science, computer science, and information science, according to the American Nurses Association. The application of computer science and information science to nursing is the definition of nursing informatics by the Canadian Nurses Association (CNA). A clinical term used to describe multidisciplinary clinical practice is Systematized NOmenclature of MEDicine—Clinical Terms (SNOMED CT); this is not how COACH defines health informatics. DIF: Understand REF: 255| 256 OBJ: Differentiate how nursing informatics differs from routine use of technologies in nursing practice. TOP: Assessment MSC: NCLEX: Safe and Effective Care Environment
Canadian Fundamentals of Nursing 6th Edition Potter Test Bank 5. The Management Information System (MIS) Group was an important event in Canadian
health care information management. Which of the following were not collected by this group? a. Demographic data. b. Clinical nursing data. c. Statistical medical data. d. Resource utilization data. ANS: B
Clinical nursing data were not collected by the MIS Group because the information to be collected was restricted to physician-driven data. The MIS Group did collect demographic data, statistical medical data, and resource utilization data. DIF: Understand REF: 256 OBJ: Discuss how health information data standards influence Canadian nursing practice. TOP: Assessment MSC: NCLEX: Safe and Effective Care Environment 6. Which institute was developed as a result of the Wilk Report? a. Hospital Medical Records Institute (HMRI). b. Statistics Canada and Health and Welfare Institute. c. Canadian Nursing Informatics Association. d. Canadian Institute for Health Information (CIHI). ANS: D
The Wilk Report (National Task Force on Health Information, 1991) had a significant effect on Canadian health information, triggering the 1993 merger of the MIS Group, HMRI, portions of Statistics Canada, and Health and Welfare Canada to create the CIHI. The Canadian Nursing InformaticN sU AR ssS ocIiaNtiG onTw Ba.s CnoOt Mdeveloped as a result of the Wilk Report. DIF: Understand REF: 256 OBJ: Discuss how health information data standards influence Canadian nursing practice. TOP: Assessment MSC: NCLEX: Safe and Effective Care Environment 7. The CIHI disseminates essential data and analysis on Canada’s health care system and the
health of Canadians. The CIHI influenced issues directly related to nursing by including issues related to which of the following? a. Nursing science and information science to support clients. b. Nursing workforce recruitment and retention. c. Clinical practice, administration, research, and education. d. Advocating for health policy. ANS: B
The CIHI is the national, independent, and not-for-profit body that records, analyzes, and disseminates data and analysis on Canada’s health care system and the health of Canadians. Although not initially attentive to nursing data, this institution later became more important to several issues directly influencing nursing, including issues related to nursing workforce recruitment and retention. Nursing science, information science, and supporting clients is part of the definition of role-centred nursing informatics. Clinical practice, administration, research, and education are part of the definition of nursing informatics related to information technology. Advocating for health policy is the mandate for the International Council of Nurses, not the CIHI.
Canadian Fundamentals of Nursing 6th Edition Potter Test Bank
DIF: Understand REF: 256 OBJ: Discuss how health information data standards influence Canadian nursing practice. TOP: Assessment MSC: NCLEX: Safe and Effective Care Environment 8. Canada Health Infoway was a key outcome of the federal–provincial–territorial partnership.
What is its mandate? a. To create an electronic health record system. b. To unify international terminology for recording nursing practice. c. To advocate for effective health policy. d. To implement provincial/territorial standards. ANS: A
Infoway has a mandate to drive forward a national agenda to create an electronic health record system. Unifying international terminology for recording nursing practice is associated with the ICNP, not Canada Health Infoway. Advocating for effective health policy is associated with the International Council of Nurses, not Canada Health Infoway. Canada Health Infoway does have a mandate to generate consensus on health information standards but not to implement provincial/territorial standards. DIF: Understand REF: 256 OBJ: Identify and compare Canadian strategies for identifying and documenting key nursing data. TOP: Implementation MSC: NCLEX: Safe and Effective Care Environment 9. A patient has had several admissions to his local hospital for asthma and was recently
transferred to another hospital to have hip replacement surgery. The benefit of a standardized EHR is a. Timely access to health dN atU a.RSINGTB.COM b. Improved coordination of care. c. Enhanced ability to work collaboratively with other organizations. d. Enhanced ability to extract best practice data from published sources. ANS: B
The patient’s health information could be transferred to the hospital where hip replacement surgery will be done. The patient would benefit from a standardized EHR by improved coordination of care, reduced repetition of health information, and reduced duplication of tests and procedures. Health care providers benefit from timely access to health data benefits. Service delivery organizations benefit from the enhanced ability to work collaboratively with other organizations. Educators benefit from the enhanced ability to extract best practice data from published sources. DIF: Analyze REF: 255| 256 OBJ: Discuss the relationship between national privacy legislation and nursing practice in a digital practice environment. TOP: Implementation MSC: NCLEX: Safe and Effective Care Environment 10. Which of the following Health Information: Nursing Component (HI:NC) can be described as
the amount and type of nursing resource used to provide care? a. Patient status. b. Nursing interventions. c. Nursing intensity.
Canadian Fundamentals of Nursing 6th Edition Potter Test Bank d. Patient outcome. ANS: C
Nursing intensity is the amount and type of nursing resource used to provide care. Patient status refers to the health status of patients. Nursing interventions are the purposeful and deliberate health-affecting interventions. Patient outcome is the patient status at a defined point after health care intervention. DIF: Understand REF: 260 OBJ: Identify and compare Canadian strategies for identifying and documenting key nursing data. TOP: Planning MSC: NCLEX: Safe and Effective Care Environment 11. Which of the following is a Health Information: Nursing Component (HI:NC)? a. Patient diagnosis. b. Educational background. c. Primary nurse identifier. d. Independent nursing judgement. ANS: C
One of the five HI:NCs is primary nurse identifier. The remaining four are patient status, nursing intervention, patient outcome, and nursing intensity. Patient diagnosis, educational background, and independent nursing judgement are not among the five HI:NCs. DIF: Understand REF: 259| 260 OBJ: Identify and compare Canadian strategies for identifying and documenting key nursing data. TOP: Planning MSC: NCLEX: Safe and Effective Care Environment 12. The seven axes of the ICNP are which of the following? Nrven RStion IN, G B.C M a. Assessment, Analysis, InteU AT ction, OOutcome, and Evaluation. b. Focus, Judgement, Action, Means, Location, Client, and Time. c. Status, Intervention, Outcome, Intensity, Identifier, Client, and Time. d. Focus, Standards, Delivery, Outcomes, Intensity, Evaluation, and Time. ANS: B
ICNP has a seven-axis model, which includes Focus, Judgement, Action, Means, Location, Client, and Time. The INCP is a standardized terminology for nursing practice and is built using the Web Ontology Language (OWL). DIF: Remember REF: 260 OBJ: Develop a beginning understanding of the scope of nursing informatics concepts and the ways in which nurses can be involved in nursing informatics. TOP: Planning MSC: NCLEX: Safe and Effective Care Environment 13. Canadian privacy legislation indicates which of the following? a. All privacy legislation is developed at the provincial/territorial level. b. Privacy legislation addresses protection only of personal health information. c. Privacy legislation cannot be violated if a nurse is practising within his or her
standards of practice. d. An understanding of the privacy legislation can assist nurses in upholding their
Code of Ethics. ANS: D
Canadian Fundamentals of Nursing 6th Edition Potter Test Bank An understanding of both the Privacy Act and the Personal Information Protection and Electronic Documents Act can assist nurses in upholding their Code of Ethics, as well as their standards for nursing practice. Some privacy legislation is developed provincially and territorially, but not all; there are also federal privacy laws. Privacy legislation addresses the protection of personal health information as well as other personal information. It is possible for a nurse to violate privacy legislation even though the nurse is fulfilling his or her standards of practice. DIF: Understand REF: 262 OBJ: Discuss the relationship between national privacy legislation and nursing practice in a digital practice environment. TOP: Planning MSC: NCLEX: Safe and Effective Care Environment 14. Which of the following is one of the three fundamental directions for the CNA’s e-Nursing
Strategy? a. Assessment. b. Participation. c. Collaborative care. d. Nursing practice integration. ANS: B
Participation is one of the three fundamental directions for the CNA’s e-Nursing Strategy. The other two directions are access and competencies. Assessment, collaborative care, and nursing practice integration are not among the three fundamental directions for the CNA’s e-Nursing Strategy. DIF: Understand REF: 264-266 OBJ: Discuss how the CanadiN anUNR urS seI sN AG ssT ocB ia. tioC n'O s nMational e-Nursing Strategy will influence current and future nursing practice. TOP: Implementation MSC: NCLEX: Safe and Effective Care Environment 15. Nurses can create discussion groups online with which current technology? a. Skype. b. NurseONE. c. NurseConnect. d. COACH. ANS: C
NurseConnect is a portal that enables nurses to create discussion groups among its subscribers. It is part of NurseONE. Skype is not a means for nurses to create discussion groups online. NurseONE is the Canadian Nurses Portal, and it provides professional links, professional development, Library, and NurseConnect. COACH is the acronym for the Canadian Organization for the Advancement of Computers in Health and is not a means for nurses to create discussion groups online. DIF: Remember REF: 265 OBJ: Discuss how the Canadian Nurses Association's national e-Nursing Strategy will influence current and future nursing practice. TOP: Implementation MSC: NCLEX: Safe and Effective Care Environment MULTIPLE RESPONSE
Canadian Fundamentals of Nursing 6th Edition Potter Test Bank
1. Health Information: Nursing Components (HI:NC) include which of the following? (Select all
that apply.) a. Nursing interventions. b. Patient status. c. Nursing intensity. d. Primary nurse identifier. e. Nursing judgement. f. Patient outcome. ANS: A, B, C, D, F
Since 1992, there has been consensus among Canadian nurses that HI:NC comprises five categories of elements: patient status, nursing interventions, patient outcomes, nursing resource intensity, and primary nurse identifier. DIF: Understand REF: 259 OBJ: Identify and compare Canadian strategies for identifying and documenting key nursing data. TOP: Planning MSC: NCLEX: Safe and Effective Care Environment
NURSINGTB.COM
Canadian Fundamentals of Nursing 6th Edition Potter Test Bank
Chapter 17: Communication and Relational Practice Potter et al: Canadian Fundamentals of Nursing, 6th Edition MULTIPLE CHOICE 1. What characterizes nurses who make the best communicators? a. Developing critical thinking skills. b. Liking different kinds of people. c. Learning effective psychomotor skills. d. Maintaining perceptual biases. ANS: A
Nurses who develop critical thinking skills make the best communicators. Critical thinking helps the nurse overcome perceptual biases or human tendencies that interfere with accurately perceiving and interpreting messages from others. Just liking people does not make someone an effective communicator because it is important to apply critical thinking standards to ensure sound effective communication. Just learning psychomotor skills does not ensure that the nurse will use those techniques, and communication involves more than psychomotor skills. Nurses who maintain perceptual biases do not make good communicators. DIF: Remember REF: 272| 273 OBJ: Describe aspects of critical thinking that are important in the communication process. TOP: Implementation MSC: NCLEX: Safe and Effective Care Environment 2. A nurse believes that the nurse–patient relationship is a partnership and that both are equal
participants. Which term should the nurse use to describe this belief? N R I G B.C M a. Critical thinking. U S N T O b. Authentic. c. Mutuality. d. Attending. ANS: C
Effective interpersonal communication requires a sense of mutuality, a belief that the nurse–patient relationship is a partnership and that both are equal participants. Critical thinking in nursing, based on established standards of nursing care and ethical standards, promotes effective communication and uses such standards as humility, self-confidence, independent attitude, and fairness. To be authentic (one’s self) and to respond appropriately to the other person are important for interpersonal relationships but do not mean mutuality. Attending is giving all attention to the patient. DIF: Understand REF: 272 OBJ: Identify significant features and therapeutic outcomes of nurse–patient helping relationships. TOP: Planning MSC: NCLEX: Safe and Effective Care Environment 3. A nurse wants to present information about flu immunizations to older persons in the
community. Which type of communication should the nurse use? a. Interpersonal. b. Public. c. Transpersonal. d. Small group.
Canadian Fundamentals of Nursing 6th Edition Potter Test Bank
ANS: B
Public communication is interaction with an audience. Nurses have opportunities to speak with groups of consumers about health-related topics, present scholarly work to colleagues at conferences, or lead classroom discussions with peers or students. Intrapersonal communication is a powerful form of communication that occurs within an individual. Transpersonal communication is interaction that occurs within a person’s spiritual domain. When nurses work on committees, lead patient support groups, form research teams, or participate in patient care conferences, they use a small-group communication process. DIF: Apply REF: 274 OBJ: Describe the five levels of communication and their uses in nursing. TOP: Planning MSC: NCLEX: Health Promotion and Maintenance 4. Which technique is being used most effectively in which scenario? a. Interpersonal communication to change negative self-talk to positive self-talk. b. Small-group communication to present information to an audience. c. Intrapersonal communication to build strong teams. d. Transpersonal communication to enhance meditation. ANS: D
Transpersonal communication is interaction that occurs within a person’s spiritual domain. Many people use prayer, meditation, guided reflection, religious rituals, or other means to communicate with their “higher power.” Interpersonal communication is one-on-one interaction between the nurse and another person that often occurs face to face. Meaningful interpersonal communication results in exchange of ideas, problem solving, expression of feelings, decision making, goal accomplishment, team building, and personal growth. Small-group communicationN isUinRter actio I GnTthBa.t oCccu Mrs when a small number of persons meet. S N O This type of communication is usually goal directed and requires an understanding of group dynamics. When nurses work on committees, lead patient support groups, form research teams, or participate in patient care conferences, they use a small-group communication process. Intrapersonal communication is a powerful form of communication that occurs within an individual. For example, you improve your health and self-esteem through positive self-talk by replacing negative thoughts with positive assertions. DIF: Evaluate REF: 273| 274 OBJ: Describe the five levels of communication and their uses in nursing. TOP: Evaluate MSC: NCLEX: Safe and Effective Care Environment 5. A nurse is standing beside the patient’s bed. The nurse asks, “How are you doing?” The
patient responds, “I don’t feel good.” In this situation, which element is the feedback? a. The nurse. b. The patient. c. “How are you doing?” d. “I don’t feel good.” ANS: D
“I don’t feel good” is the feedback because the feedback is the message the receiver returns to the sender. The sender is the person who encodes and delivers the message, and the receiver is the person who receives and decodes the message. The nurse is the sender. The patient is the receiver. “How are you doing?” is the message.
Canadian Fundamentals of Nursing 6th Edition Potter Test Bank DIF: Apply REF: 274 OBJ: Describe the basic elements of the communication process. TOP: Assessment MSC: NCLEX: Safe and Effective Care Environment 6. A nurse is sitting at the patient’s bedside documenting a nursing history. Which zone of
personal space is the nurse using? a. Intimate. b. Personal. c. Social. d. Public. ANS: B
Personal space is 45 cm to 1 m (~18 inches to ~4 feet) and is used when the nurse is sitting at a patient’s bedside, taking a patient’s nursing history, and teaching an individual patient. Intimate space is 0 to 45 cm (0 to 18 inches) and is used in such activities as performing a physical assessment, bathing, grooming, dressing, feeding, and toileting a patient. Social zone is 1 to 4 m (4 to 12 feet) and is used in such activities as making rounds with a physician, sitting at the head of a conference table, and teaching a class for patients with diabetes. Public zone is 4 m (12 feet) and greater and is used in such activities as speaking at a community forum, testifying at a legislative hearing, and lecturing. DIF: Remember REF: 276 (Box 17-2) OBJ: Describe the basic elements of the communication process. TOP: Implementation MSC: NCLEX: Safe and Effective Care Environment 7. A smiling patient angrily states, “I will not cough and deep breathe.” How will the nurse
interpret this finding? a. The patient’s personal spN acU eR wS asIvN ioG laT teB d..COM b. The patient’s affect is inappropriate. c. The patient’s vocabulary is poor. d. The patient’s denotative meaning is wrong. ANS: B
An inappropriate affect is a facial expression that does not match the content of a verbal message (e.g., smiling when describing a sad situation). The patient is smiling but is angry, which indicates an inappropriate affect. The patient’s personal space was not violated. The patient’s vocabulary is not poor. Individuals who use a common language share denotative meaning: baseball has the same meaning for everyone who speaks English, but code denotes cardiac arrest primarily to health care providers. The patient’s denotative meaning is correct for cough and deep breathe. DIF: Evaluate REF: 275| 276 OBJ: Describe the basic elements of the communication process. TOP: Evaluate MSC: NCLEX: Safe and Effective Care Environment 8. The nurse asks a patient where the pain is, and the patient responds by pointing to the area of
pain. Which form of communication did the patient use? a. Verbal. b. Nonverbal. c. Intonation. d. Vocabulary.
Canadian Fundamentals of Nursing 6th Edition Potter Test Bank ANS: B
The patient gestured (pointed), which is a type of nonverbal communication. Gestures emphasize, punctuate, and clarify the spoken word. Pointing to an area of pain is sometimes more accurate than describing its location. Verbal is the spoken word or message. Intonation or tone of voice dramatically affects the meaning of a message. Vocabulary refers to words used for verbal communication. DIF: Understand REF: 275| 276 OBJ: Describe the basic elements of the communication process. TOP: Assessment MSC: NCLEX: Safe and Effective Care Environment 9. A patient has been admitted to the hospital numerous times. The nurse asks the patient to
share a personal story about the care that has been received. Which interaction is the nurse using? a. Narrative. b. Socializing. c. Nonjudgemental. d. SBAR. ANS: A
In a therapeutic relationship, nurses often encourage patients to share personal stories. Sharing stories is called narrative interaction. Socializing is an important initial component of interpersonal communication. It helps people get to know one another and relax. It is easy, superficial, and not deeply personal. Nonjudgemental acceptance of the patient, an important characteristic of the relationship, conveys a willingness to hear a message or acknowledge feelings; it is not a technique that involves personal stories. SBAR (situation, background, assessment, and recommendation) is a popular communication tool that helps standardize N R I G B.C M communication among health cUareSproN videTrs. O DIF: Remember REF: 278 OBJ: Identify significant features and therapeutic outcomes of nurse–patient helping relationships. TOP: Assessment MSC: NCLEX: Safe and Effective Care Environment 10. Before meeting the patient, a nurse talks to other caregivers about the patient. The nurse is in
which phase of the helping relationship? a. Preinteraction. b. Orientation. c. Working. d. Termination. ANS: A
Canadian Fundamentals of Nursing 6th Edition Potter Test Bank The time before the nurse meets the patient is called the preinteraction phase. This phase can involve such things as reviewing available data, including the medical and nursing history; talking to other caregivers who have information about the patient; and anticipating health concerns or issues that can arise. During the orientation phase, the nurse and the patient meet and get to know one another. This phase can involve such things as setting the tone for the relationship by adopting a warm, empathetic, caring manner; recognizing that the initial relationship is often superficial, uncertain, and tentative; or expecting the patient to test the nurse’s competence and commitment. In the working phase, the nurse and the patient work together to solve problems and accomplish goals. This phase can involve such things as encouraging and helping the patient express feelings about his or her health, encouraging and helping the patient with self-exploration, or providing information needed to understand and change behaviour. The termination phase occurs during the ending of the relationship. This phase can involve such things as reminding the patient that the relationship is about to end, evaluating goal achievement with the patient, or reminiscing about the relationship with the patient. DIF: Understand REF: 277 (Box 17-3) OBJ: List nursing focus areas within the four phases of a nurse–patient helping relationship. TOP: Assessment MSC: NCLEX: Safe and Effective Care Environment 11. During the initial home visit, a home health nurse lets the patient know that the visits are
expected to end in about a month. The nurse is in which phase of the helping relationship? a. Preinteraction. b. Orientation. c. Working. d. Termination.
NURSINGTB.COM
ANS: B
Letting the patient know when to expect the relationship to be terminated occurs in the orientation phase. Preinteraction occurs before the nurse meets the patient. In the working phase, the nurse and the patient work together to solve problems and accomplish goals. Termination occurs during the ending of the relationship. DIF: Apply REF: 277 (Box 17-3) OBJ: List nursing focus areas within the four phases of a nurse–patient helping relationship. TOP: Assessment MSC: NCLEX: Safe and Effective Care Environment 12. A nurse and patient take action to meet health-related goals. The nurse is in which phase of
the helping relationship? a. Preinteraction b. Orientation c. Working d. Termination ANS: C
The working phase occurs when the nurse and the patient work together to solve problems and accomplish goals. Preinteraction occurs before the nurse meets the patient. During the orientation phase, the nurse and the patient meet and get to know each other. Termination occurs during the ending of the relationship. DIF: Remember
REF: 277 (Box 17-3)
Canadian Fundamentals of Nursing 6th Edition Potter Test Bank OBJ: List nursing focus areas within the four phases of a nurse–patient helping relationship. TOP: Implementation MSC: NCLEX: Safe and Effective Care Environment 13. The nurse is in the process of conducting an admission interview with the patient. At one
point in the discussion, the patient has provided information that the nurse would like to clarify. Using the technique of clarification, how does the nurse respond? a. “I’m not sure that I understand what you mean by that statement.” b. “The electrocardiogram records information about your heart’s electrical activity.” c. “Let’s look at the problem you have had with your medication at home.” d. “What’s your biggest concern at the moment?” ANS: A
In clarifying, the nurse checks whether understanding is accurate by restating an unclear message to clarify the sender’s meaning or by asking the other person to restate the message, explain further, or give an example of what the person means. The response “I’m not sure that I understand what you mean by that statement” indicates that the nurse wants to clarify what the patient is saying so that he or she can have an accurate understanding of what the patient means. The statement “The electrocardiogram records information about your heart’s electrical activity” is an example of providing information, not clarification. The statement “Let’s look at the problem you have had with your medication at home” is an example of focusing, not clarification. The question “What’s your biggest concern at the moment?” is an example of expressing empathy, not of clarification. DIF: Apply REF: 284 OBJ: Discuss effective approaches to communicating with patients at various developmental levels. TOP: Planning MSC: NCLEX: Safe and Effective Care Environment 14. The patient draws back whenNtU heRnS urIseNrG eaTcB he. sC ovOeM r the side rails to take his blood pressure.
What should the nurse do first to promote effective communication? a. Tell the patient that the blood pressure can be taken at a later time. b. Rotate the nurses who are assigned to take the patient’s blood pressure. c. Continue to perform the procedure quickly and quietly. d. Apologize for startling the patient and explain the need for contact. ANS: D
Nurses often have to enter a patient’s personal space to provide care. The nurse should convey confidence, gentleness, and respect for privacy. Apologizing for startling the patient and explaining the need for contact demonstrates respect and provides information so the patient can understand the need for personal contact. Telling the patient that the blood pressure can be taken at a later time does not promote effective communication. Rotating the nurses who are assigned to take the patient’s blood pressure impedes the nurse’s ability to form a therapeutic, helping relationship. Continuing to perform the procedure quickly and quietly may send a negative nonverbal message to the patient. It also does not promote effective communication. DIF: Apply REF: 282-285 OBJ: Identify the practices important for relational inquiry. MSC: NCLEX: Safe and Effective Care Environment
TOP: Implementation
15. A patient has trouble speaking words, and the patient’s speech is garbled. Which nursing
diagnosis is most appropriate for this patient? a. Hopelessness.
Canadian Fundamentals of Nursing 6th Edition Potter Test Bank b. Impaired verbal communication. c. Hearing loss. d. Self-care deficit. ANS: B
A patient with impaired verbal communication has defining characteristics such as an inability to articulate words, inappropriate verbalization, difficulty forming words, and difficulty comprehending. Hopelessness implies that the patient has no hope for the future. Hearing loss is not a nursing diagnosis. Just because a patient has garbled speech does not mean that a hearing loss has occurred; a physical problem such as a stroke could cause speech to be garbled. Self-care deficit does not apply in this situation because this usually relates to bathing and grooming. DIF: Analyze REF: 286 OBJ: Identify patient health states and conditions that contribute to impaired communication. TOP: Diagnosis MSC: NCLEX: Safe and Effective Care Environment 16. Which person is the best referral for a patient who speaks a foreign language? a. A family member. b. A speech therapist. c. An interpreter. d. A mental health nurse specialist. ANS: C
The services of interpreters are often necessary for patients who speak a foreign language. Use of a family member to interpret can lead to legal issues; speech therapists help patients with aphasia; and mental health nurse specialists help angry or highly anxious patients communicate more effectivelN yU . RSINGTB.COM DIF: Understand REF: 275 OBJ: Discuss nursing care measures for patients with special communication needs. TOP: Planning MSC: NCLEX: Safe and Effective Care Environment 17. A nurse is using SOLER to facilitate active listening. Which technique should the nurse use
for “R”? a. Relax. b. Respect. c. Reminisce. d. Reassure. ANS: A
In SOLER, the “R” stands for “relax.” It is important to communicate a sense of being relaxed and comfortable with the patient. Active listening enhances trust because the nurse communicates acceptance and respect for the patient, but it is not the “R” in SOLER. Reminiscence is a therapeutic communication technique, especially when used with the elderly. Reassurance can be therapeutic if the nurse helps patients understand that there are many kinds of hope and that meaning and personal growth can come from illness experiences. However, false reassurance can block communication. DIF: Understand REF: 282 OBJ: Describe qualities, behaviours, and approaches that affect interprofessional communication. TOP: Implementation MSC: NCLEX: Safe and Effective Care Environment
Canadian Fundamentals of Nursing 6th Edition Potter Test Bank
18. An older patient is wearing a hearing aid. Which technique should the nurse use to facilitate
communication? a. Speak clearly and loudly. b. Turn off the television. c. Chew gum. d. Use at least 14-point print. ANS: B
Patients who are hearing impaired benefit when nurses use the following techniques: checking for hearing aids and glasses, reducing environmental noise, getting the patient’s attention before speaking, not chewing gum, and speaking at normal volume (not shouting). Using at least 14-point print is for patients who are sight/visually impaired, not hearing impaired. DIF: Apply REF: 281 (Box 17-6) OBJ: Discuss effective approaches to communicating with patients at various developmental levels. TOP: Implementation MSC: NCLEX: Safe and Effective Care Environment 19. When making rounds, the nurse finds a patient who is not able to sleep because of anticipation
of surgery in the morning. Which therapeutic response is most appropriate? a. “It will be okay. Your surgeon will talk to you in the morning.” b. “Why can’t you sleep? You have the best surgeon in the hospital.” c. “Don’t worry. The surgeon ordered a sleeping pill to help you sleep.” d. “It must be difficult not to know what the surgeon will find. What can I do to help?” ANS: D
“It must be difficult not to knN ow suB rg.eC onOw Rwha It NthGe T Mill find. What can I do to help?” is an U S example of therapeutic communication techniques of empathy and offering of self. False reassurances (“It will be okay” and “Don’t worry”) tend to block communication. Patients frequently interpret “why” questions as accusations or think the nurse knows the reason and is simply testing them. DIF: Apply REF: 282-285 OBJ: Describe qualities, behaviours, and approaches that affect interprofessional communication. TOP: Implementation MSC: NCLEX: Safe and Effective Care Environment 20. Which situation will cause the nurse to intervene and follow up on the unregulated care
provider’s behaviour? a. The unregulated care provider is calling the older patient “honey.” b. The unregulated care provider is facing the older patient when talking. c. The unregulated care provider cleans the older patient’s glasses. d. The unregulated care provider allows time for the older patient to respond. ANS: A
Health care providers should communicate with older persons on an adult level and avoid patronizing or speaking in a condescending manner. Terms of endearment such as “honey,” “dear,” “grandma,” or “sweetheart” should be avoided. Facing an older patient, making sure the older person has clean glasses, and allowing time to respond facilitate communication with older patients and should be encouraged, not stopped. DIF: Apply
REF: 279
Canadian Fundamentals of Nursing 6th Edition Potter Test Bank OBJ: Discuss effective approaches to communicating with patients at various developmental levels. TOP: Implementation MSC: NCLEX: Safe and Effective Care Environment 21. A confused older person is wearing thick glasses and a hearing aid. Which intervention is
priority to facilitate communication? a. Focus on tasks to be completed. b. Allow time for the patient to respond. c. Limit conversations with the patient. d. Use gestures and other nonverbal cues. ANS: B
Allowing time for the patient to respond will facilitate communication, especially for an older confused patient. Focusing on tasks to be completed and limiting conversations do not facilitate communication; in fact, they block communication. Using gestures and other nonverbal cues is not effective for visually impaired or cognitively impaired patients. DIF: Apply REF: 281 (Box 17-6) OBJ: Discuss effective approaches to communicating with patients at various developmental levels. TOP: Implementation MSC: NCLEX: Safe and Effective Care Environment 22. The staff is having a hard time getting an older person to communicate. Which technique
should the nurse suggest the staff use? a. Allow the patient to reminisce. b. Try changing topics often. c. Involve only the patient in conversations. d. Ask the patient for explanations. ANS: A
NURSINGTB.COM
Encouraging older persons to share life stories and reminisce about the past has a therapeutic effect and increases their sense of well-being. Sudden shifts from subject to subject should be avoided. It is helpful to include the patient’s family and friends and to become familiar with the patient’s favourite topics for conversation. Asking for explanations is a nontherapeutic technique. DIF: Apply REF: 278-281 OBJ: Discuss effective approaches to communicating with patients at various developmental levels. TOP: Planning MSC: NCLEX: Safe and Effective Care Environment 23. Which of the following patients will cause the greatest communication concerns for a nurse? a. A patient who is alert, has strong self-esteem, and is hungry. b. A patient who is oriented, pain free, and blind. c. A patient who is cooperative, depressed, and hard of hearing. d. A patient who is dyspneic, has a tracheostomy, and is anxious. ANS: D
Canadian Fundamentals of Nursing 6th Edition Potter Test Bank Facial trauma, laryngeal cancer, or endotracheal intubation often prevents movement of air past vocal cords or mobility of the tongue, which results in inability to articulate words. A person extremely short of breath needs to use oxygen to breathe rather than speak. Persons with high anxiety are sometimes unable to perceive environmental stimuli or hear explanations. People who are alert, have strong self-esteem, and are cooperative and pain free do not cause communication concerns. Hunger, blindness, and difficulty hearing individually can cause communication concerns, but the presence of dyspnea, a tracheostomy, and anxiety can affect communication. DIF: Evaluate REF: 279| 280 OBJ: Identify patient health states and conditions that contribute to impaired communication. TOP: Evaluate MSC: NCLEX: Safe and Effective Care Environment 24. A patient is aphasic, and the nurse notices that the patient’s hands shake intermittently. Which
nursing action is most appropriate to facilitate communication? a. Use a picture board. b. Use pen and paper. c. Use an interpreter. d. Use a hearing aid. ANS: A
Using a pen and paper can be frustrating for patient who is nonverbal (aphasic) and whose handwriting is shaky; the nurse can revise the care plan to include use of a picture board instead. An interpreter is used for a patient who speaks a foreign language. A hearing aid is used for patients who are hard of hearing, not for a patient who has aphasia. DIF: Apply REF: 286 OBJ: Discuss nursing care meN asU urR esSfoIr N paG tiT enB ts. wC ithOsM pecial communication needs. TOP: Implementation MSC: NCLEX: Safe and Effective Care Environment 25. What is characteristic of a nurse using critical thinking to enhance communication with
patients? a. Showing sympathy appropriately. b. Using automatic responses fluently. c. Self-examining personal communication skills. d. Demonstrating passive remarks accurately. ANS: C
Nurses who use critical thinking skills interpret messages received from others, analyze their content, make inferences about their meaning, evaluate their effects, explain rationales for communication techniques used, and self-examine personal communication skills. Sympathy is concern, sorrow, or pity felt for the patient and is nontherapeutic. Clichés and stereotyped remarks are automatic phrases that communicate that the nurse is not taking concerns seriously or responding thoughtfully. Passive responses serve to avoid conflict or to sidestep issues. DIF: Understand REF: 272 OBJ: Describe aspects of critical thinking that are important in the communication process. TOP: Implementation MSC: NCLEX: Safe and Effective Care Environment
Canadian Fundamentals of Nursing 6th Edition Potter Test Bank 26. A patient says, “You are the worst nurse I have ever had.” Which response by the nurse is the
most assertive? a. “If I were you, I’d feel grateful for a nurse like me.” b. “I feel uncomfortable hearing that statement.” c. “How can you say that when I have been checking on you regularly?” d. “You shouldn’t say things like that, it is not right.” ANS: B
Assertive responses contain “I” messages such as “I want,” “I need,” “I think,” or “I feel.” Giving personal opinions (“If I were you”), arguing (“How can you say that?”), and showing disapproval (using words such as should, good, bad, right) are not assertive or therapeutic. DIF: Apply REF: 279 OBJ: Describe qualities, behaviours, and approaches that affect interprofessional communication. TOP: Implementation MSC: NCLEX: Safe and Effective Care Environment MULTIPLE RESPONSE 1. Which critical thinking standards should the nurse use to ensure sound, effective
communication with patients? (Select all that apply.) a. Faith. b. Supportiveness. c. Self-confidence. d. Humility. e. Independent attitude. f. Spiritual expression. ANS: C, D, E
NURSINGTB.COM
A self-confident attitude is important because the nurse who conveys confidence and comfort while communicating can more readily establish an interpersonal helping-trusting relationship. In addition, an independent attitude encourages the nurse to communicate with colleagues and share ideas about nursing interventions. An attitude of humility is necessary to recognize and communicate the need for more information before the nurse can make a decision. Faith, supportiveness, and spiritual expression are attributes of caring, not critical thinking standards. DIF: Understand REF: 272 OBJ: Describe aspects of critical thinking that are important in the communication process. TOP: Implementation MSC: NCLEX: Safe and Effective Care Environment 2. Which types of patients can cause challenging communication situations? (Select all that
apply.) a. A male patient who is cooperative with treatments. b. A female patient who is outgoing and flirty. c. An older patient who is demanding. d. An older patient who can see small print clearly. e. A teenager frightened by the prospect of impending surgery. f. A child who is developmentally delayed. ANS: B, C, E, F
Canadian Fundamentals of Nursing 6th Edition Potter Test Bank Conditions that cause challenging communication situations include flirtatiousness, being demanding, fear, or developmental delay. A child who has received little environmental stimulation may be behind in language development, which would make communication more challenging. Patients who are cooperative and those who have good eyesight (see small print) do not cause challenging communication situations. DIF: Understand REF: 281| 282 OBJ: Identify patient health states and conditions that contribute to impaired communication. TOP: Assessment MSC: NCLEX: Safe and Effective Care Environment
NURSINGTB.COM
Canadian Fundamentals of Nursing 6th Edition Potter Test Bank
Chapter 18: Patient-Centred Care: Interprofessional Collaborative Practice Potter et al: Canadian Fundamentals of Nursing, 6th Edition MULTIPLE CHOICE 1. A nurse is describing the antibiotic she is about to administer through a patient’s central line.
The patient accepts what the nurse is saying and is able to repeat it back. The patient does not internalize this information or ask questions. This is an example of which of the five ways of knowing? a. Constructed knowing. b. Received knowing. c. Subjective knowing. d. Perceived knowing. ANS: B
Received knowing refers to when a patient defers to the nurse and accepts without debate what the nurse teaches him or her. Constructive knowing refers to when knowledge is received and the patient compares this knowledge against his or her assumptions, challenging his or her own assumptions and those of others. Subjective knowing refers to when the patient listens to what the nurse is explaining and internalizes the meaning. Perceived knowing refers to when the patient receives the knowledge and considers how it relates to what he or she already understands. DIF: Apply REF: 292| 293 OBJ: Explore your own ways of learning and how these can be integrated to support your learning of how to be a professional nurse and provide care to patients. TOP: Assessment N e RCS INnvi GTronment B.COM MSC: NCLEX: Safe and EffectivU are E 2. Which of the following patterns of knowing relates to how a nurse, as a unique individual,
chooses to respond in a situation involving a patient? a. Empirical knowing. b. Personal knowing. c. Aesthetic knowing. d. Ethical knowing. ANS: C
Aesthetic knowing relates to how a nurse, as a unique individual, chooses to respond in a situation involving a patient. Empirical knowing is related to the knowledge that is applied to such situations. Personal knowing reflects the nursing understanding accumulated from previous experiences. Ethical knowing is shaped by the nurse’s own values. DIF: Understand REF: 293 OBJ: Explore your own ways of learning and how these can be integrated to support your learning of how to be a professional nurse and provide care to patients. TOP: Assessment MSC: NCLEX: Safe and Effective Care Environment 3. A nurse is planning care for a patient on the basis of Orlando’s nursing process. The nursing
process occurs in which of the following orders? a. Perception, feeling, thought, action. b. Feeling, thought, perception, action.
Canadian Fundamentals of Nursing 6th Edition Potter Test Bank c. Thought, feeling, action, perception. d. Perception, thought, feeling, action. ANS: D
The nursing process occurs in the following order: perception, thought, feeling, and action. DIF: Understand REF: 295| 296 OBJ: Gain an understanding of the frameworks that provide guidance to what nursing professional practice entails. TOP: Planning MSC: NCLEX: Safe and Effective Care Environment 4. A nurse determines that a patient is in pain and decides to give an analgesic medication that
has been ordered. The nurse then goes back to assess the patient’s pain after 30 minutes. The act of administering the analgesic medication is best described as which stage of the nursing process? a. Assessment. b. Planning. c. Intervention. d. Evaluation. ANS: C
The act of administering the analgesic medication is best described as the intervention stage. The nurse’s determining that the patient is in pain would be described as assessment; the nurse’s decision to give the analgesic reflects planning; and evaluation refers to the nurse’s returning to reassess the patient’s pain. DIF: Apply REF: 296 OBJ: Gain an understanding of the frameworks that provide guidance to what nursing professional practice entails. TOP: PlN anniR ng I GMSB C:.C NCLMEX: Safe and Effective Care Environment
U S N T
O
5. What would be the best question to ask a patient in the noticing phase of the Clinical
Judgement Model? a. “Where is the pain the worst?” b. “How are you feeling today?” c. “How was your night?” d. “What are your goals for today?” ANS: D
The most critical information to obtain from a patient is what his or her goals are. This information will help the nurse focus further exploration of the patient’s goals and plan around the patient’s goals. The other questions will elicit important information but are not the considered the most critical questions in this phase. DIF: Apply REF: 296 OBJ: Gain an understanding of the frameworks that provide guidance to what nursing professional practice entails. TOP: Assessment MSC: NCLEX: Safe and Effective Care Environment 6. A nurse gathers information through the noticing phase of the Clinical Judgement Model. The
nurse is interpreting this information by using experience, expertise, and knowledge, along with personality and environment. This is best described as which of the following? a. Intuitive interpreting. b. Narrative interpreting. c. Experiential interpreting.
Canadian Fundamentals of Nursing 6th Edition Potter Test Bank d. Analytic interpreting. ANS: A
Intuitive interpreting is described when a nurse applies his or her experience, expertise, and knowledge, along with personality and environment, to understand information. Narrative interpreting involves trying to understand the particular care and is viewed as human beings’ primary way of making sense of experience, through an interpretation of human concerns, intents, and motive. Analytic interpreting involves the integration of information gathered through the noticing phase along with the patient’s goals. Experiential interpreting is not included as a type of interpretation in this model. DIF: Understand REF: 297 OBJ: Gain an understanding of the frameworks that provide guidance to what nursing professional practice entails. TOP: Planning MSC: NCLEX: Safe and Effective Care Environment 7. Who is the expert in the patient’s management of illness? a. The nurse. b. The physician. c. The patient. d. The interprofessional team. ANS: C
Patients are the experts in their management of illness. The nurse, physician, and interprofessional team are all important partners in care. DIF: Understand REF: 298 OBJ: Gain an understanding of the frameworks that provide guidance to what nursing professional practice entails. TOP: AN ssessRmenIt GMSB C:.C NCLMEX: Safe and Effective Care Environment
U S N T
O
8. What is the cause of internalized myths that nurses may have about their own profession and
those of other health care providers? a. Role ambiguity. b. Media portrayal. c. Familial experiences. d. Socialization. ANS: D
Socialization is the most encompassing cause of internalized myths nurses may have of their own profession and those of other health care providers. Role ambiguity may contribute to these myths but is not a cause. Media portrayal and familial experiences are parts of socialization, contributing to the myths. DIF: Understand REF: 298| 299 OBJ: Explore how to integrate your growing nursing professional identity with that of an interprofessional identity. TOP: Evaluate MSC: NCLEX: Safe and Effective Care Environment 9. To clarify the role of a nurse in patient-centred care, a nurse is responsible for which of the
following? a. Asking other interprofessional team members about their roles. b. Shadowing other interprofessional team members to experience their roles. c. Being able to explain own role and the knowledge and skills that the nurse brings
Canadian Fundamentals of Nursing 6th Edition Potter Test Bank to patient-centred care. d. Being able to explain the roles of all interprofessional team members. ANS: C
Role clarification requires a nurse to be able to explain what a professional nurse’s role is and what knowledge and skills the nurse brings to patient-centred care. Asking other interprofessional team members about their roles, shadowing other interprofessional team members, and being able to explain the roles of all interprofessional team members contribute to role clarification but are not the nurse’s main responsibility when clarifying his or her own role. DIF: Understand REF: 299 OBJ: Explore how as a beginning nurse professional you can work within interprofessional teams. TOP: Implementation MSC: NCLEX: Safe and Effective Care Environment 10. A nurse is caring for six patients on a postsurgical unit. She hears a code blue being called on
one of her patients, and she runs to the patient’s room. When the nurse arrives, a code leader has been established. The first thing the nurse should do is which of the following? a. Introduce herself as the patient’s nurse, offering her specific knowledge and skills. b. Immediately jump in and assist with the airway. c. Stand back, observe the situation and wait to be given a role by the code leader. d. Acknowledge that the situation is under control and go back to her other five patients. ANS: A
The nurse should introduce herself, provide relevant information, and assist in role clarification by explaining the skills she can contribute, such as initiation of intravenous (IV) treatment or medication admN inis trati on.NG ToTfBu. ncC tion UR SI OMbest as a group, the team members need to be aware of each other’s roles. If the nurse were to jump in and assist with the airway without introducing herself, the team may not know who she is or what her role is. The nurse is not being an active team member by standing back and waiting for a role or by leaving the room during the code. It is her responsibility as the patient’s primary nurse to provide the information and skills she has to offer in this situation. DIF: Apply REF: 300 OBJ: Explore how as a beginning nurse professional you can work within interprofessional teams. TOP: Implementation MSC: NCLEX: Safe and Effective Care Environment 11. At morning rounds, a nurse explains the change in status that her patient had overnight. The
interprofessional team members all recognize that this nurse has been working in the unit for more than 10 years and is extremely competent at assessment and interpretation. This recognition influences how the team is interpreting the nurse’s report. Which aspect is being described? a. Affinity. b. Immediacy. c. Respect. d. Control. ANS: A
Canadian Fundamentals of Nursing 6th Edition Potter Test Bank Affinity refers to how the nurse is viewed on the basis of perceived competence, which influences how the team interprets the nurse’s report. Immediacy refers to the urgency of the message the nurse is relaying and was not alluded to in the question. Respect refers to trust gained over time through experience interacting with the team. Control refers to power differentials and was not alluded to in the question. DIF: Apply REF: 301 OBJ: Explore how as a beginning nurse professional you can work within interprofessional teams. TOP: Implementation MSC: NCLEX: Safe and Effective Care Environment 12. When communicating with other members of the interprofessional team, the nurse should
explain which of the following? a. Situation, background, assessment, and reassessment. b. Situation, background, assessment, and recommendations. c. Subjective information, behaviour, assessment, and recommendations. d. Socialization, behaviour, action, and reaction. ANS: B
The acronym SBAR stands for situation, background, assessment, and recommendations, all of which should be explained by the nurse when interacting with the interprofessional team. DIF: Understand REF: 301 OBJ: Explore how as a beginning nurse professional you can work within interprofessional teams. TOP: Implementation MSC: NCLEX: Safe and Effective Care Environment 13. A patient undergoing chemotherapy visits with his interprofessional team once a month. He
observes the oncologist encouraging other team members, such as the nurse, the pharmacist, or the social worker, to take oNnUthReSleIaN dG roT leBd. urC inO gM the meetings, on the basis of the patient’s concerns and goals. Which leadership element is being demonstrated by the oncologist? a. Inspiring a shared vision. b. Enabling others to act. c. Challenging the process. d. Encouraging the heart. ANS: B
The oncologist is demonstrating the leadership element of enabling others to act by encouraging other members to take on the leadership role and support patients in their decision making with the team. Inspiring a shared vision would be demonstrated by having the members focus on patient-specific goals and helping bring the ideas together in an agreed-upon plan with the patient. Challenging the process would be demonstrated by reflecting on how the team is working together and making needed changes. Encouraging the heart would be demonstrated by recognizing the positive work of all team members, including the patient, toward meeting patient-set goals. DIF: Apply REF: 303 (Box 18-4) OBJ: Explore how as a beginning nurse professional you can work within interprofessional teams. TOP: Evaluate MSC: NCLEX: Safe and Effective Care Environment 14. The Canadian Interprofessional Health Collaborative (CIHC) has four key competencies for
interprofessional collaboration. They include role clarification, team functioning, collaborative leadership, and which of the following?
Canadian Fundamentals of Nursing 6th Edition Potter Test Bank a. b. c. d.
Patient-centred care. Quality improvement. Dealing with interprofessional conflict. Reflection.
ANS: C
The four key competencies for the CIHC National Interprofessional Competency Framework are role clarification, team functioning, collaborative leadership, and dealing with interprofessional conflict. Patient-centred care, quality improvement, and reflection are not key competencies in this framework. DIF: Understand REF: 300 (Fig. 18-4) OBJ: Explore how as a beginning nurse professional you can work within interprofessional teams. TOP: Assessment MSC: NCLEX: Safe and Effective Care Environment 15. Which way of knowing is primarily affected by the ideals that are significant or important to
the person, the family, or the community? a. Empirical knowing. b. Aesthetic knowing. c. Personal knowing. d. Ethical knowing. ANS: D
Ethical knowing is shaped by values, which are the ideals that have significant meaning or importance to the person, the family, or the community. Aesthetic knowing concerns how the person, as a unique individual, chooses to respond in a situation. Empirical knowing is related to the knowledge gained that can be applied to specific situations. Personal knowing reflects the understanding accumulatN edUfR roS mIpN reG viT ouBs.eC xpOeM riences. DIF: Understand REF: 293 OBJ: Explore your own ways of learning and how these can be integrated to support your learning of how to be a professional nurse and provide care to patients. TOP: Assessment MSC: NCLEX: Safe and Effective Care Environment MULTIPLE RESPONSE 1. A nurse is caring for a patient who has been admitted after a hip replacement. The responding
phase in the Clinical Judgement Model would include which of the following? (Select all that apply.) a. Developing actions to address the patient’s health and social issues. b. Planning implementation for the actions to address the patient’s health and social issues. c. Seeking the patient’s agreement to the interventions. d. Consider the evidence necessary to determine whether certain goals have been reached. e. Assessing the patient’s response during care. f. Reflecting on the care that was provided. ANS: A, B, C
Canadian Fundamentals of Nursing 6th Edition Potter Test Bank Responding means developing actions and planning implementation to address a patient’s identified health and social issues. It also includes discussing interventions with the patient and seeking agreement to them. The patient and nurse work together to consider the evidence needed to determine whether the goals have been reached. Assessing the patient’s response during care and reflecting on the care provided both are included in the reflecting phase of the Clinical Judgement Model. DIF: Apply REF: 298 OBJ: Gain an understanding of the frameworks that provide guidance to what nursing professional practice entails. TOP: Implementation MSC: NCLEX: Safe and Effective Care Environment 2. A nurse calls the physician and says, “Dr. Green, the patient LT is complaining of increased
pain to his left leg, rating it 8/10. LT is the 8-year-old boy in room 12 who broke his femur in a snowboarding accident yesterday. His pain has been well controlled at 2 to 4/10 for the past day, but his analgesic dose was weaned after rounds this morning. His foot does not appear to be swollen below the cast, and his perfusion is okay with palpable pedal pulses and a capillary refill of 2-3 seconds to the toes on the left foot.” Which elements of SBAR communication did the nurse include? (Select all that apply.) a. Situation. b. Subjective information. c. Background. d. Assessment. e. Recommendation. f. Reassessment. ANS: A, C
NURSINGTB.COM
The communication elements of SBAR include: situation, background, assessment and recommendation. The nurse explained the situation by saying, “The patient LT is complaining of increased pain to his left leg.” The background was described by “LT is the 8-year-old boy in room 12 who broke his femur in a snowboarding accident yesterday. His pain has been well controlled at 2 to 4/10 for the past day, but his analgesic dose was weaned after rounds this morning.” Assessment was described by “His foot does not appear to be swollen below the cast, and his perfusion is okay with palpable pedal pulses and a capillary refill of 2-3 seconds to the toes on the left foot.” The nurse did not include a recommendation. Subjective information and reassessment are not elements of SBAR. DIF: Analyze REF: 301 OBJ: Explore how as a beginning nurse professional you can work within interprofessional teams. TOP: Assessment MSC: NCLEX: Safe and Effective Care Environment
Canadian Fundamentals of Nursing 6th Edition Potter Test Bank
Chapter 19: Family Nursing Potter et al: Canadian Fundamentals of Nursing, 6th Edition MULTIPLE CHOICE 1. What is an example of circular communication patterns between father and child? a. The father and child are engaged in a game of catch. b. The father uses rewards to encourage the child to go to bed. c. The father comforts the crying child. d. The child disobeys his father. ANS: C
Circular communication refers to communication between family members that is reciprocal; that is, each person influences the behaviour of the other. An example is when a parent comforts a child because the child cries. Because the parent responds to the child, the child feels safe and secure. The example of the father and child being engaged in a game of catch is not an example of circular communication. It reflects a subsystem (structural assessment) of father and child; it reflects a relationship within the family. The example of the father using rewards to encourage the child to go to bed is not an example of circular communication. It illustrates how influence (expressive functioning) is used to affect or control another person’s behaviour. Influence can be classified as instrumental (e.g., using privileges to reward good behaviour), psychological (e.g., giving praise or admonishment), or corporal (e.g., hugging or hitting). The example of the child disobeying his father is not an example of circular communication. Rather, it refers to a boundary. DIF: Analyze TOP: Assessment
REF: 31N 7 R I GOBB J:.DefC iM ne the key terms listed. U S N T O MSC: NCLEX: Safe and Effective Care Environment
2. Balancing employment and family life creates a variety of challenges in terms of child care
and household work for both parents. Why does this have major implications in health care? a. Maternal employment has been demonstrated to be harmful for children. b. Maternal employment has shifted the majority of household tasks to the male. c. Fathers now participate more fully in day-to-day parenting responsibilities. d. The number of single-parent families has decreased since 1970. ANS: C
There is no proof that maternal employment is damaging for children. However, finding quality child care is a major issue. Managing household tasks is another challenge. Although equal division of labour receives verbal approval, most household tasks remain “women’s work.” Evidence suggests that the fathering role is changing. Fathers now participate more fully in day-to-day parenting responsibilities. The number of single-parent families, which doubled from the 1940s to the 1990s, seems to be stabilizing. DIF: Understand REF: 309 OBJ: Discuss how family members influence one another's health. TOP: Diagnosis MSC: NCLEX: Safe and Effective Care Environment
Canadian Fundamentals of Nursing 6th Edition Potter Test Bank 3. A husband brings his children in to visit their mother in the hospital. The nurse asks how the
family is getting along at home without their mom around. The husband states, “None of her jobs are getting done, and I don’t do those jobs, so the house and the kids are falling apart.” The nurse suspects that this family structure is which of the following? a. Very flexible. b. Quite rigid. c. Extremely open. d. Hardy. ANS: B
A rigid structure specifically dictates who is able to accomplish a task and may limit the number of people outside the immediate family who assume these tasks. A crisis-proof, or effective, family can integrate the need for stability with the need for growth and change; has a flexible structure for task performances; and is receptive to help from outside the family system while maintaining a sense of strength and stability. An effective family exerts influence on the immediate environment of home, neighbourhood, and school, whereas a crisis-prone family may lack, or believe it lacks, control over these environments. An extremely open or very flexible structure also presents problems for the family in that consistent patterns of behaviour do not exist and enactment of roles is overly flexible. Hardiness is the internal strength and durability of the family unit characterized by a sense of control over the outcome of life and an active rather than passive orientation in adapting to stressful events. DIF: Apply REF: 314 OBJ: Discuss how family members influence one another's health. TOP: Implementation MSC: NCLEX: Health Promotion and Maintenance
N R I G B.C M
4. It is essential for family membU ers tS o reN alizT e that aOfamily’s beliefs, values, and practices
strongly influence the health-promoting behaviours of its members, and to understand which of the following? a. Canadian families are part of the same culture with the same values and beliefs. b. Economic status has little effect on a family’s ability to access adequate health care. c. Family environment in early life has a strong influence on later health practices. d. All families place a high value on good health and health practices. ANS: C
Family environment is crucial because health behaviour reinforced early in life has a strong influence on later health practices. Some families do not place a high value on good health; in fact, some families accept harmful practices. Although Canadian families exist within the same culture, they live in very different ways as a result of race, values, social class, and ethnicity. Economic stability increases a family’s access to adequate health care. DIF: Understand REF: 310 OBJ: Discuss how family members influence one another's health. TOP: Assessment MSC: NCLEX: Health Promotion and Maintenance 5. In an assessment for signs of a healthy family, what would the nurse would expect to find? a. Change is viewed as detrimental to family processes. b. The response to stressors is passive. c. The structure is flexible enough to adapt to crises.
Canadian Fundamentals of Nursing 6th Edition Potter Test Bank d. Minimal influence is exerted on the environment. ANS: C
A healthy family has a flexible structure that allows adaptable performance of tasks and acceptance of help from outside the family system. The structure is flexible enough to allow adaptability but not so flexible that the family lacks cohesiveness and a sense of stability. The healthy family is able to integrate the need for stability with the need for growth and change. It does not view change as detrimental to family processes. The healthy family demonstrates control over the environment and does not passively respond to stressors. The healthy family exerts influence on the immediate environment of home, neighbourhood, and school. DIF: Evaluate REF: 310 OBJ: Discuss how family members influence one another's health. TOP: Evaluate MSC: NCLEX: Health Promotion and Maintenance 6. An 86-year-old diabetic patient who requires daily insulin injections lives with his daughter
and her husband. To assess instrumental functioning, what would the nurse ask? a. “Mr. Casey, how do you think your daughter feels about your refusing to take your insulin?” b. “What do you feel has aggravated your difficulty in controlling the diabetes?” c. “Mr. Casey, who is responsible for administering your insulin injections?” d. “How will you deal with hypoglycemic reactions?” ANS: C
Instrumental functioning includes all normal activities of daily living, such as preparing meals, eating, sleeping, and attending to health needs. For families with health problems, these activities may include administration of medications. The question “How do you think your daughter feels about youNrUreRfus ing toTtaBk. eC you r insulin?” explores expressive SI NG OM functioning, particularly emotional communication. The question “What do you feel has aggravated your difficulty in controlling the diabetes?” explores the patient’s beliefs within the category of expressive functioning. The question “How will you deal with hypoglycemic reactions?” evaluates the family’s ability to solve problems; problem solving is in the category of expressive functioning. DIF: Apply REF: 316 OBJ: Ask assessment questions to learn relevant information about family functioning in the context of health or illness. TOP: Implementation MSC: NCLEX: Health Promotion and Maintenance 7. When initiating the care of families, what is one factor that helps organize the family approach
to the nursing process? a. The view of all patients as unique individuals. b. The realization that families have little effect on individuals. c. The realization that individuals have little effect on families. d. The realization that individuals have an effect on families. ANS: D
A change in one family member, such as an illness or health condition, affects the other family members. Family nursing promotes, supports, and provides for the well-being and health of the family and individual family members. DIF: Understand
REF: 311| 312
Canadian Fundamentals of Nursing 6th Edition Potter Test Bank OBJ: Compare family as context, family as patient and family in context, explaining how these different perspectives influence nursing practice with families. TOP: Assessment MSC: NCLEX: Safe and Effective Care Environment 8. The nurse is interviewing a patient who is being admitted to the hospital. The patient’s family
went home before the nurse’s interview. The nurse asks the patient, “Who decides where to go on vacation?” In asking this, what is the nurse trying to do? a. Assess the family structure. b. Assess the family form. c. Assess the family function. d. Make a categorical generalization. ANS: A
To assess the family structure, the nurse asks questions that determine the power structure and patterning of roles and tasks (e.g., “Who decides where to go on vacation?”). When focusing on family form, the nurse should begin the family assessment by determining the patient’s definition of family. Family function is the ability of the family to provide emotional support and to cope with health problems or situations. The question asked above will not help assess that. It is imperative to remember that categorical generalizations are misleading and should be avoided. DIF: Apply REF: 313-315 OBJ: Ask assessment questions to learn relevant information about family functioning in the context of health or illness. TOP: Implementation MSC: NCLEX: Safe and Effective Care Environment 9. When focusing on older persons, the nurse must be aware of which of the following? a. Elder abuse happens onlyNiU nR loS wI erNsoGcT ioB ec.oC noOmMic classes. b. Older persons have the same social networks as do younger people. c. Caregivers may be spouses or middle-aged children. d. Caregiver stress is minimal in the care of a parent. ANS: C
Caregivers are typically spouses, who may be older persons with declining physical stamina, or middle-aged children, who often have other responsibilities. Abuse of older persons in families occurs across all social classes. Later-life families have a different social network than younger families because friends and same-generation family members often have died or have been ill themselves. The nurse should assess for caregiver stress such as tension in relationships between family and care recipient, changes in level of health, changes in mood, and anxiety and depression. DIF: Remember REF: 310 OBJ: Compare family as context, family as patient and family in context, explaining how these different perspectives influence nursing practice with families. TOP: Assessment MSC: NCLEX: Safe and Effective Care Environment 10. When the nurse views the family as context, the primary focus is on the health and
development of an individual member existing within a specific environment (i.e., the patient’s family). Although the focus is on the individual’s health status, the nurse should do which of the following? a. Assess how much the family provides the patient’s basic needs.
Canadian Fundamentals of Nursing 6th Edition Potter Test Bank b. Assess family patterns versus individual characteristics. c. Maintain distinctions between “family as patient” and “family as context.” d. Plan care to meet not only the patient’s needs but also those of the patient’s family. ANS: A
When the nurse views the family as context, the primary focus is on the health and development of an individual member existing within a specific environment (i.e., the patient’s family). Although the focus is on the individual’s health status, the nurse assesses how much the family provides the individual’s basic needs. Family patterns are in the realm of “family as patient.” It is important to understand that although the nurse is able to make theoretical and practical distinctions between “family as context” and “family as patient,” they are not necessarily mutually exclusive. Often, the nurse will use the two simultaneously, as with the perspective of “family as system.” “Family as patient” involves planning to meet the needs of the patient and those of his or her family as well. DIF: Understand REF: 311 OBJ: Compare family as context, family as patient and family in context, explaining how these different perspectives influence nursing practice with families. TOP: Assessment MSC: NCLEX: Health Promotion and Maintenance 11. Mrs. Lily is talking about her recent diagnosis of a chronic condition and how it has affected
her emotionally, intellectually, socially, and spiritually. The nurse knows that this is referred to as which of the following? a. Reflection. b. Illness narrative. c. Comprehensive honesty. d. Complete life review. ANS: B
NURSINGTB.COM
An illness narrative is a patient’s story of how illness affects his or her whole being, including emotional, intellectual, social, and spiritual components. Reflection, comprehensive honesty, and a complete life review do not refer to these components. DIF: Understand REF: 320 TOP: Implementation
OBJ: Discuss family nursing as relational inquiry. MSC: NCLEX: Health Promotion and Maintenance
12. The nurse is caring for an older patient who apparently has no family. When questioned about
his family and his definition of family, the patient states, “I have no family. They’re all gone.” When asked, “Who prepares your meals?” he states, “I do, or I go out.” Given the three different approaches to family nursing practice, which would be most appropriate for this patient? a. Family as context. b. Family as patient. c. Family as system. d. Combination of “family as context” and “family as patient.” ANS: A
Canadian Fundamentals of Nursing 6th Edition Potter Test Bank If only one family member receives nursing care, it is realistic and practical to view the “family as context.” Although family nursing is based on the assumption that all people regardless of age are a member of some type of family form, the patient insists that he has no family. The nurse should investigate further. However, at this time, family as patient or as system, or a combination of these, is not appropriate. DIF: Analyze REF: 311 OBJ: Compare family as context, family as patient and family in context, explaining how these different perspectives influence nursing practice with families. TOP: Evaluate MSC: NCLEX: Health Promotion and Maintenance 13. The nurse is caring for an older woman and notices that she is not using her cane properly.
Which of the following statements by the nurse would most likely elicit a positive response from the patient? a. “You’re doing that all wrong. Let me show you how to do it.” b. “I don’t know who showed you how to use the cane like that, but you’re not doing it right. Let me show you again.” c. “You use the cane the way I did before I was shown a way to keep from tripping over it; do you mind if I show you?” d. “I used to use the cane the same way you are using it: the wrong way. I’ll show you the right way to do it.” ANS: C
When the nurse is respectful instead of coming across as an authority on the subject, the patient’s defences will be down, making the patient more willing to listen without feeling embarrassed. DIF: Understand REF: 31N 9URSINGTB.COM OBJ: Discuss the relational approaches and practices needed to conduct a family interview and effectively intervene with the family in nursing practice. TOP: Implementation MSC: NCLEX: Safe and Effective Care Environment 14. The nurse is providing discharge teaching for an older woman who will need dressing changes
at home. Her husband, who is also an older person, is her only source of care. The husband states that he will not be able to perform the dressing changes. What does the nurse need to arrange for? a. Home care service referrals. b. Extra dressing supplies. c. Cancellation of the discharge. d. An order for antibiotics. ANS: A
Discharge planning with a family involves an accurate assessment of what will be needed for care at the time of discharge, along with any shortcomings in the home setting. If no one can change the dressings properly, the nurse must arrange for a home care service referral. Extra dressing supplies will not help the situation if the husband cannot use them. Only under extreme situations or in an unsafe situation will the discharge be cancelled. An order for antibiotics is not a replacement for good dressing change technique. DIF: Apply REF: 311| 316 OBJ: Discuss the relational approaches and practices needed to conduct a family interview and
Canadian Fundamentals of Nursing 6th Edition Potter Test Bank effectively intervene with the family in nursing practice. MSC: NCLEX: Health Promotion and Maintenance
TOP: Implementation
MULTIPLE RESPONSE 1. Sociocultural and economic changes facing the family in today’s Canadian society are
increasing challenges to family health. Of the choices below, what are some of these changes? (Select all that apply.) a. Learning disabilities. b. Addictions. c. Family violence. d. Increase in number of families headed by same-sex couples. e. Dementia. ANS: A, B, C, E
Sociocultural and economic changes are significantly increasing challenges to family health. Many families find themselves unable to respond effectively to complex family challenges (e.g., violence, addictions, illness, learning disabilities, cognitive decline, or dementia). Many same-sex couples define their relationship in family terms, but families headed by same-sex couples are not considered a threat facing the family. DIF: Understand REF: 309 OBJ: Compare family as context, family as patient and family in context, explaining how these different perspectives influence nursing practice with families. TOP: Diagnosis MSC: NCLEX: Health Promotion and Maintenance 2. Although the family as a whole differs from individual members, the measure of family health
N R I G B.C M
is more than a summary of the U heaS lth oNf alT l membO ers. Of the following, what areas are unique to family assessment in the Calgary Family Assessment Model (CFAM)? (Select all that apply.) a. Developmental. b. Structural. c. Functional. d. Family health. e. Individual health. ANS: A, B, C
The CFAM is used to complete family assessment in developmental, structural, and functional areas. Although individual health is important, it varies from the family focus. DIF: Remember REF: 313 OBJ: State the three major categories of the Calgary Family Assessment Model (CFAM) and understand subcategories important to consider in a family assessment. TOP: Assessment MSC: NCLEX: Health Promotion and Maintenance
Canadian Fundamentals of Nursing 6th Edition Potter Test Bank
Chapter 20: Patient Education Potter et al: Canadian Fundamentals of Nursing, 6th Edition MULTIPLE CHOICE 1. A nurse is asked about the goal of patient education. What is the nurse’s best response? a. “The goal of educating others is to help people meet standards of the Nurse
Practice Act.” b. “The goal of educating others is to help people achieve optimal levels of health.” c. “The goal of educating others is to help people become dependent on the health care team.” d. “The goal of educating others is to help people provide self-care only while they are in the hospital.” ANS: B
The goal of educating people about their health is to help individuals, families, and communities achieve optimal levels of health. Although all provincial and territorial Nurse Practice Acts acknowledge that patient teaching falls within the scope of nursing practice, this is the nurse’s standard, not the goal of education. Patient education helps patients make informed decisions about their care and become healthier and more independent, not dependent. Nurses provide patients with information needed for self-care to ensure continuity of care from the hospital to the home, not only in the hospital. DIF: Remember REF: 325 TOP: Implementation
OBJ: Describe the purposes of patient education. MSC: NCLEX: Health Promotion and Maintenance
N R I G B.C M
2. A nurse is teaching a group of U healS thyN aduT lts abouOt the benefits of flu immunizations. Which
purpose of patient education is the nurse fulfilling? a. Restoration of health. b. Coping with impaired functions. c. Promotion of health and illness prevention. d. Health analogies. ANS: C
A nurse is a visible, competent resource for patients who want to improve their physical and psychological well-being. In the school, home, clinic, or workplace, nurses promote health and prevent illness by providing information and skills that enable patients to assume healthier behaviours. Injured and ill patients need information and skills to help them regain or maintain their level of health; this is referred to as restoration of health. Not all patients fully recover from illness or injury. Many have to learn to cope with permanent health alterations; this is known as coping with impaired functions. Analogies supplement verbal instruction with familiar images that make complex information more real and understandable. For example, when explaining arterial blood pressure, the nurse can use the analogy of the flow of water through a hose. DIF: Understand TOP: Planning
REF: 325 OBJ: Describe the purposes of patient education. MSC: NCLEX: Health Promotion and Maintenance
3. A nurse provides teaching about coping with long-term impaired functions. Which situation
serves as the best example?
Canadian Fundamentals of Nursing 6th Edition Potter Test Bank a. Teaching a family member to give medications through the patient’s permanent
gastric tube. b. Teaching a woman who recently had a hysterectomy about her pathology reports. c. Teaching expectant parents about physical and psychological changes in
childbearing women. d. Teaching a teenager with a broken leg how to use crutches. ANS: A
Not all patients fully recover from illness or injury. Many have to learn to cope with permanent health alterations. New knowledge and skills are often necessary for patients and family members to continue activities of daily living. Teaching family members to help the patient with health care management (e.g., giving medications through gastric tubes, doing passive range-of-motion exercises) is an example of teaching them how to cope with long-term impaired functions. Injured and ill patients need information and skills to help them regain or maintain their levels of health. Some examples of this include teaching a woman who recently had a hysterectomy about her pathology reports and expected length of recovery and teaching a teenager with a broken leg how to use crutches. In childbearing classes, expectant parents are taught about physical and psychological changes in the woman and about fetal development; this is part of health maintenance. DIF: Understand REF: 326 OBJ: Identify appropriate topics that address a patient’s health education needs. TOP: Evaluate MSC: NCLEX: Health Promotion and Maintenance 4. Which statement indicates that the nurse has a good understanding of teaching/learning? a. “Teaching and learning can be separated.” b. “Learning is an interactive process that promotes teaching.” N scio RSus, INdeli GTberate B.CsOetMof actions designed to help the c. “Learning consists of a conU
teacher.”
d. “Teaching is most effective when it responds to the learner’s needs.” ANS: D
Teaching is most effective when it responds to the learner’s needs. It is impossible to separate teaching from learning. Teaching is an interactive process that promotes learning. Teaching consists of a conscious, deliberate set of actions that help individuals gain new knowledge, change attitudes, adopt new behaviours, or perform new skills. DIF: Understand TOP: Evaluate
REF: 326 OBJ: Describe the purposes of patient education. MSC: NCLEX: Health Promotion and Maintenance
5. Which action best indicates that learning has occurred? a. A nurse presents information about diabetes. b. A patient demonstrates how to inject insulin. c. A family member listens to a lecture on diabetes. d. A primary care provider hands a diabetes pamphlet to the patient. ANS: B
Canadian Fundamentals of Nursing 6th Edition Potter Test Bank Learning is the purposeful acquisition of new knowledge, attitudes, behaviours, and skills. Complex patterns are involved in learning new skills, changing existing attitudes, transferring learning to new situations, and solving problems. A new mother exhibits learning when she demonstrates how to bathe her newborn. A nurse presenting information and a primary care provider handing a pamphlet to a patient are examples of teaching. A family member listening to a lecture does not demonstrate that learning occurred; a change in knowledge, attitudes, behaviours, or skills must be evident. DIF: Understand TOP: Evaluate
REF: 327 OBJ: Use appropriate methods to evaluate learning. MSC: NCLEX: Health Promotion and Maintenance
6. The nurse is demonstrating to the patient how to put on antiembolism stockings. In the middle
of the lesson, the patient asks, “Why have my feet been swelling?” The nurse stops and responds to the patient, adhering to which teaching principle? a. Timing. b. Setting priorities. c. Building on existing knowledge. d. Organizing teaching materials. ANS: A
The nurse who stops a demonstration of applying antiembolism stockings to answer a patient’s question is following the teaching principle of timing. If the patient has a question, it is important to answer the question right away, so that the focus can return to the task being taught. Setting priorities is important for conserving the time and energy of the patient and nurse. A patient learns best on the basis of preexisting cognitive abilities and knowledge. Organizing teaching material means that the nurse considers the order in which to present the information.
NURSINGTB.COM
DIF: Apply REF: 333| 334 TOP: Implementation
OBJ: Explain the role of the nurse in patient education. MSC: NCLEX: Health Promotion and Maintenance
7. A nurse teaches a patient with heart failure about healthy food choices. The patient states that
eating yogurt is better than eating cake. In this situation, which element represents feedback? a. The nurse. b. The patient. c. The nurse teaching about healthy food choices. d. The patient stating that eating yogurt is better than eating cake. ANS: D
Feedback should show the success of the learner in achieving objectives (i.e., the learner verbalizes information or provides a return demonstration of skills learned). The nurse is the sender. The patient is the receiver. The teaching is the message. DIF: Apply REF: 335 OBJ: Determine appropriate communication principles when providing patient education. TOP: Evaluate MSC: NCLEX: Health Promotion and Maintenance 8. While preparing a teaching plan, the nurse describes what the learner will be able to
accomplish after the teaching session. Which action is the nurse completing? a. Developing learning objectives. b. Providing positive reinforcement.
Canadian Fundamentals of Nursing 6th Edition Potter Test Bank c. Implementing interpersonal communication. d. Presenting facts and knowledge. ANS: A
Learning objectives describe what the learner will be able to do after successful instruction. Positive reinforcement follows feedback and involves the use of praise and acknowledgement of new attitudes, behaviours, or knowledge. Interpersonal communication in general is necessary for the teaching/learning process and should be used in teaching. Facts and knowledge are presented in the teaching session. DIF: Understand TOP: Planning
REF: 333 OBJ: Write learning objectives for a teaching plan. MSC: NCLEX: Health Promotion and Maintenance
9. A student nurse learns that a normal adult heartbeat is 60 to 100 beats/minute. In which
domain did learning take place? a. Kinesthetic. b. Cognitive. c. Affective. d. Psychomotor. ANS: B
Cognitive learning includes all intellectual behaviours and requires thinking. In the hierarchy of cognitive behaviours, the simplest behaviour is acquiring knowledge. The student nurse acquired knowledge, which is cognitive. Kinesthetic is a type of learner who learns best with a hands-on approach. Affective learning deals with expression of feelings and acceptance of attitudes, opinions, or values. Psychomotor learning involves acquiring skills that require integration of mental and muscular activities, such as the ability to walk or to use an eating utensil. NURSINGTB.COM DIF: Apply TOP: Assessment
REF: 327| 328 OBJ: Describe the domains of learning. MSC: NCLEX: Health Promotion and Maintenance
10. A nurse wants the patient to begin to accept the chronic nature of diabetes. Which teaching
technique should the nurse use to enhance learning? a. Lecture. b. Demonstration. c. Role play. d. Question and answer session. ANS: C
Affective learning deals with expression of feelings and acceptance of attitudes, opinions, or values. Role play and discussion (one-on-one and in groups) are effective teaching methods for the affective domain. Lectures and question-and-answer sessions are effective teaching methods for the cognitive domain. Demonstration is an effective teaching method for the psychomotor domain. DIF: Analyze REF: 328 (Box 20-4) OBJ: Describe the domains of learning. TOP: Implementation MSC: NCLEX: Health Promotion and Maintenance 11. When the nurse describes a patient’s perceived ability to successfully complete a task, which
term should the nurse use?
Canadian Fundamentals of Nursing 6th Edition Potter Test Bank a. b. c. d.
Self-efficacy. Motivation. Attentional set. Active participation.
ANS: A
Self-efficacy, a concept included in social learning theory, refers to a person’s perceived ability to successfully complete a task. Motivation is a force that acts on or within a person (e.g., an idea, an emotion, a physical need) to cause the person to behave in a particular way. An attentional set is the mental state that allows the learner to focus on and comprehend a learning activity. Learning occurs when the patient is actively involved in the educational session. DIF: Remember TOP: Diagnosis
REF: 330| 331 OBJ: Identify basic learning principles. MSC: NCLEX: Health Promotion and Maintenance
12. A toddler is going to have surgery on the right ear. Which teaching method is most
appropriate for this developmental stage? a. Encourage independent learning. b. Use discussion throughout the teaching session. c. Apply a bandage to a doll’s ear. d. Develop a problem-solving scenario. ANS: C
Use play to teach a procedure or activity (e.g., handling examination equipment, applying a bandage to a doll) to toddlers. Independent learning is for adults. Use of discussion is for older children, adolescents, and adults, not for toddlers. Use problem solving to help adolescents make choices. Problem solviN ngUiR s tS ooIaNdG vaTnB ce. dC foOr M a toddler. DIF: Analyze REF: 329 (Box 20-5) OBJ: Discuss how to integrate health education into patient-centred care. TOP: Planning MSC: NCLEX: Health Promotion and Maintenance 13. Which factors should the nurse assess to determine a patient’s ability to learn? a. Developmental capabilities and physical capabilities. b. Sociocultural background and motivation. c. Psychosocial adaptation to illness and active participation. d. Stage of grieving and overall physical health. ANS: A
Developmental and physical capabilities reflect a person’s ability to learn. Sociocultural background, motivation, psychosocial adaptation to illness, and active participation are factors in readiness to learn. Readiness to learn is also related to the stage of grieving. Overall physical health does reflect ability to learn; however, because it is paired here with stage of grieving (which is a readiness to learn factor), this is a wrong answer. DIF: Understand REF: 328-330 OBJ: Differentiate factors that determine readiness to learn from those that determine the ability to learn. TOP: Assessment MSC: NCLEX: Health Promotion and Maintenance 14. A nurse is preparing to teach a patient about heart failure. Which environment is best for
patient learning?
Canadian Fundamentals of Nursing 6th Edition Potter Test Bank a. b. c. d.
A darkened, quiet room. A well-lit, ventilated room. A private room at 30C (85F) temperature. A group room for 10 to 12 patients with heart failure.
ANS: B
The ideal environment for learning is a room that is well lit and has good ventilation, appropriate furniture, and a comfortable temperature. Although quiet is appropriate, a darkened room interferes with the patient’s ability to watch your actions, especially when you are demonstrating a skill or using visual aids such as posters or pamphlets. A room that is cold, hot, or stuffy makes the patient too uncomfortable to focus on the information being presented. Learning in a group of six or fewer is more effective than in larger groups and can prevent outburst behaviours. DIF: Apply TOP: Planning
REF: 328 OBJ: Establish an environment that promotes learning. MSC: NCLEX: Health Promotion and Maintenance
15. Which situation indicates to the nurse that the patient is ready to learn? a. A patient has sufficient upper body strength to move from a bed to a wheelchair. b. A patient has the ability to grasp and apply the elastic bandage. c. A patient with a below-the-knee amputation is motivated about how to walk with
assistive devices. d. A patient has adequate eyesight to identify the markings on a syringe and
coordination to handle a syringe. ANS: C
Motivation or readiness to learn sometimes results from social task mastery, or physical motives may be involved. OfN teU nR paStiI enNt G mT otB iv. esCaO reMphysical. Some patients are motivated to return to a level of self-sufficiency; for example, a patient with a below-the-knee amputation is motivated to learn how to walk with assistive devices. Readiness to learn should not be confused with ability to learn. All the other answers are examples of ability to learn because the capabilities mentioned often depend on the patient’s level of physical development and overall physical health. To learn psychomotor skills, a patient needs to possess a certain level of strength, coordination, and sensory acuity. For example, it is useless to teach a patient to transfer from a bed to a wheelchair if he or she has insufficient upper body strength. An older patient with poor eyesight or an inability to grasp objects tightly cannot learn to apply an elastic bandage or handle a syringe. DIF: Evaluate REF: 330 OBJ: Differentiate factors that determine readiness to learn from those that determine the ability to learn. TOP: Evaluate MSC: NCLEX: Health Promotion and Maintenance 16. A nurse is teaching the staff about nursing and teaching processes. During the teaching
process, what should the nurse do? a. Assess all sources of data. b. Realize that it is the same as the nursing process. c. Perform nursing care therapies. d. Focus on a patient’s learning needs. ANS: D
Canadian Fundamentals of Nursing 6th Edition Potter Test Bank The teaching process focuses on the patient’s learning needs and willingness and capability to learn. Nursing and teaching processes are not the same. Assessing all sources of data and performing nursing care therapies are components of the nursing process. DIF: Remember REF: 326 OBJ: Compare and contrast the nursing and teaching processes. TOP: Planning MSC: NCLEX: Health Promotion and Maintenance 17. Which nursing action is most appropriate for assessing a patient’s learning needs? a. Assess the patient’s total health care needs. b. Assess the patient’s health literacy. c. Assess all sources of patient data. d. Assess the goals of patient care. ANS: B
Because health literacy influences how you deliver teaching strategies, the nurse must assess a patient’s health literacy before providing instruction. The nursing process requires assessment of all sources of data to determine a patient’s total health care needs. Evaluation of the teaching process involves determining outcomes of the teaching/learning process and the achievement of learning objectives, not patient care. Assessing the goal of meeting patient care is the evaluation component of the nursing process. DIF: Analyze REF: 331| 338 OBJ: Compare and contrast the nursing and teaching processes. TOP: Assessment MSC: NCLEX: Health Promotion and Maintenance 18. A nurse is going to teach a patient about hypertension. Which action should the nurse
NURSINGTB.COM implement first? a. Set mutual goals for knowledge of hypertension. b. Teach what the patient wants to know about hypertension. c. Assess what the patient already knows about hypertension. d. Evaluate the outcomes of patient education for hypertension. ANS: C
Assessment is the first step of any teaching session, followed by diagnosing, planning, implementation, and evaluation. An effective assessment provides the basis for individualized patient teaching. Assessing what the patient currently knows improves the outcomes of patient education. DIF: Apply REF: 331 (Table 20-1) OBJ: Identify appropriate topics that address a patient’s health education needs. TOP: Assessment MSC: NCLEX: Health Promotion and Maintenance 19. A patient had a stroke and must use a cane for support. A nurse is preparing to teach the
patient about the cane. Which learning objective/outcome is most appropriate? a. The patient will walk to the bathroom and back to bed using a cane. b. The patient will understand the importance of using a cane. c. The patient will learn how to use a cane. d. The patient will know the correct use of a cane. ANS: A
Canadian Fundamentals of Nursing 6th Edition Potter Test Bank Outcomes refer to the patient’s ability to do something on completion of teaching, such as will empty colostomy bag, or will administer an injection. Understanding, learning, and knowing are not behaviours that can be observed or evaluated. DIF: Apply TOP: Planning
REF: 333 OBJ: Write learning objectives for a teaching plan. MSC: NCLEX: Health Promotion and Maintenance
20. Which learning objective/outcome has the highest priority for a patient with life-threatening,
severe food allergies that necessitate use of an EpiPen (epinephrine)? a. The patient will demonstrate the correct way to administer epinephrine. b. The patient will identify the main ingredients in several foods. c. The patient will list the side effects of epinephrine. d. The patient will learn about food labels. ANS: A
Once you assist in meeting patient needs related to basic survival (how to give epinephrine), you can discuss other topics, such as nutritional needs and side effects of medications. For example, a patient with a recent diagnosis of coronary artery disease may have deficient knowledge related to the illness and its implications. The patient benefits most by first learning about the correct way to take nitroglycerine and how long to wait before calling for help when chest pain occurs. Thus, in this situation, the patient benefits most by first learning about the correct way to take epinephrine. “The patient will learn about food labels” is not objective and measurable and is not specific enough. DIF: Evaluate TOP: Evaluate
REF: 333 OBJ: Write learning objectives for a teaching plan. MSC: NCLEX: Health Promotion and Maintenance
21. After a teaching session on taNkU inR gS blI ooNdGpT reB ss. urC esO, M the nurse tells the patient, “You took that
blood pressure like an experienced nurse.” What type of reinforcement did the nurse use? a. Material. b. Activity. c. Social. d. Entrusting. ANS: C
Three types of reinforcers are social, material, and activity. When a nurse works with a patient, most reinforcers are social and are used to acknowledge a learned behaviour (e.g., smiles, compliments, words of encouragement). Examples of material reinforcers include food, toys, and music. Activity reinforcers rely on the principle that a person is motivated to engage in an activity if he or she has the opportunity to engage in a more desirable activity after completion of the task. The entrusting approach is a teaching approach that provides the patient the opportunity to manage self-care. It is not a type of reinforcement. DIF: Understand REF: 336 OBJ: Determine appropriate communication principles when providing patient education. TOP: Implementation MSC: NCLEX: Health Promotion and Maintenance 22. A patient with heart failure is learning to reduce salt in the diet. When would be the best time
for the nurse to address this topic? a. At bedtime, when the patient is relaxed. b. At lunchtime while the nurse is preparing the food tray.
Canadian Fundamentals of Nursing 6th Edition Potter Test Bank c. At bath time, when the nurse is cleaning the patient. d. At medication time, when the nurse is administering patient medication. ANS: B
Appropriate times to talk about food/diet changes during routine nursing care are at breakfast, lunch, and dinner times or when the patient is filling out the menu. Many nurses find that they are able to teach more effectively while delivering nursing care. For example, while hanging blood, you explain to the patient why the blood is necessary and the symptoms of a transfusion reaction that need to be reported immediately. In this situation, because the teaching is about food, coordinating it with routine nursing care that involves food can be effective. Bedtime would be a good time to discuss routines that enhance sleep. Bath time would be a good time to describe skin care and how to prevent pressure ulcers. Medication time would be a good time to explain the purposes and side effects of the medication. DIF: Apply REF: 333 OBJ: Include patient teaching while performing routine nursing care. TOP: Implementation MSC: NCLEX: Health Promotion and Maintenance 23. A patient has been taught how to cough and deep breathe. Which evaluation method is most
appropriate? a. Return demonstration. b. Computer instruction. c. Verbalization of steps. d. Cloze test. ANS: A
To demonstrate mastery of the skill, the patient should perform a return demonstration under the same conditions that willN beUeRxpe hom Irien Gced Ba.t C Me or in the place where the skill is to be S N T O performed. Computer instruction is a programmed instruction format in which computers store response patterns for learners and select further lessons on the basis of these patterns (programs can be individualized). Computer instruction is a teaching tool rather than an evaluation tool. Verbalization of steps can be an evaluation tool, but it is not as effective for evaluating a psychomotor skill as a return demonstration is. In the Cloze test, a test of reading comprehension, patients fill in the blanks in a written paragraph. DIF: Evaluate TOP: Evaluate
REF: 336 OBJ: Use appropriate methods to evaluate learning. MSC: NCLEX: Health Promotion and Maintenance
24. A patient has been taught how to change a colostomy bag but is having trouble measuring and
manipulating the equipment and has many questions. What is the nurse’s next action? a. Refer to a mental health specialist. b. Refer to an ostomy specialist. c. Refer to a dietitian. d. Refer to a wound care specialist. ANS: B
Resources that specialize in a particular health need (e.g., wound care or ostomy specialists) are integral to successful patient education. A mental health specialist is helpful for emotional issues rather than for physical problems. A dietitian is a resource for nutritional needs. A wound care specialist provides complex wound care.
Canadian Fundamentals of Nursing 6th Edition Potter Test Bank DIF: Understand REF: 334 TOP: Implementation
OBJ: Use appropriate methods to evaluate learning. MSC: NCLEX: Health Promotion and Maintenance
25. A nurse has taught a patient about healthy eating habits. Which learning objective/outcome is
most appropriate for the affective domain? a. The patient will state three facts about healthy eating. b. The patient will identify two foods for a healthy snack. c. The patient will verbalize the value of eating healthy. d. The patient will cook a meal with low-fat oil. ANS: C
Affective learning deals with expression of feelings and acceptance of attitudes, opinions, or values. Having the patient value healthy eating habits falls within the affective domain. Stating three facts or identifying two foods for a healthy snack falls within the cognitive domain. Cooking falls within the psychomotor domain. DIF: Analyze TOP: Planning
REF: 327 OBJ: Write learning objectives for a teaching plan. MSC: NCLEX: Health Promotion and Maintenance
26. A nurse is assessing the ability to learn of a patient who has recently experienced a stroke.
Which question/statement will best assess the patient’s ability to learn? a. “What do you want to know about strokes?” b. “On a scale from 1 to 10, tell me where you rank your desire to learn.” c. “Do you feel strong enough to perform the tasks I will teach you?” d. “Please read this handout and tell me what it means.” ANS: D
A patient’s reading level affeN cts R ity tG o leB ading level is often difficult to assess CRe M Uabil SI N Tar.n.are Ooften because patients who are functionally illiterate able to conceal it by using excuses such as not having the time or not being able to see. One way to assess a patient’s reading level and level of understanding is to ask the patient to read instructions from an educational handout and then explain their meaning. Asking patients what they want to know identifies previous learning and learning needs and preferences; it does not assess ability to learn. Motivation is related to readiness to learn, not ability to learn. Just asking a patient if they feel strong is not as effective as actually assessing the patient’s strength. DIF: Evaluate REF: 329| 338| 339 OBJ: Differentiate factors that determine readiness to learn from those that determine the ability to learn. TOP: Assessment MSC: NCLEX: Health Promotion and Maintenance 27. A nurse is preparing to teach a kinesthetic learner about exercise. Which technique should the
nurse use? a. Let the patient touch and use the exercise equipment. b. Provide the patient with pictures of the exercise equipment. c. Let the patient listen to a podcast about the exercise equipment. d. Provide the patient with a case study about the exercise equipment. ANS: A
Canadian Fundamentals of Nursing 6th Edition Potter Test Bank Kinesthetic learners learn best while they are moving and participating in hands-on activities. Demonstrations and role playing work well with these learners. Patients who are visual learners learn best from pictures and diagrams that explain information. Patients who prefer auditory learning are distracted by pictures and prefer listening to information (e.g., podcasts). Patients who learn best by reasoning logically and intuitively learn better if presented with a case study that requires careful analysis and discussion with others to arrive at conclusions. DIF: Apply REF: 330 OBJ: Identify appropriate topics for a patient’s health education. TOP: Planning MSC: NCLEX: Health Promotion and Maintenance MULTIPLE RESPONSE 1. Which statements by the nurse indicate a good understanding of patient education/teaching?
(Select all that apply.) a. “Patient education is a standard for professional nursing practice.” b. “Patient teaching falls within the scope of nursing practice.” c. “Patient education is an essential component of safe, patient-centred care.” d. “Patient education is not effective with children.” e. “Patient teaching can increase health care costs.” f. “Patient teaching should be documented in the chart.” ANS: A, B, C, F
Patient education has long been a standard for professional nursing practice. Practice acts acknowledge that patient teaching falls within the scope of nursing practice. Patient education is an essential component of providing safe, patient-centred care. It is important to document evidence of successful patienN t ed tion tieCnts ’ medical records. Patient education is Ruca I GinTpBa. M U S N O effective for children; different techniques must be used for teaching children. Creating a well-designed, comprehensive teaching plan that fits a patient’s unique learning needs reduces health care costs, improves the quality of care, and ultimately changes behaviours to improve patient outcomes. DIF: Understand REF: 324-326| 340 OBJ: Identify the role of the nurse in patient education. MSC: NCLEX: Health Promotion and Maintenance
TOP: Evaluate
2. Which situation will cause the nurse to postpone a teaching session? (Select all that apply.) a. The patient is mildly anxious. b. The patient is fatigued. c. The patient is asking questions. d. The patient is hurting. e. The patient is febrile (high fever). f. The patient is in the acceptance phase. ANS: B, D, E
Canadian Fundamentals of Nursing 6th Edition Potter Test Bank Any condition (e.g., pain, fatigue) that depletes a person’s energy also impairs his or her ability to learn, so the session should be postponed until the pain is relieved and the patient is rested. Teaching should also be postponed when an illness becomes aggravated by complications such as a high fever or respiratory difficulty. A mild level of anxiety motivates learning. When patients are ready to learn, they frequently ask questions. When the patient enters the stage of acceptance, the stage compatible with learning, introduce a teaching plan. DIF: Apply REF: 338| 339 OBJ: Include patient teaching while performing routine nursing care. TOP: Implementation MSC: NCLEX: Health Promotion and Maintenance
NURSINGTB.COM
Canadian Fundamentals of Nursing 6th Edition Potter Test Bank
Chapter 21: Developmental Theories Potter et al: Canadian Fundamentals of Nursing, 6th Edition MULTIPLE CHOICE 1. A child is assessed as being extremely active, requiring a highly structured environment, and
irregular in her habits. According to Chess and Thomas (1995), an accurate interpretation of the child’s category of temperament is which of the following? a. Easy child. b. Difficult child. c. Hyperactive child. d. Slow-to-warm up child. ANS: B
According to Chess and Thomas, “difficult child” would accurately describe this child’s behaviour. The easy child is even-tempered, regular, and predictable. The temperament category “hyperactive child” does not exist within Chess and Thomas’s common behaviour categories. The slow-to-warm-up child typically reacts negatively and with mild intensity to new stimuli. DIF: Analyze REF: 346 OBJ: Discuss factors influencing growth and development. MSC: NCLEX: Health Promotion and Maintenance
TOP: Assessment
2. The nursing instructor needs to provide further instruction to the student who makes which
statement? N R I G B.C M a. “Intellectual development iU s afS fectN ed bT y cogniO tive processes.” b. “Socioemotional processes can influence an individual’s growth and development.” c. “Breast development is an example of a change resulting from biological processes.” d. “An individual’s biological processes determine physical characteristics and do not affect growth and development.” ANS: D
Human growth and development are now seen as processes in which sociocultural, biological, and psychological forces interact with the individual over time. Cognitive processes comprise changes in intelligence, use of language, and development of thinking. Socioemotional processes consist of variations in personality, emotions, and relationships with others. Height and weight, development of gross and fine motor skills, and sexual maturation resulting from hormonal changes during puberty are examples of changes resulting from biological processes. DIF: Understand REF: 344 OBJ: Discuss factors influencing growth and development. MSC: NCLEX: Health Promotion and Maintenance
TOP: Assessment
3. Which of these statements would be most appropriate for a nurse to state when assessing an
adult patient for growth and developmental delays? a. “How many times per week do you exercise?”
Canadian Fundamentals of Nursing 6th Edition Potter Test Bank b. “Are you able to stand on one foot for 5 seconds?” c. “Would you please describe your usual activities during the day?” d. “How many hours a day do you spend watching television or sitting in front of a
computer?” ANS: C
Understanding normal growth and development helps nurses predict, detect, and prevent deviations from patients’ own expected patterns. The nurse can then compare patterns of activity expected on the basis of age with the patient’s stated activity patterns to determine deviations from the patient’s own expected patterns. Asking the patient to describe his or her usual daily activities will elicit useful information about the patient’s own expected patterns. How many hours are spent watching television or in front of a computer and how many times the patient exercises in a week are closed-ended questions. These questions would not provide the nurse with as much information about the patient’s expected patterns when his or her stated patterns are compared with expected patterns for the patient’s age group to detect delays. DIF: Apply REF: 343 OBJ: Discuss factors influencing growth and development. MSC: NCLEX: Health Promotion and Maintenance
TOP: Assessment
4. Which of the following is a priority reason for being knowledgeable about biophysical
developmental theories? a. Understanding how the physical body grows. b. Predicting definite patterns of cognitive development. c. Anticipating how patients’ social behaviours develop. d. Describing the process of psychological development. ANS: A
N R I G B.C M U S N T O
Biophysical development concerns how people’s physical bodies grow and change. Nurses and other health care providers are able to quantify the changes that occur as a newborn grows into adulthood and compare them with established norms to detect abnormalities. Biophysical development is concerned with physical growth, not cognitive development, social behaviours, or psychological development. DIF: Knowledge REF: 344| 345 OBJ: Name and describe the major developmental theories associated with each tradition. TOP: Assessment MSC: NCLEX: Health Promotion and Maintenance 5. While assessing an 18-month-old toddler, the nurse distinguishes normal from abnormal
findings by remembering that Gesell’s theory of development accounts for which of the following statements? a. “The developmental stage of the toddler is affected solely by environmental influence.” b. “Developmental patterns are not affected by gene activity.” c. “Skill development should be identical to that of other toddlers in the playroom.” d. “Environmental influence does not affect the sequence of development.” ANS: D
Canadian Fundamentals of Nursing 6th Edition Potter Test Bank Gesell’s theory of development states that environment plays a part in child development, but it does not have any part in the sequence of development. Other factors influencing growth and development include biological, cognitive, and socioemotional processes. Environmental factors support, change, and modify the pattern of development, but they do not generate progressions of development. Each child’s pattern of growth and development is unique and is directed by gene activity. Not every child develops certain skills at the same time. Children grow according to their own genetic blueprint. DIF: Understand REF: 345| 346 OBJ: Name and describe the major developmental theories associated with each tradition. TOP: Assessment MSC: NCLEX: Health Promotion and Maintenance 6. Which statement is in accordance with Freud’s psychoanalytical/psychosocial theory? a. Adult personality is the result of resolved conflicts between sources of sexual
pleasure and the mandates of reality. b. Development occurs throughout the lifespan and focuses on psychosocial stages. c. The genital stage precedes the phallic stage of development. d. Problems evident in adult life are due to early successes and resolution of earlier
developmental stages. ANS: A
Freud believed that adult personality is the result of how an individual resolved conflicts between sources of sexual pleasure and the mandates of reality. Freud had a strong influence on Erik Erikson, but Erikson’s theory differed from Freud’s in that it focused on psychosocial stages rather than psychosexual stages. Freud’s five stages of psychoanalytical development in sequential order include oral, anal, phallic, latency, and genital. The phallic stage precedes the genital stage. In theory, problems in adult life would be due to unresolved conflicts and N R I G B.C M U S N T O failures. DIF: Knowledge REF: 349| 350 OBJ: Name and describe the major developmental theories associated with each tradition. TOP: Assessment MSC: NCLEX: Health Promotion and Maintenance 7. The nurse is teaching a young adult couple about promoting the health of their 8-year-old
child. The nurse knows that the parents understand the developmental stage their child is in according to Erikson when they state which of the following? a. “We should provide proper support for learning new skills.” b. “We should encourage devoted relationships with others.” c. “We should limit choices and provide harsh punishment for mistakes.” d. “We should not leave our child at school for longer than 3 hours at a time.” ANS: A
An 8-year-child would be in the industry versus inferiority stage of development. During this stage, the child needs to be praised for accomplishments such as learning new skills. Developing devoted relationships is part of the identity versus role confusion stage, usually occurring during puberty. During the autonomy versus shame and doubt stage, limiting choices and harsh punishment lead to feelings of shame and doubt. Separation anxiety is usually a part of the trust versus mistrust stage. DIF: Understand REF: 350-352 OBJ: Name and describe the major developmental theories associated with each tradition.
Canadian Fundamentals of Nursing 6th Edition Potter Test Bank TOP: Evaluate
MSC: NCLEX: Health Promotion and Maintenance
8. Jean Piaget’s cognitive developmental theory focuses on four stages of development,
including which of the following? a. Formal operations. b. Intimacy versus isolation. c. Latency. d. The postconventional level. ANS: A
Jean Piaget’s theory includes four stages in sequential order: sensorimotor, preoperational, concrete operations, and formal operations. Intimacy versus isolation is part of Erik Erikson’s psychosocial theory of development. Latency is stage 4 of Freud’s five-stage psychosexual theory of development. The postconventional level of reasoning is part of Kohlberg’s theory of moral development. DIF: Knowledge REF: 347 (Table 21-2) OBJ: Name and describe the major developmental theories associated with each tradition. TOP: Assessment MSC: NCLEX: Health Promotion and Maintenance 9. According to Piaget’s formal operations level, what is a 13-year-old likely to do? a. Hit other students to deal with environmental change. b. Use play to understand her surroundings. c. Question her parents about an upcoming presidential election. d. Question where the ice is hiding when ice has melted in her drink. ANS: C
In the formal operations perioNd, aRdolI esceGntsBa. ndCyoM ng adults begin to think about such U Sfinding N Tjustice, Ouand subjects as achieving world peace, seeking meaning in life. Asking about a presidential election demonstrates that the adolescent is concerned about political issues that affect others besides her. Hitting would be a common schema during the sensorimotor stage of development. Using play to learn about the environment is indicative of the preoperational stage. During the concrete operations stage (ages 6 to 12 years), children are able to coordinate two concrete perspectives in social and scientific thinking, such as understanding the difference between “hiding” and “melting.” DIF: Understand REF: 347 (Table 21-2) OBJ: Name and describe the major developmental theories associated with each tradition. TOP: Assessment MSC: NCLEX: Health Promotion and Maintenance 10. According to Piaget’s theory of cognitive development, the nurse should allow a hospitalized
4-year-old patient to safely play with which item? a. The pump administering intravenous (IV) fluids. b. The blood pressure cuff. c. A baseball bat. d. A book to read alone in a quiet place. ANS: B
Canadian Fundamentals of Nursing 6th Edition Potter Test Bank A 4-year-old child would be in the preoperational period. Children at this stage are still egocentric. Play is very important to foster cognitive development. Children should be allowed to play with any equipment that is safe, such as a blood pressure cuff, and should be allowed to communicate feelings about their health care. The IV pump is not a safe piece of equipment for a 4-year-old child to play with. A baseball bat is not typically found in a hospital setting and is a potentially dangerous toy to play with in the hospital. A 4-year-old child is of preschool age and may not be able to read yet. Also, the book does not allow for any human interaction and communication if it is read alone. DIF: Apply REF: 347 (Table 21-2) OBJ: Name and describe the major developmental theories associated with each tradition. TOP: Planning MSC: NCLEX: Health Promotion and Maintenance 11. Which of these manifestations, if identified in a 6-year-old patient, should the nurse associate
with a possible developmental delay according to Piaget’s theory? a. The child speaks in complete sentences but often talks only about himself. b. The child still plays with a favourite doll that he has had since he was a toddler. c. The child continues to suck his thumb. d. The child describes an event from his own perspective, even though the entire family was present. ANS: C
This is a characteristic of the sensorimotor stage (birth to 2 years), where schemas become self-initiated activities. For example, the infant who learns that sucking achieves a pleasing result generalizes the action to suck fingers, blanket, or clothing. Successful achievement leads to greater exploration. By age 6, the child is in the preoperational stage of development. The child is expected to be egocentric, even though language ability is progressing. Play N R INGTB.COM becomes a primary means by wUhicS h children foster their cognitive development; therefore, playing with a doll is considered normal at this age. Children see objects and persons from only one point of view—their own—at this stage. DIF: Analyze REF: 347 (Table 21-2) OBJ: Name and describe the major developmental theories associated with each tradition. TOP: Assessment MSC: NCLEX: Health Promotion and Maintenance 12. An 18-month-old patient is brought into the clinic for evaluation because her mother is
concerned. The 18-month-old child hits her siblings and says only “No” when communicating verbally. According to Piaget’s theory, what recommendation should the nurse make a priority? a. Consult the social worker because the child is hitting other children. b. Reassure the mother that the child is developmentally within specified norms. c. Encourage the mother to seek psychological counselling for the child. d. Remove all toys from the child’s room until this behaviour ceases. ANS: B
Canadian Fundamentals of Nursing 6th Edition Potter Test Bank At 18 months, the child is in the sensorimotor period of development. Piaget describes hitting, looking, grasping, and kicking as normal schemas to deal with the environment. The social worker does not need to be consulted in this case, nor is psychological counselling warranted, because the child is exhibiting normal behaviours. Play is an important part of all children’s development. Removing the toys is not necessary because this child is exhibiting normal behaviours; removing toys and the opportunity to play with them may actually hinder the child’s development. DIF: Evaluate REF: 347 (Table 21-2) OBJ: Name and describe the major developmental theories associated with each tradition. TOP: Implementation MSC: NCLEX: Health Promotion and Maintenance 13. A nursing student overhears a 72-year-old patient reflecting on whether his or her life was
meaningful. The nursing student acknowledges that the patient is in which stage of Erikson’s eight stages of life? a. Initiative versus guilt b. Industry versus inferiority. c. Integrity versus despair. d. Intimacy versus isolation. ANS: C
During the integrity versus despair stage (65 years and older), older persons reflect on their life and feel satisfaction or disappointment. The question answered during this stage is “Has my life been worthwhile?” The older person reflecting on his or her life is not in the initiative versus guilt stage, the industry versus inferiority stage, or the intimacy versus isolation stage. DIF: Apply REF: 351 (Table 21-5) NaUjoR OBJ: Name and describe the m r dSeI veN loG pm TeBnt.alCthOeoMries associated with each tradition. TOP: Planning MSC: NCLEX: Health Promotion and Maintenance 14. The parents of a 14-year-old boy express concern over their child’s rebellious behaviour. The
nurse should plan to respond to the parents’ concern by informing them that their: a. Child should be referred to a juvenile correctional facility. b. Child’s behaviour is normal because the adolescent is trying to adjust to his emerging identity. c. Child’s behaviour is a matter of concern because he is likely conflicted about establishing companionship with a partner. d. Child’s behaviour is expected because he is expressing his need to support future generations. ANS: B
According to Erikson, a 14-year-old adolescent is developing his identity in relation to role confusion. A teenager is very concerned with self and is often preoccupied with body image. Frequently, teenagers express themselves rebelliously as they struggle to discover their own identities. Rebellious behaviour is very common and normal at this stage of development. A juvenile correctional facility usually is not necessary. Establishing companionship occurs in the young adult age group. Feeling the need to support future generations is usually experienced by the middle-aged adult. DIF: Apply REF: 351 (Table 21-5) OBJ: Name and describe the major developmental theories associated with each tradition.
Canadian Fundamentals of Nursing 6th Edition Potter Test Bank TOP: Planning
MSC: NCLEX: Health Promotion and Maintenance
15. The teaching plan for a 3-year-old child who is at risk for developmental delay should include
which of these instructions for the parents? a. “Encourage play as your child is exploring his or her surroundings.” b. “Insist that your child discuss various points of view, not just his or her own.” c. “Discuss world events with your child to foster language development.” d. “Actively encourage your child to read lengthy books to expedite reading and writing abilities.” ANS: A
A 3-year-old child uses play to learn about and discover the surrounding environment. Children at this age are egocentric and often are unable to see the world from any perspective other than their own. Very young children are not able to understand and comment on world events because their thinking has not advanced to abstract reasoning yet. A 3-year-old child is probably unable to read. Asking a child to perform an activity that is beyond his or her developmental abilities will probably result in frustration at not being able to complete the task. DIF: Plan REF: 347 (Table 21-2) OBJ: Name and describe the major developmental theories associated with each tradition. TOP: Application MSC: NCLEX: Health Promotion and Maintenance 16. A nurse should instruct the parents of a 10-year-old child to keep which of the following
theoretical principles in mind when dealing with a behavioural problem at home? a. Strategies that worked well with the first child will be equally as effective for the second child. N R I G B.C M b. Encourage the child to voluUnteS er soNmeTtime atOa local hospital to instill a sense of fulfillment. c. Bargaining about chores in exchange for privileges may be an effective method of encouraging helpful activities. d. Do not offer praise for accomplishments and punishment for behavioural issues. ANS: C
In the concrete operations period, children begin to cooperate and share new information about the acts they perform. Parents can adjust their approaches to guide the child into helpful activities within the home, such as bargaining about chores in exchange for privileges. With the birth of a second child, most parents find that the strategies that worked well with the first child no longer work at all. After birth, children grow according to their genetic blueprint and gain skills in an orderly manner, but each at an individual pace. The need for a sense of fulfillment is usually experienced by middle-aged adults, not children. School-aged children need praise to discourage a sense of inferiority; providing praise is the best choice for encouraging positive behaviours while nurturing growth and development. DIF: Understand REF: 347 (Table 21-2) OBJ: Discuss nursing implications associated with the application of developmental principles to patient care. TOP: Implementation MSC: NCLEX: Health Promotion and Maintenance
Canadian Fundamentals of Nursing 6th Edition Potter Test Bank 17. The parents of a 15-month-old child express concern to the nurse about their child’s
thumb-sucking habit. Which of these explanations related to the child’s age and developmental level would be most appropriate for the nurse to give the parents? a. Thumb sucking at this age indicates a developmental delay and should be further assessed. b. Sucking achieves a pleasing result for infants, and generalizing that action by thumb sucking is normal. c. Thumb sucking at this age demonstrates a transition away from egocentric thinking. d. At this age, thumb sucking will enhance language development. ANS: B
Action patterns are used by infants and toddlers to deal with the environment. For example, the infant who learns that sucking achieves pleasure generalizes that action to sucking fingers, blankets, or clothing. Children remain egocentric into the preoperational period. Thumb sucking does not indicate transition away from egocentric thinking. No statements have supported thumb sucking as enhancing language development. DIF: Apply REF: 347 (Table 21-2) OBJ: Discuss nursing implications associated with the application of developmental principles to patient care. TOP: Implementation MSC: NCLEX: Health Promotion and Maintenance 18. Which of these approaches would be most appropriate for the nurse to use when teaching a
4-year-old patient about scheduled surgery? a. Give the parents a book to read about the procedure and do not discuss the procedure with the child, to decrease anxiety. NUing RSbyINtelli GTngBthe .CcOhild M that she can ask only three b. Set boundaries before teach questions because time is limited. c. Insist that the parents wait outside the room to ensure privacy of the child. d. Allow the child to touch and hold medical equipment such as thermometers and empty syringes. ANS: D
Nursing interventions during the preoperational period (ages 2 to 7 years) should recognize that the use of play (such as handling equipment) helps the child understand the events taking place. Giving the parents a book and not involving the child is not the best option because the nurse should explain all procedures to children and their parents. Children tend to ask a lot of questions; therefore, limiting questions may increase anxiety. Parents and the child all should be involved in preoperative teaching because the parents will be the primary caregivers upon discharge. DIF: Apply REF: 347 (Table 21-2) OBJ: Discuss nursing implications associated with the application of developmental principles to patient care. TOP: Implementation MSC: NCLEX: Health Promotion and Maintenance MULTIPLE RESPONSE 1. When developing a plan of care concerning growth and development for a hospitalized
adolescent, what should the nurse do? (Select all that apply.)
Canadian Fundamentals of Nursing 6th Edition Potter Test Bank a. Stick with one developmental theory for consistency. b. Apply developmental theories when making observations of the individual’s
patterns of growth and development. c. Compare the individual’s assessment findings versus established normal findings. d. Recognize his or her own moral developmental level. e. Apply a unidimensional lifespan perspective. ANS: B, C, D
No single theory successfully describes all the intricacies of human growth and development. Today’s nurses need to be knowledgeable about several theoretical perspectives when working with patients. These theories form the basis for meaningful observation of an individual’s pattern of growth and development. They provide important guidelines for an understanding of important human processes that allows the nurse to begin to predict human responses, not medical diagnoses, and to recognize deviations from the norm. Recognizing one’s own moral developmental level is essential in separating one’s own beliefs from those of others when helping patients with their moral decision-making process. Growth and development, as supported by a lifespan perspective, is multidimensional. DIF: Understand REF: 344| 356| 357 OBJ: Discuss nursing implications associated with the application of developmental principles to patient care. TOP: Planning MSC: NCLEX: Health Promotion and Maintenance 2. The nurse is planning playroom activities for a hospitalized 6-year-old patient. Which of the
following items are age appropriate? (Select all that apply.) a. Crayons and paper. b. Children’s books. c. A 500-piece puzzle. NURSINGTB.COM d. Building blocks. e. Magazines and newspapers. ANS: A, B, D
A school-aged child thrives on feelings of accomplishment. Drawing pictures, looking at children’s books, and building blocks are all ways that a child this age could play while developing a sense of accomplishment. A 500-piece puzzle would be too difficult for a 6-year-old child to complete without the possibility of getting frustrated. Most magazines and newspapers would be written at too high a reading level for a 6-year-old child. If play items offered to the child are too difficult, the child may become frustrated and may experience a feeling of inferiority. DIF: Apply REF: 351 OBJ: Discuss nursing implications associated with the application of developmental principles to patient care. TOP: Planning MSC: NCLEX: Health Promotion and Maintenance
Canadian Fundamentals of Nursing 6th Edition Potter Test Bank
Chapter 22: Conception Through Adolescence Potter et al: Canadian Fundamentals of Nursing, 6th Edition MULTIPLE CHOICE 1. To promote parent-child attachment with a healthy newborn, what should the nurse do? a. Encourage close physical contact as soon as possible after the child’s birth. b. Do not allow the newborn to remain with parents until the second hour after
delivery. c. Never leave the newborn alone with the mother during the first 8 hours after
delivery. d. Isolate the newborn in the nursery during the first hour after delivery. ANS: A
After immediate physical evaluation and application of identification bracelets, the nurse promotes the parents’ and newborn’s need for close physical contact. Early parent-child interaction encourages parent-child attachment. Most healthy newborns are awake and alert for the first half-hour after birth. This is a good time for parent-child interaction to begin. No evidence in the question stem suggests that the baby cannot be left alone with the parents during the first 8 hours or that the baby should remain in the nursery during the first hour. DIF: Knowledge REF: 362 OBJ: Discuss physiological and psychosocial health concerns during the transition of the child from intrauterine to extrauterine life. TOP: Assessment MSC: NCLEX: Health Promotion and Maintenance
thU erRoS fI a nNeG wT boBrn.uCnO deMrstands associated health risks to her baby 2. The nurse knows that the moN when she makes which statement? a. “I need to moisten the umbilical cord every hour during the day until the cord falls off.” b. “I need to remind anyone who wants to hold the baby to wash hands.” c. “I need to leave the blankets off the baby to prevent smothering.” d. “I can throw away the bulb syringe now because my baby is breathing on her own.” ANS: B
Prevention of infection is a major concern in the care of the newborn. Good hand hygiene technique is the most important factor in protecting the newborn from infection. The umbilical stump should be kept clean and dry. Newborns are susceptible to heat loss and cold stress. The nurse should place the newborn directly on the mother’s abdomen and cover him or her in warm blankets, making sure to keep the infant’s head well covered, or place the infant unclothed in an infant warmer with a temperature probe in place. Removal of nasopharyngeal and oropharyngeal secretions remains a priority of care to keep the airway patent. DIF: Understand REF: 362| 363 OBJ: Discuss physiological and psychosocial health concerns during the transition of the child from intrauterine to extrauterine life. TOP: Assessment MSC: NCLEX: Health Promotion and Maintenance 3. What is the priority assessment immediately after an infant’s birth?
Canadian Fundamentals of Nursing 6th Edition Potter Test Bank a. b. c. d.
Assess infant-parent interactions. Promote parent-newborn physical contact. Open the infant’s airway. Assess gestational age.
ANS: C
The most extreme physiological change occurs when the newborn leaves the in utero environment and develops independent respiratory functioning. Direct nursing care includes maintaining an open airway, stabilizing and maintaining the newborn’s body temperature, and protecting the newborn from infection. After immediate physical evaluation and application of identification bracelets, the nurse promotes the parents’ and newborn’s need for close physical contact. After a comprehensive physical assessment, the nurse assesses gestational age and interactions between infant and parents. DIF: Apply REF: 362| 363 OBJ: Discuss physiological and psychosocial health concerns during the transition of the child from intrauterine to extrauterine life. TOP: Assessment MSC: NCLEX: Health Promotion and Maintenance 4. Immediate intervention is needed when the newborn exhibits which of the following? a. A soft, protuberant abdomen. b. Moulding. c. Lack of reflexes. d. Cyanotic hands and feet. ANS: C
Normal reflexes include blinking in response to bright lights, startling in response to sudden loud noises, and sucking, rooN ting aspi UR, gr SI NGng,TBya.wCnin OMg, coughing, sneezing, and hiccuping. Assessment of these reflexes is vital because the newborn depends largely on reflexes for survival and in response to its environment. Normal physical characteristics include the continued presence of lanugo on the skin of the back; cyanosis of the hands and feet for the first 24 hours; and a soft, protuberant abdomen. Moulding, or overlapping of the soft skull bones, allows the fetal head to adjust to various diameters of the maternal pelvis and is a common occurrence with vaginal births. DIF: Apply REF: 364| 365 OBJ: Discuss physiological and psychosocial health concerns during the transition of the child from intrauterine to extrauterine life. TOP: Assessment MSC: NCLEX: Physiological Integrity 5. Normal physical findings in a healthy newborn include which of the following? a. Sporadic motor movements. b. Cyanosis of the feet and hands for the first 48 hours. c. Triangle-shaped anterior fontanel. d. Weight of 4800 g. ANS: A
Movements in the newborn are generally sporadic, but they are symmetric and involve all four extremities. Cyanosis of the hands and feet is normal for the first 24 hours, not 48 hours. The anterior fontanel is diamond shaped, and the posterior fontanel is triangular; both are found between the unfused bones of the skull. The expected, normal weight of a healthy newborn is between 2700 and 4000 g (6 to 9 pounds).
Canadian Fundamentals of Nursing 6th Edition Potter Test Bank
DIF: Knowledge REF: 364| 365 OBJ: Describe characteristics of physical growth of the fetus and of the child from birth to adolescence. TOP: Assessment MSC: NCLEX: Physiological Integrity 6. When comparing physical growth patterns between school-aged children and adolescents,
what does the nurse note? a. Physical growth usually slows during the adolescent period. b. Boys usually exceed girls in height and weight by the end of the middle-school years. c. Secondary sex characteristics usually develop during the adolescent years. d. The distribution of muscle and fat remains constant during the adolescent years. ANS: C
Hormonal changes during adolescence contribute to the development of secondary sex characteristics such as pubic and axillary hair growth and voice changes. Physical growth usually slows during the school-aged period, and then a growth spurt occurs during adolescence. Boys usually exceed girls in height and weight by the end of the high school years. As height and weight increase during adolescence, the distribution of muscle and fat changes. DIF: Understand REF: 381| 382| 386| 387 OBJ: Describe characteristics of physical growth of the fetus and of the child from birth to adolescence. TOP: Assessment MSC: NCLEX: Health Promotion and Maintenance 7. Which of the following is characteristic of the cognitive changes in a preschooler? a. The ability to think in a logical manner about the here and now. b. The ability to think abstraNcU tlyRaSnI dN deG alTeB ff. ecC tiO veM ly with hypothetical problems. c. The inability to assume the view of another person and to use symbols to represent
others. d. The ability to classify objects by size or colour. ANS: D
Preschoolers demonstrate their ability to think more complexly by classifying objects according to size or colour. Cognitive changes that provide the ability to think in a logical manner about the here and now and to understand the relationships between things and ideas occur during the school-age years. It is during the teenage years when the individual thinks abstractly and deals effectively with hypothetical problems. The preschooler is unable to assume the view of another person. DIF: Knowledge REF: 377 OBJ: Describe cognitive and psychosocial development from birth to adolescence. TOP: Assessment MSC: NCLEX: Health Promotion and Maintenance 8. The nursing instructor will need to provide further instruction to the student who uses which
of these statements when describing the differences between cognitive and psychosocial development in children? a. “The preschooler develops the ability to play in small groups.” b. “The toddler may participate in parallel play.” c. “The school-aged child still requires total assistance in all activities for safety.” d. “The toddler period is a time of potential frustration manifested by temper
Canadian Fundamentals of Nursing 6th Edition Potter Test Bank tantrums.” ANS: C
The care provider should promote independence within safe limits for the school-aged child. The school-aged child, according to Erikson, is in the industry versus inferiority stage of development. School-aged children like to perform tasks by themselves when possible, and they need to be praised for those tasks. Children continue to engage in solitary play during toddlerhood but also begin to participate in parallel play, which is playing beside rather than with another child. The toddler’s strong will is frequently exhibited in negative behaviour when caregivers attempt to direct his actions. Temper tantrums result when restrictions frustrate toddlers. DIF: Understand REF: 383 OBJ: Describe cognitive and psychosocial development from birth to adolescence. TOP: Assessment MSC: NCLEX: Health Promotion and Maintenance 9. The nurse is observing his 2-year-old hospitalized patient in the playroom. What is the nurse
most likely to observe the child doing? a. Participating as the leader of a small group activity. b. Sitting beside another child while playing with blocks. c. Separating building blocks into groups by size and colour. d. Seeking out same sex children to play with. ANS: B
The child beside another child and playing is exhibiting parallel play, characteristic of a toddler. Participating as a group leader does not usually occur until approximately age 5. Preschoolers (ages 3 to 5) demonstrate their ability to think more complexly by classifying objects according to size or cN olou -yea ld O chMildren do not have this ability yet. The URr.STIwo NG TBr-.oC play of preschool children becomes more social after the third birthday as it shifts from parallel to associative play. However, gender does not become a factor until the child reaches school age, when he or she prefers same-sex peers to opposite-sex peers. DIF: Apply REF: 375 OBJ: Describe cognitive and psychosocial development from birth to adolescence. TOP: Assessment MSC: NCLEX: Health Promotion and Maintenance 10. A patient in her second trimester of pregnancy comes to the prenatal clinic for a checkup.
What is most important for the nurse to assess in caring for a woman at this stage of pregnancy? a. Detection of fetal movement. b. Observation that the uterus is below the symphysis pubis. c. Confirmation of the desire to breastfeed or bottle-feed. d. Determination of the presence of morning sickness. ANS: A
During the second trimester, between 16 and 20 weeks’ gestation, the prospective mother begins to feel fetal movement. During the second trimester, the uterus should be above the level of the symphysis pubis. Confirmation of the desire to breastfeed or bottle-feed is more likely to take place during the third trimester. Morning sickness is most likely to occur during the first trimester. DIF: Apply
REF: 362
Canadian Fundamentals of Nursing 6th Edition Potter Test Bank OBJ: Discuss physiological and psychosocial health concerns during the transition of the child from intrauterine to extrauterine life. TOP: Assessment MSC: NCLEX: Physiological Integrity 11. During infant/child development, play is best recognized as which of the following? a. A means to interact with the environment and relate to others. b. Independent of cognitive and social development. c. Nonexploratory and simply play. d. Too soon to achieve milestones. ANS: A
During infancy, play is a meaningful set of activities through which the child interacts with the environment and relates to others. Play provides opportunities for development of cognitive, social, and motor skills. Much of infant play is exploratory as infants use their senses to observe and examine their own bodies and objects of interest in their surroundings. Adults facilitate infant learning by planning activities that promote the development of milestones and by providing toys that are safe for the infant to explore with the mouth and manipulate with the hands. DIF: Knowledge REF: 367 OBJ: Explain the role of play in the development of a child. MSC: NCLEX: Psychosocial Integrity
TOP: Assessment
12. Which statement by the nurse best explains the importance of play during the toddler stage of
development? a. “Exploration can suppress the toddler’s curiosity to promote safety.” b. “Parental control during play will eliminate the frustration of learning NURSINGTB.COM self-control.” c. “Play can enhance cognitive and psychosocial development.” d. “Play will enhance the toddler’s ability to explore the environment safely without supervision.” ANS: C
Play can enhance cognitive and psychosocial development. The toddler’s curiosity is evident in his or her exploration of the environment. Children strive for independence. Their strong wills are frequently exhibited in negative behaviour when caregivers attempt to direct their actions. Temper tantrums result when parental restrictions frustrate toddlers. Parents need to provide toddlers with graded independence, allowing them to do things that do not result in harm to themselves or others. DIF: Understand REF: 375 OBJ: Explain the role of play in the development of a child. MSC: NCLEX: Psychosocial Integrity
TOP: Assessment
13. After comparing appropriate play activities for infants and preschool children, the nurse
should offer which of the following activities to an infant? a. Set of cards to organize and separate into groups. b. Set of plastic stacking rings. c. Paperback book. d. Set of sock puppets with movable eyes. ANS: B
Canadian Fundamentals of Nursing 6th Edition Potter Test Bank Play becomes manipulative as the child learns control of the hands. Adults facilitate infant learning by planning activities that promote the development of milestones and by providing toys that are safe for the infant to explore with the mouth and manipulate with the hands, such as rattles, wooden blocks, plastic stacking rings, squeezable stuffed animals, and busy boxes. Preschoolers demonstrate their ability to think more complexly by classifying objects according to size or colour, and so the cards are more appropriate for them. Neither group is ready for paperback books. The sock puppet with movable eyes could create a choking hazard if one of the eyes comes off. DIF: Understand REF: 367 OBJ: Explain the role of play in the development of a child. MSC: NCLEX: Health Promotion and Maintenance
TOP: Implementation
14. A mother expresses concern because her 5-year-old child frequently talks about friends who
don’t exist. What is the nurse’s best response to this mother’s concern? a. “Have you considered a child psychological evaluation?” b. “It’s very normal for a 5-year-old child to have imaginary playmates.” c. “You should stop your child from playing electronic games.” d. “Pretend play is a sign your child watches too much television.” ANS: B
At age 5, some children have imaginary playmates. Imaginary playmates are a sign of health and allow the child to distinguish between reality and fantasy. The child does not need a psychological evaluation on the basis of this information. Television, videos, electronic games, and computer programs help support development and the learning of basic skills; however, these should be only one part of the child’s total play activities. DIF: Apply REF: 37N 9URSINGTB.COM OBJ: Explain the role of play in the development of a child. MSC: NCLEX: Psychosocial Integrity
TOP: Implementation
15. Encouraging children to play a game of kickball would be best suited for which age group? a. Infant. b. Toddler. c. Preschool. d. School-aged. ANS: D
A game of kickball would be best suited for school-aged children because in this age group, play involves peers and the pursuit of group goals. Although solitary activities are not eliminated, group play overshadows them. Younger children typically are not able to participate cooperatively in groups yet. Infants begin to play simple social games such as patty-cake and peek-a-boo. Toddlers engage in solitary play but also begin to participate in parallel play. Preschoolers playing together engage in similar if not identical activities; however, no division of labour, rigid organization, or rules are observed. By the age of 5, the group has a temporary leader for each activity. DIF: Apply REF: 381| 383 OBJ: Explain the role of play in the development of a child. MSC: NCLEX: Psychosocial Integrity
TOP: Implementation
Canadian Fundamentals of Nursing 6th Edition Potter Test Bank 16. Which of these manifestations, if identified in a school-aged child during a routine
assessment, should a nurse associate with a possible developmental delay or problem? a. Withdrawn demeanour and verbalizing that he has no friends. b. Absence of secondary sex characteristics. c. Lack of peer relationships. d. Curiosity about his or her sexuality. ANS: A
School-aged children should begin to develop friendships and to socialize with others. Interaction with peers allows them to define their own accomplishments in relation to others as they work to develop a positive self-image. The absence of secondary sex characteristics is a major concern of adolescents, not school-aged children, because physical evidence of maturity encourages the development of masculine and feminine behaviours in the adolescent. Lack of peer relationships is also a concern of adolescents, not of school-aged children, because adolescents seek a group identity to fulfill their esteem and acceptance needs. Today many researchers believe that school-aged children have a great deal of curiosity about their sexuality. Some experiment, but this play is usually transitory. DIF: Analyze REF: 383| 384 OBJ: Explain the role of play in the development of a child. MSC: NCLEX: Psychosocial Integrity
TOP: Assessment
17. The nurse who is teaching a parent about developmental needs of the infant knows that the
parent understands an infant’s developmental needs when he or she states which of the following? a. “My child is too young to understand words.” b. “My child will begin to speak in sentences by 1 year of age.” N joy RSplay INing GTpeek-a B.CO M c. “My child will probably enU -boo.” d. “While my child is in the hospital, I should let the nurses provide most of the care.” ANS: C
By 9 months of age, infants play simple social games such as patty-cake and peek-a-boo. By 1 year of age, not only do infants recognize their own names but many are able to say three to five words and understand almost 100 words. Extended separations from parents complicate the attachment process and increase the number of caregivers with whom they must interact. Ideally, the parents provide most of the care during hospitalizations. DIF: Apply REF: 366| 367 OBJ: Discuss ways in which you can help parents meet their children's developmental needs. TOP: Assessment MSC: NCLEX: Health Promotion and Maintenance 18. During hospitalization, what should the nurse encourage the parents of an 8-month-old infant
to do? a. Provide as much care as possible. b. Not worry about attachments because the infant is too young to develop them. c. Remember that infants cannot differentiate a stranger from a familiar person. d. Relax and allow nursing staff to care for the child at all times. ANS: A
Canadian Fundamentals of Nursing 6th Edition Potter Test Bank Extended separations from parents complicate the attachment process and increase the number of caregivers with whom the infant must interact. Ideally, the parents provide most of the care during hospitalizations. Close attachment to the primary caregivers, most often parents, is usually established by the age of 8 months. Infants seek out these persons for support and comfort during times of stress. By this age, most infants are able to differentiate a stranger from a familiar person and respond differently to the two. DIF: Apply REF: 367 OBJ: Discuss ways in which you can help parents meet their children's developmental needs. TOP: Implementation MSC: NCLEX: Health Promotion and Maintenance 19. The nursing student correctly explains health promotion teaching points for parents of
toddlers when she makes which statement? a. “Setting consistent, firm limits will help the child cope with the frustration of learning self-control.” b. “Slower development of motor skills prevents the child from participating in self-care activities.” c. “Toddlers have a natural sense of right and wrong and know when they do something wrong.” d. “Temper tantrums should never be tolerated, and toddlers need to do what they are told.” ANS: A
Patience, support, and firm, consistent limits allow toddlers to develop socially acceptable behaviours and to cope with the frustration of learning self-control. Rapid development of motor skills allows the child to participate in self-care activities such as feeding, dressing, and toileting. Because children’s moral development is closely associated with their cognitive N R I dle GT B.C y Mbeginning. Toddlers do not understand abilities, the moral developmenUt ofStodN rs is onlO concepts of right and wrong. Temper tantrums result when restrictions frustrate toddlers. Parents need to provide toddlers with graded independence, allowing them to do things that do not result in harm to themselves or others. DIF: Apply REF: 372 OBJ: Discuss ways in which you can help parents meet their children's developmental needs. TOP: Implementation MSC: NCLEX: Psychosocial Integrity 20. A 5-year-old boy is admitted for surgery to have his tonsils removed. On the basis of
knowledge about children of this age, what would the nurse would plan to do? a. Allow the child to take responsibility for his own preoperative hygienic care. b. Leave the child alone to relax before the procedure. c. Allow the child to handle and look at the equipment when the nurse takes his blood pressure. d. Provide magazines and puzzles for diversion. ANS: C
Preschool children may cooperate if they are allowed to manipulate the equipment. A preschool child is unable to take responsibility for his or her own preoperative hygienic care. Leaving the preschooler alone may increase the child’s anxiety. Magazines and puzzles would be more appropriate activities for the older child. The preschool child likes to engage in pretend play by using the imagination and imitating adult behaviour.
Canadian Fundamentals of Nursing 6th Edition Potter Test Bank DIF: Apply REF: 376-379 OBJ: Describe cognitive and psychosocial development from birth to adolescence. TOP: Implementation MSC: NCLEX: Psychosocial Integrity 21. Which of these statements, if made by a parent, would necessitate further instruction? a. “I should not be surprised that my teenager has so many friends.” b. “I get worried because my teenager thinks he’s indestructible. He takes a lot of
risks.” c. “I should cover for my school-aged child when he makes a mistake until he learns
the ropes.” d. “My 10-year-old child is always hungry right after school, so I usually fix him a
nutritious snack.” ANS: C
School and home influence growth and development, which requires adjustment by parents and by the child. The child learns to cope with rules and expectations presented by the school and by peers. Parents have to learn to allow their child to make decisions, accept responsibility, and learn from life’s experiences. Teenagers typically are very social and have many friends. Adolescents seek a group identity because they need esteem and acceptance. By mid-adolescence, children believe that they are unique and the exception to all rules, which gives rise to their risk-taking behaviours. Obesity occurs because children often rush into the home after school or play and eat the most easily obtainable and appealing foods. Providing nutritious snacks is often the best way to ensure good nutritional intake. DIF: Analyze REF: 380 (Table 22-5) OBJ: Discuss ways in which you can help parents meet their children's developmental needs. TOP: Implementation MSC: NCLEX: Psychosocial Integrity
NURSINGTB.COM
22. Which of these toys, if selected by the parent of a 10-month-old child, would indicate that the
parent has a correct understanding of infant growth and development? a. A game requiring two to four players. b. Electronic games. c. Small, plastic alphabet letters and magnets. d. Plastic stacking rings. ANS: D
Adults facilitate infant learning by planning activities that promote the development of milestones and by providing toys that are safe for the infant to explore with the mouth and manipulate with the hands, such as rattles, wooden blocks, plastic stacking rings, squeezable stuffed animals, and busy boxes. Infants are not capable of participating in small group activities. By age 4, children play in groups of two or three. For the toddler (not the infant), television, videos, electronic games, and computer programs help support development and learning of basic skills. Adults should provide toys that are safe for the infant to explore with the mouth. Small, plastic letters and magnets could be choking hazards for an infant. DIF: Evaluate REF: 367 OBJ: Discuss ways in which you can help parents meet their children's developmental needs. TOP: Implementation MSC: NCLEX: Health Promotion and Maintenance MULTIPLE RESPONSE
Canadian Fundamentals of Nursing 6th Edition Potter Test Bank 1. The nurse should instruct the parents of an adolescent about which of the following health
concerns? (Select all that apply.) a. Signs of substance abuse. b. Suicide prevention. c. Safe sex practices. d. Pregnancy. e. Gonadotropic hormone stimulation. f. Voice changes. ANS: A, B, C, D
All adolescents are at risk for experimental or recreational substance use; some believe that substance use makes them more mature. Suicide is the third leading cause of death in adolescents. Sexually transmitted infections annually affect 3 million sexually active adolescents. Adolescent pregnancy continues to be a major social challenge for our nation. Gonadotropic hormones stimulate ovarian cells to produce estrogen and testicular cells to produce testosterone. These hormones are produced normally and contribute to the development of secondary sex characteristics, such as pubic and axillary hair growth and voice changes, and they play an essential role in reproduction. DIF: Understand REF: 387-392 OBJ: Discuss ways in which you can help parents meet their children's developmental needs. TOP: Implementation MSC: NCLEX: Health Promotion and Maintenance
NURSINGTB.COM
Canadian Fundamentals of Nursing 6th Edition Potter Test Bank
Chapter 23: Young to Middle Adulthood Potter et al: Canadian Fundamentals of Nursing, 6th Edition MULTIPLE CHOICE 1. The nurse is performing a physical examination on a 40-year-old patient. The nurse will
probably find that at this age, the patient is experiencing which of the following physiological changes related to normal aging? a. Decreased heart rate. b. Decreased sense of smell. c. Decreased strength of abdominal muscles. d. Decreased function of the cranial nerves. ANS: C
A physiological change related to normal aging in the middle-aged adult would be decreased strength of abdominal muscles. The middle-aged adult should not have a decreased heart rate, should have a normal sense of smell, and should have normal functioning of the cranial nerves. DIF: Apply REF: 404 (Table 23-3) OBJ: Describe normal physiological changes in young and middle adulthood, including pregnancy. TOP: Assessment MSC: NCLEX: Physiological Integrity 2. The nurse is caring for a hospitalized young man who is uninsured even though he works as a
dishwasher at a local restaurant. He states that he would like to get a better job, but he has no education. How can the nurse best assist this patient psychosocially? N ndRS I rrals GT. B.C OM a. By providing information aU refeN b. By telling the patient that he needs to go back to school. c. By focusing on the patient’s medical diagnoses. d. By expecting the patient to be flexible in his decision making. ANS: A
Support from the nurse, access to information, and appropriate referrals provide opportunities for achievement of a patient’s potential. Many young adults lack the necessary resources or support systems to facilitate further education or development of skills necessary for many positions in the workplace. As a result, some young adults have limited occupational choices. Health is not merely the absence of disease but involves wellness in all human dimensions. Insecure persons tend to be more rigid in making decisions. DIF: Analyze REF: 397| 398 OBJ: Discuss major life events and developmental tasks of young and middle-aged adults. TOP: Implementation MSC: NCLEX: Psychosocial Integrity 3. A nursing student is asked to compare major life events of young adult, middle adult, and
childbearing families. Which statement by the student demonstrates understanding? a. “Young adults have gained sexual experience and do not need sexual education.” b. “Once a woman has her baby, stress levels decrease, as does health risk.” c. “The social pressure to get married is greater now than it ever was.” d. “When both married people work, income is increased, but so is stress.”
Canadian Fundamentals of Nursing 6th Edition Potter Test Bank ANS: D
The two-career family has benefits and liabilities. Stressors result from transfer to a new city; increased expenditures of physical, mental, or emotional energy; child care demands; and household needs. To avoid stress, partners should share all responsibilities. Young adults are at risk for sexually transmitted infections; therefore, there is an increased need for education regarding mode of transmission, prevention, and symptom recognition and management for sexually transmitted infections. The stress that many women experience after childbirth has a significant effect on their health. Social pressure to get married is not as great as it once was, and many young adults do not marry until their late 20s, early 30s, later, or not at all. DIF: Understand REF: 399 OBJ: Discuss major life events and developmental tasks of young and middle-aged adults. TOP: Assessment MSC: NCLEX: Psychosocial Integrity 4. The nurse knows that the young adult patient understands the health risks that affect his or her
age group when the patient makes which statement? a. “It’s probably safe for me to start smoking. At my age, there’s not enough time for cancer to develop.” b. “I am sure that I am going to get emphysema. Both my mother and my aunt had it. It’s genetic.” c. “Controlling the amount of stress in my life may decrease the risk of illness.” d. “I don’t do drugs. I do drink coffee, but caffeine is not a drug. It is perfectly safe and has no side effects.” ANS: C
Lifestyle habits that activate the stress response increase the risk of illness. Smoking is a well-documented risk factor for pulmonary, cardiac, and vascular disease, not only in smokers N R ose IN G B.C -hand M smoke. The presence of certain but also in individuals who areUexpS d toTsecondO chronic illnesses in the family increases a family member’s risk of developing a disease. Family risk is distinct from hereditary disease. Caffeine is a drug; it is a naturally occurring legal stimulant that is readily available. Caffeine stimulates catecholamine release, which, in turn, stimulates the central nervous system; it also increases gastric acid secretion, heart rate, and basal metabolic rate. DIF: Understand REF: 400| 401 OBJ: Discuss major life events and developmental tasks of young and middle-aged adults. TOP: Evaluate MSC: NCLEX: Psychosocial Integrity 5. When choosing an appropriate topic for a young adult health fair, the nurse ranks which topic
as least relevant? a. Unplanned pregnancies. b. Menopause and climacteric factors. c. Smoking cessation. d. Alcohol and drug use. ANS: B
Canadian Fundamentals of Nursing 6th Edition Potter Test Bank The onset of menopause and the climacteric affect the sexual health of the middle-aged adult, not the young adult. Unplanned pregnancies are a continued source of stress that can result in adverse health outcomes for the mother (young adult), infant, and family. Smoking is a well-documented risk factor for pulmonary, cardiac, and vascular diseases in smokers and in individuals who are exposed to second-hand smoke, and it constitutes a health risk for young adults. Substance abuse directly or indirectly contributes to mortality and morbidity in young adults. DIF: Understand REF: 401| 405 OBJ: Discuss major life events and developmental tasks of young and middle-aged adults. TOP: Assessment MSC: NCLEX: Health Promotion and Maintenance 6. During middle adulthood, the 50-year-old patient is likely to adapt favourably to a changing
body image if he or she does what? a. Decreases the amount of physical exercise. b. Eats a diet composed of 40% fat. c. Gets less than 5 hours of sleep per night. d. Engages in good hygiene practices. ANS: D
High self-esteem, a favourable body image, and a positive attitude toward physiological changes occur when adults engage in physical exercise, balanced diets, adequate sleep, and good hygiene practices that promote vigorous, healthy bodies. DIF: Knowledge REF: 404 OBJ: Discuss major life events and developmental tasks of young and middle-aged adults. TOP: Assessment MSC: NCLEX: Health Promotion and Maintenance
NURSINGTB.COM
7. A patient states that she is pregnant, is concerned because she does not know what to expect,
and wants her husband to play an active part in the birthing process. What should the nurse tell the patient? a. Childbirth classes can prepare pregnant women and their partners for what is coming. b. The frequency of sexual intercourse is key to helping the husband feel valued. c. After the birth, the stress of pregnancy will disappear and will be replaced by relief. d. After the baby is born, the wife should accept the extra responsibilities of motherhood. ANS: A
Childbirth education can prepare pregnant women, their partners, and other support persons to participate in the birthing process. The psychodynamic aspect of sexual activity is as important as the type or frequency of sexual intercourse to young adults. The stress that many women experience after childbirth has a significant effect on their health. To avoid stress in a two-career family, partners should share all responsibilities. DIF: Apply REF: 402-404 OBJ: Discuss major life events and developmental tasks of young and middle-aged adults. TOP: Assessment MSC: NCLEX: Health Promotion and Maintenance 8. What do changing norms and values about family life in Canada reveal?
Canadian Fundamentals of Nursing 6th Edition Potter Test Bank a. b. c. d.
Basic shifts in attitudes in Canadian society. Greater resistance to cohabitation without marriage. Decreased numbers of infants born to unmarried women. Greater support and acceptance from the health care system.
ANS: A
Changing norms and values about family life in Canada reveal basic shifts in attitudes in Canadian society. The trend toward greater acceptance of cohabitation without marriage is a factor in the greater numbers of infants being born to unmarried women. Many times, parents from family structures that do not conform to the norm feel lack of support and even bias from the health care system. DIF: Knowledge REF: 399 OBJ: Discuss the significance of family in the life of the adult. MSC: NCLEX: Health Promotion and Maintenance
TOP: Assessment
9. When describing relevant family psychosocial factors in middle adulthood that cause stress,
the nurse would not include which of the following? a. Singlehood and feeling isolated. b. Choices stemming from marital changes. c. Financial security and certainty. d. Planning for the future when children leave home. ANS: C
In the middle adult years, as children depart from the household, the family enters the postparental family stage. Time and financial demands on parents decrease, and the couple faces the task of redefining their own relationship. Psychosocial factors involving the family include the stresses of singlehNood alT cB ha. ngCes, UR, SmIarit NG OMtransition of the family as children leave home, and the care of aging parents. DIF: Understand REF: 405| 406 OBJ: Discuss the significance of family in the life of the adult. MSC: NCLEX: Psychosocial Integrity
TOP: Assessment
10. What should the nurse recognize when comparing the physical changes in young and middle
adulthood? a. Fertility issues do not occur in young adulthood. b. Young adults are quite active but are at risk for illness in later years. c. Young adults tend to suffer more from severe illness. d. Exercise is less important in young adulthood than in middle adulthood. ANS: B
Young adults are generally active and have a minimum of major health problems. However, their lifestyles may put them at risk for illnesses or disabilities during their middle or older years. An estimated 10% to 15% of reproductive couples are infertile, and many are young adults. Exercise in young adulthood is increasingly important for preventing or decreasing the risk for chronic health conditions such as high blood pressure, obesity, and diabetes that develop later in life. DIF: Understand REF: 404 OBJ: Describe normal physiological changes in young and middle adulthood, including pregnancy. TOP: Assessment MSC: NCLEX: Health Promotion and Maintenance
Canadian Fundamentals of Nursing 6th Edition Potter Test Bank
11. During a routine physical assessment, the nurse obtaining a health history notes that a
50-year-old woman reports pain and redness in the right breast. What is the nurse’s best action in response to this finding? a. Explain to the patient that breast tenderness is normal at her age. b. Tell the patient that redness is not a cause for concern and is quite common. c. Assess the patient as thoroughly as possible. d. Inform her that redness is the precursor to normal unilateral breast enlargement. ANS: C
A comprehensive assessment offers direction for health promotion recommendations, as well as for planning and implementing any acutely needed intervention. Redness or pain in the breasts and increased size of one breast are abnormal physical assessment findings in middle adulthood. DIF: Apply REF: 401| 402 OBJ: Describe normal physiological changes in young and middle adulthood, including pregnancy. TOP: Assessment MSC: NCLEX: Health Promotion and Maintenance 12. A 55-year-old female presents to the outpatient clinic describing irregular menstrual periods
and hot flashes. What should the nurse explain? a. Those symptoms are normal when a woman undergoes the climacteric. b. An assessment is not really needed because these problems are normal for older women. c. The patient’s age and symptoms point toward normal menopause. d. The patient should stop regular exercise because that is probably causing her symptoms. ANS: C
NURSINGTB.COM
The most significant physiological changes during middle age are menopause in women and the climacteric in men. The nurse should continue with the examination because a comprehensive assessment offers direction for health promotion recommendations, as well as for planning and implementing any acutely needed interventions. High self-esteem, a favourable body image, and a positive attitude toward physiological changes occur when adults engage in physical exercise, balanced diets, adequate sleep, and good hygiene practices that promote vigorous, healthy bodies. DIF: Apply REF: 405 OBJ: Describe normal physiological changes in young and middle adulthood, including pregnancy. TOP: Assessment MSC: NCLEX: Health Promotion and Maintenance 13. The nurse is teaching a class to pregnant women about common physiological changes during
pregnancy. Which statement by the nurse accurately describes these changes? a. “Pregnancy enhances your ability to cope with stress.” b. “Being nauseated and feeling tired will not affect your physical body image.” c. “You and your partner may experience feelings of uncertainty about assuming the roles of parents.” d. “Returning home after delivery will rejuvenate you and foster independence.” ANS: C
Canadian Fundamentals of Nursing 6th Edition Potter Test Bank Expectant parents think about and have feelings of uncertainty about impending role changes. Parents need reassurance that childbirth and childrearing are natural and positive experiences but are also stressful. Parents often are unable to cope with particular stressors. Morning sickness and fatigue contribute to poor body image. New mothers often return home from the hospital fatigued and unfamiliar with infant care. DIF: Understand REF: 402-404 OBJ: Discuss cognitive and psychosocial changes that occur during the adult years. TOP: Implementation MSC: NCLEX: Health Promotion and Maintenance 14. A nurse is about to discuss the risks of repeated sun exposure with a young adult patient.
Which of these statements would be most expected from this patient before the discussion? a. “I’ll make an appointment with my doctor right away for a full skin check.” b. “I should consider participating in a health fair about safe sun practices.” c. “I have a mole that has been bothering me. I’ll call my family doctor for an appointment to get it checked.” d. “I’ve had this mole my whole life. So what if it changed colour? My skin is fine.” ANS: D
Young adults often ignore physical symptoms and often postpone seeking health care. Making an appointment with the doctor and participating in health fairs are not typical behaviours of young adults. DIF: Apply REF: 398 OBJ: Discuss cognitive and psychosocial changes that occur during the adult years. TOP: Implementation MSC: NCLEX: Health Promotion and Maintenance 15. When performing a thoroughNpU syRcS hoIsoNcG iaT l aBss.eC ssO mM ent on a young adult, what must the nurse
realize? a. Having a job is the best way to relieve stress. b. Although psychologically disturbing, stress does not lead to physical illness. c. Change is inevitable and is not a factor in stress-related illness. d. Psychosocial health is often related to job and family stress. ANS: D
The psychosocial health concerns of young adults are often related to job and family stressors. If stress is prolonged and the patient is unable to adapt to the stressor, health problems will develop. Job assessment also includes conditions and hours, duration of employment, changes in sleep or eating habits, and evidence of increased irritability or nervousness. When a patient seeks health care and presents stress-related symptoms, the nurse needs to assess for the occurrence of a life change event. DIF: Apply REF: 398| 399 OBJ: Describe health concerns of young and middle-aged adults. TOP: Assessment MSC: NCLEX: Psychosocial Integrity 16. A 25-year-old patient is brought to the hospital by police after crashing his car in a high-speed
chase when trying to avoid arrest for spousal abuse. What should the nurse do? a. Question the patient about drug use. b. Offer the patient a cup of coffee to calm his nerves. c. Be aware that substance abuse is usually obvious.
Canadian Fundamentals of Nursing 6th Edition Potter Test Bank d. Deal with the issue at hand, and put off asking about previous illnesses. ANS: A
Reports of arrests because of driving while intoxicated, wife or child abuse, or disorderly conduct are reasons for the nurse to investigate the possibility of drug abuse more carefully. Caffeine is a legal drug, naturally occurring in coffee, that stimulates the central nervous system and is not the choice for calming nerves. Substance abuse is not always diagnosable, particularly in its early stages. The nurse may obtain important information by making specific inquiries about past medical problems, changes in food intake or sleep patterns, and problems of emotional lability. DIF: Apply REF: 401 OBJ: Describe health concerns of young and middle-aged adults. TOP: Implementation MSC: NCLEX: Health Promotion and Maintenance 17. A 61-year-old obese patient receives diagnoses of type 2 diabetes and high blood pressure.
The patient states that he is upset about the diet restrictions imposed by the treatment regimen. What is the nurse’s best approach? a. Tell the patient that he must do what the doctor tells him. b. Offer counselling on nutrition and exercise. c. Tell the patient about what happened to other patients who did not change their lifestyle. d. Explain that he needs to accept the care provider’s advice without question if he wants to get better. ANS: B
Counselling related to physical activity and nutrition is an important component of the plan of care for overweight and obesN eU paR tien SIts.NTGoThBel.pCthe OMpatients develop positive health habits, the nurse becomes a teacher and a facilitator, providing information and positive reinforcement. Ultimately, however, the patient decides which behaviours will become habits of daily living. Scare tactics do not usually work. By providing information about how the body works and how patients form and change habits, the nurse raises the patient’s level of knowledge regarding the potential effects of behaviour on health. The nurse should encourage patients to express their feelings so as to promote problem solving and recognition of risk factors by patients themselves. DIF: Apply REF: 406| 407 OBJ: Describe health concerns of young and middle-aged adults. TOP: Implementation MSC: NCLEX: Health Promotion and Maintenance MULTIPLE RESPONSE 1. What are the most common life events that occur during young adulthood? (Select all that
apply.) a. Refining self-perception and ability for intimacy. b. Achievement of personal and occupational goals. c. Examination of life goals and relationships. d. Rejection of culture-bound definitions of health and illness. e. Women surrendering careers to raise families. ANS: A, B, C
Canadian Fundamentals of Nursing 6th Edition Potter Test Bank Between the ages of 23 and 28, the person refines self-perception and ability for intimacy. From 29 to 34, the person directs enormous energy toward achievement and mastery of the surrounding world. The years from 35 to 43 are a time of vigorous examination of life goals and relationships. Often the stresses of this re-examination result in a “midlife crisis.” Each person holds culture-bound definitions of health and illness. Knowing too little about the patient’s self-perception or beliefs regarding health and illness creates conflict between the nurse and the patient. Women often continue to work during the childrearing years, and many women struggle with the enormity of balancing three careers: wife, mother, and employee. DIF: Knowledge REF: 398 OBJ: Discuss major life events and developmental tasks of young and middle-aged adults. TOP: Assessment MSC: NCLEX: Health Promotion and Maintenance 2. Nurses need to provide competent care to young and middle-aged adult patients. Why must
nurses be knowledgeable about developmental theories to care for this group? (Select all that apply.) a. These theories provide nurses with a basis for understanding the life events and developmental tasks of young and middle-aged adults. b. It is important to understand societal structures and roles because they have not changed in the past 20 or 30 years. c. Patients present challenges to nurses, many of whom are young or middle-aged adults coping with the demands of their respective developmental period. d. Nurses need to recognize the needs of their patients even if they are not experiencing the same challenges and events. ANS: A, C, D
Developmental theories provide nurses with a basis for understanding the life events and N RSINGTB.COM developmental tasks of young U and middle-aged adults. Patients present challenges to nurses, many of whom are also young or middle-aged adults coping with the demands of their respective developmental period. Nurses need to recognize the needs of their patients even if they are not experiencing the same challenges and events. Faced with a societal structure that differs greatly from the norms of 20 or 30 years ago, both men and women are assuming different roles in today’s society. DIF: Knowledge REF: 397| 398 OBJ: Discuss major life events and developmental tasks of young and middle-aged adults. TOP: Assessment MSC: NCLEX: Health Promotion and Maintenance 3. When providing prenatal care, what information does the nurse expect to provide? (Select all
that apply.) a. Protecting against urinary infection. b. No longer needing condoms. c. Exercise patterns. d. Proper diet. e. Physical assessments only during the last trimester. ANS: A, C, D
Canadian Fundamentals of Nursing 6th Edition Potter Test Bank Prenatal care includes a thorough physical assessment of the pregnant woman during regularly scheduled intervals (not just the last trimester). Information regarding sexually transmitted infections, other vaginal infections, and urinary infections that will adversely affect the fetus, as well as counselling about exercise patterns, diet, and child care, are important for a pregnant woman. DIF: Knowledge REF: 402-404 OBJ: Describe health concerns of young and middle-aged adults. TOP: Implementation MSC: NCLEX: Health Promotion and Maintenance
NURSINGTB.COM
Canadian Fundamentals of Nursing 6th Edition Potter Test Bank
Chapter 24: Older Persons Potter et al: Canadian Fundamentals of Nursing, 6th Edition MULTIPLE CHOICE 1. As the aging population in Canada increases, which of the following is true? a. The baby boomer generation accounts for a very small percentage of this group. b. The average lifespan has also increased. c. The population segment of people older than 85 is decreasing. d. Diversity of this age group will certainly decrease. ANS: B
Part of that increase is a result of the extension of the average lifespan. In 2012, the average life expectancy at birth for Canadians was 81.1, in comparison with 70.9 in 1991. Two other factors that contribute to the projected increase in the number of older persons are the aging of the baby boom generation and the growth of the population of people older than 85. The baby boomers are the large group of persons born between 1946 and 1964. The diversity of the group older than 65 will also possibly increase. DIF: Remember REF: 411 OBJ: Discuss demographic trends related to older persons in Canada. TOP: Assessment MSC: NCLEX: Health Promotion and Maintenance 2. As a patient ages, how should the nursing plan of care change? a. It should be standardized because all geriatric patients have the same needs. b. It needs to be individualized according to the patient’s unique needs. NUolo RS IlG B.C. M c. It should be based on chron gicaN agT e aloneO d. It focuses on the disabilities that all aging persons face. ANS: B
Every older person is unique, and the nurse needs to approach each one as a unique individual. The nursing care of older persons poses special challenges because of great variation in their physiological, cognitive, and psychosocial health. Nurses need to take into account the cultural, ethnic, and racial diversity of this population (not just age) as they care for older persons from these groups. Aging does not inevitably lead to disability and dependence. DIF: Understand REF: 412 OBJ: Identify common myths and stereotypes about older persons. TOP: Assessment MSC: NCLEX: Health Promotion and Maintenance 3. Which of these findings, if identified in a patient on a gerontological unit, would be most
surprising to a culturally sensitive nurse? a. The older person’s not being functionally independent. b. Preferences in food, music, and religion. c. Use of conventions of the handshake, silence, and eye contact. d. Personal health practices and spiritual resources. ANS: A
Canadian Fundamentals of Nursing 6th Edition Potter Test Bank Most older people remain functionally independent despite the increasing prevalence of chronic disease. Examples of culturally competent nursing approaches to older persons include respect for preferences in food, music, and religion; appropriate use of conventions of the handshake, silence, and eye contact; use of interpreters; use of physical assessment norms appropriate for the ethnic group; and asking about personal health practices, family customs, lifestyle preferences, and spiritual resources. DIF: Analyze REF: 412 OBJ: Identify common myths and stereotypes about older persons. TOP: Assessment MSC: NCLEX: Health Promotion and Maintenance 4. Which of the following statements by a new graduate nurse should be corrected by an
experienced nurse? a. “Most older patients are ill and disabled. That’s why we care for so many of them in the hospital.” b. “Many older persons are still interested in sexual relations.” c. “Patients over age 65 are still lifelong learners.” d. “Many older patients remain independent enough to live alone.” ANS: A
Older persons are sometimes stereotyped as ill and disabled. However, although many experience chronic conditions or have at least one disability that limits performance of activities of daily living (ADLs), only 23% of older person describe their health as poor or fair. Older persons do report continued enjoyment of sexual relationships. Although changes in vision or hearing and reduced energy and endurance sometimes affect the process of learning, older persons are lifelong learners. Most older persons live in noninstitutional settings with family members or alone. B.C M
N R I G U S N T
O
DIF: Remember REF: 412| 413 OBJ: Identify common myths and stereotypes about older persons. TOP: Assessment MSC: NCLEX: Health Promotion and Maintenance 5. Which teaching strategy is best to utilize with older persons? a. Provide several topics of discussion at once to promote independence and making
choices. b. Avoid uncomfortable silences after questions by helping patients complete their
statements. c. Ask patients to recall past experiences that correspond with their interests. d. Speak in a high pitch to help patients hear better. ANS: C
Teaching strategies include the use of past experiences to connect new learning with previous knowledge, focusing on a single topic to help the patient concentrate, giving the patient enough time in which to respond because older persons’ reaction times are longer than those of younger persons, and keeping the tone of voice low; older persons are able to hear low-pitched sounds better than high-pitched sounds. DIF: Understand REF: 412| 413 OBJ: Identify nursing interventions related to the physiological, cognitive, and psychosocial changes of aging. TOP: Implementation MSC: NCLEX: Health Promotion and Maintenance
Canadian Fundamentals of Nursing 6th Edition Potter Test Bank
6. An older patient has fallen and broken his hip. As a consequence, the patient’s family is
concerned about his ability to care for himself, especially during his convalescence. What should the nurse do? a. Stress that older patients usually ask for help when needed. b. Inform the family that placement in a nursing centre is a permanent solution. c. Tell the family to enroll the patient in a ceramics class to maintain his quality of life. d. Provide information and answer questions as family members make choices among care options. ANS: D
Nurses assist older persons and their families by providing information and answering questions as they make choices among care options. Some older persons deny functional declines and refuse to ask for assistance with tasks that place their safety at great risk. The decision to enter a nursing centre is never final, and sometimes a resident of a nursing centre is discharged to home or to another residence for less acute care. What defines quality of life varies from person to person. Nurses must listen to what the older individual considers to be most important rather than making assumptions about the person’s priorities. DIF: Remember REF: 414 OBJ: Identify nursing interventions related to the physiological, cognitive, and psychosocial changes of aging. TOP: Implementation MSC: NCLEX: Health Promotion and Maintenance 7. What is the best suggestion a nurse could make to a family requesting help in selecting a local
nursing centre? N R I G B.C M a. Suggest choosing a nursingUcenStre N thatTis as saO nitary as possible. The closer the centre is to hospital standards, the better. b. Have family members evaluate nursing home staff according to their ability to get tasks done efficiently. c. Make sure that nursing home staff members get patients out of bed every day for the entire day. d. Explain that it is probably best for the family to visit the centre and inspect it personally. ANS: D
An important step in the process of selecting a nursing home is to visit the nursing home. The nursing home should not feel like a hospital. It is a home, a place where people live. Members of the nursing home staff should focus on the person, not the task. Residents should be out of bed and dressed according to their preferences, not staff preferences. DIF: Apply REF: 414 OBJ: Discuss common developmental tasks of older persons. MSC: NCLEX: Health Promotion and Maintenance
TOP: Implementation
8. A 70-year-old patient who suffers from worsening dementia is no longer able to live alone.
When discussing health care services and possible long-term living arrangements with the patient’s only son, what should the nurse suggest? a. An apartment setting with neighbours close by. b. Weekly home health visits.
Canadian Fundamentals of Nursing 6th Edition Potter Test Bank c. A nursing centre because home care is no longer safe. d. That placement is irrelevant because the patient is retreating to a place of
inactivity. ANS: C
Some family caregivers consider nursing centre placement when in-home care becomes increasingly difficult, or when convalescence from hospitalization requires more assistance than the family is able to provide. An apartment setting and home health visits are not appropriate because older persons who cannot take care of themselves are not safe living alone at other times. Others are unable to engage in activities designed to benefit older persons such as senior health promotion activities (such as some health visits) and thus do not receive the benefits that these programs offer. Worsening dementia does not necessarily mean a retreat into inactivity, but it does require a realistic review of strengths and limitations. DIF: Apply REF: 414 OBJ: List the types of community-based and institutional health care services available to older persons. TOP: Implementation MSC: NCLEX: Health Promotion and Maintenance 9. Several theories on aging have been put forth; how should the nurse use these theories? a. To guide nursing care. b. To explain the stochastic view of genetically programmed physiological changes. c. To select one theory to guide nursing care for all geriatric patients. d. To understand the nonstochastic views of aging as the result of cellular damage. ANS: A
Although theories on aging are in various stages of development and have limitations, the nurse should use them to incrN eas e un stan di. ngCof UR SIder NG TB OMthe phenomena affecting the health and well-being of older persons and to guide nursing care. According to stochastic theories, aging is the result of random cellular damage occurring over time. There exists no single universally accepted theory that predicts and explains the complexities of the aging process. According to nonstochastic theories, aging is the result of genetically programmed physiological mechanisms within the body. DIF: Apply REF: 413 OBJ: Describe the concepts of aging well and quality of life. MSC: NCLEX: Health Promotion and Maintenance
TOP: Implementation
10. How would the nurse correctly describe psychosocial theories on aging? a. As theories that describe changes in roles and relationships in older persons. b. As theories that emphasize that all adults age in similar ways. c. As theories that stress the need for older people to discontinue activities as they
age. d. As theories that describe behaviour patterns for all aging persons as unpredictable. ANS: A
Canadian Fundamentals of Nursing 6th Edition Potter Test Bank Psychosocial theories of aging explain changes in behaviours, roles, and relationships that occur with aging. Although some theories generalize about aging, each individual ages uniquely both biologically and psychosocially. According to the activity theory, the continuation of activities performed during middle age are necessary for successful aging. According to the continuity theory, personality remains the same and behaviour becomes more predictable as people age. DIF: Remember REF: 419 OBJ: Discuss issues related to psychosocial changes connected with aging. TOP: Assessment MSC: NCLEX: Psychosocial Integrity 11. When comparing developmental tasks of middle-aged persons versus older persons, what
should the nurse infer? a. Learning to cope with loss is most common during the middle adult years. b. After age 65, most older persons age both biologically and psychologically the same way. c. Older persons will need nursing assistance to deal with loss. d. Older persons fear and resent retirement as a disruption of their lifestyle. ANS: C
Some older persons deny their own aging in ways that are potentially problematic. For example, some older persons deny functional declines and refuse to ask for assistance with tasks that place their safety at great risk. The need to cope with loss is much greater in the older person population. Most older persons cope with the death of a spouse. Some must cope with the death of adult children and grandchildren. All experience the death of friends. The ways that older persons adjust to the changes of aging are highly individualized. Many older persons welcome retirement as a time to pursue new interests and hobbies, participate in N R edu INcati GT B.Ctart M a new business career. volunteer activities, continue thUeir S on, or sO DIF: Understand REF: 415| 419 OBJ: Discuss common developmental tasks of older persons. MSC: NCLEX: Health Promotion and Maintenance
TOP: Assessment
12. An 80-year-old male is brought to the emergency department with an exacerbation of chronic
obstructive pulmonary disease (COPD). He states that he quit smoking 30 years ago, so it can’t be COPD. He argues, “It’s just these colds I’ve been getting. They’re just getting worse and worse.” The nurse understands that a. These symptoms are more associated with normal aging than with disease. b. Older persons should be encouraged to maintain physical exercise and activity. c. The patient’s age will require adjustment of lifestyle to one of inactivity. d. Older persons usually are aware and accepting of the aging process. ANS: B
Older persons should be encouraged to maintain physical exercise and activity, as tolerated. The primary benefits of exercise include maintaining and strengthening functional ability and promoting a sense of enhanced well-being. The COPD exacerbation is associated with the disease, not normal aging. The presence of disease sometimes alters the timing of the changes or their impact on daily life. Acceptance of personal aging does not mean retreat into inactivity, but it does require a realistic review of strengths and limitations. Some older persons find it difficult to accept that they are aging.
Canadian Fundamentals of Nursing 6th Edition Potter Test Bank DIF: Apply REF: 424 OBJ: Describe common physiological changes associated with aging. TOP: Planning MSC: NCLEX: Health Promotion and Maintenance 13. During assessment of an older person’s skin integrity, expected findings include which of the
following? a. Decreased elasticity. b. Oily skin. c. Increased facial hair in men. d. Faster nail growth. ANS: A
Loss of skin elasticity is a common finding in the older person. Other common findings include pigmentation changes, glandular atrophy (oil, moisture, sweat glands), thinning hair (facial hair: decreased in men, increased in women), slower nail growth, and atrophy of epidermal arterioles. DIF: Remember REF: 416 (Table 24-1) OBJ: Describe common physiological changes associated with aging. TOP: Assessment MSC: NCLEX: Physiological Integrity 14. An older patient in no acute distress reports being less able to taste and smell. What is the
nurse’s best response to this information? a. Notify the physician immediately to rule out cranial nerve damage. b. Perform testing on the vestibulocochlear nerve and a hearing test. c. Schedule the patient for an appointment at a smell and taste disorders clinic. d. Explain to the patient that diminished senses are normal findings. ANS: D
NURSINGTB.COM
Diminished taste and smell senses are common findings in older persons. Scheduling an appointment at a smell and taste disorders clinic, testing the vestibulocochlear nerve, or an attempt to rule out cranial nerve damage is unnecessary at this time per the information provided. DIF: Apply REF: 416 (Table 24-1) OBJ: Describe common physiological changes associated with aging. TOP: Assessment MSC: NCLEX: Physiological Integrity 15. Which symptom is an expected cognitive change in the older person? a. Disorientation. b. Slower reaction time. c. Poor judgement. d. Loss of language skills. ANS: B
Slower reaction time is a common change in the older person owing to degeneration of nerve cells, decreased neurotransmitters, and decreased rate of conduction of impulses. Symptoms of cognitive impairment, such as disorientation, loss of language skills, loss of the ability to calculate, and poor judgement are not normal aging changes and require further investigation of underlying causes. DIF: Understand
REF: 416 (Table 24-1)
Canadian Fundamentals of Nursing 6th Edition Potter Test Bank OBJ: Describe common physiological changes associated with aging. TOP: Assessment MSC: NCLEX: Physiological Integrity 16. A patient with gradual, progressive cognitive impairment (dementia) is admitted to the
nursing unit after hip replacement surgery. Which of the following is a nursing care principle for care of cognitively impaired older persons? a. Maintain physical health. b. Evaluate the patient’s manifestations of standard symptoms. c. Assist patient with all ADLs. d. Isolate patients to protect others. ANS: A
The nurse works to monitor and maintain physical health. The nurse should also assess the person’s unique manifestations of the disease as it progresses while facilitating independent performance of ADLs. Social interaction based on the patient’s abilities is to be promoted. DIF: Apply REF: 418| 419 OBJ: Identify nursing interventions related to the physiological, cognitive, and psychosocial changes of aging. TOP: Planning MSC: NCLEX: Psychosocial Integrity 17. To promote physical well-being and socialization in an older person, what should the nurse
realize? a. Social isolationism is always a chosen behaviour. b. Body image plays no role in decision making by the older person. c. No community resources are focused on the older person. d. Older persons may have a functional purpose in social arenas. ANS: D
NURSINGTB.COM
Social service agencies in most communities welcome older persons as volunteers and provide the opportunity for older persons to serve while meeting their socialization or other needs. Although some older persons choose isolation or a lifelong pattern of reduced interaction with others, other older persons do not choose isolation but are vulnerable to its consequences. Some older persons withdraw from social interaction because of feelings of rejection. These older persons see themselves as unattractive and rejected because of changes in their personal appearance due to normal aging changes or because of body image changes. Many communities have outreach programs designed to make contact with isolated older persons. DIF: Understand REF: 420 OBJ: Identify nursing interventions related to the physiological, cognitive, and psychosocial changes of aging. TOP: Implementation MSC: NCLEX: Psychosocial Integrity 18. A male older patient expresses his concern and anxiety about decreased penile firmness
during erection. What is the nurse’s best response? a. Explain that over time, his libido will decrease, as will the frequency of sexual activity. b. Tell the patient to double his antidepressant medication to increase his libido. c. Tell the patient that this change is expected in aging persons. d. Tell the patient that touching should be avoided unless intercourse is planned. ANS: C
Canadian Fundamentals of Nursing 6th Edition Potter Test Bank Decreased firmness during erection is an expected change in aging persons. Libido does not necessarily decrease as one ages. Many older persons use prescription medications that depress sexual activity such as antihypertensives, antidepressants, sedatives, or hypnotics. Touch complements traditional sexual methods or serves as an alternative sexual expression when physical intercourse is not desired or possible. DIF: Apply REF: 420 OBJ: Identify nursing interventions related to the physiological, cognitive, and psychosocial changes of aging. TOP: Implementation MSC: NCLEX: Health Promotion and Maintenance 19. A patient asks the nurse what the term polypharmacy means. The nurse defines this term as a. Multiple side effects experienced when taking a medication. b. The concurrent use of many medications. c. The many adverse drug effects reported to the pharmacy. d. The risks of medication effects due to aging. ANS: B
Polypharmacy refers to the concurrent use of many medications. It does not have anything to do with side effects, adverse drug effects, or risks of medication use due to aging. DIF: Remember REF: 426 TOP: Implementation
OBJ: Describe selected health concerns of older persons. MSC: NCLEX: Health Promotion and Maintenance
20. An outcome for an older patient living alone is to be free from falls. Which of these
statements by a patient indicates that teaching on safety concerns has been effective? a. “I’ll leave my throw rugs in place so that my feet won’t touch the cold tile.” b. “I’ll take my time gettingNuU INthGe T pR frS om beBd.oC r cOhM air.” c. “I should wear my favourite smooth bottom socks to protect my feet when walking around.” d. “I will have my son dim the lighting outside to decrease the glare in my eyes.” ANS: B
Older persons taking medications with adverse effects such as postural hypotension, dizziness, or sedation need to be aware of these potential effects and to take precautions such as changing position slowly or ambulating with assistance if unsteady. Household items that are easy to trip over, such as throw rugs, are a risk factor for falls. Other risk factors include wearing shoes in poor repair or slippery soles. Impaired vision and poor lighting are other risk factors. DIF: Evaluate REF: 425 TOP: Implementation
OBJ: Describe selected health concerns of older persons. MSC: NCLEX: Health Promotion and Maintenance
21. One of the greatest challenges for the nurse caring for older persons is ensuring safe
medication use. One way to reduce the risks associated with medication usage is to a. Periodically review the patient’s list of medications. b. Inform the patient that polypharmacy is to be avoided at all cost. c. Be aware that medication is absorbed the same way regardless of patient age. d. Focus only on prescribed medications. ANS: A
Canadian Fundamentals of Nursing 6th Edition Potter Test Bank Periodic and thorough review of all medications is important to restrict the number of medications used to the fewest necessary to ensure the greatest therapeutic benefit with the least amount of harm. Although polypharmacy reflects inappropriate prescribing, the concurrent use of multiple medications is necessary in situations where an older person has multiple acute and chronic conditions. Older persons are at risk for adverse drug effects because of age-related changes in the absorption, distribution, metabolism, and excretion of drugs. Work collaboratively with the older person to ensure safe and appropriate use of all medications—both prescribed medications and over-the-counter medications. DIF: Apply TOP: Assessment
REF: 426 OBJ: Describe selected health concerns of older persons. MSC: NCLEX: Health Promotion and Maintenance
22. An older patient has developed acute confusion. The patient has been on tranquilizers for the
past week. The patient’s vital signs are normal. What should the nurse do? a. Take into account age-related changes in body systems that affect pharmacokinetic activity. b. Increase the dose of tranquilizer if the cause of the confusion is an infection. c. Note when the confusion occurs and medicate before that time. d. Restrict telephone usage to prevent further confusion. ANS: A
Sedatives and tranquilizers sometimes prescribed for acutely confused older persons sometimes cause or exacerbate confusion. Carefully administer drugs used to manage confused behaviours, taking into account age-related changes in body systems that affect pharmacokinetic activity. When confusion has a physiological cause (such as an infection), specifically treat that cause, rather than the confused behaviour. When confusion varies by time of day or is related to environmental factors, nonpharmacological measures such as N mReaSnin IN G , B.C M making the environment more U gfulT providO ing adequate light, etc., should be used. Making telephone calls to friends or family members allows older persons to hear reassuring voices, which may be beneficial. DIF: Apply REF: 426 TOP: Implementation
OBJ: Describe selected health concerns of older persons. MSC: NCLEX: Health Promotion and Maintenance
23. Which of these assessments of an older adult, who has a urinary tract infection, requires an
immediate nursing intervention? a. Presbycusis. b. Confusion. c. Death of a spouse 3 months ago. d. Temperature of 36.4°C (97.6°F). ANS: B
Confusion is a common manifestation in older persons with urinary tract infection; however, the cause requires further assessment. There may be another reason for the confusion. Confusion is not a normal finding in the older person, even though it is commonly seen with concurrent infections. Difficulty hearing, presbycusis, is an expected finding in an older person. Coping with the death of a spouse is a psychosocial concern to be addressed after the acute physiological concern in this case. Older persons tend to have lower temperatures, so the nurse needs to assess for slight elevations. A temperature of 36.4°C (97.6°F) is within normal limits.
Canadian Fundamentals of Nursing 6th Edition Potter Test Bank DIF: Apply REF: 415 OBJ: Identify nursing interventions related to the physiological, cognitive, and psychosocial changes of aging. TOP: Planning MSC: NCLEX: Physiological Integrity 24. Which of these patient statements is the most reliable indicator that an older person has the
correct understanding of health promotion activities? a. “I need to increase my fat intake and limit protein.” b. “I should discontinue my fitness club membership for safety reasons.” c. “I’m up to date on my immunizations, but at my age, I don’t need the tetanus vaccine.” d. “I still keep my dentist appointments even though I have partials now.” ANS: D
General preventive measures for the nurse to recommend to older persons include keeping periodic dental appointments to promote good oral hygiene, eating a low-fat, well-balanced diet, exercising regularly, and maintaining immunizations for influenza, pneumococcal pneumonia, and tetanus. DIF: Evaluate REF: 422 (Box 24-8) OBJ: Describe selected health concerns of older persons. MSC: NCLEX: Health Promotion and Maintenance
TOP: Evaluate
25. A 72-year-old woman was recently widowed. She worked as a teller at a bank for 40 years
and has been retired for the past 5 years. She never learned how to drive. She lives in a rural area that does not have public transportation. Which of the following psychosocial changes does the nurse focus on as a priority? a. Sexuality. b. Housing and environmenN t. URSINGTB.COM c. Retirement. d. Social isolation. ANS: D
The highest priority at this time is the potential for social isolation. This woman does not know how to drive and lives in a rural community that does not have public transportation. All of these factors contribute to her social isolation. Other possible changes she may be going through right now include sexuality related to her advanced age and recent death of her spouse; however, this is not the priority at this time. She has been retired for 5 years, so this is also not an immediate need. She may eventually experience needs related to housing and environment, but the data do not support this as an issue at this time. DIF: Analyze REF: 419| 420 OBJ: Describe the concepts of aging well and quality of life. MSC: NCLEX: Health Promotion and Maintenance
TOP: Assessment
MULTIPLE RESPONSE 1. A recently widowed 80-year-old male is dehydrated and is admitted to the hospital for
intravenous fluid replacement. During the evening shift, the patient becomes acutely confused. The nurse’s best action is to assess the patient for which of the following reversible causes? (Select all that apply.) a. Electrolyte imbalance.
Canadian Fundamentals of Nursing 6th Edition Potter Test Bank b. c. d. e.
Hypoglycemia. Drug effects. Dementia. Cerebral anoxia.
ANS: A, B, C, E
Delirium, or acute confusional state, is a potentially reversible cognitive impairment that is often due to a physiological event. Physiological causes of delirium can include electrolyte imbalances, cerebral anoxia, hypoglycemia, medications, drug effects, tumours, subdural hematomas, and cerebrovascular infection, infarction, or hemorrhage. Unlike delirium, dementia is a gradual, progressive, irreversible cerebral dysfunction. DIF: Apply REF: 418 OBJ: Differentiate among delirium, dementia, and depression. MSC: NCLEX: Physiological Integrity
NURSINGTB.COM
TOP: Implementation
Canadian Fundamentals of Nursing 6th Edition Potter Test Bank
Chapter 25: The Experience of Loss, Death, and Grief Potter et al: Canadian Fundamentals of Nursing, 6th Edition MULTIPLE CHOICE 1. A nurse encounters a family whose adult child died last year. The parents are talking about the
upcoming anniversary of their child’s death. The nurse spends time with them discussing their child’s life and death. The nurse’s action best demonstrates which nursing principle? a. Pain management technique. b. Facilitating normal mourning. c. Grief evaluation. d. Palliative care. ANS: B
Anniversaries can reopen grief processes. A nurse should openly acknowledge the loss and talk about the common renewal of grief around the anniversary of the individual’s death. This facilitates normal mourning. The nurse is not attempting to alleviate a physical pain. The actions are of open communication, not evaluation. Palliative care refers to comfort measures for relief of physical symptoms. DIF: Apply REF: 447 OBJ: Identify your role in assisting patients who have experienced loss, death, and grief. TOP: Implementation MSC: NCLEX: Psychosocial Integrity 2. A patient with cancer asks the nurse when he or she will be able to access palliative care. On
the basis of the knowledge about palliative care, what will the nurse’s response be? N nts RSwho INha GTveB.C M a. It is available only for patieU a terO minal illness. b. It can help patients achieve optimal pain management. c. It is offered when patients have less than 6 to 12 more months to live. d. It is available if indicated in an advance directive. ANS: B
Palliative care can help patients achieve optimal pain management. It is for any age, any diagnosis, and at any time, not just during the last few months of life. Hospice care is an option when the patient is expected to live less than 6 to 12 more months. An advance directive can include information about palliative care but is not required for such care to be initiated. DIF: Remember REF: 447| 448 TOP: Implementation
OBJ: Discuss principles of palliative care. MSC: NCLEX: Health Promotion and Maintenance
3. A terminally ill patient is experiencing constipation secondary to pain medication. What is the
best way for the nurse to improve the patient’s constipation problem? a. Massage the patient’s abdomen. b. Contact the provider to discontinue pain medication. c. Administer enemas twice daily for 7 days. d. Use a stimulant laxative and increase fluid intake. ANS: D
Canadian Fundamentals of Nursing 6th Edition Potter Test Bank Opioid medication is known to slow gastrointestinal transit time, which places the patient at high risk for constipation. Stimulant laxatives are indicated for opioid-induced constipation. Added water to the diet allows water to be pulled into the gastrointestinal tract, which helps soften stool. Massaging the patient’s abdomen may cause further discomfort. Discontinuing pain medication is inappropriate for a terminally ill patient. Enema administration is not the first step in the treatment of opioid-induced constipation. DIF: Apply REF: 449 TOP: Implementation
OBJ: Discuss principles of palliative care. MSC: NCLEX: Health Promotion and Maintenance
4. A severely depressed patient cannot state any positive attributes to his or her life. The nurse
patiently sits with this patient and assists the patient to identify several activities the patient would actually look forward to in life. The nurse is helping the patient to demonstrate which spiritual concept? a. Time management. b. Hope. c. Charity. d. Faith. ANS: B
The concept of hope is vital to human beings; it enables a person to anticipate positive experiences. Being patient and friendly and creating positive relationships are key concepts in all areas of nursing, but especially with depressed patients. The nurse’s actions do not address time management, charity, or faith. DIF: Understand REF: 437 OBJ: Develop a nursing care plan for a patient or family experiencing loss and grief. TOP: Implementation
NURSINGMTSBC. : C NC OLMEX: Psychosocial Integrity
5. The nurse is discussing future treatments with a patient who has a terminal illness. The nurse
notes that the patient has not been eating and responds to the nurse’s information by stating, “What does it matter?” What is the most appropriate nursing diagnosis for this patient? a. Social isolation. b. Hopelessness. c. Denial. d. Powerlessness. ANS: D
A defining characteristic for the nursing diagnosis of Powerlessness may include a statement by the patient such as “What does it matter?” when offered choices or information concerning him or her. The patient’s behaviour and verbalization is not an example of social isolation. The patient is not avoiding others or being restricted from seeing others. Hopelessness is more reflective of the comment, “I have no future” than of “What does it matter?” The patient’s behaviour and verbalization does not indicate denial. DIF: Apply REF: 442 (Box 25-7) OBJ: Develop a nursing care plan for a patient or family experiencing loss and grief. TOP: Diagnosis MSC: NCLEX: Psychosocial Integrity
Canadian Fundamentals of Nursing 6th Edition Potter Test Bank 6. Family members gather in the emergency department after learning that a family member was
involved in a motor vehicle accident. After learning of the family member’s unexpected death, the surviving family members begin to cry and scream in despair. The nurse recognizes this as which stage of Bowlby’s attachment theory? a. Numbing. b. Disorganization and despair. c. Bargaining. d. Yearning and searching. ANS: D
Yearning and searching characterize the second bereavement phase in Bowlby’s attachment theory. Emotional outbursts are common in this phase. During the numbing phase, the family may feel a sense of unreality. During disorganization and despair, the reason why the loss occurred is constantly questioned. Bargaining is part of the Kübler-Ross stages, not of the Bowlby attachment theory. DIF: Apply REF: 434 OBJ: Describe and compare the phases of grieving from Kübler-Ross (1969), Bowlby (1980), and Worden (1991). TOP: Implementation MSC: NCLEX: Psychosocial Integrity 7. After the anticipated demise of a chronically ill patient, the unit nurse is found crying in the
staff lounge. What would be the best response to her crying? a. “It is normal to feel this way. Give yourself some time to mourn.” b. “Your other patients still need you, so hurry back to them.” c. “You’re being a bad role model to the unit’s nursing students.” d. “Why don’t you take a sedative to cope?” ANS: A
NURSINGTB.COM
Nurses often witness suffering on a daily basis. Nurses, as humans, also experience grief and loss when they have been intensely involved in the patient’s suffering and death. Offer comfort and understanding to colleagues, and maintain a stable patient care environment. It is inappropriate to create guilt by telling a grieving nurse to hurry back to her patients or by indicating that she is a bad role model. Suggesting that a colleague take sedative during a shift is dangerous for the safety of patients in her care. DIF: Apply REF: 453 OBJ: Discuss your own experience of loss when caring for dying patients. TOP: Implementation MSC: NCLEX: Psychosocial Integrity 8. A family is grieving after learning of a family member’s accidental death. The transplant
coordinator requests to talk with the family about possible organ and tissue donation. The nurse recognizes which of the following? a. All religions allow for organ donation. b. Life support must be removed before organ and tissue retrieval occurs. c. The best time for organ and tissue donation is immediately after the autopsy. d. The transplant coordinator will need a private place to talk with the family. ANS: D
Canadian Fundamentals of Nursing 6th Edition Potter Test Bank The transplant coordinator should always meet with family members in a private, quiet area. Not all religions allow for organ donation. A patient may be on life support during organ removal to preserve organ tissues. Autopsy compromises organ integrity; removal should occur before that. DIF: Apply REF: 452 OBJ: Describe the procedure for care of the body after death. MSC: NCLEX: Health Promotion and Maintenance
TOP: Implementation
9. The newly graduated nurse is assigned to his or her first dying patient. How can the nurse best
prepare to care for this patient? a. Completing a course dealing with death and dying. b. Controlling his or her own emotions about death. c. Drawing on the experience of the death of a loved one. d. Developing an understanding of his or her own feelings about death. ANS: D
When caring for patients experiencing grief, it is important for the nurse to assess his or her own emotional well-being and to understand his or her own feelings about death. The nurse who is aware of his or her own feelings will be less likely to place personal situations and values before those of the patient. Although coursework on death and dying may add to the nurse’s knowledge base, it does not best prepare the nurse for caring for a dying patient. The death of a patient can raise many emotions. Being able to control one’s own emotions is important; however, it is unlikely that the nurse would be able to do so if he or she has not first acknowledged his or her own feelings about death. The death of a loved one is not a prerequisite to caring for a dying patient. The experience of caring for a dying patient may help a nurse mature in dealing with loss, or it may bring up many negative emotions if N R I G B.C M O complicated grief is present. U S N T DIF: Apply REF: 453 OBJ: Discuss your own experience of loss when caring for dying patients. TOP: Implementation MSC: NCLEX: Psychosocial Integrity 10. The palliative team’s primary obligation to a patient in severe pain includes which of the
following? a. Supporting the patient’s nurse in her grief. b. Providing post-mortem care for the patient. c. Teaching the patient the stages of grief. d. Enhancing the patient’s quality of life. ANS: D
The primary goal of palliative care is to help patients and families achieve the best quality of life. Providing support for the patient’s nurse is not the primary obligation when the patient is experiencing severe pain. Not all collaborative team members would be able to provide post-mortem care; nor would nutritionists, social workers, or pharmacists. Teaching about stages of grief should not be the focus when a patient is in severe pain. DIF: Understand REF: 447| 448 TOP: Implementation
OBJ: Discuss principles of palliative care. MSC: NCLEX: Health Promotion and Maintenance
Canadian Fundamentals of Nursing 6th Edition Potter Test Bank 11. A man is hospitalized after surgery in which both lower extremities were amputated because
of injuries sustained during military service. The nurse should recognize his need to grieve for what type of loss? a. Maturational loss. b. Situational loss. c. Perceived loss. d. Uncomplicated loss. ANS: B
Loss of a body part from injury is a situational loss. Maturational losses occur as part of normal life transitions. Perceived loss is not obvious to other people. Uncomplicated loss is not a type of loss; it is a description of normal grief. DIF: Understand REF: 433 TOP: Implementation
OBJ: List and discuss the five categories of loss. MSC: NCLEX: Psychosocial Integrity
12. “I know it seems strange, but I feel guilty being pregnant after the death of my son last year,”
said a woman during her routine obstetrical examination. The nurse spends extra time with this woman, helping her to better bond with her unborn child. This demonstrates which nursing technique? a. Facilitating mourning. b. Providing curative therapy. c. Promoting spirituality. d. Eradicating grief. ANS: A
The nurse facilitates mourning in family members who are still surviving. By acknowledging the pregnant woman’s emotioNnU s,RthSeInN urG seThB el. psCtO heMmother bond with her fetus and recognize the emotions that still exist for the deceased child. The nurse is not attempting to help the patient eradicate grief, which would be unrealistic. Curative therapy and spiritual promotion are not addressed by the nurse’s statement. DIF: Understand REF: 434 TOP: Implementation
OBJ: List and discuss the five categories of loss. MSC: NCLEX: Psychosocial Integrity
13. Three of the nurse’s patients have died during the past 2 days. Which approach is most
appropriate to manage the nurse’s sadness? a. Telling the next patients why the nurse is sad. b. Talking with a colleague or writing in a journal. c. Exercising vigorously rather than sleeping. d. Avoiding friends until the nurse feels better. ANS: B
Self-care strategies for nurses include talking with a close colleague and reflecting on feelings by writing in a journal. It is inappropriate for a nurse to talk with patients to resolve the nurse’s grief. Although exercise is important for self-care, sleep is also important. Shutting oneself away from friends is not self-care; the nurse should spend time with people who are nurturing. DIF: Understand REF: 453 OBJ: Discuss your own experience of loss when caring for dying patients. TOP: Implementation MSC: NCLEX: Psychosocial Integrity
Canadian Fundamentals of Nursing 6th Edition Potter Test Bank
14. A woman is called into her supervisor’s office regarding her deteriorating work performance
since the loss of her husband 2 years ago. The woman begins sobbing and saying that she is “falling apart” at home as well. The woman is escorted to the nurse’s office, where the nurse recognizes the woman’s symptoms as which of the following? a. Normal grief. b. Complicated grief. c. Disenfranchised grief. d. Perceived grief. ANS: B
An individual undergoing a complicated grieving process that interferes with common routines of life for excessively long periods of time is experiencing complicated (dysfunctional) grief. Normal grief is the most common reaction to death; it involves a complex range of normal coping strategies. Disenfranchised grief involves a relationship that is not socially sanctioned. Perceived grief is not a type of grief; perceived loss is a loss that is not obvious to other people. DIF: Understand TOP: Assessment
REF: 434| 435 OBJ: Describe the types of grief. MSC: NCLEX: Psychosocial Integrity
15. The father has recently begun to attend his children’s school functions since the death of his
wife. This would best be described as which task in the Worden’s grief tasks model? a. Task I. b. Task II. c. Task III. d. Task IV. ANS: C
NURSINGTB.COM
The Worden grief tasks model consists of four tasks. In Task III, the surviving family member begins to adjust to life without the deceased. Task I is accepting the reality of the loss; Task II is working through the pain of grief; and Task IV is emotionally relocating the deceased and moving on with life. DIF: Understand REF: 434 OBJ: Describe and compare the phases of grieving from Kübler-Ross (1969), Bowlby (1980), and Worden (1991). TOP: Assessment MSC: NCLEX: Psychosocial Integrity 16. An identified outcome for the family of the patient with a terminal illness is that family
members will be able to provide psychological support to the dying patient. To help the family achieve this outcome, the nurse will develop a teaching plan that will include teaching about which of the following? a. Bathing techniques. b. Application of oxygen devices. c. Recognition of the patient’s needs and fears. d. Hospice care. ANS: C
Canadian Fundamentals of Nursing 6th Edition Potter Test Bank A dying patient’s family is better prepared to provide psychological support if the nurse discusses with them ways to listen and respond to needs and fears of the dying person. Demonstration of bathing techniques and application of oxygen devices may help the family meet the patient’s physical needs but would not provide psychological support for the patient. Information on when to contact the hospice nurse is important for the family to have and may help them feel they are being supported in caring for the dying patient. However, contact information does not help the family provide psychological support to the dying patient. DIF: Apply REF: 450 (Box 25-9) OBJ: Describe how to involve family members in palliative care. TOP: Planning MSC: NCLEX: Psychosocial Integrity 17. Validation of a dying person’s life would be demonstrated by which nursing action? a. Taking pictures of visitors. b. Calling the organ donation coordinator. c. Listening to family stories about the person. d. Providing quiet visiting time. ANS: C
Listening to family members’ stories validates the importance of the dying patient’s life and reinforces the patient’s dignity. Taking pictures of visitors does not address the value of a person’s life. Calling an organ donation coordinator and providing private visiting time are components of the dying process, but they do not validate a dying person’s life. DIF: Apply REF: 450 OBJ: Develop a nursing care plan for a patient or family experiencing loss and grief. TOP: Implementation MSC: NCLEX: Psychosocial Integrity
NURSINGTB.COM
18. A couple is informed that their fetus’s condition is incompatible with life after birth. Nurses
can best help the couple with their end-of-life decision making by offering them which of the following? a. An advance directive to complete. b. Brief discussion and funeral guidance. c. Time and careful explanations. d. Instructions on how to proceed. ANS: C
Families can have limited knowledge when asked to make important ethical decisions. Nurses have the time, patience, and knowledge base to help the family understand their ethical situation and to help them make their own educated decision, rather than giving them instructions. Advance directives are completed by the person who is dying. Funeral guidance is best provided by a chaplain or a caretaker. DIF: Apply REF: 439-441 OBJ: Develop a nursing care plan for a patient or family experiencing loss and grief. TOP: Implementation MSC: NCLEX: Psychosocial Integrity 19. A correctional facility nurse is called to the scene of death of an inmate under suspicious
circumstances. The correction officer wants to move the body to the funeral home quickly because he is not comfortable with death. The inmate’s body will need to be transported where?
Canadian Fundamentals of Nursing 6th Edition Potter Test Bank a. b. c. d.
Coroner’s office for an autopsy. Police department for an investigation. Directly to the inmate’s family. Warden for inspection.
ANS: A
A physician will often request that an autopsy be performed if death occurred under unusual circumstances; as the result of foul play, homicide, or suicide; or as an accidental death, as occurs in car accidents. The nurse must understand the policies that are applied in cases of foul play death and must ensure that the decedent’s body is properly cared for after death, despite the emotional feelings of individuals in close contact with the decedent. DIF: Apply REF: 451 (Box 25-10) OBJ: Describe the procedure for care of the body after death. MSC: NCLEX: Safe and Effective Care Environment
TOP: Implementation
20. When teaching a group of older persons about cultural beliefs and death, the nurse indicates
which of the following? a. The ethical decisions surrounding a patient’s death should be based on hospital policy and not culture. b. Maintaining rituals and practices allows a sense of acceptance of the dying process. c. The nurse must decide which cultural practices will be incorporated in care of the dying. d. Regardless of culture, following hospital practices will help focus patient and family on the dying process. ANS: B
NURSINGTB.COM
Maintaining the integrity of rituals and mourning practices gives families a sense of acceptance of the patient’s death and an inner peace. The nurse should be familiar with policies and procedures, but ethical decisions should be made with an understanding and appreciation of the patient’s culture. The nurse must assess the terminally ill patient’s and family’s wishes for end-of-life care and develop a plan of care by integrating patient culture and spiritual beliefs. On the contrary, the nurse must assess the terminally ill patient’s and family’s wishes for end-of-life care and develop a plan of care that integrates patient culture and spiritual beliefs with nursing actions. DIF: Apply REF: 436 (Box 25-2) OBJ: Identify your role in assisting patients who have experienced loss, death, and grief. TOP: Implementation MSC: NCLEX: Psychosocial Integrity 21. The nurse is preparing to assist the patient in the end stage of her life. The patient is showing
signs of fatigue. What should the nurse do to provide comfort for this patient? a. Spend more time with the patient. b. Limit the use of analgesics. c. Provide larger meals with more seasoning. d. Determine valued activities, and schedule rest periods. ANS: D
Canadian Fundamentals of Nursing 6th Edition Potter Test Bank To promote comfort in a terminally ill patient, the nurse should help the patient identify values or desired tasks and then help the patient to conserve energy for those tasks. Spending more time with the patient conveys caring and allows verbalization, but it is not the best way to promote comfort for a fatigued patient. The use of analgesics should not be limited; controlling the terminally ill patient’s level of pain is a primary concern in promoting comfort. Nausea, vomiting, and anorexia may increase the terminally ill patient’s likelihood of inadequate nutrition. The nurse should serve smaller portions and bland foods, which may be more palatable. DIF: Apply REF: 447| 448 OBJ: Explain reasons for the need for improved end-of-life care for patients. TOP: Implementation MSC: NCLEX: Psychosocial Integrity 22. During a follow-up visit, a woman is describing new onset of marital discord with her
terminally ill spouse. Using the Kübler-Ross behavioural theory, the nurse recognizes that the spouse is in which stage of grief? a. Denial. b. Bargaining. c. Anger. d. Depression. ANS: C
Kübler-Ross’ traditional theory involves five stages of grief. The anger stage of adjustment to an impending death can involve resistance, anger at a deity, anger at people, and anger at the situation. Denial involves failure to accept a death. Bargaining is an action to delay acceptance of the inevitability of death. Depression would manifest as withdrawal from others. DIF: Apply REF: 43N 4URSINGTB.COM OBJ: Describe and compare the phases of grieving from Kübler-Ross (1969), Bowlby (1980), and Worden (1991). TOP: Implementation MSC: NCLEX: Psychosocial Integrity 23. Mrs. Harrison’s father died a week ago. Mr. Harrison is experiencing headaches and fatigue,
and keeps shouting at his wife to turn down the television, although he has not done so in the past. Mrs. Harrison is having trouble sleeping and has no appetite. How should the nurse interpret these assessment findings as the basis for a follow-up assessment? a. Mrs. Harrison is grieving and Mr. Harrison is angry. b. Mrs. Harrison is ill and Mr. Harrison is grieving. c. Both Mr. and Mrs. Harrison likely are in denial. d. Both Mr. and Mrs. Harrison likely are grieving. ANS: D
Each individual responds to loss differently. Some people may demonstrate anger or denial as a response to grief; Mr. Harrison’s anger is most likely to be a manifestation of grief. Common physical symptoms of grief include tightness in the chest and throat, shortness of breath, feelings of weakness and lethargy, insomnia, and loss of appetite. Denial is assessed when the person indicates that he is not accepting that the loss happened. DIF: Apply REF: 433-435 OBJ: Describe the characteristics of a person experiencing grief. TOP: Assessment MSC: NCLEX: Psychosocial Integrity
Canadian Fundamentals of Nursing 6th Edition Potter Test Bank
Chapter 26: Self-Concept Potter et al: Canadian Fundamentals of Nursing, 6th Edition MULTIPLE CHOICE 1. The nursing student can help geriatric patients’ self-concept by using which technique? a. Discussing current weather. b. Reviewing old photos with patients. c. Encouraging patients to sing. d. Allowing patients extra computer time. ANS: B
Nurses can improve older patients’ self-image by reviewing old photographs with them. This form of life review is helpful to older persons in remembering positive life events and people. Discussing weather does not involve personal reflection. Singing improves global cognition, not self-concept. Giving patients extra computer time is not applicable to improving self-concept. DIF: Apply REF: 465 (Box 26-4) OBJ: Apply the nursing process to promote a client’s self-concept. TOP: Implementation MSC: NCLEX: Psychosocial Integrity 2. While documenting an adolescent’s health history, the nurse recognizes that the patient began
to act out behaviourally and engage in risky behaviour when her parents divorced. In considering a nursing diagnosis of Altered self-concept, the nurse would gather what information? N R I G B.C M a. How long the parents wereU O maS rriedN. T b. How the patient views her behaviours. c. Why the parents are divorcing. d. Why she is acting out of control. ANS: B
A nurse can identify situational life stressors that can affect a person’s self-concept. By openly exploring a patient’s thoughts and feelings, the nurse is able to use communication skills in a therapeutic manner. This facilitates the patient’s insight into behaviours and enables the nurse to make referrals or provide needed health teaching. The length of time the parents were married and the reason for the parents’ divorce do not explain the patient’s behaviours. Why the patient is out of control is not as important as how the patient views her actions when out of control. DIF: Evaluate REF: 461| 462 OBJ: Apply the nursing process to promote a client’s self-concept. TOP: Implementation MSC: NCLEX: Psychosocial Integrity 3. The developmental self-concept task known as initiative versus guilt would occur in which
person? a. A 3-week-old neonate. b. A 5-year-old kindergarten student. c. An 11-year-old student. d. A 15-year-old high school student.
Canadian Fundamentals of Nursing 6th Edition Potter Test Bank
ANS: B
The initiative versus guilt developmental stage occurs between the ages of 3 and 6 years. If a child shows initiative, the outcome of this developmental task is to develop purpose. A neonate’s developmental task is to develop trust. An 11-year-old is in the stage of new skill mastery, and a 15-year-old is struggling with identity versus role confusion. DIF: Remember REF: 459 (Box 26-1) OBJ: Describe the components of self-concept as related to psychosocial and cognitive developmental stages. TOP: Assessment MSC: NCLEX: Psychosocial Integrity 4. A verbally abusive partner has made many negative comments to his significant other, the
patient, over the years. In the crisis centre, the nurse would anticipate that the patient may have which of the following self-concept deficits? a. Negative body image. b. Role confusion. c. Rigidity. d. Yearning. ANS: A
Consistent negative comments can cause devaluation of an individual’s self-concept. The effects of negative self-esteem can invade all areas of a person’s life, including body image. Role confusion is part of a developmental task (identity versus role confusion). Rigidity and yearning are not components of self-concept. DIF: Remember REF: 460 OBJ: Discuss factors that influN encR e theIfolG lowiB ng.cC ompMonents of self-concept: identity, body image, U S N TTOP: AsOsessment and role performance. MSC: NCLEX: Psychosocial Integrity 5. Two 50-year-old men are discussing their Saturday activities. The first man describes tutoring
children as a volunteer at a community centre. The other man says that he would never work with children and that he prefers to work out at the gym to meet young women to date. The second man’s statement reflects which developmental stage? a. Inferiority. b. Role confusion. c. Self-absorption. d. Mistrust. ANS: C
In the generativity versus self-absorption developmental task, a self-absorbed person is concerned about his or her own personal wants and desires in a self-centred manner. Mistrust versus trust is encountered in the first year of life. Industry versus inferiority is a common stage in school-aged children. Identity versus role confusion commonly occurs at the start of adolescence into young adulthood. DIF: Understand REF: 459 OBJ: Describe the components of self-concept as related to psychosocial and cognitive developmental stages. TOP: Assessment MSC: NCLEX: Psychosocial Integrity
Canadian Fundamentals of Nursing 6th Edition Potter Test Bank 6. An adult is adjusting to the idea of his chronically ill parent’s moving into the family home.
The community health nurse would assess the adult son for which potential stressor secondary to the new family living arrangement? a. Role confusion. b. Role ambiguity. c. Role performance. d. Role overload. ANS: D
Role overload involves having more roles, or more responsibilities within a role, than are manageable. Role overload is common in individuals who unsuccessfully attempt to meet the demands of work and family while trying to find some personal time. Role confusion is an aspect of the developmental task of adolescence and young adulthood (identity versus role confusion). Role ambiguity involves unclear role expectations. Role performance itself is not a stressor unless it is judged ineffective. DIF: Understand REF: 463 OBJ: Identify stressors that affect self-concept and self-esteem. TOP: Assessment MSC: NCLEX: Psychosocial Integrity 7. A nurse grimaces when seeing a patient’s colostomy opening while changing the colostomy
bag. This expression is most likely to have what effect on the patient? a. Assisting recovery by using honest communication. b. Motivating the patient to increase physical activity. c. Promoting development of a negative body image. d. Developing a kind nickname for the colostomy opening. ANS: C
NURSINGTB.COM
Negative nonverbal reactions by a nurse to a patient’s scar or surgical alterations contribute to the patient’s developing a negative body image. Honest expressions of distaste by the nurse will not facilitate recovery or ongoing communication. Encouraging physical activity or adopting a positive nickname will not promote acceptance of the colostomy opening. DIF: Understand REF: 465| 466 OBJ: Explore ways in which a nurse’s self-concept and nursing actions can affect a client’s self-concept and self-esteem. TOP: Implementation MSC: NCLEX: Psychosocial Integrity 8. “I’m such a loser. I had that job for only a month.” Identify appropriate nursing outcome
criteria for this individual. a. The patient will verbalize three life areas in which he or she functions well. b. The patient will find new employment before the next clinic visit. c. The patient will confront his or her former boss about work problems. d. The patient will identify why he or she is considered a bad employee. ANS: A
Canadian Fundamentals of Nursing 6th Edition Potter Test Bank Verbalizing three life areas in which a person functions well is an individualized measurable outcome that is realistic. Confronting a former boss could have physical and emotional repercussions for the patient. If the person is voicing that he or she has problems obtaining employment, then putting extra pressure on the person to obtain employment would be detrimental to the patient and does not reflect a supportive and caring nursing outcome. Any implication that the patient is a bad employee is inappropriate. DIF: Analyze REF: 472 (Box 26-11) OBJ: Apply the nursing process to promote a client’s self-concept. TOP: Planning MSC: NCLEX: Psychosocial Integrity 9. Children learn to live an authentic lifestyle through culturally accepted behaviours, values,
and role modelling. A child who does this is attempting to create what? a. His or her own body image. b. His or her own self-esteem. c. His or her own identity. d. His or her own role performance. ANS: C
Identity versus role confusion is a stage that occurs between the ages of 12 and 20 years. Body image and self-esteem are attitudes related to personal reflection and attitudes. Role performance is the result of creating an identity. DIF: Understand REF: 459 OBJ: Describe the components of self-concept as related to psychosocial and cognitive developmental stages. TOP: Planning MSC: NCLEX: Psychosocial Integrity
NURSINGTB.COM
10. Which of the following individuals is most likely to need the nurse’s assistance because of the
presence of identity confusion? a. A 49-year-old man with stable employment. b. A 35-year-old recently divorced mother of twins. c. A 22-year-old in the third year of college. d. A 50-year-old self-employed woman. ANS: B
Identity confusion can occur when people do not maintain a clear, consistent, and continuous consciousness of personal identity. A newly divorced woman would be trying to adapt to a new lifestyle of being single while handling parenting of twins as a single parent. This situation could lead to identity confusion. A college sophomore would have had at least 2 years to adjust to the new life setting, and a self-employed woman would probably be content with creating her own employment opportunity. There is no indication that a middle-aged man with stable employment should have identity confusion. DIF: Analyze REF: 463 OBJ: Incorporate research findings to promote evidence-informed practice for identity confusion, disturbed body image, low self-esteem, and role conflict. TOP: Planning MSC: NCLEX: Psychosocial Integrity 11. Identify the assessments suggestive of an altered self-concept. a. Limping gait and large smile.
Canadian Fundamentals of Nursing 6th Edition Potter Test Bank b. Slumped posture and poor personal hygiene. c. Verbally responding when asked a question. d. Appropriate dress with clean clothes. ANS: B
A self-concept is created by an individual’s identity, body image, and role performance. Poor personal hygiene and slumping posture best describe a person with an impaired self-concept. Smiling, appropriate responses, and appropriate appearance are all signs of normal self-concept. DIF: Understand REF: 467 ( Box 26-7) OBJ: Apply the nursing process to promote a client’s self-concept. TOP: Planning MSC: NCLEX: Psychosocial Integrity 12. A priority nursing intervention for a patient who has undergone a mastectomy is which of the
following? a. Using therapeutic silence to encourage the patient to talk. b. Using communication skills to clarify family and patient expectations. c. Telling her that you know she will do fine because many other women have. d. Rotating nursing personnel in the patient’s care, so the patient can talk to many people. ANS: B
The nurse recognizes the need to use therapeutic communication skills, allowing the patient the opportunity to talk openly about issues that are important to a person who has undergone body-altering surgery. When a nurse does not allow for the development of a patient-nurse therapeutic relationship, open and honest conversation is impossible. Silence can be useful, but building rapport and openNing onOaM re necessary first. Reassurance that a URthe SIcon NGver TBsa.tiC person will do fine dismisses any potential concerns the patient may have. Rotating nursing personnel does not allow time for the patient to build rapport with any one nurse. DIF: Understand REF: 469 OBJ: Apply the nursing process to promote a client’s self-concept. TOP: Planning MSC: NCLEX: Psychosocial Integrity 13. The nurse in an addictions clinic is working with a patient on priority setting before the
patient’s discharge from residential treatment. Which of the following would be an appropriate priority for a patient at this clinic? a. Identifying local self-help groups before discharge from the program. b. Staying away from all triggers that cause substance abuse. c. Stating a plan to never be tempted by illicit substances after discharge. d. Identifying personal areas of weakness to improve. ANS: A
Canadian Fundamentals of Nursing 6th Edition Potter Test Bank Providing the patient with resources such as local self-help groups can help turn limitations into strengths. Trying to avoid all triggers that can result in addictive behaviours is not realistic. It is also unrealistic to believe that the patient will never be tempted because temptation can arise from multiple sources. On the other hand, an appropriate priority would be to recognize that triggers will arise and that the patient should learn how to handle being confronted in the postdischarge setting. Having a person talk about his or her weaknesses without recognizing a person’s strengths could be a trigger to return to an addictive lifestyle, so this would not be the most appropriate priority. DIF: Apply REF: 467 OBJ: Apply the nursing process to promote a client’s self-concept. TOP: Planning MSC: NCLEX: Psychosocial Integrity 14. The nurse can best assess the patient’s self-concept by evaluating which of the following? a. The patient’s drug abuse history. b. The patient’s nonverbal behaviour. c. The patient’s personal journal. d. The patient’s social networking site. ANS: B
Nonverbal behaviours are key indicators of a patient’s self-concept. A history of drug abuse does not necessarily indicate current self-concept, and people who do not have a drug abuse history may have a low self-concept. It would be an invasion of privacy and trust for a nurse to read a patient’s personal journal or social networking site. DIF: Apply REF: 466| 467 OBJ: Apply the nursing process to promote a client’s self-concept. TOP: Assessment MSC: NN CLUER XS : PIsyNcG hoT soBc. ialCInOteM grity 15. A newly hired nurse is struggling with night shift work and caring for multiple family
members at home. The nurse manager calls the new nurse in to talk about how the nurse is negatively affecting patient self-concept by ignoring patients’ concerns. What should the nurse manager focus on? a. The new nurse’s role overload. b. The new nurse’s self-esteem. c. The new nurse’s ego integrity. d. The new nurse’s ethics and morals. ANS: A
Role overload involves having more roles or responsibilities than are manageable. Self-esteem is the overall feeling of self-worth. The nurse is not involved in an ethical or moral dilemma. Ego integrity is part of a developmental task associated with older persons. DIF: Apply REF: 463 OBJ: Explore ways in which a nurse’s self-concept and nursing actions can affect a client’s self-concept and self-esteem. TOP: Implementation MSC: NCLEX: Psychosocial Integrity 16. A 9-year-old is proudly telling everyone about mastering the yellow belt in her martial arts
class. Identify the appropriate developmental task. a. Initiative versus guilt.
Canadian Fundamentals of Nursing 6th Edition Potter Test Bank b. Industry versus inferiority. c. Identity versus role confusion. d. Autonomy versus shame and doubt. ANS: B
Industry versus inferiority is a stage that occurs between the ages of 6 and 12 years. It is during this developmental task that a person gains self-esteem through new skill mastery. The other self-concept development tasks occur at other stages of life and would not apply to this situation. DIF: Apply REF: 459 (Box 26-1) OBJ: Describe the components of self-concept as related to psychosocial and cognitive developmental stages. TOP: Planning MSC: NCLEX: Psychosocial Integrity 17. The nurse can assist the patient in becoming more self-aware by using which technique? a. Setting up an appointment to allow the patient to vent. b. Allowing the patient to openly explore thoughts and feelings. c. Assisting the patient to physically punch a pillow when upset. d. Providing materials for the patient to write complaint letters. ANS: B
Increasing the client's self-awareness is achieved through establishing a trusting nurse-client relationship that allows the client to openly explore thoughts and feelings. A priority nursing intervention is the expert use of communication skills to clarify client and family expectations. Open exploration can make the situation less threatening and encourages behaviours that expand self-awareness. Punching pillows does not create self-awareness. Writing complaint letters andNven g ca crB ea.teCmo Rtin I Gn T Mre internal turmoil in a person and do not U S N O promote self-awareness. DIF: Apply REF: 465 OBJ: Apply the nursing process to promote a client’s self-concept. TOP: Planning MSC: NCLEX: Psychosocial Integrity MULTIPLE RESPONSE 1. Which of the following behaviours are suggestive of altered self-concept? (Select all that
apply.) a. Avoidance of eye contact. b. Hesitant speech. c. Being independent. d. Easily making decisions. e. Unkempt appearance. f. Interest in what is happening. ANS: A, B, E
Behaviours that are suggestive of altered self-concept include avoidance of eye contact, hesitant speech, unkempt appearance, being excessively dependent, demonstrating a difficulty in making decisions, and a lack of interest in what is happening. DIF: Understand
REF: 467
Canadian Fundamentals of Nursing 6th Edition Potter Test Bank OBJ: Apply the nursing process to promote a client’s self-concept. TOP: Assessment MSC: NCLEX: Psychosocial Integrity 2. Which of the following factors have been shown to be positively associated with self-esteem
in adolescents? (Select all that apply.) a. Family income above poverty level. b. Greater number of siblings. c. Religious community. d. Emotional support from adult role models. e. Increased social media use. f. Parental support. ANS: A, B, C, D, F
The factors that have been shown to be positively associated with self-esteem in adolescents includes family income above poverty level, having a greater number of siblings, engagement with a religious community, social and emotional support from adult role models, and parental support. Increased social media use has not been positively associated with self-esteem in adolescents. DIF: Apply REF: 461 OBJ: Incorporate research findings to promote evidence-informed practice for identity confusion, disturbed body image, low self-esteem, and role conflict. TOP: Planning MSC: NCLEX: Psychosocial Integrity
NURSINGTB.COM
Canadian Fundamentals of Nursing 6th Edition Potter Test Bank
Chapter 27: Sexuality Potter et al: Canadian Fundamentals of Nursing, 6th Edition MULTIPLE CHOICE 1. A nurse is caring for a 15-year-old who in the past 6 months has had multiple male and female
sexual partners. The nurse knows that which therapeutic statement would be most effective? a. “I know you feel invincible, but STIs [sexually transmitted infections] and unwanted pregnancy are a real risk. Let’s discuss what you think is the best method for protecting yourself.” b. “Having sexual interaction with both males and females places you at higher risk for STIs. To protect yourself, you need to decide which orientation you are.” c. “Your current friends are leading you to make poor choices. You should find new friends to hang out with.” d. “I think it’s best to notify your parents. They know what’s best for you and can help make sure you practise safe sex.” ANS: A
Most teenagers feel invincible, and for this reason, some participate in risky behaviours. The nurse should acknowledge this feeling to the patient and offer education and alternatives, while giving the patient the autonomy to make his or her own decisions. The nurse should not force the patient to make a choice of orientation and should not pass judgement on a patient’s sexual orientation or social network; this would make the patient feel defensive and would eliminate the trust in the relationship. Involving parents is not the first line of action; parents should be notified only if the child is in a life-or-death situation.
N RSINGTB.COM
DIF: Apply REF: 478U TOP: Implementation
OBJ: Identify high-risk and safer sex behaviours. MSC: NCLEX: Health Promotion and Maintenance
2. A nurse is caring for a patient who expresses a desire to have an elective abortion. The nurse’s
religious and ethical values are strongly opposed. How should the nurse best handle the situation? a. Continue to care for the patient, and limit conversation as much as possible. b. Refer the patient to a family planning centre or health care professional. c. Attempt to educate the patient about the consequences of abortion. d. Inform the patient that because of moral issues, another nurse will have to care for her. ANS: B
The nurse must be aware of personal beliefs and values and is not required to participate in counselling or procedures that compromise those values. However, the patient is entitled to nonjudgemental care and should be referred to someone who can create a trusting environment. The nurse should not care for a patient if the quality of care could be jeopardized. The nurse should not attempt to project personal values onto a patient. The nurse also should not create tension by informing the patient that she does not have the same morals; this could cause the patient to feel guilty or defensive when receiving care from any health care professional. DIF: Apply REF: 483 OBJ: Discuss the nurse's role in maintaining or enhancing a patient's sexual health.
Canadian Fundamentals of Nursing 6th Edition Potter Test Bank TOP: Implementation
MSC: NCLEX: Safe and Effective Care Environment
3. Which patient is most in need of a nurse’s referral to adoption services? a. A patient considering abortion for an unwanted pregnancy. b. An infertile couple religiously opposed to artificial insemination. c. A woman who suffered miscarriage during her first pregnancy. d. A couple who has been attempting conception for 3 months. ANS: B
Adoption is ideal for someone with infertility, especially if infertility treatments are unavailable owing to religious or financial constraints. A patient who wishes to have an elective abortion may be educated about all the possibilities, but the nurse should approach the patient in a nonjudgemental manner and should accept the patient’s decision. When a patient has recently miscarried, the nurse should assess the patient’s feelings about the loss and should address any concerns the patient may have about fertility. Infertility is the inability to conceive after 1 year of unprotected intercourse; therefore, talking about adoption after one miscarriage or after only 3 months of attempting conception would be too soon. DIF: Analyze REF: 483 OBJ: Identify potential referral resources for patients' sexual concerns outside the nurse's level of expertise. TOP: Implementation MSC: NCLEX: Health Promotion and Maintenance 4. When responding to an adolescent girl’s inquiry related to emergency contraception, the nurse
tells her that the most effective method is which of the following? a. Levonorgestrel for emergency contraception (LNG-EC). b. Ulipristal acetate for emergency contraception (UPA-EC). c. Hormonal intrauterine deN viU ceR. SINGTB.COM d. Copper intrauterine device. ANS: D
The most effective method of emergency contraception is the use of a copper intrauterine device, which can be inserted and is effective within 7 days after unprotected intercourse. Hormonal intrauterine devices are not used as emergency contraception. Both LNG-EC and UPA-EC are used as emergency contraception but are not as effective as the copper intrauterine device. DIF: Apply REF: 482 OBJ: Use critical thinking skills to assist patients in meeting their sexual needs. TOP: Implementation MSC: NCLEX: Health Promotion and Maintenance 5. An 18-year-old male patient informs the nurse that he isn’t sure if he is homosexual because
he is attracted to both genders. With which statement can the nurse establish a trusting relationship with the patient? a. “Don’t worry. It’s just a phase you will grow out of.” b. “Those are abnormal impulses. You should seek therapy.” c. “At your age, it is normal to be curious about both genders.” d. “Having questions about sexuality is normal. Tell me about your sexual experience.” ANS: C
Canadian Fundamentals of Nursing 6th Edition Potter Test Bank Young adults have questions about sexuality. The patient will feel most comfortable discussing his sexual concerns further if the nurse establishes that it is normal to ask questions about sexuality. The nurse can then discuss in greater detail. Although it is normal for young adults to be curious about sexuality, the nurse should use caution in giving advice on taking sexual action. The nurse should promote safe sex practices. Telling the patient not to worry dismisses his concern. Telling the patient that he is abnormal might offend the patient and prevent him from establishing an open relationship. Asking the patient about his sexual experience is also inappropriate in this situation and is not likely to assist in building a trusting relationship with the patient. DIF: Understand REF: 478 OBJ: Describe key concepts of sexual development across the lifespan. TOP: Education | Teaching MSC: NCLEX: Safe and Effective Care Environment 6. A nurse is caring for a 35-year-old female patient who recently started taking antidepressants
after repeated attempts at fertility treatment. The patient tells the nurse, “I feel happier, but my sex drive is gone.” Which nursing diagnosis has the highest priority? a. Sexual dysfunction. b. Ineffective coping. c. Deficient knowledge about contraception. d. Risk for self-directed violence. ANS: A
Antidepressants have adverse effects on sexual desire and response. The nurse should be sure to educate the patient on the potential for these side effects and how to correct for them (e.g., using lubricant to ease discomfort). The patient has taken steps toward effective coping by seeking therapy. The patient has not expressed a reason for the nurse to be concerned about N ldRaSlwa IN G ssess B.C M contraceptives. The nurse shouU ys aT foO r concerns about violence in a patient’s life. Although some antidepressants have been related to self-directed violence, this patient states that she is feeling happy. DIF: Apply REF: 484 OBJ: Define appropriate nursing Diagnosis for patients with alterations in sexuality. TOP: Planning MSC: NCLEX: Psychosocial Integrity 7. An adolescent female student who is sexually active visits the office of the school nurse. The
nurse knows that teaching has been effective when the student states which of the following? a. “My boyfriend is able to withdraw before ejaculation, and that prevents me from getting pregnant.” b. “I take my temperature every morning, and when it goes down for at least two days, we have unprotected sex.” c. “We use ‘foam’ before each time we have sex, and I haven’t gotten pregnant yet.” d. “We use a condom with a water-based lubricant.” ANS: D
Canadian Fundamentals of Nursing 6th Edition Potter Test Bank As a more effective contraceptive method, the water-based lubricant reduces the risk of the condom’s breaking. The patient is verbalizing understanding. Any act of unprotected intercourse can result in pregnancy. Withdrawal is not an effective contraceptive method; any act of unprotected intercourse can result in pregnancy. The statement “I take my temperature every morning, and when it goes down for at least two days, we have unprotected sex” also does not demonstrate understanding of the basal body temperature method of contraception. Using spermicidal foam alone is not recommended. The patient should use a condom and foam to be more effective in preventing pregnancy. DIF: Apply REF: 482 (Table 27-1) OBJ: Identify and describe nursing interventions to promote sexual health. TOP: Education | Teaching MSC: NCLEX: Health Promotion and Maintenance 8. A patient who has had several sexual partners in the past month expresses a desire to use a
contraceptive. Which contraceptive method should the nurse recommend? a. Spermicide. b. Condom. c. Diaphragm. d. Oral contraceptive. ANS: B
Condoms are both a contraceptive and a barrier against STIs and human immunodeficiency virus (HIV) infection; proper use greatly reduces the risk of infection. Spermicides, diaphragms, and oral contraceptives all protect against pregnancy; however, they do not prevent bodily fluids from coming in contact with the patient during sexual intercourse. DIF: Understand REF: 481 OBJ: Use critical thinking skilN lsU toRaS ssI istNpG atT ienBts.iC nm OeMeting their sexual needs. TOP: Education | Teaching MSC: NCLEX: Health Promotion and Maintenance 9. A woman who has been in a monogamous relationship for the past 6 months presents to clinic
with herpes on her labia. The patient is distraught because she believes her partner must have cheated on her. Which response by the nurse is most effective in establishing an open rapport with a patient? a. Sharing an anecdote. b. Informing the patient that all discussions with the nurse are confidential. c. Telling the patient that she must be honest about every sexual experience she has had. d. Asking the patient what concerns or fears she has related to the visit. ANS: B
If effective open communication is to be established with the patient, the patient must know that she can trust her health care provider. By telling the patient that all discussions with the nurse are confidential, the nurse establishes trust. After establishing that trust, the nurse can further build the relationship by asking the patient open questions about her fear. Forcing the patient to confide by sharing every sexual encounter may hinder the development of a trusting relationship. DIF: Apply TOP: Assessment
REF: 483 OBJ: Assess a patient’s sexuality. MSC: NCLEX: Safe and Effective Care Environment
Canadian Fundamentals of Nursing 6th Edition Potter Test Bank 10. A patient admits that he knew for several months that he had an STI but did not report it. Why
did he behave this way? a. Many patients with STIs do not acknowledge the importance of STI testing and treatment. b. Many patients with STIs are hesitant to admit they contracted an STI. c. Many patients with STIs have lifestyles that subject them to repeated exposure. d. Many patients with STIs are uncomfortable discussing sexual health issues with a same-sex health care provider. ANS: B
Many patients feel guilt, embarrassment, and anxiety about contracting an STI; this can cause them to defer care until the problem becomes severe. Although knowledge about the severity of STIs may be insufficient, this should not deter seeking care. Regardless of lifestyle, STIs should be treated immediately to prevent further transmission. The role of the health care provider is to establish a trusting relationship. A person may be open to discussing health promotion behaviours but may still be hesitant to discuss STIs. DIF: Understand REF: 480 OBJ: Identify and describe nursing interventions to promote sexual health. TOP: Implementation MSC: NCLEX: Health Promotion and Maintenance 11. The nurse is leading a seminar about menopause and age-related changes. The nurse knows
that a patient does not fully understand the changes of aging when the patient makes which statement? a. “I will no longer ovulate after menopause.” b. “Orgasms are no longer achievable after menopause.” c. “Hormones of sexual regulation such as estrogen decrease with age.” No R I llyGpTerform B.COmay M decrease.” d. “As men age, their ability tU seS xuaN ANS: B
Orgasms are achievable at any age. Estrogen levels do decrease, and this may impede sexual desire. The nurse should ask the patient whether she has any questions on action that will assist in healthy sexual patterns. The other statements indicate that the patient does have an understanding of age-related changes. DIF: Understand REF: 478| 479 OBJ: Use critical thinking skills to assist patients in meeting their sexual needs. TOP: Evaluate MSC: NCLEX: Health Promotion and Maintenance 12. A patient who had an ostomy placed 1 month ago states that he is feeling depressed and does
not want to participate in sexual activities anymore because he is afraid that his partner is not physically attracted to him. Which nursing intervention will be most effective in helping this patient resume sexual activity? a. Reassure the patient that lots of people resume sex the same week the ostomy is placed. b. Inform the patient about a support group for people with ostomies. c. Teach the patient about intimate activities that can be done to incorporate the ostomy. d. Discuss ways to adapt to new body image so the patient will be comfortable in resuming intimacy.
Canadian Fundamentals of Nursing 6th Edition Potter Test Bank ANS: D
The nurse should address the patient’s need to be comfortable with his own body image; once this is done, the patient can resume sexual activity. Reassuring the patient that others manage to have sexual intercourse with an ostomy may help to decrease anxiety but may have the unintended effect of making the patient feel as though he is abnormal because he has not yet resumed sexual activity. Support groups may be helpful for the patient, but this is not the most effective intervention a nurse can provide. The patient should be discouraged from manipulating the ostomy during sexual activity; this can cause infection, trauma, or prolapse to the stoma. DIF: Apply REF: 485 OBJ: Use critical thinking skills to assist patients in meeting their sexual needs. TOP: Implementation MSC: NCLEX: Psychosocial Integrity 13. A mother brings her 10-year-old daughter into a clinic and inquires about getting her a human
papillomavirus (HPV) vaccine that day. What information does the nurse give the mother? a. The HPV vaccine is safe for children over the age of 9. b. The HPV vaccine is recommended only after a girl or woman becomes sexually active. c. The HPV vaccine will prevent a girl or woman from ever getting cervical cancer. d. The HPV vaccine is not currently available for 10-year-old girls. ANS: A
The HPV vaccine is effective against the four most common types of HPV that can cause cervical cancer. The recommendation is that girls and women between the ages of 9 and 26 be offered this vaccine; it is important that the vaccine be given before the initiation of sexual activity. The vaccine is most effective if administered before sexual activity or exposure.
NUR I G B.C M S N T O
DIF: Understand REF: 480 OBJ: Identify and describe nursing interventions to promote sexual health. TOP: Implementation MSC: NCLEX: Health Promotion and Maintenance 14. A nursing student is providing education to a group of older persons who are in an
independent living retirement village. Which of the following statements made by the nursing student should prompt the nursing professor to intervene? a. “Avoiding alcohol use will enhance your sexual functioning.” b. “You do not need to worry about getting a sexually transmitted infection at this point in your life.” c. “You need to tell your partner how you feel about sex and any fears you may have.” d. “Using pillows and taking pain medication if needed before having sexual intercourse often helps alleviate pain associated with intercourse and improves sexual functioning.” ANS: B
Any person who has sexual intercourse can develop an STI at any age. People with multiple sex partners are at greater risk. Therefore, the instructor needs to intervene when the student tells the older persons that they are not at risk for developing a STI. Sexual functioning is enhanced by avoiding the use of alcohol; using pillows; taking pain medications before intercourse if needed; and communicating thoughts, fears, and feelings about sex.
Canadian Fundamentals of Nursing 6th Edition Potter Test Bank DIF: Evaluate REF: 480| 481 OBJ: Identify and describe nursing interventions to promote sexual health. TOP: Implementation MSC: NCLEX: Health Promotion and Maintenance 15. While teaching adolescents about chlamydia, what does the school nurse tell the group? a. Chlamydia is a viral infection that cannot be cured. b. Chlamydia can be treated with a full course of antibiotics. c. Chlamydia can be contracted via bloodborne exchange. d. Chlamydia can be prevented with the use of a spermicide. ANS: B
Chlamydia is the most common of all bacterial STIs and is treated with an antibiotic. Chlamydia is caused by bacteria, not by a virus, and can be treated with an antibiotic. STIs such as chlamydia are transmitted from infected individuals to partners during intimate sexual contact. STIs are contracted not via bloodborne exchange but rather through body fluids. Chlamydia is not prevented with the use of a spermicide. DIF: Apply REF: 480 OBJ: Identify common sexually transmitted infections (STIs). MSC: NCLEX: Health Promotion and Maintenance
TOP: Implementation
16. A patient asks the nurse what signs and symptoms are associated with chlamydia. How should
the nurse respond? a. The first signs that chlamydia presents are frequency and burning upon urination. b. Symptoms of chlamydia usually affect only women. c. Small red blisters appear first and then multiply. d. Dementia results if chlamydia goes untreated too long. ANS: A
NURSINGTB.COM
Urinary problems and abnormal discharge are often the first signs of chlamydia. Both men and women can experience these symptoms, although it is known as the “silent STI” because symptoms may not manifest for some time after the disease is contracted. Small red blisters are associated with genital warts and herpes. Chlamydia does not cause dementia. DIF: Remember REF: 480 OBJ: Identify common sexually transmitted infections (STIs). MSC: NCLEX: Health Promotion and Maintenance
TOP: Assessment
17. What is an example of a quality in the critical thinking model for sexuality assessment? a. Displaying curiosity. b. Applying an ethic of care. c. Accepting the sexual orientation of the patient. d. Determining the patient’s sexual concerns. ANS: A
One of the three qualities in the critical thinking model for sexuality assessment is displaying curiosity. The other two qualities are displaying integrity and taking risks if necessary. Applying an ethic of care is an example of a standard in the model. Accepting a patient’s sexual orientation and determining the patient’s sexual concerns are examples of knowledge in the model. DIF: Apply
REF: 486
Canadian Fundamentals of Nursing 6th Edition Potter Test Bank OBJ: Use critical thinking skills to assist patients in meeting their sexual needs. TOP: Implementation MSC: NCLEX: Safe and Effective Care Environment 18. A patient states that she is afraid that she and her husband will not be able to maintain a
healthy sexual relationship now that they have a baby in the house. To assist these patients, what should the nurse know? a. Whether they have similar parenting beliefs. b. How long they have been married. c. That effective communication about sexuality requires a nondiscriminatory attitude. d. The level of knowledge the patients have regarding healthy sexual relationships. ANS: C
In response to identified concerns, the nurse may initiate discussion. Effective communication about sexuality requires caring, sensitivity, tact, compassion, the use of appropriate language, and a nondiscriminatory attitude. It is important not to have preconceived notions about the patients’ sexual identity or activity. Having similar parenting beliefs will have less effect on their sexual relationship than the use of effective communication; to assist the patients in adjusting to the change of becoming a family, the nurse should explore their communication patterns. How long they have been married would be less significant. The level of knowledge the patients have regarding healthy sexual relationships would not affect their sexual relationship as much as their ability to discuss their feelings with one another. DIF: Apply REF: 483| 486| 487 OBJ: Identify and describe nursing interventions to promote sexual health. TOP: Implementation MSC: NCLEX: Health Promotion and Maintenance 19. A 68-year-old female asks thN eU nuRrsSeI ifNuG siT ngBa.cC onOtrM aceptive is still necessary. What is the best
response by the nurse? a. “A barrier such as a condom is recommended if you are unsure whether your sexual partners have been tested for STIs.” b. “No, because you have gone through menopause and can no longer become pregnant.” c. “Let’s discuss this further; what is your current level of sexual activity?” d. “Yes, as long as using a barrier does not decrease sexual performance.” ANS: C
The nurse should first assess the patient’s risk for contracting an STI. If the patient has been in a monogamous relationship, and both parties are STI free, there is no need for a barrier. However, even though a patient has gone through menopause, there is still risk for an STI. The best way for a nurse to be sensitive to a patient’s needs is to find out more about the patient. Discussing sexual performance in relation to using a barrier method does not answer the patient’s question. DIF: Analyze REF: 486| 487 OBJ: Describe key concepts of sexual development across the lifespan. TOP: Implementation MSC: NCLEX: Physiological Integrity 20. When conducting an education session on the use of contraceptives, the nurse informs the
group that which method has the lowest initial rate of effectiveness? a. Use of a male condom.
Canadian Fundamentals of Nursing 6th Edition Potter Test Bank b. Tubal ligation. c. Use of a female condom. d. Use of a vaginal contraceptive ring. ANS: C
Of the answer choices, the female condom has the lowest rate of effectiveness, beginning at 79%. The initial rate of effectiveness of the male condom is 86%. The initial rate of effectiveness of a tubal ligation 99.5%. The initial rate of effectiveness of the vaginal contraceptive ring is 98%. DIF: Apply REF: 482 OBJ: Identify and describe nursing interventions to promote sexual health. TOP: Implementation MSC: NCLEX: Health Promotion and Maintenance 21. A patient expresses concern that her partner no longer finds her attractive and is considering
having three-way sex to spice up the relationship. Which response is the best option for the nurse in this situation? a. “Please help me understand how you are feeling about your relationship right now.” b. “If your partner truly loves you, he doesn’t need to have sex with another person.” c. “Nonmonogamous sex is immoral; here is the number for the local church.” d. “Have you considered the consequences of adding a third party to a sexual relationship?” ANS: A
The nurse should be nonjudgemental, and asking for clarification of the patient’s feelings will help the nurse better understand the situation. Making assumptions about the patient’s partner is not appropriate. Passing juN dge mS enI tN orGimp ngOtM he nurse’s morals on the patient is UR TBo.siC unethical. Asking the patient to consider the consequences of three-way sex is judgemental and implies that nontraditional sex is inappropriate. DIF: Apply REF: 483| 486| 487 OBJ: Identify and describe nursing interventions to promote sexual health. TOP: Implementation MSC: NCLEX: Health Promotion and Maintenance
Canadian Fundamentals of Nursing 6th Edition Potter Test Bank
Chapter 28: Spirituality in Health and Health Care Potter et al: Canadian Fundamentals of Nursing, 6th Edition MULTIPLE CHOICE 1. The word spirituality derives from the Latin word spiritus, which refers to breath or wind.
Today, spirituality can be described as which of the following? a. Transcendence beyond self, everyday living, and suffering. b. Less important than coping with the patient’s illness. c. Patient centred and having no bearing on the nurse’s belief patterns. d. Equated with formal religious practice and having a minor effect on health care. ANS: A
Today, spirituality is often defined as a connectedness with self, others, a higher power or nature, or a combination of these; a sense of meaning in life; and transcendence beyond self, everyday living, and suffering. Spirituality is an important factor that helps individuals achieve the balance needed to maintain health and well-being and to cope with illness. It positively affects and enhances health, quality of life, health promotion behaviours, and disease prevention activities. Nurses need an awareness of their own spirituality to provide appropriate and relevant spiritual care to others. The concepts of spirituality and religion are often discussed interchangeably, but spirituality is a much broader and more unifying concept than religion. The human spirit is powerful, and spirituality has different meanings for different people. DIF: Understand REF: 494 OBJ: Compare and contrast the concepts of religion and spirituality. N R : Ps IN G social B.CInOtegrity M TOP: Assessment MSC: NCLUEXS ychoT 2. The nurse is caring for a patient who claims that he does not believe in God and does not
believe in an “ultimate reality.” The nurse realizes what about this patient? a. The patient is devoid of spirituality. b. The patient is an atheist/agnostic. c. The patient finds no meaning through relationships with others. d. The patient believes that what he does is meaningless. ANS: B
Some individuals do not believe in the existence of God (atheist) or do not know whether God or another ultimate reality exists (agnostic). This does not mean that spirituality is not an important concept for the atheist or the agnostic. Atheists search for meaning in life through their work and their relationships with others. Agnostics discover meaning in what they do or how they live because they find no ultimate meaning for the way things are. They believe that people bring meaning to what they do. DIF: Remember REF: 496 OBJ: Compare and contrast the concepts of religion and spirituality. TOP: Assessment MSC: NCLEX: Psychosocial Integrity 3. The nurse is caring for a patient who is terminally ill with very little time left to live. The
patient states, “I always believed that there was life after death. Now, I’m not so sure. Do you think there is?” The nurse states, “I believe there is.” What has the nurse attempted to do?
Canadian Fundamentals of Nursing 6th Edition Potter Test Bank a. b. c. d.
Strengthen the patient’s religion. Provide hope. Support the patient’s agnostic beliefs. Support the horizontal dimension of spiritual well-being.
ANS: B
When a person has the attitude of something to look forward to, hope is present. Religion is the system of organized beliefs and worship that a person practises to outwardly express spirituality. This is not evident here. Agnostics do not know whether God or another ultimate reality exists and are uncertain whether there is life after death. The horizontal dimension of spiritual well-being describes positive relationships and connections people have with others. In this case, the patient is more concerned with the vertical dimension, which supports the transcendent relationship with God or some other higher power. DIF: Apply REF: 497| 503 OBJ: Discuss research findings that suggest a relationship between spiritual practices and patients' health status. TOP: Implementation MSC: NCLEX: Psychosocial Integrity 4. When discussing spirituality with adolescent patients, of what should the nurse be aware? a. That many adolescents have a good concept of a supreme being. b. That many adolescents question religious practices and values. c. That adolescents fully accept the higher meaning of their faith. d. That adolescents do good deeds for others as a spiritual task. ANS: B
Adolescents often reconsider their childlike concept of a spiritual power, and in the search for an identity, they may either qN ues tion es.aC ndOvMalues or find the spiritual power as the UR SIpra NGctic TB motivation to seek a clearer meaning to life. Adolescents do not necessarily have a good concept of a supreme being; this conceptualization occurs as people mature. Adolescents do not necessarily fully accept the higher meaning of their faith. Older persons, not adolescents, often turn to important relationships and doing good deeds for others as spiritual tasks. DIF: Apply REF: 496 OBJ: Discuss the role of the nurse in providing spiritual nursing care. TOP: Implementation MSC: NCLEX: Psychosocial Integrity 5. The nurse is admitting a patient to the hospital. The patient states that he is a very spiritual
person but does not practise any specific religion. What should the nurse understand about these statements? a. That they are contradictory. b. That they indicate a strong religious affiliation. c. That they indicate a lack of faith. d. That they are reasonable. ANS: D
These statements are reasonable and are not contradictory. Many people tend to use the terms spirituality and religion interchangeably. Although closely associated, these terms are not synonymous. Religious practices encompass spirituality, but spirituality does not need to include religious practice. When a person has the attitude of something to live for and look forward to, hope is present.
Canadian Fundamentals of Nursing 6th Edition Potter Test Bank DIF: Understand REF: 496 OBJ: Compare and contrast the concepts of religion and spirituality. TOP: Assessment MSC: NCLEX: Psychosocial Integrity 6. Which of the following is an intervention that will facilitate connectedness? a. Restricting visits from pets. b. Placing memorabilia in the patient’s view. c. Avoiding physical touch and massage. d. Implementing relevant rituals and traditions. ANS: B
An example of an intervention to facilitate connectedness is to place memorabilia in view of the patient. Visits from pets should be encouraged, not restricted. Physical touch and massage are interventions to facilitate connectedness and therefore should be implemented, not avoided. Implementing relevant rituals and traditions is an example of an intervention to facilitate hope, not connectedness. DIF: Understand REF: 497| 498 OBJ: Discuss the role of the nurse in providing spiritual nursing care. TOP: Planning MSC: NCLEX: Psychosocial Integrity 7. Which statement about religion and spirituality is true? a. Religion is a unifying theme in people’s lives. b. Spirituality is unique to the individual. c. Spirituality encompasses religion. d. Religion and spirituality are synonymous. ANS: B
NURSINGTB.COM
Spirituality is a complex concept that is unique to each individual. Religion is the system of organized beliefs and worship that a person practises to outwardly express spirituality. People from different religions view spirituality differently. Although closely associated, spirituality and religion are not synonymous. Religious practices encompass spirituality, but spirituality does not need to include religious practice. DIF: Remember REF: 495| 496 OBJ: Compare and contrast the concepts of religion and spirituality. TOP: Assessment MSC: NCLEX: Psychosocial Integrity 8. What is the best statement by a nurse who is assessing the spiritual needs of a 76-year-old
patient who was just admitted to the nursing unit with terminal cancer of the liver? a. “I notice you have a Bible. Is that a source of spiritual strength to you?” b. “What do you believe happens to your spirit when you die?” c. “We would allow members of your church to visit you whenever you desire.” d. “Has your terminal condition made you lose your faith or beliefs?” ANS: A
Canadian Fundamentals of Nursing 6th Edition Potter Test Bank Stating the observation of the patient having a Bible opens communication regarding the patient’s source of strength. Assessing a patient’s sources of strength and faith can direct interaction with him or her, including medical treatment plans. “What do you believe happens to your spirit when you die?” is not the best response, as it does not provide information that would assist the nurse in meeting the patient’s spiritual needs. “We would allow members of your church to visit you whenever you desire” is not the best response, as it implies both that the patient goes, or should go, to church and the assumption that church members are a source of strength for the patient. It does not provide assessment information to determine the patient’s spiritual needs. “Has your terminal condition made you lose your faith or beliefs?” has a negative connotation and does not help assess the patient’s sources of strength or the beliefs of the patient. DIF: Understand REF: 499 OBJ: Describe a process for understanding patients' spirituality. TOP: Assessment MSC: NCLEX: Psychosocial Integrity 9. The nurse’s initial action while working with a patient to assess and support the patient’s
spirituality is which of the following? a. Referring the patient to the agency chaplain. b. Helping the patient use faith to get well. c. Providing a variety of religious literature. d. Determining the patient’s perceptions and belief system. ANS: D
To assess and support the patient’s spirituality, the nurse should first determine the patient’s perceptions and belief system. Exploring the patient’s spirituality may reveal responses to health problems that necessitate nursing intervention, or it may reveal the existence of a strong N Rient INtoGcTope B.C M set of resources that enable theUpatS effO ectively. Although the agency chaplain may be a source for referral, such a referral is not the first action the nurse should take in assessing and supporting a patient’s spirituality. The nurse needs first to assess a patient’s spirituality to determine the patient’s perceptions and belief system before attempting to help the patient use faith to get well. Providing a variety of religious literature may be ineffective because it does not address the patient as an individual and does not help assess the patient’s personal spiritual needs. DIF: Apply REF: 499 OBJ: Describe a process for understanding patients' spirituality. TOP: Implementation MSC: NCLEX: Psychosocial Integrity 10. If a patient is identified as following the traditional health care beliefs of Judaism, while
planning patient care, which of the following should be incorporated in nursing care? a. Observance of the Sabbath. b. Faith healing. c. The removal of religious symbols, d. Sacraments of Holy Communion. ANS: A
Canadian Fundamentals of Nursing 6th Edition Potter Test Bank Observance of the Sabbath is important to a patient who follows the traditional health care beliefs of Judaism. This patient may refuse treatments scheduled on the Sabbath. Followers of the Islamic or Christian faith may use faith healing in response to illness. Religious symbols should not be removed from individuals who practise any religion. Sacraments of Holy Communion are practised by followers of the Christian faith. DIF: Analyze REF: 502 (Table 28-1) OBJ: Discuss the role of the nurse in providing spiritual nursing care. TOP: Planning MSC: NCLEX: Psychosocial Integrity 11. Which of the following attributes of spiritual care in nursing practice is described as a
direction to place patients’ needs first and protect vulnerability? a. Altruism. b. Interpersonalism. c. Intuitiveness. d. Integrativeness. ANS: A
Altruism is one attribute of spiritual care, meaning direction to place patients’ needs first and protect vulnerability. Interpersonalism means willing to be present and engage in matters of spirituality without fear. Intuitiveness means discernment of spiritual matters relevant to patient. Integrativeness is an ethic of care that pervades all aspects of nursing. DIF: Understand REF: 503 (Figure 28-2) OBJ: Describe the relational attributes that facilitate spiritual nursing care. TOP: Planning MSC: NCLEX: Psychosocial Integrity 12. The patient is admitted with N chU roR nS icI bN acG kT paBin.. C ThOeMnurse who is caring for this patient should
do which of the following? a. Focus on finding quick remedies for the back pain. b. Look at how pain influences the patient’s ability to function. c. Realize that the patient’s only goal is relief of the back pain. d. Help the patient realize that there is little hope of relief from chronic pain. ANS: B
Do not just look at the patient’s back pain as a problem to solve with quick remedies; rather, look at how the pain influences the patient’s ability to function and achieve goals established in life (not just pain relief). Mobilizing the patient’s hope is central to a healing relationship. DIF: Apply REF: 497| 504 OBJ: Describe a process for understanding patients' spirituality. TOP: Implementation MSC: NCLEX: Psychosocial Integrity 13. In caring for the patient’s spiritual needs, what should the nurse understand? a. Establishing presence is part of the art of nursing. b. Presence involves “doing for” the patient. c. A caring presence involves listening to the patient’s wishes only. d. The nurse must use her expertise to make decisions for the patient. ANS: A
Canadian Fundamentals of Nursing 6th Edition Potter Test Bank Establishing presence is part of the art of nursing. Presence involves “being with” a patient versus “doing for” a patient. The nurse should demonstrate a caring presence by listening to the patient’s concerns and willingly involving family in discussions about the patient’s health. The nurse should also show self-confidence when providing health instruction and support patients as they make decisions about their health. DIF: Understand REF: 496| 499 OBJ: Describe the relational attributes that facilitate spiritual nursing care. TOP: Planning MSC: NCLEX: Psychosocial Integrity 14. When asked about his religious affiliation, a patient responds, “That’s personal; why do you
want to know?” Which of the following is the most appropriate nursing response? a. “You need not answer my question if you prefer not to share that information.” b. “By knowing your religious preferences, I can best meet your spiritual needs.” c. “All information that you provide will be kept in strict confidence.” d. “I did not mean to offend you; we ask that question of all of our new admissions.” ANS: B
Information about religious and spiritual beliefs and practices should be obtained from patients when they enter health care facilities. Nursing care should take into account the biological, psychological, social, cultural, and spiritual needs of patients. Applying knowledge of spiritual concepts and using therapeutic communication skills will help the nurse recognize and understand the patient’s spiritual beliefs, so “You need not answer my question if you prefer not to share that information” is not an appropriate response. Although it is true that patient information is kept confidential, it is important to understand the patient’s belief system to treat the patient with dignity and respect. In the assessment of spiritual caregiving, it is important for the nurse to inform the patient that he or she is obtaining information to N ’sRnSeed IN G d B.C M ensure sensitivity to the patientU s anT give tO he patient the opportunity to communicate his or her spiritual needs. DIF: Apply REF: 499| 500 OBJ: Discuss the role of the nurse in providing spiritual nursing care. TOP: Implementation MSC: NCLEX: Psychosocial Integrity 15. Which of the following can nurses used to develop their own spiritual awareness while
facilitating the spiritual practices of others? a. Hosting a church group on the medical unit. b. Engaging in mindfulness. c. Taking part in other religions’ practices. d. Taking a university-level course on religion. ANS: B
Mindfulness can be an important strategy for developing spiritual practice, both for oneself and others. Nurses should learn about various religious practices to be able to support their patients’ spiritual health, but they are not required to host a church group, take a university-level course on religion, or take part in other religions’ practices. DIF: Understand REF: 497 OBJ: Discuss the role of the nurse in providing spiritual nursing care. TOP: Implementation MSC: NCLEX: Psychosocial Integrity
Canadian Fundamentals of Nursing 6th Edition Potter Test Bank 16. The patient is having a difficult time dealing with his diagnosis of acquired immune
deficiency syndrome (AIDS). He states, “It’s not fair. I’m totally isolated from my family because of this. Even my father hates me for this. He won’t even speak to me.” What does the nurse need to do? a. Assure the patient that his father will accept his situation soon. b. Use therapeutic communication to establish trust and caring. c. Point out that the patient has no control and that he has to face the consequences. d. Tell the patient, “If your father can’t get over it, forget it. You have to move on.” ANS: B
The nurse needs to use therapeutic communication to establish trust and a caring presence because providing spiritual care requires caring, compassion, and respect. The nurse should not offer false hope. The nurse should help the patient maintain feelings of control. The nurse should encourage renewing relationships if possible and establishing connections with self, significant others, and a higher being. DIF: Apply REF: 500| 501 OBJ: Describe the relational attributes that facilitate spiritual nursing care. TOP: Implementation MSC: NCLEX: Psychosocial Integrity 17. The nurse is caring for a patient who is in the final stages of his terminal disease. The patient
is very weak but refuses to use a bedpan and wants to get up to use the bedside commode. What should the nurse do? a. Explain to the patient that he is too weak and needs to use the bedpan. b. Insert a rectal tube so that the patient no longer needs to actively defecate. c. Enlist assistance from family members if possible and assist the patient to get up. d. Put the patient on a bedpan and stay with him until he is finished. ANS: C
N R I G B.C M U S N T O
Establishing presence is part of the art of nursing. Presence involves “being with” a patient versus “doing for” a patient. Demonstrate a caring presence by listening to the patient’s concerns and willingly involving family in discussions about the patient’s health. The nurse should support patients as they make decisions about their health. If at all possible, the nurse should encourage the patient to maintain as much independence as possible. Inserting a rectal tube involves “doing for” instead of “being with.” Placing the patient on the bedpan is against the patient’s wishes and is another form of “doing for.” DIF: Apply REF: 496| 499 OBJ: Describe the relational attributes that facilitate spiritual nursing care. TOP: Implementation MSC: NCLEX: Psychosocial Integrity 18. In assessing the spiritual health of her patients, what should the nurse understand? a. Spiritual beliefs change as patients grow and develop. b. Spiritual health in older persons does not lead to peace and acceptance of others. c. Older persons often express spirituality by focusing on themselves. d. Beliefs among older people are focused on one or two factors. ANS: A
Canadian Fundamentals of Nursing 6th Edition Potter Test Bank Spiritual beliefs change as patients grow and develop. Health spirituality in older persons leads to peace and acceptance of self. However, older persons often express their spirituality by turning to important relationships and giving of themselves to others. Beliefs among older people vary based on many factors, such as gender, past experience, religion, economic status, and ethnic background. DIF: Understand REF: 496 OBJ: Discuss appropriate ways to facilitate patients' spiritual practices. TOP: Assessment MSC: NCLEX: Psychosocial Integrity MULTIPLE RESPONSE 1. When a patient’s risk for spiritual crises is evaluated, which of the following are part of the
evaluation process? (Select all that apply.) a. Reviewing the patient’s self-perception regarding spiritual health. b. Reviewing the patient’s view of his or her purpose in life. c. Discussing with family and associates the patient’s connectedness. d. Asking whether the patient’s expectations are being met. e. Impressing on the patient that spiritual health is permanent once obtained. ANS: A, B, C, D
According to one critical thinking model for spiritual health evaluation, the evaluation process includes a review of the patient’s self-perception regarding spiritual health, the patient’s view of his or her purpose in life, discussion with the family and close associates about the patient’s connectedness, and determining whether the patient’s expectations are being met. Attainment of spiritual health is a lifelong goal.
N R I G B.C M
U S N T O DIF: Evaluate REF: 500-504 OBJ: Describe a process for understanding patients' spirituality. TOP: Evaluate MSC: NCLEX: Psychosocial Integrity 2. Which of the following are included in the five Cs of relational practice? (Select all that
apply.) a. Compassion. b. Curiosity. c. Commitment. d. Communication. e. Competence. f. Corresponding. ANS: A, B, C, E, F
The five Cs of relational practice are compassion, curiosity, commitment, competence, and corresponding. Communication is not included in this framework. DIF: Understand REF: 503 (Box 28-7) OBJ: Discuss the role of the nurse in providing spiritual nursing care. TOP: Planning MSC: NCLEX: Psychosocial Integrity
Canadian Fundamentals of Nursing 6th Edition Potter Test Bank
Chapter 29: Stress and Adaptation Potter et al: Canadian Fundamentals of Nursing, 6th Edition MULTIPLE CHOICE 1. In a natural disaster relief facility, the nurse observes that an elderly man has a recovery plan,
whereas a 25-year-old man is still overwhelmed by the disaster situation. These different reactions to the same situation would be explained best by which of the following? a. Restorative care. b. Strong financial resources. c. Maturational and sociocultural factors. d. Immaturity and intelligence factors. ANS: C
Maturational factors and sociocultural factors can affect people differently, depending on their life experiences. An older individual would have had more life experiences to draw from and to determine which strategies were successful, whereas a younger individual would have had fewer such experiences. Nothing in the scenario implies that either man is in restorative care, has strong financial resources, or is immature or intelligent. DIF: Apply REF: 514 OBJ: Discuss the integration of stress theory with nursing theories. TOP: Planning MSC: NCLEX: Psychosocial Integrity 2. A woman who was sexually assaulted more than a month ago presents to the emergency
department with complaints of recurrent nightmares, fear of going to sleep, repeated vivid N, an RS Iabil GT B.C elMmuch emotion. The nurse recognizes the memories of the sexual assaultU d inN ity to feO signs and symptoms of which medical problem? a. General adaptation syndrome. b. Post-traumatic stress disorder (PTSD). c. Developmental crisis. d. Alarm reaction. ANS: B
PTSD is characterized by vivid recollections of the traumatic event and emotional numbing and often is accompanied by nightmares. Symptoms of PTSD are more persistent than those of acute stress disorder, having lasted at least 1 month. General adaptation syndrome is the expected reaction to a major stressor. Developmental crisis occurs as a person moves through life stages rather than in response to a trauma. Alarm reaction involves physiological events such as increased activation of the sympathetic nervous system, which would have occurred at the time of the sexual assault. DIF: Apply REF: 512 OBJ: Differentiate acute stress disorder and post-traumatic stress disorder. TOP: Assessment MSC: NCLEX: Psychosocial Integrity 3. The nurse teaches stress reduction and relaxation training to a health education group of
patients after cardiac bypass surgery. The nurse is performing which level of intervention? a. Primary. b. Secondary.
Canadian Fundamentals of Nursing 6th Edition Potter Test Bank c. Tertiary. d. Quad level. ANS: C
Tertiary interventions assist patients in adapting to life with an illness. Tertiary prevention focuses on the person who already has the disease and is recovering or rehabilitating, and the goals are to slow down the disease process, prevent further damage or pain from the disease, and prevent the current disease from creating other health problems. Primary interventions include stress prevention, promotion of wellness, and risk factor reduction before illness occurs. Secondary interventions are implemented after symptoms appear and help the person develop resources to manage illness and stress. “Quad level” interventions do not exist. DIF: Understand REF: 513 OBJ: Discuss the integration of stress theory with nursing theories. TOP: Assessment MSC: NCLEX: Health Promotion and Maintenance 4. A nurse is teaching guided imagery to a prenatal class. Which of the following is an example
of guided imagery? a. Singing. b. Back massage. c. Sensory peaceful words. d. Listening to music. ANS: C
Guided imagery is used as a means to create a relaxed state through the person’s imagination, often with the use of sensory words. Imagination allows the person to create a soothing and peaceful environment. Singing, back massage, and listening to music are other types of stress management techniques. NURSINGTB.COM DIF: Understand REF: 520 OBJ: Describe stress management techniques beneficial for coping with stress. TOP: Assessment MSC: NCLEX: Psychosocial Integrity 5. After a natural disaster occurred, an emergency worker referred a family for crisis intervention
services. One family member refused to attend the services, stating, “No way, I’m not crazy.” The best response the nurse can give is which of the following? a. “Many times, disasters can create mental health problems, so you really should participate with your family.” b. “Crisis intervention is a short-term problem-solving type of help, and seeking this help does not mean that you have a mental illness.” c. “Don’t worry now. The psychiatrists are well trained to help.” d. “Crisis intervention will help your family communicate better.” ANS: B
Crisis intervention is a type of brief therapy that is more directive than typical psychotherapy or counselling. It focuses on problem solving and involves only the problem created by the crisis. The goal of crisis intervention is to create stability for the person involved in the crisis while promoting self-reliance. The other options do not properly reassure the patient and build trust. DIF: Apply REF: 521 TOP: Implementation
OBJ: Discuss the process of crisis intervention. MSC: NCLEX: Psychosocial Integrity
Canadian Fundamentals of Nursing 6th Edition Potter Test Bank
6. Pediatric stressors related to self-esteem and changes in family structure reflect which
maturational school age category? a. Elementary school age. b. Preadolescence. c. Adolescence. d. Early adulthood. ANS: B
Children of preadolescent age experience stress related to self-esteem issues, changing family structure due to divorce or death of a parent, or hospitalization. Adolescent stressors include identity issues with peer groups and separation from parents. Elementary school age stressors include friends, family, and school relations. Adult stressors centre on life events. DIF: Apply REF: 514 TOP: Implementation
OBJ: Describe how stress is conceptualized. MSC: NCLEX: Psychosocial Integrity
7. During the evaluation stage of the critical thinking model applied to a patient coping with
stress, what will the nurse do? a. Select nursing interventions to promote the patient’s adaptation to stress. b. Establish short- and long-term goals with the patient experiencing stress. c. Identify stress management interventions for achieving expected outcomes. d. Reassess patient’s stress-related symptoms and compare with expected outcomes. ANS: D
During the evaluation stage, the nurse compares current stress-related symptoms against established measurable outcomes to evaluate the effectiveness of the intervention. Selecting appropriate interventions andNeU stR abSliI shN inG gT gB oa. lsCaO reMpart of the planning process. DIF: Understand REF: 522 OBJ: Develop a care plan for patients experiencing stress. MSC: NCLEX: Psychosocial Integrity
TOP: Implementation
8. The nurse is involved in crisis intervention with a family in which the father has just lost his
job and is experiencing periods of depression. The mother has a chronic debilitating illness that has put added responsibilities on the adolescent child, who is having behavioural problems. How does the nurse help the family focus on their feelings? a. Pointing out the connection between the situation and their responses. b. Encouraging the use of the family’s usual coping skills. c. Working on time management skills. d. Discussing past experiences. ANS: A
When using a crisis intervention approach, the nurse helps the patient make the mental connection between the stressful event and the patient’s reaction to it. Because an individual’s or family’s usual coping strategies are ineffective in managing the stress of the precipitating event in a crisis situation, the use of new coping mechanisms is required. Time management skills will not help reduce the stress of the precipitating event in a crisis situation. Past experiences may not be helpful in managing the stress of the precipitating event in a crisis situation. DIF: Analyze
REF: 521
OBJ: Discuss the process of crisis intervention.
Canadian Fundamentals of Nursing 6th Edition Potter Test Bank TOP: Implementation
MSC: NCLEX: Psychosocial Integrity
9. A senior university student contacts the campus health clinic about a first-year student living
on the same dormitory floor. The senior student reports that the first-year student is crying and is not adjusting to university life. The clinic nurse recognizes this as a combination of situational and maturational stress factors. The best comment to the senior student would be which of the following? a. “I’d better call 911 because your friend is suicidal.” b. “Give her this list of university and community resources.” c. “You must make an appointment for the student to obtain medications.” d. “I’d recommend you help the student pack her bags to go home.” ANS: B
A health care provider can help to reduce situational stress factors for individuals. Providing the student with a list of resources is one way to begin this process, as part of secondary prevention strategies. This is not a medical or psychiatric emergency, so calling 911 is not necessary. Not everyone who has sadness needs medications; some need counselling only. Not enough information is given to indicate whether leaving the university is the best solution for the student. DIF: Apply REF: 513 OBJ: Develop a care plan for patients experiencing stress. MSC: NCLEX: Health Promotion and Maintenance
TOP: Implementation
10. When working in a highly stressful nursing unit and accepting additional shifts, a new nursing
graduate should develop a strategy to prevent burnout. What would the best strategy for the new nurse be? N R I G B.C M a. Identify limits and scope ofUwoS rk rN espT onsibiliO ties. b. Write for 10 minutes in a journal every day. c. Use progressive muscle relaxation. d. Delegate complex nursing tasks to licensed professional nurses. ANS: A
An important step in preventing burnout is acknowledging one’s own limitations, as well as what one’s scope of work is while on the job. By doing this, the nurse will help to prevent emotional exhaustion and will limit the effects of chronic stress. Journalling and muscle relaxation are good stress-relieving techniques but are not directed at the cause of the workplace stress. Delegating, if not applicable, is an inappropriate coping mechanism. DIF: Apply REF: 521 OBJ: Discuss how stress in the workplace can affect health care providers. TOP: Implementation MSC: NCLEX: Psychosocial Integrity 11. A teenager with celiac disease continues to eat food she knows will make her ill several hours
after ingestion. In view of appropriate tertiary level interventions, what would be the nursing intervention? a. Teach the patient about Canada’s Food Guide to Healthy Eating. b. Administer antidiarrheal medications with meals. c. Assist the teen in meeting dietary restrictions while eating foods similar to those eaten by her friends. d. Admonish the teen and her parents regarding her consistently poor diet choices.
Canadian Fundamentals of Nursing 6th Edition Potter Test Bank
ANS: C
Tertiary interventions help the patient adapt to life with an illness. By adjusting the diet to meet dietary guidelines and also addressing adolescent emotional needs, the nurse will help the teenager to follow an appropriate diet without health complications and see herself as a “typical and normal” teenager. Teaching about Canada’s Food Guide to Healthy Eating will not address the real issue, which is that the teen is still eating what she knows will make her ill. Administering antidiarrheal medications may help but is not a tertiary intervention. Admonishing the teenager and parents is not a tertiary intervention, and because this approach is nontherapeutic, it may cause communication problems. DIF: Apply REF: 513 OBJ: Develop a care plan for patients experiencing stress. MSC: NCLEX: Health Promotion and Maintenance
TOP: Implementation
12. A trauma survivor is requesting sleep medication because of “bad dreams.” Concerned about
PTSD, the nurse would ask which of the following? a. “Are you reliving your trauma?” b. “Are you having chest pain?” c. “Can you describe your phobias?” d. “Can you tell me when you wake up?” ANS: A
People who have PTSD often have flashbacks, re-experiencing the trauma. The other answers involve assessment of problems not specific to PTSD. DIF: Apply REF: 512 OBJ: Differentiate acute stressNdisR ordeI r anG d poB st. -trC aumM atic stress disorder. TOP: Implementation
U S N MTSC: NCOLEX: Psychosocial Integrity
13. A 23-year-old patient who recently suffered a head injury in a motor vehicle accident is
unconscious. The physiological component that is primarily responsible for the patient’s level of consciousness is a. Medulla oblongata. b. Reticular formation. c. Pituitary gland. d. External stress response. ANS: B
The reticular formation is primarily responsible for an individual’s level of consciousness. The medulla oblongata controls vital functions such as heart rate, blood pressure, and respiration. The pituitary gland supplies hormones that control vital functions. The pituitary gland produces hormones necessary for adaptation to stress. The external stress response is not primarily responsible for a person’s level of consciousness. DIF: Analyze REF: 510 OBJ: Define the key biological systems involved in stress responses. TOP: Assessment MSC: NCLEX: Physiological Integrity 14. An example of an assessment finding of caregiver strain would be which of the following? a. Caregiver routinely creates a weekly menu plan. b. Caregiver has not received medical care when ill.
Canadian Fundamentals of Nursing 6th Edition Potter Test Bank c. Caregiver can identify respite care provider. d. Caregiver attends religious service. ANS: B
A nurse will identify a caregiver’s lack of self-care as a potential example of caregiver role strain. Sacrificing their own health to care for the identified patient places caregivers at risk for becoming ill themselves. If caregivers jeopardize their own health, they may not be able to care for the actual patient. In all of the other options, the caregiver is handling caregiver stress appropriately. DIF: Apply REF: 513 (Box 29-3) OBJ: Develop a care plan for patients experiencing stress. MSC: NCLEX: Psychosocial Integrity
TOP: Implementation
15. Nurses in the medical centre are working with patients experiencing PTSD after losing family
members from severe acute respiratory syndrome. Which of the following approaches would be appropriate to incorporate into the plan of care? a. Suppression of anxiety-producing memories. b. Reinforcement that the PTSD is short term. c. Promotion of relaxation strategies. d. Focusing on physical needs. ANS: C
Teaching the patient relaxation strategies can help reduce the stress of anxiety-provoking thoughts and events, as seen in PTSD, and reinforces an adaptive coping strategy. In the presence of anxiety-provoking thoughts and events, a common physiological symptom is muscle tension. Physiological tension is diminished through a systematic approach to releasing tension in major muNscl e gr oup s. T SB up. pC ress ion would be a maladaptive coping UR SI NG OM mechanism. PTSD is not a short-term problem but a long-term one; the focus should be on developing adaptive coping mechanisms and lowering the individual’s anxiety level. The primary focus for the patient who is experiencing PTSD is not on physical needs. DIF: Apply REF: 520 OBJ: Differentiate acute stress disorder and post-traumatic stress disorder. TOP: Implementation MSC: NCLEX: Health Promotion and Maintenance 16. The nursing student gave a wellness lecture on the importance of accurate assessment and
intervention from a personal, family, and community perspective. The other nursing students enjoyed the lecture about which nursing theory? a. Ego defence model. b. Situational model. c. Evidence-informed practice model. d. Neuman’s systems model. ANS: D
Neuman’s systems model is based on an individual’s/family’s/community’s relationship to stress and the reaction to stress. This model promotes wellness on primary, secondary, and tertiary levels. The other items listed as models are not nursing theories. Ego defence mechanisms are unconscious coping mechanisms. Situational factors include stressors such as relocation or family job changes. Evidence-informed practice consists of relying on data or other reputable information sources to guide nursing care.
Canadian Fundamentals of Nursing 6th Edition Potter Test Bank
DIF: Apply REF: 513 OBJ: Discuss the integration of stress theory with nursing theories. TOP: Implementation MSC: NCLEX: Health Promotion and Maintenance 17. An adult who was in a motor vehicle accident is brought into the emergency department by
paramedics, who report the following in-transit vital signs: • Oral temperature: 37.2°C (99.0°F) • Pulse: 102 beats per minute • Respiratory rate: 26 breaths per minute • Blood pressure: 140/106 The nurse can identify that which hormones are the likely causes of the abnormal vital signs? a. Antidiuretic hormone (ADH) and adrenocorticotropic hormone (ACTH). b. ACTH and epinephrine. c. ADH and norepinephrine. d. Epinephrine and norepinephrine. ANS: D
Epinephrine and norepinephrine are catecholamine hormones secreted by the adrenal medulla that rapidly elevate heart rate and blood pressure. ACTH originates from the anterior pituitary gland and stimulates cortisol release; ADH originates from the posterior pituitary gland and increases renal reabsorption of water. ACTH, cortisol, and ADH do not increase heart rate. DIF: Apply REF: 510| 511 OBJ: Discuss the integration of stress theory with nursing theories. TOP: Implementation MSC: NCLEX: Physiological Integrity 18. The nurse is working with paN tiU enR tsSinIaNnGoT utB pa.tiC enOt M health care setting. One of the patients is
experiencing job-related stress. Which of the following is a factor that may indicate chronic stress? a. Avoiding discussion of job problems. b. Saying negative things about other colleagues. c. Experiencing chronic pain and irritable bowel syndrome. d. Blaming the boss for being unreasonably demanding. ANS: C
Chronic pain disorders, irritable bowel syndrome, somatization disorder, and clinical depression have all been linked to chronic stress. A patient’s avoiding discussion of the problem, negative comments about his or her colleagues, and blaming of the boss may not be related to chronic stress. DIF: Understand TOP: Assessment
REF: 510-512 OBJ: Describe how stress is conceptualized. MSC: NCLEX: Psychosocial Integrity
19. A 72-year-old patient is in a long-term care facility after having had a cerebrovascular
accident. The patient is noncommunicative, enteral feedings are not being absorbed, and respirations are becoming laboured. What stage of the general adaptation syndrome is this patient experiencing? a. Resistance stage. b. Exhaustion stage. c. Reflex pain stage.
Canadian Fundamentals of Nursing 6th Edition Potter Test Bank d. Alarm reaction stage. ANS: B
The exhaustion stage occurs when the body can no longer resist the effects of the stressor and when the energy necessary to maintain adaptation is depleted. During the resistance or adaptation stage, the body stabilizes. Reflex pain response is not a stage of the general adaptation syndrome. During the alarm reaction, increasing hormone levels result in increased blood volume, epinephrine and norepinephrine amounts, heart rate, blood flow to muscles, oxygen intake, and mental alertness. DIF: Understand REF: 509 OBJ: Discuss the integration of stress theory with nursing theories. TOP: Implementation MSC: NCLEX: Physiological Integrity 20. While giving a lecture on attention-deficit–hyperactivity disorder, the nurse encourages which
of the following to reduce children’s stress regarding homework assignments? a. Time management skills. b. Prevention of iron deficiency anemia. c. Routine preventive health visits. d. Speech articulation skills. ANS: A
Time management skills are most helpful in reducing homework-related stress. Anemia prevention will improve energy levels but not alleviate stress. Routine health visits are important but do not directly affect ability to complete homework. Speech and other developmental aspects need to be developed if the child is to be successful, but skill development will not directly reduce homework-related stress.
NURSINGTB.COM
DIF: Apply REF: 520 OBJ: Describe stress management techniques beneficial for coping with stress. TOP: Implementation MSC: NCLEX: Health Promotion and Maintenance
Canadian Fundamentals of Nursing 6th Edition Potter Test Bank
Chapter 30: Vital Signs Potter et al: Canadian Fundamentals of Nursing, 6th Edition MULTIPLE CHOICE 1. How does the posterior hypothalamus help control temperature? a. By causing vasoconstriction. b. By shunting blood to the skin and extremities. c. By increasing sweat production. d. By causing vasodilation. ANS: A
If the posterior hypothalamus senses that the body’s temperature is lower than the set point, the body initiates heat conservation mechanisms. Vasoconstriction of blood vessels reduces blood flow to the skin and extremities. The anterior hypothalamus controls heat loss by inducing sweating, vasodilation of blood vessels, and inhibition of heat production. DIF: Remember REF: 528 OBJ: Explain the physiology of normal regulation of blood pressure, pulse, oxygen saturation, and respirations. TOP: Assessment MSC: NCLEX: Physiological Integrity 2. Of the following mechanisms of heat loss by the body, identify the mechanism that transfers
heat away by using air movement? a. Radiation. b. Conduction. c. Convection. NURSINGTB.COM d. Evaporation. ANS: C
Convection is the transfer of heat away from the body by air movement. Conduction is the transfer of heat from one object to another with direct contact. Radiation is the transfer of heat from the surface of one object to the surface of another without direct contact between the two. Evaporation is the transfer of heat energy when a liquid is changed to a gas. DIF: Understand REF: 529 OBJ: Explain the physiology of normal regulation of blood pressure, pulse, oxygen saturation, and respirations. TOP: Assessment MSC: NCLEX: Physiological Integrity 3. The patient has a temperature of 40.7°C (105.2°F). The nurse is attempting to lower his
temperature by providing tepid sponge baths and placing cool compresses in strategic body locations. The nurse is attempting to lower the patient’s temperature through the use of which of the following? a. Radiation. b. Conduction. c. Convection. d. Evaporation. ANS: B
Canadian Fundamentals of Nursing 6th Edition Potter Test Bank Applying an ice pack or bathing a patient with a cool cloth increases conductive heat loss. Radiation is the transfer of heat from the surface of one object to the surface of another without direct contact between the two. Evaporation is the transfer of heat energy when a liquid is changed to a gas. Convection is the transfer of heat away from the body by air movement. DIF: Apply REF: 529 OBJ: Explain the physiology of normal regulation of blood pressure, pulse, oxygen saturation, and respirations. TOP: Implementation MSC: NCLEX: Physiological Integrity 4. When focusing on temperature regulation of newborns and infants, what should the nurse
know? a. Temperatures are basically the same for infants and older persons. b. Infants have well-developed temperature-regulating mechanisms. c. The normal temperature range gradually increases as the person ages. d. Newborns need to wear a cap to prevent heat loss. ANS: D
A newborn loses up to 30% of body heat through the head and therefore needs to wear a cap to prevent heat loss. Temperature control mechanisms in newborns are immature and respond drastically to changes in the environment. The normal temperature range gradually drops with age. DIF: Understand REF: 529 OBJ: Describe factors that cause variations in body temperature, pulse, oxygen saturation, respirations, and blood pressure. TOP: Assessment MSC: NCLEX: Physiological IN ntU egR riS tyINGTB.COM 5. The nurse working the night shift on a surgical unit is making rounds at 0400 hours. She
notices that the patient’s temperature is 36°C (96.8°F), whereas at 1600 hours the preceding day, it was 37°C (98.6°F). What should the nurse do? a. Call the physician immediately to report a possible infection. b. Realize that this is a normal temperature variation. c. Provide another blanket to conserve body temperature. d. Provide medication to lower the temperature further. ANS: B
Body temperature normally changes 0.5°C to 1°C (0.9°F to 1.8°F) during a 24-hour period. Unless the patient is complaining of being cold, there is no physiological need for providing an extra blanket or medication to lower the body temperature further. There is also no need to call a physician to report a normal temperature variation. DIF: Apply REF: 530 OBJ: Describe factors that cause variations in body temperature, pulse, oxygen saturation, respirations, and blood pressure. TOP: Implementation MSC: NCLEX: Physiological Integrity 6. The nurse is caring for a patient who has a temperature reading of 38°C (100.4°F). His last
two temperature readings were 37°C (98.6°F) and 36°C (96.8°F). The nurse should a. Call the physician and anticipate an order to treat the fever. b. Assume that the patient has an infection and order blood cultures.
Canadian Fundamentals of Nursing 6th Edition Potter Test Bank c. Wait an hour and recheck the patient’s temperature. d. Be aware that temperatures this high are harmful and affect patient safety. ANS: C
Waiting an hour and rechecking the patient’s temperature would be the most appropriate action in this case. A fever usually is not harmful if it stays below 39°C (102.2°F), and a single temperature reading does not always indicate a fever. In addition to physical signs and symptoms of infection, a fever determination is based on several temperature readings at different times of the day in comparison with the usual value for that person at that time. Mild temperature elevations enhance the body’s immune system by stimulating white blood cell production. Staff nurses usually do not order blood cultures, and nurses should base actions on knowledge, not on assumptions. DIF: Apply REF: 530 OBJ: Describe factors that cause variations in body temperature, pulse, oxygen saturation, respirations, and blood pressure. TOP: Implementation MSC: NCLEX: Physiological Integrity 7. When heat loss mechanisms of the body are unable to keep pace with excess heat production,
the result is known as a. Pyrexia. b. The plateau phase. c. The set point. d. Becoming afebrile. ANS: A
Pyrexia, or fever, occurs because heat loss mechanisms are unable to keep pace with excess heat production, which resultN sU inRan abn orm al.riC seOiM n body temperature. The set point is the SI NG TB temperature point determined by the hypothalamus. When pyrogens trigger immune system responses, the hypothalamus reacts to raise the set point, and the body produces and conserves heat. During the plateau phase, chills subside and the person feels warm and dry as heat production and loss equilibrate at the new level. When the fever “breaks,” the patient becomes afebrile. DIF: Remember REF: 530 OBJ: Identify ranges of acceptable vital sign values for an infant, a child, and an adult. TOP: Assessment MSC: NCLEX: Physiological Integrity 8. The nurse is caring for a patient who has an elevated temperature. What should the nurse
know? a. Fever and hyperthermia are the same thing. b. Hyperthermia occurs when the body cannot reduce heat loss. c. Hyperthermia is an upward shift in the set point. d. Hyperthermia occurs when the body cannot reduce heat production. ANS: D
Elevation in body temperature that is related to the body’s inability to promote heat loss or reduce heat production is hyperthermia. Fever and hyperthermia are not the same thing. Fever, not hyperthermia, is a shift in the set point. DIF: Understand REF: 530| 531 OBJ: Describe factors that cause variations in body temperature, pulse, oxygen saturation,
Canadian Fundamentals of Nursing 6th Edition Potter Test Bank respirations, and blood pressure. MSC: NCLEX: Physiological Integrity
TOP: Assessment
9. The patient is restless and has a temperature of 39°C (102.2°F). What is one of the first things
the nurse should do? a. Place the patient on oxygen. b. Restrict fluid intake. c. Increase patient activity. d. Increase patient’s metabolic rate. ANS: A
During a fever, cellular metabolism increases, and oxygen consumption rises. Myocardial hypoxia produces angina. Cerebral hypoxia produces confusion. Interventions during a fever include oxygen therapy. Dehydration is a serious problem through increased respiration and diaphoresis, and a dehydrated patient is at risk for fluid volume deficit. Fluids should not be restricted. Increasing activity would increase the metabolic rate further, which would not be advisable. DIF: Apply REF: 531 OBJ: Describe factors that cause variations in body temperature, pulse, oxygen saturation, respirations, and blood pressure. TOP: Implementation MSC: NCLEX: Physiological Integrity 10. The patient needs temperatures measured every two hours. Which of the following cannot be
delegated to an unregulated care provider? a. Selecting appropriate route and device. b. Obtaining temperature measurement at ordered frequency. c. Being aware of the usual N vaUluReS sI foN r tG heTpBa. tieCnO t. M d. Assessing changes in body temperature. ANS: D
The nurse is responsible for assessing changes in body temperature. The nurse instructs an unregulated care provider to select the appropriate route and device to measure temperature, to obtain temperature measurement at ordered frequency, and to be aware of the usual values for the patient. DIF: Apply REF: 532 OBJ: Appropriately delegate vital sign measurement to unregulated care providers. TOP: Implementation MSC: NCLEX: Safe and Effective Care Environment 11. The patient requires routine temperature assessment but is confused and easily agitated and
has a history of seizures. The nurse’s best option would be to take his temperature a. Orally. b. Tympanically. c. Rectally. d. By the axillary method. ANS: B
Canadian Fundamentals of Nursing 6th Edition Potter Test Bank The tympanic route is easily accessible, necessitates minimal patient repositioning, and often can be used without disturbing the patient. Its measurement time is also very rapid. Oral temperature measurements require patient cooperation and are not recommended for patients with a history of epilepsy. Rectal temperature measurements require positioning and may increase patient agitation. Axillary temperature measurements take a long time and continuous positioning by the nurse. The patient’s agitation state may not allow for long periods of attention. DIF: Apply REF: 537 (Box 30-4) OBJ: Explain variations in technique used to assess an infant’s, a child’s, and an adult’s vital signs. TOP: Implementation MSC: NCLEX: Physiological Integrity 12. The patient is being admitted to the emergency department after a motor vehicle accident. His
jaw is broken, and he has several broken teeth. He is ashen, and his skin is cool and diaphoretic. To obtain an accurate temperature, the nurse uses which of the following routes? a. Oral. b. Axillary. c. Rectal. d. Temporal. ANS: C
The rectal route is argued to be more reliable when oral temperature cannot be obtained. Oral temperature measurement is not performed in patients who have had oral surgery, trauma, history of epilepsy, or shaking chills. Axillary temperature measurement is affected by exposure to the environment, including time to place the thermometer; it also takes a long time. Temporal artery temperature is affected by skin moisture such as diaphoresis or sweating.
NURSINGTB.COM
DIF: Apply REF: 537 (Box 30-4) OBJ: Accurately assess pulse, respirations, oxygen saturation, and blood pressure. TOP: Implementation MSC: NCLEX: Physiological Integrity 13. The nurse is caring for an infant and is measuring vital signs. The best site for the nurse to
obtain the infant’s pulse would be which artery? a. Radial. b. Brachial. c. Femoral. d. Popliteal. ANS: B
The brachial or apical pulse is the best site for assessing an infant’s or a young child’s pulse because other peripheral pulses such as the radial, femoral, and popliteal arteries are deep and are difficult to palpate accurately. DIF: Apply REF: 542 OBJ: Explain variations in technique used to assess an infant’s, a child’s, and an adult’s vital signs. TOP: Implementation MSC: NCLEX: Physiological Integrity 14. The patient is found to be unresponsive and not breathing. To determine the presence of
central blood circulation and circulation of blood to the brain, the nurse checks which pulse? a. Radial.
Canadian Fundamentals of Nursing 6th Edition Potter Test Bank b. Brachial. c. Posterior tibial. d. Carotid. ANS: D
The heart continues to deliver blood through the carotid artery to the brain as long as possible. The carotid pulse is easily accessible during physiological shock or cardiac arrest. The radial pulse is used to assess peripheral circulation or to assess the status of circulation to the hand. The brachial site is used to assess the status of circulation to lower arm. The posterior tibial pulse is used to assess the status of circulation to the foot. DIF: Apply REF: 542 OBJ: Accurately assess pulse, respirations, oxygen saturation, and blood pressure. TOP: Implementation MSC: NCLEX: Physiological Integrity 15. The nurse needs to measure the radial pulse in a patient. For accuracy, what must the nurse
do? a. Place the tips of the nurse’s first two fingers over the groove along the thumb side
of the patient’s wrist. b. Place the thumb over the groove along the thumb side of the patient’s wrist. c. Apply a very light touch so that the pulse is not obliterated. d. Apply very strong pressure to detect the pulse. ANS: A
The nurse places the tips of the first two or middle three fingers over the groove along the radial or thumb side of the patient’s inner wrist. Fingertips are the most sensitive parts of the hand for palpating arterial pulsation. The thumb has a pulsation that interferes with accuracy. The pulse is more accuratelyNass esse wG ithTm Bo.dCerat UR SId N OMe pressure. Too much pressure occludes the pulse and impairs blood flow. DIF: Apply REF: 544 OBJ: Accurately assess pulse, respirations, oxygen saturation, and blood pressure. TOP: Implementation MSC: NCLEX: Physiological Integrity 16. While the nurse is assessing the patient’s respirations, it is important for the patient to do
what? a. Be aware of the procedure being done. b. Not know that respirations are being assessed. c. Understand that respirations are estimated to save time. d. Not be touched until the entire process is finished. ANS: B
Do not let a patient know that respirations are being assessed. A patient who is aware of the assessment can alter the rate and depth of breathing. Respirations are the easiest of all vital signs to assess, but they are often the most haphazardly measured. Do not estimate respirations. Accurate measurement requires observation and palpation of chest wall movement. DIF: Apply REF: 549 OBJ: Accurately assess pulse, respirations, oxygen saturation, and blood pressure. TOP: Implementation MSC: NCLEX: Physiological Integrity
Canadian Fundamentals of Nursing 6th Edition Potter Test Bank 17. The patient’s blood pressure is 140/60. How would the nurse would record the pulse pressure? a. As 140. b. As 60. c. As 80. d. As 200. ANS: C
The difference between the systolic pressure and the diastolic pressure is the pulse pressure. For a blood pressure of 140/60, the pulse pressure is 80 (140 – 60). The systolic pressure is 140. The diastolic pressure is 60. The systolic pressure (140) added to the diastolic pressure (60) is 200, but this has no clinical significance. DIF: Understand REF: 557 OBJ: Accurately record and report vital sign measurements. MSC: NCLEX: Physiological Integrity
TOP: Implementation
18. The thickness or viscosity of the blood affects the ease with which blood flows through small
vessels. Examining what value might help determine the amount of blood viscosity? a. Hematocrit. b. Cardiac output. c. Arterial size. d. Blood volume. ANS: A
The hematocrit, or the percentage of red blood cells in the blood, determines blood viscosity. Blood pressure also depends on the cardiac output or volume pumped by the heart, but cardiac output does not affect viscosity. Differences in arterial size help modify blood pressure; the smaller lumen of a vessel incN reas ular esC istan UResSvIasc NG TBr. OMce but does not affect viscosity. Blood volume also affects blood pressure, but it does not directly affect viscosity. DIF: Understand REF: 557 OBJ: Explain the physiology of normal regulation of blood pressure, pulse, oxygen saturation, and respirations. TOP: Assessment MSC: NCLEX: Physiological Integrity 19. The patient is being admitted to the emergency department with complaints of shortness of
breath. The patient has had chronic lung disease for many years but still smokes. What should the nurse do? a. Administer high levels of oxygen. b. Use oxygen cautiously in this patient. c. Place a paper bag over the patient’s face to allow rebreathing of carbon dioxide. d. Administer CO2 via mask. ANS: B
Because low levels of arterial O2 provide the stimulus that allows the patient to breathe, administration of high oxygen levels would be fatal for patients with chronic lung disease. Oxygen must be used cautiously in these types of patients. Patients with chronic lung disease have ongoing hypercarbia (elevated CO2 levels) and do not need to have CO2 administered or “rebreathed.” DIF: Apply REF: 549 OBJ: Explain the physiology of normal regulation of blood pressure, pulse, oxygen saturation, and respirations. TOP: Implementation
Canadian Fundamentals of Nursing 6th Edition Potter Test Bank MSC: NCLEX: Physiological Integrity 20. Which statement is true of the ovulation phase? a. Progesterone levels are below normal. b. Body temperature is below baseline levels. c. Body temperature is at previous baseline levels or higher. d. Intense body heat and sweating occur. ANS: C
Progesterone levels rise and fall cyclically during the menstrual cycle. When progesterone levels are low, the body temperature is a few tenths of a degree below the baseline. The lower temperature persists until ovulation occurs. During ovulation, greater amounts of progesterone enter the circulatory system and raise the body temperature to previous baseline levels or higher. These temperature variations help predict a woman’s most fertile time, in order to achieve pregnancy. Women who undergo menopause (cessation of menstruation) often experience periods of intense body heat and sweating (hot flashes) lasting from 30 seconds to 5 minutes. DIF: Remember REF: 530 OBJ: Describe factors that cause variations in body temperature, pulse, oxygen saturation, respirations, and blood pressure. TOP: Assessment MSC: NCLEX: Physiological Integrity 21. The nurse is caring for a patient who has a pulse rate of 44. His blood pressure is within
normal limits. In trying to determine the cause of the patient’s low heart rate, what would the nurse suspect? a. That the patient has a fever. b. Possible hemorrhage or bN leU edRinSgI . NGTB.COM c. That the patient is taking calcium channel blockers or digitalis medications. d. Chronic obstructive pulmonary disease (COPD). ANS: C
Negative chronotropic drugs such as digitalis, beta-adrenergic agents, and calcium channel blockers can slow down pulse rate. Fever, bleeding, hemorrhage, and COPD all increase the body’s need for oxygen, which leads to an increase in heart rate. DIF: Understand REF: 546 OBJ: Describe factors that cause variations in body temperature, pulse, oxygen saturation, respirations, and blood pressure. TOP: Assessment MSC: NCLEX: Physiological Integrity 22. The patient was found unresponsive in her apartment and is being brought to the emergency
department. She has arm, hand, and leg edema; her temperature is 35.3°C (95.6°F); and her hands are cold secondary to her history of peripheral vascular disease. It is reported that she has a latex allergy. To quickly measure the patient’s oxygen saturation, what should the nurse do? a. Attach a finger probe to the patient’s index finger. b. Place a nonadhesive sensor on the patient’s ear lobe. c. Attach a disposable adhesive sensor to the bridge of the patient’s nose. d. Place the sensor on the same arm that the electronic blood pressure cuff is on. ANS: B
Canadian Fundamentals of Nursing 6th Edition Potter Test Bank The nurse should measure oxygen saturation on the ear or bridge of the nose in an adult patient who has a history of peripheral vascular disease. Sensors should not be attached to fingers, ears, or the bridge of the nose if area is edematous or skin integrity is compromised. Disposable adhesive probes should be avoided if the patient has latex allergy. The probe should not be attached to fingers that are hypothermic. The sensor should not be placed on the same extremity as the electronic blood pressure cuff because blood flow to finger will be temporarily interrupted when cuff inflates. DIF: Apply REF: 555 OBJ: Describe factors that cause variations in body temperature, pulse, oxygen saturation, respirations, and blood pressure. TOP: Planning MSC: NCLEX: Physiological Integrity 23. The patient is admitted with shortness of breath and chest discomfort. Which of the following
laboratory values could account for the patient’s symptoms? a. Hemoglobin level of 80 g/L. b. Hematocrit level of 45%. c. Red blood cell count of 5.0 million/mm3. d. Pulse oximetry value of 90%. ANS: A
The concentration of hemoglobin reflects the patient’s capacity to carry oxygen. Normal hemoglobin levels range from 10 to 18 g/100 mL (135 to 180 g/L) in men and from 12 to 16 g/100 mL (120 to 160 g/L) in women. A hemoglobin level of 8.0 (80 g/L) is low and indicates a decreased ability to deliver oxygen to meet bodily needs. All other values listed are considered normal. DIF: Analyze REF: 55N 0|U5R 51SINGTB.COM OBJ: Describe factors that cause variations in body temperature, pulse, oxygen saturation, respirations, and blood pressure. TOP: Implementation MSC: NCLEX: Physiological Integrity 24. Of the following values, which value would be considered as a high normal reading? a. 98/50 in a 7-year-old child. b. 115/70 in an infant. c. 135/85 in an older person. d. 120/80 in a middle-aged adult. ANS: C
An adult’s blood pressure tends to rise with advancing age. In most people, blood pressure should be lower than 140/90. Blood pressure greater than 140/90 is defined as hypertension. Blood pressure of 98/50 is normal for a child, and 115/70 can be normal for an infant. DIF: Evaluate REF: 557 OBJ: Describe factors that cause variations in body temperature, pulse, oxygen saturation, respirations, and blood pressure. TOP: Evaluate MSC: NCLEX: Physiological Integrity 25. A nurse is caring for a patient who smokes and drinks caffeine. Which point is important for
the nurse to understand before she assesses the patient’s blood pressure (BP)? a. Neither caffeine nor smoking affects blood pressure. b. She needs to insist that the patient stop smoking for at least 3 hours.
Canadian Fundamentals of Nursing 6th Edition Potter Test Bank c. The nurse should have the patient perform mild exercises. d. Caffeine and smoking can cause false BP elevations. ANS: D
Smoking immediately increases BP, and this increase lasts up to 15 minutes. Caffeine increases BP for up to 3 hours. Both affect a patient’s blood pressure. The patient should rest at least 5 minutes before BP is measured. DIF: Understand REF: 561 TOP: Implementation
OBJ: Describe ethnic variations in blood pressure. MSC: NCLEX: Physiological Integrity
26. When taking the pulse of an infant, the nurse notices that the rate is 145 beats/min and the
rhythm is regular. The nurse realizes that this rate is which of the following? a. Normal for an infant. b. The proper rate for a toddler. c. Too slow for an infant. d. The same as that of a normal adult. ANS: A
The normal pulse rate for an infant is 120 to 160 beats/min. The rate obtained (145 beats/min) is within the normal range for an infant. The normal rate for a toddler is between 90 and 140 beats/min. The normal rate for an adult is between 60 and 100 beats/min. DIF: Remember REF: 547 (Table 30-3) OBJ: Identify ranges of acceptable vital sign values for an infant, a child, and an adult. TOP: Assessment MSC: NCLEX: Physiological Integrity 27. The nurse is caring for an eldN erlyRpatien that his temperature is 36°C (96.8°F). This I tGandBn.oCtes M
O patient is in which condition? U S N T a. Suffering from hypothermia. b. Expressing a normal temperature. c. Hyperthermic relative to his age. d. Demonstrating the increased metabolism that accompanies aging. ANS: B
The average body temperature of older persons is approximately 36°C (96.8°F). This is not hypothermia or hyperthermia. Older persons have poor vasomotor control, reduced amounts of subcutaneous tissue, and reduced metabolism. The end result is lowered body temperature. DIF: Understand REF: 529| 530 OBJ: Identify ranges of acceptable vital sign values for an infant, a child, and an adult. TOP: Assessment MSC: NCLEX: Physiological Integrity 28. When assessing the temperature of newborns and children, the nurse decides to utilize a
temporal artery thermometer. Why is this preferable to methods used for adults? a. It is accurate even when the forehead is covered with hair. b. It is not affected by skin moisture. c. It reflects rapid changes in radiant temperature. d. There is no risk of injury to patient or nurse. ANS: D
Canadian Fundamentals of Nursing 6th Edition Potter Test Bank The temporal artery thermometer is especially beneficial when used in premature infants, newborns, and children because there is no risk of injury to the patient or nurse. However, it is inaccurate when the head is covered or when hair is on the forehead, and measurement is affected by skin moisture such as diaphoresis or sweating. The temporal artery thermometer provides very rapid measurement and reflects rapid changes in core temperature, not radiant temperature. DIF: Understand REF: 537 (Box 30-4) OBJ: Explain variations in technique used to assess an infant’s, a child’s, and an adult’s vital signs. TOP: Assessment MSC: NCLEX: Physiological Integrity 29. Which artery is the most appropriate for assessing the pulse of a small child? a. Radial. b. Femoral. c. Brachial. d. Ulnar. ANS: C
The brachial or apical pulse is the best site for assessing an infant’s or a young child’s pulse because other peripheral pulses are deep and difficult to palpate accurately. DIF: Apply REF: 542 OBJ: Explain variations in technique used to assess an infant’s, a child’s, and an adult’s vital signs. TOP: Implementation MSC: NCLEX: Physiological Integrity 30. The nurse is caring for a newborn in the hospital nursery. She notices that the infant is
breathing rapidly but is pink, warm, and dry. The nurse knows that the normal respiratory rate GiTnuBt. for a newborn is how many bN reU atR hsSpIeN rm e?COM a. 30 to 60. b. 25 to 32. c. 16 to 19. d. 12 to 20. ANS: A
The acceptable respiratory rate range for a newborn is 30 to 60 breaths per minute. An infant (6 months) is expected to have a rate between 30 and 50 breaths per minute. A toddler’s respiratory range is 25 to 32 breaths per minute. A child should breathe 20 to 30 times a minute. An adolescent should breathe 16 to 19 times a minute. An adult should breathe 12 to 20 times a minute. DIF: Remember REF: 553 OBJ: Explain variations in technique used to assess an infant’s, a child’s, and an adult’s vital signs. TOP: Assessment MSC: NCLEX: Physiological Integrity 31. While attempting to obtain oxygen saturation readings on a toddler, what should the nurse do? a. Place the sensor on the earlobe. b. Place the sensor on the bridge of the nose. c. Determine whether the toddler has a tape allergy. d. Ignore any variation between the oximeter pulse rate and the patient’s apical pulse
rate. ANS: C
Canadian Fundamentals of Nursing 6th Edition Potter Test Bank The nurse should determine whether the patient has latex allergy because disposable adhesive probes should not be used on patients with latex allergies. Sensors should not be placed on the earlobe or bridge of the nose in infants and toddlers because of skin fragility. Oximeter pulse rate and the patient’s apical pulse rate should be the same. Any difference necessitates re-evaluation of oximeter sensor probe placement and reassessment of pulse rates. DIF: Apply REF: 555 OBJ: Explain variations in technique used to assess an infant’s, a child’s, and an adult’s vital signs. TOP: Implementation MSC: NCLEX: Safe and Effective Care Environment 32. The nurse is preparing to assess the blood pressure of a 3-year-old. How should the nurse
proceed? a. Choose the cuff that says “Child” instead of “Infant.” b. Obtain the reading before the child has a chance to “settle down.” c. Use the diaphragm portion of the stethoscope to detect Korotkoff sounds. d. Explain to the child what the procedure will be. ANS: D
Preparing the child for the blood pressure cuff’s unusual sensation increases cooperation. Most children will understand the analogy of a “tight hug on your arm.” Different arm sizes require careful and appropriate cuff size selection. Do not choose a cuff on the basis of the name of the cuff. An “infant” cuff is too small for some infants. Readings are difficult to obtain in restless or anxious infants and children. Allow at least 15 minutes for children to recover from recent activities and become less apprehensive. Korotkoff sounds are difficult to hear in children because of low frequency and amplitude. A pediatric stethoscope bell is often helpful. DIF: Apply REF: 56N 6URSINGTB.COM OBJ: Explain variations in technique used to assess an infant’s, a child’s, and an adult’s vital signs. TOP: Implementation MSC: NCLEX: Physiological Integrity 33. What is one benefit of using a stationary automatic blood pressure device? a. It fits over clothing. b. It is extremely reliable. c. It is the method of choice for irregular heart rhythms. d. It is more reliable when pressure is less than 90 mm Hg systolic. ANS: A
The cuff fits over clothing. However, the reliability of stationary machines is limited. Electronic blood pressure measurement is not recommended with irregular heart rates or when systolic blood pressure is less than 90 mm Hg. DIF: Remember REF: 568 OBJ: Describe the benefits and precautions involving self-measurement of blood pressure. TOP: Assessment MSC: NCLEX: Physiological Integrity 34. The nurse is caring for a patient who is being discharged from the hospital after being treated
for hypertension. The patient is instructed to measure his blood pressure three times a day and to keep a record of the readings. The nurse recommends that the patient purchase a portable electronic blood pressure device. What other instruction should the nurse give the patient? a. The patient can apply the cuff in any manner he chooses because the machine is
Canadian Fundamentals of Nursing 6th Edition Potter Test Bank designed to be used by nonprofessionals. b. The patient must not adjust his medication dosages without consulting the health
care provider. c. The cuff can be placed over clothing if necessary. d. The machine is accurate when blood pressures are low. ANS: B
Patients must be advised that because of possible inaccuracies in the blood pressure devices, they must not adjust their medication regimens without consulting their health care provider. Because of their sensitivity, improper cuff placement or movement of the arm causes electronic devices to give incorrect readings. Stationary blood pressure devices are often found in public places, and the cuff fits over clothing. The same is not true for portable devices. Electronic blood pressure measurement is not recommended when systolic blood pressure is less than 90 mm Hg. DIF: Understand REF: 568| 569 OBJ: Describe the benefits and precautions involving self-measurement of blood pressure. TOP: Assessment MSC: NCLEX: Health Promotion and Maintenance 35. The nurse is caring for a patient who complains of feeling light-headed and “woozy.” The
nurse checks the patient’s pulse and finds that it is irregular. The patient’s blood pressure is 100/72. It was 113/80 an hour earlier. What should the nurse do? a. Call the physician immediately. b. Perform an apical/radial pulse assessment. c. Apply more pressure to the radial artery to assess the pulse. d. Use own thumb to detect the patient’s pulse. ANS: B
NURSINGTB.COM
If the pulse is irregular, the nurse should assess the apical/radial pulse to detect a pulse deficit. If pulse count differs by more than 2, a pulse deficit exists, which sometimes indicates alterations in cardiac output. The nurse needs to gather as much information as possible before calling the physician. The radial pulse is more accurately assessed with moderate pressure. Too much pressure occludes the pulse and impairs blood flow. Fingertips are the most sensitive parts of the hand to palpate arterial pulsations. The thumb has a pulsation of its own that interferes with accuracy. DIF: Apply REF: 544| 547| 548 OBJ: Identify when vital signs should be measured. MSC: NCLEX: Physiological Integrity
TOP: Implementation
36. Of the following patients, which one is the best candidate to have his temperature measured
orally? a. A 27-year-old postoperative patient with an elevated temperature. b. A teenage boy who has just returned from outside “for a smoke.” c. An 87-year-old confused male suspected of hypothermia. d. A 20-year-old male with a history of epilepsy. ANS: A
Canadian Fundamentals of Nursing 6th Edition Potter Test Bank An elevated temperature needs to be evaluated, and no contraindication is described for this patient. Ingestion of hot/cold fluids or foods, smoking, or receiving oxygen by mask/cannula can necessitate delays in measuring oral temperature. Oral temperatures are not measured in patients who have just had oral surgery or trauma, have a history of epilepsy, or have shaking chills, and they should not be measured in infants, small children, or confused patients. DIF: Evaluate REF: 537 (Box 30-4) OBJ: Identify when vital signs should be measured. MSC: NCLEX: Physiological Integrity
TOP: Assessment
37. The physician’s order reads “Lopressor (metoprolol) 50 mg PO daily. Do not give if blood
pressure is less than 100 mm Hg systolic.” The patient’s blood pressure is 92/66. The nurse does not give the medication; what else does the nurse do? a. Does not tell the patient what the blood pressure is. b. Documents only what the blood pressure was. c. Documents that the medication was not given owing to low blood pressure. d. Does not need to inform the health care provider that the medication was withheld. ANS: C
The nurse must document any interventions initiated as a result of vital sign measurement, such as withholding an antihypertensive drug. The nurse should inform the patient of the blood pressure value and the need for periodic reassessment of the blood pressure. Documenting only the blood pressure is not sufficient; any intervention must be documented as well. Abnormal findings must be reported to the nurse in charge or to the health care provider. DIF: Apply REF: 570| 571 OBJ: Accurately record and reN poUrtRvS itaIl N sigGnT mB ea.suCreOmMents. MSC: NCLEX: Physiological Integrity
TOP: Implementation
38. After measuring the patient’s temperature, the nurse documents the value and the route used
to obtain the reading. Why is this done? a. Temperatures are the same regardless of the route used. b. Temperatures vary depending on the route used. c. Temperatures are cooler when taken rectally than when taken orally. d. Axillary temperatures are higher than oral temperatures. ANS: B
Temperatures obtained vary depending on the site used. Rectal temperatures are usually 0.5°C (0.9°F) higher than oral temperatures, and axillary temperatures are usually 0.5°C (0.9°F) lower than oral temperatures. DIF: Understand REF: 532 OBJ: Accurately record and report vital sign measurements. MSC: NCLEX: Physiological Integrity
TOP: Implementation
39. When temperature assessment is required, which of the following cannot be delegated to an
unregulated care provider? a. Temperature measurement. b. Assessment of changes in body temperature. c. Selection of appropriate route and device.
Canadian Fundamentals of Nursing 6th Edition Potter Test Bank d. Consideration of factors that falsely raise temperature. ANS: B
The nurse is responsible for assessing changes in body temperature. The skill of temperature measurement can be delegated. The nurse instructs an unregulated care provider to select the appropriate route and device to measure temperature and to consider specific factors that falsely raise or lower temperature. DIF: Understand REF: 527| 532 OBJ: Appropriately delegate vital sign measurement to unregulated care providers. TOP: Implementation MSC: NCLEX: Safe and Effective Care Environment 40. The unregulated care provider is measuring vital signs and reports that a patient’s blood
pressure is abnormally low. What should the nurse do? a. Have the unregulated care provider retake the blood pressure. b. Ignore the report and have it rechecked at the next scheduled time. c. Retake the blood pressure herself and assess the patient’s condition. d. Have the unregulated care provider assess the patient’s other vital signs. ANS: C
The unregulated care provider should report abnormalities to the nurse, who should further assess the patient. The unregulated care provider should not remeasure the blood pressure or other vital signs because the nurse needs to assess the patient. The report cannot be ignored. Assessment must be done by the nurse. DIF: Apply REF: 527 OBJ: Appropriately delegate vital sign measurement to unregulated care providers. TOP: Implementation : C NCLMEX: Physiological Integrity N R I GMSBC.
U S N T
O
MULTIPLE RESPONSE 1. Of the following sites, which are used for obtaining a core temperature? (Select all that
apply.) a. Oral. b. Rectal. c. Tympanic. d. Axillary. e. Pulmonary artery. ANS: B, C, E
Sites of measurement of core temperature include rectum, tympanic membrane, temporal artery, esophagus, pulmonary artery, and urinary bladder. Sites of measurement of surface temperature include the skin, mouth, and axillae. DIF: Understand REF: 528 OBJ: Accurately assess temperature via multiple routes: tympanic, oral, temporal, rectal, and axillary. TOP: Assessment MSC: NCLEX: Physiological Integrity
Canadian Fundamentals of Nursing 6th Edition Potter Test Bank 2. The patient has new-onset restlessness and confusion. His pulse rate is elevated, as is his
respiratory rate. His oxygen saturation, however, is 94% according to the portable pulse oximeter. The nurse ignores the oximeter reading and calls the physician to obtain an order for measuring arterial blood gases (ABGs). The nurse does this because many things can cause inaccurate pulse oximetry readings, including which of the following? (Select all that apply.) a. Oxygen saturations (SaO2) higher than 70%. b. Carbon monoxide inhalation. c. Nail polish. d. Hypothermia at the assessment site. e. Intravascular dyes. ANS: B, C, D, E
Inaccurate pulse oximetry readings can be caused by outside light sources, carbon monoxide (caused by smoke inhalation or poisoning), patient motion, jaundice, intravascular dyes (methylene blue), nail polish, artificial nails, metal studs, or dark skin. Other factors include peripheral vascular disease (atherosclerosis), hypothermia at the assessment site, the use of pharmacological vasoconstrictors (e.g., epinephrine), low cardiac output, hypotension, peripheral edema, and tight probes. DIF: Understand REF: 556 (Box 30-11) OBJ: Accurately assess tympanic, oral, rectal, and axillary temperatures. TOP: Assessment MSC: NCLEX: Physiological Integrity 3. The nurse is assessing the patient and his family for probable familial causes of the patient’s
hypertension. The nurse begins by analyzing the patient’s personal history, as well as family history and current lifestyle situation. Which of the following issues would be considered risk factors? (Select all that apply.) N R I G B.C M a. Obesity. U S N T O b. Cigarette smoking. c. Recent weight loss. d. Heavy alcohol consumption. e. Low blood cholesterol levels. ANS: A, B, D
Obesity, cigarette smoking, heavy alcohol consumption, high blood cholesterol and triglyceride levels, and continued exposure to stress are risk factors linked to hypertension. Weight loss and low blood cholesterol levels are not risk factors for hypertension. DIF: Analyze TOP: Evaluate
REF: 560 OBJ: Describe ethnic variations in blood pressure. MSC: NCLEX: Health Promotion and Maintenance
4. The patient is being encouraged to purchase a portable automatic blood pressure device so he
can monitor his own blood pressure at home. What are some of the benefits of self-monitoring? (Select all that apply.) a. Blood pressures can be obtained if pulse rates become irregular. b. Patients can provide information about patterns to health care providers. c. Patients can actively participate in their treatment. d. Self-monitoring helps with compliance and treatment. e. The risk of obtaining an inaccurate reading is decreased. ANS: B, C, D
Canadian Fundamentals of Nursing 6th Edition Potter Test Bank Self-measurement of blood pressure has several benefits. Sometimes elevations in blood pressure are detected in persons previously unaware of a problem. Persons with prehypertension provide information about the pattern of blood pressure values to their health care provider. Patients with hypertension benefit from participating actively in their treatment through self-monitoring, which promotes compliance with treatment. Disadvantages of self-measurement include the risk of inaccurate readings. Use of electronic devices is not recommended for patients who have an irregular heart rate. DIF: Understand REF: 568| 569 OBJ: Describe the benefits and precautions involving self-measurement of blood pressure. TOP: Assessment MSC: NCLEX: Health Promotion and Maintenance 5. When the nurse records the patient’s respiratory status, what must be recorded? (Select all that
apply.) a. Respiratory rate. b. Character of respirations. c. Amount of oxygen therapy. d. Only normal findings. e. Only in the graphic section. ANS: A, B, C
Respiratory rate and character are recorded in nurses’ notes or on the vital sign flow sheet. If oxygen therapy is used during assessment, the type and amount of oxygen should be indicated. Respiratory assessment should be documented in narrative form in nurses’ notes after administration of specific therapies. The nurse should document normal and abnormal findings. DIF: Remember REF: 55N 2URSINGTB.COM OBJ: Accurately record and report vital sign measurements. MSC: NCLEX: Safe and Effective Care Environment
TOP: Implementation
Canadian Fundamentals of Nursing 6th Edition Potter Test Bank
Chapter 31: Pain Assessment and Management Potter et al: Canadian Fundamentals of Nursing, 6th Edition MULTIPLE CHOICE 1. What is the most appropriate way to assess the pain of a patient who is oriented and has
recently had surgery? a. Assess the patient’s body language. b. Observe cardiac monitor for increased heart rate. c. Ask the patient to rate the level of pain. d. Ask the patient to describe the effect of pain on the ability to cope. ANS: C
Pain is a subjective measure. Therefore, the best way to assess a patient’s pain is to ask the patient to rate the pain. Nonverbal communication, such as body language, is not as effective in assessing pain, especially when the patient is oriented. Heart rate sometimes increases when a patient is in pain, but this is not a symptom that is specific to pain. Pain sometimes affects a patient’s ability to cope, but assessing the effect of pain on coping assesses the patient’s ability to cope; it does not assess the patient’s pain. DIF: Understand REF: 575| 585 OBJ: Perform an assessment of a patient experiencing pain. MSC: NCLEX: Physiological Integrity
TOP: Assessment
2. A nurse is caring for a patient who recently had an abdominal hysterectomy and states that she
is experiencing severe pain. The patient’s blood pressure is 110/60, and her heart rate is 60. In N ppe RS I G inB.C M addition, the patient does not aU ar toNbeT any dOistress. Which response by the nurse is most therapeutic? a. “Your vitals do not show that you are having pain; can you describe your pain?” b. “You do not look like you are in pain.” c. “OK, I will go get you some narcotic pain relievers immediately.” d. “What would you like to try to alleviate your pain?” ANS: D
The nurse must believe that a patient is in pain whenever the patient reports that he or she is in pain, even if the patient does not appear to be in pain. Whenever the patient reports pain, the nurse needs to collaborate with the patient to determine the best method of pain relief, whether it be medication, meditation, or repositioning. The nurse must be careful to not judge the patient on the basis of vital signs or nonverbal communication and must not assume that the patient is seeking narcotics. The patient is a partner in pain management, and so going to get narcotics to treat the pain without consulting with the patient first is not appropriate. DIF: Understand REF: 585| 590| 591 OBJ: Describe guidelines for selecting and individualizing pain interventions. TOP: Implementation MSC: NCLEX: Physiological Integrity 3. Which of the following statements made by a patient indicates that the patient understands the
relationship between the gate control theory of pain and the use of meditation to relieve pain? a. “Meditation controls pain by blocking pain impulses from coming through the gate.”
Canadian Fundamentals of Nursing 6th Edition Potter Test Bank b. “Meditation will help me sleep through the pain because it opens the gate.” c. “Meditation stops the occurrence of pain stimuli.” d. “Meditation alters the chemical composition of pain neuroregulators, which closes
the gate.” ANS: A
According to the gate theory, pain impulses cause pain when they get through gates that are open. Pain is blocked when the gates are closed. Nonpharmacological pain relief measures, such as meditation, work by closing the gates, which keeps pain impulses from coming through. Meditation does not open pain gates or stop pain from occurring. Meditation also does not have an effect on pain neuroregulators. DIF: Evaluate TOP: Evaluate
REF: 578 OBJ: Describe the physiology of pain. MSC: NCLEX: Physiological Integrity
4. A nursing student is planning care for an older patient who is experiencing pain. Which of the
following statements made by the nursing student indicates the need for the nursing professor to clarify the nursing student’s knowledge? a. “Older patients often have difficulty determining what is causing their pain.” b. “It is safe to administer opioids to older persons as long as you start with small doses and frequently assess the patient’s response to the medication.” c. “As adults age, their ability to perceive pain decreases.” d. “Patients who have dementia probably experience pain, and their pain is not always well controlled.” ANS: C
Aging does not affect the ability to perceive pain. Sometimes older persons have difficulty interpreting their pain and deN teU rmRini ngNiG tsT caBu. seCbO ecMause multiple diseases and vague SI symptoms affect similar parts of the body. Opioids are safe to use in older persons as long as they are slowly titrated and the nurse frequently monitors the patient. Current evidence shows that patients with dementia probably experience unrelieved pain because their pain is difficult to assess. DIF: Evaluate REF: 581 (Box 31-4) OBJ: Identify barriers to effective pain management. MSC: NCLEX: Physiological Integrity
TOP: Planning
5. The nurse is caring for two patients; both are having a hysterectomy. The first patient is
having the hysterectomy after a complicated birth. The second patient has uterine cancer. What will most likely influence the experience of pain for these two patients? a. Neurological factors. b. Competency of the surgeon. c. Meaning of pain. d. Postoperative support personnel. ANS: C
Canadian Fundamentals of Nursing 6th Edition Potter Test Bank The patient’s perception of pain is influenced by psychological factors, such as anxiety and coping, which in turn influence the patient’s experience of pain. Each patient’s experience is different. The degree and quality of pain perceived by a patient are related to the meaning of the pain. Neurological factors can interrupt or influence pain perception, but neither of these patients is experiencing alterations in neurological function. The knowledge, attitudes, and beliefs of nurses, physicians, and other health care personnel about pain affect pain management but do not necessarily influence a patient’s pain perceptions. DIF: Understand TOP: Assessment
REF: 577 OBJ: Identify components of the pain experience. MSC: NCLEX: Physiological Integrity
6. The nurse anticipates administering an opioid fentanyl patch to which patient? a. A 15-year-old adolescent with a broken femur. b. A 30-year-old adult with cellulitis. c. A 50-year-old patient with prostate cancer. d. An 80-year-old patient with a broken hip. ANS: C
A fentanyl patch is an extended-relief opioid that provides pain relief for 24 hours a day. This is ideal for patients who have chronic severe pain, such as those who have cancer. The other patients are expected to experience acute pain. Therefore, they will probably benefit more from oral or intravenous (IV) opioids for short-term pain relief. DIF: Apply REF: 602 OBJ: Explain the various pharmacological approaches to treating pain. TOP: Implementation MSC: NCLEX: Physiological Integrity 7. What nursing intervention is N mU osRt S efI feN ctG ivT eB in.pC reO veMnting injury to a patient after
administration of epidural anaesthetic? a. Keeping the reversal agent in a syringe in the patient’s bedside table. b. Applying a gauze dressing to the epidural catheter insertion site. c. Labelling the tubing that leads to the epidural catheter. d. Asking the nursing assistive personnel to check on the patient at least once every 2 hours. ANS: C
To reduce the accidental administration of IV medications into the epidural catheter, the tubing that leads to the epidural catheter needs to be labelled clearly. Medications used to reverse the action of the anaesthetic medication need to be kept in a secured location, not in the patient’s room in an unsecured location. The epidural insertion site needs to be covered by a clear occlusive dressing to prevent infection and allow the nurse to assess the site. Patients receiving epidural anaesthetic need to be monitored every 15 minutes until stabilized and then at least hourly. DIF: Apply REF: 601 OBJ: Discuss nursing implications for administering analgesics. TOP: Implementation MSC: NCLEX: Safe and Effective Care Environment 8. A 24-year-old Asian woman is in labour and refuses to receive any sort of anaesthetic. Which
alternative treatment is best for this patient? a. Relaxation and guided imagery.
Canadian Fundamentals of Nursing 6th Edition Potter Test Bank b. Transcutaneous electrical nerve stimulation (TENS). c. Herbal supplements with analgesic effects. d. Pudendal block. ANS: A
In some cultures, nonpharmacological measures are preferred for pain control. In the case of a patient in labour, relaxation with guided imagery is often an effective supplement for pain management because it provides women with a sense of control over their pain. Relaxation and guided imagery can be used during any phase of health or illness. TENS units are typically used to manage postsurgical and procedural pain. Herbal supplements need to be evaluated for safety during pregnancy; in addition, some patients consider herbal supplements to be another form of medication, and they are not typically used to control acute pain. A pudendal block is a type of regional anaesthetic; administering it shows a lack of respect for the patient’s wishes for nonpharmacological pain control. DIF: Apply REF: 594 OBJ: Describe applications for the use of nonpharmacological pain interventions. TOP: Implementation MSC: NCLEX: Psychosocial Integrity 9. Which of the following statements made by the patient indicates to the nurse that teaching on
a patient-controlled analgesia (PCA) device has been effective? a. “This is the only pain medication I will need to be on.” b. “I can administer the pain medication as frequently as I need to.” c. “I feel less anxiety about the possibility of overdosing.” d. “I will need the nurse to notify me when it is time for another dose.” ANS: C
A PCA device allows the patN ient rmin URtoSdIete NG TBe.thCeOleMvel of pain relief delivered, which reduces the risk of oversedation. Its use often eases anxiety because the patient is not reliant on the nurse for pain relief. Other medications, such as oral analgesics, can be given in addition to the PCA device. The PCA device does have a minimum time limit to prevent overdose, but the patient can lengthen the amount of time between doses. One benefit of PCA is that the patient does not need to rely on the nurse to administer pain medication; the patient determines when to take the medication. DIF: Evaluate REF: 598| 599 OBJ: Explain the various pharmacological approaches to treating pain. TOP: Implementation MSC: NCLEX: Health Promotion and Maintenance 10. A nurse is caring for a patient who is experiencing pain after abdominal surgery. What
information is important for the nurse to tell the patient when providing patient education about effective pain management? a. “To prevent overdose, you need to wait to ask for pain medication until you begin to experience pain.” b. “You should take your medication after you walk to make sure you do not fall while you are walking.” c. “We should work together to create a regular schedule of medications that does not allow for breakthrough pain.” d. “You need to take oral pain medications when you experience severe pain.” ANS: C
Canadian Fundamentals of Nursing 6th Edition Potter Test Bank The best way to manage pain is to develop a schedule of medications that are given around the clock to prevent breakthrough pain. The nurse should not wait until the patient is in pain because it takes medications 10 to 30 minutes to begin to relieve pain. The nurse administers pain medications before painful activities, such as walking, and administers intravenous medications when pain is severe. DIF: Apply REF: 591 OBJ: Discuss nursing implications for administering analgesics. TOP: Implementation MSC: NCLEX: Health Promotion and Maintenance 11. A nurse is caring for a patient who recently had spinal surgery. The nurse knows that patients
usually experience acute pain after this type of surgery. The patient refuses to get up and walk and is not moving around in the bed. However, the patient is stoic and denies experiencing pain at this time. What most likely explains this patient’s behaviour and response to surgery? a. The surgery successfully cured the patient’s pain. b. The patient’s culture is possibly influencing the patient’s experience of pain. c. The patient is experiencing urinary retention because of manipulation of the spine during surgery; this is preventing the patient from experiencing pain. d. The nurse is allowing personal beliefs about pain to influence pain management at this time. ANS: B
A patient’s culture can influence the patient’s expression of pain. In this case, the patient has just had surgery, and the nurse knows that this surgical procedure usually causes pain. It is important at this time for the nurse to examine cultural and ethnic factors that are possibly affecting the patient’s denial of pain at this time. Even if surgery corrects neurological factors that create chronic pain, surgery causes pain in the acute period. Urinary retention usually N R gica INl G B.C M creates pain and does not maskUsurS paTin. TheOnurse is not allowing personal beliefs to influence pain management because the nurse is attempting to determine the reason why the patient is not verbalizing the experience of pain. DIF: Apply REF: 583| 584 OBJ: Explain how cultural factors influence the pain experience. TOP: Assessment MSC: NCLEX: Psychosocial Integrity 12. A nurse is providing discharge teaching for a patient with a fractured humerus. The patient is
going home with hydrocodone bitartrate and acetaminophen (Vicodin). What important patient education does the nurse provide? a. “Be sure to eat a meal high in fat before taking the medication, to avoid a stomach ulcer.” b. “Narcotics can be addictive, so do not take them unless you are in severe pain.” c. “You need to drink plenty of fluids and eat a diet high in fibre.” d. “As your pain severity lessens, you will begin to give yourself once-daily intramuscular injections.” ANS: C
Canadian Fundamentals of Nursing 6th Edition Potter Test Bank A common side effect of opioid analgesics is constipation. Therefore, the nurse encourages the patient to drink fluids and eat fibre to prevent constipation. Although medications can be irritating to the stomach, a diet high in fat does not prevent gastric ulcers. To best manage pain, the patient needs to take pain medication before painful procedures or activities or before pain becomes severe. As the patient’s pain gets better, the strength of the medications will decrease. Intramuscular, IV, and topical analgesics are used for more severe and chronic pain. DIF: Analyze REF: 592| 597| 598 OBJ: Discuss nursing implications for administering analgesics. TOP: Implementation MSC: NCLEX: Physiological Integrity 13. A patient arrives at the emergency department experiencing a headache and rates the pain as 7
on a 0-to-10 pain scale. What nonpharmacological intervention does the nurse implement for this patient while awaiting orders for pain medication from the health care provider? a. Frequently reassesses the patient’s pain scores. b. Reassures the patient that the provider will come to the emergency department soon. c. Softly plays music that the patient finds relaxing. d. Teaches the patient how to do yoga. ANS: C
The appropriate nonpharmacological pain management intervention is to quietly play music that the patient finds relaxing. Music diverts a person’s attention away from pain and creates relaxation. Reassessing the patient’s pain scores is done during evaluation. Building the patient’s expectation of the provider’s arrival does not address the patient’s pain. Although yoga promotes relaxation, nurses teach relaxation techniques only when a patient is not N seRtSheIpNatie GTntB.C M experiencing acute pain. BecauU is haO ving acute pain, this is not an appropriate time to provide patient teaching. DIF: Apply REF: 595 OBJ: Describe applications for the use of nonpharmacological pain interventions. TOP: Implementation MSC: NCLEX: Psychosocial Integrity 14. A patient who has had type 2 diabetes for 26 years is beginning to experience peripheral
neuropathy in the feet and lower legs, which is causing the patient to have a decreased ability to feel pain in the lower extremities. The nurse is providing education to the patient to prevent injury to the feet. The nurse tells the patient to always wear shoes or slippers when walking. Which of the following statements made by the nurse best explains the rationale for this instruction? a. “Wearing shoes blocks pain perception and helps you adapt to pain, which ends up protecting your feet.” b. “Shoes provide nonpharmacological pain relief to people with diabetes and peripheral neuropathy.” c. “Since you cannot feel pain as much in your feet, you need to open your neurological gates to allow pain sensations to come through. Wearing shoes helps to open those gates, which protects your feet.” d. “You have lost the ability to withdraw from pain because of your peripheral neuropathy. If you step on something and are not wearing shoes, you will not feel it; this could possibly cause injury to your foot.”
Canadian Fundamentals of Nursing 6th Edition Potter Test Bank ANS: D
This patient is losing the ability to feel pain because of peripheral neuropathy. The patient will no longer have protective reflexes to prevent injury to the feet. Wearing shoes prevents the patient from injuring the feet because they protect the feet. Shoes do not block pain perception, nor do they help people adapt to pain. Shoes are not a form of nonpharmacological pain relief. Wearing shoes will not have an effect on opening or closing the pain gates. DIF: Apply REF: 576-578 TOP: Implementation
OBJ: Describe the physiology of pain. MSC: NCLEX: Health Promotion and Maintenance
15. A nurse is assessing a patient who started to have severe pain 3 days ago. When the nurse asks
the patient to describe the pain, the patient states, “The pain feels like it is in my stomach. It is a burning pain, and it spreads out in a circle around the spot where it hurts the most.” What type of pain does the nurse document that the patient is having at this time? a. Superficial pain. b. Idiopathic pain. c. Chronic pain. d. Visceral pain. ANS: D
Visceral pain comes from visceral organs, such as those of the gastrointestinal tract. Visceral pain is diffuse and radiates in several directions. Superficial pain has a short duration and is usually a sharp pain. Pain of an unknown cause is idiopathic pain. Chronic pain lasts longer than 6 months. DIF: Analyze REF: 587 (Table 31-5) OBJ: Perform an assessment of a patient experiencing pain. MSC: NCLEX: Physiological IN ntU egR riS tyINGTB.COM
TOP: Assessment
16. A patient who had a motor vehicle accident 2 days ago is experiencing pain and is receiving
patient-controlled analgesia (PCA). How does the nurse know that the patient is experiencing effective pain management with the PCA? a. The patient is sleeping and is difficult to arouse. b. The patient rates pain at an acceptable level of 3 on a 0-to-10 scale. c. Sufficient medication is left in the PCA syringe. d. The patient presses the control button to deliver pain medication. ANS: B
The effectiveness of pain relief measures is determined by the patient’s generally rating pain at the level of 3 or less on a 0-to-10 pain scale. If the patient is satisfied with the amount of pain relief, then pain measures are effective. A patient who is sleeping and is difficult to arouse is possibly oversedated; the nurse needs to assess this patient further. The amount of medication left in the PCA syringe does not indicate whether pain management is effective. Pressing the button shows that the patient knows how to use the PCA but does not evaluate pain management. DIF: Evaluate TOP: Evaluate
REF: 598| 599 OBJ: Evaluate a patient’s response to pain interventions. MSC: NCLEX: Physiological Integrity
17. The nurse recognizes that which of the following is a modifiable contributor to a patient’s
perception of pain?
Canadian Fundamentals of Nursing 6th Edition Potter Test Bank a. b. c. d.
Age and gender. Anxiety and fear. Culture. Previous pain experience.
ANS: B
The nurse can take measures to ease the patient’s anxiety and fear related to pain. Age, gender, culture, and previous pain experience are all nonmodifiable factors that the nurse can help the patient to understand in relation to pain, but the nurse cannot alter them. DIF: Understand TOP: Assessment
REF: 583 OBJ: Identify components of the pain experience. MSC: NCLEX: Psychosocial Integrity
18. The nurse is evaluating the effectiveness of guided imagery for pain management as used for a
patient who has second- and third-degree burns and needs extensive dressing changes. Which situation best indicates that guided imagery is controlling the patient’s pain effectively during dressing changes? a. The patient’s need for analgesic medication decreases during the dressing changes. b. The patient rates pain during the dressing change as a 6 on a scale of 0 to 10. c. The patient’s facial expressions are stoic during the procedure. d. The patient can tolerate more pain, so dressing changes can be performed more frequently. ANS: A
The purpose of guided imagery is to allow the patient to alter the perception of pain. Guided imagery works in conjunction with analgesic medications, potentiating their effects. If the patient needs less pain medication during dressing changes, then guided imagery is helping to manage the patient’s pain. AN raU tiR ngSoIf N 6G onTaB0.-tC o-O1M 0 pain scale indicates that the patient is having moderate pain and is not experiencing pain relief at this time. A person who is stoic is not showing feelings, which makes it difficult to know whether the patient is experiencing pain. The ability to change dressings more frequently is not a way to evaluate the effectiveness of guided imagery. DIF: Evaluate TOP: Evaluate
REF: 594 OBJ: Evaluate a patient’s response to pain interventions. MSC: NCLEX: Psychosocial Integrity
19. A nurse is providing medication education to a patient who just started taking ibuprofen, a
nonselective nonsteroidal anti-inflammatory drug (NSAID). Which statement made by the nurse best indicates how ibuprofen works? a. “Ibuprofen helps to remove factors that cause or stimulate pain.” b. “Ibuprofen reduces anxiety, which will help you better cope with your pain.” c. “Ibuprofen helps to decrease the production of prostaglandins.” d. “Ibuprofen binds with opiate receptors to reduce your pain.” ANS: C
NSAIDs such as ibuprofen probably work by decreasing the synthesis of prostaglandins to inhibit cellular responses to inflammation. Ibuprofen does not remove factors that cause pain, nor does it enhance coping with pain. Opioids bind with opiate receptors to modify perceptions of pain. DIF: Understand
REF: 597
Canadian Fundamentals of Nursing 6th Edition Potter Test Bank OBJ: Explain how the physiology of pain relates to selecting interventions for pain relief. TOP: Implementation MSC: NCLEX: Physiological Integrity 20. A nurse has brought the patient his scheduled pain medication. The patient asks the nurse to
wait to give pain medication until the time for the dressing change, which is 2 hours away. Which response by the nurse is most therapeutic? a. “This medication will still be providing you relief at the time of your dressing change.” b. “OK, swallow this pain pill, and I will return in a minute to fill your wound.” c. “Would you like medication to be given for dressing changes on top of your regularly scheduled medication?” d. “Your medication is scheduled for this time, and I can’t adjust the time for you. I’m sorry, but you must take your pill right now.” ANS: C
As-needed (prn) doses of medication can be given to patients in certain circumstances, as with an extensive dressing change. By asking to hold off on the dose, the patient is indicating that the dressing changes are extremely painful. The regularly scheduled dose might not be as effective for the patient. Oral medications take 30 to 60 minutes to take effect. If the nurse began the dressing change right then, the medication would not have been absorbed yet. The patient has the right to refuse to take a medication. It is the nurse’s responsibility to communicate with the provider and with the patient about a pain control plan that works for both. DIF: Evaluate REF: 598 OBJ: Explain various pharmacological approaches to treating pain. TOP: Implementation MSC: NCLEX: Health Promotion and Maintenance
NURSINGTB.COM
21. A nurse receives an order from a health care provider to administer Vicodin ES, which
contains 750 mg acetaminophen and 7.5 mg hydrocodone, to a patient who is experiencing postsurgical pain rated 8 on a 0-to-10 scale. The order is to give 2 tablets every 6 hours by mouth as needed for pain. What is the nurse’s best next action? a. Give the Vicodin ES to the patient immediately because the patient is experiencing severe pain. b. Ask the health care provider to verify the dosage and frequency of the medication. c. Ask the health care provider for an order for a nonsteroidal anti-inflammatory drug (NSAID). d. Ask the health care provider for an order to play music for the patient, in addition to providing the pain medication. ANS: B
The maximum safe 24-hour dosage for acetaminophen is 4 g. If the patient took 2 tablets of Vicodin ES every 6 hours, the patient would be taking in 6 g of acetaminophen in 24 hours. This exceeds the safe dosage of acetaminophen, so the best action is to question this order. Giving the medication as ordered would possibly result in acetaminophen overdose. Acetaminophen overdose can result in liver failure. NSAIDs are used to treat mild to moderate pain. At this moment, the patient is experiencing severe pain. Implementing music therapy is a nursing intervention and is an independent nursing action. Thus an order to start music therapy is not needed. DIF: Evaluate
REF: 597
Canadian Fundamentals of Nursing 6th Edition Potter Test Bank OBJ: Discuss nursing implications for administering analgesics. TOP: Implementation MSC: NCLEX: Physiological Integrity 22. The nurse knows that which technique is best for assessing pain in a child who is 4 years of
age? a. Ask the parents if they think their child is in pain. b. Use the FACES scale. c. Ask the child to rate the level of pain on a 0-to-10 pain scale. d. Check to see what previous nurses have charted. ANS: B
Assessing pain intensity in children requires special techniques. Young children often have difficulty expressing their pain. The FACES scale is used to assess pain in children who are verbal. Because most 4-year-olds are verbal, this is an appropriate scale to use with this child. Parents’ statement of pain is not an effective way to assess pain in children because children’s statements are the most important. The 0-to-10 pain scale is too difficult for a 4-year-old child to understand. Previous documentation by nurses will tell you what the child’s pain has been but will not tell you the child’s current pain intensity. DIF: Understand TOP: Assessment
REF: 586-588 OBJ: Describe the components of pain assessment. MSC: NCLEX: Physiological Integrity
23. Which statement made by a nursing educator best explains why it is important for nurses to
determine a patient’s medical history and recent drug use? a. “Health care providers have a responsibility to prevent drug seekers from gaining access to drugs.” b. “This information is useful in determining what type of pain interventions will most likely be effective inNpUrR ovSidIinNgGpT aiB n. reC lieOf.M ” c. “Some recreational drugs have pharmaceutical counterparts that may be more effective in managing pain.” d. “Getting this information gives the nurse an opportunity to provide patient teaching about drug abstinence.” ANS: B
In providing effective pain management, it is important to understand the patient’s history, what drugs the patient has already tried, and what interventions work best or have negative actions. It is not the nurse’s responsibility to judge or question a patient’s pain or label the patient as a “drug seeker.” Nurses need to avoid labelling patients as drug seekers because this term is poorly defined and creates bias and prejudice among other health care providers. Although certain recreational drugs do have pharmaceutical counterparts, this is not the sole purpose of assessing drug use. The nurse needs more information beyond a patient’s medical and medication history to determine whether a patient needs teaching about drug abstinence. DIF: Evaluate REF: 585| 603| 604 OBJ: Identify barriers to effective pain management. MSC: NCLEX: Physiological Integrity
TOP: Assessment
24. A nurse is supervising a student who is caring for a patient with chronic pain. Which
statement by the student indicates an understanding of pain management? a. “This patient says her pain is a 5, but she is not acting like it. I am not going to give her any pain medication.”
Canadian Fundamentals of Nursing 6th Edition Potter Test Bank b. “The patient is sleeping, so I pushed her PCA button for her.” c. “I need to reassess the patient’s pain 1 hour after administering oral pain
medication.” d. “It wasn’t time for the patient’s medication, so when she requested it, I gave her a
placebo.” ANS: C
Because the action of oral medications usually peaks in about an hour, you need to reassess the patient’s pain within an hour of administration. Nurses must believe any patient report of pain, even if nonverbal communication is not consistent with pain ratings. The patient is the only person who should push the PCA button. Pushing the PCA when a patient is sleeping is dangerous and may lead to narcotic overdose or respiratory depression. Giving the patient a placebo and telling her it is her medication is unethical. DIF: Evaluate REF: 598 OBJ: Discuss nursing implications for administering analgesics. TOP: Evaluate MSC: NCLEX: Health Promotion and Maintenance 25. The nurse is assessing how a patient’s pain is affecting mobility. Which assessment question
is most appropriate? a. “Have you considered working with a physiotherapist?” b. “What activities, if any, has your pain prevented you from doing?” c. “Would you please rate your pain on a scale from 0 to 10 for me?” d. “What effect does your pain medication typically have on your pain?” ANS: B
Because the nurse is interested in knowing whether the patient’s pain is affecting mobility, the priority assessment question N is U toRask tie.nC t ho w the pain affects his or her ability to SIthe NGpa TB OM participate in normal activities of daily living. Although a physical therapist is a good resource, especially if pain is severely affecting mobility, considering working with a physical therapist does not describe the effect of pain on the patient’s mobility. Assessing quality of pain and effectiveness of pain medication does not help the nurse to understand how it is affecting the patient’s mobility. DIF: Understand REF: 584| 585 OBJ: Perform an assessment of a patient experiencing pain. MSC: NCLEX: Physiological Integrity
TOP: Assessment
26. The nurse is teaching a student nurse about pain assessment scales. Which statement by the
student indicates correct understanding? a. “You cannot use a pain scale to compare the pain of my patient with the pain of your patient.” b. “When patients say they don’t need pain medication, they aren’t in pain.” c. “Pain assessment scales indicate the quality of a patient’s pain.” d. “A patient’s behaviour is more reliable than the patient’s report of pain.” ANS: C
Canadian Fundamentals of Nursing 6th Edition Potter Test Bank To gain a better understanding of a patient’s current pain status and to determine what interventions are needed, the nurse should assess both current and previous pain scores. A patient who rates pain at 4 might find the pain manageable if 24 hours earlier he had rated his pain at 10. Some patients do not express their pain or do not wish to take medications to relieve the pain. This does not mean they are not in pain; the nurse can try nonpharmacological therapies for such patients. DIF: Evaluate REF: 586 OBJ: Perform an assessment of a patient experiencing pain. MSC: NCLEX: Physiological Integrity
TOP: Assessment
27. The nurse is administering pain medication to several patients. Which patient does the nurse
administer medication to first? a. The patient who needs to take a scheduled dose of maintenance pain medication. b. The patient who needs to be premedicated before walking. c. The patient with a PCA running who needs to have the syringe replaced. d. The patient who is experiencing 8/10 pain and has a STAT order for pain medication. ANS: D
STAT medications need to be given as soon as possible. In addition, this patient is the priority because of the report of severe pain. The other patients need pain medication, but their situations are not as high a priority as that of the patient with the STAT medication order. DIF: Analyze REF: 591 OBJ: Discuss nursing implications for administering analgesics. TOP: Implementation MSC: NCLEX: Physiological Integrity
NURSINGTB.COM
28. The nurse is assessing a patient for opioid tolerance. Which finding supports the nurse’s
assessment? a. Increasingly higher doses of opioid are needed to control pain. b. The patient needed a substantial dose of naloxone (Narcan). c. The patient asks for pain medication close to the time it is due around the clock. d. The patient no longer experiences sedation from the usual dose of opioid. ANS: A
In opioid tolerance, increasing doses of an opioid are needed to control pain. Naloxone (Narcan) is an opioid antagonist that is given to reverse the effects of opioid overdose. Taking pain medications regularly around the clock is an effective way to control pain. The pain medication for this patient is probably managing the patient’s pain effectively because the patient is not asking for the medication before it is due. No longer experiencing a side effect of an opioid does not indicate opioid tolerance. DIF: Understand TOP: Assessment
REF: 602 OBJ: Evaluate a patient’s response to pain interventions. MSC: NCLEX: Physiological Integrity
29. A nurse is caring for a patient with rheumatoid arthritis who is now going to be taking 2
acetaminophen (Tylenol) tablets every 6 hours to control pain. Which part of the patient’s social history is the nurse most concerned about? a. Patient drinks 1 to 2 glasses of wine every night. b. Patient smokes 2 packs of cigarettes a day.
Canadian Fundamentals of Nursing 6th Edition Potter Test Bank c. Patient occasionally smokes marijuana. d. Patient takes antianxiety medications. ANS: A
The major adverse effect of acetaminophen is hepatotoxicity. Both alcohol and acetaminophen are metabolized by the liver; when taken together, they can cause liver damage. Smoking cigarettes and smoking marijuana are not healthy behaviours, but their effects on health are not affected by acetaminophen. Antianxiety medications can be taken with acetaminophen. DIF: Apply REF: 597 OBJ: Discuss nursing implications for administering analgesics. TOP: Assessment MSC: NCLEX: Physiological Integrity 30. The nurse is caring for a patient who suddenly experiences chest pain. What is the nurse’s first
priority? a. Call the rapid response team. b. Ask the patient to rate and describe the pain. c. Raise the head of the bed. d. Administer pain relief medications. ANS: B
The nurse’s ability to establish a nursing diagnosis, plan and implement care, and evaluate the effectiveness of care depends on an accurate and timely assessment. The other responses are all interventions; the nurse cannot know which intervention is appropriate until the nurse completes the assessment, makes a nursing diagnosis, and plans care. DIF: Apply REF: 584| 585 OBJ: Describe guidelines for sN elecR tingIandGindB ividualizMing pain interventions. U SPhysiological N T .CIntegrity O TOP: Assessment MSC: NCLEX: 31. The nurse is caring for a patient who recently had surgery to repair a hernia. The patient’s pain
was rated 7 on a scale of 0 to 10 before pain medication was administered. One hour after receiving an oral opioid, the patient ranks his pain at 3. The patient asks the nurse why he isn’t receiving more pain medication. Which is the nurse’s best response? a. “This medication can be given only every 4 hours. It is not time for you to have any other pain medication right now.” b. “I will notify the health care provider to come perform an assessment if your pain doesn’t improve in 30 minutes.” c. “If the pain becomes severe, we may need to transfer you to an intensive care unit.” d. “It can take longer for oral pain medication to work, and your pain is going down. Let’s try boosting you up in bed and putting an ice pack on the incision to see if that helps.” ANS: D
Canadian Fundamentals of Nursing 6th Edition Potter Test Bank The patient is responding well to the oral pain medication, and it can take longer for oral medications to relieve pain. Trying nonpharmacological interventions as an addition to opioid medications is appropriate at this time. If nonpharmacological interventions combined with the oral opioid are ineffective, the nurse needs to notify the health care provider and ask for a change in the medication or for additional pain medication. Saying that the patient has to wait 4 hours for additional pain medication is inaccurate because the nurse needs to provide further nursing interventions if pain is not relieved to an acceptable level for the patient. Admission to an intensive care unit is not typically necessary to manage pain after surgery for a hernia. DIF: Apply REF: 598 OBJ: Describe guidelines for selecting and individualizing pain interventions. TOP: Implementation MSC: NCLEX: Health Promotion and Maintenance 32. Which of the following is the best way for the nurse to manage pain for a patient with chronic
pain from arthritis? a. Administer pain medication before any activity. b. Provide intravascular bolus as needed for breakthrough pain. c. Give medications around-the-clock. d. Administer pain medication only when nonpharmacological measures have failed. ANS: C
When a patient with arthritis has chronic pain, the best way to manage pain is to take medication regularly throughout the day to maintain constant pain relief. “Before activity” is nonspecific, and the medication may not have time to work before activity. If the patient waits until having pain to take the medication, pain relief takes longer. Nonpharmacological measures are used in conjunction with medications unless requested otherwise by the patient. DIF: Understand REF: 57N 9|U5R 98SINGTB.COM OBJ: Describe guidelines for selecting and individualizing pain interventions. TOP: Implementation MSC: NCLEX: Health Promotion and Maintenance 33. A nurse is caring for a patient who fell on the ice and has connective tissue damage in the
wrist and hand. What type of pain is the patient experiencing? a. Visceral pain. b. Somatic pain. c. Peripherally generated pain. d. Centrally generated pain. ANS: B
Somatic pain comes from bone, joint, or muscle. Visceral pain arises from the visceral organs such as the gastrointestinal tract and pancreas. Peripherally generated pain can be caused by polyneuropathies or mononeuropathies. Centrally generated pain results from injury to the central or peripheral nervous system. DIF: Remember REF: 579| 580 OBJ: Perform an assessment of a patient experiencing pain. MSC: NCLEX: Physiological Integrity
TOP: Assessment
34. The nurse is caring for an infant in the intensive care unit. Which of the following is the most
accurate description of factors that will influence the perception and management of pain for this patient?
Canadian Fundamentals of Nursing 6th Edition Potter Test Bank a. b. c. d.
Infants cannot tolerate analgesics owing to an underdeveloped metabolism. Infants have an increased sensitivity to pain in comparison with older children. Pain cannot be accurately assessed in infants. Infants respond behaviourally and physiologically to painful stimuli.
ANS: D
Infants cannot verbally express their pain, but they do express pain with behavioural cues and physiological indicators. Infants can tolerate analgesics, but proper dosing and close monitoring are essential. Infants and older children have the same sensitivity to pain. Pain can be assessed even though the neonate cannot verbalize; the nurse can observe behavioural clues. Nurses use behavioural cues and physiological responses to assess pain in infants. DIF: Understand TOP: Assessment
REF: 581| 589 OBJ: Identify components of the pain experience. MSC: NCLEX: Physiological Integrity
MULTIPLE RESPONSE 1. The nurse is administering ibuprofen (Advil) to an older patient. Which of the following
assessment data causes the nurse to withhold the medication? (Select all that apply.) a. Past medical history of gastric ulcer. b. Patient states last bowel movement was 4 days ago. c. Stated allergy to aspirin. d. Patient states has intermittent joint pain rated 2 on a 0-to-10 scale. e. Patient experienced respiratory depression after administration of an opioid medication. ANS: A, C
NURSINGTB.COM
NSAIDs can cause bleeding, especially in the gastrointestinal (GI) tract; therefore, NSAIDs are probably contraindicated in this patient. Some patients with an allergy to aspirin are also allergic to other NSAIDs. The nurse needs to verify that the health care provider is aware of the history of GI bleeding and of allergy to aspirin before administering ibuprofen. NSAIDs do not interfere with bowel function and are used for the treatment of mild to moderate acute intermittent pain. NSAIDs also do not suppress the central nervous system. DIF: Understand REF: 597 OBJ: Explain the various pharmacological approaches to treating pain. TOP: Assessment MSC: NCLEX: Physiological Integrity
Canadian Fundamentals of Nursing 6th Edition Potter Test Bank
Chapter 32: Health Assessment and Physical Examination Potter et al: Canadian Fundamentals of Nursing, 6th Edition MULTIPLE CHOICE 1. A nurse is a preceptor for a nurse who just graduated from nursing school. When caring for a
patient, the new graduate nurse begins to explain to the patient the purpose of completing a physical assessment. Which of the following statements made by the new graduate nurse necessitates intervention by the preceptor? a. “I will use the information from my assessment to figure out if your antihypertensive medication is working effectively.” b. “Nursing assessment data are used only to provide information about the effectiveness of your medical care.” c. “Nurses use data from their patient’s physical assessment to determine a patient’s educational needs.” d. “Information gained from physical assessment helps nurses better understand their patients’ emotional needs.” ANS: B
Nursing assessment data are used to evaluate the effectiveness of all aspects of a patient’s care, not just the patient’s medical care. Assessment data help to evaluate the effectiveness of medications and to determine a patient’s health care needs, including the need for patient education. Nurses also use assessment data to identify patients’ psychosocial and cultural needs. DIF: Evaluate REF: 61N 2 R I GOBB J:.DiC scM uss the purposes of physical assessment. U S N T O TOP: Communication and Documentation MSC: NCLEX: Health Promotion and Maintenance 2. For a weak patient with bilateral basilar pneumonia, which is the best position for a complete
geriatric physical examination? a. Prone position. b. Sims’s position. c. Supine position. d. Lateral recumbent. ANS: C
Supine position is the most normally relaxed position. It will not further compromise the patient’s breathing. If the patient becomes short of breath easily, the head of the bed can be raised. This position would be easiest for a weak older person to get into for an examination. Lateral recumbent and prone positions cause respiratory difficulty for any patient with respiratory difficulties. Sims’s position is used for assessment of the rectum and the vagina. DIF: Understand REF: 617 (Table 32-3) OBJ: List techniques for preparing a patient physically and psychologically before and during an examination. TOP: Planning MSC: NCLEX: Health Promotion and Maintenance
Canadian Fundamentals of Nursing 6th Edition Potter Test Bank 3. During an annual gynecological examination, a college student discusses her upcoming
college break at a tropical location. After the student receives an oral contraceptive prescription, the nurse identifies the importance of skin cancer prevention by discussing which evidence-informed prevention technique? a. Applying water-based sunscreen only before swimming. b. Using tanning bed daily for 7 days before college break trip. c. Applying broad-spectrum sunscreen of SPF 5. d. Taking extra precautions in the sun secondary to the prescription. ANS: D
Oral contraceptives can make the skin more sensitive to the sun. For this reason, the patient should be educated about the need for sun protection with such techniques as the use of wide-brimmed hats, use of broad-spectrum sunscreen of SPF 15 or higher, not tanning during midday, and not using tanning beds. Broad-spectrum sunscreens should be applied 15 minutes before a person goes out into the sun and after swimming or perspiring. DIF: Understand REF: 628 (Box 32-9) OBJ: Discuss ways to incorporate health promotion and health teaching into the examination. TOP: Planning MSC: NCLEX: Health Promotion and Maintenance 4. A head and neck physical examination is completed on a 50-year-old woman. All physical
findings are normal except that she has fine, brittle hair. On the basis of the physical findings, which of the following laboratory tests would the nurse expect to be ordered? a. Liver function test. b. Lead level. c. Thyroid-stimulating hormone test. d. Complete blood cell count (CBC). G B.C M ANS: C
N R I U S N T
O
Thyroid disease can make hair thin and brittle. Liver function testing is indicated for a patient who has jaundice. Lead levels and a CBC are not indicated for the finding of brittle hair. DIF: Understand REF: 627 OBJ: Identify how nurses use physical assessment skills during routine nursing care. TOP: Planning MSC: NCLEX: Physiological Integrity 5. A febrile preschool-aged child presents to the after-hours clinic. Varicella is diagnosed on the
basis of the illness history and the presence of small, circumscribed skin lesions filled with serous fluid. The nurse documents the varicella lesions as which type of skin lesion? a. Vesicle. b. Wheal. c. Papule. d. Pustule. ANS: A
Vesicles are circumscribed, elevated skin lesions filled with serous fluid that are smaller than 1 cm in diameter. Wheals are irregularly shaped, elevated areas of superficial localized edema that vary in size. They are common with bug bites and hives. Papules are palpable, circumscribed, solid elevations in the skin that are smaller than 1 cm in diameter. Pustules are elevations of skin similar to vesicles, but they are filled with pus. DIF: Understand
REF: 626 (Box 32-7)
Canadian Fundamentals of Nursing 6th Edition Potter Test Bank OBJ: Describe physical measurements made in assessing each body system. TOP: Assessment MSC: NCLEX: Physiological Integrity 6. A school nurse recognizes a belt buckle–shaped ecchymosis on a 7-year-old student. When
privately asked about how the injury occurred, the student described falling on the playground. Upon suspecting abuse, the school nurse’s best next action is which of the following? a. Interviewing the patient in the presence of his or her teacher. b. Ignoring the findings because child abuse is a declining problem. c. Realizing that abuse victims usually report abusive situations. d. Contacting Social Services and reporting suspected abuse. ANS: D
Most provinces and territories mandate a report to a social service centre if nurses suspect abuse or neglect. When abuse is suspected, the nurse interviews the patient in private. Abuse of children, women, and older persons is a growing health problem. It is difficult to detect abuse because many victims do not complain or report that they are in an abusive situation. DIF: Apply REF: 620 OBJ: Identify how nurses use physical assessment skills during routine nursing care. TOP: Implementation MSC: NCLEX: Psychosocial Integrity 7. A nurse identifies Pediculosis humanus capitis. Considering the possible complications of
treatment, the nurse knows to not use which of the following treatment products? a. Fine-toothed comb. b. Pediculicide. c. Lindane-based shampoo. NURSINGTB.COM d. Vinegar hair rinse. ANS: C
Products containing lindane, a toxic ingredient, often cause adverse reactions and neurotoxic effects. Patients who have head lice are instructed to shampoo thoroughly with pediculicide (shampoo available at drugstores) in cold water, comb thoroughly with a fine-toothed comb, and discard the comb. A dilute solution of vinegar and water helps loosen nits. DIF: Apply REF: 629 (Box 32-10) OBJ: Describe physical measurements made in assessing each body system. TOP: Implementation MSC: NCLEX: Health Promotion and Maintenance 8. A parent calls the school nurse with questions regarding the recent school vision screening.
Snellen chart examination revealed 20/60 for both eyes in the child. Considering the visual acuity results, what does the nurse inform the parent? a. That the child should have an optometric examination. b. That the child is suffering from strabismus. c. That the child may have presbyopia. d. That the child has vision issues most likely due to cataracts. ANS: A
Canadian Fundamentals of Nursing 6th Edition Potter Test Bank Normal vision is 20/20. The larger the denominator, the poorer the patient’s visual acuity. For example, a value of 20/60 means that the patient, when standing 20 feet away, can read a line that a person with normal vision can read from 60 feet away. Strabismus is a (congenital) condition in which both eyes do not focus on an object simultaneously: these eyes appear crossed. Acuity may not be affected. Presbyopia is impaired near vision that occurs in middle-aged and older persons and is caused by loss of elasticity of the lens. Cataracts develop slowly and progressively after age 35 or suddenly after trauma. DIF: Apply REF: 634 OBJ: Identify preventive screenings and the appropriate age(s) for each screening to occur. TOP: Implementation MSC: NCLEX: Health Promotion and Maintenance 9. During a routine pediatric history documentation and physical examination, the parents report
that their child was a premature infant and was so small that he had to stay in the neonatal intensive care unit longer than usual. They state that the infant was yellow when born and that he developed an infection that required “every antibiotic under the sun” to cure him. Considering the neonatal history, the nurse determines that it is especially important to perform which focused examination? a. Cardiac. b. Respiratory. c. Ophthalmic. d. Hearing acuity. ANS: D
Risk factors for hearing problems include low birth weight, nonbacterial intrauterine infection, and excessively high bilirubin levels. Hearing loss due to ototoxicity (injury to auditory nerves) can result from high maintenance doses of antibiotics. Cardiac, respiratory, and eye N essm RSen INtsGbT B.CotMrelevant to this case. examinations are important assU ut are nO DIF: Apply REF: 636 (Table 32-13) OBJ: Identify data to collect from the nursing history before an examination. TOP: Implementation MSC: NCLEX: Physiological Integrity 10. During a presentation about sexually transmitted infections to high school students, the nurse
recommends the human papillomavirus (HPV) vaccine series to prevent which of the following? a. Cervical cancer. b. Genital lesions. c. Vaginal discharge. d. Swollen perianal tissues. ANS: A
HPV infection increases the risk for cervical cancer. HPV vaccine is recommended by the Public Health Agency of Canada for female patients aged 9 to 26 years. Vaginal discharge, painful or swollen perianal tissues, and genital lesions are signs and symptoms that may indicate a sexually transmitted infection. DIF: Understand REF: 664 (Table 32-26) OBJ: Identify preventive screenings and the appropriate age(s) for each screening to occur. TOP: Implementation MSC: NCLEX: Health Promotion and Maintenance
Canadian Fundamentals of Nursing 6th Edition Potter Test Bank 11. A male student comes to the college health clinic. He hesitantly describes that his testis has a
pea-sized hard lump. The nurse recognizes this as a potential sign of which of the following? a. Inguinal hernia. b. Sexually transmitted infection. c. Testicular cancer. d. Diuretic use. ANS: C
Irregular lumps of the testes may indicate testicular cancer. Testicular cancer is cancer that begins in the testicles. Testicular cancer is the most common form of cancer in men between the ages of 15 and 35 years. A hernia manifests with bulging in the scrotum. Sexually transmitted infections often manifest with genital lesions. Use of diuretics, sedatives, or antihypertensives can cause difficulty in achieving an erection or ejaculation but does not usually cause lumps. DIF: Understand REF: 667 OBJ: Identify self-screening examinations commonly performed by patients. TOP: Implementation MSC: NCLEX: Physiological Integrity 12. The nurse is urgently called to the gymnasium regarding an injured student. The student is
crying in severe pain with a malformed fractured lower leg. What is the proper sequence for the nurse’s initial assessment? a. Deep palpation, light palpation, inspection. b. Light palpation, deep palpation, inspection. c. Inspection, light palpation. d. Auscultation, deep palpation, light palpation. ANS: C
NURSINGTB.COM
Inspection is the use of vision and hearing to distinguish normal from abnormal findings. Light palpation determines areas of tenderness and skin temperature, moisture, and texture. Deep palpation is used to examine the condition of organs, such as those in the abdomen; it must be done with caution and is done after light palpation. Auscultation is used to evaluate sound. DIF: Apply REF: 613| 614 OBJ: Demonstrate the techniques used with each physical assessment skill. TOP: Implementation MSC: NCLEX: Physiological Integrity 13. The nurse is caring for a female victim of rape. To perform the proper evaluation, the nurse
should place the patient in which of the following positions? a. Sitting. b. Dorsal recumbent. c. Lithotomy. d. Knee-chest. ANS: C
Canadian Fundamentals of Nursing 6th Edition Potter Test Bank Lithotomy is the position for examination of female genitalia. The lithotomy position provides for the maximum exposure of genitalia and allows the insertion of a vaginal speculum. Sitting does not allow adequate access for speculum insertion and is better used to visualize upper body parts. Dorsal recumbent is used to examine the head and neck, anterior thorax and lungs, breasts, axillae, heart, and abdomen. Knee-chest provides maximal exposure of the rectal area but is embarrassing and uncomfortable. DIF: Apply REF: 617 (Table 32-3) OBJ: List techniques for preparing a patient physically and psychologically before and during an examination. TOP: Implementation MSC: NCLEX: Safe and Effective Care Environment 14. On admission, a patient weighs 113.4 kg (250 pounds). The weight is recorded as 116.1 kg
(256 pounds) on the second inpatient day. The nurse should evaluate the patient for a. Fluid retention. b. Fluid loss. c. Decreased nutritional reserves. d. Anorexia. ANS: A
This patient has gained 2.7 kg (6 pounds) in a 24-hour period. A weight gain of 2.3 kg (5 pounds) or more in a day indicates fluid retention problems. A downward trend may indicate a reduction in nutritional reserves that may be caused by decreased intake such as anorexia or by fluid loss. DIF: Apply REF: 620 OBJ: Identify how nurses use physical assessment skills during routine nursing care. NURSINGMTSBC:.C TOP: Implementation NC OLMEX: Physiological Integrity 15. The patient is a 50-year-old African Canadian man who has come in for his routine annual
physical examination. Which of the following preventive screenings does the nurse recommend? a. Digital rectal examination of the prostate (DRE) annually. b. Blood test for cancer antigen 125 once a year. c. Complete eye examination every year. d. Colonoscopy every 3 years. ANS: A
Men need to have a digital rectal examination of the prostate every year beginning at 50 years of age. Ca-125 blood tests are indicated for women at high risk for ovarian cancer. Because this patient is a man, the cancer antigen 125 test is not needed. Patients older than 65 need to have complete eye examinations yearly. Colonoscopy every 10 years is recommended in patients 50 years of age and older. DIF: Apply REF: 619 (Table 32-4) OBJ: Discuss ways to incorporate health promotion and health teaching into the examination. TOP: Implementation MSC: NCLEX: Health Promotion and Maintenance 16. An older patient is being seen for a chronic entropion. The nurse realizes that entropion places
the patient at risk for which of the following? a. Ectropion.
Canadian Fundamentals of Nursing 6th Edition Potter Test Bank b. Infection. c. Exophthalmos. d. Strabismus. ANS: B
Entropion can cause the lashes of the lids to irritate the conjunctiva and cornea, and irritation can lead to infection. In ectropion, eyelid margins turn outward so that the lashes do not irritate the conjunctiva. Exophthalmos is a bulging of the eyes and usually indicates hyperthyroidism. Strabismus, or crossing of the eyes, results from neuromuscular injury or inherited abnormalities. DIF: Apply REF: 632 OBJ: Identify how nurses use physical assessment skills during routine nursing care. TOP: Implementation MSC: NCLEX: Physiological Integrity 17. During a school physical examination, the nurse reviews the patient’s current medical history,
which is positive for asthma, eczema, and allergic rhinitis. Which physical finding should the nurse expect on nasal examination? a. Polyp. b. Yellow discharge. c. Pale nasal mucosa. d. Puffiness of nasal mucosa. ANS: C
Pale nasal mucosa with clear mucoid discharge indicates allergic rhinitis. Polyps are tumour-like growths. Yellow discharge would be seen with infection. Habitual use of intranasal cocaine and opioids causes puffiness and increased vascularity of the nasal mucosa.
NURSINGTB.COM
DIF: Analyze REF: 637 OBJ: Identify data to collect from the nursing history before an examination. TOP: Assessment MSC: NCLEX: Physiological Integrity 18. Objective physical data describe air moving through small airways, auscultated over the
lung’s periphery. The expected inspiratory-to-expiratory phase of this normal vesicular breath sound is which of the following? a. The inspiratory phase lasts exactly as long as the expiratory phase. b. The expiration phase is longer than the inspiration phase. c. The expiration phase is two times longer than the inspiration phase. d. The inspiratory phase is three times longer than the expiratory phase. ANS: D
Vesicular breath sounds are normal breath sounds heard over the lung’s periphery, caused by air moving through smaller airways; the inspiratory phase is three times longer than the expiratory phase. In bronchovesicular breath sounds, the inspiratory phase is equally as long as the expiratory phase. In bronchial breath sounds, the expiration phase is longer than the inspiration phase, at a 3:2 ratio. DIF: Analyze REF: 646 (Table 32-18) OBJ: Discuss normal physical findings in a young, a middle-aged, and older persons. TOP: Assessment MSC: NCLEX: Physiological Integrity
Canadian Fundamentals of Nursing 6th Edition Potter Test Bank 19. A teenaged female patient reports intermittent abdominal pain that has lasted 12 hours. No
dysuria is present. During an abdominal assessment, what should the nurse do? a. Recommend that the patient take more laxatives. b. Ask the patient about the colour of her stools. c. Avoid sexual references such as possible pregnancy. d. Palpate first the spots that are most tender. ANS: B
Black or tarry stools (melena) indicate gastrointestinal alteration. The nurse should caution patients about the dangers of excessive use of laxatives or enemas. The nurse should also determine whether the patient is pregnant and note her last menstrual period. Pregnancy causes changes in abdominal shape and contour. Painful areas should be assessed last to minimize discomfort and anxiety. DIF: Apply REF: 661 (Table 32-25) OBJ: Discuss normal physical findings in a young, a middle-aged, and older persons. TOP: Implementation MSC: NCLEX: Physiological Integrity 20. During a genitourinary examination of a 30-year-old man, the nurse identifies a small amount
of a white, thick substance on the patient’s uncircumcised glans penis. What is the nurse’s next step? a. Notify his provider about a suspected sexually transmitted infection (STI). b. Recognize this as a normal finding. c. Tell the patient to avoid doing self-examinations until symptoms clear. d. Avoid embarrassing questions about sexual activity. ANS: B
A small amount of thick, whiN teUsR mS egIma Ces NGsom TBe.tim OMcollects under the foreskin of an uncircumcised male. Penile pain or swelling, genital lesions, and urethral discharge are signs and symptoms that may indicate STI. All men 15 years and older need to perform a male genital self-examination monthly. The nurse needs to assess a patient’s sexual history and use of safe sex habits. Sexual history reveals risks for STI and human immunodeficiency virus (HIV) infection. DIF: Apply REF: 666 OBJ: Discuss normal physical findings in a young, a middle-aged, and older persons. TOP: Implementation MSC: NCLEX: Physiological Integrity 21. In preparation for a rectal examination, a nonambulatory male patient is informed of the need
to be placed in which position? a. Sims’s position. b. Forward bending with flexed hips. c. Knee-chest. d. Dorsal recumbent. ANS: A
Nonambulatory patients are best examined in a side-lying Sims’s position. Forward bending would require the patient to be able to stand upright. Knees to chest would be difficult for a nonambulatory man to maintain and is embarrassing and uncomfortable. The dorsal recumbent position does not provide adequate access for a rectal examination and is used for abdominal assessment because it promotes relaxation of abdominal muscles.
Canadian Fundamentals of Nursing 6th Edition Potter Test Bank
DIF: Understand REF: 617 ( Table 32-3) OBJ: List techniques for preparing a patient physically and psychologically before and during an examination. TOP: Planning MSC: NCLEX: Physiological Integrity 22. A teenaged patient is tearful and reports locating lumps in her breasts. Other history obtained
is that she is currently menstruating. Physical examination reveals soft and movable cysts in both breasts that are painful to palpation. The nurse also notes that the patient’s nipples are erect, but the areolae are wrinkled. The next nursing step is which of the following? a. Reassure patient that her symptoms are normal. b. Consult a breast surgeon because of the abnormal nipples and areolae. c. Discuss fibrocystic disease as the likely cause. d. Tell the patient that the symptoms may get worse when her period ends. ANS: C
A common benign condition of the breast is benign (fibrocystic) breast disease. This patient has symptoms of fibrocystic disease, in which the breasts are bilaterally lumpy and painful, and sometimes nipple discharge is present. Symptoms are more apparent during the menstrual period. When palpated, the cysts (lumps) are soft, well differentiated, and movable. Deep cysts feel hard. Although it is a common condition, benign breast disease is not normal; therefore, the nurse does not tell the patient that this is a normal finding. During examination of the nipples and areolae, the nipple sometimes becomes erect with wrinkling of the areola. Therefore, consulting a breast surgeon to treat her nipples and areolae is not appropriate. DIF: Apply REF: 660 OBJ: Describe physical measurements made in assessing each body system. TOP: Assessment MSC: NCLEX: Physiological Integrity
NURSINGTB.COM
23. Asking a person what the statement “A stitch in time saves nine” means to him is a mental
status examination technique; what is it used to assess? a. Knowledge. b. Long-term memory. c. Abstract thinking. d. Recent memory. ANS: C
For an individual to explain common phrases such as “A stitch in time saves nine” requires a high level of intellectual function. Knowledge-based assessment is factual. The nurse assesses knowledge by asking how much the patient knows about his illness or the reason for seeking health care. To assess memory, the patient is asked to recall the maiden name of the patient’s mother, a birthday, or a special date in history. It is best to ask open-ended questions rather than simple yes-or-no questions. Patients demonstrate immediate recall by repeating a series of numbers in the order in which they are presented or in reverse order. DIF: Understand REF: 675 OBJ: Describe physical measurements made in assessing each body system. TOP: Planning MSC: NCLEX: Psychosocial Integrity 24. During a routine physical examination of a 70-year-old patient, a blowing sound is auscultated
over the carotid artery. The nurse notifies the medical provider of the unexpected physical finding, which is known as which of the following?
Canadian Fundamentals of Nursing 6th Edition Potter Test Bank a. b. c. d.
Clubbing. Bruit. Right-sided heart failure. Phlebitis.
ANS: B
A bruit is the sound of turbulence of blood passing through a narrowed blood vessel. A bruit can reflect cardiovascular disease in the carotid artery of middle-aged to older persons. Clubbing of the fingers is due to insufficient oxygenation at the periphery resulting from conditions such as chronic emphysema and congenital heart disease. Jugular venous distension, not bruit, is a possible sign of right-sided heart failure. In some patients with heart disease, the jugular veins are distended when the patients sit. Phlebitis is an inflammation of a vein that occurs commonly after trauma to the vessel wall, infection, immobilization, and prolonged insertion of intravenous catheters. It affects predominantly peripheral veins. DIF: Understand REF: 652 OBJ: Describe physical measurements made in assessing each body system. TOP: Planning MSC: NCLEX: Physiological Integrity 25. The nurse believes that several new female patients should receive additional health education
about the need for more frequent Papanicolaou (Pap) smears and gynecological examinations. Which of the following assessment findings reveals the patient at highest risk for cervical cancer and thus having the greatest need for patient education? a. 13 years old, nonsmoker, not sexually active b. 15 years old, social smoker, celibate c. 22 years old, smokes 1 pack of cigarettes per day, has multiple sexual partners d. 50 years old, stopped smoking 30 years ago, history of hysterectomy ANS: C
N R I G B.C M U S N T O
Female patients considered to be at higher risk for cervical cancer include those who smoke and are older than 21 with weak immune systems, multiple sex partners, and a history of sexually transmitted infections. Of all the assessment findings listed, those of the 22-year-old smoker with multiple sexual partners include the greatest number of risk factors for cervical cancer. The other patients are at lower risk. DIF: Apply REF: 664 (Table 32-26) OBJ: Discuss ways to incorporate health promotion and health teaching into the examination. TOP: Planning MSC: NCLEX: Physiological Integrity 26. The paramedics transport an adult involved in a motor vehicle accident to the emergency
department. On physical examination, the patient’s level of consciousness is reported as opening eyes to pain and responding to painful stimuli with inappropriate words and flexion withdrawal. The nurse correctly identifies the patient’s Glasgow Coma Scale score as a. 5. b. 7. c. 9. d. 11. ANS: C
Canadian Fundamentals of Nursing 6th Edition Potter Test Bank According to the guidelines of the Glasgow Coma Scale, the patient has a score of 9. Opening eyes to pain is 2 points; inappropriate word use is 3 points; and flexion withdrawal is 4 points. The total for this patient is 2 + 3 + 4 = 9. DIF: Apply REF: 674 (Table 32-33) OBJ: Identify how nurses use physical assessment skills during routine nursing care. TOP: Assessment MSC: NCLEX: Physiological Integrity 27. While assessing the skin of an 82-year-old male patient, a nurse discovers nonpainful ruby red
papules on the patient’s trunk. What is the nurse’s next action? a. Explain that the patient has basal cell carcinoma and should watch for spread. b. Document cherry angiomas as a normal geriatric skin finding. c. Tell the patient that he has a benign squamous cell carcinoma. d. Document the presence of edema. ANS: B
The skin is normally free of lesions, except for common freckles or age-related changes such as skin tags, senile keratosis (thickening of skin), cherry angiomas (ruby red papules), and atrophic warts. Basal cell carcinoma is most common in sun-exposed areas and frequently occurs in a background of sun-damaged skin; it almost never spreads to other parts of the body. Squamous cell carcinoma is more serious than basal cell and develops on the outer layers of sun-exposed skin; these cells may travel to lymph nodes and throughout the body. The nurse should report abnormal lesions to the health care provider for further examination. Edema is an area of skin that becomes swollen or edematous from a buildup of fluid in the tissues. This has nothing to do with cherry angiomas. DIF: Apply REF: 625 OBJ: Discuss normal physicalNfiU ndRinS gI s iN nG a yToB un. g,CaO mM iddle-aged, and older persons. TOP: Assessment MSC: NCLEX: Physiological Integrity 28. During a preschool readiness examination, the nurse prepares to perform visual acuity
screenings. Given the children’s age, the best equipment to test central vision is which of the following? a. Snellen test. b. E chart. c. Reading test. d. Penlight. ANS: B
The E chart is used when an individual is unable to read, as is often the case for preschool-aged children. A Snellen chart and a reading test are too advanced for a preschooler’s education level. A penlight is used to check light perception; it shines a light into the eye, and then it is turned off. If the patient notes when the light is turned on or off, light perception is intact. DIF: Apply REF: 634 OBJ: Identify preventive screenings and the appropriate age(s) for each screening to occur. TOP: Assessment MSC: NCLEX: Physiological Integrity 29. A nurse suspects an abnormal thyroid shape during the physical examination. Why does the
nurse offer the patient a glass of water and watch her as she drinks?
Canadian Fundamentals of Nursing 6th Edition Potter Test Bank a. b. c. d.
To visualize an enlarged thyroid gland. To evaluate for exostosis. To test the patient’s gag reflex. To visualize the uvula and soft palate.
ANS: A
This technique is used to visualize an abnormally large thyroid gland. Normally, the thyroid cannot be visualized. An exostosis is a bony growth between the two palates that is noted when the oral cavity is examined. The patient’s gag reflex is tested by placing a tongue depressor on the posterior tongue. The uvula and soft palate are visualized with the use of a penlight. Both structures should rise centrally as the patient says, “Ah.” DIF: Understand REF: 642 OBJ: Demonstrate the techniques used with each physical assessment skill. TOP: Assessment MSC: NCLEX: Physiological Integrity 30. The patient is a 54-year-old man with a medium frame. He weighs 67.1 kg (148 pounds) and
is 173 cm (5 feet 8 inches) tall, with a body mass index (BMI) of 22 kg/m2. The nurse realizes that this patient is which of the following? a. Overweight. b. Underweight. c. At his desired weight. d. Obese. ANS: C
A normal BMI is 18.5 to 24.9 kg/m2. This patient is at his desired weight. He is not overweight, underweight, or obese.
NURSINGTB.COM
DIF: Remember REF: 622 (Figure 32-1) OBJ: Discuss normal physical findings in a young, a middle-aged, and older persons. TOP: Assessment MSC: NCLEX: Physiological Integrity 31. A patient in the emergency department is complaining of left lower abdominal pain. The
comprehensive abdominal examination would include, in proper order, which of the following? a. Inspection, palpation, auscultation. b. Percussion, inspection, auscultation. c. Inspection, palpation, percussion. d. Inspection, auscultation, palpation. ANS: D
The order of an abdominal examination differs slightly from that of other assessments. It begins with inspection, followed by auscultation. Performing auscultation before palpation lessens the chance of altering the frequency and character of bowel sounds. DIF: Apply REF: 660| 661 OBJ: Demonstrate the techniques used with each physical assessment skill. TOP: Assessment MSC: NCLEX: Physiological Integrity 32. What is the best term for breath sounds created by air moving through large lung airways? a. Bronchovesicular. b. Rhonchi.
Canadian Fundamentals of Nursing 6th Edition Potter Test Bank c. Bronchial. d. Vesicular. ANS: A
Bronchovesicular breath sounds are created by air moving through large airways. Vesicular sounds are created by air moving through smaller airways. Bronchial sounds are created by air moving through the trachea close to the chest wall. Rhonchi are abnormal lung sounds that are loud, low-pitched, rumbling coarse sounds heard during inspiration or expiration that sometimes clear by coughing. DIF: Remember REF: 646 (Table 32-18) OBJ: Describe physical measurements made in assessing each body system. TOP: Assessment MSC: NCLEX: Physiological Integrity 33. The patient presents to the clinic with dysuria and hematuria. How does the nurse proceed to
assess for kidney inflammation? a. Lightly palpating each abdominal quadrant. b. Inspecting abdomen for abnormal movement or shadows, with indirect lighting. c. Using deep palpation posteriorly. d. Percussing posteriorly the costovertebral angle at the scapular line. ANS: D
With the patient sitting or standing erect, direct or indirect percussion is used to assess for kidney inflammation. With the ulnar surface of the partially closed fist, the nurse percusses posteriorly the patient’s costovertebral angle at the scapular line. If the kidneys are inflamed, the patient feels tenderness during percussion. A systematic palpation approach for each quadrant of the abdomen is used to assess for muscular resistance, distension, abdominal tenderness, and superficial orN gan orIma sses LiC ght URs S NG TB. . OMpalpation would not reveal kidney tenderness because the kidneys sit deep within the abdominal cavity. The lower ribs and heavy back muscles protect the kidneys, so they cannot be palpated posteriorly. Kidney inflammation does not cause abdominal movement; however, to inspect the abdomen for abnormal movement or shadows, the nurse should stand on the patient’s right side and inspect from above the abdomen, using direct light over the abdomen. DIF: Apply REF: 662 OBJ: Demonstrate the techniques used with each physical assessment skill. TOP: Assessment MSC: NCLEX: Physiological Integrity 34. The advanced practice nurse is conducting a comprehensive eye examination on an
80-year-old African Canadian woman. Which of the following findings is an indication that the nurse should contact the patient’s physician for further examination? a. A thin white ring along the margin of the iris. b. A black pupil. c. Dilated pupils. d. A black fundus of the eye. ANS: C
Canadian Fundamentals of Nursing 6th Edition Potter Test Bank Dilation of the pupils results from glaucoma, trauma, neurological disorders, eye medication, or withdrawal from opioids. Shining a beam of light through the pupil and onto the retina stimulates the third cranial nerve and causes the muscles of the iris to constrict. Any abnormality along the nerve pathways from the retina to the iris alters the ability of the pupils to react to light. A thin white ring along the margin of the iris, called an arcus senilis, is common with aging but is abnormal in anyone younger than 40. The pupils are normally black, round, regular, and equal in size. The fundus of African Canadian patients can be black. DIF: Apply REF: 633 OBJ: Discuss cultural diversity, cultural competency, and cultural safety as these relate to the provision of culturally competent health and physical assessment and improved patient health outcomes. TOP: Assessment MSC: NCLEX: Physiological Integrity 35. An elderly patient has been taking high doses of antibiotics and is experiencing a sudden loss
of hearing. The nurse should contact the health care provider and do what else? a. Stop antibiotic use until the physician responds. b. Tell the patient that older patients often lose low-frequency hearing. c. Explain that hearing loss usually occurs with thinning of the eardrum. d. Assure the patient that rapid hearing loss is normal in the elderly. ANS: A
Older persons are especially at risk for hearing loss as a result of ototoxicity (injury to auditory nerve) caused by high maintenance doses of antibiotics (e.g., aminoglycosides). Continuation of the medications is a physician responsibility. Older persons experience an inability to hear high-frequency sounds and consonants. Deterioration of the cochlea and thickening of the tympanic membrane cause older persons to lose hearing acuity gradually. DIF: Apply REF: 63N 6URSINGTB.COM OBJ: Identify how nurses use physical assessment skills during routine nursing care. TOP: Assessment MSC: NCLEX: Physiological Integrity 36. The patient has had a stroke that has affected her ability to speak, and she becomes extremely
frustrated when she tries to speak. She responds correctly to questions and instructions but cannot form words coherently. This patient is showing signs of which type of aphasia? a. Expressive. b. Receptive. c. Sensory. d. Combination. ANS: A
The two types of aphasia are sensory (or receptive) and motor (or expressive). The patient cannot express herself in words and is showing signs of expressive aphasia. She responds correctly to questions and instructions, indicating that she does not have receptive or sensory aphasia. Patients sometimes suffer a combination of receptive and expressive aphasia, but this is not the case here. DIF: Understand REF: 675 OBJ: Describe physical measurements made in assessing each body system. TOP: Assessment MSC: NCLEX: Physiological Integrity
Canadian Fundamentals of Nursing 6th Edition Potter Test Bank 37. The school nurse is assessing the tympanic membranes of a 3-year-old. Which of the
following is a normal assessment finding? a. Presence of a yellow waxy substance. b. Swollen meatus. c. Discomfort with palpation of the auricle. d. Tenderness in the mastoid area. ANS: A
A yellow, waxy substance called cerumen is common. Tenderness in the mastoid area indicates mastoiditis. The nurse should inspect the opening of the ear canal for size and presence of discharge. If discharge is present, the nurse should wear clean gloves during the examination. Swelling or occlusion of the meatus is not normal. Yellow or green, foul-smelling discharge indicates the presence of infection or a foreign body. DIF: Apply REF: 635| 636 OBJ: Discuss normal physical findings in a young, a middle-aged, and older persons. TOP: Assessment MSC: NCLEX: Physiological Integrity
NURSINGTB.COM
Canadian Fundamentals of Nursing 6th Edition Potter Test Bank
Chapter 33: Infection Control Potter et al: Canadian Fundamentals of Nursing, 6th Edition MULTIPLE CHOICE 1. The patient and the nurse are discussing Rickettsia rickettsii, the cause of Rocky Mountain
spotted fever. Which patient statement to the nurse indicates understanding regarding the mode of transmission of this disease? a. “When I go camping, I will be sure to wear sunscreen.” b. “When I go camping, I will drink bottled water.” c. “When I go camping, I will be sure to wear insect repellent.” d. “When I go camping, I will be sure to use hand gel on my hands.” ANS: C
Each infectious disease has a specific mode of transmission: a component of the chain of infection. Rocky Mountain spotted fever is caused by bacteria transmitted by the bite of ticks. Wearing a repellent that is designed for repelling ticks, mosquitoes, and other insects can help in preventing transmission of this disease. Drinking plenty of uncontaminated water, wearing sunscreen, and using alcohol-based hand gels for cleaning hands are all important activities to participate in while camping, but they do not contribute to or prevent transmission of this disease. DIF: Understand REF: 685 OBJ: Explain the relationship between the chain of infection and transmission of infection. TOP: Evaluate MSC: NCLEX: Health Promotion and Maintenance
NcUatR 2. The nurse is providing an edu ioS naIl N seG ssT ioB n. foCr O aM group of preschool workers. The nurse reminds the group that the most important thing to do to prevent the spread of infection is what? a. Encourage preschool children to eat a nutritious diet. b. Encourage parents to provide a multivitamin to the children. c. Clean the toys every afternoon before putting them away. d. Perform hand hygiene between each interaction with children. ANS: D
The most important thing that individuals can do to prevent the spread of infection is to perform hand hygiene before and after eating, going to the bathroom, changing a diaper, and wiping a nose, as well as after cleaning toys or tables, after picking up after the children, and between touching each individual child. It is important for preschool children to have a nutritious diet; a healthy individual can fight infection more effectively. A physician, along with the parent, makes decisions about dietary supplements. Cleaning the toys can decrease the number of pathogens but is not the most important way to prevent the spread of infection. DIF: Remember REF: 700 OBJ: Give an example of preventing infection for each element of the infection chain. TOP: Implementation MSC: NCLEX: Health Promotion and Maintenance 3. The nurse is admitting a patient with an infectious disease process. What question would be
appropriate for a nurse to ask this patient? a. “Do you have a chronic disease, and how long have you had it?”
Canadian Fundamentals of Nursing 6th Edition Potter Test Bank b. “Do you have any children living in the home?” c. “What is your marital status—single, married, or divorced?” d. “Do you have any cultural or religious beliefs that will influence your care?” ANS: A
Some factors increase the susceptibility of an individual to infection. These include age, nutritional status, presence of chronic disease, trauma, and smoking. The other questions are part of an admission assessment process but are not pertinent to the infectious disease process. DIF: Understand REF: 692| 693 OBJ: Give an example of preventing infection for each element of the infection chain. TOP: Assessment MSC: NCLEX: Health Promotion and Maintenance 4. The patient underwent a surgical procedure, and povidone-iodine (Betadine) was utilized in
the surgical preparation. Two days postoperatively, the nurse’s assessment indicates that the incision is red and has a small amount of purulent drainage. The patient reports tenderness at the incision site. The patient’s temperature is 38.1°C (100.5°F) and the white blood cell count (WBC) is 10,500/mm3 (10.5 109/L). Which nursing action should the nurse take? a. Plan to change the surgical dressing during the shift. b. Check to see what solution was used for skin preparation in surgery. c. Collect supplies to culture the surgical incision. d. Communicate the patient’s condition to the health care provider. ANS: D
Organisms enter the body in several different ways. Proper skin preparation for surgery is essential for decreasing the chance of infection. The nursing assessment indicates signs and symptoms of infection, and the health care provider should be notified. Changing the dressing may be a need during the shiN ft U buRt S isInN otGaTfiB rs. t pCrO ioM rity. Checking to see about the skin preparation used 2 days ago may or may not be useful information at this time. Collecting supplies for culture may be necessary after the nurse talks to the physician. DIF: Apply REF: 691| 712 OBJ: Give an example of preventing infection for each element of the infection chain. TOP: Implementation MSC: NCLEX: Physiological Integrity 5. The nurse is providing an education session to an adult community group about the effects of
smoking. Which of the following is the most important point to be included in the educational session? a. Smoke from tobacco products clings to your clothing and hair. b. Smoking affects the cilia lining the upper airways in the lungs. c. Smoking tobacco products can be very expensive. d. Smoking can affect the colour of the patient’s fingernails. ANS: B
A normal defence mechanism against infection in the respiratory tract is the cilia lining the upper airways of the lungs and normal mucus. When a patient inhales a microbe, the cilia and mucus trap the microbe and sweep them up and out to be expectorated or swallowed. Smoking may alter this defence mechanism and increase the patient’s potential for infection. Smoking can be expensive, the smell does cling to hair and clothing, and the tar within the smoke can alter the colour of a patient’s nails. These points can be included in the education but do not constitute the most important information.
Canadian Fundamentals of Nursing 6th Edition Potter Test Bank
DIF: Understand REF: 688 OBJ: Identify the body’s normal defences against infection. MSC: NCLEX: Physiological Integrity
TOP: Implementation
6. A woman presents to the clinic with reports of a white discharge and itching in the vaginal
area. During the health history, which of these questions should the nurse prioritize? a. “When was the last time you visited the physician?” b. “Has this condition affected your eating habits?” c. “What medications are you currently taking?” d. “Are you able to sleep at night?” ANS: C
The body contains normal flora (microorganisms) that live on the surface of skin, saliva, oral mucosa, gastrointestinal tract, and genitourinary tract. The normal flora of the vagina causes vaginal secretions to achieve a low pH. This inhibits the growth of many microorganisms. Antibiotics and oral contraceptives can disrupt normal flora in the vagina, which can lead to an overgrowth of Candida albicans in that area. It is important to ask the patient about current medications to obtain information that may assist with diagnosis. Visiting the physician is important for the patient’s health maintenance. Learning about the patient’s eating and sleeping habits will assist in the plan of care. DIF: Apply REF: 688 OBJ: Identify the body’s normal defences against infection. MSC: NCLEX: Physiological Integrity
TOP: Assessment
7. The nurse is caring for a school-aged child who has injured his leg after a bicycle accident. To
determine whether the child iN sU exRpS erI ieN ncG inTgBa. loC caOliM zed inflammatory response, the nurse should assess for which of these signs and symptoms? a. Fever, malaise, anorexia, and nausea and vomiting. b. Chest pain, shortness of breath, and nausea and vomiting. c. Dizziness and disorientation to time, date, and place. d. Edema, redness, tenderness, and loss of function. ANS: D
The body’s cellular response to an injury is seen as inflammation. Inflammation can be triggered by physical agents, chemical agents, or microorganisms. Signs of localized inflammation include swelling, redness, heat, pain or tenderness, and loss of function in the affected body part. Systemic signs of inflammation include fever, malaise, and anorexia, as well as nausea and vomiting. Chest pain, shortness of breath, and nausea and vomiting are signs and symptoms of a cardiac alteration. Dizziness and disorientation to time, date, and place may indicate a neurologic alteration. DIF: Remember TOP: Assessment
REF: 688| 689 OBJ: Discuss the events in the inflammatory response. MSC: NCLEX: Physiological Integrity
8. Which interventions utilized by the nurse would indicate the ability to recognize the
inflammatory response? a. Rest, ice, compression, and elevation. b. Turn, cough, and deep breathe. c. Orient to date, time, and place.
Canadian Fundamentals of Nursing 6th Edition Potter Test Bank d. Passive range-of-motion exercises. ANS: A
One sign of the inflammatory response, particularly after an injury, is swelling or edema. Resting the affected injured area, applying ice as ordered, wrapping the area to provide support—particularly if it is an extremity—and elevating the injured area will help decrease swelling or edema. Turning, coughing, and deep breathing are utilized for postoperative patients and for immobilized patients to help prevent an infectious process such as pneumonia. Orientation to date, time, and place is assessed in many patients who may be confused. Passive range of motion is utilized for individuals who need to improve movement of their extremities, including immobilized patients. DIF: Understand REF: 689 TOP: Implementation
OBJ: Discuss the events in the inflammatory response. MSC: NCLEX: Physiological Integrity
9. The nurse is caring for a group of medical-surgical patients. Which patient is most at risk for
developing an infection? a. The patient who is in observation for chest pain. b. The patient who is recovering from a right total hip arthroplasty. c. The patient who has been admitted with dehydration. d. The patient who has been admitted for stabilization of atrial fibrillation. ANS: B
The patient who is recovering from a right total hip arthroplasty has had a surgical procedure wherein bone was removed from the body and an implant was placed within the patient. The patient has a large incision from surgery. The patient also has an intravenous infusion to provide fluids and medication. All these breaks in the skin increase the likelihood of infection. The patient has been given anNaU esR thS etI icN sG anT dB m.eC diO caM tion for pain. Both of these depress the respiratory system and have the potential to decrease the expansion of alveoli and to increase the chance of infection in the respiratory system. The other patients may have one break in the skin when an intravenous infusion is used. DIF: Apply TOP: Assessment
REF: 688| 692 OBJ: Identify patients most at risk for infection. MSC: NCLEX: Physiological Integrity
10. The nurse is caring for a patient with leukemia and is preparing to provide fluids through a
vascular access device. Which nursing intervention is priority in this procedure? a. Positioning the patient comfortably. b. Maintaining aseptic technique. c. Gathering available supplies. d. Reviewing the procedure with the patient. ANS: B
Patients with disease processes of the immune system are at particular risk for infection. These diseases include leukemia, acquired immune deficiency syndrome (AIDS), lymphoma, and aplastic anemia. These disease processes weaken the defences against an infectious organism. Any time an intravenous device is accessed, aseptic technique must be maintained with wearing of appropriate personal protective equipment, preparation of the skin, and use of sterile gloves, sterile supplies, appropriate flushing, and appropriate discontinuation. Reviewing the procedure with the patient, positioning the patient, and gathering the supplies are all important steps in the procedure but are not the priority in the procedure.
Canadian Fundamentals of Nursing 6th Edition Potter Test Bank
DIF: Apply REF: 692| 693 TOP: Implementation
OBJ: Identify patients most at risk for infection. MSC: NCLEX: Physiological Integrity
11. The nurse is caring for an adult patient in the clinic who is a victim of flooding and has been
evacuated. The patient presents with signs and symptoms of a urinary tract infection. Along with needed education surrounding this diagnosis, the nurse teaches the patient about rest, exercise, eating properly, and how to utilize deep breathing and visualization. Which of the following explanations would best support these nursing interventions? a. Urinary tract infections are painful, and these techniques would help with managing the pain. b. Interventions listed are standard topics taught during health care visits. c. Stress for long periods of time can lead to exhaustion and decreased resistance to infection. d. The patient requested this information to teach to extended family at home. ANS: C
The body responds to emotional or physical stress by the general adaptation syndrome. If stress extends for long periods of time, this can lead to exhaustion, whereby energy stores are depleted and the body has no defences against invading organisms. Techniques of deep breathing and visualization may be helpful with pain, but none of the interventions listed is a standard intervention taught at every health care visit. DIF: Analyze TOP: Evaluate
REF: 692 OBJ: Identify patients most at risk for infection. MSC: NCLEX: Health Promotion and Maintenance
12. The nurse is caring for a patient who is susceptible to infection. Which of the following
nursing interventions will assN isU t iR nS dI ecN reG asT inB g. thCeO riM sk of infection? a. Teaching the patient about fall prevention. b. Teaching the patient to select nutritious foods. c. Teaching the patient to measure temperature. d. Teaching the patient about the effects of alcohol. ANS: B
When protein intake is inadequate as a result of poor diet, the rate of protein breakdown exceeds that of tissue synthesis. A reduction in the intake of protein and other nutrients such as carbohydrates and fats reduces the body’s defences against infection and impairs wound healing. Teaching the patient about fall prevention, how to measure temperature, or about the effects of alcohol does not decrease the risk of infection. DIF: Apply REF: 692 TOP: Implementation
OBJ: Identify patients most at risk for infection. MSC: NCLEX: Health Promotion and Maintenance
13. A patient with diabetes presents to the clinic for a dressing change. The wound is located on
the right foot and has purulent yellow drainage. Which of these interventions would be most appropriate for the nurse to provide? a. Position the patient comfortably on the stretcher. b. Explain the procedure for dressing change to the patient. c. Don gloves and other appropriate personal protective equipment. d. Review the medication list that the patient brought from home. ANS: C
Canadian Fundamentals of Nursing 6th Edition Potter Test Bank Localized infections are most common in the skin or with mucous membrane breakdown. The nurse should wear gloves and other personal protective equipment as appropriate when examining or providing treatment to localized infected areas. Positioning the patient, explaining the procedure, and reviewing the medication list are all tasks that need to be completed, but preventing the spread of infection takes precedence. DIF: Apply REF: 687 OBJ: Describe the signs and symptoms of a localized and a systemic infection. TOP: Implementation MSC: NCLEX: Physiological Integrity 14. Which of these interventions would take priority and should be included in a plan of care for a
patient who presents with pneumonia? a. Observe the patient for decreased activity tolerance. b. Assume that the patient is in pain and treat accordingly. c. Maintain the ambient temperature at 18.3°C (65°F). d. Provide the patient ice chips as requested. ANS: A
Systemic infection causes more generalized symptoms than does local infection. This type of infection can result in fever, fatigue, nausea and vomiting, and malaise. The nurse should be alert for changes in the patient’s level of activity and responsiveness. Respiratory infection may result in a productive cough with purulent sputum, shortness of breath, and activity intolerance. Nurses do not assume but assess and communicate with the patient about pain, temperature, and ice chips. Asking these questions would not be a priority as much as assessing the patient and determining the effect that the systemic infection is having on the patient. DIF: Understand REF: 69N 4URSINGTB.COM OBJ: Describe the signs and symptoms of a localized and a systemic infection. TOP: Implementation MSC: NCLEX: Physiological Integrity 15. The nurse is inserting a peripherally inserted central catheter (PICC) into the patient. Aware of
the potential for health care–associated infection, the nurse is careful to do which of the following? a. Prepare the skin with 2% chlorhexidine gluconate. b. Select a catheter of appropriate size for the appropriate vein. c. Use nonallergenic tape and dressings on the patient. d. Utilize local anaesthetic on the site as ordered. ANS: A
One of the sites for health care–associated infection is the bloodstream. Bloodstream infection can be caused by improper care of the needle insertion site. Two percent chlorhexidine gluconate is an antiseptic solution that, when applied properly and allowed to dry, reduces microbial counts at the insertion site. Selecting the correct catheter size, using nonallergenic tape and dressings, and utilizing local anaesthetic are important steps for individualized patient care and are typically part of the procedure, but they do not affect the cause of a health care–associated infection by, for example, decreasing microbial counts at the insertion site. DIF: Apply REF: 691 (Box 33-3) OBJ: Explain conditions that promote the transmission of health care–associated infection. TOP: Implementation MSC: NCLEX: Physiological Integrity
Canadian Fundamentals of Nursing 6th Edition Potter Test Bank
16. The infection control nurse is reviewing data for the medical-surgical unit. The nurse notices a
spike in postoperative infections on this unit and categorizes this type of health care– associated infection as which infections? a. Iatrogenic. b. Exogenous. c. Endogenous. d. Nosocomial. ANS: B
An exogenous organism is one that is present outside the patient. A postoperative infection is an exogenous infection because the organism that has caused the infection originates from outside the body. An example is Staphylococcus aureus. An endogenous organism is part of the normal flora of residing virulent organisms that could cause infection. An endogenous infection can occur when part of the patient’s flora becomes altered, and overgrowth results. Iatrogenic infection results from a diagnostic or therapeutic procedure such as a colonoscopy. Nosocomial infection is the term formerly used for health care–acquired infection. DIF: Remember REF: 689 OBJ: Explain conditions that promote the transmission of health care–associated infection. TOP: Evaluate MSC: NCLEX: Physiological Integrity 17. Which of the following nursing actions would most increase a patient’s risk for developing a
health care–associated infection? a. Use of surgical aseptic technique to suction an airway. b. Placement of a urinary catheter drainage bag below the level of the bladder. c. Clean technique for inserting a urinary catheter. N tion RSmo INreGtThan B.C M d. Use of a sterile bottled soluU oncO e within a 24-hour period. ANS: C
Using clean technique (medical asepsis) to insert a urinary catheter would place the patient at risk for a health care–associated infection. Urinary catheters need to be inserted with sterile technique, also referred to as surgical asepsis. This involves eliminating all microorganisms, including pathogens and spores, from an object or area. Placing a catheter into a sterile body cavity such as the bladder necessitates sterile technique. Sterile technique should also be used in suctioning an airway because the airway is considered a sterile body cavity. Keeping the urinary catheter drainage bag below the bladder helps decrease the risk of developing a health care–associated infection because it prevents reflux of urine from the bag back into the bladder. Bottled solutions may be used repeatedly during a 24-hour period; however, special care is needed to ensure that the solution in the bottle remains sterile. After 24 hours, the solution should be discarded. DIF: Analyze REF: 697| 712 OBJ: Explain conditions that promote the transmission of health care–associated infection. TOP: Evaluate MSC: NCLEX: Safe and Effective Care Environment 18. The nurse is caring for a patient in the labour and delivery area. When an assessment of the
patient for dilatation and effacement is nearly completed, the electronic infusion device being used on the IV infusion sounds an alarm. Which of these actions is most appropriate for the nurse to take? a. Complete the assessment, remove gloves, and silence the alarm.
Canadian Fundamentals of Nursing 6th Edition Potter Test Bank b. Discontinue the assessment, and assess the intravenous infusion. c. Complete the assessment, remove gloves, wash hands, and assess the intravenous
infusion. d. Discontinue the assessment, remove gloves, use hand gel, and assess the
intravenous infusion. ANS: C
Medical asepsis or clean technique includes procedures to decrease the number of organisms present and to prevent the transfer of organisms. Wearing gloves while assessing the dilatation and effacement of a patient in labour, removing gloves, washing hands after contact with body fluids, and then assessing the IV infusion will assist in the prevention and transfer of any potential organisms to this intravenous line. Completing the assessment, removing gloves, and silencing the alarm leaves out the crucial step of decontaminating and washing the hands. Discontinuing the assessment and assessing the infusion leaves out removing the gloves and decontamination, as well as completing the assessment for the patient. Discontinuing the assessment, removing gloves, using hand gel, and assessing the infusion is incorrect because, upon exposure to body fluids, washing hands is appropriate. DIF: Understand REF: 697| 712 OBJ: Explain the difference between medical and surgical asepsis. TOP: Implementation MSC: NCLEX: Safe and Effective Care Environment 19. The nurse is dressed and is preparing to care for a patient in the perioperative area. The nurse
has scrubbed her hands and has donned a sterile gown and gloves. Which action would indicate a break in sterile technique? a. Touching protective eyewear. b. Standing with hands folded on chest. B.C M N R I G c. Accepting sterile supplies fU romStheNsurT geon. O d. Staying with the sterile table once it is open. ANS: A
Once a nurse is gowned and gloved with sterile gown and gloves, touching nonsterile protective eyewear would indicate a break in sterile technique. Sterile objects remain sterile only when touched by another sterile object. Standing with hands folded on chest is common practice and prevents arms and hands from touching unsterile objects. Accepting sterile supplies from the surgeon who has opened them with the appropriate technique is acceptable. Staying with a sterile table once opened is a common practice to ascertain that no one or nothing has contaminated the table. DIF: Understand REF: 712| 713 OBJ: Explain the difference between medical and surgical asepsis. TOP: Implementation MSC: NCLEX: Safe and Effective Care Environment 20. The nurse is caring for a patient with an incision. Which of the following actions would best
indicate an understanding of medical and surgical asepsis? a. Donning sterile gown and gloves to remove the wound dressing. b. Utilizing clean gloves to remove the dressing and using sterile supplies for the new dressing. c. Donning clean goggles, gown, and gloves to dress the wound. d. Utilizing clean gloves to remove the dressing and clean supplies for the new dressing.
Canadian Fundamentals of Nursing 6th Edition Potter Test Bank
ANS: B
The nurse should use clean gloves (medical asepsis) to remove contaminated dressings and should use sterile supplies, including gloves and dressings (surgical asepsis–sterile technique), to reapply sterile dressings. Wearing sterile gowns and gloves is not necessary for removing soiled dressings. Donning clean gloves to dress a sterile wound would contaminate the sterile supplies. Utilizing clean supplies for a sterile dressing would not help in decreasing the number of microbes at the incision site. DIF: Understand REF: 697| 712 OBJ: Explain the difference between medical and surgical asepsis. TOP: Implementation MSC: NCLEX: Safe and Effective Care Environment 21. The nurse is caring for a patient in the endoscopy area. The nurse observes the technician
performing these tasks. Which of these observations would necessitate intervention by the nurse? a. Washing hands after removing gloves. b. Placing the endoscope in a container for transfer. c. Removing gloves to transfer the endoscope. d. Disinfecting endoscopes in the workroom. ANS: C
Routine practices are used to prevent and control the spread of infection. Transferring contaminated equipment without the protection of gloves can help spread microbes to inanimate objects and to the person doing the transfer. Utilizing gloves, washing hands, covering contaminated supplies during transfer, and disinfecting equipment in the appropriate way in the appropriate places reflect the principles of basic medical asepsis and routine practices and can break the cN hainRof I infeG ctioB n..C M
U S N T
O
DIF: Apply REF: 697| 712 OBJ: Explain the rationale and components of routine practices. TOP: Implementation MSC: NCLEX: Safe and Effective Care Environment 22. The nurse is caring for a patient with a nursing diagnosis of Risk for infection. Aware of the
need for routine practices, the nurse is careful to do what? a. Teach the patient about good nutrition. b. Wear eyewear when emptying a urinary drainage bag. c. Avoid contact with intact skin without wearing gloves. d. Don gloves when wearing artificial nails. ANS: B
Routine practices include the wearing of eyewear whenever there is a possibility of a splash or splatter. Teaching the patient about good nutrition is positive but does not apply to routine practices. The term routine practices applies to all blood and body fluids except sweat, even if blood is not present. It also applies to nonintact skin and mucous membranes. DIF: Understand REF: 705 OBJ: Explain the rationale and components of routine practices. TOP: Implementation MSC: NCLEX: Safe and Effective Care Environment 23. The nurse is caring for a patient who has just given birth. The nurse is checking the patient for
excessive vaginal drainage. It is important for the nurse to utilize which type of precautions?
Canadian Fundamentals of Nursing 6th Edition Potter Test Bank a. b. c. d.
Contact precautions. Protective precautions. Droplet precautions. Routine practices.
ANS: D
Routine practices apply to contact with blood, body fluid, nonintact skin, and mucous membranes of all patients. Contact precautions apply to individuals with colonization of infection such as methicillin-resistant S. aureus (MRSA). Protective precautions apply to individuals who have undergone transplantations. Droplet precautions focus on diseases that are transmitted by large droplets. DIF: Remember REF: 705 OBJ: Explain the rationale and components of routine practices. TOP: Implementation MSC: NCLEX: Safe and Effective Care Environment 24. The nurse is caring for a patient in the hospital. The nurse observes the unregulated care
provider turning off the handle faucet with his hands. What professional practice supports the need for follow-up with the unregulated care provider? a. The nurse is responsible for providing a safe environment for the patient. b. This is a key step in the procedure for washing hands. c. Allowing the water to run is a waste of resources and money. d. Different scopes of practice allow modification of procedures. ANS: A
The nurse is responsible for providing a safe environment for the patient. The effectiveness of infection control practices depends on conscientiousness and consistency in using effective aseptic technique. It is humanNna eyOpMrocedural steps or to take shortcuts. URture SItoNfGorg TBet.kC However, failure to comply with basic procedures places the patient at risk for infection that can impair recovery or lead to death. After washing hands, the health care provider should turn off a handle faucet with a dry paper towel and avoid touching the handles with his or her own hands to assist in preventing the transfer of microorganisms. Wet towels and hands enable the transfer of pathogens from faucet to hands. The principles and procedures for washing hands are universal and apply to all members of health care teams. Being resourceful and aware of the cost of health care is important, but taking shortcuts that may endanger an individual’s health is not a prudent practice. DIF: Analyze TOP: Evaluate
REF: 701-703 OBJ: Perform proper procedures for hand hygiene. MSC: NCLEX: Safe and Effective Care Environment
25. The nurse is caring for a patient who becomes nauseated and vomits without warning. The
nurse has contaminated hands. What is the nurse’s best next step? a. Cleaning hands with wipes from the bedside table. b. Washing hands with an antimicrobial soap and water. c. Using an alcohol-based waterless hand gel. d. Instructing the patient to wash his face and hands. ANS: B
Canadian Fundamentals of Nursing 6th Edition Potter Test Bank The U.S. Centers for Disease Control and Prevention recommend that when hands are visibly soiled, the health care provider should wash with a plain soap or with antimicrobial soap. Cleaning hands with wipes or using waterless hand gel does not meet this standard. If hands are not visibly soiled, an alcohol-based waterless antiseptic agent should be used for routinely decontaminating hands. The patient may very well need to wash his face and hands, but this is not the best next step. DIF: Apply TOP: Planning
REF: 700 OBJ: Perform proper procedures for hand hygiene. MSC: NCLEX: Safe and Effective Care Environment
26. The nurse is performing hand hygiene before assisting a physician with insertion of a chest
tube. While washing hands, the nurse touches the sink. What is the next action the nurse should take? a. Inform the physician and recruit another nurse to assist. b. Rinse and dry hands, and begin assisting the physician. c. Repeat hand hygiene, using antiseptic soap. d. Extend the hand hygiene procedure to 5 minutes. ANS: C
The inside of the sink and the counter at the edges of the sink, faucet, and handles are considered contaminated areas. If the hands touch any of these areas during hand hygiene, the nurse should repeat the hand hygiene procedure with antiseptic soap. There is no need to inform the physician or be relieved of this assignment. If the hands are contaminated from touching the sink, drying hands and proceeding with the procedure could possibly contaminate materials, contribute to increased microbial counts during the procedure, and result in infection in the patient. Extending the time for washing the hands (although this is what will happen when the procedure is repeated) is not the focus. The focus is to repeat the N Rh an INtise GTptic B.C M whole hand hygiene procedureUwitS soaOp. DIF: Apply REF: 701-703 TOP: Implementation
OBJ: Perform proper procedures for hand hygiene. MSC: NCLEX: Safe and Effective Care Environment
27. The nurse is observing a family member changing a dressing for a patient in the home
environment. Which of these observations would indicate that the family member has a correct understanding of how to manage contaminated dressings? a. The family member removes gloves and gathers items for disposal. b. The family member places the used dressings in a plastic bag. c. The family member saves part of the dressing because it is clean. d. The family member wraps the used dressing in toilet tissue before placing in the garbage. ANS: B
Contaminated dressings and other infectious items should be placed in impervious plastic or brown paper bags and then disposed of properly in garbage containers. Gloves should be worn during this process. Parts of the dressing should not be saved, even though they may seem clean, because microbes may be present. DIF: Evaluate REF: 711 (Box 33-18) OBJ: Explain how infection-control measures in the home may differ from those in the hospital. TOP: Evaluate MSC: NCLEX: Health Promotion and Maintenance
Canadian Fundamentals of Nursing 6th Edition Potter Test Bank 28. The home health nurse is teaching a patient and family about hand hygiene in the home. The
nurse is sure to emphasize washing hands at what time? a. Before and after shaking hands. b. Before and after treatments. c. Before opening the refrigerator. d. Before and after using a computer. ANS: B
Patients should perform hand hygiene before and after treatments and when coming in contact with body fluids. Depending on the patient, holding hands does not necessitate washing of hands before, but washing hands afterwards is good practice, especially before touching eyes, nose, or mouth. Washing hands before and after opening the refrigerator and using the computer is not required, but during cold and flu season, it might be advisable. DIF: Apply REF: 711 (Box 33-18) OBJ: Explain how infection-control measures in the home may differ from those in the hospital. TOP: Implementation MSC: NCLEX: Health Promotion and Maintenance 29. The nurse has been caring for a patient in the perioperative area for several hours. The
surgical mask the nurse is wearing has become moist. The nurse’s best next step is what? a. Change the mask when relieved. b. Air-dry the mask while at lunch, and reapply. c. Ask for relief, step out of the surgical area, and apply a new mask. d. Not change the mask, if the nurse is comfortable. ANS: C
A mask should fit snugly around the face and nose. After the mask is worn for several hours, it can become moist. The maN skUsR hou ldNbG eT chB an.gC edOaMs soon as possible because moisture SI encourages the growth of microorganisms. Waiting to change the mask, air-drying it, or wearing it because it is comfortable does not support the principles of infection control. DIF: Apply REF: 709 OBJ: Properly don a surgical mask, sterile gown, and sterile gloves. TOP: Implementation MSC: NCLEX: Safe and Effective Care Environment 30. The nurse is caring for a patient for whom contact precautions are ordered. Which of the
following actions would be appropriate to prevent the spread of disease? a. Wearing a gown, gloves, face mask, and goggles for interactions with the patient. b. Using a dedicated blood pressure cuff that stays in the room and is used for that patient only. c. Placing the patient in a room with negative airflow. d. Transporting the patient quickly when going to the radiology department. ANS: B
Canadian Fundamentals of Nursing 6th Edition Potter Test Bank Contact precautions are a type of isolation precaution used for patients with illness that can be transmitted through direct or indirect contact. A patient is placed on contact precautions if a disease is present that can be transmitted through direct or indirect contact. Patients who are on contact precautions should have dedicated equipment wherever possible. This would mean, for example, that one blood pressure cuff and one stethoscope would stay in the room with the patient and would be used for that patient only. A gown and gloves may be required for interactions with a patient who is on contact precautions, but a face mask and goggles are not part of contact precautions. A room with negative airflow is needed for patients placed on airborne precautions; it is not necessary for a patient on contact precautions. When a patient on contact precautions needs to be transported, he or she should wear clean gowns, and wheelchairs or gurneys should be covered with an extra layer of sheets. Anyone who might come in contact with the patient needs to be protected, and equipment must be cleaned with an approved germicide after patient use and before another patient uses the shared equipment. DIF: Apply REF: 697| 705 OBJ: Explain the rationale and components of routine practices. TOP: Implementation MSC: NCLEX: Safe and Effective Care Environment 31. The nurse is caring for a patient whose cultures were positive for Clostridium difficile. Which
of the following nursing actions would be appropriate in view of the presence of this organism? a. Instructing assistive personnel to use soap and water rather than sanitizer to clean hands. b. Placing the patient on droplet precautions. c. Wearing an N95 respirator when entering the patient room. d. Teaching the patient cough etiquette. ANS: A
NURSINGTB.COM
C. difficile is a spore-forming organism that can be transmitted through direct and indirect patient contact. Because C. difficile is a spore-forming organism, hand sanitizer is not effective in preventing its transmission. Hands must be washed with soap and water to prevent transmission. This organism is not transmitted via the droplet route; therefore, droplet precautions are not needed. An N95 respirator is used primarily for providing care to patients with airborne illness. All patients should be taught cough etiquette; this action is not one to specific to a patient who has C. difficile infection. DIF: Apply REF: 689 OBJ: Explain the rationale and practices for additional (isolation) precautions. TOP: Implementation MSC: NCLEX: Safe and Effective Care Environment 32. The nurse is changing linens for a postoperative patient and feels a stick in her hand. A
nonactivated safe needle is noted in the linens. In this scenario, the nurse may be at risk for which of the following infections? a. Hepatitis B. b. C. difficile. c. Methicillin-resistant S aureus (MRSA). d. Diphtheria. ANS: A
Canadian Fundamentals of Nursing 6th Edition Potter Test Bank Bloodborne pathogens such as those associated with hepatitis B and C are most commonly transmitted by contaminated needles. C. difficile is spread by contact with and ingestion of this microbe, and MRSA is spread by contact. Diphtheria is spread by droplets when a person is within 1 m (3 feet) of the patient. DIF: Remember REF: 700| 704 OBJ: Explain conditions that promote the transmission of health care–associated infection. TOP: Diagnosis MSC: NCLEX: Safe and Effective Care Environment 33. The nurse is caring for a patient who has a bloodborne pathogen. The nurse splashes blood
above the glove onto intact skin while discontinuing an intravenous infusion. The nurse’s best next step is which of the following? a. To obtain an alcohol swab, remove the blood with an alcohol swab, and continue care. b. To immediately wash the site with soap and running water, and seek guidance from the manager. c. To delay washing of the site until the nurse is finished providing care to the patient. d. To do nothing; accidentally getting splashed with blood happens frequently and is part of the job. ANS: B
After getting splashed with blood from a patient who has a known bloodborne pathogen, the nurse must cleanse the site immediately and thoroughly with soap and running water and notify the manager and employee health for guidance on next steps in the process. Removing the blood with an alcohol swab, delaying washing, and doing nothing because the splash was to intact skin could possibly spread the blood within the room and could spread the infection. N ined RSI G diately B.C tOoMprevent contact spread. Contamination should be contaU imN meT DIF: Apply REF: 706 (Table 33-7) OBJ: Explain the rationale and components of routine practices. TOP: Implementation MSC: NCLEX: Safe and Effective Care Environment MULTIPLE RESPONSE 1. The nurse is caring for a patient with an order for routine practices. The nurse includes hand
hygiene as part of the plan of care to do which of the following? (Select all that apply.) a. Provide an uninterrupted chain of infection. b. Decrease the incidence of health care–associated infection. c. Protect the nurse from transmission of the microbes. d. Decrease the transmission of microbes to other patients. e. Prevent contamination of clean supplies. f. Decrease the drying effects of soap. ANS: B, C, D, E
Canadian Fundamentals of Nursing 6th Edition Potter Test Bank Hand hygiene is part of routine practices and assists in interrupting the chain of infection. Washing hands can assist in decreasing the incidence of health care–associated infection, protect the nurse from the transfer of microorganisms, decrease the transmission of microbes to other patients, and prevent contamination of clean supplies. Hands are a common vehicle of transmission of bacteria from one place to another. Proper hand hygiene does not decrease the drying effects of soap; in fact, it increases the drying effects of soap. DIF: Remember TOP: Planning
REF: 700| 705 OBJ: Perform proper procedures for hand hygiene. MSC: NCLEX: Safe and Effective Care Environment
2. The home health nurse is assessing a new patient. To decrease the risk of infection, which of
these questions would be most appropriate to ask? (Select all that apply.) a. “Will you demonstrate how to wash your hands?” b. “Do you have a working refrigerator?” c. “Can you explain the risk for infection in your home?” d. “What are the signs and symptoms of infection?” e. “Who runs errands for you?” f. “Are you able to walk to the mailbox?” ANS: A, B, C, D
In the home setting, the objective is that the patient and or family will utilize proper infection control techniques. Asking the patient and family about hand hygiene, risk of infection, and signs and symptoms of infection is important in evaluating the patient’s knowledge base on infection control strategies. Refrigeration is essential in keeping perishables cold to prevent foodborne illnesses and in allowing storage of enteral feedings or refrigerated medications. Activity assessment is important for evaluation of the overall status of the patient, and knowing who runs errands gives you information on who is helping to meet the needs of the N R I G B.C ngMthe risk of infection. patient, but neither of these areUrelaStedNto dTecreasiO DIF: Remember REF: 711 (Box 33-18) OBJ: Explain how infection-control measures in the home may differ from those in the hospital. TOP: Assessment MSC: NCLEX: Health Promotion and Maintenance 3. The circulating nurse in the perioperative area is observing the surgical technologist while
applying a sterile gown and gloves to care for a patient having an appendectomy. Which of the following behaviours indicate to the nurse that the procedure has been done correctly? (Select all that apply.) a. Surgical cap and face mask are in place. b. Surgical technologist ties the back of the gown. c. Surgical technologist touches only inside of gown. d. Surgical technologist slips arms into arm holes simultaneously. e. Surgical technologist uses hands covered by sleeves to open gloves. f. Fingers are extended fully into both gloves. ANS: C, D, E, F
Canadian Fundamentals of Nursing 6th Edition Potter Test Bank To maintain sterility, the surgical technologist touches only the inside of the gown that will be against the body. Arms are slipped simultaneously into the gown to prevent contamination. Using the sleeves to cover the hands while gloves are donned maintains the principle of sterile only touching sterile. Extending the fingers fully into both gloves ensures that the surgical technologist has full dexterity while using the sterile gloved hand. The surgical technologist applies the surgical cap, face mask, and eyewear before entering the surgical area and completing the surgical scrub. Reaching behind to tie the back of the gown will contaminate the sterile area of the gown. DIF: Apply REF: 712-724 OBJ: Properly don a surgical mask, sterile gown, and sterile gloves. TOP: Assessment MSC: NCLEX: Safe and Effective Care Environment 4. The nurse is preparing to insert a urinary catheter. The nurse is using open gloving to don the
sterile gloves. Which steps are included in this process? (Select all that apply.) a. Lay glove package on clean flat surface above waistline. b. Remove outer glove package by tearing the package open. c. Glove the dominant hand of the nurse first. d. While putting on the first glove, touch only the outside surface of the glove. e. With gloved dominant hand, slip fingers underneath second glove cuff. f. After second glove is on, interlock hands. ANS: A, C, E, F
Sterile objects held below the waist are considered contaminated. Gloving the dominant hand helps improve dexterity. Slipping the fingers underneath the second glove cuff helps protect the gloved fingers. Sterile touching sterile prevents glove contamination. Interlocking fingers ensures a smooth fit over the fingers. To open sterile supplies, the sides of the package are N apa RS I G B.C M carefully separated and peeled U rt; thNis pTreventsOthe sterile contents from accidentally opening and touching contaminated objects. Touching the outside of the glove surface will contaminate the sterile item; only the inside of the glove—the piece that will be against the skin—should be touched. DIF: Apply REF: 723| 724 OBJ: Properly don a surgical mask, sterile gown, and sterile gloves. TOP: Implementation MSC: NCLEX: Safe and Effective Care Environment 5. The nurse and the student nurse are caring for two patients on the medical-surgical unit. One
patient is on airborne precautions, and the other is on contact precautions. The nurse explains to the student different interventions for care. What should the nurse include in teaching? (Select all that apply.) a. Be consistent in nursing interventions; there is only one difference in the precautions. b. Wash hands before entering and leaving both of the patients’ rooms. c. Dispose of supplies to prevent the spread of microorganisms. d. Apply the knowledge the nurse has of the disease process to prevent the spread of microorganisms. e. Patients in airborne precautions wear a mask during transportation to departments. f. Checking the working order of the negative-pressure room is done on admission and at the time of discharge. ANS: B, C, D, E
Canadian Fundamentals of Nursing 6th Edition Potter Test Bank Washing hands, properly disposing of supplies, applying knowledge of the disease process, and having patients on airborne precautions wear a mask during transfer are all principles to follow when caring for patients in isolation. Multiple differences are evident between these types of isolation, including the type of room used for the patient and what the nurse wears while caring for the patient. It is important to check the working order of a negative-pressure room before admitting a patient to the room, each shift the patient is in the room, and if and when the device sounds an alarm. Even when no patient is in this type of room, regular and routine checks are important to ensure the working order. DIF: Apply REF: 705| 706 OBJ: Explain the rationale and practices for additional (isolation) precautions. TOP: Planning MSC: NCLEX: Safe and Effective Care Environment OTHER 1. The nurse is caring for a patient who needs a protective environment. The nurse has provided
the care needed and is now leaving the room. Select the correct order for removal of the personal protective equipment and associated tasks. (All answers are used.) a. Remove eyewear/face shield and goggles. b. Perform hand hygiene. c. Remove gloves. d. Untie gown, allow gown to fall from shoulders, and do not touch outside of gown; dispose of properly. e. Remove mask by strings; do not touch outside of mask. f. Dispose of all contaminated supplies and equipment in designated receptacles. g. Leave room and close the door. I G B.C M
NURS N T
O
ANS:
C, A, D, E, B, G, F The correct order for removing personal protective equipment for a patient in a protective environment and for performing associated tasks is to remove gloves, remove eyewear, remove gown, remove mask, perform hand hygiene, leave room and close doors, and dispose of all contaminated supplies and equipment in a manner that prevents the spread of microorganisms. DIF: Remember REF: 706-708 OBJ: Explain the rationale and practices for additional (isolation) precautions. TOP: Implementation MSC: NCLEX: Safe and Effective Care Environment 2. The nurse manager is evaluating current infection control data for the intensive care unit. The
nurse compares past patient data with current data to look for trends. The nurse manager examines the chain of infection for possible solutions. Arrange these items in the proper order of the chain of infection. (All answers are used.) a. A mode of transmission. b. An infectious agent or pathogen. c. A susceptible host. d. A reservoir or source for pathogen growth. e. A portal of entry to a host. f. A portal of exit from the reservoir.
Canadian Fundamentals of Nursing 6th Edition Potter Test Bank ANS:
B, D, F, A, E, C The nurse manager is evaluating the chain of infection to determine actions that could be implemented to influence the spread of infection in the intensive care unit. Understanding the spread of infection and directing actions toward those steps have the potential to decrease infection in the setting. For spread of infection, the chain has to be uninterrupted; the links in the chain are an infectious agent, a reservoir and portal of exit, a mode of transmission, a portal of entry, and a susceptible host. DIF: Remember REF: 684-687 OBJ: Explain the relationship between the chain of infection and transmission of infection. TOP: Evaluate MSC: NCLEX: Safe and Effective Care Environment
NURSINGTB.COM
Canadian Fundamentals of Nursing 6th Edition Potter Test Bank
Chapter 34: Medication Administration Potter et al: Canadian Fundamentals of Nursing, 6th Edition MULTIPLE CHOICE 1. A nurse knows that patient education has been effective when the patient makes which
statement? a. “I must take my parenteral medication with food.” b. “If I am 30 minutes late taking my medication, I should skip that dose.” c. “I will rotate the location where I give myself injections.” d. “Once I start feeling better, I will stop taking my medication.” ANS: C
Rotating injection sites provides greater consistency in absorption of medication. Parenteral medication absorption is not affected by the timing of meals. Taking a medication 30 minutes late is within the 60-minute window of the time medications should be taken. Medications should be stopped in accordance with the provider’s orders. With some medications, such as antibiotics, it is crucial that the full course of medication is taken to avoid relapse of infection. DIF: Apply REF: 752| 791| 792 OBJ: Examine the nurse’s role and responsibilities regarding medication administration. TOP: Evaluate MSC: NCLEX: Health Promotion and Maintenance 2. Which statement by the patient is an indication to use the Z-track method? a. “I’m really afraid that a big needle will hurt.” b. “The last shot like that turned my skin different colours.” NUicat RSions IN.”GTB.COM c. “I am allergic to many med d. “My legs are too obese for the needle to go through.” ANS: B
The Z-track is indicated when the medication being administered has the potential to irritate sensitive tissues. The Z-track method is not meant to reduce discomfort from the procedure. If a patient is allergic to a medication, it should not be administered. If a patient has additional subcutaneous tissue to go through, a needle of a different size may be selected. DIF: Understand REF: 794 OBJ: Describe factors to consider when choosing routes of medication administration. TOP: Planning MSC: NCLEX: Physiological Integrity 3. The physician orders that a 2-year-old have ear irrigation performed daily. How does the
nurse correctly perform the procedure? a. Pulling the auricle down and back to straighten the ear canal. b. Pulling the auricle upward and outward to straighten the ear canal. c. Instilling the irrigation solution by holding the syringe just inside the ear canal. d. Holding the fluid in the canal for 2 to 3 minutes with a cotton swab. ANS: A
Canadian Fundamentals of Nursing 6th Edition Potter Test Bank Children up to 3 years of age should have the auricle pulled down and back; children 4 years of age and up and adults should have the auricle pulled upward and outward. Irrigation solution should be instilled 1 cm (0.4 in) above the opening of the ear canal. Irrigation solution should be allowed to drain freely during instillation. DIF: Understand REF: 769 (Box 34-21) OBJ: Prepare and administer subcutaneous, intramuscular, and intradermal injections; intravenous medications; hypodermoclysis infusions; oral and topical skin preparations; eye, ear, and nose drops; vaginal instillations; rectal suppositories; and inhalants. TOP: Planning MSC: NCLEX: Physiological Integrity 4. The patient is to receive phenytoin (Dilantin) at 0900 hours. The nurse knows that the ideal
time to measure the trough level is when? a. 0800 hours. b. 0830 hours. c. 0900 hours. d. 0930 hours. ANS: B
Trough levels are generally measured 30 minutes before the drug is administered. If the medication is to be administered at 0900 hours, the trough should be measured at 0830 hours. DIF: Understand REF: 733 TOP: Implementation
OBJ: Discuss factors that influence medication actions. MSC: NCLEX: Physiological Integrity
5. A physician orders 1000 mL of normal saline to be infused at a rate of 50 mL/hr. The nurse
plans on hanging a new bag at what time? a. 2 hours. NURSINGTB.COM b. 5 hours. c. 10 hours. d. 20 hours. ANS: D
It will take 20 hours for a litre (1000 mL) of fluid to infuse at a rate of 50 mL/hr. After 2 hours, only 100 mL would have infused. After 5 hours, only 250 mL would have infused. At 10 hours, 500 mL would have infused. DIF: Remember REF: 738 TOP: Implementation
OBJ: Calculate a prescribed medication dose. MSC: NCLEX: Safe and Effective Care Environment
6. The nurse is preparing to administer a 0.5-mL rabies vaccine into the deltoid muscle of a
pediatric patient. Which needle size is best for the procedure? a. 20 gauge 3.8 cm ( inch). b. 23 gauge 1.3 cm ( inch). c. 25 gauge 1.6 cm ( inch). d. 27 gauge 1 cm ( inch). ANS: C
Canadian Fundamentals of Nursing 6th Edition Potter Test Bank For an intramuscular injection into the deltoid, a 25-gauge, 1.6-cm ( -inch) needle (depending upon the size of the child) is recommended. The other choices did not include the correct gauge of 25. DIF: Remember REF: 786 OBJ: Describe factors to consider when choosing routes of medication administration. TOP: Implementation MSC: NCLEX: Safe and Effective Care Environment 7. The nurse is giving an intramuscular (IM) injection. The nurse notices blood return in the
syringe. What should the nurse do? a. Administer the injection at a slower rate. b. Withdraw the needle and prepare the injection again. c. Pull the needle back slightly and inject the medication. d. Give the injection and hold pressure over the site for 3 minutes. ANS: B
Blood return indicates improper placement, and the injection should not be given. Instead, the nurse should withdraw the needle, dispose of the syringe and needle properly, and prepare the medication again. Administering IM medication into a blood vessel could have dangerous adverse effects, and the medication will be absorbed faster than intended because of increased blood flow. Holding pressure is not an appropriate intervention. Pulling back the needle slightly does not guarantee proper placement of the needle and medication administration. DIF: Apply REF: 789 OBJ: Describe the importance of safe medication techniques. MSC: NCLEX: Safe and Effective Care Environment
TOP: Implementation
8. The nurse is planning to admN inU isR teS r aItN ubGeT rcB ul. inCteOsM t with a 27-gauge, 1-cm (3/8-inch) needle.
The nurse should insert the needle at which angle? a. 15 degrees. b. 45 degrees. c. 90 degrees. d. 180 degrees. ANS: A
A 27-gauge, 1-cm (3/8-inch) needle is used for intradermal injections such as a tuberculin test, which should be inserted at a 5- to 15-degree angle, just under the dermis of the skin, about 3 mm. Placing the needle at 45 degrees, 90 degrees, or 180 degrees will cause the medication to go in too deep. DIF: Understand REF: 794 OBJ: Describe factors to consider when choosing routes of medication administration. TOP: Implementation MSC: NCLEX: Safe and Effective Care Environment 9. The nurse knows to assess for signs of medication toxicity within older persons because of
which physiological change? a. Reduced blood albumin level. b. Delayed esophageal clearance. c. Decreased gastric peristalsis. d. Decreased cognitive function. ANS: A
Canadian Fundamentals of Nursing 6th Edition Potter Test Bank The degree to which medications bind to serum proteins, such as albumin, affects the medication's distribution. Medications bound to albumin cannot exert pharmacological activity. The unbound, or free, medication is the active form of the medication. Older persons have a decrease in albumin in the bloodstream, which is probably the result of a change in liver function. The same is true for patients who have liver disease or malnutrition, who, along with older persons, have the potential for more medication to be unbound and thus may be at risk for an increase in medication activity or toxicity, or both. DIF: Understand REF: 731 TOP: Implementation
OBJ: Discuss factors that influence medication actions. MSC: NCLEX: Physiological Integrity
10. A registered nurse interprets that a scribbled medication order reads 25 mg. The nurse
administers 25 mg of the medication to a patient, and then discovers that the dose was incorrectly interpreted and should have been 15 mg. Who is ultimately responsible for the error? a. Physician. b. Pharmacist. c. Nurse. d. No fault. ANS: C
Ultimately, the person administering the medication is responsible for ensuring that it is correct. The nurse administered the medication, so in this case it is the nurse. This is the importance of verifying the Ten Rights of Medication Administration. DIF: Understand REF: 743| 744 OBJ: Compare and contrast the roles of the physician, the pharmacist, and the nurse in medication ReSnI administration. TOP: ImNpU lem tatN ioG n TB.COM MSC: NCLEX: Safe and Effective Care Environment 11. A patient is to receive medication through a nasogastric tube. What is the most important
nursing action to ensure effective absorption? a. Thoroughly shake the medication before administering. b. After all medications are administered, flush tube with 15 to 30 mL of water. c. Position patient in the supine position for 30 minutes. d. Clamp suction for 30 to 60 minutes after medication administration. ANS: D
Absorption time for a medication administered through a nasogastric tube is the same as for an oral medication: 30 to 60 minutes. Therefore, the nurse would need to hold the suction for that amount of time to let the medication absorb. Thoroughly shaking the medication mixes the medication before administration but does not affect absorption. Flushing the medications ensures that all were administered. Patients with nasogastric tubes should never be positioned supine but instead should be positioned at a 30- to 90-degree angle to prevent aspiration, provided that no contraindication to such positioning is known. DIF: Analyze REF: 762 (Box 34-19) OBJ: Describe the importance of safe medication techniques. MSC: NCLEX: Physiological Integrity
TOP: Implementation
Canadian Fundamentals of Nursing 6th Edition Potter Test Bank 12. Aspirin is an analgesic, antipyretic, antiplatelet, and anti-inflammatory agent. A physician
writes for aspirin, 650 mg every 4 to 6 hours, “prn [as needed]: febrile.” For which patient would this order be appropriate? a. A 7-year-old with hemophilia. b. A 21-year-old with a sprained ankle. c. A 35-year-old with a severe headache. d. A 62-year-old with pneumonia. ANS: D
The provider wrote for the medication to be given for a fever. Hemophilia is a bleeding disorder; therefore, antiplatelet agents would be contraindicated. Although it can be used for inflammatory problems and pain, this is not what the order was written for. DIF: Evaluate REF: 732 OBJ: Discuss factors to use when assessing a patient’s needs for and response to medication therapy. TOP: Implementation MSC: NCLEX: Safe and Effective Care Environment 13. A patient is in need of immediate pain relief for a severe headache. The nurse knows that
which medication will be absorbed the quickest? a. Tylenol, 650 mg PO. b. Morphine, 4 mg SQ. c. Ketorolac (Toradol), 8 mg IM. d. Hydromorphone (Dilaudid), 4 mg IV. ANS: D
The intravenous (IV) route is the fastest route for absorption because of the increase in blood flow. Oral (PO), subcutaneous (SQ), and IM are other ways to deliver medication but with less blood flow. N R I G B.C M
U S N T
DIF: Understand REF: 731 TOP: Implementation
O
OBJ: Discuss factors that influence medication actions. MSC: NCLEX: Physiological Integrity
14. A drug requires a low pH to be metabolized. Knowing this, the nurse anticipates that the
medication will be administered by which route? a. Oral. b. Parenteral. c. Buccal. d. Inhalation. ANS: A
An oral medication would pass through to the stomach, which is an area of low pH. The nurse would question an order for a medication for which an acidic environment is needed to be metabolized. Buccal, inhalation, and parenteral routes provide neutral or alkaline environments. DIF: Analyze REF: 731 OBJ: Describe factors to consider when choosing routes of medication administration. TOP: Implementation MSC: NCLEX: Physiological Integrity 15. The nurse knows that a patient is having an idiosyncratic reaction to the stimulant
pseudoephedrine (Sudafed) when what happens? a. The patient experiences blurred vision while driving.
Canadian Fundamentals of Nursing 6th Edition Potter Test Bank b. The patient falls asleep during daily activities. c. The patient presents with a pruritic rash. d. The patient develops xerostomia. ANS: B
An idiosyncratic reaction is a reaction opposite to what the side effects of the medication normally are, or an overreaction or underreaction to the medication. Blurred vision is a toxic effect. A rash could indicate an allergic reaction. Dry mouth is a typical response to a stimulant. DIF: Apply REF: 732 OBJ: Differentiate among different types of medication actions. TOP: Assessment MSC: NCLEX: Physiological Integrity 16. An order is written for phenytoin (Dilantin), 500 mg IM q3-4h prn for pain. The nurse
recognizes that treatment of pain is not a standard therapeutic indication for this drug. The nurse believes that the prescriber meant to write for hydromorphone (Dilaudid). What should the nurse do? a. Give the patient Dilaudid, as it was meant to be written. b. Call the prescriber to clarify and justify the order. c. Administer the medication and monitor the patient frequently. d. Refuse to give the medication and notify the nurse supervisor. ANS: B
If the nurse is apprehensive about the drug, dose, route, or reason for a medication, the nurse should first call the prescriber and clarify. The nurse should not change the order without the prescriber’s consent. Ultimately, the nurse can be held responsible for administering an incorrect medication. If the pN resc isGunw lliC ngOtM o change the order and does not justify the URribe SIr N TBi. order in a reasonable and evidence-informed manner, the nurse may refuse to give the medication and notify her supervisor. DIF: Apply REF: 743-746 OBJ: Compare and contrast the roles of the physician, the pharmacist, and the nurse in medication administration. TOP: Implementation MSC: NCLEX: Safe and Effective Care Environment 17. A patient needs assistance excreting a gaseous medication. What is the correct nursing action? a. Encourage the patient to cough and deep breathe. b. Suction the patient’s respiratory secretions. c. Administer the antidote via inhalation. d. Administer 100% fraction of inspired oxygen (FiO2) via simple face mask. ANS: A
Gaseous and volatile medications are excreted through gas exchange. Deep breathing and coughing will assist in clearing the medication more quickly. DIF: Understand REF: 731 OBJ: Describe the physiological mechanisms of medication action, including absorption, distribution, metabolism, and excretion of medications. TOP: Implementation MSC: NCLEX: Physiological Integrity
Canadian Fundamentals of Nursing 6th Edition Potter Test Bank 18. The nurse knows that patient education about a buccal medication has been effective when the
patient makes which statement? a. “I should let the medication dissolve completely.” b. “I can only drink water, not juice, with this medication.” c. “For faster distribution, I should chew my medication first.” d. “I should place the medication in the same location.” ANS: A
Buccal medications should be placed in the side of the cheek and allowed to dissolve completely. Buccal medications act with the patient’s saliva and mucosa. The patient should not chew or swallow the medication. Gastric secretions may destroy some medications. The patient should rotate sides of the check to avoid irritating the mucosal lining. DIF: Understand REF: 734 OBJ: Discuss methods of educating a patient about prescribed medications. TOP: Evaluate MSC: NCLEX: Physiological Integrity 19. What is the nurse’s priority action to protect a patient from medication error? a. Requesting that the prescriber write out an order, rather than giving a verbal order. b. Asking anxious family members to leave the room before a medication is
administered. c. Checking the patient’s room number against the medication administration record. d. Administering as many of the medications as possible at one time. ANS: A
Verbal orders should be limited to urgent situations in which written communication is unavailable. The nurse should explain the reasons and logistics of a procedure to calm anxious family members and should aNskUR fam ilyNm erC s no Gem SI TBb. OMt to distract the person administering medication, for the patient’s safety. After proper education, if the family members are creating an unsafe environment, the nurse may ask them to step out of the room. The medication administration record should be checked against the patient’s hospital identification band; a room number is not an acceptable identifier. Medications should be given when scheduled, and medications with special assessment indications should be separated. DIF: Analyze REF: 740 (Box 34-5) OBJ: Implement nursing actions to prevent medication errors. MSC: NCLEX: Safe and Effective Care Environment
TOP: Implementation
20. The patient is in severe pain and is requesting a prn medication before the prn time interval
has elapsed. What is the nurse’s priority? a. Give the medication early for any pain score greater than 8. b. Call the prescriber and request a stat order. c. Explain to the patient why he will have to wait for the medication. d. Document the patient’s request and pain score. ANS: B
Canadian Fundamentals of Nursing 6th Edition Potter Test Bank The nurse should use clinical judgement to advocate for the patient by requesting a stat order for the patient’s breakthrough pain. The nurse cannot give a medication without an order because this violates the “Right Time” portion of the Ten Rights of Medication Administration. If a nurse determines that a patient is in severe pain, she must use clinical judgement to find that patient a means of pain relief. Although the nurse should document the patient’s request and pain score, this is not the priority. DIF: Analyze REF: 747 TOP: Implementation
OBJ: Explain the rights of medication administration. MSC: NCLEX: Physiological Integrity
21. A patient is at risk for aspiration. What nursing action is most appropriate? a. Hold the patient’s cup for him so he can concentrate on taking pills. b. Thin out liquids so they are easier to swallow. c. Give the patient a straw to control the flow of liquids. d. Have the patient self-administer the medication. ANS: D
Aspiration occurs when food, fluid, or medication intended for gastrointestinal administration inadvertently enters the respiratory tract. To minimize aspiration risk, allow the patient, if capable, to self-administer medication. Patients should also hold their own cup to control how quickly they take in fluid. Liquids should be thickened to reduce the risk of aspiration. Patients at risk for aspiration should not be given straws because use of a straw decreases the control the patient has over volume intake. DIF: Understand REF: 756 (Box 34-18) OBJ: Discuss factors to use when assessing a patient’s needs for and response to medication therapy. TOP: Implementation MSC: NCLEX: Safe and Effective Care Environment
NURSINGTB.COM
22. A confused patient refuses his medication. What is the nurse’s first response? a. Agree with the patient’s decision and document it in his chart. b. Educate the patient about the importance of the medication. c. Discreetly hide the medication in the patient’s favourite Jell-O. d. Inform the patient that he must take the medication whether he wants to or not. ANS: B
Much of a patient’s apprehension about medication comes from lack of understanding, and educating the patient may lead to better compliance. Ultimately, the patient does have the right to refuse the medication; however, the nurse should first try to educate the patient. Hiding, deceiving, or forcing a patient into taking a medication is unethical and violates his right to autonomy. DIF: Apply REF: 748 OBJ: Discuss methods of educating a patient about prescribed medications. TOP: Planning MSC: NCLEX: Safe and Effective Care Environment 23. A patient who is being discharged today is going home with an inhaler. The patient is to
administer 2 puffs of the inhaler twice daily. The inhaler contains 200 puffs. When should the nurse appropriately advise the patient to refill the medication? a. As soon as he leaves the hospital. b. When the inhaler is half empty. c. Six weeks after the patient starts using the inhaler.
Canadian Fundamentals of Nursing 6th Edition Potter Test Bank d. Fifty days after discharge. ANS: C
The inhaler should last the patient 50 days; the nurse should advise the patient to refill the prescription when he has 7 to 10 days of medication remaining. Refilling it as soon as he leaves the hospital or when the inhaler is half empty is too early. If the patient waits 50 days, the patient will run out of medication before it can be refilled. DIF: Understand REF: 775 TOP: Implementation
OBJ: Calculate a prescribed medication dose. MSC: NCLEX: Health Promotion and Maintenance
24. Why does a subcutaneous injection take longer to absorb than IV injection? a. Fewer blood vessels are found under the subcutaneous level. b. Adipose tissue takes longer to metabolize medication. c. Connective tissue holds medication in place longer. d. Some medication leaks out after instillation. ANS: A
How quickly a medication is absorbed is dependent on blood flow to the site. Locations with less blood supply take longer to absorb. Absorption is not based on adipose tissue; however, excessive adipose tissue may cause the medication to take longer to reach the blood supply. The connective tissue is not part of medication absorption. If a medication is properly administered, none of it should be wasted. DIF: Understand TOP: Planning
REF: 785-792 OBJ: Discuss factors that influence medication actions. MSC: NCLEX: Physiological Integrity
25. The nurse realizes which patiN entR is aI t greGateB st riC sk fM or an unintended synergistic effect? U four S different N T .specialists. O a. A 72-year-old who is seeing b. A 4-year-old who has mistakenly taken the entire packet of his mother’s birth
control pills. c. A 50-year-old who was prescribed a second blood pressure medication. d. A 35-year-old drug addict who has ingested “meth” mixed with several household
chemicals. ANS: A
A synergistic effect occurs when two medications potentiate each other, creating a greater effect than a single medication on its own. Polypharmacy is likely to occur when the 72-year-old is seeing four different health care providers. Polypharmacy places the patient at risk for unintended mixing of medications that potentiate each other. The child taking too much of a medication by mistake could experience overdose or toxicity. The 50-year-old is prescribed two different blood pressure medications for their synergistic effect, but this is a desired event. A drug addict mixing chemicals can be toxic. DIF: Analyze REF: 733| 754| 755 OBJ: Differentiate among different types of medication actions. TOP: Assessment MSC: NCLEX: Physiological Integrity 26. Which patient using an inhaler would benefit most from using a spacer? a. A 3-year-old with a cleft palate. b. A 25-year-old with multiple sclerosis. c. A 50-year-old with hearing impairment.
Canadian Fundamentals of Nursing 6th Edition Potter Test Bank d. A 72-year-old with left-sided hemiparesis. ANS: B
A spacer is indicated for a patient who has limited coordination or function. Individuals with multiple sclerosis often lose motor control and function and have difficulty seeing. Children often have difficulty using a spacer, and so a simple face mask is preferred for infants and children younger than 4. Hearing impairment may make teaching the patient to use the inhaler difficult, but it does not indicate the need for a spacer. A patient with one-sided weakness would have a difficult time assembling and administering an inhaler by using a spacer, but the patient could use the inhaler single-handedly. DIF: Analyze REF: 768-775 OBJ: Describe factors to consider when choosing routes of medication administration. TOP: Implementation MSC: NCLEX: Physiological Integrity 27. The prescriber wrote an order for a 40-kg child to receive 25 mg of medication four times a
day. The therapeutic range is 5 to 10 mg/kg/day. What is the nurse’s priority? a. Administer the medication because it is within the therapeutic range. b. Notify the physician that the prescribed dose is in the toxic range. c. Notify the physician that the prescribed dose is below the therapeutic range. d. Change the dose to one that is within range. ANS: C
The dosage range for a 40-kg patient is 200 to 400 mg a day. The prescribed dose is 100 mg/day, which is below therapeutic range. The nurse should notify the physician first and ask for clarification on the order. The dose is not above the therapeutic range and is not at a toxic level. The nurse should never alter an order without the prescriber’s approval and consent.
NURSINGTB.COM OBJ: Calculate a prescribed medication dose.
DIF: Apply REF: 737-739 TOP: Implementation
MSC: NCLEX: Safe and Effective Care Environment
28. The nurse is administering an IV medication that is to be infused over 10 minutes. Which
method should the nurse choose to efficiently administer the medication? a. Place the medication in a large-volume catheter-tipped syringe. b. Mix the medication into the patient’s maintenance fluids. c. Attach separate tubing and set the medication syringe in a mini-infusion pump. d. Stand at the patient’s bedside and carefully watch the clock while pushing the medication. ANS: C
To administer this medication efficiently, the nurse should use an infusion pump to run the medication in over a prolonged time. This method is more accurate and is more time efficient than other methods because the nurse can leave the room. The nurse should not mix medication into the maintenance bag without pharmacist and physician approval. Pushing the medication is not a time-efficient method for the nurse. A catheter-tipped syringe is an inappropriate device for administration of a medication. DIF: Analyze REF: 796 OBJ: Describe factors to consider when choosing routes of medication administration. TOP: Implementation MSC: NCLEX: Safe and Effective Care Environment
Canadian Fundamentals of Nursing 6th Edition Potter Test Bank 29. The nurse is preparing to administer medications to two patients with the same last name.
After the first administration, the nurse realizes that she did not check the identification of the patient before administering medication. Which of the following actions should the nurse complete first? a. Return to the room to check and assess the patient. b. Administer the antidote to the patient immediately. c. Alert the charge nurse that a medication error has occurred. d. Complete proper documentation of the medication error in the patient’s chart. ANS: A
The nurse’s first priority is to establish the safety of the patient by assessing the patient. Second, the nurse should notify the charge nurse and the physician. The antidote should be administered if required. Finally, the nurse needs to complete proper documentation. DIF: Evaluate REF: 743| 747 OBJ: Implement nursing actions to prevent medication errors. MSC: NCLEX: Safe and Effective Care Environment
TOP: Implementation
30. The nurse knows that caring for two patients with the same last name can lead to a medication
error involving which right of medication administration? a. Right medication. b. Right patient. c. Right dose. d. Right route. ANS: B
The nurse should ask the patient to verify his or her identity and should check the patient’s ID bracelet against the medicatioNnUrR ecor reCacc SId NtoGens TBu. OMuracy. Acceptable patient identifiers include the patient’s name, an identification number assigned by a health care agency, and a telephone number. The patient’s room number should not be used as an identifier. To identify a patient correctly in an acute care setting, the nurse should compare the patient identifiers on the medication administration record with the patient’s identification bracelet while at the patient’s bedside. Right medication, right dose, and right route are equally as important, but this example outlines right patient. DIF: Remember REF: 746-749 TOP: Implementation
OBJ: Explain the rights of medication administration. MSC: NCLEX: Safe and Effective Care Environment
31. A patient states that she would prefer not to take her daily allergy pill this morning because it
makes her too drowsy throughout the day. How may the nurse respond therapeutically? a. “The physician ordered it; therefore, you must take your medication every morning at the same time whether you’re drowsy or not.” b. “Let’s change the time you take your pill to 9 p.m., so the drowsiness occurs when you would normally be sleeping.” c. “You can skip this medication on days when you need to be awake and alert.” d. “Try to get as much done as you can before you take your pill, so you can sleep in the afternoon.” ANS: B
Canadian Fundamentals of Nursing 6th Edition Potter Test Bank The nurse should use knowledge about the medication to educate the patient about potential response to medications. Then the medication schedule can be altered based on that knowledge, after the physician has been notified. It is the patient’s right to refuse her medication; however, the nurse should educate the patient on the importance and effects of her medication. Asking a patient to change her entire life schedule around a medication is unreasonable and will decrease compliance. The nurse should be supportive and should offer solutions to manage medication effects. DIF: Apply REF: 747 OBJ: Discuss factors to use when assessing a patient’s needs for and response to medication therapy. TOP: Implementation MSC: NCLEX: Physiological Integrity 32. A provider has ordered a STAT medication to be administered. The nurse knows that the best
route of administration is a. IV. b. IM. c. SQ. d. PO. ANS: A
IV medications have the quickest effect because they receive the most blood flow. A STAT order is to be carried out as quickly as possible, so the effect should be as immediate as possible. Oral, subcutaneous (SQ), and IM are other ways to deliver medication but with less blood flow. DIF: Understand REF: 731 OBJ: Describe the physiological mechanisms of medication action, including absorption, TOP: Implementation distribution, metabolism, and exNcU reR tioS nI ofNmGeT dicBa. tioC nsO .M MSC: NCLEX: Physiological Integrity 33. A nurse is attempting to administer medication to a child, but the child refuses to take the
medication. The nurse asks for the parent’s cooperation by saying which of the following? a. “Please hold your child’s arms down at her sides, so I can get the full dose of medication into her mouth.” b. “I will prepare the medication for you and observe if you would like to try to administer the medication.” c. “Let’s turn the lights off and give the child a moment to fall asleep before administering the medication.” d. “Since your child loves applesauce, let’s add the medication to it, so your child doesn’t resist.” ANS: B
Children often have difficulties taking medication, but it is less traumatic for the child if the parent administers the medication. Holding down the child is not the best option because it may further upset the child. Never administer an oral medication to a sleeping child. Don’t mix medications into the child’s favourite foods, because the child might start to refuse the food. DIF: Evaluate REF: 754 TOP: Implementation
OBJ: Discuss factors that influence medication actions. MSC: NCLEX: Health Promotion and Maintenance
Canadian Fundamentals of Nursing 6th Edition Potter Test Bank 34. A 64-year-old quadriplegic patient needs an IM injection of antibiotic. What is the best site
for the administration? a. Deltoid. b. Dorsal gluteal. c. Ventrogluteal. d. Vastus lateralis. ANS: D
Vastus lateralis is a large muscle that is easily accessible from the supine position. Because this patient does not walk, the ventrogluteal muscle atrophies and is not the ideal location. The dorsal gluteal site is a location for a subcutaneous injection, and this patient requires an IM injection. The deltoid is easily accessible, but this muscle is not well developed in many adults. DIF: Apply REF: 793 (Box 34-27) OBJ: Discuss factors that influence medication actions. MSC: NCLEX: Physiological Integrity
TOP: Implementation
35. Which nursing action is the number one priority for ensuring that medication stays in the
target therapeutic range? a. Measuring the peak and trough levels at the same time each day. b. Administering a double dose after a dose was missed. c. Delivering the same amount of the drug at the same time each day. d. Increasing absorption by holding all other medications 1 hour before administration. ANS: A
The quantity and distributionNofUR a me ferent body compartments change SIdic NGatio TBn .inCdif OM constantly. Measuring peak and trough levels allows health care providers to see whether the current medication dosage is effective for the patient, or if it needs to be adjusted. Administering a double dose is dangerous and could cause the medication levels to cross the toxic threshold. Delivering the same amount each day may not be therapeutic or may be toxic for the patient. Holding all other medications should not affect the peak or half-life of the medications, assuming that they are compatible. DIF: Apply REF: 733 OBJ: Discuss factors to use when assessing a patient’s needs for and response to medication therapy. TOP: Implementation MSC: NCLEX: Physiological Integrity 36. Which of the following demonstrates proper oral medication administration? a. Removing the medication from the wrapper and placing it in a cup labelled with
the patient’s information. b. Using the edge of the medicine cup to fill with 0.5 mL of liquid medication. c. Placing all of the patient’s medications in the same cup, except medications with
assessments. d. Combining liquid medications from 2 single dose cups into 1 medicine cup. ANS: C
Canadian Fundamentals of Nursing 6th Edition Potter Test Bank Placing medications that require preadministration assessment in a separate cup serves as a reminder to check before the medication is given, which makes it easier for the nurse to withhold medication if necessary. Medications should not be removed from their package until they are in the patient’s room because this makes identification of the medications easier and reduces contamination. When measuring a liquid, the nurse should use the meniscus level to measure, not the edge. In addition, liquid medications measuring less than 10 mL should be drawn up in a needleless syringe. Single-dose medications should not be transferred to medicine cups, to reduce unnecessary manipulation of the dose. DIF: Understand REF: 758 OBJ: Describe the importance of safe medication techniques. MSC: NCLEX: Safe and Effective Care Environment
TOP: Implementation
37. A patient who is receiving IV fluids notifies the nurse that his arm feels tight. Upon
assessment, the nurse notes that the arm is swollen and cool to the touch. What should the nurse’s first action be? a. Discontinue the IV site, and apply a warm compress. b. Attach a syringe, and pull back on the plunger to aspirate the IV fluid. c. Start a new IV site distal from the site. d. Stop the IV fluids, and notify the physician immediately. ANS: A
An IV site that is puffy, swollen, and cool to the touch indicates infiltration. The IV site should be discontinued immediately because it is no longer a viable access point. Pulling back on the syringe will not result in fluid return because there is no longer venous access. A new IV site should be established in the opposite arm after the old IV line has been removed. The IV line should be removed; it is not sufficient to only stop the fluids.
N R I G B.C M U S N T O
DIF: Apply REF: 729 OBJ: Describe the importance of safe medication techniques. MSC: NCLEX: Safe and Effective Care Environment
TOP: Implementation
38. The physician orders 4 mg of oxycodone to be delivered every 6 hours. After 4 hours, the
patient is complaining that she is in more pain. The nurse advises the physician to make which medication adjustment? a. Add an additional narcotic on top of the oxycodone. b. Divide the dose in half and administer 2 mg every 3 hours. c. Give another 4 mg of oxycodone after 4 hours. d. Change the medication being administered for pain relief. ANS: B
The patient’s metabolism causes the peak effect to occur when the medicine is at its highest concentration. After reaching its peak, the serum concentration of the medication falls progressively. Spreading out the dose ensures that the patient will receive constant pain relief. Changing the medication, increasing the dose, or adding another medication is not the best course of action. DIF: Apply REF: 804 OBJ: Discuss factors to use when assessing a patient’s needs for and response to medication therapy. TOP: Implementation MSC: NCLEX: Physiological Integrity
Canadian Fundamentals of Nursing 6th Edition Potter Test Bank MULTIPLE RESPONSE 1. Which of the following are methods to reduce the risk of needlestick injury? (Select all that
apply.) a. Recap the needle after giving an injection. b. Use needleless systems when available c. Use two hands to dispose of sharps into the disposal. d. Never force a needle into the sharps disposal. e. Clearly mark sharps disposal containers. f. Use needleless devices whenever possible. ANS: B, D, E, F
To prevent the risk of needlesticks, the nurse should never recap needles. Needleless systems or sharps with engineered sharps injury protections (SESIP) safety devices should be used when available. Needles should not be forced into the box. Receptacles should be marked clearly to warn of danger. Using needleless systems when possible will further reduce the risk of needlestick injury. DIF: Remember REF: 796 (Box 34-28) OBJ: Describe the importance of safe medication techniques. MSC: NCLEX: Safe and Effective Care Environment
NURSINGTB.COM
TOP: Implementation
Canadian Fundamentals of Nursing 6th Edition Potter Test Bank
Chapter 35: Complementary and Alternative Approaches in Health Care Potter et al: Canadian Fundamentals of Nursing, 6th Edition MULTIPLE CHOICE 1. A patient describes practising a complementary and alternative therapy involving
concentrating and controlling his respiratory rate and pattern, recognizing that breath work is to yoga as which of the following? a. The “zone” is to acupressure. b. Massage therapy is to Ayurveda. c. Reiki therapy is to therapeutic touch. d. Prayer is to tai chi. ANS: C
This is an analogy that compares different therapies within specific categories. Both yoga and breath work are mind-body therapies, whereas both reiki and therapeutic touch therapies are energy field therapies. The other options have different design structures; thus, they do not fit the analogy. DIF: Apply REF: 816 OBJ: Describe the various treatments associated with complementary and alternative medicine. TOP: Planning MSC: NCLEX: Health Promotion and Maintenance 2. A teenager with an anxiety disorder is referred for biofeedback because her parents do not
want her on anxiolytics. The nurse recognizes that the teenager understands her health education on biofeedback when she makes which statement? Ne R I GTyB.C M a. “Biofeedback will allow mU toSdireNct m enerO gies in an intentional way when I’m stressed.” b. “Biofeedback will allow me to manipulate my stressed-out joints.” c. “Biofeedback will help me with my thoughts, feelings, and physiological responses to stress.” d. “Biofeedback will let me assess and redirect my energy fields.” ANS: C
By using electromechanical instruments, a person can receive information or feedback on his or her stress level. Having this knowledge allows the patient to develop awareness and voluntary control over his or her physiological symptoms. Biofeedback does not address energy fields. Directing energies is therapeutic touch. Manipulation of body alignment and joints is done by a chiropractor. DIF: Apply REF: 818 OBJ: Understand how complementary and alternative approaches assist in health promotion and disease prevention. TOP: Planning MSC: NCLEX: Psychosocial Integrity 3. A 70-year-old patient is newly admitted to a skilled nursing facility with the diagnoses of
Alzheimer’s dementia, lipidemia, and hypertension and a history of pulmonary embolism. Medications brought on admission included lisinopril, hydrochlorothiazide, warfarin, low-dose aspirin, ginkgo biloba, and echinacea. The nurse contacts the patient’s medical provider over which potential drug-drug interaction? a. Lisinopril and echinacea.
Canadian Fundamentals of Nursing 6th Edition Potter Test Bank b. Warfarin and ginkgo biloba. c. Echinacea and warfarin. d. Lisinopril and hydrochlorothiazide. ANS: B
Warfarin and blood thinners interact with ginkgo biloba as designed to improve memory. All herbal supplements should be evaluated with current pharmacological medications. The drugs in the other options do not interact with each other. DIF: Analyze REF: 826 OBJ: Recognize botanical treatments as an approach to complementary and alternative medicine. TOP: Planning MSC: NCLEX: Physiological Integrity 4. An acquaintance of a nurse asks for a nonmedical approach for excessive worry and work
stress. The most appropriate complementary and alternative medicine (CAM) therapy that the nurse can recommend is which of the following? a. Meditation. b. Ayurvedic herbs. c. Acupuncture. d. Chiropractic therapy. ANS: A
Meditation is indicated for stress-related illness. A person can learn to calm down and cope with stress through the use of meditation. Ayurvedic herbs have been used for centuries to treat illness. Acupuncture focuses on redirecting qi via the body’s meridian energy lines to influence deeper internal organs. Chiropractic therapy involves manipulation of the spinal column and includes physiotherapy and diet therapy.
NURSINGTB.COM
DIF: Analyze REF: 821 OBJ: Describe the various treatments associated with complementary and alternative medicine. TOP: Planning MSC: NCLEX: Psychosocial Integrity 5. The therapy that is more effective in treating physical ailments than in preventing disease or
managing chronic illness is which medicine? a. Allopathic. b. Complementary. c. Alternative. d. Mind-body. ANS: A
Allopathic medicine is synonymous with traditional Western medicine, which is highly effective in treating numerous physical ailments, but it is in general less effective in preventing disease, decreasing stress-induced illness, managing chronic disease, and caring for the emotional and spiritual needs of individuals. Complementary, alternative, and mind-body types of medicine can be used in tandem with allopathic medicine but are distinctly different. DIF: Understand REF: 815 OBJ: Identify the differences between complementary and alternative approaches in health care. TOP: Planning MSC: NCLEX: Health Promotion and Maintenance
Canadian Fundamentals of Nursing 6th Edition Potter Test Bank 6. Which of the following is considered a healing practice that promotes harmony within a
community and in the physical and spiritual worlds through sweating and purging, herbal remedies, and shamanic healing? a. Latin American practices. b. Traditional Chinese medicine. c. Ayurveda. d. Traditional Indigenous medicine. ANS: D
Traditional Indigenous medicine is a healing practice that promotes harmony within a community and in the physical and spiritual worlds through sweating and purging, herbal remedies, and shamanic healing. Latin American practice includes a humoral model for classifying food, activity, drugs, illnesses, and a series of folk illnesses. Traditional Chinese medicine promotes health and treats disease through acupuncture, herbal remedies, massage, acupressure, qigong, and moxibustion. Ayurveda incorporates a combination of remedies, such as herbs, purgatives, and oils to treat disease. DIF: Understand REF: 817 (Table 35-1) OBJ: Describe the various treatments associated with complementary and alternative medicine. TOP: Assessment MSC: NCLEX: Health Promotion and Maintenance 7. A patient asks about the new clinic in town that is staffed by allopathic and complementary
practitioners. What type of clinic would this be? a. Integrative medical clinic. b. Ayurvedic clinic. c. Naturopathic medical clinic. d. Healing intention clinic. ANS: A
NURSING TB.COM
An integrative medical program allows health care consumers to be treated by a team of providers consisting of both allopathic and complementary practitioners. The other options are solely complementary clinics. DIF: Understand REF: 817 OBJ: Identify the differences between complementary and alternative approaches in health care. TOP: Planning MSC: NCLEX: Health Promotion and Maintenance 8. Which of the following can be described as a range of practices that relax the body and calm
the mind? a. Mindfulness. b. Imagery. c. Meditation. d. Biofeedback. ANS: C
Meditation refers to a range of practices that relax the body and calm the mind. Mindfulness helps build awareness of the present moment with an attitude of openness. With imagery, patients treat pathological conditions by concentrating on an image or series of images. Biofeedback involves the use of instruments to provide a person with visual or auditory information about autonomic physiological functions. DIF: Understand
REF: 817 (Table 35-1)
Canadian Fundamentals of Nursing 6th Edition Potter Test Bank OBJ: Describe the various treatments associated with complementary and alternative medicine. TOP: Planning MSC: NCLEX: Health Promotion and Maintenance 9. A therapeutic touch practitioner scans the patient’s body to identify what? a. Blocked chakra. b. Accumulated tension. c. The flow of qi. d. Structural and functional imbalance. ANS: B
The therapeutic touch practitioner scans the body to identify areas of accumulated tension. The practitioner will then attempt to redirect these accumulated energies back into balance. Chiropractic therapy involves balancing structural and functional imbalance through spinal manipulation. Qi is involved in traditional Chinese medicine. Chakras are involved in reiki therapy. DIF: Understand REF: 822 OBJ: Describe the various treatments associated with complementary and alternative medicine. TOP: Planning MSC: NCLEX: Health Promotion and Maintenance 10. Which of the following diets is believed to have anticancer properties and consists of 40 to
60% whole cereal grain, 20 to 30% vegetables, and 5 to 10% beans? a. Macrobiotic diet. b. Gerson therapy. c. Mediterranean diet. d. Paleo diet. ANS: A
NURSINGTB.COM
The macrobiotic diet is a predominantly vegan diet (but it also includes white meat fish, occasional fruits, seeds, and nuts) believed to have anticancer properties. The diet consists of 40 to 60% whole cereal grain, 20 to 30% vegetables, and 5 to 10% beans. The Gerson therapy advocates a low-salt, high-potassium organic diet of fruit juices, raw vegetables, and nutritional supplements and is used primarily in the treatment of cancer. The Mediterranean diet is high in whole grains, nuts, fruits, vegetables, and omega-3 essential fatty acids, producing a protective benefit for cardiac health and various inflammatory diseases. The Paleo diet does not include grains or legumes. DIF: Understand REF: 817 (Table 35-1) OBJ: Describe the various treatments associated with complementary and alternative medicine. TOP: Planning MSC: NCLEX: Health Promotion and Maintenance 11. What is a basic foundational principle of chiropractic care? a. Structure and function coexist. b. Chiropractic care is risk free. c. Human beings need external hands-on care to be healthiest. d. Subluxation will cause permanently restricted joint movement. ANS: A
Canadian Fundamentals of Nursing 6th Edition Potter Test Bank Chiropractors use their hands as instruments to restore structural and functional balance. Practitioners of chiropractic care believe that general health is affected via the nervous system. Chiropractic care is not risk free, just as allopathic medical care is not risk free. Subluxation eventually can cause permanently restricted joint movement, but this is not a basic foundational principle of chiropractic care. Although hands-on care is used in chiropractic care, a natural diet and regular exercise are critical components for the body to function properly. DIF: Understand REF: 822 OBJ: Understand how complementary and alternative approaches assist in health promotion and disease prevention. TOP: Planning MSC: NCLEX: Physiological Integrity 12. A holistic nurse would be a nurse who does what? a. Recommends a vegan diet for all patients. b. Recognizes the mind-body-spirit connection. c. Provides spiritual literature to patients. d. Knows about resources for fresh herbs. ANS: B
The mind-body-spirit connection is important to a nurse with a holistic style of nursing. Nursing involves caring for the entire patient. A vegan diet is an aspect of dietary treatment, but it does not allow for alternative viewpoints or well-rounded care. Spiritual literature and knowing about resources are excellent alternative aspects of allopathic medicine, but they are not specific to holistic nurses. DIF: Understand REF: 819 OBJ: Identify the differences between complementary and alternative approaches in health care. IeNalG TOP: Planning MSC: NN CLUER XS :H thT PB ro. mC otO ionMand Maintenance 13. Drawbacks of complementary and alternative therapies would be all of the following except
which one? a. Lack of evidence-informed guidelines. b. Minimal supportive research studies. c. Strong support by allopathic medical providers. d. Lack of a long tradition of therapies taught in health care education. ANS: C
Allopathic medical providers have not been traditionally educated in complementary and alternative therapies; however, that is beginning to change with courses included in medical and nursing curricula. Many allopathic providers feel uncomfortable recommending CAM because of that lack of knowledge and training. In addition, research on CAM is limited, which makes guidelines difficult to create; this is another obstacle to the use of CAM, but the research base is beginning to expand. DIF: Understand REF: 824 OBJ: Identify the differences between complementary and alternative approaches in health care. TOP: Planning MSC: NCLEX: Health Promotion and Maintenance 14. Physiological signs of a stress response include all of the following except which one? a. Constricted pupils. b. Tachycardia.
Canadian Fundamentals of Nursing 6th Edition Potter Test Bank c. Tachypnea. d. Elevated blood pressure. ANS: A
Pupils dilate during stressful situations to increase visual capacity and sight, especially in darkened conditions; this is a survival mechanism. The physiological cascade of changes associated with the stress response includes increased heart and respiratory rates, elevated blood pressure, muscle tightening, increased metabolic rate, a sense of foreboding, fear, nervousness, irritability, and a negative mood. DIF: Understand REF: 821 OBJ: Explain the scope of naturopathic and chiropractic medicine. TOP: Planning MSC: NCLEX: Physiological Integrity 15. Which of the following is a long-term outcome for an individual who is learning relaxation
therapy? a. Identifying tension in the body and consciously releasing the tension. b. Having no tension in his or her life. c. Increasing delta brain activity. d. Increasing the focus on himself or herself. ANS: A
Long-term relaxation therapy focuses on active recognition and release of stress. It is not realistic to expect a tension-free life. Delta brain waves are high-amplitude brain waves associated with the deepest stages of sleep. The outcome of relaxation therapy is not to put a person to sleep or to increase the focus on self. DIF: Understand REF: 82N 1URSINGTB.COM OBJ: Understand how complementary and alternative approaches assist in health promotion and disease prevention. TOP: Planning MSC: NCLEX: Psychosocial Integrity 16. What is one benefit of meditation over other forms of behavioural therapy? a. Meditation improves communication skills. b. Meditation cures hypertension. c. Meditation does not require memorization. d. Meditation balances insulin and other body hormones. ANS: C
Meditation involves relaxing the body and stilling the mind, which anyone can do through a variety of measures. Meditation does not improve a person’s communication skills, cure any illness, or balance any bodily hormones. DIF: Understand REF: 821 OBJ: Understand how complementary and alternative approaches assist in health promotion and disease prevention. TOP: Planning MSC: NCLEX: Psychosocial Integrity 17. Which one of the following meals would be high in phytochemicals? a. Sandwich with whole-grain bread and lean turkey. b. Grilled salmon and whole-grain rice. c. Spinach salad with roasted beets. d. Chicken breast and mashed potatoes.
Canadian Fundamentals of Nursing 6th Edition Potter Test Bank ANS: C
Phytochemicals can be found in several plant-based foods, such as spinach, carrots, oranges, beets, red peppers, blueberries, and more. The Canadian Cancer Society advises choosing a variety of colourful foods each day. DIF: Apply REF: 824 OBJ: Identify the differences between complementary and alternative approaches in health care. TOP: Planning MSC: NCLEX: Health Promotion and Maintenance 18. Which substance could cause an abnormal drug interaction in a patient taking an
antidepressant medication? a. Digoxin. b. Aspirin. c. Chamomile. d. Ginger. ANS: C
Chamomile is known to cause drowsiness. Other herbal supplements can affect serotonin levels in the brain and can affect antidepressant medication. A patient should check with a provider before combining herbal supplements with medications. Antidepressants do not interact with digoxin, aspirin, or ginger. DIF: Understand REF: 825 OBJ: Recognize botanical treatments as an approach to complementary and alternative medicine. TOP: Planning MSC: NCLEX: Physiological Integrity 19. Which of the following statements is true concerning the regulation of herbal therapies in
Canada? NURSINGTB.COM a. Herbal remedies are harmless and do not cause side effects because they are natural plants. b. A natural health product that has been approved under the Canadian Food and Drugs Act will carry a Drug Identification Number (DIN) on its label. c. All herbal therapy manufacturers must follow strict quality control and manufacturing guidelines. d. There is no need to be concerned about concurrent use of herbal therapies and prescription or over-the-counter medications. ANS: B
The federal Food and Drugs Act states that all drugs must be proved safe and effective before they can be sold to the public. Natural health products that have been approved for sale under the new regulations have been assigned a DIN (DIN-HM for homeopathic medicines) or a Natural Product Number (NPN). These numbers certify that the product has passed a review of their formulation, labelling, and instructions for use. Health Canada advises Canadians to use only health products that carry a DIN, DIN-HM, or NPN on the label. Many people believe, incorrectly, that because herbs are natural plants, they will not cause harm or side effects. Not all companies follow strict quality control and manufacturing guidelines, which are standards for acceptable levels of pesticides, residual solvents, bacterial levels, and heavy metals. For this reason, herbal medicine should be purchased from reputable manufacturers. Concurrent use of herbal or other natural products with prescription or over-the-counter medications should be monitored. Herbs can inhibit or enhance a particular medication’s site of action.
Canadian Fundamentals of Nursing 6th Edition Potter Test Bank
DIF: Apply REF: 825 OBJ: Identify the differences between complementary and alternative approaches in health care. TOP: Assessment MSC: NCLEX: Safe and Effective Care Environment 20. The complementary and alternative therapy that is known to alter immune function is which
of the following? a. Biofeedback. b. Imagery. c. Breath work. d. Acupuncture. ANS: B
Imagery and visualization techniques produce a powerful psychophysiological response. Imagery is commonly used by patients with cancer to improve their immune system despite radical treatments such as chemotherapy. Biofeedback teaches the patient to analyze triggers of stress. Acupuncture involves puncture of the skin, which may increase risk of infection, and should be used with caution in someone who is immunocompromised. Breath work involves using a variety of breathing patterns to relax, invigorate, or open emotional channels. DIF: Understand REF: 821 OBJ: Describe the various treatments associated with complementary and alternative medicine. TOP: Planning MSC: NCLEX: Psychosocial Integrity
NURSINGTB.COM
Canadian Fundamentals of Nursing 6th Edition Potter Test Bank
Chapter 36: Activity and Exercise Potter et al: Canadian Fundamentals of Nursing, 6th Edition MULTIPLE CHOICE 1. The coordinated efforts of the musculoskeletal and nervous system maintain balance, posture,
and body alignment. What does body alignment refer to? a. A low centre of gravity balanced over a wide base of support. b. The result of weight, centre of gravity, and balance. c. The relationship of one body part to another. d. The force that occurs in a direction to oppose movement. ANS: C
Body alignment refers to the relationship of one body part to another body part along a horizontal or vertical line. Body balance is the state in which a relatively low centre of gravity is balanced over a wide, stable base of support. Coordinated body movement is a result of weight, centre of gravity, and balance. Friction is a force that occurs in a direction to oppose movement. DIF: Remember REF: 833 OBJ: Describe the role of the musculoskeletal and nervous systems in the regulation of activity and exercise. TOP: Assessment MSC: NCLEX: Physiological Integrity 2. A structural curvature of the spine associated with vertebral rotation is known as which of the
following? a. Scoliosis. b. Osteogenesis. c. Osteomalacia. d. Arthritis.
NURSINGTB.COM
ANS: A
Scoliosis is a structural curvature of the spine associated with vertebral rotation. Osteogenesis imperfecta is an inherited disorder in which bones are porous, short, bowed, and deformed. Osteomalacia is an uncommon metabolic disease characterized by inadequate and delayed mineralization; as a result, bone is compact and spongy. Arthritis is an inflammatory joint disease characterized by inflammation or destruction of the synovial membrane and articular cartilage and by systemic signs of inflammation. DIF: Remember REF: 837 OBJ: Discuss physiological and pathological influences on body alignment and joint mobility. TOP: Assessment MSC: NCLEX: Physiological Integrity 3. How is joint degeneration unlike arthritis? a. Joint degeneration results only from noninflammatory disease. b. Joint degeneration results only from inflammatory disease. c. Joint degeneration involves overgrowth of bone at the articular ends. d. Joint degeneration affects mostly non–weight-bearing joints. ANS: C
Canadian Fundamentals of Nursing 6th Edition Potter Test Bank Joint degeneration, which can occur with inflammatory and noninflammatory disease, is marked by changes in articular cartilage combined with overgrowth of bone at the articular ends. Degenerative changes commonly affect weight-bearing joints. DIF: Understand REF: 837 OBJ: Discuss physiological and pathological influences on body alignment and joint mobility. TOP: Assessment MSC: NCLEX: Physiological Integrity 4. The nurse is providing care to a patient who is bedridden. To avoid becoming fatigued, the
nurse raises the height of the patient’s bed. The nurse understands that raising the bed helps maintain balance in what way? a. It prevents a shift in the nurse’s base of support. b. It narrows the nurse’s base of support. c. It allows the nurse to bring his or her feet close together. d. It shifts the centre of gravity farther away from the nurse’s base of support. ANS: A
By raising the height of the bed when performing a procedure, the nurse prevents bending too far at the waist and causing a shift in the base of support. Balance is maintained by maintaining proper body alignment and posture through two simple techniques: first, the base of support is widened by separating the feet to a comfortable distance; second, balance is increased by the shift of the centre of gravity closer to the base of support. DIF: Understand REF: 841 OBJ: Discuss physiological and pathological influences on body alignment and joint mobility. TOP: Assessment MSC: NCLEX: Physiological Integrity 5. Approximately what percentaNgU eR ofSaI ll N baGcT kB pa. inCiO s aMssociated with manual lifting tasks? a. More than 10%. b. More than 20%. c. More than 40%. d. More than 50%. ANS: D
More than half of all cases of back pain are associated with manual lifting tasks. DIF: Remember REF: 859 OBJ: Describe how to maintain and use proper body mechanics. TOP: Assessment MSC: NCLEX: Physiological Integrity 6. The nurse is preparing to position an immobile patient. Before doing so, the nurse must
understand which of the following? a. Manual lifting is the easier method and should be tried first. b. Following body mechanics principles alone will prevent back injury. c. Body mechanics can be ignored when patient-handling equipment is used. d. Body mechanics alone are not sufficient to prevent injuries. ANS: D
Canadian Fundamentals of Nursing 6th Edition Potter Test Bank Body mechanics alone are not sufficient to prevent musculoskeletal injuries when the nurse positions or transfers patients. The use of patient-handling equipment in combination with proper body mechanics is more effective than either one in isolation. Body mechanics cannot be ignored even when patient-handling equipment is being used. Manual lifting is the last resort, and it is used only when it does not involve lifting most or all of the patient’s weight. DIF: Understand REF: 859 OBJ: Describe how to use proper body mechanics and ergonomics to prevent musculoskeletal injuries. TOP: Assessment MSC: NCLEX: Physiological Integrity 7. The nurse is preparing to reposition a patient. Before doing so, what must the nurse do? a. Assess the weight to be lifted and the assistance needed. b. Attempt to manually lift the patient alone before asking for assistance. c. Attempt a manual lift only when lifting most or all of the patient’s weight. d. Not use the agency lift team if a mechanical lift is available. ANS: A
Before lifting, assess the weight to be lifted and determine the assistance needed and the resources available. Manual lifting is the last resort, and it is used when the task at hand does not involve lifting most or all of the patient’s weight. Use safe patient-handling equipment in conjunction with agency lift teams to reduce the risk of injury to the patient and members of the health care team. DIF: Apply REF: 859 OBJ: Describe how to maintain and use proper body mechanics. TOP: Implementation MSC: NCLEX: Physiological Integrity
IeNtrGicTaBre.thCrO 8. Isotonic, isometric, and resisN tivUeR isS om eeMcategories of exercise. They are classified according to the type of muscle contraction involved. Of the following exercises, which are considered isotonic? a. Bicycling, swimming, walking, jogging, dancing. b. Tightening or tensing of muscles without moving body parts. c. Push-ups, hip lifting, pushing feet against a footboard on the bed. d. Quadriceps set exercises and contraction of the gluteal muscles. ANS: A
Examples of isotonic exercises are walking, swimming, dance aerobics, jogging, bicycling, and moving arms and legs with light resistance. Isometric exercises involve tightening or tensing of muscles without moving body parts. Examples include quadriceps set exercises and contraction of the gluteal muscles. Examples of resistive isometric exercises are push-ups and hip lifting, as well as placing a footboard on the foot of the bed for patients to push against with their feet. DIF: Remember REF: 834 OBJ: Describe the benefits of implementing an exercise program for the purpose of health promotion. TOP: Assessment MSC: NCLEX: Physiological Integrity 9. In planning a physical activity program for a patient, what must the nurse must understand? a. Isotonic exercises cause contraction without changing muscle length. b. The best program includes a combination of exercises. c. Isometric contraction involves the movement of body parts.
Canadian Fundamentals of Nursing 6th Edition Potter Test Bank d. Resistive isometric exercises can lead to bone wasting. ANS: B
The best program of physical activity includes a combination of exercises that produce different physiological and psychological benefits. Isotonic exercises cause muscle contractions and changes in muscle length. Isometric exercises involve tightening or tensing of muscles without moving body parts. Resistive isometric exercises help promote muscle strength and provide sufficient stress against bone to promote osteoblastic activity. DIF: Understand REF: 834 OBJ: Describe the benefits of implementing an exercise program for the purpose of health promotion. TOP: Assessment MSC: NCLEX: Health Promotion and Maintenance 10. An active lifestyle is important for maintaining and promoting health. In developing an
exercise program, what should the nurse know? a. Physical exercise is contraindicated for patients with chronic illnesses. b. Regular physical activity is beneficial only for the body part that is exercised. c. Physical exercise has no effect on psychological well-being. d. Physical activity enhances functioning of all body systems. ANS: D
Regular physical activity and exercise enhance the functioning of all body systems, including cardiopulmonary functioning, musculoskeletal fitness, weight control and maintenance, and psychological well-being. It is also essential in treatment for chronic illness. DIF: Understand REF: 834 OBJ: Describe the benefits of implementing an exercise program for the purpose of health promotion. TOP: AssessmeN nt R I GMSB C:.C NCLMEX: Health Promotion and Maintenance
U S N T
O
11. The nurse is developing an exercise program for older patients living in a nursing home. To
develop a beneficial health promotion program, what must the nurse need to understand about older people? a. Exercise is of very little benefit because the patients are old. b. It is important to disregard their current interests in favour of exercise. c. No physical benefit can be gained without a formal exercise program. d. Adjustments to exercise programs may have to be made to prevent problems. ANS: D
Exercise is extremely beneficial for older persons, but adjustments to an exercise program may have to be made for those of advanced age to prevent problems. When developing an exercise program for any older person, the nurse should consider not only the person’s current activity level, range of motion, muscle strength and tone, and response to physical activity but also the person’s interests, capacities, and limitations. Older persons who are unable to participate in a formal exercise program are able to achieve the benefits of improved joint mobility and enhanced circulation by simply stretching and exaggerating movements during performance of routine activities of daily living. DIF: Understand REF: 849| 860 OBJ: Describe important factors to consider when planning an exercise program for patients across the lifespan and for those with specific chronic illnesses. TOP: Assessment MSC: NCLEX: Physiological Integrity
Canadian Fundamentals of Nursing 6th Edition Potter Test Bank 12. The nurse is attempting to start an exercise program in a local community as a health
promotion project. In explaining the purpose of the project, what does the nurse explain to community leaders? a. A sedentary lifestyle contributes to the development of health-related problems. b. The recommended frequency of workouts should be twice a day. c. An exercise prescription should incorporate aerobic exercise only. d. The purpose of weight training is to bulk up muscles. ANS: A
A sedentary lifestyle contributes to the development of health-related problems. A holistic approach is taken to develop overall fitness and includes warm-ups, aerobic exercise, resistance training, weight training, and so forth. The recommended frequency of aerobic exercise is three to five times per week, or every other day, for approximately 30 minutes. Cross-training is recommended for the patient who prefers to exercise every day. Some patients use weight training to bulk up their muscles. However, the purposes of weight training from a health perspective are to develop tone and strength and to simulate and maintain healthy bone. DIF: Understand REF: 856 OBJ: Describe the benefits of implementing an exercise program for the purpose of health promotion. TOP: Assessment MSC: NCLEX: Health Promotion and Maintenance 13. The patient is eager to begin his exercise program with a 3.2-km (2-mile) jog. The nurse
instructs the patient to warm up with stretching exercises. The patient states that he is ready and does not want to waste time with a “warm-up.” What does the nurse explain about the warm-up? a. It allows the body to readjust gradually to baseline functioning. N crea RSses INthe GTpotenti B.CO M b. It prepares the body and deU al for injury. c. It should not involve stretching exercises because they can lead to injury. d. It should be performed with high intensity to prepare for the coming challenge. ANS: B
The warm-up activity prepares the body for activity and decreases the potential for injury. It usually lasts about 5 to 10 minutes and may include stretching, calisthenics, or aerobic activity, or a combination of these, performed at a low intensity. The cool-down period allows the body to readjust gradually to baseline functioning and provides an opportunity to combine movement such as stretching with relaxation-enhancing mind-body awareness. DIF: Understand REF: 856 OBJ: Describe the benefits of implementing an exercise program for the purpose of health promotion. TOP: Assessment MSC: NCLEX: Health Promotion and Maintenance 14. Many patients find it difficult to incorporate an exercise program into their daily lives because
of time constraints. For these patients, it is beneficial to reinforce that many activities of daily living (ADLs) are used to accumulate the recommended 30 minutes or more per day of moderate-intensity physical activity. When instructing these patients, what should the nurse explain? a. Housework is not considered an aerobic exercise. b. To strengthen back muscles, the patient should bend using back muscles. c. Daily chores should begin with gentle stretches. d. The patient should stick to one chore until it is done before beginning a new one.
Canadian Fundamentals of Nursing 6th Edition Potter Test Bank
ANS: C
Daily chores should begin with gentle stretches. Housework is considered aerobic exercise. To make it more aerobic, the person can work faster and scrub harder. People should bend their legs rather than their back to prevent back injury. Cleaning activities should be alternated to prevent overworking the same muscle groups. DIF: Understand REF: 856 OBJ: Describe important factors to consider when planning an exercise program for patients across the lifespan and for those with specific chronic illnesses. TOP: Assessment MSC: NCLEX: Health Promotion and Maintenance 15. The nurse is developing an exercise plan for a patient with heart failure and a nursing
diagnosis of Exercise intolerance. In doing so, what should the nurse include? a. Plan for 20 minutes of continuous aerobic activity and increase as tolerated. b. Perform 5-minute walks at the patient’s pace at least 2 times a day. c. Instruct the patient that he should not take his -adrenergic blocker medication on exercise days. d. Encourage a high-calorie diet to plan for extra calorie expenditure. ANS: B
A patient with the nursing diagnosis of Exercise intolerance should begin by performing 5-minute walks at his own pace at least twice a day. The patient would not be able to tolerate 20 minutes of continuous aerobic activity. Patients should be instructed to take medications as ordered. Low-calorie, low-sodium, and high-protein diets are best for this type of patient. DIF: Apply REF: 857 OBJ: Describe important factorsNto R consider planning program for patients across Ironi Gwhen B.C M an exercise T the lifespan and for those with speUcifiS c chN c illnessO es. TOP: Implementation MSC: NCLEX: Health Promotion and Maintenance 16. Which of the following exercise activities would probably provide the opportunity for
mind-body awareness? a. Warm-up activity. b. Resistance training. c. Aerobic exercise. d. Cool-down activity. ANS: D
The cool-down period allows the body to readjust gradually to baseline functioning and provides an opportunity to combine movement such as stretching with relaxation-enhancing mind-body awareness. The warm-up activity prepares the body and decreases the potential for injury. Aerobic exercise includes running, bicycling, and jumping rope and is the main portion of exercise activity; it precedes the cool-down period. Resistance training increases muscle strength and endurance and is associated with improved performance of daily activities but not with enhancing mind-body awareness. DIF: Analyze REF: 860 OBJ: Describe important factors to consider when planning an exercise program for patients across the lifespan and for those with specific chronic illnesses. TOP: Evaluate MSC: NCLEX: Physiological Integrity
Canadian Fundamentals of Nursing 6th Edition Potter Test Bank 17. The patient is brought to the emergency department with possible injury to his shoulder. To
help determine the degree of injury, what should the nurse evaluate? a. The patient’s gait. b. The patient’s range of motion. c. Fine motor coordination. d. Activity tolerance. ANS: B
Evaluating range of motion is one assessment technique used to determine the degree of damage or injury to a joint. Gait is the manner or style of walking; it may have little bearing on the shoulder damage. Assessing fine motor coordination would be beneficial in helping to assess the patient’s ability to perform tasks but would not help in evaluating the shoulder. Activity tolerance refers to the type and amount of exercise or activity a person is able to perform. Damage to the shoulder would affect this, but this would not have a direct bearing on the amount of damage done to the shoulder. DIF: Apply REF: 854 OBJ: Describe how to assess patients for activity intolerance. MSC: NCLEX: Physiological Integrity
TOP: Implementation
18. The nurse is examining a patient who is admitted to the emergency department with severe
elbow pain. Of the following situations, which would cause the nurse to suspect a ligament tear or joint fracture? a. Range of motion of the elbow is limited. b. Joint motion is greater than normal. c. The patient has arthritis. d. The elbow cannot be moved (frozen). B.C M ANS: B
N R I G U S N T
O
Increased mobility (beyond normal) of a joint may indicate connective tissue disorders, ligament tears, or possible joint fractures. Limited range of motion often indicates inflammation such as arthritis, fluid in the joint, altered nerve supply, or contractures (frozen joints). DIF: Analyze REF: 854 OBJ: Describe how to assess patients for activity intolerance. MSC: NCLEX: Physiological Integrity
TOP: Evaluate
19. The patient has been bedridden for several months because of severe heart disease. In
determining a plan of care for this patient that will address his activity level, the nurse formulates which of the following nursing diagnoses? a. Fatigue related to poor physical condition. b. Impaired gas exchange related to decreased cardiac output. c. Decreased cardiac output related to decreased myocardial contractility. d. Activity intolerance related to physical deconditioning. ANS: D
Canadian Fundamentals of Nursing 6th Edition Potter Test Bank When activity and exercise are problematic for a patient, nursing diagnoses often focus on the individual’s ability to move. The diagnostic label directs nursing interventions. In this case, physical deconditioning must be addressed in relation to activity level; perhaps an early goal is 6-minute walks twice a day. Physical deconditioning is the cause of fatigue as well, so it would take priority over a diagnosis of fatigue. Decreased cardiac output and myocardial contractility are serious concerns that must be addressed before activity intolerance to keep the patient safe and to help determine the level of exercise that the patient can tolerate, but reconditioning of the patient’s body will help improve contractility and cardiac output. DIF: Apply REF: 833| 837 OBJ: Develop a nursing care plan for a patient with impaired mobility and activity intolerance. TOP: Implementation MSC: NCLEX: Physiological Integrity 20. The patient weighs 204.5 kg (450 pounds) and complains of shortness of breath with any
exertion. His health care provider has recommended that he begin an exercise program. He states that he can hardly get out of bed and just cannot do anything around the house. To focus on the cause of the patient’s complaints, the nurse devises which of the following nursing diagnoses? a. Activity intolerance related to excessive weight. b. Activity intolerance related to bed rest. c. Impaired gas exchange related to shortness of breath. d. Imbalanced nutrition: less than body requirements. ANS: A
The diagnostic label directs nursing interventions. This requires the correct selection of related factors. For example, Activity intolerance related to excess weight requires very different interventions than if the related factor is prolonged bed rest. In this case, the intolerance is N R eigh INt.GHTeB.C M related to the patient’s excessivUe wS is noO t on bed rest, although he states that it is difficult for him to get out of bed. Shortness of breath is a symptom, not a cause, of Impaired gas exchange, and so this nursing diagnosis is not applicable. The patient certainly has an imbalance of nutrition, but it is more than body requirements. DIF: Apply REF: 855 OBJ: Develop a nursing care plan for a patient with impaired mobility and activity intolerance. TOP: Implementation MSC: NCLEX: Physiological Integrity 21. To promote resistive isometric exercise, the nurse helps the patient on bed rest by encouraging
which one of the following? a. Quadriceps setting. b. Gluteal muscle contraction. c. Moving the arms and legs in circles. d. Pushing against a footboard. ANS: D
In resistive isometric exercises, the individual contracts the muscle while pushing against a stationary object or resisting the movement of an object. An example of a resistive isometric exercise is pushing against a footboard. Quadriceps setting is an example of an isometric exercise. Gluteal muscle contraction is an example of an isometric exercise. Moving the arms and legs in a circle is an example of isotonic exercise. DIF: Apply
REF: 834
Canadian Fundamentals of Nursing 6th Edition Potter Test Bank OBJ: Develop a nursing care plan for a patient with impaired mobility and activity intolerance. TOP: Implementation MSC: NCLEX: Physiological Integrity 22. The patient is admitted with a stroke. The prognosis is uncertain, but the patient is unable to
move his right arm and leg. What should the nurse keep in mind? a. Active range of motion is the only thing that will prevent contractures from forming. b. Passive range of motion must be instituted to help prevent contracture formation. c. Range-of-motion exercises should be started 2 days after the patient is stable. d. Range-of-motion exercises should be done on major joints only. ANS: B
When patients cannot participate in active range of motion, passive range of motion must be instituted to maintain joint mobility and prevent contractures. Passive range-of-motion exercises can be substituted for active exercises when needed. For the patient who does not have voluntary motor control, passive range-of-motion exercises are the exercises of choice. Unless contraindicated, the nursing care plan includes exercising each joint (not just major joints) through as nearly a full range of motion as possible. The nurse should initiate passive range-of-motion exercises as soon as the patient loses the ability to move the extremity or joint. DIF: Understand REF: 838| 861 OBJ: Describe the interventions for maintaining activity tolerance and mobility during the acute, restorative, and continuing care of patients. TOP: Assessment MSC: NCLEX: Physiological Integrity 23. A 40-year-old healthy woman has presented to the clinic for a physical examination. When
.C the patient asks the nurse to rN ecUoR mS mI enNdGthTeBam ouO nM t of time she should spend per week doing moderate-intensity physical activity, what should the nurse recommend? a. One hour per week or 15 minutes a day for 4 days. b. Thirty minutes or more a day, for a weekly total of 150 minutes. c. One hour per day, for a weekly total of 7 hours. d. Ten minutes a day, for a total of 60 minutes per week. ANS: B
According to the Canadian Diabetes Association, adults should accumulate 150 minutes or more per week of moderate-intensity (brisk) physical activity. One hour per week, or 15 minutes a day for four days, and 10 minutes a day, for a total of 1 hour per week, are not considered adequate amounts of time for exercise for adults. One hour per day, for a weekly total of 7 hours, exceeds the recommended amount of exercise time for adults, but the statement does not specify the intensity of the physical activity. DIF: Apply REF: 862 (Table 36-3) OBJ: Describe the interventions for maintaining activity tolerance and mobility during the acute, restorative, and continuing care of patients. TOP: Implementation MSC: NCLEX: Health Promotion and Maintenance 24. The nurse is ambulating a patient in the hall when she notices that he is beginning to fall.
What should the nurse do? a. Grab the patient and hold him tight to prevent the fall. b. Gently lower the patient to the floor.
Canadian Fundamentals of Nursing 6th Edition Potter Test Bank c. Jump back and let the patient fall naturally. d. Push the patient against the wall and guide him to the floor. ANS: B
If the patient has a fainting episode or begins to fall, assume a wide base of support with one foot in front of the other, thus supporting the patient’s body weight. Then extend one leg and let the patient slide against the leg, and gently lower the patient to the floor, protecting the patient’s head. Grabbing the patient will shift the nurse’s centre of gravity and may lead to a back injury. Allowing the patient to fall could lead to head injury for the patient. Pushing the patient against the wall could also cause the patient to hit his head and cause injury. DIF: Apply REF: 862 OBJ: Describe how to use proper body mechanics and ergonomics to prevent musculoskeletal injuries. TOP: Implementation MSC: NCLEX: Safe and Effective Care Environment 25. In assisting the patient to exercise, which of the following should the nurse do? a. Expect that pain will occur with exercise of unused muscle groups. b. Set the pace for the exercise. c. Force muscles or joints to go just beyond resistance. d. Stop the exercise if the patient experiences pain. ANS: D
The nurse should assess for pain, shortness of breath, or a change in vital signs. If any of these is present, the exercise should be stopped. Each patient should be allowed to exercise at his or her own pace. The nurse assesses for joint limitations and must not force a muscle or a joint during exercise.
N R I G B.C M
U S N T O DIF: Apply REF: 860 OBJ: Describe the interventions for maintaining activity tolerance and mobility during the acute, restorative, and continuing care of patients. TOP: Implementation MSC: NCLEX: Health Promotion and Maintenance 26. The nurse is developing a plan of care for a patient with a diagnosis of Activity intolerance. Of
the following strategies, which has the best chance of maintaining patient compliance? a. Performing 20 minutes of aerobic exercise daily with 10-minute warm-up and cool-down periods b. Instructing the patient to use an exercise log to record day, time, duration, and responses to exercise activity c. Instructing the patient on the evils of not exercising, and getting her to take responsibility for her current health status d. Arranging for the patient to join a gym for which she will have to pay, so that she does not need to depend on insurance ANS: B
Keeping a log may increase adherence to an exercise prescription. Cross-training (combination of exercise activities) provides variety to combat boredom and increases the potential for total body conditioning, as opposed to daily aerobic exercise. “Blaming” a patient for his or her health status is usually counterproductive. Instead, the nurse should instruct the patient about the physiological benefits of a regular exercise program. Developing a plan of exercise that the patient may perform at home may improve compliance.
Canadian Fundamentals of Nursing 6th Edition Potter Test Bank DIF: Evaluate REF: 858 OBJ: Evaluate the nursing care plan for maintaining activity and exercise for patients across the lifespan and with specific chronic illnesses. TOP: Evaluate MSC: NCLEX: Health Promotion and Maintenance 27. The nurse is working with the patient in developing an exercise plan. The patient tells the
nurse that she just will not participate in a formal exercise program. The nurse then suggests that exercise activities can be incorporated into activities of daily living. The patient seems to be agreeable to that concept. Of the following activities, which would be considered a formal resistance training activity? a. Doing laundry. b. Making the bed. c. Ironing. d. Kneading bread. ANS: D
Formal resistance training includes weight training, but the same benefits can be obtained by performing ADLs such as pushing a vacuum cleaner, raking leaves, shovelling snow, and kneading bread. DIF: Evaluate REF: 856 OBJ: Evaluate the nursing care plan for maintaining activity and exercise for patients across the lifespan and with specific chronic illnesses. TOP: Evaluate MSC: NCLEX: Health Promotion and Maintenance MULTIPLE RESPONSE 1. Bones perform five functionsNiU nR thS eI boNdG y:TsuBp. poCrO t, M protection, movement, mineral storage,
and hematopoiesis. In the discussion of body mechanics, which are the most important? (Select all that apply.) a. Support. b. Protection. c. Movement. d. Mineral storage. e. Hematopoiesis. ANS: A, C
Bones perform five functions in the body: support, protection, movement, mineral storage, and hematopoiesis. In the discussion of body mechanics, two of these functions—support and movement—are most important. In support, bones serve as the framework and contribute to the shape, alignment, and positioning of body parts. In movement, bones together with their joints constitute levers for muscle attachment. As muscles contract and shorten, they pull on bones, producing joint movement. Protection involves encasing the soft tissue organs in a protective cage. Mineral storage helps to strengthen bones but also helps regulate blood levels of certain nutrients. Hematopoiesis is the formation of blood cells. DIF: Remember REF: 834 OBJ: Describe the role of the musculoskeletal and nervous systems in the regulation of activity and exercise. TOP: Assessment MSC: NCLEX: Physiological Integrity
Canadian Fundamentals of Nursing 6th Edition Potter Test Bank 2. When assessing the activity tolerance of a patient, the nurse would evaluate which of the
following? (Select all that apply.) a. Skeletal abnormalities b. Emotional factors. c. Age. d. Pregnancy status. e. Race. ANS: A, B, C, D
Factors influencing activity tolerance include physiological factors such as skeletal abnormalities, emotional factors such as anxiety/depression, developmental factors such as age and gender, and pregnancy status. Race is not a factor because people of all races are faced with similar factors that affect their activity tolerance. DIF: Apply REF: 854 (Box 36-7) OBJ: Describe how to assess patients for activity intolerance. MSC: NCLEX: Physiological Integrity
TOP: Implementation
3. In developing a nursing care plan for increasing activity tolerance in a patient, what should the
nurse include? (Select all that apply.) a. Use of generalized therapies because they work for everyone. b. Consultation with members of the health care team. c. Avoidance of isometric exercise. d. Involving the patient and the patient’s family in designing an exercise plan. e. Consideration of the patient’s ability to increase activity level. ANS: B, D, E
When planning care, the nursN eU shRoul coGnsu /cC olla borate with members of the health care SId N TBlt. OM team to increase activity, involve the patient and family in designing an activity and exercise plan (especially if family members are also providers of care), and consider the patient’s ability to increase activity level. Therapies should be individualized to the patient’s activity tolerance. The best program of physical activity includes a combination of isotonic, isometric, and resistive isometric exercises, if tolerated by the patient. DIF: Apply REF: 856 OBJ: Develop a nursing care plan for a patient with impaired mobility and activity intolerance. TOP: Implementation MSC: NCLEX: Health Promotion and Maintenance
Canadian Fundamentals of Nursing 6th Edition Potter Test Bank
Chapter 37: Quality and Patient Safety Potter et al: Canadian Fundamentals of Nursing, 6th Edition MULTIPLE CHOICE 1. A home health nurse is performing a home assessment for safety. Which of the following
comments by the patient would indicate a need for further education? a. “I will schedule an appointment with a chimney inspector next week.” b. “Daylight savings is the time to change batteries on the carbon monoxide detector.” c. “If I feel dizzy when using the heater, I need to have it inspected.” d. “If I’m cooking for only myself, I don’t need to wash my hands.” ANS: D
Nurses should teach basic techniques for safe food handling, such as hand hygiene and checking spoilage. Hand hygiene should be done before all food preparation. Checking the chimney and heater, changing the batteries on the detector, and following up on symptoms such as dizziness, nausea, and fatigue are all statements that would indicate that the individual has understood the education. DIF: Apply REF: 875 OBJ: Discuss methods to prevent and reduce safety risks. MSC: NCLEX: Health Promotion and Maintenance
TOP: Evaluate
2. The nurse is caring for an elderly patient admitted with nausea, vomiting, and diarrhea. Upon
completing the health history, which priority concern would require collaboration with social Ns he RSalth INcar GT B.C? M services to address the patient’U e needsO a. The electricity was turned off 2 days ago. b. The water comes from the county water supply. c. A son and family recently moved into the home. d. The home is not furnished with a microwave oven. ANS: A
Electricity is needed for refrigeration of food, and lack of electricity could have contributed to the nausea, vomiting, and diarrhea by potentially leading to food poisoning. This discussion about the patient’s electrical needs can be referred to social services. The water supply, the increased number of individuals in the home, and not having a microwave may or may not be concerns but do not pertain to the current health care needs of this patient. DIF: Understand REF: 888 OBJ: Discuss methods to prevent and reduce safety risks. MSC: NCLEX: Health Promotion and Maintenance
TOP: Implementation
3. A homeless adult patient presents to the emergency department. The nurse obtains the
following vital signs: temperature 34.9°C (94.8°F), blood pressure 100/56, apical pulse 56, respiratory rate 12. Which of the vital signs should be addressed immediately? a. Respiratory rate. b. Temperature. c. Apical pulse. d. Blood pressure.
Canadian Fundamentals of Nursing 6th Edition Potter Test Bank
ANS: B
Hypothermia is defined as a core body temperature of 35°C (95°F) or below. Homeless individuals are more at risk for hypothermia because of exposure to the elements. DIF: Apply REF: 888 OBJ: Describe assessment activities designed to identify patients’ physical, psychosocial, and cognitive statuses as they pertain to his or her safety. TOP: Assessment MSC: NCLEX: Physiological Integrity 4. An ambulatory patient is admitted to the extended-care facility with a diagnosis of
Alzheimer’s disease. In the Hendrich II Fall Risk Model, what is the most significant indicator of risk for falls? a. Confusion/disorientation/impulsivity. b. Dizziness/vertigo. c. Symptomatic depression. d. Altered elimination. ANS: A
According to the Hendrich II Fall Risk Model, the most significant indicator of risk for falls is confusion/disorientation/impulsivity. Dizziness/vertigo, symptomatic depression, and altered elimination are also risk factors; however, they are not the most significant indicators. DIF: Remember REF: 879 (Table 37-1) OBJ: Describe nursing interventions specific to patients’ age for reducing safety risks. TOP: Implementation MSC: NCLEX: Health Promotion and Maintenance 5. A 1-year-old child is scheduled to receive an intravenous (IV) line. The most appropriate type
NU INven GTt B.C of restraint to use for this patie ntRtoSpre removOalMof the IV line is which of the following? a. A wrist restraint. b. A jacket restraint. c. An elbow restraint. d. A mummy restraint. ANS: D
A mummy restraint is used in the short term for a small child or infant for examination or treatment involving the head and neck. This would be the most appropriate type of restraint to use for a 1-year-old who is going to receive an IV line. The wrist restraint maintains immobility of an extremity to prevent the patient from removing a therapeutic device, such as an IV tube. It would not be the best choice for starting an IV infusion on a 1-year-old. The jacket restraint is often used to prevent a patient from getting up and falling. This, too, is not the best choice for starting an IV line. An elbow restraint is commonly used with infants and children to prevent elbow flexion, as in cases in which an IV line is already in place. DIF: Apply REF: 894 OBJ: Describe nursing interventions specific to patients’ age for reducing safety risks. TOP: Implementation MSC: NCLEX: Physiological Integrity 6. A visiting nurse completes an assessment of the ambulatory patient in the home and
determines the nursing diagnosis of Risk for injury related to decreased vision. On the basis of this assessment, the patient will benefit the most from which of the following actions? a. Installing fluorescent lighting throughout the house.
Canadian Fundamentals of Nursing 6th Edition Potter Test Bank b. Evaluating the need to reposition furniture. c. Maintaining complete bed rest in a hospital bed with side rails. d. Applying physical restraints. ANS: B
Evaluating the positioning of furniture in the room and stairways is the best intervention to help prevent falls for the patient with decreased vision. Attempts should be made to reduce glare. Light bulbs that are 60 W or less may be changed to 75 W nonglare to help improve visibility. The best intervention to prevent falls is first to orient the patient to the surroundings. Maintaining complete bed rest is not the best option. Complete bed rest can cause other health problems because of a lack of mobility. The patient should not be restrained for poor vision. Attempts should be made to help compensate for the decreased vision to prevent falls. DIF: Analyze REF: 879 OBJ: Describe nursing interventions specific to patients’ age for reducing safety risks. TOP: Diagnosis MSC: NCLEX: Health Promotion and Maintenance 7. When teaching a parent about interventions for accidental poisoning, what instruction should
be included regarding flushing a child’s eye, in relation to the water temperature? a. Cold. b. Lukewarm. c. Room temperature. d. Above room temperature. ANS: C
The nurse should teach the parent that an eye flush is to be done with water at room temperature. The water for an eye flush is not to be cold, lukewarm, or above room temperature. N R I G B.C M
U S N T
O
DIF: Apply REF: 885 (Box 37-8) OBJ: Describe nursing interventions specific to patients’ age for reducing safety risks. TOP: Implementation MSC: NCLEX: Health Promotion and Maintenance 8. The nurse knows that children in late infancy and toddlerhood are at risk for injury from
which of the following? a. Learning to walk. b. Trying to pull up on furniture. c. Being dropped by a caregiver. d. Growing ability to explore and oral activity. ANS: D
Injury is a leading cause of death in children over age 1, which is closely related to normal growth and development because of the child’s increased oral activity and growing ability to explore the environment. DIF: Understand REF: 883 OBJ: Discuss methods to prevent and reduce safety risks. MSC: NCLEX: Health Promotion and Maintenance
TOP: Evaluate
9. A nurse is teaching a community group of school-aged parents about safety. The most
important item to prioritize and explain is how to check the proper fit of which of the following?
Canadian Fundamentals of Nursing 6th Edition Potter Test Bank a. b. c. d.
A bicycle helmet. Swimming goggles. Soccer shin guards. Baseball sliding shorts.
ANS: A
Bicycle-related injuries are a major cause of death and disability among children. Proper fit of the helmet helps to decrease head injuries resulting from bicycle accidents. Goggles, shin guards, and sliding shorts are important sports safety equipment and should fit properly, but they do not protect from this leading cause of death. DIF: Understand REF: 886 OBJ: Discuss methods to prevent and reduce safety risks. TOP: Implementation | Teaching | Learning MSC: NCLEX: Health Promotion and Maintenance 10. The nurse is presenting an educational session on safety for parents of adolescents. The nurse
should include which of the following teaching points? a. Adolescents need unsupervised time with friends two to three times a week. b. Parents and friends should teach adolescents how to drive. c. Adolescents need information about the effects of beer on the liver. d. Adolescents need to be reminded to use seatbelts on long trips. ANS: C
Providing information about drugs and alcohol is important because adolescents may choose to participate in risk-taking behaviours. Adolescents need to socialize but need supervision. Parents can encourage and support learning processes associated with driving, but organized classes can help to decrease m ehicG le aB cc. idC entM s. Seatbelts should be used all the time. NotoRr vI
U S N T
O
DIF: Understand REF: 877 OBJ: Discuss methods to prevent and reduce safety risks. MSC: NCLEX: Health Promotion and Maintenance
TOP: Implementation
11. The nurse discussed threats to adult safety with a college group. Which of the following
statements would indicate understanding of the topic? a. “Our campus is safe; we leave our dorms unlocked all the time.” b. “As long as I have only two drinks, I can still be the designated driver.” c. “I am young, so I can work nights and go to school with 2 hours’ sleep.” d. “I guess smoking even at parties is not good for my body.” ANS: D
Lifestyle choices frequently affect adult safety. Smoking conveys great risk for pulmonary and cardiovascular disease. It is prudent to secure belongings. When an individual has been determined to be the designated driver, that individual does not consume alcohol, beer, or wine. Sleep is important no matter the age of the individual and is important for rest and integration of learning. The average young adult needs to 8 hours of sleep each night. DIF: Understand REF: 877 OBJ: Describe nursing interventions specific to patients’ age for reducing safety risks. TOP: Evaluate MSC: NCLEX: Health Promotion and Maintenance
Canadian Fundamentals of Nursing 6th Edition Potter Test Bank 12. The nurse is caring for a hospitalized patient. Which of the following behaviours alerts the
nurse to consider the need for restraint? a. The patient refuses to call for help to go to the bathroom. b. The patient continues to remove the nasogastric tube. c. The patient gets confused regarding the time at night. d. The patient does not sleep and continues to ask for items. ANS: B
Restraints are utilized only when alternatives have been exhausted, the patient continues a behaviour that can be harmful to self or others, and the restraint is clinically justified. In this circumstance, continuing to remove a needed nasogastric tube would meet these criteria. Refusing to call for help, although unsafe, is not a reason for restraint. Getting confused at night regarding the time or not sleeping and bothering the staff to ask for items is not a reason for restraint. DIF: Understand REF: 888 OBJ: Describe methods to evaluate interventions designed to maintain or promote safety. TOP: Assessment MSC: NCLEX: Health Promotion and Maintenance 13. The nurse is discussing with a patient’s physician the need for restraint. The nurse indicates
that alternatives have been utilized. What behaviours would indicate that the alternatives are working? a. The patient continues to get up from the chair at the nurses’ station. b. The patient apologizes for being “such a bother.” c. The patient folds three washcloths over and over. d. The sitter leaves the patient alone to go to lunch. ANS: C
NURSINGTB.COM
Offering diversionary activities such as something to hold is a way to keep the hands busy and provides an alternative to restraints. Assigning a room near the nurses’ station or a chair at the desk can be an alternative for continuous monitoring. Getting up constantly can be cause for concern. Apologizing is not an alternative to restraints. Having a sitter sit with the patient to keep him occupied can be an alternative to restraints, but the sitter needs to be there continuously. DIF: Understand REF: 890 (Box 37-11) OBJ: Describe assessment activities designed to identify patients’ physical, psychosocial, and cognitive statuses as they pertain to his or her safety. TOP: Evaluate MSC: NCLEX: Safe and Effective Care Environment 14. The nurse is caring for a patient who suddenly becomes confused and tries to remove an
intravenous infusion. The nurse begins to develop a plan to care for the patient. Which nursing intervention should take priority? a. Gather restraint supplies. b. Try alternatives to restraint. c. Assess the patient. d. Call the physician for a restraint order. ANS: C
Canadian Fundamentals of Nursing 6th Edition Potter Test Bank When a patient becomes suddenly confused, the priority is to assess the patient, including checking laboratory test and oxygen status, and treating and eliminating the cause of the change in mental status. If interventions and alternatives are exhausted, the nurse working with the physician may determine the need for restraints. DIF: Apply REF: 892 OBJ: Describe assessment activities designed to identify patients’ physical, psychosocial, and cognitive statuses as they pertain to his or her safety. TOP: Planning MSC: NCLEX: Psychosocial Integrity 15. Equipment-related accidents are risks in the health care agency. The nurse assesses for this
risk when using which of the following? a. IV pumps. b. A measuring device that measures urine. c. Computer-based documentation. d. A manual medication-dispensing device. ANS: A
A dysfunctional IV pump can cause an equipment-related accident, such as too-rapid infusion of IV fluids. Measuring devices used by the nurse to measure urine, computer documentation, and manual dispensing devices can break or malfunction but are not used directly on a patient. DIF: Remember REF: 876 OBJ: Describe the most common safety risks in health care. MSC: NCLEX: Safe and Effective Care Environment
TOP: Assessment
16. The nurse is discussing measures to minimize the risk of injury from an automobile accident
with an 84-year-old independNent mN anGwh ves alone and claims to drive only to church, URwo SI TBo.liC OM to the doctor’s office, and for groceries. What change has the greatest potential for affecting the patient’s safety? a. Taking public transportation whenever it is available. b. Planning all trips around church and doctor appointments. c. Planning to drive for short trips and only during daylight hours. d. Arranging for family and friends to drive the patient whenever possible. ANS: C
The nurse should educate the patient regarding safe driving tips (e.g., driving shorter distances and only during daylight hours, using side- and rear-view mirrors carefully, and looking toward the “blind spot” before changing lanes). Taking public transportation may not meet the needs of an independent patient. Planning all trips around church and doctor appointments may not be realistic. Arranging for family and friends to drive the patient may not meet her needs. DIF: Analyze REF: 887 OBJ: Describe nursing interventions specific to patients’ age for reducing safety risks. TOP: Implementation MSC: NCLEX: Health Promotion and Maintenance 17. A patient with an intravenous infusion requests a new gown after bathing. Which of the
following actions is most appropriate? a. Disconnect the intravenous tubing, thread the end through the sleeve of the old gown and through the sleeve of the new gown, and reconnect.
Canadian Fundamentals of Nursing 6th Edition Potter Test Bank b. Thread the intravenous bag and tubing through the sleeve of the old gown and
through the sleeve of the new gown without disconnecting. c. Inform the patient that a new gown is not an option while receiving an intravenous
infusion in the hospital. d. Call the charge nurse for assistance because linen use is monitored and this is not a common procedure. ANS: B
Procedure-related accidents such as contamination of sterile items can occur in the health care setting. Keeping the intravenous tubing intact without breaks in the system is imperative for decreasing the risk of infection while a patient’s request to change a gown is satisfied. DIF: Apply REF: 877 (Box 37-3) OBJ: Describe the most common safety risks in health care. MSC: NCLEX: Safe and Effective Care Environment
TOP: Implementation
18. A confused patient needs to have restraints applied to prevent him from pulling out his Foley
catheter. Which of the following options can the nurse delegate to an unregulated care provider (UCP)? a. Applying restraints. b. Obtaining a physician’s order to restrain the patient. c. Documenting the events that led to restraining the patient. d. Evaluating the effectiveness of the restraints. ANS: A
Although the UCP can apply the restraints under the nurse’s direction, he or she cannot document, evaluate, or take physicians’ orders. The nurse is always responsible for assessment of patients’ safetyNne eds. UR SIANUGCP TBc.anCnot OMtake physicians’ orders, document the events that led to restraining the patient, or evaluate the effectiveness of the restraints. DIF: Apply REF: 892 OBJ: Develop a nursing care plan for patients whose safety is threatened. TOP: Implementation MSC: NCLEX: Safe and Effective Care Environment 19. According to the Hendrich II Fall Risk Model, a patient with a risk score of 6 is considered to
be at which risk level? a. No risk. b. Low risk. c. Medium risk. d. High risk. ANS: D
According to the Hendrich II Fall Risk Model, a score of 5 or greater indicates that the patient is at high risk for a fall, not “no risk,” “low risk,” or “medium risk.” DIF: Analyze REF: 879 (Table 37-1) OBJ: Describe nursing interventions specific to patients’ age for reducing safety risks. TOP: Assessment MSC: NCLEX: Safe and Effective Care Environment 20. An older patient presents to the emergency department after stepping in front of a car at a
crosswalk. After the patient has been examined in triage, the nurse interviews the patient. Which of the following comments would necessitate follow-up by the nurse?
Canadian Fundamentals of Nursing 6th Edition Potter Test Bank a. b. c. d.
“I try to exercise, so I walk that block almost every day.” “I waited and stepped out when the traffic sign said go.” “The car was going too fast; the speed limit is 20.” “I was so surprised; I didn’t see or hear the car coming.”
ANS: D
As patients age, sensory impairment can increase the risk for injury. The patient did not see or hear the car coming. This statement necessitates follow-up by the nurse; the patient needs hearing and eye examinations. Exercise is important at every stage of development. The patient seemed to comprehend how to cross an intersection correctly and was able to determine the speed of the car. DIF: Apply REF: 877 (Box 37-4) OBJ: Describe assessment activities designed to identify patients’ physical, psychosocial, and cognitive statuses as they pertain to his or her safety. TOP: Diagnosis MSC: NCLEX: Physiological Integrity 21. An age-related musculoskeletal change that predisposes the older person to accidents is which
of the following? a. Increase in muscle function. b. Increase in joint mobility. c. Increase in nocturia. d. Decrease in muscle strength. ANS: D
Musculoskeletal system changes associated with aging that increase the risk of accidents in older people include decreases in muscle strength and function, decreases in joint mobility, increasing brittleness of boneN s,UpR ostu ng. esC, O anMd limited range of motion. Rather than IralNGcha B S T increasing with age, muscle function decreases. Joints become less mobile with age, not more mobile. Nocturia is common in older persons and increases their risk of injury; however, it is a genitourinary change, not a musculoskeletal system change. DIF: Understand REF: 877 (Box 37-4) OBJ: Describe nursing interventions specific to patients’ age for reducing safety risks. TOP: Assessment MSC: NCLEX: Physiological Integrity 22. The patient is confused, is trying to get out of bed, and is pulling at the IV infusion tubing.
These data would help to support which nursing diagnosis? a. Risk for poisoning. b. Knowledge deficit. c. Impaired home maintenance. d. Risk for injury. ANS: D
The patient’s behaviours support the nursing diagnosis of Risk for injury. The patient is confused, is pulling at the IV line, and is trying to climb out of bed. Injury could result if the patient falls out of bed or begins to bleed as a result of a pulled line. Nothing in the scenario indicates that this patient lacks knowledge or is at risk for poisoning. Nothing in the scenario refers to the patient’s home maintenance. DIF: Understand REF: 880 OBJ: Identify relevant nursing Diagnosis associated with risks to safety.
Canadian Fundamentals of Nursing 6th Edition Potter Test Bank TOP: Diagnosis
MSC: NCLEX: Safe and Effective Care Environment
23. A confused patient is restless and continues to try to remove his oxygen and urinary catheter.
What are the priority nursing diagnosis and intervention to implement for this patient? a. Risk for injury: Prevent harm to patient; use restraints if alternative strategies fail. b. Deficient knowledge: Explain the purpose of oxygen therapy and the urinary catheter. c. Disturbed body image: Encourage patient to express concerns about body. d. Caregiver role strain: Identify resources to assist with care. ANS: A
The priority nursing diagnosis is Risk for injury. This patient could cause harm to himself by interrupting the oxygen therapy or by damaging the urethra by pulling the urinary catheter out. Before restraining a patient, it is important to implement and exhaust alternative strategies. Such strategies can include distraction and providing companionship or supervision. Patients may be moved to a location closer to the nurses’ station; trained sitters or family members may be involved. Nurses need to ensure that patients are provided adequate food, liquid, toileting, and relief from pain. If these and other strategies fail, the patient may need restraints; in this case, an order must be obtained for the restraint. This patient may have deficient knowledge, and educating the patient about treatments could be considered as an alternative to restraints; however, the highest priority is preventing injury. This scenario does not indicate that the patient has a disturbed body image or that the patient’s caregiver is strained. DIF: Apply REF: 881| 882 OBJ: Identify relevant nursing Diagnosis associated with risks to safety. TOP: Diagnosis MSC: NCLEX: Safe and Effective Care Environment
NURSINGTB.COM
24. Which of the following assessment findings is most critical for a patient who is currently
being restrained with mechanical wrist restraints? a. Angry, loud crying b. Urinary incontinence c. Reddened areas on wrists d. Hands cool to the touch ANS: D
Although the use of any restraint may be associated with serious complications, including pressure ulcers, constipation, pneumonia, urinary and fecal incontinence, and urinary retention, the most serious concerns are contractures, nerve damage, and circulatory impairment. Coolness of the patient’s hands would indicate poor circulation, which can result in permanent damage. Angry, loud crying, urinary incontinence, and reddened areas on the wrists are concerns, but they are not the most critical concerns for the patient with wrist restraints. DIF: Analyze REF: 890 OBJ: Describe methods to evaluate interventions designed to maintain or promote safety. TOP: Assessment MSC: NCLEX: Safe and Effective Care Environment 25. The nurse is providing information regarding safety and accidental poisoning to a
grandmother who will be taking custody of a 1-year-old grandchild. Which of the following comments would indicate that the grandmother needs further instruction?
Canadian Fundamentals of Nursing 6th Edition Potter Test Bank a. “If I think my grandchild has come into contact with a poison, I will call my local
poison control centre.” b. “Never induce vomiting if my grandchild drinks bleach.” c. “I should call 911 if my grandchild loses consciousness.” d. “If my grandchild eats a plant, I should provide syrup of ipecac.” ANS: D
Syrup of ipecac to induce vomiting after ingestion of a poison has not proved effective in preventing poisoning. This medication should not be administered to the child. Phone numbers for poison control centres across Canada can be accessed at http://www.aboutkidshealth.ca/En/HealthAZ/ TestsAndTreatments/Resources/Pages/Poison-Information-Centres-in-Canada.aspx. After a caustic substance such as bleach has been drunk, vomiting can cause further burning and injury as the substance is eliminated. Loss of consciousness in association with poisoning necessitates calling 9-1-1. DIF: Understand REF: 885 (Box 37-8) OBJ: Describe nursing interventions specific to patients’ age for reducing safety risks. TOP: Evaluate MSC: NCLEX: Health Promotion and Maintenance 26. An elderly patient presents to the hospital with a history of falls, confusion, and stroke. The
nurse determines that the patient is at high risk for falls. Which of the following interventions is most appropriate for the nurse to take? a. Place the patient in restraints. b. Lock beds and wheelchairs when transferring. c. Place a bath mat outside the tub. d. Silence fall alert alarm upon request of family. ANS: B
N R I G B.C M U S N T O
Locking the bed and wheelchairs when the patient is transferred helps prevent these pieces of equipment from moving during transfer and will assist in the prevention of falls. Patients are not automatically placed in restraints. The restraint process consists of many steps, including thorough assessment and exhausting of alternative strategies. All mats and rugs should be secured to help prevent falls. Silencing alarms upon the request of family is not appropriate and could endanger the patient. DIF: Understand REF: 888 OBJ: Describe nursing interventions specific to patients’ age for reducing safety risks. TOP: Implementation MSC: NCLEX: Safe and Effective Care Environment 27. The nurse has been called to a hospital room where a patient was using a hair dryer from
home. The patient received an electrical shock from the dryer and is now unconscious and not breathing. What is the best next step? a. Ask the family to leave the room. b. Check for a pulse. c. Begin compressions. d. Defibrillate the patient. ANS: B
Canadian Fundamentals of Nursing 6th Edition Potter Test Bank In this scenario, the patient is in a hospital setting, and it has been determined that the patient is not conscious and is not breathing. The next step is to check the pulse. An electrical shock can interfere with the heart’s normal electrical impulses and can cause arrhythmias. Checking the pulse helps to determine the need for cardiopulmonary resuscitation (CPR) and defibrillation. DIF: Apply REF: 898 OBJ: Describe nursing interventions specific to patients’ age for reducing safety risks. TOP: Implementation MSC: NCLEX: Physiological Integrity 28. The nurse determines that the patient may need a restraint and recognizes which one of the
following? a. An order for a restraint may be implemented indefinitely until it is no longer required by the patient. b. Restraints may be ordered on an as-needed basis. c. No order or consent is necessary for restraints in long-term care facilities. d. Restraints are to be periodically removed to have the patient re-evaluated. ANS: D
Restraints must be periodically removed, and the nurse must reassess the patient to determine whether the restraints continue to be needed. It is not true that an order for a restraint may be implemented indefinitely until it is no longer required by the patient. A physician’s order for restraints must have a limited time frame. If the orders are to be renewed, this should be done within a specified time frame according to the agency’s policy. Restraints are not to be ordered as needed. The use of restraints must be part of the patient’s medical treatment and must be ordered according to provincial or territorial legislation and agency policy. In some settings, physicians and nurses may order restraints.
N R I G B.C M U S N T O
DIF: Evaluate REF: 891 OBJ: Discuss methods to prevent and reduce safety risks. MSC: NCLEX: Safe and Effective Care Environment
TOP: Evaluate
MULTIPLE RESPONSE 1. The nurse is caring for a patient in restraints. Which of the following pieces of information
about restraints must be documented by the nurse in the medical record? (Select all that apply.) a. The patient states that her gown is soiled and needs changing. b. Previous attempts to distract the patient with television were unsuccessful. c. The patient was placed in bilateral wrist restraints at 0815. d. One family member has gone to lunch. e. Bilateral radial pulses present, 2+, hands warm to touch. f. Released from restraints, active range-of-motion exercises complete. ANS: B, C, E, F
Attempts at alternative strategies are documented in the medical record, as are type of restraint and time restrained. Assessments related to oxygenation, orientation, skin integrity, circulation, and position are documented, along with release from restraints and patient response. Comments about hygiene or the activities of one family member are not necessarily required in nursing documentation of restraints.
Canadian Fundamentals of Nursing 6th Edition Potter Test Bank DIF: Apply REF: 892 (Skill 37-1) OBJ: Describe nursing interventions specific to patients’ age for reducing safety risks. TOP: Implementation MSC: NCLEX: Safe and Effective Care Environment
NURSINGTB.COM
Canadian Fundamentals of Nursing 6th Edition Potter Test Bank
Chapter 38: Hygiene Potter et al: Canadian Fundamentals of Nursing, 6th Edition MULTIPLE CHOICE 1. A number of factors influence a patient’s personal preferences for hygiene. Because of this,
what is important for the nurse to realize? a. No two individuals perform hygiene care in the same manner. b. It is important to standardize a patient’s hygienic practices. c. Hygiene care is always routine and expected. d. Hygiene care is not the time to learn about patient needs. ANS: A
No two individuals perform hygiene care in the same manner; it is important to individualize the patient’s care according to knowledge about the patient’s unique hygiene practices and preferences. Hygiene care by nurses is never routine; this care requires intimate contact with the patient and communication skills to promote the therapeutic relationship. In addition, during hygiene care, the nurse should take time to learn about the patient’s health promotion practices and needs, emotional needs, and health care education needs. DIF: Remember REF: 904 OBJ: Discuss common factors that influence personal hygiene practices. TOP: Assessment MSC: NCLEX: Health Promotion and Maintenance 2. Social groups influence hygiene preferences and practices, including the type of hygienic
products used and the nature and frequency of personal care. Which of the following Nikel RS I G nfluenc B.COedMby family customs? developmental stages is most lU y toNbe iT a. Adolescent. b. Toddler. c. Adult. d. Older person. ANS: B
During childhood, family customs influence hygiene. As children enter their adolescent years, peer group behaviour often influences personal hygiene. During the adult years, involvement with friends and work groups shapes the expectations people have about their personal appearance. Some older persons’ hygiene practices change because of living conditions and available resources. DIF: Analyze REF: 904 OBJ: Discuss common factors that influence personal hygiene practices. TOP: Assessment MSC: NCLEX: Psychosocial Integrity 3. The patient received a diagnosis of diabetes 12 years ago. When admitted, the patient is
unkempt and is in need of a bath and foot care. When questioned about his hygiene habits, the patient tells the nurse that baths are taken once a week where he comes from, although he takes a sponge bath every other day. To provide ultimate care for this patient, what must the nurse understand? a. Personal preferences determine hygiene practices and are unchangeable. b. Patients who appear unkempt place little importance on hygiene practices.
Canadian Fundamentals of Nursing 6th Edition Potter Test Bank c. The patient’s illness may require teaching of new hygiene practices. d. All cultures value cleanliness with the same degree of importance. ANS: C
Each patient has individual desires and preferences about when to bathe, shave, and perform hair care. However, they are not unchangeable. In addition, the nurse must assist the patient in developing new hygiene practices when indicated by an illness or condition. For example, the nurse will need to teach a patient with diabetes proper foot hygiene. Patients who appear unkempt often need further assessment regarding their ability to participate in daily hygiene. Patients with certain types of physical limitations or disabilities often lack the physical energy or dexterity, or both, to perform hygienic care. Culturally, maintaining cleanliness does not hold the same importance for some ethnic groups as it does for others. DIF: Understand REF: 904| 905 OBJ: Discuss common factors that influence personal hygiene practices. TOP: Assessment MSC: NCLEX: Psychosocial Integrity 4. The nurse is caring for a patient who refuses “A.M. care.” When asked why, the patient tells
the nurse that she always bathes in the evening. What should the nurse do? a. Defer the bath until evening and pass on the information to the next shift. b. Tell the patient that she must bathe in the morning because that is the “normal” routine. c. Explain to the patient the importance of maintaining morning hygiene practices. d. Cancel hygiene care for the day and attempt again in the morning. ANS: A
Each patient has individual desires and preferences about when to bathe, shave, and perform hair care. Knowing the patienNt’s ef. erC enc es assists the nurse in providing URpers SIona NGl pr TB OM individualized care for the patient. Hygiene care is never routine. Maintaining individual personal preferences is important unless new hygiene practices are indicated by an illness or condition. Cancelling hygiene care is not an option. Practices must be adapted to meet individual needs. No evidence demonstrates greater benefit with morning or evening hygiene care. DIF: Apply REF: 912 OBJ: Discuss common factors that influence personal hygiene practices. TOP: Implementation MSC: NCLEX: Psychosocial Integrity 5. Successful critical thinking requires synthesis of knowledge, experience, information gathered
from patients, critical thinking qualities, and intellectual and professional standards. Once the assessment has been done, what is important for the nurse to understand? a. The nursing diagnoses never change. b. The patient’s condition never changes. c. Critical thinking is ongoing. d. Hygiene care needs to become a simple routine. ANS: C
A patient’s condition is always changing, which necessitates ongoing critical thinking and changing of nursing diagnoses. Because hygienic care is so important for a patient to feel comfortable, refreshed, and renewed, the nurse should avoid making hygiene care a simple routine.
Canadian Fundamentals of Nursing 6th Edition Potter Test Bank
DIF: Understand REF: 905 OBJ: Discuss the role that the nursing process and critical thinking play in the provision of hygiene care. TOP: Assessment MSC: NCLEX: Safe and Effective Care Environment 6. When the nurse provides hygiene care for an older patient, why is it important for the nurse to
closely assess the skin? a. As people age, skin becomes more resilient. b. As people age, sweat glands become more active. c. As people age, skin becomes less subject to bruising. d. As people age, less frequent bathing may be required. ANS: D
Daily bathing, as well as bathing with water that is too hot or soap that is harsh, causes the skin to become excessively dry. As the patient ages, the skin loses its resiliency and moisture, and sebaceous and sweat glands become less active. The epithelium thins, and elastic collagen fibres shrink, which makes the skin fragile and prone to bruising and breaking. DIF: Understand REF: 911 OBJ: Describe how hygiene care for the older patient may differ from that for the younger patient. TOP: Assessment MSC: NCLEX: Physiological Integrity 7. The nurse is bathing a patient and notices movement in the patient’s hair. The nurse should a. Ignore the movement and continue. b. Use gloves or a tongue blade to inspect the hair. c. Examine the hair without gloves to make picking lice easier. d. Shave the hair off of the patient’s head. ANS: B
NURSINGTB.COM
In community health and home care settings, it is particularly important to inspect the hair for lice so that appropriate hygienic treatment can be provided. Suspicions cannot be ignored. If pediculosis capitis (head lice) is suspected, the nurse must protect himself or herself against self-infestations by hand hygiene and by using gloves or tongue blades to inspect the patient’s hair. Shaving hair off affected areas is the treatment for pediculosis pubis (crab lice) and is rarely used for head lice. DIF: Apply REF: 908 OBJ: Conduct a comprehensive assessment of a patient’s total hygiene needs. TOP: Implementation MSC: NCLEX: Health Promotion and Maintenance 8. The patient has been brought to the emergency department after a motor vehicle accident. The
patient is unresponsive. His driver’s licence states that he needs glasses to operate a motor vehicle, but no glasses were brought in with the patient. What should the nurse do? a. Assume that the glasses were lost during the accident. b. Stand to the side of the patient’s eye and observe the cornea. c. Assume that the patient was not wearing glasses while driving. d. Assume that the ambulance personnel have them. ANS: B
Canadian Fundamentals of Nursing 6th Edition Potter Test Bank An important aspect of an eye examination is to determine if the patient wears contact lenses, especially in patients who are unresponsive. To determine whether a contact lens is present, stand to the side of the patient’s eye and observe the cornea for the presence of a soft or rigid lens. It is also important to observe the sclera to detect the presence of a lens that has shifted off the cornea. An undetected lens causes severe corneal injury when left in place too long. Nurses must never assume that glasses were lost or were not worn. Contacting ambulance personnel takes time. The patient’s eyes should be examined. DIF: Apply REF: 910 OBJ: Conduct a comprehensive assessment of a patient’s total hygiene needs. TOP: Implementation MSC: NCLEX: Safe and Effective Care Environment 9. When assessing a patient’s skin, what does the nurse need to know? a. Restricted movement can increase blood circulation. b. Paralyzed patients have normal sensory function. c. Loss of subcutaneous tissue may increase the rate of wound healing. d. Moisture on the skin can lead to skin maceration. ANS: D
Moisture on the surface of the skin serves as a medium for bacterial growth and causes irritation, softens epidermal cells, and leads to skin maceration. When restricted from moving freely, dependent body parts are exposed to pressure that reduces circulation to affected tissues. Nurses should know which patients require help to turn and change positions. Patients with paralysis, circulatory insufficiency, or local nerve damage are unable to sense an injury to affected parts of the skin. In patients with limited caloric and protein intake, the skin becomes thinner and less elastic, with loss of subcutaneous tissue, which results in impairment of or delay in wound healing.
N URSINGTB.COM
DIF: Remember REF: 908 (Box 38-2) OBJ: Describe conditions that place the patient at risk for problems related to the hair, scalp, integument, feet, nails, eyes, ears, nose, mouth, and throat, and discuss their related interventions. TOP: Assessment MSC: NCLEX: Physiological Integrity 10. The nurse is caring for a patient who is immobile. Why is the nurse aware that the patient is at
risk for impaired skin integrity? a. Pressure reduces circulation to affected tissue. b. Patients with limited caloric intake develop thicker skin. c. Inadequate blood flow leads to decreased tissue ischemia. d. Local nerve damage leads to pain sensation. ANS: A
Body parts exposed to pressure have reduced circulation to affected tissue. In patients with limited caloric and protein intake, the skin becomes thinner and less elastic, with loss of subcutaneous tissue. Inadequate blood flow causes ischemia and breakdown. Patients with paralysis, circulatory insufficiency, or local nerve damage are unable to sense an injury to affected parts of the skin. DIF: Understand REF: 908 (Box 38-2) OBJ: Describe conditions that place the patient at risk for problems related to the hair, scalp, integument, feet, nails, eyes, ears, nose, mouth, and throat, and discuss their related interventions. TOP: Assessment MSC: NCLEX: Physiological Integrity
Canadian Fundamentals of Nursing 6th Edition Potter Test Bank
11. The nurse is caring for a patient who has diabetes mellitus and circulatory insufficiency, with
peripheral neuropathy and urinary incontinence. What does the nurse know about patients with these conditions? a. They have decreased pain sensation and are at increased risk for skin impairment. b. They are at decreased risk of developing infection because of urinary pH level. c. They have decreased caloric intake, which results in accelerated wound healing. d. They have impaired venous return, which allows for greater circulation and less breakdown. ANS: A
Patients with paralysis, circulatory insufficiency, or local nerve damage are unable to sense an injury to affected parts of the skin. The presence of perspiration, urine, watery fecal material, and wound drainage on the skin results in breakdown and infection. In patients with limited caloric and protein intake, the skin becomes thinner and less elastic, with loss of subcutaneous tissue. This results in impairment in or delay of wound healing. Impaired venous return decreases circulation to the extremities. Inadequate blood flow causes ischemia and breakdown. DIF: Understand REF: 908 (Box 38-2) OBJ: Describe conditions that place the patient at risk for problems related to the hair, scalp, integument, feet, nails, eyes, ears, nose, mouth, and throat, and discuss their related interventions. TOP: Assessment MSC: NCLEX: Physiological Integrity 12. The nurse is caring for a patient who has undergone external fixation of a broken leg and has a
cast in place. To prevent skin impairment, what should the nurse do? a. Not allow the patient to turn in bed because that may lead to redislocation of the
NURSINGTB.COM leg. b. Restrict the patient’s dietary intake to reduce the number of times on the bedpan. c. Assess all surfaces exposed to the cast for pressure areas. d. Keep the patient’s blood pressure low to prevent overperfusion of tissue. ANS: C
Assess all surfaces exposed to casts, cloth restraints, bandages and dressings, tubing, or orthopedic braces. An external device applied to or around the skin exerts pressure or friction on the skin, which leads to skin impairment. When restricted from moving, dependent body parts are exposed to pressure that reduces circulation to affected tissues. Nurses should know which patients require assistance to turn and change positions. In patients with limited caloric and protein intake, wound healing becomes impaired or is delayed. Keeping the blood pressure artificially low may decrease arterial blood supply, which leads to ischemia and breakdown. DIF: Apply REF: 908 (Box 38-2) OBJ: Describe conditions that place the patient at risk for problems related to the hair, scalp, integument, feet, nails, eyes, ears, nose, mouth, and throat, and discuss their related interventions. TOP: Implementation MSC: NCLEX: Physiological Integrity 13. Of the following interventions, which would be the most important for preventing skin
impairment in a mobile patient with local nerve damage? a. Turning the patient every 2 hours. b. Limiting caloric and protein intake.
Canadian Fundamentals of Nursing 6th Edition Potter Test Bank c. Inserting an indwelling urinary catheter. d. Assessing for pain while the patient is bathing. ANS: D
While a patient is bathing, the nurse should assess the status of sensory nerve function by checking for pain, tactile sensation, and temperature sensation. When restricted from moving freely, dependent body parts are exposed to pressure that reduces circulation. However, a mobile patient is able to change positions. Limiting caloric and protein intake may cause wound healing to become impaired or delayed. The presence of perspiration, urine, watery fecal material, and wound drainage on the skin also causes wound healing to become impaired or delayed. However, a mobile patient can use bathroom facilities or a urinal. DIF: Analyze REF: 908 (Box 38-2) OBJ: Describe conditions that place the patient at risk for problems related to the hair, scalp, integument, feet, nails, eyes, ears, nose, mouth, and throat, and discuss their related interventions. TOP: Assessment MSC: NCLEX: Physiological Integrity 14. Of the following disorders, which is caused by a virus? a. Corns. b. Plantar warts. c. Athlete’s foot. d. Callus. ANS: B
Plantar warts appear on the sole of the foot and are due to the papillomavirus. Corns are caused by friction and pressure from ill-fitting or loose shoes. Athlete’s foot (tinea pedis) is a fungal infection. A callus is caused by local friction or pressure.
N R I G B.C M
U S N T O DIF: Remember REF: 909 (Table 38-3) OBJ: Describe conditions that place the patient at risk for problems related to the hair, scalp, integument, feet, nails, eyes, ears, nose, mouth, and throat, and discuss their related interventions. TOP: Assessment MSC: NCLEX: Physiological Integrity 15. The nurse is caring for a patient who is complaining of severe foot pain from corns. The
patient states that she has been using oval corn pads to self-treat the corns, but they seem to be getting worse. What does the nurse explain? a. Corn pads are an adequate treatment and should be continued. b. The patient should avoid soaking her feet before using a pumice stone. c. Tighter shoes would help to compress the corns and make them smaller. d. Depending on severity, surgery may be needed to remove the corns. ANS: D
Depending on severity of pain and the size of the corn, surgical removal may be necessary. Oval corn pads should be avoided because they increase pressure on the toes and reduce circulation. Warm water soaks help soften corns before gentle rubbing with a callus file or pumice stone. Wider and softer shoes, especially shoes with a wider toe box, are helpful. DIF: Understand REF: 909 (Table 38-3) OBJ: Describe conditions that place the patient at risk for problems related to the hair, scalp, integument, feet, nails, eyes, ears, nose, mouth, and throat, and discuss their related interventions. TOP: Planning MSC: NCLEX: Physiological Integrity
Canadian Fundamentals of Nursing 6th Edition Potter Test Bank 16. The patient receives a diagnosis of athlete’s foot (tinea pedis). The patient says that she is
relieved because it is “only athlete’s foot” and it can be treated easily. What does the nurse explains about athlete’s foot? a. It is generally isolated to the feet and never recurs. b. It is contagious and frequently recurs. c. It is caused by the papillomavirus. d. It is treated with salicylic acid or electrodesiccation. ANS: B
Athlete’s foot spreads to other body parts, especially the hands. It is contagious and frequently recurs. It is caused by a fungus, not the papillomavirus. It is not treated with salicylic acid or electrodesiccation; those are treatments for plantar warts. DIF: Understand REF: 909 (Table 38-3) OBJ: Describe conditions that place the patient at risk for problems related to the hair, scalp, integument, feet, nails, eyes, ears, nose, mouth, and throat, and discuss their related interventions. TOP: Planning MSC: NCLEX: Physiological Integrity 17. When assessing a patient’s feet, the nurse notices that the toenails are thick and separated
from the nail bed. What is this condition caused by? a. Fungi. b. Nail polish. c. Friction. d. Nail polish remover. ANS: A
Inflammatory lesions and fungus of the nail bed cause nails to become thickened and horny and to separate from the nail N bed a fe mT alB e p.aC tien UR. IfSI NG OMt is affected, the nurse should ask whether she frequently polishes her nails and uses polish remover because chemicals in these products cause excessive nail dryness. Friction and pressure from ill-fitting or loose shoes causes keratosis (corns). It is seen mainly on or between toes, over bony prominences. DIF: Understand REF: 911 OBJ: Describe conditions that place the patient at risk for problems related to the hair, scalp, integument, feet, nails, eyes, ears, nose, mouth, and throat, and discuss their related interventions. TOP: Assessment MSC: NCLEX: Physiological Integrity 18. The nurse is providing education about the importance of proper foot care to a patient who has
diabetes mellitus. Why is this important? a. Plantar warts can develop from foot fungi. b. Poor foot care leads to neuropathy. c. A strong dorsalis pedis pulse indicates poor blood flow. d. Foot ulcers are the most common precursor to amputation. ANS: D
Foot ulceration is the most common single indication for lower extremity amputation among persons with diabetes. Plantar warts are caused by the papillomavirus, not a fungus. Palpation of the dorsalis pedis and posterior tibial pulses indicates that adequate blood flow is reaching peripheral tissues. Neuropathy is a degeneration of the peripheral nerves usually that is due to poor control of blood glucose levels; it is not a direct result of poor foot care. DIF: Understand
REF: 927
Canadian Fundamentals of Nursing 6th Edition Potter Test Bank OBJ: Understand the importance of foot care for the diabetic patient. TOP: Assessment MSC: NCLEX: Physiological Integrity 19. The nurse is providing oral care to an unconscious patient and notes that the patient has
extremely bad breath. What is the term for “bad breath”? a. Alopecia. b. Halitosis. c. Dental caries. d. Neuropathy. ANS: B
Halitosis is the term for “bad breath.” Alopecia is hair loss. Dental caries is tooth decay. Neuropathy is a degeneration of peripheral nerves that leads to loss of sensation in the extremities. DIF: Remember REF: 908| 932 OBJ: Describe conditions that place the patient at risk for problems related to the hair, scalp, integument, feet, nails, eyes, ears, nose, mouth, and throat, and discuss their related interventions. TOP: Assessment MSC: NCLEX: Physiological Integrity 20. The nurse is caring for an unresponsive patient who has a nasogastric tube in place for
continuous tube feedings. Why does the nurse assess the patient’s oral hygiene? a. It helps prevent gingivitis. b. It may cause glossitis. c. It may lead to halitosis. d. It causes tongue coating. ANS: A
NURSINGTB.COM
Early identification of poor oral hygiene practices and common oral problems reduces the risk for gum disease and dental caries. Patients frequently develop common oral problems as a result of inadequate oral care or as a consequence of disease (e.g., oral malignancy) or as a side effect of treatments such as radiation therapy and chemotherapy. These problems include receding gum tissue, inflamed gums (gingivitis), coating of the tongue, glossitis (inflammation of the tongue), discoloration of teeth (particularly along gum margins), dental caries, missing teeth, and halitosis (foul-smelling breath). DIF: Understand REF: 908 OBJ: Describe conditions that place the patient at risk for problems related to the hair, scalp, integument, feet, nails, eyes, ears, nose, mouth, and throat, and discuss their related interventions. TOP: Assessment MSC: NCLEX: Physiological Integrity 21. The patient is being treated for cancer with weekly radiation and chemotherapy treatments.
The nurse is aware that the patient’s oral mucosa needs to be assessed because radiation therapy and chemotherapy can have what effects? a. Increase saliva production. b. Decrease the risk of oral inflammation. c. Decrease drying of oral mucosa. d. Lead to oral problems. ANS: D
Canadian Fundamentals of Nursing 6th Edition Potter Test Bank Patients frequently develop common oral problems as a result of inadequate oral care or as a consequence of disease (e.g., oral malignancy) or as a side effect of treatments such as radiation and chemotherapy. DIF: Understand REF: 908 OBJ: Describe conditions that place the patient at risk for problems related to the hair, scalp, integument, feet, nails, eyes, ears, nose, mouth, and throat, and discuss their related interventions. TOP: Assessment MSC: NCLEX: Physiological Integrity 22. In providing oral care to an unconscious patient, what is an important action by the nurse? a. Moistening the patient’s mouth with lemon-glycerin sponges. b. Holding the patient’s mouth open with his or her fingers. c. Rinsing the patient’s mouth and immediately suctioning the oral cavity. d. Using foam swabs to help remove plaque. ANS: C
When providing oral hygiene care to an unconscious patient, the nurse needs to protect him or her from choking and aspiration. Two nurses should provide care: One nurse does the actual cleaning, and the other removes secretions with suction equipment. The nurse can delegate some tasks to unregulated care providers. Some agencies use equipment that combines a mouth swab with the suction device. This device can be used safely by one nurse to provide oral care. Commercially made foam swabs are ineffective in removing plaque. Lemon-glycerin sponges should not be used because they dry mucous membranes and erode tooth enamel. While cleansing the oral cavity, the nurse should use a small oral airway or a padded tongue blade to hold the patient’s mouth open. The nurse should never use his or her own fingers to hold the patient’s mouth open. A human bite contains multiple pathogenic microorganisms.
NURSINGTB.COM
DIF: Apply REF: 931 OBJ: Describe conditions that place the patient at risk for problems related to the hair, scalp, integument, feet, nails, eyes, ears, nose, mouth, and throat, and discuss their related interventions. TOP: Implementation MSC: NCLEX: Physiological Integrity 23. The nurse is teaching the patient about flossing and oral hygiene. Which of the following is
the nurse’s instruction? a. Flossing needs to be done at least three times a day. b. To prevent bleeding, the patient should use waxed floss. c. Flossing removes plaque and bacteria from the teeth. d. Applying toothpaste to the teeth before flossing is harmful. ANS: C
Dental flossing removes plaque and bacteria between teeth. To prevent bleeding, the patient should use unwaxed floss. Flossing once a day is sufficient. If toothpaste is applied to the teeth before flossing, fluoride will come in direct contact with tooth surfaces, aiding in cavity prevention. DIF: Apply REF: 931 OBJ: Describe conditions that place the patient at risk for problems related to the hair, scalp, integument, feet, nails, eyes, ears, nose, mouth, and throat, and discuss their related interventions. TOP: Implementation MSC: NCLEX: Physiological Integrity
Canadian Fundamentals of Nursing 6th Edition Potter Test Bank 24. The nurse is caring for a patient who has head lice (pediculosis capitis). What should the nurse
know about treating this condition? a. Products containing lindane are most effective. b. Head lice may spread to furniture and other people. c. Treatment must be repeated in 7 to 10 days. d. Manual removal is not a realistic option as treatment. ANS: B
Head lice are difficult to remove and spread to furniture and other people if not treated. Products containing lindane should not be used because the ingredient is toxic and is known to cause adverse reactions. Treatments need to be repeated 12 to 24 hours after the initial treatment. Manual removal is the best option when treatment has failed. DIF: Understand REF: 910 (Table 38-4) OBJ: Describe conditions that place the patient at risk for problems related to the hair, scalp, integument, feet, nails, eyes, ears, nose, mouth, and throat, and discuss their related interventions. TOP: Assessment MSC: NCLEX: Physiological Integrity 25. Scaling of the scalp accompanied by itching is known as which of the following? a. Dandruff. b. Pediculosis. c. Alopecia. d. Ticks. ANS: A
Dandruff is scaling of the scalp that is accompanied by itching. Pediculosis (lice) consists of tiny, greyish-white parasitic insects that infest mammals. Alopecia is hair loss or balding. Ticks are small, grey-brown N paraR siteI s thG at bB ur. roC w inMto the skin and suck blood.
U S N T
O
DIF: Remember REF: 910 (Table 38-4) OBJ: Describe conditions that place the patient at risk for problems related to the hair, scalp, integument, feet, nails, eyes, ears, nose, mouth, and throat, and discuss their related interventions. TOP: Assessment MSC: NCLEX: Physiological Integrity 26. In finding pediculosis capitis (head lice) in a patient, what would the nurse expect to observe? a. Greyish-white parasites with red legs. b. Pustules or bites behind ears and at the hairline. c. Balding patches in periphery of the hairline. d. Brittle and broken hair. ANS: B
Head lice are on the scalp, attached to hair stands. Bites or pustules may be observed behind the ears and at the hairline. Greyish-white parasites with red legs are pediculosis pubis (crab lice), not head lice, and are found in pubic hair. Alopecia (hair loss) is found in people of all ethnicities and is characterized by brittle and broken hair and balding patchiness in the periphery of the hairline. DIF: Remember REF: 910 (Table 38-4) OBJ: Describe conditions that place the patient at risk for problems related to the hair, scalp, integument, feet, nails, eyes, ears, nose, mouth, and throat, and discuss their related interventions. TOP: Assessment MSC: NCLEX: Physiological Integrity
Canadian Fundamentals of Nursing 6th Edition Potter Test Bank 27. The nurse is caring for a patient who has multiple ticks on her legs and body. To rid the
patient of ticks, what should the nurse do? a. Burn the ticks in an ashtray once removed. b. Use blunt tweezers and pull upward with steady pressure. c. Allow the ticks to drop off by themselves. d. Use products containing lindane to kill the ticks. ANS: B
Using blunt tweezers, the nurse grasps the tick as close to its head as possible and pulls upward with even, steady pressure. The nurse holds the tick until it pulls out, usually for about 3 to 4 minutes. The tick should be saved in a plastic bag and put in the freezer if necessary to identify the type of tick. Because ticks transmit several diseases to people, they must be removed. Allowing them to drop off by themselves is not an option. Lindane is an ingredient that was used in treatment for pediculosis capitis (head lice); it should no longer be used because the ingredient is toxic and is known to cause adverse reactions. DIF: Apply REF: 910 (Table 38-4) OBJ: Describe conditions that place the patient at risk for problems related to the hair, scalp, integument, feet, nails, eyes, ears, nose, mouth, and throat, and discuss their related interventions. TOP: Implementation MSC: NCLEX: Physiological Integrity 28. The patient received a diagnosis of pediculosis capitis (head lice), was treated upon admission
and re-treated 24 hours later, and yet the patient still has the infestation. What should be the nurse’s next action? a. Re-treat the patient with a medicated shampoo for eliminating lice. b. Use a product containing lindane to get rid of the lice. c. Manually remove the lice using a fine-toothed comb. NUow RSerItN G B.C M d. Have the patient bathe or sh horoTughly. O ANS: C
Manual removal is the best option when treatment has failed. Re-treating with a medicated shampoo may lead to adverse reactions and should not be done without consulting the care provider. Products containing lindane should not be used because the ingredient is toxic and is known to cause adverse reactions. Although bathing or showering is a good idea, this is usually considered a treatment for pediculosis corporis (body lice), not pediculosis capitis (head lice). DIF: Apply REF: 910 (Table 38-4) OBJ: Describe conditions that place the patient at risk for problems related to the hair, scalp, integument, feet, nails, eyes, ears, nose, mouth, and throat, and discuss their related interventions. TOP: Implementation MSC: NCLEX: Physiological Integrity 29. The nurse is caring for an older patient with Alzheimer’s disease who is ambulatory but
requires total assistance with his activities of daily living (ADLs). The nurse notices that his skin is dry and wrinkled. What should the nurse do? a. Make sure that the patient is receiving daily baths. b. Reduce the number of baths per week if possible. c. Be aware that sweat glands become more active with aging. d. Be sure that the patient is using soap with his bath. ANS: B
Canadian Fundamentals of Nursing 6th Edition Potter Test Bank Decreasing the number of baths per week may help prevent further drying of the skin. As people age, the skin loses its resiliency and moisture, and sebaceous and sweat glands become less active. Daily bathing as well as bathing with water that is too hot or soap that is harsh causes the skin to become excessively dry. DIF: Apply REF: 918 (Box 38-9) OBJ: Describe how hygiene care for the older patient may differ from that for the younger patient. TOP: Implementation MSC: NCLEX: Health Promotion and Maintenance 30. A self-sufficient bedridden patient unable to reach all body parts needs which type of bath? a. Complete bed bath. b. Bag bath. c. Sponge bath. d. Partial bed bath. ANS: D
A partial bath consists of washing body parts that the patient cannot reach, including the back, and providing a back rub. Dependent patients in need of partial hygiene care or self-sufficient bedridden patients who are unable to reach all body parts receive a partial bed bath. Complete bed baths are administered to totally dependent patients in bed. The bag bath contains several soft, nonwoven cotton cloths that are premoistened in a solution of no-rinse surfactant cleanser and emollient. It can be used for any patient. The sponge bath involves bathing from a bath basin or a sink with the patient sitting in a chair. DIF: Remember REF: 918 OBJ: Discuss the different approaches used in maintaining a patient’s comfort and safety during hygiene care. TOP: Assessment MSC: NCLEX: Health Promotion and Maintenance
NURSINGTB.COM
31. The nurse is preparing to provide a complete bed bath to an unconscious patient. The nurse
decides to use a bag bath. She does this for which of the following reasons? a. Washbasins can harbour Gram-negative organisms. b. Bag baths entail the use of soaps that enhance cleansing. c. Bag baths do not contain emollients. d. Bag baths increase skin flaking and scaling. ANS: A
When washbasins are not cleaned and dried completely after use, they are more likely to be contaminated by Gram-negative organisms. Successive uses of the washbasin cause the patient’s skin to harbour more gram-negative organisms. Bag baths do not contain soap. Instead, they contain a no-rinse surfactant, a humectant to trap moisture, and an emollient that significantly reduces overall skin dryness, especially skin flaking and scaling. DIF: Understand REF: 926 OBJ: Discuss the different approaches used in maintaining a patient’s comfort and safety during hygiene care. TOP: Assessment MSC: NCLEX: Health Promotion and Maintenance 32. The female nurse is caring for a male patient who is uncircumcised and not ambulatory,
although he has full function of arms and hands. The nurse is providing the patient with a partial bed bath. What statement is true for perineal care for this patient? a. It is not necessary because he is not circumcised. b. It should be postponed because it may cause him embarrassment.
Canadian Fundamentals of Nursing 6th Edition Potter Test Bank c. It should be done by the patient. d. It should be done by the nurse. ANS: C
Patients most in need of perineal care are those at greatest risk for acquiring an infection, such as uncircumcised male patients. If a patient is able to perform perineal self-care, the nurse should encourage this independence. Embarrassment should not cause the nurse to overlook the patient’s hygiene needs. The nurse should provide this care only if the patient is unable to do so. DIF: Apply REF: 926 OBJ: Discuss the different approaches used in maintaining a patient’s comfort and safety during hygiene care. TOP: Implementation MSC: NCLEX: Health Promotion and Maintenance 33. After the patient’s bath, what should the nurse do? a. Not offer a back rub because it is not therapeutic. b. Routinely give back rubs of 2 minutes’ duration or less. c. Assume that all patients need back rubs after their bath. d. Not offer a back rub for 48 hours after the patient has undergone coronary artery
bypass surgery. ANS: D
Consult the medical record for any contraindications to a massage (e.g., fractured ribs, burns, heart surgery). A back rub of 3 minutes’ duration actually enhances patient comfort and relaxation and thus is very therapeutic. It is important to ask whether a patient would like a back rub because some individuals dislike physical contact.
N R I G B.C M
U S N T O DIF: Apply REF: 927 (Box 38-10) OBJ: Discuss the different approaches used in maintaining a patient’s comfort and safety during hygiene care. TOP: Implementation MSC: NCLEX: Physiological Integrity 34. The nurse is providing a complete bed bath to a patient with a commercial bath cleansing pack
(bag bath). What should the nurse do? a. Use one towel for the entire bath. b. Dry the skin with a towel. c. Allow the skin to air dry. d. Not use a bath blanket or towel. ANS: C
The nurse should allow the skin to air dry for 30 seconds. Drying the skin with a towel removes the emollient that is left behind after the water/cleanser solution evaporates. It is permissible to lightly cover the patient with a bath blanket or towel to prevent chilling. The cleansing pack contains 8 to 10 premoistened towels for cleansing. A single towel is used for each general body part cleansed. DIF: Apply REF: 926 OBJ: Successfully perform hygiene care of the integument; perineum; feet, hands, and nails; mouth; and eyes, ears, and nose. TOP: Implementation MSC: NCLEX: Health Promotion and Maintenance
Canadian Fundamentals of Nursing 6th Edition Potter Test Bank 35. In providing perineal care to a female patient, how should the nurse wash? a. Upward from rectum to pubic area. b. From back to front. c. From pubic area to rectum. d. In a circular motion. ANS: C
Cleansing from pubic area to rectum (front to back) reduces the transfer of microorganisms to the urinary meatus and decreases the risk of urinary tract infection. Cleansing from rectum to pubic area or back to front increases the risk of urinary tract infection. Circular motions are used in male perineal care. DIF: Apply REF: 923 OBJ: Successfully perform hygiene care of the integument; perineum; feet, hands, and nails; mouth; and eyes, ears, and nose. TOP: Implementation MSC: NCLEX: Health Promotion and Maintenance 36. The nurse is providing perineal care to an uncircumcised male patient. When providing such
care, how should the nurse treat the foreskin? a. Leave the foreskin alone because there is little chance of infection. b. Retract the foreskin for cleansing and allow it to return on its own. c. Retract the foreskin and keep retracted. d. Retract the foreskin and return it to its natural position when done. ANS: D
Return the foreskin to its natural position. Keeping the foreskin retracted leads to tightening of the foreskin around the shaft of the penis, causing local edema and discomfort. The foreskin may not return to its natural pNosi tion . Pa tients at greatest risk for infection are UR SIonNGitsToBw.nC OM uncircumcised boys and men. DIF: Apply REF: 923 OBJ: Successfully perform hygiene care of the integument; perineum; feet, hands, and nails; mouth; and eyes, ears, and nose. TOP: Implementation MSC: NCLEX: Health Promotion and Maintenance 37. Patients with diabetes mellitus need special foot care to prevent the development of ulcers.
Knowing this, what is the nurse’s action? a. Trimming the patient’s toenails daily. b. Having the patient soak his or her feet twice a day. c. Requesting a consult with a nail care specialist. d. Assessing the brachial artery. ANS: C
Patients with peripheral vascular disease or diabetes mellitus often require nail care from a specialist to reduce the risk of infection. Some agencies allow cutting of nails with a provider’s order; however, most do not. Patients with diabetes do not soak hands and feet; soaking increases the risk of infection because of maceration of the skin. When assessing the patient’s feet, the nurse palpates the dorsalis pedis of the foot, not the brachial artery. DIF: Apply REF: 926-927 OBJ: Successfully perform hygiene care of the integument; perineum; feet, hands, and nails; mouth; and eyes, ears, and nose. TOP: Implementation
Canadian Fundamentals of Nursing 6th Edition Potter Test Bank MSC: NCLEX: Health Promotion and Maintenance 38. The uncooperative patient is resisting attempts by the nurse to provide oral hygiene care. To
provide the needed care, what may the nurse do? a. Use a padded tongue blade. b. Hold the patient’s mouth open with his or her own fingers. c. Position the patient on his or her back. d. Use undiluted hydrogen peroxide as a cleaner. ANS: A
If the patient is unconscious or uncooperative or is having difficulty keeping the mouth open, a padded tongue blade can help. It is inserted over the tongue to keep the teeth apart when the patient is relaxed. The nurse must not use force. The nurse must never place fingers into the mouth of an unconscious or debilitated patient; the normal response of the patient is to bite down. The patient should be positioned on the side, or the head turned, to allow for drainage. Placing the patient on the back could lead to aspiration. Hydrogen peroxide and sodium bicarbonate effectively remove debris but, if not diluted carefully, may cause superficial burns. DIF: Apply REF: 934 OBJ: Successfully perform hygiene care of the integument; perineum; feet, hands, and nails; mouth; and eyes, ears, and nose. TOP: Implementation MSC: NCLEX: Safe and Effective Care Environment 39. How is basic eye care provided? a. The nurse cleanses the eye with soap and water. b. The nurse applies pressure directly to the eyeball. c. The nurse cleanses from N inU neR rS caI ntN hG usTtoBo.uC teO r cManthus. d. The nurse provides less frequent care to unconscious patients. ANS: C
When cleansing the patient’s eyes, the nurse should obtain a clean washcloth and cleanse from inner canthus to outer canthus. A different section of the washcloth should be used for each eye. Cleansing simply involves washing with a clean washcloth moistened in water. Soap causes burning and irritation. Direct pressure must never be applied over the eyeball because this causes serious injury. Unconscious patients often require more frequent eye care than do conscious patients. DIF: Apply REF: 939 OBJ: Successfully perform hygiene care of the integument; perineum; feet, hands, and nails; mouth; and eyes, ears, and nose. TOP: Implementation MSC: NCLEX: Physiological Integrity 40. The nurse is teaching a patient about contact lens care. The patient has plastic lenses, so what
does the nurse instruct the patient to do? a. Use tap water to clean lenses. b. Keep the lenses is a cool dry place when not being used. c. Reuse storage solution for up to a week. d. Wash and rinse lens storage case daily. ANS: D
Canadian Fundamentals of Nursing 6th Edition Potter Test Bank The lens storage case should be thoroughly washed and rinsed on a daily basis. It should be cleaned periodically with soap or liquid detergent, rinsed thoroughly with warm water, and allowed to air dry. Tap water should not be used to clean lenses because tap water is not sterile and can introduce microorganisms. Lenses should be kept moist or wet when not worn. Fresh solution is used daily when lenses are stored and disinfected. DIF: Apply REF: 939 (Box 38-14) OBJ: Successfully perform hygiene care of the integument; perineum; feet, hands, and nails; mouth; and eyes, ears, and nose. TOP: Implementation MSC: NCLEX: Health Promotion and Maintenance 41. The patient complains to the nurse about a perceived decrease in hearing. When the nurse
examines the patient’s ear, she notices a large amount of cerumen (ear wax) buildup at the entrance to the ear canal. What should the nurse do? a. Apply gentle, downward retraction of the ear canal. b. Tell the patient to use a bobby pin to extract earwax. c. Teach the patient how to use cotton-tipped applicators. d. Instill hot water into the ear canal to melt the wax. ANS: A
When cerumen is visible, gentle, downward retraction at the entrance to the ear canal causes the wax to loosen and slip out. The patient must never use sharp objects such as bobby pins or paper clips to remove earwax; use of such objects traumatizes the ear canal and ruptures the tympanic membrane. Cotton-tipped applicators should be avoided as well because they cause earwax to become impacted within the canal. Instilling cold or hot water into the ear causes nausea, vomiting, or both. DIF: Apply REF: 94N 0URSINGTB.COM OBJ: Successfully perform hygiene care of the integument; perineum; feet, hands, and nails; mouth; and eyes, ears, and nose. TOP: Implementation MSC: NCLEX: Physiological Integrity 42. The patient is being fitted with a hearing aid. In teaching the patient how to care for the
hearing aid, the nurse instructs the patient to do which of the following? a. Wear the hearing aid 24 hours per day except when sleeping. b. Change the battery every day or as needed. c. Avoid the use of hairspray, but aerosol perfumes are allowed. d. Adjust the volume for a talking distance of 1 metre. ANS: D
The volume is adjusted to a comfortable level for talking at a distance of 1 metre. Initially, the patient should wear a hearing aid for 15 to 20 minutes; then wear time is gradually increased to 10 to 12 hours per day. Batteries last 1 week with daily wear of 10 to 12 hours. People who wear hearing aids should avoid the use of hairspray and perfume; residue from the spray can cause the hearing aid to become oily and greasy. DIF: Apply REF: 941 (Box 38-15) OBJ: Successfully perform hygiene care of the integument; perineum; feet, hands, and nails; mouth; and eyes, ears, and nose. TOP: Implementation MSC: NCLEX: Physiological Integrity
Canadian Fundamentals of Nursing 6th Edition Potter Test Bank 43. The patient is complaining of an inability to clear his nasal passages. How should the nurse
instruct the patient? a. To blow his nose forcefully to clear the passage. b. To insert a cotton-tipped applicator as far as possible. c. To apply gentle suction with a pediatric bulb suction device. d. To use a dry washcloth to absorb secretions. ANS: C
Excessive nasal secretions can be removed with gentle suctioning. However, patients usually remove secretions from the nose by gentle blowing into a soft tissue. The patient should be cautioned against harsh blowing, which creates pressure capable of injuring the eardrum, the nasal mucosa, and even sensitive eye structures. If the patient is unable to remove nasal secretions, the nurse can assist by using a wet washcloth or a cotton-tipped applicator moistened in water or saline. The applicator must never be inserted beyond the length of the cotton tip. DIF: Apply REF: 934 OBJ: Successfully perform hygiene care of the integument; perineum; feet, hands, and nails; mouth; and eyes, ears, and nose. TOP: Implementation MSC: NCLEX: Health Promotion and Maintenance 44. Of the following hearing aids, which interferes the most with wearing eyeglasses and using a
phone? a. In-the-canal hearing aid. b. In- the-ear hearing aid. c. Behind-the-ear hearing aid. d. They are all equally useful. ANS: C
I G B.C M NURS N T O
The behind-the-ear aid hooks around and behind the ear and is connected by a short, clear, hollow plastic tube to an ear mould inserted into the external auditory canal. It is useful for patients with rapidly progressive hearing loss or manual dexterity difficulties, but it is more visible and interferes with wearing eyeglasses and using a phone. An in-the-canal aid is the newest, smallest, and least visible type of hearing aid; it fits entirely in the ear canal. It does not interfere with wearing eyeglasses or using the telephone, but it does not accommodate progressive hearing loss, and manual dexterity is needed to operate it. An in-the-ear aid does not interfere with wearing of eyeglasses or phone usage, but it is more noticeable than the in-the-canal aid and is not useful for persons with skin problems in the ear canal. DIF: Evaluate REF: 940 OBJ: Successfully perform hygiene care of the integument; perineum; feet, hands, and nails; mouth; and eyes, ears, and nose. TOP: Assessment MSC: NCLEX: Health Promotion and Maintenance MULTIPLE RESPONSE 1. The use of critical thinking attitudes is necessary to design a plan of care to meet the patient’s
hygiene needs. Which of the following are considered critical thinking attributes? (Select all that apply.) a. Curiosity. b. Communication principles.
Canadian Fundamentals of Nursing 6th Edition Potter Test Bank c. Prior experience. d. Humility. e. Knowledge of cultural variations. ANS: A, D
Use of critical thinking attitudes, such as curiosity and humility, is necessary to design a plan of care to meet the patient’s hygiene needs. Communication principles and knowledge of cultural variations in hygiene are considered knowledge elements, and prior experience is part of the experience elements of the critical thinking model for hygiene assessment. DIF: Remember REF: 906 OBJ: Discuss the role that the nursing process and critical thinking play in the provision of hygiene care. TOP: Implementation MSC: NCLEX: Safe and Effective Care Environment 2. Of the following patients, which are in need of help with perineal care? (Select all that apply.) a. A patient with urinary and fecal incontinence. b. A circumcised male who is ambulatory. c. A patient with rectal and perineal surgical dressings. d. A patient with an indwelling catheter. e. A morbidly obese patient. ANS: A, C, D, E
Patients at greatest risk for skin breakdown in the perineal area are those with urinary or fecal incontinence, rectal and perineal surgical dressings, indwelling urinary catheters, or a great deal of fat in the perineal area. Circumcised boys and men are not at high risk for acquiring infection, and most ambulatory patients can provide perineal self-care. DIF: Evaluate REF: 92N 6URSINGTB.COM OBJ: Successfully perform hygiene care of the integument; perineum; feet, hands, and nails; mouth; and eyes, ears, and nose. TOP: Assessment MSC: NCLEX: Health Promotion and Maintenance 3. The patient has been forcefully blowing his nose and now has a nosebleed. The nurse is
concerned about the patient’s condition and assesses the patient for which possible negative issues? (Select all that apply.) a. Clearance of nasal passages. b. Injury to the tympanic membrane (eardrum). c. Damage to nasal mucosa. d. Eye injury. e. Decreased nasal passage pressure. ANS: B, C, D
Caution the patient against harsh blowing that creates pressure capable of injuring the eardrum, nasal mucosa, and even sensitive eye structures. Clearing the nasal passages is the goal of nose blowing and is not a negative issue. Harsh blowing increases (not decreases) nasal passage pressure. DIF: Evaluate REF: 940 OBJ: Successfully perform hygiene care of the integument; perineum; feet, hands, and nails; mouth; and eyes, ears, and nose. TOP: Assessment MSC: NCLEX: Physiological Integrity
Canadian Fundamentals of Nursing 6th Edition Potter Test Bank
Chapter 39: Cardiopulmonary Functioning and Oxygenation Potter et al: Canadian Fundamentals of Nursing, 6th Edition MULTIPLE CHOICE 1. What is the structure that is responsible for returning oxygenated blood to the heart? a. Pulmonary artery. b. Pulmonary vein. c. Superior vena cava. d. Inferior vena cava. ANS: B
The pulmonary vein carries oxygenated blood to the heart. The pulmonary artery carries deoxygenated blood from the heart to the lungs. Both venae cavae return blood to the right atrium of the heart. DIF: Remember REF: 958 OBJ: Describe the structure and function of the cardiopulmonary system. TOP: Planning MSC: NCLEX: Physiological Integrity 2. Where are chemical receptors that stimulate inspiration located? a. Brain. b. Lungs. c. Aorta. d. Heart. ANS: C
NURSINGTB.COM
Chemical receptors in the aorta send signals to begin the inspiration process. The brain, lungs, and heart all are affected by this chemical reaction. DIF: Remember REF: 957 OBJ: Describe the neural and chemical regulation of respiration. TOP: Planning MSC: NCLEX: Physiological Integrity 3. The nurse knows that the primary function of the alveoli is which of the following? a. Carry out gas exchange. b. Store oxygen. c. Regulate tidal volume. d. Produce hemoglobin. ANS: A
The alveolus is a capillary membrane that allows gas exchange of oxygen and carbon dioxide during respiration. The alveoli do not store oxygen, regulate tidal volume, or produce hemoglobin. DIF: Remember REF: 956 OBJ: Describe the structure and function of the cardiopulmonary system. TOP: Knowledge MSC: NCLEX: Physiological Integrity 4. What will anemia result in? a. Hypoxemia.
Canadian Fundamentals of Nursing 6th Edition Potter Test Bank b. Impaired ventilation. c. Hypovolemia. d. Decreased lung compliance. ANS: A
Patients who are anemic do not have normal oxygen-carrying capacity. As a result, oxygen cannot properly perfuse the tissues, which results in hypoxemia. Ventilation is impaired when oxygen/carbon dioxide exchange occurs at the alveolar level. Hypovolemia is a decrease in circulating blood volume. Lung compliance refers to the elasticity of the lung tissue. DIF: Understand REF: 959 OBJ: Identify and describe clinical outcomes occurring as a result of disturbances in conduction, altered cardiac output, impaired valvular function, myocardial ischemia, and impaired tissue perfusion. TOP: Assessment MSC: NCLEX: Physiological Integrity 5. The process of exchanging gases through the alveolar capillary membrane is known as which
of the following? a. Disassociation. b. Diffusion. c. Perfusion. d. Ventilation. ANS: B
Diffusion is the process of gas exchange across the alveoli and capillaries of body tissues. Disassociation is not related to oxygenation. Perfusion is the ability of the cardiovascular system to carry oxygenated blood to tissues and return deoxygenated blood to the heart. Ventilation is the process of moving gases into and out of the lungs.
N R I G B.C M
U S N T O DIF: Remember REF: 958 OBJ: Identify the physiological processes involved in ventilation, perfusion, and exchange of respiratory gases. TOP: Assessment MSC: NCLEX: Physiological Integrity 6. In care for a patient who was in a motor vehicle accident that resulted in trauma to C4, what
would the nurse expect to find? a. Decreased tidal volumes. b. Increased perfusion. c. Increased use of accessory muscles. d. Decreased hemoglobin. ANS: A
A C4 injury would result in damage to the phrenic nerve and would cause a decrease in inspiratory lung expansion. Accessory muscles are also be damaged by a C4 injury. The patient may exhibit decreased perfusion and increased hemoglobin to compensate for hypoxemia. DIF: Understand REF: 970 (Table 39-3) OBJ: Describe the neural and chemical regulation of respiration. TOP: Assessment MSC: NCLEX: Physiological Integrity 7. In which condition would the nurse would expect to see increased ventilations? a. Increased oxygen saturation. b. Decreased carbon dioxide levels.
Canadian Fundamentals of Nursing 6th Edition Potter Test Bank c. Decreased pH. d. Increased hemoglobin levels. ANS: C
Retained CO2 creates H+ byproducts that lower pH. This sends a chemical signal to increase respiratory rate and would result in increased ventilation. All the other options would cause the ventilation rate to normalize or decrease in order to increase carbon dioxide retention or as the result of delivery of higher levels of oxygen to tissues. DIF: Understand REF: 970 (Table 39-3) OBJ: Describe the neural and chemical regulation of respiration. TOP: Assessment MSC: NCLEX: Physiological Integrity 8. Why does the nurse recommend that a patient install a carbon monoxide detector in the home? a. It is required by law. b. Carbon monoxide tightly bonds to hemoglobin, causing hypoxia. c. Carbon monoxide signals the cerebral cortex to cease ventilations. d. Carbon monoxide combines with oxygen in the body and produces a deadly toxin. ANS: B
Carbon monoxide has a high affinity for hemoglobin; therefore, oxygen is not able to bond to hemoglobin and be transported to tissues. A carbon monoxide detector is not required by law, and carbon monoxide does not signal the cerebral cortex to cease ventilations, and combines not with oxygen but with hemoglobin to produce a toxin. DIF: Remember REF: 968 OBJ: Identify nursing care interventions in the primary care, acute care, and restorative and continuing care settings that proN motR e oxI ygeG natioBn. .C M TOP: Planning MSC: NCLEX: Physiological IntU egriS ty N
T
O
9. While performing an assessment, the nurse hears crackles in the patient’s lung fields. The
nurse also learns that the patient is sleeping on three pillows. What do these symptoms most likely indicate? a. Left-sided heart failure. b. Right-sided heart failure. c. Atrial fibrillation. d. Myocardial ischemia. ANS: A
Left-sided heart failure results in pulmonary congestion, the signs and symptoms of which include shortness of breath, crackles, and discomfort in the supine position. Right-sided heart failure is systemic and results in peripheral edema and hepatojugular distension. Atrial fibrillation is an irregular heart rate. Myocardial ischemia most often results in chest pain, along with shortness of breath, nausea, and fatigue. DIF: Understand REF: 961 OBJ: Identify the physiological processes involved in ventilation, perfusion, and exchange of respiratory gases. TOP: Assessment MSC: NCLEX: Physiological Integrity 10. The nurse knows that a myocardial infarction is an occlusion of what blood vessel? a. Pulmonary artery. b. Ascending aorta.
Canadian Fundamentals of Nursing 6th Edition Potter Test Bank c. Coronary artery. d. Carotid artery. ANS: C
A myocardial infarction is the lack of blood flow as a result of obstruction to the coronary artery, which supplies the heart with blood. The ascending aorta is a vessel that leads from the heart to perfuse the brain. The pulmonary artery supplies blood to the lungs. The carotid artery supplies blood to the brain. DIF: Remember REF: 963 OBJ: Identify the physiological processes of cardiac output, myocardial blood flow, and coronary artery circulation. TOP: Assessment MSC: NCLEX: Physiological Integrity 11. Myocardial blood flow is unidirectional; the nurse knows that the correct pathway is which of
the following? a. Right atrium, right ventricle, left ventricle, left atrium. b. Right atrium, left atrium, right ventricle, left ventricle. c. Right atrium, right ventricle, left atrium, left ventricle. d. Right atrium, left atrium, left ventricle, right ventricle. ANS: C
Unoxygenated blood flows through the venae cavae into the right atrium, where it is pumped down to the right ventricle; the blood is then pumped out the pulmonary artery and is returned oxygenated via the pulmonary vein to the left atrium, where it flows to the left ventricle and is pumped out to the rest of the body via the aorta. DIF: Remember REF: 954 OBJ: Describe the structure anN d fuR nctiI on oG f the cardiopMulmonary system. U SPhysiological N TB.CIntegrity O TOP: Pathology MSC: NCLEX: 12. The nurse caring for a patient with ischemia to the left coronary artery would expect to find
which sign? a. Increased ventricular diastole. b. Increased stroke volume. c. Decreased preload. d. Decreased afterload. ANS: D
The left coronary artery supplies the muscles of the left ventricle; the strength of the muscle affects the contractility of the heart. The other options are not affected by the muscles of the left ventricle. DIF: Understand REF: 963 OBJ: Identify the physiological processes of cardiac output, myocardial blood flow, and coronary artery circulation. TOP: Assessment MSC: NCLEX: Physiological Integrity 13. Normal cardiac output is 2.5 to 4 L/minute/m3 in a healthy adult at rest. Which of the
following is the correct formula to calculate cardiac output? a. Stroke volume heart rate. b. Stroke volume/body surface area. c. Body surface area cardiac index. d. Heart rate/stroke volume.
Canadian Fundamentals of Nursing 6th Edition Potter Test Bank
ANS: A
Cardiac output can be calculated by multiplying the stroke volume and the heart rate. The other options are not measures of cardiac functioning. DIF: Remember REF: 955 OBJ: Identify the physiological processes of cardiac output, myocardial blood flow, and coronary artery circulation. TOP: Assessment MSC: NCLEX: Physiological Integrity 14. The nurse is careful to monitor a patient’s cardiac output because this helps the nurse to
determine which of the following? a. Peripheral extremity circulation. b. Oxygenation requirements. c. Cardiac arrhythmias. d. Ventilation status. ANS: A
Cardiac output indicates how much blood is being circulated systemically. Oxygen status would be determined by pulse oximetry and the presence of cyanosis. Cardiac arrhythmias are electrical impulses monitored through 5-lead electrocardiography (ECG). Ventilation status is not solely dependent on cardiac output. DIF: Apply REF: 955 OBJ: Describe the relationship among cardiac output, preload, afterload, contractility, and heart rate. TOP: Assessment MSC: NCLEX: Physiological Integrity 15. A nurse is assisting a patient with ambulation. The patient becomes short of breath and begins
to complain of sharp chest pain. Which action by the nurse is the first priority? N R eIteNam GTtoB.C M a. Call for the emergency respUonsS bringOthe defibrillator. b. Have the patient sit down in the nearest chair. c. Return the patient to the room and apply 100% oxygen. d. Ask a co-worker to get the ECG machine STAT. ANS: B
The patient is experiencing cardiac distress for reasons unknown. The nurse should first secure the safety of the patient and decrease the workload on the patient’s heart by putting him in a resting position; this will increase cardiac output by decreasing afterload. Once the patient is stable, the nurse can obtain oxygen to put on the patient. Next, the nurse can begin to monitor the patient’s oxygen and cardiac status. If necessary, the emergency team may be activated to defibrillate. DIF: Apply REF: 964| 967 OBJ: Identify nursing care interventions in the primary care, acute care, and restorative and continuing care settings that promote oxygenation. TOP: Implementation MSC: NCLEX: Physiological Integrity 16. A patient has inadequate stroke volume related to decreased preload. What action does the
nurse anticipate? a. Placing the patient on oxygen monitoring. b. Administering vasodilators. c. Verifying that the blood consent form has been signed. d. Preparing the patient for dialysis.
Canadian Fundamentals of Nursing 6th Edition Potter Test Bank
ANS: C
Preload is affected by the circulating volume; if the patient has decreased fluid, it will need to be replaced with fluid or blood therapy. Before administering blood products, typing and matching should be performed. Monitoring the patient’s oxygenation status will not affect preload. Administering vasodilators affects afterload. Dialysis would further remove fluid from the patient, thus decreasing preload. DIF: Apply REF: 955 OBJ: Describe the relationship among cardiac output, preload, afterload, contractility, and heart rate. TOP: Planning MSC: NCLEX: Physiological Integrity 17. When caring for a patient with atrial fibrillation, the nurse is most concerned with which vital
sign? a. Heart rate. b. Pain. c. Oxygen saturation. d. Blood pressure. ANS: C
Atrial fibrillation results in pooling of blood in the atria, forming emboli that can be pumped out to the rest of the body. The most common manifestations are stroke, myocardial infarction, and pulmonary embolus. A sudden and drastic drop in oxygenation and blood pressure can indicate both pulmonary embolus and myocardial infarction. DIF: Apply REF: 960 OBJ: Identify and describe clinical outcomes occurring as a result of disturbances in conduction, altered cardiac output, impairedNvalR vulaI r funGctioB n,.mCyocM ardial ischemia, and impaired tissue perfusion. TOP:
AssessmenU t
S N MTSC: NCOLEX: Physiological Integrity
18. The nurse would expect a patient with right-sided heart failure to have which of the
following? a. Peripheral edema. b. Basilar crackles. c. Chest pain. d. Cyanosis. ANS: A
Right-sided heart failure results from inability of the right side of the heart to pump effectively, which leads to a systemic backup. Peripheral edema and hepatojugular distension are signs of right-sided failure. Basilar crackles can indicate pulmonary congestion from left-sided heart failure. Cyanosis and chest pain result from inadequate tissue perfusion. DIF: Remember REF: 970 (Table 39-4) OBJ: Identify nursing care interventions in the primary care, acute care, and restorative and continuing care settings that promote oxygenation. TOP: Assessment MSC: NCLEX: Physiological Integrity 19. The P wave is represented by which portion of the conduction system? a. Sinoatrial (SA) node. b. Atrioventricular (AV) node. c. Bundle of His.
Canadian Fundamentals of Nursing 6th Edition Potter Test Bank d. Purkinje network. ANS: A
The SA node initiates electrical conduction through the atria. The AV node conducts down through the bundle of His and the Purkinje network to cause ventricular contraction. DIF: Remember REF: 955 OBJ: Describe the relationship among cardiac output, preload, afterload, contractility, and heart rate. TOP: Evaluate MSC: NCLEX: Physiological Integrity 20. Which statement by the patient indicates an understanding of atelectasis? a. “It is important to do breathing exercises every hour to prevent atelectasis.” b. “If I develop atelectasis, I will need a chest tube to drain excess fluid.” c. “Atelectasis affects only people with chronic conditions such as emphysema.” d. “Hyperventilation will open up my alveoli, preventing atelectasis.” ANS: A
Atelectasis develops when alveoli do not expand. Breathing exercises increase lung volume and open the airways. Deep breathing opens the pores of Kohn between the alveoli to allow sharing of oxygen between alveoli. This prevents atelectasis from developing. DIF: Analyze REF: 957 OBJ: Identify and describe clinical outcomes of hyperventilation, hypoventilation, and hypoxemia. TOP: Evaluate MSC: NCLEX: Physiological Integrity 21. A nurse is caring for a patient whose temperature is 37.9°C (100.2°F). The nurse expects this
patient to hyperventilate for which reason? a. Increased metabolic demands. I G B.C M N R U S N T O b. Anxiety over illness. c. Decreased drive to breathe. d. Infection destroying lung tissues. ANS: A
Fever increases the metabolic demands of the body, increasing production of carbon dioxide. The body hyperventilates to get rid of excess carbon dioxide. Anxiety can cause hyperventilation, but this is not because of fever. Hyperventilation decreases the drive to breathe. The cause of the fever in this question is unknown. DIF: Apply REF: 964 OBJ: Identify and describe clinical outcomes of hyperventilation, hypoventilation, and hypoxemia. TOP: Assessment MSC: NCLEX: Physiological Integrity 22. What assessment finding is the earliest sign of hypoxia? a. Restlessness. b. Decreased blood pressure. c. Cardiac dysrhythmias. d. Cyanosis. ANS: A
Canadian Fundamentals of Nursing 6th Edition Potter Test Bank Hypoxia is due to inadequate tissue oxygen at the cellular level. The earliest sign of hypoxia is restlessness; as it progresses, mental status changes, cardiac changes, and cyanosis can occur. Early hypoxia results in elevations in blood pressure. In later hypoxia, vital sign changes such as increases in heart and respiratory rates occur. Cyanosis is a late sign of hypoxia. DIF: Understand REF: 964 OBJ: Identify and describe clinical outcomes of hyperventilation, hypoventilation, and hypoxemia. TOP: Assessment MSC: NCLEX: Physiological Integrity 23. A 5-year-old who has “strep throat” was given aspirin for fever. The nurse knows to expect
which change in the child’s respiratory pattern? a. Hyperventilation to decrease serum levels of carbon dioxide. b. Hypoventilation to compensate for metabolic alkalosis. c. Flail chest to decrease the work of breathing. d. Shallow respirations to decrease serum pH. ANS: A
Aspirin causes an increase in carbon dioxide; the body compensates for this by increasing ventilations to blow off excess carbon dioxide. Hypoventilation would cause the body to retain even more carbon dioxide and therefore respiratory acidosis. Flail chest results from trauma to the chest wall. Shallow respirations would increase serum pH. DIF: Apply REF: 964 OBJ: Describe the effect of a patient’s level of health, age, lifestyle, and environment on cardiopulmonary oxygenation. TOP: Assessment MSC: NCLEX: Physiological Integrity
t wh uffe mC yoc ardial infarction to the left coronary artery. 24. A nurse is caring for a patienN URoSsI NGredTBa . OM Upon assessment, what would the nurse expect to find? a. Blood in the sputum. b. Distended jugular vein. c. Peripheral edema. d. Crackles in the lungs. ANS: D
The left coronary artery supplies the left ventricle of the heart; damage to the muscle in the left ventricle results in pulmonary congestion and frothy sputum, and crackles may be heard. A distended jugular vein and peripheral edema are associated with damage to the right side of the heart. Blood in the sputum is indicative of an infection such as tuberculosis. DIF: Analyze REF: 961 OBJ: Identify and describe clinical outcomes occurring as a result of disturbances in conduction, altered cardiac output, impaired valvular function, myocardial ischemia, and impaired tissue perfusion. TOP: Assessment MSC: NCLEX: Physiological Integrity 25. A nurse is caring for a patient who has poor tissue perfusion as the result of hypertension.
When the patient asks what he should eat for breakfast, what should the nurse recommend? a. A bowl of cereal with whole milk and a banana. b. A cup of nonfat yogurt with granola, and a handful of dried apricots. c. Whole wheat toast with butter, a side of cottage cheese. d. Omelette with sausage, cheese, and onions.
Canadian Fundamentals of Nursing 6th Edition Potter Test Bank ANS: B
Diets high in potassium, fibre, and calcium and low in fat are best for someone who is managing hypertension. Nonfat yogurt with granola is a good source of calcium, fibre, and potassium; dried apricots add a second source of potassium. Although cereal and a banana provide fibre and potassium, skim milk should be substituted for whole milk to decrease fat. An omelette with sausage and cheese is high in fat. Butter is high in fat. DIF: Apply REF: 966 OBJ: Identify nursing care interventions in the primary care, acute care, and restorative and continuing care settings that promote oxygenation. TOP: Planning MSC: NCLEX: Health Promotion and Maintenance 26. Upon auscultation, the nurse hears a whooshing sound at the fifth intercostal space. What does
this sound represent? a. The beginning of the systolic phase. b. The opening of the aortic valve. c. S3, the third heart sound. d. Regurgitation of the mitral valve. ANS: D
A whooshing sound at the fifth intercostal space is a murmur; a prolapsed valve allows regurgitation that is heard as a whooshing sound. The systolic phase begins with ventricular filling and closing of the aortic valve, which is heard as the first heart sound (S1). The third heart sound (S3) is heard with heart failure. DIF: Understand REF: 954 OBJ: Identify and describe clinical outcomes occurring as a result of disturbances in conduction, altered cardiac output, impairedNval vula r fun UR SI NGctio TBn,.mCyoc OMardial ischemia, and impaired tissue perfusion. TOP: Application MSC: NCLEX: Physiological Integrity 27. A nurse caring for a patient with COPD knows that which oxygen delivery device is most
appropriate? a. Nasal cannula. b. Simple face mask. c. Partial non-rebreather mask. d. Non-rebreather mask. ANS: A
Nasal cannulas deliver oxygen from 1 to 6 L/min. A patient with COPD should never receive more than 3 L/min because this decreases the drive to breathe, resulting in hypoventilation. All other devices are intended for flow rates greater than 6 L/min. DIF: Analyze REF: 979| 1002 OBJ: Identify nursing care interventions in the primary care, acute care, and restorative and continuing care settings that promote oxygenation. TOP: Planning MSC: NCLEX: Physiological Integrity 28. The nurse determines that an elderly patient is at risk for infection due to decreased immunity.
Which plan of care best addresses the prevention of infection for the patient? a. Encourage the patient to stay up to date on all vaccinations. b. Inform the patient of the importance of finishing the entire dose of antibiotics. c. Schedule patient to get annual tuberculosis skin testing.
Canadian Fundamentals of Nursing 6th Edition Potter Test Bank d. Create an exercise routine to run 30 minutes every day. ANS: A
A nursing care plan for preventative health measures should be reasonable and feasible. Keeping up to date on vaccinations is important because vaccine reduces the severity of illnesses and serious complications. Although it is important to finish the full course of antibiotics, it is not a preventative health measure. Scheduling annual tuberculosis skin tests does not address prevention. The exercise routine should be reasonable to increase compliance. DIF: Apply REF: 977 OBJ: Identify nursing care interventions in the primary care, acute care, and restorative and continuing care settings that promote oxygenation. TOP: Planning MSC: NCLEX: Health Promotion and Maintenance 29. The nurse would expect which change in cardiac output for a patient with fluid volume
overload? a. Increased preload. b. Decreased afterload. c. Decreased tissue perfusion. d. Increased heart rate. ANS: A
Preload refers to the stretch of the ventricle related to the volume of blood; an increase in circulating volume would increase the preload of the heart. Afterload refers to resistance; increased pressure would lead to increased resistance, and afterload would increase. A decrease in tissue perfusion would be seen with hypovolemia. A decrease in fluid volume would cause an increase in hN eartR rateIas G the B bo.dC y is M attempting to increase cardiac output.
U S N T
O
DIF: Understand REF: 955 OBJ: Describe the relationship of cardiac output, preload, afterload, contractility, and heart rate. TOP: Planning MSC: NCLEX: Physiological Integrity 30. A nurse is caring for a patient with chronic obstructive pulmonary disease (COPD) who is
recovering from a myocardial infarction. Which of the following nursing actions is the priority? a. Initiate continuous cardiac monitoring. b. Administer 6 L/min of oxygen via nasal cannula. c. Deep suction the patient every 2 hours. d. Assess bilateral lung sounds every hour. ANS: A
In a patient who has a recent myocardial infarction, heart rhythm can convert back to a deadly pattern, and the patient needs to be placed on continuous cardiac monitoring. The patient has COPD and should not given oxygen at a rate higher than 3 L/min. Proper cardiac functioning allows oxygenated blood to be distributed to tissues. Patients with recent myocardial infarction should not be suctioned. This patient does not have any indicators to warrant hourly assessment of lung fields. DIF: Analyze REF: 960 OBJ: Identify nursing care interventions in the primary care, acute care, and restorative and continuing care settings that promote oxygenation. TOP: Implementation
Canadian Fundamentals of Nursing 6th Edition Potter Test Bank MSC: NCLEX: Physiological Integrity 31. The nurse expects which of the following in a patient with angina pectoris? a. Experience feelings of indigestion after eating a heavy meal. b. Have decreased oxygen saturation during rest. c. Hypoventilate during periods of acute stress. d. Complain of tingling in the left arm that lasts throughout the morning. ANS: A
Angina pectoris is chest pain that results from limited oxygen supply. Often pain is precipitated by activities such as exercise, stress, and eating a heavy meal and lasts from 1 to 15 minutes. Hyperventilation may occur to compensate for decreased oxygen perfusion and during periods of acute stress. Symptoms of angina pectoris are relieved by rest, nitroglycerine, or both. Oxygen saturation, pain, and tingling in the arm should be relieved by rest. Pain or arm tingling that persists could be a sign of myocardial infarction. DIF: Understand REF: 963 OBJ: Identify and describe clinical outcomes occurring as a result of disturbances in conduction, altered cardiac output, impaired valvular function, myocardial ischemia, and impaired tissue perfusion. TOP: Planning MSC: NCLEX: Physiological Integrity 32. Which of the following is a nonmodifiable risk factor for lung disease? a. Allergies. b. Smoking. c. Stress. d. Asbestos exposure. ANS: A
NURSINGTB.COM
A nonmodifiable risk factor is one over which the patient has no control. Allergies can be managed, but immune-mediated responses cannot be controlled. Smoking, stress, and asbestos exposure are all modifiable risk factors. DIF: Understand REF: 969 OBJ: Describe the effect of a patient’s level of health, age, lifestyle, and environment on cardiopulmonary oxygenation. TOP: Assessment MSC: NCLEX: Physiological Integrity 33. The nurse is creating a plan of care for an obese patient who is suffering from fatigue related
to ineffective breathing. Which intervention best addresses a short-term goal that the patient could achieve? a. Running 30 minutes every morning. b. Stopping smoking immediately. c. Sleeping on two to three pillows at night. d. Limiting the diet to 1500 calories a day. ANS: C
Canadian Fundamentals of Nursing 6th Edition Potter Test Bank To achieve a short-term goal, the nurse should plan a lifestyle change that the patient can make immediately that will have a quick effect. Sleeping on several pillows at night will immediately relieve orthopnea and open the patient’s airway, thereby reducing sleep apnea and reducing fatigue. Running 30 minutes a day will improve cardiopulmonary health, but a patient needs to build up exercise tolerance. Smoking cessation is another helpful process, but many people have difficulty quitting immediately; quitting often occurs as a slow progression, beginning with reduction of frequency. A more realistic short-term goal would be to gradually reduce the number of cigarettes smoked. Limiting caloric intake can help a patient lose weight, but this is a gradual process and is not reasonable for a short-term goal. DIF: Analyze REF: 968 OBJ: Identify nursing care interventions in the primary care, acute care, and restorative and continuing care settings that promote oxygenation. TOP: Planning MSC: NCLEX: Physiological Integrity 34. A nurse is caring for a patient with left-sided hemiparesis who has developed bronchitis and
has a heart rate of 105, blood pressure of 156/90, and a respiration rate of 30. Which nursing diagnosis is the priority for this patient? a. Activity intolerance. b. Risk for skin breakdown. c. Impaired gas exchange. d. Risk for infection. ANS: C
The most important nursing intervention is to maintain airway and circulation for this patient; therefore, Impaired gas exchange is the first-priority nursing diagnosis. Activity intolerance is a concern but is not the priority in this case. Risk for skin breakdown and Risk for infection are NUssRaSnIimNme GTdiate B.CimOpairment M also important but do not addre with physiological integrity. DIF: Analyze REF: 973 OBJ: Identify nursing care interventions in the primary care, acute care, and restorative and continuing care settings that promote oxygenation. TOP: Planning MSC: NCLEX: Physiological Integrity 35. Which nursing intervention is most effective in preventing hospital-acquired pneumonia in an
elderly patient? a. Assisting the patient to cough, turn, and deep breathe every 2 hours. b. Encouraging patient to drink through a straw to prevent aspiration. c. Discontinuing humidification delivery to keep excess fluid from the lungs. d. Monitoring oxygen saturation and frequently assessing lung bases. ANS: A
The goal of the nursing action should be the prevention of pneumonia; the action that best addresses this is to help the patient cough, turn, and deep breathe to keep secretions from pooling at the base of the lungs. Drinking through a straw increases the risk of aspiration. Humidification thins respiratory secretions, which makes them easier to expel. Monitoring oxygen status is important but is not a method of prevention. DIF: Analyze REF: 978| 979 OBJ: Identify nursing care interventions in the primary care, acute care, and restorative and continuing care settings that promote oxygenation. TOP: Implementation
Canadian Fundamentals of Nursing 6th Edition Potter Test Bank MSC: NCLEX: Health Promotion and Maintenance 36. The nurse is assessing a patient with emphysema. Which assessment finding requires further
follow-up with the physician? a. Clubbing of the fingers. b. Increased anterior-posterior diameter of the chest. c. Hemoptysis. d. Tachypnea. ANS: C
Hemoptysis is an abnormal occurrence in emphysema, and further diagnostic studies are needed to determine the cause of blood in the sputum. Clubbing of the fingers, barrel chest, and tachypnea are all typical findings in a patient with emphysema. DIF: Apply REF: 968 OBJ: Identify nursing care interventions in the primary care, acute care, and restorative and continuing care settings that promote oxygenation. TOP: Assessment MSC: NCLEX: Physiological Integrity 37. A patient with COPD asks the nurse why he is having increased difficulty with his fine motor
skills, such as buttoning his shirt. Which response by the nurse is most therapeutic? a. “Your body isn’t receiving enough oxygen to send down to your fingers; this causes them to club and makes dexterity difficult.” b. “Your disease process makes even the smallest tasks seem exhausting. Try taking a nap before getting dressed.” c. “Often patients with your disease lose mental status and forget how to perform daily tasks.” d. “Your disease affects botN hU yoRuS r lI unNgG sT anBd.yC ouOr M heart, and not enough blood is being pumped, so you are losing sensory feedback in your extremities.” ANS: A
Clubbing of the nail bed is a common symptom of COPD and can make activities of daily living difficult. Taking a nap decreases fatigue but does not help the patient perform fine motor skills. Loss of mental status is not a typical finding with COPD. Low oxygen level, not low circulating blood volume, is the problem in COPD. DIF: Apply REF: 969 OBJ: Describe the effect of a patient’s level of health, age, lifestyle, and environment on cardiopulmonary oxygenation. TOP: Patient Teaching MSC: NCLEX: Physiological Integrity 38. The nurse is caring for a patient with a tracheostomy tube. Which nursing intervention is most
effective in promoting effective airway clearance? a. Suctioning respiratory secretions several times every hour. b. Administering humidified oxygen through a tracheostomy collar. c. Instilling normal saline into the tracheostomy to thin secretions before suctioning. d. Deflating the tracheostomy cuff before allowing the patient to cough up secretions. ANS: B
Canadian Fundamentals of Nursing 6th Edition Potter Test Bank Humidification of air will help keep the mucous membranes moist and will make secretions easier to expel. Suctioning should be done only as needed; too frequent suctioning can damage the mucosal lining, which would result in thicker secretions. Normal saline should never be instilled into a tracheostomy because this could lead to infection. The purpose of the tracheostomy cuff is to keep secretions from entering the lungs; the nurse should not deflate the tracheostomy cuff unless instructed to do so by the physician. DIF: Apply REF: 1002 OBJ: Identify nursing care interventions in the primary care, acute care, and restorative and continuing care settings that promote oxygenation. TOP: Implementation MSC: NCLEX: Physiological Integrity 39. The nurse is educating a student nurse on caring for a patient with a chest tube. The nurse
knows that teaching has been effective when the student makes which statement? a. “I should strip the drains on the chest tube every hour to promote drainage.” b. “If the chest tube becomes dislodged, the first thing I should do is notify the physician.” c. “I should clamp the chest tube when giving the patient a bed bath.” d. “I should report if I do not see continuous bubbling in the water-seal chamber.” ANS: D
Correct care of a chest tube involves knowing normal and abnormal functioning of the tube. Bubbling in the water-seal chamber is expected. Stripping the drain requires a presecriber order. If the chest tube becomes dislodged, the nurse should immediately apply occlusive pressure over the insertion site. The chest tube should not be clamped unless necessary; if so, the length of time clamped would be minimal to reduce the risk of pneumothorax. DIF: Analyze REF: 99N 8URSINGTB.COM OBJ: Identify nursing care interventions in the primary care, acute care, and restorative and continuing care settings that promote oxygenation. TOP: Planning MSC: NCLEX: Physiological Integrity 40. Which nursing diagnosis is the priority in the care of a patient with a traumatic brain injury in
whom a tracheostomy is placed? a. Risk for skin breakdown. b. Impaired gas exchange. c. Ineffective airway clearance. d. Risk for infection. ANS: C
Patients with tracheostomies rely on the tracheostomy tube to provide a stable open airway. The nurse is also concerned that the patient would not be able to cough up his or her own secretions, which could occlude the tracheostomy, putting the patient at risk for Ineffective airway clearance. Nursing priorities are airway, breathing, and circulation. Nursing diagnoses that occur frequently should be addressed before “Risk” diagnoses. Skin breakdown and infection are not immediately life-threatening. DIF: Analyze REF: 980| 1005 OBJ: Identify nursing care interventions in the primary care, acute care, and restorative and continuing care settings that promote oxygenation. TOP: Planning MSC: NCLEX: Physiological Integrity
Canadian Fundamentals of Nursing 6th Edition Potter Test Bank 41. The nurse knows that the most effective method for suctioning a patient with a tracheostomy
tube is which of the following? a. Setting the suction regulator at 150 to 200 mm Hg. b. Liberally lubricating the end of the suction catheter with a water-soluble solution. c. Limiting the length of suctioning to 10 to 15 seconds. d. Applying suction while gently rotating and inserting the catheter. ANS: C
Suctioning passes should be limited to 10 to 15 seconds to avoid oxygen desaturation. Suction for a tracheostomy should be set at 100 to 150 mm Hg. Excessive lubrication can clog the catheter or occlude the airway. Suction should not be applied until after the catheter has been inserted. DIF: Apply REF: 983-986 OBJ: Identify nursing care interventions in the primary care, acute care, and restorative and continuing care settings that promote oxygenation. TOP: Implementation MSC: NCLEX: Physiological Integrity 42. The nurse is assessing a patient with a right pneumothorax. Which finding would the nurse
expect? a. Bilateral expiratory crackles. b. Absence of breath sounds on the right side. c. Right-sided wheezes on inspiration. d. Trachea deviated to the right. ANS: B
A right pneumothorax is a collection of air in the pleural space, which causes the lung to collapse; therefore, no breathNsou URnds SIare NGhea TBrd.oCnOthMat side. Crackles indicate pneumonia. Wheezes are asthma related. Collapse of the right lung would cause the trachea to deviate to the left. DIF: Apply REF: 998 OBJ: Identify the physiological processes involved in ventilation, perfusion, and exchange of respiratory gases. TOP: Assessment MSC: NCLEX: Physiological Integrity 43. While the nurse is changing the ties on a tracheostomy collar, the patient coughs, dislodging
the tracheostomy tube. What is the nurse’s first nursing action? a. Press the emergency response button. b. Place the patient on a face mask delivering 100% oxygen. c. Insert a spare tracheostomy tube. d. Manually occlude the tracheostomy with sterile gauze. ANS: C
The nurse’s first priority is to establish a stable airway by inserting a spare tracheostomy tube into the patient’s airway. The nurse could activate the emergency response team if the patient is still unstable after the tracheostomy tube is placed. A patient with a tracheostomy has an impairment that causes him or her not to be able to breathe normally; a face mask would not be an effective method of getting air into the lungs. Manually occluding pressure over the tracheostomy site is not appropriate. DIF: Apply REF: 997 OBJ: Identify nursing care interventions in the primary care, acute care, and restorative and
Canadian Fundamentals of Nursing 6th Edition Potter Test Bank continuing care settings that promote oxygenation. MSC: NCLEX: Physiological Integrity
NURSINGTB.COM
TOP: Implementation
Canadian Fundamentals of Nursing 6th Edition Potter Test Bank
Chapter 40: Fluid, Electrolyte, and Acid–Base Balances Potter et al: Canadian Fundamentals of Nursing, 6th Edition MULTIPLE CHOICE 1. Approximately two thirds of the body’s total water volume exists in which fluid? a. Intracellular. b. Interstitial. c. Intravascular. d. Transcellular. ANS: A
Intracellular fluid accounts for approximately two thirds of the fluids in the body and about 42% of total body weight. Interstitial fluid, intravascular fluid, and transcellular fluid constitute extracellular fluid, which is the fluid outside a cell. DIF: Remember REF: 1016 OBJ: Describe the distribution, composition, movement, and regulation of body fluids. TOP: Assessment MSC: NCLEX: Physiological Integrity 2. What is the process of passively moving water from an area of lower particle concentration to
an area of higher particle concentration? a. Hydrolysis. b. Osmosis. c. Filtration. d. Active transport. ANS: B
NURSINGTB.COM
The process of moving water from an area of low particle concentration to an area of higher particle concentration is known as osmosis. Hydrolysis is a term unrelated to fluid and electrolyte balance. Filtration is mediated by fluid pressure from an area of higher pressure to an area of lower pressure. Active transport requires metabolic activity and is not passive. DIF: Remember REF: 1017 OBJ: Describe the distribution, composition, movement, and regulation of body fluids. TOP: Assessment MSC: NCLEX: Physiological Integrity 3. The nurse knows that edema in a patient who has venous congestion from right-sided heart
failure is facilitated by an imbalance with regard to which pressure? a. Hydrostatic. b. Osmotic. c. Oncotic. d. Concentration. ANS: A
Venous congestion increases capillary hydrostatic pressure. Increased hydrostatic pressure causes increases in movement of fluid into the interstitial area, which results in edema. Osmotic and oncotic pressures involve the concentrations of solutes and can contribute to edema in other situations. Concentration pressure is not a nursing term. DIF: Remember
REF: 1018
Canadian Fundamentals of Nursing 6th Edition Potter Test Bank OBJ: Describe common fluid, electrolyte, and acid–base imbalances, and identify related risk factors. TOP: Assessment MSC: NCLEX: Physiological Integrity 4. The nurse understands that administering a hypertonic solution to a patient will shift water
from which space to which space? a. Intracellular to extracellular. b. Extracellular to intracellular. c. Intravascular to intracellular. d. Intravascular to interstitial. ANS: A
The concentration of a hypertonic solution is greater than those of normal body fluids, and so water will shift out of cells because of the osmotic pull of the extra particles. Movement of water into cells occurs when hypotonic fluids are administered. Distribution of fluid between intravascular and interstitial spaces occurs by filtration, the net sum of hydrostatic and osmotic pressures. DIF: Understand REF: 1052 OBJ: Describe the distribution, composition, movement, and regulation of body fluids. TOP: Assessment MSC: NCLEX: Physiological Integrity 5. Which patient is most at risk for sensible water loss? a. A 7-year-old child with asthma. b. A 24-year-old adult with constipation. c. A 56-year-old patient with gastroenteritis. d. An 80-year-old patient with pneumonia. ANS: D
NURSINGTB.COM
Sensible water loss is the loss of fluids from the skin through visible perspiration, such as that occurring with a resolving fever related to pneumonia. Asthma would be insensible water loss through respiration. Gastroenteritis causes diarrhea with its large volume loss. Constipation does not affect fluid loss. DIF: Apply REF: 1018 OBJ: Describe common fluid, electrolyte, and acid–base imbalances, and identify related risk factors. TOP: Assessment MSC: NCLEX: Physiological Integrity 6. What is the most abundant cation in the blood? a. Sodium. b. Potassium. c. Chloride. d. Magnesium. ANS: A
Sodium is the most abundant cation in the blood. Potassium is the predominant intracellular cation. Chloride is an anion (negatively charged) rather than a cation (positively charged). Magnesium is found predominantly inside cells and in bone. DIF: Remember REF: 1016 OBJ: Describe the processes involved in regulating acid–base balance. TOP: Assessment MSC: NCLEX: Physiological Integrity
Canadian Fundamentals of Nursing 6th Edition Potter Test Bank 7. The nurse receives the patient’s most recent blood work results. Which laboratory value is of
greatest concern? a. Sodium level of 145 mmol/L (145 mEq/L). b. Calcium level of 4.5 mmol/L (17.5 mg/dL). c. Potassium level of 3.5 mmol/L (3.5 mEq/L). d. Chloride level of 100 mmol/L (100 mEq/L). ANS: B
Normal calcium levels range from 2.25 to 2.75 mmol/L (8.5 mg/dL to 10.5 mg/dL); therefore, a value of 4.5 mmol/L (17.5 mg/dL) is abnormally high and of concern. The rest of the laboratory values are within their normal ranges: normal sodium levels range from 135 to 145 mmol/L (135 to 145 mEq/L); normal potassium levels range from 3.5 to 5.0 mmol/L (3.5 to 5.0 mEq/L); and normal chloride levels range from 97 to 107 mmol/L (98 to 106 mEq/L). DIF: Remember REF: 1020 OBJ: Choose appropriate clinical assessments for fluid, electrolyte, and acid–base balances. TOP: Assessment MSC: NCLEX: Physiological Integrity 8. The nurse would expect a patient with increased levels of serum calcium to also have which
abnormal values? a. Increased potassium levels. b. Decreased phosphate levels. c. Decreased sodium levels. d. Increased magnesium levels. ANS: B
Serum calcium and phosphate have an inverse relationship. When one level is elevated, the other is decreased, except in N som thCend URe SpaItien NGtsTwBi. OM-stage renal disease. Increased level of serum calcium would not necessarily cause changes in levels of potassium, sodium, or magnesium. DIF: Remember REF: 1020 OBJ: Choose appropriate clinical assessments for fluid, electrolyte, and acid–base balances. TOP: Assessment MSC: NCLEX: Physiological Integrity 9. The nurse knows that an imbalance of which ion causes acid–base impairment? a. Hydrogen. b. Calcium. c. Magnesium. d. Sodium. ANS: A
The concentration of hydrogen ions determines pH. Low pH designates an acidic environment. High pH designates an alkaline environment. Calcium, magnesium, and sodium are ions, but their imbalances are not direct acid–base impairments. DIF: Remember REF: 1020 OBJ: Choose appropriate clinical assessments for fluid, electrolyte, and acid–base balances. TOP: Assessment MSC: NCLEX: Physiological Integrity 10. The nurse would expect a patient with respiratory acidosis to have an excessive amount of
which of the following?
Canadian Fundamentals of Nursing 6th Edition Potter Test Bank a. b. c. d.
Carbon dioxide. Bicarbonate. Oxygen. Phosphate.
ANS: A
Respiratory acidosis occurs when the lungs are not able to excrete enough carbon dioxide. Carbon dioxide and water create carbonic acid. A buildup of carbonic acid causes the extracellular fluid to become more acidic, which lowers the pH. Bicarbonate level is normal with uncompensated respiratory acidosis or elevated with compensated respiratory acidosis. Excessive oxygen and phosphate levels are not characteristic of respiratory acidosis. DIF: Remember REF: 1021 OBJ: Describe the processes involved in acid–base balance. MSC: NCLEX: Physiological Integrity
TOP: Assessment
11. A 2-year-old child was brought into the emergency department after ingesting several
morphine tablets from a bottle in his mother’s purse. The nurse knows that the child is at greatest risk for which acid–base imbalance? a. Respiratory acidosis. b. Respiratory alkalosis. c. Metabolic acidosis. d. Metabolic alkalosis. ANS: A
Morphine overdose can cause respiratory depression and hypoventilation. Hypoventilation results in retention of CO2 and respiratory acidosis. Respiratory alkalosis would result from hyperventilation, which causN es aRdecI U S reas NGeTinBC.OC2OleMvels. Metabolic acid–base imbalance would be a result of kidney dysfunction, vomiting, diarrhea, or other conditions that affect metabolic acids. DIF: Analyze REF: 1025 OBJ: Describe common fluid, electrolyte, and acid–base imbalances, and identify related risk factors. TOP: Assessment MSC: NCLEX: Physiological Integrity 12. A patient was admitted for a bowel obstruction and has had a nasogastric tube set to low
intermittent suction for the past 3 days. The patient’s respiratory rate has decreased to 12 breaths per minute. The nurse would expect the patient to have which of the following arterial blood gas values? a. pH of 7.78, PaCO2 of 40 mm Hg, HCO3– level of 30 mmol/L (30 mEq/L). b. pH of 7.52, PaCO2 of 48 mm Hg, HCO3– level of 28 mmol/L (28 mEq/L). c. pH of 7.35, PaCO2 of 35 mm Hg, HCO3– level of 26 mmol/L (26 mEq/L). d. pH of 7.25, PaCO2 of 47 mm Hg, HCO3– level of 29 mmol/L (29 mEq/L). ANS: B
Canadian Fundamentals of Nursing 6th Edition Potter Test Bank In compensated metabolic alkalosis, the pH and HCO3– (metabolic) values are alkaline, and CO2 level is slightly acidic (compensatory respiratory acidosis). In this case, the pH of 7.52 is alkaline (normal, 7.35 to 7.45), partial pressure of arterial carbon dioxide (PaCO2) is acidic (normal, 35 to 45 mm Hg), and HCO3– level is elevated (normal, 22 to 26 mmol/L [22 to 26 mEq/L]). When the pH is 7.78, PaCO2 is 40 mm Hg, and HCO3– is 30 mmol/L (30 mEq/L), the condition is uncompensated metabolic alkalosis. A pH of 7.35, a PaCO2 of 35 mm Hg, and a HCO3– level of 26 mmol/L (26 mEq/L) are within normal limits. When the pH is 7.25, PaCO2 is 47 mm Hg, and HCO3– is 29 mmol/L (29 mEq/L), the condition is compensated respiratory acidosis. DIF: Analyze REF: 1025 (Table 40-5) OBJ: Interpret basic fluid, electrolyte, and acid–base laboratory values. TOP: Assessment MSC: NCLEX: Physiological Integrity 13. The nurse would not expect full compensation to occur for which acid–base imbalance? a. Respiratory acidosis. b. Respiratory alkalosis. c. Metabolic acidosis. d. Metabolic alkalosis. ANS: B
Usually the cause of respiratory alkalosis is a temporary event (e.g., an asthma or anxiety attack). The kidneys take about 24 hours to compensate for an event, so it is unlikely that much if any compensation for respiratory alkalosis is observable. Respiratory acidosis usually results from longer term conditions such as chronic lung disease, narcotic overdose, or another event that causes respiratory depression. The kidneys do not respond for about the first 24 hours of the event. For both metabolic imbalances, the respiratory system is quick to attempt Ny R I G culty B.C M to compensate; however, it maU haS ve dNiffiT suO staining that compensation. DIF: Understand REF: 1025 OBJ: Describe the processes involved in regulating acid–base balance. TOP: Assessment MSC: NCLEX: Physiological Integrity 14. A nurse is caring for a patient whose electrocardiogram displays changes characteristic of
hypokalemia. Which assessment finding would the nurse expect? a. Thready peripheral pulses. b. Abdominal distension. c. Dry mucous membranes. d. Flushed skin. ANS: B
Signs and symptoms of hypokalemia are muscle weakness and fatigue, abdominal distension, decreased bowel sounds, and cardiac dysrhythmias. Thready peripheral pulses indicate hypovolemia. Dry mucous membranes and flushed skin are indicative of dehydration and hypernatremia. DIF: Apply REF: 1023 (Table 40-4) OBJ: Choose appropriate clinical assessments for fluid, electrolyte, and acid–base balances. TOP: Assessment MSC: NCLEX: Physiological Integrity 15. In which patient would the nurse expect to see a positive Chvostek sign?
Canadian Fundamentals of Nursing 6th Edition Potter Test Bank a. b. c. d.
A 7-year-old child admitted for severe burns. A 24-year-old adult admitted for chronic alcohol abuse. A 50-year-old patient admitted for an acute exacerbation of hyperparathyroidism. A 75-year-old patient admitted for a broken hip related to osteoporosis.
ANS: B
A positive Chvostek sign is representative of hypocalcemia or hypomagnesemia. Hypomagnesemia is common with alcohol abuse. Hypocalcemia can be brought on by alcohol abuse and pancreatitis (which also can be affected by alcohol consumption). Patients with burns frequently experience extracellular fluid volume (ECV) deficit. Hyperparathyroidism causes hypercalcemia. Immobility is associated with hypercalcemia. DIF: Apply REF: 1023 (Table 40-4) OBJ: Choose appropriate clinical assessments for fluid, electrolyte, and acid–base balances. TOP: Assessment MSC: NCLEX: Physiological Integrity 16. Which organ system is responsible for compensation of respiratory acidosis? a. Respiratory. b. Renal. c. Gastrointestinal. d. Endocrine. ANS: B
The kidneys are responsible for respiratory acidosis compensation. A problem with the respiratory system causes respiratory acidosis, and so another organ system (renal) needs to compensate. Problems with the gastrointestinal and endocrine systems can cause acid–base imbalances, but these systems cannot compensate for an existing imbalance.
NURSINGTB.COM
DIF: Understand REF: 1025 OBJ: Describe the processes involved in regulating acid–base balance. TOP: Assessment MSC: NCLEX: Physiological Integrity 17. Which laboratory value should the nurse examine when evaluating a patient with
uncompensated respiratory alkalosis? a. Partial pressure of arterial oxygen (PaO2). b. Anion gap. c. PaCO2. d. HCO3–. ANS: C
Uncompensated respiratory imbalances are revealed by the PaCO2 levels. PaO2 indicates oxygen status. High anion gap is indicative of metabolic acidosis. HCO3– levels are studied to evaluate compensation for respiratory imbalances or uncompensated metabolic imbalances. DIF: Remember REF: 1026 (Table 40-6) OBJ: Choose appropriate clinical assessments for fluid, electrolyte, and acid–base balances. TOP: Assessment MSC: NCLEX: Physiological Integrity 18. The nurse is caring for a patient who has diabetes and is in renal failure. Which laboratory
findings would the nurse expect? a. pH of 7.3, PaCO2 of 36 mm Hg, HCO3– level of 19 mEq/L (19 mmol/L). b. pH of 7.5, PaCO2 of 35 mm Hg, HCO3– level of 35 mEq/L (35 mmol/L).
Canadian Fundamentals of Nursing 6th Edition Potter Test Bank c. pH of 7.3, PaCO2 of 47 mm Hg, HCO3– level of 23 mEq/L (23 mmol/L). d. pH of 7.35, PaCO2 of 40 mm Hg, HCO3– level of 25 mEq/L (25 mmol/L). ANS: A
Patients in renal failure develop metabolic acidosis. The laboratory values that reflect this are pH of 7.3, PaCO2 of 36 mm Hg, and HCO3– level of 19 mmol/L (19 mEq/L). Laboratory findings of pH of 7.5, PaCO2 of 35 mm Hg, and HCO3– level of 35 mmol/L (35 mEq/L) reflect metabolic alkalosis. Laboratory findings of pH of 7.3, PaCO2 of 47 mm Hg, and HCO3– level of 23 mmol/L (23 mEq/L) reflect respiratory acidosis. Laboratory findings of pH of 7.35, PaCO2 of 40 mm Hg, and HCO3– level of 25 mmol/L (25 mEq/L) are within the normal ranges. DIF: Analyze REF: 1031 OBJ: Choose appropriate clinical assessments for fluid, electrolyte, and acid–base balances. TOP: Assessment MSC: NCLEX: Physiological Integrity 19. The nurse is assessing a patient and finds crackles in the lung bases and neck vein distension.
The nurse gives the patient a diuretic. What electrolyte imbalance is the nurse most concerned about? a. Potassium imbalance. b. Sodium imbalance. c. Calcium imbalance. d. Phosphate imbalance. ANS: A
Use of a diuretic can cause excess excretion of potassium, unless it is a potassium-sparing diuretic. The other electrolytes are not excreted in the same way with diuretics.
NURSINGTB.COM
DIF: Understand REF: 1023 OBJ: Describe common fluid, electrolyte, and acid–base imbalances, and identify related risk factors. TOP: Assessment MSC: NCLEX: Physiological Integrity 20. A patient receiving chemotherapy has gained 2 kg (5 pounds) in 2 days. Which assessment
question by the nurse is most appropriate? a. “Are you having difficulty sleeping at night?” b. “How many calories a day do you consume?” c. “Do you have dry mouth or feel thirsty?” d. “How many times a day do you urinate?” ANS: D
A rapid gain in weight usually indicates ECV excess if the person began with normal ECV. The patient’s description of urination habits will indicate whether the body is trying to excrete the excess fluid or whether renal dysfunction is contributing to ECV excess. Difficulty sleeping at night can occur if the body builds up excessive fluid in the lungs; however, it could also mean that the patient is getting up frequently to urinate, so the question is not specific enough. Caloric intake does not account for rapid weight changes. Dry mouth and thirst accompany ECV deficit, which would be associated with rapid weight loss. DIF: Apply REF: 1032 (Table 40-8) OBJ: Choose appropriate clinical assessments for fluid, electrolyte, and acid–base balances. TOP: Implementation MSC: NCLEX: Physiological Integrity
Canadian Fundamentals of Nursing 6th Edition Potter Test Bank 21. Which fluid order should the nurse question for a patient with a traumatic brain injury? a. 0.45% sodium chloride. b. 0.9% sodium chloride. c. Lactated Ringer’s solution. d. Dextrose 5% in 0.9% sodium chloride. ANS: A
A fluid of 0.45% sodium chloride is a hypotonic solution, and hypotonic solutions cause cells to swell, which can cause increased intracranial pressure. This can be life-threatening for a patient with a traumatic brain injury. The other solutions are physiologically isotonic sodium-containing solutions that expand ECV but do not cause cell swelling. In the fluid container, dextrose 5% in 0.9% sodium chloride is hypertonic, but the dextrose enters cells rapidly, leaving isotonic 0.9% sodium chloride. DIF: Analyze REF: 1039 (Table 40-9) OBJ: Discuss the purpose of, and procedure for, initiating, maintaining, and discontinuing intravenous therapy and peripheral vascular access devices. TOP: Implementation MSC: NCLEX: Physiological Integrity 22. The physician asks the nurse to monitor the fluid volume statuses of a patient with heart
failure and a patient at risk for clinical dehydration. What is the most effective nursing intervention for monitoring both of these patients? a. Weigh the patients every morning before breakfast. b. Ask the patients to record their intake and output. c. Measure the patients’ blood pressure every 4 hours. d. Assess the patients for edema in extremities. ANS: A
NURSINGTB.COM
An effective measure of fluid retention or loss is daily weights; each kilogram (2.2 pounds) gained or lost is equivalent to 1 litre of fluid gained or lost. This measurement should be performed at the same time every day with the same scale and the same amount of clothing. Although intake and output records are important assessment measures, some patients are not able to keep their own records themselves. Blood pressure can decrease with ECV deficit but does not necessarily increase with recent ECV excess (heart failure patient). Edema occurs with ECV excess but not with clinical dehydration. DIF: Apply REF: 1042 OBJ: Identify and discuss nursing interventions for patients with fluid, electrolyte, and acid–base imbalances. TOP: Implementation MSC: NCLEX: Physiological Integrity 23. A nurse is caring for a patient with cancer who presents with anorexia, blood pressure of
100/60, elevated white blood cell count, and oral candidiasis. The nurse knows that the purpose of starting total parenteral nutrition (TPN) is what? a. Replace fluid, electrolytes, and nutrients in the patient. b. Stimulate the patient’s appetite to eat. c. Provide medication to raise the patient’s blood pressure. d. Deliver antibiotics to fight off infection. ANS: A
Canadian Fundamentals of Nursing 6th Edition Potter Test Bank TPN is an intravenous (IV) solution composed of nutrients and electrolytes to replace the ones the patient is not eating. TPN does not stimulate the appetite, and it does not contain blood pressure medication or antibiotics. DIF: Apply REF: 1038 OBJ: Discuss the purpose of, and procedure for, initiating, maintaining, and discontinuing intravenous therapy and peripheral vascular access devices. TOP: Implementation MSC: NCLEX: Physiological Integrity 24. A patient presents to the emergency department with the complaint of vomiting and diarrhea
for the past 48 hours. The nurse anticipates which fluid therapy initially? a. 0.9% sodium chloride. b. Dextrose 10% in water. c. Dextrose 5% in water. d. 0.45% sodium chloride. ANS: A
Patients with prolonged vomiting and diarrhea become hypovolemic. The best solution to replace ECV is 0.9% sodium chloride, which is an isotonic solution. Dextrose 10% in water, dextrose 5% in water, and 0.45% sodium chloride act as hypotonic solutions in the body. The first consideration is replacing ECV to oxygenate tissues. DIF: Apply REF: 1039 (Table 40-9) OBJ: Discuss the purpose of, and procedure for, initiating, maintaining, and discontinuing intravenous therapy and peripheral vascular access devices. TOP: Implementation MSC: NCLEX: Physiological Integrity 25. A patient with a lower respirN ator y in fect ionTB ha.s C a pH UR SI NG OM of 7.25, a PaCO2 of 55 mm Hg, and a
HCO3– level of 20 mmol/L (20 mEq/L). The physician has been notified. Which is the priority nursing intervention for this patient? a. Check the colour of the patient’s urine output. b. Place the patient in Trendelenburg position. c. Encourage the patient to increase respirations. d. Place the patient in high Fowler’s position. ANS: C
The patient has respiratory acidosis from CO2 retention. Increasing rate and depth of respiration enables the patient to blow off excess carbon dioxide, and this will begin to correct the imbalance. Checking the urine colour is not a necessary assessment. Trendelenburg position would probably make it more difficult for the patient to breathe and should be avoided. High Fowler’s position may be more comfortable for the patient, but it is not necessary. DIF: Apply REF: 1025 OBJ: Identify and discuss nursing interventions for patients with fluid, electrolyte, and acid–base imbalances. TOP: Implementation MSC: NCLEX: Physiological Integrity 26. The nurse knows that IV fluid therapy has been effective for a patient with hypernatremia
when what happens? a. Serum sodium concentration returns to normal. b. Systolic and diastolic blood pressure decrease.
Canadian Fundamentals of Nursing 6th Edition Potter Test Bank c. Large amounts of emesis and diarrhea decrease. d. Urine output increases to 150 mL/hr. ANS: A
Hypernatremia is a condition of elevated serum sodium concentration. Blood pressure is not an accurate indicator of hypernatremia. Emesis and diarrhea will not stop because of IV therapy. Urine output is influenced by many factors, including ECV. A large dilute urine output can cause further hypernatremia. DIF: Apply REF: 1023 OBJ: Identify and discuss nursing interventions for patients with fluid, electrolyte, and acid–base imbalances. TOP: Implementation MSC: NCLEX: Physiological Integrity 27. The nurse would select the dorsal venous plexus of the foot as an IV site for which patient? a. A 2-year-old child. b. A 22-year-old adult. c. A 50-year-old patient. d. An 80-year-old patient. ANS: A
Use of the foot as an IV site is common in children but is avoided in adults because of the risk for thrombophlebitis. DIF: Understand REF: 1053 (Figure 40-17) OBJ: Discuss the purpose of, and procedure for, initiating, maintaining, and discontinuing intravenous therapy and peripheral vascular access devices. TOP: Implementation MSC: NCLEX: Physiological Integrity G B.C M
N R I U S N T
O
28. Which assessment finding should cause a nurse to question administering a sodium-containing
isotonic IV fluid? a. Blood pressure of 102/58. b. Dry mucous membranes. c. Poor skin turgor. d. Pitting edema. ANS: D
Pitting edema indicates that the patient may be retaining excess extracellular fluid, and the nurse should question the order for solution meant to rehydrate the patient. All other options are consistent with ECV deficit, in which case the patient would benefit from a sodium-containing isotonic solution that expands ECV. DIF: Understand REF: 1052 OBJ: Discuss the purpose of, and procedure for, initiating, maintaining, and discontinuing intravenous therapy and peripheral vascular access devices. TOP: Implementation MSC: NCLEX: Physiological Integrity 29. A patient is to receive 1500 mL of 0.9% sodium chloride intravenously at a rate of 125 mL/hr.
The nurse is using microdrip gravity drip tubing. What is the minute flow rate (drops per minute)? a. 12 gtt/min. b. 24 gtt/min.
Canadian Fundamentals of Nursing 6th Edition Potter Test Bank c. 125 gtt/min. d. 150 gtt/min. ANS: C
Microdrip tubing delivers 60 gtt/mL. To obtain a rate of 125 mL/hr with microdrip tubing, the calculation is 125 mL/hr 60 gtt/mL 60 min = 125 gtt/min. DIF: Apply REF: 1054 OBJ: Discuss the purpose of, and procedure for, initiating, maintaining, and discontinuing intravenous therapy and peripheral vascular access devices. TOP: Implementation MSC: NCLEX: Physiological Integrity 30. A nurse begins infusing a 250-mL bag of IV fluid at 1845 hours on Monday and programs the
pump to infuse at 20 mL/hr. At what time should the infusion be completed? a. 0645 Tuesday. b. 0675 Tuesday. c. 0715 Tuesday. d. 0735 Tuesday. ANS: C
This answer would be calculated as follows: • 250 mL divided by 20 mL/hr = 12.5 hr • 0.5 hr 60 min = 30 min • 1845 + 12 hr 30 min = 3115, which would be 0715 on Tuesday, the following day. DIF: Apply REF: 1055 OBJ: Discuss the purpose of, and procedure for, initiating, maintaining, and discontinuing intravenous therapy and peripheral vascular devices. TOP: Implementation NUR INGaccess B.C MO S T MSC: NCLEX: Physiological Integrity 31. A nurse is caring for a patient with diabetes who has a bowel obstruction and has orders to
ensure that the volume of intake matches the output. In the past 4 hours, the patient received dextrose 5% with 0.9% sodium chloride through a 22-gauge catheter infusing at 150 mL/hr and has eaten 200 mL of ice chips. The patient also has a nasogastric suction tube set to low continuous suction, which yielded an output of 300 mL. The patient has voided 400 mL of urine. After reporting these values to the physician, what orders does the nurse anticipate? a. Add a potassium supplement to replace loss from output. b. Decrease the rate of IV fluids to 100 mL/hr. c. Discontinue the nasogastric suctioning. d. Administer a diuretic to prevent fluid volume excess. ANS: A
The total fluid intake and output equals 700 mL, which meets the provider goals. The nurse should record half the volume of ice chips when calculating intake. Patients with nasogastric suctioning are at risk for potassium deficit, so the nurse would anticipate a potassium supplement to correct this condition. The other measures would be unnecessary because the net fluid volume is equal. DIF: Analyze REF: 1018 OBJ: Identify and discuss nursing interventions for patients with fluid, electrolyte, and acid–base imbalances. TOP: Implementation MSC: NCLEX: Physiological Integrity
Canadian Fundamentals of Nursing 6th Edition Potter Test Bank
32. A nurse is caring for a patient who is in hypertensive crisis. When the nurse is flushing the
patient’s peripheral IV line, the patient complains of pain. Upon assessment, the nurse notices a red streak near the IV site that is warm to the touch. What is the nurse’s initial action? a. Notify the physician. b. Administer pain medication. c. Discontinue the IV. d. Start a new IV line. ANS: D
The IV site is phlebitic. The nurse should start a new IV infusion before discontinuing the old one because it is important to always have an IV access site in a patient who is in hypertensive crisis. Then the physician can be notified. Pain medication may or may not need to be administered. DIF: Understand REF: 1040 OBJ: Discuss complications associated with intravenous therapy. TOP: Implementation MSC: NCLEX: Physiological Integrity 33. A patient was admitted for hypovolemia and has IV fluid running at 250 mL/hr. The patient
complains of burning at the IV insertion site. Upon assessment, the nurse does not find redness, swelling, heat, or coolness. The nurse suspects which of the following? a. IV infiltration has occurred. b. The IV catheter has caused phlebitis. c. Fluid is infusing too quickly. d. Patient is allergic to the fluid. ANS: C
NURSINGTB.COM
The infusion may be flowing faster than the vein can handle, which causes discomfort. The nurse should slow down the infusion. With infiltration, the skin around the IV insertion site becomes blanched, cool, and edematous. Pain, warmth, erythema, and a palpable venous cord are all symptoms of phlebitis. Allergic response to the fluid could involve a combination of itching, flushing, hypotension, and dyspnea, depending on the severity. DIF: Apply REF: 1041 OBJ: Discuss complications associated with intravenous therapy. TOP: Assessment MSC: NCLEX: Physiological Integrity 34. The nurse is caring for a patient with sepsis. The plan of care for the patient is to administer
antibiotics three times a day for 4 weeks. What device will be used to administer these antibiotics? a. A continuous infusion. b. A peripheral catheter with a heparin lock. c. A peripherally inserted central catheter (PICC) line. d. An implanted port catheter. ANS: C
Canadian Fundamentals of Nursing 6th Edition Potter Test Bank A PICC line is a type of central venous device that can be introduced into a peripheral vein for administration of IV antibiotics for an extended period, over the course of several weeks. A continuous infusion would not be appropriate if the patient were to receive antibiotics only three times daily. A peripheral catheter would not be necessary or have a heparin lock. An implanted port catheter is intended for long-term use of venous access over months, or even years. DIF: Apply REF: 1051 OBJ: Discuss the purpose of, and procedure for, initiating, maintaining, and discontinuing intravenous therapy and peripheral vascular access devices. TOP: Implementation MSC: NCLEX: Physiological Integrity 35. A patient had an acute intravascular hemolytic reaction during a blood transfusion. After
discontinuing the blood transfusion, what is the nurse’s next action? a. Run normal saline through the existing tubing. b. Start normal saline at to-keep-open (TKO) rate with new tubing. c. Discontinue the IV catheter. d. Return the blood to the blood bank. ANS: B
The nurse should first attach new tubing and begin running in normal saline at a rate to keep the vein open, in case any sorts of medications need to be delivered through that IV site. The existing tubing should not be used because that would infuse the blood in the tubing into the patient. It is necessary to preserve the IV catheter in place for IV access to treat the patient. After the patient has been assessed and stabilized, the blood can be returned to the blood bank. DIF: Understand REF: 1058 OBJ: Discuss the purpose of, aNnU dR prS ocI edN urG eT foB r,. inC itiO atM ing a blood transfusion and interventions to manage transfusion reaction. TOP: Implementation MSC: NCLEX: Physiological Integrity 36. A nurse is assessing a patient who is receiving a blood transfusion and finds that the patient is
anxiously fidgeting in bed. The patient is afebrile and dyspneic. The nurse auscultates crackles in both lung bases and sees jugular vein distension. The nurse recognizes that the patient is experiencing which transfusion complication? a. Anaphylactic shock. b. Septicemia. c. Fluid volume overload. d. Hemolytic reaction. ANS: C
The signs and symptoms are consistent with fluid volume overload. Anaphylactic shock would have manifested with urticaria, dyspnea, and hypotension. Septicemia would include a fever. A hemolytic reaction would consist of flank pain, chills, and fever. DIF: Understand REF: 1064| 1065 OBJ: Discuss the purpose of, and procedure for, initiating a blood transfusion and interventions to manage transfusion reaction. TOP: Implementation MSC: NCLEX: Physiological Integrity 37. The nurse selects appropriate tubing for a blood transfusion by ensuring that the tubing has
which of the following?
Canadian Fundamentals of Nursing 6th Edition Potter Test Bank a. Two-way valves to allow the patient’s blood to mix and warm the incoming
transfused blood. b. An injection port to mix additional electrolytes into the blood. c. An air vent to let bubbles in the blood escape. d. A filter to ensure that clots do not enter the patient. ANS: D
All blood transfusions must have a filter to prevent microemboli from being administered to the patient. The patient’s blood should not be aspirated to mix with the infusion blood. The blood should not have air bubbles to vent; if a bag of blood does have bubbles, the nurse should promptly return the blood to the blood bank. The only substance compatible with blood is normal saline; no additives should be mixed with the infusing blood. DIF: Understand REF: 1065 OBJ: Discuss the purpose of, and procedure for, initiating a blood transfusion and interventions to manage transfusion reaction. TOP: Implementation MSC: NCLEX: Physiological Integrity 38. The nurse is caring for a patient with hyperkalemia. Which body system would be most
important for the nurse plan to monitor closely? a. Gastrointestinal. b. Neurological. c. Cardiac. d. Respiratory. ANS: C
Potassium balance is necessary for cardiac function. Hyperkalemia places the patient at risk for potentially serious dysrhyNthm g of URias. SIMo NGnito TBri.nC OMgastrointestinal, neurological, and respiratory systems would be indicated for other electrolyte imbalances. DIF: Apply REF: 1024 OBJ: Identify and discuss nursing interventions for patients with fluid, electrolyte, and acid–base imbalances. TOP: Assessment MSC: NCLEX: Physiological Integrity 39. A patient has the following laboratory values: sodium level, 145 mmol/L (145 mEq/L);
potassium level, 4.5 mmol/L (4.5 mEq/L); calcium level, 1.05 mmol/L (4.5 mg/dL). What would the nurse expect to find in the assessment? a. Lightheadedness when the patient stands up. b. Weak quadriceps muscles. c. Tingling of the extremities and tetany. d. Decreased deep tendon reflexes. ANS: C
This patient has hypocalcemia; the normal calcium range is 2.25 to 2.75 mmol/L (8.4 to 10.5 mg/dL). Sodium and potassium values are within their normal ranges (sodium, 135 to 145 mmol/L [135 to 145 mEq/L]; potassium, 3.5 to 5.0 mmol/L [3.5 to 5.0 mEq/L]). Hypocalcemia causes muscle tetany, Trousseau’s sign, and tingling of the extremities. Lightheadedness when a patient stands up is a manifestation of ECV deficit or sometimes hypokalemia. Weak quadriceps muscles are associated with potassium imbalances. Decreased deep tendon reflexes are related to hypercalcemia or hypermagnesemia. DIF: Apply
REF: 1026| 1027
Canadian Fundamentals of Nursing 6th Edition Potter Test Bank OBJ: Choose appropriate clinical assessments for fluid, electrolyte, and acid–base balances. TOP: Assessment MSC: NCLEX: Physiological Integrity 40. A patient informs the nurse that he has the type of diabetes that does not have to do with
“blood sugar.” The nurse advises the patient to make which dietary change? a. Drink plenty of fluids throughout the day to stay hydrated. b. Avoid food high in acid to avoid metabolic acidosis. c. Reduce the quantity of carbohydrates ingested to lower blood glucose levels. d. Include a serving of dairy in each meal to elevate calcium levels. ANS: A
The patient is indicating that he has diabetes insipidus, which places him at risk for dehydration and hypernatremia. To prevent dehydration, the patient should drink plenty of fluids to replace the extra water excreted in the urine. Foods high in acid should be avoided by a patient with gastroesophageal reflux disease (GERD). A reduction in carbohydrates applies to patients with type 2 diabetes mellitus. Calcium-rich dairy products would be recommended for a patient with hypocalcemia. DIF: Analyze REF: 1030 OBJ: Identify and discuss nursing interventions for patients with fluid, electrolyte, and acid–base imbalances. TOP: Planning MSC: NCLEX: Physiological Integrity MULTIPLE RESPONSE 1. When selecting a site to insert an IV catheter on an adult, the nurse should follow which
procedure? (Select all that apply.) a. Start proximally and move distally on the arm. N R I G B.C M O b. Choose a vein with minimaU l cuS rvaN tureT . c. Choose the patient’s dominant arm. d. Check for contraindications to using the extremity. e. Select a vein that is rigid. f. Avoid areas of flexion. ANS: B, D, F
The nurse should start distally and move proximally, choosing the nondominant arm if possible. The vein should be relatively straight to avoid catheter occlusion. Contraindications to starting an IV catheter are conditions such as mastectomy, arteriovenous fistula, and central line in the extremity. The nurse should feel for the best location; a good vein should feel spongy. A rigid vein should be avoided because it might have had previous trauma or damage. DIF: Apply REF: 1038-1040 OBJ: Discuss the purpose of, and procedure for, initiating, maintaining, and discontinuing intravenous therapy and peripheral vascular access devices. TOP: Implementation MSC: NCLEX: Physiological Integrity 2. Which of the following assessments would indicate infiltration of a patient’s IV line? (Select
all that apply.) a. Edema of the extremity near the insertion site. b. Skin discoloured or bruised in appearance. c. Pain and warmth at the insertion site. d. Skin cool to the touch.
Canadian Fundamentals of Nursing 6th Edition Potter Test Bank e. Reddish streak proximal to the insertion site. f. Numbness or loss of sensation. g. Palpable venous cord. ANS: A, B, D, F
Infiltration causes skin near the IV insertion site to become edematous. The affected skin is cool to the touch and may be bruised or discoloured, and the patient may experience some numbness. Pain, warmth, erythema, a reddish streak, and a palpable venous cord are all symptoms of phlebitis. DIF: Remember REF: 1049| 1051 (Table 40-11) OBJ: Discuss complications associated with intravenous therapy. TOP: Assessment MSC: NCLEX: Physiological Integrity 3. How does the nurse discontinue peripheral IV access? (Select all that apply.) a. Use scissors to remove the IV site dressing and tape. b. Keep the catheter parallel to the skin while removing it. c. Apply firm pressure with sterile gauze during removal. d. Stop the infusion before removing the IV catheter. e. Wear sterile gloves and a mask. f. Apply pressure to the site for 2 to 3 minutes after removal. ANS: B, D, F
The nurse should stop the infusion before removing the IV catheter, so that the fluid does not drip on the patient’s skin; keep the catheter parallel to the skin while removing it, to reduce trauma to the vein; and apply pressure to the site for 2 to 3 minutes after removal, to decrease bleeding from the site. Scissors should not be used because they may accidentally cut the catheter or tubing or may injuNreUtR heSpIatie NGnt.TBD.urCing OMremoval of the IV catheter, light pressure, not firm pressure, is indicated to prevent trauma. Clean gloves are used for discontinuing peripheral IV access because gloved hands will handle the external dressing, tubing, and tape, which are not sterile. DIF: Remember REF: 1061 OBJ: Discuss the purpose of, and procedure for, initiating, maintaining, and discontinuing intravenous therapy and peripheral vascular access devices. TOP: Implementation MSC: NCLEX: Physiological Integrity
Canadian Fundamentals of Nursing 6th Edition Potter Test Bank
Chapter 41: Sleep Potter et al: Canadian Fundamentals of Nursing, 6th Edition MULTIPLE CHOICE 1. The physiological processes of sleep are complex. Which of the following statements is the
most appropriate one regarding this process? a. Circadian rhythms occur in a cycle longer than 24 hours. b. Non–rapid eye movement (NREM) refers to the cycle that most people experience when in a high-stimulus environment. c. The reticular activating system is partly responsible for the level of consciousness of a person. d. The bulbar synchronizing region causes the rapid eye movement (REM) sleep in most normal adults. ANS: C
Circadian rhythms are cyclical rhythms that are part of everyday life. The most familiar rhythm is the 24-hour day–night cycle known as the diurnal or circadian rhythm. It is not longer than 24 hours. The NREM stage is the part of the sleep cycle that most people experience in a low-stimulus environment. The ascending reticular activating system located in the upper brainstem is believed to contain special cells that maintain alertness and wakefulness. The bulbar synchronizing region is the area of the brain where serotonin is released to produce sleep. It is not responsible for REM sleep. DIF: Apply REF: 1073 OBJ: Explain the effect of the 24-hour sleep–wake cycle on biological functions. N R : Ph IN G logical B.CIOntegrity M TOP: Assessment MSC: NCLUEXS ysioT 2. The nurse is alert to patients who may be predisposed to obstructive sleep apnea. This
includes which of the following individuals? a. Patients with heart disease. b. Patients with respiratory infections. c. Patients with nasal polyps. d. Patients who are obese. ANS: C
Individuals with heart disease may have sleep disorders. Women with heart disease have the majority of sleep disturbances, but not apnea in particular. Respiratory infections do not predispose a patient to obstructive sleep apnea. Structural abnormalities such as nasal polyps, certain jaw configurations, enlarged tonsils, or a deviated septum predispose a patient to obstructive sleep apnea. Obesity, smoking and alcohol use, and a history of obstructive sleep apnea greatly increase the risk of developing sleep apnea, but they do not necessarily predispose the person to sleep apnea. DIF: Understand REF: 1077 OBJ: Discuss the characteristics of common sleep disorders. MSC: NCLEX: Physiological Integrity
TOP: Assessment
3. When a patient is deprived of sleep, the nurse might assess which of the following symptoms? a. Elevated blood pressure.
Canadian Fundamentals of Nursing 6th Edition Potter Test Bank b. Confusion and irritability. c. Inappropriateness and rapid respirations. d. Decreased temperature and talkativeness. ANS: B
Psychological symptoms of sleep deprivation include confusion and irritability. A decrease in capacity for reasoning and judgement could lead to inappropriate behaviour. Elevated blood pressure, rapid respirations, and decreased temperature are not signs of sleep deprivation. Many patients with sleep deprivation are withdrawn, not talkative. DIF: Understand REF: 1079 (Box 41-7) OBJ: Discuss the characteristics of common sleep disorders. MSC: NCLEX: Physiological Integrity
TOP: Assessment
4. The parents of a newborn wonder when she should start to sleep through the night. The
nurse’s response should be that in infants, a nighttime pattern of sleep usually develops by which age? a. 1 month. b. 2 months. c. 3 months. d. 6 months. ANS: D
Infants do not usually develop a nighttime pattern of sleep by 1 month, 2 months, or 3 months of age. Infants usually develop a nighttime pattern of sleep before 6 months of age. DIF: Understand REF: 1079 OBJ: Compare the sleep requiN remR ents of dG ifferent age groups. UandSI N TB.C OM MSC: NCLEX: Health Promotion Maintenance
TOP: Planning
5. The mother of a preschooler tells the nurse that the child has started crying and resisting going
to sleep at the scheduled bedtime. Which of the following actions should the nurse advise the parent to take? a. Offer the child a bedtime snack. b. Eliminate one of the naps during the day. c. Allow the child to sleep longer in the mornings. d. Maintain consistency in the same bedtime ritual. ANS: D
It is most important that the parent maintain a consistent bedtime routine. If a bedtime snack is already part of that routine, then this is allowable. If it is not, then the child might use having a snack only as a measure of procrastination. After 3 years of age, the child may give up daytime naps. A bedtime routine used consistently will be more effective in helping the child resist going to sleep during the day. Allowing the child to sleep longer in the mornings will not aid in establishing a consistent sleep pattern. The same regular bedtime and wake-up schedule should be maintained. DIF: Understand REF: 1088 OBJ: Compare the sleep requirements of different age groups. MSC: NCLEX: Health Promotion and Maintenance
TOP: Implementation
Canadian Fundamentals of Nursing 6th Edition Potter Test Bank 6. An 11-year-old child in middle school is currently experiencing fatigue related to sleep
deprivation during classes. Which of the following responses by the school nurse is the most appropriate one when counselling the child’s parents regarding this assessment? a. “What are the child’s usual sleep patterns?” b. “Establish bedtimes for the child, and withhold his allowance whenever he does not adhere to those bedtimes.” c. “We need to explore other health-related problems, as sleep problems are probably not the cause of the child’s fatigue.” d. “The bulbar synchronizing region of the child’s central nervous system (CNS) is causing these insomniac problems.” ANS: A
A school-age child will be tired the following day if allowed to stay up later than usual. The nurse should first ask a question to assess the child’s usual sleep patterns. The response “Establish bedtimes for the child, and withhold his allowance whenever he does not adhere to those bedtimes” is not appropriate because the nurse is assuming that the boy is not adhering to a bedtime. A sleep problem is often the cause of fatigue, not a health-related problem. The response about the bulbar synchronizing region is incorrect because the nurse is assuming the child is experiencing insomnia. DIF: Apply REF: 1088 OBJ: Compare the sleep requirements of different age groups. MSC: NCLEX: Health Promotion and Maintenance
TOP: Assessment
7. In describing the sleep patterns of older persons, the nurse recognizes that which of the
following statements is true? a. Older persons are more difficult to arouse. N R I G B.C M b. Older persons require moreUsleS ep tN hanT middleO -aged adults. c. Older persons take less time to fall asleep. d. Older persons have a decline in slow-wave or deep sleep. ANS: D
As people age, they do not become more difficult to arouse. Older people do not require more sleep than do middle-aged adults. Older people awaken more often during the night, and it may take more time for an older person to fall asleep. As people age, slow-wave (deep) sleep declines progressively. DIF: Understand REF: 1080 OBJ: Compare the sleep requirements of different age groups. MSC: NCLEX: Physiological Integrity
TOP: Assessment
8. A patient who is currently taking a diuretic should be informed by the nurse that he or she
may experience which of the following effects? a. Nocturia. b. Nightmares. c. Increased daytime sleepiness. d. Reduced REM sleep. ANS: A
Canadian Fundamentals of Nursing 6th Edition Potter Test Bank The nurse should inform the patient who is currently taking a diuretic that he or she might experience nighttime awakening because of nocturia. Diuretics do not cause nightmares, increase daytime sleepiness, or reduce REM sleep. DIF: Apply REF: 1080 OBJ: Identify factors that normally promote sleep and factors that normally disrupt sleep. TOP: Assessment MSC: NCLEX: Physiological Integrity 9. As a result of recent studies regarding infant safety during sleep, which of the following
instructions does the nurse give the parents? a. Provide a stuffed toy for comfort. b. Apply a loose-fitting plastic mattress cover. c. Place the infant on his or her back. d. Use small pillows in the crib. ANS: C
To reduce the chance of suffocation, pillows, stuffed toys, or the ends of loose blankets should not be placed in cribs. Loose-fitting plastic mattress covers can cause suffocation. Infants are usually placed on their backs to prevent suffocation or on their sides to prevent aspiration of stomach contents. DIF: Apply REF: 1090 OBJ: Identify nursing interventions designed to promote normal sleep cycles for individuals of all ages. TOP: Implementation MSC: NCLEX: Health Promotion and Maintenance 10. A 74-year-old patient reports having sleeping difficulties. To have a better idea of the
patient’s problem, how should the nurse respond? a. “What do you do just befN orU eR goSinIgNtoGbTeB d?.”COM b. “Let’s make sure that your bedroom is completely darkened at night.” c. “Why don’t you try napping more during the daytime?” d. “You should always eat something just before bedtime.” ANS: A
To assess the patient’s sleeping problem, the nurse should inquire about predisposing factors, such as by asking, “What do you do just before going to bed?” Assessment is aimed at understanding the characteristics of any sleep problem and the patient’s usual sleep habits so that ways for promoting sleep can be incorporated into nursing care. Older persons may prefer to sleep in softly lit rooms. Napping more during the daytime is often not the best solution. The nurse should first assess the patient’s sleeping problem. The patient does not always have to eat something before going to bed. DIF: Apply TOP: Assessment
REF: 1080 OBJ: Conduct a sleep history for a patient. MSC: NCLEX: Health Promotion and Maintenance
11. Which of the following information provided by the patient’s bed partner is most associated
with sleep apnea? a. Restlessness. b. Talking during sleep. c. Sleepwalking. d. Excessive snoring. ANS: D
Canadian Fundamentals of Nursing 6th Edition Potter Test Bank Restlessness is not the symptom most associated with sleep apnea. Sleep talking is associated with sleep–wake transition disorders, not sleep apnea. Sleepwalking is associated with parasomnias (specifically arousal disorders and sleep–wake transition disorders). Partners of patients with sleep apnea often complain that the patient’s snoring disturbs their sleep. DIF: Apply REF: 1084 (Box 41-10) OBJ: Discuss the characteristics of common sleep disorders. MSC: NCLEX: Physiological Integrity
TOP: Assessment
12. Which of the following instructions does the nurse give the patient in teaching methods to
promote positive sleep habits at home? a. Use the bedroom only for sleep or sexual activity. b. Eat a large meal 1 to 2 hours before bedtime. c. Exercise vigorously before bedtime. d. Stay in bed if sleep does not come after 30 minutes. ANS: A
The nurse should explain that if possible, the bedroom should not be used for intensive studying, snacking, TV watching, or other nonsleep activity, other than sexual activity. The nurse should instruct the patient to avoid heavy meals for 3 hours before bedtime; a light snack may help. The nurse should instruct the patient to try to exercise daily, preferably in the morning or afternoon, and to avoid vigorous exercise in the evening within 2 hours of bedtime. The patient who does not fall asleep within 30 minutes of going to bed should be advised by the nurse to rise and do some quiet activity until he or she feels sleepy enough to return to bed. DIF: Apply REF: 1084 OBJ: Identify nursing intervenN tiU onR sS apI prN opGriT atB e f. orCpO atM ients with sleep alterations. TOP: Implementation MSC: NCLEX: Health Promotion and Maintenance 13. The nurse is discussing sleep habits with the patient in the sleep assessment clinic. Of the
following activities performed before sleeping, the nurse informs the patient that which one of the following can potentially interfere with the patient’s sleep? a. Listening to classical music. b. Finishing office work. c. Reading novels. d. Drinking warm milk. ANS: B
Noise should be kept to a minimum. Soft music may be used to mask noise if necessary. At home, a patient should not try to finish office work or resolve family problems before bedtime. Reading a light novel, watching an enjoyable TV program, or listening to music can help a person to relax. Relaxation exercises can be useful at bedtime. A dairy product snack, such as warm milk, contains L-tryptophan and may be helpful in promoting sleep. DIF: Understand REF: 1084 OBJ: Identify nursing interventions appropriate for patients with sleep alterations. TOP: Implementation MSC: NCLEX: Health Promotion and Maintenance
Canadian Fundamentals of Nursing 6th Edition Potter Test Bank 14. The patient needs pharmacological treatment to assist with his sleep patterns. The nurse
anticipates that treatment with an anxiety-reducing, relaxation-promoting medication will include the use of which one of the following? a. Barbiturates. b. Amphetamines. c. Benzodiazepines. d. Tricyclic antidepressants. ANS: C
Withdrawal from CNS depressants such as barbiturates can cause insomnia and must be managed carefully. Patients can also develop tolerance of and dependence on barbiturates. CNS stimulants, such as amphetamines, should be used sparingly and only under medical management. Amphetamine sulphate may be used to treat narcolepsy; prolonged use may lead to drug dependence. The benzodiazepines cause relaxation, reduce anxiety, and have hypnotic effects by facilitating the action of neurons in the CNS that suppress responsiveness to stimulation, thereby decreasing levels of arousal. Tricyclic antidepressants can cause insomnia when discontinued and should be managed carefully. They are used primarily to treat depression. DIF: Understand REF: 1092 OBJ: Identify nursing interventions appropriate for patients with sleep alterations. TOP: Planning MSC: NCLEX: Physiological Integrity 15. The nurse is completing an assessment of the patient’s sleep patterns. Which of the following
is a specific question that the nurse should ask to determine the potential presence of sleep apnea? a. “How easily do you fall asleep?” G B.C M N R I b. “Do you have vivid, lifelikU O e drS eamN s?”T c. “Do you ever experience loss of muscle control or falling?” d. “Do you snore loudly or experience headaches?” ANS: D
The question “How easily do you fall asleep?” is directed at assessing the potential presence of insomnia. The questions “Do you have vivid, lifelike dreams?” and “Do you ever experience loss of muscle control or falling?” are directed at determining the potential presence of narcolepsy. To assess for sleep apnea, the nurse may ask, “Do you snore loudly?” and “Do you experience headaches after awakening?” A positive response may indicate that the patient experiences sleep apnea. DIF: Understand REF: 1083 (Box 41-9) OBJ: Conduct a sleep history for a patient. MSC: NCLEX: Physiological Integrity
TOP: Assessment
16. Which of the following is an effective brief method for assessing sleep quality? a. Sleep history. b. Sleep monitor. c. Visual analogue scale. d. Sleep questionnaire diary. ANS: C
Canadian Fundamentals of Nursing 6th Edition Potter Test Bank One effective brief method for assessing sleep quality is the use of a visual analogue scale. Documenting sleep history and keeping a sleep questionnaire diary are not brief, and they help assess more than sleep quality. A sleep monitor is not necessarily used to measure sleep quality, and it too is not brief. DIF: Apply TOP: Assessment
REF: 1083 OBJ: Describe ways to evaluate sleep therapies. MSC: NCLEX: Physiological Integrity
17. Which sleep stage begins the initial period of deep sleep? a. Stage 1. b. Stage 2. c. Stage 3. d. Stage 4. ANS: C
Stage 3 is the stage that begins the initial period of deep sleep. DIF: Understand TOP: Assessment
REF: 1074 OBJ: Describe the stages of a normal sleep cycle. MSC: NCLEX: Physiological Integrity
18. Which stage of sleep accounts for the greatest proportion of total sleep time in adults? a. Stage 1. b. Stage 2. c. Stage 3. d. Stage 4. ANS: B
Stage 2 accounts for the greaN testR proI portG ionB (45 to 5M5%) of total sleep time in adults. Stages 1, S Nproportion T .COof 3, and 4 do not account for theU greatest total sleep time in adults. DIF: Remember TOP: Assessment
REF: 1074 OBJ: Describe the stages of a normal sleep cycle. MSC: NCLEX: Physiological Integrity
19. Which of the following is a determinant of dreams? a. Gender. b. Age. c. Height. d. Weight. ANS: B
Age is a determinant of dreams, along with personality, physiological conditions, psychological conditions, pharmacological therapies, and situational factors. Height, weight, and gender are not identified determinants of dreams. DIF: Understand TOP: Assessment
REF: 1075 OBJ: Compare the characteristics of rest and sleep. MSC: NCLEX: Physiological Integrity
20. Which of the following sleep disorders is a parasomnia disorder of arousal? a. Sleep apnea. b. Narcolepsy. c. Sleepwalking. d. Early-morning awakening.
Canadian Fundamentals of Nursing 6th Edition Potter Test Bank
ANS: C
Sleepwalking is an example of a parasomnia disorder of arousal. Sleep apnea is a sleep-related breathing disorder. Narcolepsy is an example of excessive sleepiness. Early-morning awakening is an example of an insomnia sleep disorder. DIF: Understand REF: 1077 (Box 41-4) OBJ: Discuss the characteristics of common sleep disorders. MSC: NCLEX: Physiological Integrity
NURSINGTB.COM
TOP: Assessment
Canadian Fundamentals of Nursing 6th Edition Potter Test Bank
Chapter 42: Nutrition Potter et al: Canadian Fundamentals of Nursing, 6th Edition MULTIPLE CHOICE 1. The energy needed to maintain life-sustaining activities for a specific period of time at rest is
known as which of the following? a. Basal metabolic rate. b. Resting energy expenditure. c. Nutrients. d. Nutrient density. ANS: A
The basal metabolic rate (BMR) is the energy needed to maintain life-sustaining activities for a specific period of time at rest. The resting energy expenditure (REE), or resting metabolic rate, is the amount of energy that an individual needs to consume over a 24-hour period for the body to maintain all of its internal working activities while at rest. Nutrients are the elements necessary for body processes and function. Nutrient density is the proportion of essential nutrients to the number of kilocalories. Foods with high nutrient density provide a large number of nutrients in relation to kilocalories. DIF: Remember REF: 1098 OBJ: Explain the importance of maintaining a balance between energy intake and expenditure. TOP: Assessment MSC: NCLEX: Physiological Integrity 2. In general, when energy requirements are completely met by kilocalorie intake in food, what
NURSINGTB.COM happens? a. Weight increases. b. Weight decreases. c. Weight does not change. d. Kilocalories are not a factor in energy requirements. ANS: C
In general, when energy requirements are completely met by kilocalorie intake in food, weight does not change. When kilocalories ingested exceed a person’s energy demands, the individual gains weight. If kilocalories ingested fail to meet a person’s energy requirement, the individual loses weight; therefore, kilocalories are a factor in energy requirements. DIF: Understand REF: 1099 OBJ: Explain the importance of maintaining a balance between energy intake and expenditure. TOP: Assessment MSC: NCLEX: Physiological Integrity 3. In determining kcal expenditure, the nurse knows that carbohydrates and proteins provide 4
kcal of energy per gram ingested. The nurse also knows that fats provide how many kilocalories per gram? a. 3 kcal. b. 4 kcal. c. 6 kcal. d. 9 kcal.
Canadian Fundamentals of Nursing 6th Edition Potter Test Bank ANS: D
Fats (lipids) are the most calorie-dense nutrient, providing 9 kcal per gram. DIF: Remember REF: 1100 OBJ: List the end products of carbohydrate, protein, and fat metabolism. TOP: Assessment MSC: NCLEX: Physiological Integrity 4. Some proteins are manufactured in the body, and others are not. Those that must be obtained
through diet are known as which of the following? a. Amino acids. b. Dispensable amino acids. c. Triglycerides. d. Essential amino acids. ANS: D
The simplest form of protein is the amino acid. The body does not synthesize essential amino acids, so these must be provided in the diet. The body synthesizes nonessential (dispensable) amino acids. Triglycerides are made up of three fatty acids attached to a glycerol. DIF: Remember REF: 1099 OBJ: List the end products of carbohydrate, protein, and fat metabolism. TOP: Assessment MSC: NCLEX: Physiological Integrity 5. Knowing that protein is required for tissue growth, maintenance, and repair, the nurse must
understand that for optimal tissue healing to occur, the patient must be in which state? a. Negative nitrogen balance. b. Positive nitrogen balance. c. Total dependence on protN ein kca l pro ucCtion URforSI NG TBd. OM. d. Neutral nitrogen balance. ANS: B
When intake of nitrogen is greater than output, the body is in positive nitrogen balance. Positive nitrogen balance is required for growth, normal pregnancy, maintenance of lean muscle mass and vital organs, and wound healing. In negative nitrogen balance, the body loses more nitrogen than it gains. In neutral nitrogen balance, nitrogen gain equals nitrogen loss; this state is not optimal for tissue healing. Protein can provide energy (4 kcal/g), but because of its essential role in growth, maintenance, and repair, the diet should provide adequate kilocalories from nonprotein sources. Protein is spared as an energy source when carbohydrate in the diet is sufficient to meet the energy needs of the body. DIF: Understand REF: 1100 OBJ: Explain the significance of saturated, unsaturated, polyunsaturated and trans fats. TOP: Assessment MSC: NCLEX: Physiological Integrity 6. In providing diet education for a patient on a low-fat diet, it is important for the nurse to
understand which of the following? a. Saturated fats are found mostly in vegetable sources. b. Saturated fats are found mostly in animal sources. c. Unsaturated fats are found mostly in animal sources. d. Linoleic acid is a saturated fatty acid. ANS: B
Canadian Fundamentals of Nursing 6th Edition Potter Test Bank Most animal fats have high proportions of saturated fatty acids, whereas vegetable fats have higher amounts of unsaturated and polyunsaturated fatty acids. Linoleic acid, an unsaturated fatty acid, is the only essential fatty acid in humans. DIF: Understand REF: 1100 OBJ: Explain the significance of saturated, unsaturated, polyunsaturated and trans fats. TOP: Assessment MSC: NCLEX: Physiological Integrity 7. Fats are composed of triglycerides and fatty acids. Which of the following statements is true? a. Triglycerides are made up of three fatty acids. b. Triglycerides can be saturated. c. Triglycerides can be monounsaturated. d. Triglycerides can be polyunsaturated. ANS: A
Triglycerides circulate in the blood and are made up of three fatty acids attached to a glycerol. Fatty acids (not triglycerides) can be saturated or unsaturated (monounsaturated or polyunsaturated). DIF: Remember REF: 1100 OBJ: Explain the significance of saturated, unsaturated, polyunsaturated and trans fats. TOP: Assessment MSC: NCLEX: Physiological Integrity 8. The patient has been diagnosed with cardiovascular disease and placed on a low-fat diet. The
patient asks the nurse, “How much fat should I have? I guess the less fat, the better.” What does the nurse need to explain? a. Fats have no significance in health and the incidence of disease. b. All fats come from externNaU l sR ouSrI ceN sG anT dB so.cCaO nM be easily controlled. c. Deficiencies occur when fat intake falls below 20% of daily total fat intake. d. Vegetable fats are the major source of saturated fats and should be avoided. ANS: C
The acceptable macronutrient distribution range, the range associated with reduced risk of chronic illness while providing essential intakes of total fat, is 20% to 35% for adults. Various types of fatty acids have significance for health and for the incidence of disease and are referred to in dietary guidelines. Linoleic acid and arachidonic acid, which are important for metabolic processes, are manufactured by the body when linoleic acid is available. Most animal fats have high proportions of saturated fatty acids, whereas vegetable fats have higher amounts of unsaturated and polyunsaturated fatty acids. DIF: Understand REF: 1100 OBJ: Explain the significance of saturated, unsaturated, polyunsaturated and trans fats. TOP: Assessment MSC: NCLEX: Physiological Integrity 9. When inserting a nasoenteric tube, the nurse will rotate the tube how much? a. 45 degrees. b. 90 degrees. c. 180 degrees. d. 360 degrees. ANS: C
The tube is to be rotated 180 degrees while being inserted.
Canadian Fundamentals of Nursing 6th Edition Potter Test Bank
DIF: Apply REF: 1138 OBJ: Describe the procedure for initiating and maintaining tube feedings, and avoiding related complications. TOP: Implementation MSC: NCLEX: Physiological Integrity 10. The patient states that she is a lacto-ovo-vegetarian. Which of the following types of food
does a lacto-ovo-vegetarian consume? a. Fish and poultry b. Only plant foods c. Milk and fish d. Eggs and milk ANS: D
Ovo-lacto-vegetarians avoid meat, fish, and poultry but consume eggs and milk. Vegans consume only plant food. DIF: Understand REF: 1112 OBJ: Identify the potential nutritional deficits associated with vegetarian diets, with special consideration to vegan and ovo-lactate diets. TOP: Assessment MSC: NCLEX: Physiological Integrity 11. The nurse is providing nutrition teaching to a Korean patient. In doing so, the nurse must
understand that the focus of the teaching should be on which of the following? a. Changing the patient’s diet to a more conventional Canadian diet. b. Discouraging the patient’s ethnic food choices. c. Food preferences of the patient, including racial and ethnic choices. d. Comparing the patient’s eNthnR ic pI refeG rencBe. sw CithMCanadian dietary choices.
U S N T
O
ANS: C
The nurse needs to make sure to consider the food preferences of patients from different racial and ethnic groups, vegetarians, and others when planning diets. Initiation of a balanced diet is more important than conversion to what may be considered a Canadian diet. Ethnic food choices may be just as nutritious as “Canadian” choices. Foods should be chosen for their nutritive value and should not be compared with the “Canadian” diet. DIF: Understand REF: 1111 OBJ: Describe Eating Well with Canada's Food Guide and its value in planning nutritious meals. TOP: Assessment MSC: NCLEX: Health Promotion and Maintenance 12. When teaching a patient about current dietary guidelines for the general population, the nurse
explains referenced daily intakes and daily reference values, otherwise known as daily values. In providing this information, the nurse understands what about daily values? a. They have replaced recommended daily allowances (RDAs). b. They have provided a more understandable calculation of RDAs for the public. c. They are based on percentages of a diet consisting of 1200 kcal/day. d. They are not usually easy to find and computer experience is required. ANS: B
Canadian Fundamentals of Nursing 6th Edition Potter Test Bank Daily values did not replace RDAs but provide a separate, more understandable calculation for the public. Daily values are based on percentages of a diet consisting of 2000 kcal/day; these values constitute the daily values used on food labels, which are easy for anyone to find. Computer experience is not required. DIF: Understand REF: 1106 OBJ: Specify recommended dietary intake for age and sex groups in Canada to ensure that patients meet the varied essential vitamins, minerals, and nutritional requirements throughout their growth and development. TOP: Assessment MSC: NCLEX: Health Promotion and Maintenance 13. The nurse is teaching the patient about dietary guidelines. In discussing the four components
of dietary reference intakes (DRIs), it is important to understand which of the following? a. The estimated average requirement (EAR) is appropriate for 100% of the population. b. The RDA meets the needs of the individual. c. Adequate intake determines the nutrient requirements of the RDA. d. The tolerable upper intake level is not a recommended level of intake. ANS: D
The tolerable upper intake level is the highest level that probably poses no risk of adverse health events. It is not a recommended level of intake. The EAR is the recommended amount of a nutrient that appears sufficient to maintain a specific body function for 50% of the population according to age and gender. The RDA reflects the average needs of 98% of the population, not the exact needs of the individual. Adequate intake is the suggested intake for individuals that is based on observed or experimentally determined estimates of nutrient intakes and is used when evidence is insufficient for setting of the RDA. DIF: Understand REF: 11N 03URSINGTB.COM OBJ: Specify recommended dietary intake for age and sex groups in Canada to ensure that patients meet the varied essential vitamins, minerals, and nutritional requirements throughout their growth and development. TOP: Assessment MSC: NCLEX: Health Promotion and Maintenance 14. In teaching mothers-to-be about infant nutrition, what does the nurse instruct patients to do? a. Give cow’s milk during the first year of life. b. Supplement breast milk with corn syrup. c. Add honey to infant formulas for increased energy. d. Remember that breast milk or formula is sufficient for the first 6 months. ANS: D
Breast milk or formula provides sufficient nutrition for the first 6 months of life. Infants should not have regular cow’s milk during the first year of life. Cow’s milk causes gastrointestinal bleeding, is too concentrated for the infant’s kidneys to manage, increases the risk for developing milk product allergies, and is a poor source of iron and vitamins C and E. Honey and corn syrup are potential sources of botulism toxin and should not be used in the infant diet. DIF: Remember REF: 1107 OBJ: Specify recommended dietary intake for age and sex groups in Canada to ensure that patients meet the varied essential vitamins, minerals, and nutritional requirements throughout their growth and development. TOP: Assessment MSC: NCLEX: Health Promotion and Maintenance
Canadian Fundamentals of Nursing 6th Edition Potter Test Bank 15. To counter obesity in adolescents, increasing physical activity is often more important than
curbing intake. Sports and regular, moderate to intense exercise necessitate dietary modifications to meet increased energy needs for adolescents. The nurse understands that these modifications include which of the following? a. Decreasing carbohydrates to 25% to 30% of total intake. b. Decreasing protein intake to 0.75 g/kg/day. c. Drinking water before and after exercise. d. Providing vitamin and mineral supplements. ANS: C
Adequate hydration is very important for all athletes. They need to drink water before and after exercise to prevent dehydration, especially in hot, humid environments. Carbohydrates, both simple and complex, are the main source of energy, providing 55% to 60% of total daily kilocalories. Protein needs increase to 1.0 to 1.5 g/kg/day. Vitamin and mineral supplements are not required, but intake of iron-rich foods is necessary to prevent anemia. DIF: Understand REF: 1109 OBJ: Specify recommended dietary intake for age and sex groups in Canada to ensure that patients meet the varied essential vitamins, minerals, and nutritional requirements throughout their growth and development. TOP: Assessment MSC: NCLEX: Physiological Integrity 16. In providing prenatal care to a patient, what does the nurse teach the expectant mother? a. Protein intake needs to decrease to preserve kidney function. b. Calcium intake is especially important in the first trimester. c. Folic acid is needed to help prevent birth defects and anemia. d. The mother should take in as many extra vitamins and minerals as possible. ANS: C
NURSINGTB.COM
Folic acid intake is particularly important for DNA synthesis and growth of red blood cells. Inadequate intake may lead to fetal neural tube defects (such as anencephaly) or maternal megaloblastic anemia. Protein intake throughout pregnancy needs to increase to 60 g daily. Calcium intake is especially critical in the third trimester, when fetal bones are mineralized. Prenatal care usually includes vitamin and mineral supplementation to ensure daily intakes; however, pregnant women should not take additional supplements beyond prescribed amounts. DIF: Understand REF: 1109 OBJ: Specify recommended dietary intake for age and sex groups in Canada to ensure that patients meet the varied essential vitamins, minerals, and nutritional requirements throughout their growth and development. TOP: Assessment MSC: NCLEX: Physiological Integrity 17. The patient is an 80-year-old man who is visiting the clinic today for his routine physical
examination. The patient’s skin turgor is fair, but he has been complaining of fatigue and weakness. The skin is warm and dry, pulse rate is 126 beats per minute, and urinary sodium level is slightly elevated. After assessment, the nurse should recommend what to the patient? a. Decrease his intake of milk and dairy products to decrease the risk of osteoporosis. b. Drink more grapefruit juice to enhance vitamin C intake and medication absorption. c. Drink more water to prevent further dehydration. d. Eat more meat because meat is the only source of usable protein. ANS: C
Canadian Fundamentals of Nursing 6th Edition Potter Test Bank Thirst sensation diminishes with age, which leads to inadequate fluid intake or dehydration. Symptoms of dehydration in older persons include confusion, weakness, hot dry skin, furrowed tongue, and high urinary sodium. Milk continues to be an important food for older women and men, who need adequate calcium to protect against osteoporosis. After age 70, osteoporosis affects men and women equally. Older persons should be cautioned to avoid grapefruit and grapefruit juice because these will decrease absorption of many drugs. Some older persons avoid meats because of cost or because they are difficult to chew. Cream soups and meat-based vegetable soups are nutrient-dense sources of protein. DIF: Analyze REF: 1111 OBJ: Specify recommended dietary intake for age and sex groups in Canada to ensure that patients meet the varied essential vitamins, minerals, and nutritional requirements throughout their growth and development. TOP: Assessment MSC: NCLEX: Physiological Integrity 18. The nurse is assessing a patient for nutritional status. In doing so, what must the nurse do? a. Choose a single objective tool that fits the patient’s condition. b. Combine multiple objective measures with subjective measures. c. Forego the assessment in the presence of chronic disease. d. Use the Mini Nutritional Assessment for pediatric patients. ANS: B
Using a single objective measure is ineffective in predicting risk of nutritional problems. Combine multiple objective measures with subjective measures related to nutrition to adequately screen for nutritional problems. Chronic disease and increased metabolic requirements are risk factors for the development of nutritional problems; these patients may be in critical need of this assessment. The Mini Nutritional Assessment was developed to use for screening older persons in home care programs, nursing homes, and hospitals.
N UR SI NG TB.C OM
DIF: Apply REF: 1113-1116 OBJ: Discuss the major methods of nutritional assessment. MSC: NCLEX: Physiological Integrity
TOP: Assessment
19. The patient has a calculated body mass index (BMI) of 34. How would the patient be
classified? a. As unclassifiable. b. As being of normal weight. c. As being overweight. d. As being obese. ANS: D
BMI greater than 30 is defined as obesity. BMI between 25 and 30 is classified as overweight. BMI less than 25 is considered normal or underweight. All BMIs can be classified; they are calculated as weight in kilograms divided by their height in metres squared. DIF: Analyze REF: 1109 (Table 42-2) OBJ: Discuss the major methods of nutritional assessment. MSC: NCLEX: Physiological Integrity
TOP: Assessment
20. A patient is seen in the outpatient clinic for follow-up of a nutritional deficiency. In planning
for the patient’s dietary intake, the nurse includes a complete protein. Which one of the following is an example of a complete protein?
Canadian Fundamentals of Nursing 6th Edition Potter Test Bank a. b. c. d.
Cheese. Oats. Legumes. Vegetables.
ANS: A
A complete protein contains all essential amino acids in sufficient quantity to support growth and maintain nitrogen balance. Cheese, chicken, fish, and soybeans are examples of complete proteins. Incomplete proteins lack a sufficient quantity of one or more of the nine essential amino acids and include cereals, legumes, and vegetables. DIF: Remember REF: 1100 OBJ: List the end products of carbohydrate, protein, and fat metabolism. TOP: Assessment MSC: NCLEX: Physiological Integrity 21. The patient is an older woman and has been given a nursing diagnosis of Imbalanced
nutrition: less than body requirements. What role should the nurse play in her treatment regimen? a. Encourage weight gain as rapidly as possible. b. Encourage large meals three times a day. c. Decrease fluid intake to prevent feeling full. d. Encourage fibre intake. ANS: D
Increasing fibre intake deters constipation and enhances appetite. Weight gain should be slow and progressive. The patient should be encouraged to eat frequent small meals to increase dietary intake and to help offset anorexia. Older persons need eight 8-ounce glasses of fluid per day from beverage and foNoU dR soS urIces . TB.COM NG DIF: Remember REF: 1121-1123 OBJ: Identify three major nutritional problems, the patients who are at risk, and related nutrition therapy. TOP: Assessment MSC: NCLEX: Physiological Integrity 22. In determining the nutritional status of a patient and developing a plan of care, it is important
to evaluate the patient according to what data? a. Published standards. b. Nursing professional standards. c. Absence of family input. d. Patient input only. ANS: A
Published standards are based on scientific findings and are important references to use when a plan of care is developed. Nursing standards cannot be used alone. Other health care providers must be consulted to adopt interventions that reflect the patient’s needs. Family should be involved in evaluation and design of interventions. Although patient input is important, synthesis of patient information from multiple sources is necessary for devising an individualized approach to care that is relevant to the patient’s needs. DIF: Apply REF: 1098| 1112 OBJ: Formulate a plan of care to help meet the specific nutritional needs of infants, toddlers, preschoolers, school-aged children, adolescents, adults, and older persons. TOP: Implementation MSC: NCLEX: Safe and Effective Care Environment
Canadian Fundamentals of Nursing 6th Edition Potter Test Bank
23. In creating a plan of care to meet the nutritional needs of the patient, the nurse needs to
explore the patient’s feelings about weight and food. Why must the nurse do this? a. To determine which category of plan to use. b. To set realistic goals for the patient. c. To mutually plan goals with patient and team. d. To prevent the need for a dietitian consult. ANS: C
Mutually planned goals negotiated by patient, registered dietitian, and nurse ensure success. Individualized planning cannot be overemphasized. Preplanned and categorical care plans are not effective unless they are individualized to meet patient needs. It is important to explore patients’ feelings about weight and food to help them set realistic and achievable goals. The nurse does not set goals for the patient. The plan should reflect the combined effort of patient, nurse, and dietitian, and so a dietitian consult is required. DIF: Apply REF: 1119 OBJ: Formulate a plan of care to help meet the specific nutritional needs of infants, toddlers, preschoolers, school-aged children, adolescents, adults, and older persons. TOP: Implementation MSC: NCLEX: Safe and Effective Care Environment 24. The patient is admitted with facial trauma, including a broken nose, and has a history of
esophageal reflux and of aspiration pneumonia. In view of this information, which of the following tubes is appropriate for this patient? a. Nasogastric tube. b. Percutaneous endoscopic gastrostomy (PEG) tube. c. Nasointestinal tube. NURSINGTB.COM d. Jejunostomy tube. ANS: D
Patients with gastroparesis or esophageal reflux or with a history of aspiration pneumonia may require placement of tubes past the stomach and into the intestine. The nasogastric tube and the PEG tube are placed in the stomach, and placement could lead to aspiration. The nasointestinal tube and the nasogastric tube may be contraindicated by the facial trauma and the broken nose. The jejunostomy tube is the only tube in the list that is placed beyond the stomach and is not contraindicated by facial trauma. DIF: Understand REF: 1135 OBJ: Describe the procedure for initiating and maintaining tube feedings, and avoiding related complications. TOP: Assessment MSC: NCLEX: Physiological Integrity 25. The nurse is preparing to insert a nasogastric tube in a patient who is semiconscious. To
determine the length of the tube needed to be inserted, the nurse measures what length? a. From the tip of the nose to the xiphoid process of the sternum. b. From the earlobe to the xiphoid process of the sternum. c. From the tip of the nose to the earlobe. d. From the tip of the nose to the earlobe to the xiphoid process. ANS: D
The nurse measures the distance from the tip of the nose to the earlobe to the xiphoid process of the sternum. This approximates the distance from the nose to the stomach in 98% of patients. For duodenal or jejunal placement, an additional 20 to 30 centimetres is required.
Canadian Fundamentals of Nursing 6th Edition Potter Test Bank
DIF: Apply REF: 1138 (Skill 42-3) OBJ: Describe the procedure for initiating and maintaining tube feedings, and avoiding related complications. TOP: Implementation MSC: NCLEX: Physiological Integrity 26. Before giving the patient an intermittent tube feeding, what should the nurse do? a. Make sure that the tube is secured to the gown with a safety pin. b. Have the tube feeding at room temperature. c. Inject air into the stomach via the tube and auscultate. d. Place the patient in a supine position. ANS: B
Cold formula causes gastric cramping and discomfort because the mouth and the esophagus cannot warm the liquid. Safety pins should not be used; they can become unfastened and may cause harm to the patient. Auscultation is no longer considered a reliable method for verification of tube placement because a tube inadvertently placed in the lungs, pharynx, or esophagus transmits sound similar to that of air entering the stomach. Place the patient in high-Fowler’s position, or elevate the head of the bed at least 30 degrees to help prevent aspiration. DIF: Apply REF: 1142 (Skill 42-4) OBJ: Describe the procedure for initiating and maintaining tube feedings, and avoiding related complications. TOP: Implementation MSC: NCLEX: Physiological Integrity 27. At present, the most reliable method for verification of placement of small-bore feeding tubes
is a. b. c. d.
Auscultation. Aspiration of contents. Radiography. pH testing.
NURSINGTB.COM
ANS: C
At present, the most reliable method for verification of placement of small-bore feeding tubes is x-ray examination. Aspiration of contents and pH testing are not infallible. The nurse would need a more precise indicator to help differentiate the source of tube feeding aspirate. Auscultation is no longer considered a reliable method for verification of tube placement because a tube inadvertently placed in the lungs, pharynx, or esophagus transmits sound similar to that of air entering the stomach. DIF: Understand REF: 1148 OBJ: Describe the procedure for initiating and maintaining tube feedings, and avoiding related complications. TOP: Assessment MSC: NCLEX: Physiological Integrity 28. The nurse is concerned about pulmonary aspiration when providing her patient with tube
feedings. What should the nurse do? a. Verify tube placement before the feeding. b. Lower the head of the bed to a supine position. c. Add blue food colouring to the enteral formula. d. Run the formula over 12 hours to decrease volume.
Canadian Fundamentals of Nursing 6th Edition Potter Test Bank ANS: A
A major cause of pulmonary aspiration is regurgitation of formula. The nurse needs to verify tube placement and elevate the head of the bed 30 to 45 degrees during feedings and for 2 hours afterward. Blue food colouring is no longer added to enteral formula to assist with detection of aspirate. The formula should not hang longer than 4 to 8 hours. Formula becomes a medium for bacterial growth after that length of time. DIF: Apply REF: 1150-1155 OBJ: Describe the procedure for initiating and maintaining tube feedings, and avoiding related complications. TOP: Implementation MSC: NCLEX: Physiological Integrity 29. The patient is to receive multiple medications via the nasogastric tube. The nurse is concerned
that the tube may become clogged. What can the nurse do to prevent this? a. Irrigate the tube with 60 mL of water after all medications are given. b. Check with the pharmacy to find out whether liquid forms of the medications are available. c. Instill nonliquid medications without diluting. d. Mix all medications together to decrease the number of administrations. ANS: B
Crushed medication should be avoided if a liquid formulation is available. Each tube is irrigated with 30 mL of water before and after each medication. Crushed medications, if used, should be diluted. The nurse should read pharmacological information on compatibility of drugs and formula before mixing medications. DIF: Apply REF: 1141 (Skill 42-3) OBJ: Describe the procedure fN orUinRitia ting ning tube feedings, and avoiding related SI NGand TBm.aiCntaiOM complications. TOP: Implementation MSC: NCLEX: Physiological Integrity 30. The patient has just started enteral feedings but is complaining of abdominal cramping. What
should the nurse do? a. Slow the rate of tube feeding. b. Instill cold formula to “numb” the stomach. c. Place the patient in a supine position. d. Change the tube feeding to a high-fat formula. ANS: A
One possible cause of abdominal cramping is a rapid increase in rate or volume. Lowering the rate of delivery may increase tolerance. Another possible cause of abdominal cramping is use of cold formula. The nurse should warm the formula to room temperature. The nurse should maintain the head of the bed at least 30 degrees. High-fat formulas are also a cause of abdominal cramping. DIF: Apply REF: 1152 (Table 42-9) OBJ: Describe the procedure for initiating and maintaining tube feedings, and avoiding related complications. TOP: Implementation MSC: NCLEX: Physiological Integrity 31. The patient has just been started on an enteral feeding and has developed diarrhea after being
on the feeding for 2 hours. Which of the following is the most likely cause of the diarrhea?
Canadian Fundamentals of Nursing 6th Edition Potter Test Bank a. b. c. d.
Clostridium difficile infection. Antibiotic therapy. Formula intolerance. Bacterial contamination.
ANS: C
Hyperosmolar formulas can cause diarrhea. If that is the case, the solution is to lower the rate, dilute the formula, or change to an isotonic formula. Antibiotics destroy normal intestinal flora and disturb the internal ecology, allowing for C. difficile toxin buildup. However, this takes time, and the description does not suggest that this patient was taking antibiotics. Proximity to the start of the enteral feedings is more suspicious. Bacterial contamination of the feeding usually occurs when feedings are left hanging for longer than 8 hours. DIF: Apply REF: 1151 (Table 42-9) OBJ: Describe the procedure for initiating and maintaining tube feedings, and avoiding related complications. TOP: Assessment MSC: NCLEX: Physiological Integrity 32. In providing teaching for a patient with type 1 diabetes mellitus, the nurse gives the patient
which instruction? a. Insulin is the only consideration that must be taken into account. b. Saturated fat should be limited to less than 30% of total calories. c. Cholesterol intake should be greater than 200 mg/day. d. Nonnutritive sweeteners can be used without restriction. ANS: B
The patient with diabetes should limit daily fat to less than 30% of total calories and cholesterol intake to less than 200 mg/day. Patients with type 1 diabetes require both insulin and dietary restrictions for opNtim ontr No.nC nut ritive sweeteners can be eaten as long as URalScI NGol.TB OM the recommended daily intake levels are followed. DIF: Apply REF: 1127 (Box 42-9) OBJ: Identify three major nutritional problems, the patients who are at risk, and related nutrition therapy. TOP: Implementation MSC: NCLEX: Health Promotion and Maintenance 33. The parent of an 8-year-old asks the nurse about any special nutritional needs for children in
this age group. The nurse mentions that children in this age group need to do which of the following? a. Increase their intake of B vitamins. b. Significantly increase iron intake. c. Maintain a sufficient intake of protein and vitamins A and C. d. Increase carbohydrate intake to meet increased energy needs. ANS: C
School-aged children’s diets should be carefully assessed for adequate protein and vitamins A and C. School-aged children frequently fail to eat a proper breakfast, and their intake at school is unsupervised. An increase in B-complex vitamins is needed to support heightened metabolic activity of the adolescent, but not the school-aged child. The pregnant woman needs to increase iron intake significantly, but the school-aged child does not. Increased energy needs are expected in the adolescent period, not in the school-aged group. Therefore, an 8-year-old child does not need to increase carbohydrates to meet increased energy needs.
Canadian Fundamentals of Nursing 6th Edition Potter Test Bank DIF: Analyze REF: 1108 OBJ: Specify recommended dietary intake for age and sex groups in Canada to ensure that patients meet the varied essential vitamins, minerals, and nutritional requirements throughout their growth and development. TOP: Assessment MSC: NCLEX: Physiological Integrity 34. The nurse is providing home care for a patient who has acquired immune deficiency
syndrome (AIDS). In preparing meals for this patient, what should the nurse do? a. Provide small, frequent nutrient-dense meals. b. Encourage intake of fatty foods to increase caloric intake. c. Prepare hot meals because they are more easily tolerated. d. Avoid salty foods and limit liquids to preserve electrolytes. ANS: A
Small, frequent, nutrient-dense meals in which fatty foods and overly sweet foods are limited are easier to tolerate. Patients benefit from eating cold foods and drier or saltier foods with fluid in between. DIF: Apply REF: 1128 OBJ: Identify three major nutritional problems, the patients who are at risk, and related nutrition therapy. TOP: Implementation MSC: NCLEX: Physiological Integrity 35. To provide successful nutritional therapies to patients, what must the nurse understand? a. Patients will have to change diet preferences drastically to be successful. b. The patient will tell the nurse when to change the plan of care. c. Expectations of nurses frequently differ from those of the patient. d. Nurses should never alter the plan of care regardless of outcome. ANS: C
NURSINGTB.COM
Expectations and health care values held by nurses frequently differ from those held by patients. Successful interventions and outcomes depend on recognition of this fact, in addition to nursing knowledge and skill. If ongoing nutritional therapies are not resulting in successful outcomes, patients expect nurses to recognize this fact and alter the plan of care accordingly. Working closely with patients enables the nurse to redefine expectations that are realistically met within the limits of conditions and treatments and to identify their dietary preferences and cultural beliefs. DIF: Understand REF: 1129 OBJ: Discuss diet counselling and patient teaching in relation to patient expectations. TOP: Implementation MSC: NCLEX: Health Promotion and Maintenance 36. In measuring the effectiveness of nutritional interventions, what must the nurse remember? a. To expect results to occur rapidly. b. Not to be concerned with physical measures such as weight. c. To expect to maintain a course of action regardless of changes in condition. d. To evaluate outcomes according to the patient’s expectations and goals. ANS: D
Canadian Fundamentals of Nursing 6th Edition Potter Test Bank The nurse should measure the effectiveness of nutritional interventions by evaluating the patient’s expected outcomes and goals of care. Nutrition therapy does not always produce rapid results. Ongoing comparisons need to be made with baseline measures of weight, serum albumin or prealbumin, and protein and kilocalorie intake. Changes in condition may indicate a need to change the nutritional plan of care. DIF: Apply REF: 1129 OBJ: Discuss diet counselling and patient teaching in relation to patient expectations. TOP: Implementation MSC: NCLEX: Health Promotion and Maintenance 37. What action should the nurse take when expected nutritional outcomes are not being met? a. Revise the nursing measures or expected outcomes. b. Alter the outcomes on the basis of nursing standards. c. Ensure that patient expectations are congruent with the nurse’s expectations. d. Readjust the plan to exclude cultural beliefs. ANS: A
When expected outcomes are not met, the nurse should revise the nursing measures or expected outcomes according to the patient’s needs or preferences, not solely on the basis of nursing standards. Expectations and health care values held by nurses frequently differ from those held by patients. Working closely with patients enables the nurse to redefine expectations that are realistically met within the limits of conditions and treatments and to identify their dietary preferences and cultural beliefs. DIF: Apply REF: 1129 OBJ: Discuss diet counselling and patient teaching in relation to patient expectations. TOP: Implementation MSC: NCLEX: Health Promotion and Maintenance
NURSINGTB.COM
MULTIPLE RESPONSE 1. DRIs are evidence-informed criteria for an acceptable range of amounts of vitamins and
nutrients for each gender and age group. Components of DRIs include which of the following? (Select all that apply.) a. EAR. b. RDA. c. The Food Guide Pyramid. d. Adequate intake. e. The tolerable upper intake level. ANS: A, B, D, E
DRIs are evidence-informed criteria for an acceptable range of amounts of vitamins and nutrients for each gender and age group. DRIs have four components. The EAR is the recommended amount of a nutrient that appears sufficient to maintain a specific body function for 50% of the population based on age and gender. The RDA indicates the average needs of 98% of the population, not the exact needs of the individual. Adequate intake is the suggested intake for individuals that is based on observed or experimentally determined estimates of nutrient intakes and is used when evidence is insufficient to allow the RDA to be set. The tolerable upper intake level is the highest level that probably poses no risk of adverse health events. It is not a recommended level of intake. The Food Guide Pyramid is not a component of the DRIs.
Canadian Fundamentals of Nursing 6th Edition Potter Test Bank DIF: Remember REF: 1103 OBJ: Specify recommended dietary intake for age and sex groups in Canada to ensure that patients meet the varied essential vitamins, minerals, and nutritional requirements throughout their growth and development. TOP: Assessment MSC: NCLEX: Physiological Integrity 2. The patient is asking the nurse about the best way to stay healthy. The nurse explains to the
patient that from a nutritional point of view, the patient should do which of the following? (Select all that apply.) a. Maintain body weight in a healthy range. b. Increase physical activity. c. Increase intake of meat and other high-protein foods. d. Keep total fat intake to 10% or less. e. Choose and prepare foods with little salt. ANS: A, B, E
Canada's Food Guide notes the importance of physical activity in maintaining energy balance and recommends that adults spend 30 to 60 minutes per day carrying out some physical activity and children and youth spend at least 90 minutes per day. The guide provides advice on the use of vitamin and mineral supplementation when the recommended food intake pattern does not ensure adequate amounts and to maintain a healthy body weight. Although not specified in the guide, Canadians are advised to limit salt to “healthy” levels, alcohol to no more than 5% of total energy, and caffeine to no more than 400 mg for the general population. DIF: Apply REF: 1103-1106 OBJ: Describe Eating Well with Canada's Food Guide and its value in planning nutritious meals. TOP: Implementation MSC: NCLEX: Health Promotion and Maintenance 3. To create a new nutritional pN laU nR ofScI arN eG foT r aBp.aC tieOnM t, the nurse needs to do which of the
following? (Select all that apply.) a. Utilize the characteristics of a normal nutritional status. b. Evaluate previous patient responses to nursing interventions. c. Exclude established expected outcomes to evaluate patient responses. d. Design innovative interventions to meet the patient’s needs. e. Follow through with evaluation and counselling. ANS: A, B, D, E
To create a new nutritional plan of care, the nurse must utilize characteristics of a normal nutritional status to gauge effectiveness of the plan. The nurse must be aware of previous patient responses to nursing interventions for altered nutrition to determine the probability of success. The nurse must use established expected outcomes to evaluate the patient’s response to care (e.g., patient’s weight increases by 0.5 kg/week). The nurse must also be creative when designing innovative nursing interventions to meet the patient’s nutritional needs and must demonstrate responsibility by following through with evaluation and counselling to successfully reach goals. DIF: Apply REF: 1119| 1120 OBJ: Discuss diet counselling and patient teaching in relation to patient expectations. TOP: Implementation MSC: NCLEX: Health Promotion and Maintenance
Canadian Fundamentals of Nursing 6th Edition Potter Test Bank
Chapter 43: Urinary Elimination Potter et al: Canadian Fundamentals of Nursing, 6th Edition MULTIPLE CHOICE 1. If obstructed, which component of the urination system would cause peristaltic waves? a. Kidneys. b. Ureters. c. Bladder. d. Urethra. ANS: B
Ureters drain urine from the kidneys into the bladder; if they become obstructed, peristaltic waves attempt to push the obstruction into the bladder. The kidneys, bladder, and urethra do not produce peristaltic waves. Obstruction of both bladder and urethra typically does not occur. DIF: Remember TOP: Evaluate
REF: 1167 OBJ: Describe the process of urination. MSC: NCLEX: Physiological Integrity
2. When reviewing laboratory results, the nurse should immediately notify the health care
provider about which finding? a. Glomerular filtration rate of 20 mL/min b. Urine output of 80 mL/hr c. pH of 6.4 d. Protein level of 2 mg/100 mL I G B.C M ANS: A
N R U S N T
O
Normal glomerular filtration rate should be approximately 125 mL/min; a severe decrease in renal perfusion could indicate a life-threatening problem such as shock or dehydration. Normal urine output is 1000 to 2000 mL/day; an output of 30 mL/hr or less for 2 or more hours would be cause for concern. The normal pH of urine is between 4.6 and 8.0. Protein levels up to 8 mg/100 mL are acceptable; however, values in excess of this could indicate renal disease. DIF: Apply REF: 1172 OBJ: Describe the nursing implications of common diagnostic tests of the urinary system. TOP: Implementation MSC: NCLEX: Physiological Integrity 3. A patient is experiencing oliguria. Which action should the nurse perform first? a. Increase the patient’s intravenous fluid rate. b. Encourage the patient to drink caffeinated beverages. c. Assess for bladder distension. d. Request an order for diuretics. ANS: C
Canadian Fundamentals of Nursing 6th Edition Potter Test Bank The nurse first should gather all assessment data to determine the potential cause of oliguria. It could be that the patient does not have adequate intake, or it could be that the bladder sphincter is not functioning and the patient is retaining water. Increasing fluids is effective if the patient does not have adequate intake, or if dehydration occurs. Caffeine can work as a diuretic but is not helpful if an underlying pathological process is present. An order for diuretics can be obtained if the patient was retaining water, but this should not be the first action. DIF: Analyze REF: 1174| 1175 OBJ: Describe characteristics of normal and abnormal urine. MSC: NCLEX: Physiological Integrity
TOP: Assessment
4. A patient requests the nurse’s assistance to the bedside commode and becomes frustrated
when unable to void in front of the nurse. The nurse understands that the patient is unable to void for which reason? a. Anxiety can make it difficult for abdominal and perineal muscles to relax enough to void. b. The patient does not recognize the physiological signals that indicate a need to void. c. The patient is lonely, and calling the nurse in under false pretenses is a way to get attention. d. The patient is not drinking enough fluids to produce adequate urine output. ANS: A
Attempting to void in the presence of another person can cause anxiety and tension in the muscles, which makes voiding difficult. The nurse should give the patient privacy and adequate time if appropriate. No evidence suggests that an underlying physiological or N R I G B.C M O psychological condition exists.U S N T DIF: Understand REF: 1168 (Box 43-1) OBJ: Identify factors that commonly influence urinary elimination. TOP: Implementation MSC: NCLEX: Psychosocial Integrity 5. The nurse knows that urinary tract infection (UTI) is the most common health care–associated
infection for which of the following reasons? a. Catheterization procedures are performed more frequently than indicated. b. Escherichia coli pathogens are transmitted during surgical or catheterization procedures. c. Perineal care is often neglected by nursing staff. d. Bedpans and urinals are not stored properly and transmit infection. ANS: B
E. coli is the leading pathogenic cause of UTIs; this pathogen enters during invasive procedures. Sterile technique is imperative to prevent the spread of infection. Frequent catheterizations can place a patient at high risk for UTI; however, infection is caused by bacteria, not by the procedure itself. Perineal care is important, and buildup of bacteria can lead to infection, but this is not the primary cause. Bedpans and urinals may become bacteria ridden and should be cleaned frequently; however, bedpans and urinals are not inserted into the urinary tract, and so they are unlikely to be the primary cause of UTI. DIF: Understand
REF: 1169
Canadian Fundamentals of Nursing 6th Edition Potter Test Bank OBJ: Compare common alterations in urinary elimination. MSC: NCLEX: Physiological Integrity
TOP: Implementation
6. An 86-year-old patient tells the nurse that she is experiencing uncontrollable leakage of urine.
Which nursing diagnosis should the nurse include in the patient’s plan of care? a. Urinary retention. b. Hesitancy. c. Urgency. d. Urinary incontinence. ANS: D
Age-related changes such as loss of pelvic muscle tone can cause involuntary loss of urine, known as urinary incontinence. Urinary retention is the inability to empty the bladder. Hesitancy occurs as difficulty initiating urination. Urgency is the feeling of needing to void immediately. DIF: Apply REF: 1169 OBJ: Identify nursing Diagnosis appropriate for patients with alterations in urinary elimination. TOP: Planning MSC: NCLEX: Physiological Integrity 7. A patient has fallen several times in the past week when attempting to get to the bathroom.
The patient informs the nurse that he gets up three or four times a night to urinate. Which recommendation by the nurse is most appropriate in correcting this urinary problem? a. Clear the path to the bathroom of all obstacles before bed. b. Leave the bathroom light on to illuminate a pathway. c. Limit fluid and caffeine intake before bed. d. Practise Kegel exercises to strengthen bladder muscles. ANS: C
NURSINGTB.COM
Reducing fluids, especially caffeine and alcohol, before bedtime can reduce nocturia. Clearing a path to the restroom or illuminating the path, or shortening the distance to the restroom, may reduce falls but will not correct the urination problem. Kegel exercises are useful if a patient is experiencing incontinence. DIF: Apply REF: 1169 OBJ: Discuss nursing measures to promote normal micturition and to reduce episodes of incontinence. TOP: Implementation MSC: NCLEX: Physiological Integrity 8. When caring for a patient with urinary retention, the nurse would anticipate an order for which
of the following? a. Limited fluid intake. b. A urinary catheter. c. Diuretic medication. d. Renal angiography. ANS: B
A urinary catheter would relieve urinary retention. Reducing fluids would reduce the amount of urine produced but would not alleviate the urine retention. Diuretic medication would increase urine production and may worsen the discomfort caused by urine retention. Renal angiography is an inappropriate diagnostic test for urinary retention.
Canadian Fundamentals of Nursing 6th Edition Potter Test Bank DIF: Apply REF: 1169 (Table 43-1) OBJ: Discuss nursing measures to promote normal micturition and to reduce episodes of incontinence. TOP: Implementation MSC: NCLEX: Physiological Integrity 9. Upon palpation, the nurse notices that the bladder is firm and distended; the patient expresses
an urge to urinate. The nurse should follow up by asking which question? a. “When was the last time you voided?” b. “Do you lose urine when you cough or sneeze?” c. “Have you noticed any change in your urination patterns?” d. “Do you have a fever or chills?” ANS: A
To obtain an accurate assessment, the nurse should first determine the source of the discomfort. Urinary retention causes the bladder to be firm and distended. Further assessment to determine the pathological process of the condition can be performed later. Questions concerning fever and chills, changing urination patterns, and losing urine during coughing or sneezing focus on specific pathological conditions. DIF: Apply REF: 1171| 1181 OBJ: Obtain a nursing history for a patient with urinary elimination problems. TOP: Implementation MSC: NCLEX: Physiological Integrity 10. Which of the following is the primary function of the kidneys? a. Metabolizing and excreting medications. b. Maintaining fluid and electrolyte balance. c. Storing and excreting urine. d. Filtering blood cells and N prU otR eiS nsI . NGTB.COM ANS: B
The main purpose of the kidneys is to maintain fluid and electrolyte balance by filtering waste products and regulating pressures. The kidneys filter the by-products of medication metabolism. The bladder stores and excretes urine. The kidneys help maintain red blood cell volume by producing erythropoietin. DIF: Understand TOP: Assessment
REF: 1166 OBJ: Describe the process of urination. MSC: NCLEX: Physiological Integrity
11. While receiving a shift report on a patient, the nurse is informed that the patient has urinary
incontinence. Upon assessment, what would the nurse expect to find? a. An in-dwelling Foley catheter. b. Reddened irritated skin on the buttocks. c. Tiny blood clots in the patient’s urine. d. Foul-smelling discharge indicative of a UTI. ANS: B
Urinary incontinence is uncontrolled urinary elimination; if the urine has prolonged contact with the skin, skin breakdown can occur. An indwelling Foley catheter is a solution for urine retention. Blood clots and foul-smelling discharge are often signs of infection. DIF: Apply REF: 1169| 1171 OBJ: Compare common alterations in urinary elimination.
TOP: Assessment
Canadian Fundamentals of Nursing 6th Edition Potter Test Bank MSC: NCLEX: Physiological Integrity 12. Which nursing diagnosis related to alterations in urinary function in an older person should be
a nurse’s first priority for action? a. Self-care deficit related to decreased mobility. b. Risk of infection. c. Anxiety related to urinary frequency. d. Impaired self-esteem related to lack of independence. ANS: B
In many older persons, muscle tone is poor, which leads to an inability of the bladder to empty completely. Residual urine greatly increases the risk of infection. According to Maslow’s hierarchy of needs, physical health risks should be addressed before emotional/cognitive risks such as anxiety and self-esteem. Decreased mobility can lead to self-care deficit; the nurse’s priority concern for this diagnosis would be infection because the older person must rely on others for basic hygiene. DIF: Apply REF: 1173 OBJ: Identify nursing Diagnosis appropriate for patients with alterations in urinary elimination. TOP: Planning MSC: NCLEX: Physiological Integrity 13. A patient asks about treatment for urinary urge incontinence. The nurse’s best response is
which advice to the patient? a. Perform pelvic floor exercises. b. Drink cranberry juice. c. Avoid voiding frequently. d. Wear an adult diaper. ANS: A
NURSINGTB.COM
Poor muscle tone leads to an inability to control urine flow. The nurse should recommend pelvic muscle strengthening exercises such as Kegel exercises; this solution best addresses the patient’s problem. Drinking cranberry juice is a preventive measure for urinary tract infection. The nurse should not encourage the patient to reduce voiding; residual urine in the bladder increases the risk of infection. Wearing an adult diaper could be considered if attempts to correct the root of the problem fail. DIF: Analyze REF: 1183 OBJ: Identify nursing Diagnosis appropriate for patients with alterations in urinary elimination. TOP: Planning MSC: NCLEX: Physiological Integrity 14. The nurse suspects that a urinary tract infection has progressed to cystitis when the patient
complains of which symptom? a. Dysuria. b. Flank pain. c. Frequency. d. Fever and chills. ANS: C
Cystitis is inflammation of the bladder; associated symptoms include hematuria and urgency/frequency. Dysuria is a common symptom of a lower urinary tract infection. Flank pain, fever, and chills are all signs of pyelonephritis.
Canadian Fundamentals of Nursing 6th Edition Potter Test Bank DIF: Understand REF: 1168| 1169 OBJ: Compare common alterations in urinary elimination. MSC: NCLEX: Physiological Integrity
TOP: Assessment
15. Which assessment question should the nurse ask if stress incontinence is suspected? a. “Does your bladder feel distended?” b. “Do you empty your bladder completely when you void?” c. “Do you experience urine leakage when you cough or sneeze?” d. “Do your symptoms increase with consumption of alcohol or caffeine?” ANS: C
Stress incontinence can be related to intra-abdominal pressure that causes urine leakage, as would happen during coughing or sneezing. The patient’s response about the fullness of the bladder would rule out retention and overflow. An inability to void completely can refer to urge incontinence. Physiological causes and medications can affect elimination, but this is not related to stress incontinence. DIF: Apply REF: 1170 (Table 43-2) OBJ: Compare common alterations in urinary elimination. MSC: NCLEX: Physiological Integrity
TOP: Planning
16. To obtain a clean-voided urine specimen for a female patient, the nurse should teach the
patient to do which of the following? a. Cleanse the urethral meatus from the area of most contamination to least. b. Initiate the first part of the urine stream directly into the collection cup. c. Hold the labia apart while voiding into the specimen cup. d. Drink fluids 5 minutes before collecting the urine specimen. ANS: C
NURSINGTB.COM
The patient should hold the labia apart to reduce bacterial levels in the specimen. The urethral meatus should be cleansed from the area of least contamination to greatest contamination (front to back). The initial steam flushes out microorganisms in the urethra and prevents bacterial transmission in the specimen. Patients should drink fluids 30 to 60 minutes before giving a specimen. DIF: Understand TOP: Planning
REF: 1178-1180 (Skill 43-1) MSC: NCLEX: Physiological Integrity
OBJ: Obtain urine specimens.
17. When viewing a urine specimen under a microscope, what would the nurse expect to see in a
patient with a urinary tract infection? a. Bacteria. b. Casts. c. Crystals. d. Protein. ANS: A
Bacteria indicate a urinary tract infection. Crystals would be seen with renal stone formation. Casts indicate renal alterations. Protein is not visible under a microscope, but its presence indicates renal disease. DIF: Remember REF: 1180| 1181 (Table 43-5) OBJ: Describe characteristics of normal and abnormal urine.
TOP: Assessment
Canadian Fundamentals of Nursing 6th Edition Potter Test Bank MSC: NCLEX: Physiological Integrity 18. The nurse would expect the urine of a patient with uncontrolled diabetes mellitus to be which
of the following? a. Cloudy. b. Discoloured. c. Sweet smelling. d. Painful. ANS: C
Incomplete fat metabolism and buildup of ketones give urine a sweet or fruity odour. Cloudy urine may indicate infection or renal failure. Discoloration of urine may result from various medications. Painful urination indicates an alteration in urinary elimination. DIF: Understand REF: 1177 OBJ: Describe characteristics of normal and abnormal urine. MSC: NCLEX: Physiological Integrity
TOP: Assessment
19. What signs and symptoms would the nurse expect to observe in a patient with excessive white
blood cells present in the urine? a. Fever and chills. b. Difficulty with urinary elimination. c. Increased blood pressure. d. Abnormal blood glucose level. ANS: A
The presence of white blood cells in urine indicates a urinary tract infection. Difficulty with urinary elimination indicatesN bloR Uckag SIeNoGr ren TBa.l dCam OMage. Increase in blood pressure is associated with renal disease or damage and some medications. Abnormal blood glucose levels would be seen in someone with ketones in the urine, as this finding indicates the presence of diabetes. DIF: Understand REF: 1204 OBJ: Describe characteristics of normal and abnormal urine. MSC: NCLEX: Physiological Integrity
TOP: Assessment
20. For a patient who has severe flank pain and calcium phosphate crystals revealed on urinalysis,
the nurse would anticipate an order for which diagnostic test? a. Renal ultrasonography. b. Bladder scan. c. Kidney, ureter, and bladder (KUB) radiography. d. Intravenous pyelography. ANS: D
Flank pain and calcium phosphate crystals are associated with renal calculi. Intravenous pyelography allows the health care provider to observe pathological problems such as obstruction of the ureter. Renal ultrasonography is performed to identify gross structures. A bladder scan helps measure the amount of urine in the bladder. A KUB radiograph shows size, shape, symmetry, and location of the kidneys. DIF: Understand REF: 1161| 1182 (Table 43-6) OBJ: Describe the nursing implications of common diagnostic tests of the urinary system.
Canadian Fundamentals of Nursing 6th Edition Potter Test Bank TOP: Planning
MSC: NCLEX: Physiological Integrity
21. A nurse is caring for a patient who just underwent intravenous pyelography that revealed a
renal calculus obstructing the left ureter. What is the nurse’s first priority in caring for this patient? a. Turn the patient on the right side to alleviate pressure on the left kidney. b. Encourage the patient to increase fluid intake to flush the obstruction. c. Administer narcotic medications to alleviate pain. d. Monitor the patient for fever, rash, and difficulty breathing. ANS: D
To perform intravenous pyelography, iodine-based dye is administered to reveal functionality of the urinary system. Many individuals are allergic to shellfish; therefore, the first nursing priority is to assess the patient for an allergic reaction that could be life-threatening. The nurse should then encourage the patient to drink fluids to flush dye used in the procedure. Narcotics can be administered but are not the first priority. Turning the patient on the side does not affect patient safety. DIF: Analyze REF: 1182 (Table 43-6) OBJ: Describe the nursing implications of common diagnostic tests of the urinary system. TOP: Planning MSC: NCLEX: Physiological Integrity 22. Which statement by the patient about upcoming computed tomographic (CT) scanning
indicates a need for further teaching? a. “I’m allergic to shrimp, so I should monitor myself for an allergic reaction.” b. “I will complete my bowel prep program the night before the scan.” c. “I will be anaesthetized so that I lie perfectly still during the procedure.”
My anxiety.” d. “I will ask the technicianN toUpRlaSyI mNuG sicTtB o. eaCseOm ANS: C
Patients are not put under anaesthesia for a CT scan; instead, the nurse should educate patients about the need to lie perfectly still and about possible methods of overcoming feelings of claustrophobia. The other options are correct. Patients need to be assessed for an allergy to shellfish if they are to receive contrast material for the CT scan. Bowel cleansing is often performed before CT scanning. Listening to music will help the patient relax and remain still during the examination. DIF: Apply REF: 1182 (Table 43-6) OBJ: Describe the nursing implications of common diagnostic tests of the urinary system. TOP: Planning MSC: NCLEX: Safe and Effective Care Environment 23. The nurse is visiting the patient who has a nursing diagnosis of Alteration in urinary
elimination, retention. On assessment, the nurse anticipates that this patient will exhibit which of the following? a. Severe flank pain and hematuria. b. Pain and burning on urination. c. A loss of the urge to void. d. A feeling of pressure and voiding of small amounts. ANS: D
Canadian Fundamentals of Nursing 6th Edition Potter Test Bank With urinary retention, urine continues to collect in the bladder, stretching its walls and causing feelings of pressure, discomfort, tenderness over the symphysis pubis, restlessness, and diaphoresis. The sphincter temporarily opens to allow a small volume of urine (<75 mL) to escape, with no real relief of discomfort. Severe flank pain and hematuria are data indicating an upper UTI (pyelonephritis). Pain and burning on urination are symptoms of a lower UTI (such as a bladder infection). Data indicating reflex incontinence would include a loss of the urge to void. DIF: Apply REF: 1171 OBJ: Describe the nursing implications of common diagnostic tests of the urinary system. TOP: Planning MSC: NCLEX: Physiological Integrity 24. A nurse anticipates urodynamic testing for a patient with which symptom? a. Involuntary urine leakage. b. Severe flank pain. c. Presence of blood in urine. d. Dysuria. ANS: A
Urodynamic testing is used to evaluate the muscle function of the bladder and to look for the cause of urinary incontinence. Severe flank pain indicates renal calculi; CT scan or intravenous pyelography would be a more efficient diagnostic test. Blood indicates trauma to the urethral or bladder mucosa. Pain on elimination may warrant cultures to check for infection. DIF: Understand REF: 1183 (Table 43-6) OBJ: Describe the nursing implications of common diagnostic tests of the urinary system. TOP: Planning MSC: NN CLUER XS : PIhN ysG ioT loB gi. caC l IO ntM egrity 25. A patient is having difficulty voiding in a bedpan but states that she feels her bladder is full.
To stimulation micturition, which nursing intervention should the nurse try first? a. Exiting the room and informing the patient that the nurse will return in 30 minutes to check on the patient’s progress. b. Utilizing the power of suggestion by turning on the faucet and letting the water run. c. Obtaining an order for a Foley catheter. d. Administering diuretic medication. ANS: B
To stimulate micturition, the nurse should attempt noninvasive procedures first. Running warm water or stroking the inner aspect of the upper thigh promotes sensory perception that leads to urination. A patient should not be left alone on a bedpan for 30 minutes because this could cause skin breakdown. Catheterization places the patient at increased risk for infection and should not be the first intervention attempted. Diuretics are useful if the patient is not producing urine, but they do not stimulate micturition. DIF: Apply REF: 1186 OBJ: Discuss nursing measures to promote normal micturition and to reduce episodes of incontinence. TOP: Planning MSC: NCLEX: Physiological Integrity
Canadian Fundamentals of Nursing 6th Edition Potter Test Bank 26. A nurse is caring for an 8-year-old patient who is embarrassed about urinating in his bed at
night. Which intervention should the nurse suggest to reduce the frequency of this occurrence? a. “Drink your nightly glass of milk earlier in the evening.” b. “Set your alarm clock to wake you every 2 hours, so you can get up to void.” c. “Line your bedding with plastic sheets to protect your mattress.” d. “Empty your bladder completely before going to bed.” ANS: A
Nightly incontinence and nocturia are often resolved by limiting fluid intake 2 hours before bedtime. Setting the alarm clock to wake does not correct the physiological problem; nor does lining the bedding with plastic sheets. Emptying the bladder may help with early nighttime urination, but will not affect urine produced throughout the night from late-night fluid intake. DIF: Apply REF: 1186 OBJ: Discuss nursing measures to promote normal micturition and to reduce episodes of incontinence. TOP: Implementation MSC: NCLEX: Physiological Integrity 27. Many individuals have difficulty voiding in a bedpan or urinal while lying in bed for which
reason? a. They are embarrassed that they will urinate on the bedding. b. They would feel more comfortable assuming a normal voiding position. c. They feel they are losing their independence by asking the nursing staff to help. d. They are worried about acquiring a urinary tract infection. ANS: B
Assuming a normal voiding pNosi tion relax and be able to void; lying in bed is R Ihel GpsTpBat.ieCntsOM U S N not the typical position in which people void. Men usually are most comfortable when standing; women are most comfortable when sitting and squatting. Embarrassment at using the bedpan and worrying about a urinary tract infection are not related to the lying-in-bed position. Fear of loss of independence is not related to use of the bedpan or urinal. DIF: Understand REF: 1186 OBJ: Discuss nursing measures to promote normal micturition and to reduce episodes of incontinence. TOP: Planning MSC: NCLEX: Physiological Integrity 28. The nurse would anticipate inserting a coudé catheter for which patient? a. An 8-year-old boy undergoing anaesthesia for a tonsillectomy. b. A 24-year-old woman who is going into labour. c. A 56-year-old man admitted for bladder irrigation. d. An 86-year-old woman admitted for a urinary tract infection. ANS: C
A coudé catheter has a curved tip that is used for patients with enlarged prostates. This would be indicated for a middle-aged man who needs bladder irrigation. Coudé catheters are not indicated for children or women. DIF: Apply REF: 1189 TOP: Implementation
OBJ: Insert a urinary catheter. MSC: NCLEX: Physiological Integrity
Canadian Fundamentals of Nursing 6th Edition Potter Test Bank 29. The nurse knows that which in-dwelling catheter procedure places the patient at greatest risk
for acquiring a urinary tract infection? a. Emptying the drainage bag every 8 hours or when half full. b. Kinking the catheter tubing to obtain a urine specimen. c. Placing the drainage bag on the side rail of the patient’s bed. d. Failing to secure the catheter tubing to the patient’s thigh. ANS: C
Placing the drainage bag on the side rail of the bed could allow the bag to be raised above the level of the bladder and urine to flow back into the bladder. The urine in the drainage bag is a medium for bacteria; its re-entrance into the bladder can cause infection. The drainage bag should be emptied and output recorded every 8 hours or when needed. Urine specimens are obtained by temporarily kinking the tubing; a prolonged kink could lead to bladder distension. Failure to secure the catheter to the patient’s thigh places the patient at risk for tissue injury from catheter dislodgement. DIF: Apply REF: 1189 OBJ: Discuss nursing measures to reduce urinary tract infection. TOP: Implementation MSC: NCLEX: Safe and Effective Care Environment 30. A nurse notifies the provider immediately if a patient with an in-dwelling catheter does which
of the following? a. Complains of discomfort upon insertion of the catheter. b. Places the drainage bag higher than the waist while ambulating. c. Has not collected any urine in the drainage bag for 2 hours. d. Is incontinent of stool and contaminates the external portion of the catheter. ANS: C
NURSINGTB.COM
If the patient has not produced urine in 2 hours, the physician needs to be notified immediately because this could indicate renal failure. Discomfort upon catheter insertion is unpleasant but unavoidable. The nurse is responsible for maintaining the integrity of the catheter by ensuring that the drainage bag is below the patient’s bladder. Stool left on the catheter can cause infection and should be removed as soon as it is noticed. The nurse should ensure that frequent perineal care is being provided. DIF: Apply TOP: Evaluate
REF: 1169 OBJ: Insert a urinary catheter. MSC: NCLEX: Physiological Integrity
31. The nurse would question an order to insert a urinary catheter on which patient? a. A 26-year-old patient with a recent spinal cord injury at T2. b. A 30-year-old patient requiring drug screening for employment. c. A 40-year-old patient undergoing bladder repair surgery. d. An 86-year-old patient requiring monitoring of urinary output for renal failure. ANS: B
Urinary catheterization places the patient at increased risk for infection and should be performed only when necessary. Urine can be obtained via clean-catch technique for a drug screening or urinalysis. Spinal cord injury, surgery, and renal failure with critical monitoring of intake and output are all appropriate reasons for catheterization. DIF: Apply REF: 1169 (Table 43-1) OBJ: Insert a urinary catheter. TOP: Implementation MSC: NCLEX: Physiological Integrity
Canadian Fundamentals of Nursing 6th Edition Potter Test Bank
32. When caring for a hospitalized patient with a urinary catheter, which nursing action best
prevents the patient from acquiring an infection? a. Inserting the catheter with strict clean technique. b. Performing hand hygiene before and after providing perineal care. c. Fully inflating the catheter’s balloon according to the manufacturer’s recommendation. d. Disconnecting and replacing the catheter drainage bag once per shift. ANS: B
Hand hygiene helps prevent infection in patients with a urinary catheter. A catheter should be inserted in the hospital setting with sterile technique. Inflating the balloon fully prevents dislodgement and trauma, not infection. Disconnecting the drainage bag from the catheter creates a break in the system and an open portal of entry and increases risk of infection. DIF: Apply REF: 1172 OBJ: Discuss nursing measures to reduce urinary tract infection. TOP: Implementation MSC: NCLEX: Physiological Integrity 33. An 86-year-old patient asks the nurse what lifestyle changes will reduce the chance of a
urinary tract infection. Which response is accurate? a. Urinary tract infections are unavoidable in older people because of a weakened immune system. b. Decreasing fluid intake will decrease the amount of urine with bacteria produced. c. Making sure to cleanse the perineal area from back to front after voiding will reduce the chance of infection. d. Increasing consumption of acidic foods such as cranberry juice will reduce the NURSINGTB.COM chance of infection. ANS: D
Cranberry juice and other acidic foods decrease adherence of bacteria to the bladder wall. Urinary tract infections are avoidable in older people with proper knowledge and hygiene. Perineal skin should be cleansed from front to back to avoid spreading fecal matter to the urethra. Increasing fluid intake will help flush out bacteria, thus preventing them from residing in the bladder for prolonged periods of time. DIF: Apply REF: 1173 (Box 43-2) OBJ: Discuss nursing measures to reduce urinary tract infection. TOP: Implementation MSC: NCLEX: Physiological Integrity 34. A nurse is providing education to a patient being treated for a urinary tract infection. Which of
the following statements by the patient indicates an understanding? a. “Because I’m taking medication, I do not need to worry about proper hygiene.” b. “I should drink 15 to 20 glasses of fluid a day to help flush the bacteria out.” c. “My medication may discolour my urine; this should resolve once the medication is stopped.” d. “I should not have sexual intercourse until the infection has resolved.” ANS: C
Canadian Fundamentals of Nursing 6th Edition Potter Test Bank Some medications turn urine colours; this is normal and will dissipate as the medication leaves the system. Even if the patient is on medication, hygiene is important to prevent spread or reinfection. Fluid intake should be increased to help flush out bacteria; however, 15 to 20 glasses is too much. Sexual intercourse is allowed with a urinary tract infection, as long as good hygiene and safe practices are used. DIF: Apply REF: 1176 OBJ: Discuss nursing measures to reduce urinary tract infection. TOP: Evaluate MSC: NCLEX: Physiological Integrity 35. To reduce patient discomfort during closed catheter irrigation, what should the nurse do? a. Use room temperature irrigation solution. b. Administer the solution as quickly as possible. c. Allow the solution to sit in the bladder for at least 1 hour. d. Raise the bag of irrigation solution at least 30 centimetres above the bladder. ANS: A
Using cold solutions, instilling solutions too quickly, and prolonging filling of the bladder can cause discomfort and cramping. To reduce this, ensure that the solution is at room temperature, lower the solution bag so that it is instilled slowly, and drain the bladder fully after an ordered amount of time. DIF: Apply REF: 1201 (Skill 43-4) OBJ: Irrigate a urinary catheter. TOP: Implementation MSC: NCLEX: Physiological Integrity 36. Which observation by the nurse best indicates that bladder irrigation for urinary retention has
been effective? a. An output that is larger thNaU nR thS eI amNoG unTtB in. stC ilO ledM. b. The presence of blood clots or sediment in the drainage bag. c. Reduction in discomfort from bladder distension. d. Visualizing clear urinary catheter tubing. ANS: A
An output that is greater than what was irrigated into the bladder shows progress that the bladder is draining urine. The other observations do not objectively help measure the increase in urine output. DIF: Analyze REF: 1176 TOP: Implementation
OBJ: Irrigate a urinary catheter. MSC: NCLEX: Physiological Integrity
37. The nurse anticipates urinary diversion from the kidneys to a site other than the bladder for
which patient? a. A 12-year-old girl with severe abdominal trauma. b. A 24-year-old man with severe genital warts around the urethra. c. A 50-year-old man who has recently undergone prostatectomy. d. A 75-year-old woman with end-stage renal disease. ANS: A
Urinary diversion would be needed in a patient with abdominal trauma, which might cause injury to the urinary system. Genital warts are not a reason for urinary diversion. Patients with a prostatectomy may require intermittent catheterization after the procedure. End-stage renal disease would not be affected by rerouting the flow of urine.
Canadian Fundamentals of Nursing 6th Edition Potter Test Bank
DIF: Apply REF: 1171 OBJ: Compare common alterations in urinary elimination. MSC: NCLEX: Physiological Integrity
TOP: Implementation
MULTIPLE RESPONSE 1. Which nursing actions are acceptable when a urine specimen is collected? (Select all that
apply.) a. Growing urine cultures for up to 12 hours. b. Labelling all specimens with date, time, and initials. c. Wearing gown, gloves, and mask for all specimen handling. d. Allowing the patient adequate time and privacy to void. e. Squeezing urine from diapers into a urine specimen cup. f. Transporting specimens to the laboratory in a timely manner. g. Placing a plastic bag over the child’s urethra to catch urine. ANS: B, D, F, G
All specimens should be labelled appropriately and processed in a timely manner. Patients should be allowed time and privacy to void. Children may have difficulty voiding; attaching a plastic bag gives the child more time and freedom to void. Urine cultures can take up to 48 hours to develop. Gown, gloves, and mask are not necessary for specimen handling unless otherwise indicated. Urine should not be squeezed from diapers. DIF: Understand REF: 1177-1180 TOP: Implementation
OBJ: Obtain urine specimens. MSC: NCLEX: Physiological Integrity
N R I G B.C M
2. Which of the following are indU icatS ionsNforTirrigatiO ng a urinary catheter? (Select all that apply.) a. Sediment occluding within the tubing. b. Blood clots in the bladder after surgery. c. Rupture of the catheter balloon. d. Presence of renal calculi. ANS: A, B
Catheter irrigation is used to flush and remove blockage that may be impeding the catheter from properly draining the bladder. It is also used to remove blood clots in the bladder after surgery. A ruptured catheter balloon will involve extensive follow-up and possible surgery to remove the particles. Renal calculi obstruct the ureters and therefore the flow of urine before it reaches the bladder. DIF: Apply REF: 1198-1200 TOP: Implementation
OBJ: Irrigate a urinary catheter. MSC: NCLEX: Physiological Integrity
3. Which of the following symptoms are most closely associated with uremic syndrome? (Select
all that apply.) a. Fever. b. Nausea and vomiting. c. Headache. d. Altered mental status. e. Dysuria.
Canadian Fundamentals of Nursing 6th Edition Potter Test Bank ANS: B, C, D
Uremic syndrome is associated with end-stage renal disease. Signs and symptoms include headache, altered mental status, coma, seizures, nausea, vomiting, and pericarditis. DIF: Remember REF: 1171| 1172 OBJ: Compare common alterations in urinary elimination. MSC: NCLEX: Physiological Integrity
TOP: Implementation
4. The nurse understands that peritoneal dialysis and hemodialysis involve which processes to
clean the patient’s blood? (Select all that apply.) a. Gravity. b. Osmosis. c. Diffusion. d. Filtration. ANS: B, C
Osmosis and diffusion are the two processes used to clean the patient’s blood in both types of dialysis. In peritoneal dialysis, osmosis and dialysis occur across the semi-permeable peritoneal membrane. In hemodialysis, osmosis and dialysis occur through the filter membrane on the artificial kidney. In peritoneal dialysis, the dialysate flows by gravity out of the abdomen. Gravity has no effect on cleansing of the blood. Filtration is the process that occurs in the glomerulus as blood flows through the kidney. DIF: Understand REF: 1172 OBJ: Identify two modalities of renal replacement therapy. MSC: NCLEX: Physiological Integrity
NURSINGTB.COM
TOP: Implementation
Canadian Fundamentals of Nursing 6th Edition Potter Test Bank
Chapter 44: Bowel Elimination Potter et al: Canadian Fundamentals of Nursing, 6th Edition MULTIPLE CHOICE 1. The nurse knows that most nutrients are absorbed in which portion of the digestive tract? a. Stomach. b. Duodenum. c. Ileum. d. Cecum. ANS: B
Most nutrients are absorbed in the duodenum with the exception of certain vitamins, iron, and salt (which are absorbed in the ileum). Food is broken down in the stomach. The cecum is the beginning of the large intestine. DIF: Remember REF: 1212 OBJ: Discuss the role of gastrointestinal organs in digestion and elimination. TOP: Implementation MSC: NCLEX: Physiological Integrity 2. The nurse would expect the least formed stool to be present in which portion of the digestive
tract? a. Ascending. b. Descending. c. Transverse. d. Sigmoid. ANS: A
NURSINGTB.COM
The path of digestion goes from the ascending, across the transverse, to the descending and finally passing into the sigmoid; therefore, the least formed stool would be in the ascending. DIF: Understand REF: 1212 OBJ: Explain the physiological aspects of normal defecation. MSC: NCLEX: Physiological Integrity
TOP: Assessment
3. Which of the following is not a function of the large intestine? a. Absorbing nutrients. b. Absorbing water. c. Secreting bicarbonate. d. Eliminating waste. ANS: A
Nutrient absorption takes place in the small intestine. The other options are all functions of the large intestine. DIF: Remember REF: 1213| 1214 TOP: Implementation
OBJ: Describe the functions of the large intestine. MSC: NCLEX: Physiological Integrity
4. The nurse is caring for a patient who is confined to the bed. The nurse asks the patient if he
needs to have a bowel movement 30 minutes after eating a meal for what reason? a. The digested food needs to make room for recently ingested food.
Canadian Fundamentals of Nursing 6th Edition Potter Test Bank b. Ingestion of food triggers the digestive system to begin peristalsis. c. The smell of bowel elimination in the room would deter the patient from eating. d. More ancillary staff members are available after meal times. ANS: B
Peristalsis occurs only a few times a day; the strongest peristaltic waves are triggered by ingestion of food. The intestine can hold a great deal of food. A patient’s voiding schedule should not be based on the staff’s convenience. DIF: Understand REF: 1214| 1216 OBJ: List the nursing measures included in bowel training. MSC: NCLEX: Physiological Integrity
TOP: Implementation
5. A nurse is assisting a patient in making dietary choices that promote healthy bowel
elimination. Which menu option should the nurse recommend? a. Grape and walnut chicken salad sandwich on whole wheat bread. b. Broccoli and cheese soup with potato bread. c. Dinner salad topped with hard-boiled eggs, cheese, and fat-free dressing. d. Turkey and mashed potatoes with brown gravy. ANS: A
A healthy diet for the bowel should include foods high in bulk-forming fibre. Whole grains, fresh fruit, and fresh vegetables are excellent sources. Foods without much fibre and with high levels of fat can slow down peristalsis, causing constipation. DIF: Apply REF: 1216 OBJ: List the nursing measures that promote normal elimination. TOP: Implementation : C NCLMEX: Physiological Integrity N R I GMSBC.
U S N T
O
6. A patient informs the nurse that she was using laxatives three times daily to lose weight. After
stopping use of the laxative, the patient had difficulty with constipation and wonders if she needs to take laxatives again. With which statement does the nurse educate the patient? a. “Long-term laxative use causes the bowel to become less responsive to stimuli, and constipation may occur.” b. “Laxatives can cause trauma to the intestinal lining and scarring may result, leading to decreased peristalsis.” c. “Natural laxatives such as mineral oil are safer than chemical laxatives for relieving constipation.” d. “Laxatives cause the body to become malnourished, so when the patient begins eating again, the body absorbs all of the food, and no waste products are produced.” ANS: A
Long-term laxative use can lead to constipation. Increasing fluid and fibre intake can help with this problem. Laxatives do not cause scarring. Natural laxatives like mineral oil come with their own set of risks, such as rendering the body unable to absorb fat-soluble vitamins. Even if malnourished, the body will produce waste if substance is consumed. DIF: Understand REF: 1225 (Box 44-5) OBJ: Use critical thinking in the provision of care to patients with alterations in bowel elimination. TOP: Implementation MSC: NCLEX: Physiological Integrity
Canadian Fundamentals of Nursing 6th Edition Potter Test Bank 7. A patient with a hip fracture is having difficulty defecating into a bed pan while lying in bed.
Which action by the nurse would assist the patient in having a successful bowel movement? a. Administering laxatives to the patient. b. Raising the head of the bed. c. Preparing to administer a barium enema. d. Withholding narcotic pain medication. ANS: B
Lying in bed is an unnatural position for defecation; raising the head of the bed assists the patient into a more normal position that allows proper contraction of muscles for elimination. Laxatives would not give the patient control over bowel movements. A barium enema is used for a diagnostic test; it is not used as an intervention to promote defecation. Pain relief measures should be given; however, action should be taken to prevent constipation. DIF: Apply REF: 1215 OBJ: Discuss the psychological and physiological factors that influence the elimination process. TOP: Implementation MSC: NCLEX: Physiological Integrity 8. Which patient is most at risk for increased peristalsis? a. A 5-year-old child who ignores the urge to defecate owing to embarrassment. b. A 21-year-old patient with three final examinations on the same day. c. A 40-year-old woman with major depressive disorder. d. An 80-year-old man in an assisted-living environment. ANS: B
Stress can stimulate digestion and increase peristalsis. Ignoring the urge to defecate, depression, and age-related changes in older people are causes of constipation.
NURSINGTB.COM
DIF: Understand REF: 1224| 1227 OBJ: Describe the common physiological alterations in elimination. TOP: Implementation MSC: NCLEX: Psychosocial Integrity 9. A patient expresses concerns over having black stool. The fecal occult test is negative. Which
response by the nurse is most appropriate? a. “This is probably a false-negative result; we should rerun the test.” b. “Do you take iron supplements?” c. “You should schedule a colonoscopy as soon as possible.” d. “Sometimes severe stress can alter stool colour.” ANS: B
Certain medications and supplements, such as iron, can alter the colour of stool. The fecal occult test takes three separate samples over a period of time and is a fairly reliable test. A colonoscopy is health prevention screening that should be done every 5 to 10 years; it is not the nurse’s initial priority. Stress alters gastrointestinal (GI) motility and stool consistency, not colour. DIF: Apply REF: 1221 (Table 44-2)| 1223 (Table 44-4) OBJ: Use critical thinking in the provision of care to patients with alterations in bowel elimination. TOP: Implementation MSC: NCLEX: Physiological Integrity 10. Which physiological change can cause a paralytic ileus? a. Chronic cathartic abuse.
Canadian Fundamentals of Nursing 6th Edition Potter Test Bank b. Surgery with anaesthesia for Crohn’s disease. c. Suppression of hydrochloric acid from medication. d. Fecal impaction. ANS: B
Surgical manipulation of the bowel can cause a paralytic ileus. DIF: Remember REF: 1220 OBJ: Describe the common physiological alterations in elimination. TOP: Implementation MSC: NCLEX: Physiological Integrity 11. Fecal impactions occur in which portion of the colon? a. Ascending. b. Descending. c. Transverse. d. Rectum. ANS: D
A fecal impaction is a collection of hardened feces wedged in the rectum that cannot be expelled. It results from unrelieved constipation. Feces at this point in the colon contain the least amount of moisture. Feces found in the ascending, transverse, and descending colon still consist mostly of liquid and do not form a hardened mass. DIF: Remember TOP: Assessment
REF: 1225 OBJ: Assess a patient’s pattern of elimination. MSC: NCLEX: Physiological Integrity
12. The nurse knows that a bowel elimination schedule would be most beneficial in the plan of
care for which patient? NURSINGTB.COM a. A 40-year-old patient with an ileostomy. b. A 25-year-old patient with Crohn’s disease. c. A 30-year-old patient with C. difficile infection. d. A 70-year-old patient with stool incontinence. ANS: D
A bowel retraining program is helpful for a patient with incontinence. It helps the person who still has neuromuscular control defecate normally. An ileostomy, Crohn’s disease, and C. difficile infection all relate to uncontrollable bowel movements, for which a schedule of elimination is of no use. DIF: Analyze REF: 1229 OBJ: List the nursing measures included in bowel training. MSC: NCLEX: Physiological Integrity
TOP: Planning
13. Which nursing intervention is most effective in promoting normal defecation for a patient who
has muscle weakness in the legs that prevents ambulation? a. Elevating the head of the bed 45 degrees 60 minutes after breakfast. b. Using a mobility device to place the patient on a bedside commode. c. Giving the patient a pillow to brace against the abdomen while bearing down. d. Administering a soapsuds enema every 2 hours. ANS: B
Canadian Fundamentals of Nursing 6th Edition Potter Test Bank The best way to promote normal defecation is to assist the patient into a posture for defecating that is as normal as possible. The use of a mobility device promotes nurse and patient safety. Elevating the head of the bed would be appropriate if the patient were to void with a bedpan. However, the patient’s condition does not necessitate use of a bedpan. Giving the patient a pillow may reduce discomfort, but this is not the best way to promote defecation. A soapsuds enema is indicated for a patient who needs assistance to stimulate peristalsis. It promotes nonnatural defecation. DIF: Apply REF: 1218 OBJ: List the nursing measures included in bowel training. MSC: NCLEX: Physiological Integrity
TOP: Implementation
14. The nurse is devising a plan of care for a patient with the nursing diagnosis of Constipation
related to opioid use. Which of the following outcomes would the nurse evaluate as successful for the patient to establish normal defecation? a. The patient reports eliminating a soft, formed stool. b. The patient has quit taking opioid pain medication. c. The patient’s lower left quadrant is tender to the touch. d. The nurse hears bowel sounds in all four quadrants. ANS: A
The nurse’s goal is for the patient to continue taking opioid medication and to have normal bowel elimination. Normal stools are soft and formed. Ceasing pain medication is not a desired outcome for the patient. Tenderness in the left lower quadrant indicates constipation and does not further address bowel elimination. Presence of bowel sounds indicate that the bowels are moving; however, they are not an indication of defecation. DIF: Apply REF: 12N 14URSINGTB.COM OBJ: List the nursing Diagnosis related to alterations in elimination. TOP: Evaluate MSC: NCLEX: Physiological Integrity 15. The nurse is emptying an ileostomy pouch for a patient. Which assessment finding would the
nurse report immediately? a. Liquid consistency of stool. b. Presence of blood in the stool. c. Noxious odour from the stool. d. Continuous output from the stoma. ANS: B
Blood in the stool may indicate a problem with the surgical procedure, and the physician should be notified. All other options are expected findings for an ileostomy. DIF: Apply REF: 1221 OBJ: Use critical thinking in the provision of care to patients with alterations in bowel elimination. TOP: Assessment MSC: NCLEX: Physiological Integrity 16. The nurse would anticipate which diagnostic examination for a patient with black, tarry
stools? a. Ultrasonography. b. Barium enema study. c. Upper endoscopy.
Canadian Fundamentals of Nursing 6th Edition Potter Test Bank d. Flexible sigmoidoscopy. ANS: C
Black tarry stools are an indication of ulceration or bleeding in the upper portion of the GI tract; upper endoscopy would enable visualization of the bleeding. No other option would enable upper GI visualization. DIF: Understand REF: 1221 (Table 44-3) OBJ: Use critical thinking in the provision of care to patients with alterations in bowel elimination. TOP: Planning MSC: NCLEX: Physiological Integrity 17. The nurse has attempted to administer a tap water enema to a patient with fecal impaction,
with no success. What is the next priority nursing action? a. Preparing the patient for a second tap water enema. b. Donning gloves for digital removal of the stool. c. Positioning the patient on the left side. d. Inserting a rectal tube. ANS: B
When enemas are not successful, digital removal of the stool is occasionally necessary to break up pieces of the stool or to stimulate the anus to defecate. Tap water enemas should not be repeated because of risk of fluid imbalance. Positioning the patient on the left side does not promote defecation. A rectal tube is indicated for a patient with liquid stool incontinence but would not be applicable or effective for this patient. DIF: Apply REF: 1235 (Box 44-10) OBJ: Use critical thinking in the provision of care to patients with alterations in bowel elimination. TOP: Implementation : C NCLMEX: Physiological Integrity N R I GMSBC.
U S N T
O
18. The nurse should question which order? a. A normal saline enema to be repeated every 4 hours until stool is produced. b. A hypertonic solution enema with a patient with fluid volume excess. c. A Kayexalate enema for a patient with hypokalemia. d. An oil retention enema for a patient using mineral oil laxatives. ANS: C
Kayexalate binds to and helps excrete potassium, so it would be contraindicated in patients who are hypokalemic (have low potassium). Normal saline enemas can be repeated without risk of fluid or electrolyte imbalance. Hypertonic solutions are intended for patients who cannot handle large fluid volume, and they are contraindicated for dehydrated patients. Because mineral oil laxatives and an oil retention enema have the same intended effect of lubricating the colon and rectum, an oil retention enema is not needed. DIF: Apply REF: 1232 OBJ: Use critical thinking in the provision of care to patients with alterations in bowel elimination. TOP: Planning MSC: NCLEX: Physiological Integrity 19. The nurse is preparing to perform a fecal occult blood test. The nurse plans to properly
perform the examination by doing which of the following? a. Applying liberal amounts of stool to the guaiac paper. b. Not allowing patients to collect their own specimens c. Reporting any abnormal findings to the care provider.
Canadian Fundamentals of Nursing 6th Edition Potter Test Bank d. Applying sterile disposable gloves. ANS: C
Abnormal findings such as a positive test result should be reported to the care provider. A fecal occult blood test is a clean procedure; sterile gloves are not needed. A thin specimen smear is all that is required. Patients can collect their own specimens if possible. DIF: Understand REF: 1221| 1222 (Box 44-1) OBJ: Describe the nursing implications for common diagnostic examinations of the gastrointestinal tract. TOP: Planning MSC: NCLEX: Safe and Effective Care Environment 20. A nurse is preparing a patient for magnetic resonance imaging (MRI). Which nursing action is
most important? a. Ensuring that the patient does not eat or drink 2 hours before the examination. b. Removing all of the patient’s metallic jewellery. c. Administering a colon-cleansing product 12 hours before the examination. d. Obtaining an order for a pain medication before the test is performed. ANS: B
No jewellery or metal products should be in the same room as an MRI machine because of the high-power magnet used in the machine. The patient needs to be on NPO status (nothing by mouth) 4 to 6 hours before the examination. Colon cleansing products are not necessary for MRIs. Pain medication is not needed before the examination is performed. DIF: Apply REF: 1223 (Box 44-3) OBJ: Describe nursing implications for common diagnostic examinations of the gastrointestinal tract. TOP: Planning MSC: NCLEX: Safe and Effective Care Environment
N R I G B.C M
21. After a patient returns from a bU ariuS m sN walT low stuO dy, what is the nurse’s priority? a. Encourage the patient to increase fluid intake to flush out the barium. b. Monitor stools closely for bright red blood or mucus, which indicates trauma from
the procedure. c. Inform the patient that the bowel movements are radioactive and that the patient
should be sure to flush the toilet three times. d. Thicken all the patient’s drinks to prevent aspiration. ANS: A
The nurse should encourage the patient to increase fluid intake to flush and remove excess barium from the body. Barium swallow studies are noninvasive procedures that do not involve trauma, and so no blood or mucus would result, and aspiration risk would not be increased. Barium is not a radioactive substance, and so multiple flushes are not needed. DIF: Apply REF: 1223 (Box 44-3) OBJ: Describe the nursing implications for common diagnostic examinations of the gastrointestinal tract. TOP: Planning MSC: NCLEX: Physiological Integrity 22. A nurse is educating a patient on how to irrigate an ostomy bag. Which statement by the
patient indicates the need for further instruction? a. “I can use a Fleet enema to save money because it contains the same irrigation solution.” b. “Sitting on the toilet lets the irrigation sleeve eliminate into the bowl.” c. “I should never attempt to reach into my stoma to remove fecal material.”
Canadian Fundamentals of Nursing 6th Edition Potter Test Bank d. “Using warm tap water will reduce cramping and discomfort during the
procedure.” ANS: A
Enema applicators should never be used in the stoma because they can cause damage. A special coned irrigation device is used for ostomies. Irrigating a stoma into the toilet is an effective management technique. Fingers and other objects should not be placed into the stoma because they may cause trauma to the intestinal wall. Warm tap water will reduce cramping during irrigation. DIF: Apply REF: 1237| 1238 OBJ: Discuss the nursing measures required for patients with a bowel diversion. TOP: Implementation MSC: NCLEX: Physiological Integrity 23. A patient receives a diagnosis of bowel obstruction. The nurse chooses which type of tube for
gastric decompression? a. Salem sump. b. Dobhoff. c. Sengstaken-Blakemore. d. Small bore. ANS: A
A bowel obstruction causes a backup into the gastric area; a nasogastric tube may be inserted to decompress secretions and gases from the gastrointestinal tract. The Salem sump has the width and functionality needed to both feed and suction, and it is ideal for a bowel obstruction. A Dobhoff tube is used for instillation of feedings. A Sengstaken-Blakemore tube is used to compress stomach contents to prevent hemorrhage. A small bore is intended only for nutritional feedings and dN oeUsR nS otIhN avG eT suBc. tioCnOcM apacity. DIF: Apply REF: 1232 OBJ: Use critical thinking in the provision of care to patients with alterations in bowel elimination. TOP: Implementation MSC: NCLEX: Physiological Integrity 24. A patient had an ileostomy surgically placed 2 days ago. Which diet would the nurse
recommend to the patient to ease the transition to use of the new ostomy? a. Eggs over easy, whole wheat toast, and orange juice with pulp. b. Chicken fried rice with stir fried vegetables and iced tea. c. Turkey meatloaf with white rice and apple juice. d. Fish sticks with macaroni and cheese and soda. ANS: C
During the first week or so after ostomy placement, the patient should consume easy-to-digest, low-fibre foods such as poultry, rice and noodles, and strained fruit juices. Fried foods can irritate digestion and can cause blockage. Foods high in fibre will be useful later in the recovery process but can cause blockage if the GI tract is not accustomed to digesting with an ileostomy. DIF: Apply REF: 1239 OBJ: Discuss the nursing measures required for patients with a bowel diversion. TOP: Implementation MSC: NCLEX: Physiological Integrity 25. The nurse knows that the ideal time to change an ostomy pouch is when?
Canadian Fundamentals of Nursing 6th Edition Potter Test Bank a. b. c. d.
Before the patient eats a meal, when the patient is comfortable. When the patient feels that he or she needs to have a bowel movement. When ordered in the patient’s chart. After the patient has ambulated the length of the hallway.
ANS: A
The nurse wants to change the ostomy appliance when as little output as necessary ensures a smooth procedure. Patients with ostomies do not feel the urge to defecate because the sensory portion of the anus is not stimulated. The decision to change the ostomy pouch involves nursing judgement. After a patient ambulates, stool output is increased. DIF: Apply REF: 1240 (Skill 44-2) OBJ: Discuss the nursing measures required for patients with a bowel diversion. TOP: Assessment MSC: NCLEX: Physiological Integrity 26. The nurse administers a cathartic to a patient. The nurse determines that the cathartic has had
a therapeutic effect when what happens? a. The patient has a decreased level of anxiety. b. The patient experiences pain relief. c. The patient has a bowel movement. d. The patient passes flatulence. ANS: C
A cathartic is a laxative that stimulates a bowel movement. It would be considered effective if the patient experiences a bowel movement. The other options are not outcomes of administration of a cathartic. DIF: Apply REF: 12N 30 R I G B.C M U S N T O OBJ: Use critical thinking in the provision of care to patients with alterations in bowel elimination. TOP: Planning MSC: NCLEX: Physiological Integrity 27. An older person’s perineal skin appears to be dry and thin with mild excoriation. When
providing hygiene after a bowel movement, what should the nurse do? a. Thoroughly scrub the skin with a washcloth and hypoallergenic soap. b. Apply a skin protective lotion after perineal care. c. Tape an occlusive moisture barrier pad to the patient’s skin. d. Massage the skin with deep kneading pressure. ANS: B
For proper skin care and perineal cleaning, the nurse gently cleans the skin and applies a moistening barrier cream. Tape and occlusive dressings can damage skin. Excessive pressure and force are inappropriate and may cause skin breakdown. DIF: Apply REF: 1230 OBJ: List the nursing measures that promote normal elimination. TOP: Planning MSC: NCLEX: Physiological Integrity 28. The patient is seen in the gastroenterology clinic after having experienced changes in his
bowel elimination. A colonoscopy is ordered, and the patient has questions about the examination. What information should the nurse give the patient before the colonoscopy? a. No special preparation is required. b. Light sedation is normally used.
Canadian Fundamentals of Nursing 6th Edition Potter Test Bank c. No metallic objects are allowed. d. Swallowing of an opaque liquid is required. ANS: B
Light sedation is required for a colonoscopy, as is special preparation. Clear liquids are given the day before, to be followed by some form of bowel cleanser. Enemas until only clear fluid is passed also may be ordered. No restriction of metallic objects is made for a colonoscopy. A colonoscopy does not require the swallowing of an opaque liquid. DIF: Apply REF: 1223 (Box 44-3) OBJ: Describe the nursing implications for common diagnostic examinations of the gastrointestinal tract. TOP: Implementation MSC: NCLEX: Physiological Integrity 29. A nurse is providing discharge teaching for a patient who is going home with a guaiac test.
Which statement by the patient indicates the need for further education? a. “If I get a positive result, I have gastrointestinal bleeding.” b. “I should not eat red meat before my examination.” c. “I should schedule to perform the examination when I am not menstruating.” d. “I will need to perform this test three times if I have a positive result.” ANS: A
A positive result does not mean GI bleeding; it could be a false-positive finding caused by consuming red meat, some raw vegetables, or nonsteroidal anti-inflammatory drugs. Proper patient education is important for viable results. The patient needs to avoid certain foods to rule out a false-positive finding. If the test result is positive, the patient will need to repeat the test at least three times. Menses and hemorrhoids can also lead to false-positive results. DIF: Apply REF: 12N 21| 1222 UR SINGTB.COM OBJ: Describe nursing implications for common diagnostic examinations of the gastrointestinal tract. TOP: Evaluate MSC: NCLEX: Physiological Integrity 30. A nurse is caring for an older patient with fecal incontinence caused by cathartic use. The
nurse is most concerned about which complication that carries the greatest risk for severe injury? a. Rectal skin breakdown. b. Contamination of existing wounds. c. Falls from attempts to reach the bathroom. d. Cross-contamination into the upper GI tract. ANS: C
The nurse is most concerned about the worst injury the patient could receive, which involves falling while attempting to get to the bathroom. To reduce injury, the nurse should clear the path to the bathroom and reinforce use of the call light. The question concerns the greatest risk of injury, not the most frequent occurrence or the event most likely to occur. DIF: Apply REF: 1230 OBJ: Use critical thinking in the provision of care to patients with alterations in bowel elimination. TOP: Planning MSC: NCLEX: Safe and Effective Care Environment 31. The nurse is caring for a patient with Clostridium difficile infection. Which of the following
nursing actions is most efficacious in preventing the spread of bacteria? a. Monthly in-service education about contact precautions.
Canadian Fundamentals of Nursing 6th Edition Potter Test Bank b. Placing all contaminated items in biohazard bags. c. Mandatory cultures for all patients. d. Proper hand hygiene techniques. ANS: D
Proper hand hygiene is the best way to prevent the spread of bacteria. Monthly in-service education places emphasis on education, not on action. Biohazard bags are appropriate but cannot be used on every item that C. difficile comes in contact with, such as a human. Mandatory cultures are expensive and unnecessary and would not prevent the spread of bacteria. DIF: Apply REF: 1219 OBJ: Discuss the psychological and physiological factors that influence the elimination process. TOP: Planning MSC: NCLEX: Physiological Integrity 32. A nurse is caring for a patient who has had diarrhea for the past week. Which additional
assessment finding would the nurse expect? a. Increased energy levels. b. Distended abdomen. c. Decreased serum bicarbonate level. d. Increased blood pressure. ANS: C
Chronic diarrhea can result in metabolic acidosis, which is diagnostic of low serum bicarbonate. Patients with chronic diarrhea are dehydrated and have decreased blood pressure. Diarrhea also causes loss of electrolytes, nutrients, and fluid, which decreases energy levels. A distended abdomen would indicate constipation. DIF: Apply TOP: Planning
NURSINGTB.COM
REF: 1226 OBJ: Assess a patient’s pattern of elimination. MSC: NCLEX: Physiological Integrity
33. The nurse is caring for a patient who had a colostomy placed yesterday. The nurse should
report which assessment finding immediately? a. Stoma is protruding from the abdomen. b. Stoma is moist. c. Stool is discharging from the stoma. d. Stoma is purple. ANS: D
A purple stoma may indicate strangulation or poor circulation to the stoma and may require surgical intervention. A stoma should be reddish-pink and moist in appearance. It can be flush with the skin, or it can protrude. Stool is an expected outcome of stoma placement. DIF: Apply REF: 1242 (Skill 44-2) OBJ: Discuss the nursing measures required for patients with a bowel diversion. TOP: Assessment MSC: NCLEX: Physiological Integrity 34. A patient has constipation and hypernatremia. The nurse prepares to administer which type of
enema? a. Oil retention. b. Carminative. c. Saline.
Canadian Fundamentals of Nursing 6th Edition Potter Test Bank d. Tap water. ANS: D
Tap water enema would draw fluid into the system, inasmuch as it is hypotonic and would help flush out excess sodium. Oil retention would not address sodium problems. Carminative enemas are used to provide relief from distension caused by gas. A saline enema would worsen hypernatremia. DIF: Apply REF: 1232 OBJ: List the nursing measures that promote normal elimination. TOP: Implementation MSC: NCLEX: Physiological Integrity 35. A guaiac test has been ordered. The nurse knows that this is a test for which of the following? a. Bright red blood. b. Dark black blood. c. Blood that contains mucus. d. Blood that cannot be seen. ANS: D
Fecal occult blood tests are used to test for blood that may be present in stool that cannot be seen by the naked eye. The presence of such blood is usually indicative of GI bleeding. DIF: Understand REF: 1221| 1222 OBJ: Describe the nursing implications for common diagnostic examinations of the gastrointestinal tract. TOP: Assessment MSC: NCLEX: Physiological Integrity 36. The nurse should place the patient in which position when preparing to administer an enema? a. Left Sims’s position. NURSINGTB.COM b. Fowler’s. c. Supine. d. Semi-Fowler’s. ANS: A
Side-lying Sims’s position allows the enema solution to flow downward by gravity along the natural curve of the sigmoid colon. This helps to improve retention of the enema. Administering an enema in a sitting position may allow the curved rectal tube to scrape the rectal wall. DIF: Remember REF: 1233 (Skill 44-1) OBJ: List the nursing measures that promote normal elimination. TOP: Implementation MSC: NCLEX: Physiological Integrity 37. The nurse is assessing a patient 2 hours after a colonoscopy. For this procedure, what focused
assessment will the nurse include? a. Bowel sounds. b. Presence of flatulence. c. Bowel movements. d. Nausea. ANS: A
Canadian Fundamentals of Nursing 6th Edition Potter Test Bank The nurse does want to hear the presence of bowel sounds; absence of bowel sounds may indicate a complication from the surgery. Bowel movements and flatulence are not expected in the hours after surgery. The nurse does want to hear the presence of bowel movements. Nausea is not a problem after colonoscopy. DIF: Apply REF: 1224 OBJ: Describe the nursing implications for common diagnostic examinations of the gastrointestinal tract. TOP: Assessment MSC: NCLEX: Physiological Integrity
NURSINGTB.COM
Canadian Fundamentals of Nursing 6th Edition Potter Test Bank
Chapter 45: Mobility and Immobility Potter et al: Canadian Fundamentals of Nursing, 6th Edition MULTIPLE CHOICE 1. A patient has been on prolonged bed rest, and the nurse is observing for signs associated with
immobility. While assessing the patient, the nurse is alert to which of the following signs? a. Increased blood pressure. b. Decreased heart rate. c. Increased urinary output. d. Decreased peristalsis. ANS: D
Immobility disrupts the normal functioning of the gastrointestinal system, which results in decreased appetite and slowed peristalsis. In the immobilized patient, circulating fluid volume decreases, blood pools in the lower extremities, and autonomic response decreases. These factors result in decreased venous return, followed by a decrease in cardiac output, which is reflected by a decline in blood pressure. Recumbency increases cardiac workload and results in an increased pulse rate. Fluid intake can diminish with immobility, and this, combined with other causes, such as fever, increases the risk of dehydration. Urinary output may decline on or about the fifth or sixth day after immobilization, and the urine is often highly concentrated. DIF: Remember REF: 1249 OBJ: Identify changes in physiological and psychosocial function associated with immobility. TOP: Assessment MSC: NCLEX: Physiological Integrity 2. A 61-year-old patient recentlN yU haRdSleI ftN -sG idT edBp.aC raO lyM sis from a cerebrovascular accident (CVA;
also known as stroke). In planning care for this patient, the nurse would do which of the following? a. Encourage an even gait when walking in place. b. Assess the extremities for unilateral swelling and muscle atrophy. c. Encourage holding the breath frequently to hyperinflate the lungs. d. Teach the use of a two-point crutch technique for ambulation. ANS: B
Because edema moves to dependent body regions, assessment of the immobilized patient should include observation of the sacrum, legs, and feet. Unilateral increases in calf diameter can be an early indication of thrombosis. The patient who has suffered a CVA with left-sided paralysis may not be capable of an even gait. Having the patient hold his or her breath frequently is not an appropriate nursing intervention. To prevent stasis of pulmonary secretions, the patient’s position should be changed every 2 hours, and fluid intake should be increased to 2000 mL, if not contraindicated. The patient should deep breathe and cough every 1 to 2 hours to promote chest expansion. Two-point crutch technique would not be appropriate for the patient with left-sided paralysis. The patient would be more likely to ambulate safely with a walker or a cane. If crutches are used, the patient should use a three-point support. DIF: Analyze REF: 1272 (Skill 45-1) OBJ: Assess for correct and impaired body alignment and mobility. TOP: Implementation MSC: NCLEX: Physiological Integrity
Canadian Fundamentals of Nursing 6th Edition Potter Test Bank
3. When a patient with impaired physical mobility is in the recumbent position, what angle of
lateral position is recommended? a. 15 degrees. b. 30 degrees. c. 45 degrees. d. 90 degrees. ANS: B
When a patient with impaired physical mobility is in the recumbent position, the 30-degree lateral position reduces pressure from the sacral area and reduces the risk of skin breakdown. For a patient in the recumbent position with impaired physical mobility, the correct angle for a patient in the recumbent position with impaired physical mobility is not 15, 45, or 90 degrees. DIF: Apply REF: 1265 (Box 45-7)| 1272 OBJ: Assess for correct and impaired body alignment and mobility. TOP: Assessment MSC: NCLEX: Physiological Integrity 4. The patient has sequential compression stockings in place. The nurse evaluates that the
stockings have been implemented appropriately by the new staff nurse when the nurse observes what? a. Intermittent pressure is set at 40 mm Hg. b. Initial measurement is made around the patient’s calves. c. Stockings are wrapped directly over the leg from ankle to knee. d. Stockings are removed every hour during application. ANS: A
Inflation pressure averages 4N 0m mS HIg.NIG nitia urement is made around the largest part of R Bl.mCeasOM U T the patient’s thigh. A protective stockinette is placed over the patient’s leg. Then the stocking is wrapped around the leg, starting at the ankle, with the opening over the patella. Stockings are not removed every hour. For optimal results, sequential compression or intermittent pneumatic compression devices are used as soon as possible and maintained until the patient becomes fully ambulatory. The stockings should be removed periodically to assess the condition of the patient’s skin. DIF: Apply REF: 1268 OBJ: Develop individualized nursing care plans for patients with impaired mobility. TOP: Evaluate MSC: NCLEX: Physiological Integrity 5. The patient with torticollis would exhibit a. Exaggeration of the lumbar spine curvature. b. Increased convexity of the thoracic spine. c. Abnormal anteroposterior and lateral curvature of the spine. d. Contracture of the sternocleidomastoid muscle with a head incline. ANS: D
Torticollis is inclining of the head to the affected side, in which the sternocleidomastoid muscle is contracted. Lordosis is an exaggeration of the lumbar spine curvature. Kyphosis is an increased convexity in the curvature of the thoracic spine. Kyphoscoliosis is an abnormal anteroposterior and lateral curvature of the spine. DIF: Understand
REF: 1248 (Table 45-1)
Canadian Fundamentals of Nursing 6th Edition Potter Test Bank OBJ: Discuss physiological and pathological influences on body alignment and joint mobility. TOP: Assessment MSC: NCLEX: Physiological Integrity 6. The nurse expects to maintain the patient’s legs in abduction after total hip replacement
surgery with the use of which of the following? a. Foot boot. b. Wedge pillow. c. Trochanter roll. d. Sandbag. ANS: B
A wedge pillow is a triangular pillow made of heavy foam used to maintain the legs in abduction after total hip replacement surgery. A foot boot maintains feet in dorsiflexion. A trochanter roll prevents external rotation of the hips. Sandbags can be used to shape body contours and immobilize an extremity. DIF: Apply REF: 1271 OBJ: Develop individualized nursing care plans for patients with impaired mobility. TOP: Implementation MSC: NCLEX: Physiological Integrity 7. The patient is getting up for the first time after a period of bed rest. What is the initial nursing
action? a. Assess respiratory function. b. Obtain a baseline blood pressure. c. Assist the patient to sit at the edge of the bed. d. Ask the patient if he or she feels lightheaded. ANS: B
NURSINGTB.COM
When getting a patient up for the first time after a period of bed rest, the nurse should document orthostatic changes. The nurse first obtains a baseline blood pressure. Assessing the patient’s respiratory function is not the nurse’s first intervention when getting a patient up for the first time after prolonged bed rest. After the nurse assesses the patient’s blood pressure, the nurse can assist the patient to a sitting position at the side of the bed. After the patient is in the sitting position at the side of the bed, the nurse should ask the patient if he or she feels lightheaded. DIF: Apply REF: 1261| 1272 (Skill 45-1) OBJ: Develop individualized nursing care plans for patients with impaired mobility. TOP: Implementation MSC: NCLEX: Physiological Integrity 8. Immobilized patients frequently have hypercalcemia, which increases their risk for what? a. Osteoporosis. b. Renal calculi. c. Pressure ulcers. d. Thrombus formation. ANS: B
Canadian Fundamentals of Nursing 6th Edition Potter Test Bank Renal calculi are calcium stones that lodge in the renal pelvis or pass through the ureters. Immobilized patients are at risk for calculi because they frequently have hypercalcemia. Osteoporosis is caused by accelerated bone loss. A pressure ulcer is an impairment of the skin that results from prolonged ischemia (decreased blood supply) within tissues. A thrombus is an accumulation of platelets, fibrin, clotting factors, and cellular elements of the blood attached to the interior wall of a vein or artery, which sometimes occludes the lumen of the vessel. DIF: Remember REF: 1252 OBJ: Identify changes in physiological and psychosocial function associated with immobility. TOP: Assessment MSC: NCLEX: Physiological Integrity 9. Patients on bed rest or otherwise immobile are at risk for what condition? a. Increased metabolic rate. b. Increased diarrhea (peristalsis). c. Altered metabolic function. d. Increased appetite. ANS: C
Immobility disrupts normal metabolic functioning: decreasing the metabolic rate; altering the metabolism of carbohydrates, fats, and proteins (nutritional function); causing fluid, electrolyte, and calcium imbalances; and causing gastrointestinal disturbances such as decreased appetite and slowing of peristalsis. DIF: Remember REF: 1250 OBJ: Identify changes in physiological and psychosocial function associated with immobility. TOP: Assessment MSC: NCLEX: Physiological Integrity
NURSINGTB.COM
10. In caring for a patient who is immobile, what is important for the nurse to understand? a. The effects of immobility are the same for everyone. b. Immobility helps maintain sleep-wake patterns. c. Changes in role and self-concept may lead to depression. d. Immobile patients are often eager to help in their own care. ANS: C
Many immobilized patients become depressed because of changes in role and self-concept. Every patient responds to immobility differently. Immobility or bed rest frequently affects coping and creates sleep-wake alterations because of changes in routine or in the environment. Because immobilization removes the patient from a daily routine, he or she has more time to worry about disability. Worrying quickly increases the patient’s depression, causing withdrawal. Many withdrawn patients do not want to participate in their own care. DIF: Understand REF: 1252 OBJ: Identify changes in physiological and psychosocial function associated with immobility. TOP: Assessment MSC: NCLEX: Psychosocial Integrity 11. Immobility is a major risk factor for pressure ulcers. In caring for the patient who is
immobilized, the nurse needs to be aware of which of the following? a. Breaks in skin integrity are easy to heal. b. Preventing a pressure ulcer is more expensive than treating one. c. A 30-degree lateral position is recommended
Canadian Fundamentals of Nursing 6th Edition Potter Test Bank d. Pressure ulcers are caused by a sudden influx of oxygen to the tissue. ANS: C
Immobility is a major risk factor for pressure ulcers. Any break in the integrity of the skin is difficult to heal. Preventing a pressure ulcer is much less expensive than treating one; therefore, preventive nursing interventions are imperative. A 30-degree lateral position is recommended for patients at risk for pressure ulcers. Tissue metabolism depends on the supply of oxygen and nutrients to and the elimination of metabolic wastes from the blood. Pressure affects cellular metabolism by decreasing or totally eliminating tissue circulation. DIF: Understand REF: 1265 (Box 45-7)| 1272 OBJ: Identify changes in physiological and psychosocial function associated with immobility. TOP: Assessment MSC: NCLEX: Physiological Integrity 12. The nurse is caring for a patient who has suffered a stroke. As part of her ongoing care, what
should the nurse do? a. Encourage the patient to perform as many self-care activities as possible. b. Provide a complete bed bath to promote patient comfort. c. Place the patient on bed rest to prevent fatigue. d. Understand that the patient will not eat because energy needs are decreased. ANS: A
Nurses should encourage the patient to perform as many self-care activities as possible, thereby maintaining the highest level of mobility. Sometimes nurses inadvertently contribute to a patient’s immobility by providing unnecessary help with activities such as bathing and transferring. Placing the patient on bed rest without sufficient ambulation leads to loss of mobility and functional decline, resulting in weakness, fatigue, and increased risk for falls. Anorexia and insufficient assN isU taR ncSeI wN itG hT eaB tin.gCleOaM d to malnutrition. DIF: Apply REF: 1247| 1253 OBJ: Identify changes in physiological and psychosocial function associated with immobility. TOP: Implementation MSC: NCLEX: Health Promotion and Maintenance 13. The nurse is assessing the way the patient walks. The manner of walking is known as what? a. Activity tolerance. b. Body alignment. c. Range of motion. d. Gait. ANS: D
The term gait describes a particular manner or style of walking. Activity tolerance is the type and amount of exercise or work that a person is able to perform. Body alignment refers to the position of the joints, tendons, ligaments, and muscles while the person is standing, sitting, and lying. Range of motion is the maximum amount of movement available at a joint in one of the three planes of the body: sagittal, frontal, or transverse. DIF: Remember REF: 1254-1259 OBJ: Assess for correct and impaired body alignment and mobility. TOP: Assessment MSC: NCLEX: Physiological Integrity 14. When assessing the body alignment of a patient while he or she is standing, the nurse is aware
of which of the following?
Canadian Fundamentals of Nursing 6th Edition Potter Test Bank a. b. c. d.
When observed posteriorly, the hips and shoulders form an S pattern. When observed laterally, the spinal curves align in a reversed S pattern. The arms should be crossed over the chest or in the lap. The feet should be close together with toes pointed out.
ANS: B
When the patient is observed laterally, the head is erect and the spinal curves are aligned in a reversed S pattern. When observed posteriorly, the shoulders and hips are straight and parallel. The arms hang comfortably at the sides. The feet are slightly apart to achieve a base of support, and the toes are pointed forward. DIF: Understand REF: 1259 OBJ: Assess for correct and impaired body alignment and mobility. TOP: Assessment MSC: NCLEX: Physiological Integrity 15. The nurse is evaluating the body alignment of a patient in the sitting position. In this position,
how is the body aligned? a. The body weight is directly on the buttocks only. b. Both feet are supported on the floor with ankles flexed. c. The edge of the seat is in contact with the popliteal space. d. The arms hang comfortably at the sides. ANS: B
Both feet are supported on the floor, and the ankles are comfortably flexed. With patients of short stature, a footstool is used to ensure that the ankles are comfortably flexed. Body weight is evenly distributed on the buttocks and thighs. A 2.5- to 5.0-cm (1- to 2-inch) space is maintained between the edge of the seat and the popliteal space on the posterior surface of the knee to ensure that no pressuN reUisRpla SIced NGonTBth.e Cpop OMliteal artery or nerve. The patient’s forearms are supported on the armrest, in the lap, or on a table in front of the chair. DIF: Understand REF: 1259| 1260 OBJ: Assess for correct and impaired body alignment and mobility. TOP: Assessment MSC: NCLEX: Physiological Integrity 16. The nurse is assessing body alignment for a patient who is immobilized. What must the nurse
do? a. b. c. d.
Place the patient in the supine position. Remove the pillow from under the patient’s head. Insert positioning supports to help the patient. Place the patient in a lateral position.
ANS: D
For a patient who is immobilized or bedridden, body alignment is assessed with the patient in the lateral position, not supine. The nurse should remove all positioning support from the bed, except for the pillow under the patient’s head. DIF: Apply REF: 1260 OBJ: Assess for correct and impaired body alignment and mobility. TOP: Implementation MSC: NCLEX: Physiological Integrity 17. The nurse must assess the patient for hazards of immobility by performing a head-to-toe
physical assessment. When the respiratory system is assessed, what should the nurse do?
Canadian Fundamentals of Nursing 6th Edition Potter Test Bank a. b. c. d.
Assess the patient at least every 4 hours. Inspect chest wall movements during the expiratory cycle only. Auscultate the entire lung region to assess lung sounds. Focus auscultation on the upper lung fields.
ANS: C
The nurse auscultates the entire lung region to identify diminished breath sounds, crackles, or wheezes. A respiratory assessment is performed at least every 2 hours for patients with restricted activity. The nurse inspects chest wall movements during the full inspiratory-expiratory cycle. Auscultation is focused on the dependent lung fields because pulmonary secretions tend to collect in these lower regions. DIF: Apply REF: 1260 OBJ: Assess for correct and impaired body alignment and mobility. TOP: Implementation MSC: NCLEX: Physiological Integrity 18. The nurse is aware that patients who are immobile are at increased risk of developing deep
vein thromboses (DVTs). Because of this, what action does the nurse take? a. Make sure that elastic stockings are not removed. b. Measure the calf circumference of both legs. c. Dorsiflex the foot while assessing for patient discomfort. d. Measure both ankles to determine size. ANS: B
The nurse measures bilateral calf circumference and records it daily as an assessment for deep vein thrombosis (DVT). Homans’s sign, or calf pain on dorsiflexion of the foot, should not be assessed in patients when a DVT is suspected; it is no longer considered a reliable indicator in assessing for DVT, and it is pNres entSI inN oG ther onCditi UR TBc. OMons. The patient’s elastic stockings or sequential compression devices, or both, should be removed every 8 hours and the calves observed for redness, warmth, and tenderness. Bilateral calf (not ankle) circumferences should be measured daily to detect unilateral increases that may be an early indication of thrombosis. DIF: Apply REF: 1261 OBJ: Assess for correct and impaired body alignment and mobility. TOP: Implementation MSC: NCLEX: Physiological Integrity 19. A patient is admitted to the medical unit after a CVA. There is evidence of left-sided
hemiparesis, and the nurse will be following up on range-of-motion (ROM) and other exercises performed in physiotherapy. Which of the following principles of ROM exercises does the nurse correctly teach the patient and family members? a. Flex the joint to the point of discomfort. b. Work from proximal to distal joints. c. Move the joints quickly. d. Provide support to the extremity. ANS: D
While the patient is performing ROM exercises, support should be provided to the extremity. The joint should be flexed to the point of resistance, not to the point of discomfort. When performing ROM exercises, the patient should begin at distal joints and work toward proximal joints. Joints should be moved slowly through the ROM. Quick movement could cause injury. DIF: Apply
REF: 1282-1284
Canadian Fundamentals of Nursing 6th Edition Potter Test Bank OBJ: Describe essential techniques when assisting with active and passive range-of-motion (ROM) exercises, assisting a patient to move up in bed, repositioning a patient, assisting a patient to a sitting position, and transferring a patient from a bed to a chair or from a bed to a stretcher. TOP: Implementation MSC: NCLEX: Health Promotion and Maintenance 20. The nurse is caring for an older patient with the diagnosis of urinary tract infection (UTI). The
patient is confused and agitated. It is important for the nurse to realize that confusion in older people is which of the following? a. Not a normal expectation. b. Purely psychological in origin. c. Not a common manifestation with UTIs. d. Acceptable and needs no further assessment. ANS: A
Acute confusion in older persons is not normal; a thorough nursing assessment is the priority. Abrupt changes in personality often have a physiological cause such as surgery, a medication reaction, a pulmonary embolus, or an acute infection. For example, the primary symptom of compromised older patients with an acute urinary tract infection or fever is confusion. Identifying confusion is an important component of the nurse’s assessment. DIF: Understand REF: 1262 OBJ: Identify changes in physiological and psychosocial function associated with immobility. TOP: Assessment MSC: NCLEX: Psychosocial Integrity 21. In preparing to create a nursing diagnosis for a patient who is immobile, what is important for
the nurse to understand? a. Physiological issues should be the major focus. b. Psychosocial issues shouN ldUbR eS thI em ajoTrBfo.cC usO .M NG c. Developmental issues should be the major focus. d. All dimensions are important to health. ANS: D
Often the physiological dimension is the major focus of nursing care for patients with impaired mobility. Thus the psychosocial and developmental dimensions are neglected. However, all dimensions are important in health. DIF: Understand REF: 1263 OBJ: Formulate appropriate nursing Diagnosis for impaired mobility. TOP: Assessment MSC: NCLEX: Health Promotion and Maintenance 22. Many patients who experience an alteration in mobility have one or more nursing diagnoses.
For whom would the nurse would use the diagnosis of Impaired physical mobility? a. A patient who is not completely immobile. b. A patient who is completely immobile. c. A patient who is at risk for multisystem problems. d. A patient who is at risk for single-system involvement. ANS: A
Canadian Fundamentals of Nursing 6th Edition Potter Test Bank The diagnosis of Impaired physical mobility applies to the patient who has some limitation but is not completely immobile. The diagnosis of Risk for disuse syndrome applies to the patient who is immobile and at risk for multisystem problems because of inactivity. Beyond these diagnoses, the list of potential diagnoses is extensive because immobility affects multiple body systems. DIF: Apply REF: 1263 OBJ: Formulate appropriate nursing Diagnosis for impaired mobility. TOP: Implementation MSC: NCLEX: Health Promotion and Maintenance 23. The patient has the nursing diagnosis of Impaired physical mobility related to pain in the left
shoulder. This diagnosis means that the nurse should do what? a. Encourage the patient to perform self-care. b. Keep the patient as mobile as possible. c. Encourage the patient to perform ROM exercises. d. Assist the patient with comfort measures. ANS: D
The diagnosis related to pain requires the nurse to assist the patient with comfort measures so that the patient is then willing and better able to move. Pain must be controlled before the patient will be willing to initiate movement. The diagnosis of Reluctance to initiate movement necessitates interventions aimed at keeping the patient as mobile as possible and encouraging the patient to perform self-care and ROM exercises. This cannot be accomplished until comfort is achieved. DIF: Apply REF: 1263 OBJ: Formulate appropriate nursing Diagnosis for impaired mobility. NURSINGMTSBC. TOP: Implementation : C NC OLMEX: Health Promotion and Maintenance 24. In developing an individualized plan of care for a patient, what is important for the nurse to
do? a. b. c. d.
Set goals that are a little beyond the capabilities of the patient. Use his or her judgement and not be swayed by family desires. Establish goals that are measurable and realistic. Explain that without taking alignment risks, there can be no progress.
ANS: C
The nurse must develop an individualized plan of care for each nursing diagnosis and must set goals that are individualized, realistic, and measurable. The nurse should set realistic expectations for care and should include the patient and family when possible. The goals focus on preventing problems or risks to body alignment and mobility. DIF: Apply REF: 1264 OBJ: Develop individualized nursing care plans for patients with impaired mobility. TOP: Implementation MSC: NCLEX: Health Promotion and Maintenance 25. When the nurse creates a plan of care for a patient who is experiencing alterations in mobility,
which of the following is true? a. The nurse cannot delegate interventions to nursing assistive personnel. b. The nurse is solely responsible for modifying activities of daily living (ADLs). c. The nurse consults other health care team members to help plan therapy.
Canadian Fundamentals of Nursing 6th Edition Potter Test Bank d. The nurse consults wound care specialists only when wounds are apparent. ANS: C
The nurse should collaborate with other health care team members such as physiotherapists or occupational therapists when considering mobility needs. Nurses often delegate some interventions to unregulated care providers. Unregulated care providers may perform such tasks as turning and positioning patients, applying elastic stockings, and helping patient use the incentive spirometer. Occupational therapists are a resource for planning ADLs that patients need to modify or relearn. It is especially important in priority setting to account for potential complications. Many times, actual problems such as pressure ulcers are addressed only after they develop. They should be addressed before they develop. DIF: Apply REF: 1282 OBJ: Develop individualized nursing care plans for patients with impaired mobility. TOP: Implementation MSC: NCLEX: Safe and Effective Care Environment 26. The patient is being admitted to the neurological unit with the diagnosis of stroke. When
should the nurse should begin discharge planning? a. At the time of admission. b. The day before the patient is to be discharged. c. As soon as the patient’s discharge destination is known. d. When outpatient therapy will no longer be needed. ANS: A
Discharge planning begins when a patient enters the health care system. In anticipation of the patient’s discharge from an institution, the nurse makes appropriate referrals or consults a case manager or a discharge planner to ensure that the patient’s needs will be met at home. Referrals to home care or outN paUtiR enSt I thN erG apTyBa. reCoO ftM en needed. DIF: Apply REF: 1264 OBJ: Develop individualized nursing care plans for patients with impaired mobility. TOP: Implementation MSC: NCLEX: Health Promotion and Maintenance 27. Of the following nursing goals, which is the most appropriate for a patient who has had a total
hip replacement? a. The patient will walk 305 m (1000 feet), using her walker, by the time of discharge. b. The patient will ambulate by the time of discharge. c. The patient will ambulate briskly on the treadmill by the time of discharge. d. The nurse will assist the patient to ambulate in the hall. ANS: A
A goal of walking 305 m (1000 feet) with the use of her walker by the time of discharge is individualized, realistic, and measurable. “The patient will ambulate by the time of discharge” is not measurable because it does not specify the distance. Even though the patient will ambulate, this does not quantify how far. “Ambulating briskly on a treadmill” is not realistic for this patient. The last option focuses on the nurse, not the patient, and is not measurable. DIF: Analyze REF: 1264 OBJ: Develop individualized nursing care plans for patients with impaired mobility. TOP: Assessment MSC: NCLEX: Health Promotion and Maintenance
Canadian Fundamentals of Nursing 6th Edition Potter Test Bank 28. Prevention of plantar flexion through the use of pillows to support the lower legs and elevate
the toes is a priority intervention for which patient? a. A 54-year-old with a diagnosis of osteoarthritis in all lower extremity joints. b. A 25-year-old with a fractured pelvis as a result of a motorcycle accident. c. A 78-year-old who has experienced left-sided paralysis caused by a CVA. d. A 15-year-old who has been comatose for 2 years as a result of a head injury sustained from a fall off a roof. ANS: C
The patient who has suffered a CVA with resulting left-sided paralysis (hemiplegia) is at risk for footdrop. Neither the 54-year-old patient with osteoarthritis in all lower extremity joints nor the 25-year-old patient with a fractured pelvis as a result of a motorcycle accident would have damage to the nerve necessary to cause footdrop. There is little chance that the 15-year-old patient who has been comatose for 2 years as a result of a head injury sustained from a fall off a roof will ever be capable of mobility. DIF: Analyze REF: 1251| 1252 OBJ: Develop individualized nursing care plans for patients with impaired mobility. TOP: Implementation MSC: NCLEX: Physiological Integrity 29. The patient is immobilized after undergoing hip replacement surgery. Which of the following
would place the patient at risk for hemorrhage? a. Thick, tenacious pulmonary secretions. b. Low-molecular-weight heparin doses to prevent DVT. c. Sequential compression devices wrapped around the legs to prevent DVT formation. d. Elastic stockings (thromboembolic disease [TED] hose) to promote venous return. ANS: B
N R I G B.C M U S N T O
Heparin and low-molecular-weight heparin are the most widely used drugs in the prophylaxis of DVT. Because bleeding is a potential side effect of these medications, the nurse must assess the patient continually for signs of bleeding. Pulmonary secretions that become thick and tenacious are difficult to remove and are a sign of inadequate hydration, but not of bleeding. Sequential compression devices consist of sleeves or stockings made of fabric or plastic that are wrapped around the leg and are secured with Velcro. They decrease venous stasis by increasing venous return through the deep veins of the legs. They do not usually cause bleeding. Elastic stockings also aid in maintaining external pressure on the muscles of the lower extremities and in promoting venous return, and they also do not usually cause bleeding. DIF: Analyze REF: 1265 (Box 45-7)| 1269 OBJ: Identify changes in physiological and psychosocial function associated with immobility. TOP: Implementation MSC: NCLEX: Physiological Integrity 30. The nurse needs to transfer the patient from the bed to the chair. What should the nurse
remember? a. To avoid using a transfer or gait belt around the patient’s waist before transfer. b. Not to allow the patient to help in any way because resistance can lead to injury. c. To assess for the need of a mechanical lift and help. d. To ensure that the patient has stockings on the feet for transfer.
Canadian Fundamentals of Nursing 6th Edition Potter Test Bank ANS: C
Careful assessment of the patient’s ability to assist in the positioning technique to be used is extremely important. The use of a mechanical lift should be considered. The nurse’s role in assisting the patient to a sitting position is to guide and instruct. If the patient can bear weight and move to a sitting position independently, he or she should be allowed to do so, and the nurse may offer assistance. A transfer belt maintains stability of the patient during transfer and reduces risk for falls. The nurse must ensure that the patient has stable nonskid shoes on the feet. DIF: Apply REF: 1281 OBJ: Describe essential techniques when assisting with active and passive range-of-motion (ROM) exercises, assisting a patient to move up in bed, repositioning a patient, assisting a patient to a sitting position, and transferring a patient from a bed to a chair or from a bed to a stretcher. TOP: Implementation MSC: NCLEX: Safe and Effective Care Environment 31. The nurse is caring for a patient with a spinal cord injury and notices that the patient’s hips
have a tendency to rotate externally when the patient is supine. To help prevent injury secondary to this rotation, what can the nurse use? a. A trochanter roll. b. The trapeze bar. c. Hand rolls. d. Hand-wrist splints. ANS: A
A trochanter roll prevents external rotation of the hips when the patient is in a supine position. Hand rolls maintain the thumb in slight adduction and in opposition to the fingers. Hand-wrist splints are individually moulded for the patient to maintain proper alignment of the thumb and the wrist. The trapeze bar is aNtU riaRnS guIlaNrG deTvB ic. eC thO atMhangs down from a securely fastened overhead bar that is attached to the bed frame. It allows the patient to pull with the upper extremities to raise the trunk off the bed, to assist in transfer from bed to wheelchair, or to perform upper arm exercises. DIF: Remember REF: 1281 OBJ: Describe essential techniques when assisting with active and passive range-of-motion (ROM) exercises, assisting a patient to move up in bed, repositioning a patient, assisting a patient to a sitting position, and transferring a patient from a bed to a chair or from a bed to a stretcher. TOP: Implementation MSC: NCLEX: Physiological Integrity 32. The patient is unable to move himself and needs to be pulled up in bed. For this repositioning
to be done safely, what must the nurse understand? a. The procedure can be done by one person if the bed is in the flat position. b. Side rails should be in the up position to prevent the patient from falling out. c. The pillow should be placed under the patient’s head and shoulders. d. Assistive devices or additional nurses should be used. ANS: D
Canadian Fundamentals of Nursing 6th Edition Potter Test Bank This is not a one-person task. Helping a patient move up in bed without help from other coworkers or without the aid of an assistive device (e.g., friction-reducing pad) is not recommended and is not considered safe for the patient or the nurse. When a patient is pulled up in bed, the bed should be flat or in the Trendelenburg position (when tolerated) for gravity assistance, and the side rails should be down. The pillow should be removed from under the patient’s head and shoulders and placed at the head of the bed to keep the patient’s head from striking against the head of the bed. DIF: Understand REF: 1272-1280 (Skill 45-1) OBJ: Describe essential techniques when assisting with active and passive range-of-motion (ROM) exercises, assisting a patient to move up in bed, repositioning a patient, assisting a patient to a sitting position, and transferring a patient from a bed to a chair or from a bed to a stretcher. TOP: Implementation MSC: NCLEX: Safe and Effective Care Environment 33. The nurse is caring for a patient who is immobile and needs to be turned every 2 hours. The
patient has poor lower extremity circulation, and the nurse is concerned about irritation of the patient’s toes. What is one strategy that the nurse could use? a. A foot cradle. b. A trochanter roll. c. The trapeze bar. d. Hand rolls. ANS: A
A foot cradle may be used for patients with poor peripheral circulation as a means of reducing pressure on the tips of a patient’s toes. A trochanter roll prevents external rotation of the hips when the patient is in a supine position. Hand rolls maintain the thumb in slight adduction and in opposition to the fingers. The trapeze bar is a triangular device that hangs down from a securely fastened overhead bN arUthRaS t iI sN atG tacThB ed.tC oO thM e bed frame. It allows the patient to pull with the upper extremities to raise the trunk off the bed, to assist in transfer from bed to wheelchair, or to perform upper arm exercises. DIF: Apply REF: 1275 (Skill 45-1) OBJ: Describe essential techniques when assisting with active and passive range-of-motion (ROM) exercises, assisting a patient to move up in bed, repositioning a patient, assisting a patient to a sitting position, and transferring a patient from a bed to a chair or from a bed to a stretcher. TOP: Implementation MSC: NCLEX: Physiological Integrity 34. In applying for a job on a nursing unit that requires frequent patient positioning, of what
should the nurse should be aware? a. That nurses are at low risk for back injury. b. That nurses are especially at risk for upper back injuries. c. That nurses should be aware of agency policies. d. That nurses should not need to use assistive devices. ANS: C
Nurses and other health care staff are especially at risk for injury to lumbar muscles when lifting, transferring, or positioning immobilized patients. Therefore, the nurse should be aware of agency policies and protocols that protect staff and patients from injury. Current evidence supports that using mechanical or other ergonomic assistive devices is the safest way to reposition and lift patients who are unable to perform these activities themselves.
Canadian Fundamentals of Nursing 6th Edition Potter Test Bank DIF: Understand REF: 1247 OBJ: Describe essential techniques when assisting with active and passive range-of-motion (ROM) exercises, assisting a patient to move up in bed, repositioning a patient, assisting a patient to a sitting position, and transferring a patient from a bed to a chair or from a bed to a stretcher. TOP: Implementation MSC: NCLEX: Safe and Effective Care Environment 35. When the nurse is preparing a plan of care for an immobilized patient, what should the nurse
keep in mind? a. To use established expected outcomes to evaluate the patient’s response to care. b. To display an air of professional superiority when interventions are not successful. c. Never to vary from interventions that have been successful for other patients. d. To use only objective data in determining whether interventions have been successful. ANS: A
The nurse should use established expected outcomes to evaluate the patient’s response to care. The nurse should use creativity when designing new interventions to improve the patient’s mobility status and should display humility when identifying the interventions that were not successful. The nurse should ask whether the patient’s expectations of care are being met and use objective data to determine the success of interventions. DIF: Apply REF: 1295 (Figure 45-22) OBJ: Evaluate the nursing care plan for maintaining body alignment and mobility. TOP: Implementation MSC: NCLEX: Physiological Integrity 36. It has been determined that each of the following patients is at risk for falling. Which one
requires the nurse’s priority for ambulation? a. A 16-year-old with a spraN ine d an UR SIkleNGbein TBg.dCisch OMarged from the emergency department. b. A 54-year-old who has taken the initial dose of an antihypertensive medication. c. A 45-year-old postoperative patient up for the first time since knee surgery. d. An 81-year-old who is asthmatic and had a hip replaced 18 months ago. ANS: C
Disease, injury, pain, physical development (e.g., age), and life changes (e.g., pregnancy) compromise the ability to remain balanced. Medications that cause dizziness and prolonged immobility also affect balance. Although all of the options represent a potential risk for falling, the postoperative patient has both prolonged immobility and physical injury (surgery) and so is at greatest risk. The 16-year-old with a sprained ankle being discharged from the emergency department, the 54-year-old who has taken the initial dose of an antihypertensive medication, and the 81-year-old who is asthmatic and had a hip replaced 18 months ago do not receive priority for ambulation. DIF: Analyze REF: 1251 OBJ: Develop individualized nursing care plans for patients with impaired mobility. TOP: Implementation MSC: NCLEX: Physiological Integrity 37. The nurse needs to reposition a 136.1 kg (300-pound) patient. Which of the following
strategies is most likely to prevent back injury? a. Turn the patient alone using the lift pad and applying pillows. b. Put the bed in the Trendelenburg position and pull from the head of the bed.
Canadian Fundamentals of Nursing 6th Edition Potter Test Bank c. Assess and obtain the number of people needed to help. d. Bend at the waist and pull the lift pad, using the arms. ANS: C
The nurse must assess and determine the number of people needed; to prevent injury, the task should not be started until it can be completed safely. The nurse should assess the situation and not turn the patient alone if this cannot be done safely. The nurse’s trunk should be erect and the knees bent, so that multiple muscle groups (not just the arms) work together in a coordinated manner. This is not a one-person task: the nurse must not pull from the head of the bed. DIF: Analyze REF: 1273 (Skill 45-1) OBJ: Describe essential techniques when assisting with active and passive range-of-motion (ROM) exercises, assisting a patient to move up in bed, repositioning a patient, assisting a patient to a sitting position, and transferring a patient from a bed to a chair or from a bed to a stretcher. TOP: Implementation MSC: NCLEX: Safe and Effective Care Environment 38. The nurse is caring for a patient who has had a stroke that caused total paralysis of the right
side. To help maintain joint function and to prevent contractures, passive ROM exercises will be initiated. When should therapy begin? a. After the acute phase of the disease has passed. b. As soon as the ability to move is lost. c. Once the patient enters the rehabilitation unit. d. No ROM exercise is needed. ANS: B
Passive ROM exercises should begin as soon as the patient’s ability to move the extremity or joint is lost. The nurse shouldNnoR U t wa SIitNfoGrTthBe.acCute OMphase to end. It may be some time before the patient enters the rehabilitation unit, and contractures could form by then. ROM exercise is certainly needed by this patient. DIF: Understand REF: 1254| 1284 OBJ: Develop individualized nursing care plans for patients with impaired mobility. TOP: Implementation MSC: NCLEX: Physiological Integrity 39. The nurse is admitting a patient who has had a stroke. The physician writes orders for “ROM
as needed.” What does the nurse understand about this situation? a. The nurse will have to move all the patient’s extremities. b. The patient is unable to move his extremities. c. Further assessment of the patient is needed. d. The patient needs to restrict his mobility as much as possible. ANS: C
Further assessment of the patient is needed. Some patients are able to move some joints actively, whereas the nurse must help other joints with passive movement. With a weak patient, the nurse may have to support an extremity while the patient performs the movement. In general, exercises need to be as active as the patient’s health and mobility allow. DIF: Understand REF: 1253 (Figure 45-5) OBJ: Develop individualized nursing care plans for patients with impaired mobility. TOP: Implementation MSC: NCLEX: Health Promotion and Maintenance
Canadian Fundamentals of Nursing 6th Edition Potter Test Bank 40. While performing passive ROM exercises, the nurse stands at the side of the bed closest to the
joint being exercised and does what else? a. Forces the joint just a bit beyond the point of resistance. b. Moves the joint until the patient complains of pain. c. Repeats each movement twice. d. Carries out movements slowly and smoothly. ANS: D
The nurse carries out movements slowly and smoothly, just to the point of resistance. ROM exercises should not cause pain. A joint should never be forced beyond its capacity. Each movement needs to be repeated five times during the session. DIF: Apply REF: 1284 OBJ: Develop individualized nursing care plans for patients with impaired mobility. TOP: Implementation MSC: NCLEX: Physiological Integrity 41. Two nurses are standing on opposite sides of the bed to move the patient up in bed with a
drawsheet. Which of the following describes the correct position for the nurses in order to safely position the patient? a. The nurses should face the patient. b. The nurses should face the direction of movement. c. The nurses should face each other. d. The nurses should face opposite the direction of movement. ANS: B
Facing the direction of movement ensures proper balance and prevents twisting of the nurses’ bodies while moving the patient. Facing the patient, facing each other, and facing opposite the direction of movement are noNt U thR eS coIrrN ecGt T poBs. itiC onOsM for the nurses to take. DIF: Apply REF: 1274 (Skill 45-1) OBJ: Develop individualized nursing care plans for patients with impaired mobility. TOP: Implementation MSC: NCLEX: Physiological Integrity 42. The patient has suffered a spinal cord injury and needs to be repositioned through the
log-rolling technique to keep the spinal column in straight alignment. Which of the following is the proper technique for nurses to perform log-rolling? a. Obtaining assistance from at least two or three other people. b. Having the patient reach for the opposite side rail when turning. c. Moving the top part of the patient’s torso, then the bottom part. d. Not using pillows after turning because the softness causes misalignment. ANS: A
At least three to four people are needed to perform this skill safely. The patient crosses arms on the chest to prevent injury to the arms. The patient should be moved as one unit in a smooth, continuous motion on the count of three. The nurses gently let the patient as a unit lean back toward pillows for support. DIF: Apply REF: 1274| 1279 (Skill 45-1) OBJ: Describe essential techniques when assisting with active and passive range-of-motion (ROM) exercises, assisting a patient to move up in bed, repositioning a patient, assisting a patient to a sitting position, and transferring a patient from a bed to a chair or from a bed to a stretcher. TOP: Implementation MSC: NCLEX: Physiological Integrity
Canadian Fundamentals of Nursing 6th Edition Potter Test Bank
MULTIPLE RESPONSE 1. Correct body alignment reduces strain on musculoskeletal structures and contributes to
balance. Balance control is attained by which of the following? (Select all that apply.) a. Keeping the body’s centre of gravity high. b. Maintaining a wide base of support. c. Keeping the body’s centre of gravity low. d. Maintaining correct body posture. e. Maintaining immobility to prevent falls. ANS: B, C, D
Without balance control, the centre of gravity is displaced, thus creating risk for falls and injuries. Balance is enhanced by keeping the body’s centre of gravity low (not high) with a wide base of support and by maintaining correct body posture. Prolonged immobility leads to impaired balance. DIF: Understand REF: 1247 OBJ: Discuss physiological and pathological influences on body alignment and joint mobility. TOP: Assessment MSC: NCLEX: Physiological Integrity 2. The nurse is caring for a patient with the nursing diagnosis of Impaired physical mobility. The
nurse needs to be alert for which of the following potential complications? (Select all that apply.) a. Pulmonary emboli. b. Pneumonia. c. Impaired skin integrity. N R I G B.C M U S N T O d. Somnolence. e. Increased socialization. ANS: A, B, C
Immobility leads to complications such as pulmonary emboli or pneumonia. Other possible complications include impaired skin integrity. Insomnia and social isolation are more common than somnolence or increased socialization. DIF: Apply REF: 1263 OBJ: Formulate appropriate nursing Diagnosis for impaired mobility. TOP: Implementation MSC: NCLEX: Health Promotion and Maintenance 3. The nurse is caring for a patient who has had a recent stroke and is paralyzed on his left side.
He has no respiratory or cardiac issues, but he cannot walk. He becomes extremely frustrated when he cannot button his shirt and cannot feed himself because he was left-handed. He has shown no signs of dysphagia, but he has been eating very little and has lost 0.9 kg (2 pounds). He asks the nurse, “How can I go home like this? I’m not getting better. I can’t ask my wife to take care of me like a baby.” Of the following list of health care team members, which member would the nurse need to consult? (Select all that apply.) a. Physiotherapy. b. Occupational therapy. c. Respiratory therapy. d. Cardiac rehabilitation. e. Psychology services.
Canadian Fundamentals of Nursing 6th Edition Potter Test Bank
ANS: A, B, E
Physiotherapists are a resource for planning ROM or strengthening exercises, and occupational therapists are a resource for planning ADLs that patients need to modify or relearn. Referral to a mental health advanced practice nurse, a licensed social worker, or a physiologist to assist with coping or other psychosocial issues is also wise. Because the patient exhibits good cardiac and respiratory function, respiratory therapy and cardiac rehabilitation probably are not needed at this time. DIF: Apply REF: 1264 OBJ: Develop individualized nursing care plans for patients with impaired mobility. TOP: Implementation MSC: NCLEX: Safe and Effective Care Environment
NURSINGTB.COM
Canadian Fundamentals of Nursing 6th Edition Potter Test Bank
Chapter 46: Skin Integrity and Wound Care Potter et al: Canadian Fundamentals of Nursing, 6th Edition MULTIPLE CHOICE 1. The nurse is working on a medical-surgical unit that has been participating in a research
project associated with pressure injuries. The nurse recognizes that the risk factors that predispose a patient to pressure injury development include which of the following? a. A diet low in calories and fat. b. Alteration in level of consciousness. c. Shortness of breath. d. Muscular pain. ANS: B
Patients who are confused or disoriented or who have changing levels of consciousness are unable to protect themselves. The patient may feel the pressure but may not understand what to do to relieve the discomfort or to communicate that he or she is feeling discomfort. Impaired sensory perception, impaired mobility, shear, friction, and moisture are other predisposing factors. Shortness of breath, muscular pain, and a diet low in calories and fat are not among the predisposing factors. DIF: Remember REF: 1291| 1292 OBJ: Discuss the risk factors that contribute to pressure injury formation. TOP: Assessment MSC: NCLEX: Physiological Integrity 2. The nurse is caring for a patient who was involved in an automobile accident 2 weeks ago.
N R I G B.C M
The patient sustained a head inU jurySandNis T unconscOious. What is the major element involved in the development of a pressure injury? a. Pressure. b. Resistance. c. Stress. d. Weight. ANS: A
Pressure is the main element that causes pressure injury. Three pressure-related factors contribute to pressure injury development: pressure intensity, pressure duration, and tissue tolerance. When the intensity of the pressure exerted on the capillary exceeds 12 to 32 mm Hg, this occludes the vessel, causing ischemic injury to the tissues it normally feeds. High pressure over a short time and low pressure over a long time cause skin breakdown. Resistance (the ability to remain unaltered by the damaging effect of something), stress (worry or anxiety), and weight (individuals of all sizes, shapes, and ages acquire skin breakdown) are not major causes of pressure injury. DIF: Remember REF: 1290 OBJ: Discuss the risk factors that contribute to pressure injury formation. TOP: Assessment MSC: NCLEX: Physiological Integrity 3. Which nursing observation would indicate that the patient was at risk for pressure injury
formation? a. The patient ate two thirds of breakfast.
Canadian Fundamentals of Nursing 6th Edition Potter Test Bank b. The patient has fecal incontinence. c. The patient has a raised red rash on the right shin. d. The patient’s capillary refill is less than 2 seconds. ANS: B
The presence and duration of moisture on the skin increase the risk of pressure injury formation by making it susceptible to injury. Moisture can originate from wound drainage, excessive perspiration, and fecal or urinary incontinence. Bacteria and enzymes in the stool can enhance the opportunity for skin breakdown because the skin is moistened and softened, which causes maceration. Eating a balanced diet is important for nutrition, but eating just two thirds of the meal does not indicate that the individual is at risk. A raised red rash on the leg is a concern and can affect the integrity of the skin, but it is located on the shin, which is not a high-risk area for skin breakdown. Pressure can influence capillary refill, leading to skin breakdown, but this capillary response is within normal limits. DIF: Understand REF: 1292 OBJ: Discuss the risk factors that contribute to pressure injury formation. TOP: Implementation MSC: NCLEX: Physiological Integrity 4. The wound care nurse visits a patient in the long-term care unit. The nurse is monitoring a
patient with a stage 3 pressure injury. The wound seems to be healing, and healthy tissue is observed. How would the nurse stage this pressure injury? a. Stage 1 pressure injury. b. Healing stage 2 pressure injury. c. Healing stage 3 pressure injury. d. Stage 3 pressure injury. ANS: C
NURSINGTB.COM
When a pressure injury has been staged and is beginning to heal, the pressure injury keeps the same stage and is labelled with the words “healing stage.” Once a pressure injury has been staged, the stage endures even as the pressure injury heals. This pressure injury was labelled a stage 3; it cannot return to a previous stage such as stage 1 or 2. This pressure injury is healing, so it is no longer labelled just stage 3. DIF: Remember TOP: Assessment
REF: 1299 OBJ: Recognize the stages of pressure injury. MSC: NCLEX: Physiological Integrity
5. The nurse is admitting an older patient from a nursing home. During the assessment, the nurse
notes a shallow open pressure injury without slough on the right heel of the patient. How would this pressure injury be staged? a. Stage 1. b. Stage 2. c. Stage 3. d. Stage 4. ANS: B
Canadian Fundamentals of Nursing 6th Edition Potter Test Bank This would be a stage 2 pressure injury because it manifests as partial-thickness skin loss involving epidermis, dermis, or both. The pressure injury is superficial and appears clinically as an abrasion, blister, or shallow crater. In a stage 1 injury, skin is intact with nonblanchable redness over a bony prominence. With a stage 3 pressure injury, subcutaneous fat may be visible, but bone, tendon, and muscles are not exposed. Stage 4 involves full-thickness tissue loss with exposed bone, tendon, or muscle. DIF: Remember REF: 1299-1301 (Table 46-3) OBJ: Recognize the stages of pressure injury. MSC: NCLEX: Physiological Integrity
TOP: Assessment
6. The nurse is completing a skin assessment on a patient with darkly pigmented skin. Which of
the following would be used first to assist in staging a pressure injury on this patient? a. Cotton-tipped applicator. b. Disposable measuring tape. c. Sterile gloves. d. Halogen light. ANS: D
When a patient with darkly pigmented skin undergoes a skin assessment, proper lighting is essential to accurately complete the first step in assessment—inspection—and the whole assessment process. Natural light or a halogen light is recommended. Fluorescent light sources can produce blue tones on darkly pigmented skin and can interfere with an accurate assessment. Other items that could possibly be used during the assessment include gloves for infection control, a disposable measuring device to measure the size of the wound, and a cotton-tipped applicator to measure the depth of the wound, but these items not the first item used.
NURSINGTB.COM
DIF: Apply REF: 1291 (Box 46-2) OBJ: Recognize the stages of pressure injury. MSC: NCLEX: Physiological Integrity
TOP: Assessment
7. The nurse is caring for a patient with a stage 4 pressure injury. The nurse recalls that a
pressure injury takes time to heal and that the healing process is an example of which of the following? a. Primary intention. b. Partial-thickness wound repair. c. Full-thickness wound repair. d. Tertiary intention. ANS: C
Pressure injuries are full-thickness wounds that extend into the dermis and heal by scar formation because the deeper structures do not regenerate; hence the need for full-thickness repair. The full-thickness repair has three phases: inflammatory, proliferative, and remodelling. A wound heals by primary intention when wounds such as surgical wounds have little tissue loss; the skin edges are approximated or closed; and the risk for infection is low. Partial-thickness repairs are done on partial-thickness wounds that are shallow, involving loss of the epidermis and maybe partial loss of the dermis. These wounds heal by regeneration because the epidermis regenerates. In tertiary intention, a wound is left open for several days, and then the wound edges are approximated; wound closure is delayed until risk of infection is resolved.
Canadian Fundamentals of Nursing 6th Edition Potter Test Bank
DIF: Understand TOP: Planning
REF: 1302| 1303 OBJ: Discuss the normal process of wound healing. MSC: NCLEX: Physiological Integrity
8. The nurse is caring for a patient with a large abrasion from a motorcycle accident. The nurse
recalls that if the wound is kept moist, it can resurface in how long? a. 4 days. b. 2 days. c. 1 day. d. 7 days. ANS: A
A partial-thickness wound repair has three compartments: the inflammatory response, epithelial proliferation and migration, and re-establishment of the epidermal layers. Epithelial proliferation and migration start at all edges of the wound, allowing for quick resurfacing. Epithelial cells begin to migrate across the wound bed soon after the wound occurs. A wound left open to air resurfaces within 6 to 7 days, whereas a wound that is kept moist can resurface in 4 days. One or 2 days is too soon for this process to occur, moist or dry. DIF: Remember TOP: Planning
REF: 1303 OBJ: Discuss the normal process of wound healing. MSC: NCLEX: Physiological Integrity
9. The nurse is caring for a patient who is undergoing a full-thickness repair. The nurse would
expect to see which of the following in this type of repair? a. Eschar. b. Slough. c. Granulation. NURSINGTB.COM d. Purulent drainage. ANS: C
Granulation tissue is red, moist tissue composed of new blood vessels, the presence of which indicates progression toward healing. Soft yellow or white tissue is characteristic of slough—a substance that needs to be removed for the wound to heal. Black or brown necrotic tissue is called eschar, which also needs to be removed for a wound to heal. Purulent drainage is indicative of an infection and must be resolved for the wound to heal. DIF: Understand TOP: Assessment
REF: 1303 OBJ: Discuss the normal process of wound healing. MSC: NCLEX: Physiological Integrity
10. The nurse is caring for a patient who has undergone a laparoscopic appendectomy. The nurse
recalls that this type of wound heals by which process? a. Tertiary intention. b. Secondary intention. c. Partial-thickness repair. d. Primary intention. ANS: D
Canadian Fundamentals of Nursing 6th Edition Potter Test Bank A clean surgical incision is an example of a wound with little loss of tissue that heals with primary intention. The skin edges are approximated or closed, and the risk for infection is low. Partial-thickness repairs are done on partial-thickness wounds that are shallow, involving loss of the epidermis and maybe partial loss of the dermis. These wounds heal by regeneration because the epidermis regenerates. In tertiary intention, a wound is left open for several days, and then the wound edges are approximated; wound closure is delayed until the risk of infection is resolved. A wound involving loss of tissue such as a burn, a pressure injury, or a laceration heals by secondary intention. The wound is left open until it becomes filled with scar tissue. It takes longer for a wound to heal by secondary intention; thus the chance of infection is greater. DIF: Remember REF: 1301-1303 OBJ: Describe the differences between wounds that heal by primary and secondary intention. TOP: Planning MSC: NCLEX: Physiological Integrity 11. The nurse is caring for a patient in the burn unit. The nurse recalls that this type of wound
heals by which process? a. Tertiary intention. b. Secondary intention. c. Partial-thickness repair. d. Primary intention. ANS: B
A wound involving loss of tissue such as a burn or a pressure injury or laceration heals by secondary intention. The wound is left open until it becomes filled with scar tissue. It takes longer for a wound to heal by secondary intention; thus the chance of infection is greater. A clean surgical incision is an example of a wound with little loss of tissue that heals by primary N R I Gd B.C M intention. The skin edges are apUproSximNateT or cloO sed, and the risk for infection is low. Partial-thickness repair are done on partial-thickness wounds that are shallow, involving loss of the epidermis and maybe partial loss of the dermis. These wounds heal by regeneration because the epidermis regenerates. In tertiary intention, a wound is left open for several days, and then the wound edges are approximated; wound closure is delayed until the risk of infection is resolved. DIF: Remember REF: 1301-1303 OBJ: Describe the differences between wounds that heal by primary and secondary intention. TOP: Planning MSC: NCLEX: Physiological Integrity 12. Which nursing observation would indicate that a wound healed by secondary intention? a. Minimal scar tissue b. Minimal loss of tissue function c. Permanent dark redness at site d. Severe scarring. ANS: D
A wound healing by secondary intention takes longer than one healing by primary intention. The wound is left open until it becomes filled with scar tissue. If the scarring is severe, loss of function is often permanent. Wounds that heal by primary intention heal quickly with minimal scarring. Scar tissue contains few pigmented cells and has a lighter colour than normal skin. DIF: Understand
REF: 1301-1303
Canadian Fundamentals of Nursing 6th Edition Potter Test Bank OBJ: Describe the differences between wounds that heal by primary and secondary intention. TOP: Assessment MSC: NCLEX: Physiological Integrity 13. The nurse is caring for a patient who has undergone a total hysterectomy. Which nursing
observation would indicate that the patient was experiencing a complication of wound healing? a. The incision site has started to itch. b. The incision site is approximated. c. The patient has pain at the incision site. d. The incision has a mass, bluish in colour. ANS: D
A hematoma is a localized collection of blood underneath the tissues. It appears as swelling; a change in colour, sensation, or warmth; or a mass that often takes on a bluish discoloration. A hematoma near a major artery or vein is dangerous because it can put pressure on the vein or artery and obstruct blood flow. Itching of an incision site can be associated with clipping of hair, dressings, or possibly the healing process. Incisions should be approximated with edges together. After surgery, when nerves in the skin and tissues have been traumatized by the surgical procedure, it is expected that the patient would experience pain. DIF: Understand TOP: Assessment
REF: 1306| 1308 OBJ: Describe the complications of wound healing. MSC: NCLEX: Physiological Integrity
14. Which of these findings if seen in a postoperative patient should the nurse associate with
dehiscence? a. Complaint by patient that something has given way. b. Protrusion of visceral organs through a wound opening. c. Chronic drainage of fluidNthUrR ouSgI hN thG eT inB ci. sioCnOsM ite. d. Drainage that is malodorous and purulent. ANS: A
Dehiscence is a development in which a wound fails to heal properly and the layers of skin and tissue separate. It often involves abdominal surgical wounds and occurs after a sudden strain such as coughing, vomiting, or sitting up in bed. Affected patients often report feeling as though something has given way. In evisceration, vital organs protrude through a wound opening. A fistula is an abnormal passage between two organs or between an organ and the outside of the body that can be characterized by chronic drainage of fluid. Infection is characterized by drainage that is malodorous and purulent. DIF: Understand TOP: Assessment
REF: 1309 OBJ: Describe the complications of wound healing. MSC: NCLEX: Physiological Integrity
15. A patient has developed a pressure injury. What laboratory data would be important to gather? a. Serum albumin level. b. Creatine kinase level. c. Vitamin E level. d. Potassium level. ANS: A
Canadian Fundamentals of Nursing 6th Edition Potter Test Bank Normal wound healing necessitates proper nutrition. Serum proteins are biochemical indicators of malnutrition, and serum albumin is probably the most frequently measured of these parameters. The best measurement of nutritional status is prealbumin level because it reflects not only what the patient has ingested, but also what the body has absorbed, digested, and metabolized. Measurement of creatine kinase helps in the diagnosis of myocardial infarctions and has no known role in wound healing. Potassium is a major electrolyte that helps to regulate metabolic activities, cardiac muscle contraction, skeletal and smooth muscle contraction, and transmission and conduction of nerve impulses. Vitamin E is a fat-soluble vitamin that prevents the oxidation of unsaturated fatty acids. It is believed to reduce the risk of coronary artery disease and cancer. Vitamin E has no known role in wound healing. DIF: Understand REF: 1292 OBJ: Explain the factors that impede or promote wound healing. TOP: Assessment MSC: NCLEX: Physiological Integrity 16. Which of the following would be the most important piece of assessment data to gather with
regard to wound healing? a. Muscular strength assessment. b. Sleep assessment. c. Pulse oximetry assessment. d. Sensation assessment. ANS: C
Oxygen fuels the cellular functions essential to the healing process; the ability to perfuse tissues with adequate amounts of oxygenated blood is critical in wound healing. Blood flow through the pulmonary capillaries provides red blood cells for oxygen attachment. Oxygen diffuses from the alveoli into the pulmonary blood; most of the oxygen attaches to N heRrSedIbNloo GTdB.C M hemoglobin molecules within tU cells.O Red blood cells carry oxygenated hemoglobin molecules through the left side of the heart and out to the peripheral capillaries, where the oxygen detaches, depending on the needs of the tissues. Pulse oximetry measures the oxygen saturation of blood. Assessment of muscular strength and sensation, although useful for fitness and mobility testing, does not provide any data with regard to wound healing. Sleep, although important for rest and for integration of learning and restoration of cognitive function, also does not provide any data with regard to wound healing. DIF: Apply REF: 1292 OBJ: Explain the factors that impede or promote wound healing. TOP: Assessment MSC: NCLEX: Physiological Integrity 17. The nurse is caring for a patient with a healing stage 3 pressure injury. Upon entering the
room, the nurse notices an odour and observes a purulent discharge, along with increased redness at the wound site. What is the next best step for the nurse? a. Complete the head-to-toe assessment, and include current treatment, vital signs, and laboratory results. b. Notify the charge nurse about the change in status and the potential for infection. c. Notify the physician by utilizing Situation, Background, Assessment, and Recommendation (SBAR). d. Notify the wound care nurse about the change in status and the potential for infection. ANS: A
Canadian Fundamentals of Nursing 6th Edition Potter Test Bank The patient is showing signs and symptoms associated with infection in the wound. It is serious and necessitates treatment but is not a life-threatening emergency, for which care is needed immediately or the patient will suffer long-term consequences. The nurse should complete the assessment; gather all data such as current treatment modalities, medications, vital signs such as temperature, and laboratory results such as the most recent complete blood cell count or white blood cell count. The nurse can then notify the physician and receive treatment orders for the patient. It is important to notify the charge nurse and consult the wound nurse on the patient’s status and on any new orders. DIF: Apply REF: 1306| 1307 OBJ: Explain the factors that impede or promote wound healing. TOP: Implementation MSC: NCLEX: Physiological Integrity 18. The nurse is collaborating with the dietitian in treatment of a patient with a stage 3 pressure
injury. After the collaboration, the nurse orders a meal plan that includes increased levels of what? a. Fat. b. Carbohydrates. c. Protein. d. Vitamin E. ANS: C
Protein needs are especially increased in supporting the activity of wound healing. The physiological processes of wound healing depend on the availability of protein, vitamins (especially A and C), and the trace minerals of zinc and copper. A balanced diet of fat and carbohydrates, along with protein, vitamins, and minerals, is needed in any diet. Wound healing does not require increased amounts of fats or carbohydrates. Vitamin E has no known N R I G B.C M role in wound healing. U S N T O DIF: Apply REF: 1317 OBJ: Explain the factors that impede or promote wound healing. TOP: Implementation MSC: NCLEX: Physiological Integrity 19. The nurse is completing an assessment on an individual who has a stage 4 pressure injury. The
wound is malodorous, and a drain is currently in place. The nurse determines that the patient is experiencing issues with self-concept when the patient states which of the following? a. “I think I will be ready to go home early next week.” b. “I am so weak and tired; I want to feel better.” c. “I am ready for my bath and linen change as soon as possible.” d. “I am hoping there will be something good for dinner tonight.” ANS: C
The patient’s psychological response to any wound is part of the nurse’s assessment. Body image changes can influence self-concept. Factors that affect the patient’s perception of the wound include the presence of scars, drains, odour from drainage, and temporary or permanent prosthetic devices. The wound is malodorous, and a drain is in place. The patient who is asking for a bath and change in linens gives you a clue that he or she may be concerned about the smell in the room. Statements about wanting to feel better, going home, and wondering what is for dinner could be interpreted as positive statements that indicate progress along the health journey.
Canadian Fundamentals of Nursing 6th Edition Potter Test Bank DIF: Analyze REF: 1292| 1293 OBJ: Explain the factors that impede or promote wound healing. TOP: Implementation MSC: NCLEX: Psychosocial Integrity 20. A patient presents to the emergency department with a laceration of the right forearm caused
by a fall. After determining that the patient is stable, what is the next best step? a. Inspecting the wound for bleeding. b. Inspecting the wound for foreign bodies. c. Determining the size of the wound. d. Determining the need for a tetanus antitoxin injection. ANS: A
After determining that a patient’s condition is stable, the nurse should inspect the wound for bleeding. An abrasion will have limited bleeding, a laceration can bleed more profusely, and a puncture wound bleeds in relation to the size and depth of the wound. The nurse should first address any bleeding issues. Then the nurse inspects the wound for foreign bodies; traumatic wounds are dirty and may need to be addressed. A large open wound may expose bone or tissue and be protected, or the wound may need suturing. When the wound is caused by a dirty penetrating object, the nurse should determine the need for a tetanus vaccination. DIF: Apply REF: 1308| 1309 OBJ: Describe the differences in nursing care for acute and chronic wounds. TOP: Implementation MSC: NCLEX: Physiological Integrity 21. The nurse is caring for a patient on the medical-surgical unit with a wound that has a drain
and a dressing that needs changing. Which of these actions should the nurse take first? a. Don sterile gloves. b. Provide analgesic medicaN tiU onRs S asIoN rdGeT reB d..COM c. Avoid accidentally removing the drain. d. Gather supplies. ANS: B
Because removal of dressings is painful, it often helps to give the patient an analgesic at least 30 minutes before the wound is exposed and the dressing is changed. The sequence of the next events includes gathering supplies for the dressing change, donning gloves, and avoiding the accidental removal of the drain during the procedure. DIF: Apply REF: 1319 (Skill 46-3)| 1326 OBJ: Describe the differences in nursing care for acute and chronic wounds. TOP: Implementation MSC: NCLEX: Physiological Integrity 22. The nurse is caring for a patient who has a wound drain with a collection device. The nurse
notices that the collection device has a sudden decrease in drainage. What would be the nurse’s next best step? a. Remove the drain; a drain is no longer needed. b. Call the physician; a blockage is present in the tubing. c. Call the charge nurse to look at the drain. d. As long as the evacuator is compressed, do nothing. ANS: B
Canadian Fundamentals of Nursing 6th Edition Potter Test Bank Because a drainage system needs to be patent, look for drainage flow through the tubing, as well as around the tubing. A sudden decrease in drainage through the tubing may indicate drain blockage, and the nurse must notify the physician. The health care provider determines the need for drain removal and removes drains. Notifying the charge nurse, although important for communication, is not the next step in providing care for this patient. The evacuator may be compressed when a blockage is present. DIF: Apply REF: 1307 OBJ: Describe the differences in nursing care for acute and chronic wounds. TOP: Implementation MSC: NCLEX: Physiological Integrity 23. The nurse is caring for a patient who has a stage 4 pressure injury and is awaiting plastic
surgery consultation. Which of the following specialty beds would be most appropriate? a. Standard mattress. b. Nonpowered redistribution air mattress. c. Low-air-loss therapy unit. d. Lateral rotation. ANS: B
A static air mattress, or nonpowered redistribution mattress, is utilized for the patient at high risk for skin breakdown, as in this scenario. A low-air-loss therapy unit is utilized for stage 4 pressure injury and when prevention or treatment of skin breakdown is needed. If the patient has a stage 3 or stage 4 pressure injury or a postoperative myocutaneous flap, the low-air-loss therapy unit would be an appropriate selection. A standard mattress is utilized for an individual who does not have actual or potential altered or impaired skin integrity. Lateral rotation is used for treatment and prevention of pulmonary complications associated with mobility.
NURSINGTB.COM
DIF: Understand REF: 1313 OBJ: Describe the differences in nursing care for acute and chronic wounds. TOP: Assessment MSC: NCLEX: Physiological Integrity 24. The nurse is caring for a patient with a pressure injury on the left hip. The pressure injury is
black. The nurse recognizes that the next step in caring for this patient includes which of the following? a. Monitoring of the wound. b. Irrigation of the wound. c. Débridement of the wound. d. Management of drainage. ANS: C
Débridement is the removal of nonviable necrotic tissue. Removal of necrotic tissue is necessary to rid the pressure injury of a source of infection, to enable visualization of the wound bed, and to provide a clean base for healing. A wound will not move through the phases of healing if it is infected. Irrigating the wound with noncytotoxic cleaners will not damage or kill fibroblasts and healing tissue and will help to keep the wound clean after débridement. When the nurse is treating a pressure injury, it is important to monitor and reassess the wound at least every 8 hours. Management of drainage helps keep the wound clean. DIF: Understand
REF: 1317
Canadian Fundamentals of Nursing 6th Edition Potter Test Bank OBJ: Describe the differences in nursing care for acute and chronic wounds. TOP: Planning MSC: NCLEX: Physiological Integrity 25. The nurse is caring for a patient with a healing stage 3 pressure injury. The wound is clean
and granulating. Which of the following orders would the nurse question? a. Use a low-air-loss therapy unit. b. Consult a dietitian. c. Irrigate with hydrogen peroxide. d. Utilize hydrogel dressing. ANS: C
Pressure injuries should be cleaned with noncytotoxic cleansers such as normal saline, which will not kill fibroblasts and healing tissue. Cytotoxic cleansers such as Dakin’s solution, acetic acid, povidone-iodine, and hydrogen peroxide can hinder the healing process and should not be utilized on clean granulating wounds. Consulting a dietitian for the nutritional needs of the patient, utilizing a low-air-loss therapy unit to decrease pressure, and applying hydrogel dressings to provide a moist environment for healing are all appropriate orders. DIF: Analyze REF: 1316 OBJ: Describe the differences in nursing care for acute and chronic wounds. TOP: Implementation MSC: NCLEX: Physiological Integrity 26. The nurse is completing an assessment of the skin’s integrity, which includes which of the
following? a. Pressure points. b. All pulses. c. Breath sounds. d. Bowel sounds.
NURSINGTB.COM
ANS: A
The nurse continually assesses the skin for signs of pressure injury development. Assessment of tissue pressure damage includes visual and tactile inspection of the skin. The nurse should observe pressure points such as bony prominences and areas next to invasive treatments such as a binasal cannula and the nares. Assessment of pulses, breath sounds, and bowel sounds is part of a head-to-toe assessment and could influence the function of the body and ultimately skin integrity; however, this assessment is not a specific part of a skin assessment. DIF: Understand REF: 1296 (Skill 46-1) OBJ: Complete an assessment for a patient with impaired skin integrity. TOP: Assessment MSC: NCLEX: Physiological Integrity 27. The nurse is using the Braden scale to complete a skin risk assessment. The patient has some
sensory impairment and skin that is rarely moist, walks occasionally, and has slightly limited mobility, along with excellent intake of meals and no apparent problem with friction and shear. What would be the patient’s total Braden scale score? a. 15. b. 17. c. 21. d. 23. ANS: C
Canadian Fundamentals of Nursing 6th Edition Potter Test Bank The Braden scale is composed of six subscales: sensory perception, moisture, activity, mobility, nutrition, and friction and shear. The total score ranges from 6 to 23, and a lower total score indicates a higher risk for pressure injury development. On the Braden scale, the patient receives 3 for slight sensory impairment, 4 for skin’s rarely being moist, 3 for walking occasionally, 3 for slightly limited mobility, 4 for excellent intake of meals, and 4 for no problem with friction and shear. The total score is 21. DIF: Apply REF: 1298 (Table 46-2) OBJ: Complete an assessment for a patient with impaired skin integrity. TOP: Assessment MSC: NCLEX: Physiological Integrity 28. The nurse is caring for a patient with a stage 4 pressure injury. The nurse assigns which of the
following nursing diagnoses? a. Readiness for enhanced nutrition. b. Impaired physical mobility. c. Impaired skin integrity. d. Chronic pain. ANS: C
After the assessment is completed and the information that the patient has a stage 4 pressure injury is gathered, a diagnosis of Impaired skin integrity is selected. Readiness for enhanced nutrition would be selected for an individual with an adequate diet that could be improved. Impaired physical mobility and Chronic pain, as well as the nutrition nursing diagnosis, could well be the nursing diagnoses selected for this patient, but the physical data in the question strongly support Impaired skin integrity. DIF: Understand REF: 1309| 1310 Bp.aiCreO OBJ: List nursing Diagnosis aN ssU ocR iaS teI dw thT im dM skin integrity. NiG TOP: Planning MSC: NCLEX: Physiological Integrity 29. The nurse has collected the following assessment data: right heel with reddened area that does
not blanch. What nursing diagnosis would the nurse assign? a. Ineffective tissue perfusion. b. Risk for infection. c. Imbalanced nutrition: less than body requirements. d. Acute pain. ANS: A
The area on the heel has experienced a decreased supply of blood and oxygen (tissue perfusion), which has resulted in tissue damage. The most appropriate nursing diagnosis with this information is Ineffective tissue perfusion. Risk for infection, Acute pain, and Imbalanced nutrition may be other nursing diagnoses, but the data provided do not support them. DIF: Understand REF: 1309| 1310 OBJ: List nursing Diagnosis associated with impaired skin integrity. TOP: Planning MSC: NCLEX: Physiological Integrity 30. The nurse is caring for a patient with a stage 3 pressure injury. The nurse has assigned a
nursing diagnosis of Risk for infection. Which intervention would be most important for this patient? a. Teach the family how to manage the odour associated with the wound.
Canadian Fundamentals of Nursing 6th Edition Potter Test Bank b. Discuss with the family how to prepare for care of the patient in the home. c. Encourage thorough hand hygiene by all individuals caring for the patient. d. Encourage increased quantities of carbohydrates and fats. ANS: C
The number one way to decrease the risk of infection by breaking the chain of infection is to wash hands. Encouraging fluid and food intake helps with overall wellness and wound healing, especially protein, but an increase in carbohydrates and fats is not related to the risk of infection. If the patient will be discharged before the wound is healed, the family will certainly need education on how to care for the patient. Teaching the family how to manage the odour associated with a wound is certainly important, but these interventions do not directly relate to the risk of infection and breaking the chain of the infectious process. DIF: Apply REF: 1326 OBJ: List nursing Diagnosis associated with impaired skin integrity. TOP: Implementation MSC: NCLEX: Health Promotion and Maintenance 31. The patient in medical-surgical acute care has received a nursing diagnosis of Impaired skin
integrity. Which health care provider does the nurse consult? a. Respiratory therapist. b. Registered dietitian. c. Chaplain. d. Case manager. ANS: B
Assessment and a plan for the patient to optimize the diet are essential. Adequate calories, protein, vitamins, and minerals promote wound healing. The nurse is the coordinator of care, and collaborating with the diN etiti anSw ldT reBsu.ltCinOM planning the best meals for the patient. The Iou UR NG respiratory therapist can be consulted when a patient has issues with the respiratory system. Case management can be consulted when the patient has a discharge need. A chaplain can be consulted when the patient has a spiritual need. DIF: Apply REF: 1311| 1331 OBJ: List nursing Diagnosis associated with impaired skin integrity. TOP: Implementation MSC: NCLEX: Health Promotion and Maintenance 32. The nurse is caring for a patient with a stage 2 pressure injury and has assigned a nursing
diagnosis of Risk for infection. The patient is unconscious and bedridden. The nurse is completing the plan of care and is writing goals for the patient. What is the best goal for this patient? a. The patient’s family will demonstrate specific care of the wound site. b. The patient will state what to look for with regard to an infection. c. The patient will remain free of an increase in temperature and of malodorous or purulent drainage from the wound. d. The patient’s family members will wash their hands when visiting the patient. ANS: C
Canadian Fundamentals of Nursing 6th Edition Potter Test Bank Because the patient has an open wound and the skin is no longer intact to protect the tissue, the patient is at increased risk for infection. The nurse will be assessing the patient for signs and symptoms of infection, including an increase in temperature, an increase in white blood cell count, and malodorous and purulent drainage from the wound. The patient is unconscious and is unable to communicate the signs and symptoms of infection; also, enabling the patient to communicate is an intervention, not a goal, for this diagnosis. It is important for the patient’s family to be able to demonstrate how to care for the wound and wash their hands, but these statements are interventions, not goals or outcomes for this nursing diagnosis. DIF: Apply REF: 1314-1316 OBJ: List nursing Diagnosis associated with impaired skin integrity. TOP: Planning MSC: NCLEX: Health Promotion and Maintenance 33. On inspection of the patient’s wound, the nurse notes that it has a large amount of exudate.
Which of the following is an appropriate dressing for the nurse to select? a. Foam. b. Hydrogel. c. Hydrocolloid. d. Transparent film. ANS: A
Foam dressings are useful for wounds with large amounts of drainage or to assist with autolytic débridement because they can be left in place for as long as 7 days. This would be the most appropriate type of dressing for this wound. A hydrogel dressing donates moisture to the wound bed. This type of dressing hydrates the wound and absorbs some smaller amounts of exudates, but it would not be appropriate for a wound with a large amount of exudate. A hydrocolloid dressing is adhesive and occlusive. This type of dressing supports healing in N RSos IN Geful B.C M clean, granulating wounds, andUis m t usT on O shallow to moderately deep wounds. It should not be used in heavily draining wounds, full-thickness wounds, or infected wounds. Transparent film traps the wound’s moisture over the wound, which provides a moist environment. It is used for small, superficial wounds; it is not appropriate for a wound with a large amount of exudate. DIF: Apply REF: 1323| 1324 (Table 46-9) OBJ: List nursing Diagnosis associated with impaired skin integrity. TOP: Implementation MSC: NCLEX: Physiological Integrity 34. The home health nurse is caring for a patient with impaired skin integrity in the home. The
nurse is reviewing dressing changes with the caregiver. Which intervention assists in managing the expenses associated with long-term wound care? a. Sterile technique. b. No-touch technique. c. Double bagging of contaminated dressings. d. Ability of the caregiver. ANS: B
Canadian Fundamentals of Nursing 6th Edition Potter Test Bank The caregiver can use the same no-touch technique for dressing changes that is used for changing surface dressings without touching the wound or the surface that might come in contact with the wound. Double bagging is required for the disposal of contaminated dressings. The dressings go in a bag, which is fastened and then placed in the household trash. The ability of the caregiver certainly is a component of the success of home treatment, but it does not influence the cost of supplies. DIF: Apply REF: 1312 (Box 46-10) OBJ: Develop a nursing care plan for a patient with impaired skin integrity. TOP: Implementation MSC: NCLEX: Health Promotion and Maintenance 35. The nurse is caring for a patient who has suffered a stroke and has residual mobility problems.
The patient is at risk for skin impairment. Which initial interventions should the nurse select to decrease this risk? a. Gentle cleaners and thorough drying of the skin. b. Absorbent pads and garments. c. Positioning with use of pillows. d. Therapeutic beds and mattresses. ANS: A
Assessment and skin hygiene are two initial defences for preventing skin breakdown. The nurse should avoid soaps and hot water when cleansing the skin and should instead use gentle cleansers with nonionic surfactants. After the patient is bathed, the nurse makes sure to dry the skin completely and to apply moisturizer to keep the epidermis well lubricated. The use of absorbent pads and garments is controversial and should be considered only when other alternatives have been exhausted. Positioning the patient reduces pressure and shearing force to the skin and is part of the plan of care but is not one of the initial components. Depending N Ralty INbed GTmay B.CbeOneeded, M on the needs of the patient, a spUeciS but again, this does not provide the initial defence against skin breakdown. DIF: Apply REF: 1311| 1312 OBJ: Develop a nursing care plan for a patient with impaired skin integrity. TOP: Implementation MSC: NCLEX: Physiological Integrity 36. The nurse is caring for a patient who is at risk for skin impairment. The patient is able to sit up
in a chair. The nurse includes this intervention in the plan of care. How long should the nurse schedule the patient to sit in the chair? a. At least 3 hours. b. Not longer than 30 minutes. c. Less than 2 hours. d. As long as the patient remains comfortable. ANS: C
When patients are able to sit up in a chair, the nurse should make sure to limit the amount of time to 2 hours or less. The chair sitting time should be individualized. In the sitting position, pressure on the ischial tuberosities is greater than in a supine position. Foam, gel, or an air cushion may be used to distribute weight. Sitting longer than 2 hours can increase the chance of ischemia. DIF: Apply REF: 1313 OBJ: Develop a nursing care plan for a patient with impaired skin integrity.
Canadian Fundamentals of Nursing 6th Edition Potter Test Bank TOP: Implementation
MSC: NCLEX: Physiological Integrity
37. The nurse is staffing a medical-surgical unit that is assigned most of the patients with pressure
injuries. The nurse has become competent in the care of pressure wounds and recognizes that which of the following is a staged pressure injury that does not require a dressing? a. Stage 1. b. Stage 2. c. Stage 3. d. Stage 4. ANS: A
Stage 1 intact pressure injuries that resolve slowly without epidermal loss over 7 to 14 days do not require a dressing. This allows visual inspection and monitoring. A transparent dressing could be used to protect the patient from shear but cannot be used in the presence of excessive moisture. A composite film, hydrocolloid, or hydrogel can be utilized on a clean stage 2 pressure injury. A hydrocolloid, hydrogel covered with foam, calcium alginate, gauze, and growth factors can be utilized with a clean stage 3 injury. Hydrogel, calcium alginate, gauze, and growth factors can be utilized with a clean stage 4 injury. For an unstageable wound covered with eschar, the nurse should utilize a dressing of adherent film or gauze with an ordered solution of enzymes. In rare cases when eschar is dry and intact, no dressing is used, but this is an unstaged pressure injury. DIF: Understand REF: 1300 (Table 46-3) OBJ: List appropriate nursing interventions for a patient with impaired skin integrity. TOP: Diagnosis MSC: NCLEX: Physiological Integrity 38. The nurse is caring for a patient with a wound. The patient appears anxious as the nurse is
preparing to change the dressN inU gR .W ouBld.tC heOnMurse do to decrease the patient’s anxiety? ShIaNt sGhT a. Tell the patient to close his eyes. b. Explain the procedure. c. Turn on the television. d. Ask the family to leave the room. ANS: B
Explaining the procedure educates the patient regarding the dressing change and involves him in his care, thereby allowing the patient some control in decreasing anxiety. Telling the patient to close his eyes and turning on the television are distractions that do not usually decrease a patient’s anxiety. If the family is a support system, asking support systems to leave the room can actually increase a patient’s anxiety. DIF: Understand REF: 1319 (Skill 46-3) OBJ: List appropriate nursing interventions for a patient with impaired skin integrity. TOP: Intervention MSC: NCLEX: Psychosocial Integrity 39. The nurse is cleansing a wound site. As the nurse is doing so, what intervention should be
included? a. Allowing the solution to flow from the most contaminated area to the least contaminated area. b. Scrubbing vigorously when applying solutions to the skin. c. Cleansing in a direction from the least contaminated area. d. Utilizing clean gauge and clean gloves to cleanse a site.
Canadian Fundamentals of Nursing 6th Edition Potter Test Bank
ANS: C
The nurse cleanses surgical or traumatic wounds by applying noncytotoxic solution with sterile gauze or irrigations. Cleansing proceeds in a direction from the least contaminated area. The nurse should use gentle friction when applying solutions to the skin and allow irrigation to flow from the least to the most contaminated area. DIF: Apply REF: 1327 OBJ: List appropriate nursing interventions for a patient with impaired skin integrity. TOP: Intervention MSC: NCLEX: Physiological Integrity 40. The nurse is caring for a patient after an open repair of an abdominal aortic aneurysm. The
nurse requests an abdominal binder and carefully applies the binder. What is the best explanation for the nurse to use when teaching the patient the reason for the binder? a. The binder creates pressure over the abdomen. b. The binder supports the abdomen. c. The binder reduces edema at the surgical site. d. The binder secures the dressing in place. ANS: B
The patient has a large abdominal incision. This incision will need support to remain closed, and an abdominal binder will support this wound, especially during movement, as well as during deep breathing and coughing. A binder can be used to create pressure over a body part: for example, over an artery after it has been punctured, but this is not the best explanation for using the binder in this example. A binder can be used to prevent edema—for example, in an extremity—but is not used to reduce edema at a surgical site. A binder can be used to secure dressings such as elastic webbing applied around a leg after vein stripping, but this is not the best explanation for using theNbinRderIin tG his B ex.aC mplM e.
U S N T
O
DIF: Apply REF: 1330| 1331 OBJ: List appropriate nursing interventions for a patient with impaired skin integrity. TOP: Intervention MSC: NCLEX: Physiological Integrity 41. The nurse determines that the patient’s wound may be infected. In order to obtain a
quantitative swab for wound culture, which of the following actions should the nurse take? a. Collect the superficial drainage. b. Collect the culture before cleansing the wound. c. Obtain a Culturette tube and use sterile technique. d. Use the same technique as for collecting an anaerobic culture. ANS: C
To collect a quantitative swab for a wound culture, the nurse uses a sterile swab from a Culturette tube and sterile technique. The nurse never collects a wound culture sample from old or superficial drainage; resident colonies of bacteria from the skin grow in superficial drainage and may not be the true causative organisms of a wound infection. The nurse should clean the wound first with normal saline to remove skin flora, before obtaining the culture. The nurse uses different methods of specimen collection for aerobic or anaerobic organisms. DIF: Apply REF: 1308 (Box 46-7) OBJ: List appropriate nursing interventions for a patient with impaired skin integrity. TOP: Intervention MSC: NCLEX: Physiological Integrity
Canadian Fundamentals of Nursing 6th Edition Potter Test Bank 42. The patient has been provided a nursing diagnosis of Risk for skin impairment and has a score
of 15 on the Braden scale upon admission. The nurse has implemented interventions for this nursing diagnosis. Upon reassessment, which Braden score would be the best sign that the risk for skin breakdown is decreasing? a. 12. b. 13. c. 20. d. 23. ANS: D
The Braden scale is composed of six subscales: sensory perception, moisture, activity, mobility, nutrition, and friction and shear. The total score ranges from 6 to 23, and a lower total score indicates a higher risk for pressure injury development. The cutoff score for onset of pressure injury risk with the Braden scale in the general adult population is 18. The best sign is a perfect score of 23. DIF: Analyze REF: 1295| 1298 (Table 46-2) OBJ: List appropriate nursing interventions for a patient with impaired skin integrity. TOP: Evaluate MSC: NCLEX: Physiological Integrity MULTIPLE RESPONSE 1. The nurse is caring for a patient with a stage 2 pressure injury and, as the coordinator of care,
understands the need for a multidisciplinary approach. The nurse evaluates the need for several consults. Which of the following should always be included in the consults? (Select all that apply.) a. Registered dietitian. NURSINGTB.COM b. Enterostomal and wound care nurse. c. Physiotherapist. d. Case management personnel. e. Chaplain. f. Pharmacist. ANS: A, B, C, D
A registered dietitian works with the nurse to determine a meal plan that will support wound healing. An enterostomal or wound care nurse specializes in caring for the needs of the patient with wounds. Physiotherapy can assist an immobile patient to progress toward mobility and decrease the risk for pressure injury. Pressure injuries take a long time to heal and usually require continued therapy in the home. Case management personnel are useful in obtaining care for the patient outside the home. If the patient has a spiritual need, the chaplain can assist. If the patient has a need associated with medications, the pharmacist can assist. However, chaplains and pharmacists usually are not part of the wound care multidisciplinary team, unless a special need arises. DIF: Analyze REF: 1311 OBJ: List appropriate nursing interventions for a patient with impaired skin integrity. TOP: Evaluate MSC: NCLEX: Safe and Effective Care Environment 2. The nurse is caring for a patient with wound healing by tertiary intention. Which factors does
the nurse recognize as influencing wound healing? (Select all that apply.)
Canadian Fundamentals of Nursing 6th Edition Potter Test Bank a. b. c. d. e. f.
Nutrition. Evisceration. Tissue perfusion. Infection. Hemorrhage. Age.
ANS: A, C, D, F
Normal wound healing requires proper nutrition. Oxygen and the ability to provide adequate amounts of oxygenated blood are crucial for wound healing. Wound infection prolongs the inflammatory phase, delays collagen synthesis, prevents epithelialization, and decreases the production of pro-inflammatory cytokines, which leads to additional tissue destruction. As patients age, all aspects of wound healing are delayed. Hemorrhage and evisceration are complications of, not influences on, wound healing. DIF: Remember REF: 1314-1327 OBJ: Explain factors that impede or promote wound healing. MSC: NCLEX: Physiological Integrity
TOP: Assessment
3. The nurse is caring for a patient with potential skin breakdown. Which components would the
nurse include in the skin assessment? (Select all that apply.) a. Mobility. b. Hyperemia. c. Induration. d. Blanching. e. Temperature of skin. f. Nutritional status. ANS: B, C, D, E
NURSINGTB.COM
Assessment of the skin includes both visual and tactile inspection. The nurse assesses for hyperemia and abnormal reactive hyperemia (when the skin turns red after an obstruction of blood flow returns and vasodilatation causes the tissue to turn red). The nurse also assesses for indurated (hardened) areas on the skin and palpates reddened areas for blanching. Changes in temperature can indicate changes in blood flow to that area of the skin. Mobility and nutritional status are certainly part of the overall assessment for pressure injury risk but are not part of the actual skin assessment. DIF: Apply REF: 1295-1299 OBJ: Complete an assessment for a patient with impaired skin integrity. TOP: Assessment MSC: NCLEX: Physiological Integrity 4. The nurse is caring for a patient who will have both a large abdominal bandage and an
abdominal binder. The nurse’s responsibilities and activities before applying the bandage and binder include which of the following? (Select all that apply.) a. Inspecting the skin for abrasions and edema. b. Covering exposed wounds. c. Assessing condition of current dressings. d. Assessing the skin at underlying areas for circulatory impairment. e. Marking the sites of all abrasions. f. Cleansing the area with hydrogen peroxide.
Canadian Fundamentals of Nursing 6th Edition Potter Test Bank ANS: A, B, C, D
Before applying a bandage or a binder, the nurse has several responsibilities. The nurse would need to inspect the skin for abrasions, edema, and discoloration or exposed wound edges. The nurse also is responsible for covering exposed wounds or open abrasions with a sterile dressing and assessing the condition of underlying dressings and changing them if they are soiled. To check for signs of circulatory impairment, the nurse assesses the skin of underlying areas that will be distal to the bandage so that a comparison can be made after bandages are applied. Marking the sites of all abrasions is not necessary. Although it is important for the skin to be clean, and even though it may need to be cleaned with a noncytotoxic cleanser, cleansing with hydrogen peroxide can interfere with wound healing. DIF: Apply REF: 1330-1331 OBJ: List appropriate nursing interventions for a patient with impaired skin integrity. TOP: Intervention MSC: NCLEX: Physiological Integrity
NURSINGTB.COM
Canadian Fundamentals of Nursing 6th Edition Potter Test Bank
Chapter 47: Sensory Alterations Potter et al: Canadian Fundamentals of Nursing, 6th Edition MULTIPLE CHOICE 1. A nurse is administering a vaccine to a 4-year-old child who is visually impaired. After the
needle enters the arm, the child says, “Ow, that was sharp!” The nurse knows that the ability to recognize and interpret stimuli is known as which of the following? a. Sensation. b. Reception. c. Perception. d. Reaction. ANS: C
Perception is the awareness of stimuli and interpretation of information signals. Reception refers to receiving stimuli and creating a nerve impulse. Reaction is how a person responds to a perceived stimulus. Sensation is a general term that refers to awareness of sensory stimuli through the body’s sense mechanisms. DIF: Understand REF: 1341 OBJ: Differentiate among the processes of reception, perception, and reaction to sensory stimuli. TOP: Assessment MSC: NCLEX: Physiological Integrity 2. What is the involuntary motion of retracting the body from painful stimuli? a. Sensation. b. Reception. NURSINGTB.COM c. Perception. d. Reaction. ANS: D
Reaction is how a person responds to a perceived stimulus. Perception is the consciousness of stimuli and interpretation of information signals. Reception refers to receiving stimuli and creating a nerve impulse. Sensation is the combination of all three combined. DIF: Understand REF: 1340| 1341 OBJ: Differentiate among the processes of reception, perception, and reaction to sensory stimuli. TOP: Assessment MSC: NCLEX: Physiological Integrity 3. A nurse is caring for a patient with a nursing diagnosis of Hearing deficit related to
presbycusis. Which assessment of the patient would indicate an adaptation to the sensory deficit? a. The patient frequently cleans out his or her ears with a cotton swab. b. The patient turns one ear toward the nurse during conversation. c. The patient isolates himself or herself from social situations. d. The patient asks the nurse to speak loudly during conversations. ANS: B
Canadian Fundamentals of Nursing 6th Edition Potter Test Bank Adaptation for a sensory deficit indicates that the patient alters his or her behaviour to accommodate for the sensory deficit, such as turning the less affected ear toward the speaker. Cleaning the ear would not have an effect for a patient with presbycusis. Avoiding others because of a sensory deficit is maladaptive. Asking the nurse to speak loud alters the environment but is not an adaptation of the patient’s behaviour. DIF: Understand REF: 1356| 1357 OBJ: Conduct an assessment of an individual’s sensory status. MSC: NCLEX: Physiological Integrity
TOP: Assessment
4. The nurse would be most concerned about the risk of malnutrition for a patient with which
sensory deficit? a. Xerostomia. b. Disequilibrium. c. Cataracts. d. Peripheral neuropathy. ANS: A
Xerostomia is a decrease in production of saliva; this decreases the ability and desire to eat and can lead to nutritional problems. The other options do not address taste- or nutrition-related concerns. DIF: Remember REF: 1343 OBJ: Discuss common causes and effects of sensory alterations. TOP: Assessment MSC: NCLEX: Physiological Integrity 5. Which of the following sensory changes are normal with aging? a. Impaired night vision. NURSINGTB.COM b. Difficulty hearing low pitch. c. Increase in taste discrimination. d. Heightened sense of smell. ANS: A
Night vision becomes impaired as physiological changes in the eye occur. Older persons lose the ability to distinguish high-pitched noises and consonants. Senses of smell and taste are also decreased with aging. DIF: Remember REF: 1345 OBJ: Discuss common sensory changes that normally take place as individuals age. TOP: Assessment MSC: NCLEX: Physiological Integrity 6. A nurse is caring for an older patient who was in a motor vehicle accident because he thought
the traffic light was green. The patient asks the nurse if he should no longer drive. Which response by the nurse is most therapeutic? a. “Yes, you should stop driving. As you age, your cognitive function declines, and becoming confused puts everyone else on the road at risk.” b. “Yes, you should ask family members to drive you around from now on. Your reflex skills have declined so much you can’t avoid an accident.” c. “No, as you age, you lose the ability to see colours. You need to think about traffic lights in a new way. If the top is illuminated, it means stop, and if the bottom is illuminated, it means go.”
Canadian Fundamentals of Nursing 6th Edition Potter Test Bank d. “No, instead you should see your ophthalmologist and get some glasses to help
you see better.” ANS: C
Part of the normal aging process is an inability to see colours. Much as with a younger adult who is colour blind, the nurse should teach the patient new ways to adapt to his deficit. This patient’s accident was not due to impaired cognitive function or reflexes. Glasses will not assist the patient in colour discrimination. DIF: Apply REF: 1345 OBJ: Discuss common sensory changes that normally take place as individuals age. TOP: Assessment MSC: NCLEX: Physiological Integrity 7. A nurse is caring for a patient who recently had a stroke and is going to be discharged at the
end of the week. The nurse notices that the patient is having difficulty with eating his meal and is becoming tearful. The nurse includes which intervention in the patient’s plan of care? a. Teach the patient about special devices used to assist patients with eating meals. b. Order the patient food that does not require utensils. c. Place a consult for a home health nurse. d. Obtain an order for antidepressant medications. ANS: A
The nurse should include implementations that help the patient adapt to his deficit while maintaining independence. Teaching the patient to use assistive devices allows the patient to care for himself. Changing the type of food the patient eats may not work for a culture in which touching food with fingers is unacceptable, or the patient may not enjoy eating foods that do not require utensils. The services of a home health nurse are not necessary as long as the patient is able to care for N him stea d. ofCpla cing the patient on antidepressants, the URself SI. In NG TB OM nurse should assist the patient in attempting to adapt behaviour to the sensory deficit. DIF: Analyze REF: 1346| 1358 OBJ: Develop a plan of care for individuals with sensory alterations. TOP: Planning MSC: NCLEX: Health Promotion and Maintenance 8. Which nursing diagnosis addresses psychological concerns for a patient with both hearing and
visual sensory impairment? a. Self-care deficit. b. Risk for falls. c. Social isolation. d. Impaired physical mobility. ANS: C
In focusing on the psychological aspect of care, the nurse is most concerned about social isolation for a patient who may have difficulty communicating because of visual and hearing impairment. Both self-care deficit and fall risk are physiological risks for the patient. Impaired physical mobility would not apply to this patient. DIF: Understand REF: 1351| 1352 OBJ: Develop a plan of care for individuals with sensory alterations. TOP: Planning MSC: NCLEX: Psychosocial Integrity
Canadian Fundamentals of Nursing 6th Edition Potter Test Bank 9. A patient informs the nurse that she often becomes nauseated when riding in motor vehicles.
The nurse knows that this is related to which sensory deficit? a. Neurological deficit. b. Visual deficit. c. Hearing deficit. d. Balance deficit. ANS: D
Vertigo is a result of vestibular dysfunction and often is precipitated by a change in head position. This disequilibrium can cause nausea and vomiting. The other options would not result in nausea based on movement. DIF: Understand REF: 1343 OBJ: Discuss common causes and effects of sensory alterations. TOP: Assessment MSC: NCLEX: Physiological Integrity 10. A home health nurse is assembling a puzzle with an older patient and notices that the patient
is having difficulty connecting two puzzle pieces. The nurse knows that this is most likely related to which aspect of sensory deprivation? a. Perceptual. b. Cognitive. c. Affective. d. Social. ANS: A
Alterations in spatial orientation and in visual/motor coordination are signs of perceptual dysfunction. Cognitive function is the ability to think and the capacity to learn; the patient is not disoriented or unable to lN earn AI ffec tive roC blem UR. S NG TBp. OMs include boredom and restlessness; the patient is participating in an activity. The patient is socializing with the home health nurse, so isolation is not a problem. DIF: Understand REF: 1344 (Box 47-1) OBJ: Discuss common causes and effects of sensory alterations. TOP: Assessment MSC: NCLEX: Physiological Integrity 11. Which assessment question should the nurse ask to best understand how visual alterations are
affecting the patient’s self-care ability? a. “Have you stopped reading books or switched to books on audiotape?” b. “Are you able to prepare a meal or write a cheque?” c. “How do you protect yourself from injury at work?” d. “How does your vision impairment make you feel?” ANS: B
To best understand how vision is affecting self-care ability, the nurse wants to target questions to encompass what self-care tasks the patient has difficulty doing, such as preparing meals and writing cheques. Switching from reading to books on audiotape gives the nurse an idea of the severity of the deficit but not its effect on activity of daily living. Assessing whether the patient is taking measures to protect himself is important, but this does not address self-care activities. Emotional assessment of a patient is also important but does not properly address the goal of determining the effect of visual alterations on self-care ability. DIF: Analyze
REF: 1344-1346
Canadian Fundamentals of Nursing 6th Edition Potter Test Bank OBJ: Discuss the relationship of sensory function to an individual’s level of wellness. TOP: Assessment MSC: NCLEX: Health Promotion and Maintenance 12.
Which nursing assessment best measures cognitive functioning? a. Administering a Mini-Mental Status Examination (MMSE). b. Asking the patient his name, where he is, and what month it is. c. Asking the patient’s family if the patient is behaving normally. d. Evaluating the patient’s ability to read the newspaper. ANS: A
The MMSE is a formal diagnostic tool that is used to assess a patient’s level of cognitive functioning. Asking the patient orientation questions helps evaluate only the patient’s orientation to self and surroundings, not abstract reasoning or critical thinking ability. Family members are not the most reliable source of information about the patient, although information received from the family should be considered. Reading a paper is not a means of comprehensive assessment; in addition, a patient may have high cognitive function and not know how to read English. DIF: Understand REF: 1349 OBJ: Conduct an assessment of an individual’s sensory status. MSC: NCLEX: Psychosocial Integrity
TOP: Assessment
13. During a community screening, the nurse notes that a 50-year-old patient is currently taking
steroid medications. How often does the nurse recommend that this patient have an eye examination? a. Every 3 to 4 months. b. Every 6 months. NURSINGTB.COM c. Every 1 to 2 years. d. Every 4 years. ANS: C
Patients between the ages of 40 and 64 years should have an eye examination every 1 to 2 years if there is a family history of glaucoma, if the patient has had a serious eye injury in the past, or if the patient is taking steroid medication. DIF: Understand REF: 1348 (Box 47-6) OBJ: Conduct an assessment of an individual’s sensory status. MSC: NCLEX: Health Promotion and Maintenance
TOP: Implementation
14. A new nurse is caring for a patient who is undergoing chemotherapy for cancer. The patient is
becoming malnourished because nothing tastes good. Which recommendation by the nurse would be most appropriate for this patient? a. “Practice good oral hygiene to keep your taste buds well hydrated.” b. “Blend foods together in interesting flavour combinations.” c. “Eat soft foods that are easy to chew and swallow.” d. “Avoid adding spices or aromatic ingredients to food to prevent nausea.” ANS: A
Good oral hygiene is important for stimulating and hydrating taste buds. Having an unpleasant taste in the mouth discourages the patient from eating. Foods should not be blended together because this confuses the ability to discriminate flavours and taste. Texturized, spicy, and aromatic foods stimulate eating and make it more enjoyable.
Canadian Fundamentals of Nursing 6th Edition Potter Test Bank
DIF: Apply REF: 1357 OBJ: Develop a plan of care for individuals with sensory alterations. TOP: Implementation MSC: NCLEX: Health Promotion and Maintenance 15. The nurse is creating a plan of care for a patient with glaucoma. Which nursing diagnosis
addresses the complication of the sensory deficit that places the patient at greatest risk for injury? a. Risk for falls. b. Body image disturbance. c. Social isolation. d. Fear. ANS: A
A visual disturbance poses great risk for injury from falling as a result of impaired depth perception and inability to see obstacles. Body image disturbance, Social isolation, and Fear are all valid nursing diagnoses that apply to a patient with vision deficit; however, they do not address the greatest risk for injury. DIF: Analyze REF: 1351| 1352 OBJ: Develop a plan of care for individuals with sensory alterations. TOP: Planning MSC: NCLEX: Safe and Effective Care Environment 16. A patient has hyperesthesia associated with a neurological trauma. Which one of the
following is an appropriate nursing intervention in regard to the patient’s sense of touch? a. Reminding the patient of the need to have frequent tactile contact. b. Keeping the patient loosely covered with sheets and blankets. c. Allowing the patient to liN eU mR otS ioI nlN esG s.TB.COM d. Using touch as a form of therapy. ANS: B
If a patient is overly sensitive to tactile stimuli (hyperesthesia), the nurse must minimize irritating stimuli. Keeping bed linens loose to minimize direct contact with the patient and protecting the patient’s skin from exposure to irritants are helpful measures. Frequent tactile contact and allowing the patient to lie motionless are not appropriate interventions for the patient with hyperesthesia. Using touch as a form of therapy would not be an appropriate nursing intervention for the patient with hyperesthesia. DIF: Apply REF: 1357 OBJ: Identify interventions for preventing sensory deprivation and controlling sensory overload. TOP: Implementation MSC: NCLEX: Psychosocial Integrity 17. The nurse is caring for a patient with conductive hearing loss in one ear resulting from
prolonged cerumen impaction. Which intervention by the nurse is most important in establishing effective communication with the patient? a. Speaking in a loud voice, enunciating every syllable. b. Having direct conversation with the patient in his affected ear. c. If the patient does not understand what the nurse is saying, repeating the phrase again. d. Speaking with hands, face, and expressions. ANS: D
Canadian Fundamentals of Nursing 6th Edition Potter Test Bank Using gestures other than just speaking helps the patient understand what you are saying and makes it a meaningful stimulus. Speaking in loud tones can distort a patient’s ability to hear; the nurse should speak in normal low tones. If the patient does not understand the first time, the nurse should try rephrasing instead of repeating the message. The nurse can direct conversation toward the patient’s unaffected ear. DIF: Understand REF: 1357 (Box 47-11) OBJ: Describe conditions in the health care setting or a patient’s home that you can adjust to promote meaningful sensory stimulation. TOP: Implementation MSC: NCLEX: Safe and Effective Care Environment 18. The home health nurse is caring for a patient with a tactile deficit; the nurse is concerned
about injury related to inability to feel harmful stimuli. The nurse determines that the patient is able to care for himself safely when the patient demonstrates which action? a. Places coloured stickers on faucet handles to indicate temperature and keeps a thermometer near the tub. b. Asks the nurse to test the temperature of the water before entering the bath. c. Replaces all lace-up shoes with Velcro ones and purchases shampoo caps. d. Dispenses all medications onto a plate for easy access in the morning. ANS: A
By placing colour-coded stickers and other reminders about dangerous stimuli, the patient is able to safely keep up hygiene. Asking the nurse to test the water does not promote independence, although it does promote safety. Zipper and Velcro clothing is easier for a patient with a tactile deficit to wear, and shower caps allow the patient to stay well-groomed with minimal effort, but neither promotes safety. Leaving the lids off of medications can be dangerous, as can placing all medications out at once. It may be difficult for the patient to sort N Rlect INthe GTcorrect B.COtypes M and numbers of pills. through mixed medications andUseS DIF: Evaluate REF: 1357 OBJ: Discuss ways to maintain a safe environment for patients with sensory deficits. TOP: Evaluate MSC: NCLEX: Safe and Effective Care Environment 19. The nurse is caring for a patient in acute respiratory distress. The patient has multiple
monitoring systems on that constantly beep and make noise. The patient is becoming agitated and frustrated over inability to sleep. Which action by the nurse is most appropriate for this patient? a. Providing the patient with a therapeutic back rub. b. Turning off the alarms on the monitoring devices. c. Administering an opioid medication to help the patient sleep. d. Providing the patient with earplugs. ANS: D
Giving the patient control over stimuli helps to decrease the frustration that results from sensory overload. Adding more stimuli such as a back rub can increase sensory overload. Turning off monitors and alarms is unsafe; the nurse needs to be aware of critical situations. Opioid medications should not be the first option; however, antianxiety medications and sleep aids may be considered. DIF: Apply REF: 1351| 1355 OBJ: Identify interventions for preventing sensory deprivation and controlling sensory overload.
Canadian Fundamentals of Nursing 6th Edition Potter Test Bank TOP: Implementation
MSC: NCLEX: Health Promotion and Maintenance
20. The nurse is caring for a patient with expressive aphasia caused by a traumatic brain injury.
Which desired outcome should be included in the plan of care? a. Patient will recover full use of speech vocabulary in 1 week. b. Patient will carry a pen and a pad of paper around for communication. c. Patient will thicken drinks to prevent aspiration. d. Patient will communicate nonverbally. ANS: D
Patients with expressive aphasia may take a prolonged time to regain speech function, depending on the cause of the incident. To adapt to expressive aphasia, the nurse and the patient need to work on ways to communicate nonverbally through means such as pointing and gestures. A patient who has expressive aphasia may not be able to speak or write words. Thickening drinks prevents aspiration risk and is not included in a plan of care for this patient. DIF: Apply REF: 1347 OBJ: Develop a plan of care for individuals with sensory alterations. TOP: Planning MSC: NCLEX: Physiological Integrity 21. The nurse is aware that which patient is most at risk for sensory deprivation? a. A patient in the critical care unit (CCU) under constant monitoring after a
myocardial infarction. b. A patient on the unit with tuberculosis on airborne precautions. c. A patient who recently had a stroke and has left-sided weakness. d. A patient receiving hospice care for end-stage brain cancer. ANS: B
NURSINGTB.COM
Sensory deprivation occurs when a person has decreased stimulation and limited sensory input. A patient in isolation is at risk for sensory deprivation because he or she has limited exposure to meaningful stimuli. A patient in the CCU would be at risk for sensory overload with all the monitors and visitors. A patient with a stroke or with brain cancer may have difficulty with tactile sensation and may have sensory deficits but is not at risk for sensory deprivation. DIF: Evaluate REF: 1358 OBJ: Discuss common causes and effects of sensory alterations. TOP: Planning MSC: NCLEX: Psychosocial Integrity 22. What nursing action can the nurse implement to comfort an older patient with sensory
deprivation to improve meaningful stimuli? a. Placing a “Do not disturb” sign on the patient’s door. b. Offering the patient a back rub. c. Asking the patient whether he or she would like a newspaper to read. d. Placing the patient in the room farthest from the nurses’ station. ANS: B
The patient with sensory deprivation needs meaningful stimuli, and therapeutic massage helps establish a humanistic relationship that the patient is missing. All of the other options do not promote interaction and promote further social isolation. DIF: Understand
REF: 1358
Canadian Fundamentals of Nursing 6th Edition Potter Test Bank OBJ: Identify interventions for preventing sensory deprivation and controlling sensory overload. TOP: Implementation MSC: NCLEX: Psychosocial Integrity 23. The nurse is caring for a patient who is a well-known surgeon at the hospital. Because of his
status, all the hospital’s physicians want to be sure to pay him a visit. The nurse notices the patient becoming more agitated and withdrawn with each group of visitors. The nurse asks the patient if he would like a “Do not disturb” sign placed on the door. A few hours later, the nurse notices a physician who is not involved in the patient’s care attempting to enter the room. Which response by the nurse is most appropriate? a. Allowing the physician to enter because he has higher authority than the nurse. b. Calling for security to remove the visitor. c. Firmly explaining that the patient does not wish to have visitors at this time, so do not enter the room. d. Scolding the physician for not obeying the signs on the door and respecting the patient’s wishes. ANS: C
The nurse acts as an advocate for the patient (who is experiencing sensory overload and would benefit from a quiet environment) by firmly and politely asking the visitor to leave, regardless of position in the hospital. The nurse should not allow anyone to enter unless the patient approves it. Security is not a necessary measure at this time. The nurse should act with professionalism when addressing the visitor; scolding the visitor is not appropriate. DIF: Understand REF: 1340 OBJ: Describe conditions in the health care setting or a patient’s home that you can adjust to promote meaningful sensory stimulation. TOP: Implementation MSC: NCLEX: Health Promotion and Maintenance
NURSINGTB.COM
24. The nurse is caring for a patient who is recovering from a traumatic brain injury and
frequently becomes disoriented to everything except her location. Which nursing intervention would be effective in orienting a patient with neurological deficit? a. Assessing the patient’s level of consciousness and documenting every 4 hours. b. Keeping a day-by-day calendar at the patient’s bedside and having the patient manage it. c. Placing a patient observer in the patient’s room for safety. d. Informing the patient that she cannot be discharged unless she is awake, alert, and oriented. ANS: B
Keeping a calendar in the patient’s room helps to orient the patient to the dates and gives the patient a sense of control over her environment. Assessing the patient’s level of consciousness is not an action that will directly affect the patient’s confusion. A patient observer is unnecessary unless the patient is in danger from the confusion. The nurse should encourage the patient toward recovery but should be sensitive to the time it takes for progression. DIF: Apply REF: 1344 OBJ: Discuss ways to maintain a safe environment for patients with sensory deficits. TOP: Implementation MSC: NCLEX: Psychosocial Integrity 25. A nurse is establishing a relationship with a patient who is visually impaired. Which is the
most appropriate method to teach the patient how to contact the nurse for assistance?
Canadian Fundamentals of Nursing 6th Edition Potter Test Bank a. Placing a raised Braille sticker on the call button, and instructing the patient to
press for assistance. b. Instructing the patient to yell at the top of his lungs to get the attention of the staff. c. Explaining to the patient that a staff person will stop by once an hour to see
whether the patient needs anything. d. Sharing cell phone numbers with the patient so he can call the nurse if he needs
her. ANS: A
The nurse should devise a plan of care that accommodates the patient’s visual deficit. Placing a sticker on the call light allows the patient to find it and page the nurse for assistance as needed. Yelling at the top of the lungs is stressful for the patient and for surrounding patients. Making hourly rounds is not sufficient; the nurse needs to ensure that the patient can get in touch with her at any time. Sharing personal phone numbers with the patient is inappropriate. DIF: Apply REF: 1355| 1356 OBJ: Develop a plan of care for individuals with sensory alterations. TOP: Implementation MSC: NCLEX: Safe and Effective Care Environment 26. The nurse is developing a plan of care for a patient who is having a prosthetic eye placed.
Which nursing diagnosis related to patient safety is the priority for the nurse to include in the plan of care? a. Self-care deficit. b. Risk for injury. c. Anxiety. d. Body image disturbance. ANS: B
NURSINGTB.COM
The patient with a prosthetic eye will require a period of adjustment to new depth perception and visual sensation. Until the patient adapts, preventing injury should be the nurse’s priority. The other options are not directly related to the safety of the patient for eye surgery. DIF: Analyze REF: 1351| 1352 OBJ: Develop a plan of care for individuals with sensory alterations. TOP: Planning MSC: NCLEX: Health Promotion and Maintenance
Canadian Fundamentals of Nursing 6th Edition Potter Test Bank
Chapter 48: Care of Surgical Patients Potter et al: Canadian Fundamentals of Nursing, 6th Edition MULTIPLE CHOICE 1. The nurse is serving as preceptor for a student nurse and explains that perioperative nursing
care occurs when or where? a. Before, during, and after surgery. b. In preadmission testing. c. During the surgical procedure. d. In the postanaesthesia care unit (PACU). ANS: A
Perioperative nursing care occurs before, during, and after surgery. Preadmission testing occurs before surgery and is considered preoperative. Nursing care provided during the surgical procedure is considered intraoperative, and in the PACU, it is considered postoperative. All of these are parts of the perioperative phase, but each individual phase does not explain the term completely. DIF: Remember REF: 1363 OBJ: Understand the aspects of perioperative nursing care. MSC: NCLEX: Safe and Effective Care Environment
TOP: Implementation
2. The nurse is caring for a patient who is scheduled to undergo a surgical procedure. The nurse
is completing an assessment and reviews the patient’s laboratory test results and allergies. In which perioperative nursing phase would this work be completed? N R I G B.C M a. Perioperative. U S N T O b. Preoperative. c. Intraoperative. d. Postoperative. ANS: B
Reviewing the patient’s laboratory tests and allergies is done before surgery in the preoperative phase. Perioperative means before, during, and after surgery. Intraoperative means during the surgical procedure in the operating suite; postoperative means after the surgery and could occur in the PACU, in the ambulatory surgical area, or on the hospital unit. DIF: Understand REF: 1374 OBJ: Understand the aspects of perioperative nursing care. MSC: NCLEX: Safe and Effective Care Environment
TOP: Assessment
3. The nurse is caring for a patient in the PACU. The patient has developed profuse bleeding
from the surgical site, and the surgeon has determined the need to return to the operative area. This procedure would be classified as which of the following? a. Elective. b. Urgent. c. Emergency. d. Major. ANS: C
Canadian Fundamentals of Nursing 6th Edition Potter Test Bank An emergency procedure must be done immediately to save life or preserve function of a body part. An example would be repair of a perforated appendix, repair of a traumatic amputation, or control of internal hemorrhaging. An urgent procedure is necessary for a patient’s health and often prevents additional problems from developing. An example would be excision of a cancerous tumour, removal of a gallbladder for stones, or vascular repair for obstructed artery. An elective procedure is performed on the basis of the patient’s choice; it is not essential and is not always necessary for health. An example would be a bunionectomy, plastic surgery, or hernia reconstruction. A major procedure involves extensive reconstruction or alteration in body parts; it poses great risks to well-being. An example would be a coronary artery bypass or colon resection. DIF: Remember REF: 1366 (Table 48-1) OBJ: Differentiate between classifications of surgery and types of anaesthesia. TOP: Assessment MSC: NCLEX: Safe and Effective Care Environment 4. The nurse is caring for a patient in preadmission testing. The patient has been assigned a
physical status classification of PS-III by the American Society of Anesthesiologists. Which of the following assessments would support this classification? a. Denial of any major illnesses or conditions. b. Normal, healthy patient. c. History of hypertension, 36 kg (80 pounds) overweight, history of asthma. d. History of myocardial infarction that limits activity. ANS: C
A classification of PS-III means that a patient has a severe systemic disease. Patients with hypertension, obesity, diabetes mellitus, and asthma fit into this category. PS-I is the classification for a normal, healthy patient. PS-II is the classification for a patient with mild Nclas RSsific INatio GTnB.C M systemic disease. PS-IV is the U for a O patient with severe systemic disease that is a constant threat to life. DIF: Understand REF: 1366 (Table 48-2) OBJ: Differentiate between classifications of surgery and types of anaesthesia. TOP: Assessment MSC: NCLEX: Safe and Effective Care Environment 5. The patient has arrived at the ambulatory surgery centre for a colonoscopy. The patient is
scheduled to receive moderate sedation (conscious sedation) during the procedure. Moderate sedation is used routinely for procedures that require which of the following? a. An outpatient setting. b. A depressed level of consciousness. c. Loss of sensation in an area of the body. d. Immobility of the patient. ANS: B
Moderate sedation (conscious sedation) is used routinely for procedures that require not complete anaesthesia but rather a depressed level of consciousness. Not all patients who are treated on an outpatient basis receive moderate sedation. Regional anaesthesia such as local anaesthesia provides loss of sensation in an area of the body. General anaesthesia is used for patients who need to be immobile and to not remember the surgical procedure. DIF: Remember REF: 1397 OBJ: Differentiate between classifications of surgery and types of anaesthesia.
Canadian Fundamentals of Nursing 6th Edition Potter Test Bank TOP: Assessment
MSC: NCLEX: Safe and Effective Care Environment
6. The nurse is caring for a patient in the PACU who has undergone a left total knee
arthroplasty. The anaesthesia provider has indicated that the patient received a left femoral peripheral nerve block. Which assessment would be an expected finding for a patient with this type of regional block? a. Decreased pulse at the left posterior tibia. b. Left toes cool to touch and slightly cyanotic. c. Sensation decreased in the left leg. d. Patient report of pain in the left foot. ANS: C
Induction of regional anaesthesia results in loss of sensation in an area of the body. The peripheral nerve block influences the portion of sensory pathways that are anaesthetized in the targeted area of the body. Decreased pulse, coolness of toes to touch, and cyanosis are indications of decreased blood flow and are not expected findings. Reports of pain in the left foot may indicate that the block is not working or is subsiding and is not an expected finding in the immediate postoperative period. DIF: Understand REF: 1397 OBJ: Differentiate between classifications of surgery and types of anaesthesia. TOP: Assessment MSC: NCLEX: Physiological Integrity 7. The nurse is preparing a patient for surgery. Aims of assessment before surgery include which
of the following? a. Establishing a patient’s baseline of normal function. b. Planning for care after the procedure. RiS c. Educating the patient andNfU am lyIaN boG uT t tB he.pC roOcM edure. d. Gathering appropriate equipment for the patient’s needs. ANS: A
The aim of assessment of the patient before surgery is to establish the patient’s normal preoperative function to prevent and minimize possible postoperative complications. Gathering appropriate equipment, planning care, and educating the patient and family are all important interventions that must be provided for the surgical patient; they are part of the nursing process but are not the reason for completing an assessment of the surgical patient. DIF: Understand REF: 1400 OBJ: List factors to include in the preoperative, intraoperative, and postoperative assessment of a surgical patient. TOP: Assessment MSC: NCLEX: Safe and Effective Care Environment 8. The nurse is documenting a medication history for the surgical patient in preadmission testing.
Which of the following medications should the nurse instruct the patient to withhold in preparation for surgery? a. Ibuprofen. b. Acetaminophen. c. Vitamin C. d. Miconazole. ANS: A
Canadian Fundamentals of Nursing 6th Edition Potter Test Bank Nonsteroidal anti-inflammatory drugs (NSAIDs) such as ibuprofen inhibit platelet aggregation and prolong bleeding time, thereby increasing susceptibility to postoperative bleeding. Acetaminophen is a pain reliever; vitamin C actually assists in wound healing; and miconazole is an antifungal. None of these three has any special implications for surgery. DIF: Apply REF: 1371 (Table 48-5) OBJ: List factors to include in the preoperative, intraoperative, and postoperative assessment of a surgical patient. TOP: Assessment MSC: NCLEX: Safe and Effective Care Environment 9. The nurse is caring for a potential surgical patient in the preadmission testing unit. The
medication history indicates that the patient is currently taking warfarin (Coumadin). Which of the following actions should the nurse take? a. Consult with the physician regarding a radiological examination of the chest. b. Consult with the physician regarding an international normalized ratio (INR). c. Consult with the physician regarding blood urea nitrogen (BUN). d. Consult with the physician regarding a complete blood count (CBC). ANS: B
Warfarin is an anticoagulant that is utilized for different maladies, but its action is to increase the time it takes for the blood to clot. This action can put the surgical patient at risk for bleeding. Typically, if at all possible, this medication is withheld for several days before a surgical procedure to decrease this risk. INR, prothrombin time, activated partial thromboplastin time, and platelet counts reveal the clotting ability of the blood. Chest radiography, BUN measurement, and CBC are diagnostic screening tools for surgery but are not specific to warfarin. DIF: Apply REF: 1374 (Table 48-6) OBJ: List factors to include inNthU eR prS eoIpN erG atiT veB ,. intC raO opMerative, and postoperative assessment of a surgical patient. TOP: Assessment MSC: NCLEX: Physiological Integrity 10. The nurse is encouraging the postoperative patient to utilize diaphragmatic breathing. Reasons
for this intervention include which of the following? a. Management of pain. b. Decreased healing time. c. Prevention of atelectasis. d. Decreased thrombus formation. ANS: C
During general anaesthesia, the lungs are not fully inflated during surgery and the cough reflex is suppressed, and so mucus collects within airway passages. After surgery, patients may have reduced lung volume and may require greater effort to cough and deep breathe; inadequate lung expansion can lead to atelectasis and pneumonia. Purposely utilizing diaphragmatic breathing can decrease this risk. Diaphragmatic breathing, except for the components of distraction, minimal increased level of oxygen, and minimal chest wall movement, does not influence pain, healing time, or thrombus formation. More effective interventions are available for those situations. DIF: Understand REF: 1404| 1405 (Table 48-10) OBJ: Demonstrate postoperative exercises: diaphragmatic breathing, coughing, turning, and leg exercises. TOP: Intervention MSC: NCLEX: Physiological Integrity
Canadian Fundamentals of Nursing 6th Edition Potter Test Bank 11. The nurse is caring for a postoperative patient on the medical-surgical floor. To prevent
venous stasis and the formation of thrombus after general anaesthesia, what does the nurse encourage the patient to do? a. Coughing. b. Diaphragmatic breathing. c. Incentive spirometry. d. Leg exercises. ANS: D
After general anaesthesia, circulation slows, and when the rate of blood slows, the tendency for clot formation is greater. Immobilization results in decreased muscular contractions in the lower extremities; these promote venous stasis. Coughing, diaphragmatic breathing, and incentive spirometry are utilized to decrease the risk of atelectasis. DIF: Remember REF: 1385 (Skill 48-1)| 1404| 1405 (Table 48-10) OBJ: Demonstrate postoperative exercises: diaphragmatic breathing, coughing, turning, and leg exercises. TOP: Intervention MSC: NCLEX: Physiological Integrity 12. The nurse is caring for a preoperative patient. The nurse teaches the principles and
demonstrates leg exercises for the patient. The patient is unable to perform leg exercises correctly. What is the nurse’s best next step? a. Assess for the presence of anxiety, pain, or fatigue. b. Ask the patient why he does not want to do the exercises. c. Encourage the patient to practise at a later date. d. Assess the educational methods used to educate the patient. ANS: A
If the patient is unable to perN form afte r sound educational principles and URleg SIex NGerci TBse.s C OM demonstration are provided, the nurse should look for circumstances that may be affecting the patient’s ability to learn. In this case, the patient can be anticipating the upcoming surgery and may be experiencing anxiety. The patient may also be in pain or may be fatigued; both of these can affect the ability to learn. Assessment of educational methods may be needed, but in this case, sound principles and demonstration are being utilized. Asking anyone why he or she is not cooperating can cause defensiveness and may not help in attaining the answer. In this case, the patient really may want to participate and may not know why he is unable to learn. The nurse is aware that the patient is unable to do the exercises; encouraging the patient to practise later will not help the patient meet goals. DIF: Apply REF: 1389 (Skill 48-1) OBJ: Demonstrate postoperative exercises: diaphragmatic breathing, coughing, turning, and leg exercises. TOP: Intervention MSC: NCLEX: Psychosocial Integrity 13. Which nursing assessment would indicate that the patient is performing diaphragmatic
breathing correctly? a. Hands placed on border of rib cage with fingers extended will touch as chest wall contracts. b. Hands placed on chest wall with fingers extended will separate as chest wall contracts. c. The patient will feel upward movement of the diaphragm during inspiration. d. The patient will feel downward movement of the diaphragm during expiration.
Canadian Fundamentals of Nursing 6th Edition Potter Test Bank ANS: A
Positioning the hands along the borders of the rib cage allows the patient to feel movement of the chest and abdomen as the diaphragm descends and the lungs expand. As the patient takes a deep breath and slowly exhales, the middle fingers will touch while the chest wall contracts. The fingers will separate as the chest wall expands. The patient will feel normal downward movement of the diaphragm during inspiration and normal upward movement during expiration. DIF: Apply REF: 1385-1387 (Skill 48-1) OBJ: Demonstrate postoperative exercises: diaphragmatic breathing, coughing, turning, and leg exercises. TOP: Assessment MSC: NCLEX: Health Promotion and Maintenance 14. The nurse is caring for a postoperative patient with an abdominal incision. A pillow is used
during coughing to provide which of the following? a. Pain relief. b. Splinting. c. Distraction. d. Anxiety reduction. ANS: B
Surgical incisions cut through muscles, tissues, and nerve endings. Deep breathing and coughing exercises place additional stress on the suture line and cause discomfort. Splinting incisions with hands and a pillow provides firm support and reduces incisional pull. Providing a pillow during coughing does not provide distraction, reduce anxiety, or provide pain relief. Coughing can increase anxiety because it can cause pain. DIF: Understand REF: 1387 (Skill 48-1) OBJ: Demonstrate postoperatiN veUeR xeS rcI isN esG : dT iaB ph.raCgO mM atic breathing, coughing, turning, and leg exercises. TOP: Intervention MSC: NCLEX: Physiological Integrity 15. The nurse is encouraging a reluctant postoperative patient to deep breathe and cough. What
explanation can the nurse provide that may encourage the patient to cough more effectively? a. “If you don’t deep breathe and cough, you will get pneumonia.” b. “Deep breathing and coughing will clear out the anaesthesia.” c. “Coughing will not harm the incision if done correctly.” d. “You will need to cough only a few times during this shift.” ANS: C
If coughing is done correctly with proper support of the incision, it will not harm the incision. Deep breathing and coughing help clear out mucus in the respiratory system that has been caused by the anaesthetic. Although it is correct that a patient may experience atelectasis and pneumonia if deep breathing and coughing are not performed, the statement sounds threatening; the information could be communicated in a more therapeutic manner. Deep breathing and coughing is encouraged every 2 hours while the patient is awake. DIF: Apply REF: 1387 (Skill 48-1) OBJ: Demonstrate postoperative exercises: diaphragmatic breathing, coughing, turning, and leg exercises. TOP: Intervention MSC: NCLEX: Health Promotion and Maintenance
Canadian Fundamentals of Nursing 6th Edition Potter Test Bank 16. The nurse and the unregulated care provider are helping a postoperative patient turn in the
bed. To assist in minimizing discomfort, which instruction should the nurse provide to the patient? a. “Close your eyes and think about something pleasant.” b. “Hold your breath and count to three.” c. “Hold my shoulders with your hands.” d. “Place your hand over your incision.” ANS: D
The nurse should instruct the patient to place one hand over the incisional area to splint it, providing support and minimizing pulling during turning. Closing one’s eyes, holding one’s breath, and holding the nurse’s shoulders do not help support the incision during a turn. DIF: Apply REF: 1387 (Skill 48-1) OBJ: Demonstrate postoperative exercises: diaphragmatic breathing, coughing, turning, and leg exercises. TOP: Intervention MSC: NCLEX: Physiological Integrity 17. An obese patient admitted for abdominal surgery is more susceptible to the postoperative
complication of which of the following? a. Anemia. b. Seizures. c. Protein loss. d. Dehiscence. ANS: D
An obese patient is susceptible to poor wound healing and wound infection because of the structure of fatty tissue, which contains a poor blood supply. This increases the risk for dehiscence. A patient who isN mal URnou SIrish NGed,TBno.tCanOoMbese patient, is more susceptible to being anemic. An obese patient is not more susceptible to seizures or to protein loss. A patient with liver disease may have altered protein metabolism. DIF: Apply REF: 1369| 1370 OBJ: Describe the rationales for nursing interventions designed to prevent postoperative complications. TOP: Intervention MSC: NCLEX: Physiological Integrity 18. The nurse is providing preoperative teaching for the ambulatory surgical patient who will
have a cyst removed from the right arm. Which would be the best explanation for diet progression after surgery? a. “Start with clear liquids, soup, and crackers. Advance to a normal diet as you tolerate.” b. “There is no limitation on your diet. You can have whatever you want.” c. “Stay on clear liquids for 24 hours. Then you can progress to a normal diet.” d. “Start with clear liquids for 2 hours, then full liquids for 2 hours. Then progress to a normal diet.” ANS: A
Canadian Fundamentals of Nursing 6th Edition Potter Test Bank The type of surgery that patients undergo determines how quickly they can resume normal physical activity and regular eating habits. It is normal to progress gradually in activity and eating, and if the patient tolerates activity and diet well, he or she can progress more quickly. A common complication after surgery is nausea and vomiting. This can be caused by the anaesthetic, fluid imbalance from being on NPO (nothing by mouth) status, and pain. The gastrointestinal tract may be hypoactive as a result of anaesthesia. It is best to start with a clear liquid to see whether the patient can tolerate the liquid without vomiting. If so, it is appropriate for the patient to progress to soup and crackers and advancing as tolerated. Starting with a heavy, greasy meal could cause nausea and vomiting. There is no need to stay on clear liquids for 24 hours after this procedure. Putting a time frame on the progression is too prescriptive. Progression should be adjusted for the patient’s needs. DIF: Apply TOP: Intervention
REF: 1407 OBJ: Design a preoperative teaching plan. MSC: NCLEX: Physiological Integrity
19. The nurse explains the pain relief measures available after surgery during preoperative
teaching for a surgical patient. Which of the following comments from the patient indicates the need for additional education on this topic? a. “I will take the pain medication as the physician prescribes it.” b. “I will be asked to rate my pain on a pain scale.” c. “I will have minimal pain because of the anaesthesia.” d. “I will take my pain medications before doing postoperative exercises.” ANS: C
Pain after surgery is expected and is one of the patient’s fears. Anaesthesia will be induced during the procedure itself, and the patient should not experience pain during the procedure. Pain management is utilized after the postoperative phase. The nurse should inform the patient NainRS I G B.C M of interventions available for pU relieNf, inTcludingOmedication, relaxation, and distraction. The patient needs to know and understand how to take the medications that the physician will prescribe postoperatively. During the stay in the facility, the level of pain is frequently assessed by the nurses. Coordinating pain medication with postoperative exercises helps to minimize discomfort and allows the exercises to be more effective. DIF: Analyze TOP: Evaluate
REF: 1391 OBJ: Design a preoperative teaching plan. MSC: NCLEX: Health Promotion and Maintenance
20. The nurse is making a preoperative education appointment with a patient. The patient asks
whether he should bring family with him to the appointment. What is the best response by the nurse? a. “There is no need for an additional person at the appointment.” b. “Your family can come and wait with you in the waiting room.” c. “We recommend including family in this appointment to ease everyone’s anxiety.” d. “It is required that you have a family member at this appointment.” ANS: C
Canadian Fundamentals of Nursing 6th Edition Potter Test Bank It is ideal to attempt perioperative education before admission, during the hospital stay, and after discharge. Including family members in perioperative education is advisable. Often a family member is a coach for postoperative exercises when the patient returns from surgery. If anxious relatives do not understand routine postoperative events, it is likely that their anxiety will heighten the patient’s fears and concerns. Perioperative preparation of family members before surgery helps minimize anxiety and misunderstanding. An additional person is needed at the appointment if at all possible, and he or she needs to be involved in the process, not just waiting in the waiting room; however, it is certainly not a requirement for actually completing the surgery that someone comes to this appointment. DIF: Apply TOP: Intervention
REF: 1365 OBJ: Design a preoperative teaching plan. MSC: NCLEX: Psychosocial Integrity
21. The nurse is reviewing the surgical consent with the patient during preoperative education.
The patient indicates that he does not understand what procedure will be completed. What is the nurse’s best next step? a. Notify the physician about the patient’s question. b. Explain the procedure that will be completed. c. Ask the patient to sign the form. d. Continue with preoperative education. ANS: A
Surgery cannot be legally or ethically performed until the patient understands the need for a procedure, the steps involved, the risks, expected results, and alternative treatments. It is the surgeon’s responsibility to explain the procedure and obtain informed consent. It is important for the nurse to pause to notify the physician of the patient’s questions. It is not within the nurse’s scope to explain the procedure for the first time. The nurse can certainly reinforce Ned,RS ItheGT B.C tion M needs to come from the physician. It is what the physician has explainU but N informaO not prudent to ask a patient to sign a form for a procedure that he or she does not understand. DIF: Apply TOP: Intervention
REF: 1378 OBJ: Design a preoperative teaching plan. MSC: NCLEX: Safe and Effective Care Environment
22. During preoperative assessment for a 0730-hour case, the patient indicates to the nurse that he
had a cup of coffee at 0700 hours. The nurse reports this information to the anaesthesia provider, anticipating what? a. A delay in or cancellation of surgery. b. Questions regarding components of the coffee. c. Additional questions about why the patient had coffee. d. Instructions to determine what education was provided in the preoperative visit. ANS: A
A patient traditionally takes nothing by mouth (NPO) after midnight on the morning of surgery to keep the stomach empty and thus reduce the risk of vomiting and aspiration. It is recommended that patients fast from clear liquids at least 2 hours before elective procedures that necessitate general anaesthesia, regional anaesthesia, or sedation/analgesia. A delay in or cancellation of surgery will be in order for this case. Questions regarding components of the coffee (e.g., milk; can determine the length of time for a delay), asking why, and evaluating the preoperative education may all be items to be addressed, especially from a performance improvement perspective, but at this time in caring for this patient, a delay or cancellation is in order.
Canadian Fundamentals of Nursing 6th Edition Potter Test Bank
DIF: Understand TOP: Intervention
REF: 1391 OBJ: Prepare a patient for surgery. MSC: NCLEX: Physiological Integrity
23. An appendectomy is appropriately documented by the nurse as which type of surgery? a. Diagnostic surgery. b. Palliative surgery. c. Ablative surgery. d. Reconstructive surgery. ANS: C
Ablative surgery is the excision or removal of a diseased body part, such as an appendectomy. Diagnostic surgery is surgical exploration that allows the physician to confirm a diagnosis. This type of surgery may involve removal of tissue for further diagnostic testing. An example would be a breast mass biopsy. Palliative surgery relieves or reduces the intensity of disease symptoms. It will not produce a cure. An example is resection of nerve roots. Reconstructive surgery restores function or appearance to traumatized or malfunctioning tissues. An example is internal fixation of a hip fracture. DIF: Apply REF: 1366 (Table 48-1) OBJ: Differentiate between classifications of surgery and types of anaesthesia. TOP: Intervention MSC: NCLEX: Safe and Effective Care Environment 24. The nurse has completed a preoperative assessment for a patient about to undergo surgery and
gathers assessment data. Of the following, which would be the most important next step? a. Notify the operating suite that the patient has a latex allergy. b. Document that the patient had a bath at home this morning. c. Ask the nursing assistantN toUoRbS taI inNvG itaTl B sig.nC s.OM d. Administer the ordered preoperative intravenous antibiotic. ANS: A
Innumerable products that contain latex are used in the operating suite and the PACU. For a patient with this allergy, special considerations are required for preparation of the room and the types of tubes, gloves, drapes, and instruments utilized. Ensuring that the patient has a safe environment takes time, and if the correct supplies are not available, awaiting their arrival may cancel or delay the case. Obtaining vital signs, documenting, and administering medications are all part of the process and should be done—with the latex allergy in mind. However, making sure that the operating suite and PACU are safe environments is the first step. DIF: Apply TOP: Assessment
REF: 1371 OBJ: Prepare a patient for surgery. MSC: NCLEX: Safe and Effective Care Environment
25. The nurse is preparing a patient for a surgical procedure on the right great toe. Which of the
following actions would be most important to include in this patient’s preparation? a. Ascertain that the surgical site has been correctly marked. b. Ascertain where the family will be located during the procedure. c. Place the patient in a clean surgical gown. d. Ask the patient to remove all hairpins and cosmetics. ANS: A
Canadian Fundamentals of Nursing 6th Edition Potter Test Bank Because errors have occurred in the past with patients undergoing the wrong surgery on the wrong site, the universal protocol has been implemented and is used with all invasive procedures. Part of this protocol includes marking the operative site with indelible ink. Knowing where the family is during a procedure, placing the patient in a clean gown, and asking the patient to remove all hairpins and cosmetics are important but are not most important in this list of items. DIF: Apply TOP: Assessment
REF: 1394 OBJ: Prepare a patient for surgery. MSC: NCLEX: Safe and Effective Care Environment
26. The nurse is caring for a patient intraoperatively. Primary roles of the circulating nurse
include which of the following? a. Establishing and implementing the plan of care. b. Maintaining a sterile field. c. Assisting with applying sterile drapes. d. Handing sterile instruments and supplies to the surgeon. ANS: A
The circulating nurse must be a registered nurse and has the responsibilities of preoperative assessment, establishing and implementing the plan of care, evaluating the care provided, and ensuring continuity of care postoperatively. The scrub nurse, who can be a registered nurse, a licensed practical nurse, or a surgical technologist, maintains the sterile field, assists with applying the sterile drapes, and hands sterile instruments and supplies to the surgeon. DIF: Understand TOP: Assessment
REF: 1395 OBJ: Explain the nurse’s role in the operating room. MSC: NCLEX: Safe and Effective Care Environment
27. The nurse is caring for a patiN enU tR inS thIeN pG reT opB er.aC tivOeM holding area of an ambulatory surgery
centre. Which nursing action would be most appropriate for this area? a. Monitor vital signs every 15 minutes. b. Empty the urinary drainage bag. c. Apply a warm blanket. d. Check the surgical dressing. ANS: C
The temperature in the preoperative holding area and in adjacent operating suites is usually cool. The nurse should offer the patient an extra warm blanket. The main activities in this area include verification of the patient’s identity, the surgery to be performed, and physical and emotional readiness for the procedure. The intravenous catheter is usually inserted, and the preoperative checklist is reviewed. Vital signs are not normally monitored unless there is a specific reason, such as a medication being administered. Ambulatory surgical patients do not typically come to the holding area with a urinary drainage bag or a surgical dressing. Taking care of those, if appropriate, is performed in the PACU. DIF: Apply TOP: Intervention
REF: 1396 OBJ: Explain the nurse’s role in the operating room. MSC: NCLEX: Safe and Effective Care Environment
28. The nurse is caring for a patient in the operating suite. Which of the following outcomes
would be most appropriate for this patient? a. At the end of the intraoperative phase, the patient will be free of burns at the grounding pad.
Canadian Fundamentals of Nursing 6th Edition Potter Test Bank b. At the end of the intraoperative phase, the patient will be free of infection. c. At the end of the intraoperative phase, the patient will be free of nausea and
vomiting. d. At the end of the intraoperative phase, the patient will be free of pain. ANS: A
A primary focus of intraoperative care is to prevent injury and complications related to anaesthesia, surgery, positioning, and equipment use, including use of the electrical cautery grounding pad for prevention of burns. The perioperative nurse is an advocate for the patient during surgery and protects the patient’s dignity and rights at all times. Evaluation of many goals and outcomes does not occur until after surgery. Signs and symptoms of infection do not have the time to manifest during the intraoperative phase. During the intraoperative phase, the patient is anaesthetized and unconscious, and an endotracheal tube that prevents conversation and complaints is in place. Nausea, vomiting, and pain typically begin in the postoperative phase of the experience. DIF: Understand TOP: Planning
REF: 1396 OBJ: Explain the nurse’s role in the operating room. MSC: NCLEX: Safe and Effective Care Environment
29. The nurse is caring for a postoperative patient with a history of obstructive sleep apnea. The
nurse monitors for which of the following? a. Choking and noisy, irregular respirations. b. Shallow respirations. c. Moaning and reports of pain. d. Disorientation. ANS: A
One of the greatest concerns N afU teR r gSeI neNraGl T anBa. esC thO esM ia is airway obstruction. Choking and noisy, irregular respirations are classic signs and symptoms of airway obstruction. A number of factors contribute to obstruction, including a history of obstructive sleep apnea; weakening of pharyngeal/laryngeal muscle tone by anaesthetics; secretions in the pharynx, bronchial tree, or trachea; and laryngeal or subglottic edema. In the postanaesthetic patient, the tongue is a major cause of airway obstruction. Shallow respirations are indicative of respiratory depression. Moaning and reports of pain are common in all surgical patients and are expected. Disorientation is common when the patient first awakens from anaesthesia, but it can be a sign of hypoxia. DIF: Understand REF: 1399| 1400 OBJ: Describe the rationales for nursing interventions designed to prevent postoperative complications. TOP: Planning MSC: NCLEX: Physiological Integrity 30. The nurse is caring for a patient in the operating suite who is exhibiting tachypnea,
tachycardia, premature ventricular contractions, and muscle rigidity. The nurse suspects which condition in this patient? a. Hypoxia. b. Malignant hyperthermia. c. Fluid imbalance. d. Hemorrhage. ANS: B
Canadian Fundamentals of Nursing 6th Edition Potter Test Bank A life-threatening, rare complication of anaesthesia is malignant hyperthermia. Malignant hyperthermia causes tachycardia, tachypnea, premature ventricular contractions, unstable blood pressure, cyanosis, skin mottling, and muscular rigidity. It often occurs during induction of anaesthesia. Hypoxia would manifest with decreased oxygen saturation. Fluid imbalance would be assessed with intake and output and can manifest with tachycardia and blood pressure fluctuations but is not characterized by muscle rigidity. Hemorrhage can manifest with tachycardia and decreased blood pressure, along with a thready pulse; usually some sign or symptom of blood loss is noted (e.g., from drains, incision, orifice, and abdomen). DIF: Analyze REF: 1401 OBJ: Describe the rationales for nursing interventions designed to prevent postoperative complications. TOP: Evaluate MSC: NCLEX: Physiological Integrity 31. The nurse is caring for a postoperative patient who has undergone a carpal tunnel repair. The
patient has a temperature of 36.1°C and is shivering. Which of the following is the best reason for this condition? a. The patient is dressed only in a gown. b. Anaesthesia lowers metabolism. c. The surgical suite has laminar flow. d. The open body cavity contributed to heat loss. ANS: B
The operating suite and recovery room environments are cool. The patient’s anaesthetically depressed level of body function results in lowering of metabolism and a fall in body temperature. The patient being dressed in a gown and laminar flow in the surgical suite can contribute to a decrease in temperature, but the length of time required for this procedure would minimize this effect. Also, the patient in this type of case does not have a large open N R I G B.C M O body cavity to contribute to heU at loS ss. N T DIF: Analyze REF: 1401| 1406 OBJ: Describe the rationales for nursing interventions designed to prevent postoperative complications. TOP: Evaluate MSC: NCLEX: Physiological Integrity 32. The nurse is monitoring a patient in the PACU for postoperative fluid and electrolyte
imbalance. Which of the following actions would be most appropriate for this patient? a. Encouraging copious amounts of water. b. Weighing the patient and comparing with preoperative weight. c. Measuring and recording all intake and output. d. Starting an additional intravenous (IV) line. ANS: C
Accurate recording of intake and output reflects renal and circulatory function. The nurse should measure and record all sources of intake and output. Copious amounts of water in a postoperative patient might produce nausea and vomiting. In the PACU, it is impractical to weigh the patient who is waking from surgery, but in the days afterward, it is a good assessment parameter for fluid imbalance. Starting an additional IV line is not necessary and is not important at this juncture. DIF: Apply REF: 1407 OBJ: Describe the rationales for nursing interventions designed to prevent postoperative complications. TOP: Intervention MSC: NCLEX: Physiological Integrity
Canadian Fundamentals of Nursing 6th Edition Potter Test Bank
33. The nurse is caring for a patient in the PACU. The patient asks for a bedpan and states to the
nurse, “I feel like I need to go to the bathroom, but I can’t.” Which of the following nursing interventions would be most appropriate? a. Encourage the patient to wait a minute and try again. b. Call the physician and obtain an order for catheterization. c. Assess the patient’s intake and the patient for bladder distension. d. Inform the patient that everyone feels this way after surgery. ANS: C
Depending on the surgery, some patients do not regain voluntary control over urinary function for 6 to 8 hours after anaesthesia. The nurse should assess the amount of fluid that the patient obtained while in surgery and palpate the lower abdomen just above the symphysis pubis for bladder distension. If fluid intake is not excessive and the bladder is nondistended, allowing some time might be appropriate. Not all postsurgical patients feel as if they need to urinate but cannot. If the bladder is distended and the patient is unable to void, catheterization might be in order. DIF: Apply REF: 1407 OBJ: Describe the rationales for nursing interventions designed to prevent postoperative complications. TOP: Intervention MSC: NCLEX: Physiological Integrity 34. The ambulatory surgical nurse calls to check on the patient at home the morning after surgery.
The patient is reporting continued nausea and vomiting. Which of the following discharge education points should be reviewed with the patient? a. Instruct the patient to take deep breaths. b. Instruct the patient to drink ginger ale and eat crackers. IN GT M to connect the patient with the c. Instruct the patient to callNtU heRpS hys icia n B.C and atO tempt physician. d. Instruct the patient to go to the emergency department. ANS: C
Postoperative nausea and vomiting sometimes occur once the patient is at home even if symptoms were not present in the surgery centre. Options for therapy include medications. Instructing the patient to call the physician and connecting the patient with the physician can help the patient obtain relief. Taking deep breaths, drinking ginger ale, and eating crackers are interventions that may be helpful, but this patient needs additional help. Instructing the patient to go to the emergency department is an option with continued nausea and vomiting. DIF: Understand REF: 1365| 1408 OBJ: Explain the differences and similarities in caring for ambulatory (day) surgical patients versus inpatient surgical patients. TOP: Intervention MSC: NCLEX: Physiological Integrity MULTIPLE RESPONSE 1. The nurse is serving as precept for a new nurse in the perioperative area. The nurse explains
that perioperative nursing is based on certain principles and includes which of the following? (Select all that apply.) a. Purchasing the correct equipment. b. Providing high-quality and patient safety–focused care.
Canadian Fundamentals of Nursing 6th Edition Potter Test Bank c. d. e. f.
Scheduling the right types of patients. Conducting multidisciplinary teamwork. Ensuring effective therapeutic communication. Providing advocacy for the patient.
ANS: B, D, E, F
Perioperative nursing is a fast-paced, changing, and challenging field and is based on the nurse’s understanding of several important principles, including high-quality, patient safety– focused care; multidisciplinary teamwork; effective therapeutic communication and collaboration with the patient, the patient’s family, and the surgical team; effective and efficient assessment and intervention in all phases of surgery; advocacy for the patient and the patient’s family; and understanding of cost containment. Purchasing the correct equipment is important in any specialty of nursing. Perioperative nursing cares not for only the “right” types of patients but for all patients with surgical needs. DIF: Remember REF: 1363| 1364 OBJ: Understand the aspects of perioperative nursing care. MSC: NCLEX: Safe and Effective Care Environment
TOP: Planning
2. The nurse is caring for an ambulatory surgical patient. To be discharged home, which criteria
must the patient meet? (Select all that apply.) a. Able to drink fluids. b. Able to eat crackers. c. Manageable pain. d. Able to void. e. Dry and intact dressing. f. Able to dress self. ANS: A, C, D, E
NURSINGTB.COM
To be discharged home, patients need to meet certain criteria. These criteria include meeting phase I criteria of activity, circulation, respiration, consciousness, and O2 saturation, as well as phase II criteria of dressing dry and intact, manageable pain, ambulation, able to drink fluids, and voiding. Eating and the ability to dress self are not included in these criteria. DIF: Remember REF: 1399 (Table 48-9) OBJ: Describe the rationales for nursing interventions designed to prevent postoperative complications. TOP: Assessment MSC: NCLEX: Physiological Integrity 3. The nurse is preparing for a patient who will undergo surgery. The nurse screens for risk
factors that can increase a person’s risks in surgery. What risk factors are included in the nurse’s screening? (Select all that apply.) a. Age. b. Nutrition. c. Race. d. Obesity. e. Pregnancy. f. Ambulatory surgery. ANS: A, B, D, E
Canadian Fundamentals of Nursing 6th Edition Potter Test Bank Very young and old patients are at risk during surgery because of immature or declining physiological status. Normal tissue repair and resistance to infection depend on adequate nutrition. Obesity increases surgical risk by reducing respiratory and cardiac function. During pregnancy, the concern is for the mother and the developing fetus. Because all major systems of the mother are affected during pregnancy, risks for operative complications are increased. Race and ambulatory surgery are not risks associated with a surgical procedure. DIF: Remember REF: 1402| 1403 OBJ: List factors to include in the preoperative, intraoperative, and postoperative assessment of a surgical patient. TOP: Assessment MSC: NCLEX: Physiological Integrity 4. The nurse is providing preoperative education and reviews with the patient what it will be like
to be in the surgical environment. What points should the nurse include? (Select all that apply.) a. The surgical area is cold but warm blankets will be provided. b. The surgical staff will be dressed in special clothing with hats and masks. c. The operative suite will be very dark. d. Families are not allowed in the operating suite. e. The operating table or bed will be comfortable and soft. f. The nurses will be there to assist you through this process. ANS: A, B, D, F
The operating suite itself is kept cool to decrease microbial growth, so it can be very cold to patients as they enter the suite, particularly with limited clothing. The surgical staff is dressed in special clothing, hats, and masks—all for infection control. Families are not allowed in the operating suite for several reasons, which include infection control and the emotional effect of seeing a loved one in that condition. The nurse is there as the coordinator and patient advocate N R I GTB.COM during a surgical procedure. ThUe roSomN s are very bright so that everyone can see, and the operating table is very uncomfortable for the patient. DIF: Apply TOP: Intervention
REF: 1395| 1396 OBJ: Design a preoperative teaching plan. MSC: NCLEX: Safe and Effective Care Environment
5. The nurse is caring for a patient in the operating suite. The nurse assists in positioning the
patient for what reasons? (Select all that apply.) a. To obtain access to the operative site. b. To sustain adequate circulatory and respiratory function. c. To ensure patient safety and skin integrity. d. To support the use of equipment. e. To maintain neuromuscular structures. f. To provide warmth and comfort. ANS: A, B, C, E
Ideally the patient’s position provides good access to the operative site, sustains adequate circulatory and respiratory function, and ensures patient safety and skin integrity. It should not impair neuromuscular structures. Warmth and comfort are always concerns, but the other options are more important because they relate to positioning. Positioning does not support the use of equipment; rather the use of equipment complements the position of the patient to maintain patient safety. DIF: Understand
REF: 1396
Canadian Fundamentals of Nursing 6th Edition Potter Test Bank OBJ: Describe the rationales for nursing interventions designed to prevent postoperative complications. TOP: Planning MSC: NCLEX: Safe and Effective Care Environment
NURSINGTB.COM